Download as pdf or txt
Download as pdf or txt
You are on page 1of 717

Mastering the BDS IIIrd Year

(Last 25 Years Solved Questions)


Mastering the BDS IIIrd Year
(Last 25 Years Solved Questions)
Thoroughly Revised and Updated According to the Latest Syllabus of DCI

7th Edition

Hemant Gupta mdS


(Oral and Maxillofacial Pathology, Microbiology and Forensic Odontology)
General Practitioner and Consultant
Shivom Multispeciality Dental Clinic
Indore, Madhya Pradesh, India

JAYPEE BROTHERS Medical Publishers


The Health Sciences Publisher
New Delhi | London | Panama
Jaypee Brothers Medical Publishers (P) Ltd

Headquarters
Jaypee Brothers Medical Publishers (P) Ltd
4838/24, Ansari Road, Daryaganj
New Delhi 110 002, India
Phone: +91-11-43574357
Fax: +91-11-43574314
Email: jaypee@jaypeebrothers.com

Overseas offices
J.P. Medical Ltd Jaypee-Highlights Medical Publishers Inc
83, Victoria Street, London City of Knowledge, Bld. 235, 2nd Floor,
SW1H 0HW (UK) Clayton, Panama City, Panama
Phone: +44 20 3170 8910 Phone: +1 507-301-0496
Fax: +44 (0)20 3008 618 Fax: +1 507-301-0499
Email: info@jpmedpub.com Email: cservice@jphmedical.com
Jaypee Brothers Medical Publishers (P) Ltd Jaypee Brothers Medical Publishers (P) Ltd
17/1-B Babar Road, Block-B, Shyamoli Bhotahity, Kathmandu, Nepal
Mohammadpur, Dhaka-1207 Phone: +977-9741283608
Bangladesh Email: kathmandu@jaypeebrothers.com
Mobile: +08801912003485
Email: jaypeedhaka@gmail.com
Website: www.jaypeebrothers.com
Website: www.jaypeedigital.com
© 2019, Jaypee Brothers Medical Publishers
The views and opinions expressed in this book are solely those of the original contributor(s)/author(s) and do not necessarily represent
those of editor(s) of the book.
All rights reserved. No part of this publication may be reproduced, stored or transmitted in any form or by any means, electronic,
mechanical, photocopying, recording or otherwise, without the prior permission in writing of the publishers.
All brand names and product names used in this book are trade names, service marks, trademarks or registered trademarks of their
respective owners. The publisher is not associated with any product or vendor mentioned in this book.
medical knowledge and practice change constantly. This book is designed to provide accurate, authoritative information about the
subject matter in question. However, readers are advised to check the most current information available on procedures included and
check information from the manufacturer of each product to be administered, to verify the recommended dose, formula, method and
duration of administration, adverse effects and contraindications. It is the responsibility of the practitioner to take all appropriate safety
precautions. Neither the publisher nor the author(s)/editor(s) assume any liability for any injury and/or damage to persons or property
arising from or related to use of material in this book.
This book is sold on the understanding that the publisher is not engaged in providing professional medical services. If such advice or
services are required, the services of a competent medical professional should be sought.
Every effort has been made where necessary to contact holders of copyright to obtain permission to reproduce copyright material. If any
have been inadvertently overlooked, the publisher will be pleased to make the necessary arrangements at the first opportunity. The CD/
DVD-ROM (if any) provided in the sealed envelope with this book is complimentary and free of cost. Not meant for sale.
Inquiries for bulk sales may be solicited at: jaypee@jaypeebrothers.com
Mastering the BDS IIIrd Year (Last 25 Years Solved Questions)
Seventh Edition: 2019
ISBN: 978-93-5270-406-4
Dedicated to
Almighty SAI BABA
My grandparents Shri HD Gupta and Smt Vijaylakshmi Gupta
In loving memory of my parents
Late Shri VK Gupta and Late Smt Anju Gupta
To my wife Smita Sharma Gupta
for being so much understanding and
Last but not least my lovely angel son Meetaan Gupta
for making life worthwhile
Preface to the Seventh edition

It is a matter of great pride and pleasure to introduce the seventh edition of Mastering the BDS IIIrd Year (Last 25
Years Solved Questions). The aim of this text is to enable the student of dentistry to learn fundamentals. All the sec-
tions are rewritten and the answers of each and every section are revised as per the latest syllabus. This new edition
is updated and expanded, bringing forth new information gained since production of last edition. The text has
been made more clinically oriented so as to better correlate the text with clinical aspects. The text consists of a large
number of illustrations, which enhances the understanding of written description. I, as an author, wish to express
my hope that material presented is clear and understandable. The book is never meant to replace any of the text-
book. All the respective textbooks of all subjects should be read thoroughly to gain the deep knowledge of subject.
This book provides an idea of questions and answers in BDS examinations and multiple choice questions (MCQs)
in pre-PG examinations. I hope that the content will be enough to stimulate the insight and new trends of thoughts
in all the subjects of IIIrd year.
Any of the suggestions and criticism should be welcomed at macrocyte@gmail.com.

 Hemant Gupta
Preface to the first edition

It is a matter of great pride and pleasure to introduce the first edition of Mastering the BDS IIIrd Year. As our previous
books have got continued support and good response, we have kept the same basic pattern, but the sequence of
chapters has been arranged in a simpler way for a wider and systematic coverage of the topics.
The subjects of IIIrd year still ring fear in the minds of students—baseless fear that rest on silent assumptions
and those that distort thinking. However, self-study, dedication, motivation and hard work are the virtues that
go a long way in the making of a genius—a success. Listen, think, read and analyze with an open mind and you
definitely cannot go wrong. I would like to clarify that this book is not meant to replace your standard textbooks,
but yet coupled with your effort and sincerity, it will definitely make you clinch and help you put your best foot
forward to reach great heights of success.
And last but not least, I thank our publisher, Shri Jitendar P Vij (Group Chairman) of M/s Jaypee Brothers
Medical Publishers (P) Ltd, New Delhi, India, for his whole-hearted support and help to make this book a reality.

Hemant Gupta
Acknowledgments

Achievement of this book was possible by the help and support of Almighty SAI BABA, my grandparents, parents,
my wife, teachers and friends.
Special thanks to those who remain behind the curtain and help in arrangement of study material for the book.
Heartily thanks to Dr Deepak Aggarwal (MDS), Mrs Sangeeta Surange, and Mrs Kriti Gorkhe, for helping in
arranging the appropriate question papers.
Thanks to Mr Anupam Prasad from M/s Jaypee Brothers Medical Publishers (P) Ltd, Indore, Madhya Pradesh,
India, for his whole-hearted support.
I am very grateful to Shri Jitendar P Vij (Group Chairman), Mr Ankit Vij (Managing Director), Ms Ritu Sharma
(Director–Content Strategy), Ms Sunita Katla (PA to Group Chairman, and Publishing Manager), Ms Samina
Khan (Executive Assistant to Director–Content Strategy), Dr Pinki Chauhan, Dr Nidhi Sinha, Dr Ambika Kapoor,
Dr Neeti Swarup (Development Editors), Ms Seema Dogra (Cover Visualizer), and the whole team of M/s Jaypee
Brothers Medical Publishers (P) Ltd, New Delhi, India, for all their support to work in this project and make it a
success. Without their cooperation, I could not have completed this project.
Contents

SECTION 1: General Medicine 25. Sterilization 463


26. Upper Limb Ischemia 466
1. Introduction 3 27. Rectum and Anal Canal 466
2. Diseases of Gastrointestinal Tract 9 28. Miscellaneous 466
3. Diseases of Liver 20 • Multiple Choice Questions 470
4. Diseases of Cardiovascular System 37 • Fill in the Blanks 479
5. Diseases of Respiratory System 70 • Viva-Voce Questions for Practical Examination 482
6. Diseases of Renal System 99 • Additional Information 485
7. Diseases of Blood 110
8. Diseases of Endocrine System 137 SECTION 3: Oral Pathology
9. Nutrition and Metabolic Defects 143
I.  Disturbances of Development and Growth 493
10. Diseases of Nervous System 165
1. Developmental Disturbances of
11. Tropical and Infectious Diseases 189 Oral and Para oral Structures 493
12. Miscellaneous 214 2. Benign and malignant tumors of Oral cavity 512
• Multiple Choice Questions 231 3. Tumors of Salivary Glands 540
• Fill in the Blanks 243 4. Odontogenic tumors 548
• Viva-Voce Questions for Practical Examination 250 5. Cysts of Oral Cavity 553
• Additional Information 253 II.  Disturbances of Microbial Origin 562
SECTION 2: General Surgery 6. Bacterial Infections of oral Cavity 562
7. Viral Infections of Oral cavity 568
1. Wound, Sinus and Fistula 261 8. Fungal Infections of Oral Cavity 573
2. Acute Infections 273 9. Diseases of periodontium 576
3. Specific Infections 281 10. Dental Caries 580
4. Ulcer 297 11. Diseases of the Pulp and Periapical Tissues 590
5. Lymphatics and Lymph Node Enlargement 302 III.  Injuries and Repair 598
6. Skin Tumors 312 12. Spread of Oral infection 598
7. Bleeding Disorders 313 13. Physical and Chemical injuries
8. Shock 322 of the oral cavity 601
9. Burn 335 14. Regressive Alterations of the teeth 603
10. Arterial Diseases 337 15. Healing of Oral Wounds 610
11. Venous Diseases 342 IV.  Disturbance of the Metabolism 615
12. Cyst 343 16. Oral Aspects of Metabolic Diseases 615
13. Oral Cavity, Lip and Palate 357 V.  Diseases of Specific System 618
14. Cleft Lip and Palate 376 17. Diseases of Bone and Joints 618
15. Oral Cancer 383 18. Diseases of blood and blood forming organs 625
16. Neck Swelling 397 19. Diseases of Skin 631
17. Salivary Gland 402 20. Diseases of nerves and muscles 642
18. Thyroid and Parathyroid Gland 409 VI.  Forensic Odontology 644
19. Tonsils 425 21. Forensic Odontology 644
20. STOMACH 426 • Multiple Choice Questions 630
21. Operative Surgery 426 • Fill in the Blanks 638
22. Neurological Injuries 434 • Viva-Voce Questions for Practical Examination 640
23. Fractures of Bone 444 • Additional Information 644
24. Advanced Methods of Surgery and Radiology 460 • Important Classifications 652
1
sECTION

General Medicine

1. Introduction 9. Nutrition and Metabolic Defects


2. Diseases of Gastrointestinal Tract 10. Diseases of Nervous System
3. Diseases of Liver 11. Tropical and Infectious Diseases
4. Diseases of Cardiovascular System 12. Miscellaneous
5. Diseases of Respiratory System Multiple Choice Questions as per DCI and Examina-
6. Diseases of Renal System tion Papers of Various Universities

7. Diseases of Blood Viva-voce Questions for Practical Examination

8. Diseases of Endocrine System Additional Information


General Medicine
♦♦ Miscellaneous
1. Introduction • Hereditary
• Idiopathic
Q.1. Enumerate the causes of clubbing. • Unilateral: Pancoast tumor, subclavian and innominate
 (Mar 2001, 5 Marks) (Mar 1998, 5 Marks) artery aneurysm
Or • Unidigital: Traumatic or tophi deposit in gout
• Only in upper limbs in heroin addicts due to chronic
Enumerate the causes of clubbing of fingers.
obstructive phlebitis.
 (Feb 1999, 4 Marks)
Or Grades of clubbing
Write important cause of digital clubbing. ♦♦ Grade I: Softening of nail bed because of hypertrophy of
 (Mar 2011, 2 Marks) tissue at that site.
Ans. Clubbing is a bulbous enlargement of soft part of terminal ♦♦ Grade II: In addition to grade I changes, there is
phalanges with both transverse and longitudinal curving obliteration of angle between nail base and adjacent skin
of nails. of the finger.
♦♦ Grade III: In addition to grade II changes, nail itself
loses its longitudinal ridges, becomes convex from above
downwards and from side to side. The nails assume
shape of "parrot’s beak" or terminal segment may become
bulbous like a "drum stick".
♦♦ Grade IV: Finger changes are associated with hypertrophic
pulmonary osteoarthropathy.
Q. 2. Write short note on cyanosis. (Feb 2013, 5 Marks)
 (Mar 1996, 7.5 Marks) (Apr 2015, 3 Marks)
Or
Write short answer on cyanosis. (Apr 2018, 3 Marks)
Ans. Cyanosis is a bluish discoloration of the skin and mucus
membrane due to reduced hemoglobin (more than 5
mg%) in blood.
Fig. 1: Clubbing
Type of Cyanosis
Causes of clubbing
Generally, there are four types of cyanosis:
♦♦ Pulmonary disorders i. Central cyanosis
• Suppuration of lung ii. Peripheral cyanosis
–– Bronchiectasis iii. Cyanosis due to abnormal pigments
–– Lung abscess iv. Mixed cyanosis.
–– Suppurative pneumonia
• Tumors of lung Central Cyanosis
–– Mesothelioma
–– Primary lung cancer ♦♦ It occur because of poor oxygenation of blood in lungs
–– Metastatic lung cancer due to interference of exchange of gases, i.e. oxygen and
♦♦ Cardiac disorders carbon dioxide in respiratory failure or pulmonary edema.
• Cyanotic congenital heart diseases ♦♦ Central cyanosis is also visible in some congenital heart
• Subacute bacterial endocarditis diseases where deoxygenated blood from right side mixes
• Atrial myxoma to the oxygenated blood from left side. This brings down
♦♦ Disorders of gastrointestinal system and liver oxygen saturation of blood.
• Inflammatory bowel disease ♦♦ Central cyanosis is visible at under surface of tongue and
–– Regional ileitis mucous membrane of oral cavity and palate.
–– Ulcerative colitis
Peripheral Cyanosis
–– Malabsorption syndrome
• Cirrhosis of liver ♦♦ It occurs because of removal of oxygen from the blood
• Malignancy of liver when circulation is slow due to congestive cardiac failure
♦♦ Disorders of endocrine system or due to shock causing vasoconstriction.
• Myxedema ♦♦ This can also occur in healthy people when extremities
• Thyroid acropachy are very cold.
• Acromegaly ♦♦ It is visible in lip, nail, tip of nose, lobule of ear.
4   Mastering the BDS IIIrd Year  (Last 25 Years Solved Questions)

Cyanosis due to Abnormal Pigments Write important causes of central cyanosis.


 (Mar 2011, 2 Marks)
♦♦ Normal hemoglobin has iron in ferrous form. In Ans. Following are the causes of central cyanosis:
methemoglobinemia, iron is in the ferric form designated
♦♦ Pulmonary causes
as methemoglobin. Several substances like nitrite
• Lobar pneumonia
ingestion, sulfonamide or aniline dyes oxidize hemoglobin
• High altitudes
to methemoglobin, but this is immediately reduced back to
• Pneumothorax
hemoglobin by methemoglobin reductase I or diaphorase
• Multiple small pulmonary thromboembolism
I. If there is deficiency of diaphorase I, methemoglobin
• Chronic obstructive pulmonary disease
circulates in blood, causing cyanosis.
• Respiratory failure
♦♦ Sulfhemoglobin is an abnormal sulphur containing
• Severe acute asthma
substance, which is not normally present, but is formed by
♦♦ Cardiovascular causes
toxic action of drugs and chemicals like sulphonamides,
• Cyanotic heart disease
phenacetin and acetanilide. Sulfhemoglobin forms an
• Acute pulmonary edema
irreversible change in the hemoglobin pigment that has
• Cor pulmonale
no capacity to carry oxygen and causes cyanosis.
• Arterio venous fistula
Mixed Cyanosis ♦♦ Abnormal hepato pulmonary syndrome

Due to combination of both the factors, e.g. cor pulmonale due Q.4. Enumerate the causes of cyanosis.
to pulmonary emphysema.  (Mar 1998, 5 Marks)
Ans. Refer to Ans 2 of same chapter.
Causes of Cyanosis
Q.5. Describe briefly pulsus paradoxus.
♦♦ Central cyanosis  (Feb 1999, 3 Marks)
• Pulmonary causes Ans. The term pulsus paradoxus is used to describe
–– Lobar pneumonia dramatically fall in blood pressure during inspiration,
–– High altitudes i.e. characteristic of *tamponade, pericardial constriction
–– Pneumothorax and severe airway obstruction.
–– Multiple small pulmonary thromboembolism • When the systolic blood pressure falls less than
–– Chronic obstructive pulmonary disease 10 mm, the pulse is referred to pulsus paradoxus.
–– Respiratory failure • Pulsus paradoxus is the *exaggeration of the normal
–– Severe acute asthma phenomenon.
• Cardiovascular causes
–– Cyanotic heart disease Mechanism
–– Acute pulmonary edema
Inspiration increases venous return to right side of heart
–– Cor pulmonale
–– Arterio venous fistula
• Abnormal hepato pulmonary syndrome Pooling of blood in pulmonary vasculature as the
♦♦ Peripheral cyanosis result of lung expansion and more intrathoracic
• Congestive heart failure pressure during active phase of respiration
• Exposure to cold
• Due to arterial obstruction Decrease in return of blood to left atrium and left ventricle
• Due to venous obstruction and there is fall in left ventricular output
♦♦ Cyanosis due to abnormal pigments
• Methemoglobin formation due to ingestion
sulphonamide and aniline dye Decrease in blood pressure less than 10 mm
• Sulfhemoglobin formation due to sulphonamide, (Pulsus paradoxus)
phenacetin
Causes
♦♦ Mixed cyanosis
• Acute left ventricular failure ♦♦ Superior vena cava obstruction
• Mitral stenosis ♦♦ Lung conditions
Q.3. Enumerate causes of central cyanosis. • Asthma
 (Sep 1999, 5 Marks) • Emphysema
• Airway obstruction
Or

Q5. *Tamponade= Compression of the heart by an accumulation of fluid in the pericardial sac.    *Exaggeration= Greater than it really is
Section 1:  General Medicine  5

♦♦ Cardiac condition Q.7. How will you differentiate arterial and venous pulse?
• Pericardial effusion  (Mar 2008, 2 Marks)
• Constrictive pericarditis Ans.
• Severe congestive cardiac failure
Arterial pulse Venous pulse
Q.6. Describe briefly water hammer pulse.
• Normal pulse has a small • Normal venous pulse consists
 (Sep 2009, 4 Marks) (Sep 1999, 3 Marks) *anacrotic wave on upstroke, of three positive waves and
Ans. It is also called as Corrigan pulse. which is not felt. This is two negative waves
♦♦ Water hammer pulse is a large bounding pulse with followed by percussion wave
increased stroke volume of left ventricle and decrease which is felt by palpating finger
in the peripheral resistance, leading to wide pulse • Arterial pulse is better felt than • Venous pulse is better seen
pressure. seen in appearance than felt
♦♦ The pulse strikes palpating finger with rapid, forceful jerk • Pressure below the angle of • Pressure below the angle of
mandible has no change on mandible obliterates the wave
and quickly disappears.
the wave
♦♦ It is best felt in radial artery with patient’s arm elevated.
♦♦ It is described as having a water hammer quality because • Changes with respiration • Changes with the respiration
and changes on position are and position are present
of its sudden impact and collapsing quality because it falls
absent
away so rapidly.
• It has no effect of posture • It disappears when patient sits
♦♦ The collapsing pulse caused by artery suddenly emptying
up on bed
as some of the blood flow from aorta to ventricle.
• There is no effect of abdominal • Pressure over the liver dis-
Causes compression tends the vein

♦♦ Physiological
• Fever Q.8. Enumerate the causes of hematemesis.
• Chronic alcoholism  (Sep 1999, 4 Marks)
• Pregnancy Ans. Rapid loss of blood from a lesion in esophagus, stomach
♦♦ High output states or syndrome or duodenum above the level of ampulla of Vater will
• Anemia result in vomiting of blood.
• Beri beri
Causes
• Cor pulmonale
• Liver cirrhosis ♦♦ Common causes
• Paget’s disease • Duodenal ulcer
• Arteriovenous fistula • Esophagitis
• Thyrotoxicosis • Gastric erosion
♦♦ Cardiac lesions • Varices
• Aortic regurgitation • Gastric ulcer.
• Rupture of sinus of Valsalva into heart chamber ♦♦ Less common
• Patent ductus arteriosus • Carcinoma stomach
• Aortopulmonary window • Bleeding diathesis
• Bradycardia • Aortic aneurysm
• Systolic hypertension ♦♦ Rare cases
• Acute pancreatitis
• Angiomas
• Telangiectasia
• COPD
• Polycythemia vera
• Hypoparathyroidism.
Q.9. Write short note on hematemesis.(Mar 2009, 5 Marks)
Ans. For definition and causes refer to Ans 8 of same chapter.

Clinical Features
Fig. 2:  Water hammer pulse
♦♦ Patient presents with vomiting of blood or complaint of
passing the *tarry stools.

Q9. *Tarry= Smeared with tar


*Giddiness= Dizziness
*Syncope= Transient loss of consciousness, accompanied by inability to maintain an upright posture
6   Mastering the BDS IIIrd Year  (Last 25 Years Solved Questions)

♦♦ Many patients with coffee ground vomiting are present. ♦♦ Causes for pseudohemoptysis
♦♦ In cases of severe hemorrhage there may be fresh rectal • Trauma of mouth, pharynx and larynx
bleeding or *giddiness and *syncope due to sudden hypo­ • Tuberculosis, syphilis or pyogenic infection of mouth,
volemia. pharynx and larynx.
♦♦ Hematemesis is mixed with food particles. • Malignancy of mouth, pharynx and larynx
• Bleeding spongy gums in scurvy.
Management
Q.11. How will you differentiate hemoptysis from hemate-
General
mesis? (Apr 1999, 5 Marks)
♦♦ Put the patient to the bed. Ans.
♦♦ Arrange fresh blood transfusion.
♦♦ Maintain nutrition and hydration. Hemoptysis Hematemesis
♦♦ Pass a Ryle’s tube and do constant suction. In cases of sus-
• Symptoms and sign are of • Symptoms and signs are of
pected peptic ulcer an antacid in gel form is given too early. pulmonary and cardiac disease gastric or abdominal disease
Specific • Blood is coughed up • Blood is vomited
♦♦ Treat underlying condition appropriately. • Blood is bright red, frothy and • Blood is coffee ground, mixed
♦♦ Once the crisis is over and bleeding subsides, than treatment mixed with the sputum with food particles
is to be planned according to the basic disease. Drug-
• Blood is relatively in small • Blood is in large amount
induced hematemesis shall require symptomatic relief.
amount
Q.10. Enumerate the causes of hemoptysis. 
• Reaction is alkaline • Reaction is acidic
 (Feb 2006, 2.5 Marks)
Ans. Hemoptysis is defined as coughing out of the blood • Stool become rusty next day • Stool is tarry next day
which includes stained sputum.
Q.12. Outline the investigation and management of hemop-
Causes tysis. (Mar 2000, 5 Marks)
♦♦ Causes for true hemoptysis Ans. Hemoptysis is defined as expectoration of blood from
• Cardiac respiratory tract, spectrum varies from blood streak of
–– Mitral stenosis sputum to cough up or large amount of pure blood.
–– Aneurysm of aorta
–– Left ventricular failure and primary pulmonary Investigation
hypertension. ♦♦ Hemodynamic resuscitation and bronchoscopy is done.
• Respiratory ♦♦ Chest radiograph for TB, pneumonia, tumor, pulmonary
–– Pneumonia infarction.
–– Tuberculosis ♦♦ Full blood count and hematological tests.
–– Bronchogenic carcinoma and adenoma ♦♦ Bronchoscopy to exclude central bronchial carcinoma and
–– Pulmonary embolism to provide tissue diagnosis for the suspected.
–– Lung abscess ♦♦ CT scan: For peripheral lesion investigation which are seen
–– Bronchiectasis and other infections of lung and on chest radiograph.
bronchi
Management
–– Trauma to the airways and lung
–– AV malformations. ♦♦ Establishing a diagnosis is a first priority.
• Immunological ♦♦ When hemoptysis is maintained, adequate gas exchange
–– Goodpasture’s syndrome preventing blood from spleen into unaffected area of lung
–– Wegener’s granulomatosis and avoiding asphyxiation are the highest priority.
–– Polyarteritis nodosa. ♦♦ Keeping the patient at rest and partially suppressing cough
• Bleeding disorders are helpful to subside bleeding.
–– Thrombocytopenia ♦♦ If origin of blood is known and is limited to one lung,
–– Purpura bleeding lung should be placed in the dependent position
–– Agranulocytosis so that blood is not aspirated to the affected lung.
–– Leukemia ♦♦ Endotracheal intubation and mechanical intubation are
–– Hemophilia and anticoagulant therapy. necessary to maintain the airways.
• Iatrogenic ♦♦ Balloon catheters and inflating balloon at the bleeding site
–– Following bronchoscopy are helpful in control of the bleeding.
–– Lung biopsy ♦♦ Laser phototherapy, embolotherapy and surgical resection
–– Endotracheal intubation of involved area of lung are the other methods. Surgical
–– Anti coagulant therapy resection is done in life-threatening hemoptysis.
Section 1:  General Medicine  7

Q.13. Enumerate the causes of malena. (Mar 1998, 5 Marks) ♦♦ Imaging techniques:
Ans. Malena is defined as the passage of dark colored blood • X-ray chest: In cases with prolonged fever when initial
in stool. X-ray is normal, a second X-ray must be taken after
three weeks to rule out military tuberculosis.
Causes • Ultrasound: Excellent imaging is done in thin
♦♦ Peptic ulcer individuals and poor imaging in obese individuals,
♦♦ Portal hypertension SOL in hepatobiliary tree of more than 1 cm and
♦♦ Typhoid fever endocarditis vegetation of more than 2 mm can be
♦♦ Malignant GI tract detected.
♦♦ Ulcerative colitis • CT scan: Provide excellent imaging in obese patient.
♦♦ Bleeding diathesis, i.e. purpura, hemophilia, leukemia. SOL in liver is more than 1 cm and CNS lesion is more
than 0.2 cm.
Q.14. Enumerate common causes of fever. 
• Radionuclide scans: 99mTc-sulphur colloid is used for
 (Sep 2008, 2.5 Marks) (Mar 1998, 5 Marks)
scanning liver and spleen. 111Indium labeled leukocytes
Ans. Following are the causes of fever: are used for detection of intra-abdominal mass.
♦♦ Infections: Bacterial, viral, rickettsial, fungal, parasitic, etc.
♦♦ Neoplasms: Fever may be present with any neoplasm but Q.16. Write short note on pedal edema. (Sep 2005, 3 Marks)
commonly with hypernephroma, lymphoproliferative Ans. Pedal edema is defined as swelling of feet and ankle
malignancies, carcinoma of pancreas, lung and bone and caused by collection of fluid in the tissues and is a
hepatoma. possible sign of congestive heart failure.
♦♦ Vascular: Acute myocardial infarction, pulmonary
Causes
embolism, pontine hemorrhage, etc.
♦♦ Traumatic: Crush injury. Trauma.
♦♦ Immunological:
Clinical Features
• Collagen disease, SLE, rheumatoid arthritis.
• Drug fever ♦♦ Swelling appears on the feet and ankle.
• Serum sickness ♦♦ Pitting type of edema is present.
♦♦ Endocrine: Thyrotoxicosis, Addison's disease. ♦♦ Obstruction of inferior vena cava.
♦♦ Metabolic: Gout, porphyria, acidosis, dehydration
Treatment
♦♦ Hematological: Acute hemolytic crisis
♦♦ Physical agents: Heat stroke, radiation sickness. ♦♦ Sodium restriction is done.
♦♦ Miscellaneous: Factitious fever, habitual hyperpyrexia, ♦♦ Diuretics should be used, i.e. spironolactone.
cyclic neutropenia ♦♦ Management of underlying disorder.
♦♦ ACE inhibitors are given.
Q.15. How will you investigate a case of prolong fever?
♦♦ Leg elevation of patient.
 (Sep 2009, 5 Marks) (Feb/Mar 2004, 5 Marks)
Ans. Following are the investigations which are carried out as an Q.17. Enumerate the causes of hematuria.
important investigation in case of a prolong fever:  (Sep 2008, 2.5 Marks)
♦♦ ESR platelet correlation: If ESR is more than 100 mm / hr Ans. Hematuria is defined as presence of blood in urine.
with thrombocytosis, following diseases can be think off,
Causes
i.e.
• Tuberculosis ♦♦ Renal
• Malignancy • Glomerulonephritis
• Connective tissue diseases. –– Primary
If ESR is less than 100 mm / hr with thrombocytosis, - Mesangial proliferative
viral infection can be suspected. - Mesangiocapillary
♦♦ Assessment of alkaline phosphatase levels: If alkaline - Berger’s disease
phosphatase levels are higher following infections are –– Secondary
suspected, i.e. biliary tract infections, alcoholic hepatitis, - Systemic lupus erythematosus
primary and secondaries of liver, hypernephroma, - Polyarthritis nodosa
lymphoma, military tuberculosis, cytomegalovirus - Infective endocarditis
infection. –– Others
♦♦ Serological tests: They are helpful in assessing enteric fever, - Alport’s syndrome
hepatitis, CMV infection, tularemia, secondary syphilis, - Fabry's disease
brucellosis, Q fever, amoebiasis, HIV. - Benign familial hematuria
8   Mastering the BDS IIIrd Year  (Last 25 Years Solved Questions)

• Interstitial disease Causes of Clubbing


–– Acute pyelonephritis ♦♦ Pulmonary disorders
–– Papillary necrosis • Suppuration of lung
–– Neoplasms –– Bronchiectasis
• Cystic disease –– Lung abscess
–– Adult polycystic disease –– Suppurative pneumonia
–– Medullary cystic disease • Tumors of lung
• Renal stones –– Mesothelioma
• Trauma to kidneys –– Primary lung cancer
♦♦ Ureter –– Metastatic lung cancer
• Stone ♦♦ Cardiac disorders
• Neoplasm • Cyanotic congenital heart diseases
♦♦ Urinary bladder • Subacute bacterial endocarditis
• Neoplasm • Atrial myxoma
• Cystitis ♦♦ Disorders of gastrointestinal system and liver
• Stone • Inflammatory bowel disease
• Trauma or catheter induced –– Regional ileitis
• Schistosomiasis –– Ulcerative colitis
♦♦ Prostate –– Malabsorption syndrome
• Prostatitis • Cirrhosis of liver
• Benign enlargement of prostate • Malignancy of liver
• Neoplasm ♦♦ Disorders of endocrine system
♦♦ Urethra • Myxedema
• Injury • Thyroid acropachy
• Urethritis • Acromegaly
♦♦ Disorders of hemostasis ♦♦ Miscellaneous
• Bleeding or coagulation disorders • Hereditary
• Anticoagulants • Idiopathic
♦♦ Systemic diseases • Unilateral: Pancoast tumor, subclavian and innominate
• Diabetes artery aneurysm
• Amyloidosis • Unidigital: Traumatic or tophi deposit in gout
• Collagen disease • Only in upper limbs in heroin addicts due to chronic
• Disseminated intravascular coagulation obstructive phlebitis.
Q.18. Write short note on bronchial breathing.
 (Sep 2009, 4 Marks) Grading
Ans. Bronchial breathing is blowing or hollow in character, ♦♦ Grade I: Softening of nail bed due to the hypertrophy of
inspiratory phase equals expiratory phase and there is tissue at that particular site.
pause between the two. ♦♦ Grade II: In addition to grade I changes there is obliteration
• Bronchial breathing may be low pitched (Cavernous) of the angle of nail bed
medium pitched or high pitched (tubular). Low ♦♦ Grade III: In addition to grade II changes there is swelling
pitched bronchial breathing is heard over moderately of the subcutaneous tissues over the base of the nail causing
large cavities in the lung and in a case of open the overlying skin to become tense, shiny and wet and
pneumothorax. increasing the curvature of the nail, resulting in parrot
• High pitched or tubular breathing is heard where beak or drumstick appearance.
consolidation of lung has occurred round small ♦♦ Grade IV: Swelling of the fingers in all dimensions
sized bronchial tubes as in consolidation of lung, associated with hypertrophic pulmonary osteoarthropathy
lobar pneumonia, malignant disease, pulmonary causing pain and swelling of the hand, wrist, etc. and
infarction and pleural effusion. radiographic evidence of subperiosteal new bone
• Another variety of bronchial breathing is amphoric formation.
which is like blowing across a bottle and has a Q.20. Write short note on drug fever. (Dec 2010, 5 Marks)
distinct ‘echo like’ quality. It is heard over a large Ans. Drug fever is a prolonged fever and any belong to any
cavity with smooth wall or in a case of pneumothorax febrile pattern.
in direct contact with a bronchus.
• In drug fever there is relative bradycardia and
Q.19. Write short note on clubbing. hypotension.
 (Dec 2010, 5 Marks) (Dec 2015, 3 Marks) • Pruritus, skin rash and arthralgia may occur.
Ans. Clubbing is an enlargement of distal segment of fingers • It begins 1 to 3 weeks after the drug is started and
and toes due to increase in soft tissue. persists of 2 to 3 days after drug is withdrawn.
Section 1:  General Medicine  9

• Eosinophilia may be present. • Paralysis of 9th and 10th cranial nerve nuclei
• Almost all the drugs may lead to drug fever. –– Systemic sclerosis
• Drugs which commonly leads to drug fever –– Polymyelitis.
are sulphonamide, penicillin, iodide, anti- • Esophageal muscle weakness
tubercular drugs, methyldopa, anti-convulsants, –– Myopathy.
propylthiouracil. • Neuromuscular paralysis
Q.21. Write important causes of pitting edema. –– Myasthenia gravis.
(Mar 2011, 2 Marks) Investigations
Ans. Following are the causes of pitting edema: ♦ Complete hemogram is done to check for the anemia
• Ingestion of excessive salt ♦ Chest X-ray should be done to check for the tuberculosis,
• Due to steroids cardiomegaly, mediastinal enlargement
• Premenstrual ♦ Endoscopy of esophagus helps in detection of lesion and
• Due to portal obstruction finding its cause
• Due to obstruction of inferior vena cava ♦ Barium meal examination is carried out purely for
• In beri beri localization of lesion in esophagus
• Anemia and hypoproteinemia ♦ Esophageal manometry is carried out for assessing motility
• Epidemic dropsy disorders of esophagus.
• Pregnancy
• Miscellaneous: Dermatomyositis, Raynaud’s
phenomenon and old age
• If pitting remains for more than a minute most likely 2. Diseases of
cause is congestion
• If pitting remains for 40 seconds it is caused by
Gastrointestinal Tract
hypoalbuminemia
Q.1. Write short note on stomatitis.
Q.22. Write short note on causes and investigations of  (Mar 2000, 5 Marks) (Mar 2006, 5 Marks)
dysphagia. (Mar 2013, 3 Marks)
 (Sep 2007, 2 Marks) (Apr 2010, 5 Marks)
Ans. Dysphagia is difficulty in swallowing.  (June 2010, 5 Marks) (Aug 2012, 5 Marks)
Causes Or
♦♦ Due to narrowing of esophagus Write notes on stomatitis. (Aug 2011, 10 Marks)
• Intrinsic, i.e. obstruction inside esophagus Ans. Stomatitis is the inflammation of mouth and is caused
–– Esophageal stricture by bacterial, viral and fungal infections in persons with
–– Esophageal ulceration poor oral hygiene or in blood dyscrasias.
–– Congenital atresia of esophagus
–– Plummer vinson syndrome Causes of Stomatitis
–– Tumors either benign or malignant Local Causes
–– Tonsillitis
♦♦ Poor oral hygiene
–– Stomatitis
♦♦ Excessive use of tobacco
–– Glossitis
♦♦ Alcohol and spices
–– Esophagitis
♦♦ Use of broad spectrum antibiotics
–– Pharyngitis
♦♦ Drugs such as iodine or gold.
• Extrinsic, i.e. obstruction outside esophagus
–– Aortic aneurysm General causes
–– Retropharyngeal mass
–– Mitral stenosis which lead to left atrial enlargement The main general causes are the infectious diseases. There are
–– Thyroid gland enlargement which compresses various types of infective stomatitis:
esophagus. ♦♦ Bacterial, e.g. streptococcal stomatitis and Vincent’s
♦♦ Motor Dysphagia: stomatitis
• Paralysis of esophageal sphincter ♦♦ Viral, e.g. herpes simplex and herpes zoster
–– Esophageal spasm ♦♦ Fungal, e.g. candidiasis and actinomycosis
–– Cardiac achalasia. ♦♦ Recurrent aphthous stomatitis

Q1. *Excoriation= An abrasion of skin or the surface of other organs by scratching, traumatic injury, burn or other causes.
10   Mastering the BDS IIIrd Year  (Last 25 Years Solved Questions)

♦♦ Mucocutaneous diseases, e.g. Lichen planus, pemphigus Type A Gastritis (Less Common)
vulgaris, lupus erythematous, etc.
♦ It involves body of stomach and spars antrum.
♦♦ Miscellaneous, e.g. diabetes, uremia and drug toxicity.
♦ It is caused during autoimmune disorders like type I
Clinical Features diabetes mellitus, Sjögrens syndrome, Graves disease,
Hashimoto disease, myasthenia gravis, etc.
♦ Lip, tongue and gums are inflamed, swollen and painful.
♦ It is caused due to autoimmune activity against parietal
♦ Tongue is furred and foul smell is present.
cells.
♦ Sometimes ulceration of mucus membrane is present when
♦♦ Parietal cell antibodies can be detected in serum.
person is suffering from infectious stomatitis.
♦♦ In severe cases parietal cell atrophy leads to deficiency of
♦ Patient feels pain and difficulty in opening the mouth.
intrinsic factor which leads to pernicious anemia.
♦ There was an *excoriation and redness of mucus membrane
♦♦ Disease is asymptomatic and long-term complication is
of oral cavity.
gastric carcinoma.
Treatment Treatment
♦ If drug is the causative factor discontinuation of the drug ♦ In severe cases corticosteroids are administered.
is done. ♦ In mild cases parenteral iron should be administered.
♦ It allergen is the causative factor remove allergen.
♦ Antihistaminic drugs such as cetirizine to be given to the Type B Gastritis
patient. ♦ This is more common form of gastritis and involves antrum
♦ Topical corticosteroid application should be done. of stomach.
Triamcinolone acetate is effective. ♦ Usual cause is gram-negative bacteria H. pylori.
♦ Tetracycline mouthwash should be given which should be ♦ The condition is precursor of peptic ulcer.
used four times a day for 5 to 7 days. ♦ There is possibility of gastric carcinoma.
♦ Nutritional supplements to be given such as vitamin B12, Diagnosis
iron, folic acid.
♦♦ Gastric acid study, i.e. achlorhydria
Q.2. Enumerate the causes of stomatitis. ♦♦ Hemoglobin decreases
 (Mar 1998, 5 Marks) (Sep 1998, 5 Marks) ♦♦ Serum gastrin increases.
Ans. Refer to Ans 1 of same chapter.
Management
Q.3. Write short note on chronic gastritis. ♦♦ Anti- H. pylori treatment
 (Sep 1999, 5 Marks) • Triple drug therapy: Proton pump inhibitor or ranitidine
Ans. When the acute gastritis remain for the longer time and is 400 mg BD + Bismuth subcitrate + Amoxicillin 1 gm or
not treated, it becomes chronic and is known as chronic clarithromycin 500 mg or metronidazole 500 mg BD
gastritis. • Quadruple therapy: Omeprazole 10 mg BD + Tetracycline
500 mg QID + Bismuth subcitrate QID + Metronidazole
Etiology 500 mg TDS.
♦♦ Repeated injury to gastric mucosa by tea, coffee, alcohol, In both the cases 14 days course is preferred.
spices ♦♦ Parenteral vitamin B12 is administered.
♦♦ Infection from throat, teeth, gums and sinuses Q.4. Describe the management of acute diarrhea.
♦♦ NSAIDs  (Sep 1998, 5 Marks)
♦♦ Autoimmune pathology Ans.
♦♦ Very hot beverage
♦♦ Gastrectomy. General Management
♦♦ Rest, maintenance of fluid and electrolyte balance.
Types of Gastritis ♦♦ ORS should be given in all children as early as possible.
♦♦ Superficial ♦♦ Patient with constant vomiting or moderate to severe
♦♦ Atrophic dehydration require IV fluid.
♦♦ Hypertrophic ♦♦ Ringer lactate is ideal, normal saline may be given.
♦♦ Infectious Antimicrobial / Antidiarrheal Treatment
♦♦ Eosinophilic.
♦♦ Ciprofloxacin 500 mg BD for three days or nalidixic acid
There are mainly two types of chronic gastritis: 1 gm 6 hourly for 5 to 7 days.
A. Type A gastritis ♦♦ It may be combined with the tinidazole 300 mg BD for 6
B. Type B gastritis. days.
Section 1:  General Medicine  11

Antimotility Agent Clinical Features


This should be used in children before 5 years of age. Symptoms
♦♦ Loperamide or diphenoxylate atropine. ♦♦ Patient has diarrhea and pass 10–15 stools per day.
♦♦ Codeine could be used. ♦♦ Presence of abdominal pain.
♦♦ Sodium and water conserving agent, i.e. racecadotril is ♦♦ Stools consist of blood and mucus.
the newer drug safely given in the children and adult. It ♦♦ Flatulence is present.
reduces the loss of sodium and water in the stool. ♦♦ Fever is present between 38 and 40 degrees with rigors.
Q.5. Outline the management of amoebiasis. Signs
(Mar 1997, 7.5 Marks)
♦♦ Palpation of abdomen show diffuse tenderness.
Ans. Disease is caused by infection by protozoan parasite.
♦♦ Chronic cases show thickened tender sigmoid colon.
Entamoeba histolytica which get lodged in the large ♦♦ Amoeba is felt as a sausage shaped mass in right iliac fossa.
intestine causing dysentery and may spread to the liver ♦♦ Tender hepatomegaly is present.
and other organs.
Management
Management of Amoebiasis
Investigations
Investigations ♦♦ Stool examination: Show various forms of E. histolytica
• Stool examination: Shows various forms of E. histolytica ♦♦ Serological test
• Serological test ♦♦ Sigmoidoscopy for ulcer.
• Sigmoidoscopy for ulcer. ♦♦ Immunofluorescence test for autoantibodies
• Immunofluorescence test for autoantibodies Treatment
♦♦ Nitroimidazole classes of antimicrobial agents are main
Treatment of Amoebiasis
drugs of treatment.
♦♦ Nitroimidazole classes of antimicrobial agents are main • Metronidazole 400 to 800 mg TDS 5 to 7 days.
drugs of treatment. • Tinidazole 2 gm as single or divided three days dose.
• Metronidazole 400 to 800 mg TDS 5 to 7 days. • Secnidazole 2 gm as single dose.
• Tinidazole 2 gm as single or divided three days dose. • Diloxanide furoate: It should be added because it
• Secnidazole 2 gm as single dose. has effect on cyst. It is given 500 mg TDS for 10 days.
• Diloxanide furoate: It should be added because it has Metronidazole has no effect on cyst.
effect on cyst. It is given 500 mg TDS for 10 days. ♦♦ General supportive measures, fluid and electrolyte
Metronidazole has no effect on cyst. imbalance correction is there.
♦♦ General supportive measures, fluid and electrolyte ♦♦ Diet given should be soft, liquid or semi-liquid.
imbalance correction is there. Q.7. Outline the management of Amoebic liver abscess. 
♦♦ Diet given should be soft, liquid or semi-liquid.  (Sep 1999, 4 Marks)
Q.6. Describe briefly amoebic dysentery.  Ans. Investigation
 (Sep 1998, 5 Marks) (Mar 1996, 5 Marks) • TLC and DLC: Leukocytosis is present.
• Stool study shows cyst.
Or
• Sigmoidoscopy: Ulcers are seen.
Write short note on amoebiasis. • USG.
 (Mar 2006, 5 Marks) (Feb 2006, 2.5 Marks) • Aspiration study.
Ans. Amoebic dysentery is caused by a protozoan called • Immunofluorescence test for autoantibodies
Entamoeba histolytica.
Management of Amoebic Liver Abscess / Hepatic
Entamoeba histolytica exist in two forms, i.e. trophozoite
Amoebiasis
and cystic stage. ♦♦ Early cases are responding well with metronidazole 800
Trophozoites are responsible for production for disease mg TID for 5 days or tinidazole 2 gm daily for three days.
and cysts are responsible for transmission of disease. ♦♦ Luminal amoebicide: Diloxanide furoate 500 mg 8 hourly for
Reservoir of infection: Man is the only known reservoir.
10 days should be given to determine luminal cyst.
♦♦ If abscess is large and not respond to chemo­therapy
Mode of transmission: Fecal-oral route, i.e. through

repeated aspiration under ultrasonic guidance is required.
contaminated food and water. ♦♦ Rupture of abscess into peritoneal cavity requires
Incubation period is two to four months or longer. immediate aspiration or surgical drainage.
12   Mastering the BDS IIIrd Year  (Last 25 Years Solved Questions)

Q.8. Write short note on malabsorption syndrome. ♦♦ General features like anemia, sore mouth, loss of weight,
 (May/June 2009, 5 Marks) (Mar 2009, 5 Marks) fatigue and lethargy.
 (Sep 2007, 2 Marks) (Mar 2000, 5 Marks) ♦♦ Bone pain may be present.
Or ♦♦ Skin changes like pellagra are present
♦♦ Patient also suffers from peripheral neuropathy, irritability
What is malabsorption syndrome, causes of malabsorp- and lack of confidence.
tion, clinical features and its management.
 (Nov 2011, 8 Marks) Investigations
Ans. Malabsorption syndrome comprises a large number of Tests are carried out to detect the nutritional deficiency. These
pathological condition in which there is disturbance of tests indicate the malabsorption of particular nutrient and not
processes by which nutrients are transferred from lumen its cause.
of intestine into circulation. ♦♦ Fecal fat stimulation: It confirms steatorrhea and fat
malabsorption. Sudan III stain may show an increase in
Etiology the stool fat. Quantitative estimation of fat in the stool
is more reliable and sensitive. A 72-hour stool collection
♦♦ Stomach:
while the patient is on a defined diet is used for fat
• Precipitate emptying after postgastrectomy dumping.
estimation. Excretion of more than l0 g fat per day suggests
• Lack of intrinsic factor.
fat malabsorption.
• Excess acid secretion in Zollinger-Ellison syndrome.
♦♦ Schilling test: This is useful in the diagnosis of cobalamin
♦♦ Pancreatic: Inadequate enzyme and bicarbonate secretion.
(B12) malabsorption and its causes like pernicious
♦♦ Biliary: Due to defective micelle formation.
anaemia, chronic pancreatitis, achlorhydria and bacterial
♦♦ Endocrine diseases
overgrowth. In this test radio-labelled cobalamin
♦♦ Parasitic or drug.
(l mg 68°C) should be given orally and its excretion
Various diseases along with their etiologies can cause
in urine is measured. 1 mg cobalamin is administered
malabsorption as:
intramuscularly to saturate hepatic binding sites so that
♦♦ Disorders of intraluminal digestion
all radiolabelled cobalamin is excreted in the urine. The
• Pancreatic enzyme deficiency in chronic pancreatitis,
test is abnormal, if less than l0% of the radio-labelled
cystic fibrosis and pancreatic carcinoma
cobalamin is excreted in the urine in 24 hours. This will
• Disturbances of gastric function after gastroenterostomy
help in differentiating the various defects responsible for
and partial gastrectomy
malabsorption of cobalamin.
• Deficiency of bile acids in Crohn’s disease, resection
♦♦ D-xylose test: It detects carbohydrate malabsorption. 25 g
of terminal ileum, stagnant loop syndrome or blind
of D-xylose is given orally and its excretion is measured in
loop syndrome.
urine. Excretion of less than 4.5 g in 5 hours is suggestive
♦♦ Disorders of transport in the intestinal mucosal cell
of malabsorption.
• With histologically abnormal mucosa (infiltration,
♦♦ Upper gastrointestinal endoscopy and biopsy of small intestinal
inflammation or infection of mucosa) in coeliac
mucosa: It is essential for the diagnosis of conditions like
disease, tropical sprue, lymphoma, whipple’s disease,
tropical sprue, celiac sprue, Whipple’s disease, and Crohn’s
giardiasis and radiation enteritis
disease.
• With histologically normal mucosa (genetic diseases)
♦♦ Barium meal contrast radiography: Radiological assessment
in lactase deficiency, pernicious anemia
of the small intestine with barium contrast is helpful in
• Disorders of transport from mucosal cell in abdominal
evaluation of structural abnormalities in Crohn’s disease,
lymphoma, tuberculosis, telangiectasia of mesenteric
diverticulae and strictures.
lymphatics, A beta lipoproteinemia,
♦♦ Pancreatic exocrine functions: They should be carried out in
• Impaired nutrient uptake in lymphatic obstruction,
patients with steatorrhea.
cardiac heart failure, and pericarditis
♦♦ Serological studies: In some of the conditions such as celiac
• Miscellaneous: Diabetes mellitus, hyperthyroidism,
sprue and pernicious anemia autoantibodies are detected.
hyperparathyroidism
♦♦ Small intestinal biopsy (duodenal or jejunal): It is carried out
for conformational diagnosis.
Clinical Features
♦♦ *Steatorrhea is presenting symptom. Management
♦♦ Diarrhea or abdominal discomfort. ♦♦ Diet: High protein and low fat diet is taken.
♦♦ Nutritional deficiencies, i.e. deficiency of vitamin A, D, ♦♦ Digestants: Pancreatic enzyme preparations are
B12 and K administered after meals.

Q8. *Steatorrhea= Defective absorption of fat


Section 1:  General Medicine  13

♦♦ Treatment of anemia: All three hematinics vitamin B12, folic Management


acid and iron is given. ♦♦ Oral rehydration therapy is given.
♦♦ Vitamin supplements: Vitamin B complex and vitamin D ♦♦ IV replacement of water and electrolyte loss is necessary.
are given. ♦♦ Ciprofloxacin 500 mg 12 hourly for two days is given.
♦♦ Treatment of diarrhea: Codeine and loperamide are ♦♦ Antidiarrheal medication is given.
administered.
♦♦ Steroids: Prolong therapy with prednisolone is given. Q.11. Differentiate between gastric ulcer and duodenal
♦♦ Elimination of bacterial overgrowth: Tetracycline 250 mg TDS ulcer. (Feb 2006, 3 Marks)
for one week. Ans.
Characteristic Gastric ulcer Duodenal ulcer
Q.9. Enumerate causes of acute diarrhea. 
 (Feb 2006, 2.5 Marks) Age It occur in It occur in individuals
individuals more from age range of 20
Or than 40 years to 50 years
Enumerate causes of diarrhea. (Jan 2012, 5 Marks) Sex It is equal in both It is more in females
Ans. sexes
Course of illness It is less remittent It is more remittent
I. Acute Diarrhea
Duration of episode Duration is long Duration is short
♦♦ Infective diarrhoea of pain
• Bacterial: The bacteria which causes acute diarrhea are Nature of pain Diffuse Sharp pointing pain
Shigella, Salmonella, E.coli, etc. present in epigastrium
• Viral: The viruses which results in diarrhea are Occurrence of pain Pain occur after Pain occur on
parvovirus, Hawaii, etc. meal fastening
• Protozoal: The protozoans are Ehistolytica, giardia, etc. Hemorrhage More common Less common
• Toxic and systemic causes
Malena Less common More common
• Some drugs result in acute diarrhea, i.e. broad
spectrum antibiotics. Anorexia and nausea Present Less often
♦♦ Non-infective diarrhoea Hematemesis Present Less common
• Crohn’s disease
Heart burn Less common More common
• Irritable bowel syndrome
• Acute psychological stress Q.12. Discuss causes of acute gastritis. 
• Drugs such as amoxicillin, antacids, cholinergics, etc.  (Feb/Mar 2004, 5 Marks)
Ans. The causes of acute gastritis are:
II. Chronic Diarrhea
♦♦ NSAIDs and analgesics: They not only cause gastric erosion
♦♦ Ulcerative colitis by predisposing to peptic ulcer.
♦♦ HIV associated enteritis ♦♦ Antimitotic drugs
♦♦ Whipple’s disease ♦♦ Renal failure
♦♦ Pancreatic insufficiency ♦♦ H. pylori initial infection
♦♦ Zollinger-Ellison syndrome ♦♦ Other infections, i.e. streptococcal, viral, fungal, etc.
♦♦ Medullary carcinoma of thyroid ♦♦ Iron therapy
♦♦ Irritable bowel syndrome. ♦♦ Stress and burns
Q.10. Write short note on bacillary dysentery. ♦♦ Postoperative.
 (Mar 2000, 5 Marks) Q.13. Enumerate the causes of upper GI hemorrhage. 
Ans. Bacillary dysentery is mainly caused by Shigella.  (Sep 2010, 5 Marks)
Ans. The causes for upper GI hemorrhage are:
Etiology
♦♦ It occurs by contaminated food or flies. Esophageal Causes
♦♦ It is caused by unwashed hands after defecation. ♦♦ Esophageal varices
♦♦ It is also caused due to wars and natural calamities which ♦♦ Esophagitis
causes poor sanitation and crowding. ♦♦ Esophageal ulcers
♦♦ Esophageal cancers.
Clinical Features
♦♦ Diarrhea, colicky abdominal pain and tenesmus, i.e. spas- Gastric Causes
modic contraction of anal or bladder sphincter with pain. ♦♦ Gastric ulcer
♦♦ Fever, dehydration and weakness with tenderness over ♦♦ Gastric cancer
colon. ♦♦ Gastritis
♦♦ Arthritis may be seen. ♦♦ Gastric varices.
14   Mastering the BDS IIIrd Year  (Last 25 Years Solved Questions)

Duodenal Causes Q.15. Write short note on peptic ulcer. (Sep 2006, 5 Marks)
♦♦ Duodenal ulcer Or
♦♦ Vascular malformations Write short note on diagnosis and treatment of peptic
♦♦ *Hematobilia ulcer. (Nov 2014, 3 Marks)
♦♦ Bleeding from the pancreatic duct. Or
Write in detail about peptic ulcer and its management.
 (Mar 2007, 5 Marks)
Ans. Peptic ulcer is defined as mucosal ulceration near the
acid bearing regions of gastrointestinal tract. It is the
ulcer in duodenum and stomach.

Etiology
♦♦ Hereditary: Patient with blood group O has much incidence.
♦♦ H. pylori: Gram-negative bacteria are supposed to be main
cause accounting for 70% of gastric ulcer.
♦♦ NSAIDs: They lead to 30% of gastric ulcers. By depleting
mucosal prostaglandin levels aspirin and NSAIDs impairs
cytoprotection resulting in mucosal injury, erosion and
ulceration
♦♦ Smoking: It does not cause ulcer but more likely to cause
Fig. 4:  Upper GI hemorrhage complication and is responsible for nonhealing or delayed
healing.
Q.14. Outline the treatment of acute gastroenteritis.  ♦♦ Corticosteroids: They are responsible for silent perforation
 (Sep 2008, 2.5 Marks) (Mar 1996, 7.5 Marks) of ulcer.
Ans. Inflammation of stomach and intestinal tract that causes ♦♦ Acid-pepsin versus mucosal resistance: Cause of peptic
vomiting, diarrhea or both. ulceration is digestion of the mucosa with acid and pepsin
of gastric juice. Normal stomach is capable of resisting this
Treatment
digestion. So, the concept of peptic ulceration is acid plus
♦♦ Rehydration with liquid is keyword to avoid dehydration pepsin versus mucosal resistance. Factors which tilt this
and electrolytic imbalance. balance leads to the production of ulcers. The factors are:
♦♦ Antidiarrheal treatment is given, i.e. • Gastric hypersecretion.
• Antimicrobial agent • Severe ulceration occurs in Zollinger-Ellison syndrome,
–– Ciprofloxacin 500 mg BD for three days or which is characterized by very high acid secretion.
nalidixic acid 1 gm 6 hourly for 5 to 7 days. It • Acid secretion is more important in the etiology of
may be combined with the tinidazole 300 mg BD duodenal ulcer than in gastric ulcer.
for 6 days. ♦♦ Factors reducing mucosal resistance:
• Antimotility agent • Several drugs, particularly those used in rheumatoid
This should be used in children before 5 years of age. arthritis.
–– Loperamide or diphenoxylate atropine. • Aspirin is an important etiological factor in gastric
–– Codeine could be used. ulcer.
–– Sodium and water conserving agent, i.e. • The organism Helicobacter pylori
Racecadotril is the newer drug safely given in the • Reflux of bile and intestinal contents into stomach due
children and adult. It reduces the loss of sodium to poorly functioning pyloric sphincter.
and water in the stool. ♦♦ Other risk factors are smoking and alcohol consumption.
General Management Clinical Features
♦♦ Rest, maintenance of fluid and electrolyte balance. ♦♦ Patient presents with the recurrent abdominal pain which
♦♦ ORS should be given in all children as early as possible. consists of three characters, i.e. localization of epigastrium,
♦♦ Patient with constant vomiting or moderate to severe relationship to food and periodicity.
dehydration require IV fluid. ♦♦ Patient has epigastric pain. Pain is very sharply localized in
♦♦ Ringer lactate is ideal, normal saline may be given. the manner that the patient localize the site with one finger

Q13. *Hematobilia= Blood in the bile


Section 1:  General Medicine  15

only. This is also known as pointing sign. In its character –– H2 receptor antagonists: i.e. ranitidine 150 mg BD
the pain is burning. or 300 mg at night; Famotidine can be given 20 mg
♦♦ Hunger pain: As person remain empty stomach pain gets BD or 40mg at night. In gastric ulcers dose should
started which is relieved only by taking the foods. be given for 6 weeks followed by endoscopy.
♦♦ Night pain: Patient wakes from the sleep due to the pain –– Proton pump inhibitors are given, i.e. omeprazole
at around 3 AM. This is relieved by taking the food, milk or rabeprazole 20 to 40 mg/day; pantoprazole
or antacid. 40mg/day and lansoprazole 15 to 30mg/day is
♦♦ Periodicity of pain given. These should be given for 4 to 8 weeks.
• Pain usually occur in episodes and last for 1 to 3 Drugs omeprazole and lansoprazole should be
weeks every time for 3 to 4 times a year. In between given 30 minutes before taking a meal.
the episodes patients become asymptomatic. –– Prostaglandin analogue: Misoprostol 200 mg QDS
• Initially episodes are short in duration and are less helps to prevent NSAID induced mucosal injury.
frequent. With the time episodes get longer in duration –– Colloidal bismuth compounds: Here drugs such
and their frequency increases. as bismuth subsalicylate and colloidal bismuth
• In winter and spring seasons patients remain more subcitrate are given.
symptomatic. –– Complex salts: Sucralfate forms the protective
• In smokers relapse is more common as compared to covering for the ulcers.
non-smokers. H. pylori eradication
• Here triple drug therapy is used. Regimen includes
Management two antibiotics and a proton pump inhibitor
• Commonly given regimen consists of amoxicillin
Investigations / Diagnosis l gm twice daily along with clarithromycin 500 mg
♦♦ Endoscopy: It is the ideal method of diagnosing. Ulcer twice daily with twice a day proton pump inhibitor,
appears as severe aphthous ulcer with the creamy base. i.e. omeprazole or rabeprazole 20 mg, lansoprazole
♦♦ Barium meal: Peptic ulcer is seen in the form of crater 30 mg, pantoprazole 40 mg for 14 days. If person is
along the lesser curvature. allergic from penicillin, metronidazole may be used
♦♦ Gastric acid secretion tests: Fractional test meal is done in in place of amoxicillin.
which gruel meal is given for stimulating gastric secretion. • If infection persists after giving triple therapy,
Both free and total acidity are estimated. Augmented quadruple therapy, i.e. proton pump inhibitor,
histamine test is more specific and is used. bismuth, tetracycline, metronidazole is given.
♦♦ Test for H. pylori: It consists of invasive and non-invasive ♦♦ Long-term treatment
tests: • Intermittent treatment
• Invasive tests –– This should be given in cases where symptomatic
–– Rapid urease test relapses are less than 4 times a year.
–– Histology –– Four weeks course of one of the ulcer healing
–– Culture agents is given.
• Non-invasive tests • Maintenance treatment
–– Serology –– Continuous maintenance treatment is not needed
–– Urea – breath test after successful H. pylori eradication.
–– In minority who do require it, the lowest effective
Treatment dose should be given.
–– Long-term maintenance is with H2 receptor
Treatment of peptic ulcer is mainly medical.
antagonists, i.e. cimetidine 400 mg at night,
♦♦ General measure: ranitidine l50 mg at night, famotidine 20 mg at
• Stop smoking night or nizatidine l50 mg at night.
• Stop NSAIDs, corticosteroids and alcohols • Surgical treatment
• Avoid stress. –– In cases with gastric ulcer, partial gastrectomy
♦♦ Pharmacotherapy: with a Billroth I anastomosis is procedure of
• Short term treatment choice, in which ulcer itself and ulcer bearing area
–– Antacid and alginates: These are the antacids which of the stomach are resected.
are the combination of aluminum and magnesium –– Duodenal ulcer treatment could be truncal
compounds, i.e. aluminum hydroxide, magnesium va g o t o m y a l o n g w i t h p yl o r o p l a s t y o r
trisilicate and alginic acid. These drugs form gastroenterostomy.
protective mucosal raft. Sodium bicarbonate is –– In the emergency condition, ‘under-running’
the quickest acting antacid. Its dose is 15 to 30mL the ulcer for bleeding or ‘over sewing’, i.e. patch
liquid antacid 1 to 3 hours after the food and at repair for perforation is all that is required, in
bed time for 4 to 6 weeks. addition to taking a biopsy.
16   Mastering the BDS IIIrd Year  (Last 25 Years Solved Questions)

–– In patients with giant duodenal ulcers, partial Mechanism


gastrectomy using a ‘Polya’ or Billroth II
Relaxed or hypotonic sphincter
reconstruction may be required.
–– Elective surgery is done in gastric outflow
Decreased lower esophageal sphincter pressure
obstruction and recurrent ulcer despite the
medical treatment.
Raised intra-abdominal pressure

Q.16. Write short note on dysphagia. Impaired esophageal mucosal function


 (Mar 2007, 2 Marks) (Sep 2008, 2.5 Marks)
Or Delayed gastric emptying
Write short answer on dysphagia.(May 2018, 3 Marks)
Ans. Dysphagia is defined as difficulty in swallowing. Increased gastric contents

Causes Etiology
♦♦ Relaxed or hypotonic sphincter: It is due to diabetes mellitus,
♦♦ Disease pertaining to esophagus and its surrounding
hiatus hernia, fatty meal.
structures.
♦♦ Decreased lower esophageal sphincter pressure: It is due to
• Stomatitis
prolonged gastric tube intubation, scleroderma and use
• Pharyngitis
of certain drugs such as calcium channel blockers, nitrates.
• Glossitis
♦♦ Raised intra-abdominal pressure: It is due to ascites, obesity
• Pulmmer-Vinson syndrome
and pregnancy.
• Esophagitis
♦♦ Impaired esophageal mucosal function: It is due to usage of
• Carcinoma of esophagus
alcohol and smoking.
• Pressure by mediastinal tumor on esophagus
♦♦ Delayed gastric emptying: It is due to pyloric obstruction,
• Achalasia (Failure of reflex) of lower end of esophagus.
fatty foods and gastroparesis.
♦♦ Dysphagia in neurological disorders
♦♦ Increased gastric contents: It is due to large meals and
• Post-diphtheric paralysis
Zollinger-Ellison syndrome.
• Myasthenia gravis
♦♦ Sliding hiatus hernia: Where the esophagogastric junction
• Motor neuron disease
slides up through the diaphragm resulting in:
• Scleroderma or other related collagen disorders
• Loss of the obliquity of entry of esophagus into stomach.
• Acute bulbar paralysis.
• Loss of the reinforcing effect of intra-abdominal
pressure on the lower oesophageal sphincter.
Clinical Features
These two above mentioned factors of hiatus hernia facilitate
♦♦ Presence of anemia gastroesophageal reflux but do not directly cause it.
♦♦ Koilonychia ♦♦ Cardiomyotomy and vagotomy: They decrease the efficiency
♦♦ Glaze tongue of the lower esophageal sphincter.
♦♦ Malnutrition ♦♦ Increased intra-abdominal pressure: Pregnancy, obesity,
♦♦ Nasal regurgitation ascites, weight-lifting and straining increases the intra-
♦♦ Bulbar palsy abdominal pressure.
♦♦ Severe weight loss ♦♦ Reduced tone of lower esophageal sphincter: Cigarette smoking,
♦♦ Malignancy alcohol, fatty foods and caffeine act by reducing the lower
♦♦ Chest pain oesophageal sphincter tone.
♦♦ Hoarseness of voice ♦♦ Impaired gastric emptying: Impaired gastric emptying due to
obstruction of gastric outlet or use of anticholinergic drugs,
Treatment fatty foods and large volume meals act by increasing the
Dysphagia is a symptom complex of number of disease hence gastric content available for reflux.
treatment has to be planned depending on the etiological ♦♦ Systemic sclerosis.
factors. ♦♦ Drugs which reduce the lower esophageal sphincter tone,
e.g. aminophylline, beta-agonists, nitrates, calcium channel
Q.17. Write short note on gastroesophageal reflux disease. blockers, etc.
 (Apr 2007, 10 Marks)

Clinical Features
Ans. A chronic condition in which the lower esophageal
sphincter allows gastric acids to reflux into the ♦♦ Typical symptoms: Heart burn and acid regurgitation
esophagus, causing heartburn, acid indigestion, and ♦♦ Atypical symptoms: Dysphagia, globus sensation, non-
possible injury to the esophageal lining. cardiac chest pain, dyspepsia or abdominal pain.
Section 1:  General Medicine  17

♦♦ Extra-esophageal symptoms: Hoarseness, sore throat, • Avoid if possible medications that can worsen GERD -
sinusitis, otitis media, chronic cough, laryngitis, dental anti-cholinergic, theophylline, prostaglandin, calcium
erosion and recurrent aspiration. channel blockers, alendronate
♦♦ Malignancy: Head and neck cancer, esophageal adeno- • Weight loss if obese
carcinoma • Rabeprazole, esomeprazole provide superior gastric
acid suppression.
Complications 2. Medical treatment
♦♦ Esophagitis • In mild cases liquid antacid is used, i.e. 10 to 15 mL,
♦♦ Esophageal strictures one to three hours after the meal it provide relief in
♦♦ Esophageal ulcers heart burn.
♦♦ Aspiration pneumonia • In moderate cases, H 2 receptor antagonist, i.e.
♦♦ Iron deficiency anemia ranitidine 150 mg BD or QID with meals and before
♦♦ Barrett’s esophagus bed time for 6 weeks.
♦♦ Carcinoma of esophagus • In severe cases, proton pump inhibitors are given,
i.e. omeprazole 20 to 40 mg/day, pantoprazole 40mg/
Investigations day and rabeprazole 10 to 20 mg/day is given. These
♦♦ Endoscopy: Enables visualization of esophagitis, strictures should be given for 6 to 8 weeks. For maintenance
and Barrett’s mucosa which all can be confirmed by biopsy. therapy treatment should be given for 6 to 8 months.
♦♦ Barium meal can reveal hiatus hernia. • Metaclopramide or domperidone 10 mg TID increases
♦♦ Bernstein test is done in patients with high clinical lower esophageal sphincter tone and promote gastric
suspicion but negative endoscopy. emptying.
♦♦ Resting ECG and stress ECG to rule out ischemic heart • Repeated dilatations are used to treat esophageal
disease. strictures.
♦♦ Eesophageal motility studies. • In anemics oral iron or blood transfusion is given.
3. Surgical treatment
Management • Surgical resection of strictures should be carried out.
1. Conservative measures: • Surgical return of lower esophageal sphincter to
abdomen in patient with sliding hiatus hernia,
construction of an additional valve mechanism is done.
Q.18. Write in brief signs, symptoms and treatment of
amoebiasis. (Apr 2008, 5 Marks)
Or
Write short note on treatment of amoebiasis.
 (Feb 2013, 5 Marks)
Ans. Amoebiasis is the infection of gastrointestinal tract by
protozoan parasite Entamoeba histolytica.

Symptoms
♦♦ Patient has diarrhea and pass 10–15 stools per day.
♦♦ Presence of abdominal pain.
♦♦ Stools consist of blood and mucus.
♦♦ Flatulence is present.
♦♦ Fever is present between 38 and 40° with rigors.

Signs
♦♦ Palpation of abdomen show diffuse tenderness.
♦♦ Chronic cases show thickened tender sigmoid colon.
♦♦ Amoeba is felt as a sausage shaped mass in right iliac fossa.
Fig. 5:  Gastroesophageal reflux disease ♦♦ Tender hepatomegaly is present.
Treatment
• Abstain from eating within 2 hrs of bed time
• Elevate head of bed by 6 inches ♦♦ Diloxanide fuorate is given 500 mg TDS × 10 days.
• Sleep in left lateral decubitus position ♦♦ Metronidazole Or Ornidazole 500 mg TDS × 5 days
• Avoid: Caffeine, nicotine, alcohol, chocolate, mints, ♦♦ Secnidazole plus 2 gm single dose is given
carbonated beverages, high-fat foods, tomato or citrus- ♦♦ Nitazoxanide 500 mg BD is given
based products ♦♦ Dehydroemetine 1.5 mg / kg / day × 5 days IM
18   Mastering the BDS IIIrd Year  (Last 25 Years Solved Questions)

Q.19. Write short note on hepatic amoebiasis. Ans.


(May/Jun 2009, 5 Marks) Etiology
Ans. It is the most common complication of amoebiasis.
♦♦ Amoebiasis is caused by infection by protozoan parasite
Pathophysiology known as Entamoeba histolytica.
♦♦ Common source of transmission is water especially when
Amoeba after reaching the liver multiply and block small it is contaminated by fecal matter or sanitary conditions of
intrahepatic portal radicles, producing thrombosis and water supply being defective.
infarction resulting in necrosed areas surrounded by areas of ♦♦ Flies spread infection by feeding on fecal matter or
congestion. Necrotic area consists of degenerated liver cells, contaminated food.
leukocytes, connective tissue strands and enmeshed in between ♦♦ Auto-spread in a patient when cysts embedded underneath
Entamoeba histolytica. Cytolytic enzyme liberated from amoebae the nails and person may get infected if he had not properly
destroy the liver parenchyma and fusion of these small necrosed washed his hands.
areas results in abscess formation. An abscess generally is single
but may be multiple. Its walls are lined by a shaggy necrotic zone Clinical Features
in whose center there is thick reddish brown pus containing Symptoms
fragments of liver tissue, necrotic material and erythrocytes.
The pus typically is called “Anchovy sauce” like and is sterile ♦♦ Patient has diarrhea and pass 10–15 stools per day.
on culture. ♦♦ Presence of abdominal pain.
♦♦ Stools consist of blood and mucus.
Clinical Features ♦♦ Flatulence is present.
♦♦ Fever is present between 38 and 40° with rigors.
♦♦ Onset of Amoebic hepatitis is insidious and patient may
come with irregular or intermittent fever Signs
♦♦ There is stretching sensation in the liver area. Gradually
♦♦ Palpation of abdomen show diffuse tenderness.
with the progression of disease, anorexia hepatic pain and
♦♦ Chronic cases show thickened tender sigmoid colon.
epigastric discomfort appear
♦♦ Amoeba is felt as a sausage shaped mass in right iliac
♦♦ Examination shows a uniform tender hepatomegaly
fossa.
♦♦ There are signs of toxemia
♦♦ Tender hepatomegaly is present.
♦♦ Jaundice is not very common
♦♦ When hepatitis progresses on to a liver abscess, pain in the Management
liver area becomes a constant feature
♦♦ Intermittent fever, loss of weight, lassitude, peculiar ♦♦ Diloxanide fluorate is given 500 mg TDS × 10 days.
sallowness of skin, irritability, sleeplessness are common ♦♦ Metronidazole Or Ornidazole 500 mg TDS × 5 days
features. ♦♦ Secnidazole plus 2 gm single dose is given
♦♦ Nitazoxanide 500 mg BD is given.
Investigations ♦♦ Dehydroemetine 1.5 mg / kg / day × 5 days IM
♦♦ TLC and DLC shows leukocytosis with increase in Q.21. Write sign and symptoms of malabsorption syndrome.
polymorphs.  (Aug 2012, 5 Marks)
♦♦ Stool examination is done and cysts and trophozoites of Ans. Following are the sign and symptoms of malabsorption
E. histolytica should be look over. syndrome:
♦♦ Amoebic fluorescent antibody titer is positive.
Symptoms
Management of Amoebic Liver Abscess/Hepatic ♦♦ Presence of lassitude and laziness
Amoebiasis ♦♦ Mood fluctuation
♦♦ Early cases are responding well with metronidazole 800 ♦♦ Fatigue and weakness is present
mg TID for 5 days or tinidazole 2 gm daily for three days. ♦♦ Pain in bone is present
♦♦ Luminal amoebicide: Diloxamide furoate 500 mg 8 hourly ♦♦ Weight loss is present.
for 10 days should be given to determine luminal cyst.
Signs
♦♦ If abscess is large and not respond to chemotherapy
repeated aspiration under ultrasonic guidance is required. ♦♦ Skin: edema over legs and purpuras
♦♦ Rupture of abscess into peritoneal cavity requires ♦♦ Eyes: Xerophtalmia and night blindness
immediate aspiration or surgical drainage. ♦♦ Hematopoietic: Hemorrhagic tendencies are present,
anemia
Q.20. Describe etiology, clinical features and management ♦♦ Gastrointestinal system: Malnutrition is generalized,
of intestinal amoebiasis. (Jan 2012, 15 Marks) angular stomatitis, glossitis and diarrhea
Section 1:  General Medicine  19

♦♦ Genitourinary system: Hypotension, amenorrhea, loss of Clinical Features


libido, Nocturia
♦♦ Skeletal: Muscle wasting, tetany and presence of ♦♦ Abdominal Pain: A burning pain in the upper part of the
paresthesia. abdomen usually related to mealtimes together with
♦♦ Nervous system: Peripheral neuropathy. fullness, distension of the abdomen, bloating, with or
without nausea and generalized discomfort also known as
Q.22. Write acid peptic disease under the following headings:
“dyspepsia”. The pain is usually so sharply localized that
 (Feb 2014, 2 Marks Each)
the patient can often indicate the exact place with two or
a. Etiology
three fingers called the “pointing sign”. Gastric ulcer pain
b. Clinical features
c. Investigations is more after the ingestion of meals while duodenal ulcer
d. Treatment pain occurs more due to hunger.
♦♦ Nausea, heart burn, vomiting, loss of appetite and weight
Ans. Excessive secretion of acid and pepsin or a weakened
loss.
stomach mucosal defense is responsible for damage
to the delicate mucosa and the lining of the stomach, ♦♦ Gastric outlet obstruction: The ulcer could heal with scarring
esophagus and duodenum resulting in ulceration which and result in narrowing of the gastric or intestinal lumen.
is known as “Acid Peptic Disease”. This could cause an obstruction to food being passed
forward.
  “Acid peptic disease” is a collective term used to ♦♦ Vomiting or passing blood in stool: Signs of bleeding as
include many conditions such as gastroesophageal reflux vomiting of blood or black tarry color of the stool.
disease (GERD), gastritis, gastric ulcer, duodenal ulcer, ♦♦ Bleeding and perforation from the ulcer: Bleeding from the
esophageal ulcer, Zollinger-Ellison Syndrome (ZES) and site of the ulcer with thinning of the wall may result in
Meckel’s diverticular ulcer.
perforation.
Etiology Investigations
♦♦ Helicobacter pylori: H.pylori is responsible for around ♦♦ Blood tests: Blood tests such as ELISA help in the
60–90% of all gastric and duodenal ulcers. measurement of antibodies to H. pylori. Serum gastrin
♦♦ NSAIDs: Prostaglandins protect the mucus lining of the levels should be measured in patients with multiple
stomach. Nonsteroidal anti-inflammatory drugs (NSAIDs) ulcers to consider gastrin secreting tumors or Zollinger-
such as aspirin, diclofenac and naproxen prevent the Ellison syndrome. Tests for gastric secretion include the
production of these prostaglandins by blocking cyclo- “pentagastrin test”, the “chew and spit test” and the
oxygenase enzyme leading to ulceration and bleeding. “Hollander insulin test”.
♦♦ Smoking, alcohol and tobacco: Cigarettes, alcohol and ♦♦ Stool Test: Stool test detects the presence of H. pylori in the
tobacco cause an instant and intense acid production which feces and also establishes whether there is any recurrence
acts as though gasoline is poured over a raging fire. after antibiotic therapy.
♦♦ Blood group O: People with blood group “O” are reported ♦♦ Breath test: The urea breath test (UBT) is helpful in the
to have higher risks for the development of stomach ulcers detection of H.pylori. The patient is made to drink a liquid
as there is an increased formation of antibodies against containing carbon-labeled urea, which is broken down by the
the Helicobacter bacteria, which causes an inflammatory bacteria. The patient is subsequently asked to breathe into a
reaction and ulceration.
sealed bag, which is tested for the presence of labeled carbon.
♦♦ Heredity: Patients suffering from peptic ulcer diseases
A positive test indicates the presence of H pylori infection.
usually have a family history of the disease, particularly
♦♦ Endoscopy: Endoscopy is considered a more accurate
the development of duodenal ulcer which may occur
test for the diagnosis of “peptic ulcer diseases” and also
below the age of 20.
helps in taking biopsy of the affected area. Gastroscopy
♦♦ Steroids/other medicines: Drugs like corticosteroids,
or esophagogastroduodenoscopy (EGD) is a kind of
anticoagulants like warfarin (Coumarin), niacin, some
endoscopy which is carried out on patients to detect
chemotherapy drugs, and spironolactone can aggravate
peptic ulcer.
or cause ulcers.
♦♦ Barium radiography: X-rays are taken of the stomach,
♦♦ Diet: Low fiber diet, caffeinated drinks and fatty foods are
esophagus and duodenum after swallowing barium and
linked to peptic ulcer.
♦♦ Other diseases: Chronic liver, lung and kidney diseases the retention of contrast in the ulcer is monitored.
especially tumors of the acid producing cells all predispose
Treatment
to peptic ulcers. Zollinger-Ellison syndrome (ZES) is a rare
pre-cancerous condition which causes peptic ulcer disease. ♦♦ Eradication of H.pylori: The standard protocol to eradicate
Endocrine disorders such as hyperparathyroidism are also H.pylori involves the use of two or three antibiotics, i.e.
implicated in the development of peptic ulcers. amoxicillin, tetracycline, clarithromycin, metronidazole
♦♦ Stress: Stress and neurological problems can also be and the use of a proton pump inhibitor, i.e. esomeprazole,
associated with the Cushing's ulcer and peptic ulcer. omeprazole, lansoprazole, rabeprazole, pantoprazole with
20   Mastering the BDS IIIrd Year  (Last 25 Years Solved Questions)

or without a bismuth compound for around 2–3 weeks and ♦♦ Acute alcoholic hepatitis: This follows the period of heavy
repeated if there is recurrence. drinking. There is presence of right upper abdominal pain,
♦♦ Avoid NSAIDs or the concurrent use of a prostaglandin anorexia, nausea and vomiting, profound weakness. Liver
analogue (misoprostol) may be prescribed to prevent become enlarged and tender.
peptic ulceration due to NSAIDs. ♦♦ Autoimmune hepatitis: Prevalence is in young females.
♦♦ The use of antacids or H2 receptor antagonist (H2RAs) Presence of enlarged and tender liver, spider navei and
such as cimetidine, ranitidine, famotidine, and nizatidine enlarged spleen.
which help in the reduction of gastric acid secretion and ♦♦ Actinomycosis of liver: Here liver becomes enlarged and
in turn increase the gastric pH and reduce the secretion tender.
of pepsin. ♦♦ Weil’s disease: Spirochetes causes Weil’s disease. There is
♦♦ Treatment of peptic ulcer complications include a presence of enlarged and tender liver.
blood transfusion for hematemesis and melena, the use ♦♦ Malaria: In malignant form of malaria there is hepatomegaly
of antacids and H2 receptor antagonists for pain, the and tenderness over the liver. Liver is palpable in half of
treatment of peritonitis in case of perforation of peptic the cases. Spleen is always palpable.
ulcer disease. Circulatory Disturbances
♦♦ Congestive heart failure: Congestive heart failure is an
important cause of hepatomegaly moderate to massive.
3. Diseases of Liver The liver is firm and tender.
♦♦ Hepatic vein occlusion: It is uncommon condition and there
Q.1. Write short note on tender hepatomegaly. is presence of enlarged and tender liver.
 (Jan 2012, 5 Marks) (May/June 2009, 5 Marks)
 (Sep 2004, 3 Marks) (Mar 2003, 5 Marks) Tumors
 (Mar 2000, 5 Marks) ♦♦ Hepatocellular carcinoma: Liver becomes enlarged but
Ans. The term tender hepatomegaly itself means there is sometimes it is tender.
enlargement of liver with tenderness present in it. ♦♦ Angiosarcoma: In patients with exposure to gaseous
chemical, liver becomes enlarged and tender.
Causes
Budd-Chiari syndrome
♦♦ Infections
• Viral hepatitis Here liver is enlarged and tender. There is failure of jugular
• Amoebic abscess vein to fill when liver is pressed.
• Acute alcoholic hepatitis Q.2. Enumerate causes of tender hepatomegaly.
• Autoimmune hepatitis  (Sep 1999, 4 Marks) (Mar 1998, 5 Marks)
• Actinomycosis of liver Ans. Enumeration of causes of tender hepatomegaly
• Weil’s disease ♦♦ Infections
• Malaria • Viral hepatitis
♦♦ Circulatory disturbances • Amoebic abscess
• Congestive heart failure • Acute alcoholic hepatitis
• Hepatic vein occlusion • Autoimmune hepatitis
♦♦ Tumors • Actinomycosis of liver
• Hepatocellular carcinoma • Weil’s disease
• Angiosarcoma • Malaria
♦♦ Budd-Chiari syndrome ♦♦ Circulatory disturbances
• Congestive heart failure
Hepatitis • Hepatic vein occlusion
♦♦ Tumors
♦♦ Viral hepatitis: Inflammation of liver due to viral hepatitis is
• Hepatocellular carcinoma
common cause of producing tender hepatomegaly. There
• Angiosarcoma
is moderate enlargement of liver which is smooth with
♦♦ Budd-Chiari syndrome
consistency varying from soft to firm.
♦♦ Amoebic liver abscess: In this liver is enlarged and is tender Q.3. Write short note on ascites.
and tenderness present on lower costal cartilages on right  (Jan 2012, 5 Marks) (Mar 2011, 2 Marks)
side.  (Mar 2006, 5 Marks) (June 2010, 5 Marks)
♦♦ Bacterial liver abscess: Multiple small pyogenic abscesses or  (Mar 2007, 2 Marks)
a single large abscess involves liver mainly the right lobe Ans. Collection of the fluid in peritoneal cavity is called
producing an enlarged tender liver. ascites.
Section 1:  General Medicine  21

At least 1500 mL of fluid must collect in peritoneal cavity ♦♦ Pancreatic ascites due to retroperitoneal leakage of
before physical examination. pancreatic enzymes from a ruptured cyst or pancreatic duct.
♦♦ Bile ascites
Pathogenesis of Ascites ♦♦ Chylous ascites
♦♦ Epidemic dropsy

Investigations
♦♦ Ultrasonography: USG of abdomen shows presence of
minimum amount of fluid and is very needful when
clinical signs are absent. This used for guiding paracentesis.
♦♦ Paracentesis: Abdominal paracentesis is done which helps
in determination of etiology.
♦♦ In cirrhosis, ascitic fluid is clear and straw coloured. The
fluid is milky white in chyle ascites while it is cloudy in
infections. Hemorrhagic fluid is seen due to trauma, tumor
or tuberculosis.
♦♦ Presence of more than 500 leucocytes/µL is suggestive of
inflammatory conditions.
♦♦ Based on the specific gravity and total protein concentration,
Causes of Ascites ascitic fluid has traditionally been classified as transudative
and exudative.
♦♦ Disease of peritoneum
• Infections: Management of Ascites
–– Tuberculous peritonitis.
–– Spontaneous bacterial peritonitis. ♦♦ In every case of ascites oral diuretics, i.e. furosemide 40
–– Fungal-Candida, histoplasma. to 80 mg + spironolactone 25 to 100 mg is administered.
–– Parasitic — Schistosoma, enterobius. ♦♦ Sodium intake is restricted and diet which is low in sodium
–– Viral — Acute severe hepatitis. is given.
• Neoplasms: ♦♦ In case of massive ascites which produce cardio-
–– Primary mesothelioma. respiratory enlargement, abdominal paracentesis is done
–– Secondary carcinomatosis, e.g. adenocarcinoma, and fluid is drained slowly. Fluid should not be drained
sarcoma, teratoma, leukemia, Hodgkin's disease, quickly because it causes vasovagal attack.
lymphocytic lymphoma, myeloid metaplasia. ♦♦ P o r t a c a va l s h u n t s u r g e r y o r i m p l a n t a t i o n o f
–– Pseudomyxoma peritonei peritoneovenous shunt can be done in refractory ascites.
• Familial paroxysmal peritonitis Q.4. Enumerate causes of ascites.
• Miscellaneous  (Mar 2001, 5 Marks) (Sep 2007, 2 Marks)
–– Vasculitis — SLE and other collagen vascular Ans. Refer Ans 3 of same chapter.
diseases, allergic vasculitis (Henoch-Schonlein
purpura). Q.5. Describe causes, clinical features, diagnosis and
–– Eosinophilic gastroenteritis. management of jaundice (Apr 2017, 12 Marks)
–– Whipple’s disease. Ans. Jaundice is a condition where there is yellow pigmentation
–– Granulomatous peritonitis — Sarcoidosis, of skin or sclera due to excess bilirubin in the blood.
Crohn’s disease, starch peritonitis.
Causes
–– Peritoneal loose bodies.
–– Peritoneal encapsulation. The causes are based on the type of jaundice.
♦♦ Portal hypertension ♦♦ Hemolytic jaundice: In hemolytic jaundice there is
♦♦ Congestive heart failure excessive break down of RBC due to which bilirubin
♦♦ Hypoalbuminemia is produced in excess from hemoglobin. The causes of
• Nephrosis hemolytic jaundice are:
• Malnutrition • Jaundice of prematurity, physiological jaundice
• Protein-losing enteropathy • Defect in shape of erythrocytes, i.e. spherocytes and
♦♦ Beri-beri sickle cell anemia
♦♦ Myxedema • There is parasitic destruction of erythrocyte
♦♦ Ovarian disease: • Toxic agents, i.e. metal like lead, poison like snake
• Meigs syndrorne venom
• Struma ovari • Incompatible blood transfusion
• Ovarian overstimulation syndrome • ABO and Rh-incompatibility
22   Mastering the BDS IIIrd Year  (Last 25 Years Solved Questions)

• Excessive burns of body ♦♦ In cirrhotics look for spider naevi, white nails, enlargement
• Bacterial toxins, septicemia. of parotid glands, testicular atrophy, palmar erythema,
♦♦ Obstructive jaundice: In obstructive jaundice bilirubin gynecomastia, edema over legs and feet, and ascites.
conjugation takes place normally in the liver, but it does ♦♦ There may be scratch marks on skin suggestive of
not reach into the intestine and goes into the blood stream, cholestasis due to obstructive jaundice, bruising and
the result is rise in bilirubin level. The causes of obstructive petechial spots indicating prothrombin deficiency in
jaundice are: alcoholic or Laennec’s cirrhosis may be observed.
• Extrahepatic: ♦♦ Liver may be palpable, smooth and tender in infective
–– Obstruction within the bile ducts: The common hepatitis, hard and nodular in malignancy.
cause is gallstone. ♦♦ Gallbladder becomes palpable when obstruction at the
–– Obstruction due to change in wall of ducts: level of common bile duct is incomplete. A hard, small
Congenital obliteration of ducts during operation nodular gallbladder may be palpated in carcinoma.
procedure, sclerosing cholangitis, etc. ♦♦ In chronic cholecystitis, gallbladder may be palapable
–– Due to pressure on bile ducts: Pressure on the bile and tender (Murphy’s sign). In addition to looking
duct occur in number of disease, e.g. carcinoma for signs of disease in general examination, look
of liver, hydatid cyst or fever, etc. specifically in abdomen for ascites, liver, spleen and any
• Intrahepatic: In it there is no mechanical obstruction lymphadenopathy.
in bile ducts and it is due to intake of drugs like ♦♦ Rectal examination may be carried out for any primary
oral contraceptives, antitubercular drugs and chlor- growth in rectum.
promazine.
• Hepatocellular jaundice: In this liver cells fall to Diagnosis
conjugate and excrete all the bile pigments. The causes ♦♦ Biochemical test:
of hepatocellular jaundice are: • White cell count: Leukopenia is present in the
– Infection like viral hepatitis, yellow fever, malaria, hepatocellular jaundice. Eosinophilia is present in
typhoid, etc. drug hepatitis.
– Chemical poisons like chloroform, halothane, • Urine: Urobilinogen is absent in the obstructive
CCl4, etc. jaundice and is in excess in hemolytic jaundice.
– Alcoholic hepatitis, postnecrotic cirrhosis, etc. Bilirubin is in excess in urine in obstructive jaundice.
• Liver function test: Serum bilirubin estimation is done to
Clinical Features
asses level of jaundice. The flocculation test is positive
Symptoms for hepatocellular and is negative for hemolytic and
♦♦ Symptoms of a case of jaundice shall vary with the type of obstructive jaundice. Serum albumin levels are low
jaundice the patient is suffering from and the underlying and globulin levels are high in chronic hepatocellular
condition. jaundice.
♦♦ Commonest form of jaundice is due to hepatitis where the • Hematology: In hemolytic jaundice blood film shows
patient may start with malaise, low grade fever, vomiting immature cells and spherocytosis, erythrocyte fragility
and loss of appetite. is increased and Coombs test is positive.
♦♦ Person may take his / her morning breakfast normally and ♦♦ Radiology:
as the day passes, appetite for food almost disappears. • Imaging of liver by ultrasound technique should be
♦♦ In smokers, urge to smoke is the earliest to go. Yellowness initial technique for all the jaundiced patients.
appears first in the conjunctiva and then the mucous • Imaging of liver through CT scan is done. The dilated
membrane of the lips and palate became pale. bile ducts are seen during obstructive jaundice while
♦♦ Urine is high colored while in early stage, the stools may imaging through CT scan.
remain of normal color. ♦♦ Aspiration needle biopsy of liver:
♦♦ When a person has got features of obstructive jaundice, • It is done cautiously in jaundice. Menghini needle
color of conjunctiva is yellowish green. Stools become clay is used. The histological appearance of hemolytic,
colored and there is severe degree of itching. hepatocellular and obstructive jaundice is distinctive.
♦♦ Pulse becomes slow.
♦♦ Patient may suffer from bruises and bleeding from mucous Management
surfaces due to lack of fat soluble vitamin K.
Treatment is directed towards the underlying cause
♦♦ If jaundice remains for prolonged periods as in case of
♦♦ Patient should be given small feeds of fat free, low protein
malignancy patient suffers from marked asthenia and
and high carbohydrate diet which can be easily assimilated.
wasting.
♦♦ Additionally vitamin B and C are given orally in high
Signs dosages.
♦♦ Patient may show signs of anemia, malnutrition suggestive ♦♦ In obstructive jaundice, vitamin K should be given 10 mg
of malignancy or cirrhosis. parentrally.
Section 1:  General Medicine  23

Q.6. Write in detail about viral hepatitis. (Sep 2007, 5 Marks) ♦♦ Normal skin, sclera and urine color
Or ♦♦ Normal stool color
♦♦ Appetite improves
Write in brief the clinical features and treatment of ♦♦ Liver enlargement regresses.
viral hepatitis. (June 2010, 5 Marks)
Or Diagnosis
Diagnosis of viral hepatitis is done in a patient with history
Describe clinical features, investigations and treatment
of severe anorexia, nausea, vomiting and fever for a few days,
of viral hepatitis.
elevated serum bilirubin and value of SGPT over 500 indicates
 (Sep 2010, 15 Marks) (Feb 2002, 15 Marks)
viral hepatitis. The etiological agent is detected by serological
Ans. Viral hepatitis is a clinical entity where systemic infection
markers.
causes inflammation and hepatic cell necrosis.
♦♦ Hepatitis A:
Viruses causing viral hepatitis are:
• Non specific lab tests
♦♦ Hepatitis A virus –– Raised serum bilirubin within few days and
♦♦ Hepatitis B virus remain high up to 12 weeks.
♦♦ Hepatitis C virus –– Serum AST and ALT levels remain high for 1 to
♦♦ Hepatitis D virus 3 weeks.
♦♦ Hepatitis E virus –– Alkaline phosphatase level is mildly elevated,
Other viruses are: though it remains persistently high, it suggest
♦♦ Cytomegalovirus hepatitis associated cholestasis.
♦♦ Epstein-Barr virus • Specific test
♦♦ Herpes simplex virus, etc. –– Hepatitis A specific IgM antibody can be detected
at the onset of symptoms and at first rise in serum
Clinical Manifestations ALT. It peaks within first month and remain
Prodromal symptoms proceed the development of jaundice in positive for 3 to 6 months.
sclera and behind the tongue from few days to two weeks and –– IgG and hepatitis A virus become positive at onset
common symptoms are: of illness and is detectable for many years
–– Nucleic acid based test like PCR.
♦♦ Mild fever with or without chills
♦♦ Hepatitis B:
♦♦ Headache
In a patient with history of severe anorexia, nausea,
♦♦ Malaise
vomiting, fever for few days, elevated serum bilirubin
♦♦ Arthralgia and skin rashes particularly in HBV infection
and value of SGPT over 500 indicates HBV. The etiological
♦♦ Prominent gastroenteritis symptoms like anorexia, nausea
agent is identified by serological markers.
and vomiting
♦♦ Steady upper mild abdominal pain in right hypochondrium Following is the serological diagnosis of hepatitis B:
♦♦ Liver is not palpable. Stage of HBsAg Anti - HBc Anti-HBs
infection IgM IgG
Physical Signs
Incubation Positive Positive Negative Negative
♦♦ Liver is usually tender but is not palpable initially. Acute hepatitis B
♦♦ Enlarged cervical nodes may be found.
Early Positive Positive Negative Negative
♦♦ Splenomegaly is present, particularly in children.
♦♦ Dark urine and a yellow tint of sclera held the onset of Late or Positive Positive Positive Negative
established
jaundice.
Convalescence
Features of Jaundice 3 to 6 months Negative Positive Positive Positive or
♦♦ It is first observed in sclera in bulbar conjunctivitis or Negative
♦♦ Jaundice deepens following obstruction of bile canaliculi. Negative
♦♦ Stool become pallor 6 to 9 months Negative Negative Positive Positive
♦♦ Urine is dark Post - infection
♦♦ Liver is tender and is easily palpable More than 1 year Negative Negative Positive Positive
♦♦ At this time appetite often improves
Chronic infection Positive -------- Positive Negative
♦♦ Gastrointestinal symptoms diminished intensity.
Immunization Negative Negative Negative Positive
Thereafter jaundice recedes and all features comes to normal without exposure
in 3 to 6 weeks of time which can be revealed by: to infection
24   Mastering the BDS IIIrd Year  (Last 25 Years Solved Questions)

Markers of hepatitis B infection Diet

Markers Interpretation ♦ Nutritious diet containing 2000–3000 kcal daily is given.


♦ Light diet supplemented with glucose and food debris is
HBsAg Indicates presence of virus in body
acceptable.
HBeAg Indicates active replication of virus
♦ A good protein diet should be encouraged.
HBe antibody Indicates seroconversion and non- ♦ In case of severe vomiting, IV fluids and glucose may be
replecative stage required.
IgM anti-HBc Indicate recent infection or acute flare of ♦ Highly fatty diet should be avoided however complete
chronic infection. Low levels in chronic restriction of fatty diet is not required.
infection
HBs antibody Indicates immunity against the infection Drugs
either manual or following the vaccine
♦ Drugs should be avoided, especially in severe hepatitis.
HBV DNA quantitative Indicates viral load ♦ Sedative and hypotonic drugs, alcohol, oral contraceptives
should be especially avoided.
Viral Blood
Surgery
HBV – DNA is measured by polymerase chain reaction in blood.
Viral loads are in excess of 105 copies/mL in presence of active During acute viral hepatitis surgery carries a significant role of
viral replication. post operative liver failure. Only life saving operation should
be carried.
Other Investigations
♦♦ During early phase of hepatitis there is an increase Liver Transplantation
in more than 400 units/L increase in plasma alaline This may be required in acute or chronic liver failure due to
aminotransferase and aspartate aminotransferase. hepatitis virus.
♦♦ High levels of alkaline phosphatase is suggestive of
cholestasis. Prevention of Viral Hepatitis
♦♦ Prothrombin time is increased which indicates severe Prophylaxis of only hepatitis A virus and hepatitis B virus is
liver damage. present, i.e. active immunization.
♦♦ Hepatitis C
In active immunization recombinant hepatitis B vaccine
• Antibodies to HCV (anti-HCV)
which consists of HBsAg is available and produce active
–– Detection of antibodies to recombinant HCV
immunization in 95% of individuals.
polypeptides. Enzyme immunoassay measures
antibodies against two antigens NS4 and NS3. Q.7 Mention the complication of viral hepatitis.
–– These assays can detect antibodies within 6 to 8  (Sep 1999, 5 Marks) (Apr 2007, 5 Marks)
weeks of exposure. Ans. The complications of viral hepatitis are:
–– Average time for seroconversion is 2 to 3 weeks • Acute hepatic failure: Fatalities are rare and usually
• Recombinant immunoassay occur in this case.
• Hepatitis C virus RNA testing qualitative test • Relapsing hepatitis: There is return of signs and
• Hepatitis C virus RNA testing quantitative test symptom during recovery. It can be detected by
• HCV genotyping is helpful in predicting response clinical signs and biochemical tests. It resolves
to therapy. jaundice.
♦♦ Hepatitis D • Cholestasis: It can develop at any stage of illness and
• During Hepatitis D viral infection both IgM and IgG gives features of obstructive jaundice.
antibodies can be detected in serum in acute phase. • Gillbert’s syndrome: It may come into picture during
• HDV infection can also be detected using reverse viral hepatitis.
transcriptase polymerase chain reaction (RT-PCR) • Connective tissue disease such as polyarthritits nodosa
♦♦ Hepatitis E is observed in HBV and HCV infection.
• Identification of IgM antibodies to HEV from acute • Renal failure, i.e. glomerulonephritis can occur also
plasma serum samples. Antibodies detected are in relation to HBV and HCV infection.
against ORF-2 and ORF-3 • Henoch-Schönlein purpura and papular *acrode-
rmatitis is repeated in children.
Management • Chronic hepatitis is observed with HBV infection with
♦♦ Only the more severely affected patients require care in or without HDV and HCV viruses.
hospital. • Cirrhosis is also the complication of HBV and HCV
♦♦ Posthepatic syndrome is treated by reassurance. and follows chronic hepatitis.

Q7. *Acrodermatitis= Dermatitis of extremities


Section 1:  General Medicine  25

• Hepatocellular carcinoma: It is also the complication of Investigations


HBV and HCV following cirrhosis of liver. Serology
Q.8. Write short note on viral hepatitis.
♦♦ HBV consists of number of antigens. The three important
 (Aug 2012, 5 Marks) (Sep. 1999, 5 Marks)
antigens are hepatitis B surface antigen (HBsAg), core
Or antigen (HBeAg) and hepatitis e antigen (HBeAg).
Discuss clinical features diagnosis, management and ♦♦ Appearance of hepatitis B surface antigen (HBsAg)
complications of viral hepatitis. (Jan 2017, 12 Marks) in serum is the first evidence of infection. It normally
Ans. Refer to Ans 6 and Ans 7 of same chapter. persists for 3–4 weeks but can persist up to 6 months.
Q.9. Write short note on hepatitis B. (Mar 2001, 5 Marks) After disappearance of HBsAg, antibody against HBsAg
Or (Anti-HBs) appears and persists for years and confers
Describe briefly viral hepatitis B. (Mar 1998, 3 Marks) immunity. Presence of Anti-HBs antibody means either
 (Dec 2015, 3 Marks) (Feb 2013, 5 Marks) previous infection or vaccination.
Or ♦♦ HBeAg is not seen in the blood. However, antibody to it
Discuss mode of transmission and clinical features of (anti-HBc) appears early during the illness. Presence of
hepatitis B infection. (May 2018, 5 Marks) IgM anti-HBc indicates acute infection and IgG anti-HBc
suggests chronic infection (when HBsAg positive) or
Ans. Hepatitis B is a virus which results in causing of disease
recovery (when anti-HBs positive).
known as hepatitis in man.
♦♦ Presence of HBeAg indicates active viral replication and
Route of Transmission or Etiology high degree of infectivity. Anti-HBe appears as HBeAg
♦♦ Horizontal transmission disappears and its presence suggests low level of viral
• By use of drug injection replication and decreased infectivity.
• By infected unscreened blood products ♦♦ Above mentioned serological tests are done to identify the
• By tattooing and acupuncture needles cause of hepatitis.
• In sexually active individuals, i.e. either homosexuals
Viral Blood
or heterosexuals
♦♦ Vertical transmission HBV-DNA is measured by polymerase chain reaction in blood.
• By HBsAg positive mother Viral loads are in excess of 105 copies/mL in presence of active
• High risk groups of HBV infection are patients viral replication.
of hemodialysis, physician, dentists, surgeons,
Other Investigations
paramedical staff and persons in laboratory and
blood bank. ♦♦ During early phase of hepatitis there is an increase
in more than 400 units/L increase in plasma alaline
Clinical Features
aminotransferase and Aspartate aminotransferase.
The cases of hepatitis pass through three phases: ♦♦ High levels of alkaline phosphatase is suggestive of
1. Preicteric phase: cholestasis.
• Prodromal period of 4 to 7 days ♦♦ Prothrombin time is increased which indicates severe
• Mild fatigue fever for 2 to 5 days liver damage.
• Anorexia, nausea and vomiting
• Distaste for smoking Management
• Abdominal distress
2. Icteric phase: ♦♦ Supportive treatment:
• Jaundice on third or forth days deepens or rapidly This is given in acute hepatitis
increases • Restrict the physical activity and bed rest is strictly
• Tea colored urine is present recommended.
• Loss of weight • High calorie diet is given. Good protein intake should
• Stool becomes light in color be there. Hospitalization and intravenous fluid (10%
• Spleen is palpable glucose) are indicated, if oral intake is not proper or
3. Posticteric phase: there is marked nausea and vomiting.
• Jaundice recedes and all features come back to normal • Avoid the drugs which are hepatotoxic or those that
in 3 to 6 weeks are metabolized in the liver.
• Appetite is good. • Bile salt sequestering agent (cholestyramine) decreases
• Stool and urine regain their natural color. pruritus in cases with cholestasis.
26   Mastering the BDS IIIrd Year  (Last 25 Years Solved Questions)

• Patients having features of severe hepatic failure such Write etiology, clinical features, investigations and
as alteration in mental status (hepatic encephalopathy) management of cirrhosis.  (July 2016, 12 Marks)
and prolonged prothrombin time or bleeding time Ans. The term cirrhosis is applied to chronic diffuse liver
should be hospitalized. disease of varied etiology and is characterized by hepatic
• No specific therapy is recommended for acute viral cell necrosis, proliferation of connective tissue and
hepatitis except in acute HCV infection. Subcutaneous nodular regeneration.
interferon alpha has been shown to reduce the rate of
chronicity in acute HCV hepatitis. Causes of Cirrhosis of Liver
♦♦ Specific management: ♦♦ Common causes
• It is done in case of chronic hepatitis B • Alcohol
• Interferon inhibits the division of virus and antiviral • Viruses: Hepatitis B virus, hepatitis C virus.
drug lamivudine 100 mg OD is given. ♦♦ Other causes
• Interferon is given in dose of 5 MU daily or 10 MU • Hepatic venous congestion
three times per week for 16 days. • Veno-occlusive disease
• Lamivudine is anti-DNA polymerase agent. It has • Budd-Chairi syndrome
shown significant improvement but when the drug • Wilson’s disease
is stopped HBV replication recurs and resistance to
• Galactosemia
lamivudine is another problem.
• Glycogen storage diseases
• *Hemochromatosis
Complication / Fate of HBV • Drugs such as isoniazid, oral contraceptives, etc.

Clinical Features
♦♦ Hepatomegaly
♦♦ Jaundice
♦♦ Ascites
♦♦ Circulatory changes
♦♦ Spider angioma
♦♦ Palmar erythema
♦♦ Cyanosis
♦♦ Endocrine changes.
• Loss of libido
• Hair loss of chest.
Men: Gynecomastia, testicular atrophy, impotence
Q.10. Describe the causes, clinical features, compli­cations Women: Breast atrophy, irregular menstrual cycle,
and management in cases of cirrhosis of liver. amenorrhea
 (Apr 2010, 15 Marks) (Mar 2000, 15 Marks) ♦♦ Hemorrhage tendency: Bruises, purpura, epistaxis,
menorrhagia
Or ♦♦ Portal hypertension: Splenomegaly, collateral vessels,
Describe clinical features, etiology and management variceal bleeding
of cirrhosis of liver. (Dec 2009, 15 Marks) ♦♦ Hepatic encephalopathy
Or ♦♦ Miscellaneous: Pigmentation, clubbing, low grade fever.
Write short note on clinical features and complications Complications
of cirrhosis of liver. (Dec 2012, Marks) ♦♦ Posthepatic vein obstruction “Budd-Chiari syndrome” or
Or extrahepatic and postsinusoidal.
Write short note on cirrhosis of liver. (Dec 2015, 3 Marks) ♦♦ Intra-hepatic postsinusoidal / cirrhosis
 (Mar 2016, 3 Marks) ♦♦ Esophageal varices causes severe hemorrhage
♦♦ Development of hepatocellular failure due to hepato­
Or
cellular carcinoma.

10. *Hemochromatosis= A hereditary disorder in which iron salts are deposited in the tissues, leading to liver damage, diabetes mellitus,
and bronze discoloration of the skin.
Section 1:  General Medicine  27

♦♦ Renal failure
♦♦ Hypersplenism
♦♦ Due to infection, spontaneous bacterial peritonitis and
secondary bacterial peritonitis occur.

Investigations
♦♦ Blood examination:
• Anemia can be present secondary to bleeding, folate
deficiency, marrow suppression or hypersplenism.
Leukopenia and thrombocytopenia.
• Aminotransferases (ALT, AST) get frequently elevated
whereas a rise in the serum bilirubin and ALP may
occur later. Serum albumin is low and Prothrombin
time is frequently prolonged.
♦♦ Imaging: Ultrasonography is done to assess the liver
size and texture, ascites, portal hypertension and
splenomegaly.
♦♦ Endoscopy: Upper gastrointestinal endoscopy is being
carried out to detect esophageal varices and to exclude
other causes of upper gastrointestinal bleeding in the
stomach and duodenum.
♦♦ Liver biopsy: It is done for the assessment of severity of the
cirrhotic changes and also confirms the specific cause of
the cirrhosis. Fig. 6: Cirrhosis

Management Q.11. Write briefly complications of liver cirrhosis. 


 (Sep 1999, 3 Marks)
No treatment can reverse cirrhosis or even ensure that no further
Ans. Following are the complications of liver cirrhosis:
progression occurs, but medical therapy can improve general
♦♦ Posthepatic vein obstruction “Budd-Chiari syndrome” or
health and treat the symptom of disease effectively.
extrahepatic and postsinusoidal.
The main objectives are: ♦♦ Intra-hepatic postsinusoidal / cirrhosis
♦♦ Esophageal varices causes severe hemorrhage
♦♦ Detect treatable causes
♦♦ Development of hepatocellular failure due to hepatocellular
♦♦ Prevent and correct malnutrition
carcinoma.
♦♦ Manage chronic cholestasis
♦♦ Renal failure
♦♦ Treat the complication.
♦♦ Hypersplenism
• Treatable causes: The treatable causes are alcohol abuse,
♦♦ Due to infection, spontaneous bacterial peritonitis and
drug ingestion, *hemochromatosis and Wilson’s
secondary bacterial peritonitis occur.
disease, relief of biliary obstruction will prevent
secondary biliary cirrhosis. Q.12. Discuss management and etiopathogenesis of cirrhosis.
• Nutrition: In absence of ascites, a high energy 3000  (Oct 2003, 16 Marks)
kcal / day, protein rich (80–100 gm / day) diet should Ans. For management refers toAns 10 of same chapter.
be advised. Salt restriction is required if ascites are
Etiopathogenesis
present. Fat intake is not restricted unless cholestasis
is a feature. Complete absence of alcohol. Vitamin Hepatic cirrhosis can occur at any age and often causes prolong
and other supplements are not required when good morbidity. Any condition leading to persistent or recurrent
diet is taken. hepatocyte death may lead to hepatic cirrhosis, e.g. viral
• Drug treatment: Any drug should be avoided because hepatitis and alcohol, prolonged biliary damage and obstruction
as most of the drugs are metabolized in the liver which as occur in primary biliary cirrhosis, sclerosing cholangitis
are liable to develop toxic reaction because they will and postsurgical biliary structures will also result in cirrhosis.
unable to get metabolize. Persistent blockages of venous return from liver, e.g. veno-
• Liver transplantation: It is considered in all patients occlusive disease and Budd- Chiari syndrome, will eventually
with chronic liver disease who develop liver failure. result in liver cirrhosis.
28   Mastering the BDS IIIrd Year  (Last 25 Years Solved Questions)

Common to all cause of liver cirrhosis is activation of hepatic • Bacterial


stellate cells. These cells are widely distributed throughout the –– Typhoid
liver in the space of disse. Following activation the fat storing –– Pneumonia
stellate cells become transformed into multifunctional cells, and –– Brucellosis
cytokine synthesis. –– Tuberculosis
• Protozoal
Q.13. Differentiate between hepatocellular jaundice and
–– Amoebiasis
hemolytic jaundice.  (Feb 2006, 3 Marks)
–– Malaria
Ans. –– kala-azar
Hepatocellular jaundice Hemolytic jaundice • Spirochetal
–– Weil’s disease
• In this liver cells fails to • There is excessive brake
conjugate and excrete all the down of RBC due to which –– Syphilis
bile pigments, so the bilirubin excess bilirubin is produced –– Relapsing fever
level is increased • Parasitic
• It occurs in young adults • It occurs in infants and young –– Schistosomiasis
adults –– Echinococcus
• Pain is dragging over the liver • No pain is present
–– Clonorchiasis
• Fungal
• Rate of development of • Rate of development of –– Actinomycosis
jaundice is slow jaundice is slow but rapid in
–– Histoplasmosis
hemolytic crisis
♦♦ Metabolic
• Jaundice is orange yellow • Jaundice is lemon yellow type • Fatty liver
type
• Amyloid
• Liver is tender and enlarged • Liver is moderately enlarged • Glycogen storage disease
• Urobilinogen is present in • Urobilinogen is present in ♦♦ Congestive
excess in urine and bilirubin excess and bilirubin is absent • General
is present –– Congestive heart failure
• Flocculation is positive • Flocculation test is negative –– Tricuspid regurgitation
–– Constrictive pericarditis
Q.14. How will you differentiate between obstructive jaun- • Local
dice and infective jaundice? (Apr 1999, 5 Marks) –– Portal hypertension (cirrhosis)
Ans. –– Hepatic vein thrombosis
♦♦ Tumors
Obstructive jaundice Infective jaundice • Primary
• Obstructive jaundice occurs • In this liver cells fails to –– Benign and malignant hepatoma
due to obstruction of bile duct conjugate and excrete all –– Benign and malignant cholangioma
the bile pigments, so the –– Fibroma
bilirubin level is increased –– Sarcoma
• It occurs in middle aged or • It occurs in young adults –– Hemangioma
elderly persons • Secondary: Direct due to spread by contiguity, or
• Pain is colicky • Pain is dragging over the liver embolic metastatic
• Liver is enlarged • Liver is tender and enlarged ♦♦ Cysts
• Hydatid
• Urobilinogen is absent in urine • Urobilinogen is present in
examination but bilirubin is excess in urine and bilirubin • Polycystic
present is present ♦♦ Biliary obstruction
• Type of jaundice is greenish • Type of jaundice is orange • Gallstones
yellow yellow type • Strictures of bile ducts
• Flocculation test is negative • Flocculation test is positive ♦♦ Hematological
• Leukemias
Q.15. Enumerate the causes of hepatomegaly.  • Lymphoma
 (Dec 2010, 5 Marks) (Feb 2006, 2.5 Marks) • Myeloproliferative disorders
Ans. Enumeration of causes of hepatomegaly ♦♦ Storage disorders
♦♦ Infections • Gaucher’s disease
• Viral • Niemann - Pick's disease
–– Viral hepatitis • Amyloidosis
–– Yellow fever • Glycogen storage disease
–– Infectious mononucleosis • Gargoylism,
–– Lassa fever • Hemochromatosis
Section 1:  General Medicine  29

♦♦ Myeloid metaplasia to the doctor. Its basic aim is to arrive a diagnosis of


• Secondary carcinoma of bone disease and outline the treatment.
• Myelofibrosis • History taking is an important clinical presentation
• Myelosclerosis which helps in the diagnosis and finding the
• Multiple myeloma etiology / cause of jaundice.
• Marble-bone disease • In history taking we are mainly concerned with the
♦♦ Genetic abnormalities: Sickle cell disease abdominal pain which patient undergoes like in
♦♦ Congenital: Riedel’s lobe hepatocellular jaundice patient will not feel abdominal
Q.16. Write short note on soft tender liver. (Sep 2005, 5 Marks) colic while in obstructive and hemolytic jaundice
Ans. Soft tender liver is the clinical condition of the liver in patient feels sharp pain.
which on palpitation the liver assumes to be soft and • The second point concerned in history taking with
palpable. patient is pruritis, i.e. discoloration of skin. Pruritis is
transient in hepatocellular jaundice. It is well, marked
Etiology and is present in obstructive jaundice while it is fully
absent in hemolytic jaundice.
♦♦ Hydatid cyst
• Last and most important point in history taking
♦♦ Calcinosis cutis
is past history of patient, like patient can develop
♦♦ Nephritis and cerebellar ataxia
hepatocellular jaundice if he / she is in contact with
♦♦ Hypersensitivity reaction
jaundice patient before as well as certain drug can
♦♦ Oral contraceptives
cause hepatocellular jaundice.
♦♦ HIV
• A patient of obstructive jaundice will explain about
♦♦ Acute renal failure
pain, weight loss or any abdominal surgery before,
♦♦ Obstructive jaundice
which can give rise to the obstructive jaundice.
♦♦ Hepatocellular carcinoma
• A patient of hemolytic jaundice has a blood transfusion
♦♦ Pleural and pericardial effusion
or certain drugs which can cause hemolytic jaundice
♦♦ Gallbladder injury.
which patient had taken in past.
Clinical Features
Investigations
♦♦ Fever is present
Investigation means laboratory diagnosis.
♦♦ Hepatomegaly is present
♦♦ Muscle cramp / fatigue due to lactic acid accumulation Investigation Hepatocellular Obstructive Hemolytic
♦♦ Jaundice is seen • Urine
♦♦ Non pitting edema is present Bilirubin •  Present •  Present •  Absent
♦♦ Anorexia, ascites and anemia is present Urobilinogen •  Present •  Absent •  Present
♦♦ Amyloidosis is present.
• Stools
Diagnosis Stercobilinogen •  Present •  Absent Present

♦♦ MRI should be done • Peripheral •  Leukopenia •  Normal Reticulo-


smear   in infective cytosis and
♦♦ USG is done
 hepatitis sphero-
♦♦ X-ray of lower abdomen shows moderate enlargement
cytosis
of liver
♦♦ Lipid profile shows increase in lipid value. • Laboratory
♦♦ Blood picture reveals microcytic hypochromic anemia.    finding test
– Bilirubin •  + + •  + + •  + +
Treatment –  Alkaline •  Normal
•  Raised • Markedly
  phosphate
♦♦ Treatment of cause is done • Markedly raised
– SGOT
♦♦ No drug is given, only conservative treatment is done. raised •  Raised •  Normal
♦♦ Fluid diet is given, restriction of salt and fatty acids and • Barium meal Normal May reveal Normal
spicy foods is done. and cholangio- pancreatic
♦♦ Electrolyte balance is maintained with digestive enz- graphy growth
ymes.
• RBC survival Normal Normal Decreased
Q.17. How history taking and investigation will help in find-
ing the cause of the jaundice? (Sep 2005, 18 Marks) Q.18. Mention the complications of viral hepatitis B.
Ans. History taking of person mainly concerned with its  (Sep 2006, 5 Marks)
illness, present past and family history which he describe Ans. Refer to Ans 7 and Ans 9 of same chapter.
30   Mastering the BDS IIIrd Year  (Last 25 Years Solved Questions)

Q.19. Write short note on liver function test. Q.21. Differentiate between hepatitis A and hepatitis B viral
 (Mar 2007, 2 Marks) infection. (Mar 2008, 2 Marks)
Ans. Ans.
Liver Function Test
Hepatitis A viral infection Hepatitis B viral infection
Serum Bilirubin
• It spreads by feco oral route • It spreads by parental route
♦♦ The normal level is 1 mg / 100 mL.
• Incubation period ranges from • Incubation period ranges from
♦♦ It increases in: 15 days to 45 days 6 weeks to 6 months
• Hepatocellular injury
• Chronic infection is not • Chronic infection is present
• Posthepatic biliary obstruction.
present
Thymol Turbidity Test
• It is caused by enterovirus • It is caused by hepadna which
♦♦ Demonstrating hepatic cellular dysfunction which is RNA virus is DNA virus
♦♦ Gamma globulins are mainly responsible for positivity • For prevention passive • For prevention passive
in this test. immunity is given by immune immunity is given by
serum globulin hyperimmune serum globulin
Enzyme in Liver Disease
♦♦ Alkaline phosphate: Q.22. Write short note on jaundice.
• Elevation occurs in obstructive jaundice.  (Mar 2011, 4 Marks) (Sep 2009, 4 Marks)
• M i l d e l e va t i o n i n x a n t h o m a t o u s c irrho s is ,  (Mar 2006, 5 Marks)
hepatocellular injury, liver abscess. Or
♦♦ SGOT (serum glutamine oxalate transferase): Write briefly on signs and symptoms and treatment of
• Normal level is 5 to 40 unit jaundice. (May/June 2009, 5 Marks)
• It increases in all conditions leading to hepatic necrosis Or
and in alcoholic liver damage.
Write short answer on jaundice. (Apr 2018, 3 Marks)
♦♦ SGPT (serum glutamine pyruvic transferase)
Ans. Jaundice is a condition where there is yellow pigmentation
• It is more specific for liver disease.
of the skin or sclera by excess of bilirubin in the blood.
• It is raised in sever parenchymal damage to liver.
When levels of bilirubin exceed 2 mg or above clinical
For example, chronic acute hepatitis, alcoholic liver
jaundice become apparent.
disease, biliary obstruction.
♦♦ GGT (gamma glutamyl transpeptidase): Sensitivity test
for alcoholic liver disease and for hepatobiliary disease. Classification
♦♦ Leucine amylo peptidase: It increases in primary liver ♦♦ Hemolytic:
diseases. • Physiological: Jaundice of prematurity
♦♦ Serum pseudocholinesterase: • Congential: Spherocytosis, sickle cell anemia
• It has limited value. • Parasitic destruction of erythocytes—malaria
• Value decreases in sub-acute and chronic parenchymal • Toxins—heavy metals
diseases of liver (Cirrhosis). • Poisons—snake venom
• Drugs—sulfonamides, nitrofurantoin
Q.20. Write short note on hepatitis B importance in dental
• Bacterial toxins: Septicemia
practice. (Mar 2007, 2 Marks)
• Incompatible blood transfusions—ABO and Rh blood
Ans. Hepatitis may be defined as an infection of the liver
group incompatibility
caused by the hepatitis virus B. • Extensive burns.
The prevention for hepatitis will be ♦♦ Obstructive:
• Prevention of mode of transmission • Extrahepatic:
– Avoid infected blood transfusion, body organs, –– Obstruction within the bile ducts—gallstones,
sperms and other tissues. Blood should be neoplasm, round worm.
screened before transfusion. –– Obstruction due to changes in the wall of the ducts—
– Strict sterilization process should be ensured in congenital obstruction (biliary atresia), traumatic
clinics. (following surgery), sclerosing cholangitis.
– Presterilized needles and syringe should be used. –– Pressure from without: Carcinoma of liver (prima­
– Avoid injections unless they are absolutely ry / secondary) gumma, hydatid cyst, enlarged glands
necessary. in porta hepatis (Hodgkin’s, leukemia, tuberculosis)
– Carrier should be told not to share razors or tooth carcinoma head of pancreas, cancer stomach.
brushes, use barrier methods of contraceptions, • Intrahepatic (obstruction without mechanical cause):
avoid blood donation. –– Drugs like chlorpromazine, antitubercular drugs,
• Hepatitis B vaccination should be given. methyl testosterone, oral contraceptives.
Section 1:  General Medicine  31

–– Viral hepatitis with prolonged cholestasis. petechial spots indicating prothrombin deficiency in
–– Jaundice of pregnancy alcoholic or Laennec’s cirrhosis may be observed.
–– Primary biliary cirrhosis ♦♦ Liver may be palpable, smooth and tender in infective
–– Hepatocellular: hepatitis, hard and nodular in malignancy.
- Viral hepatitis ♦♦ Gallbladder becomes palpable when obstruction at the
- Infectious mononucleosis level of common bile duct is incomplete. A hard, small
- Yellow fever nodular gall bladder may be palpated in carcinoma.
- Bacterial diseases with fever, typhoid ♦♦ In chronic cholecystitis, gallbladder may be palapable and
- Malaria tender (Murphy’s sign). In addition to looking for signs of
- Weil’s disease disease in general examination, look specifically in abdomen
- Chemicals like chloroform, halothane, trinitro­ for ascites, liver, spleen and any lymph­adenopathy.
toluene, carbon tetrachloride. ♦♦ Rectal examination may be carried out for any primary
- Post-necrotic cirrhosis growth in rectum.
- Alcoholic hepatitis
Treatment
- Hemochromatosis.
–– Congenital hyperbilirubinemia Treatment is directed towards the underlying cause:
- Without liver pigment:
♦♦ Patient should given small feeds of fat free, low protein and
* Gilbert’s disease high carbohydrate diet which can be easily assi­milated.
*  Crigler-Najjar syndrome. ♦♦ Additionally Vitamin B and C are given orally in high
- With liver pigment: dosages.
* Dubin-Johnson syndrome ♦♦ In obstructive jaundice, vitamin K should be given 10 mg
* Rotor syndrome. parentrally.
Symptoms Q.23. Describe causes, clinical features and mana­gement of
portal cirrhosis. (Sep 2009, 4.5 Marks)
♦♦ Symptoms of a case of jaundice shall vary with the type of
Ans. It is characterized by the diffuse involvement of the
jaundice the patient is suffering from and the underlying
liver in form of necrosis of liver cells, collapse of hepatic
condition.
lobules, reticulin framework followed by diffuse fibrosis
♦♦ Most common form of jaundice is due to hepatitis where and formation of structurally abnormal nodules. This
the patient may start with malaise, low grade fever, interferes not only with liver blood flow, but also its
vomiting and loss of appetite. function. This results in portal cirrhosis which is due to
♦♦ Person may take his / her morning breakfast normally and inadequacy of liver cells and portal hypertension.
as the day passes, appetite for food almost disappears.
♦♦ In smokers, urge to smoke is the earliest to go. Yellowness Causes
appears first in the conjunctiva and then the mucous ♦♦ Hepatitis B and D
membrane of the lips and palate became pale. ♦♦ Consumption of excessive alcohol
♦♦ Urine is high colored while in early stage, the stools may ♦♦ Hemochromatosis
remain of normal color. ♦♦ Alpha-1 antitrypsin deficiency
♦♦ When a person has got features of obstructive jaundice, ♦♦ Autoimmune chronic active hepatitis
color of conjunctiva is yellowish green. Stools become clay ♦♦ Wilson’s disease
colored and there is severe degree of itching. ♦♦ Malaria
♦♦ Pulse becomes slow. ♦♦ Schistosomiasis
♦♦ Patient may suffer from bruises and bleeding from mucous ♦♦ Veno-occlusive disease
surfaces due to lack of fat soluble vitamin-K. ♦♦ Hepatic venous congestion to drugs like methyldopa etc.
♦♦ If jaundice remains for prolonged periods as in case of
Clinical Features
malignancy patient suffers from marked asthenia and
wasting. Case of cirrhosis may present either in compensated or
decompensated forms.
Signs
Compensated Form
♦♦ Patient may show signs of anemia, malnutrition suggestive
of malignancy or cirrhosis. ♦♦ A compensated case of cirrhosis has features of dyspepsia
♦♦ In cirrhotics look for spider naevi, white nails, enlargement in the form of morning anorexia, nausea, vomiting and
of parotid glands, testicular atrophy, palmar erythema, vague ill-health. This is more so when it is early stage of
gynecomastia, edema over legs and feet, and ascites. alcoholic cirrhosis.
♦♦ There may be scratch marks on skin suggestive of ♦♦ There is palmar erythema, spider naevi, splenomegaly and
cholestasis due to obstructive jaundice, bruising and hepatomegaly with a non tender liver.
32   Mastering the BDS IIIrd Year  (Last 25 Years Solved Questions)

♦♦ There is loss of weight, ill-health and edema of the ankles. Ans.


♦♦ There may be no firm signs of cirrhosis and diagnosis is
made on clinical suspicion to be confirmed by biochemical Management of Amoebic Liver Abscess
investigations and liver biopsy. Injection dehydroemetine 60 mg IM daily for 6 days.
♦♦ This stage of compensated form of cirrhosis may continue Tab. Tinidazole 600 mg TDS for 7 days.
for a variable period of time ranging from months to years
Tab. Chloroquine 500 mg TDS for 11 days.
and bleed from esophageal varices may draw attention
to the disease or some precipitating cause like severe If Amoebic liver abscess is large:
bacteremia may produce hepatocellular decompensation.
♦♦ The abscess should be aspirated and all fluid pus, etc. will
Decompensated Form be removed.
♦♦ A decompensated cirrhosis is characterized by a downhill ♦♦ Open drainage of pus may be undertaken if there is large
course, abdominal distension, ascites, weight loss, edema amount of pus secondarily infected or if there are signs of
over the dependent parts, cirrhotic facies (sunken eyes, pus, but aspiration is negative.
hollow cheeks, pinched nose), skin dry and sallow. ♦♦ Depending on the patient’s response aspiration of abscess
♦♦ Jaundice may appear indicating progressive liver cell may be carried out.
destruction. Q.25. Describe the etiology, diagnostic criterias and manage-
♦♦ Liver may be palpable with irregular surface or it may not ment of viral hepatitis. (Apr 2008, 15 Marks)
be palpable when it is shrunken. Ans. Viral hepatitis is a clinical entity where systemic infection
♦♦ Splenomegaly is present in 80% of patients. Nails are white
causes inflammation and hepatic cell necrosis.
and clubbed.
♦♦ Endocrinal changes in the form of spider naevi, palmar
Etiology
erythema, gynecomastia, loss of axillary and pubic hair,
and testicular atrophy are seen. There may be bleeding Hepatitis A,B,C,D,E, cytomegalovirus, herpes simplex virus,
spots or bruising due to prothrombin deficiency. Epstein-barr virus and yellow fever virus.
♦♦ Ascites may be massive and is disproportionate to the
edema of feet. Diagnostic Criterias
♦♦ Hepatocellular failure may supervene as liver cell necrosis
proceeds. Diagnosis is based on investigations. Following are the
♦♦ Breath becomes foul smelling often giving a mousy smell investigation which confirm the diagnosis of viral hepatitis
(Fetor hepaticus).
♦♦ Flapping tremors and encephalopathy appear.
Investigations
♦♦ Appearance of jaundice, rapid accumulation of ascites
and development of hepatic encephalopathy are poor ♦♦ Hepatitis A:
signs in cirrhosis. • Non-specific lab tests
–– Raised serum bilirubin within few days and
Management
remain high up to 12 weeks.
It is palliative: –– Serum AST and ALT levels remain high for 1 to
♦♦ Rest in bed: To maximize treatment of any reversible 3 weeks.
element of underlying liver disease and to improve renal –– Alkaline phosphatase level is mildly elevated,
perfusion. though it remains persistently high, it suggest
♦♦ Correction of any etiological factor: i.e. stoppage of alcohol, hepatitis associated cholestasis.
stoppage of drugs causing portal cirrhosis. • Specific test
♦♦ Diet: Low salt. Total daily intake of 2000 calories with –– Hepatitis A specific IgM antibody can be detected
protein intake of 120 gm. If patient can tolerate it. at the onset of symptoms and at first rise in serum
Fats and carbohydrates in normal amount. Vitamin B ALT. It peaks within first month and remain
complex. positive for 3 to 6 months.
♦♦ Symptomatic treatment: –– IgG and hepatitis A virus become positive at onset
• For anemia vitamin B12 and folic acid is given of illness and is detectable for many years
• For restlessness sedatives such as lorazepam should –– Nucleic acid based test like PCR.
be given ♦♦ Hepatitis B:
• For ascites low sodium diet and diuretics should be In a patient with history of severe anorexia, nausea,
given. vomiting, fever for few days, elevated serum bilirubin
Q.24. Write management of Amoebic liver abscess.  and value of SGPT over 500 indicates HBV. The etiological
 (Mar 2011, 4 Marks) agent is identified by serological markers.
Section 1:  General Medicine  33

Following is the serological diagnosis of hepatitis B: • Peak titers for IgM are observed during first 4 weeks
while onset of infection. A rising titer of IgG antibody
Stage of infection HBsAg Anti - HBc Anti - HBs is also diagnostic of infection.
IgM IgG For management of viral hepatitis refer to Ans 6 of
Incubation Positive Positive Negative Negative same chapter.
Acute hepatitis B Q.26. Write in brief the signs, symptoms and treatment of
ascites. (Apr 2008, 5 Marks)
Early Positive Positive Negative Negative
Ans. Ascites is the accumulation of free fluid in the peritoneal
Late or established Positive Positive Positive Negative
cavity.
Convalescence
Symptoms
3–6 months Negative Positive or Positive Positive or
negative negative ♦♦ Patient has sudden abdominal pain and fever may also
6–9 months Negative Negative Positive Positive be present.
Post-infection ♦♦ Edema over ankles and feet.
♦♦ Skin over abdomen is stretched.
More than 1 year Negative Negative Positive Positive
Chronic infection Positive -------- Positive Negative Sign
Immunization without Negative Negative Negative Positive ♦♦ Bulging of abdomen and in flanks is present.
exposure to infection ♦♦ Umbilicus is transversely stretched or everted.
♦♦ Presence of fluid thrill and shifting dullness.
Markers Interpretation ♦♦ Increase in inferior vena cava pressure.
♦♦ Diaphragm may be pushed upwards and there can be
HBsAg Indicates presence of virus in body
respiratory distress.
HBeAg Indicates active replication of virus ♦♦ Pleural effusion on right side can be present.
HBe antibody In d i ca te s se ro co nversi on a nd n on - ♦♦ Urinary output is reduced.
replecative stage
Treatment
IgM anti-HBc Indicate recent infection or acute flare of
chronic infection. Low levels in chronic ♦♦ In every case of ascites oral diuretics, i.e. furosemide 40
infection to 80 mg + spironolactone 25 to 100 mg is administered.
HBs antibody Indicates immunity against the infection ♦♦ Sodium intake is restricted and diet which is low in sodium
either manual or following the vaccine is given.
HBV DNA quantitative Indicates viral load ♦♦ In case of massive ascites which produce cardiorespiratory
enlargement, abdominal paracentesis is done and fluid
♦♦ Hepatitis C is drained slowly. Fluid should not be drained quickly
• Antibodies to HCV (anti-HCV) because it causes vasovagal attack.
–– Detection of antibodies to recombinant HCV Q.27. Write short note on fulminant hepatic failure.
polypeptides. Enzyme immunoassay measures  (Dec 2010, 5 Marks)
antibodies against two antigens NS4 and NS3.
Ans. Fulminant hepatic failure is seen in a healthy person
–– These assays can detect antibodies within 6 to 8
which develop acute hepatitis and goes into hepatic
weeks of exposure.
encephalopathy within the 8 weeks of illness.
–– Average time for seroconversion is 2 to 3 weeks
• Recombinant immunoassay Etiology
• Hepatitis C virus RNA testing qualitative test
• Hepatitis C virus RNA testing quantitative test ♦♦ Acute viral hepatitis
• HCV genotyping is helpful in predicting response ♦♦ Drugs – all hepatotoxic drugs
to therapy. ♦♦ Pregnancy with hepatitis
♦♦ Hepatitis D ♦♦ Wilson’s disease
• During Hepatitis D viral infection both IgM and IgG ♦♦ Due to Reye’s syndrome.
antibodies can be detected in serum in acute phase.
Clinical Features
• HDV infection can also be detected using reverse
transcriptase Polymerase chain reaction (RT-PCR) ♦♦ Cerebral features: Poor alertness, slurred speech, drowsiness,
♦♦ Hepatitis E: confusion, disorientation, convulsion and coma.
• Identification of IgM antibodies to HEV from acute ♦♦ Jaundice is present.
plasma serum samples. Antibodies detected are ♦♦ Fetor hepaticus
against ORF-2 and ORF-3 ♦♦ Flapping tremors are present.
34   Mastering the BDS IIIrd Year  (Last 25 Years Solved Questions)

♦♦ Signs of portal hypertension are present, i.e. ascites, edema. Q.29. Write acute viral hepatitis under following headings:
♦♦ Cerebral edema is present.  (Mar 2013, 2 Marks Each)
a. Etiology
Investigations b. Clinical features
♦ Bilirubin levels are high c. Investigations
♦ Serum transaminase levels are high d. Treatment
♦ Prothrombin time is prolonged Ans. • Etiology: Refer to Ans 25 of same chapter
♦ Urine may contain urobilinogen and bilirubin • Clinical features: Refer to Ans 6 of same chapter
♦ Serum ammonia levels are high • Investigations: Refer to Ans 25 of same chapter
♦ USG of liver shows reduced liver size. • Treatment: Refer to Ans 6 of same chapter.
Treatment Q.30. Write short note on high-risk groups and prophylaxis
♦♦ Phenobarbitone or 5 mg IV diazepam is given to patient. of hepatitis B. (Feb 2014, 3 Marks)
♦♦ Care of pulse, blood pressure, bowel and bladder is taken. Ans.
♦♦ 5 to 10% of glucose drip IV is given.
♦♦ IV vitamin K is given 10 mg for 3 days. High-risk Groups of Hepatitis B
♦♦ IV vitamin C is given 500 mg daily for prevention of
bleeding. The hepatitis B virus can infect infants, children, teens and adults.
♦♦ IV ranitidine is given 50 mg twice daily. Although everyone can be at some risk for a hepatitis B infection,
♦♦ Encephalopathy is treated by with drawl of protein intake; there are people who are at greater risk because of their ethnic
Sterilization of gut by neomycin 1 gm orally for 6 hours; background, occupation, or lifestyle choices.
increased fecal output of nitrogen by changing bacterial The following list is a guide for screening high-risk groups, but
flora with lactulose 30 to 60 mL orally after 2–3 hours till it certainly does not represent all potential risk factors.
loose stool is produced. ♦♦ Health care providers and emergency responders
♦♦ Infections should be treated with amoxicillin or ceftzoxime. ♦♦ Sexually active heterosexuals (more than 1 partner in the
Q.28. Enumerate the causes of cirrhosis liver. past six months)
(Aug 2012, 5 Marks) ♦♦ Men who have sex with men (Homosexuals)
Ans. Following are the causes of cirrhosis of liver: ♦♦ Individuals diagnosed with a sexually transmitted disease
♦♦ Viral hepatitis B and C (STD)
♦♦ Illicit drug users (injecting, inhaling, snorting, pill popping)
♦♦ Alcohol
♦♦ Sex contacts or close household members of an infected
♦♦ Cryptogenic
person
♦♦ Metabolic:
♦♦ Children adopted from countries where hepatitis B is
• Hemochromatosis
common (Asia, Africa, South America, Pacific Islands,
• Wilson’s disease
Eastern Europe, and the Middle East)
• α1—antitrypsin deficiency
♦♦ All pregnant women
• Cystic fibrosis ♦♦ Recipients of a blood transfusion before 1992
• Glycogen storage disease ♦♦ Kidney dialysis patients and those in early renal failure
• Galactosemia ♦♦ Inmates of a correctional facility
♦♦ Biliary obstruction: ♦♦ Staff and clients of institutions for the developmentally
• Primary biliary cirrhosis disabled
• Sclerosing cholangitis ♦♦ Any individual who may have other risk factors not
• Secondary biliary cirrhosis included on this list.
♦♦ Venous outflow obstruction:
• Budd-Chiari syndrome Prophylaxis of Hepatitis B
• Veno-occlusive disease Following is the prophylaxis of hepatitis B:
• Congestive heart failure ♦♦ Recombinant hepatitis B vaccine having HBsAg capable
♦♦ Drugs: of producing active immunization.
• Methotrexate ♦♦ Usually three injections of vaccine should be given IM
• Methyldopa during current, first and sixth month. These vaccinations
• Oxyphenisatin provide 90% of prophylaxis from hepatitis B virus.
• Amiodarone ♦♦ If patient is immunocompromised larger doses of
♦♦ Indian childhood cirrhosis. vaccination should be given.
Section 1:  General Medicine  35

♦♦ Passive immunization is provided by IM injection of ♦♦ Myxedema


hyperimmune serum globulins which is given within 24 ♦♦ Ovarian disease:
hours or almost within a week of exposure to infected • Meigs’syndrorne
blood. • Struma ovari
♦♦ Active along with passive immunization is provided to • Ovarian overstimulation syndrome
the paramedicos who has undergone needle stick injury, ♦♦ Pancreatic ascites due to retroperitoneal leakage of
to newborn babies of hepatitis B positive mothers and pancreatic enzymes from a ruptured cyst or pancreatic
to regular sexual partner of hepatitis B positive patient. duct.
Dosage is 500 IU for adults and 200 IU for babies. ♦♦ Bile ascites
♦♦ Chylous ascites
Following precautions are to be taken for prevention from ♦♦ Epidemic dropsy
hepatitis B:
Investigations
♦♦ Avoid infected blood transfusion, body organs, sperms and
other tissues. Blood should be screened before transfusion. ♦♦ Blood examination: Anemia can be present. Presence of
♦♦ Strict sterilization process should be ensured in clinics. neutrophilic leukocytosis indicates infection.
♦♦ Presterilized needles and syringe should be used. ♦♦ Urine examination: Massive albuminuria greater than 3.5 gm
♦♦ Avoid injections unless they are absolutely necessary. per day is present in nephritic syndrome.
♦♦ Carrier should be told not to share razors or tooth brushes, ♦♦ Stool for occult blood: It may indicate gastrointestinal
use barrier methods of contraceptions, avoid blood malignancy as cause for ascites.
donation. ♦♦ Ultrasonography: It detect ascites
♦♦ Diagnostic paracentesis: 50–100 mL of ascetic fluid is
Q.31. Write short note on causes and investigations of ascites.
aspirated and biochemical analysis is done. Bacteriological
 (Feb 2014, 3 Marks)
examination should also be done. It should detect whether
Ans.
the ascetic fluid is exudate or transudate.
Causes of Ascites ♦♦ Serum-ascites albumin gradient: Albumin present in serum
and ascetic fluid is determined for calculating the gradient.
♦♦ Disease of peritoneum Gradient > 1.1 g / dL indicates transudative ascites and
• Infections: < 1.1 g / dL indicate exudative ascites. Fluid protein < 50%
–– Tuberculous peritonitis. of serum protein indicates transudate while > 50% is
–– Spontaneous bacterial peritonitis. indicative of exudates.
–– Fungal: Candida, histoplasma.
–– Parasitic: Schistosoma, enterobius. Q.32. Describe clinical features of portal hypertension. How
–– Viral: Acute severe hepatitis. will you investigate and manage a case of cirrhosis of
• Neoplasms: liver. (Feb 2014, 8 Marks)
–– Primary mesothelioma. Ans. Following are the clinical features of portal hypertension:
–– Secondary carcinomatosis, e.g. adenocarcinoma, • Splenomegaly: Spleen gets enlarged to the size of
sarcoma, teratoma, leukemia, Hodgkin's disease, 5–6 cm.
lymphocytic lymphoma, myeloid metaplasia. • Hypersplenism: It is common leading to anemia,
–– Pseudomyxoma peritonei thrombocytopenia and leukemia. Pancytopenia can
• Familial paroxysmal peritonitis also occur.
• Miscellaneous: • Collateral vessels form at gastroesophageal junction
–– Vasculitis — SLE and other collagen vascular which may rupture leading to hematemesis or malena.
diseases, allergic vasculitis (Henoch-Schönlein • Ascites is present.
purpura). • Caput-medusae is seen around umbilicus and at
–– Eosinophilic gastroenteritis. times large umbilical collateral give rise to venous
–– Whipple’s disease. hum.
–– Granulomatous peritonitis — Sarcoidosis, • Fetor hepaticus results from shunting of portal
Crohn’s disease, starch peritonitis. blood. In fetor hepaticus foul smell from breadth is
–– Peritoneal loose bodies. present due to mercaptans.
–– Peritoneal encapsulation.
♦♦ Portal hypertension Investigation of Case of Cirrhosis of Liver
♦♦ Congestive heart failure
♦♦ Hypoalbuminemia: I. Assessment of severity of cirrhosis of liver:
• Nephrosis A. Liver biochemistry:
• Malnutrition –– It may be normal in compensated cirrhosis.
• Protein-losing enteropathy –– SGOT / SGPT reflect the activity of the disease,
♦♦ Beriberi which is low when cirrhosis is established,
36   Mastering the BDS IIIrd Year  (Last 25 Years Solved Questions)

but increased in evolution of cirrhosis due to ♦♦ Manage chronic cholestasis


autoimmune hepatitis. ♦♦ Treat the complication.
–– Alkaline phosphatase and transaminases may be • Treatable causes: The treatable causes are alcohol abuse,
slightly raised in decompensated cirrhosis. drug ingestion, *hemochromatosis and Wilson’s
–– Serum albumin reflects liver cell function, its low disease, relief of biliary obstruction will prevent
level indicates cirrhosis. secondary biliary cirrhosis.
–– Plasma bilirubin is normal in most of the cases • Nutrition: In absence of ascites, a high energy 3000
but it gets increased in decompensated cirrhosis. kcal / day, protein rich (80–100 gm/day) diet should be
–– Low albumin, i.e. <2.5 g / L and rising bilirubin advised. Salt restriction is strictly required if ascites are
are signs of progressive liver damage and these present. Fat intake is not restricted unless cholestasis
constitute bad prognostic signs. is a feature. Complete absence of alcohol. Vitamin
B. Hematology: and other supplements are not required when good
–– Anemia, leukopenia and thrombocytopenia or diet is taken.
pancytopenia may be seen in cirrhosis. • Drug treatment: Any drug should be avoided because
–– Increased prothrombin time is a bad prognostic as most of the drugs are metabolized in the liver which
sign. are liable to develop toxic reaction because they will
C. Ascitic fluid examination: Ascites is transudate in nature. unable to get metabolize.
II. Assessment of type or cause of cirrhosis of liver: • Liver transplantation: It is considered in all patients
A. Blood biochemistry with chronic liver disease who develop liver failure.
–– Viral markers, i.e. HbSAg are of value in
Q.33. Describe hepatitis-B under following headings.
identification.
 (Nov 2014, 2+2+2+2 Marks)
–– Serum autoantibodies, antinuclear, anti-smooth
muscle and anti-mitochondrial antibodies level a. Etiology
increase in cryptogenic cirrhosis and biliary b. Clinical features
cirrhosis. c. Diagnosis
–– Rise in alkaline phosphatase indicates biliary d. Prophylaxis
cirrhosis. Ans. For etiology and clinical features refer to Ans 9 of same
–– Serum immunoglobins: The IgG is increased in chapter.
autoimmune hepatitis, IgA increases in alcoholic For prophylaxis refer to Ans 30 of same chapter.
cirrhosis and IgM in primary biliary cirrhosis.
B. Imaging Diagnosis
–– Ultrasound examination: USG reveals changes in
Serology
size, shape and echotexture of the liver. Fatty
change and fibrosis produce diffuse increases ♦♦ HBV consists of number of antigens. The three important
echogenicity. Presence of ascites, varices portal antigens are hepatitis B surface antigen (HBsAg), core
vein diameter and enlarge in size of spleen can antigen (HBeAg) and hepatitis e antigen (HBeAg).
be determined on ultrasound. ♦♦ Appearance of hepatitis B surface antigen (HBsAg)
–– Barium meal swallow is done for esophageal in serum is the first evidence of infection. It normally
varices. persists for 3–4 weeks, but can persist up to 6 months.
–– Endoscopy can be done for detection and for After disappearance of HBsAg, antibody against HBsAg
treatment of varices (anti-HBs) appears and persists for years and confers
C. Liver biopsy: immunity. Presence of anti-HBs antibody means either
It is also necessary to confirm the diagnosis of previous infection or vaccination.
cirrhosis. ♦♦ HBeAg is not seen in the blood. However, antibody to it
(anti-HBc) appears early during the illness. Presence of
Management of Case of Cirrhosis of Liver IgM anti-HBc indicates acute infection and IgG anti-HBc
suggests chronic infection (when HBsAg positive) or
No treatment can reverse cirrhosis or even ensure that no further recovery (when anti-HBs positive).
progression occurs, but medical therapy can improve general ♦♦ Presence of HBeAg indicates active viral replication and
health and treat the symptom of disease effectively. high degree of infectivity. Anti-HBe appears as HBeAg
The main objectives are: disappears and its presence suggests low level of viral
♦♦ Detect treatable causes replication and decreased infectivity. Above mentioned
♦♦ Prevent and correct malnutrition serological tests are done to identify the cause of hepatitis.
Section 1:  General Medicine  37

Viral Blood ♦♦ Cardiac symptoms 3–6 weeks after infection is first to


draw attention.
HBV – DNA is measured by polymerase chain reaction in blood.
♦♦ Abdominal symptoms: Abdominal pain and tenderness,
Viral loads are in excess of 105 copies/mL in presence of active
nausea, vomiting, fever and leukocytosis.
viral replication.
♦♦ Pyrexia of unknown origin
Other Investigations ♦♦ Typhoid or influenzal mode of onset with fever
♦♦ Nodules of skin lesion.
♦♦ During early phase of hepatitis there is an increase
in more than 400 units/L increase in plasma Alaline Treatment of Acute Rheumatic Fever
aminotransferase and aspartate aminotransferase.
♦♦ Bed rest is important to reduce joint pain and cardiac
♦♦ High levels of alkaline phosphatase are suggestive of
workload. Duration of bed rest is guided by markers of
cholestasis.
inflammation like temperature, WBC count and ESR.
♦♦ Prothrombin time is increased which indicates severe
♦♦ Benzathine penicillin 1.2 mu IM 4 hourly. If patient is
liver damage.
allergic to penicillin, erythromycin 40–50 mg / kg for ten
days is given.
♦♦ Aspirin usually relieves symptom of arthritis rapidly. A
4. Diseases of starting dose of 60 mg / kg body weight per day is given
divided into 6 doses. The dose may be increased to 120
Cardiovascular System mg / kg body weight. This dose may produce severe
Q.1. Write short note on acute rheumatic fever. symptoms like vomiting, tachypnea and acidosis. Aspirin
 (Apr 2007, 5 Marks) is given till ESR comes to normal.
Or ♦♦ Corticosteroids like prednisolone produces rapid
Describe in brief about rheumatic fever.  symptomatic relief than aspirin and is indicated in cases
with severe arthritis or carditis. Prednisolone is given in
 (Dec 2015, 4 Marks)
doses of 1.2 mg / kg body weight till ESR comes to normal.
Or
Write short answer on rheumatic fever. Q.2. Outline the management of acute rheumatic fever.
 (May 2018, 3 Marks)  (Mar 2009, 5 Marks) (Mar 1998, 5 Marks)
Ans. It is an acute inflammatory disease which occurs due Or
to infection by group A hemolytic streptococci which Discuss the management of acute rheumatic fever.
involves heart, joint, skin and nervous system which  (Aug 2010, 12 Marks)
develops as autoimmune reaction to infecting organism. Ans. Management
I. Treatment of acute attack:
Etiology • Bed rest is important to reduce joint pain and
♦♦ Predisposing causes: cardiac workload. Duration of bed rest is guided
Age should be 5 to 15 years. by markers of inflammation like temperature,
Sex has equal incidence WBC count and ESR.
♦♦ Genetic factors: Family incidence known. • Benzathine penicillin 1.2 mu IM 4 hourly. If
♦♦ Social and economic factors: Dampness, overcrowding and patient is allergic to penicillin, erythromycin
under nutrition increases incidence. 40–50 mg / kg for ten days is given.
♦♦ Idiosyncrasy is presumably a factor since 3% of people are • Aspirin usually relieves symptom of arthritis
involved in streptococcal epidemics develop rheumatic rapidly. A starting dose of 60 mg/kg body weight
fever. per day is given divided into 6 doses. The dose
may be increased to 120 mg / kg body weight.
Clinical Manifestations This dose may produce severe symptoms like
♦♦ Prodromal phase: Tonsillitis or sore throat 1 to 4 weeks prior vomiting, tachypnea and acidosis. Aspirin is
to onset of acute rheumatic fever. Besides this anorexia, given till ESR comes to normal.
pallor, fatigability and nervous irritability is present. • Corticosteroids like prednisolone produces
♦♦ Latent phase: When antibodies to preceding streptococcal rapid symptomatic relief than aspirin and
infection are produced. is indicated in cases with severe arthritis or
♦♦ Phase of onset of rheumatic fever/mode of onset. carditis. Prednisolone is given in doses of 1.2
Arthritis and fever 2–3 weeks after infection. mg / kg body weight till ESR comes to normal.
38   Mastering the BDS IIIrd Year  (Last 25 Years Solved Questions)

II. Secondary prevention: To prevent further attack of


rheumatic fever, long-term prophylaxis is needed.
• Benzathine penicillin 1.2 mu IM is injected at
the interval of 21 days. Further attack is unusual
after the age of 21 years and treatment can be
stopped.
• To prevent chances of endocarditis prophylactic
antibiotic therapy should be given.
Q.3. Describe briefly diagnosis of rheumatic fever. 
 (Sep 2007, 2.5 Marks)
Or
Write short notes on Jones criteria for rheumatic fever.
 (Mar 2008, 3 Marks) (Feb 1999, 3 Marks)
Or
Write short note on duke jones criteria in acute rheu-
matic fever. (Sep 2009, 4 Marks)
Ans. Diagnosis of rheumatic fever is made by ‘Jones criteria’
which is as follows:

Major Criteria

♦♦ Carditis:
• It is pancarditis involving endocardium, myocardium
and pericardium.
• It manifests as breathlessness, palpitation and chest
pain.
• Tachycardia, cardiomegaly and new or change
murmurs
• Aortic regurgitation in 50% cases.
• Pericarditis produces frictional rub and pericardial
tenderness.
• Cardiac failure due to myocardial infarction.
♦♦ Sydenham’s chorea:
• Late neurological manifestations that occurs at least
three months after the episode of acute rheumatic fever Fig. 7:  Jone’s criteria
when all signs disappear.
• More common in female. Clinical
• It is characterized by involuntary dancing movements ♦♦ Fever
of hands, feet or face. ♦♦ Arthralgia
♦♦ Polyarthritis: ♦♦ Previous history of rheumatic fever or rheumatic heart
• Early feature of illness is non-specific. disease.
• It is characterized by acute painful symmetric and
migratory inflammation of large joints. Laboratory
• Classical presentation is acute migratory polyarthritis. ♦♦ Acute phase reactants (leucokytosis, raised ESR, C- reactive
Pain and swelling in involved joints subside or protein)
disappear as newer joints get affected. ♦♦ Prolonged PR interval in ECG.
♦♦ Erythema marginatum: Red macules which fade in center,
but remain red at the edges and occur mainly on trunk and Essential Criteria
proximal extremities on face.
♦♦ Subcutaneous nodules: They are small, dense, firm, Evidence for recent streptococcal infection as evidenced by:
painless and are best felt over tendons and bones. ♦♦ Increase in ASO titer
• Nodules appear more than 3 weeks after onset of other • > 333 Todd units (in children).
manifestations. • > 250 Todd units (in adults).
Section 1:  General Medicine  39

♦♦ Positive throat culture for streptococcal infection ♦♦ Signs of raised pulmonary capillary pressure: Pleural effusion,
♦♦ Recent history of scarlet fever. crepitation, pulmonary edema.
♦♦ Signs of pulmonary hypertension: RV heave, loud P2
Confirmation of Diagnosis ♦♦ Others: Basal crackels, ascites and pleural effusion
Result is based on Presence of two or more major criterias or
Investigations
one major and two minor criteria, in the presence of essential
criteria, is required to diagnose acute rheumatic fever. ♦♦ ECG:
• Right ventricular hypertrophy
Q.4. How will you diagnose and manage a case of rheumatic
• Left atrial hypertrophy
fever? Outline complications of rheumatic fever.
♦♦ X-ray chest:
 (Sep 1999, 15 Marks) (Sep 2009, 5 Marks)
• Prominent left atrial appendage may be seen in left
Ans. For diagnosis refer to Ans 3 of same chapter.
border of heart between pulmonary artery and left
For management refer to Ans 2 of same chapter. ventricle. It indicates left atrial enlargement.
Complications • Double shadow of enlarged left atrium on right side
of spine.
♦♦ Myocardial infarction • Signs of pulmonary venous congestion
♦♦ Mitral stenosis ♦♦ Echocardiogram:
♦♦ Tricuspid regurgitation • Show thick immobile mitral cusp
♦♦ Aortic regurgitation • Decreased diastolic filling of left ventricle
♦♦ Aortic stenosis is rare • Decreased valve orifice area
♦♦ Mitral regurgitation. • Left atrial thrombus, if it is present.
Q.5. Enumerate the causes of Jones criteria of acute rheu- ♦♦ Cardiac catheterization: is used to assess valvular lesions
matic fever. (Mar 2001, 5 Marks) and to detect coronary artery disease.
Ans. The causes of Jones criteria are: ♦♦ Doppler:
• Previous streptococcal infection • Pressure gradient across mitral valve
• Recent scarlet fever • Pulmonary artery pressure
• Positive throat culture from streptococcal A • Left ventricular function
• Increased-antistreptolysin O titer. Diagnosis
Q.6. Describe clinical features, diagnosis, investigations
It is based on physical signs and investigations.
and management of rheumatic mitral stenosis.
 (Dec 2012, 8 Marks)
Management
Ans. Mitral stenosis is a valvular heart disease.
Rheumatic mitral stenosis occurs in elderly people and is ♦♦ Medicinal:
most common in females. • Salt restriction should be done in diet or very low salt
diet is given.
Clinical Manifestations • Digitalis therapy is given. In the patient with
congestive heart failure Tab. digoxin 0.25 mg BD is
Symptoms
given.
♦♦ Patient complains of breathlessness and fatigue on • Diuretics can be given for controlling heart failure
exertion. • Anticoagulants such as heparin can be given to
♦♦ Progression of stenosis lead to dyspnea on rest and even prevent embolism
have orthopnea and paroxysmal nocturnal dyspnea. • Prophylactic oral penicillin V 250 mg BD is given
♦♦ Acute pulmonary edema can also occur. to prevent rheumatic fever. If patient is allergic of
♦♦ Hemoptysis can be present due to rupture of pulmonary penicillin erythromycin 250 mg daily orally is given.
congestion and pulmonary embolism and cough due to ♦♦ Surgical:
pulmonary congestion. When patient remains symptomatic despite of medical
♦♦ Chest pain is present due to pulmonary venous treatment or when mitral stenosis is severe, surgical
hypertension. intervention is needed:
• Mitral valvotomy:
Signs
–– Percutaneous balloon valvotomy is indicated
♦♦ Atrial fibrillation is present. when mitral valve is noncalcified and without
♦♦ Auscultation: Presence of loud first heart sound, opening regurgitation. The procedure involves the passing
snap and mid diastolic low pitched rumbling murmur best of catheter across the valve and inflation of the
heared at the apex. balloon to dilate the orifice.
40   Mastering the BDS IIIrd Year  (Last 25 Years Solved Questions)

–– Open valvotomy is carried out in patients where Q.8. Briefly describe sub-acute bacterial endocarditis.
balloon valvotomy is not possible or in cases  (Apr 2010, 5 Marks) (Nov 2008, 15 Marks)
with restenosis. In this procedure, the fusion of Or
the valve is loosened and calcium deposit and
thrombi are removed. Describe clinical features, investigations and manage-
• Mitral valve replacement: The mitral valve is replaced ment of sub-acute bacterial endocarditis.
when there is critical mitral stenosis and/or there is  (June 2010, 20 Marks)
associated mitral regurgitation. Replacement is also  (Sep 2008, 5 Marks) (Sep 2004, 20 Marks)
done when the mitral valve is severely distorted and Or
calcified.
Write short note on clinical features and treatment of
Q.7. Describe briefly clinical features and management of bacterial endocarditis.  (Sep 2006, 10 Marks)
aortic regurgitation. Ans. Sub-acute bacterial endocarditis is defined as infection
 (Sep 1999, 5 Marks) (Sep 2006, 5 Marks) and inflammation of inner lining of heart including heart
Ans. Aortic regurgitation is produced due to acute rheumatic valves. It also includes infection at the site congenital
carditis which is associated with other valve involvement heart anomaly.
and infective endocarditis.
Clinical Manifestations
Clinical Features ♦♦ General: Presence of nausea, fever, anorexia, weight loss,
Symptoms night sweat and weakness.
♦♦ Cardiovascular system: Tachycardia, cardiac murmur,
♦♦ In mild to moderate aortic regurgitation: conduction defect and cardiac failure.
• Often asymptomatic ♦♦ Blood vessels: Loss of peripheral pulse.
• On palpitation — pounding of heart is a common ♦♦ Central nervous system: Headache, hemiplegia or
symptom monoplegia and toxic encephalopathy.
• Symptoms of left heart failure appear but late ♦♦ Lungs: Pleuritic pain and hemoptysis
♦♦ In severe aortic regurgitation: ♦♦ Nails: Osler’s nodes, clubbing of finger and splinter
• Symptoms of heart failure, i.e., dyspnea, orthopnea hemorrhage.
are present at onset. ♦♦ Skin: Purpuric spot and petechial hemorrhage.
• Angina pectoris is frequent complaint. ♦♦ Eyes: Roth’s spot and subconjunctival hemorrhage.
• Arrhythmias are uncommon. ♦♦ Kidney: Hematuria and glomerulonephritis
♦♦ Spleen: Splenomegaly is present
Signs
♦♦ Blood: Anemia is present
♦♦ Collapsing or good volume pulse (wide pulse pressure)
♦♦ Bounding peripheral pulses Pathogenesis
♦♦ Dancing carotids (Corrigan’s sign)
Introduction of bacteria into blood stream via dental
♦♦ Capillary pulsation in nail beds (Quincke’s sign)
procedure
♦♦ Pistol shots sound and Duroziez's sign/murmur
♦♦ Head nodding with carotid pulse — de Musset's sign
♦♦ Cyanosis (peripheral, central or both) may be present Intravenous drug abuse, blood borne, following
♦♦ Pitting ankle edema may be present. surgery or through abscess or infection in body
♦♦ Tender hepatomegaly if right heart failure present.

Setting of bacteria at the site of already damaged


Management
endocardium or heart valve or septal defect
♦♦ Treatment of underlying causes like endocarditis and
syphilis.
Formation of vegetation over the area which consist
♦♦ Surgical: Replacement of aortic valve should be performed
of platelet, fibrin and micro organisms
before heart failure can develop. Serial evaluation of
end systolic dimensions should be made and surgery
considered when this exceeds 5 mm. Destruction of adjacent tissues
♦♦ Medical:
• Prophylaxis against bacterial endocarditis before and
after surgery Breaking of vegetation and its embolization into
various sites like kidney, spleen, etc.
• Therapy of heart failure if develops.
Section 1:  General Medicine  41

Investigations Antimicrobial Therapy for Infective Endocarditis

♦♦ Blood culture: In absence of recent or concurrent antibiotic


Microorganism Antibiotic regimen
therapy, the first 3 random blood cultures are positive in
most patients, and blood culture is positive by third day Strept. viridans Penicillin G (1.2 g IV at 4–6 hour interval for
in 90%. and Group D 4 weeks), then oral amoxicillin is given (6 g)
♦♦ Urine: Microscopic hematuria is the most common finding. streptococci daily for 2 to 6 weeks.
Slight albuminuria and hyaline and granular casts also Or
found. Penicillin G (1.2 g IV for two weeks) +
♦♦ Hematology: Normocytic normochromic anemia, usually Gentamicin 1 mg/kg at 8 hours interval for
mild. May be raised ESR and raised C-reactive protein. 2 weeks, then oral amoxicillin (6 g) daily for
2–6 weeks.
♦♦ Chest radiograph: May be diagnostic in right sided
If patient is allergic to penicillin:
endocarditis, with multiple shadows visible due to an
Cetazolin 2.0 g lV at 6–8 hours interval for 4
embolic pneumonia.
weeks,
♦♦ ECG: Myocardial infarction seen on ECG may be due
Or
coronary embolism, and a conduction defect may be due
Vancomycin 15 mg/kg IV at 12 hour interval
to development of an aortic root abscess. for 4 weeks.
♦♦ Echocardiography: Higher sensitivity in identifying
vegetation with transesophageal echocardiography as Enterococci or Penicillin G (1.5 to 2 g IV for 4–6 hours or
compared to transthoracic echocardiography. penicillin resistant ampicillin 2.0 g lV at 6 hours interval tor 4–6
• Vegetations: An echodense structure attached to the S. viridans weeks + Gentamicin 1 mg/kg at 8 hour interval
IV or IM for 5–6 weeks.
valve or its supporting structures, or lying in the track
If allergic to penicillin,
of a turbulent jet, which is irregular in shape.
Vancomycin 15 mg/kg IV at 12 hours interval
• Leaflet perforation is best seen as regurgitant jet on
for 4 weeks.
color flow mapping.
• Annular and periprosthetic echolucent spaces Pneumococci Penicillin G (0.6 to1.2 g IV in divided doses at
(abscesses) and fistula formation. or Group A 4–6 hours interval for 4 weeks).
♦♦ Chest X-ray: Shows evidence of cardiomegaly and heart streptococci If allergic to penicillin
failure. Cefazolin 2.0 g lV at 6 to 8 hours interval for
4 weeks.
Management S. aureus Natcillin 2.0 g IV at 4 hours interval for 4–6
weeks
Management is divided into three parts: Or
♦♦ Treatment during disease process Flucloxacillin 2.0 g IV 6 hourly for 2 weeks, then
♦♦ Prophylaxis 2.0 g orally 6 hourly for 4 weeks.
♦♦ Indication for cardiac surgery.
If allergic to penicillin
Cefazolin 2.0 g lV at 6 to 8 hours interval for
Treatment During Disease Process 4 weeks.
Or
♦♦ It is mainly antimicrobial treatment. Along with source
Vancomycin 15 mg/kg IV at 12 hour interval
of infection symptoms are removed as soon as possible. for 4 weeks.
♦♦ Antibiotics should preferably be bactericidal.
♦♦ Antibiotics should be administered parenterally to HACEK organism Ceftriaxone 2g/day IV as a single dose for 4
achieve high serum concentration since the vegetation (rare) weeks or ampicillin + gentamicin may be used.
is avascular.
Coxiella burnettii Tetracycline 0.5 to 1g IV or orally
♦♦ Therapy is generally of prolonged duration. (Rickettsia) Or
♦♦ Selection of antibiotics should be based on culture report
Rifampicin (600 mg 12 hourly orally) +
and minimum inhibitory concentration (MIC) values. doxycycline (100 mg 12 hourly orally).
♦♦ Empirical therapy may be initiated in acute severe
cases after drawing blood samples for culture. The
Prophylaxis Against Infective Endocarditis
antibiotics are later changed based on sensitivity reports,
if necessary. Patients with valvular and congenital heart disease who
♦♦ Treatment of infective endocarditis should be prompt and are at high or moderate risk of endocarditis should receive
adequate. The list of antibiotics commonly used, their prophylactic antibiotics before undergoing any procedure which
dosage and indications are given below in table. may cause bacteremia.
42   Mastering the BDS IIIrd Year  (Last 25 Years Solved Questions)

Antibiotic regimen for prophylaxis of endocarditis in adults ♦♦ Genitourinary and gastrointestinal tract procedures:
at moderate or high-risk is as follows: • In high risk patients: Ampicillin 2g IM or IV +
♦♦ Oral cavity, respiratory tract, or esophageal procedures gentamycin 1.5 mg/kg IV or IM within 30 min of
(in patients at high-risk, administer a half dose after the starting the procedure followed by ampicillin
initial dose). • In high-risk patients allergic to penicillin: Vanacomycin
• Standard regimen: Amoxicillin 2.0 g oral 1 hour before 1g IV over 1 to 2 hour + gentamycin 1.5 mg/kg IV or
procedure. IM combination is given within 30 min of starting the
• Inability to take oral medication of standard regimen: procedure.
Ampicillin 2.0 g IV or IM within 30 min of procedure. • In moderate risk patients: Amoxicillin 2.0 g oral 1 hour
• If patient is allergic to penicillin: before procedure Or ampicillin or amoxicillin 2.0 g IV
Clarithromycin 500 mg or azithromycin 500 mg orally or IM within 30 min of procedure.
l hour before procedure.
• In moderate risk patients allergic to penicillin: Vancomycin
Or
1g IV over 1 to 2 hours completed within 30 min of
Cephalexin or cefadroxil 2 g orally 1 hour before
starting procedure.
procedure
Or
Indication for Cardiac Surgery
Clindamycin 600 mg oral l hour before procedure
• If patient is allergic to penicillin and is unable to take ♦♦ Heart failure due to valve damage
oral medication ♦♦ Failure of antibiotic therapy, i.e. in fungal endocarditis
Clindamycin 600 mg I.V. 30 min before procedure. ♦♦ Large vegetation on left sided heart valves with evidence
Or or high-risk of systemic emboli.
Cefazolin 1.0 g I.V. or IM 30 min before procedure. ♦♦ Abscess formation

Fig. 8:  Subacute bacterial endocarditis

Q.9. Outline the management of subacute bacterial endo- Treatment During Disease Process
carditis. (Mar 1999, 5 Marks) ♦♦ It is mainly antimicrobial treatment. Along with source
Ans. Management is divided into three parts: of infection symptoms are removed as soon as possible.
1. Treatment during disease process ♦♦ Antibiotics should preferably be bactericidal.
2. Prophylaxis ♦♦ Antibiotics should be administered parenterally to achieve
3. Indication for cardiac surgery. high serum concentration since the vegetation is avascular.
Section 1:  General Medicine  43

♦♦ Therapy is generally of prolonged duration. Prophylaxis Against Infective Endocarditis


♦♦ Selection of antibiotics should be based on culture report
and minimum inhibitory concentration (MIC) values. Patients with valvular and congenital heart disease who
♦♦ Empirical therapy may be initiated in acute severe cases are at high or moderate risk of endocarditis should receive
after drawing blood samples for culture. The antibiotics prophylactic antibiotics before undergoing any procedure which
are later changed based on sensitivity reports, if necessary. may cause bacteremia.
♦♦ Treatment of infective endocarditis should be prompt and Antibiotic regimen for prophylaxis of endocarditis in adults
adequate. The list of antibiotics commonly used, their at moderate or high-risk is as follows:
dosage and indications are given in table. ♦♦ Oral cavity, respiratory tract, or esophageal procedures
Antimicrobial Therapy for Infective Endocarditis (in patients at high-risk, administer a half dose after the
initial dose).
Microorganism Antibiotic regimen • Standard regimen: Amoxicillin 2.0 g oral 1 hour before
Strept. Viridans Penicillin G (1.2 g IV at 4 to 6 hours interval procedure.
and Group D for 4 weeks), then oral amoxicillin is given (6
• Inability to take oral medication of standard regimen:
streptococci g) daily for 2 to 6 weeks.
or
Ampicillin 2.0 g IV or IM within 30 min of proce­
Penicillin G (1.2 g IV for two weeks) + dure.
Gentamicin 1 mg/kg at 8 hours interval for • If patient is allergic to Penicillin:
2 weeks, then oral amoxicillin (6 g) daily for Clarithromycin 500 mg or azithromycin 500 mg orally
2–6 weeks l hour before procedure.
If patient is allergic to penicillin: Or
Cetazolin 2.0 g lV at 6–8 hours interval for 4 Cephalexin or cefadroxil 2 g orally 1 hour before
weeks,
procedure.
Or
Or
Vancomycin 15 mg/kg IV at 12 hour interval
for 4 weeks. Clindamycin 600 mg oral l hour before procedure.
Enterococci or Penicillin G (1.5 to 2 g IV for 4–6 hours or
• If patient is allergic to penicillin and is unable to take
penicillin resistant ampicillin 2.0 g lV at 6 hours interval tor 4 to 6 oral medication.
S. viridans weeks + Gentamicin 1 mg/kg at 8 hours interval Clindamycin 600 mg IV 30 min before procedure.
IV or IM for 5 to 6 weeks Or
If allergic to penicillin: Cefazolin 1.0 g I.V. or IM 30 min before procedure.
Vancomycin 15 mg/kg IV at 12 hours interval ♦♦ Genitourinary and gastrointestinal tract procedures:
for 4 weeks
• In high-risk patients: Ampicillin 2g IM or IV +
Pneumococci Penicillin G (0.6–1.2 g IV in divided doses at
Gentamycin 1.5 mg/kg IV or IM within 30 min of
or Group A 4–6 hours interval for 4 weeks).
streptococci starting the procedure followed by ampicillin
If allergic to penicillin:
• In high-risk patients allergic to penicillin: Vancomycin
Cefazolin 2.0 g lV at 6 to 8 hours interval for
4 weeks. 1g IV over 1 to 2 hour + gentamycin 1.5 mg/kg IV or
S. aureus Nafcillin 2.0 g IV at 4 hours interval for 4–6 IM combination is given within 30 min of starting the
weeks procedure.
Or • In moderate risk patients: Amoxicillin 2.0 g oral 1 hour
Flucloxacillin 2.0 g IV 6 hourly for 2 weeks, then before procedure
2.0 g orally 6 hourly for 4 weeks. Or Ampicillin or amoxicillin 2.0 g IV or IM within 30
If allergic to penicillin min of procedure.
Cefazolin 2.0 g lV at 6–8 hours interval for 4 • In moderate risk patients allergic to penicillin: Vancomycin
weeks.
1g IV over 1 to 2 hours completed within 30 min of
Or
starting procedure.
Vancomycin 15 mg/kg IV at 12 hours interval
for 4 weeks.
Indication for Surgery
HACEK organism Ceftriaxone 2g/day IV as a single dose for 4
(rare) weeks or ampicillin + gentamicin may be used. ♦♦ Heart failure due to valve damage
Coxiella burnettii Tetracycline 0.5 to 1g IV or orally ♦♦ Failure of antibiotic therapy, i.e. in fungal endocarditis
(Rickettsia) or ♦♦ Large vegetation on left sided heart valves with evidence
Rifampicin (600 mg 12 hourly orally) + or high-risk of systemic emboli.
doxycycline (100 mg 12 hourly orally). ♦♦ Abscess formation
44   Mastering the BDS IIIrd Year  (Last 25 Years Solved Questions)

Q.10. Write short note on clinical signs of SABE. ♦♦ Auscultation: Presence of loud first heart sound, opening
 (Mar 2000, 5 Marks) snap and mid diastolic low pitched rumbling murmur best
Ans. heared at the apex.
♦♦ Signs of raised pulmonary capillary pressure: Pleural effusion,
crepitation, pulmonary edema.
♦♦ Signs of pulmonary hypertension: RV heave, Loud P2
♦♦ Others: Basal crackels, ascites and pleural effusion

Diagnostic Features

Investigations
♦♦ ECG:
• Right ventricular hypertrophy
• Left atrial hypertrophy
*
♦♦ X-ray chest:
• Prominent left atrial appendage may be seen in left
border of heart between pulmonary artery and left
ventricle. It indicates left atrial enlargement.
• Double shadow of enlarged left atrium on right side
of spine.
• Signs of pulmonary venous congestion.
♦♦ Echocardiogram:
• Show thick immobile mitral cusp
Fig. 9:  Subacute bacterial endocarditis • Decreased diastolic filling of left ventricle
♦♦ Cardiovascular system: Tachycardia, cardiac murmur, • Decreased valve orifice area
conduction defect and cardiac failure. • Left atrial thrombus, if it is present.
♦♦ Blood vessels: Loss of peripheral pulse ♦♦ Cardiac catheterization: is used to assess valvular lesions
♦♦ Central nervous system: Headache, hemiplegia or and to detect coronary artery disease.
monoplegia and toxic encephalopathy ♦♦ Doppler:
♦♦ Lungs: Pleuritic pain and hemoptysis • Pressure gradient across mitral valve
♦♦ Nails: *Osler’s nodes, clubbing of finger and splinter • Pulmonary artery pressure
hemorrhage. • Left ventricular function
♦♦ Skin: Purpuric spot and petechial hemorrhage. Q.12. Write short note on coarctation of aorta. 
♦♦ Eyes: Roth’s spot and subconjunctival hemorrhage.  (Mar 2000, 5 Marks)
♦♦ Kidney: Hematuria and glomerulonephritis
Ans. Coarctation of aorta is defined as the narrowing of aorta
♦♦ Spleen: Splenomegaly is present
at or distal to the subclavian artery.
♦♦ Blood: Anemia is present
Q.11. Enumerate clinical and diagnostic features of mitral
stenosis. (Feb 1999, 5 Marks)
Ans.

Clinical Features
♦♦ Patient complains of breathlessness and fatigue on
exertion.
♦♦ Progression of stenosis lead to dyspnea on rest and even
have orthopnea and paroxysmal nocturnal dyspnea.
♦♦ Acute pulmonary edema can also occur.
♦♦ Hemoptysis can be present due to rupture of pulmonary
congestion and pulmonary embolism and cough due to Fig. 10:  Coarctation of aorta
pulmonary congestion.
♦♦ Chest pain is present due to pulmonary venous Clinical Features
hypertension. ♦♦ In uncomplicated cases the only symptoms are *intermittent
claudication, pain, weakness and dyspnea on running.
Signs
♦♦ Headache and nose bleed
♦♦ Atrial fibrillation is present. ♦♦ Hypertension in upper limb
Q12. *Intermittent claudication= Cramping or pain in the leg muscles brought on by the predictable amount of walking and is relieved by the rest.
Section 1:  General Medicine  45

♦♦ Physical examination shows weak or impalpable femorals ♦♦ ECG may show right ventricular hypertrophy
in comparison to strong radial acting pulsation. ♦♦ Second sound may show delayed splitting
♦♦ Heart size remains normal with left ventricular forcible ♦♦ In X-ray chest boot shaped heart is seen
apex. ♦♦ Echocardiography shows that aorta is not juxtaposed with
♦♦ On auscultation: intraventricular septum.
• S1 is accentuated
• S2 normal splitting delayed A2 Complications
• S3 with left ventricular filling ♦♦ Syncope
• S4 with hypertension. ♦♦ Cerebral abscess
♦♦ Stroke due to cerebral thrombosis
Investigation
♦♦ Subacute infective endocarditis
♦♦ ECG: It shows left axis deviation representing left ♦♦ Sudden death
ventricular failure.
♦♦ X-ray: Normal sized heart with prominent ascending aorta. Treatment
Barium swallow shows characteristic E sign.
It is divided into two stages:
♦♦ Aortography: Show usually short narrow segment.
1. Management of anoxic spells
Treatment 2. Definitive treatment.
♦♦ Medical management consists of control of congestive Management of Anoxic Spells
cardiac failure in infancy.
♦♦ Knee chest position
♦♦ Definitive management is operative.
♦♦ Humidified oxygen / moist oxygen
♦♦ Operation can be done at any age, but lowest risk is
♦♦ Morphine: 0.1 to 0.2 mg / kg subcutaneous inj.
between 1–10 years.
♦♦ Correction of acidosis if ph < 7.1 by giving sodium
♦♦ Resection of narrow segment is done in operation.
bicarbonate IV
Q.13. Write short note on tetralogy of Fallot.  ♦♦ Propranolol 0.1 gm / kg IV during spells and then 0.5 to 1
 (Feb 1999, 5 Marks) mg / kg 4 to 6 hourly orally.
 (Mar 2008, 5 Marks) (Apr 2008, 5 Marks) ♦♦ Vasopressor: Methoxamine IM or IV
Ans. It is the commonest cyanotic congenital heart disease in ♦♦ Correct anemia
children above the age of 2 years. It is characterized by ♦♦ Consider operation.
four constituents:
• Ventricular septal defect. Definitive Treatment
• Pulmonary stenosis
• Overriding or dextroposed aorta It is operative. Operative treatment is of two varieties, i.e.
• Right ventricular hypertrophy. papillative and definitive.

Clinical Features
♦♦ The chief complaint is development of *anoxic spells. The
anoxic spells are dangerous for children and can occur
many times.
♦♦ It occurs prominently after walking up or following
exertion.
♦♦ Children start crying, become dyspneic, blue than before
and may loose consciousness. Convulsions may occur.
♦♦ Frequency varies from once in a few days to numerous
attacks everyday.
♦♦ Each spell is life-threatening.

Physical Signs
♦♦ Cyanosis
♦♦ Clubbing of finger and toes
♦♦ Growth is stunted
♦♦ Ejection systolic murmur is present at pulmonary area. Fig. 11:  Tetralogy of Fallot (For color version, see Plate 1)

Q13. *Anoxic spells= Paroxysmal attacks of dyspnea.


46   Mastering the BDS IIIrd Year  (Last 25 Years Solved Questions)

Q.14. Outline the management of congestive cardiac failure. thoratic are considered when medical management
 (Mar 1997, 7.5 Marks) fails. They are usually used as a bridge to cardiac
Ans. The management of cardiac failure aims to restore transplant.
balance between metabolic demands of the body and Q.15. Describe the management of acute left ventricular
person heart’s ability to meet these demands. failure. (Sep 1998, 5 Marks)
• Rest: Complete bed rest is the key stone of
Ans. Management
management. When the patient is dyspneic, bed
First aim of treatment is to find and remove the precipitating
rest is given with the head end of bed raised to 45°.
cause, i.e. arrhythmia or an intercurrent infection.
The legs should be kept below the pelvis to prevent
♦♦ Patient should be kept in sitting position, with legs
the fluid present in legs to return to vascular system
hanging along side of the bed, i.e. upright posture should
and precipitate pulmonary edema.
be maintained.
• Diet: Basic aim is to restrict sodium in the diet.
♦♦ Diet: Salt free diet is given till left ventricular failure
Quantity of salt intake per day should not exceed
improves and later on restricted salt diet is given.
0.5 gm. Salt substitutes may be used to make diet
♦♦ Sedatives: Morphine should be given in doses of 5 to 10 mg
more palatable.
along with an anti-emetic, i.e. metaclopramide 10mg I.V.
• Diuretics: In cardiac failure there is always sodium
and repeat the drug as soon as desired.
and water retention. Hence, diuretics are given
♦♦ Oxygen: About 60% of oxygen is given by facemask under
to increase sodium extraction. Furosemide 40 to
positive pressure. It should be given as 6 to 8 L/min
80 mg orally produces effect in 4 to 6 hours and
through Wolfe’s bottle.
on IV administration of furosemide 40 to 100 mg
♦♦ Loop diuretics: Furosemide 40 to 100 mg IV should be given.
produces its effect in 20 minutes. Spironolactone
♦♦ IV sodium nitroprusside 20 to 30µg/min or IV nitroglycerin
which is potassium sparing diuretic is given 25 mg
should be given in patients whose systolic blood pressure
orally 4 times a day which removes the symptoms is more than 100 mm Hg.
like hypokalemia due to action of furosemide. ♦♦ Digitalis: If digoxin is not used previously, the three fourth
Triamterene or amiloride hydrochloride is given of full dose, i.e. 0.5 to 1mg is given as IV dose.
along with spironolactone. ♦♦ Bronchodilators: Attimes aminophylline or theophylline
• Digitalis: It increases the force of myocardial in dose of 250 to 500 mg I.V. decreases brochoconstriction.
contraction and decreases work of heart. The ♦♦ In cases of severe left ventricular failure inotropes can be
commonly used drug is digoxin which is administered given.
orally 0.25 mg BD. For rapid digitalization digoxin ♦♦ If all the above measures failed then rotating tourniquet
0.5–0.75 mg is given slow IV over a period of 5 to 10 should be applied to extremities.
minutes under electrocardiographic control. ♦♦ Intra-aortic balloon counterpulsation: It is used in acute left
• Sympathomimetic amines: Dopamine at low doses of ventricular failure during cardiac procedures or cardiac
3 to 5 μg / kg / min increases contractility of heart. repairs.
• Vasodilators: Sodium nitroprusside in the dose of 5
to 10 μg / min have balance dilator effect. Besides this Q.16. Write a short note on systemic hypertension. 
hydralazine, nitrates and ACE inhibitors are used.  (Mar 2008, 5.5 Marks) (Feb 2006, 5 Marks)
• Inodilator levosimendan: This is a calcium channel Or
sensitizer. It has positive inotropic and vasodilator
Write about clinical features, etiology, complications
effect. It is given IV with loading dose of 6–12µg/kg/
and management of systemic hypertension.
min over 10 min followed by infusion 0.05 to 2 µg/
 (Sep 2005, 20 Marks)
kg/min infusion.
• Oxygen: It is given via Woulfe’s bottle at rate of 5 to Or
8 lts/min. Describe briefly on secondary hypertension.
• Miscellaneous drugs: Tranquilizers such as diazepam  (Apr 2015, 4 Marks)
2 to 5 mg TDS are given to reduce anxiety.
• Cardiac re-synchronization therapy or biventricular Or
pacing: It is used in patients with symptomatic Describe the etiology, clinical features and manage-
refractory cardiac failure with conduction ment of hypertension. (Feb 2013, 15 Marks)
abnormality or left bundle branch block. This
Or
therapy involves pacing the right atrium, right
ventricle and left ventricle to improve synchrony of Write diagnosis and management of hypertension
the cardiac chambers.  (Jan 2017, 12 Marks)
• Left ventricular assist device: Devices such as intra- Ans. Systemic hypertension is defined as systolic pressure
aortic balloon pump, impella device, heart-mate, equal to or above 140 mm Hg and diastolic pressure
Section 1:  General Medicine  47

equal to or above 90 mm Hg measured on two or more Complications


different occasions. ♦♦ Central nervous system complications:
• Transient cerebral ischemic attacks
Etiology
• Cerebrovascular accidents (strokes)
♦♦ Renal disorders:
• Subarachnoid hemorrhage.
• Renovascular stenosis
• Hypertensive encephalopathy is characterized by very
• Pa r e n c h y m a l r e n a l d i s e a s e , p a r t i c u l a r l y
high blood pressure, it is characterized by transient
glomerulonephritis disturbances in speech and vision, paresthesias,
• Polycystic kidney disease convulsion, disorientation, loss of consciousness
♦♦ Endocrinal disorders: and papilledema. The neurological symptoms are
• Pheochromocytoma reversible as blood pressure comes under control.
• Cushing’s syndrome ♦♦ Ophthalmic complications:
• Primary hyperaldosteronism • Hypertensive retinopathy: In this there is thickening of
• Aeromegaly the walls of the retinal arterioles, diffuse or segmental
• Hypothyroidism and hyperthyroidism narrowing of blood columns, varying width of the
♦♦ Drugs: light reflex from vessel walls, arteriovenous nipping
• Oral contraceptives retinal hemorrhages, soft and hard exudates and
• Corticosteroids papilledema. Severe retinopathy can cause visual field
• Sympathomimetic drugs defects and blindness.
• Cyclosporine ♦♦ Cardiovascular complications:
• NSAIDs • Coronary artery disease (angina, myocardial
♦♦ Miscellaneous: infarction)
• Toxemia of pregnancy • Left ventricular failure
• Coarctation of aorta • Aortic aneurysm
• Aortic dissection
• Raised intracranial pressure
♦♦ Renal complications:
• Obstructive sleep apnoea.
• Proteinuria
• Progressive renal failure
Clinical Features
♦♦ Malignant hypertension: It is a clinical syndrome of markedly
♦♦ Majority of patients remain asymptomatic and are high blood pressure with retinal hemorrhages and
diagnosed on routine clinical examination. exudates. It also includes confusion, headache, vomiting,
♦♦ Common symptoms are occipital headache, dizziness, visual disturbances and renal function deterioration.
palpitation and fatigue.
Diagnosis
♦♦ Some of the patients may also present with symptoms
which are related to target organ damage like epistaxis, Diagnosis is based on the investigations and clinical features
hematuria, blurred vision, angina and breathlessness. Investigations
♦♦ Various symptoms pertaining to underlying cause can be ♦♦ Basic investigations which are done in all patients:
present such as weight gain (Cushing’s syndrome), weight • Urine examination for protein, blood and glucose
loss (thyrotoxicosis), episodic headache, palpitation, and • Serum creatinine and blood urea nitrogen for assessing
sweating (pheochromocytoma) the renal functions
♦♦ History taking should include age, sex, occupation, • Serum electrolytes, i.e. serum sodium and serum
lifestyle of patient along with history of smoking, diabetes potassium
mellitus, hyperlipidemia, alcohol and drug intake and • Hematocrit
presence of hypertension in all family members. • Fasting and post prandial blood glucose for
♦♦ Physical examination: Presence of truncal obesity (Cushing’s hyperglycemia
• Serum potassium for hypokalemic alkalosis or diuretic
disease), palpable kidneys (polycystic kidneys),
therapy
radiofemoral delay (coarctation of aorta), recurrent back
• Plasma urea assessment
pain, undiagnosed fever and recurrent urinary infections
• Complete serum lipid profile
suggest (chronic pyelonephritis) abdominal bruit • ECG for left ventricular hypertrophy or ischemia
(renovascular), may help in identifying the secondary • Chest X-ray for assessing the cardiomegaly or heart
cause of hypertension. failure.
♦♦ The signs of complications of hypertension such as ♦♦ Special investigations to screen for special group of patients
heaving apex, 4th heart sound, loud aortic second heart • X-ray chest and echocardiography
sound, pulmonary crackles, and retinal changes may also • Intravenous pyelogram and renal ultrasound, if renal
be present. disease is suspected (polycystic disease)
48   Mastering the BDS IIIrd Year  (Last 25 Years Solved Questions)

• Renal arteriography for renal artery stenosis, if it is ♦♦ Beta-blockers: Propranolol was used extensively as
suspected effective antihypertensive drug. The dose administered
• 24-hour urine catecholamine for pheochromocytoma, as 40–160 mg/day.
if it is suspected ♦♦ Calcium channel blockers: Nifedipine 16 to 20 mg BD
• Plasma renin activity and aldosterone for Conn’s is administered. Side effects are headache, dizziness,
syndrome, if it is suspected flushing. Drug is contraindicated in acute myocardial
• Urinary cortisol and dexamethasone suppression test infarction, cardiogenic shock, pregnancy and is used with
for Cushings syndrome, if it is suspected caution in diabetics and edema.
• Angiography/MRI for coarctation of aorta, if it is Felodipine sulfate 5 to 10 mg/day is effective in mild to
suspected. moderate hypertension.
Amlodipine 5 to 10 gm is effective and is safe in
Management
hypertensives.
Refer to Ans 18 of same chapter. ♦♦ ACE inhibitors:
• Enalapril maleate 5 mg OD is given.
Q. 17. Enumerate the complications and outline the manage- • Captopril in combination with diuretics or a beta-
ment of systemic hypertension.  (Mar 2000, 5 Marks) blocker. 25 gm TDS of captopril is administered. It is
Ans. Refer to Ans 16 of same chapter. considered safe in asthmatics and diabetics.
• Ramipril 2.5 mg daily is used to reduce hypertension.
Q.18. Outline the management of essential hypertension. ♦♦ Alpha-1 adrenergic blockers: Prazosin is commonly used
 (Feb 1999, 5 Marks) drug. Dose is 0.5 mg TDS.
Or ♦♦ Vasodilators: Diazoxide and sodium nitroprusside are
Describe briefly management of hypertension. given as IV dosage.
 (Feb/Mar 2004, 5 Marks) Q. 19. Describe briefly management of acute myocardial
Ans. infarction. (Sep 1998, 5 Marks)
Management Or
Outline the management of acute myocardial infarc-
Nonpharmacological Treatment tion. (Mar 2010, 5 Marks) (Feb 1999, 5 Marks)
♦♦ Salt restriction: Modest sodium restriction 2.4 gm sodium Ans. The management of myocardial infarction is divided
or 6 gm NaCl is effective in controlling hypertension. into two parts:
♦♦ Weight reduction: In overweight persons, reduction of 1. Early management.
1 kg may reduce 1.6 to 1.3 mm Hg of blood pressure 2. Late management.
♦♦ Stop smoking: Smoking raises BP. It is an independent and
Early Management
most important reversible coronary risk factor.
♦♦ Diet: ♦♦ Aspirin 150–300 mg to be chewed earliest.
Lacto-vegetarian diet and high intake of poly unsaturated ♦♦ Sublingual glyceryl trinitrate 0.4–1 mg, to be repeated, if
fish oils have high potassium levels and lower blood necessary
pressure by: ♦♦ Oxygen through nasal cannula at a rate of 2–4 L/min.
• Increased sodium extraction ♦♦ Procure IV line and take blood samples for glucose, lipids
• Decreased sympathetic activity and complete haemogram.
• Decreased rennin-angiotensin secretion and direct ♦♦ Record 12-lead ECG.
dilatation of removal of renal arteries ♦♦ Pain may be relieved by IV morphine (5 mg) plus
• Adequate calcium and magnesium intake should be metoclopramide as an antimetic (10 mg).
maintained in diet ♦♦ IV beta-blockers (metoprolol 5 mg every 2–5 minutes
♦♦ Limit of alcohol intake is done for 3 doses) for ongoing chest pain, hypertension and
♦♦ Various forms of relaxation like yoga, biofeedback and tachycardia provided there is no contraindication.
psychotherapy lower the blood pressure ♦♦ Thrombolysis should be done.
♦♦ Regular exercise, relaxation exercise. ♦♦ If PTCA is planned, give GP IIb/IIIa inhibitor
♦♦ After admission immediately shift the patient to ICU or
Pharmacological Therapy ICCU

♦♦ Diuretics: Commonly used diuretics are hydrochloro­ In Hospital Treatment


thiazide 100 mg per day, chlorthalidone 50–100 mg/
day. The reduced potassium levels in body results in Hospitalization and Strict Bed Rest
hypokalemia, potassium supplements have to be given in ♦♦ Hospitalize the patient and advice strict bed rest. As early
people on long-term diuretic therapy. Potassium sparing as the patient is hospitalized, the better it is.
diuretics, i.e. spironolactone 25 mg TDS is given. ♦♦ Patient should be strictly admitted in ICCU.
Section 1:  General Medicine  49

Analgesia Beta Adrenergic Blockers


♦♦ IV morphine sulphate 10 mg and an anti emetic, i.e. Acute beta-adrenoreceptors blockade intravenous atenolol
cyclizine 50 mg is given through I.V. cannula. (5–l0 mg given over 5 minutes) or metoprolol (5–15 mg given
♦♦ The drug is repeated depending on the response till over 5 minutes) relieves pain, reduces arrhythmias, salvages
complete analgesia is received and patient feels better. myocardium and improves short term mortality in patients
who present within l2 hours of onset of pain.
Anti-platelet Drugs
Low dose aspirin, i.e. 75 to 150 mg and clopidogrel 300 mg stat Nitrates and Other Agents
and then 75 mg orally daily is given. Sublingual glyceryl trinitrate 0.4 mg is useful in threatened
infarction.
Oxygen Therapy
Inhalation of oxygen increases arterial partial pressure of Sedatives
oxygen, so there is increase in the concentration of oxygen Diazepam 5mg for three to four times a day is effective. It is
gradient which is responsible for diffusion of oxygen in ischemic given for few days.
myocardium from adjacent better perfused areas. This is given
by facemask or nasal prongs for a day or two after infarction. Diet
♦♦ For first 4 to 5 days low calorie diet which is divided into
Thrombolysis
multiple meals is given.
♦♦ Coronary thrombolysis helps to restore patency of ♦♦ If heart failure is present restrict the salt.
coronary artery preserves left ventricular function and ♦♦ From second week, food should be increased in amount.
improves survival.
♦♦ The choice of drug for thrombolysis is less important than Lipid Lowering Agent
the speed of treatment. Atorvastatin is given to reduce the LDL levels.
♦♦ Streptokinase, 1.5 million units in 100 mL of saline given in
an intravenous infusion over 1 hour, is a widely accepted Late Management
method. It is a cheap, but being antigenic, sometimes, may ♦♦ Risk stratification and further investigations: Prognosis of
cause serious allergic manifestations. Hence, it can be used patient survived after myocardial infarction depends on
once and therapy is changed if the patient requires second degree of myocardial damage, any residual ischemia and
thrombolysis within few years. presence of ventricular arrhythmias.
Alteplase (Tissue Plasminagen Activator (tPA) ♦♦ Life style modification:
• Stop smoking
♦♦ It is a potent fibrinolytic drug but is expensive. • Diet control
♦♦ It is less antigenic and does not cause hypotension. • Regular exercise.
♦♦ The current tPA regimen given over 90 minutes (bolus dose ♦♦ Secondary prevention
of 15 mg followed by 50 mg over 30 minutes and then 35 • Antiplatelet agents, i.e. aspirin
mg over next 60 minutes) is widely accepted. The other • Lipid lowering agents
drugs include reteplase given in double dose regime i.e. • Beta-blockers and ACEI in congestive cardiac failure
10 million units over 2–3 minute followed by another dose and hypertension.
of 10 million units after 30 minutes. Tenectaplase given as • Control of blood sugar in diabetes.
bolus dose of 53 mg/kg over 10 seconds. Both reteplase and ♦♦ Rehabilitation and after care:
tenectaplase are known as bolus fibrinolytics. • Physical activities should be restricted for 4 to 6 weeks
Angioplasty since infarct takes 4–6 weeks to become replaced with
fibrous tissue.
Immediate angioplasty of infarct related artery is safe and is • Gradual mobilization and return to work over 6 weeks.
effective alternative to thrombolysis. It is done in the patients • When there are complications, the regimen has to be
in whom the hazards of thrombolysis is high. modified accordingly.
• Exercise within the limits set by angina and tiredness
Anti-coagulants
will do no harm but much good. Same limits apply
♦♦ Sub-cutaneous heparin, i.e. 7,500 units twice a day for 7–10 to sexual activity.
days or till discharge of patient from the hospital can be • Control of obesity, regular exercises, cessation of
employed. In patients who do not receive thrombolytic smoking, adoption of a less frenetic way of life and
therapy to prevent venous thrombosis. control of plasma lipids by diets and drugs.
♦♦ Patients who receive thrombolytic therapy (tPA) should • Complications should be managed. Pain relief,
receive immediate and full doses of heparin (10,000 U reassurance, rest and correction of hypokalemia play
bolus plus, l000 U hourly). a major role in prevention of arrhythmias.
50   Mastering the BDS IIIrd Year  (Last 25 Years Solved Questions)

Routine Drug Therapy and peculiar facial expression. Pain is described as


tightness, heaviness and constriction.
♦♦ Low dose aspirin, i.e. 75 to 150 mg daily and is continued
• Anxiety: It is common and occurs when there is fear
indefinitely.
of impending death.
♦♦ Beta-adrenergic blocker should be given for 4 to 6 weeks
• Nausea and vomiting.
if there is no contraindication.
• Breathlessness due to fall in cardiac output.
♦♦ ACE inhibitor, i.e. captopril 25 mg TID or ramipril 2.5 to
• Collapse or syncope due to arrhythmia and profound
5 mg BD
hypertension.
♦♦ Lipid lowering agent atorvastatin is given to lower the
♦♦ Signs:
LDL levels.
• Signs of sympathetic activation
Q.20. Enumerate the complications and outline the manage-
–– Pallor
ment of acute myocardial infarction.
–– Sweating
 (Mar 2000, 5 Marks)
–– Tachycardia.
Ans.
• Signs of vagal stimulation
Enumeration of Complications of Myocardial Infarction –– Vomiting
♦♦ Immediate –– Bradycardia.
• Arrhythmias and conduction disturbances • Signs of impaired myocardial function
–– Sinus bradycardia –– Hypotension and oligouria
–– Ventricular ectopics –– Narrow pulse pressure
–– Ventricular tachycardia –– Raised jugular venous pressure
–– Ventricular fibrillation –– Third heart sound
–– Idioventricular rhythm –– Quite first heart sound
–– Atrial fibrillation –– Lung crepitations.
–– Heart block • Signs of tissue damage: Fever and arrhythmia
• Post – myocardial angina can occur in 50% patients • Signs of complications:
occur. –– Due to mitral regurgitation
• Acute circulatory failure. –– Due to pericarditis.
• Pericarditis –– Ventricular ectopic beats
• Mechanical complications include: –– Ventricular tachycardia
–– Papillary muscle dysfunction or rupture –– Heart blocks.
–– Rupture of interventricular septum
Diagnosis
–– Rupture of ventricle
• Mural thrombosis and embolism Diagnosis of acute myocardial infarction is based on history,
• Sudden death characteristics symptoms and signs and investigations.
♦♦ Late complications: Investigations
• Post-myocardial infarction syndrome (Dressler’s
syndrome) ♦♦ Electrocardiography:
• Ventricular aneurysm. • ECG is the specific method for confirming the
diagnosis.
Management • Typical changes are seen in leads which faces the
infracted area. These changes are:
Refer to Ans 19 of same chapter. –– Elevation of ST-segment
Q.21. Describe the clinical features, diagnosis and treatment –– Pathologic Q-waves appear.
of acute myocardial infarction. (Mar 2001, 15 Marks) –– T waves may become tall and peaked in very early
Or myocardial infarction. T waves are transient and
last for a few hours only.
Describe the clinical features of acute myocardial –– In contrast to transmural lesions, partial thickness
infarction. (Feb 2013, 5 Marks) or subendothelial infarction causes ST/T wave
Ans. Acute myocardial infarction is defined as irreversible changes without Q-waves or prominent ST
damage to myocardium of heart as a result of occlusive elevation.
thrombus due to rupture of atherosclerotic plaque in –– Changes in the ECG are seen which evolve in
coronary artery. predictable fashion over next few days to weeks.
♦♦ Blood test:
Clinical Features
• Plasma biochemical markers:
♦♦ Symptoms: Myocardial infarction leads to detectable rise in the
• Pain: Chest pain is more common and is severe in plasma concentration of various enzymes and proteins
comparison to angina. Pain is severe with pallor that are normally concentrated within the cardiac cells.
Section 1:  General Medicine  51

Plasma enzymes (cardiac injury enzymes) are as Types


follows: ♦♦ Stable
–– Creatine kinase (CK). ♦♦ Unstable
–– Aspartate aminotransferase (AST). ♦♦ Nocturnal
–– Lactate dehydrogenase (LDH). ♦♦ Prinzmetal’s
–– Myoglobin ♦♦ Postinfarction angina
–– Troponins (troponin I and troponin T)
• Creatine kinase starts to rise at 4–6 hours and it peaks Etiology
by about 12 hours and falls to normal in 48–72 hours.
Acute myocardial ischemia occurs when myocardial oxygen
Myocardial isoenzyme of creatine kinase is more
demand exceeds supply in following:
specific. It is useful for diagnosis of early myocardial
♦♦ Coronary atherosclerotic narrowing (most cases).
infarction.
♦♦ Non-atherosclerotic coronary artery disease—Coronary
• Aspartate aminotransferase (AST) starts to rise at
spasm, coronary thromboembolism, congenital anomalies,
about 12 hours and reaches a peak on the first or
coronary vasculitis.
second day and returns on third and fourth day.
♦♦ Valvalar heart disease — Aortic stenosis and/or
• Lactate dehydrogenase (LDH) starts to rise after 12
aortic regurgitation, mitral stenosis with pulmonary
hours, reaches a peak after 2–3 days and may remain
hypertension, mitral valve prolapse.
elevated for a week. Rise in the value of LDH I (an
♦♦ Pulmonary hypertension.
isoenzyme of LDH) is a more sensitive indicator of
♦♦ Systemic hypertension.
myocardial infarction than total LDH. It is useful in
♦♦ Hvpertrophic or dilated cardiomyopathy.
diagnosis for patients who present several days after
♦♦ Anemia—from tachycardia and reduction in O2 availability.
myocardial infarction.
• Cardiac troponins are cardiac troponin-T (cTn-T) and Precipitating Causes
cardiac troponin-I (cTn-I). Sensitivity of troponins
is similar to that of isoenzymes of creatine kinase. ♦♦ Physical exertion
Moreover, cTn-T remains elevated for 100 to 200 hours ♦♦ Heavy metal
after acute myocardial infarction and therefore, it may ♦♦ Exposure to cold
have particular utility in the evaluation of patients who ♦♦ Emotion and excitement
present sufficiently long episode after the pain in chest. ♦♦ Hyperinsulinism in diabetic patients
–– Leukocytosis with a peak on first day. ♦♦ Other causes: Straining at stools, bathing, sexual intercourse,
–– ESR is raised which may remain raised for some micturition
days.
–– C-reactive protein is elevated. Clinical Features
♦♦ Chest X-ray: It can detect acute pulmonary edema or Symptoms
congestion. It is also helpful to detect pericardial effusion,
♦♦ Anginal pain:
cardiomegaly, etc.
• Site: Most often over middle or lower sternum or over
♦♦ Radionuclide scanning: It shows site of necrosis and the
left precordium, at times in epigastrium. Sometimes
extent of impairment of ventricular function.
discomfort is located only in left shoulder or left
♦♦ Echocardiography: This is done for regional wall motion
upper arm, occasionally in lower jaw and rarely in
abnormality and ejection fraction.
interscapular area.
Treatment • Radiation of pain: May spread to right or left arm or
both neck or jaw. Occasionally, pain starts in the wrist,
Refer to Ans 19 of same chapter. upper arms or face and then spreads to the chest.
Q.22. Write short note on angina pectoris. • Character: Vice-like constriction or choking. Sometimes
 (Mar 2000, 5 Marks) only pressure or burning pain, rarely mere weakness
of one or both arms. An important characteristic is its
Or
constancy, the pain being steady while it lasts.
Describe clinical features, diagnosis and management • Duration: Most commonly l to 4 minutes. It may force
of angina pectoris. (Mar 2011, 4.5 Marks) patient to stop walking.
Ans. Angina pectoris is a symptom complex caused by • Provocation: By effort specially like walking against
transient myocardial ischemia and constitutes a clinical the wind or up a climb, hurrying after meals or
syndrome rather than a disease. unaccustomed exercise at times due to excitement
52   Mastering the BDS IIIrd Year  (Last 25 Years Solved Questions)

anger, and fear. In advanced cases, pain is provoked Management


by lying down (angina decubitus) or stooping.
It is divided into three phases:
• Relief with sublingual nitroglycerine.
♦♦ General measures.
♦♦ Dyspnea: If it occurs before the pain suggests severe
♦♦ Pharmacological treatment.
ventricular disease.
♦♦ Invasive treatment.
♦♦ Other symptoms:
• Choking sensation in throat or feeling of impending General Measures
doom.
♦♦ Do not smoke
• Belching or passage of flatus or polyuria after an
♦♦ Aim at ideal body weight
attack.
♦♦ Take regular exercise
• Dizziness, faintness or rarely syncope
♦♦ Avoid severe exertion, vigorous exercise and exercise in
• If pain is severe sweating and nausea.
cold weather
Signs ♦♦ Take sublingual nitrate before taking exertion that may
induce angina.
♦♦ At time, no signs are present.
♦♦ Signs of LV dysfunction: Atrial or third heart sound.
Pharmacological Treatment
♦♦ Dysfunction of papillary muscle: It can lead to transient
mitral regurgitation in case of ischemia. Following agents are used with successful outcome.
♦♦ Signs associated with risk factors: ♦♦ Antiplatelet agents:
• Hypertension. • Aspirin is used usually in dose of 75–150 mg daily.
• Hyperlipidemia—Arcus senilis, xanthelasma, or • Clopidogrel is used along with or without aspirin at
cholesterol deposits along tendons and in skin of dose of 75 mg daily.
palms and buttocks. ♦♦ Anti-anginal agents:
• Obesity • Sublingual glycertrinitrate effectively abort anginal
• Diabetes and its accompaniments. attack by causing coronary vasodilatation and
♦♦ During the attack—pallor and sweating with rise of BP reducing preload and cardiac output.
often tachycardia. Pressure on carotid sinus may produce • Beta-blockers improve cardiac efficiency and reduce
slowing of pulse and cessation of pain. oxygen consumption. Cardioselective agents such as
atenolol 25 to 50 mg, metoprolol 200 mg daily can
Diagnosis be used.
Investigations • Calcium-channel antagonists, i.e. amlodipine,
lacidipine. They are the vasodilators and lowers
♦♦ Resting ECG: ECG changes of myocardial ischemia are
myocardial oxygen demand by reducing blood
reflected in ST-T waves. Occassionally, there is flattening
pressure and myocardial contractility.
of T waves in some lead in patient with angina.
• Potassium-channel opener, i.e. nicorandil has atrial
♦♦ Exercise ECG or stress test: With continuous ECG monitoring
and venous dilatation property which does not exhibit
and intermittent BP recording is performed with a
tolerance.
treadmill or bicycle ergometer. Standardized protocols
are used (e.g. Bruce protocol), enabling performance to be Invasive Treatment
assessed in same patient at different times and workload
♦♦ *Percutaneous coronary *intervention or percutaneous
at onset of symptoms or ECG changes to be determined.
transluminal coronary *angioplasty is done.
An exercise ECG is abnormal, if there is horizontal or
♦♦ Coronary artery bypass grafting is done.
down-sloping ST segment depression of 0.1 mm or more
in any lead.
♦♦ Myocardial perfusion scintigraphy: The isotope cardiovascular Q.23. Outline the management of angina pectoris.
stress (usually thallium-201 or technetium—99 m) is  (Sep 1999, 5 Marks)
injected at peak exercise and images taken with a camera Ans. Refer to Ans 22 of same chapter.
immediately or shortly after exercise and compared
with rest images taken a few hours later following a Q.24. Describe briefly Marfan’s syndrome. 
second injection of tracer. Areas of myocardial ischemia  (Aug 1995, 5 Marks)
are identified by reduced isotopic uptake in the same Ans. It is a genetic metabolic disorder of connective tissue.
anatomical distribution stress images but not resting
Etiology
images (reversible defect).
♦♦ Coronary angiography: It is done before angioplasty or Disease is caused by defect in manufacture of fibrillin molecules
coronary bypass surgery. in extracellular matrix.

Q22. *Percutaneous = Effected through skin    *Intervention = One or more actions taken in order to modify an effect.
*Angioplasty = Any endovascular procedure that reopens narrow blood vessels and restores forward blood flow.
Section 1:  General Medicine  53

Clinical Features • Drugs—Digitalis, quinidine


♦♦ Patient is tall and slender. • Calcific aortic stenosis.
♦♦ The inheritance is autosomal dominant. • Trauma (penetrating).
♦♦ Extremities especially fingers and toes are long and thin. • Surgical procedures: After correction of ventricular
♦♦ Muscles are Markedly *hypotonic and joints are *hyper septal defect, or following insertion of prosthetic
extensible. valves or removal of hypertrophied septum in
♦♦ The main cause of *morbidity and *mortality is related hypertrophic cardiomyopathy.
to CVS. There is aortic aneurysm (localized abnormal • Cardiomyopathy (particularly infiltrative).
dilatation of blood vessel usually an artery due to • Syphilitic heart disease.
congenital defect or weakness in wall of vessel) and other • Infiltrative masses: Sarcoidosis, tubercles, abscesses
valve abnormalities mainly mitral incompetence. from endocarditis, gummas, tumors, amyloidosis,
♦♦ Death is due to rupture of aneurysm. hemochromatosis.
♦♦ There is presence of flat feet, *stooped shoulders and • Collagen diseases: Rheumatoid arthritis, dermatomyositis.
dislocation of optic lens. • Fistulae: Sinus of Valsalva aneurysm rupturing into
right atrium.
Management • Unknown cause: Idiopathic fibrosis.
♦♦ Beta-blockers are given to prevent *aneurysmal rupture. Clinical Features
♦♦ Treatment of complication: Aneurysm* can be operatively
treated by using dacron mesh or graft. Symptoms
♦♦ Valvular defects are treated accordingly by either repair ♦♦ Due to low cardiac output: Lassitude, fatigue, light
or replacement. headedness, and especially during exercise syncope.
Q. 25. Enumerate causes of continuous murmurs.  Symptoms of vertebrobasilar insufficiency and congestive
 (Sep 1999, 5 Marks) heart failure may be precipitated.
Ans. A continuous murmur is one which begins in systole and ♦♦ Due to increased stroke volume: Uncomfortable awareness
continues through the second heart sound into part or of heart beat, or slow palpitation, if block is intermittent.
whole of diastole. ♦♦ Due to transient circulatory arrest: Stokes-Adams attacks —
Symptoms depend on duration of standstill of circulation:
Causes of continuous murmurs are as follows:
About 5 seconds—giddiness and faintness, about 10
♦♦ Patent ductus arteriosus
seconds—convulsions. Convulsions and incontinence
♦♦ Aortopulmonary window
may suggest epilepsy, but in transient asystole pallor
♦♦ Ruptured sinus of valsalva into the right side of the heart
is often striking, patient flushes during recovery, and
♦♦ Surgically produced shunts in TOP—Blalock-Taussig shunt
consciousness is regained very rapidly; though some
♦♦ Co-arctation of aorta
permanent impairment of cerebral function may occur
♦♦ Coronary/pulmonary/systemic A-V fistula
after long or repeated episodes.
♦♦ Aortic stenosis and aortic regurgitation
♦♦ Ventricular septal defect with aortic regurgitation Signs
♦♦ Venous hum
♦♦ Slow and regular heart rate: At 30 to 50 beats per minute,
♦♦ Mammary soufflé
which does not usually increase significant with physical
♦♦ Anomalous origin of left coronary artery from pulmonary
activity or exercise.
artery
♦♦ Raised Jugular venous pressure: ‘a’ waves may be seen in the
Q.26. Describe briefly complete heart block. neck unrelated to ventricular beats.
 (Feb 2014, 4 Marks) ♦♦ Cannon waves: Giant ‘a' waves which are transmitted in the
Ans. Complete heart block is also known as third degree heart neck when the atrium contracts against a closed tricuspid
block or complete AV block valve.
♦♦ Variation in intensity of First heart sound: First heart sound
Etiology
is loudest when the interval between the preceding atrial
♦♦ Congenital: beat and the ventricular beat is short, it is faintest when
• Usually associated with ventricular septal defect, the interval is long. From time to time, there is a sharp
rarely isolated. accentuation of the first sound at the apex (cannon sound).
♦♦ Acquired: ♦♦ Tide pulse pressure: Due to increased systolic pressure and
• Rheumatic heart disease low diastolic pressure. This gives rise to water hammer
• Acute infections—rheumatic fever, diphtheria pulse and capillary pulsation.

Q24 *Hypotonic = Having abnormally low tension   *Hyper extensible = Extreme extension.
*Morbidity = State of being diseased *Mortality = The condition of subjecting the death. *Stooped = Bent forward
*Aneurysm = Localized abnormal dilatation of blood vessel usually an artery due to congenital defect or weakness in wall of vessel.
54   Mastering the BDS IIIrd Year  (Last 25 Years Solved Questions)

♦♦ Cardiac enlargement: Due to increased stroke volume, • Nonuniform pulmonary artery perfusion
hyperdynamic cardiac impulse. • Idiopathic arterial calcification in childhood.
♦♦ Systolic ejection murmur: Loudest in 2nd and 3rd left • Cyanosis
interspaces adjacent to the sternal edge, and due to • Alveolar hemorrhage syndrome with renal
increased velocity of blood flow associated with increased involvement
stroke volume. • Rheumatic fever
♦♦ Atrial sounds: They may be heard in constant relation to • Pericardial effusion
first and second heart sounds. • Ebstein anomaly.
♦♦ Apical diastolic flow murmur: It is occasional.
♦♦ ECG: There is no relation between atrial and ventricular Q.28. Describe briefly verapamil. (Sep 1999, 3 Marks)
complexes. The duration of QRS is normal. Ans. Verapamil is a Class IV anti – arrhythmic agent and an
anti – hypertensive drug.
Management
Mechanism of Action
In Acute Complete AV Block
♦♦ It leads to coronary dilatation and decreases myocardial
Acute onset of complete AV block occurs in acute myocardial oxygen consumption.
infarction, i.e. in inferior myocardial infarction and anterior ♦♦ It interferes with the inward displacement of calcium and
myocardial infarction. delays conduction within the AV node.
Complete AV Block with Inferior Myocardial Infarction Dosage
♦♦ Complete AV block occurs usually in acute inferior 40 or 80 mg TDS orally
wall infarction but are transient and less troublesome. Or
No treatment is needed in such patients who are well 5 mg to 10 mg IV paroxysmal atrial tachycardia.
and hemodynamically stable. If in such cases, clinical Side Effects
deterioration occurs, then atropine 0.6 mg IV can be given
♦♦ Constipation
as a bolus and repeated, if necessary. If this treatment fails,
♦♦ Hypotension
then a temporary pacemaker may be inserted. In most of
♦♦ Vertigo
the patients, heart block disappears under 7–10 days.
♦♦ Nervousness
Complete AV Block with Anterior Myocardial Infarction Indications
♦♦ This occurs less commonly, is dangerous and carry poor ♦♦ In supraventricular tachycardias such as atrial tachycardia
prognosis. Asystole commonly occurs in such cases and and atrial fibrillation
lead to mortality. If patients develop asystole, atropine 0.6 ♦♦ In premature heart beats
mg IV given as bolus and repeated, if needed. Temporary ♦♦ In ischemic heart disease such as angina
pacemaker is inserted immediately. Isoprenaline infusion, ♦♦ In hypertension
i.e. 1 to 5 mg in 500 mL of 5% dextrose should be started
Q.29. Describe briefly captopril. (Feb 1999, 5 Marks)
at minimum rate to produce a satisfactory heart rhythm
Ans. Captopril is an ACE inhibitor.
till temporary pacemaker is inserted.
♦♦ If block is due to drug toxicity, strictly stop the offending drug. Mechanism of Action
Chronic Complete AV block ♦♦ Drug prevent the conversion of angiotensin I into
A permanent pacemaker is indicated in patients having angiotensin II by inhibiting angiotensin converting enzyme
asymptomatic Mobitz type II complete heart block because it thus preventing action of angiotensin II, i.e. vasoconstriction
can improve their prognosis. and aldosterone synthesis and causes fall in blood pressure.
♦♦ Captopril metabolizes bradykinin which is a potent
Q.27. Enumerate the causes of massive cardiomegaly. vasodilator. It increases bradykinin levels causing
 (Feb 1999, 5 Marks) vasodilatation.
Ans. The causes of massive cardiomegaly are:
• Hypertrophic cardiomyopathy and dilated Adverse Effects
*cardiomyo­pathy It causes hypotension, hyperkalemia, dry persistent cough, loss
• Congenital insufficiency of tricuspid valve of taste sensation, urticaria, angiedema, pruritus, skin rash,
• Pediatric anesthesia agranulocytosis, proteinuria, renal insufficiency

Q27 *Cardiomyopathy = Cardiomyopathy is a chronic disease of the heart muscle (myocardium), in which the muscle is abnormally enlarged,
thickened, and/or stiffened. The weakened heart muscle loses the ability to pump blood effectively, resulting in irregular heartbeats
(arrhythmias) and possibly even heart failure.
Section 1:  General Medicine  55

Uses ♦♦ Presence of dry cough which is disturbing.


♦♦ Presence of oliguria and nocturia.
♦♦ Captopril is first line drug to be used in treatment
♦♦ Presence of tachypnea because of stiff congested lungs and
of hypertension. The captopril is more effective in
there is also presence of tachycardia because of hypoxia
renovascular hypertension.
created due to pulmonary congestion.
♦♦ It decreases preload and afterload in cardiac heart failure.
♦♦ Presence of fatigue and weakness.
♦♦ In diabetics, it improves renal function.
♦♦ Cerebral symptoms are present, i.e. altered mental state,
♦♦ Captopril is used in myocardial infarction and also causes
difficulty in concentration, memory impairment, headache,
reduction in motility.
insomnia and anxiety
Dose Physical Findings
It is given 25 mg TDS increased every week upto a maximum ♦♦ Extremities can be cold or pale.
of 450 mg/day ♦♦ There is presence of tachycardia and rapid pulse rate.
Q.30. Describe clinical features, investigation and treatment ♦♦ Presence of profuse sweating
of left ventricular failure. (Mar 2003, 15 Marks) ♦♦ Presence of tachypnea, i.e. increased respiratory rate
Or ♦♦ There is low pulse volume or pulsus alternans can be
Write short note on left ventricular failure.  present.
 (Jan 2012, 5 Marks) ♦♦ Presence of central cyanosis.
Ans. ♦♦ Third heart sound can be heard.
Left ventricular failure is defined as failure to maintain an ♦♦ There is presence of basal pulmonary rales or crackles.
effective ventricular output for a given pulomonary venous ♦♦ Presence of an expiratory wheezing.
or left atrial pressure or can do so only at the expense of an ♦♦ Presence of oliguria.
elevated left atrial filling pressure. ♦♦ Hydrothorax or pleural effusion is present.
♦♦ Presence of anxiety and depression.
Causes ♦♦ Urine can be of high specific gravity and shows proteinuria.
♦♦ Presence of massive cardiomegaly.
♦♦ Left ventricular outflow obstruction:
• Systemic hypertension Investigations
• Aortic valvular stenosis
♦♦ Electrocardiogram:
• Idiopathic hypertrophic subaortic stenosis
• Presence of left ventricular hypertrophy and left
• Coarctation of aorta
atrial hypertrophy in patients having aortic valvular
♦♦ Left ventricular inflow obstruction:
• Mitral stenosis diseases as well as mitral regurgitation.
• Left atrial myxoma • Presence of ST-T changes in patients having disease
• Endomyocardial fibrosis with stiff left ventricle of myocardium.
♦♦ Left ventricular volume overload: ♦♦ Chest X-ray:
• Mitral valve prolapsed • Presence of enlargement of cardiac shadow.
• Mitral regurgitation • Presence of pulmonary venous congestion which
• Aortic regurgitation (rheumatic and non-rheumatic) extends from hilum to periphery.
• Ventricular septal defect • There is presence of Kerley’s lines because of
• Patent ductus arteriosus interstitial edema.
• High output states ♦♦ Echocardiogram:
• Papillary muscle dysfunction • Increase in the left ventricular dimensions
♦♦ Reduced left ventricular contractility: • Left ventricular end diastolic pressure or volume of
• Cardiomyopathy predominantly involving left both can be high
ventricle • Increase in the cardiac output and stroke volume
• Anterior wall myocardial infarction • There is reduction in ejection fractions
• Left ventricle endocarditis ♦♦ Other test:
Monitor blood urea and other electrolytes
Clinical Features
♦♦ Progressive dyspnea is the earliest sign of left heart failure. Treatment
♦♦ Presence of orthopnea due to increase in venous return First aim of treatment is to find and remove the precipitating
during recumbent position. cause, i.e. arrhythmia or an intercurrent infection.
♦♦ Attacks of breathlessness which occur at night and awaken ♦♦ Patient should be kept in sitting position, with legs
the patient, i.e. paroxysmal nocturnal dyspnea. hanging alongside of the bed, i.e. upright posture should
♦♦ In severe heart failure there is a periodic respiration in be maintained.
which periods of hyperpnea alternate with apnea, i.e. ♦♦ Diet: Salt-free diet is given till left ventricular failure
Cheyne-Stokes respiration. improves and later on restricted salt diet is given.
56   Mastering the BDS IIIrd Year  (Last 25 Years Solved Questions)

♦♦ Sedatives: Morphine should be given in doses of 5 to 10 mg Inj. Digoxin (0.5. to 1 mg) is given.
along with an anti-emetic, i.e. metoclopramide 10 mg IV ♦♦ Inj. Aminophylline 0.24 gm slow IV in 10 min is effective.
and repeat the drug as soon as desired. ♦♦ Patients not responding above regimen IV nitroprusside
♦♦ Oxygen: About 60% of oxygen is given by facemask under 20–30 unit g per minute is given.
positive pressure. It should be given as 6 to 8 lts / min
through Wolfe’s bottle. Q.32. Discuss etiology, clinical features, diagnosis and man-
♦♦ Loop diuretics: Furosemide 40–100 mg IV should be given. agement of acute myocardial infarction. 
♦♦ IV sodium nitroprusside 20–30 µg/min or IV nitroglycerine  (Feb/Mar 2004, 20 Marks)
should be given in patients whose systolic blood pressure Ans. Acute myocardial infarction is defined as irreversible
is more than 100 mm of Hg. damage to myocardium of heart as a result of occlusive
♦♦ Digitalis: If digoxin is not used previously, the three fourth thrombus due to rupture of atherosclerotic plaque in
of full dose, i.e. 0.5 to 1 mg is given as IV dose. coronary artery.
♦♦ Bronchodilators: At times, aminophylline or theophylline
in dose of 250 to 500mg IV decreases bronchoconstriction. Etiology
♦♦ In cases of severe left ventricular failure, inotropes can ♦♦ The most important cause of myocardial infarction is
be given. coronary atherosclerosis.
♦♦ If all the above measures failed, then rotating tourniquet ♦♦ Obesity and hypertension
should be applied to extrimities. ♦♦ Cigarette smoking
♦♦ Intra-aortic balloon counterepulsation: It is used in acute left ♦♦ Diabetes mellitus
ventricular failure during cardiac procedures or cardiac ♦♦ Sedentary life cycle (A life cycle involving little exercise,
repairs. even of at least strenuous type)
Q.31. Describe briefly acute pulmonary edema.  ♦♦ Dyslipidemia, i.e. increased levels of LDL.
 (Mar 1998, 5 Marks) ♦♦ Hereditary susceptibility
Ans. The term acute pulmonary edema refers to collection of For clinical features and diagnosis refer to Ans 21 of same
fluid into alveoli its wall and alveolar sac due to an acute chapter.
increase in left atrial pressure.
For management refer to Ans 19 of same chapter.
Causes
♦♦ Cardiogenic: Q.33. Describe clinical diagnosis of subacute bacterial endo-
• Left ventricular failure carditis. How will you confirm diagnosis?
• Myocardial infarction  (Sep 2005, 10 Marks)
• Mitral stenosis Or
• Pulmonary infarction Write short note on diagnosis of infective endocarditis.
• Cardiac arrhythmia.  (Mar 2007, 5 Marks)
♦♦ Non-cardiogenic: Ans. For clinical diagnosis of subacute bacterial endocarditis,
• Fluid overload there is a criteria given by Duke, i.e. Duke’s criteria.
• Shock
• Inhalation of gastric acid (Mendelson’s syndrome) Duke’s Criteria for Clinical Diagnosis of SABE
• High altitude pulmonary edema.
Major Criteria
Clinical Features
♦♦ Blood culture: Positive blood culture with typical infective
♦♦ Feeling of *oppression in chest endocarditis microorganisms (Viridans streptococci, S.
♦♦ Acute and distressing dyspnea bovis, HAECK group or community acquired S. aureus or
♦♦ Short cough and pink frothy sputum, sometimes blood enterococci)
tinged fluid from mouth and nose. ♦♦ Endocardial involvement: New regurgitation murmur,
♦♦ Sweat and feeble pulse. positive ECG for SABE.
♦♦ Fall of temperature.
Minor Criteria
Management ♦♦ Predisposing cardiac or IV drug abuse.
♦♦ Bed rest is provided as the sitting position tends to reduce ♦♦ Fever
venous return. ♦♦ Vascular phenomenon: Emboli, mycotic aneurysms, petechiae
♦♦ Oxygen is provided ♦♦ Immunologic phenomenon: Glomerulonephritis and
♦♦ Inj. Morphine 15 mg subcutaneously is given. rheumatoid factor.
♦♦ Inj. Furosemide 80 mg IV relieves pulmonary edema. ♦♦ Echocardiogram: Consistent with infective endocarditis, but
♦♦ If patient has not received digoxin during last fortnight not meeting major criteria.

Q31. *Oppression= Compression


Section 1:  General Medicine  57

♦♦ Microbiology: Positive blood cultures, but not meeting ♦♦ Headache


major criteria, serological evidence of active infection with ♦♦ Cerebral hemorrhage or infarction
possible microorganisms. ♦♦ Visual disturbance
♦♦ *Hypertensive encephalopathy
Confirmation of Diagnosis ♦♦ Gastrointestinal symptoms, i.e. nausea and vomiting.
The confirmation is of three types, i.e: ♦♦ Oligurea.
♦♦ Definite:
Diagnosis
• Pathology or bacteriology of vegetations.
Or ♦♦ Complete blood count
• Both major criteria ♦♦ Coagulation profile
Or ♦♦ Electrolyte profile
♦♦ Urine output and electrolytes
• One major and three minor criteria
♦♦ TSH
Or
♦♦ Renal function test
• Five minor criteria ♦♦ Chest radiograph
♦♦ Possible: ♦♦ Head CT scan
Neither definite nor rejected ♦♦ ECG to see ischemia and infarction.
♦♦ Rejected:
• Firm alternative diagnosis Treatment
Or
Since malignant hypertension is a medical emergency, it
• Resolution on less than four days of antibiotics. requires immediate treatment.
Q.34. Describe clinical features, investigations and treatment ♦♦ Most effective agent for blood pressure reduction in
of acute rheumatic mitral stenosis.  an emergency is controlled IV infusion of sodium
 (Feb 2002, 15 Marks) nitroprusside, i.e. 0.3 to 1.0 µg/kg/min.
Ans. For clinical features, investigations and treatment of ♦♦ Alternatively diazoxide 300 mg I.V. or enalaprilat 1.25 mg
acute mitral stenosis refer to Ans 6 of same chapter. IV is given rapidly and anti-hypertensive effect is noted
Q.35. Write short note on malignant hypertension.  under 1 to 3 min. The same dose is repeated when pressure
 (Apr 2007, 5 Marks) (Feb 2002, 5 Marks) begins to elevate.
♦♦ Chewing a nefidipine l0 mg capsule is often sufficient to
Ans. Malignant hypertension is a complication of hyper­
give a graded reduction in blood pressure.
tension characterized by very elevated blood pressure
♦♦ Bed rest should be given to the patient.
and organ damage in eyes, brain, lung and kidneys.
♦♦ Sedation by IV diazepam is given.
It differs form other complication of hypertension in that ♦♦ A potent diuretic, i.e. IV frusemide 40 mg stat can also
it is accomplished by *papilledema. be given.
Systolic and diastolic blood pressure are usually greater
than 240 and 140 mm Hg. Q.36. Describe the etiology, clinical features, complica­tion
and management of acute myocardial infarction.
Malignant hypertension is an hypertensive emergency.
 (Sep 2006, 15 Marks)
Etiology Ans. For etiology refer to Ans 32 of same chapter.
For clinical features refer to Ans 21 of same chapter.
♦♦ Drugs, i.e. cocaine, beta-blockers and oral contraceptives
♦♦ Alcohol For complication refer to Ans 20 of same chapter.
♦♦ Atherosclerosis For management refer to Ans 19 of same chapter.
♦♦ Chronic diabetes mellitus Q.37. Write management of acute coronary thrombosis and
♦♦ Renal failure. its prevention. (Mar 1998, 3 Marks)
Clinical Features Ans. It is the formation of blood clot in coronary artery.

♦♦ Chest pain is present Management


♦♦ Dyspnea
♦♦ Neurological defect ♦♦ Thrombolytic agents are used, i.e. streptokinase
♦♦ Angina and myocardial infarction ♦♦ Antiplatelet drugs, i.e. heparin and aspirin
♦♦ Pulmonary edema ♦♦ Anticoagulants, i.e. warfarin

Q35. *Papilledema = Edema and inflammation of the optic nerve at its point of entrance into retina.
*Hypertensive encephalopathy = Headache, vomiting, seizure and change in mental status.
58   Mastering the BDS IIIrd Year  (Last 25 Years Solved Questions)

♦♦ Platelet glycoprotein receptor antagonist, i.e. abciximab blood pressures above normal can go undiagnosed for a long
♦♦ When a thrombus is large and life-threatening, surgical period of time.
removal is done.
Management
Prevention
Depending on one’s blood pressure and risk factors for heart
♦♦ Deep breathing exercise of patient in bed. disease, one may only need to make a few lifestyle adjustments.
♦♦ Slight elevation of foot part of bed for bed fast patients. Here are some strategies to help manage prehypertension:
♦♦ Exercise of extremities while on bed ♦♦ Lose weight, if one is overweight. Being overweight
♦♦ Maintenance of hydration increases the risk of high blood pressure. However, losing
♦♦ Correction of cardiac failure. weight can lower high blood pressure. Studies show that
modest weight loss can prevent hypertension by 20% in
Q.38. Describe the etiology, clinical features, complica­tions
overweight people with prehypertension.
and management of acute myocardial infarction. 
♦♦ Exercise regularly: Exercise helps to lose weight. Exercise
 (Sep 2008, 15 Marks)
also helps in lowering blood pressure.
Ans. Acute myocardial infarction is defined as irreversible ♦♦ Eat plenty of fruits, vegetables, whole grains, fish, and low-fat
damage to myocardium of heart as a result of occlusive dairy: Studies show high blood pressure can be lowered
thrombus due to rupture of atherosclerotic plaque in and prevented with the DASH diet. This diet is low in
coronary artery. sodium and high in potassium, magnesium, calcium,
protein, and fiber.
Etiology
♦♦ Cut back on dietary salt / sodium: A diet high in sodium (salt)
♦♦ The most important cause of myocardial infarction is can increase blood pressure. A low-sodium diet can lower
atherosclerosis. high blood pressure or prevent it. Aim for less than 2,300
♦♦ Obesity and hypertension milligrams of sodium daily (about 1 teaspoon of table salt).
♦♦ Cigarette smoking ♦♦ Eat foods low in saturated and transfat and cholesterol.
♦♦ Diabetes mellitus Diets high in saturated fat (meats and high-fat dairy),
♦♦ Sedentary life cycle. transfat (some margarine, snack foods, and pastries) and
♦♦ Dyslipidemia. cholesterol (organ meats, high-fat dairy, and egg yolks)
For clinical features, refer to Ans 21 of same chapter. may lead to obesity, heart disease, and cancer.
For complications, refer to Ans 20 of same chapter. ♦♦ Eat a plant-based or vegetarian diet: Add high-protein
For management, refer to Ans 19 of same chapter. soy foods to your diet. Increase servings of fruits and
vegetables by adding one serving at a time. You can add
Q.39. Write short note on treatment of acute myocardial a serving of fruit at lunchtime. Then add a serving of
infarction. (Mar 2007, 5 Marks) vegetables at dinner.
Ans. Refer to Ans 19 of same chapter. ♦♦ Drink only in moderation: Drinking excess alcohol can
increase blood pressure. Limit drinking to not more than
Q.40. Describe etiology, diagnostic criteria and prophylaxis
two drinks a day for men, and one drink a day for women.
of acute rheumatic fever. (Apr 2007, 15 Marks)
Ans. For etiology refer to Ans 1, for diagnostic criteria refer to Q.42. Write management of unstable angina pectoris.
Ans 3 and for prophylaxis refer to Ans 2 same chapter.  (Mar 2011, 4 Marks)

Q.41. Write short note on prehypertension and its manage- Or


ment. (Sep 2008, 2.5 Marks) Write management of unstable angina.
Ans. Prehypertension is considered to be blood pressure  (Mar 2013, 4 Marks)
readings with a systolic pressure from 120–139 mm Hg Ans. ♦  T
 o control acute disease and stabilize the thrombotic
or a diastolic pressure from 80–89 mm Hg. Readings process:
greater than or equal to 140 / 90 mm Hg are considered • Hospitalization - to facilitate rapid adjustments in
hypertension. therapy which are usually required.
• Oxygen administration and sedation.
Symptoms
• Therapy with beta blockers and calcium antagonists.
Prehypertension is often asymptomatic (without symptoms) at • Sublingual nitrates for immediate relief.
the time of diagnosis. Only extremely elevated blood pressure • IV nitrates - IV Nitroglycerin 3-10 µg / min if patient
(malignant hypertension) can, in rare cases, cause headaches, does not settle on above therapy. IV nitrates produce
visual changes, fatigue, or dizziness, but these are nonspecific hypotension and regular BP measurements are
symptoms which can occur with many other conditions. Thus, required and dose is increased.
Section 1:  General Medicine  59

• Aspirin 325 mg. OD and Clopidogrel 75 mg / day or ♦♦ Mitral regurgitation + stenosis


Prasugrel 10 mg / day. ♦♦ Congenital heart disease
• Anticoagulation with heparin. Dose 60 U / kg IV bolus ♦♦ Previous infective endocarditis.
followed by 12 U / kg / hr infusion. Low molecular
Intermediate Risk
heparin superior to unfractionated heparin in dose
of 1 mg / kg 12 hourly subcutaneous. ♦♦ Mitral valve prolapse
• Glycoprotein IIb / IIIa receptor inhibitors - which ♦♦ Mitral stenosis
block the receptors that lead to platelet aggregation ♦♦ Tricuspid valve disease
– Eptifibatide 180 µg / kg IV bolus followed by 2 ♦♦ Pulmonary valve disease
µg / kg / min. infusion. ♦♦ Asymmetrical septal hypertrophy
– Tirofiban 0.4 pg / kg / min over 30 mins, followed ♦♦ Calcific aortic sclerosis
by 0.1 µg / min. infusion. ♦♦ Non-valvular intracardiac prosthetic implants
– Abciximab 0.25 mg / kg IV bolus followed by
Very Low Risk
0.125 µg / kg / min. infusion.
♦ Long-term management of underlying coronary artery ♦♦ Atrial septal defect
disease—with revascularization procedures. ♦♦ Atherosclerotic plaques
♦♦ Post myocardial infarction thrombi, atrial thrombi and
Q.43. Describe types, etiology, sign, symptoms, inves­
ventricular aneurysms
tigations and outline treatment of infective endocar-
♦♦ Syphilitic aortitis
ditis. (Jan 2012, 12 Marks)
♦♦ Cardiac pacemakers
Or ♦♦ Surgically corrected cardiac lesions (without prosthetic
Write short note on infective endocarditis. implants)
 (Dec 2010, 5 Marks) Signs
Or
♦♦ Signs of infection:
Describe risk factors clinical features, diagnosis and • Fever: Variable, low grade.
treatment of infective endocarditis. • Anaemia: Yellow muddy discoloration of skin.
 (Apr 2015, 8 Marks) • Clubbing: of fingers and toes occurs early.
Or • Splenomegaly: after about 6 weeks of illness. Sudden
enlargement with tenderness and rub if infarction of
Discuse etiology, Clinical feature and treatment of
spleen. Rarely gross splenomegaly.
infective endocarditis. (May 2018, 5 Marks)
• Arthralgia: Sudden transient pain in joints without
Ans. Infective endocarditis is an illness caused by microbial any effusion.
infection of the cardiac endothelial surface. ♦♦ Cardiac signs:
Types Murmur:
• Organic heart murmur: due to valvular defect or
♦♦ Acute bacterial endocarditis: Caused by virulent organisms congenital cardiovascular lesions.
and run its course over days to weeks. • Development of new murmurs: due to perforation of
♦♦ Subacute bacterial endocarditis: Caused by organisms of low ventricular septum, rupture of sinus of Valsalva, acute
virulence and run its course over weeks to months. mitral regurgitation or acute aortic regurgitation.
♦♦ Non-bacterial thrombotic endocarditis: Sterile vegetations • Absence of murmur:
occur in this type. –– Endocarditis involving mural thrombus compli-
♦♦ Surgical endocarditis: May follow any type of surgery and cating healed myocardial infarction.
is caused by Staphylococcus epidermidis. –– Early acute endocarditis involving previously
normal valve.
Etiology
–– Tricuspid endocarditis when it exists, may be
Generally, the microorganisms attack an already damaged heart. murmur free.
The microorganisms responsible are Streptococcus viridans, S. • Cardiac failure—may occur due to toxic myocarditis.
faecalis and Staphylococcus aureus. Below given are all the heart ♦♦ Systemic embolism:
defects which lead to infective endocarditis. • Arterial:
–– Cerebral producing hemiplegia or mycotic
Relatively High Risk aneurysms which may subsequently rupture.
♦♦ Prosthetic heart valves. –– Renal causing colic and hematuria.
♦♦ Aortic valve disease –– Retinal with disturbing vision.
60   Mastering the BDS IIIrd Year  (Last 25 Years Solved Questions)

–– Of mesenteric arteries causing acute abdominal ♦♦ Echocardiography: Higher sensitivity in identifying


pain; splenic infarction with sudden local pain vegetation with Transoesophageal echocardiography
and perhaps friction. as compared to transthoracic echocardiography. (a)
–– Peripheral vessel resulting in gangrene of an Vegetations: An echodense structure attached to the valve or
extremity. Early prosthetic valve endocarditis. its supporting structures, or lying in the track of a turbulent
b. Pulmonary: Recurrent pneumonitis (infective emboli jet, which is irregular in shape. (b) Leaflet perforation is
cause abscesses) and arterial rupture. best seen as regurgitant jet on color flow mapping. (c)
♦♦ Immunological: Annular and periprosthetic echolucent spaces (abscesses)
• Thenar and hypothenar eminences: and fistula formation.
–– Osler’s nodes: Tender, pea-sized nodules on pads ♦♦ Chest X-ray: Shows evidence of cardiomegaly and heart
of fingers and toes. Often pale in the centre failure.
–– May occur in crops. Fade after few days usually
without breaking down or leaving any residue. Treatment
–– Either due to minute emboli in superficial
terminal vessels or due to vasculitis. For treatment refer to Ans 8 of same chapter.
–– Laneway lesions: Large non-tender macules on
palms and soles. Q.44. Describe etiology, diagnostic criteria, compli­cations
• Skin and mucous membranes: and management of rheumatic fever.
–– Petechial hemorrhages in palpebral conjunctivae,  (Feb 2013, 12 Marks)
buccal and pharyngeal mucous membrane. Ans. For etiology refer to Ans 1 of same chapter.
–– Subungual: Splinter hemorrhages For diagnostic criteria refer to Ans 3 of same chapter.
–– Finger and toe tips: Osler’s nodes For management refer to Ans 2 of same chapter
–– Retina-Roth spots: Lesions with a white centre and
red edge. Boat-shaped hemorrhage Complications
–– Renal: Glomerulonephritis leading to kidney Inflammation caused by rheumatic fever may last for a few
failure. weeks to several months. In some cases, the inflammation may
Symptoms cause long-term complications.
Rheumatic heart disease is permanent damage to the heart
♦♦ Patient with a known cardiac ailment may start complaining
caused by the inflammation of rheumatic fever. Problems are
of vague ill health, fatigue, weakness and a low-grade fever.
most common with the valve between the two left chambers of
♦♦ There is presence of pain and marked aches all over the
the heart (mitral valve), but the other valves may be affected.
body.
The damage may result in one of the following conditions:
Investigations
♦♦ Valve stenosis: This condition is a narrowing of the valve,
♦♦ Blood cultures: In absence of recent or concurrent antibiotic which results in decreased blood flow.
therapy, the first 3 random blood cultures (2–4 hours apart) ♦♦ Valve regurgitation: This condition is a leak in the valve,
are positive in most patients, and blood culture is positive which allows blood to flow in the wrong direction.
by third day in 90%. ♦♦ Damage to heart muscle: The inflammation associated with
♦♦ Urine: Microscopic hematuria most common finding. Slight rheumatic fever can weaken the heart muscle, resulting in
albuminuria and hyaline and granular casts also found. poor pumping function.
♦♦ Hematology: Normocytic. normochromic anemia, usually
Damage to the mitral valve, other heart valves or other
mild. May be raised ESR and raised C-reactive protein.
heart tissues can cause problems with the heart later in life.
♦♦ Chest radiograph: May be diagnostic in right sided
Resulting conditions may include:
endocarditis, with multiple shadows visible due to an
embolic pneumonia. ♦♦ Atrial fibrillation, an irregular and chaotic beating of the
♦♦ ECG: Myocardial infarction seen on ECG may be due to upper chambers of the heart (atria).
coronary embolism, and a conduction defect may be due ♦♦ Heart failure, an inability of the heart to pump enough
to development of an aortic root abscess. blood to the body.
Section 1:  General Medicine  61

Q.45. Write the difference between cardiac and non-cardiac chest pains. (Nov 2011, 4 Marks)
Ans.
Features Cardiac Chest Pain Non-cardiac Chest Pain
Nature of pain Pain is dull, constricting, choking and/ or crushing Pain may vary from burning to sharp, stabbing and prickling
Location Pain is usually central (towards the center of the Pain is usually localized—located at only one spot which can be
chest) and fanning outwards (diffuse) clearly pinpointed by the patient
Radiation of pain Pain radiates to the jaw, neck, shoulder, arms There may not often be any radiation of the pain
(either one or even both) or back
Precipitating factors Exertion or emotion, large meal or even extremes Spontaneous although it may be exacerbated by exertion,
of temperature, particularly cold, can trigger or Changes in posture, deep or rapid breathing or pressure may
exacerbate the pain also exacerbate the pain
Pain-relieving factors Pain is relieved by rest and responds quickly to Not relieved significantly by rest, at all
Signs and Symptoms nitrates. • Gastrointestinal: Bloating, belching, nausea vomiting and/or
• Severe shortness of breath—patient may regurgitation
report a feeling of suffocation • Respiratory: Shallow breathing, persistent cough, abnormal brea-
• Dizziness thing sounds, difficulty breathing when lying flat, expectorating
• Fainting spells (‘blackouts’) mucus or coughing up blood. (Refer to Lung Chest Pain)
• Musculoskeletal: Limited range of motion, cannot tolerate
pressure on the affected area
• Psychological: Weepy, depressed, excited, agitated, fearful

Q.46. Give definition, etiology, sign and symptoms, inves- ♦♦ Primary pulmonary hypertension.
tigation and treatment of congestive cardiac failure. ♦♦ Right ventricular infarction.
 (Feb 2013, 12 Marks) ♦♦ Pulmonary and tricuspid valve disease
Or ♦♦ Isolated right ventricular cardiomyopathy.
Write clinical features, investigations and treatment of
congestive cardiac failure. (Jan 2016, 12 Marks) Signs and Symptoms / Clinical Features
Ans. It is defined as a pathophysiologic state when heart is not
Left-sided Failure
able to maintain its cardiac output to meet the demands
of metabolizing tissues or can do so only at the expense ♦♦ Common respiratory signs are tachypnea (increased rate of
of elevating filling pressures. breathing) and increased work of breathing (non-specific
In congestive cardiac failure, patient has features of both signs of respiratory distress). Rales or crackles, heard
right- and left-sided heart failure. initially in the lung bases, and when severe, throughout
the lung fields suggest the development of pulmonary
Etiology edema (fluid in the alveoli). Cyanosis which suggests
severe hypoxemia, is a late sign of extremely severe
Left-sided Failure
pulmonary edema.
♦♦ Myocardial damage: ♦♦ Additional signs indicating left ventricular failure include
• Myocardial infarction a laterally displaced apex beat (which occurs, if the heart is
• Myocarditis enlarged) and a gallop rhythm (additional heart sounds)
• Cardiomyopathy may be heard as a marker of increased blood flow, or
• Cardiac depressant drugs. increased intracardiac pressure. Heart murmurs may
♦♦ Increased load: indicate the presence of valvular heart disease, either
• Hypertension as a cause (e.g. aortic stenosis) or as a result (e.g. mitral
• Mitral and aortic valve disease regurgitation) of the heart failure.
• Cardiac arrhythmias ♦♦ Backward failure of the left ventricle causes congestion
• Over transfusion. of the pulmonary vasculature, and so the symptoms are
predominantly respiratory in nature.
Right-sided Failure
♦♦ Patient will have dyspnea (shortness of breath) on exertion
♦♦ Pulmonary hypertension: and in severe cases, dyspnea at rest.
• Secondary to left heart failure ♦♦ Increasing breathlessness on lying flat, called orthopnea,
• Chronic lung disease occurs. It is often measured in the number of pillows
• Pulmonary embolism required to lie comfortably, and in severe cases, the patient
• Left-to-right shunts. may resort to sleeping while sitting up.
62   Mastering the BDS IIIrd Year  (Last 25 Years Solved Questions)

♦♦ Another symptom of heart failure is paroxysmal ♦♦ Blood urea and electrolytes: It is done for hypokalemia,
nocturnal dyspnea a sudden night time attack of severe hyponatremia and renal failure,
breathlessness, usually several hours after going to sleep.
♦♦ Easy fatigability and exercise intolerance are also common Treatment
complaints related to respiratory compromise.
♦♦ Rest: Complete bed rest is the key stone of management.
♦♦ Cardiac asthma or wheezing may occur.
When the patient is dyspneic, bed rest is given with the
♦♦ Compromise of left ventricular forward function may
head end of bed raised to 45°. The legs should be kept
result in symptoms of poor systemic circulation such as
below the pelvis to prevent the fluid present in legs to
dizziness, confusion and cool extremities at rest.
return to vascular system and precipitate pulmonary
Right-sided Failure edema.
♦♦ Diet: Basic aim is to restrict sodium in the diet. Quantity
♦♦ Physical examination may reveal pitting peripheral edema,
of salt intake per day should not exceed 0.5 gm. Salt
ascites, and hepatomegaly.
substitutes may be used to make diet more palatable.
♦♦ Jugular venous pressure is frequently assessed as a marker
♦♦ Diuretics: In cardiac failure, there is always sodium
of fluid status, which can be accentuated by eliciting
and water retention. Hence, diuretics are given to
hepatojugular reflux.
increase sodium extraction. Furosemide 40–80 mg orally
♦♦ Backward failure of the right ventricle leads to congestion of
produces effect in 4–6 hours and on IV administration of
systemic capillaries. This generates excess fluid accumulation
furosemide 40–100 mg produces its effect in 20 minutes.
in the body. This causes swelling under the skin (termed
Spironolactone which is potassium sparing diuretic
peripheral edema or anasarca) and usually affects the
dependent parts of the body first (causing foot and ankle is given 25 mg orally 4 times a day which removes
swelling in people who are standing up, and sacral edema the symptoms such as hypokalemia due to action of
in people who are predominantly lying down). furosemide. Triamterene or amiloride hydrochloride is
♦♦ Nocturia (frequent night time urination) may occur when given along with spironolactone.
fluid from the legs is returned to the bloodstream while ♦♦ Digitalis: It increases the force of myocardial contraction
lying down at night. and decreases work of heart. The commonly used drug
♦♦ In progressively severe cases, ascites (fluid accumulation in is digoxin which is administered orally 0.25 mg BD. For
the abdominal cavity causing swelling) and hepatomegaly rapid digitalization digoxin 0.5–0.75 mg is given slow IV
(enlargement of the liver) may develop. over a period of 5–10 minutes under electrocardiographic
♦♦ Significant liver congestion may result in impaired liver control.
function, and jaundice and even coagulopathy (problems ♦♦ Sympathomimetic amines: Dopamine at low doses of 3–5 μg/
of decreased blood clotting) may occur. kg / min increases contractility of heart.
♦♦ Vasodilators: Sodium nitroprusside in the dose of 5–10 μg/
Investigation min have balance dilator effect. Besides this hydralazine,
♦♦ Electrocardiogram (ECG): This will reveal arrhythmias, nitrates and ACE inhibitors are used.
ventricular hypertrophy and myocardial ischemia. ♦♦ Inodilator levosimendan: This is a calcium channel sensitizer.
♦♦ Chest X-ray: This will show enlargement of heart, peripheral It has positive inotropic and vasodilator effect. It is given IV
lung congestion, presence of Kerley’s lines, pulmonary with loading dose of 6–12µg/kg/min over 10 min followed
edema, hydrothorax, pulmonary hypertension, double by infusion 0.05 to 2 µg/kg/min infusion.
atrial shadow in mitral valve disease and calcification of ♦♦ Oxygen: It is given via Woulfe’s bottle at rate of 5–8
valves. L/min.
♦♦ Echocardiogram: In this ultrasound is used to image the ♦♦ Miscellaneous drugs: Tranquilizers such as diazepam 2 to
heart muscle, valve structures, and blood flow patterns. 5mg TDS are given to reduce anxiety.
The echocardiogram is very helpful in diagnosing heart ♦♦ Cardiac re-synchronization therapy or biventricular pacing: It
muscle weakness. In addition, the test can suggest possible is used in patients with symptomatic refractory cardiac
causes for the heart muscle weakness (for example, prior failure with conduction abnormality or Left Bundle Branch
heart attack, and severe valve abnormalities). Virtually all Block. This therapy involves pacing the right atrium, right
patients in whom the diagnosis of congestive heart failure ventricle and left ventricle to improve synchrony of the
is suspected should ideally undergo echocardiography cardiac chambers.
early in their assessment. ♦♦ Left ventricular assist device: Devices such as Intra-aortic
♦♦ Nuclear medicine studies assess the overall pumping balloon pump, Impella device, Heart-Mate, Thoracic are
capability of the heart and examine the possibility of considered when medical management fails. They are
inadequate blood flow to the heart muscle. usually used as a bridge to cardiac transplant.
♦♦ BNP or B-type natriuretic peptide level: This level can vary
with age and gender but is typically elevated from heart Q.47. Write on management of cardiac arrest.
failure and can aid in the diagnosis, and can be useful in (Feb 2013, 6 Marks)
following the response to treatment of congestive heart Ans. Cardiac arrest is defined as sudden failure of heart
failure. resulting in inadequate cerebral circulation.
Section 1:  General Medicine  63

Management –– Propranolol: 5–10 mg may be given intravenously


as an antiarrhythmic agent.
♦♦ Basic life support or Basic cardiac life support
–– Bretylium tosylate: It is given as 5 to 10mg/kg IV
Maintain the airway, breathing and circulation ♦♦ Specific measures
simultaneously as rapidly as possible as follows: • For ventricular asystole: If cardiac arrest persists in spite
• Airway: Airway must be patent. If any of the foreign of the above measures, external cardiac pacing is done.
body is suspected, the patient must be rolled on one If this is negative or required for a long period, internal
side and 4–5 forceful blows must be delivered rapidly cardiac pacemaker is inserted.
between the shoulder blades with the heel of the • For ventricular fibrillation: If the above drugs do not
hand. Now keep the patient in supine position and immediately revert the cardiac rhythm, or if ventricular
abdominal thrusts in an upward direction are given fibrillation is recurrent inspite of the above measures
to the patient just below the xiphisternum. After the a direct current (DC) shock is given to the heart with
foreign body is excluded, patient should be kept in 200 joules. It should be repeated, if required after a few
supine position as he may require external cardiac minutes with 300 V Joules and then up to 400 Joules.
massage and artificial respiration. The patient’s head ♦♦ Management after successful resuscitation in an
should be lifted with one hand under the neck and unconscious patient
the other hand pressing the forehead so that the head • Endotracheal Intubation with controlled ventilation
is tilted backwards to keep the upper airway patent. should be continued with the help of a ventilator to
• Breathing: Once the patency of airway is maintained, keep the partial pressure of oxygen at 100 mm Hg and
and if breathing is inadequate, artificial ventilation partial pressure of carbon dioxide at 30-40 mm Hg.
must be given. With the above position, the patient’s • Blood pressure must be maintained at 10mm Hg
nostrils must be sealed with thumb and index finger systolic, if required with the help of vasopressors like
and mouth-to-mouth respiration must be given to dopamine.
the patient. • Acid-base and fluid electrolyte balance should be
This is done by taking a deep inspiration and exhaling maintained.
in the patient’s mouth and then the patient is allowed • Mannitol 350 to 500 mL I.V. or dexamethasone 4
to exhale passively. Continue this procedure at the rate mg 6 to 8 hourly I.V. should be given in presence of
of 16-18/min. In hospitals, an Ambu bag is used. cerebral edema.
• Circulation: With the patient in supine position, neck • Phenytoin sodium, diazepam or phenobarbitone
extended and legs elevated a sudden sharp thrust is should be given if there are convulsions.
• Aspiration pneumonia should be prevented by
given on the chest wall. This may restore the effective
appropriate antibiotics.
beating of the heart especially, if the cardiac arrest
is due to cardiac standstill. In absence of response, Q.48. Write in brief sign, symptoms and treatment of rheu-
external cardiac massage is given. matic fever. (Apr 2008, 5 Marks)
♦♦ Advanced Cardiac Life Support (ACLS) Or
As the basic life support is maintained an ECG must be Write signs and symptoms of acute rheumatic fever.
taken to determine whether the cause of cardiac arrest is (Dec 2010, 5 Marks)
ventricular asystole or ventricular fibrillation. Ans. It is an acute, recurrent, inflammatory disease mainly of
• If it is ventricular asystole, while electric methods of children typically occurring 1 to 5 weeks after Group A
treatment like external cardiac pacemaker are made streptococcal infection.
available, the following drugs are given; Symptoms
–– Epinephrine (Adrenaline): 1 mL of 1:1000
♦♦ Pyrexia of unknown origin.
epinephrine is given intravenous followed by a
♦♦ Arthralgia, i.e. pain in joints. Pain in joints is fleeting and
bolus of dextrose.
migratory.
–– Calcium: 10 mL calcium gluconate 10% is injected
IV or sometimes intracardiac. It is rarely used. Signs
–– Sodium bicarbonate: 10 mL of 7.5% sodium
Carditis
bicarbonate is infused slowly intravenously to
correct metabolic acidosis. This is also rarely used. ♦♦ It is pancarditis involving endocardium, myocardium and
–– Vasopressors: Nor-epinephrine 1:1000 is initially pericardium.
given in the dose of 2 mL and later repeated, if ♦♦ It manifests as breathlessness, palpitation and chest pain.
hypotension persists. tachycardia, cardiomegaly and new or change murmurs.
• lf it is ventricular fibrillation the following drugs are ♦♦ Aortic regurgitation in 50% cases.
used in addition: ♦♦ Pericarditis produces frictional rub and pericardial
–– Lignocaine: 50–100 mg is injected intravenously tenderness.
as a bolus and may be repeated after 15–20 min. ♦♦ Cardiac failure due to myocardial infarction.
64   Mastering the BDS IIIrd Year  (Last 25 Years Solved Questions)

Sydenham’s Chorea • Weight loss, tremors, palpitation and sweating due to


hyperthyroidism or pheochromocytoma
♦♦ Late neurological manifestations that occurs at least three
• Joint pain, bronchospasm and peripheral vascular
months after the episode of acute rheumatic fever when
disease symptoms due to polyarteritis nodosa.
all signs disappear.
♦♦ More common in female. Signs
♦♦ It is characterized by involuntary dancing movements of
hands, feet or face. ♦♦ Moon’s face, buffalo hump and truncal obesity in
Cushing’s syndrome.
Arthritis ♦♦ Presence of puffy face, rough skin, obesity in myxedema
♦♦ Tremors, tachycardia, exophthalmos, thyroid dermopathy
♦♦ Early feature of illness and is non-specific.
and goiter in hyperthyroidism
♦♦ It is characterized by acute painful symmetric and
♦♦ Prognathism, clubbed hand, coarse features in acromegaly
migratory inflammation of large joints.
♦♦ Palpable kidney lump in polycystic kidney
Erythema Marginatum ♦♦ Bruit over abdominal aorta.
♦♦ Undiagnosed fever and recurrent urinary infections in
Red macules which fade in center but remain red at the edges chronic pyelonephritis.
and occur mainly on trunk and proximal extremities on face.
Treatment
Subcutaneous Nodules
Refer to Ans 18 of same chapter.
♦♦ They are small, dense and firm and painless and are best
felt over tendons and bones. Q.50. Write short note on congestive cardiac failure.
♦♦ Nodules appear more than 3 weeks after onset of other  (Mar 2006, 10 Marks) (May/June 2009, 5 Marks)
manifestations. Or
Write sign and symptoms of congestive cardiac
Respiratory failure. (Jan 2012, 5 Marks)
Epistaxis is an atypical manifestation. Or
Describe in brief congestive cardiac failure
Gastrointestinal  (Mar 2016, 4 Marks)
Ans. Refer to Ans 46 of same chapter.
Mild gastroenteritis is present. Repeated vomiting spells are
present. Q.51. Describe the definition, etiology, clinical features,
diagnostic criteria and management of acute rheumatic
Treatment
fever. (Dec 2009, 15 Marks)
Refer to Ans 1 of same chapter. Or
Q.49. Write in brief sign, symptoms and treatment of hyper- Discuss, etiology, clinical features, diagnostic criteria
tension. (May/June 2009, 5 Marks) and management of acute rheumatic fever.
Ans. Hypertension refers to increase in the blood pressure.  (Dec 2009, 15 Marks)
Ans. For definition refer to Ans 48. For etiology refer to Ans
1 of same chapter. For clinical features refer to Ans 2 of
Symptoms
same chapter. For diagnostic criteria and management
♦♦ General: Presence of headache, dizziness, palpitation and refer to Ans 3 of same chapter.
easy fatigability Q.52. Write short note on congenital heart disease.
♦♦ Symptoms referable to systemic vascular involvement:  (Jun 2010, 5 Marks)
• Epistaxis Ans. Congenital heart diseases are the abnormalities of heart
• Hematuria and great vessels due to defective development in the
• Blurring of vision or sudden blindness prenatal period.
• Dyspnea on exertion
• Anginal chest pain
• Palpitations Classification
• Transient ischemic episodes leading to weakness or ♦♦ Acyanotic:
paralysis • Acyanotic with left to right shunt:
• Hypertensive encephalopathy –– Atrial septal defect (ASD)
• Tinnitus –– Ventricular septal defect (VSD)
• Syncope –– Patent ductus arteriosus
♦♦ Symptoms related to underlying disease: –– Aorto-pulmonary window
• Edema and puffy face due to acute nephritis –– Common complete atrio-ventricular canal.
• Weight gain, hirsutism, truncal obesity due to • Acyanotic without shunt:
Cushing’s syndrome –– Pulmonary stenosis
Section 1:  General Medicine  65

–– Endocardial cushion effects • Serum biochemistry: High sodium and low potassium
–– Aortic stenosis may suggest primary hyperaldosteronism.
–– Coarctation of aorta. • Urea and creatinine: High levels suggest a degree
♦♦ Cyanotic: of renal impairment due to hypertension, or that
• Complete transposition of great vessels underlying renal disease is the cause for hypertension.
• Persistent truncus arteriosus • Lipid and glucose concentrations: To judge cardiovascular
• Tetralogy of Fallot risk status for each patient.
• Ebstein’s anomaly with right-to-left shunt • ECG: Detection of left ventricular hypertrophy and strain
• Common atrium. pattern is an important adverse prognostic indicator.
♦♦ Special tests:
Etiology
• Intravenous pyelogram and renal ultrasound, if renal
♦♦ Patent Ductus arteriosus: Maternal cause, Patau’s syndrome disease is present.
♦♦ Atrial septal defect: Rubella virus, Noonan’s syndrome, Holt- • Renal arteriography for renal artery stenosis
Oram syndrome, Down syndrome, Patau’s syndrome, • Plasma rennin activity and aldosterone for Conn
♦♦ Ventricular septal defect: Alcohol, Holt-Oram syndrome, syndrome
Down’s syndrome, Edward’s syndrome. • Angiography  /  MRI for coarctation of aorta.
♦♦ Tricuspid atresia: Alcohol
♦♦ Fallot’s Tetralogy: Down’s syndrome, lesions associated Treatment
with 22q11 deletions. For treatment refer to Ans 18 of same chapter.

Features of Various Congenital Heart Diseases Q.54. Describe aetiology, clinical features and management
of coronary artery disease. (Jan 2012, 15 Marks)
♦♦ Central cyanosis and digital clubbing is present in Ans. Coronary artery diseases are also known as ischaemic
transposition of great vessels and tetralogy of Fallot. heart disease.
♦♦ Growth retardation as well as intellectual impairment is
Following are the coronary artery diseases:
present in all congenital heart diseases.
♦♦ Syncope is present in severe right or left ventricular • Angina pectoris
outflow tract obstruction. • Acute myocardial infarction
♦♦ Short stature is commonly seen in patients with congenital • Sudden cardiac death.
heart disease. Aetiology of Coronary Artery Diseases
Q.53. Describe JNC VII criteria for hypertension. Write in ♦♦ Atherosclerotic coronary artery disease
brief investigations and outline treatment. ♦♦ Other coronary artery diseases:
 (Aug 2011, 15 Marks) • Coronary artery spasm
Ans. JNC VII criterias is the 7th report of the Joint National • Coronary arteritis
Committee on Prevention Detection, Evaluation and • Embolism
Treatment of High Blood Pressure. • Coronary AV malformation
♦♦ Valvular diseases:
JNC VII Criteria for Hypertension • Aortic stenosis and regurgitation
• Mitral valve prolapse
Category Systolic Pressure Diastolic Pressure ♦♦ Other cardiac diseases:
(mm Hg) (mm Hg) • Hypertrophic cardiomyopathy
Normal <120 <80 • Collagen disease
Pre-hypertension 120–139 80-89 • Syphilis
♦♦ Increased demands:
Hypertension
• Thyrotoxicosis
Stage1 140–159 90-99
• Anemia
Stage 2 >160 >100
• Beriberi
Hypertension should be based on average of two or more Clinical Features
readings taken at each of two or more visits after initial reading.
♦♦ Angina pectoris: Refer to Ans 22 of same chapter.
♦♦ Acute myocardial infarction: Refer to Ans 21 of same chapter
Investigations
♦♦ Sudden cardiac arrest: In this, a person with good health
♦♦ Routine tests: fall ill and die suddenly within minutes and few hours.
• Urinalysis: Detection of proteinuria and microscopic • Absence of pulses
hematuria may indicate some degree of renal arteriolar • On auscultation, cardiac impulse is not present.
necrosis and nephrosclerosis, or underlying intrinsic • Extremities are cold
renal disease such as polycystic kidneys, chronic • Cessation of respiration of patient
pyelonephritis or glomerulonephritis. • Blood pressure is not measurable.
66   Mastering the BDS IIIrd Year  (Last 25 Years Solved Questions)

Management Q.56. Write management of atrial fibrillation.


 (Dec 2012, 4 Marks)
♦♦ Angina pectoris: Refer to Ans 22 of same chapter.
♦♦ Acute myocardial infarction: Refer to Ans 19 of same chapter Ans. It is the most common sustained tachyarrhythmia seen
♦♦ Sudden cardiac arrest: Refer to Ans 47 of same chapter. in patients.

Q.55. Write classification of rheumatic heart diseases and Management


discuss the sign and symptoms of mitral stenosis. ♦♦ Ventricular rate is reduced by giving digoxin. Digoxin 0.5
 (Aug 2012, 15 Marks) mg IV slowly is given. After this, patient is kept on oral
Ans. Classification of Rheumatic Heart Diseases digitalis therapy, i.e. 0.25 mg twice daily.
Based on the severity of rheumatic heart disease. ♦♦ Antiarrhythmic drugs such as propranolol is given IV at a
rate of 1 mg every 5 min followed by maintenance of oral
Severe valvular disease dose of 20–40 mg three times a day.
Or ♦♦ Underlying causes such as thyrotoxicosis and acute chest
Priority 1 Moderate / severe valvular lesion with symptoms infections should be treated to maintain normal sinus
(Severe) Or rhythm.
Mechanical prosthetic valves, tissue prosthetic valves
♦♦ Anticoagulant therapy should be given in patients with
and valve repairs including balloon valvuloplasty
chronic atrial fibrillation.
Any moderate valve lesion in the absence of symptoms ♦♦ Defibrillation should be done. A DC shock of 100 joules
and with normal left ventricular function
Or
restore normal sinus rhythm.

Priority 2
Mild mitral regurgitation PLUS mild aortic stenosis Q.57. Write short note on prophylaxis for infective endocar-
Or ditis. (Dec 2012, 3 Marks)
(Moderate)
Mild or moderate mitral or aortic stenosis
Ans. Refer to Ans 8 of same chapter.
Or
Any pulmonary or tricuspid valve lesion coexisitng with Q.58. Describe clinical features of rheumatic fever and di-
a left-sided valve lesion agnosis of carditis with strategy of prophylaxis.
Acute rheumatic fever with no evidence of rheumatic  (Nov 2014, 8 Marks)
Priority 3 heart disease Ans. For clinical features of rheumatic fever refer to Ans 1 of
(Mild) Or same chapter.
Trivial to mild valvular disease
For diagnosis of carditis, refer to Ans 33 of same chapter
Patient with the history of acute rheumatic fever
Priority 4 and for strategy of prophylaxis refer to Ans 8 of same
(no rheumatic heart disease) for whom secondary
(Inactive)
prophylaxis has been ceased chapter.
Q.59. Write short note on PTCA. (Nov 2014, 3 Marks)
Sign and Symptoms of Mitral Stenosis Ans. Percutaneous transluminal coronary angioplasty
(PTCA) is a minimally invasive procedure to open up
Symptoms blocked coronary arteries, allowing blood to circulate
♦♦ Patient complains of breathlessness and fatigue on unobstructed to the heart muscle.
exertion. • In PTCA, balloon dilatation of coronary stenosis is
♦♦ Progression of stenosis lead to dyspnea on rest and even done.
have orthopnea and paroxysmal nocturnal dyspnea. Method
♦♦ Acute pulmonary edema can also occur.
♦♦ Hemoptysis can be present due to rupture of pulmonary Procedure of doing PTCA is commenced by passing a guide
congestion and pulmonary embolism and cough due to wire under fluoroscopic control, this wire positions the balloon
pulmonary congestion. dilatation centre at level of stenosis, now balloon dilatation
♦♦ Chest pain is present due to pulmonary venous of the stenotic segment is carried out for maintaining the
hypertension. circulation all through.
Indications
Signs
♦♦ This method provides complete or partial revascularization
♦♦ Atrial fibrillation is present. in cases of stable angina pectoris, unstable angina pectoris
♦♦ Auscultation: Presence of loud first heart sound, opening or myocardial infarction.
snap and mid diastolic low-pitched rumbling murmur ♦♦ It is indicated in patients with recurrent angina after
best heard at the apex. coronary artery bypass grafting (CABG).
♦♦ Signs of raised pulmonary capillary pressure: Pleural effusion, Complications
Crepitation, pulmonary edema. ♦♦ Blood vessel occlusion can occur by thrombosis.
♦♦ Signs of pulmonary hypertension: RV heave, Loud P2 ♦♦ Recurrent angina pectoris
♦♦ Others: Basal crackles, ascites and pleural effusion ♦♦ Restenosis can occur.
Section 1:  General Medicine  67

Q.60. Describe risk factors, clinical features, diagnosis and ♦♦ Arrythmias or conduction defects
treatment of myocardial infarction. ♦♦ Sudden cardiac death
 (Dec 2015, 8 Marks) ♦♦ Asymptomatic coronary artery disease detected on routine
Or medical check up.
Describe clinical features, diagnosis, primary care, Risk Factors
treatment and complications of acute myocardial
infarction. (Apr 2017, 12 Marks) ♦♦ Age: As age advances chances of occurrence of coronary
artery diseases increases. It is due to cumulative effects of
Ans. For clinical features and diagnosis refer to Ans 21 of same
multiple risk factors overtime.
chapter.
♦♦ Diet: Diet rich in fat, sugar, cholesterol leads to formation
For treatment of myocardial infarction, refer to Ans 19 of athroma and this leads to coronary artery diseases.
of same chapter. ♦♦ Genetic: Positive family history of sudden death, myocardial
For complications, refer to Ans 20 of same chapter. infarction, angina are points to genetic predisposition.
Risk Factors ♦♦ Personality: In persons having traits of aggressiveness,
ambition and competitiveness chances of occurrence of
♦♦ Family history of heart disease
coronary artery diseases increases.
♦♦ Patient history of heart disease
♦♦ Smoking: Persons who smoke are susceptible for coronary
♦♦ Diabetes or elevated blood glucose even in non-diabetics
artery diseases because of nicotine and carbon monoxide.
♦♦ Hypertension
♦♦ Diabetes mellitus: Due to diabetes athroma may develop
♦♦ Advanced age
in early age and patient is suffering from asymptomatic
♦♦ High lipoprotein lipids
coronary artery disease.
♦♦ Stress, smoking, sedentary lifestyle, compulsive personality
♦♦ Obesity: It is associated with increased levels of serum
♦♦ Poor diet, i.e. high sodium, high fat, high intake of alcohol;
cholesterol, blood pressure, serum triglycerides and serum
low intake of B complex vitamin, calcium, magnesium and
insulin which lead to coronary artery disease.
potassium; low intake of fruit and vegetables
♦♦ Physical activity: Persons who does not undergo regular
♦♦ Obesity.
physical activities such as brisk walking or exercise have
Primary Care chances for coronary artery diseases.
♦♦ Aspirin 150–300 mg to be chewed earliest. Diagnosis
♦♦ Sublingual glyceryltrinitrate 0.4–1 mg, to be repeated, if
In Angina Pectoris
necessary
♦♦ Oxygen through nasal cannula at a rate of 2–4 L/min. ♦♦ Resting ECG: ECG changes of myocardial ischemia are
♦♦ Procure IV line and take blood samples for glucose, lipids reflected in S-T waves. Occasionally, there is flattening of
and complete hemogram. T-waves in some lead in patient with angina.
♦♦ Record 12-lead ECG. ♦♦ Exercise ECG or Stress test: With continuous ECG monitoring
♦♦ Pain may be relieved by IV morphine (5 mg) plus and intermittent BP recording is performed with a treadmill
metoclopramide as an antimetic (10 mg). or bicycle ergometer. Standardised protocols are used (e.g.
♦♦ IV beta-blockers (metoprolol 5 mg every 2–5 minutes Bruce protocol), enabling performance to be assessed in
for 3 doses) for ongoing chest pain, hypertension and same patient at different times and work load at onset of
tachycardia provided, there is no contraindication. symptoms or ECG changes to be determined. An exercise
♦♦ Thrombolysis should be done. ECG is abnormal, if there is horizontal or down-sloping
♦♦ If PTCA is planned, give GP IIb/IIIa inhibitor ST segment depression of 0.1 mm or more in any lead.
♦♦ After admission, immediately shift the patient to ICU or ♦♦ Myocardial perfusion scintigraphy: The isotope cardiovascular
ICCU stress (usually thallium-201 or technetium—99 m) is
Q.61 Write clinical features, risk factors, diagnosis and injected at peak exercise and images taken with a camera
management of ischemic heart disease. immediately or shortly after exercise and compared
with rest images taken a few hours later following a
 (July 2016, 12 Marks)
second injection of tracer. Areas of myocardial ischemia
Ans. Ischemic heart disease occurs whenever there is an
imbalance between myocardial oxygen demand and its are identified by reduced isotopic uptake in the same
supply. anatomical distribution stress images but not resting
images (reversible defect).
Clinical Features ♦♦ Coronary angiography: It is done before angioplasty or
♦♦ Presence of asymptomatic ischemia. coronary bypass surgery.
♦♦ Angina—Stable or unstable
In Acute Myocardial Infarction
♦♦ Ischemic cardiomyopathy
♦♦ Acute myocardial infarction Diagnosis of acute myocardial infarction is based on history,
♦♦ Cardiac arrest characteristics symptoms and signs and investigations.
68   Mastering the BDS IIIrd Year  (Last 25 Years Solved Questions)

Investigations ♦♦ Chest X-ray: It can detect acute pulmonary edema or


♦♦ Electrocardiography: congestion. It is also helpful to detect pericardial effusion,
• ECG is the specific method for confirming the cardiomegaly, etc.
diagnosis. ♦♦ Radionuclide scanning: It shows site of necrosis and the
• Typical changes are seen in leads which faces the extent of impairment of ventricular function.
infracted area. These changes are: ♦♦ Echocardiography: This is done for regional wall motion
–– Elevation of ST segment abnormality and ejection fraction.
–– Pathologic Q—waves appear.
–– T waves may become tall and peaked in very early Management
myocardial infarction. T waves are transient and
♦♦ For angina pectoris, refer to Ans 22 of same chapter
last for a few hours only.
♦♦ For myocardial infarction, refer to Ans 19 of same chapter.
–– In contrast to transmural lesions, partial thickness
or subendothelial infarction causes ST/T wave Q.62. Write short answer on jugular venous pulse.
changes without Q waves or prominent ST  (Apr 2018, 3 Marks)
elevation. Ans. Jugular venous pulse is an important physical sign of
–– Changes in the ECG are seen which evolve in cardiovascular disease.
predictable fashion over next few days to weeks. This pulse is examined for pressure and wave forms.
♦♦ Blood test:
• Plasma biochemical markers: Procedure of measuring jugular venous pulse
Myocardial infarction leads to detectable rise in the
This is examined y reclining the patient against pillows at 450
plasma concentration of various enzymes and proteins
angle, neck muscles should be relaxed, upper level of pulsation
that are normally concentrated within the cardiac cells.
Plasma enzymes (cardiac injury enzymes) are as follows: of an internal jugular vein is seen, though vein may itself be
a. Creatine kinase (CK). not visible.
b. Aspartate aminotransferase (AST). In a normal person, jugular venous pulsation in above said
c. Lactate dehydrogenase (LDH). position remains either behind the clavicle or below it and rarely
d. Myoglobin extends beyond 2 to 3 cm above sternal angle.
e. Troponins (troponin I and troponin T)
–– Creatine kinase starts to rise at 4–6 hours and it Waveforms of jugular venous pulse
peaks by about 12 hours and falls to normal in
Waveform of jugular venous pulse in patient of sinus rhythm is:
48–72 hours. Myocardial isoenzyme of creatine
kinase is more specific. It is useful for diagnosis ♦♦ ‘a’ wave occur due to atrial systole. This is absent in atrial
of early myocardial infarction. fibrillation but becomes large in tricuspid stenosis, right
–– Aspartate aminotransferase (AST) starts to rise at ventricle hypertrophy and pulmonary hypertension.
about 12 hours and reaches a peak on the first or ♦♦ ‘c’ wave corresponds to tricuspid closure
second day and returns on third and fourth day.
–– Lactate dehydrogenase (LDH) starts to rise after
12 hours, reaches a peak after 2–3 days and may
remain elevated for a week. Rise in the value of
LDH I (an isoenzyme of LDH) is a more sensitive
indicator of myocardial infarction than total LDH.
It is useful in diagnosis for patients who present
several days after myocardial infarction.
–– Cardiac troponins are cardiac troponin-T (cTn-T)
and cardiac troponin-I (cTn-I). Sensitivity of
troponins is similar to that of isoenzymes of
creatine kinase. Moreover, cTn-T remains
elevated for 100–200 hours after acute myocardial
infarction and therefore, it may have particular
utility in the evaluation of patients who present
sufficiently long episode after the pain in chest.
• Leucocytosis with a peak on first day.
• ESR is raised, which may remain raised for some days.
• C-reactive protein is elevated. Fig. 12:  Waveforms of jugular venous pressure
Section 1:  General Medicine  69

♦♦ ‘v’ wave result from building of pressure in right atrium ♦♦ CNS: Disturbed sleep, fatigue, hallucination and mental
during ventricular systole when tricuspid valve is closed. depression.
This wave becomes prominent in tricuspid regurgitation ♦♦ Muscular weakness and tiredness
or congestive heart failure. ♦♦ Abrupt withdrawal can cause angina or frank myocardial
♦♦ All the above mention three waves i.e. a, c and v waves infarction.
are known as positive waves.
♦♦ There are two more waves present beside these which are Contraindications
known as negative waves or descents i.e. x and y descents. ♦♦ They are contraindicated in asthmatics and COPD.
♦♦ ‘x’ descent correspond with atrial relaxation and downward ♦♦ They are contraindicated in patients of Prinzmetal angina.
displacement of tricuspid valve at the time of ventricular ♦♦ They are contraindicated in peripheral vascular disease.
systole. It is prominent in constrictive pericarditis. ♦♦ They are contraindicated in patients with low cardiac
♦♦ Following the summit of ‘v’ wave, negative ‘y’ descent is reserve.
produced by opening of tricuspid valve and rapid flow of ♦♦ They are contraindicated in diabetic who are taking
blood in right ventricle. oral anti-diabetic drugs as they delay recovery from
♦♦ Prominent ‘y’ descent occurs in constructive pericarditis hypoglycemia.
and congestive heart failure along with ‘v’ wave, so it is
collectively known as prominent ‘v’ wave and ‘y’ descent. Uses or Indications of β Blockers
♦♦ Slow ‘y’ descent occurs in tricuspid stenosis where ‘a’ ♦♦ Hypertension: β blockers are used to treat all the grades
wave is prominent. of hypertension. They are also indicated in angina,
♦♦ Giant ‘a’ wave is a large sharp systolic wave seen in venous myocardial infarction and cardiac arrhythmias.
pulse during forcible contraction of right ventricle against ♦♦ In prophylaxis of angina and myocardial infarction: β
closed tricuspid valve. It is seen in complete heart block, blockers decreases oxygen demand of myocardium and
atrioventricular dissociation and junctional rhythm. decreases the heart rate. These drugs reduces the frequency
Q.63. Write short answer on beta blockers. of angina attacks. They are indicated in acute phase of
 (Apr 2018, 3 Marks) myocardial infarction so that they limit the size of infarct.
Ans. These drugs inhibit responses mediated by the ♦♦ In cardiac arrhythmias: They are indicated in atrial
adrenergic β blockers. arrhythmias i.e. atrial fibrillation, atrial flutter and
paroxysmal supraventricular tachycardia.
Classification ♦♦ Congestive cardiac failure: Chronic usage of β blockers
♦♦ Cardioselective (β1): Metoprolol, atenolol, acebutolol. reducec rate of mortality during chronic heart failure.
♦♦ β2 selective: Butoxamine. ♦♦ Pheochromocytoma: These drugs control cardiac
♦♦ Nonselective (β1+ β2): Propranolol, nadolol, sotalol, manifestations produced by pheochromocytoma.
timolol. ♦♦ Glaucoma: They decrease intra-ocular pressure by
decreasing secretion of aqueous humor. Timolol is the
Actions choice of drug.
♦♦ Heart: These drugs decreases force of contraction, ♦♦ In prophylaxis of migraine: They decreases the frequency
decreases AV conduction, decreases heart rate, decreases of migraine headache.
automaticity of myocardial fibers thus cardiac output is ♦♦ Hyperthyroidism: Sign and symptoms of hyperthyroidism
decreased. decreases due to blockade of β receptors.
♦♦ Blood vessels: On prolong administration BP falls in ♦♦ Essential tremors: Propranolol provides relief in tremors.
hypertensives. ♦♦ Anxiety: It acts as anti-anxiety drug.
♦♦ Respiratory system: It causes increase of bronchoconstriction ♦♦ Alcohol withdrawl: Propranolol provides benefit in alcohol
in asthmatics. withdrawl.
♦♦ Eye: β-blockers decrease aqueous humor secretion. ♦♦ Dissecting aortic aneurysm: β blockers reduce cardiac
contractility as well as development of pressure during
Adverse Reactions
systole.
♦♦ CVS: Bradycardia, heart block and can precipitate heart Q.64. Write short answer on calcium channel blocker.
failure.
♦♦ Respiratory system: these drugs can precipitate bronchial  (May 2018, 3 Marks)
asthma. Ans. Following are the calcium channel blockers:
70   Mastering the BDS IIIrd Year  (Last 25 Years Solved Questions)

Drug Action Side effects Uses


Nifedipine •  It is a potent calcium •  Headache Cardiovascular Uses:
10-50 mg orally or Sublingually antagonist. •  Lethargy •  In angina Pectoris
•  It decreases oxygen •  Flushing •  In Hypertension
consumption and reduces •  Tachycardia •  In Peripheral vascular disease
cardiac work by leading to •  Hypotension •  In achalasia cardia
peripheral vasodilatation •  In Raynaud’s phenomenon
and reducing the peripheral •  In Migraine
resistance •  In Nocturnal leg cramps
b) VERAPAMIL, AMLODIPINE •  It leads to coronary •  In High altitude pulmonary edema
dilatation and decreases •  Constipation •  In Left Ventricular Hyperplasia,
NICARDIPINE, ISRADIPLNE,
myocardial oxygen •  Hypotension arrhythmias, valvular diseases,
FELODIPINE, NIMODIPINE cerebrovascular diseases, post-
consumption. •  Vertigo
•  It interferes with inward •  Nervousnes myocardial infarction
displacement of calcium & Non-cardiovascular Uses
delays conduction within •  In Esophageal motility disorders
AV node. •  In Supraventricular arrhythmias
•  In Extrinsic bronchial asthma
DILTIAZEM •  This drug increases the •  Headache •  In Biliary or renal colic
30-60 mg TDS coronary blood flow, •  Flushing •  In Epilepsy
decreases myocardial •  Edema and hypotension. •  In Alzheimer disease
contractility, reduces •  Depression of AV nodal •  In Bone pain in transplant recipients
peripheral resistance conduction •  In Subarachnoid hemorrhage,
and blood pressure, thus •  Bradycardia nocturnal enuresis
increasing cardiac output
due to decreased afterload.
•  It increases exercise
tolerance.
PERHEXILINE MALEATE •  Actual action is not known •  Dizziness, headache
100-200 mg TDS but it: •  Hepatotoxicity
   • Reduces exercise induced •  Impotence
tachycardia. •  Polyneuropathy, myopathy
   • Causes vaso-dilatation
of systemic and coronary
vessels, decreases left
ventricular work & oxygen
consumption.
OXYFEDRINE It improves the myocardial •  Weakness, headache,
24-48 mg/day microcirculation. giddiness
•  Insomnia, nausea,
constipation

Clinical Features
5. Diseases of Symptoms
Respiratory System ♦♦ In most of the patients disease commence with high-grade
fever, chills and rigors, pleuritic chest pain and cough.
Q.1. Describe clinical features, investigations and manage- ♦♦ After some days, as the abscess cavity ruptures into a
ment of lung abscess. (Mar 2003, 15 Marks) patent bronchus, the patient suddenly starts expectorating
large quantities of sputum.
Or ♦♦ Sputum is very large in volume, purulent, foul smelling,
greenish yellow in color or it is often blood tinged.
Write in brief sign, symptoms and treatment of lung
Expectoration of cough varies with posture, i.e. more in
abscess. (May/June 2009, 5 Marks) lying down position than compared to sitting position.
Ans. It is localized pyogenic infection of the lung characterized ♦♦ In few of the cases, the lung abscess is more insidious in
by the *suppuration, destruction of lung parenchyma onset with low-grade fever, malaise, weight loss, anorexia
with cavitation and formation of abscess. and a deep-seated chest discomfort.

Q1. *Suppuration = Formation of pus.


Section 1:  General Medicine  71

♦♦ As the disease become chronic patient starts to loose his/ Duration of antibiotic therapy is variable. Some
her weight. of the patients require antibiotic therapy for 4 to 6
weeks. Injectables should be given in the emergency
Clinical Signs situations. Clindamiycin 600 mg IV 8 hourly is given
♦♦ During general examination, there is presence of anemia, ♦♦ Postural drainage:
fever, clubbing of fingers, halitosis and oronasal sepsis. • Percussion therapy or “clapping" over the site of the
♦♦ In most of the patients, there may be signs of consolidation, abscess with the patient in the postural drainage
i.e. coarse crepitations/crackles or rales, dullness on position is often effective in dislodging and expelling
percussion, increased vocal fremitus, vocal resonance, secretions from the cavity.
bronchial breathing and pleural rub appear. • Bronchoscopy: Suction is applied to the orifices of the
♦♦ In case of ruptured pyogenic abscess producing amphoric bronchi leading to segments presumed to be involved
or cavernous bronchial breathing heard over the area in the process in hope of initiating or promoting
involved, signs of pleural effusion, i.e. dull percussion note drainage. In addition any foreign material is removed
with absent breath sounds will be present. and a careful search made for a tumor.
♦♦ Signs of septicemia, i.e. fever, perspiration, tachypnea and • Oxygen inhalations: When sputum is foul because it
tachycardia are present. checks the anaerobic organisms
• Head elevation: Patient bed should be inclined to 45°
Investigations from horizontal plane in cases of altered mentation,
on mechanical ventilatory support.
♦♦ Blood examination: There can be presence of normocytic
♦♦ Surgical resection: If at the end of 3 weeks, there is no clinical
normochromic anemia. Leukocytosis is present in infective
and radiological improvement, then segmental resection
abscess. Raised ESR is also present.
of lung, lobectomy or pneumonectomy is done.
♦♦ Examination of sputum: It consists of isolation of infective
microorganisms by Gram’s stain and acid-fast bacilli by Q.2. Describe etiology, clinical features and management
Ziehl-Neelsen stain, Aerobic and anaerobic cultures and of lung abscess. (Aug 2012, 15 Marks)
sensitivity, malignant cells are detected by the special Ans. Lung abscess is the localized pyogenic infection of the
stains. lung characterized by the suppuration, destruction
♦♦ Urine examination: This is carried out for proteinuria, pus of lung parenchyma with cavitation and formation of
cells and cases. Albuminuria is indicative of amyloidosis abscess.
which is a complication of chronic lung abscess.
♦♦ Chest X-ray: It shows radiolucency in the the area of Etiology
consolidation. Wall or border of cavity completely ♦♦ Infection: Staphylococci, streptococci, Pneumococcus and
surrounds the radiolucent area and air-fluid level is seen. fusiform bacilli.
Associated involvement of pleura is noted by obliteration ♦♦ Aspiration of infected material from the oral cavity and
of CP angle. throat.
♦♦ Fiberoptic bronchoscopy: It is done to rule out bronchogenic ♦♦ As a complication of pneumonia when the organism is
cause of lung abscess. It also exclude malignancy, obtain virulent and patient’s immunity is low.
specimen for studies and for removing the secretions. ♦♦ Bronchial obstruction as in bronchogenic carcinoma.
♦♦ CT scan of thorax may detect lung abscess with certainity. ♦♦ Embolic infection with the cavity formation.
Management ♦♦ Necrosis and abscess formation of growth in the lung.
♦♦ Metastatic lung abscess.
♦♦ General measures:
♦♦ Rupture of amoebic liver abscess into lung.
• Bed rest
• Oxygen is given Complications
• High protein diet is taken
• Exercise is avoided. ♦♦ An abscess may extend to the surrounding structures
♦♦ Specific antimicrobial treatment: producing mediastinitis and pericarditis.
• Intensive antimicrobial therapy is employed ♦♦ Involvement of pleura causes *pleurisy, *empyema and
depending on the sputum culture and drug sensitivity *pyopneumothorax.
of the organism ♦♦ Pleural effusion, pneumothorax, bronchopleural fistula
• Most of the patients with lung abscess respond to ♦♦ Metastatic cerebral abscess
oral ampicillin 500 mg QDS or cotrimoxazole 960 ♦♦ Severe hemoptysis may occur.
mg BD is given. Along with this oral metronidazole ♦♦ Extrapulmonary complications include pain, abscess,
400 mg 8 hourly is given along with other antibiotics. *cachexia and amyloidosis.

Q2. *Pleurisy = Inflammation of pleural cavity       *Empyema = A collection of inflamed, infected fluid between the pleura.
*Pyopneumothorax = The presence of pus and gas in pleural cavity, *Cachexia = A state of ill-health, malnutrition and wasting
72   Mastering the BDS IIIrd Year  (Last 25 Years Solved Questions)

♦♦ Aspergilloma ♦♦ Nonspecific irritants (e.g. cigarette smoke, perfumes, paints)


♦♦ A chronic form of lung abscess may leave behind ♦♦ Drugs (e.g. B-blockers, aspirin, NSAIDs)
bronchiectasis and chronic cavity with fibrosis. ♦♦ Emotion or stress
For management and clinical features, refer to Ans 1 of ♦♦ Occupational exposure
same chapter.
Clinical Features
Q.3. Write short note on bronchial asthma. 
 (Dec 2008, 5.5 Marks) ♦♦ Symptoms of bronchial asthma can be episodic or
Or persistent.
Describe in brief on bronchial asthma. ♦♦ Asthma is characterized by paroxysms of dyspnea, cough
 (Dec 2015, 4 Marks) and wheezing commonly in children and young adults.
Or ♦♦ Attacks can be mild or severe or may last for hours, days
Describe briefly the management of bronchial asthma. or rarely weeks. Between episodes, the patients become
 (Sep 1998, 5 Marks) asymptomatic.
Or ♦♦ In older patients, asthma is chronic and persistent.
Outline the management of allergic bronchial asthma. Symptoms get worst in the early morning.
 (Sep 2010, 5 Marks) (Mar 1998, 5 Marks) ♦♦ On examination, there is tachypnea, tachycardia as well as
Or involvement of accessory respiratory muscles.
Describe clinical features, diagnosis and mana­gement ♦♦ Breath sound become harsh, vesicular with prolonged
of bronchial asthma. (Mar 2011, 4.5 Marks) expiration. Prominent wheeze should be audible in both
phases of respiration.
Or
♦♦ Acute severe asthma is a life-threatening attack of asthma
Describe clinical features of bronchial asthma. which is previously called as status asthmaticus. In
 (Feb 2013, 5 Marks) this patient may additionally have tachycardia, pulsus
Or paradoxus, cyanosis and active accessory respiratory
Discuss clinical features, complications investigations rnuscles. The air entry is drastically reduced with silent
and treatment of bronchial asthma. (Apr 2018, 5 Marks) chest on auscultation. The patient may become confused
Or or drowsy.
Write short answer on bronchial asthma. 
Investigations
 (May 2018, 3 Marks)
Ans. Bronchial asthma consists of an increased responsiveness ♦♦ Chest radiograph may be normal, or show signs of
of bronchial tree, which is characterized by frequent segmental or lobar collapse.
attacks of dyspnea due to generalized bronchial ♦♦ Full blood count reveals eosinophilia.
constriction. ♦♦ Sputum: On examination eosinophils, Charcot-Leyden
crystals and at times Curschmann‘s spirals may appear
Etiology purulent (due to eosinophilic leucocytes) in absence of
♦♦ Genetic and environmental factors also cause bronchial infection).
asthma. ♦♦ Skin tests: A patch test is done with aqueous solution of
♦♦ Early onset asthma starts in childhood, there should be substance to be tested. Positive test indicates wheal and
family history of atopy, i.e. external antigen and other flare. Tests are performed with groups of substances or
allergic disorders, i.e. urticaria, allergic rhinitis and allergens which produce asthma.
eczema. In this, skin tests are positive for allergies and ♦♦ Lung function test: Spirometry shows reduction in FEV1,
there are high levels of IgE antibody in serum. There is FEV1/FVC ratio and peak expiratory flow rate (PEFR).
positive response to provocative tests which involve in Reversibility, which is one of characteristic feature of
inhalation of specific antigen. asthma, is shown by 200 mL increase in FEV, 15 minutes
♦♦ Late onset asthma occurs in adulthood. In this, there is no after inhaled short acting β2 agonist or 2-4 weeks trial of
effect of atopy along with no family history of allergens. oral corticosteroids.
Skin tests are normal and there are no increased levels of ♦♦ Provocation (challenge) tests: Exercise challenge tests useful
IgE antibody in serum. in young adults and can be used to confirm diagnosis of
asthma, since fall in FEV, or PEFR occurs after 5-7 minutes
Various Trigger Factors for Bronchial Asthma
of vigorous exercise in most patients with asthma.
♦♦ Night or early morning ♦♦ IgE and IgE specific test: Elevation of total serum IgE
♦♦ Exercise (especially running) supports diagnosis of atopy, and measurement of fractions
♦♦ Cold air, fog of IgE specific to one allergen, radioallergosorbent test
♦♦ Viral respiratory tract infection (RAST) can be useful in some patients in whom a specific
♦♦ Allergens (e.g. house dust, mite, cat fur) allergy is suspected.
Section 1:  General Medicine  73

Complications of Bronchial Asthma cases even in the presence of CO2 retention. Oxygen
supply should be started immediately through mask,
Following are the complications of bronchial asthma
and concentration adjusted according to blood gas
♦♦ Respiratory
measurement. PaO2 of greater than 8.5 to 9.0 kPa
• From mucus plugs
should be maintained, if possible.
–– Atelectasis lobar or lobular
• High doses of salbutamol, i.e. 2.5–5 mg or terbutaline
–– Bronchiectasis
5–l0 mg by nebulizer should be given initially
• From cough
and repeated after 30 minutes, necessary. If no
–– Subcutaneous emphysema
improvement occurs with nebulised therapy, then
–– Mediastinal emphysema
250 µg of salbutamol or terbutaline may be given by
–– Spontaneous pneumothorax
IV infusion over 10 minutes.
–– Cystic degeneration of lungs
• In all severe cases of acute asthma systemic
–– Spontaneous rib fracture
corticosteroids, i.e. hydrocortisone 200 mg I.V. stat
• From infection
and then 4 to 6 hourly or oral prednisolone 40–60 mg/
–– Recurrent bronchitis and pneumonia
day to tide over the crisis.
• From uneven ventilation and pulmonary perfusion
• Systemic bronchodilators such as aminophylline
–– Respiratory failure and cor pulmonale
250 mg IV over a period of 30 minutes may be given
♦♦ Cardiac
immediately followed by either 8 hours doses or
• Dysryhtmias from hypoxia and stress of asthma,
continuous infusion not exceeding total dose of 1.5
compounded by bronchodilator therapy with β
g/day.
agonist and theophylline
• Ipratropium bromide can be used in acute severe
• Myocardial infarction rarely in severe asthma
asthma in doses of 0.5 mg added to a nebulized beta-
♦♦ Hypokalemia: Due to high dose corticosteroids, high dose
agonist.
β agonists, respiratory alkalosis of hypocapnia
• Reassess the patient by PEFR and arterial blood
♦♦ Other complications
gas analysis. If recovery is good, then continue
• Nausea and vomiting from theophylline
oxygen therapy and oral prednisolone, i.e. 40
• Acute myopathy due to high dose I.V. steroids
mg/day in decreasing doses is given. Nebulized
Management β-adrenoreceptor agonist may be continued every 4–6
hours and then replaced by metered dose inhalation.
♦♦ Identify the allergens and wherever possible, exposure to IV hydrocortisone 200 mg 6 hourly may be continued
such agents must be avoided. for 24–48 hours in severe cases followed by oral
♦♦ Drug therapy: The drugs used in asthma can be grouped steroids. lf response is not good, then shift the patient
as: (a) quick relievers, which inhibit smooth muscle in respiratory intensive care for assisted ventilation.
contraction and cause bronchodilatation and (b) long • Assisted ventilation: Mechanical ventilation can be life
term control medications, which prevent or reverse saving in few patients who are critically ill.
inflammation.
Management of Chronic Asthma
Management of Acute Severe Asthma
For management of chronic bronchial asthma, a stepwise
♦♦ Treatment of severe acute asthma at home:
approach is chosen according to the severity of disease. Once the
• Administer oxygen (40-60%) through mask, if disease is controlled, a step-down therapy should be attempted.
available.
♦♦ Step 1: Use of inhaled bronchodilators: Here inhaled short-
• Bronchodilator: IV Aminophylline, i.e. 250-375 mg in
acting β2 adrenoreceptor agonists bronchodilators such as
20 mL of saline slowly after checking blood pressure salbutamol or terbutaline 100 to 200 μg are used as needed
Or IV, Salbutamol 250 µg in 20 mL of saline over 10 for minor symptoms.
min Or IV Terbutaline 250 µg in 20 mL of saline over ♦♦ Step 2: Use of bronchodilators with regular use of inhaled anti-
10 min. inflammatory agents: If the symptoms are not controlled
• Alternatively: by inhaled adrenergic drugs, then a low dose of inhaled
–– Salbutamol (5 mg) or terbutaline (10 mg) by steroid is added i.e. beclomethasone diproprionate or
nebulizer. budesonide upto 800 μg twice a day.
–– Give hydrocortisone sodium succinate 200 mg ♦♦ Step 3: Use of bronchodilators with high doses of steroids: Dose
IV stat. of inhaled steroid is increased till 800–2000 μg a day.
–– Arrange for emergency admission to a hospital in ♦♦ Step 4: Use of high-dose corticosteroids and bronchodilators with
ambulance equipped with oxygen therapy. therapeutic drug: In addition to drugs used in Step 3, An
–– Give prednisolone 60 mg orally inhaled long-acting adrenergic agent such as salmeterol or
♦♦ In hospital treatment: formoterol can be added or sustained released theophylline
• High concentration of oxygen, i.e. 40–60% at high can be used orally or inhaled ipratropium bromide or
flow rate should be given. It is recommended in all sodium chromoglycate are tried.
74   Mastering the BDS IIIrd Year  (Last 25 Years Solved Questions)

♦♦ Step 5: Addition of oral steroids: Regular oral steroid, i.e. • Salbutamol (5 mg) or terbutaline (10 mg) by nebulizer.
prednisolone 20–30 mg/day in single dosage is added to • Give hydrocortisone sodium succinate 200 mg IV stat.
Step 4 regimen to control symptoms. • Arrange for emergency admission to a hospital in
ambulance equipped with oxygen therapy.
Patient Education and Monitoring of Therapy • Give prednisolone 60 mg orally
♦♦ Educate the patients about the nature of disease as well as
In Hospital Treatment
its treatment. Patients are trained to recognize the severity
of their disease and monitor the response to therapy with ♦♦ High conc. of oxygen, i.e. 40–60% at high flow rate
the use of peak flow meter. should be given. It is recommended in all cases even in
♦♦ Demonstrate the proper use of inhalation devices such the presence of CO2 retention. Oxygen supply should be
as metered-dose inhalers (pressurized aerosol system), started immediately through mask, and concentration
rotahaler (dry powder system) and nebulizers. adjusted according to blood gas measurement. PaO2 of
♦♦ Encourage the usage of inhaler therapy because it is greater than 8.5 to 9.0 kPa should be maintained if possible.
effective in lower dosage together with a rapid onset of ♦♦ High doses of salbutamol, i.e. 2.5–5 mg or terbutaline 5–
action and has less side effects. l0 mg by nebulizer should be given initially and repeated
after 30 minutes, necessary. If no improvement occurs with
Q.4. Write short note on status asthmaticus. 
nebulised therapy then 250 µg of salbutamol or terbutaline
 (Oct 2007, 5 Marks)
may be given by IV infusion over 10 minutes.
Or ♦♦ In all severe cases of acute asthma systemic corticosteroids
Describe briefly the management of status asthmati- i.e. hydrocortisone 200 mg IV stat and then 4 to 6 hourly
cus. (Dec 2012, 4 Marks) or oral prednisolone 40–60 mg/day to tide over the crisis.
♦♦ Systemic bronchodilators such as aminophylline 250 mg
Or IV over a period of 30 minutes may be given immediately
Outline the management of status asthmaticus.  followed by either 8 hours doses or continuous infusion
 (Sep 1999, 5 Marks) not exceeding total dose of 1.5 g/day.
Ans. Status asthmaticus is a continuous state of breadthlessness ♦♦ Ipratropium bromide can be used in acute severe asthma
without any period of relief. A prolong attack of in doses of 0.5 mg added to a nebulised beta-agonist.
continuous asthma with fluctuations also comes under ♦♦ Reassess the patient by PEFR and arterial blood gas
status asthmaticus. analysis. If recovery is good, continue oxygen therapy
and oral prednisolone, i.e. 40 mg/day in decreasing doses
Clinical Features is given. Nebulized β-adrenoreceptor agonist may be
continued every 4–6 hours and then replaced by metered
♦♦ Presence of repeated dry cough which causes aggravation
dose inhalation. IV hydrocortisone 200 mg 6 hourly may
of dyspnea and respiratory distress.
be continued for 24–48 hours in severe cases followed by
♦♦ Patient sweats heavily and there is also presence of
oral steroids. lf response is not good, then shift the patient
tachycardia.
in respiratory intensive care for assisted ventilation.
♦♦ Presence of an increased pulse rate till 120/min.
♦♦ Assisted ventilation: Mechanical ventilation can be life
♦♦ Respiratory rate is 30/min
saving in few patients, who are critically ill.
♦♦ Presence of pulsus paradoxsus
♦♦ As breathlessness is present, patient is unable to speak. Patient Education and Monitoring of Therapy
♦♦ Peak expiratory flow is falls to less than 50%.
♦♦ Educate the patients about the nature of disease as well as
♦♦ When patient develops carbon dioxide retention,
its treatment. Patients are trained to recognize the severity
hypoxemia and acidosis, there can be occurrence of life-
of their disease and monitor the response to therapy with
threatening situation.
the use of peak flowmeter.
♦♦ CNS effects such as confusion, drowsiness,
♦♦ Demonstrate the proper use of inhalation devices such
semiconciousness and cyanosis develops.
as metered-dose inhalers (pressurized aerosol system),
♦♦ In critically ill patients, chest becomes silent due to
rotahaler (dry powder system) and nebulizers.
decrease in the air entry.
♦♦ Encourage the usage of inhaler therapy because it is
Management effective in lower dosage together with a rapid onset of
action and has less side effects.
Treatment of Severe Acute Asthma at Home
♦♦ Administer oxygen (40–60%) through mask, if available. Q.5. Mention the complications of lobar pneumonia.
♦♦ Bronchodilator: IV Aminophylline i.e. 250–375 mg in 20  (Sep 2006, 5 Marks)
mL of saline slowly after checking blood pressure Or IV, Ans. Complications of Pneumonia
Salbutamol 250 µg in 20 mL of saline over 10 min Or IV. In lung:
Terbutaline 250 µg in 20 mL of saline over 10 min. • Pleural effusion
♦♦ Alternatively • Pneumothorax
Section 1:  General Medicine  75

• Empyema • Acute dilatation of stomach


• Lung abscess • Paralytic ileus
• Respiratory failure. • Peritonitis.
In cardiac involvement: Other Complications
• Pericarditis ♦♦ Pneumococcal meningitis
• Endocarditis ♦♦ Arthritis
• Myocarditis ♦♦ Parotitis
• Cardiac failure. ♦♦ *Otitis media
GIT complications: They are uncommon and may be in ♦♦ Nephritis
form of: ♦♦ Venous thrombosis.

Q.6. Write note on management of pneumonia.


 (Oct 2003, 15 Marks) (Apr 2010, 15 Marks)
Ans.

Primary pneumonias
Type of Management
pneumonia
Pneumococcal • Initially oral amoxicillin should be given 500 mg 8 hourly or erythromycin 500 mg 6 hourly.
pneumonia • If patient is very ill or gram negative or staphylococcal infection is present IV ampicillin 0.5 to 1gm 6 hourly + flucoxacillin
250–500 mg IV 6 hourly + gentamycin 60–80 mg every 8 hourly IV is given.
• Antibiotic therapy should be given for 7 to 10 days.
• Choice of antibiotic depends on the causative microorganisms.
• Oxygen therapy is given in seriously ill patients. Oxygen should be delivered at very high rate.
• Analgesics such as mefenamic acid 250–500 mg or pethidint 50–100 mg or morphine 10 to 15 mg IM or IV injections
should be given.
• Physiotherapy is given to patient by encouraging him to cough and to take deep breadth as pleuritic pain disappears
Staphylococcal • IV flucloxacillin 0.5–1 g 6 hourly or erythromycin 0.5–1 g every 6 hourly is given in cases with mild infection.
pneumonia • If infection is severe than IV sodium fusidate in dosage of 500 mg 6 hourly is given thrice a day in addition to cloxacillin
and erythromycin. Treatment should be given for 2 weeks.
Klebsiella • IV gentamycin 2–5 mg/kg is given in divided doses for every 8 hours.
pneumonia • IV Ceftazidime or cefotaxime 1 g 8 hourly is given or ciprofloxacin 200 mg after every 12 hours IV infusion for 30 to
60 min is choice of treatment.
Legionella • Erythromycin 1 gm 8 hourly IV for 13 weeks followed by 500 mg qds for 2 weeks.
pneumonia • Doxycycline 100 mg twice a day orally for 3 weeks.
• Rifampicin 600 mg twice a day orally for 3 weeks.
• In Legionella endocarditis, the treatment with antibiotics has to be continued for 3–12 months.
Primary atypical • Oral tetracycline 500 mg 6 hourly or erythromycin 500mg 6 hourly is effective.
pneumonia • If severe case is present, above drugs should be given parenterally.
Viral pneumonia • Disease is self-limiting.
• Antipyretics can be given
Secondary pneumonia
Acute broncho • Oral Amoxycillin 250–500 mg after every 8 hours Or cotrimoxazole 960 mg twice a day is usually effective.
is pneumonia • In cases with severe infection, cefotaxime or ceftriaxone 1 gm IV twice a day may be used.
• If cyanosis is present, then oxygen therapy is given.
• Physiotherapy is advised to older and debilitated patients who get recovered from acute episode.
Aspiration • Oral Amoxycillin 500 mg 8 hourly or Ampicillin 500 mg 6 hourly Or cotrimoxazole 960 mg twice a day is usually effective.
pneumonia • For anaerobes, oral metronidazole 400 mg 8 hourly is given.
• Above treatment should be given for 2 weeks.
• Analgesics should be given, if pain is present.
• Physiotherapy and postural drainage is required in cases of lung abscess.
Nosocomial • In mild cases, amoxicillin with clavulanic acid 500 mg TDS is given.
pneumonia • In severe cases IV cefuroxime 750 mg every 8 hourly + Clarithromycin 250 mg every 8 hourly is given.
P n e u m o n i a i n • Before culture and sensitivity report is received, a third generation cephalosporine, i.e. cefotaxime 1.0 g IV twice a day
immunocompro- or a quinolone, i.e. ciprofloxacin 200 mg IV + amoxycillin 500 mg IV after every 8 hours can be started.
mised host • At times, gentamicin 80 mg after every 8 hours may be added to the above regimen.
• Metronidazole can be added is anaerobic infection is present.
• After treatment, if no response is gained under a week bronchoscopy is done and lavage is sent for culture and
sensitivity
76   Mastering the BDS IIIrd Year  (Last 25 Years Solved Questions)

Q.7. Outline the investigation and management of com- ♦♦ In pneumococcal pneumonia, a homogeneous opacity
munity acquired pneumonia. (Mar 2000, 5 Marks) is seen localized to the affected lobe or segment which
Or appears within 12 to 18 hours from onset of illness.
Discuss briefly the management of community ac-
quired pneumonia. (Feb/Mar 2004, 5 Marks) Microbiological Investigations
Ans. Community acquired pneumonia is also known as
♦♦ Most of the cases of community acquired pneumonia
primary pneumonia.
are successfully managed, if they are not severe. So the
Community acquired pneumonia is defined as an infection that
need of microbiological investigation is based on clinical
begins outside the hospital or is diagnosed within 48 hours after
circumstances.
admission to hospital in a patient who has not resided in a long-
♦♦ In severe cases of community acquire pneumonia a full
term facility for 14 days or more before the onset of symptoms.
range of microbial tests should be carried out.
Investigations ♦♦ If patient does not respond to the initial therapy,
microbiological tests provide proper modification of
Blood Tests
therapy.
♦♦ WBC count is marginally raised or is normal in patients
with pneumonia caused by atypical microorganisms. Assessment of Gas Exchange
♦♦ Absolute neutrophil count is more than 15 × 109/L and this
is indicative of bacterial etiology. ♦♦ Pulse oximetry is the non – invasive method which
♦♦ A very high or low WBC count can be seen in severe measures the arterial oxygen saturation and monitors the
pneumonia. response to oxygen therapy.
♦♦ C-reactive protein is typically elevated. ♦♦ An arterial blood gas is sampled in patients with low
arterial oxygen saturation or having features of severe
Radiological Examination pneumonia to assess whether patient has evidence of
♦♦ A confident diagnosis should always need a chest X-ray. ventilator failure or acidosis.

Management of Community Acquired Pneumonias

Community acquired pneumonias


Type of Management
pneumonia
Pneumococcal • Initially oral amoxicillin should be given 500 mg 8 hourly or erythromycin 500 mg 6 hourly.
pneumonia • In severe hospitalized cases of community acquired pneumonia, IV ampicillin or amoxicillin should be given 500mg 8
hourly. IV cefotaxime or ceftriaxone may be given in penicillin-resistant strains. Vancomycin is given in infections resistant
to penicillin or cephalosporin
• Antibiotic therapy should be given for 7–10 days.
• Choice of antibiotic depends on the causative microorganisms.
• Oxygen therapy is given in seriously ill patients. Oxygen should be delivered at very high rate.
• Analgesics such as mefenamic acid 250 to 500mg or pethidine 50–100 mg or morphine 10–15 mg IM or IV injections
should be given.
• Physiotherapy is given to patient by encouraging him to cough and to take deep breath as pleuritic pain disappears.
Staphylococcal • IV flucloxacillin 0.5–1 g 6 hourly or erythromycin 0.5–1 g every 6 hourly is given in cases with mild infection.
pneumonia • If infection is severe than IV sodium fusidate in dosage of 500 mg 6 hourly is given thrice a day in addition to cloxacillin
and erythromycin. Treatment should be given for 2 weeks.
Klebsiella • IV gentamycin 2–5 mg/Kg is given in divided doses for every 8 hours.
pneumonia • IV Ceftazidime or cefotaxime 1g 8 hourly is given or ciprofloxacin 200mg after every 12 hours IV infusion for 30–60 min
is choice of treatment.
Legionella • Erythromycin 1gm 8 hourly IV for 13 weeks followed by 500 mg qds for 2 weeks.
pneumonia • Doxycycline 100 mg twice a day orally for 3 weeks.
• Rifampicin 600 mg twice a day orally for 3 weeks.
• In Legionella endocarditis, the treatment with antibiotics has to be continued for 3–12 months.
Primary • Oral tetracycline 500 mg 6 hourly or erythromycin 500 mg 6 hourly is effective.
atypical • If severe case is present, then above drugs, should be given parenterally.
pneumonia
Viral • Disease is self limiting.
pneumonia • Antipyretics can be given.
Section 1:  General Medicine  77

Q.8. Describe the clinical features and management of Diagnosis


pulmonary embolism. (Mar 1998, 10 Marks)
It is in the patients who had suspicion for underlying cause
Or for emboli formation, development of pulmonary sign and
Write short note on pulmonary embolism. symptoms as well as cardiovascular involvement, presence
 (May/June 2009, 5 Marks) (Feb 1999, 5 Marks) of thrombophlebitis in deep leg veins, prolonged bed rest,
immobilization, cardiac irregularity in form of atrial fibrillation
Or
should be considered while keeping in mind clinical picture
Enumerate etiology, clinical features, investigations, of precordial pain, breathlessness and tachycardia in patient
complications and management of pulmonary who had recently gone for major surgery. Examination of
embolism  (Aug 2011, 15 Marks) veins is mandatory in the patients who are at high risk for
Ans. Pulmonary embolism is the most common and fatal form development of deep vein thrombosis. These features along
of venous thromboembolism in which there is occlusion with investigatory features form the diagnosis. Following are
of pulmonary arterial tree by thromboemboli. the investigations:
Etiology Investigations
♦♦ Thrombotic: ♦♦ Blood examination: If pulmonary infarct is present, there
• Deep vein thrombosis can be leucocytosis or raised ESR.
• Congestive heart failure ♦♦ Chest X-ray: In massive pulmonary embolism, there is
• Right-sided endocarditis presence of diffuse infiltrates in the lung with increased
• Atrial fibrillation. bronchovascular markings. If medium size vessels are
♦♦ Non-thrombotic: involved, there will be triangular pleuropulmonary opacity
• Fat embolism in peripheral lung fields, there can also be pleural effusion
• Amniotic fluid embolism: Spontaneous delivery and present.
cesarean section ♦♦ Arterial blood gas analysis: Presence of hypoxemia and
• Tumor embolism: Choriocarcinoma hypocapnia.
• Parasitic embolism: Schistosomiasis
♦♦ D-dimer: It is a fibrin degradation product release in
• Air embolism: Pulmonary barotraumas generally in
circulation in pulmonary embolism. Presence of high levels
the sea divers.
of D-dimer is suggestive of an embolism while presence of
Clinical Features low D-dimer exclude pulmonary embolism.
♦♦ Echocardiography: It shows the right ventricular dilatation
In acute massive embolism, i.e. acute cor pulmonale
and presence of clot in it.
♦♦ Symptoms are of presence of acute dyspnea, tachypnea, ♦♦ Spiral CT scan: CT of chest along with the IV contrast
tachycardia, hemoptysis and chest pain. diagnose the pulmonary embolism. It effectively diagnose
♦♦ Signs are increase in the jugular venous pressure, presence
the large and central pulmonary embolism. Newer
of central cyanosis, Loud P2 and narrow splitting of P2,
scanners can also detect peripherally present emboli.
an ejection systolic murmur in P2 area, right ventricular
♦♦ Pulmonary angiography: It demonstrates the site of
hypertrophy, signs of shock,
obstruction of all sized blood vessels.
In small- or medium-sized pulmonary vessels embolization
♦♦ Symptoms are hemoptysis, pleuritic pain and wheeze Complications
which is the triad of pulmonary infarct. ♦♦ The foremost complication for pulmonary embolism is
♦♦ Signs are of pleural effusion i.e. reduced or absent pulmonary hypertension.
chest wall movement and expansion of chest on the ♦♦ Right-sided heart failure
side involved, activity of extrarespiratory muscles is ♦♦ Peripheral segmental infarctions.
absent, position of trachea and mediastenum is shifted to
opposite side, percussion note is stony dull on the side of Management
involvement, vocal fremitus is reduced or absent on the ♦♦ In patient of massive embolism
side involved, breath sounds are absent or diminished • If patient is in state of shock or collapse:
over the area involved. –– Vasopressors such as dopamine or dobutamine
In multiple microembolization, i.e. chronic cor pulmonale are to be given.
♦♦ Symptoms are of presence of dyspnea, weakness, fatigue –– Administer oxygen to the patient.
and syncope. –– Correct acidosis
♦♦ Signs are increase in the jugular venous pressure, presence –– If there is failure of an initial resuscitation or there
of cyanosis, edema, hepatomegaly, presence of loud P2 is hypotension or right ventricular dysfunction,
with the ejection systolic murmur in P2 area, presence of primary therapy should be administered,
narrow splitting of P2, presence of parasternal heave and i.e. dissociation of clot by thrombolysis or
right ventricular atrophy. embolectomy.
78   Mastering the BDS IIIrd Year  (Last 25 Years Solved Questions)

• If acute event is survived by the patient, ♦♦ Patient suffers from general *debility, weight loss, low
–– Streptokinase 2.5–5 lac unit IV > in dextrose or grade fever and malaise.
saline is given for 30 min followed by 1 lac IV ♦♦ Presence of lymphadenopathy and splenomegaly
for 24 hours. ♦♦ Nocturnal bronchospasm
–– Recombinant tissue plasminogen activator
tPA, i.e. Alteplase 100mg for 2 hours is a good Diagnosis
alternative to thrombolytic therapy. ♦♦ Positive history for prolong presence of patient in endemic
• Anti coagulation therapy is an initial and immediate area.
treatment of choice. ♦♦ Lacking of microfilaria in blood, in both night as well as
–– Loading dose of 80–150 units/kg is given which day samples.
is followed by 15 to 18 units/kg/hr as continuous ♦♦ Presence of high titers of filarial antibodies.
infusion. ♦♦ IgE levels are 1000 units/mL.
–– Warfarin should be added to heparin and is ♦♦ Peripheral eosinophilia with 3000 cells/mL.
continued for 5 days.
–– After 5 days taper the heparin and administer Treatment
warferin for 6 weeks to 6 months. The drug of choice is diethyl carbamazine given in dose of
–– Monitor anticoagulant therapy by PTT or INR 5 mg / kg body weight for 2 weeks
ratio or bleeding time which should be 2–3 times
♦♦ In adult dosage, three tab of 50 mg four times a day for
than the control.
5 days.
♦♦ In small embolisms
♦♦ In addition patient will require bronchodilators to relieve
• Analgesics, i.e. NSAIDs should be given to relieve
the bronchospasm.
the pain.
♦♦ Corticosteroids and anti-histaminics are employed.
• Anticoagulant is given to prevent further embolization.
• Various preventive measures should be undertaken ♦♦ Recovery after therapy is good.
such as calf muscle exercise, elastic stockings, prolong ♦♦ Repeat course of drug may have to be given after six weeks.
immobilization at bed, respiration exercises should Q.10. Write short note on acute respiratory failure.
be done. (Apr 2010, 5 Marks) (Mar 2000, 5 Marks)
Q.9. Write short note on tropical eosinophilia.  Ans. Respiratory failure is defined as failure of respiratory
 (Mar 1998, 5 Marks) system to maintain normal partial pressure of oxygen
and carbon dioxide in the blood.
Ans. Tropical eosinophilia: A disease of tropical countries (India,
Pakistan, Bangladesh, Sri Lanka, etc.) is characterized by Types of Acute Respiratory Failure
chronic cough, attack of breathlessness, lassitude and
♦♦ Type I acute respiratory failure: In this, there is acute
weight loss with rise in eosinophilic count in blood.
alteration in blood gases concentration with hypoxemia
Etiology and normo- or hypocapnia because of tachypnoea or
hyperventilation.
The most important cause of tropical eosinophilia is allergic
♦♦ Type II acute respiratory failure: It is also known as asphyxia.
reaction to filarial worm, e.g. Wuchereria bancrofti.
In this, there is hypercapnia and acute respiratory acidosis.
Pathology Causes
Patients having tropical eosinophilia lack IgG blocking anti- ♦♦ Type I acute respiratory failure:
bodies against the circulating microfilaria. Microfilaria is • Acute asthma
removed from peripheral circulation and is trapped in various • Pulmonary embolism
tissue sites by IgG dependent cell-mediated effector mechanism. • Pulmonary edema.
Antigens are released as parasites get destroyed which initiates • Acute respiratory distress syndrome
Type I hypersensitivity reaction. Peripheral eosinophilia occurs • Pneumothorax
due to a reaction which further progress to granuloma and • Pneumonia
fibrosis. ♦♦ Type II acute respiratory failure:
• An inhaled foreign body
Clinical Features • Status asthmaticus
♦♦ Chronic cough of several weeks or months duration is the • Paralysis of respiratory muscles
prominent complaint. • Fractured ribs
♦♦ Cough may be dry or mucoid to mucopurulent • Brainstem ischemia
♦♦ There is constricting sensation in the chest. • Overdose of narcotic drugs

Q9. *Debility = Weakness or lack of strength.


Section 1:  General Medicine  79

Management Physical Signs


♦♦ Removal of underlying cause is mandatory. On Inspection
♦♦ Hospitalize the patient and treat in the respiratory ♦♦ Presence of bilaterally symmetrical chest.
intensive care unit. ♦♦ Restrictions of movements of chest on the side of effusion.
♦♦ Supervise coughing in a conscious patient and change
the patient’s position from side to side which helps in On Palpation
clearing airway. ♦♦ Shifting of trachea and mediastinum to opposite side on
♦♦ By using a rolled gauze piece clear the thick secretions in pushing.
oral cavity by holding in an artery forcep. ♦♦ Diminishing of expansion of chest on involved side.
♦♦ Secretions at back of throat or in trachea are removed by ♦♦ Absence of vocal fremitus on involved side.
frequent secretion.
♦♦ High concentration of oxygen is given to the patients via On Percussion
ventimasks to improve hypoxemia and ventilation. For
Stony dull percussion note on the involved side.
small children oxygen tents are used. Oxygen therapy
should be continued till patient achieve normal level of On Auscultation
blood gases.
♦♦ Absence of breath sounds or diminishing of breath sounds
♦♦ In Type II, acute respiratory failure immediate reversal of
over area of pleural effusion.
precipitating factor should be done. In cases where reversal
♦♦ Amphoric bronchial breathing can be heard at apex of
cannot occur temporary ventilator support is required.
pleural effusion at interscapular region over the involved
♦♦ If patient get worse even after taking above-mentioned
side.
treatment, tracheostomy and endotracheal intubation
♦♦ Absence of vocal resonance.
is done and now the patient should be supported with
intermittent positive pressure ventilation. Investigations
♦♦ Mucolytic agents such as bromohexine liquefy secretion.
Acetylcystine, i.e. 1–2 mL of 20% solution is instilled via ♦♦ Examination of pleural fluid:
tracheostomy tube. • Pleural fluid is serous or straw-colored and forms
♦♦ Patients suffering from acute respiratory distress syndrome cobwebs on standing.
can be on positive end expiratory pressure. • Fluid is exudates in nature.
♦♦ Intravenous fluids and electrolyte therapy is given. • On microscopic examination, prominent cell type in
fluid is lymphocyte.
♦♦ Underlying infection should be treated by proper
♦♦ Tuberculin test is positive.
antibiotics.
♦♦ Pleural biopsy show granulomatous lesion.
♦♦ H2 blockers are given via IV drip
♦♦ Culture for acid-fast bacilli is positive.
♦♦ Patient should be slowly weaned from respirator as
♦♦ Polymerase chain reaction (PCR) test for tuberculosis is
voluntary effort is gained from ventilation as etiology is
positive.
corrected.
♦♦ X-ray chest shows uniform dense opacity with upward
Q.11. Write short note on tuberculous pleural effusion.  concavity in lower and lateral part of hemithorax pushing
 (Mar 1997, 5 Marks) the lung medially. Sharp angle between diaphragm and
Ans. Pleural effusion: Abnormal accumulation of fluid in rib cage get altered.
pleural spaces. ♦♦ ESR is high in tubercular effusion.
• When amount of fluid is more than 300 mL. ♦♦ Ultrasound localizes and detects the effusion.
• Tubercular pleural effusion occurs due to tuberculosis. ♦♦ Biochemical tests:
• In tubercular pleural effusion, the pleural fluid is • Lactate dehydrogenase get increased in tubercular
exudative fluid mixed with the blood. effusion.
• Adenosine deaminase activity gets increased in
Clinical Features tuberculous pleural effusion, i.e. it is more than 40 IU/L.
♦♦ Acute form comes in the form of acute pleurisy with • Interferon Gamma levels are more than 140 pg/mL.
constitutional symptoms such as fever, toxemia, loss of
Treatment
appetite and ill health.
♦♦ Acutely developing effusion will produce breathlessness In tuberculous pleural effusion, anti-tubercular therapy is
and dry cough. started, i.e. all four drugs isonex, rifampicin, pyrazinamide
♦♦ Chronic forms of pleurisy present with the picture of and ethambutol are given for first three months and then on
chronic ill health, low grade fever and loss of appetite. three drugs, i.e. isonex, rifampicin, ethambutol for six months.
80   Mastering the BDS IIIrd Year  (Last 25 Years Solved Questions)

♦♦ Aspiration of pleural fluid is done for diagnosis, for Q.13. Write short note on eosinophilia.
relieving dyspnea and if fever and toxemia is not subsiding  (Mar 2006, 10 Marks) (Mar 2008, 2.5 Marks)
after 4 weeks of anti-tubercular therapy.  (April 2008, 5 Marks) (Mar 2000, 5 Marks)
♦♦ Chest physiotherapy is done to encourage expansion of Ans. Rise in eosinophils in blood to abnormal levels, i.e.
lower chest. greater than 0.4 × 109/L is known as eosinophilia
Q.12. How will you diagnose and manage a case of pneumo-
Causes / Etiology of Eosinophilia
coccal pneumonia? (Mar 1997, 15 Marks)
Ans. It is defined as the inflammation of the lung parenchyma. ♦♦ Parasitic infection: Ascariasis, Hookworm disease,
Pneumococcal pneumonia is caused by the Pne­umococcus. Wuchereria bancrofti, schistosomiasis.
♦♦ Allergic conditions: Drug allergy, hay fever, asthma,
Diagnosis of Pneumococcal Pneumonia aspergillosis
♦♦ Skin disorders: Psoriasis, eczema, dermatitis herpetiformis
♦♦ Physical signs:
♦♦ Tumors: Lymphoma and Myeloproliferative disorders
• At the time of early stage of illness, there are decreased
♦♦ Collagen vascular disorders: Rheumatoid arthritis, Systemic
respiratory movements. lupus erythematosus
• Impairment of percussion node ♦♦ Hypereosinophilic syndrome
• Breadth sounds are diminished. ♦♦ Miscellaneous: Addison’s Disease, Sarcoidosis.
• Pleural rub is present on the affected side.
• Three days after the disease, signs of consolidations Types of Pulmonary Eosinophilia
are seen, i.e. high-pitched bronchial breathing and
♦♦ Cryptogenic eosinophilic pneumonia:
increased vocal resonance.
• It is seen commonly in middle age females.
• At the time of resolution, numerous coarse crackles/
• Symptoms present are of fever, malaise, nausea,
crepitations are heard this indicate liquefaction of
breadthlessness and dry cough.
alveolar exudates.
• Chest X-ray show abnormal shadows in upper zones
♦♦ Blood test: It reveals marked neutrophil leucocytosis.
simulating pulmonary tubercular infiltrates.
♦♦ Blood culture: It shows the presence of Streptococcus
• Peripheral blood and sputum on examination show
pneumonia.
eosinophilia.
♦♦ Examination of sputum: Gram staining of sputum may
• Both ESR and serum IgE levels are elvated.
demonstrate pneumococci.
• Prednisolone is given which should be withdrawn in
♦♦ Chest radiograph: In pneumococcal pneumonia, a
tapered doses after patient get normal.
homogeneous opacity is seen localized to the affected
Drug-induced bronchopulmonary eosinophilia:
lobe or segment which appears within 12–18 hours from
• As the name suggests, it occur due to drugs.
onset of illness.
• It starts with cough and long with other symptoms
♦♦ Serological test: It can detect pneumococcal antigen in the
such as fever with chills and dyspnea.
serum.
• On X-ray pulmonary infiltrates are seen.
♦♦ In some of the cases, fiberoptic, bronchoscopic aspiration
• Withdrawal of an offending drug and use of steroids.
or transthoracic needle aspiration is required.
Asthmatic bronchopulmonary eosinophilia:
• It is characterized by asthma.
Management
• Main cause is allergy to A. fumigatus or Candida
♦♦ Initially oral amoxicillin should be given 500 mg 8 hourly • Presence of transient fleeting shadows on X-ray.
or erythromycin 500 mg 6 hourly. • Peripheral blood and sputum on examination show
♦♦ If patient is very ill or gram-negative or staphylococcal eosinophilia.
infection is present IV ampicillin 0.5–1 g 6 hourly + ♦♦ Idiopathic eosinophilic syndromes:
flucoxacillin 250–500 mg IV 6 hourly + gentamycin 60– 80 Loeffler’s syndrome:
mg every 8 hourly IV is given. • This is a benign acute eosinophilic pneumonia.
♦♦ Antibiotic therapy should be given for 7–10 days. • Various parasitic infestations are associated with it.
♦♦ Choice of antibiotics depends on the causative • It is characterized by migratory pulmonary infiltrates.
microorganisms. • There is also presence of cough, fever and dyspnea.
♦♦ Oxygen therapy is given in seriously ill patients. Oxygen Hypereosinophilic syndrome:
should be delivered at very high rate. • There is presence of peripheral blood eosinophilia for
♦♦ Analgesics such as mefenamic acid 250–500 mg or 6 months or long.
pethidine 50–100 mg or morphine 10–15 mg IM or IV • In this disease, there is lack of presence of parasitic,
injections should be given. allergic or other known cause of eosinophilia.
♦♦ Physiotherapy is given to patient by encouraging him • It leads to the multisystem organ dysfunction.
to cough and to take deep breath as pleuritic pain • There is tissue infiltration by mature eosinophils as
disappears well as blood and bone marrow eosinophilia.
Section 1:  General Medicine  81

• In this syndrome, organs affected are heart, lung, liver, ♦♦ Chemicals such as isocyanides and resins, etc: Avoid
spleen, skin and nervous system. exposure to contact
• Treatment should be done by steroids. ♦♦ Occupational pollutants: Avoid pollutants and change the
Churg-Strauss syndrome: occupation.
• It is a multisystem vasculitic disorder which affects If care should be taken for these factors, asthma will be
skin, lungs, kidney and nervous system. prevented, i.e. cold air, tobacco smoke, respiratory tract
• Patient suffers from fever, malaise, anorexia, weight infection, drugs (beta-blockers, NSAIDs, aspirin, etc.), strenuous
loss, severe asthmatic attack. exercise (exercise induced asthma).
• Presence of pulmonary infiltrates on X-ray.
Q.15. Outline the clinical features of empyema. 
• Peripheral blood on examination shows eosinophilia.
 (Sep 2005, 10 Marks)
• Treatment is done by steroids and immunosuppressive
Ans. Empyema means the presence of pus in the pleural
agents.
cavity.
Clinical Features
Clinical Features of Acute Empyema
♦♦ Cases of pulmonary eosinophilia generally have
mild symptoms in the form of a cough with mucoid Symptoms
expectoration, breathlessness and mild fever, dyspnea or ♦♦ Those of primary disease: Imperfect recovery in pneumonic
orthopnea, wheezing cases, or sudden increase in fever with rigors.
♦♦ General weakness and exhaustion ♦♦ Those due to mechanical effect: Pleuritic chest pain in early
♦♦ Pain in chest which is mild and diffuse. stage, dyspnea, cough and sputum.
♦♦ Those due to toxemia: Malaise, anorexia, sweats and loss
Diagnosis
of weight.
♦♦ Blood count shows raised eosinophilic count and IgE level.
♦♦ Skin test is positive. Signs
♦♦ There is presence of high amount of eosinophils in the ♦♦ Same as pleural effusion, i.e.
sputum. • On inspection:
♦♦ X-ray chest shows diffuse miliary mottling often –– Presence of bilaterally symmetrical chest.
stimulating tuberculosis. –– Restrictions of movements of chest on the side
of effusion.
Treatment • On palpation:
By long-term steroids such as oral prednisolone 60 mg/day. –– Shifting of trachea and mediastinum to opposite
side on pushing.
Q.14. Write note on prevention of bronchial asthma.
–– Diminishing of expansion of chest on involved
 (Sep 1997, 5 Marks)
side.
Ans. A disease of airways produced by hyper-responsiveness
–– Absence of vocal fremitus on involved side.
of tracheobronchial tree to a wide variety of stimuli
• On percussion:
resulting in reversible narrowing of air passage.
–– Stony dull percussion note on the involved side.
Preventive Measures • On auscultation:
–– Absence of breath sounds or diminishing of
♦♦ Pollen breath sounds over area of pleural effusion.
• Try to avoid exposure to flowering vegetation. –– Amphoric bronchial breathing can be heard at
• Keep bed room windows clean. apex of pleural effusion at interscapular region
♦♦ House dust over the involved side.
Vacuum cleaning of mattress daily. –– Absence of vocal resonance.
Shake out blankets and bed sheets daily. ♦♦ Sometimes edema of chest wall is present.
Dust is removed from the bedroom thoroughly. ♦♦ Finger clubbing can develop within 2 to 3 weeks of onset.
♦♦ Animal dander
Avoid contact with animals especially dogs, cats, etc.
Feathers in pillow and quilts should be substituted from Clinical Features of Chronic Empyema
foam pillows and terylene quilts. ♦♦ Recurrent symptoms of chest pain and fever.
♦♦ Drugs: Avoid all preparation of relevant drugs. ♦♦ Loss of weight and anemia.
♦♦ Insect web: Do not allow the insect web to collect. ♦♦ Clubbing of fingers.
♦♦ Chest wall deformity from fibrosis.
Less Common ♦♦ Chronic sinus tracts into the skin or lungs may develop.
♦♦ Foods / Food items: Identify and eliminate them from dishes ♦♦ When bronchopleural fistula is present, air can be heard
such as fish, meat, milk, etc. (or felt) blowing through a patent sinus during coughing.
82   Mastering the BDS IIIrd Year  (Last 25 Years Solved Questions)

Q.16. Write short note on chronic bronchitis. • Aerosols such as hair spray, insecticide spray and
 (Feb 2002, 5 Marks) aerosols should be avoided.
Ans. Chronic bronchitis may be defined as a “condition where • Polluted atmosphere should be strictly avoided.
there is persistent productive cough for at least three ♦♦ Treatment of an infection:
consecutive months in at least two consecutive years”. • Ampicillin 250–500 mg every 6 hourly is given for 5
to 7 days.
Etiology • Cotrimazole 960 mg can be given as BD dose.
♦♦ Cigarette smoking • Antibiotics should be given till purulent mucous
♦♦ Air pollution and clinical factors become mucoid.
♦♦ Infections, i.e. upper respiratory tract infection in smokers • If necessary modify antibiotics as per culture and
♦♦ Occupation sensitivity test.
♦♦ Familial and genetic factors ♦♦ Bronchodilators:
♦♦ Alpha-1 antitrypsin deficiency. • In mild-to-moderate chronic bronchitis oral
theophylline 150 mg BD or inhaled salbutamol 200 μg
Clinical Features 6 hourly can be given.
• In severe bronchitis, ipratropium bromide 40–80mcg
Symptoms
6 hourly is added.
♦♦ It affects male more commonly than the females ♦♦ Mucolytic agents: Bromhexine and carbocystein are to be
♦♦ Chronic bronchitis is present with recurrent attacks of given.
cough ♦♦ Corticosteroids: Prednisolone 30 mg/day for 2 weeks is
♦♦ Cough may occur in paroxysms or is more in elderly hours given. If improvement occur by oral steroids, they are
of morning. replaced by inhalational steroids.
♦♦ Cough is dry when start but later sputum is bringing out ♦♦ Domiciliary oxygen therapy: Long-term oxygen therapy
which is mucoid to mucopurulent sometimes, it is blood in low concentration i.e. 2 L/min by nasal cannula is
tinged. given to reverse or to delay development of pulmonary
♦♦ Patient complains sense of tightness in chest, brea­ hypertension.
thlessness and asthma-like picture. Q.17. Describe clinical features and evaluate management
♦♦ Fever and toxemia appear when infection supervenes of postprimary tuberculosis. (Mar 1998, 15 Marks)
Signs Ans. Postprimary tuberculosis is also known as secondary
tuberculosis.
♦♦ There is increase in respiratory rate.
Most of the morbidity and mortality from TB is caused
♦♦ Inspiratory and expiratory ronchi and presence of
by this form disease.
crepitation at the base of lungs.
It occurs due to reactivation of dormant primary
Investigation tuberculosis; as a progressive primary lesion;
♦♦ X-ray: In early stage, it is normal and later it shows widening hematogeous spread to lungs.
of intercostal spaces, ribs placed more horizontally, At times tuberculosis remains symptom-free and is diagnosed
diaphragm displaced downwards and some patient shows on routine radiography.
patches of pneumonia.
Clinical Features
♦♦ Sputum for culture and sensitivity test is usually sterile.
♦♦ Lung function tests:
• Decrease in FEV1
• Subnormal FEV1/VC
• Decrease in PEF
• Normal lung volumes except with emphysema
• Normal diffusing capacity.
♦♦ Blood gas analysis: In severe cases, there is increase in
hypercapnia and decrease in hypoxaemia.

Management
♦♦ Bronchial irritants should be avoided, i.e.
• Smoking should be strictly stopped.
• Passive smoking is stopped.
• Gas smoke is avoided by housemakers. Fig. 13:  Post-primary tuberculosis
Section 1:  General Medicine  83

Symptoms ♦♦ Later on, there can be presence of dullness to percussion or


bronchial breathing in upper part of one or both the lungs.
♦♦ Evening rise of temperature
♦♦ In chronic cases, there is evidence of volume loss and
♦♦ Night sweats
mediastinal shift.
♦♦ Malaise and cachexia
♦♦ Hilar, mediastinal and cervical groups lymph nodes are
♦♦ Irritability and difficulty in concentration.
enlarged with splenomegaly.
♦♦ Cough and expectoration for more than three weeks.
♦♦ Pleuritic chest pain Management
♦♦ Breathlessness is the feature of advanced disease.
Refer to Ans 18 of same chapter.
♦♦ Indigestion and dyspnea
♦♦ Amenorrhea often in young women Q.18. Write short note on chemotherapy of pulmonary
♦♦ Hoarseness of voice. tuberculosis. (Sep 1997, 5 Marks)
♦♦ Sputum can be mucoid, purulent or blood stained. Ans.
♦♦ Hemoptysis is a classical feature
Chemotherapy
♦♦ Presence of localized wheeze from local ulcer or narrowing
of major bronchus. Drugs for primary chemotherapy (First line anti-tubercu-
♦♦ Presence of recurrent cold. lar drugs)
Physical Signs Drug Daily dose (Adult) Thrice weekly dose
Rifampicin >50 kg 600 mg (10 mg/kg)
♦♦ Fever and weight loss
<50 kg 450 mg (10 mg/kg)
♦♦ Tachycardia and tachypnea
♦♦ Rapid pulse rate Isoniazid 200–300 mg (5 mg/kg)
♦♦ Physical signs of collapse, consolidation, cavitation, (in military TB 10 mg/kg) (10 mg/kg)
fibrosis, bronchiectasis, pleural effusion or pneumothorax. Pyrazinamide >50 kg 2 g (35 mg/kg, max 3 gm)
♦♦ In some cases only localized *rhonchi or rales are present. (25 mg/kg max 2 g)
♦♦ Clubbing of fingers is present in chronic disease. <50 kg 1.5 g
♦♦ Most common physical sign of chest is fine crepitation in Ethambutol 25 mg/kg in initial phase (30 mg/kg)
the upper part of one or both the lungs. This is heard on 15 mg/kg in continuation
taking a deep breath after coughing. phase

Under DOTs following treatment regimen is used

Indication Intensive phase Continuation phase


Drugs Duration Drugs Duration
Category I New sputum smear-positive Isoinazid, rifampicin, Thrice weekly for 2 Isoniazid and Thrice weekly for 4
New sputum smear-negative Pyrazinamide and months rifampicin months
New extrapulmonary Ethambutol
tuberculosis
Category II Sputum smear-positive relapse Isoniazid, Rifampicin, Thrice weekly for
Sputum smear-positive failure Pyrazinamide, 2 months followed by
Sputum smear-positive Ethambutol and
treatment after default steptomycin

Isoniazid, Rifampicin, For 1 month Isoniazid, 5 months


Pyrazinamide and Rifampicin and
Ethambutol Pyrazinamide

Second-line Antituberculous Drugs Drug Dose


Drug Dose Levofloxacin 7.5–10 mg/kg/day OD orally

Streptomycin 20–40 mg/kg/day OD im Moxifloxacin 7.5–10 mg/kg/day orally


Ethionamide 15–20 mg/kg/day b orally
Kanamycin 15–30 mg/kg/day OD im
Prothionamide 15–20 mg/kg/day BD orally
Amikacin 15–22.5 mg/kg/day OD im
Cycloserine* 10–20 mg/kg/day OD or BD orally
Capreomycin 15–30 mg/kg/day OD im
PAS* 150 mg/kg/day BD or TDS orally
Ofloxacin 15–20 mg/kg/day BD orally
*Cycloserine and PAS are bacteriostatic others are bactericidal.
84   Mastering the BDS IIIrd Year  (Last 25 Years Solved Questions)

Treatment Regimen under RNTCP for MDR-TB (Multi- Second line anti-tubercular drugs
drug-resistant TB) and XDR—TB (Extensively drug
Name of drug Side – effects
resistant TB)
Cycloserine Weakness, tremor, ataxia, convulsion, slurred
♦♦ For MDR-TB: speech, brisk jerk, ankle clonus, insomnia and
• Six drugs in intensive phase for 6-9 months: Kanamycin, psychosis.
Levofloxacin, Ethionamide, Cycloserine, Pyrazinamide
PAS (p-amino • Gastrointestinal symptoms: Anorexia,
and Ethambutol. salicyclic acid nausea, vomiting, diarrhea
• Four drugs in continuation phase for 18 months: • Intolerance: Fever, skin rash and
Levofloxacin, Ethionamide, Cycloserine and lymphadenopathy
Ethambutol. • Hemopoietic: leucopenia, eosinophilia, ataxia
• Reserve drug is p-aminosalicylic acid. • Hepatic damage
♦♦ For XDR-TB • Acute renal failure
• Myxedema and Loeffler’s syndrome
• Seven drugs in intensive phase for 6-12 months:
Capreomycin, p-aminosalicylic acid, Moxifloxacin,
high-dose Isoniazid, Clofazimine, Linezolid, Q.20. Write short note on rifampicin. (Mar 2000, 5 Marks)
Amoxicillin and Clavulinic acid. Ans. Rifampicin is a macrolide antimicrobial drug.
• Six drugs in continuation phase for 18 months: It has good sterilizing and resistance preventing actions.
p-aminosalicylic acid, Moxifloxacin, high dose
lsoniazid, Clofazimine, Linezolid, Amoxicillin and Mechanism of Action
Clavulinic acid. ♦♦ Rifampicin inhibits DNA dependent RNA polymerase thus
• Reserve drugs: Clarithromycin, Thiacetazone stopping the expression of bacterial genes.
♦♦ It is bactericidal to M. tuberculosis and many other gram-
Q.19. Write short note on side effects of antitubercular positive and negative bacteria.
drugs. (Sep 2008, 5 Marks)
Side Effects
Ans.
♦♦ Liver damage
First line anti-tubercular drugs ♦♦ Influenza-like reaction
Name of drug Side-effects ♦♦ Intolerance: Fever, skin rash, diarrhea, leucopenia,
Rifampicin Nausea, vomiting, diarrhea, skin rashes and liver eosinophilia, ataxia and dizziness.
damage, influenza-like reaction, leucopenia, ♦♦ Orange to red color of urine, feces and sputum.
eosinophilia, ataxia and dizziness
Uses
Isoniazid Peripheral neuropathy, psychosis hepatic injury,
optic neuritis convulsions and dryness of mouth ♦♦ In tuberculosis
♦♦ Leprosy
Pyrazinamide Polyarthralgia, nausea, vomiting, malaise, toxic
hepatitis, hyperuricemia, gout. ♦♦ Meningiococcal carrier
♦♦ Staphylococcal septicemia
Ethambutol Blurring of vision, optic neuritis, nausea, vomiting,
♦♦ Herpes zoster
liver damage and peripheral neuropathy
♦♦ H. influenza
Second line anti-tubercular drugs ♦♦ Brucella
Name of drug Side – effects ♦♦ Mycetoma
Streptomycin Nephrotoxicity, ototoxicity, ataxia, anaphylaxis, ♦♦ Legionella
injection abscess, circumoral paresthesia, ♦♦ Chlamydia
eosinophilia, drug fever
Q.21. Write short note on nosocomial pneumonia. 
Kanamycin Ototoxicity, deafness, vertigo, nephrotoxic tinnitus,  (Nov 2014, 3 Marks) (Mar 2007, 5 Marks)
Amikacin vertigo, renal damage, cutaneous reactions, Ans. Nosocomial pneumonia is defined as hospital acquired
hypocalcemia, hypomagnesaemia and hepatitis
pneumonia.
Capreomycin Psychosis, seizures, peripheral neuropathy, ♦♦ It is the secondary pneumonia.
headache, somnolence and allergy
♦♦ Pneumonias developing in hospital in a patient who has
Ofloxacin Anorexia, nausea, vomiting, dizziness, headache, been admitted for more than 48 hours should be considered
Levofloxacin mood changes and impaired growth to be nosocomial rather than community acquired.
Moxifloxacin ♦♦ Because of change of oropharyngeal flora in hospita­
Ethionamide Skin rash, purpura, anorexia, nausea, vomiting, lized patients, it is caused by anaerobic gram-negative
Protionamide headache, anaphylactic shock, postural organism like E. coli, Klebsiella pneumoniae, Pseudo­monas,
hypotension, metallic taste, hypersalivation, more frequent. Staphylococcus aureus, pneumo­cocci and
hallucination, menstrual disorders. H. influenza are less frequent causative organisms.
Section 1:  General Medicine  85

Factors Predisposing for Nosocomial Pneumonia Signs


♦♦ Reduced host defenses against bacteria: ♦♦ Signs of lung consolidation are seen.
• Reduced immune defenses (e.g. corticosteroid ♦♦ If there is formation of lung abscess, cavitation can be felt.
treatment, diabetes, malignancy) ♦♦ There is presence of pleural rub.
• Reduced cough reflex (e.g. post-operative) ♦♦ Digital clubbing is also well appreciated.
• Disordered mucociliary clearance (e.g. anesthetic
agents) Investigations
• Bulbar or vocal cord palsy ♦♦ TLC and DLC should be carried out for checking
♦♦ Aspiration of nasopharyngeal or gastric secretions: neutrophilic leucocytosis.
• Immobility or reduced conscious level
♦♦ ESR is high.
• Vomiting, dysphagia, achalasia or severe reflux
♦♦ Chest X-ray: It shows a homogeneous opacity. If there
• Nasogastric intubation
is presence of fluid in the opacity, this indicates of an
♦♦ Bacteria introduced in to lower respiratory tract:
abscess.
• Endotracheal intubation/tracheostomy
• Infected ventilators/nebulizers/bronchoscopes ♦♦ Culture of sputum is carried out to identify micro-organisms.
• Dental or sinus infection
Treatment
♦♦ Bacteremia:
• Abdominal sepsis ♦♦ In aerobic infection ampicillin 500 mg QID or amoxicillin
• Intravenous cannula infection 500 mg TID or cotrimoxazole 960 mg BD orally provides
• Infected emboli relief.
♦♦ In anaerobic infection metronidazole 400 mg TID orally
Treatment
is added.
♦♦ In mild cases amoxicillin with clavulanic acid 500mg TDS ♦♦ Antibiotics should be prescribed based on the culture and
is given. sensitivity testing.
♦♦ In severe cases I.V. cefuroxime 750mg every 8 hourly + ♦♦ Treatment should be continued for 10 to 15 days.
Clarithromycin 250mg every 8 hourly is given. ♦♦ NSAIDs should be given for pain relief.
♦♦ Most of the patients require ventilatory support and also ♦♦ In subjects with lung abscess physiotherapy and postural
need supplemental oxygen therapy.
drainage is done.
Q.22. Write short note on aspiration pneumonia. 
 (Apr 2007, 10 Marks) Complications
Ans. Aspiration pneumonia is also known as suppurative
♦♦ Amyloidosis
pneumonia.
♦♦ Empyema
Aspiration pneumonia is the consolidation of lung in
♦♦ Bronchiectasis
which there is continued destruction of parenchyma
♦♦ Septicemia
by inflammatory cells which causes formation of
♦♦ Pulmonary fibrosis.
microabscess.
Q.23. Mention complications of pulmonary tuberculosis.
Etiology
 (Apr 2007, 5 Marks)
Aspiration pneumonia is caused by Staphylococcus aureus, Ans. The complications of pulmonary tuberculosis are as
Sterptococcal pneumonia, Streptococcus pyogens, Hemophilus follows:
influenza as well as various other anaerobic bacterias. I. Early complications
• Pneumonia
Clinical Features
• Empyema
Symptoms • Hemoptysis
♦♦ Aspiration pneumonia occurs mostly on the right side as • Laryngitis
compared to left side. • Pneumothorax.
♦♦ Patient complaints of high fever, cough and dyspnea. II. Late complications
♦♦ During coughing copious foul-smelling sputum is present, • Bronchiectasis
which is blood stained too. • Mycetomas in cavity
♦♦ There is presence of tachycardia and restlessness. • Colonization of fibrotic lung with nontuberculous
♦♦ Weight loss is too rapid. Mycobacterium
♦♦ There is also presence of perspiration. • Nonrespiratory disease, i.e. genitourinary, bone.
86   Mastering the BDS IIIrd Year  (Last 25 Years Solved Questions)

Q.24. Write short note on smoking-related disorder. larynx, esophagus, stomach, pancreas, liver, kidney, ureter,
 (Mar 2008, 2.5 Marks) urinary bladder and uterine cervix and also causes myeloid
Ans. The incidence of smoking-related diseases is greater in leukemia.
younger than in older smokers, particularly for coronary
Respiratory Disease
artery disease and smokers.
• Cardiovascular disease Cigarette smoking is responsible for 90% of COPD within 1–2
• Cigarette smokers are more likely than nonsmokers years of beginning of smoke regularly.
to develop large vessel atherosclerosis as well as Many smokers will develop inflammatory changes in small
small vessel disease. airway due to there is reduced expiratory airflow.
• Cigarette smoking also increases likelihood of
myocardial infarction and sudden cardiac death. Other Conditions
Smoking delays healing of peptic ulcer, increases risk of
Cancer
osteoporosis, senile cataract, macular degeneration, premature
Tobacco smoking causes cancer of lung, oral cavity,, nose, menopause, wrinkling in skin, gallstone, male impotence and
oro and hypopharynx, nasal cavity and paranasal sinuses, cholecystitis in women.

Fig. 14:  Smoking-related disorders


Section 1:  General Medicine  87

Q.25. Write short note on bronchitis. Q.26. Write short note on pneumococcal pneumonia.
 (Apr 2008, 5 Marks) (Mar 2008, 5.5 Marks)  (Sep 2009, 4.5 Marks)
Ans. It is caused by streptococcal pneumonia.
Ans. Bronchitis is the infection of the bronchi.
It is characterized by homogeneous consolidation of one

Types: lobe or more lobes or segments of a lung, hence, called
1. Acute bronchitis. lobar pneumonia
2. Chronic bronchitis. Clinical Features
Acute bronchitis ♦♦ It occurs at all ages but is common in early and adult life. It
is most common bacterial pneumonia following an upper
It is an acute infection of the bronchi and may be caused by the
respiratory tract infection. It is usually a sporadic disease,
infection with organisms such as Streptococcus, pneumococci, common in winter and spreads by droplet infection.
Hemophilus influenza or primarily viral in origin. ♦♦ In children, the onset is sudden often with fever, chills and
Causes rigors, vomiting and convulsions. In adults, the onset is
with fever, chills, cough, breathlessness and chest pain.
♦♦ Infection: Bacterial or viral, or descending infection from ♦♦ Shaking chills and rigors
nasal sinus or throat. ♦♦ Loss of apatite body ache and headache
♦♦ Complicating other diseases such as: measles and whooping ♦♦ Hemoptysis and weakness
cough. ♦♦ The physical signs during an early stage of illness
♦♦ Physical and chemical irritants: Inhaled dust, steam, gases show decreased respiratory movements, impairment of
such as sulphur dioxide and ether. percussion note, diminished breath sounds and often a
Symptoms pleural rub on the affected side.
• Later on, usually after 3 days, signs of consolidation
♦♦ Toxemia: Malaise, fever, ill health, tachycardia appear.
♦♦ Irritative: Cough with expectoration, at first scanty viscid • During resolution, numerous coarse crackles crepita­
sputum is present and later on it become more copious tions are heard, indicating the liquefaction of alveolar
and mucopurulent; substernal pain or raw sensation exudate.
under sternum.
♦♦ Obstructive: Chocked up feeling, paroxysms of dyspnea Investigations
following spells of coughing relieved with expectoration. ♦♦ Blood test: It reveals marked neutrophil leucocytosis.
♦♦ Blood culture: It shows the presence of Streptococcus
Physical Signs
pneumonia.
Fever, tachycardia, flushing of face, respiratory rate is slightly ♦♦ Examination of sputum: Gram staining of sputum may
increased. Crepitations are heared at the base when secretions demonstrate pneumococci.
collect in the lungs. ♦♦ Chest radiograph: In pneumococcal pneumonia, a
homogeneous opacity is seen localized to the affected
Treatment lobe or segment which appears within 12–18 hours from
♦♦ In dry stage: onset of illness.
• Bed rest ♦♦ Serological test: It can detect pneumococcal antigen in the
• Nutritious diet serum.
• Tincture benzoin inhalation ♦♦ In some of the cases, fiberoptic, bronchoscopic aspiration
• Application of vicks vaporub on chest or transthoracic needle aspiration is required.
• Capsule amoxicillin 250 mg 8 hourly for 4 to 5 days.
• Tablet aspirin or paracetamol twice a day. Complications
• Cough sedative mixture like linctus codeine to ♦♦ Pulmonary:
suppress dry cough. • Parapneumonic pleural effusion
♦♦ In moist stage: • Emphysema
• Amoxicillin and cloxacillin 500 mg 8 hourly for 4 to • Suppurative pneumonia or lung abscess
5 days. • Acute respiratory distress syndrome
• Expectorant mixture with sodium or potassium iodide • Pneumothorax
to bring out the secretions. ♦♦ Extra pulmonary
• Hepatitis, pericarditis, meningioencephalitis
Chronic Bronchitis
• Multiorgan failure
Refer to Ans 16 of same chapter. • Ectopic abscess formation
88   Mastering the BDS IIIrd Year  (Last 25 Years Solved Questions)

Management symptoms. Bacilli are transported to regional lymph


♦♦ Initially, oral amoxicillin should be given 500 mg 8 hourly nodes and parenchymal lesion in the lungs (Ghon Focus)
or erythromycin 500 mg 6 hourly. along with enlarged lymph nodes which may calcify over
♦♦ If patient is very ill or gram negative or staphylococcal a period of time and this constitutes primary complex
infection is present IV ampicillin 0.5–1g 6 hourly + (Ghon’s complex).
flucoxacillin 250–500 mg IV 6 hourly + gentamicin 60–80 ♦♦ A case of primary tuberculosis draws attention when
mg every 8 hourly IV is given. a child may present with a nonspecific pneumonia or
♦♦ Antibiotic therapy should be given for 7–10 days. bronchial obstruction because of enlarged hilar gland or
♦♦ Choice of antibiotic depends on the causative low-grade fever with pleural effusion.
microorganisms. ♦♦ Primary complex heals leaving a calcified lesion.
♦♦ Oxygen therapy is given in seriously ill patients. Oxygen ♦♦ Bacilli may remain for years and may become reactivated
should be delivered at very high rate. when body’s immunity falls as in malnutrition, debilitating
♦♦ Analgesics such as mefenamic acid 250–500 mg or disease and following severe form of measles, whooping
pethidine 50–100 mg or morphine 10–15 mg IM or IV cough.
injections should be given. Clinical Features
♦♦ Physiotherapy is given to patient by encouraging him to
cough and to take deep breath as pleuritic pain disappears. As tuberculosis remains symptom-free and is diagnosed on
routine radiography.
Q.27. Differentiate between pleural friction rub and cra­ckles
or rales. (Sep 2008, 2.5 Marks) Symptoms
Ans. ♦♦ Evening rise of temperature
♦♦ Night sweats
Pleural friction rub Crackles or rales
♦♦ Malaise and cachexia
A pleural friction rub is caused Crackles are the sounds you will ♦♦ Irritability and difficulty in concentration.
by the inflammation of the hear in a lung field that has fluid in ♦♦ Cough and expectoration for more than three weeks.
visceral and parietal pleurae the small airways or if atelectasis
♦♦ Pleuritic chest pain
is present
♦♦ Breathlessness is the feature of advanced disease.
This is a low-pitched creaking, This is small bubbling or clicking ♦♦ Indigestion and dyspnea
grating or rubbing sound heard with a stethoscope, it is ♦♦ Amenorrhea often in young women
much like the static sound heard
♦♦ Hoarseness of voice.
on the radio
♦♦ Sputum can be mucoid, purulent or blood stained.
Sound is heard more often with Sound is heard with breathing in or ♦♦ Hemoptysis is a classical feature
inhalation than expiration out and is not always a continuous
♦♦ Presence of localized wheeze from local ulcer or narrowing
sound
of major bronchus.
Sound occur when inflamed The sounds produced are created ♦♦ Presence of recurrent cold.
pleural surfaces rub together when air is forced through
during respiration respiratory passages that are Physical Signs
narrowed by fluid, mucus, or pus
♦♦ Fever and weight loss
Q.28. Write short note on primary tuberculosis and its inves­ ♦♦ Tachycardia and tachypnea
tigations. (Sep 2009, 4 Marks) ♦♦ Rapid pulse rate
Ans. Primary tuberculosis is the first lesion which develops ♦♦ Physical signs of collapse, consolidation, cavitation,
in a previously unexposed non-sensitized individual fibrosis, bronchiectasis, pleural effusion or pneumothorax.
irrespective of the age. ♦♦ In some cases, only localized *rhonchi or rales are present.
♦♦ Clubbing of fingers is present in chronic disease.
Pathology ♦♦ Most common physical sign of chest is fine crepitation in
the upper part of one or both the lungs. This is heard on
♦♦ Reinfection of a sensitized person or reactivation of a taking a deep breath after coughing.
primary dormant lesion is called secondary or post- ♦♦ Later on, there can be presence of dullness to percussion or
primary tuberculosis. bronchial breathing in upper part of one or both the lungs.
♦♦ The initial lesion after ingestion of tubercle bacilli which ♦♦ In chronic cases there is evidence of volume loss and
mainly occurs in the lungs constitutes primary tuber­ mediastinal shift.
culosis. ♦♦ Hilar, mediastinal and cervical groups lymph nodes are
♦♦ It commonly involves children and is in the form of enlarged with splenomegaly.
subpleural lesion either in the lower part of upper lobe or
upper part of lower lobe. Investigations
♦♦ The initial entry of the bacilli initiates non-specific ♦♦ Sputum examination (for acid fast bacilli): By direct smear
inflammatory response which hardly produces any examination (Ziehl-Neelsen stain). At least, three smears
Section 1:  General Medicine  89

must be examined before finally reaching a conclusion. multiple thick-walled cavities can be seen. At the time
When direct smear is negative, sputum examination be of fibrosis, trachea and mediastinum shift to same side.
done by concentration method using 24 hours collection of Fibrosis can also cause calcification
sputum. Further confirmation is done by sputum culture ♦♦ Pathological tests:
by animal inoculation which takes 4–8 weeks. If adequate • Blood examination: Peripheral blood examination shows
amount of sputum is not available, bronchoscopic monocytosis, i.e. 8 to 12%
aspiration of secretions be made and submitted for smear • ESR is elevated.
and culture examination. • Tuberculin test: It is a test to recognize prior tubercular
infection, and is done by injecting one unit of purified
♦♦ Serology: In this ELISA, technique is used which helpful
protein derivative (PPD) on the forearm and readings
in diagnosis of tuberculosis in children. PCR technique is
taken after 48 hours. Induration of more than l5 mm
more specific and sensitive serological test than ELISA,
indicates a positive test. The younger is the patient,
but PCR is less used due to its high cost. greater is significance of positive test. A negative test
♦♦ Chest X-ray: Presence of multiple nodular infiltrations or does not always exclude tubercular infection since it
ill-defined opacities in one of upper lobes is characteristic may be negative in patients of blood malignancies,
for pulmonary tuberculosis. An area of translucency in malnourishment and those on immunosuppressive
radiopacities is indicative of cavitation. Presence of cavity therapy. Tuberculin test is non-specific and only
is indicative of an active lesion. In some of the patients indicates prior infection. Its sensitivity wanes with age.
Q.29. Write difference between bronchial and cardiac asthma. (Sep 2008, 2.5 Marks) (Sep 2011, 4 Marks)
Ans.
Features Cardiac asthma Bronchial asthma
1. Past History Of hypertension, aortic disease Of previous attacks of asthma or other allergic conditions
2. Age Onset is after 40 years Any age
3. Precipitating factor Precipitated by exertion, acute myocardial infarction or Trigger factors may be infection, non-specific irritants,
hypertension external allergens, exercise or emotional factors
4. Symptoms:
a. Cough Cough and dyspnea almost simultaneous. Pink frothy Starts with dyspnea.
sputum which increases in intensity towards end of attack. Expectoration of thick sticky sputum.
b. Wheezing Rare Usual
c. Sweating asthma Prominent Rare unless acute severe
5. Signs:
a. Inspection
i. Accessory muscles Not active Active
of respiration
ii. Shape of chest Normal Emphysematous
iii. Respirations Rapid and shallow Rapid with prolong
expiration
b.  Auscultation
i. Chest Expiration not unduely prolonged. Expiration markedly prolonged.
Rales more than ronchi in early stages at lung bases, Ronchi are more than rales
gradually ascending up with progress of the attack Signs diffuse all over the lungs
ii. Heart A2 may be loud Normal A2
c.  Pulse Pulsus alterans Feeble and rapid
d.  BP Usually elevated Normal or low
e. Extremities Cold Warm

Q.30. Write short note on respiratory failure.  normo or hypocapnia because of tachypnea or
 (Apr 2008, 5 Marks) hyperventilation.
Ans. Respiratory failure is defined as failure of respiratory • Chronic: Chronic alteration in blood gases occur due to
system to maintain normal partial pressure of oxygen slow diffusion of carbon dioxide via lungs. So there is
and carbon dioxide in the blood. occurrence of hypoxemia and normocapnia. Here pH
Types and bicarbonate levels remain normal.
♦♦ Type I (Hypoxemic) respiratory failure: ♦♦ Type II (Hypercapnic) respiratory failure:
• Acute: In this, there is acute alteration in blood • Acute: It is also known as asphyxia. In this, there is
gases concentration with hypoxemia and hypercapnia and acute respiratory acidosis.
90   Mastering the BDS IIIrd Year  (Last 25 Years Solved Questions)

• Chronic: In this, there is hypoxemia, hypercapnia but is done and now the patient should be supported with
low or normal pH. intermittent positive pressure ventilation.
• Acute on chronic Type II respiratory failure: Here there is ♦♦ Mucolytic agents such as bromohexine liquefy secretion.
sudden rise of carbon dioxide and there is occurrence Acetylcystine, i.e. 1–2 mL of 20% solution is instilled via
of acidemia with acute insult from the precipitating tracheostomy tube.
factors. This is known as acute on chronic Type II ♦♦ Patients suffering from acute respiratory distress syndrome
respiratory failure. can be on positive end expiratory pressure.
♦♦ Intravenous fluids and electrolyte therapy is given.
Signs and Symptoms (Clinical Features) ♦♦ Underlying infection should be treated by proper
Due to Hypoxemia antibiotics.
♦♦ H2 blockers are given via IV drip
♦♦ Signs: ♦♦ Patient should be slowly weaned from respirator as
• Tachycardia
voluntary effort is gained from ventilation as etiology is
• Central cyanosis
corrected.
• Poor peripheral circulation
• Depress level of consciousness Chronic Respiratory Failure
• Cardiac arrhythmias.
♦♦ Nebulized solution of salbutamol, i.e. 2.5–5 mg 4 hourly
♦♦ Symptoms:
or turbutaline 5–10 mg can also be given.
• Sweating
♦♦ Bronchodilator such aminophylline is given in dose of
• Restlessness
0.25 gm IV diluted in 10 mL of 10–25% dextrose and is
• Mental confusion.
injected slowly.
Due to Hypercapnia ♦♦ Short course of antibiotics should be given.
♦♦ Signs: ♦♦ Secretions should be removed by asking the patient to
• Affect on CNS functions: Asterixis, miosis, hyporeflexia, cough or by intermittent endotracheal catheter suction.
confusion and coma ♦♦ If patient develops pulmonary edema or cor pulmonale.
• Bounding pulse ♦♦ Oxygen therapy and assisted ventilation should be given
• Muscle twitching to acute on chronic Type II respiratory failure patients till
• Elevated blood pressure patient have acceptable levels of PaO2 and PaCO2.
• Cardiac dysrhythmias. ♦♦ If condition of patient worsen mechanical ventilation by
♦♦ Symptoms: fixed volume, ventilators is given to deliver fix volume
• Breathlessness of oxygen.
• Headache ♦♦ Ventilatory assistance is slowly withdrawn as patient
• Warm extremities. returns to voluntary effort.
Q.31. Write short note on emphysema. (Apr 2008, 5 Marks)
Management
Ans. It is defined as distention of the air spaces distal to the
Acute Respiratory Failure terminal bronchiole with destruction of alveolar septa.
♦♦ Removal of underlying cause is mandatory.
Predisposing Factors/Etiology
♦♦ Hospitalize the patient and treat in the respiratory
intensive care unit. ♦♦ Smoking
♦♦ Supervise coughing in a conscious patient and change ♦♦ Environmental pollution
the patient’s position from side to side which helps in ♦♦ Genetic predisposition due to alpha-I antitrypsin deficiency
clearing airway. ♦♦ Bacterial
♦♦ By using a rolled gauze piece clear the thick secretions in ♦♦ Occupational exposure.
oral cavity by holding in an artery forceps.
Types of Emphysema
♦♦ Secretions at back of throat or in trachea are removed by
frequent secretion. ♦♦ Centri-acinar: Presence of destruction and enlargement of
♦♦ High concentration of oxygen is given to the patients via central or proximal part of respiratory unit. Predominant
ventimasks to improve hypoxemia and ventilation. For involvement of upper lobe and apices. Commonly seen
small children, oxygen tents are used. Oxygen therapy in male smokers.
should be continued till patient achieve normal level of ♦♦ Pan-acinar: Presence of uniform destruction and
blood gases. enlargement of acinus. It is predominant in lower basal
♦♦ In Type II acute respiratory failure, immediate reversal of zones. It is associated with alpha-1 antitrypsin deficiency.
precipitating factor should be done. In cases where reversal ♦♦ Paraseptal: It involves only distal acinus. Found near pleura
cannot occurs temporary ventilator support is required. often causes spontaneous pneumothorax
♦♦ If patient get worse even after taking above-mentioned ♦♦ Irregular: There is irregular type of acinus with scarring
treatment, tracheostomy and endotracheal intubation involvement.
Section 1:  General Medicine  91

Clinical Features Treatment


There are two types of patients suffering from emphysema. Type A Chronic source of infection in upper respiratory airway be
refers to pink puffers who have minimal cough and expectoration removed.
while dyspnea is prominent and Type B refers to blue bloaters. ♦♦ Patient should be taken away from cold climate
In this, patients have marked blood gas abnormalities though the ♦♦ Smoking in any form must be stopped
pulmonary diffusing capacity is within the normal limits. ♦♦ Breathing exercise be advised
Symptom Type A (Pink Type B ♦♦ Respiratory infection be prevented and prompt antibiotic
Puffers) (Blue Bloaters) treatment, e.g. ampicillin, amoxicillin.
Cough Minimal Predominant ♦♦ Bronchodilators are used, i.e. theophylline and
Expectoration Minimal Copious salbutamol.
Dyspnea Predominant Mild ♦♦ In severe respiratory insufficiency, corticosteroids are
Cyanosis Absent Present useful prednisolone
Corpulmonale Infrequent Common
♦♦ Oxygen inhalation, intermittently is useful in case of
Edema Not common Edema in dependent parts emphysema.
Jugular venous Not raised Raised
pressure Q.32. Enumerate antitubercular drugs—its dosage and com-
Diffusing capacity Decreased In normal limits plications. (Nov 2008, 10 Marks).
of lungs Ans.

First-line anti-tubercular drugs


Name of drug Dosage Complications
Rifampicin >50 kg 600 mg Nausea, vomiting, diarrhea, skin rashes and liver damage, influenza like reaction,
<50 kg 450 mg leucopenia, eosinophilia, ataxia and dizziness
Isoniazid 200–300 mg Peripheral neuropathy, psychosis hepatic injury, optic neuritis convulsions and dryness
(In miliary TB 10 mg/kg) of mouth
Pyrazinamide >50 kg 2 gm Polyarthralgia, nausea, vomiting, malaise, toxic hepatitis, hyperuricemia, gout.
<50 kg 1.5 gm
Ethambutol 25 mg/kg in initial phase Blurring of vision, optic neuritis, nausea, vomiting, liver damage and peripheral
neuropathy
Second line anti-tubercular drugs
Name of drug Dosage Complications
Streptomycin 20–40 mg/kg/day OD IM Nephrotoxicity, ototoxicity, ataxia, anaphylaxis, injection abscess, circumoral
paresthesia, eosinophilia, drug fever
Kanamycin 15–30 mg/kg/day OD IM Ototoxicity, deafness, vertigo, nephrotoxic tinnitus, vertigo, renal damage, cutaneous
reactions, hypocalcemia, hypomagnesemia and hepatitis
Amikacin 15–22.5 mg/kg/clay OD IM
Capreomycin 15–30 mg/kg/day OD l IM Psychosis, seizures, peripheral neuropathy, headache, somnolence and allergy
Ofloxacin 15–20 mg/kg/day BD orally Anorexia, nausea, vomiting, dizziness, headache, mood changes and impaired growth
Levofloxacin 7.5–10 mg/kg/day OD orally
Moxifloxacin 7.5–10 mg/kg/day orally
Ethionamide 15–20 mg/kg/day BD orally Skin rash, purpura, anorexia, nausea, vomiting, headache, anaphylactic shock, postural
hypotension, metallic taste, hypersalivation, hallucination, menstrual disorders.
Protionamide 15–20 mg/kg/day BD orally
Cycloserine 10–20 mg/kg/day OD or BD orally Weakness, tremor, ataxia, convulsion, slurred speech, brisk jerk, ankle clonus, insomnia
and psychosis.
PAS (p-amino 150 mg/kg/day BD or TDS orally • Gastrointestinal symptoms: Anorexia, nausea, vomiting, diarrhea
salicyclic acid • Intolerance: Fever, skin rash and lymphadenopathy
• Hemopoietic: leucopenia, eosinophilia, ataxia
• Hepatic damage
• Acute renal failure
• Myxedema and Loeffler’s syndrome
92   Mastering the BDS IIIrd Year  (Last 25 Years Solved Questions)

Q.33. Discuss etiology, clinical features, diagnosis and out- Investigations


line the management of acute pulmonary embolism. ♦♦ Blood examination: If pulmonary infarct is present there can
 (Dec 2009, 15 Marks) be leucocytosis or raised ESR.
Or ♦♦ Chest X-ray: In massive pulmonary embolism, there is
Write short note on acute pulmonary embolism. presence of diffuse infiltrates in the lung with increased
 (Mar 2001, 8 Marks) bronchovascular markings. If medium size vessels are
Ans. involved, there will be triangular pleuropulmonary opacity
in peripheral lung fields, there can also be pleural effusion
Etiology present.
♦♦ Thrombotic: ♦♦ Arterial blood gas analysis: Presence of hypoxemia and
• Deep vein thrombosis hypocapnia.
• Congestive heart failure ♦♦ D-dimer: It is a fibrin degradation product release in
• Right-sided endocarditis circulation in pulmonary embolism. Presence of high levels
• Atrial fibrillation. of D-dimer is suggestive of an embolism while presence of
♦♦ Non-thrombotic: low D-dimer exclude pulmonary embolism.
• Fat embolism ♦♦ Echocardiography: It shows the right ventricular dilatation
• Amniotic fluid embolism: spontaneous delivery and and presence of clot in it.
caesarean section ♦♦ Spiral CT scan: CT of chest along with the IV contrast
• Tumor embolism: Choriocarcinoma diagnose the pulmonary embolism. It effectively diagnose
• Parasitic embolism: Schistosomiasis the large and central pulmonary embolism. Newer
• Air embolism: Pulmonary barotraumas generally in scanners can also detect peripherally present emboli.
the sea divers ♦♦ Pulmonary angiography: It demonstrates the site of
obstruction of all sized blood vessels.
Clinical Features
In acute massive embolism, i.e. acute cor pulmonale Management
♦♦ Symptoms are of presence of acute dyspnea, tachypnea, ♦♦ In patient of massive embolism
tachycardia, hemoptysis and chest pain. • If patient is in state of shock or collapse:
♦♦ Signs are increased in the jugular venous pressure, presence –– Vasopressors such as dopamine or dobutamine
of central cyanosis, Loud P2 and narrow splitting of P2, are to be given.
an ejection systolic murmur in P2 area, right ventricular –– Administer oxygen to the patient.
hypertrophy, signs of shock, –– Correct acidosis
In small or medium-sized pulmonary vessels embolization –– If there is failure of an initial resuscitation, or there
♦♦ Symptoms are hemoptysis, pleuritic pain and wheeze is hypotension or right ventricular dysfunction,
which is the triad of pulmonary infarct. primary therapy should be administered,
♦♦ Signs are of pleural effusion, i.e. reduced or absent i.e. dissociation of clot by thrombolysis or
chest wall movement and expansion of chest on the embolectomy.
side involved, activity of extrarespiratory muscles is • If acute event is survived by the patient:
absent, position of trachea and mediastenum is shifted to –– Streptokinase 2.5–5 lac unit I.V> in dextrose or
opposite side, percussion note is stony dull on the side of saline is given for 30 min followed by 1 lac IV
involvement, vocal fremitus is reduced or absent on the for 24 hours.
side involved, breath sounds are absent or diminished –– Recombinant tissue plasminogen activator tPA i.e.
over the area involved. Alteplase 100 mg for 2 hours is a good alternative
to thrombolytic therapy.
Diagnosis
• Anticoagulation therapy is an initial and immediate
It is in the patients who had suspicion for underlying cause treatment of choice:
for emboli formation, development of pulmonary sign and –– Loading dose of 80–150 units/kg is given which
symptoms as well as cardiovascular involvement, presence is followed by 15–18 units/kg/hr as continuous
of thrombophlebitis in deep leg veins, prolonged bed rest, infusion.
immobilization, cardiac irregularity in form of atrial fibrillation –– Warfarin should be added to heparin and is
should be considered while keeping in mind clinical picture of continued for 5 days.
precordial pain, breathlessness and tachycardia in patient who –– After 5 days, taper the heparin and administer
had recently gone for major surgery. Examination of veins is warfarin for 6 weeks to 6 months.
mandatory in the patients who are at high risk for development –– Monitor anticoagulant therapy by PTT or INR
of deep vein thrombosis. These features along with investigatory ratio or bleeding time which should be 2 to 3 times
features form the diagnosis. Following are the investigations: than the control.
Section 1:  General Medicine  93

♦♦ In small embolisms • Pyrazinamide (Z)


• Analgesics, i.e. NSAIDs should be given to relieve • Ethambutol (E)
the pain. • Streptomycin (S)
• Anticoagulant is given to prevent further embolization. • H: Isoniazid (300 mg), R: Rifampicin (450 mg),
• Various preventive measures should be undertaken Z: Pyrazinamide (1500 mg), E: Ethambutol (1200 mg),
such as calf muscle exercise, elastic stockings, prolong S: Streptomycin (750 mg).
immobilization at bed, respiration exercises should ♦♦ Patients who weigh 60 kg or more receive additional
be done. Rifampicin 150 mg.
♦♦ Patients who are more than 50 years old receive
Q.34. Write on RNTCP classification of tuberculosis.  Streptomycin 500 mg. Patients who weigh less than 30 kg
 (Dec 2009, 10 Marks) receive drugs as per pediatric weight band boxes according
Ans. RNTCP or the Revised National Tuberculosis Control to body weight.
Program is the state-run tuberculosis control initiative of ♦♦ Numerical before a phase is the duration of that phase
the Government of India. It incorporates the principles in months.
of directly observed treatment-short course (DOTS), ♦♦ Numerical in subscript is the number of doses of that drug
per week. If there is no subscript numerical, then the drug
the global TB control strategy of the World Health
is given daily.
Organization. The program provides, free of cost, quality
anti-tubercular drugs across the country through the Treatment Regimen under RNTCP for MDR-TB (Multidrug-
numerous Primary Health Centres and the growing resistant TB) and XDR—TB (Extensively drug resistant TB)
number of private-sector DOTS-providers.
♦♦ For MDR-TB:
Objectives • Six drugs in intensive phase for 6–9 months: Kanamycin,
♦♦ Detecting at least 70% of sputum positive tuberculosis Levofloxacin, Ethionamide, Cycloserine, Pyrazinamide
patients in the community. and Ethambutol.
♦♦ Curing at least 85% of the newly detected sputum positive • Four drugs in continuation phase for 18 months:
cases. Levofloxacin, Ethionamide, Cycloserine and
RNTCP classifies tuberculosis patients into following treatment Ethambutol.
categories. • Reserve drug is p-aminosalicylic acid.
♦♦ For XDR—TB:
TB Cat- Patient Type Initial Continua- Total
• Seven drugs in intensive phase for 6-12 months:
egory phase tion phase Dura-
tion Capreomycin, p-aminosalicylic acid, Moxifloxacin,
high dose Isoniazid, Clofazimine, Linezolid,
I New sputum positive 2H3R3Z3E3 4H3R3 6 months
Or Amoxicillin and Clavulinic acid.
New smear negative • Six drugs in continuation phase for 18 months:
Or p-aminosalicylic acid, Moxifloxacin, high dose
New case with lsoniazid, Clofazimine, Linezolid, Amoxicillin and
severe form of Clavulinic acid.
extrapulmonary • Reserve drugs: Clarithromycin, Thiacetazone
tuberculosis
II Smear positive failure 2 H 3 R 3 Z - 5H3R3E3 8 months Second-line antituberculous drugs
Or ES +
3 3 3
Smear positive 1H3R3Z3E3 Drug Dose
relapse
Streptomycin 20–40 mg/kg/day OD im
Or
Sputum positive Kanamycin 15–30 mg/kg/day OD im
treatment after default Amikacin 15–22.5 mg/kg/day OD im
III Smear negative 2H3R3Z3 4H3R3 6 months
Capreomycin 15–30 mg/kg/day OD im
pulmonary T.B. with
limited parenchymal Ofloxacin 15–20 mg/kg/day BD orally
involvement Levofloxacin 7.5–10 mg/kg/day OD orally
Or
Less severe form of Moxifloxacin 7.5–10 mg/kg/day orally
extrapulmonary T.B. Ethionamide 15–20 mg/kg/day b orally
Prothionamide 15–20 mg/kg/day BD orally
Explanation of Standard Code
Cycloserine* 10–20 mg/kg/day OD or BD orallt
♦♦ Each antitubercular drug has standard abbreviation, i.e.
PAS* 150 mg/kg/day BD or TDS orally
• Isoniazid (H)
• Rifampicin (R) *Cycloserine and PAS are bacteriostatic others are bactericidal.
94   Mastering the BDS IIIrd Year  (Last 25 Years Solved Questions)

Q.35. Write in brief clinical features and treatment of pneu- Q.38. Write short note on malignancy of lung.
monia. (Jun 2010, 5 Marks) (Mar 2013, 3 Marks)
Or Ans. Malignancy of lung is known as lung cancer.
Write sign and symptoms of pneumonia.
 (Jan 2012, 5 Marks) Predisposing Factors
Ans. Pneumonia is an accumulation of secretions and
♦♦ Cigarette smoking
inflammatory cells in alveolar spaces of lungs caused
♦♦ Occupational exposure: This is due to radioactive gases,
by infection.
asbestos, arsenic, nickel, chromates, metallic iron
Clinical Features ♦♦ Atmospheric pollution: In urban areas
♦♦ Lung diseases: Chances of lung cancer increases in patients
Symptoms with cryptogenic fibrosing alveolitis.
♦ Malaise, fever, rigors, and night sweats, vomiting in the
Clinical Features
elderly confusion and disorientation.
♦♦ Dyspnea, cough, and sputum which is often blood-stained Symptoms
or rusty and difficult to expectorate. ♦♦ Non-specific: Weakness, tiredness, anorexia, loos of weight
♦♦ Pain aggravated by cough, deep breath or movement, ♦♦ Respiratory: Presence of influenza-like illness or pneumonia
usually localized to site of inflammation. distal to obstruction caused by tumor, Increased cough,
Signs mild hemoptysis, dyspnea, chest pain which is worst at
night, wheeze.
♦♦ In early stage:
• Pulse rate and heart rate increases Signs
• Alae nasi are in action
♦♦ Clubbing of fingers
• Presence of herpes on the lip
♦♦ Supraclavicular lymphadenopathy
• Movements of chest are restricted.
♦♦ Mid- inspiratory crackles over a lobe, reduction of breadth
• Percussion over the affected area is diminished.
• Breath sound are harsh with prolong expiration and sounds over a lobe and signs of lobar collapse.
few crypts. ♦♦ Wheezing sound is present
♦♦ Signs of pulmonary consolidation: ♦♦ Pleural effusion is present.
• Limitation of movement on affected side. Diagnosis
• Increased vocal fremitus
• Impaired percussion It is based on physical examination and investigations.
• Breathing sounds are bronchial, few crypts may be
audible. Investigations
• Vocal resonance is increased. ♦♦ Chest X-ray shows peripheral round mass. It is well
• Pleural rub may be heard. defined or irregular with pseudopodia or Sun-ray
♦♦ During the period of resolution: projection radiating from its surface.
• Bronchial breathing disappear ♦♦ Sputum cytology: On examination reveals presence of
• Normal breath sound appear
cancer cells.
• Coarse crepitations during both phases of respiration.
♦♦ Bronchoscopy: Fiber-optic bronchoscopy is done.
Treatment ♦♦ Thoracic CT including upper abdomen is done to see
extensions of malignancy.
For treatment refer to Ans 6 of same chapter.
Q.36. Write in brief sign, symptoms and treatment of Management
pulmonary tuberculosis. (Jun 2010, 5 Marks) ♦♦ Surgery can be done.
Ans. For sign and symptoms refer to Ans 28 of same chapter. ♦♦ Radiotherapy: When resection is not carried out of tumor,
For treatment refer to Ans 18 of same chapter. radiotherapy is employed. Continuous hyperfractional
Q.37. Write clinical features of pulmonary tuberculosis. How accelerated radiotherapy (CHART) three times daily for
will you investigate and treat a case of sputum AFB 2 weeks increases chances of survival.
positive tuberculosis? (Mar 2013, 8 Marks) ♦♦ Chemotherapy: Useful in patients with widespread disease
Ans. For clinical features refer to Ans 28 of same chapter. and no local symptoms. Commonly used combinations
For investigation refer to Ans 28 and for treatment of are mitomycin-ifosfamide-cisplatin, Mitomycin-cisplatin-
case of sputum AFB positive tuberculosis refer to Ans vincristine, cisplatin-gemcitabine and cisplatin-vinorelbine.
34 of same chapter. Three cycles are given.
Section 1:  General Medicine  95

Q.39. Write short note on first-line antitubercular drugs. Ethambutol


(Feb 2014, 3 Marks) ♦♦ Ethambutol is selectively tuberculostatic and clinically as
Ans. The first-line antitubercular drugs are Isoniazid, active as S. Fast multiplying bacilli are more susceptible
Rifampin, Pyrazinamide, Ethambutol and Strepto- as are many atypical mycobacteria.
mycin. ♦♦ Ethambutol inhibits arabinosyl transferases involved in
These drugs have high antitubercular efficacy as well as arabinogalactan synthesis and to interfere with mycolic
low toxicity and are used routinely. acid incorporation in mycobacterial cell wall.
♦♦ Patient acceptability of ethambutol is very good and side
Isoniazid
effects are few.
♦♦ Isoniazid is a first line antitubercular drug.
• It acts on extracellular as well as intracellular TB and Streptomycin
is equally effective in alkaline and acidic medium. ♦♦ It was the first clinically useful antitubercular drug.
• The most possible action of isoniazid is inhibition ♦♦ It is tuberculocidal, but less effective than rifampin;
of synthesis of mycolic acids which are unique fatty acts only on extracellular bacilli. Thus, host defense
acid component of mycobacterial cell wall. The lipid mechanisms are needed to eradicate the disease.
content of Mycobacterium exposed to isoniazid is ♦♦ It penetrates tubercular cavities, but does not cross to the
reduced. CSF, and has poor action in acidic medium.
• Isoniazid is completely absorbed orally and penetrates ♦♦ Resistance developed rapidly when streptomycin was used
all body tissues, tubercular cavities and placenta. alone in tuberculosis–most patients had a relapse.
• It is extensively metabolized in liver by acetylation.
• The metabolites are excreted in urine. Q.40. Describe various extra pulmonary tuberculosis and
how will you treat a case of tubercular meningitis. 
Rifampin  (Nov 2014, 8 Marks)
Ans.
♦♦ Rifampin is bactericidal to M. tuberculosis.
♦♦ Bactericidal action covers all subpopulations of TB bacilli, Extrapulmonary Tuberculosis
but acts best on slowly or intermittently dividing ones, as
In extrapulmonary tuberculosis which involves all the major
well as on many atypical mycobacteria. organs in body from heart to gastrointestinal tract.
♦♦ It has good sterilizing and resistance preventing actions.
Following are the various types of extrapulmonary tuberculosis:
♦♦ Rifampin inhibits DNA dependent RNA synthesis.
♦♦ It is well-absorbed orally, widely distributed in the Tuberculous Pericarditis
body: penetrates cavities, caseous masses, placenta and ♦♦ Involvement of pericardium is very common by
meninges. M. tuberculum bacteria.
♦♦ It is metabolized in liver to an active deacetylated ♦♦ It occurs in form of pericarditis, pericardial effusion and
metabolite which is excreted mainly in bile, some in urine later on there is constrictive pericarditis.
also. ♦♦ Its earliest sign is pericardial rub and fever.
♦♦ As disease exacerbate, there is formation of effusion and
Pyrazinamide when it is massive cardiac temponade may occur.
♦♦ It is weakly tuberculocidal but more active in acidic Gastrointestinal Tract
medium.
♦♦ It is more lethal to intracellularly located bacilli and to ♦♦ There is occurrence of primary lesion due to swallowing
of tubercle bacilli which lodges in the ileocecal region
those at sites showing an inflammatory response (pH is
and produce primary hypertrophic ileocecal tuberculosis.
acidic at both these locations). ♦♦ Common manifestations are tuberculous peritonitis, tabes
♦♦ It is highly effective during the first 2 months of therapy mesenterica, tuberculous enteritis, diarrhea and fistula.
when inflammatory changes are present.
♦♦ By killing the residual intracellular bacilli, it has good Skeletal Tuberculosis
‘sterilizing’ activity. ♦♦ Tuberculosis of spine, paravertebral cold abscess, sinus
♦♦ It inhibits mycolic acid synthesis, but by interacting with tract formation and involvement of weight-bearing joints
a different fatty acid synthase encoding gene. such as knees and hips.
♦♦ Pyrazinamide is absorbed orally, widely distributed, has ♦♦ When tubercular granuloma extend to the mandible or
good penetration in CSF, extensively metabolized in liver maxilla via extraction socket by means of hematological
and excreted in urine. spread, this can lead to tuberculous osteomyelitis.
96   Mastering the BDS IIIrd Year  (Last 25 Years Solved Questions)

♦♦ Early diagnosis by joint aspiration and biopsy is done to Etiology


prevent disability and to avoid surgery.
♦♦ Bacterial: Pneumococcus, Staphylococcus, Streptococcus, H.
Genitourinary Tuberculosis influenza, E.coli, Klebsiella, Pseudomonas, etc.
♦♦ Atypical: Viral, Rickettsial, mycoplasmal
♦♦ It presents as painless hematuria and sterile pyuria.
♦♦ Protozoal: E. histolytica
♦♦ In this renal parenchyma, calyces, ureter and bladder are
♦♦ Fungal: Actinomycosis, aspergillosis, histoplasmosis,
affected in descending order.
nocardiosis
♦♦ Testicular and epididymal involvement may be present
♦♦ Allergic: Loeffler’s syndrome
which causes sterility.
♦♦ Radiation
♦♦ In females Involvement of fallopian tubes causes female
♦♦ Collagenosis: Systemic lupus erythematosus, rheumatoid
infertility.
arthritis, polyarteritis nodosa
Meningeal Tuberculosis ♦♦ Chemical: Aspiration of vomitus, gases and smokes,
kerosene, paraffin and petroleum.
Tuberculoma in brain and tubercular meningitis are very
common complications and may leave behind number of Investigations
sequelae.
♦♦ X-ray chest is done for assessing the opacity in lung.
Adrenal Tuberculosis ♦♦ Examination of sputum is done by Gram’s and Ziehl –
Neelsen stains.
♦♦ It produces the picture of Addison’s disease. ♦♦ Sensitivity as well as sputum culture should be carried out
♦♦ It is seen in long standing cases of abdominal tuberculosis. for anaerobic and aerobic organisms.
♦♦ If sputum is absent, then bronchoscopic aspiration is done
Lupus Vulgaris
for both culture and sensitivity.
When tubercle bacilli invade the skin this is known as lupus ♦♦ Blood examination is done for assessing leucocytosis.
vulgaris. ♦♦ Sputum, urine and serum testing should be done for
identifying the pneumococcal antigen.
Oral Tuberculosis ♦♦ If empyema is present, then pleural fluid aspiration is done.
♦♦ Blood gas analysis should be done.
♦♦ When tubercle bacilli directly inoculated in oral tissues
Q.42. Write etiology, diagnosis and management of pulmo-
of a person who had not acquired the immunity to the
nary tuberculosis. (July 2016, 12 Marks)
tuberculosis.
Ans. Involvement of lungs by tuberculosis is known as
♦♦ In oral cavity, there is involvement of tongue, gingiva,
pulmonary tuberculosis.
extraction socket and buccal mucosa.
♦♦ In oral cavity at above mention sites, there is presence of Etiology
typical tubercular ulcer.
♦♦ Mycobacterium tuberculosis leads to the pulmonary
♦♦ Tuberculous gingivitis appears as diffuse, hyperemic or
tuberculosis.
nodular papillary proliferation.
♦♦ In immunocompromised patients or in children
Treatment of a Case of Tubercular Meningitis tuberculosis can be caused by atypical Mycobacterium.
♦♦ Antitubercular drugs: Rifampicin 600 mg/day + Isoniazide Diagnosis
(600 to 900 mg/day) + Pyrazinamide (1.5 gm) should be
It is based on clinical signs and symptoms as well as
given. Treatment with this regimen is given for 2 months.
investigations.
This is followed by rifampicin 600 mg/day + Isoniazide
(600 to 900 mg/day) for 12 to 18 months.
Symptoms
♦♦ Steroids: Prednisolone 40–60 mg/day to reduce toxicity,
pia-arachnoid adhesions and feeling of well-being. ♦♦ Evening rise of temperature
♦♦ Night sweats
Q.41. Write etiology, clinical features, investigations and
♦♦ Malaise and cachexia
treatment of pneumonia. (Feb 2015, 12 Marks)
♦♦ Irritability and difficulty in concentration.
Ans. For clinical features of pneumonia refer to Ans 35 of same ♦♦ Cough and expectoration for more than three weeks.
chapter. ♦♦ Pleuritic chest pain
For treatment of pneumonia refer to Ans 6 of same ♦♦ Breathlessness is the feature of advanced disease.
chapter. ♦♦ Indigestion and dyspnea
Section 1:  General Medicine  97

♦♦ Amenorrhea often in young women Management


♦♦ Hoarseness of voice.
♦♦ Chemotherapy: For details, refer to Ans18 of same chapter.
♦♦ Sputum can be mucoid, purulent or blood stained.
♦♦ Corticosteroids: They are to be given in the severe cases to
♦♦ Hemoptysis is a classical feature
enable them to survive till antitubercular drugs become
♦♦ Presence of localized wheeze from local ulcer or narrowing
effective. Oral prednisolone is given in doses of 20 mg
of major bronchus.
orally for 6 to 8 weeks. Steroids produce euphoria and
♦♦ Presence of recurrent cold.
increase appetite in the patients.
♦♦ Surgery: Surgical resection of infected lobe is feasible.
Physical Signs
♦♦ Symptomatic treatment:
♦♦ Fever and weight loss • Cough: If it is irritative, linctus codeine is given.
♦♦ Tachycardia and tachypnea Smoking should be stopped.
♦♦ Rapid pulse rate • Laryngitis: Rest is given to the voice. If pain is
♦♦ Physical signs of collapse, consolidation, cavitation, present anesthetic powders, spray and lozenges
fibrosis, bronchiectasis, pleural effusion or pneumothorax. are given.
♦♦ In some cases only localized *rhonchi or rales are present.
Q.43. Describe etiology, clinical features, diagnosis, compli-
♦♦ Clubbing of fingers is present in chronic disease.
♦♦ Most common physical sign of chest is fine crepitation in cations and treatment of chronic obstructive pulmonary
the upper part of one or both the lungs. This is heard on disease. (Mar 2016, 8 Marks)
taking a deep breath after coughing. Or
♦♦ Later on, there can be presence of dullness to percussion or
Write etiology, clinical features, diagnosis, and man-
bronchial breathing in upper part of one or both the lungs.
♦♦ In chronic cases, there is evidence of volume loss and agement of chronic obstructive pulmonary disease
mediastinal shift. (COPD).
♦♦ Hilar, mediastinal and cervical groups lymph nodes are  (Jan 2018, 12 Marks)
enlarged with splenomegaly. Ans. Chronic obstructive pulmonary disease is characterized
by irreversible obstruction to the airflow throughout
Investigations lungs. It consists of two important disorders of lungs, i.e.
♦♦ Serology: In this ELISA, technique is used which helpful chronic bronchitis and emphysema. These both diseases
in diagnosis of tuberculosis in children. PCR technique is coexist in a single patient.
more specific and sensitive serological test than ELISA,
but PCR is less used due to its high cost. Etiology
♦♦ Chest X-ray: Presence of multiple nodular infiltrations or
ill-defined opacities in one of upper lobes is characteristic ♦♦ Localized:
for pulmonary tuberculosis. An area of translucency in • Congenital
radiopacities is indicative of cavitation. Presence of cavity • Compensatory due to lung collapse, scarring or
is indicative of an active lesion. In some of the patients resection
multiple thick-walled cavities can be seen. At the time • Partial bronchial obstruction due to neoplasm or
of fibrosis, trachea and mediastinum shift to same side. foreign body
Fibrosis can also cause calcification. • MacLeod’syndrome
♦♦ Pathological tests: ♦♦ Generalized:
• Blood examination: Peripheral blood examination shows • Idiopathic
monocytosis, i.e. 8 to 12% • Senile
• ESR is elevated. • Familial, i.e. due to alpha-l-anti-trypsin deficiency
• Tuberculin test: It is a test to recognize prior tubercular • Associated with chronic bronchitis, asthma or
infection, and is done by injecting one unit of purified pneumoconiosis.
protein derivative (PPD) on the forearm and readings Clinical Features
taken after 48 hours. Induration of more than l5 mm
indicates a positive test. The younger is the patient, In most of the patients, chronic bronchitis is associated with
greater is significance of positive test. A negative test emphysema so two types clinical syndromes of chronic
does not always exclude tubercular infection since it obstructive pulmonary disease are present i.e.
may be negative in patients of blood malignancies, 1. Predominant chronic bronchitis with emphysema, i.e.
malnourishment and those on immunosuppressive Bluebloater type
therapy. Tuberculin test is nonspecific and only 2. Predominant emphysema with some degree of chronic
indicates prior infection. Its sensitivity wanes with age. bronchitis, i.e. pink-puffers type
98   Mastering the BDS IIIrd Year  (Last 25 Years Solved Questions)

Features Pink-puffer Bluebloater, i.e. ♦♦ Heart:


i.e. prominent prominent chronic • Apex beat may not be visible or palpable.
emphysema bronchitis • Right ventricular heave can be present.
Onset Dyspnea and cough Cough without • Heart sounds can be diminished. Second sound can
dyspnea be loud. Gallop rhythm can be heared.
Build Thin Obese • Functional tricuspid regurgitation murmur can be
present.
Sputum Scanty Profuse,
• Hyperkinetic state with warm limb and water hammer
mucopurulent
pulse is present.
Dyspnea intense with purse Relatively mild ♦♦ Miscellaneous:
lip breathing dyspnea
• Hepatomegaly can be present.
Cough After dyspnea starts Before dyspnea • Optic disc can show papilliedema
starts
♦♦ Investigations:
Cardiac failure (Cor Rarely develop Often edematous • There is presence of post-bronchodilator FEV1/FVC
pulmonale) edema or overt and easily lapse into less than 0.7 which confirms the presence of persistent
heart failure CHF
airflow limitation and thus is diagnostic of chronic
Weight loss Marked weight loss No marked weight obstructive pulmonary disease.
loss except
• Arterial blood gas show retention of carbon dioxide
terminally
in emphysema.
Bronchial infections Less frequent More frequent
• Serum alpha-1-anti-trypsin levels to diagnose alpha-1
Episodes of Often terminal Repeated anti-trypsin deficiency.
respiratory failure • X-ray chest: Presence of hypertranslucency of lung
Pulmonary None or mild Moderate to severe fields, wide intercoastal spaces, diaphragm is low and
hypertension flat, heart is tubular shaped, presence of large hilar
Course Unrelenting downhill Ambulatory shadows, diminished peripheral vascular pattern,
Chest X-ray Signs of Bronchovascular rounded areas of hypertranslucency with thin hairline
emphysema seen markings are shadow forming margins.
prominent
Complications
Hypoxemia Mild Moderate to severe
Hypercapnia Absent or mild Present
In Emphysema

Cyanosis Absent or present Present and ♦♦ Pneumothorax due to rupture of bullae in pleural space.
terminally common ♦♦ Cor pulmonale, i.e. right-sided heart failure or right
ventricular hypertrophy secondary to lung disease.
Secondary Uncommon Common
polycythemia ♦♦ Type II respiratory failure.
Diffusing capacity Decreased Normal or increased In Chronic Bronchitis

Diagnosis ♦♦ Type I and type II respiratory failure


♦♦ Cor pulmonale
♦♦ History of chronic progressive symptoms, i.e. cough or ♦♦ Pulmonary arterial hypertension
wheeze or breathlessness. ♦♦ Secondary infections
♦♦ General condition: Patient can be emaciated, cyanosed and ♦♦ Secondary polycythemia
edematous. Jugular venous pressure may show giant
a-waves. Treatment
♦♦ Chest findings:
• Chest wall process is barrel-shaped. ♦♦ Bronchial irritants should be avoided, i.e.
• Movement of chest is decreased. • Smoking should be strictly stopped.
• Centrally placed mediastinum • Passive smoking is stopped.
• Percussion is hyper-resonant. • Gas smoke is avoided by housemakers.
• Breath is diminished vesicular with prolong expiration. • Aerosols such as hair spray, insecticide spray and
• Rhonchi are heared. aerosols should be avoided.
• Vocal resonance sounds are diminished. • Polluted atmosphere should be strictly avoided.
Section 1:  General Medicine  99

♦♦ Treatment of an infection: Or
• Ampicillin 250 to 500 mg every 6 hourly is given for Describe briefly the clinical features and management
5–7 days. of acute glomerulonephritis. (Sep 1999, 5 Marks)
• Cotrimazole 960 mg can be given as BD dose.
• Antibiotics should be given till purulent mucous Or
become mucoid. Give definition, etiology, sign, symptoms, investigations
• If necessary modify antibiotics as per culture and and treatment of nephritic syndrome.
sensitivity test. (Nov 2011, 8 Marks)
♦♦ Bronchodilators: Or
• In mild-to-moderate chronic bronchitis oral Write etiology, clinical features, investigations and
theophylline 150 mg BD or inhaled salbutamol 200 treatment of nephritic syndrome.
μg 6 hourly can be given.  (Feb 2015, 12 Marks)
• In severe bronchitis, ipratropium bromide 40 to 80 μg Or
6 hourly is added.
♦♦ Mucolytic agents: Bromhexine and carbocystein are to be Write nephritic syndrome under the following
given. headings: (Feb 2014, 2 Marks; Each)
♦♦ Corticosteroids: Prednisolone 30mg/day for 2 weeks is given. a. Causes
If improvement occur by oral steroids they are replaced b. Clinical features
by inhalational steroids. c. Investigations
♦♦ Domiciliary oxygen therapy: Long-term oxygen therapy d. Treatment
in low concentration, i.e. 2L/min by nasal cannula is Ans. Acute glomerulonephritis involves mainly the
given to reverse or to delay development of pulmonary glomeruli and to lesser extent the renal tubules by an
hypertension. acute transient inflammatory process which manifests
♦♦ If cor pulmonale is present diuretics such as furosemide, clinically by acute reduction in glomerular filtration
digitalis and potassium salts might be given. rate, rapid renal failure, proteinuria and salt and water
♦♦ Chest physiotherapy should be done, and proper exercises retention.
should be taught to the patient. Acute glomerulonephritis is also known as acute
♦♦ Patients with COPD should receive influenza and nephritic syndrome or nephritic syndrome.
pneumococcal vaccines. Nephritic syndrome is characterized by oliguria,
♦♦ Non-invasive ventilation is useful in those with pronounced hematuria, proteinuria, edema, hypertension and acute
daytime hypercapnia. renal failure.
♦♦ Lung volume reduction surgery can be done. In this parts
of lungs are resected to reduce hyperinflation. Etiology
♦♦ Infectious disease:
• Post-streptococcal glomerulonephritis
6. Diseases of Renal • Non-streptococcal post-infectious glomerulonephritis
–– Bacterial: Infective endocarditis, sepsis, Pneu-
System mococcal pneumonia, typhoid fever, secondary
syphilis, meningococcemia
Q.1. Describe briefly acute glomerulonephritis.  –– Viral: Hepatitis B, infectious mononucleosis,
 (Sep 1998, 5 Marks) mumps, measles, varicella, vaccinia, echovirus
and coxsackievirus
Or
–– Parasitic: Malaria, toxoplasmosis
Write short note on acute glomerulonephritis.  ♦♦ Multisystem disease:
 (Mar 2010, 5 Marks) • Systemic lupus erythematosus
Or • Vasculitis
• Henoch-Schönlein purpura
Write briefly on clinical features and management of • Goodpasture’s syndrome
acute glomerulonephritis.  (Sep 2006, 5 Marks) ♦♦ Primary glomerular disease:
Or • Mesangiocapillary glomerulonephritis
Write in brief sign, symptom and treatment of acute • Mesangial proliferative glomerulonephritis
glomerulonephritis  (June 2010, 5 Marks) ♦♦ Miscellaneous:
• Gullain-Barré syndrome
Or • Irradiation of Wilm’s tumor
Discuss etiology, clinical features, investigations and • Diphtheria-Pertussis-Tetanus (DPT) vaccine
treatment of acute glomerulonephritis. • Serum sickness.
 (Aug 2011, 15 Marks) • IgA nephropathy
100   Mastering the BDS IIIrd Year  (Last 25 Years Solved Questions)

Clinical Features that passed in preceding 24 hours. Fruit juices rich with
potassium are given with caution.
Symptoms ♦♦ Diet: Restrict dietary protein and restrict sodium and
♦♦ Patient complains to puffiness over the face and edema potassium intake. Monitor potassium and sodium.
over the feet in early hours of morning. ♦♦ Hypertension: Moderate-to-severe hypertension is
♦♦ In some cases, headache, vomiting and abdominal pain controlled by hydralazine, beta-blockers such as atenolol or
is present. calcium channel blockers or ACE inhibitors. Salt restriction
should be done.
Signs ♦♦ Antibiotics: Injection benzathine penicillin 500,000 units
♦♦ Generalized anasarca is present. It is more over face. IM 6 hourly for 7 days.
♦♦ Oligouria is present. ♦♦ Diuretics: It is not indicated unless there is acute LVF or
♦♦ Proteinuria is present. It is less than 1 g/day. pulmonary edema is present. Frusemide 40 mg IV daily
♦♦ Hypertension is present since there is retention of salt for few days, followed by oral substitution till diuretic
and water. phase is induced.
♦♦ Hematuria is present. ♦♦ In patients with progressive renal failure or if fluid
♦♦ Presence of circulatory congestion. overload is present, dialysis may have to be employed.
♦♦ Occurrence of circulatory congestion due to capillaritis, Q.2. Write short note on glomerulonephritis.
increased cardiac output and short circulation time.  (Jan 2012, 5 Marks)
Investigations Ans. Involvement of glomeruli in the kidney either by the
process of inflammation or immunologically mediated
♦♦ Urine examination: injury or part of generalized systemic diseases constitute
• Volume of urine is reduced. glomerulo­nephritis.
• Urine is dark in color or smoky when it is fresh and
after hemolysis it becomes tea colored. Classification of Glomerulonephritis
• Proteinuria is variable which is rarely more than
2.5 g/day. ♦♦ Clinical:
• Red cells and red cell casts are present in urine Acute nephritic syndrome
microscopy. There is also presence of white cells, white Sub-acute nephritis
cell casts and granular casts. Chronic nephritis
♦♦ Blood examination: Chronic renal failure
• There is presence of polymorphonuclear leucocytosis ♦♦ Morphologically based on histological examination:
• ESR is raised. Minimal change disease
• Blood urea and serum creatinine are raised. Membranous glomerulonephritis
• C3 levels are reduced. Focal segmental glomerulonephritis
♦♦ ASO titer: It is elevated in poststreptococcal nephritis. Membranoproliferative glomerulonephritis
♦♦ Antinuclear antibody is present in significant titer in lupus ♦♦ Etiologic:
nephritis. Primary glomerulonephritis
♦♦ X-ray chest shows cardiomegaly and pulmonary edema, Secondary due to systemic disease
but it is not always present. Hereditary disorders producing glomerulonephritis.
♦♦ Renal biopsy is indicative of glomerulonephritis Also refer to Ans 1 of same chapter.

Complications Q.3. Enumerate the common causes of nephrotic syn-


drome.
♦♦ Hypertensive encephalopathy (Aug 2012, 5 Marks) (Apr 2010, 5 Marks)
♦♦ Acute left heart failure Ans. It is a clinical complex with number of renal and
♦♦ Non-cardiogenic pulmonary edema extrarenal features
♦♦ Uremia • Hypoproteinemia
• Hypoalbuminemia
Management
• Generalized edema or anasarca
♦♦ Bed rest: Patient should be hospitalized and rest is given • Hypercholesterolemia
till illness is resolved. In mild cases, bed rest is given for 3 • Hypercoagulability.
weeks and in severe cases for 3 months. ♦♦ Primary glomerular diseases
♦♦ Fluid restriction: Avoid fluid overload. For first 24 hours • Minimal change nephropathy.
only 500 mL of water or glucose or barley water should • Mesangioproliferative glomerulonephritis.
be given. If volume of urine in 24 hours is less than • Membranous nephropathy.
400 mL teart for acute renal failure and if it is more than • Focal and segmental glomerulosclerosis.
400 mL limit intake of fluid to 500 mL + a volume equal to • Crescentic glomerulonephritis
Section 1:  General Medicine  101

♦♦ Idiopathic: Dose -Prednisolone 1 mg/kg/day, maximum 80


♦♦ Secondary to other diseases: mg/ day. Remission usually occurs between days
• Infections: Malaria, hepatitis B, herpes zoster, 7 and 14, though some patients need up to 16 weeks
streptococcal and staphylococcal infections, syphilis, therapy to achieve complete remission.
leprosy, schistosomiasis. Prednisolone dose is reduced to 0.5 mg/kg/day and
• Drugs: NSAIDs, Heavy metals such as gold, anti – then tapered slowly. An attempt to stop treatment
convulsants, penicillamine, ACE inhibitors, heroin, should be made after 8 weeks. In patients who
rifampicin, tolbutamide and probenecid. relapse, course of prednisolone should be repeated.
• Malignancy: Hodgkin’s disease and other lymphomas. • Immunosuppressive drugs: In steroid-resistant
• Systemic diseases: Diabetes mellitus, amyloidosis, patients, or in those in whom remission can
systemic lupus erythematosus, Henoch-Schonlein only be maintained by heavy doses of steroids,
purpura, cryoglobulinemia, polyarteritis nodosa. cyclophosphamide 1.5–2 mg/kg/ day for 8–12 weeks
• Familial disorders: Congenital (neonatal) nephrotic with concomitant prednisolone 7.5–15 mg/day.
syndrome, Alport's syndrome, Fabry’s disease • Levamisole: In corticosteroid-dependent children
• Miscellaneous conditions: Reflux nephropathy, renal 2.5 mg/ kg to maximum 150 mg on alternate days is
vein thrombosis, toxemia of pregnancy, allergic
useful in maintenance of remission.
reactions to insect bites, pollens and vaccines, renal
artery stenosis. Q.5. Describe the clinical features and diagnostic features
of nephrotic syndrome. 
Q.4. Outline the management of nephrotic syndrome. 
 (Feb 1999, 5 Marks) (Feb 2002, 5 Marks)
 (Sep 2009, 5 Marks)
Ans. Clinical Features
Ans. Management of nephrotic syndrome involves:
♦♦ Age and sex: Nephrotic syndrome is two to three times more
• Scientific treatment of underlying morphology or
common in childhood with peak incidence at 2–3 years.
causative disease
In this age group, there is a male : female ratio of 2.5 : 1,
• General measures to control nephritic complications
in adults, sex incidence is equal.
• Treatment to reduce proteinuria
♦♦ Edema: It is peripheral involving the limbs, particularly
Non-specific measures that may reduce proteinuria
lower limbs. In children, edema may be more obvious
include ACE and NSAIDs.
in the face and abdomen. Usually, massive generalized
ACE-I and ARBs (angiotensin-receptor blockers)
anasarca, the patient almost weighing double his true
reduce proteinuria and slows the rate of progression
weight. Intense edema of the scrotum or vulva may occur.
of rate of renal failure by lowering intraglomerular
There may be bilateral hydrothorax. Edema may persist
pressure and preventing development of
for many weeks or months. Spontaneous subsidence with
hemodynamically mediated focal segmental
diuresis (nephrotic crisis) may occur, to be followed again
*glomerulosclerosis.
by increase of edema.
• Edema: Advice patient to take low sodium diet i.e.
♦♦ Gastrointestinal symptoms: Anorexia causes severe
1 to 2 g/day. In mild edema thiazide induce gentle
malnutrition, Diarrhea and vomiting due to edema of
diuresis. In moderate edema frusemide in doses of
intestinal wall.
80 to 120 mg/day or torsemide 20 to 40mg/day is
given. In patients with severe edema frusemide 20 ♦♦ General symptoms: Prolonged protein loss causes anorexia,
to 40mg/day is combined with spironolactone 100 lethargy, tiredness, frequent infections and muscle wasting.
to 200mg/day for complete resorption of sodium Dyspnea may occur, if there is fluid in the pleural cavity.
throughout the nephron. ♦♦ Blood pressure: There may be periods of hypertension;
• Hypercholesterolemia: It is treated by the lipid ultimately with development of chronic nephritis
lowering agent specially statins. Atorvastatin 20 mg permanent hypertension may develop.
od or BD Diagnostic Features
• Anticoagulation is needed for patient with deep
vein thrombosis, arterial thrombosis and pulmonary Diagnosis is not difficult when there is massive generalized
embolism. edema with albuminuria, hypoproteinemia and hyper-
• Antiplatelet agents and warfarin could be advocated. cholesterolemia.
• Diet: Restricted protein diet is advised since high Diagnosis is based on clinical signs and investigations
protein diet accelerates progression of nephritic
Clinical signs
syndrome.
• Vitamin D supplementation is advisable. ♦♦ Physical examination reveals generalized edematous
• Antibiotics: Aggressive antibiotic therapy is given in person. Edema may persist for many weeks or months.
nephrotic syndrome as chances of sepsis are always ♦♦ Tachycardia is present while blood pressure is normal.
present. Mainly cephalexin are given ♦♦ Pitting edema is present over legs and feet, abdominal
• Corticosteroids: Produce rapid and complete walls and lower eyelids.
remission with clearing of proteinuria in 90% cases. ♦♦ Eyelids become puffy
102   Mastering the BDS IIIrd Year  (Last 25 Years Solved Questions)

♦♦ At places where edema is severe ascites and pleural ♦♦ Blood examination:


effusion are seen. • Anemia: It is slight normochromic.
♦♦ Kidneys are not palpable. • Hypoalbuminemia: Serum albumin usually less than
3 g/100 mL. Total serum globulin concentration
Investigations frequently lowered with often elevation of α2 and
♦♦ Urine examiation: β-globulins.
• Oliguria while edema is forming, dieresis or normal • Serum lipids show increase in LDL levels and
amount of urine during period of subsidence of cholesterol.
edema. • ESR is raised due to hyperfibrinogenemia.
• Proteinuria: It is massive, usually more than 5 g/day • Serum complement level: Serum complement C3 and C4
though variable from time to time; urine becomes levels get reduced.
almost solid on boiling. Daily loss of protein may be • Other biochemical tests, i.e. blood urea, serum
20–50 g. creatinine, creatinine clearance and electrolytes are
• 24 hour urine shows excretion of albumin or protein normal.
more than 3.5 g/day. ♦♦ Ultrasound of abdomen: It can show normal small or
• Red blood cells are absent or few are seen large kidneys which depends on the underlying cause.
• Casts: Fatty casts, tubular cells, oval fat bodies, doubly Amyloid and diabetic kidneys are large while kidney in
refractile bodies are seen glomerulonephritis is small.
♦♦ Blood examination: ♦♦ Renal biopsy: is normal on light microscopy but electron
• Anemia: It is slight normochromic. microscopy shows typical abnormalities (effacement of
• Hypoalbuminemia: Serum albumin usually less than epithelial cell foot processes).
3 g/100 mL. Total serum globulin concentration For management refer to Ans 4 of same chapter.
frequently lowered with often elevation of α2 and Q.7. Enumerate the complications and outline the manage-
β-globulins. ment of nephrotic syndrome. (Mar 2000, 5 Marks)
• Serum lipids show increase in LDL levels and
Ans. Complications of Nephrotic Syndrome
cholesterol.
♦♦ Protein malnutrition: Due to protein loss
• ESR is raised due to hyperfibrinogenemia.
♦♦ Ascites, pleural and pericardial effusion
• Serum complement level: Serum complement C3 and C4
♦♦ Accelerated atherosclerosis and coronary artery disease
levels get reduced.
♦♦ Pulmonary thromboembolism, stroke, deep vein
• Other biochemical tests, i.e. blood urea, serum
thrombosis due to hypercoagulability
creatinine, creatinine clearance and electrolytes are
♦♦ Vitamin D deficiency is responsible for hypocalcemia
normal.
♦♦ Microcytic hyperchromic anemia
♦♦ Ultrasound of abdomen: It can show normal small or
♦♦ Hypothyroidism due to depressed thyroxin level due to
large kidneys which depends on the underlying cause.
loss of thyroxin, binding globulin.
Amyloid and diabetic kidneys are large while kidney in
♦♦ Chronic renal failure eventually leading to end stage
glomerulonephritis is small.
renal diseases and require hemodialysis and renal
♦♦ Renal biopsy: It is normal on light microscopy but electron
transplantation
microscopy shows typical abnormalities (effacement of
For management refer to Ans 4 of same chapter.
epithelial cell foot processes).
Q.6. Describe briefly investigation and management of Q.8. Write short note on nephrotic syndrome.
nephrotic syndrome. (Mar 2001, 5 Marks)  (Mar 2016, 3 Marks) (Mar 2007, 2 Marks)
Ans. Investigation of Nephrotic Syndrome  (Feb/Mar 2005, 5 Marks)
♦♦ Urine examiation:
Or
• Oliguria while edema is forming, dieresis or normal
amount of urine during period of subsidence of Write note on nephrotic syndrome.
edema.  (Dec 2009, 10 Marks)
• Proteinuria: It is massive, usually more than 5 g/day Ans. Nephrotic syndrome is defined as the presence of heavy
though variable from time to time; urine becomes proteinuria and hypoalbuminemia in association with
almost solid on boiling. Daily loss of protein may be varying degrees of edema, lipiduria and hyperlipidemia.
20–50 g.
• 24-hour urine shows excretion of albumin or protein Causes
more than 3.5 g/day. ♦♦ Primary glomerular diseases:
• Red blood cells are absent or few are seen • Minimal change nephropathy.
• Casts: Fatty casts, tubular cells, oval fat bodies, doubly • Mesangioproliferative glomerulonephritis.
refractile bodies are seen • Membranous nephropathy.
Section 1:  General Medicine  103

• Focal and segmental glomerulosclerosis. • Casts: Fatty casts, tubular cells, oval fat bodies, doubly
• Crescentic glomerulonephritis refractile bodies are seen
♦♦ Idiopathic ♦♦ Blood examination:
♦♦ Secondary to other diseases: • Anemia: It is slight normochromic.
• Infections: Malaria, hepatitis B, herpes zoster, • Hypoalbuminemia: Serum albumin usually less than
streptococcal and staphylococcal infections syphilis, 3 g/100 mL. Total serum globulin concentration
leprosy, schistosomiasis. frequently lowered with often elevation of α2 and
• Drugs: NSAIDs, heavy metals such as gold, anti – β-globulins.
convulsants, penicillamine, ACE-inhibitors, heroin, • Serum lipids show increase in LDL levels and
rifampicin, tolbutamide and probenecid. cholesterol.
• Malignancy: Hodgkin’s disease and other lymphomas. • ESR is raised due to hyperfibrinogenemia.
• Systemic diseases: Diabetes mellitus, amyloidosis, • Serum complement level: Serum complement C3 and C4
systemic lupus erythematosus, Henoch-Schonlein levels get reduced.
purpura, cryoglobulinemia, polyarteritis nodosa. • Other biochemical tests, i.e. blood urea, serum creatinine,
• Familial disorders: Congenital (neonatal) nephrotic creatinine clearance and electrolytes are normal.
syndrome, Alport's syndrome, Fabry’s disease ♦♦ Ultrasound of abdomen: It can show normal small or
• Miscellaneous conditions: Reflux nephropathy, renal large kidneys which depends on the underlying cause.
vein thrombosis, toxemia of pregnancy, allergic Amyloid and diabetic kidneys are large while kidney in
reactions to insect bites, pollens and vaccines, renal glomerulonephritis is small.
artery stenosis. ♦♦ Renal biopsy: It is normal on light microscopy but electron
microscopy shows typical abnormalities (effacement of
Clinical Features epithelial cell foot processes).
♦♦ Age and sex: Nephrotic syndrome is two to three times more
Treatment
common in childhood with peak incidence at 2–3 years.
In this age group, there is a male: female ratio of 2.5 : 1, in ♦♦ Edema: Advice patient to take low sodium diet, i.e. 1–2
adults, sex incidence is equal. g/day. In mild edema thiazide induce gentle dieresis.
♦♦ Edema: It is peripheral involving the limbs, particularly In moderate edema, frusemide in doses of 80–120 mg/
lower limbs. In children, edema may be more obvious day or torsemide 20–40 mg/day is given. In patients with
in the face and abdomen. Usually, massive generalized severe edema frusemide 20–40 mg/day is combined with
anasarca, the patient almost weighing double his true spironolactone 100–200 mg/day for complete resorption
weight. Intense edema of the scrotum or vulva may occur. of sodium throughout the nephron.
There may be bilateral hydrothorax. Edema may persist ♦♦ Corticosteroids: Produce rapid and complete remission
for many weeks or months. Spontaneous subsidence with with clearing of proteinuria in 90% cases. Dosage is
diuresis (nephrotic crisis) may occur, to be followed again Prednisolone 1 mg/kg/day, maximum 80 mg/ day.
by increase of edema. Remission usually occurs between days 7 and 14, though
♦♦ Gastrointestinal symptoms: Anorexia causes severe some patients need up to 16 weeks therapy to achieve
malnutrition, Diarrhea and vomiting due to edema of complete remission.
intestinal wall. ♦♦ Prednisolone dose is reduced to 0.5 mg/kg/day and then
♦♦ General symptoms: Prolonged protein loss causes anorexia, tapered slowly. An attempt to stop treatment should be
lethargy, tiredness, frequent infections and muscle wasting. made after 8 weeks. In patients who relapse, course of
Dyspnoea may occur, if there is fluid in the pleural cavity. prednisolone should be repeated.
♦♦ Blood pressure: There may be periods of hypertension; ♦♦ Immunosuppressive drugs: In steroid-resistant patients, or in
ultimately with development of chronic nephritis those in whom remission can only be maintained by heavy
permanent hypertension may develop. doses of steroids, cyclophosphamide 1.5–2 mg/kg/ day for
8–12 weeks with concomitant prednisolone 7.5–15mg/day.
Investigations ♦♦ Levamisole: In corticosteroid dependent children 2.5 mg/
♦♦ Urine examiation: kg to maximum 150 mg on alternate days is useful in
• Oliguria while edema is forming, dieresis or normal maintenance of remission.
amount of urine during period of subsidence of ♦♦ Antibiotics: Aggressive antibiotic therapy is given in
edema. nephrotic syndrome as chances of sepsis are always
• Proteinuria: It is massive, usually more than 5 g/day present. Mainly cephalhexins are given.
though variable from time to time; urine becomes Q.9. Write short note on urine examination of renal failure.
almost solid on boiling. Daily loss of protein may be  (Oct 2003, 10 Marks)
20–50 g. Ans. Reduction in glomerular filtration rate and rise in
• 24 hour urine shows excretion of albumin or protein nitrogenous and nonnitrogenous substance in blood as
more than 3.5 g/day. a result of degenerated renal function is known as renal
• Red blood cells are absent or few are seen failure.
104   Mastering the BDS IIIrd Year  (Last 25 Years Solved Questions)

For Acute Renal Failure • Serum lipids show increase in LDL levels and
♦♦ Volume of urine is less (Oliguria) or absent, i.e. anuria. cholesterol.
♦♦ Albumin content in urine depends on the underlying • ESR is raised due to hyperfibrinogenemia.
causes. • Serum complement level: Serum complement C3 and C4
♦♦ Urine osmolality is more than 600 mOsm/L; urinary levels get reduced.
sodium excretion is less than 20 mmol/L; urine/plasma • Other biochemical tests, i.e. blood urea, serum
urea ratio is more than 10:1 creatinine, creatinine clearance and electrolytes are
♦♦ Dipstick for blood, protein or both is suggestive of renal normal.
inflammatory process. ♦♦ Ultrasound of abdomen: It can show normal small or
♦♦ Urine microscopy: Presence of red cell cast is suggestive of large kidneys which depends on the underlying cause.
glomerulonephritis. Amyloid and diabetic kidneys are large while kidney in
glomerulonephritis is small.
For Chronic Renal Failure ♦♦ Renal biopsy: It is normal on light microscopy but electron
microscopy shows typical abnormalities (effacement of
♦♦ Volume of urine passed daily is in the form of polyuria.
epithelial cell foot processes).
♦♦ Appearance and color is normal.
♦♦ There is no odor present Q.11. Outline the management of urinary tract infection. 
♦♦ Specific gravity of urine is low and fixed.  (Apr 2010, 5 Marks)
♦♦ Albuminuria is present Ans. Infection of kidneys, ureter or bladder by microorganism
♦♦ Microscopically, there are hyaline and broad cell cast in that either ascends from the urethra or spread to kidney
urine from bloodstream.
♦♦ Serum creatinine increases
Management
Q.10. How will you diagnose and investigate a case of ne­
ph­rotic syndrome? (Sep 2005, 10 Marks) ♦♦ Rest should be given to the patient.
Ans. Diagnosis is based on clinical signs and investigations: ♦♦ Mild-to-moderate cases should be treated by giving
antibiotics, i.e. nitrofurantoin, ciprofloxacin, gentamicin,
Clinical Signs cotrimoxazole, norfloxacin.
♦♦ In severe cases, parenteral antibiotics, i.e. inj. carbenicillin
♦♦ Physical examination reveals generalized edematous
is given.
person. Edema may persist for many weeks or months.
♦♦ Fluid intake should be high, alkalization of urine with
♦♦ Tachycardia is present while blood pressure is normal.
potassium citrate solution to alleviate symptoms
♦♦ Pitting edema is present over legs and feet, abdominal
♦♦ To reduce pain, antispasmodic drug is given.
walls and lower eyelids.
♦♦ Eyelids become puffy Q.12. Outline the management of acute gout. 
♦♦ At places where edema is severe ascites and pleural  (Feb 1999, 5 Marks)
effusion are seen. Ans. Gout is an abnormality of metabolism which results in
♦♦ Kidneys are not palpable. the deposition of monosodium urate crystals in joints
and other tissues.
Investigations
♦♦ Urine examiation: Management
• Oliguria while edema is forming, dieresis or normal Management of Acute Attack
amount of urine during period of subsidence of
edema. ♦♦ NSAIDs: Any of the NSAID should be given except aspirin
• Proteinuria: It is massive, usually more than 5 g/day as it causes uric acid retention. Selective COX-2 inhibitors
though variable from time to time; urine becomes e.g. Etoricoxib 120 mg OD or Valdecoxib 20–40 mg BD
almost solid on boiling. Daily loss of protein may be can be given. Colchicine 0.5 mg postoperatively every
20–50 g. 2 hours, up to 4–6 mg/day is now reserved for patients
• 24 hour urine shows excretion of albumin or protein without renal, hepatic or marrow disease, in whom the
more than 3.5 g/day. more effective NSAIDs contraindicated or poorly tolerated.
• Red blood cells are absent or few are seen ♦♦ Corticosteroids: Methyl prednisolone acetate 5–25 mg per
• Casts: Fatty casts, tubular cells, oval fat bodies, doubly joint as Intra-articular injection is given. Systemic oral
refractile bodies are seen prednisolone 20 mg/ day tapered off over 4–10 days or IM
♦♦ Blood examination: triamcinolone 60 mg/ day repeated in 1–4 days, are highly
• Anemia: It is slight normochromic. effective relatively safe alternatives.
• Hypoalbuminemia: Serum albumin usually less than ♦♦ Restrict allopurinol or uricosuric drugs till the acute attack
3 g/100 mL. Total serum globulin concentration has settled for 2–3 weeks as they can prolong the acute
frequently lowered with often elevation of α2 and attack or trigger their episodes. Patients should be advised
β-globulins. to avoid diuretics and/ or salicylates.

Q10. *Debility = Weakness


Section 1:  General Medicine  105

Long-term Management kidney disease (e.g. diabetic nephropathy), a further


reduction in proteinuria can be achieved by adding a
♦♦ Diet: It should given low in purine and fats.
mineralocorticoid receptor antagonist (MRA) such as
♦♦ Weight reduction in obese patient, avoid alcohol.
eplerenone or spironolactone.
♦♦ Anti-hyperuricemic therapy:
♦♦ Immunosuppressants such as cyclophosphamide and
• Hypouricemic drug: Allopurinol 300 mg daily with a
azathioprine should be reserved for patients with
NSAID or colchicines 0.5 mg BD.
progressive renal insufficiency or with vasculitic lesions
• Febuxostat 40 mg OD to reduce and maintain serum
on renal biopsy.
uric acid levels below 6 mg/dL. If serum uric acid levels
♦♦ Patients with moderate-to-severe proteinuria are usually
are more than 6 mg/dL after 2 to 4 weeks treatment
fluid overloaded and require diuretic therapy along with
with the drug 80 mg/day, dose is increased to 120mg/
dietary salt restriction.
day.
♦♦ There are recommendations for no restrictions or only mild
• Probenecid 0.5 to 1gm BD or Sulfinpyrazone 100mg
restriction in protein intake, i.e.0.8–1 g/kg daily.
TDS as an alternative to allopurinol with colchicines
if renal function is not impaired. Q.14. Write short note on renal failure.(Mar 2007, 2 Marks)
• Benzbromarone 100 mg daily in patients with Ans. The deterioration of renal function resulting in decline
moderate renal impairment where other uricosuric in GFR and rise in urea and non-nitrogenous substances
agents are ineffective. in blood is called renal failure.
• If patient do not tolerate or fail the full dose of other Types:
treatments IV pegloticase is given 8 mg for every
1. Acute renal failure.
2 weeks. It dramatically reduces the serum uric acid.
2. Chronic renal failure.
Q.13. Write the investigation and management of proteinu-
Acute Renal Failure
ria. (Mar 2000, 5 Marks)
Ans. Proteinuria is defined as presence of protein usually
albumin in the urine, this finding may be transient and
entirely benign or a sign of severe renal disease.

Investigations of Proteinuria
♦♦ Heat coagulation method: This is done by heating upper
portion of urine in a test tube. White coagulum present at
top of the urine is suggestive of proteinuria.
♦♦ Dipstick test: This is a bedside test, which patient can
perform by himself if his/her color vision is normal. Color
change in strip is compared to the color on bottle which
quantify loss of proteins.
♦♦ Electrophoresis of proteins: It detects the globulins in urine.
♦♦ Immunoelectrophoresis: It is carried out to identify fragments
of immunoglobulins when there is a monoclonal peak on
routine urine paper electrophoresis.
♦♦ 24 hours urine for proteinuria: It is done to separate the cases
of nephrotic syndrome, i.e. massive proteinuria more than
3.5 g/day from other etiology of proteinuria in which there
is mild proteinuria, i.e. 1 to 2 g/day.
♦♦ Radioimmunoassay: This test is done for detection of
microalbuminuria.
Management of Proteinuria
Proteinuria is not a specific disease. So its treatment depends
on identifying and managing its underlying cause.
♦♦ Angiotensin-converting enzyme (ACE) inhibitors
and angiotensin receptor blockers (ARBs) reduce
intraglomerular pressure by inhibiting angiotensin II
mediated efferent arteriolar vasoconstriction. These groups
of drugs have a proteinuria-reducing effect independent
of their antihypertensive effect.
♦♦ When treatment with an ACE inhibitor or ARB does not
adequately control proteinuria in a patient with chronic Fig. 15:  Renal failure
106   Mastering the BDS IIIrd Year  (Last 25 Years Solved Questions)

♦♦ There is acute fall in GFR over days or weeks. ♦♦ Hematuria can occur.
♦♦ Invariable reversible ♦♦ In chronic infection, there are periods of acute exacerbation
♦♦ Causes of acute renal failure may be pre renal, renal or in addition to malaise, low-grade fever and ill health.
postrenal. ♦♦ In acute cases, tachycardia and tenderness develops on
♦♦ Oligouria and anuria are characteristics. deep palpation on renal region.
♦♦ Symptoms and signs of uremia are of recent onset.
Investigations
♦♦ Parameters of acute reduction of GFR, i.e. edema,
hypertension, salt and water retention are presenting ♦♦ Urine examination: A mid stream specimen is taken for
features. examination. A heavily infected urine may look hazy to
♦♦ Broad casts or renal failure casts are absent. naked eye. It may have a fishy smell in E. coli infection
♦♦ Specific gravity of urine is high and ammonical in Proteus infection. Reaction of urine
♦♦ Dialysis is rewarding required for short period. usually is acidic. Albumin is present in traces. Microscopic
examination will show clumps of pus cells. Urine should
Chronic Renal Failure be cultured for type of organisms. Colony count done and
♦♦ GFR falls gradually over a period of months or years. sensitivity of the organism to various drugs.
♦♦ Invariably irreversible ♦♦ Plain X-ray abdomen is done for renal/bladder/ureteric
♦♦ Causes are mostly renal but may be extrarenal. calculi
♦♦ Polyuria and nocturia are commonly seen. ♦♦ Intravenous pyelography for any congenital anomalies,
♦♦ Symptoms and signs of uremia are of more than three calculi.
months of duration. ♦♦ Ultrasonography for renal size, calculi and any other
♦♦ Proof of chronicity is present, such as long duration abnormality.
of symptoms and signs of uremia, small sized kidney,
Management
anemia, hypertension, renal osteodystrophy.
♦♦ Broad casts or renal failure casts present. ♦♦ Rest should be given to the patient.
♦♦ Specific gravity of urine is low and fixed. ♦♦ Mild-to-moderate cases should be treated by giving
♦♦ Repeated chronic maintenance and dialysis is required. antibiotics, i.e. nitrofurantoin, ciprofloxacin, gentamicin,
♦♦ Renal transplantation is final answer. cotrimoxazole, norfloxacin.
♦♦ In severe cases, parenteral antibiotics, i.e. inj. carbenicillin
Q.15. Write briefly clinical features and management of is given.
nephrotic syndrome. (Apr 2007, 5 Marks) ♦♦ Fluid intake should be high. Alkalization of urine with
Ans. For clinical features refer to Ans 5 and for management potassium citrate solution to alleviate symptoms.
refer to Ans 4 of same chapter. ♦♦ To reduce pain, antispasmodic drug is given.
Q.16. Write short note on UTI. (Sep 2007, 2 Marks) Q.17. Differentiate acute and chronic nephritis. 
Ans. UTI- Urinary Tract Infection  (Mar 2008, 2.5 Marks)
The urinary tract infection is divided into two general Ans.
anatomic categories:
1. Lower tract infection (Urethritis and cystitis). Features Acute nephritis Chronic nephritis
2. Upper tract infections (Acute pyelonephritis, 1. Onset Acute Insidious
prostatitis, intra renal and perinephric abscess).
2. History History of sore throat History of acute or
Etiology or skin infection in sub-acute nephritis
previous weeks
Gram-negative organisms are mainly responsible, i.e. E.coli,
3. Clinical features Puffiness of face, Pallor, hypertension,
Proteus, Klebsiella, Enterobacter, etc. minimal edema, cardiomegaly and no
♦♦ UTI is more common in females as compared to males. pallor, hypertension, ascites
♦♦ Pain is the most common sign. It can be mild, constant or cardiomegaly
intense and colicky. 4. Urine Smoky and dark Polyuria, specific
♦♦ There is also abrupt onset of frequency of micturition and colored and specific gravity is low and
dysuria. gravity is raised fixed, albumin
♦♦ Patient suffers from fever with rigors and chills, malaise, is traced
loss of apetite and vomiting. 5. Blood chemistry Serum protein and Blood urea and
♦♦ There is presence of scalding pain in urethra during level are normal cholesterol levels are creatinine
micturition. increased
♦♦ After urinary bladder get empty, there is desire to pass
6. Ultrasonography Kidney is normal Kidney is small
more urine.
Section 1:  General Medicine  107

Q.18. Define nephrotic syndrome its etiology, clinical ♦♦ Hypertension is invariably present in chronic renal
features, pathological blood and urine findings and failure. Patient may develop hypertensive heart failure,
management. (Nov 2011, 8 Marks) myocarditis or pericarditis.
Or ♦♦ Patient has acidotic breathing.
♦♦ Repeated chest infections are common. Uremic lung
Describe the etiology, clinical features and manage-
develops soon.
ment of nephrotic syndrome. (Dec 2010, 15 Marks)
♦♦ Skin has a yellowish-brown pigmentation and patient
Ans. This syndrome refers to massive proteinuria more complains of intractable itching.
than 3.5 g/day mainly of albumin, reduced albumin ♦♦ Because of disturbances in calcium metabolism
concentration, edema, hyperlipidemia, lipiduria and (osteomalacia, osteoporosis, renal osteodystrophy) and
hypercoagulability. development of secondary hyperparathyroidism, patient
• For etiology refer to Ans 3 of same chapter. has aches and pains in the bones.
• For clinical features refer to Ans 5 of same chapter ♦♦ Cases of chronic renal failure suffer from anaemia which
• For blood and urine findings refer to Ans 6 of same is normocytic normochromic. Bleeding tendency is often
chapter present and patient may bleed from various sources.
• For management refer to Ans 4 of same chapter. ♦♦ Menstrual irregularities in women (amenorrhea, infertility)
Q.19. Describe causes, clinical features and mana­ge­ment of are common while men may complain of impotence.
chronic renal failure. (Sep 2009, 4.5 Marks) ♦♦ Physical examination reveals generally an ill, looking
Ans. Persistent impairment of tubular and glomerular function person, anaemic, pale. Skin is shallow with a peculiar
of gradual onset so that kidneys are unable to maintain earthy color.
their normal physiological functions constitutes chronic ♦♦ Tongue is brown, dry and furred.
renal failure. ♦♦ Breath has a peculiar ammoniuremic smell (uraemic fetor).
♦♦ Hiccough is present.
Causes Management
♦♦ Congenital or heredofamilial disorders: Polycystic disease of
kidney, Alport syndrome ♦♦ The aim is to delay the progression of renal failure
♦♦ Vascular diseases of kidney: Vasculitis, polyarteritis nodosa, and main thought is to correct water and electrolyte
systemic lupus erythematosus. disturbances, prevent endogenous breakdown of protein
♦♦ Glomerular diseases: Proliferative glomerulonephritis, and retention all its end products as well as controlling
Crescentric glomerulonephritis, membranous blood pressure levels and improving the quality of life.
glomerulonephritis, glomerulosclerosis, diabetic ♦♦ Diet: Adequate caloric intake by encouraging patient to
nephropathy. consume high caloric, carbohydrate foods such as sweetened
♦♦ Tubulointerstitial diseases: Chronic pyelonephritis, rice, sugar, sweetened biscuits, corn-flour, bread etc.
tuberculosis of kidney, analgesic nephropathy Restriction of dietary proteins (15-18 g per day) essential to
♦♦ Obstructive renal diseases: Pelvic tumor, benign enlargement reduce the rate of production of nitrogenous waste products.
of prostrate, retro-peritoneal fibrosis High carbohydrate diet gives energy and so it is essential
that patient takes adequate amount of calories.
Clinical Features ♦♦ Fluids and electrolytes: Patients in chronic renal failure have to
maintain balance in their salt and water intake. Fluid intake
♦♦ Cases of chronic renal failure may remain asymptomatic
should be sufficient so that patient passes at least 2–2.5 liters
for a long time and it is often either an intercurrent
of urine per day. Overhydration as well as dehydration must
infection or exacerbation of the disease process or some
be prevented. Salt intake has to be restricted in presence of
complications which draw attention to the patients illness. edema, hypertension and congestive cardiac failure. Intake
Symptoms are varied and involve all the major systems of potassium has to be restricted.
of body. ♦♦ Anemia: Anemia in chronic renal failure requires fresh
♦♦ Patient has got marked weakness, lethargy and restlessness. blood transfusions. It is better to use packed cells.
There is anorexia, nausea and vomiting. ♦♦ Renal bone disease: There is hypocalcemia along with
♦♦ Sleep rhythm is disturbed. Nausea, retching is most features of hyperparathyroidism. Calcium orally is given
marked in the early hours of morning. to act as phosphate binder from the gut. Large doses of
♦♦ Patient develops revulsion towards food. Dehydration is vitamin D or D3 are taken to help in the absorption of
invariably present. calcium from the gut.
♦♦ Neurological features include headache, lassitude, ♦♦ Hypertension: Control of hypertension is essential, since it
neuropathy, muscular weakness. In those with severe shall worsen the renal failure as well as produce various
degree of hypertension, convulsions, muscular twitchings, complications. Angiotensin converting enzyme (ACE)
irritability and in late stages of renal failure, loss of inhibitors are the one which, are preferred for treating
consciousness even leading to coma. renal hypertension.
108   Mastering the BDS IIIrd Year  (Last 25 Years Solved Questions)

Q.20. Write about diet in renal failure. ( Nov 2008, 10 Marks) Signs
Ans. Following is the diet in renal failure: ♦♦ Physical examination reveals generalized edematous
♦♦ Adequate caloric intake by encouraging patient to consume person.
high caloric, carbohydrate foods such as sweetened rice, ♦♦ Tachycardia is present while blood pressure is normal.
sugar, sweetened biscuits, corn-flour, bread, etc. ♦♦ Pitting edema is present over legs and feet, abdominal
♦♦ Restriction of dietary proteins (15–18 g per day) essential walls and lower eyelids.
to reduce the rate of production of nitrogenous waste ♦♦ Eye lids become puffy
products. ♦♦ At places where edema is severe ascites and pleural
♦♦ High carbohydrate diet gives energy and so it is essential effusion are seen.
that patient takes adequate amount of calories. ♦♦ Kidneys are not palpable.
♦♦ Patients in chronic renal failure have to maintain balance
in their salt and water intake. Treatment
♦♦ Fluid intake should be sufficient so that patient passes Refer to Ans 4 of same chapter.
at least 2–2.5 liters of urine per day. Overhydration as
well as dehydration must be prevented. Salt intake has Q.23. Write sign and symptoms of uremia.
to be restricted in presence of edema, hypertension and  (Dec 2010, 5 Marks)
congestive cardiac failure. Ans. Following are the signs and symptoms of uremia:
♦♦ Intake of potassium has to be restricted if levels of serum
Symptoms
potassium are high.
♦♦ Diet consists of whole meal bread, marmalade or honey, ♦♦ Malaise, weakness, lethargy and fever.
small quantity of fish, fruits and vegetables. ♦♦ Nausea, vomiting, anorexia, diarrhea
♦♦ Milk about 200 mL per day is permitted. ♦♦ Polyuria and breathlessness when patient gets exerted.
♦♦ Patient can take good quantities of sugar, jam, honey, tea ♦♦ Headache, confusion and disorientation
and lemonade. ♦♦ Irregular menstrual cycle and loss of libido.
Q.21. Write about causes of anasarca and define and discuss
Signs
nephrotic syndrome. (Nov 2008, 15 Marks)
Ans. Anasarca is a form of generalized edema. There is mas­ ♦♦ Presence of fluid and electrolyte disturbances.
sive collection of fluid in the subcutaneous interstitial ♦♦ Neurological: Cramps, lethargy, myopathy, seizures and
spaces and the various sacs of the body. coma
The various causes of anasarca are: ♦♦ Cardiovascular: hypertension, pulmonary edema,
pericarditis
Causes of Anasarca ♦♦ Gastrointestinal tract: Gastritis and enteritis
♦♦ Heart disease—right heart failure. ♦♦ Hematological: Anemia and bleeding diathesis
♦♦ Kidney disease—nephritis. nephrosis, ♦♦ Endocrinal: Secondary hyperparathyroidism, amenorrhea,
♦♦ Severe anaemia—tropical diseases (epidemic dropsy) osteodystrophy.
♦♦ Nutritional causes—wet beri-beri, hypoproteinemia Q.24. Write short note on acute nephritis.
♦♦ Endocrine disorder—myxedema
 (Nov 2011, 3 Marks)
♦♦ In congenital—general edema.
For nephrotic syndrome, refer to Ans 19 of same chapter. Ans. If the kidneys suddenly become inflamed, a condition
is developed called acute nephritis. Acute nephritis can
Q.22. Write in brief on sign, symptoms and treatment of lead to kidney failure.
nephrotic syndrome. (May/Jun 2009, 5 Marks)
Ans. Nephrotic syndrome is a clinical complex with number Types
of renal and extra-renal features
There are several types of acute nephritis.
Symptoms
Interstitial Nephritis
♦♦ Gradual onset of generalized edema with puffiness of
In interstitial nephritis, the spaces between the renal tubules
eyelids.
that form urine become inflamed. The kidneys swell from the
♦♦ Patient complains of loss of appetite, malaise and
inflammation.
generalized weakness.
♦♦ Swelling over abdomen is present
Pyelonephritis
♦♦ Breathlessness is present
♦♦ Diarrhea is also present. Pyelonephritis is an infection in the bladder that travels up the
♦♦ Amount of passing of urination is decreased in 24 hours. ureters and spread into the kidneys.
Section 1:  General Medicine  109

Glomerulonephritis ♦♦ Proteinuria is less, i.e. 1 g/day


♦♦ Hypertension is present.
This type of acute nephritis produces inflammation in the
♦♦ Circulatory congestion can occur.
glomeruli. Inflamed glomeruli may not filter the blood properly.
Treatment
Symptoms
♦♦ Bed rest: Patient should be hospitalized and rest is given
♦♦ Pain in the pelvis
till illness is resolved. In mild cases, bed rest is given for 3
♦♦ Pain or a burning sensation while urinating weeks and in severe cases for 3 months.
♦♦ Frequent need to urinate ♦♦ Fluid restriction: Avoid fluid overload. For first 24 hours
♦♦ Cloudy urine only 500 mL of water or glucose or barley water should
♦♦ Blood or pus may be present in urine be given. If volume of urine in 24 hours is less than 400
♦♦ Pain in the kidney area and/or abdomen mL teart for acute renal failure and if it is more than 400
♦♦ Swelling in the body, commonly in the face, legs, and feet mL limit intake of fluid to 500 mL + a volume equal to
♦♦ Vomiting that passed in preceding 24 hours. Fruit juices rich with
♦♦ Fever. potassium are given with caution.
♦♦ Diet: Restrict dietary protein and restrict sodium and
Diagnosis potassium intake. Monitor potassium and sodium.
Various diagnostic tests may be needed to confirm a case of ♦♦ Hypertension: Moderate-to-severe hypertension is
acute nephritis. These tests include the following: controlled by hydralazine, beta-blockers such as atenolol or
♦♦ A biopsy of the kidneys. A biopsy is a small tissue sample calcium-channel blockers or ACE inhibitors. Salt restriction
taken from an organ and examined in a laboratory setting. should be done.
♦♦ Antibiotics: Injection benzathine penicillin 500,000 units
♦♦ Urine and blood testing. These tests may detect and locate
IM 6 hourly for 7 days.
bacteria and signs of infection. Abnormal blood cells may
♦♦ Diuretics: It is not indicated unless there is acute LVF or
be present to show signs of infection.
pulmonary edema is present. Frusemide 40 mg IV daily
♦♦ A CT scan may be used to take pictures of your pelvis
for few days, followed by oral substitution till diuretic
and abdomen. phase is induced.
Treatment ♦♦ In patients with progressive renal failure or if fluid
overload is present, dialysis may have to be employed.
♦♦ Medications:
Q.26. Write about acute renal failure under following head-
• Antibiotics and pain relievers may be, used if
ings. (Apr 2015, 2 + 2 Marks)
pyelonephritis is present.
a. Causes
• If high blood pressure is present, calcium channel
b. Management
blockers should be taken.
Ans.
• Corticosteroids or other immune suppressing
♦♦ Causes:
medications should also be given.
• Pre-renal causes: The causes for renal hypoperfusion
♦♦ Home Care:
are septicaemia, hypovolemia, hemorrhage, shock,
• Drink more and more water: Water helps your kidneys burn, crushing injury, hemolysis inside the vessels,
flush out any waste products that may be hampering rhabdomyolysis.
the recovery. • Intra-renal causes: The causes for intrinsic renal disease
• Eat less sodium to prevent fluid retention. are renovascular obstruction, glomerulonephritis,
Q.25. Write short note on clinical features and treatment of vasculitis, tubulointerstitial nephritis, acute tubular
post-streptococcal glomerulonephritis. necrosis.
 (Mar 2013, 3 Marks) • Obstructive causes: Obstruction of the urinary tract at
Ans. It follows an acute streptococcal infection of throat or any place.
skin by Group-A beta hemolytic streptococci. ♦♦ Management:
Management of acute renal failure is divided into three
Clinical Features phase, i.e.
• Oligouric phase:
♦♦ It is commonly seen in children. –– Water and electrolyte balance should be
♦♦ In morning, there is presence of puffiness over the face maintained. One liter of fluid should be started
and edema over the feet. and CVP is also maintained. Fluids which are to
♦♦ Presence of headache, vomiting and abdominal pain be replaced should be calculated by loss to urine
♦♦ Oligouria is present. output of yesterday.
110   Mastering the BDS IIIrd Year  (Last 25 Years Solved Questions)

––Proteins should be given, i.e. 20–40 g of proteins ♦♦ Bone marrow aplasia


per day are given. • Aplastic anaemia
–– In cases of infections, antibiotics are to be given • Pure red cell aplasia
based on culture and sensitivity test. ♦♦ Anemia due to systemic infections or systemic disorders:
–– Sodium bicarbonate should be given. • Anaemia due to chronic infection
–– Dialysis should be done when in oliguria is • Anaemia due to chronic renal disease
present, hyperkalemia, metabolic acidosis etc. • Anaemia due to chronic liver disease
• Diuretic phase • Disseminated malignancy
–– Fluid and electrolytes should be replaced. • Endocrinal diseases
–– IV glucose or glucose saline is given. ♦♦ Anemia due to bone marrow infiltration
–– 70–80 g of protein per day is given. • Leukemias
–– Patient is told to have plenty of fruit juices. • Lymphomas
• Recovery phase • Myelofibrosis/myelosclerosis
–– After 7–20 days recovery occurs. • Multiple myeloma
• Congenital sideroblastic anemia
Q.27. Write in detail about nephrotic syndrome under the ♦♦ Anemia due to increased red cell destruction (haemolytic
headings: anaemias)
a. Definition • Intracorpuscular defect (hereditary or acquired)
b. Etiology • Extracorpuscular defect (acquired)
c. Clinical features
Morphological Classification of Anemia
d. Diagnosis
e. Treatment Based on the red cell size, hemoglobin content and red cell
indices, anemias are classified into 3 types:
Ans. a. Definition: Nephrotic syndrome is defined as the 1. Microcytic, hypochromic: MCV, MCH, MCHC are all
presence of heavy proteinuria and hypoalbuminemia reduced, e.g. in iron-deficiency anemia and in certain
in association with varying degrees of edema, noniron deficient anemias (sideroblastic anaemia,
lipiduria and hyperlipidemia. thalassaemia, anaemia of chronic disorders).
b. For etiology refer to Ans 3 of same chapter. 2. Normocytic, normochromic: MCV MCH, MCHC are all
c. For clinical features refer to Ans 5 of same normal e.g. after acute blood loss, haemolytic anaemias,
chapter. bone marrow failure, anaemia of chronic disorders.
d. For diagnosis refer to Ans 10 of same chapter. 3. Macrocytic: MCV is raised, e.g. in megaloblastic anemia
due to deficiency of vitamin B12 or folic acid.
e. For treatment refer to Ans 4 of same chapter.
Physiological Classification
This is based on the reticulocyte production index:
7. Diseases of Blood ♦♦ Anemia with reticulocyte production count less than 2.5
Q.1. Write short note on anemia. (Feb 2006, 5 Marks) • Normocytic-normochromic: Iron deficiency anemia,
aplastic anemia, anemia of inflammatory, renal and
Or endocrine disease.
Write clinical features, type and management of anemia • Microcytic or Macrocytic
 (July 2016, 12 Marks) Microcytic: Sideroblastic, thalassemia
Ans. Anemia is a state in which the hemoglobin concentration Macrocytic: Vitamin B12 or folate deficiency
falls below the accepted normal range depending on age ♦♦ Anemia with reticulocyte production count more than 2.5
and sex. Hemolytic/hemorrhagic: Anemia due to blood loss,
Hemoglobinopathies, autoimmune hemolytic, etc.
Classification of Anemia/Types of Anemia
Clinical Features
Etiological Classification of Anemia
♦♦ Blood loss: Symptoms
• Post-hemorrhagic ♦♦ General: Lassitude and fatigue
• Chronic blood loss due to piles, hematemesis, ♦♦ Gastrointestinal tract: Nausea, weight loss, anorexia,
menorrhagia flatulence and constipation
• Hookworm disease ♦♦ Cardiovascular symptoms: Palpitations, exertional dyspnea,
♦♦ Deficiency of hemopoietic factors angina, throbbing in head and ear.
• Iron deficiency ♦♦ Central nervous system symptoms: Tinnitus, headache,
• Folate and vitamin B12 deficiency dizziness, insomnia, numbness, tingling sensation in hand
• Protein deficiency, i.e., diarrhoea, malabsorption and feet.
Section 1:  General Medicine  111

♦♦ Genitourinary tract: Amenorrhea or menorrhagia, loss of • Haematinic should be started only after adequate
libido. blood examination, since response to a haematinic
may obscure the blood picture.
Signs
• The specific hematinic should be given alone.
♦♦ Presence of pallority on skin, mucous membrane and • The hematinic should be given in adequate doses for
conjunctiva. a sufficient period of time.
♦♦ Tachycardia
♦♦ Presence of collapsing pulse Q.2. Write note on iron-deficiency anemia. 
♦♦ Midsystolic flow murmur across aortic and the pulmonary  (Apr 2010, 5 Marks) (Mar 2011, 3 Marks)
valves. Or
♦♦ In severe anemia, there is presence of cardiomegaly and Write short note on clinical features and investigations
congestive heart failure. of iron deficiency anemia. (Mar 2007, 5 Marks)
♦♦ Edema can be present.
Or
Investigations Write iron-deficiency anemia under following head-
Every case of anemia should have the following investigations ings: (Dec 2012, 2 Marks each)
to detect degree and cause of anemia: a. Etiology
♦♦ Hemoglobin count decreases. b. Clinical Features
♦♦ RBC count, packed cell volume, mean corpuscular volume c. Investigations
d. Treatment
and mean corpuscular hemoglobin concentration (MCHC).
Total leukocyte count and differential leukocyte count. Or
♦♦ Peripheral blood film for type of anemia and shape of RBCs Write etiology, clinical features and treatment of iron
and presence of any abnormal cells. deficiency anemia. (Jun 2014, 12 Marks)
♦♦ Clotting time and bleeding time in hemolytic anemia. Ans. Iron deficiency anemia is commonest cause of anemia
♦♦ Blood platelets. and is a microcytic hypochromic anemia.
Bone marrow examination is done when cause of anemia
requires further investigation especially to detect type of Causes / Etiology
erythropoiesis. ♦♦ Due to increased blood loss:
• Gastrointestinal: Peptic ulcer, piles, hookworm
Other Investigations
manifestation, Carcinoma of stomach, acute erosive
♦♦ Stools for parasites: Test for presence of blood and stools is gastritis, ulcerative colitis
done in patients suspected to chronic blood loss. • Lung: Due to hemoptysis
♦♦ Urine for albumin, bile salt, pigments and urobilinogen • Renal: Haemoglobinuria and hematuria
♦♦ Gastric analysis: Histamine fast achlorhydria in pernicious • Uterine: Menorrhagia, post menopausal uterine
anemia and megaloblastic anemia. bleeding.
♦♦ Studies for detecting steatorrhea and malabsorption • Nose: Epistaxis
studies. ♦♦ Due to increased body demands:
♦♦ Schilling test for vitamin B12 absorption in megaloblastic • In adolescence
and dimorphic anemias. • During prematurity
♦♦ Figlu test is done to assess folic acid deficiency. About • In pregnancy and lactation.
15 g of histidine hydrochloride is given by mouth and the ♦♦ Due to inadequate dietary intake:
urine in which it is excreted it is collected over next eight • In low socioeconomic status
hours. Normal excretion is 1–17 mg • In elder patients with loss of teeth
• Anorexia of pregnancy.
Management ♦♦ Decreased absorption:
♦♦ Correction ofdietary deficiency, i.e. Faulty dietary habits, • In achlorhydria
chronic alcoholism and malnourishment. • In patients with malabsorption
♦♦ Treatment of underlying cause i.e. Ankylostomiasis, piles, • In cases with gastrectomy.
menorrhagia, infection, chronic kidney failure, leukemia, Clinical Features
liver disease, collagen disease or endocrine deficiency,
surgical correction of intestinal abnormalities, e.g. blind loop. Symptoms
♦♦ Removal of toxic chemical agent or drug, i.e. in some cases ♦♦ Tiredness
of haemolytic anemia or aplastic anemia. ♦♦ Weakness
♦♦ Blood transfusion: Its chief value is its immediate effect. ♦♦ Lethargy
♦♦ Administration of substances specifically lacking, i.e. ♦♦ Loss of appetite
112   Mastering the BDS IIIrd Year  (Last 25 Years Solved Questions)

♦♦ Headache and bodyache ♦♦ Due to increased blood loss:


♦♦ Inability to concentrate, giddiness • Gastrointestinal: Peptic ulcer, piles, hookworm
♦♦ Breathlessness manifestation, carcinoma of stomach, acute erosive
♦♦ Epigastric discomfort. gastritis, ulcerative colitis
• Lung: Due to hemoptysis
Signs
• Renal: Hemoglobinuria and hematuria
♦♦ Pallor • Uterine: Menorrhagia, postmenopausal uterine
♦♦ Palpitation bleeding.
♦♦ Angular stomatitis • Nose: Epistaxis
♦♦ Atrophic gastritis ♦♦ Due to increased body demands:
♦♦ Flattening or spoon-shaped nails, i.e. platynychia and • In adolescence
koilonychia • During prematurity
♦♦ Tongue is pale and smooth • In pregnancy and lactation.
♦♦ Glossitis ♦♦ Due to inadequate dietary intake:
♦♦ Hepatosplenomegaly • In low socioeconomic status
♦♦ Pulmmer-Vinson syndrome, i.e. dysphagia and cricoid webs. • In elder patients with loss of teeth
• Anorexia of pregnancy.
Investigations ♦♦ Decreased absorption:
• In achlorhydria
♦♦ Blood picture and red cell indices:
• In patients with malabsorption
• Hemoglobin: Fall on hemoglobin concentration.
• In cases with gastrectomy.
• Red cells: RBCs in blood film are hypochromatic and
microcytic and there is anisocytosis, poikilocytosis and Q.4. Describe clinical signs and symptoms of iron deficiency
elliptocytosis. anemia. (Sep 1998, 5 Marks)
• Reticulocyte count: Normal or reduced. Or
• Absolute values: MCV, MCH and MCHC are decreased. Write signs and symptoms of iron deficiency anemia.
• Leukocytes: Usually normal  (Dec 2010, 5 Marks)
• Platelets: Usually normal but raised if bleeding is cause Ans. Refer to Ans 2 of same chapter.
of anemia. Q.5. Write short note on hemolytic anemia. 
• ESR: Value of ESR is low  (Sep 1999, 5 Marks) (Sep 2006, 5 Marks)
♦♦ Bone marrow findings: (Sep 2009, 4 Marks)
• Marrow cellularity: Marrow cellularity is increased due Ans. Hemolytic anemia is a reduction in number of circulating
to erythroid hyperplasia. red cells from there premature destruction.
• Erythropoiesis: Normoblastic erythropoiesis.
• Marrow iron is deficient. Etiological Classification
♦♦ Biochemical findings:
♦♦ Acquired or extracorpuscular
• Serum iron level is low.
• Immunohemolytic anemia:
• Total iron-binding capacity is high.
–– Autoimmunohemolytic anemia
• Serum ferritin is very low.
- Warm antibody autoimmune hemolytic anemia
Management - Cold antibody autoimmune hemolytic anemia
–– Drug induced immunohemolytic anemia
♦♦ Proper physical and mental rest. –– Iso immune hemolytic anemia.
♦♦ Good nourishing diet with supplementation of foods rich • Mechanical trauma: Micro-angiopathic hemolytic
in iron. anemia
♦♦ Oral iron therapy: Ferrous sulphate 200 mg TID is given in • Direct toxic effect: Malaria, bacteria, infection and other
between the meals. If after taking the drug, there is any agents
abdominal pain, nausea, vomiting or constipation, the salt
• Acquired red cell membrane abnormalities: Paroxysmal
is changed to ferrous gluconate or ferrous fumarate. Oral
nocturnal hemoglobinuria.
iron therapy should be given for 6 months.
• Splenomegaly.
♦♦ Parenteral iron therapy: Iron sorbitol ciric acid complex 1.5
♦♦ Hereditary or Intracorpuscular
mg/kg body weight is given as IM or Iron dextran in 5%
• Abnormalities of red cell membrane:
glucose is given as IV
–– Hereditary spherocytosis
♦♦ Blood transfusion: Packed red cells are transfused.
–– Hereditary elliptocytosis
Q.3. Enumerate causes of iron-deficiency anemia.  –– Hereditary stomatocytosis.
 (Sep 1999, 4 Marks) • Disorders of red cell interior:
Ans. Following are the causes of iron-deficiency anemia: –– Red cell enzyme defect:
Section 1:  General Medicine  113

- Defects in HMP shunt: G6PD deficiency ♦♦ Patients having hereditary spherocytosis may undergo
- Defects in glycolytic pathway: Pyruvate kinase splenectomy if they have moderate to severe disease or
deficiency. have experienced episodes of hemolytic crisis or gall
–– Disorders of hemoglobin: stones.
- Structurally abnormal hemoglobin, sickle syn­ Q.6. Describe briefly etiological classification of hemolytic
drome and other hemoglobinopathies. anemia. (Mar 2001, 5 Marks)
- Reduced globin chain synthesis: Thalassemias. Ans. Refer to Ans 5 of same chapter.
Clinical features Q.7. Enumerate causes of agranulocytosis. 
♦♦ There is presence of high fever, toxemia, marked  (Sep 2008, 2.5, Marks)
*prostration, shock and hemoglobinuria Ans. It is defined as an acute disease marked by a deficit or
♦♦ Acute renal failure may develop absolute lack of granulocytic WBCs, i.e. neutrophils,
♦♦ Chronic form includes jaundice varying from mild to basophils and eosinophils.
severe form, mongoloid faces, splenomegaly, chronic leg
ulcers and pigment stones in gallbladder. Causes

Investigations Or Diagnosis for Hemolytic Anemia ♦♦ Endocrinal causes like hyperpituitarism, hypoadrenal­
ism.
Test of Increased Red Cell Breakdown
♦♦ Agranulocytosis due to drugs: This is a very important
♦♦ Blood film shows normocytic normochromic or dimorphic cause and is related to dosage of drugs as well as sensitivity
anemia. reaction. The main drugs are:
♦♦ Unconjugated serum bilirubin is raised • Anti-cancer drugs
♦♦ Urine urobilinogen is raised but bilirubinuria is absent • Anti-inflammatory drugs
♦♦ Serum hepatoglobin is reduced • Phenothiazines and tranquilizers
♦♦ Plasma LDH is increased • Sulfonamides and cotrimoxazole
♦♦ There is evidence of intravascular hemolysis i.e. • Anti-thyroid drugs
hemoglobinaemia, hemoglobinuria, metnemoglobinaemia, • Anti-diabetic drugs
hemosiderinuria. • Anti-histaminics
• Anti-epileptic drugs
Test of Increased Red Cell Production • Anti-microbial agents
♦♦ Reticulocyte count get increased ♦♦ Diuretics:
♦♦ Blood film shows macrocytosis, polychromasia and • Deficiency of vitamin B12 and folate
presence of normoblasts • It is caused due to obliteration of bone marrow due
♦♦ Bone marrow examination shows erythroid hyperplasia to myelofibrosis, lymphoma and sarcoma.
with raised iron store. It is caused due to bone marrow damage due to X-ray
♦♦ X – ray of bones show expansion of marrow space in bones radiation.
such as skull. ♦♦ Due to viral infections, i.e. hepatitis, HIV, influenza, EBV
♦♦ Due to bacterial infections, i.e. enteric fever, tuberculosis
Test of Damage to Red Cells and gram-negative bacterial septicemia
♦♦ Blood film show microspherocytes and fragmented RBCs. ♦♦ Due to protozoal diseases, i.e. malaria and Kala-azar.
♦♦ Osmotic fragility is increased. ♦♦ Due to autoimmune diseases i.e. SLE, chronic autoimmune
♦♦ Electrophoresis test is done for abnormal hemoglobin neutropenia
♦♦ Congenital i.e. in Kostmann’s syndrome
♦♦ Estimation of Hemoglobin A2 should be done
♦♦ Estimation of Hemoglobin F should be done Q.8. Describe clinical manifestations and management of
♦♦ Test for sickling is done agranulocytosis. (Mar 2003, 10 Marks)
♦♦ Screening test for G6PD deficiency is done Ans. It is defined as an acute disease marked by a deficit or
absolute lack of granulocytic WBCs, i.e. neutrophils,
Test for Shortened Red Cell Survivor basophils and eosinophils.
♦♦ Chromium labeled method show short red cell life span.\
Clinical Manifestations
Treatment of hemolytic anemia ♦♦ Females are more commonly affected than males
♦♦ Patients with compensated hemolytic process need no ♦♦ Early manifestations of agranulocytosis are may be in form
treatment. of sore throat or pain.
♦♦ Mainly management is general and specific. ♦♦ There may be fever going up to 130°F, sometimes coming
♦♦ Folic acid 5mg is given routinely and lifelong in patients with rigor chills, body ache and pain and extreme degree
with inherited hemolytic disorders of prostration.

Q5. *Prostration= Absolute exhaustion


114   Mastering the BDS IIIrd Year  (Last 25 Years Solved Questions)

♦♦ In large number of cases ulceromembranous lesions appear Clinical Manifestations


on throat, tonsils, gum, tongue and genitalia ♦♦ Females are more commonly affected as compared to males
♦♦ These are often covered with grayish black exudates and ♦♦ Early manifestations of agranulocytosis are may be in form
may become gangrenous. of sore throat or pain.
♦♦ Lymph gland generally cervical groups and in some, there ♦♦ There may be fever going up to 130°F, sometimes coming
are generalized lymphadenopathy with rigor chills, body ache and pain and extreme degree
♦♦ Liver and spleen may become enlarged. of prostration.
♦♦ As disease progresses severe toxemia develops and patient ♦♦ In large number of cases ulceromembranous lesions appear
may go into shock. on throat, tonsils, gum, tongue and genitalia
Management ♦♦ These are often covered with grayish black exudates and
may become gangrenous.
♦♦ Firstly, the cause is removable which is identifiable. ♦♦ Lymph gland generally cervical groups and in some, there
♦♦ Secondly, the infection is controlled and patient is put on are generalized lymphadenopathy
isolation ward and barrier nursing is done. ♦♦ Liver and spleen may become enlarged.
♦♦ Anti-bacterial drugs such as penicillin 5 mega units IM or ♦♦ As disease progresses severe toxemia develops and patient
IV 4 hourly or ciprofloxacin 500 mg IV 4 hourly is started may go into shock.
immediately.
♦♦ In addition metrogyl 500 mg every 6 hourly to take care Investigations
of infection ♦♦ Peripheral blood film show complete absence of
♦♦ Anabolic steroids are also given. neutrophils.
♦♦ In cases where toxemia is severe corticosteroids are em­ ♦♦ Bone marrow is normocellular and show decrease mature
ployed, i.e. injection dexamethasone 4 mg IV 6 hourly. neutrophils.
♦♦ Granulocytes transfusion is given to tide over crisis. This
is given daily for 5–7 days. Management
Q.9. Describe briefly diagnosis of agranulocytosis.  ♦♦ Firstly, the cause is removable which is identifiable.
 (Apr 2010, 5 Marks) ♦♦ Secondly, the infection is controlled and patient is put on
Ans. The diagnosis of agranulocytosis is based on clinical isolation ward and barrier nursing is done.
history and physical examination. ♦♦ Anti-bacterial drugs such as penicillin 5 mega units IM or
• Person with history of sore throat, fever with rigor and IV 4 hourly or ciprofloxacin 500 mg IV 4 hourly is started
chills, body ache and extreme degree of prostration immediately.
(absolute exhaustion). ♦♦ In addition metrogyl 500 mg every 6 hourly to take care
• In large number of cases ulceromembranous lesions of infection
are present on throat, tonsil, gums, tongue and ♦♦ Anabolic steroids are also given.
genitalia. The lesions are covered with grayish black ♦♦ In cases where toxemia is severe corticosteroids are
exudates and become gangrenous. employed, i.e. injection dexamethasone 4 mg IV 6 hourly.
• Confirmation of diagnosis is made by laboratory ♦♦ Granulocytes transfusion is given to tide over crisis. This
investigations. is given daily for 5 to 7 days.
• Peripheral blood film shows complete absence of Q.11. Write note on hemophilia. (June 2010, 5 Marks)
neutrophils. (Sep 2007, 5 Marks) (Feb 2006, 5 Marks)
• Bone marrow is hypocellular and there is depletion
Ans. Hemophilia is an inherited disorder which affects males
of myeloid elements.
and is transmitted by females.
Q.10. Discuss in short neutropenia. (Mar 1997, 5 Marks) • It is a X-linked recessive disorder caused by
Ans. The presence of abnormally small number of neutrophils deficiency of factor VIII. This type of hemophilia is
in the blood usually less than 1500 to 2000 per micro liter. called classic hemophilia.
• The another disorder is hemophilia B or Christmas
Causes disease which involves males inherited to X-linked
♦♦ Starvation and *debility. recessive trait. It is caused due to deficiency of factor
♦♦ Overwhelming infections and toxemia in older people. IX.
♦♦ Infections: Typhoid, measles, malaria, kala-azar, hepatitis,
influenza, HIV and miliary TB. Clinical Features
♦♦ Hypersplenism, liver cirrhosis. ♦♦ At birth: Newborn with hemophilia is usually healthy,
♦♦ Bone marrow failure: In aplastic anemia, leukemia and though bleeding from cord and cephalohematoma may
myelofibroma occur.
♦♦ Drugs: Sulfonamides, antibiotic, analgesics, anti-thyroids, ♦♦ In infant: Hemophilia is asymptomatic till 6 to 12 months,
anti-convulsants, etc. when bruising becomes more obvious. Bleeding from the
♦♦ Anaphylactic shock. mouth is common.
Section 1:  General Medicine  115

♦♦ During childhood: In severly affected individuals, ♦♦ Splenic sequestration


spontaneous bleeding may occur in joint and muscles, ♦♦ Dilution loss
including psosas muscle.
Oral Manifestations
♦♦ In adults: Frequency of spontaneous bleeding decreases,
but joints may already have been damaged. Intra-cranial ♦♦ Excessive bleeding after tooth extraction.
hemorrhage is a life-threatening complication. Spontaneous ♦♦ Extensive spontaneous gingival bleeding may be seen.
bleeds are common in mildly affected individuals, but after ♦♦ Petechiae do not blench on pressure
injury bleeding occurs till appropriate therapy has begun.
Management
Investigations ♦♦ Corticosteroids that is prednisolone 60 mg/dL.
♦♦ APTT is prolonged in hemophilia A and B, but PT is normal. ♦♦ Splenectomy, if there is no response to prednisolone to
♦♦ Specific factor assay demonstrate reduced factor VIII c in prednisolone in 3–4 days.
hemophilia A and factor IX in hemophilia B. If the clinical ♦♦ Local hemostatics.
history is strongly suggestive of hemophilia, normal APTT Hemophilia
does not exclude the diagnosis.
Etiology
Treatment ♦♦ It is a hereditary disorder.
♦♦ It occurs due to the deficiency to reduce activity of factor
♦♦ It is done by fresh blood transfusion when bleeding is in
VIII in hemophilia A or factor IX in hemophilia B.
the large mass.
♦♦ Factor VIII concentrate is employed in case of hemophilia Oral Manifestations
A and factor IX concentrate is employed in case of
♦♦ Prolonged bleeding after tooth extraction hematoma of
hemophilia B.
floor of mouth and larynx with subsequent respiratory
♦♦ A hemophilic will require treatment before any dental
embarrassment.
procedure. Such patient is managed by IV infusion or
♦♦ Physiological process of tooth eruption may be associated
cryoprecipitate or factor VIII combined with aminocaproic
with severe and prolonged hemorrhage.
acid 4 to 6 g QDS.
♦♦ For any major surgery or periodontal surgeries or extraction Hemophilia
of tooth hemophilic patient should be hospitalized.
Management
Infusion of factor VIII concentrate is given before surgery
and is continued for 48 to 72 hours. Antibiotic should also ♦♦ It is done by fresh blood transfusion when bleeding is in
be given to the patient the large mass.
♦♦ Factor VIII concentrate is employed in case of hemophilia
Complications of Hemophilia
A and factor IX concentrate is employed in case of
♦♦ Hepatitis in patients who received multiple transfusion of hemophilia B.
FFP or cryoprecipitate. ♦♦ A hemophilic will require treatment before any dental
♦♦ AIDS in a patient not screened for HIV and treated with procedure. Such patient is managed by IV infusion or
FFP or cryoprecipitate. cryoprecipitate or factor VIII combined with aminocaproic
♦♦ Anemia from excessive bleeding. acid 4 to 6 g QDS.
♦♦ Contractures in muscles following intramuscular ♦♦ For any major surgery or periodontal surgeries or extraction
hematomas. of tooth hemophilic patient should be hospitalized.
♦♦ Spontaneous intracranial hemorrhage in severe hemophilia. Infusion of factor VIII concentrate is given before surgery
Q.12. Write short note on bleeding disorders in dental and is continued for 48 to 72 hours. Antibiotic should also
practice. (Mar 2005, 5 Marks) (Sep 2008, 7 Marks) be given to the patient
Or
von Willebrand’s Disease
Write short note on bleeding disorder.
Etiology
(Aug 2012, 5 Marks)
Ans. Bleeding disorders: Bleeding disorders or hemorrhagic Hereditary coagulation disorder occurring due to qualitative
diatheses are a group of disorders characterized by or quantitative defects in von Willebrand factor.
defective hemostasis with abnormal bleeding. Oral Manifestations
♦♦ Gingival bleeding and postextraction bleeding
Bleeding Disorders
♦♦ Disease may be discovered after dental extraction.
Idiopathic Thrombocytopenic Purpura
Management
Etiology ♦♦ Mild episodes of bleeding can be treated with desmopressin
♦♦ Impaired platelet production which increases vWF level which leads to secondary
♦♦ Accelerated platelet destruction increase in Factor VIII.
116   Mastering the BDS IIIrd Year  (Last 25 Years Solved Questions)

♦♦ For more serious bleeds during dental procedure • Non-Hodgkin’s lymphoma


hemostasis is achieved by Factor VIII concentrate which • Parasitic infection
contain considerable quantities of vWF in addition to • Vitamin C deficiency
factor VIII. • Benign tumor
• Acute leukemia
Hereditary Hemorrhagic Telangectasia • Cellulitis
It is transmitted as autosomal dominant trait and is characterized • Disseminated intravascular coagulation.
by bleeding from mucous membrane. Q.14. Write short note on leukemia.  (Sep 2008, 7 Marks)
Ans. Leukemia is defined as clone of malignant cells derived
Management from myeloid or lymphoid stem cells.
♦♦ In patient having repeated attacks of epistaxis septal Etiology
dermoplasty should be done. In septal dermoplasty,
involved mucosa get removed and skin grafting is done. ♦♦ Genetic factors such as familial, identical twins, congenital
♦♦ If spontaneous hemorrhages are present or nasal bleeding disorders.
♦♦ Environmental factors, i.e. atomic radiation and pollution
is present during dental procedure, it is controlled by
♦♦ Ionizing radiations
giving pressure packs.
♦♦ Retroviruses
♦♦ Sclerosing agents, i.e. sodium tetradecyl sulphate if injected
♦♦ Chemical agents, i.e. alkylating agents, cytotoxic drugs.
intra lesionally stop bleeding.
♦♦ Electrocautery is done. It helps in arresting bleeding. Types of Leukemias
Disseminated Intravascular Coagulation Acute Leukemia
It is a condition that results when the clotting system is activated ♦♦ Acute lymphoblastic leukemia
in all or a major part of vascular system. Despite widespread ♦♦ Acute myeloid leukemia
fibrin production the major clinical problem is bleeding not
thrombosis. Disseminated intravascular coagulation (DIC) Chronic Leukemia
is associated with a number of disorders such as infection, ♦♦ Chronic lymphatic leukemia
obstetric complications, cancer and snakebite. ♦♦ Chronic myeloid leukemia.
Management Clinical Features
♦♦ Correction of hemodynamic instability by fluid therapy, ♦♦ Anemia
transfusion of packed cells or whole blood. ♦♦ Fatigue and lethargy
♦♦ Factor replacement: This is the specific therapy, in this ♦♦ Fever
fresh-frozen plasma, cryoprecipitate, platelet concentrate ♦♦ Bone and joint pain
transfusions are essential. Fresh-frozen plasma is given at ♦♦ Combination of pallor, petechiae or purpura is present.
the dose of 15 mL/kg. Platelet is transfused at the dosage ♦♦ Mucus membrane is bleeding
of 0.1 unit/kg. ♦♦ Hepatomegaly, splenomegaly and renomegaly is present.
♦♦ There is tenderness over other bones.
Q.13. Write differential diagnosis of skin bleed. 
 (Oct 2003, 10 Marks) Investigations
Ans. Differential Diagnosis of skin Bleed
♦♦ Bone marrow picture shows hypercellular reaction is
• Staphylococcal scalded skin syndrome present with premature and primitive cells.
• Bacterial skin infection ♦♦ There is presence of biochemical changes with presence
• Viral skin disease: Measles, rubella of Philadelphia chromosome.
• Dermatitis ♦♦ Peripheral blood film show normocytic normochromic
• Lichen planus picture with an abundance of neutrophil, myelocyte,
• Fungal skin infection metamyelocyte.
• Squamous cell carcinoma ♦♦ White cell counts are elevated
• Urticaria
• Tick bite Treatment
• Spirochetal infection ♦♦ Chemotherapy, bone marrow transplantation or both
• Hypersensitivity and inflammatory diseases should be done.
• Hemoptysis ♦♦ New regimes are derived regularly and are tailored to
• Acne and related disorders specific illness
• Iron deficiency anemia ♦♦ Treatment is given in several phases, with a period of
• Cancer of skin (Melanoma, Kaposi’s sarcoma) induction chemotherapy to induce remission followed by
• Rheumatoid arthritis maintenance and consolidation phases.
Section 1:  General Medicine  117

♦♦ This multiphase treatment is designed for further deplete Management


malignant cells from bone marrow to achieve complete
cure. Chemotherapy
♦♦ Induction phase: Initial high dose chemotherapy in order
Q.15. Briefly describe acute leukemia. (Mar 1996, 7 Marks)
to reduce leukemic cells below the levels of morphogenic
Or detection
Describe the etiology, clinical features and manage-
ment of acute leukemia. (Feb 2013, 15 Marks) Treatment Regimen for Acute Lymphoblastic Leukemia
Ans. Acute leukemia is defined as an uncontrolled growth • Vincristine 1.4 mg/m2 IV weekly for 4 weeks.
of immature hemopoietic cells at the exposure normal • Prednisolone 60 mg/day for 4 weeks.
marrow tissue. • L- Asparaginase 50–200 ku/kg IV for 4 weeks.
• Daunorubicin 30 mg/m2 IV daily for two weeks.
Etiology
Treatment Regimen for Acute Myeloid Leukemia
♦♦ Radiation: Association between radiation-induced genetic
damage to the hemopoietic progenitors and development • Daunorubicin IV alternate days 3 doses.
of myelodysplasia and acute leukemia is seen after nuclear • Cytosine arabinoside IV BD for 10 days
disease. • Thioguanine oral BD for 10 days.
♦♦ Chemical and drugs: Chronic benzene exposure and use of • In this, blood transfusion and platelet transfusion are
cytotoxic and immunosuppressive agents. required.
♦♦ Oncogens and cytogenic abnormalities ♦♦ CNS prophylaxis: Intra – thecal methotrexate is given in
♦♦ Genetic factors, i.e. genetic disorders such as Down’s acute lymphoblastic leukemia
syndrome, Klinefelter’s syndrome, etc. ♦♦ Consolidation: Another dose of chemotherapy to reduce
♦♦ Viruses: Infection by human lymphotrophic virus (HTLV-1). leukemic burden

Clinical Features In Acute Lymphoblastic Leukemia

♦♦ Symptoms due to anemia, i.e. tiredness, weakness and • IV daunorubicin


marked pallor. • IV cytosine arabinoside
♦♦ Hemorrhagic manifestation: Petechiae, bleeding from gums • IV Etoposide
and nose, persistent bleeding after tooth extraction. • Methotrexate, 6 – thioguinine
♦♦ Infection: It causes infective lesions of mouth and throat, i.e. • Dexamethasone
ulceration of mouth and pharynx, herpes simplex infection In Acute Myloid Leukemia
of face and infection of respiratory tract such as bronchitis
and pneumonia. • High dose IV Cytosine arabinoside is given
♦♦ Symptoms of cellular hyperviscosity ♦♦ Maintenance: Low dose chemotherapy for 18 months to
♦♦ There are tissue deposits of leukemic cells causing gum 2 years
hypertrophy which is common in myelomonocytic and In Acute Lymphoblastic Leukemia
monocytic variety of AML.
♦♦ Lymphadenopathy and splenomegaly are common in • Oral Prednisolone
acute lymphoid leukemia. • IV Vincristine
♦♦ Signs of organ infiltration are present, i.e. • Oral 6 – mercaptopurine
• CNS: Meningeal involvement occurs in children with • Oral Methotrexate
acute lymphocytic leukemia. In Acute Myeloid Leukemia
• Skin: Bluish nodules or dusky red patches are present.
• Kidneys: Presence of kidney failure. Post remission therapy is given, i.e. myeloablative therapy
• Other sites: Testes, ovary, liver, gut and serous followed by bone marrow transplantation in relapse or
membranes such as pleura and peritoneum. those with high-risk chromosomal changes.
♦♦ Bone pain are present i.e. tenderness of sternum, osteolytic ♦♦ Bone marrow transplantation
bone lesions and pathologic fractures may occur. In Acute Lymphoblastic Leukemia
♦♦ Constitutional symptoms, i.e. fever, malaise and Allogenic bone marrow transplantation is an option
prostration. in acute lymphocytic leukemia patients entering first
♦♦ Roth’s spot, i.e. presence of white central retinal remission who have an HLA – identical sibling, provided
hemorrhages in acute myeloid leukemia the sibling is fit and is less than 55 years.
118   Mastering the BDS IIIrd Year  (Last 25 Years Solved Questions)

In Acute Myeloid Leukemia Investigations


In adult patient-high dose chemotherapy with autologus ♦♦ Peripheral blood: WBC count is 10–500 × 10 9/L, with
transplantation of hemopoietic stem cells derived from excess of neutrophils, myelocytes and blasts. Basophilia
peripheral blood or bone marrow can be done. and eosinophilia are prominent and thrombocytosis is
♦♦ Supportive care common.
• Hemoglobin level should not allow falling below ♦♦ Bone marrow is hypercellular with marrow fibrosis and
8 g/dL by transfusing 4 units of packed RBCs. This is gaucher like cells. High myeloid to erythroid ratio, i.e. 15
done to avoid anemia. to 20 : 1.
• If bleeding is present, then there is transfusion of ♦♦ Neutrophil alkaline phosphatase is low.
pooled or single donor platelets. ♦♦ Serum vitamin B12 is increased.
• Good nursing, prophylactic gastrointestinal tract ♦♦ Serum uric acid is increased.
decontamination, antibiotics and attention to fluid
Treatment
balance are given to prevent infections.
• If disseminated intravascular coagulation is present, Chronic Phase
then fibrinogen replacement, platelet transfusion twice ♦♦ Tyrosine kinase inhibitor therapy with Imatinib 400 mg
daily and anticoagulants are given. OD as first line.
• Pneumocystis jiroveci pneumonia is a risk during the ♦♦ Other second generation: Tyrosine kinase inhibitor advocated
treatment of acute lymphocytic leukemia maintenance in first line therapy.
therapy. ♦♦ Ponatinib used in case of resistant to first and second
• Hyperuricemia is prevented by adequate hydration generation Tyrosine kinase inhibitor.
and pretreatment with allopurinol which should be ♦♦ Zinatinib is now used in cases of very high counts.
continued till peripheral blood is cleared of blast cells. ♦♦ Allogenic stem cell transplant with HLA-atched-related
Q.16. Write short note on chronic myeloid leukemia.  or unrelated donor, in case of Tyrosine kinase inhibitor
 (Feb 1999, 4 Marks) intolerance
 (Feb/Mar 2005, 5 Marks) (Apr 2010, 4 Marks) ♦♦ Omacetaxine can also be given.
Ans. Chronic myeloid leukemia is a clonal disorder of ♦♦ Alpha-interferon therapy can be given to maintain
pluripotent stem cell. remission in chronic phase of disease in patient less
than 70 years of age. It is given either intramuscularly or
The most important characteristic feature of chronic
subcutaneously in dose of 3 to 9 mega units daily, then
myeloid leukemia is demonstration of Philadelphia
dose is reduced and majority of patients tolerate the dose
chromosome in leukemic blast cells.
of 3 mega units/3 times a week without any side effect.
Clinical Features Advanced Phase Disease
Common Features ♦♦ Treatment of lymphoid blast crisis is done: Treatment similar
♦♦ Nonspecific: Loss of weight, fatigue, malaise, excessive to Acute lymphoblastic leukemia is done.
perspiration. ♦♦ Myeloid crisis: Prognosis is poor and few of drugs used in
♦♦ Splenomegaly: Size of spleen is enlarged treatment of AML offer more than temporary relief.
♦♦ Bleeding, excessive menstrual or other bleeding. Q.17. Write short note on oral manifestation of leukemia. 
♦♦ Anemia  (Mar 1998, 5 Marks)
♦♦ Bone pain: It is due to extension of hemopoiesis through Ans. Oral Manifestations of leukemia
long bones.
Oral Manifestations of Acute Leukemia
Rare Features
♦♦ Bleeding from gingiva is present. Gingiva becomes boggy,
♦♦ Splenic infarction edematous and red in color.
♦♦ Leukostasis ♦♦ Presence of paresthesia of lower lip.
♦♦ Gout ♦♦ Crustation over lips is seen.
♦♦ Retinal hemorrhage ♦♦ Mobility of permanent teeth is present.
♦♦ Fever. ♦♦ Oral mucosa appears pale with ulceration along with
petechiae and ecchymosis.
Etiology
Chronic myeloid leukemia results from translocation of genetic Oral Manifestations of Chronic Leukemia
material between chromosome 9 and 22. ♦♦ Gingival hypertrophy is present. Ulceration of gingiva
The translocation result in production of abnormal tyrosine with necrosis is present.
kinase that makes affected cell immortal. ♦♦ Tongue is dark and is swollen.
Section 1:  General Medicine  119

♦♦ Presence of mobility of teeth is seen. Enumerate five causes of splenomegaly.


♦♦ Necrosis of PDL is seen  (Dec 2009, 5 Marks)
♦♦ Alveolar bone destruction is also present. Ans. Refer to Ans 18 of same chapter.
Q.18. Write short note on splenomegaly.  Q.20. Enumerate the causes of cervical lymphadeno­pathy.
 (Dec 2010, 5 Marks) (Jan 2016, 6 Marks)  (Sep 1998, 5 Marks)
Or Or
Write short answer on splenomegaly.  Enumerate the causes of generalized lymphadeno­
 (Apr 2018, 3 Marks) pathy. (Dec 2010, 5 Marks) (Feb 1999, 4 Marks)
Ans. Splenomegaly is defined as the enlargement of the Or
spleen. Spleen can be mildly, moderately and massively
Enumerate the causes of lymphadenopathy.
enlarged.
 (Jan 2012, 5 Marks)
Ans.
Classification of Splenomegaly as Per its Size
♦♦ Infectious Diseases:
♦♦ Mild splenomegaly: In it spleen is mildly enlarged and • Viral infections:
weighs up to 500 gm. –– Infectious hepatitis
♦♦ Moderate splenomegaly: In it the spleen is moderately –– Infectious mononucleosis
enlarged weighs up to 500 to 1000 gm. –– AIDS
♦♦ Massive splenomegaly: In it the spleen is massively –– Rubella
enlarged and weighs up to greater than 1000 gm. –– Varicella
–– Herpes zoster.
Causes of Splenomegaly • Bacterial infections:
Infections –– Streptococci
–– Staphylococci
♦♦ Bacterial: Septicemia, typhoid, infective endocarditis, TB, –– Salmonella
syphilis. –– Brucella
♦♦ Viral: Hepatitis, infectious mononucleosis. –– Listeria monocytogenes.
♦♦ Protozoan: Malaria and kala-azar • Fungal infections:
♦♦ Parasitic: Hydatid. –– Coccidioidomycosis
–– Histoplasmosis
Circulatory
–– Chlamydial Infections
♦♦ Congestive cardiac failure –– Lymphogranuloma venereum
♦♦ Portal hypertension –– Trachoma.
♦♦ Hepatic or portal vein thrombosis • Mycobacterial infections:
♦♦ Splenic vein obstruction. –– Tuberculosis
–– Leprosy
Hematological –– Parasitic infestations
♦♦ Hemolytic disorders: Hereditary spherocytosis, ellipto- –– Microfilariasis
cytosis, pyruvic kinase deficiency, etc. –– Toxoplasmosis.
• Spirochetal diseases
♦♦ Hematological malignancies: Acute leukemia, chronic
–– Syphilis
myeloid leukemia and lymphomas
–– Yaws
♦♦ Myeloproliferative disorders: Polycythemia vera
–– Leptospirosis.
♦♦ Inflammatory and collagen disorders: Acute rheumatic ♦♦ Immunologic Diseases:
fever, lupus erythematosus • Rheumatoid arthritis
♦♦ Granulomatous disorders: Sarcoidosis and berylliosis • Systemic lupus erythematosus
♦♦ Metabolic storage diseases, i.e. Gaucher’s disease • Dermatomyositis
♦♦ Splenomegaly of unknown etiology: Tropical spleno­megaly, • Serum sickness
non-tropical splenomegaly. • Drug reactions: Phenytoin, hydralazine
Q.19. Describe briefly causes of splenomegaly.  • Primary biliary cirrhosis
 (Sep 1998, 5 Marks) • Chronic active hepatitis.
♦♦ Malignant Disorders:
Or
• Haematologic disorders:
Enumerate the causes of splenomegaly.  –– Hodgkin’s lymphoma
 (Dec 2010, 5 Marks) (Aug 2012, 5 Marks) –– Myeloid leukaemia—blastic crisis
Or –– Chronic lymphatic leukaemia
120   Mastering the BDS IIIrd Year  (Last 25 Years Solved Questions)

• Metastatic tumours: Q.22. Write short note on approach to investigate a case of


–– Melanoma lymphadenopathy. (Sep 2004, 5 Marks)
–– Kaposi’s sarcoma Or
–– Tumours
–– Lung How will you investigate a case of lymphadeno­pathy?
–– Breast (Feb/Mar 2004, 5 Marks)
–– Prostate Or
–– Kidney Write short answer on lymphadenopathy.
–– Head and neck  (May 2018, 3 Marks)
–– Gastrointestinal tract.
Ans. History
♦♦ Endocrine disease:
• Age: Tuberculous lymphadenitis in childhood,
• Hyperthyroidism.
♦♦ Lipid Storage disease: secondary carcinoma in old age.
• Gaucher’s disease • Occupation: Tularemia in hunters and butchers;
• Niemann-Pick disease. sporotrichosis in farmers and gardeners.
♦♦ Miscellaneous disorders: • Duration:
• Sarcoidosis – Acute swelling of a few days duration mostly
• Amyloidosis pyogenic.
• Sinus histiocytosis. – Subacute lymphadenitis of 3–4 weeks duration
may be due to streptococcal infection,
Q.21. Describe the clinical and diagnostic features of chronic
tuberculosis, secondary syphilis, infectious
myeloid leukemia. (Feb 2002, 5 Marks)
mononucleosis or tularemia.
Ans. For clinical features, refer to Ans 16 of same chapter.
– Chronic lymphadenopathies include tuberculosis,
Diagnostic Features lymphomas, leukemia, primary lymphatic
tumours and secondary carcinoma. History
Diagnosis is based on the clinical findings and investigations. of previous radiation treatment or operative
removal.
Investigations
♦♦ Normocytic-normochromic anemia Physical Examination
♦♦ Mean WBC count is 220 × 109/L (range 9.5 to 600 × 109/L) Local
or (2-6 lakhs/µL.) ♦♦ Number of glands: Single gland may appear to be affected
♦♦ Mean platelet count is 445 × 109/L (range 162–2000 × 109/L) for some time in tuberculosis, Hodgkin’s and secondary
♦♦ Leucocyte alkaline phosphatase: Absent in granulocytes carcinoma. Multiple in tuberculosis, Hodgkin’s disease,
in CML leukemia.
♦♦ Plasma uric acid and alkaline phosphatase are increased ♦♦ Site:
♦♦ Serum B12 level is increased due to increase in transcobalamin • Neck usual site for tuberculous lymphadenitis,
III which is present in neutrophil granules. lymphosarcoma, and most other lymphadenopathies.
♦♦ Bone marrow shows increased cellularity especially • Inguinal gland enlargements may be due to syphilis,
myeloid and megakaryocytic. Marrow and blood show lymphogranuloma inguinale or chancroid.
basophilia, eosinophilia and monocytosis. • The infraclavicular glands are seldom so enlarged as to
♦♦ Disease acceleration is denoted by be palpable except in secondary cancer or Hodgkin’s
• Blasts 10–19% in blood or bone marrow disease.
• Basophils > 20% in blood or bone marrow • Supratrochlear (epitrochlear) lymphadenopathy in
non-Hodgkin’s lymphoma, chronic lymphocytic
• Platelets < 100,000/µL unrelated to therapy or = 10,
leukemia, infectious mononucleosis, secondary
00,000/µL. unresponsive to therapy
syphilis, sarcoidosis, iv drug abuse.
• Increasing splenic size
• Hilar and superior mediastinal lymphadenopathy
• Increasing WBC count unresponsive to therapy
in tuberculosis, histoplasmosis, sarcoidosis,
• Cytogenetic clonal evolution pneumoconiosis, malignancy and cryptococcosis.
• Progressive anemia. ♦♦ Character: Discrete in Hodgkin’s disease and leukemia,
♦♦ Blastic crisis is established by: primary tumors of lymphatic tissue and so called
• Blasts > 20 % in bone marrow or peripheral blood “lymphadenoid” form of tuberculous lymphadenitis.
smear Moveable, discrete and painless in sarcoidosis. Matted
• Extramedullary blast formation. together in tuberculosis and lymphogranuloma.
• Large foci or clusters of blasts in bone marrow. ♦♦ Cold abscess: In tuberculosis, lymphogranuloma, tularemia
♦♦ All patients have evidence of translocation by cytogenetics, and sporotrichosis. Tuberculous glands may break through
fluorescent in situ hybridization, or by molecular methods. to give a typically indolent ulcer with undermined edges.
Section 1:  General Medicine  121

♦♦ Primary cause: In the area drained by the enlarged glands; • Hemoglobin count decreases.
e.g. scalp if occipital or posterior auricular glands, fauces • RBC count, packed cell volume, mean corpuscular
and pharynx in upper anterior cervical group, etc. Healed volume and mean corpuscular hemoglobin
scar at portal of entry, scar of operative removal, or of concentration (MCHC). Total leukocyte count and
radiation treatment. differential leukocyte count.
• Peripheral blood film for type of anemia and shape
Systemic of RBCs and presence of any abnormal cells.
♦♦ Skin: • Clotting time and bleeding time in hemolytic anemia.
• Cutaneous tumors mostly on the face, usually in • Blood platelets.
chronic lymphatic leukemia. Sometimes generalized Bone marrow examination is done, when cause of anemia
erythroderma, polymorphic rashes and purpura. requires further investigation, especially to detect type
• Rash of secondary syphilis. of erythropoiesis.
• Painless papules without surrounding erythema may
Other Investigations
be found on face, arms and legs in sarcoidosis.
• Eruption of lupus erythematosus. ♦♦ Stools for parasites: Test for presence of blood and stools is
♦♦ Lungs: Pulmonary or mediastinal tuberculosis, lesions of done in patients suspected to chronic blood loss.
sarcoidosis or metastatic or primary deposits in carcinoma. ♦♦ Urine for albumin, bile salt, pigments and urobilinogen
♦♦ Abdomen: Abdominal glands may be palpable in ♦♦ Gastric analysis: Histamine fast achlorhydria in pernicious
tuberculosis. Enlargement of spleen and liver in leukemia anemia and megaloblastic anemia.
and Hodgkin’s disease. ♦♦ Studies for detecting steatorrhea and malabsorption studies.
♦♦ Genitalia: Scar of primary sore of syphilis, or “chancre” in ♦♦ Schilling test for vitamin B12 absorption in megaloblastic
lymphogranuloma. and dimorphic anemias.
♦♦ Icterus: Jaundice with lymphadenopathy may be met ♦♦ Figlu test is done to assess folic acid deficiency. 15 gram
with in viral hepatitis (cervical glands), lymphoma, acute of histidine hydrochloride is given by mouth and the urine
lymphocytic leukemia, disseminated TB. in which it is excreted it is collected over next eight hours.
♦♦ (f) Temperature: Raised in Hodgkin’s disease, infectious Normal excretion is 1–17 mg
mononucleosis and tularemia. Q.24. Enumerate the causes of iron deficiency anemia. How
will you treat iron deficiency anemia? 
Investigations  (Sep 2005, 10 Marks)
♦♦ Blood picture: For diagnosis of leukemia and infectious Ans. For enumeration of causes of iron deficiency anemia
mononucleosis. Positive ANA and reduced complement refer to Ans 3 of same chapter
C4 levels in SLE.
♦♦ Special tests: Serologic tests for syphilis. Paul-Bunnell Treatment of Iron Deficiency Anemia
or monospot test for infectious mononucleosis, ♦♦ Oral iron therapy: Ferrous sulphate 200 mg TID is given in
Agglutination reaction and animal inoculation in between the meals. If after taking the drug, there is any
tularemia. Autoantibodies in SLE. abdominal pain, nausea, vomiting or constipation, the salt
♦♦ Liver biopsy useful in sarcoidosis and infectious is changed to ferrous gluconate or ferrous fumarate. Oral
mononucleosis. Serological tests for HIV infection. iron therapy should be given for 6 months. Iron absorption
♦♦ Radiography of lungs and gastrointestinal tract. Skeletal is enhanced by combining iron salts with hydrochloric
changes in Boeck’s sarcoid and sporotrichosis (multiple acid, ascorbic acid, succinic acid, fructose, cysteine, isonine
small areas of decalcification). and cobalt. Administration of iron after food minimizes
♦♦ Biopsy: Needle aspiration biopsy is useful for initial gastric upset.
evaluation of superficial lymphadenopathy. It is however not ♦♦ Parenteral iron therapy: Iron sorbitol ciric acid complex 1.5
helpful in diagnosis of lymphomas and other hematologic mg/Kg body weight is given as IM or Iron dextran in 5%
malignancies. Lymph-node biopsy tissue should be glucose is given as IV A small test dose should be given
processed for culture of appropriate organisms, frozen in IV before giving total dose. Total dose of iron should not
liquid nitrogen for lymphocyte typing or special studies for exceed 2.5 g
malignant cell types, and for routine histological studies. ♦♦ Blood transfusion: Packed red cells are transfused.
♦♦ CT scan of abdomen in lymphoma. Q.25. Discuss causes of bleeding. (Feb/Mar 2004, 5 Marks)
♦♦ Lymphangiography of value in diagnosing site, extent, Ans. •  Bleeding can be the result of inability to form a tem­
and, in certain cases, even the nature of primary lymph porary clot or the inability to form a definitive clot.
node enlargement.
• Inability to form a temporary clot results from
Q.23. How will you investigate a case of anemia?  inadequate platelet count, i.e. thrombocytopenia or
 (Feb/Mar 2004, 5 Marks) abnormal platelet function, i.e. thrombocytopathy.
Ans. Every case of anemia should have the following inves­ • Inability to form a definitive clot results from
tigations to detect degree and cause of anemia: abnormalities in clotting factor.
122   Mastering the BDS IIIrd Year  (Last 25 Years Solved Questions)

• Disseminated intravascular coagulation and –– Liver enlarged but not grossly like spleen.
hemophilia results in bleeding. –– Anaemia.
• Idiopathic thrombocytopenic purpura –– Loss of hair and pigmentation of skin.
• Aplastic anemia –– Generalized lymphadenopathy especially in
• Leukemia. children. Nodes are soft, non-tender.
Q.26. Write short note on massive splenomegaly. –– Other features are cough, hemorrhagic features
 (Sep 2005, 5 Marks) –– LD bodies—on stained material from bone
marrow or splenic aspirate.
Ans. A clinically palpable spleen is called splenomegaly and
it may be mild, moderate and massive. • Sub-acute infective endocarditis:
–– Unexplained fever.
When weight of spleen is over 1000 g, it is massive.
–– Presence of cardiac murmur.
Causes –– Presence of petechiae, anemia, peripheral emboli,
clubbing of fingers.
♦♦ Chronic myeloid leukemia –– Red cells in urine.
♦♦ Chronic malaria –– Positive blood culture.
♦♦ Kala-azar • Brucellosis:
♦♦ Myelofibrosis –– History of ingestion of raw milk, or occupation
♦♦ Hairy cell leukemia hazard in veterinary surgeons, laboratory
♦♦ Banti’s disease (tropical splenomegaly) personnel or slaughter house workers.
♦♦ Myeloid metaplasia –– Patient not toxic inspite of high fever.
♦♦ Gaucher’s disease –– Spleen of moderate size, rarely massive.
♦♦ Hepatic vein obstruction. –– Liver may be enlarged, particularly, if spleen is
Q.27. Write short note on differential diagnosis of massive very large.
splenomegaly. –– Back pain common.
 (Sep 2006, 5 Marks) (Mar 2010, 5 Marks) –– Culture of organism from blood or bone marrow.
Complement fixation and anti-human globulin
Ans. When there is enlargement of spleen with its weight
tests in chronic infection.
greater than 1000 g, it is known as splenomegaly.
• Tuberculous splenomegaly: In rare cases, tuberculous
Differential Diagnosis enlargement of spleen occurs with little involvement of
other organs. Blood picture shows anemia, leucopenia
♦♦ Cirrhosis of liver: or thrombocytopenia either single or in combination.
• Symptoms and signs of hepatocellular failure: Spider nevi,
Weakness, lassitude, loss of weight and often pyrexia.
liver palms, alopecia, gynecomastia and testicular
Bleeding may occur. X-ray of spleen may demonstrate
atrophy in males, icterus, Foetor hepaticus. Palpable
areas of calcification.
enlarged liver.
• Evidence of portal hypertension: Ascites, prominent veins Q.28. Write short note on hemophilia A. (Sep 2006, 5 Marks)
on abdomen, hematemesis, piles. Ans. It is the most common congenital disorder of coagulation
• Diagnosis by liver biopsy demonstration of oesophageal caused by reduction of factor VIII.
varices by barium swallow, laparoscopy and • It is a sex-linked disorder with factor VIII gene
scanning. located on X-chromosome.
♦♦ Infections, subacute and chronic: • Women are the carrier and males are affected.
• Chronic malaria:
–– History of fever with rigors with classical features Clinical Features
of the attack like cold stage, hot stage, sweating ♦♦ At birth: Newborn with hemophilia is usually healthy,
stage. though bleeding from cord and cephalohematoma may
–– Spleen very large and firm. occur.
–– Liver may be enlarged.
♦♦ In infant: Hemophilia is asymptomatic till 6 to 12 months,
–– Severe anemia.
when bruising becomes more obvious. Bleeding from the
–– Malarial parasites in peripheral blood or sternal
mouth is common.
marrow.
♦♦ During childhood: In severly affected individuals,
–– Leucopenia.
–– Therapeutic test with adequate dose of anti- spontaneous bleeding may occur in joint and muscles,
malarial drug during fever. including psosas muscle.
• Kala-azar: ♦♦ In adults: Frequency of spontaneous bleeding decreases,
–– Residence in endemic area. but joints may already have been damaged. Intra-cranial
–– Splenomegaly: which may be massive. hemorrhage is a life-threatening complication. Spontaneous
–– Recurrent fever: Double rise of temperature in 24 bleeds are common in mildly affected individuals, but after
hours may be seen. injury bleeding occurs till appropriate therapy has begun.
Section 1:  General Medicine  123

Investigations Laboratory Findings


♦♦ APTT is prolonged in hemophilia A, but PT is normal. Platelet count is usually less than 100, 000 per cumm.
♦♦ Specific factor assay demonstrate reduced factor VIII in
haemophilia A. Treatment

Management ♦♦ If patient are asymptomatic and platelet count is about


40,000 per cumm, treatment is unnecessary.
♦♦ It is done by fresh blood transfusion when bleeding is in ♦♦ For symptomatic patients, treatment regimen include high
the large mass. dose corticosteroid, IV immunoglobulin, splenectomy.
♦♦ Factor VIII concentrate is employed in case of hemophilia A
Q.30. Write short note on thrombocytopenia. 
♦♦ A hemophilic will require treatment before any dental
 (Mar 2008, 5.5 Marks)
procedure. Such patient is managed by IV infusion or
Or
cryoprecipitate or factor VIII combined with aminocaproic
Write short answer on thrombocytopenia. 
acid 4 to 6 g QDS.
 (Apr 2018, 3 Marks) (May 2018, 3 Marks)
♦♦ For any major surgery or periodontal surgeries or extraction
of tooth, hemophilic patient should be hospitalized. Ans. Thrombocytopenia means decreased platelet count, i.e.
Infusion of factor VIII concentrate is given before surgery less than 1,50,000 per cumm.
and is continued for 48 to 72 hours. Antibiotic should also Etiology
be given to the patient.
♦♦ Desmopressin 0.3 µg/kg IV is combined with an anti- ♦♦ Impaired platelet production:
fibrolytic agent which increases Factor VIII levels in • Due to impaired platelet production:
hemophilia A and may ne used to avoid treatment with –– Bone marrow failure:
blood products. - Aplastic anemia
- Leukemia
Complications of Hemophilia A - Megaloblastic anemia
♦♦ Hepatitis in patients who received multiple transfusion of - Myelofibrosis
FFP or cryoprecipitate. - Marrow infiltration.
♦♦ AIDS in a patient not screened for HIV and treated with –– Selective suppression of platelet production
FFP or cryoprecipitate. - Drugs such as sulpha drugs, rifampicin, thi­
♦♦ Anemia from excessive bleeding. azides, etc.
♦♦ Contractures in muscles following intramuscular ♦♦ Increased consumption or destruction of platelets.
hematomas. • Disseminated intravascular coagulation
♦♦ Spontaneous intracranial hemorrhage in severe • Thrombotic thrombocytopenic purpura
hemophilia. • Idiopathic thrombocytopenic purpura
Q.29. Write short note on purpura. (Apr 2007, 5 Marks) • Gram-negative septicemia
Ans. Any rash in which blood cells leak into the skin or mucus • Viral infection.
membrane, usually at multiple sites. Purpuric rashes ♦♦ Increased splenic sequestration.
are often associated with disorders of coagulation or • Hypersplenism
thrombosis. –– Lymphoma
–– Liver diseases.
Pin point purpuric lesions are called as petechiae, large
hemorrhage into the skin are called as ecchymoses. Clinical Features
Types of Purpura ♦♦ As the count of platelets is above 1 lakh per cumm of
blood, patient remain asymptomatic and bleeding time
♦♦ Allergic purpura: Any of a group of purpuras caused by
is also normal.
a variety of agents, including bacteria, drugs and food.
♦♦ As count of platelets is in between 50,000 and 1 lakh per cu
♦♦ Anaphylactic purpura or Henöch-Schönlein purpura: A form of
mm of blood, bleeding is increased. At this stage, bleeding
small vessel vasculitis that affects children more commonly
occur with severe trauma.
than adults, it is marked by abdominal pain, polyarticular
♦♦ As count of platelets reaches below 50,000, bruising is
joint disease and purpuric lesions of the lower extremities.
present along with purpura. At this stage, bleeding occur
♦♦ Idiopathic thrombocytopenic purpura or hemorrhagic purpura:
along with minor trauma.
It is hemorrhagic autoimmune disease in which there
♦♦ Platelet count less than 20,000 per cu mm of blood causes
is destruction of circulating platelets, caused by auto-
spontaneous bleeding.
antibodies that bind with antigen on the platelet membrane.
. 31. Describe drug treatment of AML. (Oct 2007, 5 Marks)
Q
Symptoms Ans. Drug treatment
♦♦ Bleeding from nose, the gums or the gastrointestinal tract. • Induction of remission
♦♦ Physical findings include petechiae, especially on the lower Daunorubicin IV alternate days 3 doses.
extremities and ecchymoses. Cytosine arabinoside IV BD for 10 days
124   Mastering the BDS IIIrd Year  (Last 25 Years Solved Questions)

Thioguanine oral BD for 10 days. • Hypoparathyroidism can cause osteoporosis and


• Consolidation phase: Repeat cycle of drugs used for fractures
inducing remission, consolidation phase ranges upto • Diabetes mellitus due to iron deposits in Islet of
2 years. Langerhans
• Cranial prophylaxis: Not required since cranial
involvement only in few cases. Investigations
• Maintenance phase: Generally maintenance therapy ♦♦ Hematological:
is not required since intensive treatment in • Anemia: It is moderate-to-severe with 10 to 12 g/dL.
remission and consolidation phase sufficient to RBCs are microcytic hypochromic. Target cells are
give relief. If relapse occurs use of ablative therapy present and basophilic stippling is common. Also
supported by allogenic or autologous bone marrow presence of tear drop, elliptical, fragments in red cells
transplantation. and at times red cell with Howell-Jolly body.
Q.32. Write short note on thalassemia. (Nov 2011, 3 Marks) • Reticulocytosis is present.
Ans. •  Thalassemias are genetic disorders of hemoglobin • Leucocytosis with few metamyelocytes and myelocytes
synthesis in which there is reduced production of ♦♦ Biochemical:
one or more chains of hemoglobin. • Reduced serum haptoglobins
• This results in a relative excess production of either • Bilirubin (unconjugated) increased and urine
a chains or b chains, which without their partner urobilinogen increased
chains are unstable and precipitate in RBCs or their • Iron status
precursors. –– Serum iron and ferritin markedly increased.
• The inclusion bodies produced by this process –– Total iron binding capacity (TIBC) is reduced.
increase the rigidity of RBCs and result in their ♦♦ Bone marrow:
destruction, either in the marrow or the circulation or • Erythroid hyperplasia with reversed M : E ratio
both. Hence the anaemia of thalassemia results from • Normoblastic erythropoiesis
ineffective erythropoiesis due to intramedullary RBC • Ineffective erythropoiesis — Some normoblasts die in
destruction, and a shortened RBC survival caused the marrow without maturing into red cells
• Myelopoiesis and Megakaryopoiesis
by haemolysis.
• Increased bone marrow iron
• Thalassemias are classified according to the
♦♦ Other special tests:
particular globin chain that is ineffectively produced.
• HbF levels are high.
• In alpha thalassemia there is reduced rate of chain
• Hb electrophoresis: Bands of both HbA and HbF in
synthesis.
β-thalassemia.
• Beta thalassemias are associated with synthesis of
• Global chain synthesis: α-β globin chain synthesis ratio
beta chains.
altered (normal l : l) due to lack of synthesis of β chains.
• Beta thalassemias is divided into two forms i.e.
• DNA analysis-useful for predicting disease severity
thalassmias minor and thalassemia major
and diagnosis.
Clinical Features • Liver spectrometry for detecting hemosiderosis of
liver.
♦♦ Onset: Affected children fail to thrive from about third
month and become progressively more anaemic. Management
♦♦ Increasing pallor is present
♦♦ Blood transfusions should be done to keep level of
♦♦ Splenomegaly: Haern siderosis with extramedullary
hemoglobin between 9–11 gm%, if infant’s Hb. 6–7 gm%
haematopoiesis
and failure to thrive. Transfusions are given every 2 to 4
♦♦ Facies: Frontal bossing due to thickening of cranial weeks.
bones and prominent cheek bones due to overgrowth of ♦♦ Chelating agent such as desferrioxamine as S.C. infusion
zygomatic bones. using a syringe driver pump/infuser.
♦♦ Mild haemolytic jaundice ♦♦ Splenectomy: Hypersplenism due to splenomegaly causes
♦♦ Increased susceptibility to infections neutropenia and increased need for blood transfusion.
♦♦ Hepatomegaly: Due to extramedullary haemopoiesis Splenectomy reduces severity of neutropenia and
in first 3-4 years. Later on further enlargement due to subsequent infections. Splenectomy should be done as
haemosiderin deposits in Kupffer cells. late as possible.
♦♦ Cardiac involvement: Myocardial haemosiderin may result ♦♦ Bone marrow transplantation: Indication for bone marrow
in arrhythmias and cardiac failure. transplantation in cases where matched siblings are
♦♦ Endocrines: available in a family, if not available, to look for a matched
• Stunted growth from growth hormone deficiency related donor. This is curative for the patient.
• Delayed puberty due to hypothyroidism ♦♦ Folic acid supplements should be given to the patient.
Section 1:  General Medicine  125

Q.33. Write causes of hemolytic anemia.(Feb 2013, 6 Marks) • Disseminated intravascular coagulation (DIC): It is a
Ans. Refer to Ans 6 of same chapter. condition that results when the clotting system is
activated in all or a major part of vascular system.
Q.34. Describe various bleeding disorders and their manage-
Despite widespread fibrin production the major clinical
ment in detail. (Apr 2008, 15 Marks)
problem is bleeding not thrombosis. DIC is associated
Or with a number of disorders such as infection, obstetric
Write notes on bleeding disorders. complications, cancer and snakebite.
 (Aug 2011, 10 Marks) Management of Bleeding Disorders
Ans.
Hereditary Hemorrhagic Telangectasia
Etiology of Bleeding Disorders It is transmitted as autosomal dominant trait and is characterized
♦♦ Vascular Defects by bleeding from mucous membrane.
Bleeding disorders caused by vascular defects may be
Management
caused by structural malformation of vessels. Hereditary
disorders of connective tissue and acquired connective ♦♦ In patient having repeated attacks of epistaxis septal
tissue disorders. Vascular defects rarely cause serious dermoplasty should be done. In septal dermoplasty,
bleeding. Bleeding into skin or mucous membrane starts involved mucosa get removed and skin grafting is done.
immediately alter trauma but ceases within 24 to 48 ♦♦ If spontaneous hemorrhages are present or nasal bleeding
hours. The vascular defects are hereditary hemorrhagic is present, then it is controlled by giving pressure packs.
telangectasia, Henoch-Schönlein purpura. ♦♦ Sclerosing agents,, i.e. sodium tetradecyl sulphate, if
♦♦ Platelet Disorder: injected intralesionally stop bleeding.
♦♦ Electrocautery is done. It helps in arresting bleeding.
It can be of two types:
♦♦ Reduction in number—Thrombocytopenic purpura. If the Idiopathic Thrombocytopenic Purpura (ITP)
total number of circulating platelets falls below 50,000 per
Steroid Treatment Protocol
mm3 of blood the patient can have bleeding. In some cases,
the total platelet count is reduced by unknown mechanism, ♦♦ Initial steroid treatment protocol for ITP: Initial steroid treatment
this is called primary or idiopathic thrombocytopenic protocol l mg/kg/day prednisone, PO for 2-6 weeks.
purpura (ITP). Chemicals, radiation and various systemic ♦♦ Subsequent steroid treatment protocol for ITP: Prednisone
disease (e.g. leukemia) may have direct effect on the bone dose is individualized for every patient. Usually, the dose
marrow and may result in secondary thrombocytopenia. of prednisone is tapered to less than l0 mg per day for
3 months and then withdrawn. Splenectomy is done, if
• Defect in quality: Nonthrombocytopenic purpura, e.g.
discontinuation of prednisone causes a relapse.
von Willebrand’s disease, Bernard-Soulier disease,
♦♦ Follow the ’rule of twos’ for major dental treatment and
Glanzmann’s thrombasthenia von Willebrand’s
provide extra steroids prior to surgery, if the patient is
disease (pseudohemophilia) is the most common currently on steroids or has used steroids for 2 weeks
inherited bleeding disorder. Unlike hemophilia, it can longer within the past 2 years.
occur in females. This is a disease of both coagulation
factors and platelets. It is caused by an inherited Minor Surgery
defect involving platelet adhesion. Platelet adhesion ♦♦ Hemostasis after minor surgery is usually adequate, if
is affected because of a deficiency of von Willebrand`s platelet levels are above 50,000 cells/mm3.
factor. ♦♦ Platelets can be replaced or supplemented by platelet
• Various drugs such as carbamazepine, aspirin, methyl transfusions; though sequestration of platelets occurs
dopa, phenytoin can also lead to platelet disorders. rapidly. Platelet transfusion is indicated for established
♦♦ Coagulation Defects thrombocytopenic bleeding.
• Hemophilia A: It is the most common coagulation ♦♦ When given prophylactically platelets should be given
defect. It is inherited as X-linked recessive trait. The half before surgery to control capillary bleeding and half
hemostatic abnormality in hemophilla A is caused by at the end of the operation to facilitate the placement of
adequate sutures.
a deficiency/defect of factor VIII. Until recently, factor
♦♦ Platelets should be used within 6–24 hours after collection
VIII was thought to be produced by endothelial cells
and suitable preparations include platelet rich plasma (PRP),
and not by the liver as most of coagulation factors.
which contains about 90% of the platelets from a unit of fresh
The defective gene is located on the X-chromosome. blood and platelet-rich concentrate (PRC), which contains
• Hemophilia B (Christmas disease): Factor IX is deficient about 50% of the platelets from a unit of fresh whole blood.
or defective. It is inherited as X-linked recessive trait. ♦♦ PRC is thus the best source of platelets. Platelet infusions
Like Hemophilia A, the disease primarily affects males carry the risk of isoimmunization, infection with blood-
and the clinical manifestations of the two are identical. borne viruses and, rarely, graft-versus-host disease.
126   Mastering the BDS IIIrd Year  (Last 25 Years Solved Questions)

♦♦ Where there is immune destruction of platelets (e.g. in ♦♦ In hemophilia B human dried factor IX concentrate is
ITP), platelet infusions are less effective. supplied as power which is to be mixed with distill water
♦♦ The need for platelet transfusions can be reduced by and administer IV.
local hemostatic measures and the use of Desmopressin
von Willebrand Disease
or tranexamic acid or topical administration of platelet
concentrates. ♦♦ It is the most common inherited bleeding disorder. It is
♦♦ Absorbable hemostatic agents such as oxidized regenerated inherited as autosomal dominant but a severe form of
cellulose (Surgical), synthetic collagen (lnstat) or disease may be inherited as a sex-linked recessive trait.
microcrystalline collagen (Avitene) may be put in the ♦♦ It is caused due to the deficiency or defect in Von
socket to assist clotting in postextraction socket. Willebrand factor.
♦♦ Drugs that affect platelet function, such as gentamicin, ♦♦ Types of Von-Willebrand diseases are: Type I, Type II A
antihistamines and aspirin should be avoided. and II B, Type III

Major Surgery Management


♦♦ Surgical procedures can be performed in patients with
For major surgery platelet levels over 75,000 cells/mm3 are
mild von Willebrand disease by using DDAVP and EACA.
desirable.
Patients with severe Von Willebrand disease requires
Hemophilia cryoprecipitate and Factor VIII concentrate.
♦♦ Bleeding should be controlled by using local measures
It is a hereditary disorder of blood coagulation characterized such as pressure packs, gelfoam with thrombin, tranexamic
by excessive hemorrhage due to increased coagulation time. It acid, etc.
is of two types, i.e. hemophilia A and hemophilia B. ♦♦ Aspirin and NSAIDs are avoided and acetaminophen can
be given to patients.
Management
♦♦ In majority of patients with von Willebrand disease
♦♦ Local anesthesia is contraindicated. So intrapulpal hemostatic defect is controlled with desmopressin via
anesthesia, intraligamentary anesthesia should be used. nasal spray.
Sedation with diazepam or NO2-O2 sedation can be given. ♦♦ Type I von Willebrand disease is treated with desmopressin
♦♦ Endodontic procedures should be carried out and care while Type II A and B and Type III require clotting factor
is taken not to do instrumentation beyond apex. If replacement.
hemorrhage is present, it should be controlled by 1:1000 Disseminated Intravascular Coagulation
aqueous epinephrine on paper point.
♦♦ Restorative treatments can be carried out by proper Management
application of rubber dam for avoiding trauma to gingiva ♦♦ Correction of hemodynamic instability by fluid therapy,
and other soft tissues. In case if rubber dam is not present transfusion of packed cells or whole blood.
an epinephrine impregnated hemostatic cord is kept ♦♦ Factor replacement: This is the specific therapy, in this
in gingival sulcus before preparation of crown or inlay fresh frozen plasma, cryoprecipitate, platelet concentrate
margin. transfusions are essential. Fresh-frozen plasma is given at
♦♦ Complete dentures and removable partial dentures can the dose of 15 mL/kg. Platelet is transfused at the dosage
be given to hemophilic patients and are well tolerated by of 0.1 unit/kg.
them. Patient has to take care for proper maintenance of Q.35. Discuss causes, diagnosis and management of iron
hygiene of prosthesis. deficiency anemia. (May/June 2009, 15 Marks)
♦♦ Conservative periodontal treatment should be done rather Ans. For causes and management refer to Ans 2 of same
than attempting for periodontal surgeries. chapter.
♦♦ In case, if oral surgical procedures are to be done local
Diagnosis
hemostatic agents should be used, pressure surgical packs
should be employed, sutures, topical thrombin is used. It is based on the clinical signs and symptoms and investigations
After removal of tooth, socket is packed with mechanical
Clinical Signs and Symptoms
splint. Post-operative use of anti-fibrinolytic agent is used
to support clot maintenance. Symptoms
♦♦ In cases of Hemophilia A Human freeze-dried factor VIII ♦♦ Tiredness
concentrate or new recombinant factor VIII is used. ♦♦ Weakness
Section 1:  General Medicine  127

♦♦ Lethargy Contd…
♦♦ Loss of appetite
Hematological Oral manifestation
♦♦ headache and bodyache disorder
♦♦ Inability to concentrate Agranulocytosis • Gangrenous ulceration of gingiva, buccal
♦♦ Giddiness mucosa, soft palate and lip.
♦♦ Breathlessness • Wound healing is delayed
♦♦ Epigastric discomfort. Polycythemia • Hematoma formation and ulceration of
mucosa, gingiva and tongue.
Signs
• Petechiae and ecchymosis
♦♦ Pallor • Spontaneous gingival bleeding
♦♦ Palpitation • Purplish-red discoloration of tongue, cheek
♦♦ Angular stomatitis and lips
♦♦ Atrophic gastritis Iron deficiency • Mucosal atrophy
♦♦ Flattening or spoon-shaped nails, i.e. platonychia and anemia • Pallor
koilonychia • Bald tongue
♦♦ Tongue is pale and smooth • Atrophic glossitis
• Angular cheilitis
♦♦ Glossitis
• Glossodynia
♦♦ Hepatosplenomegaly
Pernicious anemia • Erythematous oral mucosa with burning
♦♦ Plummer-Vinson syndrome, i.e. dysphagia and cricoid
sensation.
webs. • Beefy red tongue with depapillation or
Hunter’s glossitis
Investigations
• Focal areas of atypical mucosal erythema
♦♦ Blood picture and red cell indices • Xerostomia is present
• Hemoglobin: Fall on hemoglobin concentration. Folic acid • Depapillation of tongue with glossitis
• Red cells: The RBCs in blood film are hypochromatic deficiency anemia • Glossodynia
and microcytic and there is anisocytosis, poikilocytosis • Aphthous-like ulceration
and elliptocytosis Aplastic anemia • Presence of purpura
• Reticulocyte count: Normal or reduced. • Spontaneous gingival bleeding
• Absolute values: MCV, MCH and MCHC are decreased. • Gingival hyperplasia
• Leukocytes: Usually normal • Ulceration
• Halitosis
• Platelets: Usually normal but raised if bleeding is cause
• Severe mucosal pallor
of anemia
Thalassemia • Protrusion of upper teeth.
• ESR: value of esr is low
• Maxillary teeth have spacing in between.
♦♦ Bone marrow findings:
• Prominent premaxilla and cheek bones
• Marrow cellularity: The marrow cellularity is increased • Mongoloid faces
due to erythroid hyperplasia. Sickle cell anemia • Mucosal pallor
• Erythropoiesis: Normoblastic erythropoiesis. • Pain in mandible
• Marrow iron is deficient. • Lip paresthesia
♦♦ Biochemical findings: • Delayed eruption of teeth
• Serum iron level is low. Hemophilia • Petechaie
• Total iron binding capacity is high. • Prolonged bleeding after tooth extraction
• Serum ferritin is very low. • Excessive bleeding on physiologic eruption
and exfoliation of tooth
Q.36. Write notes on oral manifestations of hematological
Erythroblastosis • Black, brown and bluish pigmentations of
disorder. (Aug 2011, 10 Marks)
Foetalis teeth
Ans. Following are the oral manifestations of hematological • Protrusion of upper teeth
disorders: • Enamel hypoplasia
Hematological Oral manifestation
Q.37. Classify anemia and discuss nutritional anemias. 
disorder
 (Aug 2012, 12 Marks)
Leukemias • Enlargement, bleeding and necrosis of Ans. Classification of Anemia
gingiva
• Necrosis and ulceration of oral mucosa Etiological Classification of Anemia
• Ecchymosis (By Lea and Febiger, 1981)
• Profuse bleeding on trauma or following
♦♦ Loss of blood:
extraction of tooth
• Acute posthemorrhagic anemia
Contd… • Chronic posthemorrhagic anemia.
128   Mastering the BDS IIIrd Year  (Last 25 Years Solved Questions)

♦♦ Excessive destruction of red blood corpuscles: • Chronic renal failure.


• Extracorpuscular causes: • Chronic inflammatory disease:
–– Antibodies –– Infectious
–– Infections like malaria –– Non-infectious including granulomatous and
–– Splenic sequestration and destruction collagen disease.
• Cirrhosis of liver.
–– Associated diseases like lymphomas
–– Drugs, chemical and physical agents Morphological Classification of Anemia
–– Trauma to RBC.
Based on the red cell size, hemoglobin content and red cell
• Intracorpuscular hemolytic diseases: indices, anemias are classified into 3 types:
–– Hereditary 1. Microcytic, hypochromic: MCV, MCH, MCHC are all
- Disorders of glycolysis reduced, e.g. in iron deficiency anemia and in certain non-
- Faulty synthesis or maintenance of reduced iron deficient anemias (sideroblastic anemia, thalassaemia,
glutathione anemia of chronic disorders).
- Qualitative or quantitative abnormalities in the 2. Normocytic, normochromic: MCV MCH, MCHC are all
synthesis of globulin normal, e.g. after acute blood loss, hemolytic anemias,
- Abnormalities in RBC membrane bone marrow failure, anemia of chronic disorders.
- Erythropoietic porphyria. 3. Macrocytic: MCV is raised, e.g. in megaloblastic anemia
due to deficiency of vitamin B12 or folic acid.
• Acquired
–– Paroxysmal nocturnal hemoglobinuria Nutritional Anemias
–– Lead poisoning Following are the nutritional anemias:
♦♦ Impaired blood production resulting from deficiency of ♦♦ Iron deficiency anemia
substances essential for erythropoiesis: ♦♦ Macrocytic anemia.
• Iron deficiency For iron deficiency anemia in detail refer to Ans 2 of same
• Deficiency of various B vitamins: Vitamin B12 and folic chapter.
acid (pernicious anemia and megaloblastic anemia);
pyridoxine responsive anemia Macrocytic Anemia
• Protein deficiency Macrocytosis is the rise in mean cell volume or red cells above
• Possibly ascorbic acid deficiency. the normal range. It is due to vitamin B12 deficiency or folic
♦♦ Inadequate production of mature erythrocytes: acid deficiency.
• Deficiency of erythroblast
Etiology
–– Atrophy of bone marrow: Aplastic anemia
- Chemical or physical agents ♦♦ Vitamin B12 deficiency
- Hereditary • Inadequate intake:
–– In strict vegetarians
- Idiopathic.
–– In poor diet.
–– Isolated erythroblastopenia • Due to malabsorption:
- Thymoma –– Gastric:
- Chemical agents - Pernicious anemia
- Antibodies. - Congenital intrinsic factor deficiency
• Infiltration of bone marrow: - Gastrectomy.
–– Leukemia, lymphomas –– Small intestinal disease:
–– Multiple myeloma - Topical and Non-topical sprue
–– Carcinoma, Sarcoma - Crohn’s disease
- Fish tapeworm.
–– Myelofibrosis.
–– Increased requirement
• Endocrine abnormalities:
- In pregnancy.
–– Myxedema ♦♦ Folic acid deficiency
–– Addison’s disease • Inadequate intake
–– Pituitary insufficiency –– Infancy
–– Sometimes hyperthyroidism. –– Old age
Section 1:  General Medicine  129

–– Poverty Treatment
–– Alcoholism
–– Kwashiorkor. ♦♦ Vitamin B12 Deficiency:
• Malabsorption • Hydroxycobalamine 1000 µg IM is given, i.e. 6
injections in 2 to 3 weeks.
–– Coeliac Disease
• Since rapid regeneration of blood deplete marrow iron
–– Topical sprue
stores, so ferrous sulphate 200 mg daily is given after
–– Congenital folate malabsorption.
starting the therapy.
• Increased utilization or loss
• Maintenance dose of 500–1000 μg IM is given for
–– Physiological
every 3 months.
- Prematurity ♦♦ Folic acid deficiency:
- Pregnancy and lactation. • Initially Folic acid 5 mg daily orally for 4 months is given.
–– Pathological • Maintenance dose is 5 mg folic acid once a week.
- Blood disorders
- Malignancy Q.38. Write Hodgkin’s lymphoma under following head-
- Dialysis. ings: (Mar 2013, 2 Marks each)
–– Anti-folate Drugs a. Clinical Features
- Methotrexate b. Investigations
- Pyrimethamine c. Staging
d. Treatment
- Trimethoprim
Ans. Hodgkin’s disease is a clinically and histologically
- Anti-convulsant drugs.
distinct chronic lymphoproliferative disorder of
unknown etiology.
Clinical Features
Clinical Features
♦♦ Due to anemia: Shortness of breadth, anemia and
pallor. Local Signs
♦♦ Gastrointestinal: Diarrhea, loss of weight and apetite ♦♦ Lymphadenopathy: Superficial lymph nodes in neck are first
♦♦ Neurological: Vitamin B12 neuropathy and neural tube to enlarge at first one side and then other. On palpation,
defects due to deficiency of folic acid. lymph nodes are painless, leathery and discrete. In
♦♦ Gonadal dysfunction: It is due to deficiency of both advanced cases there is a pyramidal swelling with base at
Vitamin B12 and folic acid clavicle and apex at angle of the jaw.
♦♦ Epithelial cell changes: Glossitis and other epithelial ♦♦ Splenomegaly: Moderate enlargement is present.
surfaces show cellular abnormalities. ♦♦ Hepatomegaly: Moderate and non-tender

Investigations Systemic Symptoms

♦♦ Vitamin B12 Deficiency: ♦♦ Presence of cachexia and loss of weight


• Hemoglobin levels are decreased below normal range. ♦♦ Fever is present. It can be of mild grade or undulant for
several days or can be continuous.
• Mean corpuscular volume is raised.
♦♦ Night sweats are present
• Peripheral blood film examination reveals macrocy­
♦♦ Anemia can be present
tosis, poikilocytosis and hypersegmentation of the ♦♦ Generalized or refractory pruritus
neutrophils
• Bone marrow examination reveals hypercellular Features Due to Metastatic Growth
marrow with megaloblastosis, giant metamyelocytes
and platelets ♦♦ In skin: pruritus, erythema, herpes zoster
♦♦ In bones: localized pain and tenderness
• Serum iron and serum ferritin levels are raised.
♦♦ Nervous system: Paresthesia and pain
• Schilling’s test is positive.
♦♦ Respiratory tract: Paralysis of larynx, Collapse of lung,
♦♦ Folic acid deficiency:
pleural effusion
• Serum folate levels are low ♦♦ Gastrointestinal: Jaundice and ascites
• Red cell folate levels are low ♦♦ Genitourinary: Hematuria, retention of urine and pain in
• Figlu test is positive. back.
130   Mastering the BDS IIIrd Year  (Last 25 Years Solved Questions)

Due to Immunologic Changes ♦♦ Above the diaphragm, Y field therapy is given.


♦♦ Below the diaphragm, mantle or inverted Y field therapy
♦♦ Lowering of resistance to infection
♦♦ Homolytic anemia is given
♦♦ Chemotherapy should also be given.
Investigations ♦♦ Involved field radiation therapy is given to all sites of bulky
disease post-ABVD
♦♦ Blood test:
• Non-specific anemia of chronic disease is common Stage III and IV
• Lymphopenia is present.
♦♦ Mainly chemotherapy is given
• ESR is elevated
♦♦ Drugs used are MOPP combination, i.e.
♦♦ Lymph node biopsy:
• Mustine—6 mg/m2 IV daily on days 1 and 8
• Show presence of reed-sternberg cells. • Oncovine—1.4 mg/m2 IV daily on days 1 and 8
♦♦ Imaging: • Procarbazine—100 mg/m2 orally on days 1–14
• Chest X-ray to look for mediastinal lymph nodes and • Prednisolone—40 mg/m2 orally on days 1–14
pleural effusion. ♦♦ Give 6 courses with 2 weeks rest between the end of one
• CT is valuable in detecting intra-thoracic and course and beginning of next.
abdominal lymphadenopathy. It can also detect ♦♦ ABVD combination is popular these days, i.e.
presence or absence of bone marrow involvement • Adriamycin: 25 mg/m2 IV on days 1 and 15
after chemotherapy. • Bleomycin: 10 mg/m2 IV on days 1 and 15
• Bone marrow: Aspiration biopsy is indicated in stage • Vinblastine: 6 g/m2 IV on days 1 and 15
II disease and higher staging. • Dacarbazine: 150 mg/m2 I.V. on days 1 to 5
Staging • Alternating MOPP with ABVD gives excellent response.
Q.39. Describe mechanism of coagulation. Write hemophilia
Ann Arbor Staging Classification of Hodgkin’s Disease
under following headings:
Stage I I Involvement of a single lymph node region a. Definition
(A or B) IE Involvement of a single extralymphatic organ or site b. Clinical features
Stage II II Involvement of two or more lymph node regions on c. Treatment (Nov 2014, 3 + 1 + 2 +3 Marks)
(A or B) IIE the same side of the diagphragm
Ans. Coagulation is the spontaneous arrest of the bleeding.
(or) with localised contiguous involvement of an
extranodal organ of site Following are the factors which are involved in the
Stage III III Involvement of lymph node regions on both sides mechanism of coagulation of blood:
(A or B) IIIE of the diaphragm Factor I—Fibrinogen
IIIS (or) with localised contiguous involvement of an Factor II—Prothrombin
IIIES extranodal organ or site Factor III—Thromboplastin (Tissue factor)
(or) with involvement of spleen Factor IV—Calcium ions
(or) both features of IIIE and IIIS
Factor V—Labile factor
Stage IV IV Multiple or disseminated involvement of one Factor VI—Presence not approved
or
Factor VII—Stable factor
(A or B) More extralymphatic organs of tissues with
Factor VIII—Anti-hemophilic factor
or without lymphatic involvement
Factor IX—Christmas factor
A = asymptomatic; B = presence of constitutional symptoms;  E = extranodal
involvement; S = splenomegaly Factor X—Stuart-Prower factor
Factor XI—Plasma thromboplastin antecedent
Treatment Factor XII—Hegman factor
Factor XIII—Fibrin-stabilizing factor
Stage I and II Factor XIV—Prekallikrein
♦♦ Mainly radiotherapy is given, i.e. external high cobalt Factor XV—Kallikrein
radiotherapy. Factor XVI—Platelet factor
Section 1:  General Medicine  131

Coagulation Occurs in Three Stages

Fig. 16:  Coagulation of mechanism

Mechanism of coagulation Intrinsic Pathway


Blood coagulation occurs in three major stages:
Intrinsic mechanism of prothrombin activation occurs in four
♦♦ Stage 1: Activation of Stuart-Prower factor (formation of
steps:
prothrombin activator)
♦♦ Stage 2: Formation of thrombin from prothrombin
♦♦ Stage 3: Formation of fibrin from fibrinogen Step 1 (Activation of factor XII)

Activation of Stuart-Prower Factor (Factor X) Activation of intrinsic pathway starts with contact of Hageman
factor with a negatively charged surface or exposed collagen of
Activation of Stuart-Prower factor or factor X is the key to
blood coagulation. Factor Xa is called prothrombin activator the injured vessel wall.
as it activates prothrombin to form thrombin. Therefore, this 1. High molecular weight kininogen and kallikerin act as
process is also called prothrombin activation. cofactors to facilitate the activation of factor XII
This is achieved by two pathways: the intrinsic pathway and 2. Exposed collagen stimulates platelet adhesion and
the extrinsic pathway. aggregation before initiating blood coagulation.
132   Mastering the BDS IIIrd Year  (Last 25 Years Solved Questions)

Step 2 (Activation of factor XI) 1. Proteolysis of soluble fibrinogen


2. Polymerization of fibrin monomers
Activated factor XII (XIIa) converts factor XI to its active form
3. Stabilization of fibrin polymer
(XIa). This step is accelerated in the presence of high molecular
weight kininogen. Proteolysis of Soluble Fibrinogen
Step 3 (Activation of factor IX ) Fibrinogen has three domains: two peripheral (D) domains and
one central (E) domain.
Factor XIa then converts factor IX to IXa, which is accentuated
1. Thrombin binds with central domain and proteolytically
by factor VIIa. Calcium accelerates this process.
releases two fibrinopeptides A and B from aminoterminals
Step 4 (Activation of factor X) of Aα and Bβ chains of each fibrinogen molecules.
2. Release of fibrinopeptides leads to the formation of fibrin
Final step in activation of prothrombin activator is activation monomer.
of factor X. Factor IXa causes activation of factor X to Xa, the
activated Stuart prower factor. The membrane phospolipid, Mechanism of fibrin stabilization
calcium and activated factor VIII act as cofactors for the
activation of Stuart – prower factor is crucial in Stage 1.
Formation of factor VIIIa is the key to the process of activation
of factor X

Extrinsic Pathway
Extrinsic pathway of blood coagulation occurs in three steps:

Step 1 (Release of tissue thromboplastin)


Key to the clotting mechanism is the release of tissue
thromboplastin from the injured tissue. As tissue thromboplastin
is the tissue factor which is viewed as extrinsic to circulating
blood, this system of blood coagulation is called extrinsic system
of clotting.
Polymerization of Fibrin Monomers
Step 2 (Activation of VII)
Fibrin monomers join to form protofibrils.
Tissue thromboplastin converts factor VII to its active form 1 About 15 to 20 protofibrils aggregate to form thick fibers
(VIIa). This is the key step in extrinsic mechanism. Factor VIIa of fibrin.
directly activates not only factor X, but also factor IX. Thus it 2. Protofibrils also branch out to form into a meshwork
also influences intrinsic mechanism of activation of factor X. of interconnected thick fibrin fibers. This is called
polymerization of fibrin monomers.
Step 3 (Activation of X) 3. Thrombin initiates the process of polymerization and
Factor VIIa converts X to Xa. This process is accelerated in simultaneously activates factor XIII.
the presence of calcium, platelet phospholipid and tissue 4. Factor XIIIa completes the process of polymerization.
thromboplastin. Stabilization of Fibrin Polymer
Formation of Thrombin from Prothrombin Fibrin stabilizing factor (XIIIa) stabilizes the fibrin polymers by
cross – linking them.
This is the second stage of blood coagulation in which activated
1. Factor XIII is converted to XIIIa by thrombin.
Stuart-Prower factor (Xa) converts prothrombin to thrombin,
2. Calcium acts as a cofactor for this conversion.
in the presence of platelet phospolipid, calcium and activated
3. Covalent cross-linking of fibrin polymers provides
factor V (Va). Factor Va acts as cofactor for acceleration of this
adequate strength to the fibrin thread and to the fibrin
process. Therefore, it is also called proaccelerin or accelerator
meshwork.
globulin.
4. The stabilized fibrin meshwork is the blood clot.
Formation of Fibrin from Fibrinogen 5. Red cells and platelets are trapped inside the fibrin
meshwork to give the volume to the clot.
This is the final stage of blood coagulation in which thrombin
acts as an enzyme to convert fibrinogen to fibrin. In this process, Definition of Hemophilia
first, the fibrin monomers are formed and afterward, they are Hemophilia is an inherited disorder which affects males and
polymerized to fibrin thread (blood clot). This occurs in three is transmitted by females. It is an X-linked recessive disorder
steps: caused by deficiency of factor VIII.
Section 1:  General Medicine  133

Clinical Features Sources


For clinical features, refer to Ans 11 of same chapter. Sources of folic acid are yeast, fresh green vegetables, cereals,
liver, kidney, meat.
Treatment
Daily Requirement
For treatment refer to Ans 11 of same chapter.
Q.40. Write short note on agranulocytosis. 50 to 100 μg.
 (Nov 2014, 3 Marks) Deficiency of Folic Acid
Ans. It is defined as an acute disease marked by a deficit or
absolute lack of granulocytic WBCs, i.e. neutrophils, Deficiency of folic acid leads to megaloblastic anemia, glossitis,
basophils and eosinophils. diarrhea, weight loss and weakness.
• For etiology or causes refer to Ans 7 of same chapter.
Indications
• For Clinical manifestations and management refer
to Ans 8 of same chapter. ♦♦ In megaloblastic anemia caused due to folate deficiency,
• For diagnosis refer to Ans 9 of same chapter. folic acid is given in dose of 1–5 mg/day and is continued
till 3–4 months.
Q.41. Write clinical features of acute leukemia. ♦♦ In pregnancy due to increase demand, it is given 0.5 mg/
 (Feb 2015, 12 Marks) day from first trimester.
Ans. Following are the clinical features of acute leukemia: ♦♦ Folic acid antagonizes methotrexate toxicity.
I. Due to bone marrow failure Q.43. Write down common coagulation disorders. Write
• Constitutional symptoms: Fever, malaise and prostration down the clinical features, investigations and manage-
• Due to anemia: Tiredness, fatigue, pallor, weakness ment of idiopathic thrombocytopenic purpura.
and dyspnea.  (June 2015, 12 Marks)
• Bleeding manifestations: Bruises, Petechiae, bleeding Ans. Coagulation disorders deal with disruption of the body’s
from gums and nose, purpura, gastrointestinal, renal
ability to control blood clotting.
and bleeding in nervous system.
Common coagulation disorders are:
• Infections: Infective lesions of oral cavity and throat,
ulcers in oral cavity and pharynx, herpes simplex • Hemophilia, or hemophilia A (Factor VIII deficiency),
infection, Infections of respiratory tract such as an inherited coagulation disorder. This genetic
bronchitis and pneumonia, infections of skin such as disorder is carried by females but most often affects
cellulitis and boils. males.
II. Due to organ infiltration • Christmas disease, also known as hemophilia B or
• CNS: Hemorrhage, meningeal infiltration and multiple Factor IX deficiency, is less common than hemophilia
cranial nerve palsies. A with similar in symptoms.
• Skin: Bluish nodules or dusky red patches. • Disseminated intravascular coagulation disorder,
• Kidneys: Renal failure also known as consumption coagulopathy, occurs
• Heart: Cardiomyopathy and pericarditis as a result of other diseases and conditions. This
• Fundus: Roth spots and papilledema disease accelerates clotting, which can actually cause
• Testes: Swelling present in acute lymphoid hemorrhage.
leukemia • Thrombocytopenia is the most common cause of
• Bony tenderness is present especially in sternum coagulation disorder. It is characterized by a lack of
• Presence of hepatosplenomegaly and lympha- circulating platelets in the blood. This disease also
denopathy. includes idiopathic thrombocytopenia.
III. Due to leukemic cells • Von Willebrand’s disease is a hereditary disorder
• Tissue deposits of leukemic cells leads to gum with prolonged bleeding time due to a clotting factor
hypertrophy. deficiency and impaired platelet function. It is the
• Symptoms of cellular hyperviscosity are headache, most common hereditary coagulation disorder.
confusion, fits, focal neurological signs and coma. • Hypoprothrombinemia is a congenital deficiency of
• In promyelocytic variant of acute myeloid leukemia clotting factors that can lead to hemorrhage.
the release of cytoplasmic granular contents activates • Other coagulation disorders include factor XI
coagulation and fibrinolytic systems which leads to deficiency, also known as hemophilia C, and
acute hemostatic failure. factor VII deficiency. Hemophilia C afflicts one in
100,000 people and is the second most common
Q.42. Write short note on folic acid. (Apr 2015, 2 Marks) bleeding disorder among women. Factor VII is also
Ans. Folic acid is a combination of glutamic acid, para- called serum prothrombin conversion accelerator
aminobenzoic acid and pteridine nucleus. (SPCA) deficiency. One in 500,000 people may be
134   Mastering the BDS IIIrd Year  (Last 25 Years Solved Questions)

afflicted with this disorder that is often diagnosed Etiology


in newborns because of bleeding into the brain as a
For etiology of iron deficiency anemia, refer to Ans 2 of same
result of traumatic delivery.
chapter.
Idiopathic Thrombocytopenic Purpura For etiology of macrocytic anemia, refer to Ans 37 of same
chapter.
It is considered to be the autoimmune disease of platelets.
For etiology of folic acid deficiency anemia, refer to Ans 37 of
Clinical Features same chapter.
Acute idiopathic thrombocytopenic purpura For etiology of hemolytic anemia refer to etiological classification
♦♦ It is seen commonly in children who are recovering from in Ans 5 of same chapter.
the viral disease.
Clinical Features
♦♦ Onset of the disorder is sudden and symptoms are seen
2 to 3 weeks after the viral infection with appearance of For clinical features of iron deficiency anemia, refer to Ans 2
purpura or at times epistaxis. of same chapter.
Chronic idiopathic thrombocytopenic purpura For clinical features of macrocytic anemia, refer to Ans 37 of
♦♦ This is seen during 2nd to 4th decades of life. same chapter.
♦♦ It has female predilection. For clinical features of folic acid deficiency anemia, refer to Ans
♦♦ Patient has purpura or epistaxis. 37 of same chapter.
♦♦ There is presence of ecchymoses and menorrhagia For clinical features of hemolytic anemia, refer to Ans 5 of same
♦♦ Internal bleeding can also be present. chapter.
♦♦ Splenomegaly can be present.
Approach to Diagnosis
Investigations
For diagnosis of iron deficiency anemia, refer to Ans 35 of same
♦♦ Bleeding time becomes prolonged. chapter.
♦♦ Platelet count get decreased For diagnosis of macrocytic anemia, refer to Ans 37 of same
♦♦ Bone marrow examination reveals increase in the number chapter.
of megakaryocytes indicating peripheral destruction of
For diagnosis of folic acid deficiency anemia, refer to Ans 37
platelets.
of same chapter.
Management For diagnosis of hemolytic anemia, refer to Ans 5 of same
In children chapter.
♦♦ Mild idiopathic thrombocytopenic purpura requires no Treatment
treatment.
♦♦ In moderate-to-severe idiopathic thrombocytopenic For treatment of iron deficiency anemia refer to Ans 2 of same
purpura prednisolone 2 mg/kg daily oral administration chapter.
is advised. For treatment of macrocytic anemia refer to Ans 37 of same
♦♦ Platelet transfusion can be given. chapter.
♦♦ After giving steroidal therapy, if bleeding persist For treatment of folic acid deficiency anemia refer to Ans 37 of
immunoglobulins should be given. same chapter.
In adults Q.45. Describe iron deficiency anemia under following head-
♦♦ In adults prednisolone should be given 1 mg/kg as ings: (Mar 2016, 2+2+2 Marks)
standard regimen and is continued for 2 to 4 weeks, after
a. Iron metabolism
this drug should be slowly withdrawn.
♦♦ If bleeding is high platelet infusion should be done. b. Investigations
♦♦ IV immunoglobulin should be given, i.e. 1 g/kg. This is c. Treatment
to be given in the patients who are not responding to Ans. Iron metabolism: Average diet consists of 10–15 mg of
prednisolone therapy. iron out of which 5–10% is normally absorbed. Iron
♦♦ If patient has more than two remissions of the disease required for hemoglobin synthesis is derived from two
splenectomy should be done. primary sources, i.e. ingestion of food containing iron
and recycling of iron from senescent red cells. Iron is
Q.44. Describe the etiology, classification, clinical features, absorbed from duodenum and upper jejunum in ferrous
approach to diagnosis and treatment of anemias. form (Fe2+) and plays role in absorption. Iron absorbs
 (Dec 2015, 10 Marks) better from heme diet. Iron from heme is released
Ans. For classification of anemias, refer to Ans 37 of same in mucosa of small intestine after absorption for its
chapter. utilization. Iron is consumed from the diet and recycling
Section 1:  General Medicine  135

of iron released is done by catabolism of hemoglobin ♦♦ Cytochemical stains: Myeloperoxidase and Sudan black
derived from destroyed senescent red cells. Transfer are positive.
of iron to bone marrow is by iron-binding globulin, Q.47. Define and classify anemia. Discuss clinical features,
i.e. transferrin. Iron is stored in reticuloendothelial investigation and treatment of hemolytic anemia.
cells, hepatocytes and skeletal muscle cells as two iron  (Apr 2018, 5 Marks)
protein complexes, i.e. ferritin which is water soluble Ans. For definition and classification of anemia refer to Ans
and hemosiderin which is water insoluble. Iron is also 8 of same chapter.
present in myoglobin and cytochrome enzymes.
For clinical features, investigations and treatment of
 Total of 0.5–2 mg of iron is absorbed daily by normal hemolytic anemia refer to Ans 5 of same chapter.
human male or non-menstruating female. Out of which
0.5–1 mg of iron is lost in sweat, urine, stool and in Q.48. Write short answer on megaloblastic anemia.
mensis. 2 mg of iron is utilized in pregnancy. 0.5 mg is  (Apr 2018, 3 Marks) (May 2018, 3 Marks)
utilized in growth and lactation women. Ans. Megaloblastic anaemias are characterized by macrocytic
blood picture (MCV > 100 fl) and megaloblastic bone
For investigations and treatment in detail for iron
marrow.
deficiency anemia, refer to Ans 2 of same chapter.
Q.46. Write short note on clinical features and diagnosis of Causes of megaloblastic anemia
acute myeloid leukemia. (Apr 2017, 6 Marks) I. Due to Vitamin B12 deficiency
Ans. ♦♦ Inadequate intake: In strict vegetarians, poor quality diet,
in elderly
Clinical Features of Acute Myeloid Leukemia ♦♦ Impaired absorption
♦♦ It occurs in adults and it age ranges from 15–40 years. • Gastric
♦♦ It comprises of 20% of childhood leukemia. –– Pernicious anaemia
♦♦ Symptoms due to anemia, i.e. tiredness, weakness and –– Congenital intrinsic factor deficiency
marked pallor. –– Gastrectomy
♦♦ Hemorrhagic manifestations: Petechiae, bleeding from gums • Small intestinal disease
and nose, persistent bleeding after tooth extraction. –– Bacterial overgrowth
♦♦ Infection: It causes infective lesions of mouth and throat, –– Crohn's disease and resection of terminal ileum
i.e. ulceration of mouth and pharynx, herpes simplex –– Tropical sprue and non-tropical sprue
infection of face and infection of respiratory tract such as –– Selective ileal malabsorption of B12 .
bronchitis and pneumonia. –– Fish tapeworm disease
♦♦ Symptoms of cellular hyperviscosity –– Coeliac disease (folic deficiency more common)
♦♦ There are tissue deposits of leukemic cells causing gum –– Miscellaneous — HIV infection, severe pancreatic
hypertrophy which is common in myelomonocytic and disease, drugs
monocytic variety of AML. ♦♦ Increased requirement: In pregnancy and disseminated
cancer
♦♦ Lymphadenopathy and splenomegaly are present
♦♦ Signs of organ infiltration are present, i.e. II. Due to folic acid deficiency
• Skin: Bluish nodules or dusky red patches are present.
♦♦ Inadequate intake: In malnutrition, Old age, Poverty,
• Kidneys: Presence of kidney failure.
Alcoholism, Goat‘s milk, Kwashiorkor
• Other sites: Testes, ovary, liver, gut and serous
♦♦ Impaired absorption: Coeliac disease, Dermatitis
membranes such as pleura and peritoneum.
herpetiformis, Tropical sprue, Congenital folate
♦♦ Bone pain are present, i.e. tenderness of sternum, osteolytic
malabsorption and Oral contraceptives
bone lesions and pathologic fractures may occur.
♦♦ Increased requirement: Infancy, pregnancy and
♦♦ Constitutional symptoms, i.e. fever, malaise and prostration. hyperplastic marrow due to haemolytic anaemia
♦♦ Roth’s spot, i.e. presence of white central retinal ♦♦ Impaired utilization: Folic acid antagonists, methotrexate,
hemorrhages in acute myeloid leukemia pyrimethamine, trimethoprim, anticonvulsant drugs
Diagnosis of Acute Myeloid Leukemia ♦♦ Increased loss (combined folic acid and vitamin B12
deficiency): Tropical sprue, Non—tropical sprue and
♦♦ Peripheral blood picture shows increase in number of haemodialysis
typical or atypical myeloblasts. Auer rods may be found
in cytoplasm. Clinical Features
♦♦ Bone marrow aspirate shows more than 30% blast ♦♦ Due to anaemia: Shortness of breath, dyspnoea, pallor and
cells. Marrow is entirely replaced by myeloblasts and in older subjects angina or cardiac failure.
promyelocytes. ♦♦ Gastrointestinal: Diarrhoea, loss of appetite and weight.
♦♦ There is presence of low to high WBC count. Sore tongue due to glossitis and angular cheilosis. Mild
♦♦ Thrombocytopenia is moderate. jaundice may give the patient a lemon yellow tint.
136   Mastering the BDS IIIrd Year  (Last 25 Years Solved Questions)

♦♦ Neurological • Increased levels of methylmalonic acid in serum and


• Vitamin B12 neuropathy: Due to symmetrical damage urine in B12 deficiency.
to peripheral nerves and posterior and lateral columns • FIGLU excretion in urine in excess in folic acid
of spinal cord, the legs being more affected than arms. deficiency.
Psychiatric abnormalities and visual disturbances may • Deoxyuridine suppression test for deficiency of both
occur (from folate deficiency). vitamin B12 and folate.
• Neural tube defects: Folic acid supplements during • Schillings test of vitamin B12 absorption. Radioactive
pregnancy have been shown to reduce incidence of vitamin B12 is used to assess intrinsic factor and
spina bifida, encephalocele and anencephaly in the Vitamin B12 to distinguish megaloblastic anaemia
foetus. due to intrinsic factor deficiency (pernicious anaemia)
♦♦ CVS disease — Raised serum homocysteine concentrations from other causes of B12 deficiency.
have been associated with arterial obstruction and venous • Serum homocysteine levels - increased in folateand
thrombosis. B12 deficiency.
♦♦ Gonadal dysfunction — Deficiency of either B12 or
folic acid may cause sterility, which is reversible with Diagnosis of Megaloblastic Anaemia
appropriate vitamin supplements. ♦♦ Oval macrocytes in peripheral smear.
♦♦ Knuckle pigmentation ♦♦ Hypersegmented neutrophils.
♦♦ Megaloblastic erythropoiesis in bone marrow.
Laboratory Diagnosis
♦♦ Response to B12/Folate therapy.
♦♦ Blood Film
• Macrocytosis: Hemoglobin content in red cells is Treatment
proportionately increased hence normal MCHC. ♦♦ Vitamin B12 deficiency
• Peripheral smear: It shows • Initially: Hydroxycobalamin 1000 µg IM, 6 injections
–– Oval macrocytes in 2-3 weeks.
–– Anisopoikilocytosis. • Maintenance dose is 500-1000 mcg IM every three
–– Few tear drop cells and normocytes. months for life.
–– Few nucleated RBCs. • For patients sensitive to B12 injections or those who
–– Macrocytes without central pallor. refuse injections, oral B12 (cyanocobalamin) in large
–– Evidence of dyserythropoiesis: basophilic daily doses (100 µg or more).
stippling, Cabot ring and Howell-Jolly bodies. ♦♦ Folate deficiency
• Haemoglobin is decreased. • Initially folic acid 5 mg daily orally for at least 4
• Hypersegmented neutrophils may be the first months. Folic acid in such big doses should not be
evidence of megaloblastic anaemia. given until B12 deficiency has been excluded, since
• Iron ferritin level increases. folic acid may precipitate B12 neuropathy in a severely
♦♦ Bone marrow deficient B12 patient.
• Hypercellularity with marked erythroid hyperplasia. • Maintenance: Need depends on whether the
underlying cause can be reversed, e.g. gluten-free
• Megaloblasts are larger than normoblasts and
diet in coeliac disease.
have sieve like nuclear chromatin. Evidence of
dyserythropoiesis. Q.49. Describe the coagulation pathway. Discuss the clinical
• Myelopoiesis — Giant metamyelocytes and giant band features, pathogenesis and treatment of hemophilia.
forms with abnormal nuclear shapes.  (May 2018, 5 Marks)
• Megakaryocytes with hyperlobulation and immature Ans. For coagulation pathway refer to ans39 of same chapter.
nucleus.
• Dimorphic anaemia — Macrocytes and hypochromic Pathogenesis of hemophilia
microcytes in cases of combined B12/Folate and iron ♦♦ Haemophilia is caused by quantitative reduction of factor
deficiency. VIII in 90% of cases, while 10% cases have normal or
♦♦ Biochemical estimations increased level of factor VIII with reduced activity.
• Serum folate levels: Decrease determined by isotope ♦♦ Factor VIII is synthesized in hepatic parenchymal cells and
dilution method, microbiologic assay. regulates the activation of factor X in intrinsic coagulation
• Serum vitamin B12 levels — Decrease determined by pathway.
isotope dilution technique or microbiological assay. ♦♦ Factor VIII circulates in blood complexed to another larger
Section 1:  General Medicine  137

protein, von Willebrand's factor (vWF), which comprises Investigations


99% of the factor VIII— vWF complex.
♦♦ Imaging:
♦♦ Normal haemostasis requires 25% factor VIII activity.
• Radiography:
Though occasional patients with 25% factor VIII level may –– X-ray Skull shows enlarged sella turcica,
develop bleeding, most symptomatic haemophilic patients enlarged frontal sinus, increase thickness of skull,
have factor VIII levels below 5%. macrognathia and wide space teeth.
For clinical features and treatment of hemophilia refer to –– There is arrow head tufting of finger tips
Ans 11 of same chapter. –– Heel pad sign: Heel pad > 23mm thick.
• CT scan: A large adenoma is easily seen on CT scan
taken after IV contrast.
–– MRI: Sagittal view is useful in identifying the
8. Diseases of relationship between suprasellar and infrasellar
Endocrine System ♦♦
structures.
Biochemical diagnosis:
• Growth hormone levels are increased.
Q.1. Write short note on acromegaly. 
• Glucose tolerance test: It is the accepted diagnostic
 (Dec 2012, 3 Marks) (Sep 1998, 5 Marks)
method measuring glucose and growth hormone. In
Ans. If the excess of growth hormone occur after the fusion
healthy individuals growth hormone is undetectable
of epiphysis, then enlargement of acral parts, i.e. hands,
during the test.
finger, feet and toes occur leading to increase in their
• Insulin-like growth factor-I (IGF–I) levels: Growth
width rather than length.
hormone stimulates production of IGF–I
Etiology predominantly in liver. IGF-I levels assess disease
activity in acromegalics, reflecting overall growth
♦♦ In 95% of cases, it is pituitary adenoma. hormone secretion.
♦♦ Excessive secretion of growth hormone releasing hormone
from carcinoid tumors and adrenal tumors. Management
♦♦ Excessive growth hormone-secreting pancreatic islet cell ♦♦ Surgical: Surgery is the treatment of choice. Surgical
tumors. removal of tumor is done by trans sphenoidal route
♦♦ As a part of multiple endocrine neoplasia type I. followed by radiotherapy.
♦♦ Radiotherapy: It is advised when initial attempts at
Clinical Features surgery do not reduce growth hormone levels to 5 MU/L.
♦♦ General: Fatigue, weight gain, heat intolerance, increased Implantation of radioactive isotope. Yttrium-90 causes
sweating major reduction in growth hormone levels.
♦♦ Skin changes: Thickening of skin. Skin is coarse and greasy, ♦♦ Medical therapy:
perspiration, hypertrichosis • Bromocriptine 20-30 mg/day orally in divided doses
♦♦ Soft tissues: Thickening of lips and nose, macroglossia, is given.
increase in heel pad, hypertrophy of muscular system in • Octreotide decreases the growth hormone levels.
initial stages, mammary hyperplasia. Its dose is 100 µg TDS and can be increased upto
♦♦ Skeletal changes: Arthropathy of joints; Enlargement of 1500 µg/day. Sandostatin: LAR is a sustained release
hands, feet, supraorbital ridges, facial bones; prognathism formulation of acterotide. It is given as 30 mg IM for
is present; spacing apart of teeth; thick clavicles; Changes 6 weeks which decreases growth hormone levels and
in spine, i.e. Osteoporosis, Kyphosis, lordosis and scoliosis; also decreases pituitary tumor size.
carpel-tunnel syndrome; prominent ridges and furrows on • G r o w t h h o r m o n e r e c e p t o r a n t a g o n i s t , i . e .
skull; Large frontal and maxillary sinus Pegvisomant is given S C as 40 mg/day followed by self
♦♦ Cardiovascular: Hypertension, cardiac failure or acromegalic administration of 10mg/day. Liver function needs
cardiomyopathy, coronary artery disease, Arrythmias monitoring.
♦♦ Respiratory: Deep voice due to enlargement of larynx, Q.2. Outline the management of thyrotoxicosis. 
Lungs enlarge proportionately with thorax  (Sep 1999, 3 Marks) (Apr 2010, 5 Marks)
♦♦ Ophthalmologic: Visual field defects are present such as Ans. The management of thyrotoxicosis is divided into four
bitemporal hemianopia or scotomas parts as follows:
♦♦ Metabolic: Impaired glucose tolerance is present. • General:
♦♦ Malignancy: Prevalence of malignant disease, i.e. probability – Allow the patient to take mental and physical
of colonic cancer increased. rest.
138   Mastering the BDS IIIrd Year  (Last 25 Years Solved Questions)

– Maintain nutrition of patient by giving nutritious ♦♦ Emotional stress


diet. ♦♦ Gender: Females more prone than men (7 to 10:1) ratio.
– If patient is anxious alprazolam 0.25 to 0.5 mg ♦♦ Pregnancy: iodine-containing drugs
BD is given. ♦♦ Iodine and drugs: Amiodarone and iodine-containing
• Drug therapy: contrast media may precipitate Graves' disease.
–– Anti-thyroid drugs such as carbimazole, i.e. 40 ♦♦ Irradiation, e.g. radioactive iodine for multinodular
to 60 mg/day, methimazole, i.e. 100 to 150 mg 8 goiter.
hourly and propylthiouracil, i.e. 300 to 450 mg/
day can be given depending on the severity of the Clinical Features
disease. Drugs should be gradually decreased for
Symptoms
4 to 8 weeks based on FT4 levels. When FT4 levels
are normal, the carbimazole 5 to 15 mg/day or ♦♦ Weight loss with increased appetite
propylthiouracil 50 mg/day is given. Drugs can ♦♦ Heat intolerance and sweating
be given for 1 to 2 years by regular checking of ♦♦ Fatigue and weakness
FT4 and TSH levels. ♦♦ Hyperactivity, irritability, dysphoria, insomnia
–– For symptomatic relief, beta-blockers such as
♦♦ Dyspnea
propanolol 80–160 mg daily is given. It is given for
♦♦ Oligomenorrhea, loss of libido
2 to 3 weeks along with anti-thyroid treatment. It
relieves symptoms such as anxiety, tremors and ♦♦ Diarrhea/Defecation hyperclefecation
tachycardia. ♦♦ Polyuria
–– Dexamethasone 8 mg/day may be used to Signs
inhibit conversion of T4 to T3 in severe form of
thyrotoxicosis. ♦♦ Tremor, hyper-reflexia
–– Lithium carbonate 300–450 mg TDS inhibit ♦♦ Tachycarclia, atrial fibrillation in elderly
thyroid hormone secretion temporarily in patients ♦♦ Warm moist skin
who are allergic to iodides and thioamides. ♦♦ Lid retraction, lid lag (causing a stare)
–– Potassium perchlorate 500 mg BD inhibits iodine ♦♦ Thyroid ophthalmopathy is a specific feature for Grave’s
uptake by thyroid gland. It is combined with disease for which signs and symptoms are as follows:
thioamides.
• Surgery: Subtotal thyroidectomy is done in severely Symptoms Signs
affected cases. Before surgery patient should be Xerophthalmia Lid signs: Lid lag and lid retraction
made euthyroid by beta blockers and anti – thyroid resulting in staring look.
drugs. Two weeks before the surgery drugs should
Puffy eyelids Soft tissue signs: Eyelid edema,
be stopped and lugol iodine is given to reduce the
Conjunctival erythema and chemosis
vascularity.
• Radioiodine treatment: Radioactive iodine, i.e. 131I Proptosis Proptosis: It is combined with lid retraction
leads to the destruction of thyroid cells and is given Eyelid retraction Exposure keratitis
with anti-thyroid drugs to prevent thyroid storm.
–– Anti-thyroid drugs must be stopped for minimum Diplopia especially at External ophthalmoplegia
of 3 to 5 days before 131I to allow uptake of isotope extreme gaze
555 MBq to ablate thyroid. Loss of vision Absence of wrinkling on looking upwards
–– High doses are needed for large goiter in severely
thyrotoxic patients. Field loss Mösbius sign: Failure of convergence

Q.3. Write short note on Grave’s disease.  Dyschromatopsia


 (Mar 2001, 5 Marks) Ocular pressure on pain
Ans. Grave’s disease is an autoimmune disease caused by
production of autoantibodies that stimulate thyroid- Lacrimation
stimulating hormone receptor on thyroid cell membrane
resulting in excessive synthesis and secretion of thyroid Investigations
hormone. ♦♦ T3 and T4 both are elevated.
♦ Low TSH or become undetcetable
Risk factors
♦ 131
I uptake is increased, i.e. greater than 35% at 5 hours
♦♦ Genetic susceptibility: Role of hereditary factors is evidenced ♦♦ Serum cholesterol is low
by increased incidence of other autoimmune disorders in ♦♦ ECG shows tachycardia, arrhythmias, ST-T changes
members of patient’s families. ♦♦ Ultrasonography of thyroid shows diffuse goiter.
Section 1:  General Medicine  139

Management Clinical Features


Treatment for Discomfort ♦♦ Goiter is present, i.e. either diffuse or nodular.
♦♦ Artificial tears should be given for the day, i.e. ♦♦ Gastrointestinal features: Vomiting, diarrhea and weight loss
methylcellulose is given. ♦♦ Cardiovascular features: Arrhythmia, i.e. atrial fibrillation,
♦♦ Simple eye ointment should be given for the night dyspnea, wide pulse pressure
♦♦ Patient should use dark glasses with side frames. ♦♦ Dermatological manifestations: Clubbing, loss of hair, palms
♦♦ No smoking become red, increased sweating
♦♦ Reproductive features: Amenorrhea, infertility, abortion,
Medical therapy impotence
♦♦ Ophthalmological features: Exophthalmos, Diplopia, lid
♦♦ Reduction of morning lid edema by sleeping on bed with
retraction, staring look, excessive watering from eyes
its head slightly raised.
♦♦ Prednisolone 60 mg daily is given. ♦♦ Neuromuscular features: Tremors in hand, psychosis,
♦♦ Anti-thyroid drugs such as carbimazole i.e. 40–60 irritability, restlessness, nervousness, high tendon reflexes
mg/day, methimazole, i.e. 100–150 mg 8 hourly and ♦♦ Miscellaneous: Fatigue, polydypsia, heat tolerance
propylthiouracil, i.e. 300–450 mg/day can be given
Investigations
depending on the severity of the disease. Drugs should be
gradually decreased for 4 to 8 weeks based on FT4 levels. ♦♦ Serum TSH level is decreased and is the initial diagnostic
When FT4 levels are normal carbimazole 5–15 mg/day or test. Normal TSH levels exclude clinical hyperthyroidism.
propylthiouracil 50mg/day is given. Drugs can be given ♦♦ Serum total and unbound (free) T3 and T4 are increased
for 1–2 years by regular checking of FT4 and TSH levels. in hyperthyroidism.
♦♦ For symptomatic relief beta blockers such as propanolol ♦♦ In some cases, only T3 levels are raised whereas T4 is
80–160 mg daily is given. It is given for 2–3 weeks along normal (T3 toxicosis).
with anti -thyroid treatment. It relieves symptoms such as ♦♦ TSH-R antibodies levels are increased in about 75% cases.
anxiety, tremors and tachycardia. ESR may be increased in subacute thyroiditis.
♦♦ Radioactive iodine, i.e. 131I leads to the destruction of ♦♦ Uptake of radioactive iodine by thyroid is increased in
thyroid cells and is given with anti-thyroid drugs. Graves’ disease and toxic nodular goiter, whereas it is low
in subacute thyroiditis.
Surgery
♦♦ Ultrasonography of thyroid gland reveals diffuse
♦♦ Subtotal thyroidectomy is done in severely affected cases. enlargement of thyroid gland which helps in differentiating
Before surgery patient should be made euthyroid by beta- Graves’ disease from nodular goiter.
blockers and anti-thyroid drugs. Two weeks before the
surgery drugs should be stopped and lugol iodine is given Management
to reduce the vascularity. Drug Therapy
Q.4. Write short note on hyperthyroidism. 
♦♦ Anti-thyroid drugs such as carbimazole, i.e. 40–60
 (Mar 2003, 5 Marks) (Oct 2007, 10 Marks)
mg/day, methimazole, i.e. 100–150 mg 8 hourly and
(June 2010, 5 Marks) (Sep 2009, 4 Marks)
propylthiouracil, i.e. 300–450 mg/day can be given
Ans. Hyperthyroidism is defined as increased secretion of
depending on the severity of the disease. Drugs should
thyroid hormone with increase in level of T3 and T4.
be gradually decreased for 4–8 weeks based on FT4 levels.
Etiology When FT4 levels are normal carbimazole 5–15 mg/day or
propylthiouracil 50 mg/day is given. Drugs can be given
♦♦ Common causes:
for 1–2 years by regular checking of FT4 and TSH levels.
• Grave’s disease
♦♦ For symptomatic relief beta-blockers such as propanolol
• Toxic nodular goiter
–– Multinodular 80–160 mg daily is given. It is given for 2–3 weeks along
–– Solitary nodule with anti–thyroid treatment. It relieves symptoms such as
♦♦ Less common: anxiety, tremors and tachycardia.
• Thyroiditis ♦♦ Dexamethasone 8 mg/day may be used to inhibit
• Drug-induced conversion of T4–T3 in severe form of thyrotoxicosis.
• Factitious ♦♦ Lithium carbonate 300–450 mg TDS inhibit thyroid
• Iodine excess hormone secretion temporarily in patients who are allergic
♦♦ Rare: to iodides and thioamides.
• Pitutary or ectopic TSH ♦♦ Potassium perchlorate 500 mg BD inhibits iodine uptake
• Thyroid carcinoma by thyroid gland. It is combined with thioamides.
140   Mastering the BDS IIIrd Year  (Last 25 Years Solved Questions)

Surgery ♦♦ Treatment of myxedema is the life-long replacement of


thyroid hormones by L-thyroxine.
Subtotal thyroidectomy is done in severely affected cases. Before
♦♦ Initial starting dosage is 50–100 µg daily as a single dose
surgery patient should be made euthyroid by beta- blockers and
empty stomach in the morning for first 3 to 4 weeks.
anti-thyroid drugs. Two weeks before the surgery drugs should
After some time, dosage can be increased to 150 µg/day.
be stopped and lugol iodine is given to reduce the vascularity.
Adjustment of final dosage should be done after assessing
Radioactive Iodine TSH levels.
♦♦ Maximum dosage of L–thyroxine is 300 µg/day.
♦♦ Radioactive iodine, i.e. 131I leads to the destruction of
♦♦ In geriatric patients or the patients suffering with ischemic
thyroid cells and is given with anti-thyroid drugs to
heart disease, low dose of L–thyroxine 25 µg/day can be
prevent thyroid storm.
started and is increases after assessing the levels of TSH.
♦♦ Anti-thyroid drugs must be stopped for minimum of
♦♦ Since plasma half life of L–thyroxine is 7 days so increase
3–5 days before 131I to allow uptake of isotope 555MBq to
and decrease in dose should be done at an interval of
ablate thyroid.
2 weeks.
♦♦ High doses are needed for large goiters in severely
thyrotoxic patients. Q.6. Describe clinical manifestations and management of
myxedema. (Mar 2003, 10 Marks)
Q.5. Write short note on myxedema. (Mar 1998, 5 Marks)
Or
Ans. Myxedema is a clinical condition resulting from decreased
circulating levels of T3 and T4. It is characterized by Write short note on treatment of myxedema.
deposition of mucinous material causing swelling of  (Feb 2013, 5 Marks)
skin and subcutaneous tissue. Ans.
Clinical Manifestations
Clinical Features
♦♦ Cold intolerance
♦♦ General: There is tiredness, *somnolence, weight gain, cold ♦♦ Thickness and dryness of skin and hair
intolerance and goiter. ♦♦ Swelling of hands and face
♦♦ Skin and subcutaneous tissue: Coarse dry skin, puffiness ♦♦ Change in shape of face
of face with malar flush, baggy eyelids with swollen ♦♦ Thickening of lips is present
edematous appearance of supraclavicular regions, neck ♦♦ Non-pitting edema is present
and lacks of hand and feet. ♦♦ There is yellowish discoloration of skin.
♦♦ Cardiovascular and respiratory features: Bradycardia, angina, ♦♦ Hoarseness of voice is present.
cardiac failure, pericardial effusion and pleural effusion. ♦♦ There is decrease or loss of sweating
♦♦ Neuromuscular features: Aches and pains, cerebellar ♦♦ Loss of hair on outer third of eyebrows.
syndrome with slurred speech and ataxia, muscle cramps ♦♦ Slow pulse/bradycardia.
and stiffness.
♦♦ Gastrointestinal features: Constipation and ascites Management
♦♦ Developmental: Growth and mental retardation Refer to Ans 5 of same chapter.
♦♦ Reproductive system: Infertility, menorrhagia, hyper- Q.7. Describe clinical and diagnostic features of myxede-
prolactemia and galactorrhea. ma. (Feb 1999, 5 Marks)
Investigation Ans. For clinical features, refer to Ans 5 of same chapter.
Diagnosis is based on the Clinical signs of patient and the
♦♦ Serum T3 and T4 are decreased.
investigations
♦♦ Serum TSH level is high
♦♦ Creatinine level increases Diagnostic Features
♦♦ Serum cholesterol level is increased
♦♦ BMR is low Clinical Signs
♦♦ Iodine uptake by thyroid is poor
♦♦ Cold intolerance
♦♦ ECG can show bradycardia, low amplitude of QRS and
♦♦ Thickness and dryness of skin and hair,
ST-T changes.
♦♦ Swelling of hands and face
♦♦ Blood picture shows macrocytic anemia.
♦♦ Change in shape of face
♦♦ X-ray chest can be normal or shows cardiomegaly.
♦♦ Thickening of lips is present
♦♦ Photomotogram reveals delayed ankle jerk.
♦♦ Non-pitting edema is present
Management ♦♦ There is yellowish discoloration of skin.
♦♦ In patient of myxedema adequate ventilation is maintained ♦♦ Hoarseness of voice is present.
along with electrolyte balance and slow warming. ♦♦ There is decrease or loss of sweating
♦♦ Principle of therapy is replacement of deficient thyroid ♦♦ Loss of hair on outer third of eyebrows.
hormones. ♦♦ Slow pulse/bradycardia.
Section 1:  General Medicine  141

Investigations ♦♦ Neuromuscular: Ache and pain, cerebellar ataxia, myalgia,


♦♦ Thyroid function test: There is reduction in T3 and T4 levels delayed relaxation of reflexes, carpal tunnel syndrome
and rise in serum thyroid-stimulating hormone which ♦♦ Gastrointestinal: Constipation, ascites, ileus
indicates primary hypothyroidism. Reduction in T3 and ♦♦ Hematological: Iron deficiency anemia, macrocytic anemia,
T4 levels with TSH level below normal range is secondary pernicious anemia, normochromic normocytic anemia
hypothyroidism. ♦♦ Reproductive: Infertility, impotence, menorrhagia
♦♦ Serum cholesterol: It is raised in primary thyroid failure. The ♦♦ Development: Growth retardation, mental retardation and
fall in serum level is more than 50 mg/100 mL. delayed puberty.
♦♦ Tendon reflex duration is prolonged. Investigations
♦♦ In ECG bradycardia, low voltage complexes and flattened
or inverted T-waves are present. ♦♦ Serum T3 and T4 levels are low.
♦♦ Serum TSH level is high in primary hypothyroidism and
Q.8. Write short note on hypothyroidism. 
low in secondary hypothyroidism.
 (May/June 2009, 5 Marks) (Feb 2006, 5 Marks)
♦♦ Serum cholesterol levels are high.
Or ♦♦ ECG shows bradycardia, low amplitude of QRS and ST-T
Write in brief on hypothyroidism.(Mar 2016, 4 Marks) changes.
♦♦ Blood picture shows macrocytic anemia.
Or ♦♦ X-ray chest can be normal or show cardiomegaly
Write signs and symptoms of hypothyroidism.
 (Jan 2012, 5 Marks) Treatment

Or ♦♦ Treatment of hypothyroidism is life long.


♦♦ Thyroid hormones are replaced by L-thyroxine
Write all clinical features of hypothyroidism. ♦♦ Initial starting dosage is 50–100 µg daily empty stomach
 (Jun 2014, 5 Marks) as single dose in morning for first 3 to 4 weeks. Later dose
Or is increased to 150 µg daily.
♦♦ Final dose adjustment is done by TSH levels. TSH levels
Write clinical features, diagnosis, and management of
are maintained in normal range by increasing the dosage.
hypothyroidism. (Jun 2018, 12 Marks)
♦♦ Maximum dose of L-thyroxine is 300 µg.
Or
Complications of hypothyroidism
Discuss clinical features, complications, investigations
and treatment of hypothyroidism.(May 2018, 5 Marks) Untreated hypothyroidism can lead to a number of health
problems:
Ans. Hypothyroidism is defined as the clinical condition
caused by the low level of circulating thyroid hormones. ♦♦ Goiter: Constant stimulation of thyroid to release more
hormones may cause the gland to become larger — a
Types condition known as a goiter. Hashimoto's thyroiditis is one
of the most common causes of a goiter. Although generally
♦♦ Primary: When the cause lies in thyroid
not uncomfortable, a large goiter can affect appearance and
♦♦ Secondary: When hypothyroidism occurs due to disease
of anterior pituitary. may interfere with swallowing or breathing.
♦♦ Heart problems: Hypothyroidism may also be associated
Clinical Features with an increased risk of heart disease, primarily because
high levels of low-density lipoprotein (LDL) cholesterol
Symptoms
— the "bad" cholesterol — can occur in people with an
♦♦ Feeling of tiredness underactive thyroid. Even subclinical hypothyroidism, a
♦♦ Weight gain mild or early form of hypothyroidism in which symptoms
♦♦ Cold intolerance have not yet developed, can cause an increase in total
♦♦ Hoarseness of voice and lethargy cholesterol levels and impair the pumping ability of heart.
♦♦ Somnolence Hypothyroidism can also lead to an enlarged heart and
♦♦ Goiter heart failure.
♦♦ Hyperlipidemia. ♦♦ Mental health issues: Depression may occur early in
hypothyroidism and may become more severe over
Signs time. Hypothyroidism can also cause slowed mental
♦♦ Skin and subcutaneous tissues: Dry skin, puffiness of face functioning.
with malar flush, baggy eyelids, alopecia, vitiligo ♦♦ Peripheral neuropathy: Long-term uncontrolled
♦♦ Cardiovascular: Bradycardia, angina, heart failure hypothyroidism can cause damage to peripheral nerves
♦♦ Respiratory: Pericardial effusion, pleural effusion — the nerves that carry information from brain and spinal
♦♦ Psychiatric: Depression and psychosis cord to the rest of body. Signs and symptoms of peripheral
142   Mastering the BDS IIIrd Year  (Last 25 Years Solved Questions)

neuropathy may include pain, numbness and tingling in Free T4 Index


the area affected by the nerve damage. It may also cause
It represents a calculated free T4 hormone concentration. It is
muscle weakness or loss of muscle control.
calculated by multiplying total serum T4 by T4 binding ratio.
♦♦ Myxedema: This rare, life-threatening condition is the
The free T4 index is low and high in hypothyroidism and
result of long-term, undiagnosed hypothyroidism. Its
hyperthyroidism.
signs and symptoms include intense cold intolerance
and drowsiness followed by profound lethargy and Thyroid-Stimulating Hormone (TSH)
unconsciousness. A myxedema coma may be triggered by
sedatives, infection or other stress on your body. If in an It is secreted from anterior pituitary and has negative feedback
individual shows signs or symptoms of myxedema, he/she relationship with circulating T4 and T3. It is the single best test
need immediate emergency medical treatment. for diagnosing primary hypothyroidism and hyperthyroidism.
♦♦ Infertility: Low levels of thyroid hormone can interfere In former condition, serum TSH is increased and in later
with ovulation, which impairs fertility. In addition, some condition it is decreased.
of the causes of hypothyroidism — such as autoimmune Q.10. Write briefly on Addison’s disease. 
disorder — can also impair fertility.  (Apr 2010, 5 Marks)
♦♦ Birth defects: Babies born to women with untreated Ans. Addison’s disease or primary hypoadrenalism results
thyroid disease may have a higher risk of birth defects from destruction of adrenal cortex by variety of
than babies born to healthy mothers. These children are pathological processes.
also more prone to serious intellectual and developmental
problems. Infants with untreated hypothyroidism present Etiology
at birth are at risk of serious problems with both physical ♦♦ It is caused due to autoimmune adrenalitis.
and mental development. But if this condition is diagnosed ♦♦ Infections, i.e. TB, cytomegalovirus and fungal infection
within the first few months of life, the chances of normal associated with AIDS.
development are excellent. ♦♦ By tumors
Q.9. Discuss thyroid function tests. (Oct 2003, 15 Marks) ♦♦ Inherited disorders, i.e. adrenoleukodystrophies and
Ans. A laboratory thyroid profile includes measurement of familial isolated glucocorticoid deficiency.
• Serum T4
Clinical Features
• Resin T3 uptake
• T4 binding ratio ♦♦ Pigmentation of skin and mucus membrane: There is bluish
• Free T4 index black discoloration of lips, gums and posterior aspect of
• Thyroid-stimulating hormone. palate.
♦♦ Gastrointestinal symptoms: Anorexia, nausea and vomiting.
Serum T4
Constipation with intermittent diarrhea, achlorhydria and
Measured total serum T4 reflects hormone which both binds to abdominal pain
thyroid-binding globulin and free hormone. The normal range ♦♦ Cardiovascular system: Postural hypotension, faintness may
of serum T4 is 5–11.5 µg/DL. Hence, changes in either free result. Dyspnea is also present.
hormone level or concentration in thyroid-binding globulin will ♦♦ Muscular system: Muscular weakness and wastening with
alter total serum T4 but not affect free hormone level. Therefore, creatinuria. Sometimes cramp in muscles is present.
increased secretion of thyroid-binding globulin increases total ♦♦ Mental and nervous: Muscle weakness and lassitude
serum T4 whereas decrease in TBG lowers total serum T4.
(exhaustion) are first symptoms to appear. Loss of memory
Resin T3 Uptake and drowsiness.
♦♦ Genital system: Impotence and *amenorrhea are present.
It reflects the thyroid-binding globulin concentration and is
♦♦ Renal system: Renal function is impaired and the excretion
reported as percentage, i.e. 25 to 35%. A present amount of
of urine is diminished.
radioactive T3 is added to patient’s sample of serum. Radioactive
T3 binds to all liable binding sites on thyroid-binding globulin Investigation
and left over is bound to resin and is measured.
♦♦ Serum sodium and chloride level are decreased
T4 Binding Ratio ♦♦ Blood sugar level is decreased
♦♦ Blood urea increases
The resin T3 uptake is converted to T4 binding ratio by dividing
the measured resin T3 uptake by reference serum mean resin ♦♦ Urine secretion of chloride is increased.
T3 uptake which is 30%. The resin T3 uptake and to T4 binding ♦♦ Urinary secretion decreases
ratio are low and high in hypothyroidism and hyperthyroidism, ♦♦ BMR is decreased
respectively. ♦♦ Abdominal X-ray shows calcification of adrenal gland.
Section 1:  General Medicine  143

Treatment Q.15. Write short note on investigations in a case of Cushing’s


♦♦ Cortisone substitution: Hydrocortisone 20 mg TDS for 72 syndrome. (Mar 2007, 5 Marks)
hours. For maintenance, dose is 20 mg in morning and 10 Ans. Cushing’s syndrome is caused by the excessive
mg in evening. production of cortisol.
♦♦ Aldosterone substitution: Fludrocortisone 50 μg BD is started.
Investigations
♦♦ Salt: Patients with diarrhea or profuse sweating should
be given an additional 3–6 gm of sodium chloride daily. ♦♦ Urinary excretion of 11 oxysteroids and 17 ketosteroids
is increased. Dexamethasone suppression test showing
Q.11. Write short note on thyrotoxicosis (Clinical features).
failure of significant plasma cortisol suppression favors
 (Sep 2004, 3.5 Marks) (Mar 2009, 5 Marks)
adrenal tumor as the cause.
Ans. For clinical features of thyrotoxicosis, refer to Ans 4 of
♦♦ Glucose tolerance test such as diabetes.
same chapter.
♦♦ X-ray chest for evidence of thymic carcinoma.
Q.12. Describe the clinical findings of hyperthyroidism. ♦♦ X-ray skull for pituitary edema is not helpful since a
(Sep 2005, 10 Marks) pituitary basophilic adenoma does not expand and so sella
Or turcica and glenoid processes are normal.
Describe the clinical findings, diagnosis and manage- ♦♦ X-ray dorsal spine may show collapse of vertebrae
ment of hyperthyroidism. (Sep 2004, 20 Marks) producing fish spine appearance.
Ans. For clinical finding and management refer to Ans 4 of ♦♦ IV *pyelography for adrenocortical tumor. *Perirenal
same chapter. insufflation, *tomography, USG and CT scan and MRI to
detect adrenal hyperplasia.
Diagnosis Q.16. Write in brief clinical features and management of
The case with signs like exophthalmos, tremors, tachycardia and thyroid nodules. (Apr 2007, 5 Marks)
thyroid enlargement and symptoms, i.e. sweating, intolerance Ans. The term “thyroid nodule” refers to any abnormal
to heat, restlessness, increased appetite, diarrhea and weight growth that forms a lump in the thyroid gland.
loss can be diagnosed as hyperthyroidism. In anxiety state, the
Clinical Features
hands are cold and moist while in hyperthyroidism, they are
warm and moist. ♦♦ Voice hoarseness
Thyroid function test: There is raised T3 and T4 levels. ♦♦ Rapid increase in size, there are compressive symptoms
Diagnosis is solely based or investigations. For details of such as dyspnea or dysphagia
investigations refer to Ans 4 of same chapter. ♦♦ Lymphadenopathy
♦♦ Sudden, unexplained weight loss
Q.13. Write short note on parathyroid hormone.
♦♦ Nervousness
(Feb 2002, 5 Marks) ♦♦ Rapid or irregular heartbeat.
Ans. Parathyroid hormone is secreted by chief cells of
parathyroid gland. Management
♦♦ Surgery should be performed in the following instances:
Action
• Reaccumulation of the nodule despite 3 to 4 repeated
♦♦ Primary function of parathyroid hormone is to maintain FNACs
the blood calcium level within the range of 9 to 11 mg%. • Size in excess of 4 cm
♦♦ Parathyroid hormone maintains blood calcium level • Complex cyst on thyroid ultrasound (showing solid
by acting on bone, kidney and GIT; by increasing the and cystic components)
resorption of calcium from bones, by decreasing excretion • Compressive symptoms
of calcium through kidneys, by increasing absorption of • Signs of malignancy (vocal cord dysfunction,
calcium through GIT. lymphadenopathy)
♦♦ The increased activity of parathyroid glands lead to ♦♦ Oral thyroxine therapy to suppress pituitary secretion
excessive secretion of parathormone which leads to the of thyroid-stimulating hormone, thereby removing an
disorder known as hyperparathyroidism and decreased important growth factor for thyroid follicle cells.
synthesis of parathormone is called hypopara­thyroidism.
Q.14. Write short note on thyrotoxicosis. 
 (Sep 2006, 10 Marks)
9. Nutrition and
Or Metabolic Defects
Write short answer on thyrotoxicosis. 
 (Apr 2018, 3 Marks) Q.1. Write short note on vitamin A deficiency.
Ans. Refer to Ans 4 of same chapter  (Feb 2006, 5 Marks) (Mar 2009, 5 Marks)
144   Mastering the BDS IIIrd Year  (Last 25 Years Solved Questions)

Or Q.2. Write short notes on beriberi. (Sep 1999, 5 Marks)


Describe the clinical features of vitamin A deficiency. Ans. Beriberi is caused due to the deficiency of vitamin B1,
 (Feb 2013, 5 Marks) i.e. thiamine.
Ans. Deficiency of vitamin A causes interference with growth, Deficiency of vitamin B1 affects growth, nutrition and
reduced resistance to infections and interference with carbohydrate metabolism.
nutrition of cornea, conjunctiva, trachea, hair follicle and
Etiology
renal pelvis.
♦♦ Vitamin A deficiency interferes with ability of eyes to adapt Deficiency of vitamin B1 occurs due to:
to darkness and impairs visual affinity. ♦♦ Chronic alcohol intake
♦♦ Children with vitamin A deficiency will experience ♦♦ Severe malnutrition
impaired growth and development. ♦♦ Chronic debilitating diseases

Causes Pathogenesis
♦♦ Poor intake Thiamine forms an essential part of two coenzymes which are
♦♦ Malabsorption important in oxidative decarboxylation of alpha-ketoacidosis.
♦♦ Disease of liver and intestine. Deficiency of vitamin leads to accumulation of alpha-
ketoacidosis whose toxic effects results in production of beriberi.
Clinical Features
Types of Beriberi
♦♦ Earliest sign of deficiency of vitamin is difficulty in reading
or sewing at night or finding anything in darkness. ♦♦ Wet Beriberi or cardiovascular beriberi: Beriberi is wet when
♦♦ Conjunctiva becomes dry and small grayish white raised there is cardiac involvement.
spots appear. Bitot’s spots form grayish white triangular ♦♦ Dry beriberi or neuritic type: Beriberi is dry when there is
plaques on conjunctival surface lateral to cornea. CNS involvement.
♦♦ Microcytic anemia.
Clinical Features
♦♦ Cornea subsequently becomes dry and lusterless, and if
there is lack of treatment changes are irreversible. ♦♦ Beri-beri
♦♦ *Keratomalacia involving the cornea leading to the • Wet beriberi: Palpitation, dyspnea, Cardiomegaly,
ulceration and blindness may result. warm extremities, anasarca (severe generalized
♦♦ Children with vitamin A deficiency not only have retarded edema), signs of congestive heart failure in late stage.
growth but also increased tendency to chest infection. • Dry beriberi: Cramps, tingling and numbness in the
♦♦ Skin become dry and rough limbs, nystagmus (cyclic movements of eyes), wrist
♦♦ Imperfect enamel formation of teeth. and foot drop, ataxia, loss of equilibrium, parasthesia
♦♦ Hemeralopia: Patient is unable to see bright light. and confusion.
♦♦ Asteatosis, i.e. persistent dry scaling of the skin is the ♦♦ Wernick’s encephalopathy: There is involvement of brain
earliest manifestation. which is characterized by ataxia, ophthalmoplegia,
♦♦ Phrynoderma, i.e. toad skin. Brown to dark brown, confusion and disorientation.
dry, rough, hyperkeratotic follicular papules with ♦♦ Korsakoff psychosis: It occurs due to involvement of
central keratotic horny spurs. Bilateral and symmetrical mammillary bodies and confabulates with amnesia.
distribution in anterolateral aspects of thighs, posterolateral ♦♦ Presence of angular stomatitis.
aspects of arms.
Diagnosis
Treatment ♦♦ Blood thiamine is low
♦♦ Prevention of vitamin A deficiency by giving good ♦♦ Raised pyruvate and lactate levels.
nutrition, intake of fresh leafy green vegetables and ♦♦ Low urinary excretion of thiamine and its metabolites
addition of vitamin A to food stuffs. ♦♦ Measuring of whole blood or erythrocyte transketolase
♦♦ Single dose of oral retinol palmitate 60 mg or 200,000 IU activity before and after addition of thiamine. In this there
should be given. In cases of diarrhea 50 mg IV may be is low transketolase activity which increases by 15% after
given. Second dose is repeated next day and the third and addition of thiamine confirm the diagnosis.
last dose is given at the time of follow-up visit. ♦♦ There is clinical improvement after single dose of thiamine,
♦♦ Various other associated conditions such as diarrhea, i.e. 50 mg IM with increase in diastolic blood pressure and
dehydration, electrolytic imbalance, protein energy decrease in heart rate is another diagnostic criteria.
malnutrition should be treated by appropriate measures.
♦♦ For local treatment of eyes patient should be sent to Treatment
ophthalmologist. ♦♦ Vitamin B1 (50–100 mg daily) IM or IV is given.
♦♦ If primary or secondary infections are present, antibiotics ♦♦ As acute crisis is over, patient has given small dose of
should be given to the patient. 5 mg daily along with nourishing diet.
Section 1:  General Medicine  145

♦♦ Diet consists of high B complex and protein content like Management of Rickets and Osteomalacia
eggs, milk, nuts and green vegetables.
♦♦ Dietary enrichment of vitamin D in form of milk.
♦♦ Wet beriberi with cardiac involvement require digoxin and
♦♦ If tetany is present give IV calcium gluconate. Daily dose
diuretics, if congestive heart failure is present.
is 1000–2000 IU of vitamin D combined with 500–1000
Q.3. Describe briefly vitamin D deficiency.  mg of calcium.
 (Sep 1999, 3 Marks) (Mar 2010, 5 Marks)
♦♦ Curative treatment includes 2000–4000 IU of calcium daily
Ans. Vitamin D deficiency leads to rickets in growing child for 6–12 weeks followed by daily maintenance dose of
and osteomalacia in adults.
2000–4000 IU for long period.
Vitamin D is essential in calcium and phosphorus ♦♦ Patients with osteomalacia require large dose of vitamin
metabolism. It is required for normal development of
D and calcium, i.e. 40,000–1,00,000 IU of vitamin D and
bones and teeth.
15–20 gm of calcium lactate per day.
Deficiency of vitamin D causes imperfect skeletal
formation, bone disease, rickets and caries. Q.4. Write short note on protein calorie malnutrition. 
 (Jan 2012, 5 Marks)
Vitamin D Deficiency Rickets Ans. Protein calorie malnutrition leads to two diseases known
It occurs generally in growing children. as marasmus and kwashiorkor.

Clinical Features Marasmus


♦♦ In first six months of life tetany and convulsions are common. It is seen in infants and young children.
These above manifestations are due to hypo­calcaemia. In marasmus, there is total deficiency of calories including
♦♦ Wrist and ankles are swollen protein deficiency.
♦♦ Changes in bone are found in epiphyseal plates, metaphysis
and shaft. Causes
♦♦ Localized area of thinning are sometime present in skull ♦♦ Poor intake of breastfeeds and or inadequate supply of
so that a finger can produce indentation. This condition milk and other nutrients.
is called craniotabes. ♦♦ Lack of digestion
♦♦ Pigeon breast is present. ♦♦ Impaired absorption of protein.
♦♦ Developmental abnormalities of dentin and delayed ♦♦ Protein is absorbed but cannot be metabolized satisfactorily.
eruption. ♦♦ Protein is metabolized but not properly utilized.
♦♦ Higher caries index.
♦♦ Hypoplasia of enamel is present. Clinical Features
♦♦ Pulp chamber is large.
It occurs below the age of 1 year
♦♦ Malocclusion of teeth is present.
♦♦ There is progressive loss of subcutaneous fat.
Investigations ♦♦ Child is irritable and cries excessively.
♦♦ Radiological: Radiograph of wrist show widening of distal ♦♦ Typical appearance of monkey face.
ends of shaft of both radius and ulna. Zone of epiphyseal ♦♦ Sunken and lusterless eyes.
cartilage get thickened. As these changes get completely ♦♦ Delay in sitting, standing and walking.
developed, they cause saucer shaped deformity. ♦♦ Abdomen may be *distended or sunken and it does not
♦♦ Biochemical changes: has fat in wall
• Low plasma calcium and phosphate levels ♦♦ Lack of playful movements and persistent crying.
• High serum alkaline phosphatase levels ♦♦ Gross degree of muscle wasting is present
• Plasma 25 dehydroxy cholecalciferol is low or ♦♦ Limbs are thin, look like sticks and buttocks do not contain
absent. fat.
♦♦ Head is large for body and ribs are visible.
Vitamin D Deficiency Osteomalacia
It is also known as adult rickets. Only flat bones and diaphysis Management
of long bones are affected. ♦♦ Correction of fluid, electrolytes, acidosis, hypoglycemia,
hypothermia.
Clinical Features
♦♦ Look for the infections and treat them from proper
♦♦ Pelvic deformities are seen in females. antibiotic therapy.
♦♦ Remodeling of bone occur in absence of adequate calcium ♦♦ Diet is given to the child which is rich in proteins, energy
resulting in softening and distortion of skeleton. and essential nutrients. Diet therapy should be continued
♦♦ Bone pain and muscle weakness is present. till the child attains normal weight as well as nutritional
♦♦ Severe periodontitis is present. status.
146   Mastering the BDS IIIrd Year  (Last 25 Years Solved Questions)

♦♦ Muscles and fats: Presence of wasting of muscles of chest,


upper arm, legs and hips. Fat is seen plentiful at these sites.
♦♦ Vitamin deficiencies: Vitamin A deficiency is present,
folate deficiency is common, deficiency of vitamin K can
cause purpura and bleeding.

Management
♦♦ Diet
• Diet for mild-to-moderate cases: Diet should be easily
digestable. It should be rich in proteins, minerals and
vitamins with extra calories. Milk should be given.
Egg is a good flip with milk and water as a source of
first class protein Plant protein mixtures such as corn,
soya, diet milk can be given. Additional fats should be
included. If there is lactose intolerance dal, rice and
butter can be given.
• Diet for acute cases: Protein intake should be 3 to 4 g/
Kg/day and energy intake of 150 kcal/kg/day. If child
is unable to take diet from mouth go for nasogastric
intubation. Care must be taken to take sufficient amount
of plant, animal proteins and fats to maintain the weight.
♦♦ Vitamins and mineral supplementations
• Vitamin A, D, B complex and C are given at therapeutic
dosages.
• In vitamin A deficiency 30 mg of Vitamin A should
Fig. 17: Marasmus be given for 3 days.
• 0.5–1 g of potassium chloride dissolve in feeds or water
Kwashiorkor should be given daily in divided dosages.
♦♦ Maintenance of body temperature
It occurs in children between 1 to 4 years.
For maintainence of body temperature at night blankets
In Kwashiorkor there is deficiency of proteins and amino acids or room heaters are useful.
Causes ♦♦ Skin care
In cases of dermatoses skin should be kept clean and
♦♦ Prolong febrile illness, massive burns and large chronic protected.
ulcers. V. In Hospital treatment
♦♦ Stress, starvation, persisting vomiting and diarrhea.
♦♦ Chronic infections, i.e. urinary tract infections, TB and
parasitic infections.

Clinical Features (Signs and symptoms)


♦♦ Growth: Failure of growth is present, child appears lean
and thin. Presence of generalized edema.
♦♦ Hair: Change in the color of hair from black to blond,
reddish or grey. Hairs are fine, straight and show sparse
pigmentation.
♦♦ Skin: Skin become thick, pigmented, desquamated and
ulcerated. A severe case with desquamation looks like
the child is burnt. Dermatosis with fissuring is also seen
in moderate cases.
♦♦ Gastrointestinal tract: Anorexia and diarrhea are common.
Liver is enlarged and palpable due to fatty changes.
♦♦ Blood: There is presence of moderate degree of anemia.
♦♦ Mucosa changes: There is presence of stomatitis, chelosis,
smooth tongue and ulceration is present around anus.
♦♦ Mental changes: Child becomes miserable, apathetic and
has mewing cry. Fig. 18:  Kwashiorkor

Q4. *Distended = To stretch out.


Section 1:  General Medicine  147

• Correction of dehydration, electrolyte disturbances, In Children


hypoglycemia, acidosis and hypothermia.
♦♦ Diet
• Parenteral therapy with half of saline and 2.5% glucose
• Diet for mild-to-moderate cases: Diet should be easily
before diet therapy.
digestable. It should be rich in proteins, minerals and
• In anemia, packed erythrocyte infusion is given.
vitamins with extra calories. Milk should be given.
Q.5. Write causes and management of malnutrition in India. Egg is a good flip with milk and water as a source of
 (Oct 2003, 5 Marks) first class protein. Plant protein mixtures such as corn,
Ans. Causes of Malnutrition soya, diet milk can be given. Additional fats should be
♦♦ Due to defective intake of food: included. If there is lactose intolerance dal, rice and
• Loss of appetite due to depression, anxiety, anorexia butter can be given.
nervosa, etc. • Diet for acute cases: Protein intake should be 3 to 4 g/
• Due to persistent vomiting Kg/day and energy intake of 150 Kcal/kg/day. If child
• Alcohol intake is unable to take diet from mouth go for nasogastric
• Unbalanced therapeutic diet intubation. Care must be taken to take suffieicnt
• Administration of prolong IV fluids, parenteral fluids amount of plant, animal proteins and fats to maintain
in hospitalized patients the weight.
♦♦ Maldigestion and disordered absorption: ♦♦ Vitamins and mineral supplementations
• In hypochlorhydria or achlorhydria
• Vitamin A, D, B complex and C are given at therapeutic
• In steatorrhea
dosages.
• Intestinal hurry due to surgical resection
• In vitamin A deficiency 30 mg of vitamin A should be
• Due to prolong use of antibiotics
♦♦ Defective utilization: given for 3 days.
• In cirrhosis of liver • 0.5–1 gm of potassium chloride dissolve in feeds or
• In malignancy water should be given daily in divided dosages.
• In infections ♦♦ Maintenance of body temperature
• In renal failure For maintenance of body temperature at night blankets or
• Due to inborn error of metabolism, i.e. hartnup’s room heaters are useful.
disease ♦♦ Skin care
• Due to drugs such as anti-epileptics • In cases of dermatoses skin should be kept clean and
♦♦ Loss of nutrients from body: protected.
• Proteinuria in nephrotic syndrome leads to ♦♦ In hospital treatment
hypoproteinemia. • Correction of dehydration, electrolyte disturbances,
• In diabetes mellitus, glycosuria causes loss of energy hypoglycemia, acidosis and hypothermia.
and undernutrition • Parenteral therapy with half of saline and 2.5% glucose
• Due to excessive menstrual blood loss before diet therapy.
• Hypokalemia due to severe persistant diarrhea. • In anemia, packed erythrocyte infusion is given.
♦♦ Due to increased demands:
• In pregnancy, lactation, adolescence, illness and heavy
Q.6. Write short note on osteoporosis. 
manual work.
• In fever and thyrotoxicosis  (Sep 1998, 5 Marks) (Oct 2007, 5 Marks)
• In burn, surgery and trauma, there is increase in Ans. Osteoporosis is a group of skeletal disorders characterized
catabolism of proteins and vitamin C. by reduction in bone mass per unit bone volume, which
• Malignant cachexia causes loss of weight due to results in increased risk of fracture, even in absence of
increased demand. noticeable injury.

Treatment of Malnutrition
In Adults
♦♦ For mild starvation, adequate supplementation of nutrients
is necessary
♦♦ In moderately starved people, extra-feeding is needed
♦♦ In severely starved persons, food is given in small amount
at frequent intervals. Food should be staple meal, i.e. cereal
with some sugar, milk and oils, salt is restricted. Potassium,
magnesium and vitamins are adequately given. This is
continued till patient feel active. Fig. 19:  Normal cancellous bone
148   Mastering the BDS IIIrd Year  (Last 25 Years Solved Questions)

♦♦ Calcium 1.5–2 g/day, if calcium malabsorption is there,


cholecalciferol 0.25 mg, alfacalcidol 0.5 µg/day.
♦♦ Estrogen therapy: 0.635 mg daily estrogen is given in small
thin women, early women, etc.
♦♦ Biphosphates, i.e. etidronate 400 mg/day for 2 weeks
inhibit bone resorption.
♦♦ Calcitonin 100 IU SC inhibits bone resorption.
♦♦ Sodium fluoride 40–60 mg/day stimulates osteoblasts and
causes bone formation.
Q.7. Mention the complications of diabetes mellitus. 
Fig. 20:  Osteoporotic cancellous bone
 (Sep 1999, 5 Marks) (Sep 2006, 5 Marks)
Or
Causes
Write short note on complications of diabetes mellitus.
♦♦ Dietary calcium deficiency: Especially in case of women  (Mar 2016, 3 Marks) (Apr 2010, 5 Marks)
both premenopausal and postmenopausal. Apart from  (Mar 2011, 4 Marks)
increased requirements, degree of calcium absorption from
intestine decreases with age. Or
♦♦ Excessive alcohol or caffeine. Enumerate the complications of diabetes mellitus.
♦♦ Genetic influences.  (Feb/Mar 2005, 5 Marks)
Other disorders affecting mineral homeostasis: Or
♦♦ Endocrine: Cushing syndrome, hyperthyroidism, type I Write notes on ten complications of diabetes mellitus.
diabetes mellitus, hypogonadism and acromegaly (Only names) (Dec 2009, 10 Marks)
♦♦ Drugs: Glucocorticoids, thyroxin and diuretics.
Ans. Diabetes mellitus is collection of disorders that have
Clinical Features hyperglycemia as hallmark.
♦♦ Bone loss progresses for many years without causing Complications
symptoms.
Acute or Immediate Complications
♦♦ Shortened heights, bone pain, low back pain, difficulty
in getting up from chair are complaints in women at ♦♦ Diabetic ketoacidosis or coma
menopause with osteoporosis. ♦♦ Hypoglycemia or hypoglycemic coma
♦♦ Signs of osteoporosis include deformities of skeleton, ♦♦ Non-ketotic hyperosmolar diabetic coma
such as kyphosis (Dowager’s hump) and loss of height ♦♦ Lactic acidosis
especially in vertebral compression fracture. First two complications occur in Type I diabetes mellitus and
other two occur in Type II diabetes mellitus.
Diagnosis
♦♦ History of fracture: Fractures associated with osteoporosis Diabetic Ketoacidosis
are vertebral fractures, hip fractures and Colle's fracture ♦♦ It is an exclusive complication of type I diabetes. It
of distal radius. can develop in patients with severe insulin deficiency
♦♦ Radiograph: It shows prominence of vertical trabeculae, combined with glucagon excess. Failure to take insulin
generalized loss of contrast between bone and soft tissue. and exposure to stress are precipitating causes.
♦♦ Bone density measurement: Dual energy X-ray absorptiometry ♦♦ As the ketogenesis continues the excess ketone bodies
is the gold standard. It is performed in spine, both hips and produced cannot be degraded by the muscles and other
both wrists. Bone densities are then plotted against normal tissues resulting in ketosis, which manifests as anorexia,
database to provide standard deviation from normal, i.e. nausea, vomiting, deep and fast breathing, mental
T score. For post-menopausal women, T score values are: confusion and coma. However, most of the patients
1 to 0 is normal recover.
0 to –1 is normal Hypoglycemia
–1 to –2.5 is osteopenia
♦♦ It is defined as fall in blood glucose concentration below
<2.5 is osteoporosis 3.1 mmol/L.
♦♦ It is seen in type I diabetes patients due to excessive
Management
administration of insulin, missing a meal or due to stress.
♦♦ Active physical activity helps to maintain strength, ♦♦ As the hypoglycemia continues, it can lead to comma,
flexibility and co-ordination of body improves. cardiac arrhythmias and is fatal.
Section 1:  General Medicine  149

♦♦ This can lead to worsening of control of diabetes and degeneration of myelinated and non-myelinated fibers,
rebound hyperglycemia. Segmental demyelination, Schwann cell injury.
Nonketotic Hyperosmolar Diabetic Coma Diabetic Nephropathy
♦♦ It is common in type II diabetes mellitus. ♦♦ This is the most common cause of mortality and morbidity
♦♦ The clinical hallmark is hyperglycemia, hyperosmolality in diabetic patients.
and dehydration without ketoacidosis. ♦♦ 40 to 50% of type II diabetes patient develop it and 25%
♦♦ Precipitating features are infection, myocardial infarction, patients with type I diabetes develop end stage renal
drugs such as thiazides, steroids, diphenylhydantoin. disease and die of it.
♦♦ Loss of glucose in urine is so intense that the patient is ♦♦ It is divided into three stages, in first stage patient is
unable to drink sufficient water to maintain urinary fluid asymptomatic but has high GFR, in next stage there is renal
loss. hypertrophy which leads to microalbuminuria. In advance
♦♦ Because of high viscosity of blood, thrombotic and stage, patient develop macroproteinuria and passes onto
bleeding complications are frequent. Mortality rate is high nephrotic syndrome.
in this complication. ♦♦ 25% of patients with diabetic nephropathy can go directly
at end stage renal disease with hypertension and undergo
Lactic Acidosis chronic renal failure.
♦♦ It is seen in type II diabetes mellitus.
♦♦ It is caused due to excess lactate production and/or Miscellaneous
inadequate utilization. It consists of various infections, gastroparesis, arthropathy.
♦♦ This can be precipitated by metformin or other systemic
disorders such as liver or renal failure, pancreatitis or Atherosclerosis
leukemia. ♦♦ In diabetic patient development of athroma is faster as
♦♦ Cardiovascular collapse leads to mortality. compared to normal individual.
Chronic or Late Onset Complications ♦♦ Hyperlipidemia, decreased HDL levels, increased platelet
adhesiveness, obesity and associated hypertension are
These complications are due to changes in small blood vessels i.e. contributory factors for athrosclerosis.
microangiopathy or in large blood vessels, i.e. macroangiopathy. ♦♦ Atherosclerosis leads to coronary artery disease, silent
Microvascular are retinopathy, neuropathy, nephropathy myocardial infarction, cerebral stroke and gangrene of
and miscellaneous toes and feet.
Macrovascular are atherosclerosis, hypertension, peripheral Diabetic foot ulcer
vascular disease and diabetic foot ulcer
The chronic complications occur more frequently in Type II ♦♦ This is the frequent site of complication in diabetes.
♦♦ Pathogenic components of diabetic foot are neuropathy,
diabetes mellitus rather than Type I diabetes mellitus.
peripheral vascular disease causing ischemia and
Diabetic Retinopathy secondary infection causing ulceration.
♦♦ This is the very important cause of blindness in diabetic Q.8. Outline the management of diabetes mellitus. 
patients.  (Sep 1997, 7.5 Marks)
♦♦ Dilatation of retinal capillaries is earliest sign. Besides Or
these there is also presence of microaneurysms, retinal Write short note on treatment of diabetes mellitus
hemorrhage, neovascularization, hard and soft exudates,  (Apr 2017, 6 Marks)
vitreous hemorrhage and fibrosis.
Ans. Management
♦♦ Frequency, onset and severity of retinopathy vary in
diabetic patients.
I. Diet Management
♦♦ Background retinopathy is most common and proliferative
retinopathy is less common. ♦♦ Restoration of normal blood glucose and optimum levels.
♦♦ Maintenance of blood glucose levels as near to physiologic
Diabetic Neuropathy
levels to prevent onset or progression of complications.
♦♦ It can involve any part of nervous system except the brain. ♦♦ Maintenance of normal growth rate in children and
♦♦ Neuropathy is an early and common complication which adolescents as well as attainment and maintenance of
leads to morbidity and disability. reasonable body weight in adolescents and children.
♦♦ Poor glycemic control and long duration of diabetes is ♦♦ Provision of adequate nutrition for pregnant women and
associated with high incidence of neuropathy. fetus during lactation.
♦♦ Sign and symptoms are of peripheral nervous system. ♦♦ Consistency in timing of meals and snacks to prevent
♦♦ Main pathological changes in peripheral nerves are axonal inordinate swings in blood glucose level.
150   Mastering the BDS IIIrd Year  (Last 25 Years Solved Questions)

♦♦ Motivation to have small frequent meals. Free foods: All meat, fish, eggs (not fried), clear soup or meat
♦♦ Determination of meal plan appropriate for individual extracts; tea or coffee; vegetables such as cabbage, cauliflower,
and based on dietary history to have good compliance. spinach, pumpkin, brinjal, lady’s finger, turnip, French beans,
♦♦ Management of weight reduction for obese individuals cucumber, lettuce, tomato, spring onions, radish, asparagus.
with NIDDM. Spices, salt, pepper and mustard; butter and margarine. Sugar
♦♦ Improvement in the overall health of patients with diabetes substitutes for sweetening.
through optimal nutrition.
II. Oral Hypoglycemics
Total Calories These drugs are used in patients of Type II diabetes mellitus
Requirements are determined by the patients activity: (NIDDM) who do not respond to dietary management and
♦♦ Overweight NIDDM should be encouraged to establish who would otherwise require treatment with insulin in later
their weight within a desirable range. A reduction of situation, so they are also used as adjuvant drugs to insulin in
approximately 500 kcal/ day can result in loss of 1–2 kg/ overweight diabetes patients.
month. ♦♦ Insulin secretagogues i.e. drugs increasing secretion of
♦♦ Carbohydrates: Carbohydrates should be taken in form of insulin
starch and complex sugars. 100–300 g of carbohydrates • Sulfonyl ureas
should be spreaded over 3 meals, i.e. 60g each and 3 snacks,
Name of drug Intial daily Dose/ Comment
i.e. 30 g each with half litre of milk. Unrefined carbohydrates dose (mg) day
should be substituted by refined carbohydrates to the
extent possible. First generation
♦♦ Proteins: Recommended dietary allowance of 0.85 g/kg Acetohexamide 250 mg 1–2 Has diuretic and uricosuric
body weight for adult is an appropriate guide. activity
♦♦ Fat: Fat intake should be 50–150 g daily divided between Chlorpropamide 100 mg 1–2 Can potentiate ADH
the meals. Replacement of saturated with polyunsaturated
fat is desirable to reduce cardiovascular risk. Cholesterol Tolazamide 100 mg 1–2 Disulfiram-like action with
alcohol in 1/3rd of patients,
intake should be < 300 mg/day.
Has diuretic activity
♦♦ Fiber: Increased consumption of dietary fiber especially
soluble fiber are associated with lower levels of blood Second generation
glucose and serum lipids. The water insoluble fibers such Glibenclamide 2.5–20 mg 1–2 Hypoglycemia can be
as cellulose, lignin and most hemicelluloses found in whole severe
grain breads, cereals and wheat bran affect gastrointestinal
Gliclazide 40–320 mg 1–2 Metabolism/excretion by
transit time and fecal bulk with little impact on plasma liver/kidney
glucose. However highly viscous water soluble fibers such
as pectins, gums and storage polysaccharide found in Glipizide 5 mg 1–2 Mild diuretic activity
fruits, legumes, lentils roots, tubers, oat and oat bran, when Glyburide 2.5 mg 1–2 Highest risk of
eaten in purified form, reduce serum levels of glucose and hypoglycernia
insulin. Ideal recommended amount of fiber in patient’s Glemepiride 1 mg 1 Excretecd in urine and bile
diet is 35–40 g/day.
♦♦ Alternative sweeteners: Both nutritive and non-nutritive • Meglitinides:
sweeteners are acceptable in diabetes management. –– Repaglinide, i.e. 0.5–4 mg three times a day
♦♦ Sodium: It should be restricted to 1000 mg/1000 kcal, –– Netaglinide, i.e. 60–120 mg three times a day 15
to 30 min before each meal
not to exceed 3000 mg/day to minimize symptoms of
–– Voglibose, i.e. 0.2–0.3 mg, 15–30 min before each
hypertension.
meal.
♦♦ Alcohol: It should be taken in moderation and may need
♦♦ Insulin sensitizers, i.e.drugs sensitizing action of insulin
to be restricted entirely by person with diabetes and
and overcome insulin resistance
insulin—induced hypoglycemia, neuropathy, poor control
• Biguanides, i.e. metformin should be given 1.5–2.5 g/
of glucose and lipids, or obesity.
day in three divided doses after meals.
♦♦ Vitamins, minerals and antioxidants: intake should be • Thiozolidinediones, i.e.rosiglitazone should be given
encouraged. 2–8 mg or pioglitazone 15–45 mg in a single or two
Forbidden foods: Sugar, jam, jellies, honey, jaggery, tinned divided doses. They can be combined with sulfonyl
fruits and juices, sweets, chocolate, ice creams, pastries, glucose ureas or metformin
drinks, foods made with sugar, pudding, sauces. ♦♦ Alpha glucosidase inhibitors, i.e. Acarbose 25 to 100 mg
Foods allowed in moderation: Bread of all kinds and chapattis TDS or Voglibose 0.2–5 mg TDS taken orally in three main
made from wheat or millets, plain biscuits, all fresh fruits, baked meals. It can be given in combination with metformin 500
beans, breakfast cereals. mg for increased efficiency.
Section 1:  General Medicine  151

♦♦ DPP-4 inhibitor: Vildagliptin 50–100 mg daily in two dived ♦♦ Insulin purity: The impurities which may be contained
doses with meals ot saxagliptin 2.5 to 5mg OD. are proinsulin, insulin intermediates and contaminating
♦♦ Pramlinide: Initial dose is 15 μg before each meals in Type proteins from islet tissue or exocrine pancreas such as
I diabetes mellitus and 60 μg in Type II diabetes mellitus. glucagon, somatostatin and pancreatic polypeptides.
Standard insulins currently have only 10–20 PPM of
III. Insulin
proinsulin and purified monocomponent insulins less
Following are the indications for insulin therapy: than 1 PPM.
♦♦ In type I diabetes mellitus Best sites for the insulin injection are subcutaneous fat on
♦♦ In gestational diabetes abdomen, buttock, anterior thigh and dorsal area of arm.
♦♦ Hyperglycaemia despite maximum doses of oral agents
♦♦ Decompensation due to intercurrent events such as IV Exercise
infection, acute injury or stress ♦♦ Exercise must be adjusted to patient’s preference and
♦♦ Development of severe hyperglycemia with ketosis existing medical condition.
♦♦ Perioperative in patients undergoing surgery ♦♦ Aerobic exercise is preferred, i.e. swimming, cycling,
♦♦ Kidney or hepatic disease walking, running. etc
♦♦ Addition of moderate resistance should be considered.
Various Insulin Preparations
♦♦ Duration should be 20–45 minutes per session.
Insulin preparation Onset Peak Duration ♦♦ Frequency is 3–4 sessions per week is required to observe
(Hours) (Hours) (Hours) beneficial metabolic effects. 4–5 sessions per week for
Rapid acting weight reduction.
♦♦ ADA recommends 150 min/week of moderate aerobic
Regular ½–1 2–4 6–8
physical activity with no gaps longer than 2 days.
Lispro (analogue) ¼–½ 1–2 3–5 ♦♦ Program of exercise should be:
Intermediate acting • Stretching for 5–10 min
NPH (isophane) 1–4 8–10 12–20
• Warm up for 5–10 min
• Exercise for 20–45 min
Lente (insulin zinc 2–4 8–12 12–20
• Warm down for 10 min at 30% of full exercise intensity.
suspension)
Long acting Q.9. Write short note on insulin. (Feb 2006, 5 Marks)
Ans. Insulin is required for proper control of diabetes and to
Ultralente (Extended 3–5 10–16 18–24
insulin zinc keep blood sugar within limits.
suspension) Following are the indications for insulin therapy:
♦♦ In type I diabetes mellitus
Treatment Strategies with Insulin ♦♦ In gestational diabetes
♦♦ Single dose regimen: Daily injection of intermediate act ing ♦♦ Hyperglycemia despite maximum doses of oral agents
insulin given before breakfast. The starting dose 0.3–0.4 U/ ♦♦ Decompensation due to intercurrent events such as
Kg/ day; increased gradually to obtain glucose values in infection, acute injury or stress
acceptable range. Regular insulin can be added to decrease ♦♦ Development of severe hyperglycemia with ketosis
the glucose level that follows breakfast. ♦♦ Perioperative in patients undergoing surgery
♦♦ Twice daily regimen: Combination of regular and ♦♦ Kidney or hepatic disease
intermediate acting insulin BD, i.e. before breakfast and Various insulin preparations
before dinner ("Split mix" regimen).
♦♦ Multiple daily injections: For achievement of more tight Insulin preparation Onset Peak Duration
control of blood glucose which requires administration (Hours) (Hours) (Hours)
of at least 3 injections per day. These can be given with Rapid acting
use of mixture of intermediate- and short-acting insulins
Regular ½–1 2–4 6–8
(pre-mixed insulin) in the morning before breakfast,
with regular insulin before supper- and intermediate- Lispro (Analogue) ¼–½ 1–2 3–5
acting at bedtime; or again a combination of regular and Intermediate acting
intermediate-acting before dinner. Divide the total daily
into 2 equal doses following 1:1 transfer from basal insulin. NPH (Isophane) 1–4 8–10 12–20
Give half before breakfast and other half before dinner. Lente (Insulin zinc 2–4 8–12 12–20
The largest meal will require larger proportion of insulin. suspension)
Reduce total dose by 20%, if patient experiences recurrent Long acting
hypoglycemia.
♦♦ Insulin concentrations: The insulins are available in many Ultralente (Extended 3–5 10–16 18–24
concentrations. In India the commonly used is 40 U/mL insulin zinc suspension)
152   Mastering the BDS IIIrd Year  (Last 25 Years Solved Questions)

Treatment Strategies with Insulin Q.10. Outline management of diabetic comma. 


♦♦ Single dose regimen: Daily injection of intermediate acting  (Feb 1999, 5 Marks)
insulin given before breakfast. The starting dose 0.3–0.4 U/ Ans. Diabetic ketoacidosis or diabetic comma is the exaggeration
Kg/ day; increased gradually to obtain glucose values in or deranged energy metabolism due to deficiency of
acceptable range. Regular insulin can be added to decrease insulin which results in accumulation of acid metabolites
the glucose level that follows breakfast. and ketone bodies.
♦♦ Twice daily regimen: Combination of regular and
intermediate acting insulin BD, i.e. before breakfast and Management
before dinner ("Split mix" regimen). Following is the management of diabetic comma
♦♦ Multiple daily injections: For achievement of more tight ♦♦ Admission of patient
control of blood glucose which requires administration • Diagnosis is confirmed by examination of blood
of at least 3 injections per day. These can be given with glucose and ketone measurements.
use of mixture of intermediate- and short acting insulins • Initial assessment of dehydration, hyperosmolality,
(pre-mixed insulin) in the morning before breakfast, serum potassium, acidosis and kidney function is
with regular insulin before supper- and intermediate- done.
acting at bedtime; or again a combination of regular and • Fluid loss is measured by subtracting admission
intermediate acting before dinner. Divide the total daily weight by last known stable weight.
into 2 equal doses following 1:1 transfer from basal insulin. • Patient should be evaluated for sepsis.
Give half before breakfast and other half before dinner. ♦♦ Management during hour 1
The largest meal will require larger proportion of insulin. • If patient is hypovolemic and hypotensive fluid
Reduce total dose by 20%, if patient experiences recurrent administration is done, i.e. normal saline is
hypoglycemia. administered and if necessary colloids should be
♦♦ Insulin concentrations: The insulins are available in many given. Rate of administration is necessary to restore
concentrations. In India, the commonly used is 40 U/mL circulatory function. As blood pressure become
♦♦ Insulin purity: The impurities which may be contained normal and urine output becomes adequate rate of
are proinsulin, insulin intermediates and contaminating administration of normal saline is 1000 mL/hour.
proteins from islet tissue or exocrine pancreas such as • Continuous I.V. infusion of regular insulin 5–10 units/
glucagon, somatostatin and pancreatic polypeptides. hour is given.
Standard insulins currently have only 10–20 PPM of • I.V, infusion of potassium is done at 10–40 mmol/ hour
proinsulin and purified monocomponent insulins less during initiation of insulin therapy.
than 1 PPM. • Sodium bicarbonate IV is given in cases with acidosis.
Dose is 50–100 mmol/L sodium bicarbonate in 0.45%
Site of Insulin Injection saline for 30–60 minutes. Additional potassium should
Best sites for the insulin injection are subcutaneous fat on also be given with bicarbonate therapy.
abdomen, buttock, anterior thigh and dorsal area of arm. ♦♦ Management during hour 2
• Normal saline should be continued at 500 mL/ hour.
Insulin Delivery System
Plasma osmolality should be greater than 285 mmol
♦♦ Insulin pens are used in both Type I and Type II diabetes
/ L during first 12 hours. If serum sodium is greater
mellitus.
than 150 mmol/L switch to 0.45% saline.
♦♦ Continuous insulin infusion systems are used in Type II
• Blood glucose should be checked and insulin is
diabetes mellitus.
adjusted to 5 mmol/litre/hour. Blood glucose should
Complications not fall below 11.1–14.0 mmol/L. Anion gap is
decreasing and blood pH is increasing.
Following are the complications of insulin therapy: • Serum potassium should be maintained at 4–5 mmol/
♦♦ Hypoglycemia: It is the most frequent and most serious litre by addition of potassium to IV fluids.
reaction. Hypoglycemia can occur in any diabetic ♦♦ Management during hours 3 to 4
following inadvertent injection of large doses, by missing • Management should be continued as given in second
a meal or by performing various exercises hour.
♦♦ Local reaction: Swelling, erythema and stinging sometimes • Cognitive and neurological symptoms should be
occurs especially in beginning. *Lipodystrophy occurs at observed for 12 hours.
injection site after long usage. ♦♦ Management during hours 5 to 8
♦♦ Insulin allergy: This is infrequent and is due to contaminating • Normal saline should be continued at 250 mL/ hour.
proteins. Urticaria, angiedema and anaphylaxis are As blood glucose reaches 11.1–14 mmol/ L, change IV
manifestations. fluid to 500 mL/hour normal saline with 5% glucose.
♦♦ Edema: Some patients develop short-lived dependent • Insulin should be continued at maintenance dose until
edema when insulin therapy is started. ketoacidosis is cleared.
♦♦ Weight gain: Patients on long-term insulin therapy gain • Potassium should be continued at 10–40 mmol/ hour,
weight. since ketoacidosis is cleared.
Section 1:  General Medicine  153

• Phosphate replacement is done at 6 hours, if serum ♦♦ Non-islet cell tumors, e.g. hepatoma, adrenal carcinoma,
phosphate is < 2 mg/dL mesothelioma, etc
♦♦ Management during hours 8 to 24 ♦♦ Iatrogenic, i.e. drug induced due to oral hypoglycemic,
• IV repletion is continued with 0.45% saline with or salicylism, due to propranolol.
without 2.5% or 5% glucose as needed.
Clinical features
• As diabetic comma is subsided, patient should be
switched to subcutaneous insulin and stop IV and ♦♦ Cardiovascular system: Palpitation, tachycardia, anxiety,
IM insulin. cardiac arrhythmias
♦♦ Central nervous system: Tremors, confusion, headache,
Q.11. Differentiate between diabetic comma and hypo­
tiredness, difficulty in concentration, in cordination,
glycemic comma. slurred speech, drowsiness, convulsion and coma.
 (Feb 2006, 3 Marks) (Sep 2008, 2.5 Marks) ♦♦ Gastrointestinal tract: Nausea and vomiting
Ans. ♦♦ Skin: Sweating and hypothermia
Feature Diabetic comma Hypoglycemic comma Diagnosis
Onset Slow and insidious Rapid and quick ♦♦ In hypoglycemia state:
History Too little or no insulin Regular dose of insulin • Suggestive history
and no food • Dramatic response to IV glucose during attack
Precipitating Due to untreated or In severe unaccustomed • Low plasma level glucose during attack
factor hidden infection exercise • C-peptide concentration > 200pmL/L
Cause Due to inadequate Due to excess of insulin ♦♦ Of cause:
insulin • Clinical: Exclude hypopituitarism, Addison’s disease,
Symptoms Frequent vomiting with No vomit/occasional liver cirrhosis or failure, sarcoma, alcohol ingestion
abdominal pain vomit after fasting, self administration of insulin or
Signs • Tongue and skin are • Tongue and skin are sulphonylurea.
dry moist • Investigations for insulinomas:
• Weak or feable • Bounding pulse –– Overnight fasting plasma glucose and insulin
pulse • Normal breathing measurements will demonstrate spontaneous
• Breathing is rapid • Absence of air hunger hvpoglycemia and raised plasma insulin
and shallow • Smell of acetone is –– Low plasma C-peptide with high plasma insulin.
• Presence of air absent
–– High fasting plasma proinsulin.
hunger • Tendon reflexes are
• Smell of acetone is brisk Management
present • Plantars are extensor ♦♦ In acute attack:
• Tendon reflexes are
• Administration of rapidly absorbable carbohydrate
diminished
• In mild reaction, orange juice (100 mL) or corn syrup
• Plantars are normal
or candy taken orally sufficient.
Urine Glucose and ketone Glucose and ketone
• In unconscious or uncooperative patients 50 mL of
bodies are present bodies are absent
50% glucose iv if vein not available glucagon 1 mg sc
Blood Low blood glucose, High blood glucose, low
or im will cause sufficient increase in blood sugar to
bicarbonate and pH bicarbonate and pH
allow the patient to become rational and cooperative.
levels are normal.
• When recovery is slow, e.g. after overdose of insulin
Treatment Response to treatment Response to treatment is
or sulphonylurea therapy, constant infusion of 10–20%
is slow quick and is very rapid to
IV glucose
dextrose is given to maintain the blood sugar.
♦♦ Conservative treatment and prevention of acute attacks
Q.12. Write short note on hypoglycemia.  • Diet: Carbohydrate not more than 150 gms in slowly
 (Feb/Mar 2004, 5 Marks) (Sep 2010, 5 Marks) absorbable form like cereals, bread, fruits and
Ans. Hypoglycemia is defined as fall in blood glucose vegetables. Liberal protein because glucose derived
concentration below 2.5 mmol/L. from it is liberated slowly; fat to make up calories.
In hepatogenic type bedtime meal to prevent early
It can be present as asymptomatic, mild, severe or may
morning hypoglycemia.
be present as coma. • Restriction of physical exercise.
Etiology ♦♦ Surgical measures:
• Removal of islet cell tumors, or partial resection of
♦♦ Postprandial pancreas
♦♦ Alcohol ingestion –– In fulminating cases where convulsions are not
♦♦ Starvation, prolonged exercise, infection controlled by glucose.
154   Mastering the BDS IIIrd Year  (Last 25 Years Solved Questions)

–– In severe, chronic cases not controlled by diet. Diagnosis


–– Patients with marked, neuropsychiatric ♦♦ Presence of low hemoglobin
syrnptoms. ♦♦ Macrocytosis,
♦♦ Drugs ♦♦ High MCHC
• Glucagon: The efficacy depends on size of hepatic ♦♦ Megaloblastic bone marrow
glycogen stores. The drug has little effect on blood ♦♦ Gastric achlorhyclria
glucose in patients who have been fasting or are ♦♦ Serum vitamin B12 less than 100 pg/mL
hypoglycemic for long periods. IM injection of 1mL of ♦♦ Elevated homocysteine
glucagon is given, if hypoglycemia is insulin induced.
• Diazoxide and Octreotide may be used to control Treatment
symptoms while the patient is awaiting surgery or
♦♦ Vitamin B12 deficiency especially in those with megaloblastic
when the patient is not a candidate for surgery.
anemia is treated by giving hydroxycobalamin 1000
Q.13. Write short note on vitamin B12 deficiency.  microgram IM twice a week for first week. This is followed
 (Feb 2002, 5 Marks) by 1000 µg weekly for 6 weeks.
Ans. It is also known as Wonder vitamin and cyanocobalamin. ♦♦ Replacement therapy is given throughout the life in dosage
of hydroxycobalamin 1000 µg IM for every three months.
Causes of Vitamin B12 Deficiency ♦♦ For patients sensitive to injections, Oral B12 in large daily
♦♦ Due to inadequate dietary intake doses is given, i.e. 100 µg or more.
♦♦ Due to malabsorption Q.14. Write short note on scurvy. 
• Intrinsic factor deficiency in pernicious anemia,  (Dec 2012, 3 Marks) (Feb/Mar 2004, 5 Marks)
gastrectomy, congenital lack of intrinsic factor Ans. Deficiency of vitamin C leads to scurvy.
• Intestinal causes, i.e. tropical sprue, Ileal resection,
Due to vitamin C deficiency, there is defective collagen
Crohn’s disease
formation.
• Removal of vitamin B12 from intestines i.e. In
tapeworm infestation and due to bacterial proliferation Etiology
in intestinal blind loop syndrome
• Drugs such as neomycin, PAS and colchicine. ♦♦ In artificially fed infants
♦♦ It is precipitated by febrile disease, infections and diarrhea
Clinical Features Clinical Features

♦♦ Blood: Megaloblastic anemia ♦♦ Onset of the disease is usually gradual, there is fretfulness
♦♦ General features: Pallor, anorexia, diarrhea and increasing pallor, or tenderness of legs which causes
♦♦ Oral manifestations: Glossitis, Glossodynia, Angular child to cry whenever touched, anorexia and loss of
stomatitis, xerostomia weight. First symptom is inability of the child to use his
♦♦ Nervous system manifestations: legs.
• Subjective sensory disturbances: Paraesthesia, Tingling ♦♦ Symptoms due to hemorrhages:
and numbness in toes, tips of fingers, rarely • Under periosteum of long bones: “Pithed frog posture"
simultaneously in both upper and lower extremities. with thighs flexed and abducted and knees flexed,
Sometimes burning or stabbing pains or even lightning sometimes diffuse swelling above or below the knee;
pains like tabes. in severe cases, infiltration of muscles with blood
• Objective sensory loss: Sense ofvibration, posture and causing edematous limbs, i.e woody leg; hemorrhage
passive movement affected first in lower, later in upper between diaphysis and epiphysis causes separation of
limbs. Glove and stocking type of superficial sensory epiphysis from shaft.
loss. Tenderness of calf muscles. • Perifollicular hemorrhage.
• Motor symptoms: Pyramidal weakness and ataxia • In orbit, there is proptosis with swelling of eyelids
develop at variable interval after onset of sensory • Hematuria: May be an early symptom and is only
disturbances. microscopic.
• Reflexes: Ankle jerks lost, knee jerks may be absent. Both • Scorbutic beading of the ribs
exaggerated, if lateral column lesion predominates. ♦♦ Presence of anemia
Plantars extensor. ♦♦ Keratosis of hair follicles with ‘corkscrew’ hair
• Sphincter disturbances: First difficult or precipitate ♦♦ Failure of wound healing
micturition, later retention of urine or incontinence.
Impotence is early. Oral Manifestations
• Mental changes: Mild dementia, impaired memory. ♦♦ It chiefly affects the gingival and periodontal region.
Confusional psychosis or irritability or depression. ♦♦ The interdental and marginal gingiva becomes bright red,
• Bilateral primary optic atrophy in 5%. swollen, smooth, shiny producing appearance known as
Section 1:  General Medicine  155

scurvy bud. In fully developed scurvy, gingiva becomes • A balanced diet not only looks after the well-being
boggy, ulcerated and bleeds easily of person but may have role to play in prevention
♦♦ There is a typical fetid breath of a patient with fusospirochetal of certain diseases.
stomatitis. • So while planning a balanced diet, care should
♦♦ Color of the gingiva changes to violaceous red. be taken that all essential ingredient in diet are
♦♦ In severe cases, hemorrhage and swelling of periodontal adequately incorporated with fresh fruits, vegetables
ligament membrane occur followed by loss of bone and and fiber diet.
loosening of teeth which are exfoliated. • Food should be palatable, properly cooked and well
balanced providing a balance between food intake
Diagnosis and energy output.
♦♦ Low ascorbic acid level in platelets and plasma Q.16. Write note on pellagra. (Dec 2009, 10 Marks)
♦♦ Anemia is mild-to-moderate but may be severe.
Ans. It is the clinical condition produced due to the deficiency
♦♦ X-ray shows characteristic features such as
of nicotinic acid or niacin.
• Pencilled cortex
• White lines of Frankel Etiology
• Zone of Trummerfeld
♦♦ Inadequate intake or absorption of niacin.
• Halo sign of Wimberger
♦♦ Restricted or limited diet in which single serial grain, i.e.
• Subperiosteal hematoma
corn is consumed without consumption of wheat, eggs,
• Joint effusion
beef and another niacin rich food.
• Scorbutic rosary
♦♦ In chronic alcoholism.
Management
Clinical Features
♦♦ About 3 to 4 ounces of fresh orange juice or tomato juice
daily. ♦♦ Pellagra is triad of dermatitis, diarrhea and dementia.
♦♦ Ascorbic acid 100 mg or more orally or parenterally twice ♦♦ Dermatitis is bilaterally symmetrical and occur in parts
or thrice daily for 10 to 15 days. exposed to sunlight. In acute cases skin lesions may
produce vesiculation, cracking, exudation, crusting with
Q.15. Write short note on balanced diet.  ulceration and desquamation. In chronic cases, dermatitis
 (Feb/Mar 2004, 5 Marks) (Apr 2008, 5 Marks) occurs.
 (June 2010, 5 Marks) (Aug 2012, 5 Marks) ♦♦ Gastrointestinal manifestations: Presence of anorexia,
Or nausea, dysphagia. Glossitis precede the skin lesions.
Write notes on balanced diet. (Aug 2011, 10 Marks) Diarrhoea is also present.
Or ♦♦ Mental features consists of apathy, insomnia, fatigue
Write about balanced diet. (Mar 2006, 10 Marks) followed by encephalopathy characterized by confusion,
Ans. A diet adequate in energy providing substances, i.e. disorientation, loss of memory and hallucination.
carbohydrates and fats, tissue building compounds, ♦♦ Other features: Loss of appetite, irritability and burning
i.e. proteins, inorganic chemical, i.e. water and mineral sensation in different areas of body.
salts, agents that regulate or catalyze metabolic process, Oral Manifestations
i.e. vitamins and substances for certain physiological
processes such as bulk for promoting peristaltic ♦♦ Oral mucosa is fiery red and is painful.
movement of digestive tract known as balanced diet. ♦♦ Glossitis is present. Tongue is red, swollen and beefy. In
• For proper growth, maintenance and development early stage, only the tip and margins of tongue are swollen
of our body an ideal combination of essential and red but in advance cases, tongue looses the papillae
nutrient, vitamins and minerals should be aimed. and reddening is intense.
• An average Indian diet is poor in quality as well as ♦♦ Presence of angular stomatitis
quantity in meeting the daily needs. ♦♦ Superimposed acute necrotizing ulcerative gingivitis is
• A balanced diet should contain 30 cal/kg optimum present involving gingiva, tongue and mucosa.
body weight. Diagnosis
• Balanced diet should contain 60–70% carbohydrates
of total calories, 12–18% protein of total calories, ♦♦ NAD and NADP levels are low in RBCs in patients with
20–25% fats of total calories. pellagra.
• Diet should contain as much as of fresh fruits, ♦♦ Plasma tryptophan is low.
vegetables which provide nutrients.
Treatment
• Fiber is an important part of the food.
• Milk is an ideal food and contains or proximate the Initially by injection nicotinamide 50–100 mg IM daily for a
principles of balanced diet. week. Maintenance dose of 50 mg orally per day.
156   Mastering the BDS IIIrd Year  (Last 25 Years Solved Questions)

Q.17. Write in brief clinical features and management of Treatment


diabetic ketoacidosis.  (Apr 2007, 5 Marks)
The main treatment is lifestyle, i.e. caloric restriction and
Ans. Diabetic ketoacidosis or diabetic comma is the
physical activity. However, drug treatment may occasionally
exaggeration or deranged energy metabolism due to
be necessary. Generally, the individual diseases that comprise
deficiency of insulin which results in accumulation of
the metabolic syndrome are treated separately, e.g. diuretics
acid metabolites and ketone bodies.
and ACE inhibitors for hypertension. Cholesterol drugs may
Clinical Features be used to lower LDL cholesterol and triglyceride levels, if
they are elevated.
Symptoms
♦♦ Polyuria
♦♦ Thirst
♦♦ Weight loss
♦♦ Weakness
♦♦ Nausea
♦♦ Vomiting
♦♦ Leg cramps
Fig. 21:  Metabolic syndrome
♦♦ Blurred vision
♦♦ Abdominal pain.
Q.19. Write short note on diabetic ketoacidosis.
Signs  (May/June 2009, 5 Marks) (Sep 2007, 5 Marks)
 (Dec 2010, 5 Marks)
♦♦ Dehydration
♦♦ Hypotension Ans. Diabetic ketoacidosis a state of acidemia induced
♦♦ Tachycardia by excess production of ketoacids. Dehydration and
♦♦ Confusion hyperglycemia are the rule and lactic acidosis may also
♦♦ Drowsiness be present.
♦♦ Comma.
Pathophysiology
For management, refer to Ans 10 of same chapter.
Q.18. Write short note on metabolic syndrome X /dysmetabolic Diabetic ketoacidosis is caused by severe insulin deficiency
syndrome. (Apr 2007, 5 Marks) and is accentuated by excessive glucagon secretion. This leads
Ans. Metabolic syndrome is a combination of medical to major clinical and laboratory abnormalities seen in diabetic
disorders that increase one’s risk for cardiovascular ketoacidosis, which includes excess mobilization of free acids
disease and diabetes. It affects a large number of people from adipose tissue, increased glucose production from the
in a clustered fashion. liver, impaired glucose uptake and utilization by muscle. The
two major effects of uncontrolled diabetes are:
It is known under various other names, such as
♦♦ Increased glucose production which causes hyperglycemia,
(metabolic) syndrome X, insulin resistance syndrome,
osmotic diuresis, electrolyte depletion and dehydration
Reaven’s syndrome or CHAOS.
♦♦ Increased ketogenesis, resulting in metabolic acidosis
Clinical Features
Precipitating Causes
Symptoms
♦♦ Acute infection viral or bacterial single most common
♦♦ Fasting hyperglycemia cause.
♦♦ High blood pressure ♦♦ Omission of insulin or inadequate dosage.
♦♦ Central obesity (also known as visceral, male pattern or ♦♦ Vomiting.
apple-shaped adiposity), overweight with fat deposits ♦♦ Diarrhea.
mainly around the waist. ♦♦ Prolonged neglect of diabetes.
♦♦ Weakness, fatigue and restlessness. ♦♦ Indiscretions in diet.
Signs ♦♦ Surgical operations.
♦♦ Trauma.
♦♦ Fatty liver (especially in concurrent obesity), ♦♦ Myocardial infarction.
♦♦ Non-alcoholic fatty liver disease ♦♦ Pregnancy.
♦♦ Polycystic ovarian syndrome ♦♦ Thyrotoxicosis.
♦♦ Hemochromatosis (iron overload); ♦♦ Resistance to insulin.
♦♦ Acanthosis nigricans (a skin condition featuring dark ♦♦ Unnoticed interruption of insulin delivery in diabetics
patches). treated with continuous subcutaneous insulin infusion.
Section 1:  General Medicine  157

Clinical Features ♦♦ Chronic obstructive jaundice.


♦♦ Prolonged treatment with anticonvulsant drugs.
Refer to Ans 17 of same chapter.
Toxicity or Excess of Vitamin D
Diagnosis
There is presence of anorexia, lassitude, vomiting, diarrhea,
The cardinal features are:
profuse sweating, polydipsia, polyuria and headache.
♦♦ Acidosis, i.e. arterial pH ≤ 7.3 Hypercalcemia causes calcium deposition in tissues and kidneys
♦♦ Plasma anion gap ≥ 16 mmol/L which leads to renal failure.
♦♦ Serum ketone is positive
♦♦ Serum bicarbonate ≥ 15 mmol/L Q.21. Write short note on vitamin C.
♦♦ Hyperglycemia, i.e. Plasma glucose ≥ 11.1 mmol/L  (Mar 2008, 5.5 Marks) (Mar 2011, 4 Marks)
Ans. Vitamin C was named ascorbic acid owing to its anti-
Management ascorbic properties.
Refer to Ans 28 of same chapter. ♦♦ Ascorbic acid is a white crystalline water-soluble substance
with sour taste.
Q.20. Write short note on vitamin D.  ♦♦ It is easily destroyed by cooking.
 (Feb 2014, 3 Marks) (Mar 2008, 7 Marks)
Ans. Vitamin D are a group of sterol and commonly found in Absorption and Storage
animals mainly there are two types of active form: ♦♦ Ascorbic acid is easily absorbed from the small intestine,
D2 - Ergosterol/Ergocalciferol peritoneum and subcutaneous tissues.
D3 - 7- dehydrocholesterol. ♦♦ It is not stored in any particular organ and is spread
throughout the body.
Source
Sources
Fish liver oil, egg yolk, butter milk, the cheapest source is
sun light which forms vitamin D3 from its precursor present Amla, citrus fruits, tomatoes, guava, green peppers.
in the skin.
Physiological Functions
Physiological Functions ♦♦ It is involved in oxidation reaction of cell.
♦♦ Vitamin D increases the absorption of Ca and P. ♦♦ It is required for the metabolism of amino acid, e.g. tyrosine
♦♦ It increases the calcification of bones in children and adult. and tryptophan.
So, it is essential for the development of bones and normal ♦♦ It is also involved in conversion of folic acid into its active
growth of body. coenzyme.
♦♦ It increases the excretion of phosphate by kidney and helps ♦♦ It is required for iron absorption and also for the formation
in lowering of serum phosphate concentration. of collagen fibers and mucopolysaccharides of connective
♦♦ It increase the citrate levels of blood, bone, kidney and tissue.
heart tissues.
Daily Requirements
♦♦ It decreases the pH in the lower intestinal tract which
helps in increasing the absorption of calcium and ♦♦ Infants: 35 mg
phosphate. ♦♦ Children: 40 mg
♦♦ Adults: 45 mg
Daily Requirements ♦♦ Pregnant and lactating women 80 mg.
I nfants and children: 400 IU Deficiency Manifestations
Adult: 200 IU ♦♦ Severe ascorbic acid deficiency produces—Scurvy.
Pregnant and lactating women: 400 IU ♦♦ For scurvy in detail refer to Ans 14 of same chapter.
Q.22. Write short note on Thiamin (Vitamin B1). 
Deficiency of Vitamin D  (Sep 2009, 4.5 Marks)
♦♦ Rickets: Due to defect in bone growth and calcification of Ans.
long bones in children. Thiamine (Vitamin B1)
♦♦ Osteomalacia: Due to defective calcification of steroid tissue
Synonyms: Anti-beriberi factor, anti-neuritic vitamin, aneurin.
in adult.
Properties
Causes of Deficiency of Vitamin D
♦♦ Thiamine is readily soluble in water
♦♦ Dietary insufficiency or insufficient exposure to sunlight. ♦♦ It is stable in acid medium.
♦♦ Gastrointestinal disorder. ♦♦ It is destroyed when autoclave at 120°C for 30 minutes.
158   Mastering the BDS IIIrd Year  (Last 25 Years Solved Questions)

Source ♦♦ Fat: Fat intake should be 50 to 150 gm daily divided


between the meals. Replacement of saturated with
♦♦ Rich sources: Polishing rice, wheat jaw and yeast
polyunsaturated fat is desirable to reduce cardiovascular
♦♦ Good source: Cereals, pulses, nuts, oil seeds
risk. Cholesterol intake should be < 300 mg/day.
♦♦ Fair sources: Meat, fish, egg, milk, vegetable and fruit.
♦♦ Fiber: Increased consumption of dietary fiber especially
Daily Requirement soluble fiber are associated with lower levels of blood
glucose and serum lipids. The water insoluble fibers such
♦♦ Infants: 0.3 to 0.5 mg
as cellulose, lignin and most hemicelluloses found in whole
♦♦ Children: 0.7 to 1.2 mg
grain breads, cereals and wheat bran affect gastrointestinal
♦♦ Adults: 1 to 1.5 mg
transit time and faecal bulk with little impact on plasma
♦♦ Pregnant woman: 1.3 to 1.5 mg.
glucose. However, highly viscous water soluble fibers
Physiological Role such as pectins, gums and storage polysaccharide found in
fruits, legumes, lentils roots, tubers, oat and oat bran, when
♦♦ Thiamine is essential for growth.
eaten in purified form, reduce serum levels of glucose and
♦♦ It is essential for maintaining the nerves in normal
insulin. Ideal recommended amount of fiber in patient’s
condition.
diet is 35–40 g/day.
Deficiency Manifestations ♦♦ Alternative sweeteners: Both nutritive and non-nutritive
sweeteners are acceptable in diabetes management.
♦♦ Beri-beri: It is a nutritional disorder. ♦♦ Sodium: It should be restricted to 1000 mg/1000 kcal,
• Wet-beriberi not to exceed 3000 mg/day to minimize symptoms of
–– Edema is the most prominent feature hypertension.
–– Anorexia and dyspepsia are present. ♦♦ Alcohol: It should be taken in moderation and may need
• Dry beriberi to be restricted entirely by person with diabetes and
–– Essential feature is polyneuropathy insulin—induced hypoglycemia, neuropathy, poor control
–– Muscles become wasted and weak and difficulty of glucose and lipids, or obesity.
in walking. ♦♦ Vitamins, minerals and antioxidants — intake should be
• Infantile Beriberi. encouraged.
♦♦ Wernick’s encephalopathy: There is involvement of brain
which is characterized by ataxia, ophthalmoplegia, Forbidden foods: Sugar, jam, jellies, honey, jaggery, tinned
confusion and disorientation. fruits and juices, sweets, chocolate, ice creams, pastries, glucose
♦♦ Korsakoff psychosis: It occurs due to involvement of drinks, foods made with sugar, pudding, sauces.
mammilary bodies and confabulates with amnesia.
Foods allowed in moderation: Bread of all kinds and chapattis
Presence of angular stomatitis. made from wheat or millets, plain biscuits, all fresh fruits, baked
Q.23. Write short note on diabetic diet.(Nov 2011, 3 Marks) beans, breakfast cereals.
Ans. The golden rule for a diabetic on diet therapy should be Free foods: All meat, fish, eggs (not fried), clear soup or meat
to eat little and more often. extracts; tea or coffee; vegetables such as cabbage, cauliflower,
• A heavy meal is not desirable since it gives rise to spinach, pumpkin, brinj al, lady’s finger, turnip, French beans,
rise in blood lipids. cucumber, lettuce, tomato, spring onions, radish, asparagus.
• One must time the meals, food must have variety Spices, salt, pepper and mustard; butter and margarine. Sugar
and monotony in diet be avoided. substitutes for sweetening.
• Foods should be adjusted in such a way that they
form part of the family ratios. Q.24. Give definition, etiology, sign, symptoms, complication
and management of diabetes mellitus.
Requirements are determined by the patient’s activity (Feb 2013, 12 Marks)
♦♦ Overweight NIDDM should be encouraged to establish Ans. Diabetes mellitus is a group of metabolic diseases
their weight within a desirable range. A reduction of characterized by hyperglycemia resulting from defects
approximately 500 kcal/ day can result in loss of 1–2 kg/ in insulin secretion, insulin action or both.
month.
♦♦ Carbohydrates: Carbohydrates should be taken in form of Etiology
starch and complex sugars. 100–300 gm of carbohydrates ♦♦ Heredity: It is inherited as mendelian type of recessive trait
should be spreaded over 3 meals, i.e. 60 gm each and is predominant in children born to parents who are
and 3 snacks, i.e. 30 gm each with half liter of milk. diabetic. Every person who has a history of diabetes in his
Unrefined carbohydrates should be substituted by refined or her family is a carrier of disease. The risk percentage in
carbohydrates to the extent possible. first degree relatives is generally up to 20% while in second
♦♦ Proteins: Recommended dietary allowance of 0.85 g/kg degree relatives it is about 5 % and the overall percentage
body weight for adult is an appropriate guide. of diabetes among the children of diabetics goes up to 10
Section 1:  General Medicine  159

%. Identical twins of a diabetic have almost 40 % chances ♦♦ In such patients if diabetic ketoacidosis is severe patient
of developing the disease. develop mental apathy, confusion and undergo coma.
♦♦ Obesity: Obese people are more prone to suffer from
diabetes probably because obesity imposes strain on the In Type II Diabetes Mellitus
islets of Langerhans and there is a relative deficiency of Symptoms
insulin. Obese also show a relative resistance to insulin due
to reduction in the number of insulin receptors on target Patient remains asymptomatic in the beginning. Disease is
cells. Obesity results due to uninhibited indulgence in detected during routine check up.
food and lack of physical activity and imposes a constant
Sign
stress on the pancreas. Thus these people are more prone
to get diabetes. In this multiple systems are affected, so considering this various
♦♦ Race: All races are involved and suffer from diabetes signs are:
though a number of factors operate in one ethnic group ♦♦ Eyes: Errors of refraction leads to frequent change in
or the other. Jewish race has been known to be more spectacles, premature formation of cataract, retinopathy,
commonly affected than others. Some communities are recurrent sty.
known to have less incidence of diabetes as compared ♦♦ Skin: Abscess, carbuncle, boils, non-healing wounds.
to others, but here the role of diet, physical exercise ♦♦ Gastrointestinal tract: Chronic diarrhea, malabsorption and
and environmental factors come into play. A certain dilatation of stomach.
community in Japan (Ainus) was known to have practically ♦♦ Cardiovascular system: Hypertension, ischemic heart
no or little diabetes probably due to undernourishment and disease, diabetic foot, cold extremities.
poverty, but with the rapid industrialization and boom in ♦♦ Respiratory system: Pneumonia, tuberculosis and lung
the economy of the community, the incidence of diabetes abscess.
among them has gone up. ♦♦ Nervous system: Autonomic neuropathy and peripheral
♦♦ Social and environmental factors: Diabetes has been neuritis.
considered a disease of civilization and its prevalence is ♦♦ Urinary tract: Nephrotic syndrome, urinary tract infection.
closely related to the economic affluence. Diabetes occurs ♦♦ Genital tract: Pruritis vulvae, menstrual irregularity and
more in richer and affluent classes of society though the infertility.
poorer class of people are equally liable to suffer from For complications refer to Ans 7 and for management refer
early onset diabetes. Diabetes in richer class of people is to Ans 8 of same chapter.
closely related to their eating habits, lack of physical effort
and obesity. Q.25. Write briefly on sign, symptoms and treatment of
♦♦ Exercise: Lack of physical effort and exercise promotes protein malnutrition. (May/June 2009, 5 Marks)
obesity and indirectly predisposes to diabetes. Physical Ans. Protein malnutrition is caused due to deficiency of
effort and leading an active life goes a long way in keeping proteins in body.
one self trim and helps in the proper utilization of body
Symptoms
glucose and maintains a homeostatic balance.
♦♦ Diet: Excessive intake of carbohydrates and refined sugars ♦♦ Patient has high urge for food and water.
produces strain on the pancreas and this combined with ♦♦ Patient becomes weak, exhausted and fatigued.
sedentary occupation goes a long way in predisposing to ♦♦ Presence of loss of libido
diabetes. ♦♦ Patient feels cold
♦♦ Parity: Women with repeated pregnancies are more liable ♦♦ Amenorrhea is present.
to develop diabetes since too many pregnancies are a ♦♦ Patient looks old and mature and face is expressionless.
strain on the carbohydrate metabolism and often there is ♦♦ Diarrhea may occur.
hormonal imbalance. ♦♦ Patient in inactive and is depressed.
♦♦ There is loss of libido
Sign and Symptoms / Clinical Features
Signs
In Type I Diabetes Mellitus ♦♦ Muscle wastening is present.
♦♦ Extremities of patient are cold.
Symptoms ♦♦ Skin is dry, lustureless and become fissured.
♦♦ Polydypsia, polyuria and polyphagia ♦♦ Subcutaneous fat is absent and bony prominences are
♦♦ Weight loss prominent.
♦♦ Weakness and lassitude. ♦♦ Circumference of arm is subnormal.
♦♦ In females edema is present which is not due to
Sign hypoalbuminemia.
♦♦ Severe emaciation with wastening of muscles ♦♦ Body temperature is subnormal
♦♦ Ribs are prominent. ♦♦ Tendon jerks become diminished.
160   Mastering the BDS IIIrd Year  (Last 25 Years Solved Questions)

♦♦ Pulse of patient is slow and blood pressure becomes low. Q.26. Discuss the clinical features, diagnosis and management
♦♦ Abdominal distention is commonly present. of diabetes mellitus.
 (Jan 2018, 12 Marks)(Jun 2010, 15 Marks)
Treatment Ans. For clinical features refer to sign and symptoms of Ans
24 of same chapter. For management refer to Ans 8 of
In Adults
same chapter.
♦♦ For mild starvation adequate supplementation of nutrients
is necessary Diagnosis
♦♦ In moderately starved people extra-feeding is needed Diagnosis is based on symptoms, signs and laboratory tests. In
♦♦ In severely starved persons food is given in small amount presence of signs and symptoms confirmation is done by finding
at frequent intervals. Food should be staple meal, i.e. cereal random blood glucose higher than 200 mg/dL.
with some sugar, milk and oils, salt is restricted. Potassium,
magnesium and vitamins are adequately given. This is Investigations
continued till patient feel active. ♦♦ Blood sugar estimation: Both fasting and post-prandial
(2 hours after a meal) levels of blood sugar be estimated.
In Children Mean value of blood sugar in healthy adults is 70 to 80
mg%. When fasting blood sugar level exceeds 110 mg%
♦♦ Diet
• Diet for mild to moderate cases: Diet should be easily diagnosis is clear. Post-prandial blood sugar level is further
digestable. It should be rich in proteins, minerals and screening test, if value exceeds 120 mg% it is strongly
vitamins with extra calories. Milk should be given. suggestive of diabetes mellitus. Random blood sugar
Egg is a good flip with milk and water as a source of estimation is not of much help as screening test. Value
first class protein. Plant protein mixtures such as corn, exceeding 160 mg% in presence of glycosuria is suggestive
soya, diet milk can be given. Additional fats should be but values about 200 mg% is diagnostic.
included. If there is lactose intolerance dal, rice and ♦♦ Oral glucose tolerance test: In this first sample is taken
butter can be given. after an overnight fast of 8 hours following which patient
• Diet for acute cases: Protein intake should be 3 to 4g/ is given glucose 1g/kg body weight. Blood is then collected
kg/day and energy intake of 150kcal/Kg/day. If child at 30, 60, 120 and 180 minutes. In diminished glucose
is unable to take diet from mouth go for nasogastric tolerance, level of blood glucose is raised at 180 minutes.
intubation. Care must be taken to take sufficient ♦♦ Glycosylated hemoglobin: It measures the long-term
amount of plant, animal proteins and fats to maintain glycemic control. Slow non-enzymatic attachment of
the weight. glucose to hemoglobin result in formation of glycosylated
♦♦ Vitamins and mineral supplementations: hemoglobin.
• Vitamin A, D, B complex and C are given at therapeutic
♦♦ Microalbumin: Patients with microalbumineria are
dosages.
on greater risk in developing kidney failure as well as
• In vitamin A deficiency 30 mg of vitamin A should be
cardiovascular damage.
given for 3 days.
♦♦ Urine protein/creatinine ratio: It provides information of
• 0.5 to 1gm of potassium chloride dissolve in feeds or
water should be given daily in divided dosages. proteinuria in patients with diabetic nephropathy. More
♦♦ Maintainence of body temperature is the ratio greater is the damage.
For maintainence of body temperature at night blankets ♦♦ Insulin levels: Elevated blood glucose levels with low
or room heaters are useful. insulin levels indicate insufficient insulin levels for
♦♦ Skin care adequate control of blood glucose. High insulin levels
In cases of dermatoses skin should be kept clean and with low blood glucose indicate change in dosage of drug.
protected. ♦♦ Insulin antibody test: In this there is quantitative
♦♦ In hospital treatment: determination of antibodies against insulin in serum.
• Correction of dehydration, electrolyte disturbances, ♦♦ GAD – 65 antibody: These antibodies are common
hypoglycemia, acidosis and hypothermia. in newly diagnosed diabetic patient and often appear
• Parenteral therapy with half of saline and 2.5% glucose years before clinical onset in the disease. Presence of this
before diet therapy. antibody is the strong predictive marker for onset of type
• In anemia packed erythrocyte infusion is given. I diabetes mellitus.
Section 1:  General Medicine  161

Criterias given by WHO in 1999 • Management during hour 2


♦♦ Presence of classic symptoms of diabetes and causal – Normal saline should be continued at 500 mL/
plasma glucose greater than or equal to 200 mg/dL. Causal hour. Plasma osmolality should be greater than
is any time of day without regard to time since last meal. 285 mosmol / L during first 12 hours. If serum
Classic symptoms of diabetes include polyuria, polydypsia sodium is greater than 150 mmol/L switch to
and unexplained weight loss. 0.45% saline.
Or – Blood glucose should be checked and insulin
♦♦ Fasting plasma glucose greater than or equal to 126 mg/dL. is adjusted to 5 mmol/L/hour. Blood glucose
Fasting is defined as no caloric intake for 8 hours. should not fall below 11.1–14.0 mmol/L. Anion
Or gap is decreasing and blood pH is increasing.
♦♦ Two hours plasma glucose greater than or equal to 200 mg/dL – Serum potassium should be maintained at 4–5
during an oral glucose tolerance test. Test is performed as mmol/L by addition of potassium to IV fluids.
described by World Health Organization by using glucose • Management during hours 3 to 4
load containing equivalent of 75 gm anhydrous glucose – Management should be continued as given in
dissolved in water. second hour.
In absence of unequivocal hyperglycemia with acute – Cognitive and neurological symptoms should
metabolic decompensation above criterias are confirmed by
be observed for 12 hours.
repeated testing on other days.
• Management during hours 5 to 8
Q.27. Write sign and symptoms of diabetes mellitus. – Normal saline should be continued at 250 mL/
 (Aug 2012, 5 Marks) hour. As blood glucose reaches 11.1–14 mmol/
Or lt, change IV fluid to 500 mL/hour normal saline
Describle clinical features of diabetes mellitus. with 5% glucose.
 (Feb 2013, 5 Marks) – Insulin should be continued at maintenance dose
Ans. Refer to Ans 24 of same chapter. until ketoacidosis is cleared.
Q.28. Write management of diabetic ketoacidosis. – Potassium should be continued at 10–40 mmol/
 (Mar 2013, 4 Marks) hour since ketoacidosis is cleared.
Ans. Following is the management of diabetic ketoacidosis – Phosphate replacement is done at 6 hours if
• Admission of patient serum phosphate is < 2 mg/dL
– Diagnosis is confirmed by examination of blood • Management during hours 8 to 24
glucose and ketone measurements. – IV repletion is continued with 0.45% saline with
– Initial assessment of dehydration, hyperosmo­ or without 2.5% or 5% glucose as needed.
lality, serum potassium, acidosis and kidney – As ketoacidosis is subsided patient should be
function is done. switched to sub-cutaneous insulin and stop IV
• Fluid loss is measured by subtracting admission
and IM insulin.
weight by last known stable weight.
– Patient should be evaluated for sepsis. Q.29. Write classification, clinical features, investigations and
• Management during hour 1 treatment of diabetes mellitus. (Jun 2014, 12 Marks)
– If patient is hypovolemic and hypotensive fluid Ans. Diabetes mellitus is a clinical syndrome of hyperglycemia
administration is done, i.e. normal saline is with glycosuria due to lack of insulin or insulin resistance
administered and if necessary colloids should is termed as diabetes mellitus.
be given. Rate of administration is necessary to
restore circulatory function. As blood pressure Classification
become normal and urine output becomes
adequate rate of administration of normal saline Primary Diabetes
is 1000 mL/hour. ♦♦ Type I insulin dependent diabetes mellitus (IDDM). It is
– Continuous IV infusion of regular insulin 5–10 sub-divided:
units/hour is given. • Immuned mediated (islet cell antibodies).
– IV infusion of potassium is done at 10–40 mmol/ • Non-immune (no antibody).
hour during initiation of insulin therapy. ♦♦ Type II non-insulin dependent diabetes mellitus (NIDDM)
– Sodium bicarbonate IV is given in cases with It is sub-divided:
acidosis. Dose is 50–100 mmol/L sodium • Obese (insulin resistance with relative insulin
bicarbonate in 0.45% saline for 30–60 minutes. deficiency).
Additional potassium should also be given with • Non-obese (insulin secretory detect with insulin
bicarbonate therapy. resistance).
162   Mastering the BDS IIIrd Year  (Last 25 Years Solved Questions)

Secondary Diabetes Q.30. Write short note on obesity. (Feb 2013, 5 Marks)
Ans. Obesity is the condition in which there is excessive
♦♦ Pancreatic diabetes due to pancreatitis.
accumulation of body fat.
♦♦ Hormonal or endocrinal abnormalities, i.e., acromegaly,
shing’s syndrome, pheochromocytoma, etc. Obesity is the common disorder of the nutrition of
♦♦ Drugs induced (Iatrogenic) due to steroids and thiazides. affluent societies.
♦♦ Insulin receptors antibodies. Etiology
♦♦ Genetic syndromes, i.e., Lipodystrophies, muscular
dystrophies, Klinefelter’s syndrome, Turner’s syndrome, ♦♦ Genetic and Environmental factors: Genetic and environmental
Down’s syndrome, DIDMOAD (Diabetes insipidus, factors play important role in causing the obesity. Familial
diabetes mellitus, optic atrophy and deafness) syndrome. predisposition is also seen. Overeating and other factors
also cause obesity. If both the parents of a child are
Clinical Features overweight then chances of child being obese are 80%.
For clinical features refer to Ans 24 of same chapter. ♦♦ Socioeconomic status: Obesity is the disorder of affluent
societies. Some occupations also predisposes to obesity,
Investigations such as cooks. In some of the societies obesity is considered
♦♦ Blood sugar estimation: Both fasting and post-prandial (2 as the sign of heritage.
hours after a meal) levels of blood sugar be estimated. ♦♦ Energy intake: Minute excess of calories causes accumulation
Mean value of blood sugar in healthy adults is 70 to 80 of fat. If there is lag of physical activity, this also causes
mg%. When fasting blood sugar level exceeds 110 mg% obesity.
♦♦ Drugs: Various drugs such as steroids, oral contraceptives,
diagnosis is clear. Post-prandial blood sugar level is further
etc causes increase in apetite, this leads to weight gain.
screening test, if value exceeds 120 mg% it is strongly
♦♦ Endocrine: During pregnancy there is increase in the body
suggestive of diabetes mellitus. Random blood sugar
fat which is under hormonal influence. Obesity can also
estimation is not of much help as screening test. Value
occur in hypothyroidism due to decrease calorie demands.
exceeding 160 mg% in presence of glycosuria is suggestive
but values about 200 mg% is diagnostic. Clinical Features
♦♦ Oral glucose tolerance test: In this first sample is taken after
an overnight fast of 8 hours following which patient is ♦♦ Type II diabetes mellitus, gallbladder stones, gout and
various other metabolic disorders are associated with
given glucose 1 g/kg body weight. Blood is then collected
obesity.
at 30, 60, 120 and 180 minutes. In diminished glucose
♦♦ Varicose vein, hernia and osteoarthritis are the mechanical
tolerance, level of blood glucose is raised at 180 minutes.
disorders associated with obesity.
♦♦ Glycosylated hemoglobin: It measures the long-term glycemic
♦♦ Various respiratory infections are also associated with the
control. Slow non-enzymatic attachment of glucose to
obesity along with the sleep disorders.
hemoglobin result in formation of glycosylated hemoglobin.
♦♦ Hypertension, atherosclerosis and ischemic heart diseases
♦♦ Microalbumin: Patients with microalbuminria are on greater
are the cardiovascular diseases which are associated with
risk in developing kidney failure as well as cardiovascular
the obesity.
damage.
♦♦ Urine protein/Creatinine ratio: It provides information of Management
proteinuria in patients with diabetic nephropathy. More
♦♦ Diet: Calories uptake should depend on the age, sex,
is the ratio greater is the damage. occupation and urgency of the weight reduction. An obese
♦♦ Insulin levels: Elevated blood glucose levels with low person should spare from fats, butter, ghee, cream dryfruits
insulin levels indicate insufficient insulin levels for (nuts) vegetables (potatoes and peas). Person should follow
adequate control of blood glucose. High insulin levels strict diet regime and maintain discipline in choice of foods
with low blood glucose indicate change in dosage of drug. to which he/she takes and the amount of calories consumed
♦♦ Insulin antibody test: In this there is quantitative per day. Food should be taken in intervals, chewed slowly.
determination of antibodies against insulin in serum. ♦♦ Exercise: Habit of regular physical exercise should be
♦♦ GAD – 65 antibody: These antibodies are common in newly cultivated by an obese person. An obese person should
diagnosed diabetic patient and often appear years before do morning and evening walks, jogging, stretch exercise
clinical onset in the disease. Presence of this antibody is the and taking part in outdoor games.
strong predictive marker for onset of type I diabetes mellitus. ♦♦ Drug therapy: In this recent era serotonergic drugs are used.
DL-fenfluramine which was the first generation drug was
Treatment
used to reduce weight. These days it is replaced by second
For treatment refer to Ans 8 of same chapter. generation drugs dL-fenfluramine, fluoxetine which is a
Section 1:  General Medicine  163

slow releasing serotonin antagonist, sibutramine which than140 mg, then it warrants second step of administration of
is a slow serotonin and noradrenaline reuptake inhibitor. 100 g glucose and measure the serum glucose in fasting state
These drugs increase the satiety and potentiate serotonin and at 1, 2 and 3 hrs. Normal plasma glucose concentration is
in hypothalamus. given below:
♦♦ Surgical treatment: This method is employed in the persons ♦♦ Fasting <105 mg/L.
having severe obesity. The surgical methods employed are: ♦♦ At 1 hr < 190 mg/L.
• Wiring of jaws: This is carried out to prevent eating and ♦♦ At 2 hr <165 mg/L
only taking liquids. This method is good but as wires ♦♦ At 3 hr <145 mg/L.
are removed patient regains the weight. Deviations from the normal indicate gestational diabetes
• Gastric plcation: In this stomach is reduced by creating mellitus.
a small pouch due to stapling of stomach to abdominal
wall. Good results can be attained. Management
• Bariatric surgery: It is done in persons with extreme ♦♦ High dietary protein intake. Salt restriction if edema is
obesity. present.
Q.31. Write short note on gestational diabetes. ♦♦ Exercise should be done regularly
 (Nov 2014, 3 Marks) (Dec 2015, 3 Marks) ♦♦ Insulin therapy is essential.
Ans. Gestational diabetes is defined as development of ♦♦ Oral hypoglycemic should be avoided except metformin
hyperglycemia, first time in pregnancy. which is effective in women with PCOD to aid conception.
• It occurs in women who are genetically predisposed Q.32. Describe briefly on Type II diabetes.
to type 1 and type 2 diabetes.  (Apr 2015, 4 Marks)
• The condition is asymptomatic. Ans. Type II diabetes is also known as non- insulin dependent
• Repeated hyperglycemia in every pregnancy lead diabetes mellitus.
the women to undergo suffering from permanent
diabetes. Etiology
• Gestational diabetes can occur in any women, so at ♦♦ Genetic: It is inherited as mendelian type of recessive trait
the time of pregnancy during each trimester random and is predominant in children born to parents who are
blood sugar screening is done by oral glucose diabetic. Every person who has a history of diabetes in his
tolerance test. In this test blood glucose is more than or her family is a carrier of disease. The risk percentage
126 mg%. in first degree relatives is generally up to 20 %t while in
Obstetric Complications Associated with Gestational second degree relatives it is about 5 %. Type II diabetes
Diabetes Mellitus is caused due to abnormal insulin secretion and insulin
resistance.
♦♦ Maternal risk ♦♦ Due to pancreatic beta cell failure: In Type II diabetes there
• Maternal hypertension is reduction of pancreatic beta cell mass with reduction in
• Eclampsia insulin levels. As beta cells decreases alpha cells secrete
• Nephropathy, retinopathy and vascular complications glucagon and this causes hyperglycemia.
can worsen. ♦♦ Environmental factors: Physical inactivity and obesity
♦♦ Fetal risk are the factor leading to type II diabetes in genetically
• Stillbirth susceptible individuals.
• Neonatal hypoglycemia
♦♦ Age: Type II diabetes occur after age of 30 years.
• Neonatal hypocalcemia
• Macrosome Clinical Features
• Hyaline membrane disease
Symptoms
• Growth retardation
• Congenital anomalies Patient remains asymptomatic in the beginning. Disease is
detected during routine check up.
Diagnostic Criteria
Sign
Condition Glucose load FBS PDG (Oral glucose
tolerance test) In this multiple systems are affected, so considering this various
IGT 75 gm NA 140 mg/dL signs are:
♦♦ Eyes: Errors of refraction leads to frequent change in
DM 75 gm 140 mg/dL 200 mg/dL
spectacles, premature formation of cataract, retinopathy,
A two step strategy has been proposed for establishment recurrent sty.
of diagnosis of gestational diabetes. First step is to give 50 mg ♦♦ Skin: Abscess, carbuncle, boils, non-healing wounds.
oral glucose and measure serum glucose at 60 min; if glucose is ♦♦ Gastrointestinal tract: Chronic diarrhea, malabsorption and
found less than 140 mg then test is normal and if found greater dilatation of stomach.
164   Mastering the BDS IIIrd Year  (Last 25 Years Solved Questions)

♦♦ Cardiovascular system: Hypertension, ischemic heart ♦♦ Insulin sensitizers, i.e.drugs sensitizing action of insulin
disease, diabetic foot, cold extremities. and overcome insulin resistance
♦♦ Respiratory system: Pneumonia, tuberculosis and lung • Biguanides, i.e. metformin should be given 1.5 to
abscess 2.5gm/day in three divided doses after meals.
♦♦ Nervous system: Autonomic neuropathy and peripheral • Thiozolidinediones, i.e.Rosiglitazone should be given
neuritis 2 to 8mg or pioglitazone 15 to 45 mg in a single or two
♦♦ Urinary tract: Nephrotic syndrome, urinary tract infection. divided doses. They can be combined with sulfonyl
♦♦ Genital tract: Pruritus vulvae, menstrual irregularity and ureas or metformin.
infertility. ♦♦ Alpha glucosidase inhibitors, i.e. Acarbose 25 to 100 mg
TDS or Voglibose 0.2 to 5 mg TDS taken orally in three
Management main meals. It can be given in combination with metformin
500 mg for increased efficiency.
♦♦ Low energy and weight reducing diet is given to the patient ♦♦ DPP – 4 inhibitor: Vildagliptin 50 to 100 mg daily in two
of obese type II diabetes mellitus. dived doses with meals ot saxagliptin 2.5 to 5 mg OD.
♦♦ Non-obese type II diabetes should have to maintain their ♦♦ Pramlinide: Initial dose is 15μg before each meals in
weight, so they have to take weight maintenance diet. Type I diabetes mellitus and 60 μg in Type II diabetes
These drugs are used in patients of Type II diabetes mellitus mellitus.
(NIDDM) who do not respond to dietary management and
Q.33. Write short note on tetany.
who would otherwise require treatment with insulin in later
 (Apr 2015, 3 Marks)(Mar 2016, 3 Marks)
situation, so they are also used as adjuvant drugs to insulin in
overweight diabetes patients. Ans. Tetany is a clinical condition with low levels of ionized
calcium causing increased neuromuscular excitability.
♦♦ Insulin secretagogues, i.e. drugs increasing secretion of
insulin Etiology
• Sulfonyl ureas
Low levels of calcium, potassium, magnesium causes tetany.
Name of drug Intial daily Dose/ Comment
♦♦ Hypocalcemia: This is due to following factors, i.e. malab­
dose (mg) day
sorption of calcium, osteomalacia, hypoparathyroidism,
First generation acute pancreatitis, anti-convulsant drugs.
Acetohexamide 250 mg 1–2 Has diuretic and ♦♦ Hypokalemia: This is caused due to repeated vomiting,
uricosuric activity excessive intake of alkali, due to acute anion load, primary
Chlorpropamide 100 mg 1–2 Can potentiate ADH hyperaldosteronism, due to acute anion load.
♦♦ Acute hypocalcemia is caused by sepsis, burns, alkalosis,
Tolazamide 100 mg 1–2 Disulfiram-like action parathyroid surgery, malignancy, hypomagnesemia, etc.
with alcohol in 1/3rd of
pts, has diuretic activity Clinical Features
Second generation ♦♦ In children there is presence of triad of symptoms, i.e.
Glibenclamide 2.5 to 20 mg 1–2 Hypoglycemia can be carpopedal spasm, stridor and convulsion.
severe ♦♦ In carpopedal spasm the hands adopt a posture in which
Gliclazide 40 to 320 mg 1–2 Metabolism/excretion flexion occurs at metacarpophalangeal joint and extension
by liver/kidney at interphalangeal joint and opposition of thumb occur.
Glipizide 5 mg 1–2 Mild diuretic activity ♦♦ Stridor occurs due to closure of glottis.
♦♦ In case of adults tingling sensation occur in peripheral part
Glyburide 2.5 mg 1–2 Highest risk of
of limbs and around the mouth.
hypoglycemia
♦♦ Trousseau’s sign: Increasing the blood pressure by the
Glimepiride 1 mg 1 Excretead in urine and inflation of sphygmomanometer cuff at above the systolic
bile
level leads to carpopedal spasm in 2 to 5 minutes.
• Meglitinides Chvostek’s sign: Tapping at the facial nerve especially at angle
–– Repaglinide, i.e. 0.5 to 4 mg three times a day of mandible leads to twitching of facial muscles.
–– Netaglinide, i.e. 60 to 120 mg three times a day 15
Management
to 30 min before each meal
–– Voglibose, i.e. 0.2 to 0.3 mg, 15 to 30 min before In management etiological factor leading to tetany is eliminated
each meal. and hypocalcemia and alkalosis should be treated.
Section 1:  General Medicine  165

Treatment of Hypocalcemia as complication of septic state or due to


involvement of neurotrophic virus. The pain is
10% calcium gluconate 20 mL IM is given. For long lasting effect
confined to face and scalp.
10 mL of drug is given additionally.
– Arthralgia of TMJ: It is in the form of rheumatic
Treatment of Alkalosis arthritis or ankylosing spondylitis (a chronic
progressive inflammatory disorder unlike other
♦♦ If vomiting is present IV isotonic saline should be given. rheumatological disorders) when there is pain
♦♦ If alkali is the etiological factor, alkalis should be and swelling of joint.
withdrawn, after withdrawal if effect is not proper – Trigeminal neuralgia: This is a disease seen
ammonium chloride 2 gm orally is given after every 4 commonly in middle and elderly individuals
hours. and is characterized by attacks of sever pain in
♦♦ If hyperventilation is the etiological factor, patient is asked distribution of trigeminal nerve and its branches
to inhale 5% carbon dioxide in oxygen. especially in maxillary and mandibular branches.
Q.34. What are the non-communicable diseases. Write – Postherpetic neuralgia: Herpes zoster commonly
down the diagnostic criteria of diabetes mellitus, involves the ophthalmic division of 5th nerve,
complication and management of type-2 diabetes characterized by vesicular eruption on the face.
mellitus. (June 2015, 12 Marks) – Temporal arthritis: It is a form of collagen disorder
Ans. Non-communicable disease is a medical condition or of unknown etiology which involves mainly
disease that is non-infectious or non-transmissible. the arteries. It occurs in elderly age group.
Non-communicable disease can refer to chronic diseases Patient may complain of pain on the face, jaw,
which last for long periods of time and progress slowly. mouth and tongue in distribution of branches of
The most non-communicable diseases are obesity, external carotid artery.
diabetes, cancer, cardiovascular, chronic respiratory and – Miscellaneous causes: These include lesions of
neurological diseases. trigeminal nerve in brain stem, *syringobulbia
and thrombosis of posterior inferior cerebellar
For diagnostic criteria of diabetes mellitus refer to Ans
artery. Tabes dorsalis is another cause of pain
26 of same chapter.
coming in attacks over the face.
For complications of diabetes mellitus refer to Ans 7 of
same chapter. Q.2. Write short note on trigeminal neuralgia.
 (Mar 2001, 5 Marks) (Apr 2007, 5 Marks)
For management of diabetes mellitus refer to Ans 32 of
 (Feb 2013, 5 Marks) (Sep 2007, 5 Marks)
same chapter.
 (Apr 2017, 6 Marks) (Mar 2011, 4 Marks)
Q.35. Write diagnosis, clinical features and complications of Ans. Trigeminal neuralgia is also called as Tic Douloureux.
diabetes mellitus. (Jan 2016, 12 Marks) A disorder characterized by the *paroxysmal attacks of
Ans. Diagnosis is based on investigations, for investigations neuralgic pain with affection of one or more division of
in detail refer to Ans 29 of same chapter and for criteria trigeminal nerve. The pain involves the third and second
of diagnosis refer to Ans 26 of same chapter divisions equally and rarely the first.
For clinical features of diabetes mellitus refer to Ans 24
of same chapter.
For complications of diabetes mellitus refer to Ans 7 of
same chapter.

10. Diseases of
Nervous System
Q.1. Enumerate causes of facial pain.(Sep 2008, 2.5 Marks)
Ans. Causes of facial pain are: Fig. 22:  Trigeminal neuralgia

• Neuritis of cutaneous nerves of face and scalp.


• Arthralgia of temporomandibular joint Clinical Features
• Trigeminal neuralgia. ♦♦ Pain is unilateral and is confined to one of the three
• Post herpetic neuralgia. divisions of nerve. Pain is sharp and onset is sudden. The
• Temporal arthritis. pain is only of a few seconds.
• Miscellaneous causes: ♦♦ During the attacks there is flushing of face, i.e. redness
– Facial neuralgia: It is a form of inflammation of the face.
of nerve of face and scalp. It generally occurs ♦♦ Dilatation of pupil is present.
166   Mastering the BDS IIIrd Year  (Last 25 Years Solved Questions)

♦♦ There is excessive lacrimation Lower Motor Neuron Palsy or Bell’s Palsy


♦♦ After repeated attacks skin becomes shiny and hair in the
Refer to Ans 4 of same chapter.
area becomes gray.
♦♦ Sometimes secretion of nasal mucus and saliva may occur Q.4. Write short note on Bell’s palsy.
in the side of pain.  (Dec 2013, 3 Marks) (Apr 2015, 3 Marks)
 (Dec 2010, 5 Marks) (Sep 2006, 5 Marks)
Etiology Or
Trigeminal neuralgia is spontaneous and following exposure Write causes and clinical features of Bell’s palsy.
to cold wind, blow on face, or chewing or eating, drinking hot (Jan 2012, 6 Marks)
or cold fluid and washing the face. Ans. Bell’s palsy is an acute apparently isolated, lower motor
neuron facial palsy.
Management Etiology
♦♦ Elimination of all possible sources of infection. ♦♦ Cold: It usually occurs after exposure to cold.
♦♦ Drugs: ♦♦ Trauma: Extraction of teeth or injection of local anesthetic
• Analgesics: Potent analgesics must be used with may damage to the nerve and subsequent paralysis.
caution because of danger of habituation. ♦♦ Surgical procedure: Such as removal of parotid gland tumor
• Carbamazepine: 100–200 mg BD a day and increasing in which the facial nerve is sectioned can also cause facial
the dose to 600–800 mg per day. paralysis.
• Phenytoin sodium: 0.1 gm TDS when carbamazepine ♦♦ Tumors: Tumors of the cranial base, parapharyngeal space
is not tolerated. and infratemporal fossa after cause 7th nerve palsy.
• Vitamin B12: 1000 μg IM daily for two weeks. ♦♦ Familial: Familial and hereditary occurrence is also
♦♦ Injection of alcohol: It is given in affected nerve, or gasserian reported in case of Bell’s palsy.
ganglion. If more than one division is affected inject 10 ♦♦ Facial canal and middle ear neoplasm.
minims of 90% alcohol after local anesthesia with 2–3 ♦♦ Herpes simplex—viral infection.
drops of procaine.
♦♦ Microvascular decompression: In this there is separation of Clinical Features
blood vessels is done which are in contact with trigeminal ♦♦ Symptoms:
nerve roots. There is also insertion of non – absorbable • Sudden following exposure to chill or without any
sponge which provide relief of pain in most of the patients. apparent precipitating causing, maximum paralysis
♦♦ Radiofrequency thermocoagulation: This procedure is in 24 hours.
carried out at the trigger spot or site of pain origin which • Postauricular pain is common.
is localized by electric stimulation of needle inserted in • Spontaneous complaints of loss of sense of taste,
trigeminal ganglion and leads to permanent relief. *hyperacusis.
♦♦ Surgery: In this selective or complete preganglionic section • Sweating is less on the affected side.
of trigeminal root is done. This technique can lead to ♦♦ Signs:
disadvantages such as permanent dysesthesiae. Another • Forehead is not wrinkled and frowning is lost.
better technique is percutaneous electrocoagulation of • Eye of the affected side is not closed and on attempting
preganglionic rootlets corresponding to trigger zone, the closure eyeball turns upwards and outwards.
temperature of probe being so regulated as to coagulate • On showing teeth the lips do not separate on the
small thinly myelinated pain fibers but preserving most affected side. Whistling is not possible. Nasolabial
heavily myelinated touch fibers. fold is flattened out. Angle of mouth on affected side
Q.3. Write short note on facial palsy. (Mar 2008, 7 Marks) droops with dribbling of saliva.
 (Sep 2005, 5 Marks) (Sep 1999, 5 Marks) • Cheeks puff out with the expiration because of
Ans. Facial palsy refers to the paralysis of facial muscles. buccinator paralysis and food collects between the
It is of two types: teeth and paralyzed cheek. Fluid runs out while
1. Upper motor neuron palsy drinking.
2. Lower motor neuron palsy or Bell’s palsy: • Base of tongue is lowered.
• Deafness may result.
Upper Motor Neuron Palsy
Investigations
♦♦ It affects mainly muscles of lower part of face and is never
complete. Electromyography is of prognostic importance.
♦♦ It is seldomly isolated palsy. ♦♦ If signs of denervation are present after 10 days, i.e. axonal
♦♦ The emotional movements are preserved. degeneration is present and recovery is incomplete or
♦♦ There is no muscle *contracture. delayed.
♦♦ There is no reaction of degeneration. ♦♦ If there is incomplete denervation in less than 7 days the
♦♦ Electromyography and nerve conduction is normal. prognosis is good.
Section 1:  General Medicine  167

♦♦ Fibrillation potential after 2 weeks is suggestive of ♦♦ Corticosteroids: If seen under a week of onset. Prednisolone
wallerian degeneration. 40mg/day for 4 days and in tapering doses for over next 6
Management days helps by reducing secondary edema.
♦♦ Anti–virals: Acyclovir, Valacyclovir or Famciclovir in
♦♦ Local heat: Infrared or moist heat over the face or parotid combination with steroids, if started within 3 days of onset.
region or both if there is tenderness of nerve trunk. ♦♦ Surgery: Decompression of facial nerve in second or third
♦♦ Local treatment of muscles: The patient should massage the week cannot influence favorably natural course of Bell’s
facial muscles with bland oil for twice a day for 5 min. palsy. Cases which fail to recover after 9 months in them
The massaging movements should start from the chin and anastomosis of facial nerve with accessory or preferably
lower lip and are directed upwards. With return of function hypoglossal nerve is considered, or plastic surgery in cases
the patient should practice movements of various muscles of total paralysis with atrophy of muscle.
of face before a mirror.
♦♦ Prevention of facial sagging: Application of strips of adhesive Q.5. Write short note on migraine.
tape is done to lift up the angle of mouth. Tape is attached  (Feb 1999, 5 Marks) (Sep 1998, 5 Marks)
to the temple and extends down in a V shaped fashion to  (Dec 2015, 3 Marks) (Mar 2009, 5 Marks)
upper and lower lips. Ans. Migraine is defined as recurrent attacks of headache
♦♦ Protection of eye: It is done with dark glass or eye patch. varied in intensity frequency and duration and is
Mild zinc boric solution is used to wash the eye to prevent commonly unilateral in onset and is associated with
conjunctivitis. anorexia and sometimes with nausea and vomiting.

Fig. 23:  Pathways of migraine


168   Mastering the BDS IIIrd Year  (Last 25 Years Solved Questions)

Clinical Features • Drugs:


♦♦ The headache follows and the pain is confined on one side –– Serotonin (5-HT) inhibitors: Calcium antagonists
by occasionally it may be bilateral. such as Flunarizine 10 mg/day Or Cyproheptadine
♦♦ Nausea and vomiting may be present and last for the few 4 mg tds. Or Pizotifen 0.5 mg tds or 1.5 mg nocte.
hours. Or Methysergide 1–2 mg t,d.s. is the most effective
♦♦ Light and noise sensitivity is present. drug in this group, but should be used under
♦♦ Fatigue and stress are present in the case. supervision in courses not exceeding 3–4 months.
♦♦ There is presence of polyuria. –– Topiramate: It is an antiepileptic drug used in
prophylaxis of migraine, Dose: 2.5 to 5 mg BD
Simplified Diagnostic Criteria for Migraine –– Divalproex (valproic acid) 200 mg BD
–– Tricyciic agents: Amitryptiline 25 mg TDS may
Repeated attacks of headaches lasting for 4 to 72 hours which
be effective irrespective of the presence of
have features:
depression.
♦♦ Normal physical examination
–– Ergotamine tartrate: For histamine cephalgia 1
♦♦ No other reasonable cause for headache
mg by mouth or 0.25 mg by self-administered
♦♦ At least two of:
injection or by suppository used regularly last
• Unilateral pain
thing at night can be continued for many weeks
• Throbbing pain
without harmful effects, 2 days being left without
• Aggravation of pain by movement
treatment each week.
• Moderate or severe intensity
–– Hormones: Progesterone given for last eight days
♦♦ At least one of:
may be useful for migraine occurring in the
• Nausea or vomiting
immediate premenstrual period or at beginning
• Photophobia or phonophobia
of catamenia. When migraine begins or becomes
Management worse at the time of menopause, estrin, given in
small doses as continuous therapy sometimes
♦♦ During attack helpful.
• Analgesics: NSAIDs, e.g. diclofenac can be given orally –– Schedule:
or im and is particularly useful when severe vomiting - Propranolol 40–160 mg/day or Flunarizine 5–10
is a feature. Sublingual Piroxicam has significant mg/day as first line of therapy.
analgesic effects in acute migraine without aura with - In patients with episodic and chronic migraine
excellent tolerability. Topiramate 50–100 mg/day.
• Ergotamine: Ergotamine tartrate 0.25 to 0.5 mg IM or - In patients with episodic migraine Divalproex
orally 1–2 mg. tablet preferably in combination with 250–750 mg/day.
100 mg caffeine –2 tablets at onset followed by l tablet - For mixed migraine and tension type headache:
after 30 minutes, if necessary, or Dihydroergotamine 1 Amitryptiline 10–25 mg/ day.
mg IM, or 1–2 mg by mouth. Whichever preparation is After 6–12 months of prophylaxis gradual
used, a high dose often causes nausea and vomiting. withdrawal should be considered.
These may be prevented by giving cyclizine 50 mg.
Q.6. Classify headache. Discuss etiology, symptoms, signs,
or chlorpromazine 25 mg.
pathogenesis of cluster headache.(Mar 1994, 15 Marks)
• 5-HT1 agonists: Sumatriptan 6 mg s.c. gives relief
from headache in 60 minutes, with corresponding Ans.
improvement in nausea, vomiting and photophobia.
Oral dose of 100 mg provides relief within 2 hours. Classification of Headache
Headaches recur within 48 hours in little less than The classification is given by International Headache Society,
half the patients. Rizatriptan given orally acts faster which is as follows:
than sumatriptan. Zolmitriptan nasal spray 5 mg gives ♦♦ Migraine (Classical common, opthalmoplegic and basilar
relief in 5 minutes. artery)
• General: Lying in a darkened and quiet room and ice ♦♦ Tension type, headache (e.g. episodic or chronic)
pack to the head may help. ♦♦ Cluster headache and chronic hemicrania
♦♦ Reducing frequency and severity of subsequent attacks ♦♦ Miscellaneous headache not associated with structural
• Elimination of trigger factors: Sleeping late, irregular and lesion (e.g., cold stimulus induced, cough headache,
hurried meals, certain foods, especially chocolate and idiopathic, sexual activity induced)
fried food, or missing of meals, psychological stress, ♦♦ Traumatic headache
contraceptive pills. Treatment of cervical spondylosis ♦♦ Headache associated with vascular disorders (e.g. CVA,
• Relaxation exercises: They may include biofeedback intracranial hemorrhage or hematoma, arteritis, venous
from a temporalis electromyogram. Yoga, Pranayama. sinus thrombosis and other vascular disorders).
Section 1:  General Medicine  169

♦♦ Hedache associated with nonvascular intracranial diseases ♦♦ It induces attacks with oxygen desaturation which
(e.g. high or low CSF pressure, intracranial infections or reaches the threshold level of oxygen causing activation
neoplasm). of chemoreceptor and stimulate the nuclei of 7th and 10th
♦♦ Headache associated with substance abuse or their cranial nerve in respiratory center.
withdrawal. ♦♦ It results in stimulation of peripheral secretory and other
♦♦ Headache associated with systemic infection (e.g. viral receptors innervated by the cranial nerves causing cluster
and bacterial). headache.
♦♦ Headache associated with metabolic diseases (e.g. hypoxia, Q.7. Outline the management of headache. 
hypercapnia, hypoglycemia, dialysis, etc.)  (Sep 1997, 6.5 Marks)
♦♦ Headache due to referred pain (e.g., disorders of eyes, ear, Ans. The management of headache is as follows:
nose and sinuses, teeth, jaw and temporomandibular joint) • Mild headache responds to rest, massage,
♦♦ Cranial neuralgia (e.g., trigeminal, glossopharyngeal or acetaminophen or listening to relaxing music.
occipital neuralgia). • Moderate headache typically requires NSAID
♦♦ Unclassified headache.
therapy
Cluster Headache / Migranous Neuralgia • Caffeine helps ameliorate many mild-to-moderate
headache.
This is distinctive and treatable vascular headache syndrome • Antie me tic s s uc h as p ro c hlo rp er azine or
which is characterized by one to three short lived attacks of metoclopramide helps to relieve moderate to severe
periorbital pain per day over 4 to 8 weeks. headache especially those accompanied by nausea.
Etiology • Ergotamine and triptan drugs are suited to treating
migraine.
Headache associated with at least one of the following on the • Cluster headache often resolves after treatment with
painful site namely: corticosteroid or high flow oxygen.
♦♦ Conjunctival injection • The headache of temporal arteritis also responds
♦♦ Nasal *congestion to high dose steroids, but these agents must be
♦♦ Forehead and facial flushing continued for months or years until syndrome
♦♦ *Miosis remits.
♦♦ *Ptosis
♦♦ Eyelid edema Q.8. Write short note on status epilepticus.
♦♦ Rhinorrhea.  (Sep 1999, 5 Marks) (Sep 2006, 5 Marks)
Or
Symptoms Write management of status epilepticus.
♦♦ There is periodic, severe and unilateral periorbital pain.  (Nov 2011, 4 Marks) (Mar 2011, 4 Marks)
♦♦ Reddening of eye is present. Or
♦♦ Nasal *stuffiness is present.
Outline the management of status epilepticus.
♦♦ Nausea is present.
 (Feb 2002, 5 Marks)
♦♦ Pain occur after 1 or 2 hours when patient fallen asleep.
Ans. Status epilepticus is an alternate period of convulsions
Signs and unconsciousness without any intervening normal
period. It is a medical emergency because if not rapidly
♦♦ Conjunctiva infection
controlled it may be fatal.
♦♦ Rhinorrhea
♦♦ Nasal congestion It is most commonly referred to the tonic clonic seizures.
♦♦ Lacrimation Status epilepticus is never the presenting feature of
♦♦ Miosis idiopathic epilepsy but may be precipitated by:
♦♦ Ptosis • Abrupt withdrawal of antiepileptic drugs.
♦♦ Flushing, sweating • Structural lesion in brain
♦♦ Edema of face. • Acute metabolic disturbance.

Pathogenesis Clinical Features

Carotid body plays major role in cluster headache. Status epilepticus is divided into four stages depending on the
♦♦ Disturbance in some specific areas in hypothalamus duration for which seizures continue.
due to cyclic cluster periods which causes disturbances ♦♦ Prodromal phase: In patients with established epilepsy,
in sympathetic and parasympathetic supplies to body. tonic-clonic status epilepticus seldom develops without
It causes increase in vasomotor tone which activate warning. There is usually a prodromal stage in which
peripheral chemoreceptors. seizures become more frequent.
170   Mastering the BDS IIIrd Year  (Last 25 Years Solved Questions)

♦♦ Early status: Once status epilepticus has been developed Q.9. Write short note on temporal epilepsy. 
the first 30 minutes comprised of early stage.  (Mar 2003, 5 Marks) (Apr 2010, 5 Marks)
♦♦ Established status: It is a status which has continued for Ans. Temporal epilepsy is the form of epilepsy where aura (A
30min inspite of early stage treatment. subjective, but recognizable sensation that precedes and
♦♦ Refractory status: The stage is reached, if seizures continue signals the onset convulsion) is either auditory, visual,
for 60 to 90 min after initiation of therapy. olfactory or gustatory.
Management Clinical Features
♦♦ General and immediate measures: ♦♦ There is feeling of unusual smell, an emotional feeling or
• Move away the person from danger such as fire, water, *hallucinations.
machinery, etc. ♦♦ Motor activity stops and patient looks vacant.
• After convulsions cease, turn into recovery position. ♦♦ There is unilateral dystonic posturing of the limb.
• Ensure clear airway ♦♦ There is temporary cessation of the activity followed by
• Do not insert anything in the mouth. lip smoking, chewing movements or the patient may walk
• If convulsions continue for more than 5 minutes urgent aimlessly. When seizure and amnesia is present.
medical attention is required.
• Patient should not be left alone until he is recovered. Management
• Establish intravenous access in large veins. Take ♦♦ General:
blood for electrolytes, glucose, calcium, magnesium, • Avoid physical exertion, regular habits of eating and
full blood count, anti-epileptic drug levels, alcohol sleeping.
and toxicology screen and cultures as appropriate. • Avoid alcohol.
Urea and electrolytes, blood glucose, calcium and ♦♦ Pharmacological:
phenytoin levels are obtained urgently. • Clonazepam 1.5 mg/day reduces excitability of
• Check glucose and immediately correct any neurons.
hypoglycemia with 50% glucose up to 50 mL I.V. • Methsuximide 500 mg daily is effective.
over 1 to 2 min in a large vein. • Clobazam 10 to 20 mg has antiepileptic activity.
• If poor nutrition or alcohol abuse is suspected, • Gabapentin 300 mg TDS is used to control epilepsy.
administer thiamine 250 mg IV over 10min. • Lamotrigine 50 mg daily for two weeks followed by
• Check for the blood gases. 100 mg/day given in two divided doses for 2 weeks.
♦♦ Pharmacological management After dose is increased by 100 mg every 1 to 2 weeks
• Lorazepam 4 mg IV over 2–3 minutes or IV Diazepam till response is obtained.
5 mg IV. Repeat dose, if necessary. If IV access not
Q.10. Write short note on syncope. 
possible give Midazolam 10 mg intranasally or IM.
 (Mar 2016, 3 Marks) (Feb 2014, 3 Marks)
• Give IV phenytoin 15–20 mg/kg or IV Fosphenytoin
 (Nov 2008, 10 Marks) (Aug 2012, 5 Marks)
20 mg/kg in infusion at rate of less than 50 mg/min
 (May/June 2009, 5 Marks)
(phenytoin).
Ans. Syncope is a transient loss or impairment of consciousness
• Alternatives to IV phenytoin are Valproate 500–1000
with inability to maintain postural tone due to acute
mg IV bolus or Levetiracetam 1gm bolus.
decrease in cerebral blood flow.
Refractory status—if seizures continue (more than 20
minutes), then intubate and give one of the following: Causes
• Midazolam drip IV. Load with 0.2 mg/kg and infuse
♦♦ Due to decreased cerebral perfusion
at rate of 0.03–0.2 mg/kg/hr
• Inadequate vasoconstrictive mechanisms
Or
–– Vasovagal
• Propofol IV. Load 1–2 mg/kg repeat every 5 mins till –– Postural hypotension
seizures stop, followed by IV infusion 2–10 mg/hr. –– Carotid sinus syncope
Or –– Antihypertensive drugs, i.e., hydralazine, etc.
• Phenobarbitone IV bolus 6–8 mg/kg at a rate not more • Hypovolemia
than 60 mg/min. –– Hemorrhage (blood loss)
Partial (local) minor status epilepticus consists of –– Addison's disease
frequent seizures involving an extremity or facial • Reduction of venous return
muscles with preservation of consciousness and no –– Cough
tendency for generalization. –– Micturition
• Phenytoin orally during an 8 to 12 hours period –– Mediastinal compression
followed by maintenance dose of 300 to 400 mg/day. –– Straining at stool evacuation (defecation)
Section 1:  General Medicine  171

• Reduced cardiac output Carotid Sinus Syncope


–– Aortic stenosis or hypertrophic subaortic stenosis ♦♦ It is due to vagal mediated cardiac slowing or due to fall
–– Myocardial infarction in blood pressure due to depressor response.
–– Cardiac temponade due to pericardial effusion ♦♦ Carotid sinus is sensitive to stretch, so it is induced in
–– Pulmonary embolism sensitive patient by turning head to one side or by tight
• Arrhythmias fitting cervical collar.
–– AV blocks (2nd degree and 3rd degree or complete ♦♦ It last for few seconds to few minutes.
AV block)
–– Ventricular asystole Cough Syncope
–– Ventricular tachycardia and fibrillation ♦♦ It occurs in patients with chronic bronchitis or chronic
–– Supraventricular tachycardia obstructive pulmonary disease.
• Cerebrovascular disturbance ♦♦ It happens due to the decrease venous return to heart due
–– Transitory ischemic attack to rise in intra-thoracic pressure at the time of coughing.
–– Hypertension
–– Vertebrobasilar insufficiency Syncope of Cerebrovascular Disease
♦♦ Noncirculatory causes ♦♦ It occurs due to narrowing of large arteries.
• Hypoxia ♦♦ This is the manifestation of vertebrobasilar insufficiency.
• Anaemia
♦♦ Excessive physical activity or exercise decreases blood flow
• Prolonged bed rest
to upper part of brainstem leading ot syncope.
• Anxiety neurosis
Types of Syncope Clinical Features
Vasovagal Syncope ♦♦ Tingling or numbness in the limbs, sudden darkness before
eyes and patient has feeling of *blacking out.
♦♦ This is the most common syncope. ♦♦ The patient is cold and sweating and fall suddenly to
♦♦ It tends to occur at the time of emotional stress or after an ground and become unconsciousness.
injury or accident or severe pain. ♦♦ Patient’s respiration is *sighing.
♦♦ It involves cardiac slowing because of vagal stimulation ♦♦ Pulse of the patient is slow and limbs are cold and
and sudden reduction in systemic peripheral resistance clammy.
due to withdrawal of sympathetic tone. ♦♦ Pupils may retract to light.
Postural Hypotension with Syncope Management
♦♦ It occurs in patients who have chronic illness or instability ♦♦ All the medical/dental procedure or treatment is
of vasomotor reflexes. stopped.
♦♦ Fall in blood pressure is present in erect posture due to loss ♦♦ Remove instruments from oral cavity such as rubber dam,
of vasoconstrictive reflexes leading to pooling of blood in gauze, cotton etc.
capacitance vessels of lower limbs. ♦♦ Patient is kept in Trendelenburg position, i.e. patient is
♦♦ It usually occurs on sudden arising from supine position kept in a head low and feet up position.
or in the standing position. ♦♦ Loose tighten clothing of patient.
Micturition Syncope ♦♦ Aromatic fumes inhalation is given or sprinkle cold water
on face of patient for reflex stimulation.
♦♦ It is seen in elder patients during or after urination mainly ♦♦ If recovery is gained escort patient home.
after arising from sitting position. ♦♦ If recovery is not gained Injection Atropine 0.6 mg IM or
♦♦ It occurs due to the release of intravesical pressure at the I.V. is given.
time of micturition which leads to sudden vasodilatation ♦♦ If still recovery is not gained look for hypoglycemia and
which become augmented by standing causing syncope. Addison’s crisis
Cardiac Syncope ♦♦ Start basic life support
♦♦ Summon medical help.
♦♦ It occurs due to reduction in sudden cardiac output.
♦♦ It occurs due to abnormal decrease or increase in the Q.11. Write short note on Brown-Sequard’s syndrome.
heart rate.  (Aug 1995, 5 Marks)
♦♦ Supraventricular tachycardias, cerebrovascular disease, Ans. Brown-Sequard’s syndrome is a spinal cord lesion. In it
valvular heart disease can cause syncope. hemisection (Bisection) of spinal cord is present.
172   Mastering the BDS IIIrd Year  (Last 25 Years Solved Questions)

Q.12. Enumerate the cause of epilepsy. (Feb 2006, 2 Marks)


Ans. The causes of epilepsy are:
Follwing are the causes of epilepsy in different age groups.

In Neonates (0 to 2 years)
♦♦ Perinatal hypoxia, or ischemia
♦♦ Birth injury
♦♦ Acute infections, i.e., meningitis, encephalitis metabolic
disturbance, i.e hypoglycemia, hypocalcemia,
hypomagnesemia structural lesions such as congenital
vascular malformations
♦♦ Familial or genetic disorders.
In Chidren (2 to 12 years)
♦♦ Idiopathic
♦♦ Acute infections, i.e., meningitis, encephalitis,
toxoplasmosis, cerebral abscess
♦♦ Head injury or trauma
♦♦ Febrile convulsions

Fig. 24:  Brown-Sequard syndrome In Adolescents (12 to 18 years)


♦♦ Idiopathic
Etiology
♦♦ Head trauma
♦♦ Compression of cord ♦♦ Drugs, i.e., amphetamines, antidepressants, phenothiazines,
♦♦ Intramedullary neoplasm etc.
♦♦ Due to *stab in back ♦♦ Alcohol withdrawal
♦♦ Bullet wounds ♦♦ Arteriovenous malformations
♦♦ Vertebral fracture and caries ♦♦ Infections, i.e., meningitis, encephalitis, cerebral abscess,
♦♦ Vascular causes, i.e. *arachnoiditis.
toxoplasmosis
Clinical Features
In Person of Age 18 to 35 years
♦♦ Below the site of lesion ♦♦ Head injury or trauma
• Motor changes: Spastic paralysis, i.e. ♦♦ Alcoholism
–– Positive Babinski’s sign
♦♦ Brain tumors, cysts, hydrocephalus, aneurysms, AV
–– Clasp knife rigidity
malformations
–– Exaggerated tender jerks and loss of voluntary
power. All the above features are seen in unilateral ♦♦ Inflammatory disorders such as sarcoidosis, multiple
side of lesion. The sign are due to interruption of sclerosis, SLE.
already crossed pyramidal tract. In Older Adults and Old Persons
• Sensory changes:
–– Loss of fine touch, tactile sensation and vibration ♦♦ Brain tumours
sense is loss in ipsilateral side. This is due to loss ♦♦ Cerebrovascular accidents (thrombosis, infarction,
of lemniscal fibers. hemorrhage)
–– Loss of crude touch, pressure, pain and ♦♦ Alcoholism
temperature sense on contralateral side. This is ♦♦ Uremia
due to loss of spinothalamic tract fiber. ♦♦ Hepatic encephalopathy
♦♦ At the site of lesion ♦♦ Hypertensive encephalopathy
• Motor changes: Lower motor neuron paralysis is ♦♦ Electrolyte disturbances
segment corres­ponding to level of lesion on ipsilateral ♦♦ Hypoglycemia
side, due to loss of anterior horn cells of segment.
Q.13. Write short note on pyogenic meningitis. 
• Sensory changes: There is anesthesia or ipsilateral
*dermatomes due to loss of posterior root of  (Mar 1998, 5 Marks) (Oct 2007, 10 Marks)
corresponding spinal cord segment. Or

Q11. *Stab= Wound produced by piercing with a knife or pointed instrument.     *Arachnoiditis= Inflammation of arachnoid membrane.
Section 1:  General Medicine  173

Describe briefly clinical features and management of ♦♦ Mental retardation


pyogenic meningitis. ♦♦ Epilepsy.
 (Sep 1999, 5 Marks) (Sep 2006, 5 Marks)
Treatment
Ans. Inflammation of the meninges is called as meningitis.
Pyogenic meningitis occurs due to Streptococcus pneu- Emperical treatment should be given before CSF culture and
moniae, staphylococcus aureus, hemophilus influenzae. Gram stain report. Treatment should be directed to the most
Pathology: Infection once it reaches the protective wall common microorganism present in particular age group.
of meninges, rapidly spread over the surface of brain, Antibiotic Treatment
spinal cord and ependymal lining of ventricles.
♦♦ Brain and spinal cord are swollen and congested. ♦♦ Ceftriaxone or cefotaxime is given against S. pneumonia,
♦♦ Cortical veins become conjusted and convolutions on H. influenz, Group B streptococci and N. meningitides. In
the surface of brain become flattened because of internal this vancomycin can be added to cover cephalosporin
hydrocephalus which is due to inflammatory adhesions resistant S. pneumonia. Ampicillin can be added to cover L.
obstructing outflow of CSF from fourth ventricle, whole monocytogens in neonates of less than 3 months and more
of CSF become turbid and purulent. than 55 years of age.
♦♦ Ceftazidime is active against P. aeuroginosa and is preferred
Clinical Features over ceftriaxone or cefotaxime in hospital acquired
Symptoms meningitis.
♦♦ Choice of empirical antibiotics in pyogenic meningitis is:
♦♦ Fever coming with rigors.
• In neonates or infants of less than 3 months: Ampicillin
♦♦ Headache which is very severe (bursting in character)
mainly in frontal region radiating down to back. 100 to 50 mg/dL + Ceftriaxone 500 to 1000 mg/kg/day
♦♦ Vomiting or cefotaxime 50 mg/kg
♦♦ Convulsion in children • In children and adults: Ceftriaxone 500 to 1000 mg/kg/
♦♦ Malaise day or cefotaxime 50 mg/kg + vancomycin 60 mg/kg
♦♦ Severe photophobia • Adults more than 55 years: Ampicillin 3 gm tds or
♦♦ Ptosis: Due to raised intracranial tension QDS + Ceftriaxone 2 gm BD or cefotaxime 50mg/kg +
♦♦ Stiffness in neck and back vancomycin 1gm 8 hourly
♦♦ Pain in neck • In hospital acquired meningitis, post-traumatic or
♦♦ Impairment of consciousness i.e. confusion, delirium and post-surgical, immunocompromised patients—
coma. Ampicillin 3 gm tds or qds + Ceftazidime 2gm 8 hourly
+ vancomycin 1 gm 8 hourly.
Signs
Duration of Antibiotic Therapy
♦♦ Head retraction is present in infants and children.
♦♦ Neck rigidity, i.e. bending of neck causes pain and spasm ♦♦ One week for H. influenzae and N. meningitides infection
of neck muscles or it is difficult to bend the neck. ♦♦ S. pneumonia for two weeks
♦♦ Kernig’s sign is positive ♦♦ L. monocytogenes and Gram-negative bacilli infections
♦♦ Brudzinski’s sign is also positive if patient is conscious. for 3 weeks.
♦♦ Presence of papilledema
♦♦ Presence of cranial nerve palsies Adjunctive Therapy
Dexamethasone 0.4mg/kg BD for 4 days with first dose of
Investigation
antibiotic.
Blood count: Leukocytosis with rise in polymorph count.
Supportive Therapy
CSF Examination
♦♦ Patients having raised intra-cranial pressure should be
♦♦ CSF pressure is raised.
treated in ICU.
♦♦ It is turbid and purulent pus like.
♦♦ IV mannitol, hyperventilation and elevation of patient’s
♦♦ Formation of coagulum
head to 30° is done to decrease raised intra-cranial pressure.
♦♦ Protein content is markedly increased.
♦♦ Glucose content is decreased. Q.14. Enumerate the etiological factors of bacterial men-
♦♦ CSF protein is raised ingitis. Describe clinical features, complica­tions and
♦♦ Gram staining of CSF is positive management of a case of meningococcal meningitis.
 (Mar 2001, 20 Marks) (Sep 2009, 5 Marks)
Complications Ans. Etiology of Bacterial Meningitis
♦♦ Cranial nerve paralysis Bacterial meningitis is caused by various bacteria which
♦♦ Deafness are as follows:

Q11. *Dermatome= A band or region of skin supplied by single sensory nerve.


174   Mastering the BDS IIIrd Year  (Last 25 Years Solved Questions)

♦♦ In neonates or infants Patients allergic to penicillin are treated with chlora­


• Gram negative bacilli, i.e. E. coli and B. proteus mphenicol 1 gm IV 6 hourly
• Group B streptococci ♦♦ Treatment is continued for 7 to 10 days.
• Listeria monocytogenes ♦♦ For raised intracranial tension IV mannitol is given which
♦♦ In adolescents or adults is accompanied by high doses of dexamethasone 4 mg IV
• Streptococcus pneumoniae 6 hourly.
• Neisseria meningitidis The supportive treatment is to maintain nutrition, fluid
• Mycobacterium tuberculosis and electrolytic balance.
• Staphococcus aureus Q.15. Describe the management of tubercular meningitis.
• Hephilus influenzae  (Sep 1998, 5 Marks)
♦♦ In old age
Ans. Management of Tubercular Meningitis
• Hemophilus influenzae
• General management:
• Neisseria meningitides
• Streptococcus pneumoniae – Maintenance of nutrition, hydration and
• Mycobacterium tuberculosis electrolyte balance.
♦♦ In immunocompromised – Case of bowel and bladder
• Listeria monocytogenes – Nursing should be good
• Gram negative bacilli – If there are convulsions, anticonvulsants are given.
• Streptococcus pneumoniae • Treatment:
• Mycobacterium tuberculosis – Antitubercular drugs: Rifampicin 600 mg/day +
• Cryptococcus neoformans Isoniazide (600 to 900 mg/day) + Pyrazinamide
(1.5 gm) should be given. Treatment with this
Clinical Features of Meningococcal Meningitis regimen is given for 2 months. This is followed
♦♦ Meningococcal rash, petechial rash on skin, mucus by rifampicin 600 mg/day + Isoniazide (600 to
membrane and conjunctiva. 900 mg/day) for 12 to 18 months.
♦♦ Acute fulminant illness with adrenal insufficiency. – Steroids: Prednisolone 40–60 mg/day to reduce
♦♦ Hypotension, shock and patient go quickly in comma. This toxicity, pia-arachnoid adhesions and felling of
is called as Water house–Friderichsen syndrome. well-being.
♦♦ It is due to necrosis in adrenal gland during course of Q.16. Describe clinical features and evaluate manage­ment
meningococcal septicemia.
of meningitis. (Mar 1997, 12.5 Marks)
♦♦ Signs of meningeal irritation, i.e. Kernig’s and Brudzinski’s
Ans. For clinical features and management of pyogenic
sign are positive.
♦♦ In children and adults it is mainly present. meningitis refer to Ans 13 of same chapter.
♦♦ Neck rigidity
♦♦ Vomiting Clinical Features of Tuberculous Meningitis
♦♦ Fever with rigor Symptoms
♦♦ Dilatation of pupil.
♦♦ Headache
Complications of Meningococcal Meningitis ♦♦ Vomiting
♦♦ Neurological defect like hemiplegia, aphasia, ocular ♦♦ Low grade fever
nateor, hemianopia, blindness and deafness. ♦♦ Lassitude, i.e. weariness or exhaustion
♦♦ Mental deterioration ♦♦ Depression
♦♦ Cerebritis, brain abscess ♦♦ Confusion
♦♦ Focal fits ♦♦ Behavior changes.
♦♦ Auditory impairment
♦♦ Sub-dural empyema
Signs
♦♦ Internal hydrocephalus
♦♦ Spinal cord compression due to arachnoiditis. ♦♦ Meningism may be present
♦♦ Occulomotor palsies
Management of Meningococcal Meningitis ♦♦ Papilledema
♦♦ For adult patients penicillin G 5 to 10 million units IV ♦♦ Depression of conscious level
6 hourly. ♦♦ Focal hemisphere signs.
♦♦ Cephalosporins, i.e. cefotaxime 2 gm IV or ceftriaxone For management of tuberculous meningitis refer to Ans
2 gm IV OD is also effective. 18 of same chapter.
Section 1:  General Medicine  175

Q.17. Differentiate between tubercular meningitis and pyo- Q.19. Describe the clinical and diagnostic features of tube­r­
genic meningitis. (Feb 2006, 2.5 Marks) culous meningitis. (Feb 2002, 5 Marks)
Ans. Ans. For clinical features refer to Ans 16 of same chapter.
Tubercular meningitis Pyogenic meningitis Diagnostic Features
• Most common organism • Most common organism for
♦♦ Person with history of contact with tubercular patient
for causing tubercular causing pyogenic meningitis
meningitis is M. tuberculum is S. pneumoniae presenting with low grade fever, ill health, weight loss, odd
behavior, headache should make one suspect tubercular
• It is common in childhood • It is most common in adults
meningitis.
• Neck rigidity, vomiting and • Neck rigidity, vomiting and
convulsions are lower in convulsions are higher in
♦♦ Diagnosis shall be confirmed by the lumbar puncture.
intensity intensity. Muscular spasm is CSF examination reveals following results:
more common
• It is associated with infection • It is associated with infection ♦♦ CSF is straw colored, clear but when allowed to stand, a
in lungs, bowel and mesen- in pneumonia, chronic otitis fine clot, i.e. spider web is formed.
teric glands media, sinusitis and head ♦♦ Lymphocyte count is high
injury ♦♦ Protein content is high
• CSF is clear or slightly opal- • CSF is thick, greenish fluid. It ♦♦ Glucose is low
escent and is under pres- is turbid and pus like. The CSF ♦♦ In acute cases, polymorphs may predominate
sure. Cob web forms are pressure is raised. Polymorphs ♦♦ AFB stain can be positive. Culture or AFB is positive in
seen if it is allowed to stand are the most common cells
80% of cases
overnight. In it mainly lym- found
phocytes are seen in CSF CT or MRI brain may show meningeal enhancement or
hydrocephalous
• In it antitubercular treatment • Penicillin G or cefotaxime for
is given at least 7 days is given Q.20. Outline the management of migraine. 
 (Feb 2002, 5 Marks) (Mar 2009, 5 Marks)
• Not specific vaccine is • Vaccination with hib vaccine
available or present for reduces the incidence Ans.
tubercular meningitis ♦♦ During attack:
• Analgesics: NSAIDs, e.g. diclofenac can be given orally
Q.18. Write briefly on myasthenia gravis.(Aug 1995, 4 Marks) or i.m. and is particularly useful when severe vomiting
Ans. An acquired autoimmune disorder causing skeletal is a feature. Sublingual Piroxicam has significant
muscle fatigability and weakness which can be present analgesic effects in acute migraine without aura with
at any age is called as myasthenia gravis. excellent tolerability.
Symptoms and Signs • Ergotamine: Ergotamine tartrate 0.25 to 0.5 mg IM or
orally 1–2 mg. Tablet preferably in combination with
Muscular weakness: There is repetitive contraction with tendency
100 mg caffeine - 2 tablets at onset followed by l tablet
to recovery of motor power after inactivity
after 30 minutes, if necessary, or Dihydroergotamine 1
♦♦ Ocular muscles: The ocular muscles are first to be involved.
mg IM, or 1–2 mg by mouth. Whichever preparation is
There is presence of double vision or Ptosis. Symptoms
used, a high dose often causes nausea and vomiting.
are asymmetrical.
These may be prevented by giving cyclizine 50 mg.
♦♦ Limb weakness: It may involve proximal or distal limbs.
or chlorpromazine 25 mg.
♦♦ Bulbar muscle weakness: It leads to the loss of facial
• 5-HT1 agonists: Sumatriptan 6 mg s.c. gives relief
expression, inability to whistle, difficulty with speech,
from headache in 60 minutes, with corresponding
chewing and swallowing.
improvement in nausea, vomiting and photophobia.
♦♦ Respiratory muscle involvement: It leads to shortness of
Oral dose of 100 mg provides relief within 2 hours.
breath and ventilatory failure.
Headaches recur within 48 hours in little less than
Management half the patients. Rizatriptan given orally acts faster
♦♦ Cholinesterase inhibitors, i.e. pyridostigmine 30 to 120 mg than sumatriptan. Zolmitriptan nasal spray 5 mg gives
or neostigmine 15 to 45 mg TDS is given. relief in 5 minutes.
♦♦ Thymectomy is performed as soon as feasible in any • General: Lying in a darkened and quiet room and ice
antibody positive patient with symptoms not confined to pack to the head may help.
extraocular muscles. ♦♦ Reducing frequency and severity of subsequent attacks
♦♦ Plasma exchange is done • Elimination of trigger factors: e.g. Sleeping late,
♦♦ IV immunoglobin 0.4 gm/kg/day for 5 days is given. irregular and hurried meals, certain foods, especially
♦♦ Immunosuppressant: Azathioprine 2.5 mg/kg body weight chocolate and fried food, or missing of meals,
is helpful in reducing dosage of steroids necessary and psychological stress, contraceptive pills. Treatment
allows steroids to withdraw. of cervical spondylosis
176   Mastering the BDS IIIrd Year  (Last 25 Years Solved Questions)

• Relaxation exercises: They may include biofeedback –– Neonatal: Other herpes virus, Epstein-Barr virus,
from a temporalis electromyogram. Yoga, Pranayama. Varicella zoster virus
• Drugs: –– Uncommon: Cytomegalovirus, HIV, Lymphocytic
–– Serotonin (5-HT) inhibitors: Calcium antagonists choriomeningitis virus
such as Flunarizine 10 mg/day Or Cyproheptadine –– Rare: Adenovirus Types 3 and 7, Arbovirus
4 mg t.d.s. Or Pizotyfen 0.5 mg t.d.s. or 1.5 mg • Protozoa: Naegleria
nocte. Or Methysergide 1–2 mg t.d.s. is the most • Fungal: Cryptococcus neoformans, Candida
effective drug in this group but should be used • Spirochaetal: Leptospirosis, Syphilis and Lyme disease
under supervision in courses not exceeding 3–4 • Ricketssial: Typhus fever
months. ♦♦ Non-infectious causes
–– Topiramate: It is an antiepileptic drug used in • Malignant: Leukemic meningitis
prophylaxis of migraine. Dose: 2.5 to 5 mg BD • Other non-infectious causes: Sarcoidosis, connective
–– Divalproex (valproic acid) 200 mg BD tissue disease, systemic lupus erythematosus,
–– Tricyclic agents: e.g. amitryptiline 25 mg TDS Sjögren’s syndrome
may be effective irrespective of the presence of • Vasculitis: Granulomatous polyangiitis, Eosinophilic
depression. granulomatous polyangiitis, CNS vasculitis
–– Ergotamine tartrate—for histamine cephalalgia 1
Clinical Features
mg by mouth or 0.25 mg by self-administered
injection or by suppository used regularly last Symptoms
thing at night can be continued for many weeks ♦♦ Fever coming with rigors
without harmful effects, 2 days being left without ♦♦ Headache which is severe mainly in frontal region
treatment each week. radiating down to back
–– Hormones: Progesterone given for last eight ♦♦ Vomiting
days may be useful for migraine occurring in the ♦♦ Convulsion in children
immediate premenstrual period or at beginning ♦♦ Malaise
of catamenia. When migraine begins or becomes ♦♦ Severe photophobia
worse at the time of menopause, estrin, given in ♦♦ Ptosis – due to raised intra – cranial tension
small doses as continuous therapy sometimes ♦♦ Stiffness in neck and back
helpful. ♦♦ Pain in neck
–– Schedule: ♦♦ Impairment of consciousness, i.e. confusion, delirium
- Propranolol 40–160 mg/day or flunarizine 5–10 and coma
mg/day as first line of therapy.
- In patients with episodic and chronic migraine Signs
Topiramate 50–100 mg/day. ♦♦ Head retraction is present in infants and children.
- In patients with episodic migraine Divalproex ♦♦ Neck rigidity, i.e. bending of neck causes pain and spasm
250–750 mg/day of neck muscles or it is difficult to bend the neck.
- For mixed migraine and tension type head­ ♦♦ Kernig’s sign is positive
ache—amitryptiline 10–25 mg/ day. ♦♦ Brudzinski’s sign is also positive if patient is conscious.
After 6–12 months of prophylaxis gradual ♦♦ Presence of papilledema
withdrawal should be considered. ♦♦ Presence of cranial nerve palsies
Q.21. Describe causes, clinical features, treatment and Treatment
complication of meningitis. (Sep 2004, 20 Marks)
Ans. Inflammation of the meninges is called as meningitis. Empirical treatment should be given before CSF culture and
Gram stain report. Treatment should be directed to the most
Causes common microorganism present in particular age group.
♦♦ Infectious causes:
Antibiotic Treatment
• Bacterial:
–– Common: N. meningitides, S. pneumoniae, H. ♦♦ Ceftriaxone or cefotaxime is given against S. pneumonia,
influenza, M tuberculosis H. influenzae, Group B streptococci and N. meningitides.
–– Neonatal: Group B streptococcus, E. coli, L. In this vancomycin can be added to cover cephalosporin
monocytogenes resistant S. pneumonia. Ampicillin can be added to cover L.
–– Uncommon: S. aureus, Ps. Areuginosa monocytogens in neonates of less than 3 months and more
–– Rare: Salmonella, Shigella, N. gonorrhea than 55 years of age.
• Viral: ♦♦ Ceftazidime is active against P. aeuroginosa and is preferred
–– Common: Mumps, Echovirus, Coxsackie virus A over ceftriaxone or cefotaxime in hospital acquired
and B, Genital herpes virus 1 and 2 meningitis.
Section 1:  General Medicine  177

♦♦ Choice of empirical antibiotics in pyogenic meningitis is: Q.22. Discuss causes and management of unconsciousness.
♦♦ In neonates or infants of less than 3 months: Ampicillin 100  (Feb/Mar 2004, 10 Marks)
to 50 mg/dL + Ceftriaxone 500 to 1000 mg/kg/day or Ans. Causes of Unconsciousness
cefotaxime 50 mg/kg • Decrease in the cerebral perfusion:
♦♦ In children and adults: Ceftriaxone 500 to 1000 mg/kg/day – Inadequate vasoconstriction mechanism
or cefotaxime 50 mg/kg + vancomycin 60 mg/Kg - Postural hypotension
♦♦ Adults more than 55 years: Ampicillin 3 gm TDS or QDS + - Vasovagal shock
Ceftriaxone 2gm BD or cefotaxime 50 mg/kg + vancomycin - Anti-hypertensive drugs
1gm 8 hourly - Carotid sinus syncope.
♦♦ In hospital acquired meningitis, post-traumatic or post- – Hypovolemia:
surgical, immunocompromised patients: Ampicillin 3 gm - Addison’s disease
TDS or QDS + Ceftazidime 2 gm 8 hourly + vancomycin - Due to blood loss i.e. hemorrhage.
1gm 8 hourly. – Decrease in the venous return
- Mediastinal compression
Duration of Antibiotic Therapy
- Micturition
♦♦ One week for H. influenzae and N. meningitides infection - Cough
♦♦ S. pneumoniae for two weeks - Straining during defecation.
♦♦ L. monocytogenes and Gram-negative bacilli infections for – Decrease in cardiac output
3 weeks - Myocardial infarction
- Pulmonary embolism
Adjunctive Therapy
- Aortic stenosis
Dexamethasone 0.4 mg/kg BD for 4 days with first dose of - Cardiac temponade.
antibiotic. – Arrhythmias
- In AV blocks
Supportive Therapy - Supraventricular tachycardia
- Ventricular asystole
♦♦ Patients having raised intra-cranial pressure should be
- Ventricular tachycardia.
treated in ICU.
♦♦ IV mannitol, hyperventilation and elevation of patient’s – Cerebrovascular disturbances
head to 30° is done to decrease raised intracranial - Hypertension
pressure. - Transitory ischemic attack
- Vertebrobasilar insufficiency.
For treatment of tubercular meningitis refer to ans13 of same
chapter • Non-circulatory causes:
- Anemia
- Anxiety neurosis
Complications
- Hypoxia.
♦♦ Neurological deficiencies: Hemiplegia, aphasia, hemianopia, For management refer to Ans 10 of same chapter.
blindness, deafness.
♦♦ Mental deterioration Q.23. Describe the CSF picture of pyogenic meningitis and
♦♦ Cerebritis, brain abscess, focal fits, auditory impairments, tubercular meningitis. (Sep 2005, 10 Marks)
sub-dural empyema, internal hydrocephalus. Ans.

Content Normal value Pyogenic meningitis Tubercular meningitis


Glucose 40–70 mg/dL Low<40 mg/dL Low or normal
Lactate 10–20 mg/dL Low Low or normal
Protein Up to 45 mg/dL Elevated Elevated
Albumin 6.6–42.2 mg/dL Elevated Elevated
Ammonia 25–80 µg/dL Elevated Elevated
CSF pressure 50–180 mm/Hg Raised and turbid Normal
CSF volume 150 mL Raised Raised
Gram staining — Positive Negative
Lymphocyte 60–70% Normal Raised
Polymorphs None Present —
178   Mastering the BDS IIIrd Year  (Last 25 Years Solved Questions)

Q.24. Discuss the causes of headache.  ♦♦ Tension (muscle contraction) headache:


 (Feb/Mar 2004, 5 Marks) Pain resulting from sustained contraction of skeletal
Ans. The causes of headache are: muscles of the neck, frontalis, occipital muscles due to
♦♦ Intracranial and local extracranial emotional tension.
• Trauma leads to contusional or post-traumatic ♦♦ Exertional headache:
headache Headache may come on during exertion and persist for
• Intracranial inflammations: Meningitis, encephalitis, few hours afterwards.
cerebral abscess. ♦♦ Other primary headaches:
• Vascular headaches: Hypertension, cerebral or sub- • Hypnic headache syndrome is a late onset disorder
arachnoid hemorrhage, intracranial aneurysm; and usually wakes up the patient from sleep at around
vasodilator drugs like nitrites and histamine, the same time every night. The headache is usually
adrenaline. treatable with flunarizine and lithium.
• Menopausal: Alcohol hangover or withdrawal, coffee • Exploding head syndrome can occur any time during
withdrawal. Giant cell arteritis (temporal arteritis), day or night.
thrombosis of intracranial venous sinus. Q.25. Write short note on definition and classification of
• Traction headache: Pain produced by intracranial epilepsy. (Mar 2007, 5 Marks)
arterial displacement and distortion of the dura, Ans. Epilepsy is defined as condition characterized by the
usually caused by space occupying lesions or raised recurrent episodes primarily of cerebral origin in which
intracranial pressure or low intracranial pressure there is disturbance of movement, sensation, behavior
(intracranial hypotension). and consciousness.
• Post-lumbar puncture headache: Low CSF pressure
Classification
headache.
• Cough headache: A benign syndrome of severe headache Classification of epileptic seizures modified in 1981
which accompanies coughing, straining or sneezing ♦♦ Partial or focal seizures:
can be due to posterior fossa tumour. • Simple partial seizures (awareness preserved)
♦♦ Cranial neuritis and neuralgias: This is of sensory Depending on the concomitant signs, they are:
nerves of scalp, e.g. orbital neuralgia, or neuralgia of –– Motor
auriculotemporal, posterior auricular or great occpital –– Sensory
nerves, herpes of Gasserian ganglion. –– Visual
♦♦ General or systemic causes –– Versive
• Anoxemia: Anemia, carbon monoxide or carbon –– Psychomotor
dioxide poisoning. • Complex partial seizures (awareness lost)
• Toxic: Fevers, uremia, eclampsia, metallic poisoning, Depending on the area involved due to spread, they
"alcoholic" hangover, post- convulsive, drugs like are:
quinine, tobacco, cocaine, morphine, sulphonamides. –– Temporal lobe
Pelvic or gallbladder disease, constipation, intestinal –– Frontal lobe
stasis. Nervous exhaustion. • Secondary generalized partial seizures
• Metabolic factors: Hypoglycemia, alkalosis or acidosis. ♦♦ Primary generalized seizures:
• Hemopoietic factors: Essential polycythemia, • Tonic – clonic (grand mal)
thromobasthenia. • Tonic
♦♦ Referred pain • Absence (petit mal)
• Eyes: Errors ofrefraction, glaucoma, iritis, etc. • Akinetic
• Ears: Otitis, mastoiditis, vestibular nerve lesions, • Myoclonic
Eustachian tube block, tumors of middle and inner ear. • Infantile spasms
• Teeth: Impacted teeth, infected tooth sockets and ♦♦ Unclassified seizures:
dental roots. Seizures which do not fit into above two categories;
• Paranasal sinuses: Infection of paranasal sinuses may • Neonatal seizures
cause localized pain. • Infantile spasms
• Neck: Diseases of upper cervical spine may be Q.26. Write short note on panic attack. (Mar 2007, 5 Marks)
associated with both occipital and frontal pain. Ans. Panic attacks are discrete episodes of paroxysmal
♦♦ Psychogenic: Common cause of headache in depression. severe anxiety and are characterized by severe and
Section 1:  General Medicine  179

frightening autonomic symptoms, i.e. shortness of


breadth, palpitations, excessive perspiration; dizziness,
faintness and chest pain. Many patients believe they
are in immediate danger of death or collapse and seek
urgent medical attention.

Typical Feature of Panic Attack


♦♦ It is of sudden onset and short duration
♦♦ It shows rapidly escalating physical and psychological
symptoms
♦♦ Presence of incapacitating symptoms of breadthness and/
or palpitations
♦♦ Fear of impending death, collapse or loss of control
♦♦ Rapid escape from situation where attack is occurred
♦♦ Sometimes panic attacks are labeled nocturnal when they
occur at night only
♦♦ Panic attack in social phobia is restricted to feared social
situations
♦♦ Panic attack in panic disorder occurs unexpectedly in social
encounters and when person is alone.
Diagnostic Guideline for Panic Attack
A panic attack is characterized by all of the following: Fig. 25:  Panic attack
♦♦ A discrete episode of intense fear or discomfort
♦♦ It starts abruptly Q.27. Write short note on depression. (Sep 2007, 5 Marks)
♦♦ It reaches maximum intensity within a few minutes Ans. Depression is defined as depressed mood on a daily basis
♦♦ It lasts for at least several minutes for a minimum duration of two weeks.
♦♦ At least four symptoms are present (including at least one Depression is present in a quarter to half of all mental
autonomic symptom) patients.
♦♦ The attack is not caused by a physical disease, an
organic mental disorder, or another condition such as
Etiology
schizophrenia, mood disorder or somatoform disorder.
♦♦ Panic attack may accompany any anxiety disorder, but a ♦♦ Genetic factors: They play a major role in mood disorders
specific diagnosis of panic disorder can be made if they and their effect is stronger in patients with more severe
occur frequently and unexpectedly. biological symptoms. In twin studies of bipolar disorder,
average concordance rate is 65% in monozygotic twins
Management and 14% in dizygotic twins.
♦♦ Psychological: ♦♦ Social factors: Events associated with depression are
• Helping patients to understand that their symptoms generally ’loss events' such as loss of a job, relative or
are not caused by serious physical ailment. tend, money, health or status. Other factors that adversely
• Relaxation training can be helpful, but severely ill affect the response to the events can be, a working class
patients are more likely to benefit from cognitive background, lack of confiding relationship with a spouse,
behavior therapy. unemployment, loss of a parent before l l years of age.
• Exposure therapy can be formed under supervision ♦♦ Biological markers for depression: They show strong
of a behavior therapist. associations, particularly with the somatic (endogenous)
♦♦ Drugs: syndrome. False-positive results occur in the presence of
• High dose benzodiazepines, e.g. alprazolam are various medical disorders. Dexamethasone suppression
effective but can cause substantial depression and test is the most important biological marker.
should be prescribed in severely ill patients who have ♦♦ Circadian rhythms and related markers: They have also
not responded to other treatment approaches. been found abnormal in depression. This is suggested by
• Anti-depressant drugs, i.e. imipramine, clomipramine the diurnal variations in mood, early morning waking
and selective serotonin re-uptake inhibitors, i.e. and the sometimes periodic course, for example, yearly
paroxetine are as efficacious in reducing anxiety attacks of illness.
symptoms, lessening agoraphobia and minimizing ♦♦ Psychological factors: Repeated trauma, stressful life events
overall impairment. and disturbed marital and interpersonal relationships.
180   Mastering the BDS IIIrd Year  (Last 25 Years Solved Questions)

Clinical Features but have a wide range of side effects. These lead to non-
♦♦ General: Hopelessness, helplessness, low mood, low self compliance and limit usefulness in illnesses of mild to
esteem, reduced energy, suicidal thoughts, loss of interest, moderate severity. The dose of tricyclics should be initially
poor concentration, guilt, pessimism, depersonalization. low and increased gradually.
♦♦ Somatic: Appetite disturbance, weight change, constipation, ♦♦ Selective serotonin re-uptake inhibitors: They are effective
amenorrhea, low libido, sleep disturbance drugs and are better tolerated at therapeutic doses than
♦♦ Anxiety: Tension, apprehension, phobias. other compounds.
♦♦ If patient is not responding to anti-depressant drugs
Management ECT treatment should be given. Usually 6 to 10 ECTs
♦♦ Antidepressants are used in all phases of the treatment are effective in resolving acute depression. As course is
of major depression, acute management, continuation completed prophylactic treatment should be given to
therapy and maintenance or prophylactic treatment. prevent relapse
♦♦ Tricyclic drugs: They increase recovery rate significantly, ♦♦ Congenitive psychotherapy should also be done.

Fig. 26:  Cycle of depression

Q.28. Write short note on anxiety neurosis.  • Occur in absence of stressful circumstances
 (Mar 2008, 7 Marks) (Feb 2013, 5 Marks) • Impair physical, social or occupational functioning
Ans. Anxiety is a normal response to threat or stressful
Features of Anxiety Neurosis
events and is usually short lived and controllable.
Psychological
It probably functions as an ‘alarm mechanism’ to
prepare an individual for a physical response to ♦♦ Fear and apprehension
♦♦ Inner tension and restlessness
perceived danger (the fight or—flight’ response). Anxiety
♦♦ Irritability
symptoms are considered clinically significant when
♦♦ Impaired ability to concentrate
they: ♦♦ Increased startle response
• Are abnormally severe ♦♦ Increased sensitivity to physical sensations
• Are unusually prolonged ♦♦ Disturbed sleep
Section 1:  General Medicine  181

Physical behavior, headache should make one suspect tubercular


meningitis.
♦♦ Increased muscle tension
♦♦ Diagnosis shall be confirmed by the lumbar puncture. CSF
♦♦ Tremor
examination reveals following results:
♦♦ Sweating
• CSF is straw colored, clear but when allowed to stand,
♦♦ Palpitations
a fine clot, i.e. spider web is formed.
♦♦ Chest tightness and discomfort
• Lymphocyte count is high
♦♦ Shortness of breath
• Protein content is high
♦♦ Dry mouth
• Glucose is low
♦♦ Difficulty in swallowing
• In acute cases, polymorphs may predominate
♦♦ Diarrhea
• AFB stain can be positive. Culture or AFB is positive
♦♦ Frequency of micturition
in 80% of cases
♦♦ Loss of sexual interest
–– CT or MRI brain may show meningeal
♦♦ Dizziness
enhancement or hydrocephalous
♦♦ Numbness and tingling
♦♦ Faintness Management
Management ♦♦ General management
• Maintenance of nutrition, hydration and electrolyte
Definitive need for treatment depends on severity of symptoms,
balance.
degree of personal distress, level of occupational and social
• Case of bowel and bladder
impairment.
• Nursing should be good
♦♦ Benzodiazepines are effective anxiolytic drugs but can
• If there are convulsions, anticonvulsants are given.
cause sedation and have potential for dependence. They
♦♦ Treatment
should be given in short courses.
• Antitubercular drugs: Antitubercular drugs—
♦♦ Other drugs include certain tricyclic antidepressants e.g.
Rifampicin 600 mg/day + Isoniazide (600 to 900
imipramine (50 – 300 mg/d), buspirone (15–45 mg/d)
mg/day) + Pyrazinamide (1.5 gm) should be given.
and venlafaxine (serotonin—noradrenaline re – uptake
Treatment with this regimen is given for 2 months.
inhibitor 75 mg/d) none of which have same potential for
This is followed by rifampicin 600 mg/day + Isoniazide
psychological or physical dependence.
(600 to 900 mg/day) for 12 to 18 months.
♦♦ Behavior therapy in form of relaxation training, systemic
• Steroids: Prednisolone 40–60 mg/day to reduce toxicity,
desensitization and cognitive therapy. Psychotherapy and
pia-arachnoid adhesions and felling of well-being.
family education is must.
Q.30. Write short note on examination of cerebrospinal
Q.29. Describe clinical features, diagnosis and management fluid. (Sep 2009, 4 Marks)
of tubercular meningitis. (Mar 2011, 4.5 Marks)
Ans. Normal CSF is colorless, clear.
Ans.
• Presence of blood indicates either local trauma or
Clinical Features subarachnoid hemorrhage especially fresh.
• Xanthochromia or yellowish coloration of CSF is
Symptoms found in cerebral hemorrhage and when pus is
♦♦ Headache present in CSF in considerable amount.
♦♦ Vomiting • Turbidity when present indicates excess of
♦♦ Low grade fever polymorphonuclear cells (meningitis).
♦♦ Lassitude, i.e. weariness or exhaustion • A clot or cob web may form in cases of tuberculous
♦♦ Depression meningitis.
♦♦ Confusion • Of the biochemical tests, protein content is 30–40
♦♦ Behavior changes. mg/dL and sugar 80 mg/DL.
• Normal CSF contains a small number of cells mostly
Signs
lymphocytes (0–5/cumm).
♦♦ Meningism may be present: • Excess of cells in the CSF indicate meningeal
♦♦ Oculomotor palsies irritation.
♦♦ Papilledema • In cases of meningitis, CSF is examined for bacteria
♦♦ Depression of conscious level by gram’s staining.
♦♦ Focal hemisphere signs. • A culture examination of CSF is often carried out in
suspected infective process for identifying the causal
Diagnosis
organism.
♦♦ Person with history of contact with tubercular patient • CSF examination has an important role in diagnosing
presenting with low grade fever, ill health, weight loss, odd bacterial and viral diseases, but its role in cases of
182   Mastering the BDS IIIrd Year  (Last 25 Years Solved Questions)

cerebral hemorrhage is now being taken over by CT ♦♦ Psychological theories: In cognitive behavioral theory,
scan because of hazards involved. obsessions are considered anxiogenic. OCD patients cannot
• Colloidal reactions in the form of colloidal gold escape this anxiety and therefore develop compulsion in
reaction is of value in cases of general paralysis of an attempt to reduce or prevent the feared consequences.
insane (GPI) and differentiates it from other form of Reduction of anxiety reinforces the compulsive behavior.
neurosyphilis.
• Serological reactions like wasserman reaction are Symptoms and Signs
also of help in cases of neurosyphilis. Common obsessive thoughts in obsessive-compulsive disorder
Q.31. Differentiate between syncope and epilepsy. (OCD) include:
 (Sep 2008, 2.5 Marks) ♦♦ Fear of being contaminated by germs or dirt or
Ans. contaminating others.
♦♦ Fear of causing harm to yourself or others.
Features Epilepsy Syncope ♦♦ Intrusive sexually explicit or violent thoughts and images.
Precipitant Unusual Emotional, painful and ♦♦ Excessive focus on religious or moral ideas.
stressful event ♦♦ Fear of losing or not having things you might need.
♦♦ Order and symmetry: the idea that everything must line up
Circumstances Any Usually upright
posture, crowded or “just right.”
hot environment ♦♦ Superstitions; excessive attention to something considered
lucky or unlucky.
Onset Usually abrupt Usually gradual with
feeling of faintness, Common compulsive behaviors in obsessive-compulsive
nausea, sweating and disorder (OCD) include:
grayishness of vision ♦♦ Excessive double-checking of things, such as locks,
Skin color Pale or flushed Pale appliances, and switches.
Breathing Stertorous, foaming Shallow ♦♦ Repeatedly checking in on loved ones to make sure they’re
Incontinence Common Unusual
safe.
♦♦ Counting, tapping, repeating certain words, or doing other
Tongue biting Common Unusual
senseless things to reduce anxiety.
Vomiting Unusual Common ♦♦ Spending a lot of time washing or cleaning.
Injury Common Unusual ♦♦ Ordering or arranging things “just so.”
Postictal Drowsy, confused, Rapid recovery ♦♦ Praying excessively or engaging in rituals triggered by
headache, sleep religious fear.
Duration of Minutes Seconds ♦♦ Accumulating “junk” such as old newspapers or empty
unconsciousness food containers.

Diagnostic Criteria
Q.32. Write short note on obsessive compulsive disorder.
 (Sep 2009, 4 Marks) ♦♦ Obsessional thoughts, compulsive acts or both should be
Ans. Obsessive-compulsive disorder (OCD) is an anxiety present on most days for at least two weeks.
disorder characterized by intrusive thoughts that ♦♦ They are recognized by patients as their own
produce uneasiness, apprehension, fear, or worry; by ♦♦ Patients have tried to resist unsuccessfully at least one
repetitive behaviors aimed at reducing the associated obsessive thought or compulsive act.
anxiety; or by a combination of such obsessions and ♦♦ The thoughts or act are not pleasurable.
compulsions. ♦♦ The thoughts, images, impulses and acts are unpleasantly
repetitive.
Etiology
♦♦ Genetic: Obsessive compulsive disorder is widely Management
accepted to result from genetic vulnerability and/or Behavioral Therapy
chemical changes in some areas of the brain. The precise
pathogenesis is not completely understood. Twin studies The specific technique used in BT/CBT is called exposure and
show concordance rates as high as 87% in monozygous ritual prevention (also known as “exposure and response
twins compared with 47% in dizygous pairs. prevention”) or ERP; this involves gradually learning to tolerate
♦♦ Neurobiology: In OCD patients have shown abnormalities in the anxiety associated with not performing the ritual behavior.
orbitofrontal cortex, cingulate cortex and caudate nucleus. Exposure ritual/response prevention (ERP) has a strong
In some children and adolescents, OCD develops after β evidence base. It is considered the most effective treatment
hemolytic streptococcal infection, an autoimmune reaction for OCD. However, this claim has been doubted by some
similar to that of rheumatic fever. researchers criticizing the quality of many studies.
Section 1:  General Medicine  183

More recent behavioral work has focused on associative • Most commonly occlusion is in the common carotid
splitting. artery and one may be able to appreciate diminished
pulsation in the vessel in the neck. When obstruction
Medication is severe a bruit may be auscultated at the site.
♦♦ Medications as treatment include selective serotonin • Obstruction of other arteries like anterior cerebral
reuptake inhibitors (SSRIs) such as paroxetine (60 mg/day), artery, middle cerebral artery and posterior cerebral
sertraline, fluoxetine, escitalopram and fluvoxamine and artery produce picture almost like above except for
the tricyclic antidepressants, in particular clomipramine little variations depending on the occlusion site.
(150–300 mg/day). SSRIs prevent excess serotonin from • On the other hand involvement of posterior
circulation (basilar artery, vertebral artery, post-
being pumped back into the original neuron that released
inferior cerebellar artery) produces a picture
it. Over a period of several weeks, the increased levels of
of crossed hemiplegia, hemisensory loss and
serotonin downregulate the receptors, making them less
hemianopic visual loss. In addition patient has
responsive to 5-HT. This downregulation is concurrent
impairment of consciousness, small fixed pupils,
with the onset of any therapeutic benefits from SSRIS,
pseudobulbar palsy and quadriplegia seen mainly.
from 2–3 weeks. • In basilar artery lesions while cases with posterior
♦♦ The atypical antipsychotics olanzapine, quetiapine, and inferior cerebellar artery involvement are associated
risperidone have also been found to be useful as adjuncts to with severe vertigo, vomiting, dysphasia and
an SSRI in treatment-resistant OCD. However, these drugs diplopia. In addition there is some degree of cerebellar
are often poorly tolerated, and have significant metabolic deficiency with hypotonia and incoordination on the
side effects that limit their use. side of lesion, analgesia and thermoanesthesia on
the face on the side of lesion and on the trunk and
Cognitive Behavior Therapy
limbs on opposite side.
Main approach in OCD is graded exposure and self imposed • Again neurological deficits shall be depending on
response prevention. This require patient’s to face their feared which branch of the vessel is involved.
absessions without undoing them with their compulsions. Q.34. Write short note on facial pain. (Nov 2011, 3 Marks)
Exposure should be of sufficient duration to be effective. An
Ans. Various number of conditions are involved in the pain
effective method known as thought stopping may also be used.
localized to face. These may range from pain arising from
Combined Therapy diseases of teeth, gums, sinuses, tem­poromandibular
joint to various causes.
Combination of cognitive behavior therapy and medication can
be more effective than either alone. Facial Neuritis

Psychosurgery ♦♦ It is a form of inflammation of the nerve of the face and


scalp.
For some, medication, support groups and psychological ♦♦ It generally occurs as a complication of a septicemic estate
treatments fail to alleviate obsessive-compulsive symptoms. or due to involvement by a neurotropic virus.
These patients may choose to undergo psychosurgery as a last ♦♦ Onset is usually acute and pain is confined to the face and
resort. In this procedure, a surgical lesion is made in an area of scalp, occurring in paroxysms lasting for several hours
the brain (the cingulate cortex). and very often till the end of the day when the patient is
exhausted.
Q.33. Write short note on transient ischemic attack. ♦♦ If the character of pain is dull aching which is intensified by
 (Sep 2009, 5 Marks) exposure to cold and often occurs in the form of shooting
Ans. These are transient attacks of loss of function of one part pains in the distribution of the nerve. Sometimes the pain
of the brain coming suddenly and lasting for variable is so severe that the patient is unable to sleep.
period of time ranging from minutes to hours. ♦♦ Physical examination shows presence of hyperalgesia in
• Since arteries supplying the brain are end arteries the distribution of nerves including face and scalp. Nerve
so any pathology which produces obstruction to the trunk is tender on pressure.
flow of that vessel shall produce symptoms in the
distribution of that blood vessel. Myofascial Pain
• Internal carotid artery is one of the commonest ♦♦ It is a form of dull constant pain with local tenderness of
cerebral vessel which is involved by atherosclerosis the muscles of jaw.
and patient complains of transient disturbances ♦♦ There is often pain and difficulty in opening the mouth.
due to localized cortical ischemia in the form of ♦♦ This pain is related to bad and improper habits of clenching
confusion contralateral hemiparesis and sensory and grinding of teeth. This type of habit is present amongst
loss. There may be aphasia (if lesion on left side) hysterical persons especially women who often clench and
and hemianopic visual loss. grind their teeth.
184   Mastering the BDS IIIrd Year  (Last 25 Years Solved Questions)

♦♦ There is no physical finding in such people except that Q.35. Write short note on temporal lobe epilepsy.
these people have an emotionally labile personality and (Mar 2011, 3 Marks) (Dec 2012, 3 Marks)
often suffer from depression. Ans. Temporal lobe epilepsy is a form of focal epilepsy,
♦♦ Treatment consists in giving them assurance, analgesics a chronic neurological condition characterized by
and tricyclic antidepressants. recurrent seizures.
Temporal lobe epilepsy (TLE) is the most common single
Trigeminal Neuralgia
form causing refractory epilepsy.
Refer to Ans 2 of same chapter. Temporal lobe epilepsies are a group of medical
disorders in which humans and animals experience
Post Herpetic Neuralgia
recurrent epileptic seizures arising from one or both
♦♦ Herpes zoster commonly involves the ophthalmic division temporal lobes of the brain.
of the 5th nerve characterized by vesicular eruption on the
face and pain. When herpes heals it leave behind neuralgic Types
pain in the distribution of previous eruptions. Two main types are internationally recognized:
♦♦ It is a form of continuous aching or burning pain at that
♦♦ Medial temporal lobe epilepsy (MTLE): arises in the
site on face and patient is often in great agony.
hippocampus, parahippocampal gyrus and amygdala
♦♦ Treatment is by analgesics. Sometimes codeine phosphate
which are located in the inner aspect of the temporal lobe.
may have to be given.
♦♦ Lateral temporal lobe epilepsy (LTLE): arises in the neocortex
♦♦ In some severe cases the course of post herpetic neuralgia
on the outer surface of the temporal lobe of the brain.
may be prolonged one.

Migrainous Neuralgia Symptoms and Signs


♦♦ It is often called ‘Facio plegic migraine’ where there are ♦♦ Simple partial seizures (SPS) involve small areas of the
attacks of severe pain especially at night and the pain is temporal lobe such as the amygdala or the hippocampus.
confined to face and around one eye. This form of disease The term “simple” means that consciousness is not
is common in men especially middle aged and the attack altered. In temporal lobe epilepsy SPS usually only cause
may be brought on after a bout of alcohol. sensations. These sensations may be mnestic such as
♦♦ The pain may last for our. Because of associated vomiting déjà vu (a feeling of familiarity), jamais vu (a feeling of
and paroxysmal nature of disease it is called ‘migrainous unfamiliarity), a specific single or set of memories, or
neuralgia’ or ‘facioplegic migraine. There is headache and amnesia. The sensations may be auditory such as a sound
patient complains of some degree of congestion in the face. or tune, gustatory such as a taste, or olfactory such as a
♦♦ Treatment is by analgesics but response is poor. smell that is not physically present. Sensations can also
be visual, involve feelings on the skin or in the internal
Arthralgia of Temporomandibular Joint
organs. The latter feelings may seem to move over the
♦♦ It may be in the form of rheumatoid arthritis or ankylosing body. Dysphoric or euphoric feelings, fear, anger, and
spondylitis when there is pain and swelling of the joint. other sensations can also occur during SPS. Often, it is
♦♦ Movements at the joint are limited and patient complains hard for persons with SPS of TLE to describe the feeling.
of pain at the site as well as along the jaw confined to face. SPS are often called “auras” by lay persons who mistake
♦♦ Involvement of other joints in the body shall favor the them for a warning sign of a subsequent seizure. In fact,
diagnosis. they are actual seizures in and of themselves. Persons
♦♦ Treatment is by heat and anti-inflammatory drugs as experiencing only SPS may not recognize what they are
well as by exercises of the joint involved with adequate or seek medical advice about them. SPS may or may not
periods of rest. progress to the seizure types listed below.
♦♦ Complex partial seizures (CPS) by definition are seizures
Temporal Arteritis which impair consciousness to some extent. This is to say
♦♦ It is a form of collagen disorder of unknown etiology which that they alter the person’s ability to interact with his or
involves mainly the arteries. her environment. They usually begin with an SPS, but then
♦♦ It commonly occurs in elderly age group. the seizure spreads to a larger portion of the temporal lobe
♦♦ Patient may complain of pain on the face, jaw, mouth and resulting in impaired consciousness. Signs may include
tongue in the distribution of branches of external carotid motionless staring, automatic movements of the hands or
artery. mouth, altered ability to respond to others, unusual speech,
♦♦ This pain worsens off if on eating and opening of the or unusual behaviors.
mouth. ♦♦ Seizures which begin in the temporal lobe but then spread
♦♦ Since temporal arteritis is a collagen disorder there is a to the whole brain are known as secondarily generalized
form of inflammation of the arteries. Tonic-Clonic Seizures (SGTCS).
Section 1:  General Medicine  185

Treatment • As convulsion ceases, patient is turned into semiplane


position and make air passage clear.
♦♦ In temporal lobe epilepsy, the most commonly used drugs
• Patient is advised to consult the doctor for medical
are phenytoin, carbamazepine, primidone, valproate and
advice.
phenobarbital.
♦♦ Drug therapy:
♦♦ Newer drugs, such as gabapentin, topiramate, levetiracetam,
Choice of the drug depends on type of seizure.
lamotrigine, pregabalin, tiagabine, lacosamide, and
• In localization related epilepsy first line prophylactics
zonisamide promise similar effectiveness, possibly with
are lamotrigine, carbamazepine (slow-release),
fewer side-effects.
oxcarbamazepine, levetiracetam.
♦♦ For patients with medial TLE whose seizures remain
• In primary generalized, symptomatic generalized
uncontrolled after trials of several antiepileptic drugs,
and unclassified epilepsies, initial therapy should
resective surgery should be considered. be lamotrigine or valproate (broad spectrum gents).
Q.36. Write in brief signs, symptoms and treatment of Valproic acid is useful in absences and benzodiazepines
epilepsy. (Apr 2008, 5 Marks) in myoclonic jerks.
Or • Patients with refractory generalized epilepsy may
benefit from adjunctive treatment with topiramate or
Write short note on epilepsy. (Jan 2016, 5 Marks)
levetiracetam, zonisamide, clonazepam, pregabalin.
Ans. Epilepsy is defined as the group of disorders in which
• Various anti-epileptic drugs along with their dosages
there are recurrent episodes of altered cerebral functions
used in epilepsy are:
associated with paroxysmal and hypersynchronous
electrical discharge of cerebral neurons. Name of the drug Dosage
In generalized tonic clonic (grand mal) or partial (focal) seizures
Symptoms
Phenytoin 200 to 400 mg
♦♦ Aura is present which means there is warning of the attack.
Adult 3 to 5 mg/kg given in BD dose
♦♦ Loss of consciousness
♦♦ Patient fall over the ground Carbamazepine 600 to 1200 mg TDS
♦♦ Irritability Valproic acid 800 to 2000 mg TDS
♦♦ Depression and abnormal feelings Phenobarbital 60 to 180 mg BD
♦♦ Giddiness and abdominal cramps
Primidone 750 to 1500 mg TDS
♦♦ When patient awakes he complains of headache
Lamotrigine 200 to 500 mg BD
Signs
Topiramate 200 to 400 mg BD
♦♦ Tonic contraction of muscles and epileptic cry produced by Oxcarbazepine 900 to 1800 mg BD
forceful expiration through the partly closed vocal cords.
Levetiracetam 1000 to 3000 mg BD
♦♦ Tonic convulsions of two sides of the body occur with head
and eyes rotated to one side. Zonisamide 200 to 600 mg BD
♦♦ Tonic phase followed by clonic phase, i.e. after completion Pregabaline 150 to 180 mg BD
of tonic phase patient remain unconscious for few minutes
Absence (Petit mal) seizures
to half an hour.
♦♦ Frothing of mouth and increased salivation from the Ethosuximide 500 to 1500mg TDS
mouth. Valproic acid 1500 to 2000mg TDS
♦♦ During clonic phase patient may bite his tongue and pass Clonazepam 1 to 6mg OD or BD
urine and stools.
Myoclonic seizures
Treatment Valproic acid 1500 to 2000mg TDS
Treatment of epilepsy is directed at the elimination of the cause Clonazepam 1 to 6mg OD or BD
of seizures, suppressing the expression of seizures and dealing
with psychosocial consequences.
Plan of Treatment
♦♦ Immediate treatment of a seizure:
• Patient should always be migrated to the safer place. ♦♦ Initial regimen: The drug selected must be used in
• Loose the clothes around the neck and move people monotherapy. The drug should be introduced in small
away from the place so that patient can breathe fresh doses, since rapid introduction may cause side effects.
air easily. ♦♦ Maintenance treatment: The aim should be to find the
• At the time of convulsion, helpers are not allowed lowest dose which achieves complete seizure control
to put the fingers inside the mouth. Tongue biting without side effects which may be either idiosyncratic
should be prevented by putting tightly rolled piece or due to intoxication, or chronic. Serum anticonvulsant
of cloth in mouth. levels are a useful guide to therapy – phenytoin (40–80),
186   Mastering the BDS IIIrd Year  (Last 25 Years Solved Questions)

carbamazepine (20–50), phenobarbitone (40–170), ♦♦ Prevention of facial sagging: Application of strips of adhesive
ethosuximide (20–600). Valproate 300-600, oxcarbazepine tape is done to lift up the angle of mouth. Tape is attached
50–125, lamotrigine 4–60 mmol/L. If the optimum level to the temple and extends down in a V shaped fashion to
of a single, first line drug does not control seizures, or if upper and lower lips.
side-effects develop, the initial drug should be substituted ♦♦ Protection of eye: It is done with dark glass or eye patch.
with another first line anticonvulsant. If the second drug Mild zinc boric solution is used to wash the eye to prevent
also fails to control seizures monotherapy with a third conjunctivitis.
anticonvulsant, or combination therapy with two first line ♦♦ Corticosteroids: If seen under a week of onset. Prednisolone
drugs should be tried. 40 mg/day for 4 days and in tapering doses for over next
If a combination of two first-line drugs is unsuccessful, one 6 days helps by reducing secondary edema.
of the second-line drugs may be considered. ♦♦ Anti-virals: Acyclovir, Valacyclovir or Famciclovir in
combination with steroids, if strated within 3 days of onset.
Drug Withdrawl ♦♦ Surgery: Decompression of facial nerve in second or third
It should take place slowly over 2 to 3 months. If patient is week cannot influence favorably natural course of Bell’s
receiving more than one drug, each drug should be withdrawn palsy. Cases which fail to recover after 9 months in them
individually. anastomosis of facial nerve with accessory or preferably
hypoglossal nerve is considered, or plastic surgery in cases
Q.37. Write in brief signs, symptoms and treatment of facial of total paralysis with atrophy of muscle.
palsy. (Apr 2008, 5 Marks)
Q.38. Write on causes of meningitis and discuss in detail
Or
bacterial meningitis. (Nov 2008, 15 Marks)
Write signs and symptoms of facial palsy.
 (Aug 2012, 5 Marks) Or
Ans. It is the paralysis of the facial nerve. Describe the etiology, clinical features and management
of bacterial meningitis. (Dec 2010, 15 Marks)
Symptoms Ans. Following are the causes of meningitis:
♦♦ Post auricular pain is common and may precede paralysis • Neonates: E.coli
by 2 days. • Children:
♦♦ Spontaneous complains of loss of taste sensation, – H. influenzae
hyperacusis and watering of eyes. – S. pneumoniae
♦♦ Sweating is less over the affected side. – N. meningitidis.
• Adults:
Signs – Young people: Meningococcus
♦♦ Forehead cannot be wrinkled; frowning lost. – Older people: S. pneumonia.
♦♦ Eye of affected cannot be closed. On attempting closure, • Elderly and immunocompromised persons:
eyeball turns upwards and outwards (Bell’s phenomena). – Pneumococcus
♦♦ On showing the teeth, the lips do not separate on affected – Listeria
side. Whistling not possible. Articulation of labial – Tuberculosis
components difficult. Naso-labial fold flattened out. Angle – Gram-negative organism
of mouth on affected side droops with dribbling of saliva. – Cryptococcus.
♦♦ Cheek puffs out with expiration because of buccinator • Viral:
paralysis. Food collects between teeth and paralyzed cheek. – Enterovirus
Fluid runs out while drinking. – Herpes simplex virus
♦♦ Base of tongue is lowered. – Mumps virus
♦♦ Vesicles within the external auditory meatus and ear – Influenza virus
drum in Ramsay Hunt syndrome. Pain may precede facial – Japanese encephalitis virus
weakness. Deafness may result. – Arbo viruses
– Rabies virus
Treatment – HIV.
♦♦ Local heat: Infrared or moist heat over the face or parotid • Nosocomial and post-traumatic meningitis:
region or both if there is tenderness of nerve trunk. – Klebsiella pneumonia
♦♦ Local treatment of muscles: The patient should massage the – E.coli
facial muscles with bland oil for twice a day for 5 min. – Pseudomonas aeruginosa
The massaging movements should start from the chin and – S. aureus.
lower lip and are directed upwards. With return of function • Meningitis in special situation:
the patient should practice movements of various muscles – CSF shunts – staphylococcal
of face before a mirror. – Spinal procedures- pseudomonas.
Section 1:  General Medicine  187

Bacterial Meningitis Management


Pathophysiology Involves ♦♦ For adult patients penicillin G 5 to 10 million units IV 6
hourly.
♦♦ Transmission, colonization and invasion of nasopharyngeal
♦♦ Cephalosporins, i.e. cefotaxime 2 gm IV or ceftriaxone 2
epithelium.
gm IV OD is also effective.
♦♦ Survival in the blood stream by evading host immune
Patients allergic to penicillin are treated with
response.
chloramphenicol 1 gm IV 6 hourly
♦♦ Meningeal invasion. Bacteremia may be rapidly followed
♦♦ Treatment is continued for 7 to 10 days.
by seeding of meningeal pathogens and secondary
♦♦ For raised intracranial tension IV mannitol is given which
infection of the meninges.
is accompanied by high doses of dexamethasone 4 mg IV
♦♦ CSF inflammatory response.
6 hourly.
♦♦ Cerebral edema and thrombosis.
The supportive treatment is to maintain nutrition, fluid
♦♦ Bacterial meningitis causes loss of cerebrovascular
and electrolytic balance.
anticoagulation.
Q.39. Enumerate five causes of headache.(Dec 2009, 5 Marks)
Clinical Features Or
Symptoms Enumerate causes of headache. (Jan 2012, 5 Marks)
Ans. Following are the causes of headache:
♦♦ Fever, malaise headache and vomiting. • Intracranial and local extracranial
♦♦ Pain over the neck – Trauma
♦♦ Stiffness over the neck
– Intracranial inflammations
♦♦ Confusion, delirium and coma.
– Vascular headaches: Hypertension, cerebral
Signs or subarachnoid haemorrhage, intracranial
aneurysm, vasodilator drugs like nitrites and
♦♦ Kernig’s sign is positive.
histamine, adrenaline.
♦♦ Neck rigidity, i.e. when neck is bended there is presence
– Traction headache: Pain produced by intracranial
of pain and there is spasm of neck muscles.
arterial displacement and distortion of the dura
♦♦ Brudzinski’s sign is also positive.
– Post-lumbar puncture headache: Low CSF
♦♦ Photophobia is present
pressure headache.
♦♦ Cranial nerve palsies most commonly IIIrd, IVth, VIth
– Cough headache: A benign syndrome of severe
and VIIth.
headache which accompanies coughing,
♦♦ Focal neurological deficits such as nystagmus, aphasia, straining or sneezing can be due to posterior
ataxia and peripheral nerve palsies. fossa tumor.
♦♦ Partial or generalized seizures tend to be more common • Cranial neuritis and neuralgias of sensory nerves of
in Strep. pneumoniae and HIV meningitis. scalp, e.g. orbital neuralgia, neuralgia of auriculo-
♦♦ Purpura or petechiae in meningococcal meningitis, with temporal, posterior auricular or great occipital nerves.
or without features of septic shock. • General or systemic causes:
– Anoxemia: Anaemia, carbon monoxide or carbon
Diagnosis
dioxide poisoning.
♦♦ Examination of CSF – Toxic: Fever, uremia, metallic poisoning,
Typical CSF findings in acute bacterial meningitis: “alcoholic” hangover etc
• Raised WBC count (usually 100–60,000 cells/mL – Metabolic factors: Hypoglycemia, alkalosis or
(predominantly neutrophils) acidosis
• Reduced CSF glucose (30–40% serum glucose level) – Hemopoietic factors: Essential polycythemia,
• Raised CSF protein (0.5–5 g/L) thrombasthenia.
• Gram-staining of CSF is positive in over 90% of cases • Referred pain
of hematologically acquired meningitis. – Eyes: Errors of refraction, glaucoma, iritis, etc.
♦♦ Blood culture should be performed in all patients with – Ears: Otitis, mastoiditis, vestibular nerve lesions,
suspected meningitis, and latex agglutination bacterial – Teeth: Impacted teeth, infected tooth sockets and
antigen test or polymerase chain reaction analysis (to dental roots
detect bacterial DNA) may be performed on blood or CSF – Paranasal sinuses: Infection of paranasal sinuses
to try to obtain a diagnosis. Such tests remain positive for may cause localised pain
several days after administration of antibiotics. Laboratory – Neck: Diseases of upper cervical spine may be
markers of poor prognosis include low peripheral WBC associated with both occipital and frontal pain.
count, thromobocytopenia, absence of CSF pleocytosis and • Psychogenic:
high CSF protein levels. Common cause of headache in depression.
188   Mastering the BDS IIIrd Year  (Last 25 Years Solved Questions)

• Tension (muscle contraction) headache: Pain ♦♦ Vascular pain:


resulting from sustained contraction of skeletal • Migraine: Episodes of severe and continuous pain,
muscles of the neck, frontalis, occipital muscles due often burning in character in or behind one eye, or
to emotional tension. in cheek, forehead and temple. Often suffusion of
• Exertional headache: Headache may come on during conjunctiva and blocking of nostril on that side.
exertion and persist for few hours afterwards. • Temporal arteritis: Pain of dull, throbbing nature
Q.40. Write in brief sign, symptoms and treatment of with associated scalp tenderness. Thickened temporal
bacterial meningitis. (Jun 2010, 5 Marks) arteries with reduced or absent pulsations.
Ans. Refer to Ans 38 of same chapter. ♦♦ Other neuralgias
• Post-herpetic neuralgia: Pain is continuous in nature.
Q.41. Enumerate causes of meningitis and describe clinical Vesicles and scars of herpetic infection.
features, complications and treatment of tubercular • Glossopharyngeal neuralgia: Pain in tonsillar fossa, back
meningitis. (Aug 2011, 15 Marks) of throat and larynx; may radiate to ear on affected
Ans. For causes of meningitis refer to Ans 38 of same chapter. side. Swallowing is the stimulus most likely to produce
For clinical features refer to Ans 16 and for management pain.
refer to Ans 15 of same chapter. ♦♦ Atypical facial pain
Intermittent but long – lasting pain of aching character
Complications which affects the cheek and upper jaw, often bilateral
♦♦ Infection spreads in parenchyma of brain causing and occurs almost exclusively in young and middle—
meningoencephalitis. aged women. Generally believed to be a manifestation of
♦♦ Involvement of cerebral vessel causes obliterative endarteritis depression or anxiety.
♦♦ Thrombosis of cerebral vessels lead to cerebral infarction ♦♦ Miscellaneous
♦♦ Delayed complications are development of hydrocephalus • Clonic facial spasm: Sometimes painful, usually
optic atrophy, spinal cord compression and cranial nerve associated with intermittent twitching of eyelid
palsy. and face on one side. Platysma usually involved in
Q.42. Write notes on facial pain. (Aug 2011, 10 Marks) twitching.
Ans. For various types of facial pains refer to ans34 of same • Neuralgic pain: Occasionally associated with facial
chapter. hemiatrophy.
• Idiopathic trigeminal neuropathy: It is commonly
Differential Diagnosis of Facial Pain associated with muscle wasting of masseter.
♦♦ Symptomatic trigeminal neuralgia Q.43. Write short note Grand-mal epilepsy.
• Neuralgia indistinguishable from the idiopathic  (Jan 2012, 5 Marks)
variety may occur as a result of compression of Ans. It is a type of a primary generalized seizure.
trigeminal root or ganglion, e.g. meningioma, acoustic • It is a true seizure.
neuroma, aneurysm of basilar artery, arteriovenous • It is also known as tonic clonic seizure.
malformations, basilar invagination, epidermoid • Grand mal epilepsy comes suddenly without
cholesteatomas in the cerebellopontine angle, as a warning in some patients while in others it is
result of Paget's disease or osteomalacia. preceded by various phases or symptoms.
• Paratrigeminal neuralgia (Raeder’s syndrome): Severe • Following are the phases:
pain in and around one eye accompanied by Horner’s – Prodromal phase: In this phase patient become
syndrome on the affected side. It is continuous and uneasy and irritable for some hours to days
progressive and is usually caused by a structural before an attack.
lesion, often malignant in the base of the skull – Aura: It is seen when partial seizure become
involving the paratrigeminal region. generalized so the symptoms of partial seizures
• Multiple sclerosis: Diagnosis of multiple sclerosis such as flashing of light, hallucination of
should always be suspected in a young patient with hearing words or sounds, Tingling or numbness
trigeminal neuralgia. sensation, pain in epigastrium and unnatural
• Syringobulbia. sensations in some part of the body. Aura is
• Painful superior orbital fissure syndrome (Tolosa- warning of an attack and is produce by an
Hunt syndrome) caused by granulomatous tissue activation of epileptic discharge in brain. It
involving nerves III, IV and VI. Pain with development remains for some seconds or minutes.
of ocular palsies and loss of first division trigeminal – Tonic clonic phase: Tonic convulsions of two side
sensation. of body occur with head and eyes rotated to
♦♦ Referred pain: Paranasal sinuses, toothache, aural infection. one side. Tonic Phase occur for 10–30 seconds
Temporomandibular joint dysfunction (Costen’s syndrome) and there is presence of flexion of arms, tonic
— abnormality of bite and aggravated by chewing. contraction of muscles, a cry due to spasm of
Section 1:  General Medicine  189

respiratory muscles and extension of legs. Tonic


phase is followed by clonic phase which last
11. Tropical and
for 1–5 minutes. In clonic phase there is violent Infectious Diseases
jerking of face and limbs, biting of tongue and
passing of urine and stools. Clonic phase is Q.1. Enumerate the causes of enteric fever. 
followed by unconsciousness which lasts for few  (Mar 1998, 5 Marks) (Apr 2010, 5 Marks)
minutes to half an hour. Ans. Enteric fever is an infectious disease
– Postictal phase: This phase last for few minutes
to hours. There is presence of unconsciousness Causative Agent
with flaccid limbs, corneal reflexes are lost and
♦♦ Salmonella typhi and Salmonella paratyphi
plantar extensor response may occur during
♦♦ The organisms are gram-negative nonspore forming bacilli.
this phase. In this patient has severe headache,
confusion and automatic behavior. Predisposing Factors

Diagnosis of Grand mal Epilepsy ♦♦ Organisms: A large number of organisms are ingested by
the healthy person to suffer from typhoid. Smaller inocula
♦♦ Diagnosis of grand mal epilepsy is made by careful may produce the disease. If the organisms are very virulent
assessment of patient’s history documented by the or if the resistance of host is poor.
diagnostic studies. ♦♦ Stomach acidity: Acid in the stomach destroys Salmonella.
♦♦ These include blood test to assess for the metabolic Hence, patients having achlorhydria or who take large
disarray, brain imaging using MRI or CT scan and amount of antacids to neutralize the acid in stomach suffer
EEG. Normally EEG shows series of small alpha waves more often from the typhoid.
about 10per second and occasionally small beta waves, ♦♦ Intestinal flora: Normal intestinal flora produces short chain
but during attack in Grand mal epilepsy series of sharp fatty acids which are lethal to Salmonella. When these are
spikes is present. Between the attacks 3 Hz 5W interictal reduced by antibiotics the patient is more prone to typhoid.
epileptiform activity there is presence of intermittent
irregular slow waves in grand mal epilepsy. Q.2. Describe clinical signs and symptoms of enteric
♦♦ It also depends upon its onset and symptoms like biting fever. (Sep 1998, 5 Marks)
of tongue, passing of urine in clothes, injury to the patient, Ans. Clinical signs and symptoms of enteric fever.
loss of consciousness and postepileptic features.
Invasion (1st Week)
Management ♦♦ Onset: Lassitude, headache, bodyache and anorexia.
♦♦ Tongue is coated with raw tips and edges.
♦♦ Immediate treatment of an attack of fit.
♦♦ Abdominal discomfort and distention occurs with nausea,
• Patient should be protected from the injury. He should
vomiting and constipation which are followed by diarrhea.
be moved away from fire and sharp and hard object.
♦♦ Hepatomegaly
• Padded mouth gag is inserted between the teeth to
♦♦ Fever: It may show step ladder rise.
avoid tongue injury.
♦♦ Signs of bronchitis are common. Epistaxis may occur.
• Clear airway should be maintained. ♦♦ Pulse: Relative bradycardia and *dicrotic pulse.
• Diazepam 5 to 10 mg slow IV injection is given.
♦♦ Long-term drug therapy. Advance (2nd Week)
• Phenytoin sodium 200 to 400 mg daily. ♦♦ General state: Listlessness and *apathy.
• Carbamazepine 600 to 1800 mg daily in divided ♦♦ Abdomen:
dose. • Spleen become palpable
• Sodium valproate 0.25 to 1 mg daily. • Increased abdominal distention and discomfort
• Phenobarbitone 60 to 180 mg daily. • Usually there is diarrhea.
• Primidone 750 to 1500 mg daily in divided dose. ♦♦ Temperature: It is high with slight morning remissions.
♦♦ Social and psychological aspects. ♦♦ Rash (Rose spots): These are erythematous maculopapular
• Patients and relatives should be told about the illness, lesions 2–4 mm in diameter which *blanch on pressure
its precipitating factors and consequences. usually seen on the upper abdomen, back and chest.
• Restriction should be in children as they are more
likely to be in danger. Cycling, driving and swimming Decline (3rd Week)
is avoided. Mild case: Toxemia *abates and gradual fall of temperature.
• Patient should be advised to take occupation in which Severe case: Increased toxemia, intestinal hemorrhage or
neither he nor the community is on risk. perforation.
190   Mastering the BDS IIIrd Year  (Last 25 Years Solved Questions)

Q.3. Describe briefly complications of enteric fever and Q.4. Write short note on typhoid.
their management.  (Dec 2015, 5 Marks) (Feb 2006, 3 Marks)
 (Mar 2001, 5 Marks) (Oct 2007, 5 Marks) Or
Ans. Complications of enteric fever with their management. Write short note on enteric fever.
 (Dec 2010, 5 Marks) (Apr 2008, 5 Marks)
Hemorrhage
Ans. Typhoid is an acute systemic illness caused by infection
Seen at the end of 2nd week and early 3rd week from gastric due to salmonella typhi.
ulcer.
Epidemiology
Treatment
Typhoid germs are contracted from food or drink contaminated
♦♦ Absolute bed rest. with excreta from carriers or patients. Spread is facilitated by
♦♦ Repeated blood transfusion poor environmental hygiene.
♦♦ Morphine. 15 mg SC.
Clinical Features
Shock
Invasion (1st Week)
Treatment
♦♦ Onset: Lassitude, headache, bodyache and anorexia.
♦♦ Plasma transfusion ♦♦ Tongue is coated with raw tips and edges.
♦♦ Oxygen ♦♦ Abdominal discomfort and distention occurs with nausea,
♦♦ Vasopressor drugs vomiting and constipation which are followed by diarrhea.
♦♦ IV hydrocortisone or dexamethasone. ♦♦ Hepatomegaly
♦♦ Fever: It may show step ladder rise.
Perforation
♦♦ Signs of bronchitis are common. Epistaxis may occur.
It is most dangerous complication leading to peritonitis. ♦♦ Pulse: Relative bradycardia and *dicrotic pulse.

Treatment Advance (2nd Week)


♦♦ Gastric suction ♦♦ General state: Listlessness and *apathy.
♦♦ IV fluids ♦♦ Abdomen:
♦♦ Broad spectrum gram-negative agents. • Spleen become palpable
• Increased abdominal distention and discomfort
Hepatitis • Usually there is diarrhea.
♦♦ It is represented by hepatomegaly and jaundice. ♦♦ Temperature: It is high with slight morning remissions.
♦♦ Treatment is symptomatic. ♦♦ Rash (Rose spots): These are erythematous maculopapular
lesions 2–4 mm in diameter which blanch on pressure
Cholecystitis usually seen on the upper abdomen, back and chest.
Salmonella typhi has affinity for gallbladder and can produce Decline (3rd Week)
inflammation.
♦♦ Mild case: Toxemia *abates and gradual fall of temperature.
Treatment ♦♦ Severe case: Increased toxemia, intestinal hemorrhage or
perforation.
Ampicillin or ciprofloxacin is required.
Cholecystectomy is done if above treatment fails. Investigations

Toxemia First Week


♦♦ Normochromic normocytic anemia, leukopenia and
Treatment
albuminuria
Hydrocortisone 200 mg or Dexamethasone 8 mg parenterally ♦♦ Blood culture may be positive in 70–90% of cases.
followed by 45 mg prednisolone.
Second Week
Meningitis ♦♦ Anemia, leukopenia may persist
♦♦ Widal test become positive, may show four fold rise in
Treatment
agglutinins against somatic ‘O’ antigen. It is not specific
Cefotaxime 2 gm IV 4 hourly or ceftazidine 2 gm 6 hourly and but rising titers are diagnostic.
gentamicin. ♦♦ Blood culture may be positive only in 50%.
Section 1:  General Medicine  191

Third Week Q.5. Describe briefly typhoid vaccination.


♦♦ Anemia and leukopenia persist. Leukocytosis occur in  (Sep 1999, 3 Marks)
severe septicemia Ans. Following are the typhoid vaccinations present:
♦♦ Blood culture is positive in 30–45% of cases. ♦♦ Anti-triple typhoid vaccine containing in each mL. 1000
♦♦ Positive WIDAL test with rising titers million typhoid organisms, 250 million each of para-
♦♦ Positive stool culture and urine test for S. typhi. typhoid ‘A' and ‘B’ organisms. Course of 3 injections of
0.5 mL, 1 mL and 1 mL subcutaneously at intervals of not
Complications less than 7 days or not more than 28 days. Immunity lasts
for about 12 months.
Following are the complications of typhoid fever:
♦♦ V1 capsular polysaccharide typhoid vaccine: 25 mg in each
♦♦ Intestinal complications:
dose. Single dose (0.5 mL) SC or IM gives protection for
• Hemorrhage 3 years. However, the V1 antigen does not invariably
• Perforation provoke V1 antibody. Booster every 2 years.
• Paralytic ileus ♦♦ TY21a—oral vaccine: TY21a is a galactose epimerase mutant
• Peritonitis S. typhi given as oral enteric coated capsules. The bacilli
♦♦ Extra – intestinal complications: invade mononuclear cells and undergo 4–5 cell divisions
• Meningitis in intestinal tract. This stimulates immunity, but the bacilli
• Bone and joint infection do not survive within the cells, as they lack the essential
• Cholecystitis enzyme UDP—galactose-4 epimerase and are therefore
• Encephalopathy avirulent. The vaccine stimulates cell mediated immunity
• Pneumonia and also stimulates intestinal IgA.
• Granulomatous hepatitis ♦♦ Dose: One capsule on days 1, 3 and 5 irrespective of age one
• Nephritis hour before meal with milk or water. Not recommended
• Myocarditis for children under 6 years of age. Protection commences
2 weeks after last capsule and lasts for at least 3 years.
Treatment Booster is given every 5 years.
Specific Q.6. Describe briefly management of malaria. 
♦♦ Ciprofloxacin: It is given in the dose of 500 mg BD for 7 to  (Sep 1999, 5 Marks)
10 days it is avoided in children because of risk of cartilage Or
damage and tendonitis. If absolutely required, low dose Write short note on treatment of malaria.
can be used for not more than 3 days.  (Feb 2013, 5 Marks)
♦♦ Ceftriaxone: It is 3rd generation cephalosporin and Ans. Malaria is a common topical disease caused by protozoa,
improves the condition rapidly. It is given in the dose of Plasmodium.
1 gm BD for 10 to 14 days. Acute febrile illness characterized by paroxysms of fever
♦♦ Azithromycin: 1 gm OD for 5 days. as a result of asexual production of Plasmodium within
the red cells.
Supportive Treatment
♦♦ Treatment of fever – paracetamol Management
♦♦ Good nursing care. General Management
♦♦ Nutritious diet should be given. 3000 calories per day
♦♦ Measurement of glucose and if possible lactate and arterial
should be given
blood gases.
♦♦ Fluid and electrolyte balance should be maintained.
♦♦ Fluid balance should be maintained because both
♦♦ For severe toxemia and peripheral circulatory failure:
dehydration and overhydration can occur as a result of
Dexamethasone 3 mg/kg stat followed by 8 doses of 1mg/
disease or treatment.
kg 6 hourly for 48 hours each given by IV infusion over ♦♦ Treatment of convulsions with diazepam.
30 min. ♦♦ Attention should be given to hypoglycemia and
Treatment of Carrier hyponatremia.
♦♦ Blood should be taken for cross-matching and coagulation
Patients who are asymptomatic, but constantly releases bacteria studies
in stool because bacteria are persisting in gallbladder. ♦♦ Parameters for monitoring treatment include twice daily
♦♦ Ampicillin 500 mg QID for six weeks. parasite counts, regular pH and blood gas measurements
♦♦ Ciprofloxacin 500 mg BD for 2 to 4 weeks. and when appropriate, measurement of glucose (during
♦♦ Cholecystectomy if above measure fails. iv quinine therapy), lactate, CRP and kidney function.
192   Mastering the BDS IIIrd Year  (Last 25 Years Solved Questions)

Specific Treatment sensitivity of P.falciparum to anti-malarial drugs, i.e.


♦♦ Treatment of chloroquine susceptible P. vivax, Quinine or chloroquine is given as an intravenous
P. falciparum, P. ovale, P. malariae chloroquine: infusion over 2 to 4 hours to avoid acute circulatory
This is given in the dose of 600 mg base followed by 300 failure or acute malarial encephalopathy. Quinine
mg at 6th, 24th and 48th hours. It is useful in trating all should be used in chloroquine-resistant cases. The
types of malaria. It is curative for P. falciparum malaria but starting dose of chloroquine is 5 mg/kg and of quinine
cannot prevent relapses due to exoerythrocytic cycles of is 10 mg/kg. The dose should be repeated at intervals
P. vivax malaria. of 8–12 hours until the patient can take the drug orally.
♦♦ Treatment of chloroquine resistant P. falciparum: The total dose of chloroquine is 25 mg/kg. The quinine
• Quinidine IV 10 mg/kg dissolved in 300 mL normal is continued orally in the same dose for 7–10 days.
saline infused over 1 to 2 hour. Recently artemisinin (artemether) has become first
• Quinine hydrochloride: 600 mg TDS for 3–7 days is line treatment of severe malaria as mentioned by
useful. WHO. Artesunate 2.4 mg/kg IV then 1.2 mg/kg/day
If required for cerebral malaria, this drug can given IV or artemether 3.2 mg/kg, then 1.6 mg/kg IM daily is
the dose is 7 mg/kg over 30 min, followed by 10 mg/kg given for 3–5 days.
over 4 hours and then 10 mg/kg over 8 hours or until • Single dose of parenteral phenobarbitone 5–20 mg/kg
the patient can complete a week of oral treatment. is given to prevent convulsion.
• Mefloquine: It provides rapid schizonticidal action in • If severe anaemia is present then transfusion with
single dose of 15 mg/kg orally maximum dose is 1000 packed red cells is done.
to 1250mg. • If oliguria develops, frusemide or an infusion of
• Halofantrine: mannitol is given to prevent renal failure.
Dosage: Adult 500 mg BD for 4–6 days. • Intravenous fluids are given, if necessary should be
Children: 8 mg/kg guided by central venous pressure because pulmonary
• Artemether: This drug is rapidly acting, safe and edema may develop if the patient is over-infused.
is effective against multidrug resistant infections. • Exchange blood transfusions is life saving in
Artemether 3.2 mg/kg IM is given followed by 1.6 complicated very severe infection (over 10% of RBCs
mg/kg IM every 12 to 24 hours until patient wakes are infected).
up. Artesunate 2 mg/kg IV stat followed by 1 mg/Kg • Hypoglycemia and septicemia may be treated
12 hourly. appropriately. All patients treated with quinine
• Sulfadoxine and pyrimethamine: Combination of should be monitored for blood sugar and should
sulfadoxime 1500mg and pyrimethamine 25mg helps receive 5–10% glucose as continuous infusion during
to cure an acute attack of chloroquine resistant malaria. treatment.
♦♦ Treatment of chloroquine resistant P.vivax • If renal failure develops dialysis should be done.
Oral mefloquine and halofantrine. • Care of unconscious patient or if patient undergo
♦♦ Treatment of persistent hypnozoites in P. vivax or P. coma.
ovale infection
• Primaquine: It is given in the dose of 7.5mg BD for 14 Q.8. Write short note on diphtheria.  (Apr 2007, 5 Marks)
days usually after doing a G6PD test Ans. Diphtheria is an acute infectious disease caused by
• Bulaquine: It is given 25mg OD for 5 days. Corynebacterium diphtheriae and is characterized by the
local exudates on the mucous membrane of nose, throat
Q.7. Outline the management of F. malaria.
 (Feb 1999, 4 Marks) and larynx and systemic toxemia.
Ans. For complications refer to Ans 9 of same chapter.
♦♦ Treatment of chloroquine susceptible P. falciparum
Types of Diphtheria
Chloroquine: This is given in the dose of 600 mg base
followed by 300 mg at 6th, 24th and 48th hours. Pharyngeal Diphtheria
♦♦ Treatment of chloroquine resistant P. falciparum:
♦♦ It is characterized by the toxemia, *congestion and edema
• Quinidine IV 10 mg/kg dissolved in 300 mL normal
saline infused over 1 to 2 hours. of palate.
• Quinine hydrochloride: 600 mg TDS for 3-7 days is ♦♦ There is formation of a membrane in pharynx which is
useful. generally thin glistening pearly white in early stage and
• Mefloquine: It provides rapid schizonticidal action in become thick, grayish and opaque later on.
single dose of 15 mg/kg orally maximum dose is 1000 ♦♦ There is lymphadenopathy in neck and breath is foul
to 1250 mg. smelling.
♦♦ Treatment of complicated P. falciparum malaria ♦♦ In second week there is lethargy and restlessness. Pulse
• Cerebral malaria caused by P. falciparum infection is becomes irregular. Respiration is rapid and shallow.
a medical emergency. So depending on prevailing Repeated vomiting takes place.
Section 1:  General Medicine  193

Nasal Diphtheria A test dose is usually given before giving the injection to
♦♦ There is presence of unilateral and bilateral nasal exclude hypersensitivity.
discharge, at first serous and often blood stained, later ♦♦ Antibiotics: A course of ampicillin or erythromycin 500
thick, mucopurulent and foul smelling. mg 6 hourly should be given to eradicate the diphtheria
♦♦ Thick membrane may be visible on the mucosa of the bacillus.
anterior part of nasal septum. ♦♦ General management:
• Diet: In mild cases, normal diet may be allowed
Laryngeal Diphtheria –– In moderate to severe cases initially fluids are
♦♦ It is common in young children given orally.
♦♦ It is characterized by the hoarseness, brassy cough followed –– If there is palatal palsy semisolid diet is preferred
by attack of inspiratory stridor and laryngeal spasm to liquids because liquids may be regurgitate
♦♦ Membrane is usually limited to larynx. from nose.
–– If swallowing is affected, feeding should be with
Cutaneous Diphtheria
Ryle’s tube.
When corneum bacteria enters into the abrasion or wounds • Care of Mouth: The mouth should be cleaned.
they produced punched out ulcers which is covered by the ♦♦ Treatment of complication:
grayish membrane. • Cardiac failure: Diuretics and digitalis may have to
be given.
Facial Diphtheria
• Palatal palsy: Head low position may be given to drain
♦♦ Mild: secretion of the mouth.
• Reddening of one or both the tonsils. • Laryngeal obstruction: Tracheostomy may be required.
• Small membrane formation on one or both the tonsils. • Respiratory paralysis: Oxygen and ventilator may be
♦♦ Moderate: required.
• Membrane on both tonsils. ♦♦ Prophylaxis: Acute immunization should be given to all the
• Localized tonsillar lymph node enlargement. children at age of 4th, 5th and 6th month in form of DPT.
♦♦ Severe:
• Rapidly spreading firmly membrane on the palate and Q.9. Mention the complications of diphtheria. 
roof of the mouth.  (Sep 1999, 5 Marks)
• There is gross edema of facial and palatal tissue. Ans.
Complications
Conjunctival Diphtheria
Acute Circulatory Failure
♦♦ Due to direct involvement of the eyes by the organism or
spread from the nose. ♦♦ Toxic myocarditis.
♦♦ It is characterized by the severe congestion in the eyes ♦♦ It is characterized by the tachycardia, feeble heart sounds,
and discharge. cardiac enlargement, tic-tac rhythm and arrhythmias.
♦♦ Sudden death may occur.
Clinical Features ♦♦ Congestive heart failure.

♦♦ Patient complains of presence of sore throat. Respiratory Complications


♦♦ Low grade fever is present. ♦♦ Bronchitis
♦♦ Presence of headache and malaise. ♦♦ Bronchopneumonia
♦♦ Hoarseness of voice, dyspnea in cases with laryngeal ♦♦ Respiratory obstruction
diphtheria. ♦♦ Respiratory paralysis.
♦♦ Presence of gross cervical lymphadenopathy. Toxic Neurological Damage
♦♦ Formation of bluish white or grayish green
pseudomembrane at the site of infection. ♦♦ Paralysis of palate
♦♦ Punched out ulcerations are present if skin is involved. ♦♦ Paralysis of accommodation
♦♦ Foul smelling serosanguinous discharge in cases of nasal ♦♦ Facial paralysis
diphtheria. ♦♦ Bulbar paralysis
♦♦ Cyanosis is seen in cases with laryngeal diphtheria. ♦♦ Paralysis of muscles of respiration
♦♦ Peripheral neuropathy.
Management of Diphtheria Renal Complications
♦♦ Bed rest: Usually bed rest is required for 3 to 6 weeks ♦♦ Toxic nephritis.
♦♦ Antitoxin: Anti-diphtheria serum is given subcutaneously ♦♦ Vascular involvement especially middle cerebral artery
or IM in the dose of 10,000–1,00,000 units depending on leading to a picture of thrombosis or monoplegia.
the severity of the disease. ♦♦ Other complication include otitis media and arthritis.
194   Mastering the BDS IIIrd Year  (Last 25 Years Solved Questions)

Q.10. Enumerate the causes of hyperplasia of gums.  – Dental caries: It may be obvious or hidden
 (Feb 1999, 4 Marks) between the teeth and get irrigating the gums.
Ans. According to etiologic factors and pathologic changes, – Tartar
gingival enlargements are: – Pyorrhea alveolitis is the result of septic infection
extending down into the sockets, loosening the
Inflammatory Enlargement teeth, causing the gum margins to recede by
♦♦ Chronic erosion and leading to purulent discharge. In
♦♦ Acute. severe cases gums bleed on the slightest touch.
– Tuberculous gingivitis
Drug Induced Enlargement – Stomatitis.
• General conditions:
Enlargement Associated with Systemic Disease
– Scurvy: Spongy bleeding of the gums, teeth
♦♦ Conditional enlargement: covered by exuberant blood is the prominent
• Pregnancy feature of scurvy due to lack of vitamin C.
• Puberty – Syphilis: In second stage will produce bleeding
• Vitamin C deficiency from the gums.
• Plasma cell gingivitis – Purpura
• Non-specific conditional enlargement. – Blood dyscrasis.
♦♦ Systemic diseases causing gingival enlargement:
Q.12. Briefly describe hookworm infestation. 
• Leukemia
 (Mar 1996, 7 Marks)
• Granulomatous disease.
Ans. Hookworm is a small, grayish white and cylindrical
Neoplastic Enlargement in form. The egg is oval in shape and contains four
blastomeres.
♦♦ Benign tumors
♦♦ Malignant tumors There are three species of hookworm which are
pathogenic:
False Enlargement • Ankylostoma duodenale
♦♦ According to location and distribution gingival • Necator americans
enlargement are classified as: • Ankylostoma brazilenses
• Localized: Gingival enlargement limited to one or Habitat: Adult worm lives in jejunum. Here they suck,
more teeth. the blood from jejunal wall.
• Generalized: Entire mouth, gingiva is enlarged.
• Marginal: limited to marginal gingiva. Clinical Features
• Papillary: Confined to inter-dental papilla.
• Diffuse: Involves all parts of gingiva that is marginal, Causes Clinical features
attached and inter-dental. Penetration of skin and Local erythema, macules, papules
subcutaneous migration of
• Discrete: Isolated sessile or pedunculated tumor like
filariform larvae
enlargement.
Migration of larvae via lungs, •  Bronchitis
♦♦ According to degree of gingival enlargement
bronchi and trachea to •  Pneumonitis
• Grade 0: No sign of gingival enlargement. esophagus
• Grade I: Enlargement confirmed to inter-dental papilla Injury to intestinal mucosal •  Anorexia
• Grade II: Enlargement involves papilla and marginal surface by attachment of •  Pica
gingiva. adult worm •  Epigastric pain and tenderness
• Grade III: Enlargement covers three quarters or more •  Gastrointestinal hemorrhage
of the crown. •  Diarrhea
Q.11. Enumerate causes of bleeding gums.  Sucking of worms causing •  Iron deficiency anemia
 (Mar 1998, 5 Marks) (Oct 2007, 5 Marks) chronic intestinal blood loss • Protein losing enteropathy
causing hypoalbuminemia and
Or
edema
Enumerate five causes of gum bleeding.  Chronic iron deficiency Exertional dyspnea and edema
 (Dec 2009, 5 Marks) anemia
Ans. Causes Loss of nutrients in intestine Hypoalbuminaemia, edema,
• Local Cause: growth retardation
– Minor injury: Use of hard new tooth brush may Iron deficiency • Impaired work productivity and
cause minor local injury to gums and leads to learning
bleeding. •  Cognitive development
Section 1:  General Medicine  195

Diagnosis Ans. Threadworm is also called as pinworm or seatworm. It


♦♦ Stool examination: It shows adult worm or characteristic is enterobius vermicularis. It is a nematode and sexes
hookworm eggs. are separate. Adult worm is small, spindle shaped and
♦♦ Blood examination: Hypochromic anemia and eos­ white in color.
inophilia.
Mode of Entry
Treatment Eggs entry into mouth through contaminated fingers during
♦♦ This consists of expulsion of the worms and treatment of scratching in the perianal region. This is auto- infection.
anemia. If anemia is severe, it should be treated first.
♦♦ Anemia: Anemia should be treated with oral iron therapy Clinical Features
along with vitamin B complex and folic acid. If it is severe ♦♦ Pruritis, i.e. itching in the perianal regions.
blood transfusion should be given. ♦♦ Salpingitis: Inflammation of fallopian tube.
♦♦ *Nocturnal enuresis.
Effective Drugs
♦♦ Appendicitis.
♦♦ Tetrachloroethylene 5 mL is given after an overnight fast
and may be repeated in heavy infestations. Treatment
♦♦ Bephenium hydroxynaphthoate 5 gm is given orally with ♦♦ Mebendazole: Single dose of 100 mg.
fruit juices because it is very bitter. ♦♦ Pyrantel: Single dose of 10 mg/kg
♦♦ Mebendazole 100 mg twice daily is given for 3 days. ♦♦ Sporadic infections are usually cured by one treatment.
Q.13. Outline the treatment of hook worm infestation. ♦♦ In intensive and symptomatic infection drug therapy
 (Mar 2010, 5 Marks) should be repeated after 2 weeks, and then if necessary
Or every 2 months.
Outline management of hookworm infestation. Q.16. Write short note on chickenpox prevention.
 (Mar 1998, 3 Marks)  (Oct 2003, 10 Marks)
Ans. Refer to Ans 12 of same chapter. Ans. Chicken pox is an acute infectious disease.
Q.14. Outline the management of tapeworm infestation. 
Etiological Agent
 (Feb 1999, 4 Marks)
Ans. Tapeworms are ribbon like segmented worms. Chickenpox is caused by varicella virus which is closely related to
The common tapeworm affecting human are Taenia the herpes simplex virus, cytomegalovirus and Epstein-Barr virus.
saginata, Taenia solium. Prevention of Chickenpox
Humans are definitive host.
Varicella-zoster immunoglobin is given to the following, i.e.
Clinical Features
Indication
♦♦ Mild abdominal pain
♦♦ Nausea ♦♦ Following significant exposure to chickenpox in immuno-
♦♦ Change in appetite compromised and susceptible children.
♦♦ Weakness ♦♦ Susceptible adolescents and adults particularly pregnant
♦♦ Weight loss. women.
♦♦ Newborn infants mothers have chickenpox within 5 days
Management before delivery.
♦♦ Premature infants of less then 28 weeks gestation.
♦♦ Niclosamide
♦♦ Premature infants whose mother do not have a history
Adults: Single dose of 2 gm.
of chickenpox
Children: 0.5 to 1 gm
♦♦ Dose: 125 units per 10 kg body weight IM within 48 hours
• The tablets should be given on empty stomach, chewed
and not later than 96 hour after exposure
thoroughly and washed down with a little water. ♦♦ Maximum suggested dose is 625 units.
• A purgative is recommended if the dead segments are
not passed out within few years. Vaccine
♦♦ Praziquantel: Single dose of 5–10 mg/kg after a light
♦♦ Live attenuated Varicella-zoster.
breakfast.
♦♦ Safe and highly protective in both healthy and immuno-
♦♦ For cerebral cysticercosis praziquantel 50 mg/kg/day for
compromised children.
15 days or albendazole 15 mg/kg/day for 30 days. ♦♦ Dose: In children of 2 to 12 years, who have not had chicken
Q.15. Outline treatment of threadworm infestation. pox, two doses are recommended first at 12 to 15 months
 (Mar 1996, 6.5 Marks) of age and second at 4 to 6 years of age.
196   Mastering the BDS IIIrd Year  (Last 25 Years Solved Questions)

Q.17. Write short note on measles. (Feb 2006, 3 Marks) ♦♦ In stained smears of sputum or nasal secretions or urine
Ans. It is highly infective disease. there is presence of multinucleated giant cells in which
Measles are caused by RNA paramyxovirus group. measles virus is isolated on appropriate cell cultures.
♦♦ Measles antigen is detected by fluorescent antibody test in
Clinical Features stained respiratory or urinary epithelial cell.
♦♦ In patient having encephalitis along with measles, CSF
♦♦ Prodromal stage (4 to 5 days)
examination is done which show rise in protein with cell
• Fever is present and there is abrupt rise in temperature
count along with normal range. CSF shows lymphocytosis.
to 400 C.
• *Catarrh: *Coryza, conjunctivitis, photophobia and Management
hacking cough ♦♦ Bedrest is given.
• Koplik’s spots: They appear on second day as minute ♦♦ Frequent fluid intake.
pin point bluish white specks with slight reddish ♦♦ Paracetamol for the fever.
mottled areola around them, on buccal mucosa usually ♦♦ Irrigation of eyes with the boric lotion.
opposite to lower molars. They looks like grain of ♦♦ Cough linctus to suppress the dry cough.
salt. They are variable in number. These spots begin ♦♦ Antibiotics such as amoxicillin if there are complications
to fade with appearance of rash. Red blotches may such as Otitis media or pneumonia.
be seen on soft palate. Koplik’s spots may sometime ♦♦ Vitamin A 200,000 IU orally for 2 days will prevent ocular
occur on lover lip in front of lower incisors, in severe complication and res­piratory infections.
cases of palate and rest of mucosa are peppered with Q.18. Write short note on mumps. 
these spots.  (Mar 2001, 5 Marks) (Oct 2007, 5 Marks)
• Laryngeal involvement: Hoarseness and laryngeal  (Sep 2009, 4 Marks) (Dec 2009, 10 Marks)
stridor is present.  (Nov 2011, 3 Marks) (Apr 2008, 5 Marks)
• Gastrointestinal: Persistent vomiting and diarrhea.  (Nov 2008, 10 Marks)
• Fleeting rashes: Either urticarial or erythematous. Ans. Mumps is a widely prevalent infectious disease.
♦♦ Exanthematous stage:
• On 5th day red macules appear first behind the ear, Etiology
along hair line, posterior part of cheeks and spread It is caused by mumps virus which belongs to group of
rapidly in few hours all over the body. Macules paramyxovirus. Humans are the only natural host and infection
appear in crop which by confluence form blotches spreads by droplet infection as well as by direct contact with
with crescentric or thumb nail edge. respiratory secretions of the patient. Incubation period usually
• In severe disease rashes are confluent, face get swollen ranges from 12 to 22 days average being 16–18 days.
and disfigured and along with photophobic eyes
creates typical measly appearance. Clinical Features
• Mucous membrane involvement: It includes conjunctivitis, ♦♦ Onset of disease:
rhinitis, stomatitis, laryngitis, tracheitis and bronchitis. • Presence of moderate fever, sore throat, drawing or
♦♦ Stage of Defervescence puckering feeling at angle of jaw.
• Temperature falls by crisis or rapid lysis in 24 to 48 • Swelling of face first draw the attention.
hours. • Onset with rigor or convulsion.
• Rashes fade from face and leaves brown staining • Onset with meningeal reaction, i.e. cerebral mumps
followed by branny desquamation. ♦♦ Early signs:
• At times normal rash of measles instead of fading • Presence of pain or tenderness on pressure beneath
become deep purple and this can persists for week angle of lower jaw.
or two. • Redenning of parotid duct orifice
♦♦ Other features:
Types of Measles • Usually one parotid gland is affected followed by the
♦♦ Toxic: This is malignant form of disease and is manifested other gland after varying interval or only one gland is
by high fever, delirium, circulatory, fever and may be fatal. affected throughout. Swelling reaches to its maximum
♦♦ Pulmonary: Respiratory infection with high temperature in 3 days, remain its peak for 2 days and slowly receds.
and rapid respiration. Lobe of ear is in center of swelling which is tender on
♦♦ Hemorrhage: It is rare. There is hemorrhage into the mucus pressure.
• After appearance of parotitis fever rise to 400 C. Fever
membrane, skin and subcutaneous tissue.
falls by lysis in 3 to 7 days.
Laboratory Findings • Other symptoms are diminished salivation, furred
tongue and foul breadth.
♦♦ Leukopenia is frequent. If leukocytosis is present it is • Enlargement of parotid may cause trismus and
indicative of superadded bacterial infection. deafness.
Section 1:  General Medicine  197

Diagnosis ♦♦ Meningitis: Neck rigidity can occur in both tetanus and


♦♦ Viral isolation: From saliva or nasopharynx in acute illness meningitis, signs of meningeal irritation, diagnostic CSF.
or from CSF in mumps meningitis. ♦♦ Rabies: Dysphagia associated with spasms of inspiratory
♦♦ Antibody titer: Four fold rise in 1 to 2 weeks after infection. and pharyngeal muscles also occur in rabies. History of
dog bite is present
Treatment ♦♦ Tetany: Spasm starts in periphery with carpopedal spasm.
♦♦ It is mainly supportive and symptomatic. ♦♦ Drug dystonia: Dystonic relations to drug such as
♦♦ Child is put to bed rest. phenothiazines and metoclopramide. Grimacing,
♦♦ Mouth hygiene must be maintained meticulously. spasmodic and neck retraction and torticollis, wide
♦♦ Analgesics and antipyretics are the main stay of treatment, opening of mouth and eyes. Dystonia prolong muscular
i.e. paracetamol/nimulid syrup 1–2 teaspoonful two to contractions that may cause twisting of body parts,
three times a day repetitive body movements and increased muscular tone.
♦♦ Diet should be soft, bland and preferably in liquid form. ♦♦ Acute peritonitis: Board like abdominal rigidity as in
♦♦ In cases where complications like orchitis or neurological tetanus, but in tetanus there is little or no tenderness.
complications develop, prednisolone 40–60 mg per day ♦♦ Functional muscle spasms: Bizarre movement of posture,
in divided doses for about seven days is given along absence of constant rigidity of involved muscles, history
with antibiotics, i.e. Ampicillin/Amoxycillin/Cefazolin/ of previous personal disorders.
Ceftizoxime. ♦♦ Catatonic schizophrenia: It might cause confusion in absence
of background information.
Complications Q.20. Write clinical examination of tetanus patient. 
♦♦ Common complications:  (Mar 2000, 10 Marks)
• Orchitis and epididymitis: It is unilateral. Common Ans. Tetanus initially presents with muscle stiffness. The
in young adults, may occur without parotitis. Fever distribution may vary with the type of tetanus. The masseter
return and go up to 40–50°C. Testis is swollen, tender muscles are commonly involved with an accompanying
and tense with or without epididymitis. Last for 10 headache. Neck stiffness, difficulty swallowing, generalized
days. Can result in sterility. muscle spasms including the abdominal and back muscles
• Meningitis: It follows parotitis and can occur at same and sweating may be seen later in the disease. In severe
time or before salivary gland enlargement. cases, respiratory paralysis may develop, which presents
• Oophoritis: It is less common than orchitis. with apnea, hypoxia and hypercapnia.
• Acute pancreatitis: Occur in 2nd week. Occasionally Physical examination of a patient with tetanus may
disease can be present with pancreatitis without reveal the following:
salivary gland involvement.
♦♦ Rare complications: General Appearance
• Neurological: Meningoencephalitis, cranial nerve ♦♦ Severe muscular spasm (location varies with the type)
involvement, polyneuritis, other CNS problems, i.e. • Opisthotonos
cerebellar ataxia, facial palsy, transverse myelitis, • Leg extension with arm flexion
hydrocephalus, flaccid paralysis and behavioral changes. • Risus sardonicus
• Arthritis: Occassionally one or more large joints, but ♦♦ May be in respiratory distress
there is no permanent damage.
Vitals
• Mastitis: Mild and transient enlargement of breasts.
• Prostatitis in males ♦♦ Fever
• Thyroiditis ♦♦ Tachycardia
• Nephritis ♦♦ Elevated blood pressure
• Fetal endocardial fibroelastosis
Musculoskeletal
Q.19. Write differential diagnosis of lock jaw.  ♦♦ Spasms of the diaphragm and intercostals
 (Oct 2003, 10 Marks) ♦♦ Stiff abdominal wall
Ans. Lock jaw or tetanus is caused by the powerful neurotoxin,
Respiratory
i.e. tetanospasmin which is released by the C. tetani.
This disease is characterized by the muscular rigidity ♦♦ Tachypnea
and spasms. ♦♦ Dyspnea

Differential Diagnosis Cardiovascular


♦♦ Other causes of trismus: Irritant local lesions of teeth, throat, ♦♦ Normal S1 and S2
temporomandibular joint, masseter muscle and cervical ♦♦ Hypertension
lymph nodes. ♦♦ Arrhythmia
198   Mastering the BDS IIIrd Year  (Last 25 Years Solved Questions)

Autonomic negative test result would normally be a gag reflex attempting


to expel the foreign object.
Intervals of bradycardia and hypotension are accompanied by
that of tachycardia and hypertension. Q.21. How will you differentiate tetany from tetanus?
 (Aug 2005, 5 Marks)
Abdominal
Ans.
Stiffening of the abdominal muscles.
Tetany Tetanus
The physical examination may vary according to the type of • It is the hyper excitability • It occurs in three forms, i.e.
tetanus. Specific findings associated with the various types of of nervous system causing mild, moderate and severe.
tetanus may include: spasmodic, painful contrac- It is a local muscle rigidity
tion with spasm of muscles leading to trismus, change
Local Tetanus in facial expression, pain of
spine of muscle
♦♦ Limited area of spasm
• Numbness and stiffness • Dysphagia and trismus of jaw
♦♦ The affected area is in close proximity to a contaminated around the mouth with
wound muscle cramping
♦♦ Contraction is usually painful and associated with swelling • It is caused due to low calcium • No effect on calcium serum
♦♦ Generalized tetanus may follow localized tetanus serum level, i.e. 4–8 mg% level
• Elevated serum phosphorus • No effect on serum
Cephalic Tetanus
levels phosphorus levels
♦♦ Ear infection or head injury may be seen • It is caused due to deficiency • It is caused due to bacterial
♦♦ Trismus of calcium infection by Clostridium tetani
♦♦ Signs of CN III, IV, VI, VII and XII involvement including: • Loss of hair, nail, enamel • No such signs are present in
• Tilting of the mouth defects in teeth, severe head- patient
• Inability to close the eye ache diminished vision and
• Inability to move the tongue mental retardation is present
• Diplopia • It is treated by IV infusion or • It is treated by giving ATS of
♦♦ Abnormal eye movements: Dysphagia tetanus toxoid calcium
♦♦ Confusion
♦♦ Symptoms of stroke Q.22. How will you differentiate rheumatoid arthritis from
rheumatic arthritis? (Aug 2005, 5 Marks)
Generalized Tetanus Ans.
♦♦ Descending spasm
♦♦ Trismus may present initially: Characteristic Rheumatoid Rheumatic
feature arthritis arthritis
• Followed by stiffness of the neck
• Difficulty in swallowing • Age of onset 20–45 years 2–15 years
• Stiffness of abdominal muscles • Causative factor Autoimmunization S.hemolyticans
♦♦ Other symptoms include: • Onset Insidious Rapid
• Elevated temperature
• Sweating • Number of joint Multiple Multiple but not
involved migratory
• Elevated blood pressure
• Episodic rapid heart rate • Other symptoms Weakness, fatigue, Fever, chills, pallor,
♦♦ Spasms may occur frequently and last for several minutes low fever low anemia
♦♦ Spasms may continue for 3–4 weeks • Stiffness of joint Mild to moderate Usually not marked
♦♦ Complete recovery may take months • Swelling and Fusiform swelling Present
Neonatal Tetanus effusion of finger joints and
effusion of bigger
♦♦ Unhealed, unhygienic umbilical stump joints
♦♦ Trismus (spasm of masseter muscle) • Muscle atrophy Marked in later stage Absent
♦♦ Risus sardonicus (spasm of facial muscles) • Skin changes Skin may be smooth Red due to
♦♦ Clenched hands and shiny inflammation
♦♦ Dorsiflexion of the feet
• Deformities Flexion deformity, Absent
♦♦ Opisthotonus (spasm of spinal muscles) ulnar duration of
hand, later fibrous
The Spatula Test or bony ankylosis
The "spatula test" is a clinical test for tetanus that involves • Distribution Bilateral and sym- Multiple migratory
touching the posterior pharyngeal wall with a sterile, soft-tipped metrical proximal flitting involvement of
instrument, and observing the effect. A positive test result is interphalangeal are elbows, wrists, knees
commonly involved and ankles
the contraction of the jaw (biting down on the "spatula"), and a
Section 1:  General Medicine  199

Q.23. Write short note on viral exanthema.  patient all that may be seen is prominence of hair follicles
 (Sep 2008, 2.5 Marks) giving a goose pimpled appearance.
Ans. Viral infections associated with skin lesions are known
as viral exanthems. Q.24. Describe briefly AIDS.
Viral exanthems lead to maculopapular rashes sparing  (Mar 1998, 3 Marks) (Sep 2008, 7 Marks)
palms as well as soles. Or
Following are the viral exanthems:
Write short note on AIDS. (Feb 2006, 5 Marks)
• Measles
• Rubella  (Dec 2015, 3 Marks)
• Varicella-zoster infection, i.e. chicken pox and Ans. AIDS stands for acquired immunodeficiency
shingles syndrome
• Exanthem infectiosum A CD4 count less than 200/µl in HIV infected individual
• Exanthem Subitum is defined as AIDS.
• Enteroviruses
• Infectious mononucleosis Etiology
• Adenovirus
Both HIV – 1 and HIV – 2, members of lentivirus family of
• Reovirus
retroviruses causes AIDS, but HIV – 2 appears to be less virulent
• Arbovirus.
progress slowly and is less commonly transmitted vertically.
Exanthematous Stage in Measles
Transmission
♦♦ In exanthematous stage on 5th day the red macules appear
behind the ear, along hair line and on posterior parts of ♦♦ Sexual: Most HIV infection occurs in homosexual man.
cheeks and spread rapidly in a few hours all over the Multiple heterosexual contacts often prostitutes.
body. Macules appear in the crops which by confluence ♦♦ Contacts with blood and body fluids, contaminated blood
from bloatches with crescentric or thumbnail edge. Fully and blood products.
erupted rash deepens in color, petechiae may occur. In ♦♦ Contaminated needles and syringes.
severe measles rash is confluent, the face is swollen and ♦♦ Through organ and tissue donation
disfigured and together with photophobic eyes create ♦♦ From mother to child: In uterus at birth, breast milk.
typical measly appearance.
♦♦ Mucous membrane involvement: It consists of conjunctivitis, Stages of HIV Infection by WHO
rhinitis, stomatitis, laryngitis, tracheitis and bronchitis. Primary HIV Infection
Exanthema in Chicken Pox ♦♦ Asymptomatic
♦♦ Evolution: In form of crops, first at back, then chest, ♦♦ Acute retroviral syndrome: Fever with maculopapular rash
abdomen, face and lastly limbs. primarily on trunk with small aphthous lesions on oral
♦♦ Character: At first macule appear then in few hours dark and genital mucosa.
pink papule which soon turn into vesicle. They also get
Clinical Stage 1
collapsed if pierced and vesicles turn into pustules in 24
hours and to scabs in 2 to 5 days. ♦♦ Asymptomatic
♦♦ Distribution: It is centripetal, i.e. more on upper arm and ♦♦ Persistent generalized Lymphadenopathy
thighs, upper part of face and in concavities and flexures.
Clinical Stage 2
♦♦ Cropping: Rash mature quickly and most spots dry up
within 48 hours of appearance. For 2 to 3 days new ♦♦ Unexplained moderate weight loss (< 10% of presumed
spots continue to appear on any area of body vesicles, body weight)
pustules and scabs are found side by side. ♦♦ Infections
♦♦ Recurrent respiratory tract infections
Exanthema in Rubella ♦♦ Herpes zoster
In exanthema rash occurs more often in older children and ♦♦ Fungal infections of finger nails: Oral lesions
adults on first or second day of illness, first on face and behind ♦♦ Recurrent oral ulcerations
the ears, and then spreads downwards to trunk and limbs. ♦♦ Angular cheilitis: Itchy dermatosis
The rash is variable, but commonly starts as discrete, pink, ♦♦ Papularpruritic eruptions
punctate, erythematous, perifollicular macules that rapidly ♦♦ Seborrheic dermatitis
become confluent. Alternatively, there may be blotchy pink Clinical Stage 3
rash or confluent blush. Rash seldom persists for more than 4
days and is not followed by staining or desquamation. Rubella Conditions where a presumptive diagnosis can be made on the
without rash is common in young children. In a dark skinned basis of clinical signs or simple investigations.
200   Mastering the BDS IIIrd Year  (Last 25 Years Solved Questions)

♦♦ Unexplained symptoms: ♦♦ Investigations for virological monitoring: In this HIV


• Chronic diarrhea for > 1 month viral load tests are done such as HIV – RNA by PCR, HIV
• Persistent fever, intermittent or constant for > 1 month – RNA by bDNA, HIV – RNA by NASBA. These all tests
♦♦ Severe weight loss (> 10% of presumed or measured body measure the actively replicating HIV virus. These cells also
weight) deflect the response of antiretroviral drugs. These test are
♦♦ Infections: excellent for diagnosis of acute HIV infection before its
• Severe presumed bacterial infections seroconversion.
• Pulmonary tuberculosis diagnosed in last 2 years
♦♦ Oral lesions: Treatment
• Oral candidiasis
The medical management of AIDS is done by HAART, i.e. highly
• Oral hairy leukoplakia
active antiretro viral therapy.
• Acute necrotizing ulcerative stomatitis, gingivitis or
periodontitis It causes suppression of HIV replication and prolonging life
♦♦ Conditions where confirmatory diagnostic testing is as well as improving the quality of life of the patient.
necessary The drugs used are:
• Unexplained anemia (< 8 g/l or neutropenia (< 500 ♦♦ Nucleoside reverse transcriptase inhibitors
µl) or thrombocytopenia (< 50,000 µl) for > 1 month • Zidovudine: Dose: 600 mg daily in two divided doses
Clinical Stage 4 • Didanosine: Dose: 400 mg OD. To be taken before
meals
Conditions where a presumptive diagnosis can be made on the • Zalcitabine: It is used in combination with zidovudine:
basis of clinical signs and simple investigations Dose is a 0.75 mg TDS.
♦♦ HIV wasting syndrome • Lamivudine 150 mg orally BD
♦♦ Infections ♦♦ Nucleotide reverse transcriptase inhibitors
• Pneumocystis pneumoniae • Tenofovir: Dose: 300 mg orally OD
• Recurrent severe or radiological bacterial pneumonia ♦♦ Non-nucleoside reverse transcriptase inhibitors
• Chronic herpetic simplex infection (oral, labial, genital • Nevirapine: 200 mg orally daily for two weeks and
or anorectal of > 1 month duration). then 200 mg BD
• Esophageal candidiasis • Efavirenz: 600 mg orally daily
• Extrapulmonary tuberculosis ♦♦ Protease inhibitors
♦♦ Neoplasms: Kaposi's sarcoma • Amprenavir – 1200 mg orally BD
♦♦ Neurological disease
• Atazanavir – 400 mg orally daily
• CNS toxoplasmosis
• Indinavir – 800 mg orally TDS
• HIV encephalopathy
♦♦ Protease inhibitors boosted with ritonavir
Laboratory Diagnosis • Lopinavir R – 6 capsules per day
• Fosamprenavir – 700 mg + 100mg ritonavir twice daily
♦♦ Investigations for the diagnosis of HIV infection:
• HIV-ELISA: This is the most commonly used • Ritonavir – 100 mg BD orally
screening test for HIV infection. If this test is positive ♦♦ Fusion inhibitors
confirmation should be done by western blot test. • Enfuvirtide – 90 mg SC injection twice daily.
• HIV-rapid antibody test: They are for rapid diagnosis, Q.25. Write clinical examination of advanced HIV disease
i.e. within 10 to 15 minutes. It is also a screening test, in patient. (Mar 2000, 10 Marks)
as if this test comes positive confirmation is done by Ans. Clinical examination of advanced HIV disease.
western blot.
• Western blot: It is the confirmatory test for HIV External Examination
infection.
Patient is markedly cachectic with:
• Blood cell count: Since neutropenia, anemia and
pancytopenia are associated with HIV infection, this ♦♦ Generalized lymphadenopathy
test is done. ♦♦ Nodular skin lesion all over the body.
♦♦ Investigations for monitoring progress of HIV infection: ♦♦ White patches in the oral cavity.
• Absolute CD4 lymphocyte count: This is most commonly
Internal Examination
used. As its count decreases occurrence of opportunistic
infection and malignancy is high, i.e. if count is less ♦♦ Skull: Meninges are congested and inflamed.
than 200 cells/μL. ♦♦ Thorax: There was presence of dense adhesion between the
• CD4 lymphocyte percentage: In this if count is less pleura and the chest wall. Mediastinal lymph nodes were
than 14% occurrence of opportunistic infection and enlarged and matted. Both lungs show marked destruction
malignancy is high, if treatment is not given to patient. of parenchyma.
Section 1:  General Medicine  201

♦♦ Abdomen: Serosal surfaces of liver, spleen and kidney were Complications


studded with tubercle. Mesenteric lymph nodes were ♦♦ Cerebral ataxia
enlarged and matted. ♦♦ Myocarditis
♦♦ Urinary bladder: Serosal surface shows presence of ♦♦ Hepatitis
tubercles. ♦♦ Acute glomerulonephritis
♦♦ Genital system: It is studded with tubercles. ♦♦ Pneumonia
♦♦ Adrenals: Enlarged, congested, shows presence of multiple ♦♦ Arthritis
tuburcles. ♦♦ Corneal lesion
Q.26. Write short note on chickenpox.  ♦♦ Bleeding diathesis.
 (Sep 2004, 5 Marks) (Sep 2009, 5 Marks) Management
Ans. It is an erythematous or vesiculopapular lesion caused
by varicella-zoster virus. ♦♦ No need to confine patient to bed unless symptoms are
severe.
It spreads through aerosol route.
♦♦ For pruritus Calamine lotion with or without phenol (0.4%)
Its incubation period is of 14 to 15 days and sedative antihistaminics by mouth. If there is much
scabbing, gauze soaked in 1 in 5,000 solution of potassium
Clinical Features permanganate which is changed every 4 hours may be
♦♦ Stage of invasion or prodomata: applied to areas most affected.
• Presence of headache, sore throat and fever for 24 ♦♦ For secondary infection antibiotics should be given.
hours. ♦♦ For true varicella pneumonia oxygen is given.
• Erythematous or urticarious prodromal rashes are ♦♦ For encephalitis oxygen and corticosteroids are given
seen. ♦♦ Paracetamol is given for fever.
♦♦ Stage of eruption: ♦♦ Oral acyclovir should be initiated within 24hours of rash
• Enanthem: Earliest lesions are on buccal and pharyngeal results in a decrease in the duration and magnitude of the
mucosa. fever, and in the number and duration of skin lesions. Dose
• Exanthem is 20 mg/kg qds.with maximum of 800 mg qds.
–– Evolution: In form of crops, first at back, then chest, Q.27. Write short note on oral manifestations of HIV/AIDS.
abdomen, face and lastly limbs. (Sep 2006, 5 Marks)
–– Character: At first macule appear then in few hours Or
dark pink papule which soon turn into vesicle.
Describe oral manifestations of HIV
They also get collapsed if pierced and vesicles
 (Feb 2013, 5 Marks)
turn into pustules in 24 hours and to scabs in 2
Ans. Classification of Oral Manifestations By EC-Clearing-
to 5 days.
house
–– Distribution: It is centripetal, i.e. more on upper
arm and thighs, upper part of face and in Group 1: Strongly Associated With HIV Infection
concavities and flexures.
♦♦ Candidiasis: Erythematous, pseudomembranous, angular
–– Cropping: Rash mature quickly and most spots dry
cheilitis
up within 48 hours of appearance. For 2 to 3 days
♦♦ Hairy leukoplakia
new spots continue to appear on any area of body
♦♦ Kaposi’s sarcoma
vesicles, pustules and scabs are found side by side.
♦♦ Non-Hodgkin’s lymphoma
♦♦ Other symptoms:
♦♦ Periodontal diseases: Linear gingival erythema, necrotizing
• Pruritus of varying degree. gingivitis, necrotizing periodontitis.
• Generalized lymphadenopathy may occur.
• Enlargement of suboccipital and posterior cervical Group 2: Less Commonly Associated with HIV Infection
lymph nodes. ♦♦ Bacterial infections: Mycobacterium avium-intracellulare,
Mycobacterium tuberculosis
Laboratory Diagnosis
♦♦ Melanotic hyperpigmentation
♦♦ A Tzanck smear, performed by scraping the base of an ♦♦ Necrotizing ulcerative stomatitis
acute lesion and staining with Giemsa or Papanicolaou’s ♦♦ Salivary gland disease: Dry mouth, unilateral or bilateral
stain may demonstrate multinucleated giant cells having swelling of major salivary glands
intranuclear inclusions. ♦♦ Thrombocytopenia purpura
♦♦ Other tests are fluorescent antibody against membrane ♦♦ Oral ulcerations NOS (not otherwise specified)
antigen, immune adherence hemagglutination and ♦♦ Viral infections: Herpes simplex, human papillomavirus,
ELISA. varicella-zoster
202   Mastering the BDS IIIrd Year  (Last 25 Years Solved Questions)

Group 3: Seen in HIV Infection ♦♦ Molluscum contagiosum: It is caused by pox virus. Lesions
are small, waxy, dome shaped papules which demonstrate
♦♦ Bacterial infections: Actinomyces israelii, Escherichia coli,
central depressed crater.
Klebsiella, pneumonia
♦♦ Cat-scratch disease (Bartonella henselae) Q.28. Write short note on HIV importance in dental practice.
♦♦ Epithelioid (bacillary) angiomatosis (Bartonella henselae) (Mar 2007, 2 Marks)
♦♦ Drug reactions: Ulcerative, erythema multiforme, lichenoid, Ans. The HIV is the virus which results in the causation of
toxic epidermolysis the AIDS.
♦♦ Fungal infections other than candidiasis: Cryptococcus
Prophylactic measures to be adopted by dental health
neoformans, Geotrichum candidum, Histoplasma capsulatum,
care workers while treating AIDS patient.
Mucoraceae (mucormycosis/zygomycosis), Aspergillus
• Care in handling sharp objects like needles, blades.
flavus
• All cuts and abrasions in an HIV patient should be
♦♦ Neurologic disturbances: Facial palsy, trigeminal neuralgia
covered with a waterproof dressing
♦♦ Recurrent apthous stomatitis
• Minimal parenteral injections
♦♦ Viral infections: Cytomegalovirus, Molluscum contagiosum
• Equipments and areas which are contaminated
Description of Oral Manifestations with secretions should be wiped with sodium
hypochlorite solution or 2% glutaraldehyde.
♦♦ Candidiasis is the most common oral manifestation of • Contaminated gloves, cottons should be incinerated.
HIV infection. All the three types. i.e. erythematous, • E q u i p m e n t s s h o u l d b e d i s i n f e c t e d w i t h
pseudomembranous and hyperplastic forms are seen. glutaraldehyde.
Erythematous candidiasis is seen when the CD4 count • Disposable equipments (drapes,scalpels,etc.) should
drops below 400 cells/mm 3 and pseudomembranous be used whenever possible.
develop when CD4 count drop below 200 cells/mm3. • Walls and floor should be cleaned properly with
♦♦ Hairy leukoplakia: Presence of soft painless plaque on the soap water.
lateral border of tongue with corrugated surface. • Separate operation theater and staff to do surgeries
♦♦ Kaposi’s sarcoma: Single or multiple bluish swellings are to HIV patients is justifiable
seen with or without ulceration over gingiva and palate. • Avoid shaving whenever possible before surgery in
♦♦ Angular chelitis: Linear fissures or linear ulcers are seen at HIV patients.
the angle of mouth. • All people inside the theater should wear disposable
♦♦ Linear gingival erythema: It is fiery red band along the gowns, plastic aprons, goggles, overshoes and
gingival margin and attached gingiva with profuse gloves.
bleeding. • Surgeons, assistants and scrub nurse should wear
♦♦ Necrotizing ulcerative gingivitis: Destruction of inter-dental in addition double gloves.
papillae is seen. • Suction bottle should be half—filled with freshly
♦♦ Necrotizing ulcerative periodontitis: There is advanced prepared glutaraldehyde solution.
necrotic destruction of periodontium, rapid bone loss, loss • Soiled body fluids should be diluted with
of periodontal ligament and sequestration. glutaraldehyde.
♦♦ Oral ulcerations: Single or multiple major recurrent • Accidental puncture area in surgeon or scrub nurse
aphthous ulcers are seen with white pseudomembrane should immediately washed with soap and water
surrounding the erythematous halo. thoroughly
♦♦ Non-Hodgkin’s lymphoma: It is the malignancy of HIV • Theater should be fumigated after surgery to HIV
infected individuals. It occurs in extranodal locations patient.
and CNS is the common site. Intra-osseous involvement
Q.29. Write short note on HIV. (Sep 2007, 2 Marks)
is also seen.
Or
♦♦ Mycobacterial infection: Mycobacterial infection in form of
Write on mode of transmission and clinical features of
tuberculosis is seen. When present tongue is affected most
HIV. (Apr 2018, 5 Marks)
commonly. Affected areas show common ulcerations.
Ans. HIV disease is an infectious disease caused by human
♦♦ Herpes simplex virus: Recurrent or secondary herpes
immunodeficiency virus.
simplex infection is seen in the patients. Herpes simplex
lesions increase when CD4 cell count drops below Late stage of HIV infection is AIDS when CD4 + T
50 cells/mm3. lymphocyte count is < 200/µL.
♦♦ Herpes zoster: It is common in HIV infected individuals.
Etiology
Orally, involvement is severe and leads to sequestration
of bone as well as loss of teeth. Both HIV – 1 and HIV – 2, members of lentivirus family of
♦♦ Histoplasmosis: It is the fungal infection caused by retroviruses causes AIDS, but HIV – 2 appears to be less
histoplasma capsulatum. Sign and symptoms of disease are virulent progress slowly and is less commonly transmitted
fever, weight loss, splenomegaly and pulmonary infiltrate. vertically.
Section 1:  General Medicine  203

Transmission ♦♦ Unexplained symptoms


• Chronic diarrhoea for > 1 month
♦♦ Sexual: Most HIV infection occurs in homosexual man.
• Persistent fever, intermittent or constant for > 1 month
Multiple heterosexual contacts often prostitutes.
♦♦ Severe weight loss (> 10% of presumed or measured body
♦♦ Contacts with blood and body fluids, contaminated blood
weight)
and blood products.
• Infections
♦♦ Contaminated needles and syringes.
♦♦ Severe presumed bacterial infections
♦♦ Through organ and tissue donation
♦♦ Pulmonary tuberculosis diagnosed in last 2 years
♦♦ From mother to child: In uterus at birth, breast milk.
• Oral lesions
Pathogenesis of HIV Infection ♦♦ Oral candidiasis
♦♦ Oral hairy leukoplakia
♦♦ HIV virus infects target cells (CD4 + T cells, monocytes, ♦♦ Acute necrotizing ulcerative stomatitis, gingivitis or
macrophages and dendrite cells) through CD4 receptors. periodontitis
♦♦ On entering T cells, the virus integrates its RNA genome • Conditions where confirmatory diagnostic testing is
into the host cell genome by first transcribing this genome necessary
into DNA (HIV provirus) with the help of enzyme reverse ♦♦ Unexplained anemia (< 8 g/L or neutropenia (< 500 µL) or
transcriptase. thrombocytopenia (< 50,000 µL) for > 1 month
• Provirus is then transcribed and translated along
Clinical Stage 4
with the host cell DNA to synthesize specific viral
components, which eventually assemble to produce Conditions where a presumptive diagnosis can be made on the
complete virus particles. basis of clinical signs and simple investigations
• At this time patient is seronegative, i.e. antibodies ♦♦ HIV wasting syndrome
against the virus are not present. But patient is highly ♦♦ Infections
infectious. This period is labelled ”window period". • Pneumocystis pneumonia
• Although some virions are killed, HIV continues to • Recurrent severe or radiological bacterial pneumonia
multiply infecting increasing number of CD4 cells. • Chronic herpetic simplex infection (oral, labial, genital
♦♦ In early stages of immune destruction, the patient is or anorectal of > 1 month duration).
asymptomatic. • Esophageal candidiasis
♦♦ As the immunosuppression progresses over a period of • Extrapulmonary tuberculosis
time patient becomes symptomatic. ♦♦ Neoplasms
• Kaposi's sarcoma
Stages of HIV Infection by WHO ♦♦ Neurological disease
Primary HIV Infection • CNS toxoplasmosis
• HIV encephalopathy
♦♦ Asymptomatic
Q.30. Give definition, etiology, signs, symptoms, diagnosis,
♦♦ Acute retroviral syndrome: Fever with maculopapular rash
differential diagnosis and complications of enteric
primarily on trunk with small aphthous lesions on oral fever. (Jan 2012, 12 Marks)
and genital mucosa.
Ans. It is an acute systemic illness caused by infection due
Clinical Stage 1 to salmonella typhi. Typhoid fever is characterized by
fever, malaise, pain abdomen, rash, splenomegaly
♦♦ Asymptomatic
and leukopenia. The untreated patients may develop
♦♦ Persistent generalized lymphadenopathy
complications during 2nd and 3rd week due to toxemia
Clinical Stage 2 and septicemia. In some cases it may be fatal.
♦♦ Unexplained moderate weight loss (<10% of presumed For etiology refer to Ans 1 of same chapter.
body weight) For sign and symptoms refer to Ans 2 of same chapter.
♦♦ Infections For complications refer to Ans 4 of same chapter.
♦♦ Recurrent respiratory tract infections Diagnosis
♦♦ Herpes zoster
♦♦ Fungal infections of finger nails First Week
• Oral lesions ♦♦ Normochromic normocytic anemia, leukopenia and
♦♦ Recurrent oral ulcerations albuminuria
♦♦ Angular cheilitis ♦♦ Blood culture may be positive in 70–90% of cases.
• Itchy dermatosis Second Week
♦♦ Papularpruritic eruptions
♦♦ Seborrhoeic dermatitis ♦♦ Anemia, leukopenia may persist
♦♦ Widal test become positive, may show four fold rise in
Clinical Stage 3 agglutinins against somatic ‘O’ antigen. It is not specific
Conditions where a presumptive diagnosis can be made on the but rising titers are diagnostic.
basis of clinical signs or simple investigations. ♦♦ Blood culture may be positive only in 50%.
204   Mastering the BDS IIIrd Year  (Last 25 Years Solved Questions)

Third Week Clinical Stage 1


♦♦ Anemia and leucopenia persist. Leukocytosis occur in ♦♦ Asymptomatic
severe septicemia ♦♦ Persistent generalized lymphadenopathy
♦♦ Blood culture is positive in 30-45% of cases. Clinical Stage 2
♦♦ Positive WIDAL test with rising titers
♦♦ Unexplained moderate weight loss (< 10% of presumed
♦♦ Positive stool culture and urine test for S. typhi.
body weight)
Differential Diagnosis ♦♦ Infections
♦♦ Recurrent respiratory tract infections
♦♦ Para typhoid fever: Mode of onset often acute and ♦♦ Herpes zoster
atypical; Wider remissions of temperature; Eruption is ♦♦ Fungal infections of finger nails
more profuse; less toxaemia; sweating and rigours more • Oral lesions
common; Intestinal complications rare ♦♦ Recurrent oral ulcerations
♦♦ Short fever: A fever lasting for 8 to 10 days; no associated ♦♦ Angular cheilitis
signs, probably of viral origin. Subside spontaneously. • Itchy dermatosis
No complications. ♦♦ Papular pruritic eruptions
♦♦ Amoebic liver abscess: Pain in right hypochondrium ♦♦ Seborrheic dermatitis
and lower chest, moderate fever, enlarged tender liver
or compression tenderness over right lower intercostal Clinical Stage 3
spaces. Right hemidiaphragm may be elevated and Conditions where a presumptive diagnosis can be made on the
immobile on fluoroscopy. basis of clinical signs or simple investigations.
♦♦ Viral hepatitis: In pre-icteric stage there is marked nausea ♦♦ Unexplained symptoms
and vomiting, hepatic tenderness, high colored urine. • Chronic diarrhea for > 1 month
♦♦ Tuberculous meningitis: Absence of abdominal • Persistent fever, intermittent or constant for > 1 month
discomfort, greater frequency of vomiting, persistence ♦♦ Severe weight loss (> 10% of presumed or measured body
of headache after first week, irritability, irregular pupils, weight)
CSF changes. • Infections
♦♦ Miliary tuberculosis: Increased respirations, irregular ♦♦ Severe presumed bacterial infections
temperature, tachycardia, cough and cyanosis, symptoms ♦♦ Pulmonary tuberculosis diagnosed in last 2 years
referable to alimentary tract less pronounced. Early loss of • Oral lesions
flesh. Diagnostic CXR. ♦♦ Oral candidiasis
♦♦ Heat fever: Not uncommon in children and aged. Fever ♦♦ Oral hairy leukoplakia
may be continuous or touch normal for some hours every ♦♦ Acute necrotizing ulcerative stomatitis, gingivitis or
day. Absence of other physical signs. Response to lowered periodontitis
temperature. • Conditions where confirmatory diagnostic testing is
♦♦ Subacute infective endocarditis: Fever seldom continuous necessary
or high; Frequent chills with septic type of temperature; ♦♦ Unexplained anemia (< 8 g/l or neutropenia (< 500 µl) or
cardiac signs; anaemia; embolic phenomenon; positive thrombocytopenia (< 50,000 µL) for > 1 month
blood culture.
♦♦ E. coli infection: Pyelitis or septicemia—high fever, Clinical Stage 4
though not of continuous type. May last for 2–3 weeks.
Leukocytosis, tenderness in loins, pus in urine or positive Conditions where a presumptive diagnosis can be made on the
blood culture. basis of clinical signs and simple investigations
♦♦ Malaria: Sudden onset, wide diurnal variation, early ♦♦ HIV wasting syndrome
splenic enlargement, malarial parasites in blood, response ♦♦ Infections
to antimalarial drugs. • Pneumocystis pneumonia
• Recurrent severe or radiological bacterial pneumonia
Q.31. Write short note on WHO criteria for diagnosis of • Chronic herpetic simplex infection (oral, labial, genital
AIDS. (Mar 2008, 2.5 Marks) or anorectal of > 1 month duration).
Ans. WHO criteria for diagnosis of AIDS. • Esophageal candidiasis
• Extrapulmonary tuberculosis
Primary HIV Infection
♦♦ Neoplasms
♦♦ Asymptomatic • Kaposi's sarcoma
♦♦ Acute retroviral syndrome: Fever with maculopapular rash ♦♦ Neurological disease
primarily on trunk with small aphthous lesions on oral • CNS toxoplasmosis
and genital mucosa • HIV encephalopathy
Section 1:  General Medicine  205

Q.32. Write short note on risk factors for candidiasis. For investigations or diagnosis refer to Ans 30 of same
(Sep 2008, 3 Marks) chapter.
Ans. Following are the risk factors for candidiasis. For complications refer to Ans 4 of same chapter.
Any condition that weakens the immune system, such as:
Treatment
• Diabetes
• Organ transplant Specific
• Chemotherapy ♦♦ Ciprofloxacin: It is given in the dose of 500 mg BD for 7 to
• AIDS 10 days it is avoided in children because of risk of cartilage
• Daily corticosteroid use damage and tendonitis. If absolutely required, low dose
• Breaks in the skin or mucous membranes can be used for not more than 3 days.
• Kidney dialysis ♦♦ Ceftriaxone: It is 3rd generation cephalosporin and
• Intravenous catheters improves the condition rapidly.
• Intravenous drug abuse It is given in the dose of 1 gm BD for 10 to 14 days.
• Obesity ♦♦ Azithromycin: 1 gm OD for 5 days.
• Peptic ulcer disease Treatment of complications: Refer to Ans 3 of same chapter.
• Severe burns
• Urinary catheters Supportive Treatment
Q.33. Enumerate etiology, clinical features, investigations, ♦♦ Treatment of fever – paracetamol
complications and management of enteric fever. ♦♦ Good nursing care
 (Nov 2008, 15 Marks) ♦♦ Nutritious diet should be given
Or ♦♦ Fluid and electrolyte balance should be maintained.
♦♦ For severe toxemia and peripheral circulatory failure.
Discuss clinical feature, complication and management
corticosteroids may be used, i.e. Dexamethasone 3 mg/kg
of enteric fever. (May/Jun 2009, 15 Marks)
stat followed by 8 doses of 1 mg/kg 6 hourly for 48 hours
Or each given by IV infusion over 30 min.
Write etiology, clinical features, investigations, treat-
Treatment of Carrier
ment and complications of enteric fever (Typhoid).
 (Jun 2014, 12 Marks) Patients who are asymptomatic, but constantly releases bacteria
Or in stool because bacteria are persisting in gall- bladder.
♦♦ Ampicillin 500 mg QID for six weeks.
Describe the complications and treatment of enteric
♦♦ Ciprofloxacin 500 mg BD for 2 to 4 weeks.
fever
♦♦ Cholecystectomy if above measure fails.
 (Sep 2005, 10 Marks)
Or Q.34. Write in brief on gum hypertrophy.
 (Nov 2008, 10 Marks)
Write enteric fever under following headings:
 (Dec 2012, 2 Marks Each) Ans. Gum hypertrophy means increase in the size of gums.
a. Etiology Classification
b. Clinical features
c. Complications  ccording to etiologic factors and pathologic changes, gingival
A
d. Treatment enlargements are:
Or
Infmmatory Enlargement
Write etiology, clinical features, investigation and
treatment of enteric fever. (Jan 2016, 12 Marks) ♦♦ Chronic
Or ♦♦ Acute.

Describe enteric fever under following headings: Drug Induced Enlargement


 (Mar 2016, 1+2+1+2 Marks) Enlargement Associated with Systemic Disease
a. Etiology
b. Clinical features ♦♦ Conditional enlargement
c. Complications • Pregnancy
d. Treatment • Puberty
Or • Vitamin C deficiency
• Plasma cell gingivitis
Write etiology, clinical features, diagnosis and manage- • Non-specific conditional enlargement.
ment of enteric fever. (Jan 2018, 12 Marks) ♦♦ Systemic diseases causing gingival enlargement
Ans. For etiology refer to Ans 1 of same chapter. • Leukemia
For Clinical features refer to Ans 2 of same chapter. • Granulomatous disease.
206   Mastering the BDS IIIrd Year  (Last 25 Years Solved Questions)

Neoplastic Enlargement Symptoms


♦♦ Benign tumors ♦♦ Patient complains of presence of sore throat.
♦♦ Malignant tumors ♦♦ Low grade fever is present.
♦♦ Presence of headache and malaise.
False Enlargement
♦♦ Hoarseness of voice, dyspnea in cases with laryngeal
♦♦ According to location and distribution gingival diphtheria.
enlargement are classified as:
• Localized: Gingival enlargement limited to one or Signs
more teeth.
♦♦ Presence of gross cervical lymphadenopathy.
• Generalized: Entire mouth, gingiva is enlarged.
♦♦ Formation of bluish white or grayish green
• Marginal: limited to marginal gingiva.
pseudomembrane at the site of infection.
• Papillary: Confined to inter-dental papilla.
♦♦ Punched out ulcerations are present if skin is involved.
• Diffuse: Involves all parts of gingiva that is marginal,
♦♦ Foul smelling serosanguinous discharge in cases of nasal
attached and inter-dental.
diphtheria.
• Discrete: Isolated sessile or pedunculated tumor like
♦♦ Cyanosis is seen in cases with laryngeal diphtheria.
enlargement.
♦♦ According to degree of gingival enlargement: Treatment
• Grade 0: No sign of gingival enlargement.
• Grade I: Enlargement confirmed to inter-dental papilla Refer to Ans 8 of same chapter.
• Grade II: Enlargement involves papilla and marginal
gingiva. Q.36. Write in brief clinical features and treatment of
• Grade III: Enlargement covers three quarters or more syphilis. (Jun 2010, 5 Marks)
of the crown. Ans. Syphilis is a sexually transmitted disease caused by
Treponema palladium.
Clinical Features
Clinical Features
♦♦ Presence of slight ballooning of interdental papilla and
marginal gingiva Acquired syphilis is divided into three stages, based on this
♦♦ The enlargement progress slowly and painlessly clinical features are as follows:
♦♦ It occurs as a discrete sessile or pedunculated mass ♦♦ Primary syphilis
resembling a tumor, it may be inter proximal or on • A single painless macule is seen over penis which
marginal or attached gingiva later on become papular and then ulcerates forming
♦♦ The lesions are slow growing masses punched out ulcer with well-defined margins,
♦♦ The lesions undergo spontaneous reduction in size, indurated base and fail to bleed on trauma. It is known
followed by exacerbation and continued enlargement as chancre.
♦♦ Painful ulceration sometimes occur in fold between mass • In men, the ulcer is found on the coronal sulcus or
and adjacent gingiva. on glans penis and in women on vulva, vaginal walls
or cervix.
Treatment • Bilateral inguinal lymphadenopathy occur. Lymph
nodes are discrete, rubbery in consistency and are
♦♦ Scaling and curettage: If the size of enlargement does
non-tender.
not interfere with complete removal of deposits, the
• In very short time, generalized lymphadenopathy
enlargement caused due to inflammation is treated by
occurs and as the lesion heals lymphadenopathy
scaling and curettage.
persists for a longer time.
♦♦ Surgical removal: Indicated for two reasons.
• Untreated ulcers resolve spontaneously in 3 to 8
• In enlargement with significant fibrotic component
weeks, usually without leaving a scar.
that does not undergo shrinkage following shrinkage
♦♦ Secondary syphilis
and curettage.
• In one-third of patients, primary lesions are still
• If size of enlargement interferes with access to root
evident when secondary manifestations occur.
surface deposits.
• One to three months after appearance of primary
Q.35. Write in brief signs, symptoms and treatment of diph- chancre secondary stage develops.
theria. (May/June 2009, 5 Marks) • Patient complains of malaise, headache and fever.
Ans. Diphtheria is an acute infectious disease caused by non- • A macula-papular rash appears over trunk and
motile, non-sporing generally aerobic Gram-positive extremities which is symmetrical, dull red in color
rods C. diphtheriae. and is nonitchy.
Section 1:  General Medicine  207

• Condylomata lata are seen over round the anus, on • Dysuria is presence and increased frequency of
labia, between buttocks, on lateral aspect of scrotum micturition.
and other warm moist areas of body. • Pus is greenish yellow in color.
• Ulcers are seen over mucous membrane of mouth, ♦♦ In females:
these ulcers may coalesce to form snail track ulcers. • Presence of dysuria, vaginal discharge, abnormal
• Laryngeal lesions involving the vocal cords may give menstrual bleeding and rectal discomfort are the
rise to hoarseness of voice symptoms.
• Either regional or generalized lymphadenopathy is • Leucorrhea is present and feature of pelvic infection
present. are present. Pelvic infection may lead to abscess
♦♦ Late syphilis formation and toxemia.
It is divided into two, i.e. late latent syphilis and tertiary • Extension of infection leads to salpingitis, bartholinitis
syphilis and abdominal pain.
Late latent syphilis
It is associated with no clinical manifestations, but time Diagnosis
elapsed since acquisition of disease is more than 2 years.
♦♦ History of sexual contact with pus discharge from urethra
Tertiary syphilis in males and from vagina in females make diagnosis more
• It takes years to develop. likely.
• Gumma is a painless round swelling which is rubbery ♦♦ It is made by demonstration of intracellular gram-negative
in consistency and involves deeper tissues such as diplococcic in smears obtained from urethral discharge
muscle or bone and later on manifests as solitary, deep and staining with Gram stain.
punched out mucosal ulcer. ♦♦ Immunofluoroscent antibody test gives quick diagnosis
• Mucosal gummas affect submucous tissues of mouth, ♦♦ ELISA is done to detect gonococcal antigens.
throat, palate, larynx, pharynx and nasal septum. They
can ulcerate with punched out appearance and lesions Complications
have a sloughy base.
♦♦ In males:
• Bony gummas are diffuse subperiosteal reactions
• Local complications
which often occur in long bones, particular in anterior
–– Epididymitis
margin of tibia.
–– Prostatitis
• Formation of lytic lesions can lead to perforation of
–– Inguinal lymphadenitis
hard palate or nasal septum which causes collapse of
–– Periurethral abscess and later on periurethral
nasal bridge.
fistula.
Treatment • Systemic complications
–– Arthritis
Primary, Secondary and Early Latent Syphilis
–– Bacteremia.
♦♦ Benzathine penicillin G 2.4 million units IM usually in two ♦♦ In females:
sites (4 mL in each buttock) at a single visit. Additional • Local complications
dosages of 2.4 million units should be given 7 to 14 days –– Premature birth
later for latent syphilis of unknown duration.. –– Salpingitis
♦♦ If patient is allergic to penicillin give ceftriaxone 125 mg –– Pelvic infection.
IM OD for 10 days and azithromycin 1 gm orally OD Or • Systemic complications
oral doxycycline 100 mg BD for 14 days. –– Perihepatitis
Tertiary Syphilis –– Pustular eruption.

♦♦ Benzathine penicillin G 2.4 million units IM weekly for Treatment


three weeks. ♦♦ IM injection of 2.4 g of procaine penicillin with 1 g of
♦♦ If patient is allergic to penicillin give oral doxycycline 100
probenecid orally.
mg BD for 14 days.
♦♦ Alternative therapy oral amoxicillin or ampicillin 3 g as
Q.37. Write short note on gonorrhea. (Jan 2012, 5 Marks) single dose with 1 g of probenecid orally Or cotrimoxazole
Ans. It is a sexually transmitted disease caused by Neisseria 4 g in single dose.
gonorrhoea. ♦♦ For penicillin resistant gonorrhea ciprofloxacin is given
250 to 500 mg orally as single dosage Or Cefotaxime 0.5
Clinical Features to 1 g IM as single dosage.
♦♦ In males: ♦♦ For complicated gonorrhea patient is hospitalized.
• Earliest symptom present is burning or tingling Penicillin G IV 10 million units is given for 5 days;
sensation in urethra followed by discharge which ciprofloxacin 500 mg BD for 5 days; ceftriaxone 1 g IV for
becomes muco-purulent. 5 days is given.
208   Mastering the BDS IIIrd Year  (Last 25 Years Solved Questions)

Q.38. Write short note on risk factors and prophylaxis of HIV Three clinical stages are present:
infection. (Mar 2013, 3 Marks) ♦♦ Cold Stage: It remains for half an hour. Patient feels intense
Ans. Following are the risk factors of HIV infection cold and shivers from head to foot, teeth of patient chatter.
Patient cover himself with blanket. Fever raises rapidly
Risk Factors and is as high as 41°C.
♦♦ People who have unprotected vaginal or anal sex. ♦♦ Hot stage: It follows cold stage and last for 3 to 4 hours.
♦♦ People who have sex with many partners, thereby Patient feels intense heat and throws blanket. Patient has
increasing the chance that they will encounter a partner flushed face, headache, vomiting, dry skin.
who is HIV infected. ♦♦ Sweating Stage: Patient has excessive perspiration,
♦♦ People who share needles, for example for intravenous temperature is declined and patient feels relief.
drug use, tattooing or body piercing Diagnosis
♦♦ Babies of mothers who are HIV infected.
♦♦ People who have another STD, especially STDs that cause ♦♦ Clinical: Periodic fever with chills, sweating, anemia and
open sores or ulcers such as herpes, chancroid or syphilis. splenomegaly.
♦♦ Hemophiliacs and other people who frequently receive ♦♦ Blood film: Malarial parasites are identified in thick and
blood products (this risk is now very much diminished, thin blood smears. Common microscopic characters of
but there are still countries where blood is not adequately falciparum malaria are high concentration of parasites,
screened) predominant thin ring-shaped trophozoites
♦♦ Health care workers, where precautions are neglected or ♦♦ Malarial antigen spot test using parasite LDH: P. falciparum
fail for example through not wearing gloves or accidental antigens are taken from blood by finger prick and are
needle stick injuries. exposed to monoclonal antibodies to detect antigens and
read out color bands.
Prophylaxis ♦♦ Immunofluoroscent microscopy and PCR (Polymerase
chain reaction)
♦♦ Practice safer sex. This includes using a condom unless
♦♦ Latex agglutination assay.
you are in a relationship with one partner who does not
have HIV or other sex partners. Treatment
♦♦ Never share intravenous (IV) needles, syringes, cookers,
♦♦ Treatment of chloroquine susceptible P. vivax, P. falciparum,
cotton, cocaine spoons, or eyedroppers with others if you
P. ovale, P. malariae chloroquine:
use drugs.
This is given in the dose of 600 mg base followed by 300
♦♦ Do not donate blood, plasma, semen, body organs, or
mg at 6th, 24th and 48th hours. It is useful in treating all
body tissues.
types of malaria. It is curative for P. falciparum malaria but
♦♦ Do not share personal items, such as toothbrushes, razors,
cannot prevent relapses due to exoerythrocytic cycles of
or sex toys, that may be contaminated with blood, semen,
P. vivax malaria
or vaginal fluids.
♦♦ Treatment of chloroquine resistant P. falciparum:
♦♦ The risk of a woman spreading HIV to her baby can be
• Quinidine IV 10 mg/kg dissolved in 300 mL normal
greatly reduced if she: is on medicine that reduces the saline infused over 1 to 2 hour.
amount of virus in her blood to undetectable levels during • Quinine hydrochloride: 600 mg TDS for 3-7 days is
pregnancy; continues treatment during pregnancy; Does useful.
not breast-feed her baby. If required for cerebral malaria, this drug can given IV
♦♦ Healthcare workers should take universal precautions the dose is 7 mg/kg over 30 min, followed by 10 mg/kg
while treating HIV positive patient. over 4 hours and then 10 mg/kg over 8 hour or until
Q.39. Discuss the signs, symptoms, diagnosis and treatment the patient can complete a week of oral treatment.
of malaria. (Aug 2012, 12 Marks) • Mefloquine: It provides rapid schizonticidal action in
Or single dose of 15 mg/kg orally maximum dose is 1000
to 1250mg.
Write short note on malaria. • Halofantrine
 (Mar 2016, 3 Marks) (Dec 2015, 3 Marks) Dosage: Adult 500 mg BD for 4–6 days.
Ans. Malaria is the acute febrile illness which is characterized Children: 8 mg/kg
by paroxysms of fever as a result of asexual reproduction • Artemether: This drug is rapidly acting, safe and
of plasmodia in red cells. is effective against multidrug resistant infections.
Artemether 3.2 mg/Kg IM is given followed by 1.6mg/
Symptoms and Signs
Kg IM every 12 to 24 hours until patient wakes up.
Onset of malaria is with lassitude, anorexia, headache and Artesunate 2 mg/Kg IV stat followed by 1mg/Kg 12
fever with chills. hourly.
Section 1:  General Medicine  209

• Sulfadoxine and pyrimethamine: Combination of ♦♦ Take vitamin C supplements, if citrus fruits and juices
sulfadoxime 1500mg and pyrimethamine 25 mg helps cannot be taken.
to cure an acute attack of chloroquine resistant malaria. ♦♦ If dentures make gums bleed, wear them only during meals.
♦♦ Treatment of Chloroquine resistant P.vivex ♦♦ Gum bleeding can be controlled by applying pressure with a
Oral mefloquine and halofantrine. gauze pad soaked in ice water directly to the bleeding gums.
♦♦ Treatment of persistent hypnozoites in P. vivex or P. ♦♦ Brush teeth gently after every meal. Use a soft brush, or a
ovale infection special vibrating brush to clean the teeth without irritating
• Primaquine: It is given in the dose of 7.5 mg BD for 14 the gums.
days usually after doing a G6PD test ♦♦ Use gumpaint regularly to control bleeding from the gums.
• Bulaquine: It is given 25 mg OD for 5 days.
Q.40. Write short note on clinical features and treatment of Q.42. Write short note on herpes zoster infection.
P. vivex malaria. (Dec 2012, 3 Marks)  (Feb 2013, 5 Marks)
Or
Ans. Write short answer on herpes zoster infection.
Clinical Features  (Apr 2018, 3 Marks)
Onset of malaria is with lassitude, anorexia, headache and Ans. It is also known as shingles.
fever with chills. Herpes zoster is the secondary form of infection.
Three clinical stages are present: After primary infection virus remains dormant in the
dorsal root or cranial nerve ganglia. The disease recurs
♦♦ Cold Stage: It remains for half an hour. Patient feels intense
in the localized form which remains to its dermatome
cold and shivers from head to foot, teeth of patient chatter.
innervated by spinal or cranial sensory ganglion or it
Patient covers himself with blanket. Fever raises rapidly
can affect the motor nerve causing facial nerve palsy.
and is as high as 41°C.
♦♦ Hot stage: It follows cold stage and last for 3 to 4 hours. Predisposing Factors
Patient feels intense heat and throws blanket. Patient has
flushed face, headache, vomiting, dry skin. Herpes zoster is more common in people with diminished cell
♦♦ Sweating Stage: Patient has excessive perspiration, mediated immunity. This includes elderly people, patients with
temperature is declined and patient feels relief. lymphoma receiving chemotherapy or steroids, and individual
with HIV.
Treatment
Clinical Features
♦♦ Treatment of chloroquine susceptible P. vivax:
• Chloroquine: This is given in the dose of 600 mg ♦♦ It consists of two stages, i.e.
followed by 300 mg at 6th, 24th and 48th hours. • Pre-eruptive stage: Pain with hyperaesthesia along the
♦♦ Treatment of Chloroquine resistant P.vivax: course of the nerve and fever.
• Oral mefloquine with dosage of three 250 mg tablets • Eruptive stage: May be the first manifestation of disease
which are repeated after 6 hours. in some cases.
♦♦ Treatment of persistent hypnozoites in P. vivax infection: ♦♦ There are several edematous patches along the course of
• Primaquine 30 mg per day orally for 14 days is given. a nerve with intervening clear areas, these are very tender
Q.41. Write short note on causes and treatment of bleeding and painful.
gums. (Feb 2014, 3 Marks) ♦♦ A few hours later these patches are surmounted with small
Ans. Causes of Bleeding Gums vesicles in cluster. Vesicles occur in crops. The contents
Following are the causes of bleeding gums: may become purulent. The vesicles crust over, and in
• Gingivitis and gingivostomatitis absence of secondary infection, clear within a week.
• Bleeding disorders, i.e. Thrombocytopenia, ♦♦ Chronic ulcerative lesions may be seen in HIV-infected
hemophilia, leukemia and anticoagulants. patients.
• Abnormalities of vessel wall: Scurvy, Henoch Schönlein ♦♦ Regional glands are painful and tender.
purpura, dysproteinaemia ♦♦ An attack lasts for 2–3 weeks.
• Connective tissue disorders: Ehlers-Danlos syndrome, ♦♦ The rash is usually unilateral and leaves behind
pseudoxanthoma elasticum. pigmentation and scarring.
♦♦ Thoracic and lumbar dermatomes are the sites most
Treatment of Bleeding Gums commonly affected.
♦♦ Bleeding gums can be treated by the removal of the source ♦♦ An eruption in the mandibular and maxillary distribution
of bacteria. of trigeminal nerve is associated with oral, palatal and
♦♦ Proper maintenance of the teeth by the patient is a must. pharyngeal lesions. If there is involvement of ophthalmic
♦♦ Sore and bleeding gums can be aggravated by citrus fruits division, there can be keratitis or uveitis with presence of
and juices, rough or spicy food, alcohol, and tobacco. vesicles over the nose.
210   Mastering the BDS IIIrd Year  (Last 25 Years Solved Questions)

Complications ♦♦ For post herpetic neuralgia analgesics, amitriptyline,


♦♦ Post-herpetic neuralgia gabapentin, pregabalin, tramadol and carbemazepine is given.
♦♦ Keratitis and corneal ulceration Q.43. Write short note on postexposure prophylaxis in HIV/
♦♦ Neurological, i.e. encephalitis, meningitis and myelitis. AIDS. (Nov 2014, 3 Marks)
Or
Management Write post exposure prophylaxis in AIDS.
♦♦ Acyclovir 800 mg orally five times a day for a week is  (Apr 2015, 4 Marks)
given. Acyclovir cream can be applied over the lesions also. Ans. Following is the post exposure prophylaxis in HIV/AIDS:
♦♦ Prednisolone 40–60 mg in tapering doses is given for 3 • HIV post exposure prophylaxis is started within an
weeks. hour or two.
♦♦ Calamine lotion or cream is given to the patient for • Depending on the type and seriousness of the
prevention of irritation in the lesion. exposure either a basic regimen, i.e. zidovudine with
♦♦ Anti-septic powder can also be applied for preventing the lamivudine is followed or expanded drug regimen,
secondary infection. This should be applied after vesicles i.e. zidovudine + lamivudine + Indinavir is given.
break • Following is the recommended HIV postexposure
♦♦ Analgesics can be given as per need. prophylaxis:

Exposure type Small volume, i.e. few drops splashed Large Volume, i.e. major blood splash
HIV positive class I ,i.e. Asymptomatic HIV Consider Zidovudine with lamivudine Zidovudine + lamivudine + Indinavir
infection
HIV positive class II i.e. symptomatic HIV Recommended Zidovudine with lamivudine Expanded Zidovudine + lamivudine +
infection, AIDS, acute seroconversion Indinavir
Exposure type Less severe More severe
Source HIV status known No postexposure prophylaxis is warranted. Con- No postexposure prophylaxis is warranted
sider zidovudine with lamivudine for source with
HIV risk factors
Unknown source No post exposure prophylaxis is warranted. Consid- --------
erte zidovudine with lamivudine in settings where
exposure to HIV-infected person is likely

Q.44. Write etiology, clinical features, investigations and Stage of HIV infection Clinical features
complications of HIV infection. (Feb 2015, 12 Marks) Acute HIV syndrome • Fever
Ans. The presence of a reliably diagnosed disease, i.e. • Headache
atleast moderately indicative of an underlying defect • Pharyngitis
in cell-mediated immunity occurring in a person with • Malaise
no known cause for immunodeficiency other than the • Lymphadenopathy
• Diffuse cutaneous erythematous rash
presence of HIV.
Chronic asymptomatic • Headache
infection • Diffuse reactive lymphadenopathy
Etiology Early symptomatic • Fever
disease • Night sweat
♦♦ HIV infection is caused by the infection with virus HIV-1 • Chronic diarrhea
and HIV-2. • Fatigue
♦♦ HIV-1 is most common worldwide. It has many strains • Minor oral infections
due to the mutation. • Headache
• Anorexia
♦♦ HIV-2 has 40% sequence homology with HIV-1 and is very • Weight loss
closely related to simian immunodeficiency virus. • Nausea
• Vomiting
Clinical Features Late symptomatic Symptoms of Pneumocystic carnii pneu-
disease monia and other opportunistic infections
Clinical features of HIV infection as per its stage are as Advanced disease Symptoms of AIDS, i.e. protozoal, fungal,
follows: viral and bacterial infections.
Section 1:  General Medicine  211

Investigations Or
♦♦ Investigations for the diagnosis of HIV infection Write short answer on herpes simplex
• HIV-ELISA: This is the most commonly used  (May 2018, 3 Marks)
screening test for HIV infection. If this test is positive Ans.
confirmation should be done by western blot test.
• HIV-rapid antibody test: They are for rapid diagnosis, Causative Organism
i.e. within 10 to 15 minutes. It is also a screening test, Herpes simplex is caused by a DNA virus. Virus consists of two
as if this test comes positive confirmation is done by strains, i.e. Type 1 and Type 2. Type 1 virus enters through the
western blot. mouth while Type 2 virus is sexually transmitted and leads to
• Western blot: It is the confirmatory test for HIV anorectal or genital infections.
infection.
• Blood cell count: Since neutropenia, anemia and Manifestations or Clinical Features
pancytopenia are associated with HIV infection, this
♦♦ Primary infection by herpes simplex
test is done.
• Mucous membranes:
♦♦ Investigations for monitoring progress of HIV infection
• Absolute CD4 lymphocyte count: This is most commonly –– Acute gingivostomatitis: It is characterized by
used. As its count decreases occurrence of opportunistic soreness of mouth, salivation, fever and malaise.
infection and malignancy is high, i.e. if count is less Vesicles are present in the mouth.
than 200 cells/µL. –– Keratoconjunctivitis: The condition is painful and
• CD4 lymphocyte percentage: In this if count is less is unilateral. Ulceration of cornea can occur which
than 14% occurrence of opportunistic infection and leads to chronic scarring.
malignancy is high, if treatment is not given to patient. –– Genital herpes: In females, vesicles are present over
♦♦ Investigations for virological monitoring the vulva, cervix or vagina. In males, vesicles are
In this, HIV viral load tests are done such as HIV – RNA seen on glans penis or less commonly urethritis.
by PCR, HIV – RNA by bDNA, HIV – RNA by NASBA. • Skin:
These all tests measure the actively replicating HIV virus. –– Disseminated herpes simplex: Seen in new born
These cells also deflect the response of antiretroviral drugs. infants. Brain, liver, lungs and other vital organs
These test are excellent for diagnosis of acute HIV infection are affected.
before its seroconversion.
–– Eczema herpeticum: In cases with atopic dermatitis,
Complications herpes simplex develops in varicelliform.
–– Herpetic whitlow: It manifest as indolent
System Complications inflammatory arising at site of minor skin trauma
Neuropsychiatric Primary central nervous system lymphoma, on finger in form of deep, painful multiple vesicles
Chronic psychiatric disorders over the finger tip.
Head and neck Gingivitis, dental and salivary gland disease –– Anal infection: In homosexuals there is intense
pain, tenesmus and local vesicles are present.
Cardiovascular Cardiovascular disease, endocarditis
• Recurrent infection
Pulmonary Chronic obstructive pulmonary disease, lung –– Herpes labialis: Lesions appear on vermilion border
cancer (including Kaposi's sarcoma and lym-
of lip and surrounding skin. These lesions are grey
phoma)
or white vesicles which rupture quickly leaving
Gastrointestinal Viral hepatitis, lymphoma, Kaposi's sarcoma,
small red ulcerations with slight erythematous
HPV- related malignancies
halo on lip covered by brown crusts.
Renal/genitouri- Chronic kidney disease not caused by HIV-
–– Eye infection: Recur as superficial keratitis.
nary associated nephropathy
–– Genital infection: They are mild and short as
Musculoskeletal Osteopenia, osteoporosis, osteonecrosis compared to its primary form.
Hematologic or on- Lymphoma, multiple myeloma
cologic Treatment
Dermatologic Papulosquamous disorders, molluscum conta- ♦♦ 5% acyclovir cream, is applied in every 4 hours.
giosum, Kaposi sarcoma
♦♦ For severe infection oral acyclovir 200 mg 5 times a day is
Q.45. Write herpes simplex under following headings: given. Reduce the dose to half in children.
 (Apr 2015, 2+2+2 Marks) ♦♦ IV acyclovir 5 mg/kg for every 8 hours by slow infusion
a. Causative organism is given.
b. Manifestations/Clinical features ♦♦ Anti-bacterial therapy is given in mucocutaneous herpes
c. Treatment to reduce risk of secondary infection.
212   Mastering the BDS IIIrd Year  (Last 25 Years Solved Questions)

Q.46. Write short note on rose spots. (Apr 2015, 2 Marks) Management
Ans. Rose spots are the clinical sign for typhoid fever. ♦♦ In acute phase of disease, patient should be on bed rest.
• As first week of typhoid fever is over rose spots Condition resolve in 4 weeks.
appear. ♦♦ Short course of steroids can be used, i.e. 10 mg prednisolone
• Rose spots appear over the trunk. They can also be thrice daily with tapering dosages.
seen over back and chest. ♦♦ Gargles with soluble aspirin are helpful for relieving sore
• They are small macules which are 2–4 mm in throat.
diameter, red in color and on applying pressure they ♦♦ Antibiotics, i.e. erythromycin can be given, if secondary
show blanching. infection is present.
• Rose spots lasts for 2–3 days. ♦♦ Interferon-α administration, cytotoxic chemotherapy and
radiation therapy can also be used.
Q.47. Write in short treatment of esophageal candidiasis.
 (Apr 2015, 2 Marks) Q.49. Write short note on complications of enteric fever.
Ans. Treatment of esophageal candidiasis  (Sep 2006, 10 Marks)
• Oral fluconazole systemically 200 mg/day for 14–21 Ans.
days is given. ♦♦ Intestinal complications
• In cases with severe dysphagia, IV administration • Hemorrhage may occur at the end of the second week
of Fluconazole is done. and is characterized by black stools, tachycardia,
• Patients who had previous infection of esophageal hypotension and diarrhea. There is no abdominal
candidiasis, patient should be given Fluconazole pain or rigidity or obliteration of liver dullness like in
100–200 mg daily for prophylaxis. intestinal perforation. Transfusions may be needed, if
there is massive blood loss.
Q.48. Write short note on infectious mononucleosis. • Perforation may occur at the end of the second week or
 (Dec 2015, 3 Marks) in the third week. It is characterized by acute pain in
Ans. It is also known as glandular fever. the lower abdomen, vomiting, abdominal distension,
• Infectious mononucleosis is caused by Epstein-Barr hypotension and tachycardia. Liver dullness may be
virus (EBV). obliterated and the abdomen becomes tender, rigid
• Incubation period for virus is 7–20 days. and silent (absent peristalsis).
• Tympanitis
Pathogenesis • Cholecystitis
EBV undergo replication in lymphoid tissues of throat and is • Splenic infarction
transmitted to healthy person via saliva. • Rarely, appendicitis, intussusception and pyogenic
liver abscess
Clinical Features ♦♦ Extraintestinal complications
• Myocarditis, endocarditis
♦♦ There is presence of acute onset of fever with chills, core
• Osteomyelitis, arthritis, typhoid spine and Zenker’s
throat, headache, malaise and tiredness.
degeneration of rectus abdominis
♦♦ Lymph nodes become enlarged, discrete and slightly
• Pulmonary infection and embolism
tender affecting cervical and submandibular lymph
• Thrombophlebitis
nodes. • Electrolyte imbalance, shock and acute renal failure
♦♦ Non-tender splenomegaly is present. • Neurological: Meningoencephalitis, meningism,
♦♦ Petechial rash can occur at the junction of hard and soft cranial nerve palsies, myelitis, ascending paralysis,
palate on 4th day and can persist for 3–4 days. Parkinsonism, athetosis, cerebellar ataxia, neuritis
Investigations • Typhoid state: This is characterized by coma vigil,
muttering delirium, carphologia (picking up clothes
♦♦ Peripheral smear shows leucocytosis with atypical in bed) and subsultus tendinosis
lymphocytosis. • Psychosis
♦♦ Paul-Bunnell test can be positive in 1:32 dilution.
♦♦ Liver transminases are raised. Q.50. Enumerate the complications of typhoid fever.
 (Feb. Mar 2005, 5 Marks)
Complications Ans. Following are the complications of typhoid fever:
♦♦ Fatigue ♦♦ Intestinal complications
♦♦ Hemolytic anemia • Hemorrhage
♦♦ Hepatitis • Perforation
♦♦ Meningoencephalitis • Paralytic ileus
♦♦ Thrombocytopenia. • Peritonitis
Section 1:  General Medicine  213

♦♦ Extra-intestinal complications: ♦♦ HIV-related rash: Acute HIV exanthem, papular rash of HIV
• Meningitis ♦♦ Pruritic papular and follicular eruptions: Folliculitis
• Bone and joint infection Eosinophilic pustular, pityrosporum, demodectic, insect
• Cholecystitis bite reaction, Prurigo nodularis.
• Encephalopathy ♦♦ Papulosquamous disorders: Seborrheic dermatitis, psoriasis,
• Pneumonia Reiter's syndrome, atopic dermatitis, lichen planus
• Granulomatous hepatitis ♦♦ Malignancies: Kaposi’s sarcoma, squamous cell and basal
• Nephritis cell carcinoma, malignant melanoma
• Myocarditis ♦♦ Miscellaneous dermatoses: Acquired ichthyosis, chronic
photosensitivity, granuloma annulare, vitiligo,
Q.51. Write etiology, clinical features, diagnosis and manage-
immunobullous disorders, Sjögren's syndrome, porphyria,
ment of malaria. Mention complications of falciparum
telangiectases, leukocytoclastic vasculitis, scabies
malaria.  (Jan 2017, 12 Marks) (Norwegian), hidradenitis suppurativa, palmoplantar
Ans. Malaria is a common tropical disease. keratoderma.
Etiology Oral Manifestations of AIDS by EC-Clearinghouse
It is caused by protozoa, Plasmodium through the bite of a Group 1: Strongly Associated with HIV Infection
female anopheles mosquito. ♦♦ Candidiasis: erythematous, pseudomembranous, angular
Requirements for the causation of malaria are: cheilitis
♦♦ Presence of suitable female anopheles mosquito ♦♦ Hairy leukoplakia
♦♦ Reservoir of malaria infection in the particular area. ♦♦ Kaposi’s sarcoma
♦♦ Suitable non-immune or partly immune hosts ♦♦ Non-Hodgkin’s lymphoma
♦♦ An environmental temperature with suitable humidity ♦♦ Periodontal diseases: Linear gingival erythema, necrotizing
For clinical features, diagnosis and management of malaria gingivitis, necrotizing periodontitis.
refer to Ans 40 of same chapter. Group 2: Less Commonly Associated with HIV Infection
♦♦ Bacterial infections: Mycobacterium avium-intracellulare,
Complications of Falciparum Malaria
Mycobacterium tuberculosis
♦♦ Central nervous system: Cerebral malaria, i.e. convulsions Melanotic hyperpigmentation
and coma ♦♦ Necrotizing ulcerative stomatitis
♦♦ Renal system: Black water fever, acute renal failure ♦♦ Salivary gland disease: Dry mouth, unilateral or bilateral
♦♦ Blood: Severe anemia, disseminated intravascular swelling of major salivary glands
coagulation ♦♦ Thrombocytopenia purpura
♦♦ Respiratory: Adult respiratory distress syndrome ♦♦ Oral ulcerations NOS (not otherwise specified)
♦♦ Metabolic: Hypoglycemia, metabolic acidosis ♦♦ Viral infections: Herpes simplex, human papillomavirus,
♦♦ Gastrointestinal tract and liver: Diarrhea, jaundice, splenic varicella—zoster
rupture
Group 3: Seen in HIV Infection
♦♦ Miscellaneous: Hypotensive shock, hyperpyrexia
♦♦ Pregnancy: Maternal death, abortion, stillbirth and low ♦♦ Bacterial infections: Actinomyces israelii, Escherichia coli,
birth weight Klebsiella pneumoniae
♦♦ Cat-scratch disease (Bartonella henselae)
Q.52. Write short note on cutaneous and oral manifestations ♦♦ Epithelioid (bacillary) angiomatosis (Bartonella henselae)
of AIDS.  (Apr 2017, 6 Marks) ♦♦ Drug reactions: Ulcerative, erythema multiforme, lichenoid,
Ans. Cutaneous Manifestations of AIDS toxic epidermolysis
Following are the cutaneous or dermatological manifestations ♦♦ Fungal infections other than candidiasis: Cryptococcus
of AIDS: neoformans, Geotrichum candidum, Histoplasma capsulatum,
♦♦ Superficial fungal infection: Candidiasis, dermatophytosis, Mucoraceae (mucormycosis/zygomycosis), Aspergillus
pityrosporum infection flavus
♦♦ Disseminated fungal and protozoal infection: Cryptococcus ♦♦ Neurologic disturbances: Facial palsy, trigeminal neuralgia
neoformans infections, histoplasmosis, sporotrichosis, ♦♦ Recurrent aphthous stomatitis
dermal leishmaniasis, coccidioidomycosis ♦♦ Viral infections: Cytomegalovirus, molluscum contagiosum
♦♦ Bacterial and mycobacterial infections: Staph. aureus, M.
avium intracellulare, M. tuberculosis, actinomycosis. Q.53. Write short answer on candidiasis.
♦♦ Viral: HPV infection, molluscum contagiosum, herpes  (May 2018, 3 Marks)
simplex virus, varicella and herpes zoster, cytomegalovirus, Ans. Candidiasis is a fungal infection caused by an
EB virus. opportunistic fungus known as candida albicans.
214   Mastering the BDS IIIrd Year  (Last 25 Years Solved Questions)

Predisposing factors Diagnosis


♦♦ Changes in oral flora: Marked changes in oral flora ♦♦ Clinically the white lesion can be scrapped off by wet
occur due to administration of antibiotics, excessive use guaze.
of antibacterial mouth rinses, xerostomia secondary to
anti – cholinergic agent or salivary gland disease. These Treatment
chages result ininhibiton of competitive bacteria leading ♦♦ Treatment includes antiseptic mouth wash with antiseptic
to candidiasis. solution for 4-5 days.
♦♦ Local irritant: Chronic local irritants i.e. dentures, ♦♦ Nystatin tablets may be retained locally in the mouth 3-4
orthodontic appliances etc. times daily or may be crushed or mixed with glycerine for
♦♦ Drug therapy: Various drugs i.e. corticosteroids, cytotoxic local application.
drugs, immunosuppressive agents and radiation to head ♦♦ Oral administration of nystatin is needed in severe or
and neck. systemic infection. Nystatin is given 250mg TDS for two
♦♦ Acute and chronic diseases such as leukemia, diabetes, weeks followed by 1 troche per day for third week.
tuberculosis etc.
♦♦ Malnutrition states such as low serum vitamin A,

♦♦
pyridoxine and Vitamin A.
Endocrinopathy: Such as hypothyroidism, hyper
12. Miscellaneous
parathyroidism and Addison’s disease
Q.1. Write short note on anaphylactic shock. 
♦♦ Immunodeficiency states such as AIDS, Hypogamma­
 (Sep 2006, 5 Marks)
globulinemia
Or
Clinical features Write notes on anaphylaxis. (Aug 2011, 10 Marks)
♦♦ It occur in both infants and adults. In infants, oral lesion Or
occurs between 6th and 10th day after birth. Discuss anaphylaxis and its management in detail
♦♦ Common sites for infection to occur are roof of mouth,  (Jan 2017, 12 Marks)
retromolar area and mucobuccal fold. Ans. Anaphylaxis is an acute and dramatic life-threatening
♦♦ It is more common in women as compared to man. immunological reaction to a drug or other stimulus.
♦♦ Patient can complain for burning sensation.
♦♦ Lesion appears as white plaque which is pearlish white or Etiology
blue white in colour and are present on the oral mucosa.
Lesions resemble as cottage chesse or curdled milk. ♦♦ Antibiotics: Penicillin and streptomycin.
Mucosa adjacent to the lesion appears red in color and is ♦♦ Radio contrast media
moderately swollen. ♦♦ Anesthetic agent, i.e. lignocaine
♦♦ White patches are easily wiped out with wet guaze which ♦♦ Blood and blood products including sera.
leaves normal or erythematous area. This area can be ♦♦ Hormones: Insulin and growth hormone.
painful. Deep invasion of an organism leave an ulcerative ♦♦ Venoms: Bees, spiders and wasps
lesion on removal of patch. ♦♦ Others: NSAIDs, narcotic agents, heparin and thrombolytic
♦♦ In severe infection, the involvement of pharynx and agents.
oesophagus cause dysphagia.
♦♦ The fungus may travel to lower respiratory passage and Clinical Features
may involve lungs in fulminant infection.
♦♦ Onset may be instantaneous or within a few minutes after
Investigations the IV injection and 30 minutes after exposure.
♦♦ Cardiovascular: Tachycardia, arrhythmia, hypotension and
♦♦ On staining with periodic acid Schiff (PAS) method,
circulatory collapse.
candidal hyphae are readily identified. Organisms are
identified by bright magenta color. The candidal hyphae ♦♦ Respiratory: Laryngeal obstruction, angiedema,
are 2μm in diameter, vary in length and may show bronchospasm or pulmonary edema may occur singly or
branching. in combination.
♦♦ About 10–20% KOH is also used to identify organisms ♦♦ Nervous: Syncope and seizures.
readily. ♦♦ GIT: Diaphoresis, abdominal pain and diarrhea may occur.
♦♦ Cultures can be obtained readily on Sabouraud’s medium ♦♦ Skin: Wheel and erythematous lesions are seen which are
and on ordinary bacteriological culture media. Colonies circumscribed, round, discrete, erythematous areas with
are creamy white, smooth with a yeasty color. irregular borders and blanched centres.
♦♦ On corn-meal agar medium C. albicans form ♦♦ Lesions are very pruritic.
chlamydospores. ♦♦ If not treated death may occur.
Section 1:  General Medicine  215

Management It is type I hypersensitivity reaction of skin to variety of


exogenous or endogenous agents.
♦♦ Airways: Endotracheal intubation or perform tracheostomy,
if intubation is not possible. When the subcutaneous tissue involves, it is known as
♦♦ Oxygen is given in high concentration. angiedema.
♦♦ Injection adrenaline 0.5 to 1 mL of 1:1000 solution IM Etiology
♦♦ Fluids: 0.5 or 1 liter of fluid restores blood pressure but 6-9
liters is required for adequate restoration of blood volume. ♦♦ Exogenous causes:
• Ingestants, i.e. drugs and foods
♦♦ Hydrocortisone 300 mg IV.
• Inhalants, i.e. pollen, plant, dust, etc.
♦♦ Aminophylline 250 mg IV in 20 mL dextrose to revert
• Injectants, i.e. penicillin, insulin, anti sera, vaccines.
bronchospasm.
• Contactants, i.e. bee sting and bug bites.
♦♦ Diphenhydramine 50 mg IV slowly.
♦♦ Endogenous causes:
♦♦ Dopamine, if hypotension persists. • Infections, i.e. urinary tract infection, respiratory tract
♦♦ IV atropine and glucagon. infection and candida infections.
♦♦ Ventilatory support is necessary, if patient is critical. • Infestants, i.e. helminths, amoebiasis and giardiasis
Q.2. Describe briefly management of drug, induced ana- • Systemic disease, i.e. systemic lupus erythmatosus
phylaxis. (Feb/Mar 2004, 5 Marks) and lymphomas.
• Psychogenic, i.e. emotional stress.
Or
Outline treatment of anaphylaxis shock. Management
(Jan 2012, 6 Marks) Histamine-mediated Urticaria
Ans. Refer to Ans 1 of same chapter. ♦♦ First line treatment:
• Loratadine 10 mg/day
Q.3. Write short note on drug interactions. 
• Tricyclic anti-depressant doxepine 10 mg TDS or
 (Mar 2001, 5 Marks) (Aug 2012, 5 Marks) ranitidine 150 mg BD
Ans. Drug interaction has been defined as action of an ♦♦ Second line treatment
administered drug upon the effectiveness or toxicity of • Prednisolone 0.5 mg/kg 1 to 2 weeks.
another drug administered earlier, simultaneously or later. ♦♦ Third line treatment:
When two or more drugs are prescribed either • Cyclosporine A
simultaneously or in quick succession, they may exhibit • IV immunoglobin
either a synergistic or antagonistic action. • Plasmapheresis.
Immune Complex-mediated Urticaria
Types of Interaction
♦♦ Dapsone 75–100 mg/day
There are three types of interactions: ♦♦ Indomethacin 25–50 mg TDS
♦♦ Hydroxy chloroquine 200–400 mg/day.
Pharmacokinetic Interactions
Q.5. Describe briefly pyrazinamide. (Feb 1999, 3 Marks)
These results in alterations in delivery of drugs to their sites of Ans. It is an antitubercular drug.
action, i.e. impaired GI absorption, induction of hepatic enzymes • It inhibits mycolic acid synthesis.
and inhibition of cellular uptake. • Pyrazinamide is absorbed orally and has good
penetration in CSF.
Pharmacodynamic Interactions • Hepatotoxicity is more important dose-dependent
In this, the responsiveness of the target system or organ is adverse effect.
modified by other agents. • It is contraindicated in patient with the liver disease.
• Other adverse effects are arthralgia, flushing, rashes,
Pharmacodynamic and Other Interactions between Drugs fever, etc.
They are usually therapeutically useful interactions in which Q.6. Describe briefly side effects of corticosteroids. 
combined effect of two drugs is greater than that of the either  (Feb 1999, 3 Marks) (Sep 2008, 5 Marks)
drug. For example: Amlodipine + Atenolol and Sulfonamides + Ans. Side Effects
Trimethoprim • In hyperglycemia they cause precipitation of
The first two interactions are detrimental while last one is useful diabetes and glucosuria.
• They cause muscular weakness.
interaction employed in therapeutic medicine.
• They cause thinning of skin, fragile skin and purple
Q.4. Write short note on urticaria. (Feb 2006, 3 Marks) striae hirsutism.
Ans. Urticaria is a vascular reaction characterized by transient, • They cause Cushing’s habitus in which there is
evanescent, pruritic wheals occurring on any site of the moon face, narrow mouth and obesity of trunk with
body. thinning of lungs.
216   Mastering the BDS IIIrd Year  (Last 25 Years Solved Questions)

• They increase susceptibility to infection, i.e. TB and Patient with many illness become malnutritioned, if
opportunistic infection may flare up. they are unable to eat a balanced diet for more than few
• They cause delayed healing of wound and surgical weeks, however only a small percentage of these patients
incisions. clearly benefit from parenteral nutrition support.
• They causes peptic ulcer. Patient who benefit most from total parenteral nutrition
• They causes osteoporosis, glaucoma and growth are those at extremes of nutritional deficiency.
retardation in children.
• At high doses, they causes psychiatric disturbance. Q.11. Write short note on leukorrhea. (Mar 2007, 2 Marks)
• They cause suppression of hypothalamopituitary Ans. Leukorrhea is strictly defined as an excessive normal
axis. vaginal discharge.
• The excess secretion is evident from persistent vulval
Q.7. Write short note on fixed drug eruption.
moistness or staining of undergarments or need to
 (Mar 1998, 5 Marks)
wear a vulval pad.
Ans. Fixed drug eruption occurs at the same site each time • It is non-purulent and non-offensive.
the offending drug is taken. • It is non irritant and never causes pruritus.
♦♦ Causative agents are tetracyclines, sulphonamides, • The physiological basis involve in normal vaginal
phenolphthalein, oxyphenbutazone. secretion in development of an endogenous estrogen
♦♦ Lesions, which are often painful and may cause a burning level.
sensation, appear as clearly demarcated oval or round • Excessive secretion is due to:
erythematous plaques which are sometimes bullous and – Physiological excess: Normal secretion is expected
usually 20 mm in diameter, though lesions up to 10 cm in to increase in condition when the estrogen level
diameter may occur. After 1–2 days, the lesions become becomes high. The condition is during puberty,
violaceous. Typically, there are one to three lesions. Lesions during menstural cycle, pregnancy and during
fade over 7–10 days and leave a hyperpigmented patch. sexual excitement.
♦♦ Sites affected are hands, feet, glans penis or lips. – Cervical cause or cervical leukorrhea: Non infective
♦♦ Lesions always occur at the same site. cervical lesion may produce excess secretion
Q.8. Write short note on differential diagnosis.  which produces out at vulva.
 (Mar 2000, 5 Marks) – Vaginal cause or vaginal leukorrhea: Increase vaginal
Ans. Identification of disease by comparison of illness that transudation during the uterine prolapse.
share features of present illness but differ in some critical
Treatment
ways is known as differential diagnosis.
It is given on the basis of determination of cause and ♦♦ Improvement of general health
pathogenic condition. ♦♦ Cervical factor requires surgical treatment such as
electrocautery, cryosurgery
This is done by evaluating the history of disease process,
♦♦ Pelvic lesion producing vaginal leukorrhea requires
sign and symptoms, laboratory test and special test such
appropriate therapy for pathology.
as radiography and ECG.
♦♦ Pills users stop the pill temporarily
For example of differential diagnosis, refer to Ans 27 of
♦♦ Local hygiene is to be maintained meticulously.
chapter Diseases of Blood.
Q.12. Write short note on acne vulgaris. (Mar 2007, 2 Marks)
Q.9. Discuss in short lactose intolerance. 
Ans. Acne vulgaris is a disease in which the pilosebaceous
 (Mar 1997, 5 Marks) follicle becomes oversensitive to normal levels of
Ans. An assessment of person’s blood sugar and exhaled testosterone.
hydrogen level within two hours after drinking a loading
dose of lactose, i.e. is usually about 100 g of lactose for Etiology
average size adult. ♦♦ Androgens
People who readily digest the lactose will have an ♦♦ Follicular keratinization
increase in their blood sugar and little increase in exhaled ♦♦ Hereditary
hydrogen gas during the test. ♦♦ Propionibacterium acnes
People who are unable to digest will have no increase ♦♦ Immunological factors
in blood sugar and the level of hydrogen, they exhaled ♦♦ Environmental factors
will exceed 50 ppm.
Exacerbating Factors
Q.10. Write short note on total parenteral nutrition. 
 (Mar 2000, 5 Marks) Acne worsens with stress and in premenstrual period.
Ans. The intravenous provision of dextrose, amino acids, fats, In patients with aggressive or recalcitrant acne, underlying
trace elements, vitamin and minerals to patients who are cause may be a virilizing syndrome in women, acromegaly,
unable to assimilate adequate nutrition to mouth. occupational exposure to acnegenic agents.
Section 1:  General Medicine  217

Drugs that worsen acne are steroids, hormones (androgen ♦♦ Skin rashes produce itching or burning sensation.
and progesterone), antiepileptic drugs, iodides; can follow facial ♦♦ Disease often causes hyperpigmentation of skin.
massage. ♦♦ Patchy or extensive loss of hair from the scalp is very
Genetic and hormonal factors also play a role. common clinical findings.
♦♦ There is severe burning sensation in the oral mucosa and
Grading the affected area is extremely tendered to palpation.
♦♦ Mild disease: Open (black heads) and closed (white ♦♦ Formation of hemorrhagic macules in the oral mucosa that
heads) comedones with sparse inflammatory lesions. Some becomes frequently ulcerated.
comedones are deep-seated (submarine comedones). ♦♦ Fever, fatigue, malaise, vomiting, diarrhea and anorexia
♦♦ Moderate: Numerous papules and pustules are present.
♦♦ Severe: Polymorphic eruption with comedones, papules, ♦♦ Dysphagia and depression
pustules, nodules and cysts. ♦♦ Splenomegaly and lymphadenopathy.

Management Treatment
♦♦ Topical therapies are the mainstay of treatment for mild Systemic steroid therapy is the treatment of choice.
acne:
Q.14. Write note on emergency drug tray. 
• Benzoyl peroxide 5% has antibacterial and keratolytic
 (Mar 2000, 5 Marks)
properties. It treats both inflamed and non-inflamed
lesions. Ans. Medical emergencies in dental practice are of common
• Topical antibiotics, i.e. 1% clindamycin and 2% occurrence and a dental surgeon must well prepare to
erythrmomycin are used. meet them since correct and early management cannot
• Topical retinoids, i.e. tretinoin cream or gel (0.25 only prevent morbidity but also mortality in such
to 1%), Adelphane (0.1%) cream or gel or solution patients.
applied in night to entire face and leave it for 20–30 For management of any medical emergency dental clinic
minutes and then wash off with mild soap must consist of emergency drug tray.
• Sulphate calamine lotion can be used. Emergency tray contains IV fluids, administration
• Salicylic acid can be used as adjunctive therapy. set, disposable syringe and needles, stethoscope, BP
• Azelaic acid (20%) cream possess anti–microbial, anti apparatus, sterilized pad must be available.
– inflammatory and comedolytic properties. A fare amount of emergency drugs such as the following
♦♦ Systemic therapy must be readily available in the emergency drug tray.
• Antibiotics: Therapy is given for 3–6 months. • Inj. adrenaline
Tetracycline 500 mg BD or doxycycline 100 mg BD • Inj. atropine
for 2 weeks to 10 months. If tetracycline is not tolerated • Inj. avil
by the patient erythromycin 500 mg BD can be given • Inj. aminophylline
• Corticosteroids: Prednisolone or dexamethasone • Inj. methenamine
once at night is useful in patients with severe acne • Inj. dopamine
unresponsive to conventional therapy. • Inj. lidocaine
• Other drugs: Oral contraceptives, spironolactone, • Inj. digoxin
flutamide help in young women • Inj. propranolol
• Adjunctive therapy: Intralesional steroids for neurocystic • Inj. diazepam
lesions. Comedone extraction, chemical peels, • Inj. dilantin
dermabrasion, LASER and light therapy. • Inj. stemetil
Q.13. Write short note on SLE. (Mar 2007, 2 Marks) • Inj. isoproterenol
Ans. The full form of SLE is systemic lupus erythematosus. • Inj. pethidine
It is an autoimmune disorder characterized by the • Inj. pentazocine
destruction of tissue due to deposition of antibodies and • Inj. morphine
immune complexes within it. • Inj. analgin
• Inj. diclofenac sodium
SLE produces lesion in the skin and oral mucous
• Inj. dexamethasone
membrane and beside this it also involves certain body
• Oxygen
systems.
• Capsule nifedipine
• Inj. serpasil
Clinical Features
• Inj. glucose Or 5% dextrose saline.
♦♦ Skin lesions of SLE are characterized by the development of • Inj. calcium carbonate
fixed erythematous rashes that have a butterfly configuration • Dextrose 50%
over the malar region and across the bridge of the nose. • Aromatic salts.
218   Mastering the BDS IIIrd Year  (Last 25 Years Solved Questions)

Q.15. Write short note on sepsis syndrome.  suppressing of bacterial flora by tetracyclines
 (Apr 2007, 5 Marks) can result in super infection.
Ans. It is defined as the inflammatory response infection – Toxic effects: These effects are result due to over
in which there is fever, tachycardia, tachypenia and dosage or prolong use of drugs, for example,
evidence of inadequate blood flow to internal organs. comma by barbiturates, complete AV block by
The syndrome is the common cause of death in severely digoxin.
ill patients. – Drug habituation and dependence: Drugs capable
of altering the moods and feeling, are liable to
Etiology repetitive use to derive a feeling of euphoria to
♦♦ It is caused due to combined effect of virulent infection escape from the reality, social adjustment, etc.
and the powerful host response to the infection. – Drug withdrawal reaction: Sudden withdrawal
♦♦ Infection of lungs, abdomen and urinary tract. or stoppage of certain drugs can result in a type
♦♦ Infections at other body sites. of adverse reaction, e.g. withdrawal of beta
blockers can precipitate an effect of myocardial
Complications infarction.
- Withdrawal of phenytoin can precipitate
♦♦ Shock
status epilepticus.
♦♦ Organ failure
– Teratogenic effect: This refers to the ability of drug
♦♦ Disseminated intravascular coagulation
to cause congenital abnormality in the fetus,
♦♦ Altered mental status
when given during pregnancy, e.g. cleft palate
♦♦ Jaundice
following the use of corticosteroids.
♦♦ Metastatic abscess formation.
– Drug-induced disease or iatrogenic diseases: When
Treatment certain drugs are used chronologically, they can
Eradication of underlying cause of infection and support of produce disease, e.g. chronic use of aspirin can
failing organ systems. lead to production of peptic ulcer.
♦♦ Maintaining an open airway. • Unpredictable reactions: These are based on the
♦♦ IV fluid is given. peculiarities of the patient and not on the drug
♦♦ Adrenaline should be given to the patient. action.
♦♦ Draining or debriding abscess, if present. – Drug allergy: It is an immunologically mediated
♦♦ Heparin is given to lessen the risk of venous thrombosis. reaction producing stereotype symptoms which
♦♦ Antibiotics such as cephalosporins or penicillin are given. are unrelated to the effect of drug or its doses,
Q.16. Mention complications of drug reaction.  e.g. anaphylactic reactions resulting in urticaria,
 (Apr 2007, 5 Marks) etching, angiedema, asthma.
Ans. The term adverse drug reaction has been defined as any – Photosensitivity: It is a cutaneous reaction,
noxious which is suspected to be due to a drug occur at resulting from drug-induced sensitization of
doses normally used require treatment or decrease in the skin to UV radiation. Drugs that causes such
dose or indicates caution in future use of same drug. reactions are demeclocycline, chloroquine.
– Idiosyncrasy: It is generally determined as

Adverse effect of drug have been classified as:
abnormal reactivity to a chemical. Certain
• Predictable reaction: These are related to the adverse effects of some drugs are restricted
pharmacological effect of a drug. They include: to individuals with a particular genotype, e.g.
– Side effects: These are unwanted but often barbiturates causes excitement and mental
unavoidable pharmacodynamic effect that confusion in some person.
occurs at therapeutic doses.
- A side effect may be based on the same Q.17. Write short note on eczema. (Sep 2007, 2 Marks)
action of the drug, for example, dryness of Ans. Eczema is defined as an itchy red rash that initially weep
the mouth with atropine. or oozes serum and may become crusted, thickened or
- A side effect may be based on a different scaly.
facet of action, for example estrogen causes
nausea. Classification of Eczema
- An effect may be therapeutic in one contact ♦♦ Atopic
but side effect in another contact, for ♦♦ Seborrhoeic
example, codeine used for cough, produces ♦♦ Discoid
constipation as a side effect. ♦♦ Irritant
– Secondary effect: These are indirect consequences ♦♦ Allergic
of a primary action of a drug, for example, ♦♦ Asteatotic
Section 1:  General Medicine  219

♦♦ Gravitational Check breathing If breathing


♦♦ Lichen simplex (look, listen and feel) (Put in recovery position)
♦♦ Pompholyx. ↓
Symptoms of Eczema Breathe (To effective breaths)

♦♦ Acute symptoms:
• Redness and swelling with ill defined margins. Assess      Circulation present
• Papules, vesicles and more rending large blister. (Signs of circulation) (Continue rescue breathing,
• Exudation and cracking 10 seconds only)
• Scaling. ↓
♦♦ Chronic symptoms: No circulation (Compress chest 100/min)
• Less vascular and exudative lichenification, i.e. a
dry lathery thickening with increased skin margins, Q.19. Write short note on needle stick injury. 
secondary to rubbing and scratching  (Mar 2008, 2.5 Marks)
• Fissures and stretch marks Ans. Needle stick injuries are wounds caused by needles that
• Pigmentation change. accidentally puncture the skin.
• Needle stick injuries are a hazard for people who
Diagnostic Criteria work with hypodermic syringes and other needle
Itchy skin and at least three of the following: equipment.
♦♦ History of itch in skin creases • These injuries can occur at any time: during use,
♦♦ History of asthma/hay fever. disassembling or disposal.
♦♦ Dry skin • Needle stick injuries transmit infectious diseases,
♦♦ Visible flexural eczema. especially blood-borne viruses.
• In recent years, concern about AIDS, hepatitis B and
Management hepatitis C has prompted research to find out the
causes of these injuries and develop measures to
♦♦ General:
prevent them.
• Explanation, reassurance and encouragement
• Despite published guidelines and training programm,
• Avoidance of contact with irritants
needle stick injuries remain an ongoing problem.
• Regular use of greasy emollients
• Accidental punctures by contaminated needles can
♦♦ Medical:
inject hazardous fluids into the body through the
• Topical steroids, i.e. prednisolone and hydrocortisone
skin. There is potential for injection of hazardous
are used.
drugs, but injection of infectious fluids, especially
• Topical immunosuppressants are used.
blood, is by far the greatest concern.
• Blend emollients are used regularly both directly on
• Even small amounts of infectious fluid can spread
skin and in bath.
certain diseases effectively. Accidental injection of
• Sedative antihistaminics, i.e. alimemazine tartrate or
blood-borne viruses is the major hazard of needle
trimeprazine tartrate, if sleep is interrupted.
stick injuries, especially the viruses that cause AIDS
Q.18. Write short note on basic life support.  (HIV), hepatitis B and hepatitis C.
 (Mar 2008, 2.5 Marks) Causes
Ans. The aim of basic life support is to maintain the circulation, ♦♦ Equipment design: Safer innovative devices using protected
until more definitive treatment with advanced life needle devices or needle-free systems with self-sealing
support can be administered. ports would alleviate many of these injuries. Syringes with
• The management of the collapsed patient requires safety features reduce needle stick injuries.
prompt assessment and restoration of the airway, ♦♦ Recapping: Recapping can account for 25 to 30% of all needle
breathing and circulation (ABC) using basic life stick injuries. Often, it is the single most common cause.
support, with the aim of maintaining the circulation ♦♦ Improper disposal: Virtually all needle stick injuries are from
until more definitive treatment with advanced life needles that have either been lost in the workplace or thrown
support can be administered. into regular garbage. Janitors and garbage handlers can also
experience needle stick injuries or cuts from ’sharps’ when
Basic Life Support
handling trash that contains needles or scalpels.
Check responsiveness (Shake and shout)
↓ Management
Open airway (Head tilt and chin tilt) ♦♦ Stop all operative procedures.
↓ ♦♦ Identify and examine the wound
220   Mastering the BDS IIIrd Year  (Last 25 Years Solved Questions)

♦♦ Immediately wash but do not scrub the injury ♦♦ This type of shock is also known as neurogenic or
♦♦ Encourage bleeding psychogenic shock.
♦♦ Blood specimen for both patient and health care worker ♦♦ There is sudden pooling of blood in the capacitance vessels
is taken for specimen. of legs and splanchnic arterial bed. This causes hypoxia
Q.20. Differentiate Type A and Type B adverse drug reac- of vital center.
tion. (Mar 2008, 2.5 Marks) Clinical Features
Ans.
♦♦ History of emotional stress or pain of a sudden nature.
Type A Type B ♦♦ Bradycardia or pallor.
Adverse Drug Reaction Adverse Drug Reaction
♦♦ Tachypenia
1. These are based on pharma- 1. These are based on peculia- ♦♦ Fainting
cological properties of the rities of the patient and not on ♦♦ Reflexes are usually intact.
drug drug’s known action
2. It includes side effect, toxic 2. It includes allergy and idiosy- Management
effects and consequences of ncrasy ♦♦ Place the patient flat or in head low position.
drug withdrawal
♦♦ Ensure potency of airway
3. They are more common 3. They are less common ♦♦ IV atropine may be needed for persistent or increasing
4. These are dose related 4. These are often non dose bradycardia.
related
5. Mostly these are preventable 5. Generally more serious and
Hypovolemic Shock, i.e. Hemorrhagic Shock
drug
Management
Q.21. Describe treatment of septic shock. ♦♦ Control hemorrhage.
 (Oct 2007, 4 Marks) ♦♦ IV fluid is to restore circulating blood volume.
Ans. Septic Shock ♦♦ Blood transfusion.
This type of shock is mostly due to release of endotoxins ♦♦ Rising of foot end of bed.
in blood, which causes wide spread vasodilation of blood ♦♦ Oxygen inhalation.
vessels resulting in fall in the cardiac output. Fall in the ♦♦ If acidosis develops. Sodium bicarbonate is infused till
cardiac output is not initial feature and vasoconstriction ph is normalized.
is not observed. ♦♦ Recovery is indicated by improvement in conscious­
Bacteria responsible for release of endotoxins are E. coli, ness, dry and warm skin, CVP is more then 5 cm of H2O,
Pseudomonas proteus, etc. distended veins and increased in urinary output.
Clinical Features Q.23. Write short note on ACE inhibitors.
♦♦ Restlessness, anxiety  (Sep 2009, 4 Marks)
♦♦ Cyanosis Ans. The ACE inhibitors are Captopril, Enalapril and
♦♦ Cold and clammy skin Ramipril.
♦♦ Tachycardia
Mechanism of Action
♦♦ Oligouria or anuria
♦♦ Acidotic breathing. ♦♦ These drugs prevent conversion of angiotensin I into
angiotensin II by inhibiting angiotensin-converting enzyme
Management thus preventing action of angiotensin II, i.e. vasocons­triction
♦♦ Sedation with diazepam and aldosterone synthesis and causes fall in BP
♦♦ IV fluids ♦♦ ACE metabolizes bradykinin which is a potent vasodilator.
♦♦ Blood culture and sensitivity ♦♦ ACE inhibitors increase bradykinin levels causing
♦♦ Antimicrobial agents: Combination of penicillin or vasodilatation.
cephalosporins and aminoglycosides and metronidazole
Pharmacological Actions
♦♦ Injection hydrocortisone.
♦♦ ACE inhibitors decrease total peripheral resistance, and
Q.22. Write short note on management of shock, while doing
hence there is decrease in systolic and diastolic blood
dental surgery. (Oct 2007, 5.5 Marks)
pressure.
Ans. In such case there following shock occurs: ♦♦ ACE inhibitors increase blood supply to renal, cerebral
Neurogenic or Vasovagal Shock and coronary arteries.
♦♦ ACE inhibitors causes decrease in aldosterone synthesis,
This is response to sudden fear or severe pain and the effects and hence there is decrease in sodium and water
from slight fainting fit to death. retention.
Section 1:  General Medicine  221

Adverse Effects Q.26. Write short note on clinical uses and side effects of
prednisolone. (Mar 2011, 3 Marks)
ACE inhibitors produce hypotension, hyperkalemia, dry
persistent cough, loss of taste sensation, urticaria, angiedema, Ans. Prednisone is a synthetic corticosteroid drug that is
etc. particularly effective as an immunosuppressant drug.
It is used to treat certain inflammatory diseases (such as
Uses moderate allergic reactions) and (at higher doses) some
types of cancer, but has significant adverse effects.
♦♦ Hypertension: ACE inhibitors are first line drugs to be used
in the treatment of hypertension. They are used alone or Clinical Uses
in combination with other drugs. These drugs are more
♦♦ In asthma, chronic obstructive pulmonary disease
effective in renovascular hypertension.
( C O P D ) , c h r o n i c i n f l a m m a t o r y d e m ye l i n a t i n g
♦♦ Congestive heart failure: ACE inhibitors decreases preload
polyradiculoneuropathy (CIDP), rheumatic disorders,
and afterload in CHF.
allergic disorders, ulcerative colitis and Crohn’s disease,
♦♦ Diabetics: In diabetics, renal functions are improved.
adreno­cortical insufficiency, hypercalcemia due to cancer,
♦♦ Myocardial infarction: In myocardial infarction motility is
thyroiditis, laryngitis, severe tuberculosis, urticaria
reduced.
(hives), lipid pneumonitis, pericarditis, multiple sclerosis,
Q.24. Write short note on ABCD rule. (Nov 2011, 4 Marks) nephrotic syndrome, myasthenia gravis, and as part of a
Ans. This rule is given for diagnosis for malignant melanoma drug regimen to prevent rejection post organ transplant.
and skin cancers ♦♦ Prednisone has also been used in the treatment of migraine
• A is for ASYMMETRY: Half of a mole or birthmark headaches and cluster headaches and for severe aphthous
does not match the other. ulcer.
• B is for BORDER: The edges are irregular, ragged, ♦♦ Prednisone is used as an antitumor drug.
notched, or blurred. ♦♦ Prednisone is important in the treatment of acute lympho­
• C is for COLOR: The color is not the same all over, blastic leukemia, non-Hodgkin lymphomas, Hodgkin’s
but may have differing shades of brown or black, lymphoma, multiple myeloma and other hormone-sensitive
sometimes with patches of red, white, or blue. tumors, in combination with other anticancer drugs.
• D is for DIAMETER: The area is larger than ♦♦ Prednisone is also used for the treatment of the Herx­
6 millimeters (about ¼ inch) across, or the area has heimer reaction, which is common during the treatment of
been growing. syphilis, and to delay the onset of symptoms of Duchenne
Q.25. Write short note on statins. (Mar 2011, 4 Marks) muscular dystrophy and also for uveitis.
Ans. They are also known as HMG-CoA reductase inhibitors. ♦♦ Prednisone also could be used in the treatment of
decompensated heart failure
The statins are Lovastatin, Simvastatin, Pravastatin,
Atorvastatin, Rosuvastatin. Side-effects
The statins are the hypolipidaemic drugs. Major
Mechanism of Action ♦♦ Increased blood sugar for diabetics
♦♦ They decrease cholesterol synthesis by inhibition of ♦♦ Difficulty controlling emotion
3-hydroxy-3-methyl glutaryl coenzyme A (HMG-CoA). ♦♦ Difficulty in maintaining train of thought
♦♦ Statins also decrease low density lipids (LDL) and ♦♦ Immunosuppression
triglycerides. They increases high density lipids (HDL). ♦♦ Weight gain
♦♦ Facial swelling
Uses ♦♦ Depression, mania, psychosis, or other psychiatric symptoms
♦♦ Used in hyperlipidemia with raised LDL and total ♦♦ Unusual fatigue or weakness
cholesterol levels. ♦♦ Mental confusion / indecisiveness
♦♦ Used in secondary hypercholestrolemia caused due to ♦♦ Blurred vision
diabetes and nephritic syndrome. ♦♦ Abdominal pain
♦♦ Peptic ulcer
Adverse Effects ♦♦ Infections
♦♦ Headache, nausea, bowel upset and rashes ♦♦ Painful hips or shoulders
♦♦ Sleep disturbances ♦♦ Steroid-induced osteoporosis
♦♦ Rise in serum transaminase ♦♦ Stretch marks
♦♦ Muscle tenderness can occur ♦♦ Osteonecrosis
♦♦ Myopathy can occur. ♦♦ Insomnia
222   Mastering the BDS IIIrd Year  (Last 25 Years Solved Questions)

♦♦ Severe joint pain – Drug allergy: Allergy to any drug is independent


♦♦ Cataracts or glaucoma of its dose. These reactions are based on humoral
♦♦ Anxiety factor or cell mediated.
♦♦ Black stool – Photosensitivity: Exposure of skin to ultraviolet
♦♦ Stomach pain or bloating rays of sun results in cutaneous reactions from
♦♦ Severe swelling drug induced sensitization.
♦♦ Mouth sores or dry mouth – Teratogenecity: A drug is teratogenic when it
♦♦ Avascular necrosis harms the fetus. Such types of drugs should not
♦♦ Hepatic steatosis. be given to pregnant ladies.
Minor Q.29. Evaluation for the case of general anesthesia.
 (Nov 2008, 10 Marks)
♦♦ Nervousness
Or
♦♦ Acne
♦♦ Skin rash Write a short note on evaluation for the case of general
♦♦ Appetite gain anesthesia. (Aug 2012, 5 Marks)
♦♦ Hyperactivity Ans.
♦♦ Increased thirst
♦♦ Frequent urination History of Patient
♦♦ Diarrhea ♦♦ Detailed history of patient is taken about any major
♦♦ Reduced intestinal flora ailment in the past and recently. Confirm the type of
♦♦ Leg pain / cramps treatment given and if any residual effects of disease are
♦♦ Sensitive teeth still persisting.
Q.27. Write management of anaphylaxis. (Nov 2011, 4 Marks) ♦♦ History of alcohol intake and addiction to other narcotic
Ans. Refer to Ans 1 of same chapter. agents is taken.
♦♦ Some patients are allergic to certain drugs and general
Q.28. Write short note on drug reactions. anesthesia. It must be thoroughly ascertained, if the person
 (Apr 2008, 5 Marks) (June 2010, 5 Marks) is allergic to any of these drugs.
 (Jan 2012, 5 Marks) ♦♦ If a patient is chronic smoker, he / she is likely to have
Ans. While achieving therapeutic levels drug produce chronic cough and this has to be considered when giving
unwanted side effects or toxicity. This is known as general anesthesia.
adverse drug reaction. ♦♦ If there is presence of hypertension, the drugs being taken
• Drug reactions are divided in two types i.e. must be ascertained and recorded. Anesthetist should
predictable reactions in the form of side effects, assess degree of hypertension and its effect on the heart
overdosage, toxicity and unpredictable reactions in as well as any interaction of the anti-hypertensive drugs
form of idiosyncrasy and hypersensitivity reactions. with the anesthetic agents.
• Following are the adverse drug reactions: ♦♦ If the patient is suffering from cardiac insufficiency the
– Side effect: Undesirable effects which accompany heart must be evaluated and certain emergency drugs must
therapeutic effects of drug so does not necessitate be kept at hand during operative procedure. A detailed
stoppage of drug, e.g. Anti-cholinergics in history about all drugs the patient is on anticoagulants,
treatment of peptic ulcer cause xerostomia. digoxin, nitrates, diuretics or any other drug must be
– Toxic effect: They develop when drug is used enquired and recorded.
for the longer period in therapeutic doses, e.g.
Patient taking isoniazid for tuberculosis develop Physical Examination
peripheral neuropathy.
– Poisoning: It occurs when large doses of drug ♦♦ A detailed general physical examination must be done
are consumed for prolonged periods. preoperatively.
– Drug Intolerance: Some people are intolerant to ♦♦ Look for anaemia and state of nutrition. Patient who is
the drug in small therapeutic doses, It is due severely anemic is not fit for general anesthesia. Anemia
to low threshold of the person to the drug, e.g. must be corrected before. If emergency operation is to be
Chloroquine in usual doses cause nausea and done, blood transfusion is given.
vomiting while in an intolerant person it causes ♦♦ Pulse rate should be in between 60–80 beats per minute. If
vomiting with single tablet. the pulse rate is either high or slow, cause must be found
– Idiosyncrasy: It is an abnormal reaction to any out, and if possible it should be corrected.
drug which occur in individuals who are ♦♦ Blood pressure must be evaluated carefully. Neither high
genetically predisposed to it. or low blood pressure is desirable. An hypertensive must
Section 1:  General Medicine  223

be treated before going for general anesthesia and blood ♦♦ If urine contains considerable amount of albumin, this
pressure levels brought to normal or near normal levels. indicates renal pathology and anesthetist in consultation
But if complications are present such as cardiomegaly, with the physician must evaluate the risks and the type
renal or lung complications or features of left heart or of anesthesia.
congestive failure, then great caution has to be employed. ♦♦ Heart size must be evaluated. Position of apex beat,
♦♦ Lungs are examined for evidence of chronic bronchitis presence of abnormal pulsations over the precordium
with or without emphysema and other lung diseases. and cardiac murmurs must be assessed.
Look for clubbing, cyanosis and emphysema. Patient’s ♦♦ Look for patient veins especially over the ankles and
respiratory functions must be evaluated. A chronic smoker antecubital fossa. Suitable veins must be identified for
is likely to be at disadvantage. For this breadth holding intravenous purposes.
test is done. In such patients, it is desirable to institute Investigations
breathing exercises for at least a week before operation and
the patient is encouraged to continue with these exercises In every patient due for general anesthesia following
as soon as he regains consciousness. In this way, many investigations be carried out:
pulmonary complications are avoided. Breathing exercises ♦♦ Hemoglobin to assess anemia.
program must be carried out under the guidance of a ♦♦ Total and differential leucocyte count.
physiotherapist with advice from the physician. ♦♦ Bleeding time (BT) and clotting time (CT) for bleeding
♦♦ Abdomen should be examined for hepatomegaly, disorders.
splenomegaly, ascites and any organomegaly. Lymph ♦♦ Platelet count: Low platelet count indicates thrombocyto-
glands in the body should be palpated for evidence of any penia.
disease of reticuloendothelial system. ♦♦ Complete urine examination for albumin, sugar, ketone
♦♦ Bleeding tendencies must be looked into. History of bleed bodies and presence of any casts and cells.
from any source after any simple injury or bruising should ♦♦ Blood sugar (fasting and postprandial) to exclude diabetes.
be considered. ♦♦ Prothrombin time for bleeding disorders.
♦♦ Oral cavity must be carefully examined, and if any ♦♦ X-ray heart (PA view) for heart size: It shall also assess any
loose teeth are present or if there is any great degree of pathology in the lungs.
sepsis these should be taken care of before operation. In ♦♦ Electrocardiogram for any myocardial damage or
patients using dentures these must be removed before insufficiency or presence of any arrhythmia.
administering premedication since failure to do this may ♦♦ Pulmonary function tests to evaluate the lung functions.
choke the patient.
Patient preparation before giving general anesthesia
♦♦ Patient color should be carefully examined and if any
♦♦ On the day of surgery, patient should be empty stomach.
signs of anemia or polycythemia are present they must
be attended to. ♦♦ Little bowel enema is given to patient so that his / her
♦♦ Urine should be carefully tested, and if any evidence of bowels are empty.
diabetes is present, blood sugar estimations be done. Urine ♦♦ Breathing exercises are performed a week before.
be examined not only for sugar but also for albumin and ♦♦ Smokers should stop smoking 2 to 3 weeks before general
ketone bodies. Diabetes must be controlled before hand anesthesia for preventing pulmonary complications during
with insulin. In the presence of ketone bodies, it is usual and after anesthesia.
to institute 10 % glucose drip neutralizing with 25 units ♦♦ Sleeping pill is given to a patient in night so that proper
of soluble insulin in each bottle. This infusion is given till sleep is taken
urine is ketone free. It is preferable in any diabetic patient ♦♦ Preanesthetic medication is given which helps in induction
who is on oral antidiabetic drugs, to switch on to soluble of general anesthesia.
insulin 48 hours before operation. On the day of operation, Q.30. Write about medical emergencies in dental practice.
patient is put on 5% or 10% glucose drip neutralized with
 (Dec 2009, 10 Marks)
soluble insulin. After operation, patient has to be continued
on insulin injections till the wound has healed. This may Or
have to be carried out for 10–14 days. Once recovery is Write short note on medical emergencies in dental
complete and patient is ambulatory he can be switched practice. (Jun 2010, 5 Marks) (Aug 2012, 5 Marks)
on to oral antidiabetic drugs. Ans. Medical Emergencies in Dental Practice
224   Mastering the BDS IIIrd Year  (Last 25 Years Solved Questions)

Medical emergency Clinicalfeatures Treatment


Syncope • Feeling of giddiness • The operator should discontinue any procedure in progress,
• Weakness and lower the chair back while the patients legs are slightly
• Paleness and sweating elevated, thus placing the patient in a semi-reclining position
• This position aids venous return from the lower portions of the
body while preventing venous congestion in the upper body
• If the patient is conscious, then he should be instructed to take
a few deep breadth
• If any time a patient loses consciousness unexpectedly in the
dental chair the pulse, then respiration and color should be
checked
• If there is any significant change in the respiratory pattern
or pallor or tachycardia or bradycardia, then patients
respiration should be maintained with artificial ventilation. Give
supplemental oxygen
• If breathing is present, take some ammonia salt solution
in a cotton and hold it under the patient’s nose to retrieve
consciousness
• Escort patient to his home
Hyperventilation • Presence of giddiness • Patient is told to breathe slowly in a paper bag and rebreath
• Presence of rapid breathing the exhaled air
• Presence of numbness of finger and toes • Above method is not for diabetics
• Palpitations are present
Asthma • Wheezing is audible • In acute condition, i.e. pulse rate more than 110/min, respiratory
• Extremities remain blue rate more than 45/min, in such cases 4-metered dosage of
• Presence of breathlessness bronchodilator is indicated
• In life-threatening condition, following is the treatment
   –  High flow oxygen
   –  Salbutamol is given, i.e. one puff in a large volume
   –  Prednisolone 40–60 mg orally
   –  Puff is repeated in every 15 min
Angina pectoris • Difficulty in swallowing • Stop dental treatment
• Pulse is bounding • Sublingual glycertrinitrate Tablet ie 0.6 mg is given
• Presence of moderate-to-severe crushing • Repeat same dosage in 5min after checking blood pressure
chest pain which radiates to left arm • Repeat again after 5 min, if pain continues
• If improvement is not present after 15 min, condition now is
treated as acute myocardial infarction
Acute myocardial • Presence of moderate-to-severe crushing • Ambulance should be called
infarction chest pain which radiates to left arm which is • 100% oxygen is given
not relieved after taking glycertrinitrate • Tab Aspirin 300 mg is given in dissolved form and one
glycertrinitrate dosage Repeat another in 5 min
• Opioids are given for controlling of pain
Cardiac arrest • Presence of sudden unconsciousness • Ambulance should be called
• Breathing is not present • Cardiopulpmonary resuscitation initiated
• Nonpalpable to irregular pulse • Oxygen is given
• Early defibrillation is done
• Medical management is started with appropriate drugs
Epilepsy • Presence of sudden unconsciousness • Remove all the dental equipmen from the mouth as well as
• Presence of apnea and cyanosis remove all the equipment surrounding the patient
• Involuntary movement of body • Tight clothing should be loosened
• Restraining of patient should be avoided
• Patient should be turned in stable lateral position after the attack
is stopped
• Clear airway is maintained
• Aspiration is avoided

Contd…
Section 1:  General Medicine  225

Contd…

Medical emergency Clinical features Treatment


Hypoglycemia • Patient is confused • Administer glucose
• Behavior is altered • In cases with loss of consciousness, glucose or glucagon is
• Speech is slurred given parentrally
• Patient has features of sweating, hunger,
tremor, drowsiness
• Loss of consciousness
• There is rapid fall in blood pressure if
treatment is not done
Hyperglycemia • Presence of polydipsia and polyuria • Ambulance should be called
• Patient is dehydrated • Hydrate the patient
• Reduction in the conscious level and patient
may undergo comma and urinary cessation
Adrenal crisis • Presence of sudden loss of consciousness • Patient should be kept in supine position with raised legs
• Impalpable pulse is present • Airways should be cleared and oxygen is given
• There is rapid fall in blood pressure • Ambulance is called
• IV hydrocortisone is given
Anaphylaxis • Patient has sneezing and breathlessness • Adrenaline hydrochloride inj (1:1000) 0.3 and 0.5 mg SC/IM
• Presence of circulatory collapse and cardiac • IV fluids to correct hypotension
arrest • Glucocorticoids: Hydrocortisone hemisuccinate 100 mg IV
• Antihistaminic drugs: Chlorpheniramine 10–20 mg slow IV it is
given after adrenaline
• Bronchodilators:
   – Salbutamol or IV aminophylline is given in patients with
bronchospasm
   –  Supportive therapy with oxygen or assisted ventilation

Q.31. Write short note on anaphylactic shock with diagnosis Management


and management. (Jan 2012, 5 Marks)
Refer to Ans 1 of same chapter.
Ans. Anaphylaxis is defined as a state of rapid developing
immune response to an antigen mediated by IgE Q.32. Discuss the clinical presentation and management of
antibodies. anaphylactic shock. (Aug 2012, 12 Marks)
Ans. Clinical Presentation
Diagnosis
• Onset may be instantaneous or within a few minutes
Diagnosis is based on physical examination and investigations. after the IV injection and 30 minutes after exposure.
• Cardiovascular: Tachycardia, arrhythmia, hypotension
Physical Examination and circulatory collapse.
♦♦ Presence of immediate respiration difficulty, laryngeal • Respiratory: Laryngeal obstruction, angiedema,
edema with stridor and wheeze is audible. bronchospasm or pulmonary edema may occur
♦♦ Wheal and erythematous lesions are seen which are singly or in combination.
circumscribed, round, discrete, erythematous areas with • Nervous: Syncope and seizures.
irregular borders and blanched centres. • GIT: Diaphoresis, abdominal pain and diarrhea may
♦♦ Lesions are very pruritic. occur.
• Skin: Wheel and erythematous lesions are seen which
Investigations are circumscribed, round, discrete, erythematous
♦♦ Skin-Prick test: When an allergen is injected into the areas with irregular borders and blanched centres.
skin and is observed for the development of wheal and • Lesions are very pruritic.
induration within 15 minutes. There is a good correlation • If not treated death may occur.
between diameter of wheal and levels of specific IgE
antibodies in serum. Management
♦♦ In vitro radioallergosorbent test (RAST) is done to Refer to Ans 1 of same chapter.
detect allergen specific IgE for a variety of food, insect
bites / stings, venom, latex and some drugs. Q.33. Write short note on aspirin. (Feb 2013, 5 Marks)
♦♦ Serum tryptase levels are done for detection of recent Ans. Chemical name of aspirin is acetyl salicylic acid.
anaphylaxis.
226   Mastering the BDS IIIrd Year  (Last 25 Years Solved Questions)

Pharmacological Action ♦♦ It causes respiratory depression.


♦♦ It decreases the temperature by bringing temperature Q.34. Write about: (Apr 2015, 2+2+2+2+2 Marks)
regulating centre to normal when it becomes deranged. a. What is anaphylaxis
♦♦ Aspirin block pain centre in thalamus. It also inhibits b. Write etiology
synthesis of prostaglandins and prevent sensation c. Causes
of pain receptors to histamine, bradykinin and d. Symptoms and Signs
5 hydroxytryptamine, mediators of pain and inflammation. e. Treatment
♦♦ Aspirin inhibits platelet aggregation by inhibiting ADP Ans. a.  Anaphylaxis is defined as state of rapid developing
release from platelets and inhibiting the synthesis of immune response to an antigen mediated by IgE
prostaglandin, endoperoxidase and thromboxane A2. antibodies.
Indications b. Etiology: For details refer to Ans 1 of same chapter.
c. Causes: For details refer to Ans 1 of same chapter.
♦♦ It acts as keratolytic, fungistatic and mild antiseptic. d. Symptoms and signs: For details refer to Ans 1 of same
♦♦ As analgesic, antipyretic and antirheumatic. chapter.
♦♦ It prevents platelet aggregation. e. Treatment: For details refer to Ans 1 of same
Side Effects chapter.

♦♦ Nausea and vomiting Q.35. Tabulate the differences, clinical features and
♦♦ Increases prothrombin time. management of anaphylactic shock and vagal shock.
♦♦ Fatty infiltration of liver and kidney.  (June 2015, 12 Marks)
♦♦ Salicylism: Headache, dizziness, vertigo, tinnitus, Ans. Tabulation of differences between anaphylactic shock
diminished hearing and vision. and vagal shock.

Sign and symptoms Anaphylactic shock Vagal shock


Interval (after injection) Sometimes before, usually after a few seconds to a Within 30 minutes after injection; the most severe reactions begin
few minutes after the injection with the first 15 minutes
Consciousness Fainting sensation, dizziness, Loss of consciousness Anxiety, which may progress to unconsciousness in severe cases
Breathing Slow, with a few seconds of Respiratory difficulties; coughing, sneezing, wheezing, stridor
apnea in some cases
Pulse Slow and weak, but regular Rapid, weak and irregular
Skin Diaphoresis, clammy skin, pallor • Warm skin, progressing to clammy and pallor
• Pruritis and urticaria
• Swelling of face and tongue
Blood Pressure Hypotension Hypotension (systolic pressure <90 mm Hg), which may progress
to cardiovascular collapse
Gastro-Intestinal System Nausea and vomiting Nausea, vomiting, abdominal pains, diarrhea
Management Anaphylactic shock Vagal shock
• Airways: Intubation or perform tracheostomy if • Place patient in a recumbent position and elevate legs
intubation is not possible above head
• Oxygen is given in high concentration • Ventilate the room well
• Injection adrenaline 0.5 to 1 mL of 1:1000 • Place cold, damp cloth on face
solution IM • Give reassurance
• Fluids: 0.5 or 1L of fluid restores blood pressure
but 6–9 L is required for adequate restoration of
blood volume.
• Hydrocortisone 300 mg IV
• Aminophylline 250 mg IV in 20 mL dextrose to
reveal broncheospasm
• Diphenhydramine 50 mg IV slowly
• Dopamine, if hypotension persists
• IV atropine and glucagon
• Ventilatory support is necessary, if patient is
critical
Section 1:  General Medicine  227

Q.36. Write down and discuss six common comorbid diseases Seizure
where you should take medical consultation before
Seizure disorder is caused by an electrical disturbance in
dental extraction. (June 2015, 12 Marks)
the brain. Any person can experience a seizure in a stressful
Ans. Comorbidity is the presence of one or more additional
situation and approximately half of all seizures are considered
disorders or diseases co-occurring with a primary
idiopathic. Epilepsy is a condition commonly associated with
disease or disorder; or the effect of such additional
a syndrome of associated seizure types.
disorders or diseases. The additional disorder may also
When a known epileptic patient is scheduled for treatment,
be a behavioral or mental disorder.
the dental team should determine, if the patient has taken
Following are the six comorbid diseases for which his / her antiseizure medication. These patients should be
one should take medical consultation before dental scheduled for short appointments, when they are well rested.
extraction: Patients often report an aura before experiencing a seizure. This
1. Diabetes mellitus aura can be a sound, feeling, or smell. Shining a bright light in
2. Seizure the patient’s eyes should be avoided, since this has been known
3. AIDS to trigger a seizure.
4. Cardiovascular risks Patients who are taking oral anticonvulsant medications,
5. Respiratory disorders such as Dilantin, Zarontin, or Depacon, may experience gingival
6. Blood dyscrasias. hyperplasia. Emphasis must be placed on meticulous home care
Diabetes Mellitus to prevent serious periodontal problems and the increased need
for regular dental visits.
There are three types of diabetes: Type I is considered insulin
dependent, 5–10% of cases; Type II is when the body does not Cardiovascular Risks
produce enough insulin and, therefore diet and medication may Patients who state that they have experienced chest pain,
be regulated; Gestational diabetes occurs during pregnancy and shortness of breath, pain that radiates down their neck or arm,
is reversed after delivery. If the patient indicates that they have have swollen ankles, and / or have high blood pressure are at
diabetes, the type must be noted in the history. risk for cardiovascular disease. They may or may not have
Patients with uncontrolled diabetes have low resistance been diagnosed by a physician. If this patient also indicates
to infection and are prone to periodontal disease. They have that they are a smoker and / or are overweight, the risk is
poor healing response, including excessive bleeding, and may increased. It is recommended that a medical consultation or
experience hypoglycemia or hyperglycemia during dental evaluation be obtained for all patients who indicate some type
treatment. Patients who are undiagnosed diabetics may report of cardiovascular disease on their health histories. Additionally,
the following symptoms: excessive thirst and hunger, increased medical consultations should also be obtained for patients who
urination, and higher birth weights in babies. are reporting or exhibiting symptoms of cardiovascular disease,
When reviewing a health history, the following questions but have not had a definitive diagnosis. Preventing medical
should be asked of diabetic patients: crises during dental treatment is the best method of protecting
♦♦ Is your diabetes well controlled? patients who are medically compromised.
♦♦ When is the last time your blood glucose was checked? Rheumatic heart disease is a result of rheumatic fever and
♦♦ Are you currently taking oral anti-diabetic medications can cause deformities in the heart valves. Some patients will
or insulin injections? report taking antibiotics on a regular basis, although this is not
♦♦ When was the last time you ate? a required practice. If the patient reports a history of rheumatic
♦♦ Did you take your medication on time today? fever, the dental practice must consult with the patient’s
It is advisable to keep sources of sugar on hand for hyper physician to determine the extent of the heart involvement. This
or hypoglycemic episodes. Glucose gel from a pharmacy, assessment will determine the need for antibiotic prophylaxis.
granulated sugar packets, honey packets, or tubes of cake icing Congenital heart defects place the patient at high risk for
can be easily stored and used in the case of an emergency. bacterial endocarditis. Again, a statement from a physician is
When questioning diabetic patients before treatment and advisable to determine the type of defect and to verify the need
you find out they have not eaten or taken their insulin, their for prophylactic antibiotics.
appointment should be rescheduled. Make sure they understand Coronary artery disease results from atherosclerosis. Patients
the importance of these steps to maintain a normal blood sugar may report having been diagnosed with angina pectoris, or
level during dental treatment. After approximately 8 hours having chest pain after some type of physical activity or stress.
of fasting, an average range is between 70–99 mg / dL. If they These patients may be taking several medications, particularly
have eaten an average level should be less that 140 mg / DL. A vasodilators such as nitroglycerin; or beta-blockers, such as
strategy for making a dental appointment for a diabetic patient propanolol. The stress of a dental visit may cause an angina
is to keep it short and in the early to mid-morning hours when attack, therefore it is important to minimize patient stress,
sugar levels are more stable. maximize patient comfort and make certain that the patient
228   Mastering the BDS IIIrd Year  (Last 25 Years Solved Questions)

has his / her medications available, particularly nitroglycerin. patient should be able to produce the medication or inhaler just
In addition, it is recommended that local anesthetic without in case it is needed.
epinephrine or other vasoconstrictors be used for these patients,
to avoid further constriction of the blood vessels. Patients with Chronic Obstructive Pulmonary Disease (COPD)
unstable or uncontrolled angina should wait for at least 30 days The two most common forms of COPD are emphysema and
after their angina is stabilized to receive dental treatment. chronic bronchitis. Emphysema is the irreversible enlargement
Myocardial infarction is more commonly known as a heart of the air sacs in the lungs making it hard to expel all oxygen
attack. Patients who have experienced a recent myocardial during breathing. Chronic bronchitis is characterized as the
infarction should not receive any dental treatment for a irreversible condition of narrowed airways. Like patients with
minimum of 6 months after the heart attack. Most fatalities from heart failure, those with either of these types of COPD may not
myocardial infarction occur within 3–4 months after the attack. be able to breathe easily in a supine position or after walking
Stressful situations, like dental treatment can cause a rupture certain distances. Due to prolonged steroid use to treat their
in the area of infarct, leading to further health problems and COPD, these patients may have Candida infections. Before dental
possible death. extraction medical consultation of such patients is necessary.
Hypertension is a common condition characterized by high Tuberculosis (TB) is a bacterial infection that occurs primarily
blood pressure. Patients with uncontrolled hypertension are at in the lungs, but can occur in other organ systems in the body. It
risk for a stroke, kidney failure, or heart attack. Since stress will is spread through airborne particles, when an infectious patient
increase blood pressure, dental treatment is contraindicated in coughs, sneezes, talks, or sings. TB is most easily spread in
patients with severe hypertension. Patients presenting with a small confined spaces where infectious people share the same
systolic pressure between 120–139 mm hg should be advised to airspace with others. Because TB is highly infectious and its
monitor their readings for improvement and see their physician airborne transmission is difficult to control, CDC has issued
for assessment. As a rule, patients with a systolic pressure specific guidelines to help protect healthcare workers when
greater than 160 mm hg and / or a diastolic pressure greater treating patients.
than 95 mm hg should be referred for medical consultation In the case of TB, these guidelines state that elective, (non-
and treatment, dental treatment should be delayed. emergency) dental treatment should be postponed for patients
It is relatively common for patients to have undiagnosed who have or are suspected of having an active, infectious TB
hypertension, making it very important for the dental team case. If these patients can be referred for treatment in a facility
to monitor patient’s vital signs during their dental visits. that is specifically designed to treat TB patients (e.g. a hospital
A patient who reports having frequent dizziness, nosebleeds, or setting), it is acceptable for the dental practice to make this
headaches may have high blood pressure and may be in need referral and not be liable for discriminatory practices.
of medical intervention with antihypertensive drugs and / or In addition to asking patients on their health history if they
diuretics. have had TB, the patients should also be asked if they have
Heart failure was more commonly known as “congestive experienced any of the following symptoms: night sweats,
heart failure” but the American Heart Association has recently unexplained fever, weight loss, or a prolonged or bloody cough.
renamed and shortened the title to characterize a broader If a patient has experienced any of these symptoms, refer them
spectrum of the disease. Depending on the type of heart failure, to a physician for evaluation before proceeding with anything
patients will typically have swollen ankles and shortness of but emergency palliative treatment.
breath, due to poor circulation and fluids backing up in the If emergency treatment must be performed on a patient
lungs. These patients may indicate that they must sleep upright suspected of having infectious TB, the following precautions
or with several pillows. In these cases, placing them in a supine should be taken:
position for dental treatment may cause them acute distress. ♦♦ Schedule the patient at the end of the day, as the last
Heart failure patients are usually taking one or more diuretic patient treated.
medications to remove fluid. Supplemental oxygen may be ♦♦ All team members present in the treatment room must
needed for these patients and use of nitrous oxide analgesia is wear a HEPA or NIOSH N95 mask.
contraindicated. ♦♦ High volume evacuation and dental dam isolation must
be used to reduce aerosols.
Respiratory Disorders
AIDS
Asthma is a chronic respiratory disorder that results in a
narrowing of airways. An asthma attack can be triggered in AIDS is a viral infection that impairs a patient’s immune
several ways including environmental allergens, medications, system, making the patient highly susceptible to other
or exercise. Patients with asthma may be prone to an attack or infectious diseases. It is transmitted through blood and other
episode brought on by the stress of dental treatment, or exposure body fluids, which puts dental healthcare workers at some
to an allergen-like latex. The patient must always be instructed risk of infection from treating patients. Again, the practice of
to bring his / her medication / inhaler to each appointment. When standard precautions must be followed, since many patients
the health history is reviewed at the beginning of treatment, the will not disclose their HIV positive status for fear of humiliation
Section 1:  General Medicine  229

or rejection. Remember that information about a patient’s prior to treatment. Aspirin must never be prescribed for pain
HIV positive status is highly sensitive and must be protected. control for these patients, since it is a natural blood thinner and
The dental team must take every precaution to protect the increases bleeding.
confidentiality of that patient’s health history.
Q.37. Write short answer on shock. (Apr 2018, 3 Marks)
Although some dental health care workers are fearful of Ans. Shock is defined as an acute clinical syndrome
treating AIDS patients, it is illegal not to accept them into a dental characterized by a significant, systemic reduction
practice or to refer them to another practice, unless the type of in tissue perfusion, resulting in decreased tissue
treatment is not performed in that practice. For example, if a oxygen delivery and insufficient removal of cellular
general dental practice does not perform periodontal surgeries, metabolic products, resulting in tissue injury and severe
it is legal to refer HIV positive or AIDS patients to a periodontist, dysfunction of vital organs.
since non-HIV or non-AIDS patients are referred as well. It can occur either because the function of the heart itself is
However, if only the HIV positive or AIDS patients are referred impaired, or because heart is inadequately filled.
for periodontal surgery, this would be considered discriminatory.
Classification of Shock
When treating patients with full blown AIDS, who are
typically immunosuppressed, the dental team should take extra Following is the classification of shock:
precautions to protect the patient from opportunistic infections. ♦♦ Hypovolemic shock
These may include wearing sterile surgical gloves rather than ♦♦ Cardiogenic shock
non-sterile exam gloves, having the patient use a pre-treatment ♦♦ Distributive shock:
rinse of chlorhexidine gluconate or other mouthwash to prevent • Septic shock
bacteremia, and using only sterile water for irrigation rather • Anaphylactic shock
than from the air / water syringe, which may contain some • Neurogenic shock
bacterial contamination. ♦♦ Obstructive shock.

Blood Dyscrasias Clinical features

Anemia is a deficiency of red blood cells, caused by vitamin or ♦♦ Cold clammy skin, profuse sweating
iron deficiency or bone marrow problems. An anemic patient ♦♦ Hypotension (systolic BP <100 mmHg)
may have problems with slow wound healing and excessive ♦♦ Tachycardia with thready pulse
bleeding. They may report feeling weak and fatigued, and may ♦♦ Rapid, shallow respiration
appear very pale. If a patient exhibits these symptoms a medical ♦♦ Restlessness, drowsiness, confusion
evaluation should be recommended before dental extraction. ♦♦ Oliguria, may progress to anuria
♦♦ Jugular venous pressure elevated in cardiogenic shock,
Leukemia is a type of blood cancer, where there is an overgrowth
reduced in hypovolaemic and anaphylactic shock, variable
of white blood cells. These white blood cells may displace red
in septic shock
blood cells, used to transport oxygen in the blood. These
♦♦ Multiorgan failure
patients may exhibit oral signs, often before other symptoms
of their disease. These signs typically include excessive gingival Stages of Shock
irritation in the absence of other causative agents, such as heavy
plaque or calculus. Stage l: Stage of compensatory shock—by neuroendocrine
response to maintain the perfusion of the vital organs like brain,
Patients with leukemia are very prone to infections, including
heart, kidney, liver.
periodontal infections. In addition, the chemotherapeutic agents
used to treat the disease have many side effects, most notably Stage 2: Stage of decompensatory shock—where there is
xerostomia (dry mouth). Consult the treating physician before progressive shock causing persistent shock with severe
proceeding with dental treatment. hypotension (with mean arterial pressure <65 mm Hg); oliguria,
tachycardia.
Hemorrhagic disorders are ailments in which patients experience
excessive bleeding, due to a deficiency of clotting factors in Stage 3: Stage of irreversible shock - with severe hypoxia and
their blood. Common bleeding disorders are hemophilia A Multi organ dysfunction syndrome (MODS).
and B, factor II, V, VII, X, XII, and von Willebrand’s disease.
A patient with one of these disorders will bruise very easily, Causes of Shock
may experience spontaneous, excessive bleeding, including
Hypovolemic shock
unprovoked epistaxis (bleeding from the nose).
Dental treatment, such as extractions, that may cause ♦♦ Due to reduction in total blood volume. It may be due to:
bleeding can be risky for these patients. Close monitoring by • Hemorrhage:
the physician is necessary before dental extraction. Extraction –– External from wounds, open fractures
should be confined to specific areas (e.g. one tooth or quadrant –– Internal from injury to spleen, liver, mesentery
at a time) and transfusion with clotting factors may be necessary or pelvis.
230   Mastering the BDS IIIrd Year  (Last 25 Years Solved Questions)

• Severe burns, which results in loss of plasma ♦♦ Serum lactate estimation is an important prognostic factor.
• Peritonitis, intestinal obstruction Level >2 mEq/L suggest tissue ischemia.
• Vomiting and diarrhea of any cause ♦♦ USG of a part, CT/MRI of the location of pathology
♦♦ Cardiac causes: of standard focus should be done; often may require
• Acute myocardial infarction, acute carditis repeti¬tion of these imaging to assess progress.
• Acute pulmonary embolism wherein embolus blocks ♦♦ Blood urea, serum creatinine, liver function tests,
the pulmonary artery at bifurcation or one of the prothrombin time (PT), activated partial thromboplastin
major branches time (APTT), ECG monitoring are also should be done.
• Drug induced ♦♦ All these tests including platelet count and arterial blood
• Toxemia of any causes gas (ABG) should be repeated at regular intervals.
• Cardiac surgical conditions like valvular diseases,
congenital heart diseases Treatment of Shock
• Cardiac compression causes: ♦♦ Treat the cause, e.g. arrest hemorrhage, drain pus.
–– Cardiac tamponade due to collection of blood, ♦♦ Fluid replacement: Plasma, normal saline, dextrose,
pus, fluid in the pericardial space which prevents Ringer’s lactate, plasma expander (haemaccel). Dosage
the heart to expand leading to shock. is maximum l liter can be given in 24 hours. Initially
–– Trauma to heart. crystalloids then colloids are given. Blood transfusion is
♦♦ Septic shock—is due to bacterial infections which release done whenever required.
toxins leading to shock. ♦♦ Ionotropic agents: Dopamine, dobutamine, adrenaline
♦♦ Neurogenic shock—due to sudden anxious or painful infusions—mainly in distributive shock like septic shock.
stimuli causing severe splanchnic vessel vasodilatation. ♦♦ Correction of acid-base balance: Acidosis is corrected by
Here patient either goes for cardiac arrest and dies using 8.4% sodium bicarbonate intravenously.
or recovers fully spontaneously—spinal cord injury/ ♦♦ Steroid is often lifesaving. 500–l000 mg of hydrocortisone
anaesthesia can cause neurogenic shock. can be given. It improves the perfusion, reduces the
♦♦ Anaphylactic shock—is due to type I hypersensitivity capil¬lary leakage and systemic inflammatory effects.
reaction ♦♦ Antibiotics in patients with sepsis; proper control of blood
♦♦ Respiratory causes: sugar and ketosis in diabetic patients.
• Atelectasis (collapse) of lung ♦♦ Catheterization to measure urine output (30–50 mL/hour
• Thoracic injuries or > 0.5 mL/kg/hour should be maintained).
• Tension pneumothorax ♦♦ Nasal oxygen to improve oxygenation or ventilator
• Anesthetic complications. support with intensive care unit monitoring has to be done.
♦♦ Other causes: ♦♦ Central venous pressure line to perfuse adequately and to
• Acute adrenal insufficiency (Addison’s disease) monitor fluid balance. Total parentral nutrition is given
• Myxedema when required.
♦♦ Pulmonary capillary wedge pressure to monitor very
Investigations and Monitoring of Shock critical patient.
♦♦ Regular monitoring with blood pressure, pulse, heart ♦♦ Hemodialysis may be necessary when kidneys are not
rate, respiratory rate, urine output measurement (hourly) functioning.
should be done. Urine output should be more than 0.5 ml/ ♦♦ Control pain—using morphine (4 mg IV).
kg/hour. Pulse oximetry should be used. ♦♦ Ventilator and ICU/critical care management.
♦♦ Central venous pressure (CVP), pulmonary capillary ♦♦ Injection ranitidine IV or omeprazole IV or pantoprazole
wedge pressure (PCWP—an accurate assessment of left IV.
ventricular/function) monitoring should be done. ICU care ♦♦ Activated protein even though costly is beneficial as it
is needed during monitor period. But both CVP and PCWP prevents the release and action of inflammatory response.
are not accurate method of assessing tissue perfusion. ♦♦ MAST(Military Anti-shock Trouser ) provides
♦♦ Complete blood count, ESR, pH assessment, serum circumferen¬tial external pressure of 40 mm Hg. lt is
electrolyte estimation, chest X-ray (to rule out acute wrapped around lower limbs and abdomen, and inflated
respiratory distress syndrome/pulmonary problems). with required pres¬sure. It redistributes the existing blood
♦♦ Pus/urine/blood/bile/sputum cultures depending on the and fluid towards center. It should be deflated carefully
focus and need in sepsis. and gradually.
Section 1:  General Medicine  231

MULTIPLE CHOICE QUESTIONS


As per Dci and Examination Papers 1 Mark Each
of Various Universities

1. Migraine is precipitated by: 10. Mumps is caused by:


a. Bacteria a. Bacteria
b. Fungus b. Virus
c. Flash of light c. Parasite
2. Drug of choice in P. falciparum malaria is: 11. Which valve is least affected in rheumatic fever?
a. Choloroquine a. Mitral valve
b. Primaquine b. Aortic valve
c. Artemether c. Pulmonary valve
3. Deficiency of vitamin D causes: d. Tricuspid valve
12. True about autonomic neuropathy are all except:
a. Increased thickness of bones
a. Resting tachycardia
b. Osteoporosis
b. Silent MI
c. Diarrhea
c. Orthostatic hypotension
4. Lymphoma is type of:
d. Bradycardia
a. Disease of lymphatic vessels
13. Man takes peanut and develops stridor neck swelling,
b. Disease of lymph nodes and spleen tongue swelling and hoarseness of voice, most
c. Causes increased neutrophils probable diagnosis is:
5. Factor VIII deficiency causes: a. Foreign body bronchus
a. Anemia b. Parapharyngeal abscess
b. Hemophilia c. Foreign body larynx
c. Clotting disorder d. Angioneurotic edema
6. In acute nephritis urine contains: 14. The amino acid associated with atherosclerosis:
a. RBCs a. Lysine
b. Factor VIII b. Cystein
c. None of A and B c. Homocysteine
7. Vitamin B12 is: d. Alanine
a. Fat soluble 15. A 45-year-old female patient suffering from T2DM
with hypertension, which of the following anti-
b. Water soluble hypertensive should not be used?
c. None of a and b a. Lisinopril
8. Chronic hepatitis is caused by: b. Losartan
a. Hepatitis B virus c. Thiazide
b. Hepatitis C virus d. Trandolapril
c. Both of a and b 16. In stable angina;
9. Vitamin K helps: a. CK-MB is elevated
a. To stop bleeding b. Troponin T and I is elevated
b. To stop clotting c. All is elevated
c. None of a and b d. Nothing is elevated

Answers: 1. c 2. c 3. b 4. b
5. b 6. a 7. b 8. c
9. b 10. b 11. c 12. d
13. d 14. c 15. a 16. d
232   Mastering the BDS IIIrd Year  (Last 25 Years Solved Questions)

17. An obese patient is presented in causality in an 24. Palatal palsy is a complication of:
unconscious state. His blood sugar is 400mg% urine a. Syphilis
tested positive for sugar and ketone bodies. Drug most b. Rubella
useful in management is:
c. Diphtheria
a. Glibenclamide
d. Mumps
b. Pioglitazone
25. All are clinical features of cirrhosis of liver expect:
c. Miglitol
a. Increased libido
d. Insulin
b. Ascites
18. All are features of Cushing syndrome except:
c. Jaundice
a. Central obesity
d. Palmar erythema
b. Glucose intolerance
c. Episodic hypertension 26. Which of the following is not characteristic of
congenital syphilis?
d. Easy bruising
a. Interstitial keratitis
19. The best marker to diagnose thyroid-related disorder is:
b. Mulberry molars
a. T3
c. Notched incisors
b. T4
c. TSH d. Ghon‘s complex
d. Thyroglobin 27. Macrocytic anaemia is due to deficiency of:
20. A 5-year-old girl always have to wear worn socks even is a. B1
summer season, on physical examination it was noticed b. B2
that she had high blood pressure and her femorals are c. B6
weak as compared to radials and carotid pulse. Chest d. B12
X-ray showed remarkable notching of ribs along with
28. The Best test for assessment of iron status is:
lower borders. This was due to:
a. Transferrin
a. Femoral artery thrombosis
b. Plasma ferritin
b. Reynaud’s disease
c. Serum iron
c. Co-arctation of aorta
d. Hemoglobin
d. Takayasu’s arteritis
21. The most important diagnostic method for enteric fever 29. Thiamin deficiency causes:
is: a. Ophthalmoplegia
a. Widal test b. Cardiomyopathy
b. Blood culture c. Peripheral neuropathy
c. X-ray abdomen d. All the above
d. Ultrasonography of abdomen 30. “ CD4 count” term is used with which disease:
22. Peptic ulceration is strongly associated with: a. Hepatitis B
a. Family history b. AIDS
b. Irregular dietary habits c. Pernicious anemia
c. Hurry, worry and curry d. Falciparum malaria.
d. Helicobacter pylori infection 31. Which of the following drug is used in the treatment
23. All the following can cause acute gastritis except: of hyperkalemia in acute renal failure?
a. Anti-hypertensives a. Amlodipine
b. Non steroidal anti-inflammatory drugs b. Captopril
c. Salicylates c. Insulin
d. Corticosteroids d. Atenolol

Answers: 17. a 18. c 19. c 20. c


21. a 22. d 23. d 24. c
25. a 26. d 27. d 28. a
29. d 30. b 31. c
Section 1:  General Medicine  233

32. A boy presents with headache, vomiting, fever, neck a. Normal


stiffness the most important investigation is: b. Shuffling
a. Complete blood counts c. Hemiplegic
b. X-ray skull d. Drunken
c. CSF Examination 39. In thalassemia, peripheral smear for RBC morphology
d. MRI Brain shows:
33. In cardiology department, RMO makes a diagnosis a. Normochromic normocytic
of mitral stenosis. The most important sign on which b. Sickle cells
diagnosis is based is: c. Hypochromic microcytic
a. Ejection systolic murmur d. Macrocytosis
b. Mid diastolic murmur 40. A patient of thromboembolic stroke is taking warfarin.
c. Pansystolic murmur He comes to dental OPD for tooth extraction. The most
d. Third heart sound useful investigation to see effect of warfarin is:
34. A 15-year-old boy who is IDDM presents with pain in a. Hematocrit
abdomen, vomiting and shortness of breath. He also b. Platelet count
gives history of fever and sore throat two days back. c. Prothrombin time
The most likely cause of his symptoms:
d. Bleeding time
a. Renal failure
41. Hepatitis B spreads through:
b. Gastritis
a. Blood transfusion
c. Diabetic ketoacidosis
b. Alcohol consumption
d. Non-ketotic hyperosmolar coma
c. Faeco-oral route
35. A patient of pulmonary tuberculosis is taking ATT,
d. Droplet infection
presents with joint pain and raised uric acid level. The
most likely cause of symptoms is side effect of: 42. Following are the causes of cirrhosis of liver except:
a. INH a. Alcoholism
b. Pyrazinamide b. Wilson’s disease
c. Streptomycin c. Hepatitis E
d. Ethambutol d. Hemochromatosis
36. The most important investigation to confirm diagnosis 43. Chronic diarrhea is a feature of infection by:
of bronchial asthma is: a. Bacillus cereus
a. X-ray chest b. Cholera
b. Lung function test c. Giardiasis
c. Serum IgE level d. Camphylobacter
d. CBC 44. Koilonychia is a feature of:
37. A 18-year-old boy with hypertension, examination of a. Vitamin B12 deficiency
CVS reveals radiofemoral delay. The most likely cause b. Iron deficiency
of hypertension in this patient is: c. Protein deficiency
a. Coarctation of aorta d. Folic acid deficiency
b. Conn‘s syndrome 45. Schilling’s test is performed to detect deficiency of:
c. Renal artery stenosis a. Vitamin B1
d. Diabetic nephropathy b. Vitamin B2
38. Gait of a patient suffering from parkinsonism is likely c. Vitamin B6
to be: d. Vitamin B12

Answers: 32. c 33. b 34. c 35. b


36. c 37. a 38. b 39. c
40. c 41. a 42. c 43. a
44. b 45. d
234   Mastering the BDS IIIrd Year  (Last 25 Years Solved Questions)

46. Exanthematous fever characterized by vesicular erup- a. Hyperreflexia


tion is: b. Hypotonia
a. Measles c. Ataxia
b. Mumps d. Delayed relaxation of ankle jerk
c. Rubella 54. In a patient of centripital obesity, acne and hirsutism
d. Varicella the most likely diagnosis is:
47. Cerebral malaria is caused by: a. Hypogonadism
a. P. Falciparum b. Simple obesity
b. P. vivax
c. Hypothyroidism
c. P. malariae
d. Cushing’s syndrome
d. P. ovale
55. An epileptic girl is having gingival hypertrophy in
48. The most common mode of transmission of HIV
dental OPD. Antiepileptic drug which she is most
worldwide is:
likely taking is:
a. Mother to fetus
a. Na valproate
b. Injection drug use
b. Phenytoin
c. Homosexuals
c. Carbamazepine
d. Heterosexuals
d. Gabapentin
49. Intrinsic factor is secreted by:
56. A 50-year-old man is admitted in ICCU with acute
a. Parietal cells
myocardial infarction. Which of the following drugs
b. Chief cells
is used in reperfusion therapy?
c. Mucous cells
a. Warfarin
d. Endocrine cells
b. Aspirin
50. Following are vitamin K dependent clotting factors
except: c. Streptokinase
a. Factor VII d. Heparin
b. Factor VIII 57. A 18 years boy present with chronic diarrhea. Which
c. Factor IX of the following features suggests that he has irritable
bowel syndrome:
d. Factor X
51. A 40 year male present in emergency department with a. Anemia
central chest pain. His BP is 100/60 mm of Hg and pulse b. Abdominal pain relieved by defecation
is 110 BPM, low volume. He is pale and sweating pro- c. Nocturnal symptoms
fusely. The most likely diagnosis is: d. Blood in stool
a. Esophagitis 58. In a patient with history of muscle cramps and carpo-
b. Myocardial infarction pedal spasm. Which of the following serum electrolyte
c. Pleural effusion level is likely to be low?
d. Pneumothorax a. Calcium
52. In rheumatic fever, the treatment of choice for symp- b. Sodium
toms is:
c. Chloride
a. Aspirin
d. Potassium
b. Paracetamol
59. The most common side effect of quinine is:
c. Morphine
a. Coma
d. Ibuprofen
b. Deafness
53. A 40 year old female presents with the cold intolerance,
swelling all over body, hoarseness of voice and goiter. c. Headache
The most likely finding on CNS examination is: d. Tremors

Answers: 46. a 47. a 48. d 49. a


50. b 51. b 52. a 53. d
54. d 55. b 56. c 57. b
58. a 59. b
Section 1:  General Medicine  235

60. A young boy presents with fever, skin rash and diar- a. It is caused by Mycobacterium
rhea. Examination of oral cavity shows Koplik spots b. Incubation period is under a week
on mucosa. The most likely diagnosis is: c. The red macule is eroded to form an indurated
a. Measles painful ulcer
b. Chickenpox d. It resolves within 2-6 weeks without treatment
c. Smallpox 68. Long-term treatment with overdoses of corticosteroids
d. Typhoid may result in all the following except:
61. One of the following is true about herpes zoster: a. Osteoporosis
a. Itching vesicles appear around lips b. Diabetes mellitus
b. Systemic viral infection causes vesicles around penile c. Blood dyscrasias
area
d. Susceptibility to infection
c. Burning discomfort occurs in affected dermatome
69. Hepatitis A is spreaded by:
where discrete vesicles appear 3–4 days later
d. Vesicular eruptions begin on mucosal surface first a. Feco-oral route
followed by centripetal distribution b. Vertical transmission
62. Pernicious anemia is due to: c. Blood transfusion
a. Gastric atrophy d. Droplet infection
b. Vitamin B1 deficiency 70. Treatment of angular stomatitis and cheilosis is:
c. Vitamin B12 deficiency a. Pyridoxine
d. Folic acid deficiency b. Riboflavin
63. Deficiency of coagulation factor IX is associated with: c. Cyanocobalamin
a. Hemophilia A d. Vitamin C
b. Hemophilia B 71. In emergency, treatment of choice for anaphylactic
c. Henoch-schönlein purpura shock is:
d. All the above a. Ampicillin
64. Bull neck in diphtheria is due to: b. Adrenaline
a. Cellulitis c. Amiodarone
b. Laryngeal edema 72. Which parameter can be used to monitor severity of
c. Retropharyngeal abscess bronchial asthma at home?
d. Lymphadenopathy a. ECG
65. Spider nevi, palmar erythema, Gynecomastia are fea- b. PEFR
tures of: c. EEG
a. Acute Amoebic dysentery d. EMG
b. Chronic Amoebic dysentery 73. Which is responsible for rheumatic fever?
c. Acute liver disease
a. Staphylococcus
d. Chronic liver disease
b. E. Coli
66. The complication of enteric fever during second to
c. Streptococcus
third week is:
d. Clostridium
a. Deafness
74. Which drug is useful for prevention of migraine?
b. Perforation of intestine
c. Anemia a. Aspirin
d. None of the above b. Paracetamol
67. One of the following statement is true about primary c. Ergot
chancre: d. Flunarizine

Answers: 60. a 61. c 62. c 63. b


64. d 65. d 66. c 67. c
68. c 69. a 70. b 71. b
72. b 73. c 74. c
236   Mastering the BDS IIIrd Year  (Last 25 Years Solved Questions)

75. ADA criteria for diagnosis of diabetes fasting blood c. Vitamin K


glucose should be equal to or more than d. Fibrinogen
a. 50 mg/dL 83. Milk is devoid of:
b. 100 mg/dL a. Iron
c. 126 mg/dL b. Proteins
d. 102 mg/dL c. Fats
76. In acute LVF (Pulmonary edema), treatment of choice d. Calcium
in emergency is: 84. Steatorrhea is a feature of:
a. Furosemide a. Amoebic dysentery
b. Ciprofloxacin b. Malabsorption syndrome
c. Quinidine c. Bacillary dysentery
d. Thiazide d. Pseudomembranous colitis
77. Exopthalmos is clinical feature of: 85. Casal’s necklace is a feature of deficiency of:
a. Hypothyroidism a. Thiamine
b. Malaria b. Riboflavin
c. Throtoxicosis c. Pyridoxine
d. Diabetes d. Niacin
78. A continuous murmur is present in which congenital 86. Which of the following malignancy is associated with
heart disease: HIV infection?
a. Kaposi’s Sarcoma
a. ASD
b. Lymphatic leukemia
b. VSD
c. Hepatocellular carcinoma
c. Patent ductus arteriosus (PDA)
d. Chronic myeloid leukemia
d. CHB
87. For labeling, a patient of PUO, the duration of fever
79. Fever, headache, vomiting and neck rigidity is seen in
should be:
which condition:
a. 1 week
a. Pneumonia
b. 2 weeks
b. Panic attack
c. 3 weeks
c. Meningitis
d. 4 weeks
d. Anxiety 88. Generalized lymphadenopathy is a feature of:
80. Postural hypotension is common side effect of which a. Latent syphilis
drug:
b. Secondary syphilis
a. Prazosin
c. Primary syphilis
b. Dilantin d. Neurosyphilis
c. Chloroquine 89. Iron absorption mainly takes place in:
d. Clopidogrel a. Stomach
81. Following hepatitis viruses are RNA viruses except: b. Ileum
a. Hepatitis A c. Jejunum
b. Hepatitis B d. Colon
c. Hepatitis C 90. Pernicious anemia is caused by the deficiency of:
d. Hepatitis D a. Folic acid
82. Liver synthesizes following clotting factors except: b. Thiamine
a. Factor VIII c. Cyanocobalamine
b. Factor VII d. Ascorbic acid

Answers: 75. c 76. a 77. c 78. c


79. c 80. a 81. b 82. a
83. a 84. b 85. c 86. a
87. c 88. b 89. c 90. c
Section 1:  General Medicine  237

91. Chronic smoking is a risk factor for: 99. Cushing’s syndrome is due to excess of which hormone:
a. Bronchogenic carcinoma a. Glucocorticoids
b. Pneumonia b. Insulin
c. PTB c. Growth hormone
d. ARDS d. TSH
92. During examination of a patient hyper-resonant will 100. In SVT of new onset treatment of choice is:
be seen in case of:
a. Cardioversion
a. Pneumothorax
b. Digoxin
b. Pleural effusion
c. Beta-blocker
c. Pneumonia
d. Verapamil
d. PTB
101. TSH is raised in:
93. Irregularly irregular pulse is seen in a case of:
a. Hyperthyroidism
a. CHB
b. Hypothyroidism
b. VT
c. Euthyroidism
c. SVT
d. Atrial fibrillation d. All of the above
94. Which drug is used for thyrotoxicosis? 102. Gait in parkinsonism is:
a. Carbemazepine a. Hemiplegic
b. Dilantin b. Shuffling
c. Carbimazole c. Drunkin
d. Aspirin d. All of the above
95. In acute myocardial infarction which drug can be used 103. Mid-diastolic murmur is seen in:
for reperfusion? A. Mitral stenosis
a. Ceftriaxone B. Atrial stenosis
b. Streptokinase C. Aortic regurgitation
c. Warfarin D. All of the above
d. Dapsone 104. ACE inhibitors are contraindicated in:
96. Hypoglycemia can be treated in acute stage by: a. Adult + hypertension
a. Ampicillin b. Diabetic hypertension
b. Dexamethasone c. Pregnancy hypertension
c. Glucagon
d. None of the above
d. GH
105. Drug of choice in diabetic ketoacidosis is:
97. Which one is good cholesterol?
a. Sulphonyl urea
a. HDL
b. Metformin
b. LDL
c. Insulin
c. VLDL
d. All of the above
d. TG
106. Normal blood pH is:
98. Pulse pressure is wide in which valvular heart disease:
a. 7.4
a. Aortic regurgitation (AR)
b. 7.2
b. MS
c. AS c. 7.1
d. PS d. None of the above

Answers: 91. a 92. a 93. d 94. c


95. b 96. c 97. a 98. a
99. a 100. d 101. b 102. a
103. a 104. d 105. c 106. a
238   Mastering the BDS IIIrd Year  (Last 25 Years Solved Questions)

107. Increase in blood homocysteine level causes: 115. Drug of choice in gestational diabetes is:
a. Pneumonia a. Metformin
b. Nephritis b. Insulin
c. Coronary artery disease c. Sulphonyl urea
d. All of the above d. All of the above
108. Petechial hemorrhages are seen in: 116. Clubbing and chymosin both are seen in:
a. Tubercular meningitis a. Mitral stenosis
b. Pneumococcal meningitis b. Congestive heart failure
c. Meningiococcal meningitis c. Fallot’s teratology
d. All of the above d. None of the above
109. Subcutaneous nodules are seen in: 117. Exopthalmos is seen in:
a. Rheumatic fever a. Grave’s disease
b. Bacterial endocarditis b. Acromegaly
c. Angina pectoris c. Tetany
d. All of the above d. None of the above
110. Blackish discoloration of buccal mucosa is seen in: 118. Water hammer pulse is seen in:
a. Tetany a. Mitral stenosis
b. Acromegaly b. Aortic regurgitation
c. Addison’s disease c. Aortic stenosis
d. None of the above d. All of the above
111. T3 is raised in: 119. Insulin resistance is seen in:
a. Hypothyroidsm a. Type I-diabetes
b. Hyperthyroidism b. Type II-diabetes
c. Euthyroid c. Both of the above
d. All of the above d. None of the above
112. Tremors, rigidity and hypokinesia is a feature of: 120. Smoking is a risk factor for:
a. Meningitis a. Lung cancer
b. Encephalitis b. Myocardial infarction
c. Parkinsonism c. Chronic obstructive pulmonary disease
d. All of the above d. All of the above
113. Pan systolic murmur is seen in: 121. TSH is decreased in:
a. Mitral stenosis a. Hypothyroidism
b. Mitral regurgitation b. Hyperthyroidism
c. Aortic stenosis c. Euthyroidism
d. None of the above
d. All of the above
114. Drug of choice in pregnancy hypertension is: 122. Drug of choice in gestational diabetes is:
a. ACE inhibitor a. Sulphonyl urea
b. Metformin
b. ARBs c. Insulin
c. Methyldopa d. All of the above
d. All of the above

Answers:
107. c 108. c 109. a 110. c
111. b 112. c 113. b 114. c
115. a 116. d 117. a 118. b
119. b 120. d 121. b 122. c
Section 1:  General Medicine  239

123. ACE inhibitors are contraindicated in: 132. Chronic hepatitis is seen in (most commonly):
a. Adult hypertension a. Hepatitis A
b. Diabetic hypertension b. Hepatitis B
c. Pregnancy hypertension c. Hepatitis C
d. None of the above d. Hepatitis D
124. Osler’s nodes are found in: 133. Swelling of salivary glands is seen in:
a. Rheumatic fever a. Infectious mononucleosis
b. CCF b. Measles
c. Infective endocarditis c. Mumps
d. All of the above d. Rubella
125. Tremors, rigidity and hypokinesia is the feature of: 134. Xerophthalmia is seen in:
a. Parkinsonism a. Vitamin A deficiency
b. Mitral stenosis b. Vitamin B deficiency
c. Aortic regurgitation c. Vitamin C deficiency
d. All of the above d. Vitamin D deficiency
126. Hypoglycemia can be treated in acute stage by: 135. Corynebacterium diptheriae has following features:
a. Sulphonyl urea a. Non-motile
b. Glucagon b. Non-sporing
c. Dexamethasone c. Gram positive
d. Insulin d. All of the above
127. Drug of choice in thyrotoxicosis: 136. Ludwig’s angina involves spaces except:
a. Carbimazole a. Sub-arachanoid
b. Dilantin b. Bilateral sublingual
c. Carbamazepine c. Bilateral submental
d. Levothyroxine d. Bilateral submandibular
128. Water hammer pulse is seen in: 137. Generalized lymphadenopathy is the feature of:
a. Aortic regurgitation a. Agranulocytosis
b. Mitral stenosis b. Diarrhea
c. CCF c. Lymphomas
d. None of the above d. Renal failure
129. Cushing’s syndrome is due to excess of which hormone: 138. Von Willebrand disease is caused by deficiency of:
a. Insulin a. Factor VII
b. TSH b. Subendothelial factor VIII
c. Growth hormone c. Factor IX
d. Glucocorticoids d. Factor X
130. Which drug is useful for prevention of migraine? 139. In cases of agranulocytosis:
a. Aspirin a. Platelet count is increased
b. Paracetamol b. RBC count is increased
c. ACE inhibitors c. WBC count is decreased
d. Flunarzine d. WBC count is increased
131. Gum hypertrophy is caused by: 140. Hemodialysis is done in cases of:
a. Phenytoin sodium a. Congestive cardiac failure
b. Phenobarbitone b. Chronic myeloid leukemia
c. Sodium valproate c. Chronic renal failure
d. Carbamazepine d. Chronic obstructive lung disease

Answers:
123. c 124. c 125. a 126. b
127. a 128. b 129. d 130. c
131. a 132. b 133. c 134. a
135. d 136. a 137. c 138. b
139. c 140. c
240   Mastering the BDS IIIrd Year  (Last 25 Years Solved Questions)

141. Dangerous type of wear: 149. Electrical alternans is seen in:


a. Attic granulation a. Cardiac temponade
b. Central perforation b. Restrictive cardiomyopathy
c. Marginal perforation c. Left atrial myxoma
d. Multiple perforation d. COPD
142. Ototoxic drug is: 150. In Kartagener syndrome all are seen except:
a. Gentamycin a. Cystic fibrosis
b. Metrogyl b. Dextrocardia
c. Penicillin c. Sinusitis
d. Ampicillin d. Absence of cilia
143. No hepatic dose adjustment is required in: 151. Most common cause of superior vena cava obstruction
a. Rifampicin is:
b. INH a. Thrombosis
c. Ethambutol b. Extrinsic compression
d. PZA c. Mediastinal lymphoma
144. Fatty change in liver is due to accumulation of: d. Teratoma
a. Cholestrol 152. Most common sign of aspiration pneumonitis:
b. VLDL a. Tachypneoa
c. LDL b. Broncheospasm
d. Triglyceride c. Cyanosis
145. Which inhaled pollutant likely to cause pulmonary d. Crepitation
fibrosis: 153. Aspirin sensitive asthma is associated with:
a. Silica a. Extrinsic asthma
b. Tobacco b. Urticaria
c. Ozone c. Nasal polyp
d. Carbon dioxide d. Obesity
146 Not associated with cancer: 154 Ursodeoxycholic acid is a:
a. Fragile X syndrome a. Urinary stone dissolving drug
b. Fanconi’s syndrome b. Thrombolytic drug
c. Down’s syndrome c. Gallstone dissolving drug
d. Bloom syndrome d. Antiplatelet
147. Radiofrequency ablation is done for: 155. Highest concentration of K+ is seen in:
a. Ventricular tachycardia a. Dueodenum
b. PSVT b. Jejunum
c. WPW c. Ileum
d. Atrial tachycardia d. Colon
148. Nosocomial pneumonia is most commonly caused by: 156. Diabetes insipidus is due to lack of:
a. Gram negative bacilli a. ANP
b. Gram positive bacilli b. Vasopressin
c. Gram negative cocci c. Aldosterone
d. Mycoplasma d. Insulin

Answers:
141. b 142. a 143. c 144. d
145. a 146. c 147. b 148. a
149. a 150. b 151. c 152. a
153. a 154. c 155. d 156. b
Section 1:  General Medicine  241

157. Elevated glucose level especially in obese may be due 165. HAART is used in treatment of following infection:
to: a. HAV
a. DKA b. HBV
b. Glucose intolerance c. HCV
c. Insulin resistance d. HIV
d. Insulin shock 166. Community acquired pneumonia is most commonly
158. Which of the following is hypoglycemic drug: caused by:
a. Atorvastatin a. Pseudomonas aeruginosa
b. Vildagliptin b. Streptococcus pyogenes
c. Glimepride c. Streptococcus pneumonia
d. Clopidogrel d. Mycoplasma
159. “Master gland” of endocrine system located in base of 167. Autoimmune disease causing oral ulcer is:
brain: a. Liver cirrhosis
a. Apical gland b. Diabetes insipidus
b. Bartholin gland c. Systemic lupus erythematosus
c. Pituitary gland d. Hypothyroidism
d. Thyroid gland 168. Hepatitis B virus belongs to family called:
160. Glitazones are used to treat: a. Flaviviridae
a. Diabetes insipidua b. Picornavirus
b. NIDDM c. Deltaviridae
c. Infertility d. Hepadnaviridae
d. Hypothyroidism 169. Hyperglycemic hyperosmolar state is complication of:
161. Pernicious anemia is due to failure of production of: a. Hyperthyroidism
a. Bile b. Hypothyroidism
b. Insulin c. Diabetes insipidus
c. Intrinsic factor d. Diabetes mellitus
d. ACTH 170. Which of the following is thiazolidinedione:
162. Complication of herpes zoster is: a. Glimepiride
a. Haemolytic uremic syndrome b. Vildagliptin
b. Toxic mega colon c. Pioglitazone
c. Progressive multifocal leucoencephalopathy d. Metformin
d. Ramsay Hunt Syndrome 171. Common causative agent for subacute infective endo-
163. Digoxin is used in: carditis:
a. NIDDM a. Staphylococcus epidermis
b. Cardiac arrhythmia b. Neisseria gonorrhoeae
c. Diabetes insipidus c. Enterococci
d. Hypothyroidism d. Streptococcus viridians
164. Which of these is a hemolytic anemia: 172. Paroxysmal nocturnal dyspnea is a symptom of:
a. Sideroblastic anemia a. Liver cirrhosis
b. Aplastic anemia b. Hypertension
c. Hereditary anemia c. Heart failure
d. Aplastic anemia d. Renal failure

Answers:
157. c 158. b and C 159. c 160. b
161. c 162. d 163. b 164. c
165. d 166. c 167. c 168. d
169. c 170. d 171. c 172. a
242   Mastering the BDS IIIrd Year  (Last 25 Years Solved Questions)

173. Drug used in treatment of CML is: 177. Which of the following is an anti – platelet drug:
a. Imatinib mesylate a. Prednisone
b. Fludarabine b. Repaglinide
c. Rituximab c. Clopidogrel
d. Cyclophosphamide d. Atorvastatin
174. Cushing’s syndrome is due to: 178. Second line anti – tubercular drug is:
a. Increased insulin a. Athionamide
b. Hypoaldosteronism b. Isoniazid
c. Increased mineralocorticoid c. Streptomycin
d. Increased glucocorticoid d. Rifampcin
175. In nephrotic syndrome there is: 179. Status epilepticus refers to:
a. Hyperalbuminemia a. Cardiac arrhythmia
b. Massive proteinuria b. Continuous seizures
c. Mild proteinuria c. Continuous fever
d. Hypolipidemia d. Hyperglycemia
176. Bell’s palsy is a: 180. β adrenoceptor agonist are used in:
a. Upper motor neuron type of 5th nerve palsy a. Diabetes mellitus
b. Upper motor neuron type of 7th nerve palsy b. Myocardial infarction
c. Lower motor neuron type of 5th nerve palsy c. Atrial fibrillation
d. Lower motor neuron type of 7th nerve palsy d. Bronchial asthma

Answers:
173. a 174. d 175. b 176. d
177. c 178. c 179. b 180. d
Section 1:  General Medicine  243

FILL IN THE BLANKS


As per Dci and Examination Papers 1 Mark Each
of Various Universities

1. Herpes simplex is caused by ……………… 21. Mumps cause enlargement of glands called ……………
Ans. HSV-1 and HSV-2 Ans. Parotid
2. Diptheria is caused by ……………… 22. Full form of AIDS is ………………
Ans. Corynebacterium diphtheriae Ans. Acquired Immuno Deficiency Syndrome
3. Syphilis is caused by ……………… 23. Enteric fever is caused by ………………
Ans. Treponema pallidum Ans. Salmonella typhi
4. Amoebiasis is caused by ……………… 24. AIDS is caused by ………………
Ans. Entamoeba histolytica
Ans. HIV virus
5. The most common cause of pleural effusion is
25. TSH level is decreased in ………………
………………
Ans. Hyperthyroidism
Ans. Pneumonia
26. Water soluble vitamins are …………… and …………
6. Pellagra is caused by ………………
Ans. Vitamin B3 deficiency Ans. B-Complex and Vitamin C
7. Vitamin C deficiency causes ……………… 27. Vitamin B1 deficiency causes ………………
Ans. Scurvy Ans. Beriberi
8. TSH level is raised in ……………… 28. Bell’s Palsy is paralysis of …………… cranial nerve.
Ans. Hypothyroidism Ans. Facial nerve (Seventh)
9. Syncope is due to ……………… 29. Tachycardia means heart rate more than ……………
Ans. Hypotension Ans. 100 beats/min
10. Agranulocytosis means decrease of ……………… 30. Most common cause of pneumonia is ………………
Ans. White blood cells Ans. Bacteria
11. Stomatitis is the feature of deficiency of ……………… 31. Cause of bacillary dysentery is ………………
Ans. Iron Ans. Shigella Bacterium
12. Acromegaly occurs due to ……………… 32. Viral hepatitis occur due to hepatitis ………………
Ans. Secretion of excess growth hormone ………………, ……………… and ……………… virus.
13. Smoking commonly causes cancer of ……………… Ans. A, B, C, D and E
Ans. Lungs 33. Night blindness is the feature of vitamin ………………
14. Caput Medusae is seen in ……………… deficiency.
Ans. Portal hypertension Ans. Vitamin A
15. Hypertension means BP above ……………… mm Hg. 34. Cretinism occurs due to ………………
Ans. 140/90 Ans. Hypothyroidism
16. Ischemic heart disease is due to occlusion of arteries 35. Neck rigidity is seen in ………………
called ……………… Ans. Meningitis
Ans. Coronary Artery 36. Drugs causing Cushing’s syndrome is ………………
17. Anti-viral drug use in treatment of viral hepatitis B is Ans. Glucocorticoid drugs
………………
37. Full form of HIV is ………………
Ans. Lamivudine
Ans. Human Immunodeficiency Virus
18. Kernig’s sign is positive in ………………
Ans. Meningitis 38. Trigeminal neuralgia occur due to involvement of
……………… Cranial nerve.
19. Addison’s disease is due to ………………
Ans. Adrenal insufficiency Ans. Fifth nerve
20. Cushing’s syndrome is due to ……………… 39. Full form of ARDS is ………………
Ans. Excess of ACTH hormone Ans. Acute Respiratory Distress Syndrome
244   Mastering the BDS IIIrd Year  (Last 25 Years Solved Questions)

40. Infectious mononucleosis occurs due to ……………… 61. JVP stands for………………
Ans. Epstein-Barr virus Ans. Jugular venous pressure
41. Pneumothorax means……………… 62. Idiopathic facial palsy is called as………………
Ans. Collection of air in chest or pleural cavity Ans. Bell’s palsy
42. Thyrotoxicosis means……………… 63. Vitamin D deficiency caused……………… in children.
Ans. Excess of thyroid hormone in body Ans. Rickets
43. Cranial nerves originating from Pons are ……………… 64. ARDS stands for ………………
and ……………… Ans. Acute respiratory distress syndrome
Ans. Trigeminal nerve and facial nerve 65. Peptic ulcer is caused by………………
44. Hepatitis B causes cancers of ……………… Ans. Helicobacter pylori
Ans. Liver 66. Janeway lesions are seen in………………
45. Drugs used in treatment of herpes zoster is ………………
Ans. Palms or soles in infective endocarditis
Ans. Acyclovir
67. Bald tongue is the feature of………………
46. Meningiococcal meningitis is caused by ………………
Ans. Iron deficiency anemia
Ans. Bacterium Neisseria meningitidis
68. Tuberculosis is caused by………………
47. Algid malaria presents with ………………
Ans. Mycobacterium tuberculosis
Ans. Hemodynamic disorders as shock with pronounced
69. The most common cause of lower motor neuron type
metabolic changes and hypothermia.
of facial palsy is ………………
48. Acromegaly is caused by ………………
Ans. Trauma
Ans. Excessive secretion of growth hormone
70. Single most important etiological factor for COPD
49. Vitamin A deficiency causes………………
is………………
Ans. Night blindness
Ans. Smoking
50. Drug used in treatment of trigeminal neural-
71. Pulse rate in hypothyroidism is ………………
gia………………
Ans. Slow
Ans. Carbamazepine
72. Hookworm infestation causes ……………… anemia
51. The most common cause of hyperthyroidism is
……………… Ans. Nutritional
Ans. Grave's disease and toxic nodular goiter 73. Viral cause of bilateral parotid enlargement
is………………
52. Nephrotic syndrome is increased excretion of
……………… in urine Ans. Mumps
Ans. Protein 74. Neck stiffness and Kernig’s sign is seen in………………
53. Good pasture’s syndrome is simultaneous involvement Ans. Meningitis
of ……………… and lungs. 75. The most common organism causing community ac-
Ans. Kidney quired pneumonia (CAP) is ………………
54. Syphilis is caused by……………… Ans. Mycobacterium pneumoniae
Ans. Treponema pallidum 76. Addison’s disease occurs due to deficiency of
55. Drug used in treatment of petit mal epilepsy ………………
……………… Ans. Adrenocorticoids
Ans. Ethosuximide, valproic acid and clonazepam 77. Arthritis of acute rheumatic fever is………………
56. The most common cause of hepatitis is……………… Ans. Polyarthritis
Ans. Virus 78. Common infective cause of jaundice is………………
57. Commonest cause of cirrhosis in India is ……………… Ans. Viral
Ans. Alcohol
79. Osler nodes are seen in………………
58. Caput medusa is seen in………………
Ans. Infective endocarditis
Ans. Umbilicus
80. Drug used for trigeminal neuralgia is ………………
59. Emphysema is commonly seen in………………
Ans. Carbamazepine
Ans. Smokers
60. Clubbing is obliteration of ……………… 81. Cerebral malaria is caused by………………
Ans. Nailbeds Ans. Plasmodium falciparum
Section 1:  General Medicine  245

82. Endocrine disorder that causes macroglossia (large 101. Black water fever is a complication of………………
tongue) is ……………… Ans. Malaria
Ans. Acromegaly 102. In hypothyroidism, the pulse rate is………………
83. Common adverse effect of ACE inhibitor is ……………… Ans. Slow
Ans. Profound hypotension 103. Beriberi is caused by the deficiency of………………
84. For diagnosis of nephritic syndrome 24 hours urinary Ans. Vitamin B
protein should be ……………… 104. ……………… is an important clinical feature of lung
Ans. More than 3.5 g/day abscess and bronchiectasis.
85. Preferred route of drug delivery for treatment of bron- Ans. Chronic lung sepsis
chial asthma is………………
105. In thyrotoxicosis, the level of……………… is decreased.
Ans. Inhalation
Ans. TSH
86. Pneumothorax is collection of air in ……………… 106. ……………… is the most common cause of bleeding
Ans. Pleural cavity gums.
87. Night blindness is caused by deficiency of …………… Ans. Vitamin C deficiency
Ans. Vitamin A 107. Retinopathy is a complication of……………… an-
88. Changes in nail in patient with broncheictasis is dis………………
……………… Ans. Diabetes mellitus and hypertension
Ans. Clubbing 108. The most common complication of mitral stenosis
89. In vitamin B12 deficiency the color of tongue is is………………
……………… Ans. Acute pulmonary edema
Ans. Beefy or fiery red 109. Ascites is present in……………… cavity.
90. The infection of measles is spreaded by ……………… Ans. Peritoneal
method
110. Diphtheria is caused by………………
Ans. Airborne
Ans. Bacterium corynebacterium diphtheriae
91. The diagnostic feature of diphtheriais …………… on
111. Aminoglycoside antibiotics are (name………………
tonsil
Ans. Streptomycin, Gentamycin, Tobramycin, Amikacin,
Ans. Presence of grayish green membrane
Kanamycin, Sisomicin, Neomycin, Framycetin and
92. The most important etiological factor of cirrhosis liver
Netilmicin
is………………
112. Composition of oral rehydration salt/solution
Ans. Alcohol is………………
93. Enteric fever is caused by……………… Ans. Sodium chloride – 2. 6 g
Ans. Salmonella typhi
Potassium chloride – 1.5 g
94. In upper motor neuron type of facial palsy,
Sodium citrate – 2.9 g
this……………… half of the face is spared.
Glucose – 13.5 gm
Ans. Upper
Water – 1 L
95. The type of anemia of renal origin is……………… Type.
113. Drug of choice for amoebiosis is………………
Ans. Normocytic normochromic
Ans. Diloxanide fuorate
96. The name of first cranial nerve is………………
114. Write down the common investigation for day-to-day
Ans. Olfactory
tooth extraction are………………
97. In……………… air and fluid both are present in pleural
Ans. CBC, FBS and PPBS, Rapid HbSAg, Rapid HIV
cavity.
Ans. Pleural effusion 115. Write down three salient features of Graves, dis-
ease……………………..
98. In pneumonia (consolidation), the type of breathing
Ans. Diffuse goiter, sign and symptoms of hypothyroidism,
is………………
Exopthalmosis, pre-tibial myxedema
Ans. Bronchial
116. Three commonly prescribed antimalarials are
99. ……………… is an important sign of meningitis. ………………
Ans. Stiff neck Ans. Quinine, chloroquine, artemisinin
100. In jaundice, the level of……………… is increased. 117. Drugs contraindicated in asthma patients are……………
Ans. Bilirubin Ans. NSAIDs and β-blockers
246   Mastering the BDS IIIrd Year  (Last 25 Years Solved Questions)

118. Three clinical features of AIDS are……………………. 133. Mention causes of strawberry gingivitis is………………
Ans. Weight loss, anorexia, night sweats Ans. Wegner’s granulomatosis
119. Two clinical features of Stevens Johnson syndrome 134. Mention causes of strawberry tongue is………………
are……………………. Ans. Scarlet fever
Ans. Involvement of oral, genital, ocular and nasal 135. Mention causes of bald tongue is………………
involvement, Nikolsky’s sign is positive Ans. Pernicious anemia
120. Write down five anti-tubercular medicines of short 136. Mention causes of cobblestoning of buccal mucosa
course therapy………………… is……………
Ans. Rifampicin, Isoniazid, Pyrazinamide, Ethambutol, Ans. Crohn’s disease
Streptomycin
137. Mention causes of diffuse melanin pigmentation
121. Three common clinical features of migraine
is………………
…………………
Ans. Physiologic oral pigmentation
Ans. Unilateral, episodic, throbbing headache associated with
138. Mention causes of honeycomb plaques is ……………
nausea, vomiting and visual disturbances; Females are
more affected as compared to males; frequency of each Ans. Lupus erythematosus
attack is from hours to days. 139. Mention causes of floating teeth is………………
122. Drug of choice, doses and duration of treatment for Ans. Cherubism
herpes zoster…………… 140. The infection of syphilis is caused by………………
Ans. Acyclovir, 800 mg, 5 times a day for a week Ans. Treponema pallidum
123. Clinical features (any five) of cirrhosis of liver 141. Amoebiasis is caused by………………
…………… Ans. Entamoeba histolytica
Ans. Ascites, painless hepatomegaly, palmer erythema, loss 142. The name of fifth cranial nerve is………………
of libido, presence of mild-to-moderate jaundice
Ans. trigeminal nerve
124. Three commonly used medicines for acid peptic disease
143. In is……………… air and pus both are present in pleu-
are……………
ral cavity.
Ans. H 2 receptor antagonists, proton-pump inhibitors,
Ans. Pyopneumothorax
mucosal protective agents
144. HIV infection is caused by………………
125. Prophylaxis of rheumatic fever is with ……………
Ans. HIV-1 or HIV-2 viruses
Ans. Amoxicillin 50 mg/kg (max, 1 g) orally once daily for 10
days 145. The fluid in peritoneal cavity is called as………………
126. Three clinical features of acromegaly are…………… Ans. Ascites
Ans. Macroglossia, arthropathy, myopathy 146. Diphtheria is caused by………………
127. Complications of recurrent rheumatic fever episodes Ans. Corynebacterium diphtheriae
are……………… 147. Dysphagia means the difficulty in ………………
Ans. Atrial fibrillation, mitral stenosis, mitral regurgitation Ans. Swallowing
and heart failure 148. PEM meansis………………
128. Three clinical features of acute nephritis are………….. Ans. Protein energy malnutrition
Ans. Hypertension, edema over face, oliguria 149. The lower motor neuron type of facial palsy is also
129. 2 common causes of agranulocytes areis……………… known as………………
Ans. Use of cytotoxic drugs, irradiation Ans. Bell’s palsy
130. Drugs of choice for epilepsy are (any twois……………… 150. The thiamine deficiency causes………………
Ans. Carbamazepine, phenytoin Ans. Wet beriberi and dry beriberi
131. BMI (Body mass index) is calculated by formula 151. In mitral stenosis, the first heart sound is………………
is………………
Ans. Loud
Ans. Weight + Height (kg/m2)
152. TSH is……………… in hypothyroidism.
132. Four common causes of cervical lymphadenopathy are
……………… Ans. Increased
Ans. Syphilis, tuberculosis, herpes simplex infection, 153. Peripheral neuropathy is the complication of……………
lymphoma Ans. Diabetes mellitus
Section 1:  General Medicine  247

154. ……………… bronchial breathing suggests the pres- 173. ……………… is an important sign of meningitis.
ence of cavity in the lung. Ans. Brudzinski's sign
Ans. Tubular 174. Malaria spread by bite of………………..
155. Mumps is the infection of……………… Ans. Infected Anopheles mosquito
Ans. Parotid gland 175. Sydenham’s chorea is found in……………..
156. Matted lymph nodes are present in……………… Ans. Rheumatic fever
Ans. Tuberculosis
176. Erythropoietin is produced by………………
157. The most common cause of endocarditis is………………
Ans. Kidney
Ans. Streptococci
177. M o s t c o m m o n va l ve i n v o l ve m e n t i n R H D
158. Normal serum creatinine level is less than…………mg. is…………………..
Ans. 1.5 Ans. Aortic valve
159. Bradycardia means the pulse rate less than …………….. 178. Pe l l a g r a i s c a u s e d b y d e f i c i e n c y o f v i t a -
beats per minute. min……………………
Ans. 60 Ans. B3 or niacin
160. Cerebral malaria is a complication of………………
179. Acromegaly is caused by……………….. hormone ex-
Ans. Plasmodium falciparum cess.
161. Halitosis is present in……………… Ans. Growth
Ans. Oral cavity 180. R e t i n o p a t h y i s c o m p l i c a t i o n o f … … … … … .
162. M e g a l o b l a s t i c a n e m i a i s c a u s e d b y v i t a - and…………….
min………………… deficiency Ans. Diabetes mellitus and hypertension
Ans. B12 181. Warfarin causes prolongation of……………………time.
163. Name of the 7th cranial nerve is…………………. Ans. Prothrombin
Ans. Facial nerve 182. In leukemia, ……………….count is decreased.
164. Presence of blood in urine…………………… Ans. RBC
Ans. Hematuria 183. Cirrhosis is caused by hepatitis……………. and
165. Aspirin is………………..drug. ………….. virus.
Ans. Antiplatelet Ans. B and C
166. BCG vaccine is given to prevent…………………….. 184. To diagnose hypothyroidism, test used is……………..
Ans. Tuberculosis Ans. Thyroid function test
167. HIV stands for………………… 185. Pulmonary tuberculosis is caused by………………..
Ans. Human immunodeficiency virus Ans. Mycobacterium tuberculosis

168. In dengue,…………………(blood component) is de- 186. Three causes of stomatitis are………………..


creased Ans. Cheek bite, trauma due to dentures, viral infection i.e.
Ans. Platelet herpes, chemotherapy treatment for cancer
187. Beri Beri is due to deficiency of……………….
169. Diagnostic feature of diphtheria is ……………….. on
tonsil. Ans. Vitamin B
Ans. Grey membrane 188. Causative organism of syphilis is………………….

170. Increase steroid intake can cause………………. Facies. Ans. Treponema pallidum
Ans. Moon 189. Drug of choice in anaphylaxis is…………………
Ans. Epinephrine
171. Massive proteinuria is found in……………….syn-
drome. 190. Cause of rheumatic fever is……………….
Ans. Nephrotic Ans. Group A Treptococcus
172. Heart beat of less than 60 beats/min is called……………. 191. M e g a l o b l a s t i c a n e m i a i s d u e t o d e f i c i e n c y
Ans. Bradycardia of…………………
Ans. Vitamin B12
248   Mastering the BDS IIIrd Year  (Last 25 Years Solved Questions)

192. Syncope is due to……………….. 212. Vitamin C deficiency causes…………..


Ans. Insuficient flow of blood to brain Ans. Scurvy
193. Features of nephrotic syndrome are……………. 213. Dengue is caused by bite of………………….mosquito.
Ans. P r o t e i n u r i a , h y p o a l b u m i n e m i a , e d e m a , Ans. Aedes
hypercholestrolemia
214. Peptic ulcer is caused by this organism………………….
194. Endocarditis is an inflammation of……………… Ans. H. pylori
Ans. Endocardium
215. Cholera is …………….borne disease.
195. Scurvy is due to deficiency of……………….. Ans. Water
Ans. Vitamin C
216. In nephrotic syndrome, this is lost in urine………………
196. T h re e c l in ica l f e a tu r e s o f p o r ta l hyper ten - Ans. Protein
sion………………….
217. Hemophilia is treated with injection of………………
Ans. Ascites, splenomegaly, esophageal varices, hematemesis
Ans. Factor VIII or factor IX
197. Drug of choice in herpes zoster is…………………..
218. Fluconazole is a…………… drug.
Ans. Acyclovir
Ans. Antifungal
198. Agranulocytosis means…………….
219. Megaloblastic anemia is caused by…………………..
Ans. Decrease in neutrophil number
Ans. Deficiency of vitamin B12
199. CPR means………………..
220. Inhalational steroids are used in…………………
Ans. Cardiac pulmonary resuscitation
Ans. Bronchial asthma
200. Master of endocrine orchestra is…………………..
221. Enumerate four first-line antitubercular drugs
Ans. Pituitary gland
…………………
201. Three common cardiac arrhythmias are………………. Ans. Isoniazid, Rifampin, Ethambutol, Streptomycin
Ans. Atrial fibrillation, ventricular tachycardia, paroxysmal
supraventricular tachycardia 222. Treadmill test is done to diagnose……………………
Ans. Effects of exercise on heart
202. Three common heart diseases are…………………
Ans. Hear failure, heart valvular disease, congenital heart 223. EEG is done in the neurological condition………………
disease Ans. Seizure disorder
203. Body temperature is regulated by……………… 224. Enteric fever is diagnosed by the test………………..
Ans. Hypothalamus Ans. Widal
204. Hyperhidrosis means……………….. 225. CD4 counts are done for monitoring this dis-
Ans. Increased sweating ease……………….
Ans. AIDS
205. Grave's disease is due to………………..
Ans. Hyperthyroidism 226. Liver transplant is usually indicated for……………….
Ans. Liver failure
206. Painful deglutition is known as………………….
Ans. Dysphagia 227. Which drug should be avoided in dengue………………?
Ans. Aspirin
207. Beriberi is caused by deficiency of……………….
Ans. Vitamin B1 228. In primary hypothyroidism,………………… is in-
creased.
208. Molluscum contagiosum is a……………….illness.
Ans. TSH
Ans. Viral
229. Name of the fifth cranial nerve is………….
209. Tarry black stools are known as……………
Ans. Trigeminal
Ans. Melena
230. Malaria is caused by the bite of………………….
210. Hemoptysis is blood in……………… Ans. Anopheles mosquito
Ans. Cough
231. Oral ulcer, cheilosis mainly caused by the deficiency
211. Leprosy is caused by………………… of……………….vitamin.
Ans. Mycobacterium lepare Ans. B2 (Riboflavin)
Section 1:  General Medicine  249

232. Diagnostic feature of diphtheria is……………….. on Ans. Dilantin sodium


tonsil. 242. Swine flu is caused by……………………virus.
Ans. Pseudomembrane
Ans. H1N1
233. Heparin is……………..
243. Painless oral ulcer found in……………..connective
Ans. Anticoagulant
tissue disease.
234. Carditis is feature of………………… Ans. Systemic lupus erythematosus
Ans. Rheumatic fever
244. Tetany is caused by deficiency of………………………..
235. In AIDS,…………….count is decreased. Ans. Calcium
Ans. Lymphocyte
245. Increase steroid intake can cause…………………….
236. Difficulty in deglutition is called as……………… facies.
Ans. Dysphagia Ans. Moon
237. Fluid in pleural cavity is known as……………. 246. Blood in vomiting is called as………………..
Ans. Pleural effusion Ans. hemoptysis
238. Fever, neck rigidity, vomiting, headache are the fea- 247. Yellow discoloration of sclera is……………
tures of………………… Ans. Jaundice
Ans. Meningitis 248. Koilonychia is found in…………………deficiency
239. In cirrhosis which organ is mainly involved………………. anemia.
Ans. Liver Ans. Iron
240. Heart rate > 100 is known as…………………. 249. Rifampin drug is used in …………………
Ans. Tachycardia Ans. Tuberculosis.
241. Gum hypertrophy is side effect of which anti-epileptic
drug……………….
250   Mastering the BDS IIIrd Year  (Last 25 Years Solved Questions)

Viva-Voce Questions for


Practical Examination

1. Name the route of infection caused by Hepatitis E virus. 17. Name the type of ulcers seen in amoebiasis.
Ans. Feco-oral route Ans. Flask-shaped
2. Name the hepatitis virus which spreads by parenteral 18. In which disease does Koplik spots appear.
route. Ans. Measles
Ans. Hepatitis B, C and D 19. On which day rashes of measles occur.
3. Name the carcinoma by which Epstein-Barr virus is Ans. On fourth day
associated with.
20. Name the first sensation which becomes lost in leprosy.
Ans. Nasopharyngeal carcinoma
Ans. Temperature
4. Name the disease in which transaminase levels get
raised. 21. In which diseases does neutropenia occur.
Ans. Myocardial infarction Ans. Typhoid, viral infection, protozoal infections,
agranulocytosis
5. Which type of a virus is herpes virus?
22. In which age group does mumps commonly occur.
Ans. DNA virus
Ans. Children
6. Which type of virus is HIV virus?
Ans. It is a retro virus 23. Name the virus which leads to chickenpox.
7. What is the incubation period of hepatitis B virus Ans. Varicella zoster virus
infection? 24. In which disease, rose spots are seen.
Ans. Incubation period is 1 to 6 months Ans. Typhoid
8. Name the cells which get decreased in AIDS. 25. Which disease of show bitot spots?
Ans. CD4, i.e. below 200/mm3 Ans. Vitamin A deficiency
9. After how much duration of infection; the HIV is posi- 26. What is lupus vulgaris.
tive in the serological test? Ans. It is the cutaneous tuberculosis
Ans. 8 weeks 27. What is Pott’s disease?
10. What is the most common route of transmission of Ans. It is the tuberculosis of spine
AIDS?
28. What is Ghon’s focus?
Ans. Heterosexual contact
Ans. It is the presence of subpleural focus in primary
11. Name the test by which HIV is detected and also get tubercular granuloma along with central caseation.
confirmed.
29. What is Ghon’s complex?
Ans. Polymerase chain reaction (PCR) method
Ans. It is the Ghon’s focus along with regional lymph node
12. Name the most common neoplasm which occur in HIV
involvement.
positive homosexual males.
30. Name the first earliest manifestation of tetanus.
Ans. Glomus tumor
Ans. Trismus
13. What do you mean by scrofula?
31. Name the neurotoxin which is released by Clostridium
Ans. Tuberculosis of lymph nodes
tetani.
14. What is the route of spread of tubercle bacilli in the
military tuberculosis. Ans. Tetanospasmin
Ans. Bloodstream 32. What is C reactive protein?
15. Which physiological system does tertiary syphilis af- Ans. It is the acute phase reactant, which is synthesized by
fects? the liver, it opsonizes invading pathogens
Ans. Central nervous system 33. Name the drug of choice of trigeminal neuralgia.
16. Which type of ulcer is seen on the male genitalia in Ans. Carbamazepine
primary syphilis? 34. What is the other name of trigeminal neuralgia?
Ans. A punched out ulcer Ans. Tic douloureux, and Fothergill’s disease
Section 1:  General Medicine  251

35. What is parkinsonism? 52. Name the disease in which bleeding time is prolonged.
Ans. It is the disease caused by the depletion of pigmented Ans. Thrombocytopenic purpura and Von Willebrand disease
dopaminergic neurons in substantia nigra, Lewy bodies 53. How much is the normal platelet count?
and atropic changes in substantia nigra. Ans. It is 1.5 to 4 lakhs/cumm of blood.
36. Which is the drug of choice for partial and tonic clonic 54. What is critical count of platelet?
seizures?
Ans. When count of platelet is below 40000/mm3 it is critical
Ans. Phenytoin count of platelet.
37. What do you mean by hematuria? 55. What is thrombocytopenic purpura?
Ans. It is the presence of RBCs in urine. Ans. It is the abnormal decrease in the count of platelets.
38. Which is the earliest marker appearing in serum in 56. Name the components, which get destroyed in the
acute hepatitis B infection. blood.
Ans. HBsAg Ans. WBC, clotting factors and platelets
39. What is tetany. 57. Name the diseases which constitute hemoglobinopa-
Ans. It occur due to hypocalcemia and there is stridor due to thies.
spasm of glottis and not dysphagia. Ans. Sickle cell anemia and thalassemia
40. Which is the most common bacterial infection in gen- 58. Name the cardinal features of diabetic ketoacidosis.
eral medical practice.
Ans. Hyperglycemia, hyperketonemia and metabolic acidosis
Ans. Urinary tract infection
59. What is osteoporosis?
41. Name the triad of symptoms in portal hypertension.
Ans. This is a disease which is characterized by decrease in
Ans. Ascites, splenomegaly and esophageal varices the bone mass, microarchitectural destruction of bone
42. In which disease, there is increase in level of SGOT. and there is increased risk of fracture.
Ans. In liver disease 60. Name the drugs used in treatment of osteoporosis.
43. Why icterus is more marked in sclera. Ans. Bisphosphonates
Ans. It is white 61. What is glycosylated hemoglobin?
44. In which disease does dark color urine and clay stool Ans. It gives an accurate and objective measure of glycemic
is visible? control over period of weeks to months.
Ans. Obstructive jaundice 62. What is Philadelphia chromosome?
45. In which type of jaundice does conjugated and uncon- Ans. It is the chromosome, which is associated with chronic
jugated bilirubin get raised? myeloid leukemia.
Ans. Hepatocellular jaundice 63. What is the hallmark of obstructive lung disease.
46. Name the blood coagulation factor whose deficiency Ans. Decreased expiratory flow rate
leads to classical hemophilia or hemophilia A.
64. What is Kussmaul breathing?
Ans. Factor VIII
Ans. It is the increase in both the rate and depth of respiration
47. Name the blood coagulation factor whose deficiency in diabetic ketoacidosis and anemia.
leads to Christmas disease or hemophilia B.
65. What is the surfactant?
Ans. Factor IX
Ans. It reduces the surface tension and counteract tendency
48. Name the blood coagulation factor whose deficiency of alveoli to collapse
leads to hemophilia C.
66. What is asthma?
Ans. Factor XI
Ans. It is the disorder, which is characterized by the
49. Name the blood coagulation factor whose deficiency chronic airway obstruction as well as increased airway
leads to vascular hemophilia or Von Willebrand dis- responsiveness which leads to wheeze, breathlessness,
ease. cough, chest tightness
Ans. Von Willebrand factor 67. What is emphysema?
50. Name the blood coagulation factor, whose deficiency Ans. It is the pathological process in which there is permanent
leads to parahemophilia. destructive enlargement of spaces distal to the terminal
Ans. Factor V bronchioles.
51. As patients of hemophilia get operated what is given 68. What is pleural effusion?
them to stop surgical bleeding? Ans. It is the accumulation of serous fluid in pleural space.
Ans. Factor VIII concentrate
252   Mastering the BDS IIIrd Year  (Last 25 Years Solved Questions)

69. What is Ghon’s focus? 85. What do you mean by cyanosis?


Ans. It is the unilateral solitary area of consolidation of Ans. Cyanosis is the bluish discoloration of nails because of
tuberculus inside the pleura in lower part of upper lobe. increased amount of reduced hemoglobin in the capillary
70. Name the most common site for deep vein throm- blood.
bosis. 86. What is coarctation of aorta?
Ans. Calf vein Ans. It is the narrowing of aorta at isthmus just below the
71. What is lobar pneumonia? origin of left subclavian artery
Ans. It is the radiological and pathological term which refers 87. Name the most common heart valve involved in IV
to the homogeneous consolidation of one or more lung drug abuse.
lobes. Ans. Tricuspid valve
72. What is bronchiectasis? 88. What is pulsus paradoxus?
Ans. It is the irreversible dilation of bronchi Ans. It is the dramatic fall in blood pressure due to inspiration.
73. What happens in the clubbing? 89. What do you mean by hypertensiv urgency?
Ans. In clubbing, the normal angle between proximal part of Ans. It is defined as the marked elevation of blood pressure
nail and skin become lost. without the end organ damage.
74. What is hemoptysis? 90. What do you mean by hypertensive emergency?
Ans. It is the coughing of the Frank blood. Ans. It is defined as the marked elevation of blood pressure
with end organ damage.
75. What is hematemesis?
91. Name the microorganism which leads to dental caries,
Ans. It is the vomiting of the altered blood.
infective endocarditis and atherosclerosis?
76. What is epistaxis?
Ans. Streptococcus mutans
Ans. It refers to the bleeding from the nose. 92. What do you mean by sinus tachycardia?
77. What is malena? Ans. It is the sinus rate of more than 100 beats per min.
Ans. It refers to the dark tarry colored stools. 93. What do you mean by hemochromatosis?
78. What do you mean by stable angina. Ans. It is the condition in which total body iron get increased.
Ans. When there is an imbalance between the myocardial 94. What do you mean by bronze diabetes?
oxygen supply and demand, stable angina occur.
Ans. It is the diabetes mellitus which is associated with
79. What do you mean by unstable angina. hemochromatosis with iron deposit in skin, liver,
Ans. It is the rapidly worsening angina when patient is on pancreas and other viscera
rest. 95. What is gout?
80. Name the most common congenital heart disease. Ans. It is the condition which is characterized by the recurrent
Ans. Ventricular septal defect. attack of acute inflammatory arthritis.
81. Name the most common congenital cyanotic heart 96. Which node does act as a pacemaker?
disease. Ans. SA node
Ans. Tetralogy of Fallot 97. What is pulsus alternans?
82. Name the most common cause of acute rheumatic fever. Ans. It is the strong and weak beat which occur alternatively.
Ans. β-hemolytic streptococci 98. What is pulsus bisferiens?
83. Name the most common complication of subacute Ans. It is the rapidly rising amd two time beating pulse whose
bacterial endocarditis. both the waves are felt at the time of systole.
Ans. Thromboembolism 99. What is pulsus tardus?
84. What do you mean by tachycardia and bradycardia? Ans. It is felt in aortic stenosis.
Ans. When heart beat is more than 100 beats per min, it is 100. What is trilogy of Fallot?
known as tachycardia and when heart beat is less than Ans. It is the combination of ventricular septal defect,
60 per min, it is known as bradycardia. pulmonary stenosis and overriding of aorta.
Section 1:  General Medicine  253

Additional Information

Classification of AIDS Various Serum Markers in Hepatitis–B and


Their Indications
Class I • Primary infection
• Fever, arthralgia, myalgia Marker Indication
• Occurrence of primary infection 2 to 4
HBsAg Acute HBV infection
weeks after an exposure.
• Appearance of specific, Anti–HIV Anti HBsAg Previous vaccination
antibodies in serum (sero conversion)
Anti HBC (IgM) Acute HBV infection
takes place later at 3 to 12 weeks
(median 8 weeks) Anti HBC (IgG) Chronic HBV infection
Class II Asymptomatic infection HBeAg Active replication
Class III Persistent generalized lymphadenopathy. Anti HBS Ag + Anti – HBC Previous infection
It is defined as enlarged glands at
more than two inguinal sites. Various Viruses and Their Diseases
Class IV AIDS syndrome
Name of virus Name of diseases
• Characterized by opportunistic
infection, tumors etc. Herpes simplex virus Type 1 • Primary stomatitis
• Bacterial, viral and fungal infections • Keratoconjunctivitis
• Candidiasis • Encephalitis
• Pneumocystis carinii pneumonia • Herpes labialis
• Mycobacterium avium • Herpetic whitlow
• Herpes simplex
Herpes simplex virus Type 2 Genital infection
• Toxoplasmosis
• Molluscum contagiosum Cytomegalovirus • Congenital infection
• Tumors • Pneumonitis
• Kaposi sarcoma • Retinitis
• Non – hodgkin’s lymphoma • Disease in immunocompromised
• Oral hairy leukoplakia patients
Epstein Barr virus • Infectious mononucleosis
• Burkitt’s lymphoma
CD4 Cell Count in HIV Associated Diseases • Nasopharyngeal carcinoma
• Oral hairy leukoplakia
CD4 cell count HIV associated diseases
Varicella zoster virus • Chicken pox
G r e a t e r t h a n 5 0 0 • Acute primary infection • Shingles
cells/mm3 • Persistent generalized
lymphadenopathy Herpes simplex virus - 8 Kaposi’s sarcoma
• Recurrent vaginal candidiasis
Incubation Periods of Various Diseases
Less than 500 cells/ • Pulmonary tuberculosis
mm3 • Pneumococcal pneumonia Infection Incubation period
• Herpes zoster
• Kaposi sarcoma Short incubation period
• Oral hairy leukoplakia • Anthrax 2 to 5 days
• Herpes zoster • Diphtheria 2 to 5 days
• Scarlet fever 1 to 3 days
Less than 200 cells/ • Pneumocystis carinii • Typhoid
mm3 • Herpes simplex 5 days
• Cholera
• Oesophageal candidiasis 2 to 3 hours
• Miliary tuberculosis
Intermediate incubation period
• Peripheral neuropathy
• Cryptosporidium • Measles 7 to 14 days
• Microsporidium • Mumps 2 to 3 weeks
• Rubella 2 to 3 weeks
Less than 100 cells/ • Cerebral toxoplasmosis • Chicken pox
mm3 • Cryptococcal meningitis 14 to 21 days
• Non – hodgkin’s lymphoma Long incubation period
• HIV associated dementia • Hepatitis – A 2 to 6 weeks
Less than 50 cells/ Cytomegalovirus retinitis • Hepatitis – B 6 weeks to 6 months
mm3 Disseminated mycobacterium avium • Tuberculosis Months to years
intracellulare • Leprosy
2 to 5 years
254   Mastering the BDS IIIrd Year  (Last 25 Years Solved Questions)

Complications of Various Diseases Disease affecting posterior • Tabes dorsalis


column • Diabetic pseudotabes
Disease Complications
Disease affecting • Subacute degeneration of spinal
Measles Otitis media
posterolateral columns cord
Rubella Polyarthritis • Subacute myelo – optic
neuropathy
Mumps Aseptic meningitis in children; Orchitis
and Oophoritis Complete cord transaction • Trauma
• Multiple sclerosis
Streptococcal pharyngitis Rheumatic fever
Typhoid Paralytic ileus
Tetanus Obstruction of airway Various Nerve Lesions and Their Effect on Oral Structures

Various Diseases and Their Rashes Nerve lesion Oral structure affected
Tr i g e m i n a l n e r v e On protrusion jaw deviates towards side
Disease Rash lesion of lesion
Bromide poisoning Presence of acne from rash Facial nerve lesion Deviation of angle of mouth to opposite side
Chicken pox Rash is seen on second day of infection Vagus nerve lesion On elevating the palate, uvula deviates to
which appears first on trunk. Lesions can opposite side of lesion
be seen during all stages of development.
Hypoglossal nerve On protrusion tongue deviates to side of
Eczema Monk’s cowl rash present on face and neck lesion lesion
Erythema infectiosum Presence of slap cheek rash with circumoral
pallor
Difference Between Upper Motor Neuron Lesion and
Kwashiorkor Presence of crazy pavement epithelium rash Lower Motor Neuron Lesion
Measles Presence of maculopapular rash which begin
at hairline and behind ears. It spread to neck, Upper motor neuron lesion Lower motor neuron lesion
chest and over extrimities This is the lesion of intracranial This is the lesion of facial nerve or
Pellagra Sunburn rash and necklace rash portion which is proximal to its nucleus inside the pons.
pontine nucleus.
Rubella Maculopapular rash which start over face
and move to trunk Etiology Etiology
Scarlet fever Presence of diffuse erythematous rash which • Tumors • Herpes zoster infection
blanch on pressure. Rash disappear under • Cerebrovascular accidents • Parotid tumor
7 to 10 days • Demyelination • Bell’s Palsy
• Lesion in cerebellopontine
Small pox Face is involved and pitting scars are angle
present.
Clinical features Clinical features
Typhoid As first week is over, rash is seen on upper • Involvement of opposite • Involvement of half of the face
abdomen. Rose red spots are present which side of face i.e. contralateral of same side of lesion.
are sparse, slightly raised and fade on giving side. • Loss of muscles of facial
pressure • Involvement of lower half expression
of face as in lower motor • Furrowing of face is absent on
Various Types of Spots in Diseases neuron palsy while upper same side.
part of face is spared. • Patient is inable to close the eye
Disease Type of spot of same side.
Bitots spot Vitamin A deficiency. Seen in eye • Angle of mouth gets flattened.
• Mouth gets deviated towards
Forcheimer spot Rubella. Seen on soft palate the opposite side.
Koplick spot Measles. Seen on buccal mucosa Power is decreased Power is much decreased
Rose spot Typhoid. Seen on abdomen Clasp knife spasticity is present Flaccidity is present
Roth spot Infective endocarditis. Seen in eye Atrophy is minimum Atrophy is marked
Deep reflexes are exaggerated/ Deep reflexes are absent or reduced
Diseases Affecting Various Parts of Spinal Cord brisk
Clonus is present Clonus is absent
Part of spinal cord Name of disease
affected Fasciculations are absent Fasciculations are present
Disease affecting central part Syringomyelia Reaction of degeneration in Reaction of degeneration in muscle
of spinal cord muscle is absent is present
Section 1:  General Medicine  255

Drugs of Choice in Seizures Hereditary Microspherocytes


sphaerocytosis
Type of Seizure Choice of drug
Infectious mononucleosis Atypical lymphocytes
In absence seizure • Ethosuccimide
• Sodium valproate Malaria Schuffner’s dots
For partial and tonic clonic seizure Phenytoin Megaloblastic anemia Megaloblasts and hypersegmented
neutrophils
For myoclonic seizure Piracetam
After splenectomy Howell jolly bodies, target cells, spur cells,
spherocytes
Various Lobes and their Functions Sepsis Neutrophilia and Dhol bodies
Name of lobe Functions Thalassemia Microcytes, tear drops and target cells
Frontal • Behavior
• Emotional control Various Types of Breath in Various Diseases
• Language
• Personality Name of breadth Name of disease
Parietal dominant • Language Acetone breadth Diabetes mellitus
• Calculation
Alcoholic breadth Alcoholics
Parietal non – dominant • Construction skill
• Spatial orientation Musty breadth Hepatic coma

Temporal dominant • Auditory perception Uremic breadth Renal disease


• Balance
• Smell
• Verbal memory Features of Hypothyroidism and Hyperthyroidism
Temporal non – dominant • Auditory perception Hypothyroidism Hyperthyroidism
• Balance
• Smell Cold intolerance Heat intolerance
• Non - Verbal memory Menorrhagia Amenorrhoea/Oligomenorrhoea
Occipital Visual processing Weight gain, goiter, hoarseness Loss of weight despite normal
and tiredness apetite, diffuse bruit
Various Blood Disorders with Their Laboratory Predictions
B r a d y c a r d i a , h y p e r t e n s i o n , Tachycardia, dyspnoea, atrial
Disease Factor Bleeding Clotting Pro- Partial congestive heart failure fibrillation
defi- time time throm- Throm- Dry skin, alopecia, myxedema Pretibial myxedema,
cient bin bo- pigmentation, increased
time plastin sweating
time
Depression, ache and pain, Carpal Irritability, nervousness, tremor,
Hemophilia A VIII Remain Increases Remain Increas- tunnel syndrome emotional lability
normal normal es
Iron deficiency anemia and Diplopia, Exopthalmos, lid
Hemophilia B IX Remain Increases Remain Increas- macrocytosis retraction
normal normal es
Hemophilia C XI Remain Increases Remain Slightly
normal normal pro- Various Diseases and Their Effect on Serum Calcium,
longed Serum Phosphorus and Alkaline Phosphatase
Para- V Remain Increases Increases Increas-
Hemophilia normal es
Name of disease Serum Serum Alkaline
Von Williebr- Von Wil- Increases Remain Remain Slightly calcium phosphorus phospha-
and Disease liebr-and normal normal pro- tase
Factor longed
Chronic renal failure Decrease Increase Normal
Various Disorders and Their Presenting Abnormality in Hyperparathyroidism Increase Decrease Increase
Peripheral Blood Film
Hypoparathyroidism Decrease Increase Normal
Name of disorder Abnormality in peripheral blood film Osteoporosis Normal Normal Normal
Aleukemic leukemia Circulating leuco – erythroblasts Rickets Increase Decrease Increase
256   Mastering the BDS IIIrd Year  (Last 25 Years Solved Questions)

Diagnosis of Oral Glucose Tolerance Test Which are the microorganisms Chlamydia, mycoplasma,
leading to atypical pneumonia pneumocystis carini
Interpretation Fasting 2 hours after
glucose glucose load Which are the most common Gram negative bacilli
microorganisms isolated from
Fasting hyperglycemia 110 to 125mg/dl <140mg/dl
patients with nosocomial pneumonia
Impaired glucose <126 mg/dl 140 to 199 mg/dl
Which is the drug of choice for Cotrimoxazole and
tolerance
atypical pneumonia which is caused Clindamycin
Diabetes >126 mg/dl >200 mg/dl by pneumocystis carini
Which is the drug of choice for Erythromycin and
atypical pneumonia which is caused clarithromycin
Various Volumes and Capacities in Obstructive and by mycoplasma
Restrictive Respiratory Diseases
Which is the drug of choice for Clarithromycin and
Name of volume or Obstructive Restrictive community acquired pneumonia flucloxacillin
capacity respiratory respiratory Which is the drug of choice for Doxycycline and erythromycin
disease disease atypical pneumonia which is caused
Diffusion capacity Normal Decrease by Chlamydia
Forced expiratory Decrease Increase
volume in 1 second/ Various Occupational Lung Diseases
Forced vital capacity
Lung compliance Unaffected Decrease Name of the occupational Description
lung disease
Residual volume Increase Decrease
Anthracosis Caused due to deposition of carbon
Total lung capacity Normal to increase Decrease particles
Vital capacity Decrease Decrease Asbestosis Asbestos particles deposit in lungs
Bagassosis Caused due to inhalation of
Various Chest Deformities sugarcane dust
Byssinosis Caused due to deposition of cotton
Name of chest Features fiber particles
deformity
Farmer’s lung Caused due to mouldy hay
Barrel shaped chest • Seen in chronic obstructive pulmonary
disease.
• In emphysema anteroposterior Various Diseases and Color of Pleural Fluid
diameter of chest increase relatively
to lateral diameter. Diseases Color of pleural fluid
Kyphoscoliosis • Abnormality in alignment of dorsal Acute pancreatitis Blood stained
spine which is caused by polio,
trauma or congenital abnormality. Malignant disease Blood stained
• This restricts and distorts the wall of Obstruction of thoracic outlet Milky
chest.
Rheumatoid disease Turbid
Pigeon chest or pectus • Seen in severe asthma during
carinatum childhood Tuberculosis Amber
• Seen in rickets also
Funnel chest or pectus • In this body of sternum mainly lower Various Diseases and Color of Sputum
excavatum end is curved backward.
• It restrict chest expansion and Diseases Color of sputum
decreases vital capacity
Active bronchopulmonary infection Mucopurulent
Bronchial carcinoma Red
Various Important Points in Respiratory Diseases Bronchiectasis and lung abscess Purulent
Anthracosis and aspergillosis Black
Important point Answer
Pneumonia Rusty
Which is the most important cause Streptococcus pneumonia or
of community acquired pneumonia Pneumococcus pneumonia Pulmonary edema Pink frothy
Section 1:  General Medicine  257

Laennec’s Division of Pneumonia Phases Various Markers in Myocardial Infarction

Phases Description Condition Markers present


Stage of • Numerous bacteria Enzymes which are elevated in plasma CPK – MD, SGOT, LDH
congestion • Dilation and congestion of capillaries in during myocardial infarction
alveolar wall
Proteins elevated in plasma during Troponin T and Troponin I
• Alveolar fluid consist of few red cells and
myocardial infarction
neutrophils
• In air spaces there is presence of air spaces Preferred biochemical markers for Troponins and CPK – MB
myocardial infarction
Early • Replacement of edema fluid by fibrin
consolidation or • Marked cellular exudates of neutrophils by Markers arise first in myocardial infarction CPK – MB and Troponin T
Red hepatisation red cells Marker appear last in myocardial LDH
• Less prominent alveolar septa infarction
Late • Dense fibrin strands
consolidation • Red cells and neutrophils are disintegrated
or Grey • Cellular exudates often separated from walls Important Questions With Answers in Cardiovascular
hepatisation by thin clear space System
Resolution • Removal of fluid by expectoration which
Question Answer
result in restoration of lung parenchyma
• Enzymatic digestion of fibrin strands Which is the common congenital heart Ve n t r i c u l a r s e p t a l
• Engorged alveolar capillaries disease defect
Which is the common cyanotic congenital Tetralogy of fallot
heart disease
Different Types of Murmur
What is the common cause of mitral Rheumatic valvulitis
Systolic murmers stenosis, mitral regurgitation, aortic
regurgitation and tricuspid regurgitation
Ejection systole It is present in atrial systole, pulmonary systole
and atrial septal defect Name the common cause of acute β hemolytic streptococci
rheumatic fever
Pan systolic It is present in mitral regurgitation, tricuspid
regurgitation and ventricular septal defect. Name the common complication of sub Thromboembolism
This is best heard at the time of expiration acute bacterial endocarditis
Name the valves involved in rheumatic Mitral and aortic valves
Late systolic It is present in coarctation of aorta and mitral
phenomenon
valve prolapse
Which is the common valvular lesion Mitral regurgitation
Diastolic murmurs
following myocardial infarction
Mid diastolic It is present in mitral stenosis, tricuspid Which is the common cause of high output Chronic anemia
stenosis and atrial regurgitation. failure
Its character is rough rumbling, localized,
low pitch. Name the common disease complicated by R h e u m a t i c m i t r a l
sub acute bacterial endocarditis regurgitation
It is heard in mitral region with bell placed in
left lateral position. Sound increased during Name the common complication of Sub acute bacterial
expiration and exercise prosthetic valve endocarditis
Early diastolic It is present in aortic regurgitation and Name the valve commonly involved in Tricuspid valve
pulmonary regurgitation Ebstein anomaly

Types of Fallot and Their Features


Types of Endocarditis and Microorganisms Involved in
Them Name of Fallot Features
Trilogy of Fallot Ventricular septal defect + Pulmonary
Name of Endocarditis Microorganisms present stenosis + Overriding of aorta
Subacute endocarditis Streptococcus viridians group i.e. S. Tetrology of Fallot Ventricular septal defect + Pulmonary
sanguis, S. mitis, α hemolytic streptococci stenosis + Overriding of aorta + Right
Acute endocarditis Staphylococcus aureus and Streptococcus ventricular hypertrophy
pneumoniae Pentalogy of Fallot Ventricular septal defect + Pulmonary
Post – operative Staphylococcus epidermidis stenosis + Overriding of aorta + Right
endocarditis ventricular hypertrophy + Atrial septal defect
258   Mastering the BDS IIIrd Year  (Last 25 Years Solved Questions)

Various Heart Sounds T wave --------- Due to ventricular


repolarization
Name Time Mechanism Abnormality
of heart P-R interval Onset of P wave to Due to atrial
sound onset of Q wave depolarization and
conduction via AV
First or S1 Onset of Due to closure of • Loud sound
node
systole both mitral and is heard in
tricuspid valves anemia, QRS duration Onset of Q wave and Due to ventricular
pregnancy, end of S wave depolarization
thyrotoxicosis, Q-T interval Onset of Q wave and Due to electrical activity
mitral stenosis end of T wave inside ventricles
• Slow sound is
heard in mitral ST segment End of S wave and -----------
regurgitation onset of T wave
and heart failure
Second or End of systole Due to closure • Fixed wide Elements and Events of Jugular Venous Pressure
S2 of both aortic splitting: Atrial
and pulmonary septal defect Element Event
valves • Variable
splitting: Right ‘a’ wave During right atrial contraction
and left bundle ‘c’ wave Transmitted carotid impulse during onset of systole
branch block
‘v’ wave Passive atrial filling against close tricuspid valve in
Third or S3 Early diastole, Due to abrupt In heart failure and systole
just after com- cessation of mitral regurgitation
‘x’ descent During right atrial relaxation and descent or
pletion of S2 rapid filling from
displacement of tricuspid valve in systole
ventricular valve
‘y’ descent During opening of tricuspid valve and passive filling
Fourth or End of diasto- Ventricular origin • Absent in atrial
of right ventricle during start of diastole
S4 le just before fibrillation
commence- • Present in left
ment of S1 ventricular
hypertrophy
Classification of Bone Density by Misch

Type Hounsfield units Type of bone


D1 >1250 units Dense cortical bone
Waves or Segments of ECG
D2 850 to 1250 units Thick porous cortical bone on crest and
within coarse trabecular bone
Wave or Onset Cause
Segment D3 350 to 850 units Thinner porous cortical crest and fine
trabecular bone
P wave --------- Due to atrial
depolarization D4 150 to 350 units Almost no crestal cortical bone
QRS complex -------- Due to ventricular D5 <150 units Ve r y s o f t b o n e w i t h i n c o m p l e t e
depolarization mineralization
2
sECTION

General Surgery

1. Wound, Sinus and Fistula 17. Salivary Gland


2. Acute Infections 18. Thyroid and Parathyroid Gland
3. Specific Infections 19. Tonsils
4. Ulcer 20. Stomach
5. Lymphatics and Lymph Node Enlargement 21. Operative Surgery
6. Skin Tumors 22. Neurological Injuries
7. Bleeding Disorders 23. Fractures of Bone
8. Shock 24. Advanced Methods of Surgery and Radiology
9. Burn 25. Sterilization
10. Arterial Diseases 26. Upper Limb Ischemia
11. Venous Diseases 27. Rectum and Anal Canal
12. Cyst 28. Miscellaneous
13. Oral Cavity, Lip and Palate Multiple Choice Questions as per DCI and Examina-
14. Cleft Lip and Palate tion Papers of Various Universities
15. Oral Cancer Viva-voce Questions for Practical Examination
16. Neck Swelling Additional Information
General Surgery
♦♦ An alternative to suturing is the application of adhesive
1. Wound, Sinus and Fistula tape strips.
♦♦ Systemic antibiotics should be given
Q.1. Write short note on lacerated wound. ♦♦ Injection tetanus toxoid for prophylaxis against tetanus.
 (Sep 2008, 3 Marks)
Q.2. Enumerate the various types of surgical wounds and
Ans. Lacerated wound is an open wound.
describe their management.
• Lacerated wounds are caused by the blunt objects  (Apr 2010, 15 Marks) (Apr 2008, 15 Marks)
like fall on the stone or due to road traffic accident.  (Mar 1998, 16 Marks)
• Edges of lacerated wound are *jagged.
Ans. Surgical wounds are classified into four categories
• Injury involves skin or subcutaneous tissue or
according to possibility of infection occurring in them.
sometimes-deeper structures also.
• In the lacerated wound, there is crushing of tissue Management of Surgical Wounds
due to blunt nature of the object, which results in
hematoma, bruising, or even necrosis of tissue. ♦♦ Airways should be maintained.
♦♦ Bleeding of the patient should be controlled.
Management ♦♦ IV fluids should be started.
♦♦ If it is an incised wound, then primary suturing is done
Ideal form of management of incised or lacerated wound
is surgical inspection, cleaning, excision and closure, under after thorough cleaning.
appropriate anesthesia and *tourniquet in case of a limb. ♦♦ If it is a lacerated wound, then it is excised and primary
suturing is done.
♦♦ Wound must be thoroughly inspected to ensure that there
♦♦ If it is a crushed wound, there will be edema and tension in
is no damage to deep structure, or where encountered,
wound. In such cases, debridement of wound is done and all
there must be repaired.
♦♦ All dirt and foreign material must be removed. the devitalized tissues are excised, edema should subside in
♦♦ Excision of damage skin, wound margins and excision of 5 to 6 days. Afterwards delayed primary suturing is done.
devitalized tissue such as muscle and fascia. ♦♦ If the wound is deep devitalized, debridement of wound is
♦♦ After excision, wound is irrigated with antiseptic agent done and it is allowed to granulate completely. If wound is
and then suturing must be done. small secondary suturing is done. If wound is large, split
♦♦ There are precise suture placement technique for nerves, skin graft is used to cover the defect.
tendons and blood vessels. ♦♦ If wound is with tension, then fasciotomy is done.
♦♦ Muscles can be apposed in layer by mattress suture and fas- ♦♦ Antibiotics, fluid and electrolytic balance, blood trans-
cia and subcutaneous fat should be apposed by interrupted fusion, tetanus toxoid or anti-tetanus globulin injection.
absorbable suture to allow a firm platform for skin closure.
Classification of Surgical Wounds
♦♦ On the face, fine nylon suture should be placed near to the
wound margins, to be removed on the 5th day. See Table below.

Wound class Definition Examples of typical procedure Wound Infection Rate Usual organism
Clean Nontraumatic Mastectomy 2% Staphylococcus
aureus
Elective surgery Vascular procedure
Gastrointestinal, respiratory or
genitourinary tract not entered
Clean Respiratory Gastrectomy, hysterectomy, <10% Related to viscous
contaminated Genitourinary or gastrointestinal cholecystectomy entered
tracts entered but minimal
contamination
Contaminated Open, fresh, traumatic wounds. Ruptured appendix, resection of 15–30% Depend upon
Uncontrolled spillage from an unprepared bowel underlying causes
unprepared viscous. Minor break
in sterile technique.
Dirty Open, traumatic dirty wounds. Resection of gangrene 40–70% Depend upon
Traumatic perforated viscous. underlying causes
Pus in operative field

Q1. *Jagged = Unevenly cut or torn.


*Tourniquet = Any constrictor used on an extremity to apply pressure over an artery and thereby control bleeding.
262   Mastering the BDS IIIrd Year  (Last 25 Years Solved Questions)

Q.3. Write short note on keloid. (Feb 2013, 5 Marks) Classification of Wound
 (Mar 1998, 6 Marks) (Apr 2008, 5 Marks)
♦♦ Rank and Wakefield Classification
Ans. Keloid means like a claw. • Tidy wounds:
• In keloid, there is defect in maturation and –– They are wounds such as surgical incisions and
stabilization of collagen fibrils. Normal collagen wounds caused by sharp objects.
bundles are absent. –– It is incised, clean, healthy wound without any
• Keloid is common in blacks. tissue loss.
• It is more common in females. –– Usually primary suturing is done. Healing is by
• It is genetically predisposed, often familial and is primary intention.
very rare in Caucasians. • Untidy wounds:
• Keloid continues to grow even after 6 months, may –– They are due to crushing, tearing, avulsion,
be for many years. It extends into adjacent normal devitalised injury, vascular injury, multiple
skin. It is brownish black/pinkish black (due to irregular wounds, burns.
vascularity) in color, painful, tender and sometimes –– Fracture of the underlying bone may be present.
hyperaesthetic; spreads and causes itching. –– Wound dehiscence, infection, delayed healing are
• Keloid may be associated with Ehlers-Danlos common.
syndrome or scleroderma. –– Liberal excision of devitalized tissue and al-
• When keloid occurs following an unnoticed trauma lowing to heal by secondary intention is the
without scar formation is called as spontaneous management.
keloid, commonly seen in Negroes. –– Secondary suturing, skin graft or flap may be
• Some keloids occasionally become non-progressive needed.
after initial growth. ♦♦ Classification Based on Type of Wound
• Pathologically, keloid contains proliferating • Clean incised wound
immature fibroblasts, proliferating immature blood
• Lacerated wounds
vessels and type III thick collagen stroma.
• Bruising and contusion
• Keloid is common over the sternum. Other sites are
• Hematoma.
upper arm, chest wall, lower neck in front.
• Closed blunt injury.
Precipitating Factors • Puncture wounds and bites.
• Abrasion
♦ Negro race
• Traction and avulsion injury.
♦ Tuberculosis patient
• Crush injury
♦ Vaccination site.
• War wounds and gunshot injuries.
Complications • Injuries to bones and joints, may be open or closed.
• Injuries to nerves, either clean cut or crush.
♦ Ulceration
• Injuries to arteries and veins (major vessels).
♦ Infection.
• Injury to internal organs, may be of penetrating or
Treatment non-penetrating (blunt) types.
• Penetrating wounds.
♦♦ Steroid injection: Intrakeloidal triamcinolone, is injected at
♦♦ Classification based on Thickness of the Wound
regular intervals, may be once in 7–10 days, of 6—8 injec-
• Superficial wound
tions. Its sequence is Steroid injection—excision—steroid
• Partial thickness wound
injection.
• Full thickness wound
♦♦ Methotrexate and vitamin A therapy into the keloid.
• Deep wounds
♦♦ Compressive dressings with silicone gel sheets reduce the
• Complicated wounds
tendency of keloid to recurr.
• Penetrating wound.
♦♦ Laser therapy can be given.
♦♦ Classification based on Involvement of Structures
♦♦ Vitamin E/palm oil massage.
• Simple wounds
♦♦ Intralesional excision retaining the scar margin may pre-
• Combined wounds.
vent recurrence. It is ideal and better than just excision.
♦♦ Classification based on the Time Elapsed
♦♦ Excision and skin grafting may be done.
• Acute wound
Q.4. Classify wound and describes in detail the stage of • Chronic wound.
wound healing. (Sep 2006, 15 Marks) ♦♦ Classification of surgical wounds
Ans. Wound is discontinuity or break in epithelium. • Clean wound
A wound is break in the integrity of skin or tissues often • Clean contaminated wound
which may be associated with disruption of the structure • Contaminated wound
and function. • Dirty infected wound
Section 2:  General Surgery  263

Stages of Wound Healing ♦♦ Occasionally, multiple sinuses are seen.


♦♦ Epithelialization: Epithelialization occurs mainly from the ♦♦ Opening of the sinus occurs in a small triangular area in
edges of wound by process of cell migration and multipli- front of the ear at the level of the tragus.
cation. This is brought about by marginal basal cells. Thus ♦♦ Scarring is common around the opening due to repeated
within 48 hours entire wound is re-epithelialized. infection.
♦♦ Wound contraction: It starts after 4 days and usually Investigations
complete by 14 days. It is brought about by specialized
fibroblasts. Because of their contractile elements, they are ♦♦ Discharge study should be done.
called as myofibroblasts. Wound contraction readily occurs ♦♦ Sinusogram is done to assess the track.
when there is loose skin as in back. ♦♦ MR sinusogram is beneficial.
♦♦ Connective tissue formation: Formation of granulation tissue
is most important and fundamental step in wound healing. Differential Diagnosis
Injury results in release of mediators of inflammation. This ♦♦ Cold abscess
result in increased capillary permeability, later kinins and ♦♦ Sebaceous cyst
prostaglandin act and play chemotactic role for white cells
and fibroblasts. In first 48 hours, PMN leukocytes domi- Treatment
nate. They play role of scavengers by removing dead and ♦♦ If sinus infected antibiotics and drainage are required fol-
necrotic tissue. Between 3rd and 5th day, PMN leukocytes lowed by excision when infection passes off.
diminished but monocytes increases. By 5th or 6th day, ♦♦ Under general anesthesia complete excision of tract is only
fibroblasts appear which proliferate and eventually give recommended.
rise to protocollagen, which is converted in collagen in
presence of enzyme protocollagen hydroxylase; O2, fer-
rous ion and ascorbic acid are necessary. The wound is
fiber-gel-fluid system.
♦♦ IV. Scar formation: Following changes takes place during
scar formation:
• *Fibroplasia and lying of collagen is increased.
• Vascularity becomes less.
• Epithelialization continues.
• Ingrowth of lymphatics and nerve fiber takes place.
Fig. 1: Sinus
• Remodeling of collagen takes place with cicatrization
result in scar. Q.6. Write short note on mental sinus. (Sep 2006, 5 Marks)
Q.5. Write short note on Preauricular sinus.  Ans. It is a chronic infective acquired condition, wherein
 (Sep 2006, 5 Marks) (Mar 2009, 5 Marks) there is infection of roots of one or both lower incisor
Ans. It is a congenital entity occurring due to imperfect fusion teeth forming root abscess which eventually tracks
of the six tubercles which form ear cartilage. down between two halves of lower jaw in the midline
presenting as discharging sinus on the point of chin at
Clinical Features midline.
♦♦ It is seen since childhood.
Clinical Features
♦♦ It can be unilateral or bilateral.
♦♦ Often swelling appears and apparently disappears repeat- ♦♦ Usually painless discharging sinus in the midline on the
edly. point of chin.
♦♦ Pain and discharge is common. ♦♦ Often incisor infection may be revealed.
♦♦ It causes a cosmetic problem in young individual. ♦♦ Osteomyelitis of the mandible is the possible complication.
♦♦ Sinus opening may be seen at the root of the helix or on ♦♦ Pus discharges through the sinus in center of chin.
the tragus. ♦♦ Patient present with recurrent swelling in submental re-
♦♦ Track is quiet deep running backwards, slightly upwards gion, which burst open subcutaneously discharging mucus
towards the helix. It usually ends blindly. and seropurulent fluid at times.
♦♦ Outer opening of the sinus often closed causing formation ♦♦ There is repeated history of swelling, discharge and healing
of a cystic swelling which contains fluid which is often which are the common presentations.
infected.
♦♦ Preauricular sinus in no instance will communicate with Investigations
the external auditory meatus. ♦♦ Dental X-ray is diagnostic.
♦♦ Bursting of this swelling leads into formation of ulcer like ♦♦ Discharge study should be done.
lesion.

Q4. *Fibroplasia= Developing of fibrous tissue.


264   Mastering the BDS IIIrd Year  (Last 25 Years Solved Questions)

Differential Diagnosis Ans.


♦♦ Infected sebaceous cyst Sinus
♦♦ Tuberculous sinus It is a blind tract leading from the surface down into the tissues.
♦♦ Osteomyelitis It is lined with a granulation tissue.

Causes
♦♦ Congenital sinus: Preauricular sinus.
♦♦ Acquired sinus:
• Median mental sinus: Occurs as a result of tooth abscess.
• Pilonidal sinus: Occurs in the midline in the anal region.
• Osteomyelitis: Gives rise to sinus discharging pus with
or without bony spicules.

Fistula
It is an abnormal communication between the lumen of one viscus
and the lumen of another (internal) or communication of one
hollow viscus with the exterior, i.e. body surface (external fistula).
♦♦ Examples of internal fistula:
• Tracheo-esophageal fistula
• Colovesical fistula
• Examples of external fistula
Fig. 2:  Mental sinus
• Orocutaneous fistula due to carcinoma of the oral
Treatment cavity infiltrating the skin
♦♦ After doing discharge surgery, antibiotics should be given. • Branchial fistula
♦♦ Lay opening and excision of sinus tract with extraction of • Thyroglossal fistula.
incisor tooth/teeth.
Clinical Features of Sinus/Fistula
Q.7. Write short note on crushed lacerated wound.
 (Apr 2007, 10 Marks) ♦♦ Discharge from the opening of sinus
Ans. •  C rushed lacerated wound is caused by road ♦♦ No floor is present
accidents, or a machinery accidents. ♦♦ Raised indurated edge, indurated base and non-mobile
• Crushing of the parts with lacerated skin, ♦♦ Often sprouting granulation tissue over sinus opening.
devitalization or crushing of the musculature is ♦♦ Bone thickening in osteomyelitis
seen. These devitalized tissues must be excised in ♦♦ Surrounding skin may be erythematous and inflammatory.
order to obtain proper healing. Bluish in tuberculosis, excoriated in fecal fistula, pigmented
• Damage to blood vessels and nerves with associated in chronic sinuses and fistulas.
profuse bleeding is also observed. The bone or bones ♦♦ Induration is the feature of all chronic fistulas except
are shattered. tuberculosis.
• The wound is highly contaminated. Investigations
• There can be loss of soft/hard tissue.
• Treatment of crushed lacerated wound ♦♦ Fistulogram/Sinusogram using ultrafluid lipiodol or water
• Cleaning of wound soluble iodine dye.
• Removal of foreign bodies ♦♦ Biopsy from edge for tuberculosis and malignancy
• Debridement ♦♦ X-ray of the involved part.
• Hemostasis ♦♦ Proctoscopy of fistula.
• Closure in layers, i.e. primary closure Treatment
• Dressing
• Prevention of infection ♦♦ Cause should be treated
• Pain control ♦♦ Excision of sinus or fistula and specimen should be sent
• Follow up. for histology
♦♦ Antibiotics, anti-tubercular drugs, rest and adequate
Q.8. Write short note on sinus and fistula. drainage.
 (Jan 2012, 5 Marks) (Dec 2007, 3 Marks) Q.9. Write short note on complication of furunculosis. 
Or  (Dec 2007, 3 Marks)
Write short answer on sinus and fistula. Ans. Furunculosis is the reoccurring presence of pus-filled
 (June 2018, 3 Marks) sores, known as boils, on the skin.
Section 2:  General Surgery  265

Following are the complications of furunculosis: • Marjolin ulcer: As repeated breakdown of hyper-
• Carbuncles trophic scar occur it becomes Marjolin’s ulcer.
• Cellulitis
• Gangrene Q.12. Write short note on general factors affecting wound
• Necrotizing fasciitis healing. (Feb 2013, 5 Marks)
• Hidradenitis suppurativa (infection in group of hair Ans.
follicles) General Factors
• Lymphadenitis ♦♦ General factors affecting wound healing are as follows:
• Cavernous sinus thrombosis. • Age
Q.10. Describe wounds, their classification, wound healing • Malnutrition
and its treatment. (Jan 2011, 10 Marks) • Vitamin deficiency (Vitamin C, Vitamin A)
Ans. For description of wound, its classification and wound • Anemia
healing refer to Ans 4 of same chapter. • Malignancy
• Uremia
Treatment • Jaundice
♦♦ Wound is inspected and classified as per the type of • Diabetes, metabolic diseases
wounds. • HIV and immunosuppressive diseases
♦♦ If it is in the vital area, then: • Steroids and cytotoxic drugs
• The airway should be maintained. Age: In younger age group, wound healing is faster and better
• The bleeding, if present, should be controlled. while in elderly healing is delayed due to reduction in collagen
• Intravenous fluids are started. synthesis, epithelialization, growth factors and angiogenesis.
• Oxygen, if required, may be given. But final scar will be excellent in old individuals.
• Deeper communicating injuries and fractures, etc Malnutrition and vitamin deficiency: Adequate vitamin, trace
should be looked for. elements, fatty acids and proteins are essential for wound
♦♦ If it is an incised wound, then primary suturing is done healing. Vitamin A deficiency affects monocyte activation,
after thorough cleaning. inflammatory phase, collagen synthesis and growth factor
♦♦ If it is a lacerated wound, then the wound is excised and actions. Vitamin K deficiency affects synthesis of prothrombin
primary suturing is done. (ll), factors VII, IX and X. vitamin E, being an antioxidant
♦♦ If it is a crushed or devitalized wound, then there will be stabilizes the cell membrane. Vitamin C deficiency impairs
edema and tension in the wound. So after wound debride- collagen synthesis, fibroblast proliferation and angiogenesis;
ment or wound excision by excising all devitalized tissue, increases the capillary fragility and susceptibility for infection.
the edema is allowed to subside for 2–6 days. Then, delayed Zinc is an essential cofactor for RNA and DNA polymerase;
primary suturing is done. magnesium is a co-factor for synthesis of proteins and collagen;
♦♦ If it is a deep devitalised wound, after wound debridement copper is a required co-factor for cytochrome oxidase.
it is allowed to granulate completely. Later, if the wound Anemia: Hemoglobin less than 8% causes poor oxygenation of
is small secondary suturing is done. tissues preventing healing of the wounds.
♦♦ In a wound with tension, fasciotomy is done so as to Diabetes mellitus: In diabetic patients, wound healing is
prevent the development of compartment syndrome. delayed because of several factors such as microangiopathy,
♦♦ Antibiotics, fluid and electrolyte balance, blood transfusion, atherosclerosis, decreased phagocytic activity, proliferation of
tetanus toxoid (0.5 mL intramuscular to deltoid muscle), bacteria due to high blood sugar etc.
or anti-tetanus globulin (ATG) injection. Metabolic causes: Obesity causes hypoperfusion, reduced
♦♦ Wound debridement (wound toilet, or wound excision) microcirculation, increased wound tension, and hence prevents
is liberal excision of all devitalized tissue at regular wound healing.
intervals. Jaundice and uremia: Jaundiced and uraemic patients have poor
wound healing because fibroblastic repair is delayed.
Q.11. Write short note on complications of wounds. HIV and immunosuppressive diseases and malignancy: HIV
(Dec 2007, 5 Marks) and immunosuppression of varying causes, malignancy leads
Ans. Following are the complications of wounds: into poor wound healing.
• Infection: It is the most important complication which Drugs: Steroids interfere with activation of macrophages,
is responsible for delay in wound healing. Bacteria fibroblasts and angiogenesis in the early phase of healing
mainly remain endogenous. Based on pus culture and (proliferative). Nonsteroidal anti-inflammatory drugs (NSAIDs)
blood report antibiotics should be prescribed. decrease collagen production. Chemotherapeutic agents used
• Scar: Present due to infection. in oncology inhibit cellular proliferation, protein synthesis.
• Hypertrophic scar and Keloid formation due to Alcohol consumption decreases the phagocyte response and
altered collagen synthesis in wound healing process. pro-inflammatory cytokine release; diminishes host response
• Skin pigmentation and thus increasing the infection rate.
266   Mastering the BDS IIIrd Year  (Last 25 Years Solved Questions)

Q.13. Write short note on hypertrophic scar. ♦♦ Sometimes fistula ends internally as blind end.
 (May/June 2009, 5 Marks) ♦♦ It is common in children and early adolescent period.
Ans. In hypertrophic scar, there is hypertrophy of mature Equals in both sexes.
fibroblasts. Blood vessels are minimum in this condition. ♦♦ External orifice is very small with a dimple which becomes
• It can occur anywhere in the body. more prominent on dysphagia with tuck in appearance.
• Growth limits upto 6 months. ♦♦ Discharge is mucoid or mucopurulent.
• Hypertrophic scar is limited to scar tissue and does
not extend to normal skin. Treatment
• It is pale brown in color and is non-tender. Only surgical treatment is the choice.
• It is self limiting. ♦♦ Under general anesthesia, methylene blue is injected into
• It is common in wounds crossing tension lines, deep the tract. Probe is passed into the fistulous tract. Through
dermal burns, wounds heal by secondary intension. circumferential/elliptical incision around the fistula
opening, entire length of the tract is dissected until the
Complications
internal orifice. Care should be taken to safeguard carotids,
♦♦ Scar breaks repeatedly and cause infection and pain. jugular vein, hypoglossal nerve, glossopharyngeal nerve
♦♦ After repeated breakdown, it may turn into Marjolin’s ulcer. and spinal accessory nerve. Entire tract should be excised.
♦♦ Step-ladder dissection is done using two parallel incisions
Treatment
one below at lower part another above at upper part of the
♦♦ It is controlled by pressure garments or often revision neck, will make dissection easier and complete.
excision of scar and closure if required with skin graft.
Q.15. Describe briefly primary and secondary healing of
♦♦ Triamcinolone injection is second line of therapy.
wounds. (Jun 2010, 5 Marks)
Q.14. Discuss briefly branchial fistula. Ans. Primary Healing of Wound
 (May/Jun 2009, 5 Marks) ♦♦ In primary healing of wound edges are approximated by
Or surgical sutures.
Write in brief on branchial fistula.(Aug 2011, 5 Marks) • In the initial phase, there will be formation of blood
Ans. Branchial fistula is commonly a congenital lesion. clot, which helps to hold the parts of the wound
♦♦ It is persistent precervical sinus between 2nd branchial cleft together.
and 5th branchial cleft having opening in the skin at lower ♦♦ The tissue becomes edematous and an inflammatory pro-
l/3rd of neck on the inner margin of sternocleidomastoid cess starts, with the infiltration of polymor-phonuclear
muscle, often ends as a sinus just proximal to the posterior neutrophils (PMN) and lymphocytes into the area.
pillar of fauces behind tonsil which is also the site of inner ♦♦ The tissue debris collected in the wound are cleared either
opening when presents as a fistula. by the process of phagocytosis or by their lysis with the help
♦♦ Fistula runs between the structures related to second and of proteolytic enzymes, liberated by the inflammatory cells.
3rd branchial arches. From external opening at skin below, ♦♦ Once the tissue debris are cleared, granulation tissue
it runs in subcutaneous plane to pierce deep fascia at level forms that replaces the blood clot in the wound, and it
of thyroid cartilage; to travel between 2nd arch artery usually consists of young blood capillaries, proliferating
and third arch artery behind posterior digastric belly and fibroblasts, PMN and other leukocytes.
stylohyoid; outer to stylopharyngeus, hypoglossal and ♦♦ The epithelium at the edge of the wound starts to prolif-
glossopharyngeal nerves; perforates superior constrictor erate and gradually, it covers the entire wound surface.
to reach the internal opening. ♦♦ Finally, the healing process is complete with progressive
♦♦ Occasionally, acquired branchial fistula can occur due to increase in the amount of dense collagen bundles and
rupture of or after drainage of infected branchial cyst or decrease in the number of inflammatory cells in the area.
incomplete excision of the cyst tract. This type of fistula
is located outside at skin at the level of upper third of Secondary Healing of Wound
sternomastoid muscle.
♦♦ When the opposing margins of the wound cannot be
Clinical Features approximated together by suturing, the wound fills in
♦♦ It is a persistent second branchial cleft with a communication from the base with the formation of a larger amount
outside to the exterior. It is commonly a congenital fistula. of granulation tissue, such type of healing of the open
♦♦ Occasionally, the condition is secondary to incised infected wound is known as healing by “secondary intention” or
branchial cyst. “secondary healing”.
♦♦ Often it is bilateral. ♦♦ The secondary healing occurs essentially by the same
♦♦ External orifice of the fistula is situated in the lower third of process as seen in the primary healing, the only difference
the neck near the anterior border of the sternomastoid muscle. is that a more severe inflammatory reaction and an
♦♦ Internal orifice is located on the anterior aspect of the exuberant fibroblastic and endothelial cell proliferation
posterior pillar of the fauces, just behind the tonsils. occur in the later.
Section 2:  General Surgery  267

♦♦ In secondary healing, once the blood clot is removed, Drugs: Steroids interfere with activation of macrophages,
the granulation tissue fills up the entire area and the fibroblasts and angiogenesis in the early phase of healing
epithelium begins to grow over it, until the wound surface (proliferative). Non-steroidal anti-inflammatory drugs
is completely epithelized. (NSAIDs) decrease collagen production. Chemotherapeutic
♦♦ Later on, the inflammatory exudates disappear slowly and agents used in oncology inhibit cellular proliferation, protein
the fibroblasts produce large amounts of collagen. synthesis. Alcohol consumption decreases the phagocyte
♦♦ Most of the healing processes occurring due to secondary response and proinflammatory cytokine release; diminishes
intention, result in scar formation at the healing site. host response and thus increasing the infection rate.
However, in the oral cavity these are rare.
Local Factors
Q.16. Write briefly on factors influencing wound healing. 
♦♦ Local factors affecting wound healing are as follows:
 (Aug 2011, 5 Marks)
• Infection
Ans. • Presence of necrotic tissue and foreign body
General Factors • Poor blood supply
♦♦ General factors affecting wound healing are as follows: • Venous or lymph stasis
• Age • Tissue tension
• Malnutrition • Hematoma
• Vitamin deficiency (Vitamin C, Vitamin A) • Large defect or poor apposition
• Anemia • Recurrent trauma
• Malignancy • X-ray irradiated area
• Uremia • Site of wound
• Jaundice • Underlying diseases such as osteomyelitis and
• Diabetes, metabolic diseases malignancy
• HIV and immunosuppressive diseases • Mechanism and type of wound-incised/lacerated/
• Steroids and cytotoxic drugs crush/avulsion
Age: In younger age group, wound healing is faster and better • Tissue hypoxia locally reduces macrophage and
while in elderly healing is delayed due to reduction in collagen fibroblast activity
synthesis, epithelialization, growth factors and angiogenesis. Wound infection: Infection prolongs inflammatory phase,
But final scar will be excellent in old individuals. releases toxins and utilizes vital nutrients thereby prevents
Malnutrition and Vitamin deficiency: Adequate vitamin, wound epithelialization. The β-hemolytic streptococci more
trace elements, fatty acids and proteins are essential for wound than 105 per gram of tissue prevent wounds healing. Formation
healing. Vitamin A deficiency affects monocyte activation, of biofilms on the wound surface by microorganisms prevents
inflammatory phase, collagen synthesis and growth factor wound healing.
actions. Vitamin K deficiency affects synthesis of prothrombin
(ll), factors VII, IX and X. Vitamin E, being an antioxidant Presence of necrotic tissue and foreign body: Necrotic tissue
stabilizes the cell membrane. Vitamin C deficiency impairs and foreign bodies such as sutures cause intense inflammatory
collagen synthesis, fibroblast proliferation and angiogenesis; reaction and infection.
increases the capillary fragility and susceptibility for infection. Poor blood supply: Improper blood supply to wound delays
Zinc is an essential cofactor for RNA and DNA polymerase; wound healing.
magnesium is a co-factor for synthesis of proteins and collagen;
Tissue tension: It affects quantity, aggregation and orientation
copper is a required co-factor for cytochrome oxidase.
of collagen fibers.
Anemia: Hemoglobin less than 8% causes poor oxygenation of
tissues preventing healing of the wounds. Hematoma: It precipitates infection and delays wound healing.
Diabetes mellitus: In diabetic patients, wound healing is
Hypoxia: Hypoxia prevents fibroblast proliferation and collagen
delayed because of several factors such as microangiopathy,
synthesis; it also promotes bacterial invasion into the wound.
atherosclerosis, decreased phagocytic activity, proliferation of
bacteria due to high blood sugar, etc. Site of wound: Movement of wound area delays wound
Metabolic causes: Obesity causes hypoperfusion, reduced healing. e.g. wound over the joints and back has poor healing.
microcirculation, increased wound tension, and hence prevents
X-ray irradiated area: Both external radiotherapy or ionizing
wound healing.
radiation cause endarteritis, fibrosis and delay in wound
Jaundice and uremia: Jaundiced and uremic patients have poor
healing. Radiation may itself cause local tissue necrosis, sepsis
wound healing because fibroblastic repair is delayed.
and hypoxia.
HIV and immunosuppressive diseases and malignancy: HIV
and immunosuppression of varying causes, malignancy leads Poor apposition: Poor apposition leads to the infection in the
into poor wound healing. wound area which delays healing.
268   Mastering the BDS IIIrd Year  (Last 25 Years Solved Questions)

Q.17. Define and describe differentiating features of incised – Hematoma: Caused due to injury. It consist of
wound and lacerated wound. (Jan 2012, 5 Marks) reddish plasmatic fluid which can be aspirated
Ans. and if it get infected, then pus is formed.
– Closed blunt injury.
Incised wound Lacerated wound – Puncture wounds and bites.
– Abrasion: Occurs due to shearing of skin where
This wound is caused by sharp They are caused by blunt objects
objects such as knife, blade, such as fall on a stone or due to surface is rubbed off.
glass, etc road traffic accidents – Traction and avulsion injury.
– Crush injury: It is caused in wars, road traffic
Edges of wound are sharp Edges of wound are jagged
accidents, tourniquet.
Contamination is less Contamination is more - Muscle ischaemia is present.
Does not involve deeper structures Involve deeper structures - Presence of gangrene and loss of tissue.
Crushing of tissue does not occur Crushing of tissue is present – War wounds and gunshot injuries.
– Injuries to bones and joints, may be open or
Only primary suturing is done Wound excision and primary
closed.
suturing is done
– Injuries to nerves, either clean cut or crush.
– Injuries to arteries and veins (major vessels).
Q.18. Write short note on causes of delayed wound healing.
– Injury to internal organs, may be of penetrating
(Aug 2012, 5 Marks)
or non-penetrating (blunt) types.
Or – Penetrating wounds.
Write short note on delayed wound healing. • Classification based on Thickness of the Wound
 (June 2015, 5 Marks) – Superficial wound
Ans. For the causes of delayed wound healing refer to Ans 16 – Partial thickness wound
in detail – Full thickness wound
– Deep wounds
Q.19. Discuss the etiology, clinical features and management
– Complicated wounds
of wounds. (Dec 2012, 10 Marks)
– Penetrating wound.
Ans. A wound is the break in the integrity of skin or tissues • Classification based on Involvement of Structures
often which may be associated with disruption of – Simple wounds
structure and function. – Combined wounds.
• Etiology and Clinical Features • Classification based on the Time Elapsed
– Tidy wounds: – Acute wound
- They are wounds such as surgical incisions – Chronic wound.
and wounds caused by sharp objects. • Classification of Surgical Wounds.
- It is incised, clean, healthy wound without – Clean wound
any tissue loss. – Clean contaminated wound
– Untidy wounds: – Contaminated wound
- They are due to crushing, tearing, avulsion, – Dirty infected wound
devitalised injury, vascular injury, multiple
irregular wounds, burns. Management
- Fracture of the underlying bone may be ♦♦ Wound is inspected and classified as per the type of
present. wounds.
- W ound dehiscence, infection, delayed ♦♦ If it is in the vital area, then:
healing are common. • The airway should be maintained.
• Classification based on Type of Wound • The bleeding, if present, should be controlled.
– Clean incised wound: It is a clean cut wound with • Intravenous fluids are started.
linear edge. • Oxygen, if required, may be given.
– Lacerated wounds: The lacerated wounds are • Deeper communicating injuries and fractures, etc.
caused by the blunt objects such as fall on the should be looked for.
stone or due to road traffic accident. ♦♦ If it is an incised wound, then primary suturing is done
- The edges of lacerated wound are *jagged. after thorough cleaning.
- The injury involves skin or subcutaneous ♦♦ If it is a lacerated wound, then the wound is excised and
tissue or sometimes-deeper structures also. primary suturing is done.
- In the lacerated wound there is crushing of ♦♦ If it is a crushed or devitalized wound, then there will
tissue due to blunt nature of the object. be edema and tension in the wound. So after wound de-
– Bruising and contusion: It is a minor soft tissue bridement or wound excision by excising all devitalised
injury with discoloration and hematoma tissue, the oedema is allowed to subside for 2–6 days. Then
formation without skin break. delayed primary suturing is done.

Q19. *Jagged= Unevenly cut or torn.


Section 2:  General Surgery  269

♦♦ If it is a deep devitalised wound, after wound debridement Q.21. Describe pathophysiology of primary and secondary
it is allowed to granulate completely. Later, if the wound is healing. Enumerate their clinical advantages and dis­
small secondary suturing is done. If the wound is large a advantages in tabular form. List when assault becomes
split skin graft (Thiersch graft) is used to cover the defect. a cognizable offence. (June 2014, 10 Marks)
♦♦ In a wound with tension, fasciotomy is done so as to pre- Ans. Pathophysiology of primary and secondary healing
vent the development of compartment syndrome. means primary and secondary healing of wounds. For
♦♦ Vascular or nerve injuries are dealt with accordingly. details, refer to Ans 15 of same chapter.
♦♦ Vessels are sutured with 6-zero polypropylene nonabsorb-   Assault is the act of creating apprehension of an
able suture material. If the nerves are having clean-cut imminent harmful or offensive contact with a person.
wounds then it can be sutured primarily with polypropyl- Generally, cognizable offence means a police officer has
ene 6-zero or 7-zero suture material. If there is difficulty the authority to make an arrest without a warrant and
in identifying the nerve ends, or if there are crushed cut the police is also allowed to start an investigation with
ends of nerves, then marker stitches are placed using silk or without the permission of a court.
at the site and later secondary repair of the nerve is done. Following are the cognizable offences:
♦♦ Internal injuries has to be dealt with accordingly. Fractured • All grievous hurts
bone is also identified and properly dealt with. • Simple hurt by dangerous weapon
♦♦ Antibiotics, fluid and electrolyte balance, blood transfu- • Murder
sion, tetanus toxoid, antitetanus globulin (ATG) injection. • Culpable homicide
♦♦ Wound debridement is liberal excision of all devitalized • Causing death by rash or negligence act.
tissue at regular intervals until healthy, bleeding, vascular • Dowry death
tidy wound is created. • Abetment to suicide
Q.20. Enumerate differences between sinus and fistula. • Attempt to murder
(June 2014, 2 Marks) • Attempt to commit suicide
Or • Rape.
Write differences between sinus and fistula.
 (Apr 2017, 2 Marks) Primary Healing
Or Advantages Disadvantages
Differences between sinus and fistula. Easy for the patient to manage the wound Risk of wound infection
 (Jan 2017, 10 Marks)
Rapid return of function of the wounded
Ans. part final cosmetic result is superior
Sinus Fistula Tissue heal closer to normal length
It is a blind track-lined by It is an abnormal communication Blood supply is restored soon
granulation tissue leading from between the lumen of one viscus
an epithelial surface into the to another or the body surface or
Secondary healing
surrounding tissues between the vessels
Congenital cause is preauricular Congenital cause is branchial Advantages Disadvantages
sinus fistula, trachea-esophageal Wound infection is Daily dressing changes are required until the
fistula, congenital AV fistula and virtually impossible wound is healed, which may take some time, and
umbilical fistula the final result is a cicatrix that may be unsightly
Acquired causes are actinomyco- Acquired causes are traumatic, Final cosmetic result is poor. Granulation results
sis, tuberculosis, pilonidal sinus, inflammatory and malignancy in a broader scar
chronic osteomyelitis, medial,
Healing process can be slow due to presence
mental sinus
of drainage from infection
In this a single epithelialised sur- In this, two epithelialized surfaces
face is involved are involved.
Extension of sinus is from cavity Extension of fistula is from cavity Q.22. Enumerate causes for chronicity of sinus and fistula.
to cavity to outside. (Feb 2014, 3 Marks)
In oral cavity, sinus is a drainage Fistula is a passage between Ans. Following are the causes for chronicity of sinus and
passage through bone piercing to hollow cavities, e.g. oroantral fistula:
the oral mucosa, e.g. in infections fistula . • A foreign body or necrotic tissue underneath. e.g.
such as periapical abscess.
suture, sequestrum.
On inspection the location of vari- On inspection the location of
• Insufficient or non-dependent drainage
ous sinus is: fistula is:
• Persistent obstruction in lumen, e.g. In fecal fistula,
- Preauricular sinus is at root of - Branchial fistula: Sternomas-
helix of ear toid anterior border biliary fistula
- Sinus of TB is located at neck - Parotid fistula: Parotid region • Lack of rest, persistent infection
It cannot be surgically created. It can be surgically created for • Wall become lined with epithelium and endothelium
therapeutic reasons. • Dense fibrosis prevents contraction and healing
270   Mastering the BDS IIIrd Year  (Last 25 Years Solved Questions)

• Specific infections: Tuberculosis and actinomycosis ♦♦ Unhealthy granulation tissue


• Presence of malignant disease, post – irradiation. • This is pale in color with purulent discharge.
Q.23. What is primary and secondary healing, advantage of • Floor of granulation tissue is covered with slough.
• Its edges are inflamed and edematous.
one over other. What are the injuries that constitute
• It is a type of a spreading ulcer.
cognizable offence in IPC. (Apr 2015, 10 Marks)
♦♦ Unhealthy, pale, flat, granulation tissue
Ans. Primary healing: It occurs in a clean incised or surgical
• It is seen in chronic non-healing ulcer.
wound. Edges of the wound are approximated by
• Exuberant granulation tissue
sutures. Epithelial regeneration is more as compared
• It occurs in sinus where granulation tissue protrude
to fibrosis. Wound healing takes place rapidly with
out of orifice of sinus as proliferating mass.
complete closure. Scar will be linear, supple and smooth.
• Mostly associated with retained foreign body in sinus

Secondary healing: It occurs in a wound with extensive cavity.
soft tissue loss like in major trauma, burn, wound with ♦♦ Pyogenic granuloma
sepsis. Healing by secondary intention occur slowly • Here granulation tissue protrudes from an infected
with fibrosis. This also leads to the formation of wide wound or ulcer bed and presents a well localized, red
scar which is hypertrophied and contracted. This can swelling which bleed on manipulation.
also lead to disability. • It is treated by surgical excision.
For primary and secondary healing in details, refer to Q.25. Write briefly on cognizable offence.
Ans 15 of same chapter.  (Jan 2016, 2 Marks)
For advantages of one another, refer to Ans 21 of same Ans. Generally, cognizable offence means a police officer has
chapter. the authority to make an arrest without a warrant and
to start an investigation with or without the permission
Injuries that Constitute Cognizable Offence in IPC
of a court.
Cognizable offence means an offence for which a police officer • In cognizable offence, police has to record information
may arrest without a warrant. in writing.
• The police can file a First Information Report (FIR)
Following are the cognizable offences:
only for cognizable offences.
♦♦ Murder (S.302) • In cognizable offences, police can start investigation
♦♦ Causing death by rash or negligent act (S. 304A) without the order of magistrate.
♦♦ Dowry death (S.304B) • Cognizable offence is more serious and carry a
♦♦ Abetment of suicide (S. 306) sentence of 3 years or more.
♦♦ Attempt to commit suicide (S. 309)
♦♦ Causing miscarriage without women’s consent (S. 313) Q.26. Write difference between hypertrophic scar and keloid.
♦♦ Grievous hurt (S. 325, 326)  (Mar 2016, 3 Marks)
♦♦ Rape (S. 376) Ans. Following are differences between hypertrophic scar and
keloid:
♦♦ Theft (S. 379).
Features Hypertrophic scar Keloid
Q.24. Write short note on granulation tissue. Genetic Yes No
 (Feb 2015, 5 Marks) predisposition
Ans. Granulation tissue is the proliferation of newer capillaries Site of Chest wall, upper arm, Anywhere in the
along with fibroblasts which are intermingled with RBCs occurrence lower neck, ear body, common in
and WBCs with thin fibrin cover over it. flexor surfaces
Growth Continues to grow without Growth limits for 6
time limit. Extends to months. Limited to
Types of Granulation Tissue
normal skin scar tissue only
Treatment Poor response Good response to
Following are the types of granulation tissue: steroids
♦♦ Healthy granulation tissue Recurrence Very high ls uncommon
• It results in an healing ulcer. Collagen 2 0 t i m e s m o r e t h a n 3–6 times more than
• It has got a sloping edge with serous discharge. synthesis normal skin (Type III thick) normal skin (Type III
• It bleeds on manipulation. fine collagen)
• Skin graft takes well with healthy granulation tissue. Relation of size No relation. Small healed Related to size of
• Healthy granulation tissue consists of 5Ps i.e. pink, o f i n j u r y a n d scar can form large keloid injury and duration of
punctuate hemorrhage, pulseful, painless, pin head lesion healing
Contd...
Section 2:  General Surgery  271

Contd... Secondary Wound Healing


Features Hypertrophic scar Keloid ♦♦ It occurs in a wound with extensive soft tissue loss like in
Age Adolescents, middle age Children major trauma, bums and wound with sepsis.
Sex Common in females Equal in both ♦♦ It heals slowly with fibrosis.
Race More in blacks (15 times) No racial relation ♦♦ It leads into a wide scar, often hypertrophied and con-
Structure Thick collagen with Fine collagen with tracted.
increased epidermal increased alpha actin ♦♦ It may lead into disability.
hyaluronic acid ♦♦ Re-epithelialization occurs from remaining dermal ele-
Features Vascular, tender, itching Not vascular, non- ments or wound margins.
tender, no itching
Natural history Progressive Shows regression Mechanism of Primary Healing of Wound
Problems Hyperesthesia, Not much ♦♦ In primary healing of wound, edges are approximated by
ulceration surgical sutures.
Q.27. Describe differentiating features of ulcer and wound. ♦♦ In the initial phase, there will be formation of blood clot,
which helps to hold the parts of the wound together.
 (Jan 2017, 4 Marks)
♦♦ Within 10 to 15 minutes of injury vasodilatation occur
Ans. Following are the differences between ulcer and wound:
and there is increased capillary permeability mediated
Ulcer Wound by vasoactive substances released by damage cells and
An ulcer is break in the continuity A wound is a break in the integrity clot breakdown.
of covering epithelium either skin of the skin or tissues often, which ♦♦ The tissue becomes edematous and an inflammatory
or mucous membrane due to may be associated with the process starts, with the infiltration of polymorphonuclear
molecular death. disruption of the structure and neutrophils (PMN) and lymphocytes into the area.
function
♦♦ The tissue debris collected in the wound are cleared either
In ulcer, there is disruption or In wound, there is disruption of by the process of phagocytosis or by their lysis with the
break in continuity of any lining soft tissues or bone or internal help of proteolytic enzymes, liberated by the inflamma-
may be skin, mucous membrane organ
tory cells.
and others.
♦♦ Once the tissue debris are cleared, granulation tissue
Ulcer is one of the type of a Wound is a break in the integrity forms that replaces the blood clot in the wound, and it
wound of the skin or tissues
usually consists of young blood capillaries, proliferating
In an ulcer, the primary tissue In a wound, the primary tissue fibroblasts, PMN and other leukocytes.
breakdown is internal breakdown is caused by a force ♦♦ The epithelium at the edge of the wound starts to prolif-
originating from the external world
erate and gradually, it covers the entire wound surface.
Ulceration on the skin is caused by Wound is caused by injury to ♦♦ Finally, the healing process is complete with invasion of
pressure or circulatory problems the skin. wound area by fibroblast on 3rd day, progressive increase
that impede the blood flow in the
in the amount of dense collagen bundles on 5th day and
area and the surrounding tissues.
formation of scar tissue with scanty cellular and vascular
Ulcers seem more prone to host Wounds seem less prone to host elements, some inflammatory cells and epithelialized
biofilms biofilms as compared to ulcer
surface by 4th week.
Ulcers are often dressed once or Major wounds may require daily
twice weekly at walk-in clinics with changes. Mechanism of Secondary Healing of Wound
complex dressing techniques.
♦♦ When the opposing margins of the wound cannot be ap-
Mortality directly from ulcers is Mortality directly from wounds is proximated together by suturing, the wound fills in from
rare and therefore ulcers are of more and therefore wounds are the base with the formation of a larger amount of granu-
less clinical concern of more clinical concern
lation tissue, such type of healing of the open wound is
Q.28. Describe primary and secondary wound healing. Dis­ known as healing by “secondary intention” or “secondary
healing”.
cuss mechanism and various phases of wound healing.
♦♦ The secondary healing occurs essentially by the same
 (Apr 2017, 15 Marks)
process as seen in the primary healing, the only differ-
Ans. ence is that a more severe inflammatory reaction and an
Primary Wound Healing exuberant fibroblastic and endothelial cell proliferation
occur in the later.
♦♦ It occurs in a clean Incised wound or surgical wound. ♦♦ In secondary healing, once the blood clot is removed, pro-
♦♦ Wound edges are approximated with sutures. There is liferation starts, the granulation tissue fills up the entire
more epithelial regeneration than fibrosis. Wound heals area, contraction of wound occur and the epithelium begins
rapidly with complete closure. to grow over it, until the wound surface is completely
♦♦ Scar will be linear, smooth and supple. epithelized.
272   Mastering the BDS IIIrd Year  (Last 25 Years Solved Questions)

♦♦ Later on, the inflammatory exudates disappear slowly and ♦♦ During first week, fibroblasts begin secreting glycosa-
the fibroblasts produce large amounts of collagen. Most of minoglycans and proteoglycans, the ground substance
the healing processes occurring due to secondary intention, for granulation tissue, as well as collagen, in response to
result in scar formation at the healing site. However, in macrophage synthesized FGF and TGF-β, as well as PDGF.
the oral cavity scars are very rare. Fibroblasts grow and form a new, provisional extracellular
matrix (ECM) by excreting collagen and fibronectin.
Phases of Wound Healing ♦♦ There is re-epithelialization of the epidermis, in which
Following are the phases of wound healing: epithelial cells proliferate and reaches to the wound bed,
providing cover for the new tissue. This process starts
Inflammatory Phase from the basement membrane, if the basement membrane
♦♦ It begins at the time of injury and lasts 2–4 days. remains intact, otherwise the process initiates from the
♦♦ This phase is characterized by hemostasis and inflam- wound edges.
mation. ♦♦ Fibroblasts produce mainly type III collagen in this phase.
♦♦ First hemostasis occurs and there is formation of the ♦♦ As fibrin clot and provisional matrix degrades, there is
platelet plug. deposition of granulation tissue (ground substance, col-
♦♦ Injury to vascular tissue initiates the extrinsic coagulation lagen, capillaries), which continues until the wound is
pathway by releasing intracellular calcium and tissue covered. Granulation tissue formed during this phase, is
factor that activate factor VII. Resulting fibrin plug leads important in secondary wound healing.
to hemostasis which is aided by reflex vasoconstriction. Remodeling Phase
♦♦ Now this plug acts as a lattice for aggregation of platelets,
this is the indication of early inflammatory phase. ♦♦ Remodeling continues for 6–12 months after injury.
♦♦ In inflammatory phase, both bacteria and debris are ♦♦ In this, there is maturation of collagen by cross linking
phagocytosed and removed, now the factors are released and realignment of collagen fibers along the line of tension
that cause the migration and division of cells involved in which is responsible for tensile strength of scar. Vascularity
the proliferative phase. of wound reduces. Fibroblast and myofibroblast leads to
♦♦ Collagen which were exposed at the time of wound for- wound contraction. Here Type III collagen is replaced by
mation causes activation of the clotting cascade (both the Type I collagen causing maturity of collagen. Ratio of Type
intrinsic and extrinsic pathways), initiating the inflam- I collagen to Type III collagen is 4:1.
matory phase. ♦♦ Early extracellular matrix consists of fibronectin and col-
♦♦ Injured tissues, via activated phospholipase A, catalyze lagen Type III; eventually it consists of glycosaminoglycans
arachidonic acids to produce potent vasoconstrictors and proteoglycans; final matrix consists of Type I collagen.
thromboxane A2 and prostaglandin 2-alpha, which is ♦♦ Strength of scar is 3% in one week, 20% in 3 weeks, 80% in
collectively known as eicosanoids. This initial response 12 weeks. Finally, matured scar is acellular and avascular.
limits hemorrhage. Q.29. Classify wounds and discuss management of different
♦♦ After a short period of time capillary vasodilatation occurs types of wounds. (Jan 2018, 20 Marks)
which is due to local histamine release, and the cells of Ans. For classification of wounds refer to Ans 4 of same
inflammation migrate to the wound bed. chapter.
♦♦ Chemical mediators, i.e. platelets release platelet-derived
growth factor (PDGF) and and transforming growth factor Management of Different Types of Wounds
beta (TGF-β) from their alpha-granules attract neutrophils ♦♦ Wound is inspected and classified as per the type of
and macrophages. wounds.
♦♦ Now, neutrophils scavenge for bacteria and foreign debris ♦♦ If wound is in the vital area, then:
and macrophages continue to release growth factors to • The airway should be maintained.
attract fibroblasts and enter in the next phase of wound • The bleeding, if present, should be controlled.
healing. • Intravenous fluids are started.
• Oxygen, if required, may be given.
Proliferative Phase
• Deeper communicating injuries and fractures, etc.
♦♦ It starts from day 3 and last for 3 to 6 weeks. It overlaps should be looked for.
with the inflammatory phase. This phase is characterized ♦♦ If it is an incised wound, then primary suturing is done
by angiogenesis, collagen deposition, granulation tissue after thorough cleaning.
formation, and epithelialization. ♦♦ If it is a lacerated wound, then the wound is excised and
♦♦ Fibroblasts initiate angiogenesis, epithelialization, and col- primary suturing is done.
lagen formation. Fibroblasts migrate inside from wound ♦♦ If it is a crushed or devitalized wound there will be edema
margins over the fibrinous matrix which is established and tension in the wound. So after wound debridement
during this phase. or wound excision by excising all devitalized tissue, the
Section 2:  General Surgery  273

edema is allowed to subside for 2—6 days. Then delayed The most common cause is dental infection of second or third
primary suturing is done. molar teeth
♦♦ If it is a deep devitalized wound, after wound debridement
it is allowed to granulate completely. Later, if the wound is Precipitating Factors
small secondary suturing is done. If the wound is large a ♦♦ Tooth extraction
split skin graft (Thiersch graft) is used to cover the defect. ♦♦ Submandibular sialadenitis
♦♦ In a wound with tension, fasciotomy is done so as to pre- ♦♦ Trauma
vent the development of compartment syndrome. ♦♦ Peritonsillar abscess
♦♦ Vascular or nerve injuries are dealt with accordingly. Ves- ♦♦ Upper respiratory infection
sels are sutured with 6-zero polypropylene nonabsorbable ♦♦ Interventions like endotracheal intubation.
suture material. If the nerves are having clean, cut wounds
it can be sutured primarily with polypropylene 6-zero or Predisposing Factors
7-zero suture material. If there is difficulty in identifying ♦♦ Diabetes mellitus
the nerve ends or if there are crushed cut ends of nerves ♦♦ Chemotherapy
then marker stitches are placed using silk at the site and ♦♦ Oral cancer
later secondary repair of the nerve is done. ♦♦ Alcohol
♦♦ Internal injuries (intracranial by craniotomy, intrathoracic ♦♦ Neutropenia.
by intercostal tube drainage, intra-abdominal by lapa-
rotomy) has to be dealt with accordingly. Fractured bone Microscopic Organisms
is also identified and properly dealt with. ♦♦ As Ludwig’s angina is of dental origin streptococci or
♦♦ Antibiotics, fluid and electrolyte balance, blood transfu- mixed oral flora are the most commonly reported micro-
sion, tetanus toxoid (0.5 mL intramuscular to deltoid organisms.
muscle), or antitetanus globulin (ATG) injection. ♦♦ Presence of staphylococci, E. coli, Pseudomonas and anaer-
♦♦ Later definitive management is done with: Wound debride- obes including bacteroides and Peptostreptococcus, Prevotella
ment (wound toilet, or wound excision) is liberal excision species have also been isolated.
of all devitalized tissue at regular intervals (of 48–72 hours) ♦♦ Role of anaerobes as primary or synergistic organisms
until healthy, bleeding, vascular tidy wound is created. should not be omitted in the culture.

Clinical Features
2. Acute Infections ♦♦ Presence of diffuse painful swelling with woody brawny
induration of the mouth and anterior neck. Swelling is
Q.1. Describe briefly Ludwig’s angina.  non-fluctuant but with redness and tenderness. Bilateral
 (Feb 2002, 5 Marks) (Nov 2008, 5 Marks) submandibular edema with marked tenderness on palpa-
tion at suprahyoid area with bull’s neck appearance.
Or
♦♦ Toxic features such as fever, tachycardia, tachypnea is
Write short note on Ludwig’s angina. common.
 (Aug 2012, 5 Marks) (Aug 2011, 5 Marks) ♦♦ Difficulty in speech, earache, drooling of saliva and putrid
 (June 2010, 5 Marks) (Apr 2010, 5 Marks) halitosis.
 (Dec 2009, 5 Marks) (Sep 2008, 3 Marks) ♦♦ Involvement of connective tissues, muscles and fascial
 (Apr 2008, 5 Marks) (Mar 2007, 5 Marks) spaces but not glandular structures.
 (Apr 2007, 5 Marks) (Sep 2006, 6 Marks) ♦♦ Infection spread via fascial planes in continuity not by
 (Oct 2003, 10 Marks) (Feb 2013, 5 Marks) lymphatics; no lymph node enlargement.
 (Apr 2017, 4 Marks) ♦♦ Edema of the tongue with pushing against palate (eleva-
Or tion) upwards and backwards causing airway obstruction,
dysphagia and odynophagia.
Define, describe clinical feature and principles of treat­
♦♦ Stridor, respiratory distress and cyanosis may develop due
ment of Ludwig’s angina (Jan 2017, 5 Marks)
to edema of tongue and larynx.
Or
Investigations
Write short answer on Ludwig’s angina. ♦♦ CT scan or MRI is useful to identify airway block, fluid
 (June 2018, 3 Marks) collection and presence of gas.
Ans. ♦♦ Ultrasound neck is simpler method to identify same.
♦♦ Total count, blood sugar, chest X-ray and often blood gas
Ludwig’s Angina analysis (in severe cases) is done.
It is a rapidly progressive polymicrobial cellulitis of the Differential Diagnosis
sublingual and submandibular spaces involving the floor of the ♦♦ Angioneurotic edema
mouth and suprahyoid area on both sides of the neck. ♦♦ Sublingual hematoma
274   Mastering the BDS IIIrd Year  (Last 25 Years Solved Questions)

♦♦ Sialadenitis ♦♦ Blunt dissection through the skin, superficial fascia, platy-


♦♦ Lymphadenitis. sma, deep fascia and mylohyoid muscle is done to reach the
sublingual space. The sublingual space may alternatively
Complications be drained intraorally by an incision in the floor of the
♦♦ Laryngeal edema can occur due to spread of inflammation mouth if the mouth opening is adequate.
to glottis submucosa via stylohyoid tunnel. It may require ♦♦ A drain is inserted through all these layers and stabilized.
emergency tracheostomy to maintain the respiration. ♦♦ There may be no pus at all in most cases of Ludwig’s an-
♦♦ Mediastinitis due to spread of infection into mediastinum; gina. Pus may only be seen at later stages.
aspiration pneumonia. ♦♦ A loose dressing is placed which is changed everyday after
♦♦ Septicemia. careful cleaning of the drain.
♦♦ Spread of infection into the parapharyngeal space leads to
Hydration of the Patient
thrombosis of the internal jugular vein which may extend
above into the sigmoid sinus which may be fatal. ♦♦ Pyrexia and dysphagia may lead to dehydration of the
patient.
Management ♦♦ It is necessary to put the patient on IV fluids.
Main principles of treatment of Ludwig’s angina are:
Removal of Cause
♦♦ Airway maintenance
♦♦ Parenteral antibiotics ♦♦ Once the patient is stabilized and the trismus relieved to
♦♦ Surgical decompression of tissues an extent, the offending tooth may be removed.
♦♦ Hydration of patient Q.2. Write short note on carbuncle. (Aug 2012, 5 Marks)
♦♦ Removal of cause  (Dec 2010, 5 Marks) (Nov 2008, 5 Marks)
Airway Maintenance  (Apr 2007, 5 Marks) (Sep 2007, 2.5 Marks)
 (Mar 2007, 2.5 Marks) (Mar 2003, 10 Marks)
♦♦ Edema of the glottis is what leads to airway obstruction  (Mar 2001, 10 Marks) (June 2015, 5 Marks)
and death of the patient, if untreated. Maintaining a patent  (Apr 2017, 4 Marks)
airway is thus of prime importance. Or
♦♦ Intubation is done by an experienced anesthetist.
♦♦ Bulging of the posterior pharyngeal wall makes intubation
Write in short about carbuncle. (Jan 2018, 5 Marks)
difficult. Perforation may lead to aspiration of pus, if an Or
endotracheal tube is forced in blindly.
Write brief notes on carbuncle. (Apr 2017, 2 Marks)
♦♦ A tracheotomy or cricothyroidotomy may be advisable. Or
Tracheostomy is usually difficult because the anatomical Define, describe clinical features and principles of
landmarks on the neck are not easily identifiable with the treatment of carbuncle. (Jan 2017, 5 Marks)
swelling.
Ans. Word meaning of carbuncle is charcoal. It is caused by
Parenteral Antibiotics extensive infectious gangrene of the adjacent hair follicle
and subcutaneous tissue by Staphylococcus aureus.
♦♦ Penicillin is the antibiotic of choice.
• It commonly occur in diabetic patients.
♦♦ Others such as amoxycillin, cloxacillin may also be used.
• Nape of neck is the most common site followed by
♦♦ Metronidazole is given against anaerobic organisms.
back and shoulder region. Skin of the site is coarse
♦♦ Gentamycin can also be given along with penicillin for
and has poor vascularity.
coverage of Pseudomonas.

Surgical Decompression Pathology


Infection
♦♦ It is usually preferred under LA instead of GA. The main
aim is to relieve the pressure within the tissues for the Development of small vesicles
edema to reduce. Also, the pressure within the tissues
compresses the blood vessels preventing the penetration of Sieve like pattern
antibiotics in the area of interest. Thus decompression im-
proves vascularity and potentiates the action of antibiotics. Red indurated skin with discharging pus
♦♦ If there is pus, it provides a channel for the drainage of pus.
Many fuse together to form a central necrotic
♦♦ Pus obtained from a drainage may be sent for culture and
ulcer with peripheral fresh vesicle looking like
sensitivity testing.
a rosette or cribriform pattern
♦♦ Decrease in edema reduces the respiratory embarrassment.
♦♦ Ethyl chloride spray may be used or LA may be injected
Skin become black due to blockage of cutaneous vessels
locally in the area of incision.
♦♦ Bilateral submandibular incisions with a midline submen-
Disease spread to adjacent skin rapidly
tal incision may be placed.
Section 2:  General Surgery  275

Clinical Features Q.3. Write the management of carbuncle. 


 (Dec 2012, 5 Marks)
♦♦ It is common in diabetics after 40 years of age
♦♦ Surface is red and looks like a red-hot coal. Or
♦♦ Symptom like fever with chills. Discuss the management of carbuncle.
♦♦ Sever pain and swelling in nape of neck and back.  (Jan 2011, 5 Marks)
♦♦ Surrounding area is indurated. Ans. Management of Carbuncle
♦♦ Skin on center of carbuncle softens and peripheral satellite • Diabetes control preferably with injectable insulin.
vesicle appears, if rupture discharging pus and give rise • Appropriate parenteral antibiotics are given
to cribriform appearance. till complete resolution occurs. Most strains of
♦♦ Later development of large crateriform ulcer with center staphylococcal aureus are sensitive to cloxacillin,
slough. flucloxacillin, erythromycin and some of the
cephalosporins. However, methicillin Resistant
Staphylococcus aureus (MRSA) are resistant to the
drugs mentioned above. They are sensitive only to
expensive drug Vancomycin which has to be given
intravenously.
• Improve general health of the patient
• If carbuncle does not show any softening, or if it
shows evidence of healing, it is not incised. It can
be left open to the exterior or saline dressings are
applied to reduce edema. Complete resolution may
take place within l0-l5 days.
Fig. 3:  Carbuncle of neck
Surgical Management
Complications ♦♦ Surgery is required when there is pus.
♦♦ Worsening in diabetic status resulting in diabetic ketoaci- ♦♦ Cruciate incision is preferred because of multiple abscesses
dosis and extensive subcutaneous necrosis. Edges of the skin flap
♦♦ Extensive necrosis of skin overlying carbuncle. Hence, it are excised, pus is drained, loculi are broken down, slough
is included under acute infective gangrene. is excised, and cavity is treated with antiseptic agents. Once
♦♦ Septicemia and toxemia. wound granulates well, skin grafting may be required.

Management Q.4. Write short note on cold abscess. (Feb 2013, 5 Marks)
 (Jan 2011, 5 Marks) (Sep 2001, 10 Marks)
Principles of treatment of carbuncle are:  (Sep 1997, 6 Marks) (Sep 1999, 5 Marks)
♦♦ Control of diabetes Or
♦♦ Parenteral antibiotics Write in brief about cold abscess.(July 2016, 5 Marks)
♦♦ Surgical management: Ans. Cold abscess is also known as tuberculous abscess.
• Diabetes control preferably with injectable insulin.
Abscess shows no signs of acute inflammation, that is
• Appropriate parenteral antibiotics are given
why it is known as cold abscess
till complete resolution occurs. Most strains of
It occurs most commonly in the neck, groin, psoas region,
Staphylococcus aureus are sensitive to cloxacillin,
paraspinal area.
flucloxacillin, erythromycin and some of the
cephalosporins. However, methicillin resistant Areas of Origin
Staphylococcus aureus (MRSA) are resistant to the drugs
mentioned above. They are sensitive only to expensive ♦♦ In tuberculous lymphadenitis, cold abscess is commonly
drug vancomycin which has to be given intravenously. seen in anterior triangle of neck.
♦♦ In tuberculosis of cervical spine, cold abscess occur in
• Improve general health of the patient
posterior triangle of neck.
• If carbuncle does not show any softening or if it shows
evidence of healing, it is not incised. It can be left
Clinical Features
open to the exterior or saline dressings are applied to
reduce edema. Complete resolution may take place ♦♦ It is more common in younger age group and is seen at
within l0–l5 days. any age.
276   Mastering the BDS IIIrd Year  (Last 25 Years Solved Questions)

♦♦ There is presence of swelling over the neck which is Bacteriology


smooth, soft, fluctuant, non-tender, non-transilluminant
♦♦ As Ludwig’s angina is of dental origin streptococci or
and have restricting mobility.
mixed oral flora are the most commonly reported micro-
♦♦ With the change in posture when patient is seated, he
organisms.
support his head with hands and forearm. This is known
♦♦ Presence of staphylococci, E. coli, Pseudomonas and anaer-
as Rust’s sign.
obes including bacteroides and Peptostreptococcus, Prevotella
♦♦ There is also presence of evening fever, loss of weight,
appetite and anemia. species have also been isolated.
♦♦ Matted lymph nodes are present adjacent to cold abscess ♦♦ Role of anaerobes as primary or synergistic organisms
which are palpable. should not be omitted in the culture.
♦♦ Abscess shows no signs of acute inflammation. Spread of Perimandibular Space Infection
Investigations ♦♦ Infection from a lower third molar reaches the subman-
dibular space.
♦♦ ESR is raised.
♦♦ From here it spreads along the submandibular salivary
♦♦ Mantoux test is positive.
gland above the mylohyoid muscle to reach the sublin-
♦♦ Presence of anemia and lymphocytosis
♦♦ Chest X-ray shows pulmonary tuberculosis gual space.
♦♦ On FNAC epithelioid cells are seen. ♦♦ From one side of the sublingual space, it moves across the
♦♦ X-ray of neck in tuberculous cervical spine is done to genioglossus muscles and reaches the sublingual space on
identify reduced joint space, vertebral destruction and the other side. It can then cross over the mylohyoid muscle
soft tissue shadow. and reach the opposite side submandibular space.
♦♦ MRI of cervical spine, US/CT scan of neck are needed ♦♦ The submental space gets involved via the lymphatics.
for confirming the anatomical location and number of ♦♦ Since it is a cellulitis and not an abscess, it does not remain
lesions. localized but rapidly spreads along the fascial planes and
tissue spaces.
Sequelae ♦♦ After involving the three spaces, the cellulitis spreads
♦♦ Secondary infection occurs in cold abscess, and it become within the substance of the tongue posteriorly along the
tender. course of the sublingual artery in the cleft between the geni-
♦♦ Collar stud abscess forms, as pressure increases in cold oglossus and geniohyoid muscles. This reaches the region
abscess it provide a way through deep fascia to reach of the epiglottis producing edema and inflammation of the
subcutaneous plane and adhere to skin. laryngeal inlet. This causes severe airway compromise.
♦♦ Formation of sinus. ♦♦ Also from the submandibular space it can pass along the
♦♦ Disease spread to other lymph nodes and organs. investing layer of the deep cervical fascia all along the
anterior aspect of the neck to the clavicle and continue
Differential Diagnosis into the mediastinum.
♦♦ Branchial cyst and other cystic swellings of neck. ♦♦ Communication of the submandibular space with the
♦♦ Secondaries of neck lymph nodes pterygomandibular, masseteric and lateral pharyngeal
♦♦ Secondaries in cervical spine. spaces causes trismus and further enhances airway com-
promise.
Treatment
Management
♦♦ Anti-tubercular drugs should be started.
♦♦ Zig-zag aspiration by wide bore needle in non-dependent The main aspect of management is:
aspiration of cold abscess. ♦♦ Airway maintenance
♦♦ Excision of affected lymph nodes ♦♦ Parenteral antibiotics
♦♦ Cervical spine should be immobilized by plaster jacket ♦♦ Surgical decompression of tissues
for 4 months. ♦♦ Hydration of patient
♦♦ Drainage using non-dependent incision and later closure ♦♦ Removal of cause
of wound without placing the drain.
Airway Maintenance
Q.5. Describe the bacteriology of perimandibular space ♦♦ Edema of the glottis is what leads to airway obstruction
infection, its spread and management.  and death of the patient, if untreated. Maintaining a patent
 (Sep 2004, 20 Marks) airway is thus of prime importance.
Ans. Submandibular, sublingual and submental spaces are ♦♦ Intubation is done by an experienced anesthetist.
collectively called as perimandibular space. When the ♦♦ Bulging of the posterior pharyngeal wall makes intubation
perimandibular spaces are involved in an infection, it is difficult. Perforation may lead to aspiration of pus, if an
known as Ludwig’s angina. endotracheal tube is forced in blindly.
Section 2:  General Surgery  277

♦♦ A tracheotomy or cricothyroidotomy may be advisable. Bacteriology


Tracheostomy is usually difficult because the anatomical ♦♦ Commonly due to Streptococcus pyogenes and other Gram-
landmarks on the neck are not easily identifiable with the positive organisms. Release of streptokinase and hyaluro-
swelling. nidase can cause spread of infection.
Parenteral Antibiotics ♦♦ Often gram-negative organisms such as Klebsiella, Pseu-
domonas, E. coli are also involved.
♦♦ Penicillin is the antibiotic of choice.
♦♦ Others such as Amoxicillin, cloxacillin may also be used. Source of Infection
♦♦ Metronidazole is given against anaerobic organisms. ♦♦ Injuries
♦♦ Gentamycin can also be given along with penicillin for ♦♦ Graze or scratch
coverage of Pseudomonas. ♦♦ Snake bite.
Surgical Decompression Clinical Features
♦♦ It is usually preferred under LA instead of GA. The main ♦♦ Acute inflammatory lesions such as cellulitis especially
aim is to relieve the pressure within the tissues for the when situated in the dangerous area of the face are prone
edema to reduce. Also, the pressure within the tissues to spread in cavernous sinus.
compresses the blood vessels preventing the penetration of ♦♦ Patient is morbidly ill and may be delirious or semi
antibiotics in the area of interest. Thus decompression im- conscious.
proves vascularity and potentiates the action of antibiotics. ♦♦ High grade fever, headache, nausea, vomiting are common.
♦♦ If there is pus, it provides a channel for the drainage of pus. ♦♦ Local signs such as edema of conjunctiva and eyelids,
♦♦ Pus obtained from a drainage may be sent for culture and dilated and sluggishly reacting pupil may be present.
sensitivity testing. ♦♦ Also there will be movements of eyeball due to involvement
♦♦ Decrease in edema reduces the respiratory embarrassment. of 3rd, 4th and 6th cranial nerves.
♦♦ Ethyl chloride spray may be used or LA may be injected ♦♦ Proptosis and involvement of opposite site eye in advanced
locally in the area of incision. cases
♦♦ Bilateral submandibular incisions with a midline submen- ♦♦ It spreads through loose connective and interstitial tissues
tal incision may be placed. of face.
♦♦ Blunt dissection through the skin, superficial fascia, platy- ♦♦ Tender regional lymph nodes may be palpable which
sma, deep fascia and mylohyoid muscle is done to reach the signifies severity of infection.
sublingual space. The sublingual space may alternatively ♦♦ There is no edge, no pus, no fluctuation and no limit.
be drained intraorally by an incision in the floor of the
mouth if the mouth opening is adequate. Complications
♦♦ A drain is inserted through all these layers and stabilized.
♦♦ Cellulitis can drain into an abscess which needs to be
♦♦ There may be no pus at all in most cases of Ludwig’s an-
drained.
gina. Pus may only be seen at later stages.
♦♦ Necrotizing fasciitis: Certain highly invasive strains of
♦♦ A loose dressing is placed which is changed everyday after
Streptococcus pyogenes can cause extensive necrosis of
careful cleaning of the drain.
skin, subcutaneous tissue and may result in necrotizing
Hydration of the Patient fasciitis.
♦♦ Toxemia and septicemia: Streptococcal toxic shock syndrome
♦♦ Pyrexia and dysphagia may lead to dehydration of the
can result if exotoxins are produce by microorganisms.
patient.
♦♦ Cellulitis can precipitate ketoacidosis, if patient is diabetic.
♦♦ It is necessary to put the patient on IV fluids.
Treatment
Removal of Cause
Principles of treatment are:
Once the patient is stabilized and the trismus relieved to an
extent, the offending tooth may be removed ♦♦ Bed rest and elevation of limb
♦♦ Dressing of glycerine magnesium sulfate
Q.6. Write short note on cellulitis. (June 2015, 5 Marks) ♦♦ Control of diabetes mellitus
 (May/June 2009, 5 Marks) (Mar 2008, 5 Marks) ♦♦ Parenteral antibiotic therapy
Or ♦♦ Treatment of septicemia, if present
Define, describe clinical features and principle of ♦♦ Surgical decompression of tissues
treatment of cellulitis. (June 2017, 5 Marks) • Bed rest and elevation of limb or part to reduce edema,
Ans. Cellulitis is defined as spreading inflammation of so as to increase the circulation and bandaging.
subcutaneous tissues and fascial planes. • Glycerine magnesium sulfate dressing should be given
When a periapical infection fail to localize as abscess which decreases edema of affected part.
it leads to cellulitis where infection rapidly spreads • Diabetes mellitus, if present is treated with insulin
through facial tissue planes diffusely. therapy.
278   Mastering the BDS IIIrd Year  (Last 25 Years Solved Questions)

• Appropriate antibiotics such as injection crystalline periapical abscess which later spreads through cortical
penicillin 10 lakh units IM or IV 6 hourly for 5 to 7 part of bone into soft tissues and form an alveolar abscess.
days or ciprofloxacin 500 mg BD is given.
• Often patient may be in septicemia, patient in this Pathology
condition is treated with higher antibiotics, critical Disease begins inside pulp of tooth
care with fluid management, along with maintaining
adequate urine output. Pulpitis
Surgical Management Spread to root
♦♦ Since the tissues are tense and stretched, an incision and
exploration of that area decompress or relieve pressure Localized osteitis
within the tissues.
♦♦ Decompressing the tissues help improve the vascularity, Abscess formation
allowing better penetration of I.V. antibiotics to the area.
Spread into soft tissues outside cheek
Q.7. Write short note on Cellulitis in oral cavity. 
 (Sep 2009, 5 Marks) (Feb/Mar 2004, 5 Marks)
Initially, diffused later localized swelling in jaw with
Ans. Cellulitis is defined as spreading inflammation of
redness and edema of gingiva. Initial dull continuous
subcutaneous tissues and fascial planes.
pain later become severe excruciating pain
Etiology
Alpha-hemolytic streptococci is the etiologic agent. Clinical Features
♦♦ It is common odontogenic infection and constitutes 2% of
Clinical Features apical radiolucencies.
♦♦ Presence of widespread swelling, redness and tenderness ♦♦ Due to acute abscess, there is pain in the affected tooth.
without proper localization. ♦♦ Localized swelling and an erythematous change in
♦♦ Tissues become edematous and there is presence of indura- overlying mucosa is present.
tion. On palpation, tissues are firm to hard in consistency. ♦♦ The affected area of jaw may be tendered during palpitation.
♦♦ Tissues get discolored and temperature rises up. ♦♦ Pain aggravates during percussion and when pressure is
♦♦ Depending on the location and proximity of anatomical applied with the opposing tooth.
structures pus can evacuate on nose, maxillary sinus, vesti- ♦♦ Application of heat intensifies pain, whereas application
bule, floor of mouth, infratemporal fossa and fascial spaces. of cold relieves pain temporarily.
♦♦ Infection occurring in maxilla perforate outer cortical layer ♦♦ Pus discharging sinus often develops on alveolar mucosa
of bone above buccinators and leads to swelling of upper over the affected root apex and sometimes on skin overly-
half of the face, and if infection perforates outer cortical ing the jaw bone.
plate in mandible below buccinators, there is swelling in ♦♦ Infection from acute periapical abscess often spread to
lower half of face. facial spaces, leading to space infections.
♦♦ If maxillary tooth is associated with infection, then redness
can be seen in the eye. Treatment
♦♦ Rest is given
Management
♦♦ Incision and drainage under general anesthesia
♦♦ Surgical incision and drainage: This is done if pus is diag- ♦♦ Appropriate antibiotics
nosed. In large cellulitis, an erythematous area is present ♦♦ IV fluids to correct dehydration.
which consists of pus near superficial surface. These areas ♦♦ Extraction of tooth at later period
are incised and drained under local anesthesia. Knife is ♦♦ Extraction of sinus whenever required.
inserted in most inferior portion of fluctuant area. A small
sinus forceps is inserted in the wound and is opened in Complications
various directions, so that pus is drained. Rubber drain is ♦♦ Septicemia.
placed in deepest area of the wound and suturing is done. ♦♦ Spread of infection into other spaces such as parapharyn-
Dressing is given. geal spaces; sublingual and submandibular spaces caus-
♦♦ Broad-spectrum antibiotics should be given to the patient. ing Ludwig’s angina; edema of epiglottis and respiratory
Antibiotics of cephalosporin family are preferred. distress; spread to pterygoid space and along pterygoid
♦♦ Associated tooth should be extracted. muscles through emissary vein leading to cavernous
Q.8. Describe pathology, clinical features, treatment and sinus thrombosis; upper canine tooth abscess spreading
complications of alveolar abscess.(Sep 2002, 15 Marks) to corner of eye causing angular vein thrombophlebitis
Ans. Alveolar abscess occur due to spread of infection from which further progress to cavernous sinus thrombosis;
root of tooth in periapical tissue. Initially it forms submasseteric abscess.
Section 2:  General Surgery  279

♦♦ Lower incisor abscess can cause abscess in the chin and Treatment
median mental sinus; chronic osteomyelitis of the jaw with
Injection crystalline penicillin 10 lakh units 6 hourly IM/IV for
discharging sinuses. Osteomyelitis is common in mandible
5 to 10 days.
— horizontal process near the mentum, presenting with
pain, swelling, discharging sinuses, bone thickening, loose Q.12. Name etiological agent, pathognomic diagnostic fea­
tooth and trismus. Sequestrum is commonly seen. ture and treatment (only modalities) of erysipelas.
Q.9. Write briefly on alveolar abscess. (Dec 2010, 5 Marks)  (Jan 2017, 3 Marks)
Ans. Refer to Ans 8 of same chapter. Ans.

Q.10. Enumerate differences between cellulitis and erysip­ Etiological Agent


elas. (June 2014, 2 Marks)
Beta hemolytic Streptococcus pyogenes
Ans.
Pathognomic Diagnostic Feature
Cellulitis Erysipelas
When erysipelas occurs on the face, it involves pinna because
It is a spreading inflammation of It is a spreading inflammation of
subcutaneous and facial planes skin and subcutaneous tissue erysipelas is a cuticular lymphangitis. It is described as Milian’s
ear sign positive. This sign differentiates cellulitis of face from
It is caused by S.pyogenes, Kleb- It is caused by S.pyogenes
facial erysipelas. In cellulitis of face, pinna does not get involved
siella, pseudomonas and E.coli
because of close adherence of skin to cartilage.
Bacteremia, septicemia and Toxemia is present
pyemia are present Treatment (Only Modality)
Milian’s ear sign is negative Milian’s ear sign is positive
Antibiotic treatment should be given.
Cutaneous lymphangitis with Cutaneous lymphangitis with
development of rose pink rash development of rose pink rash Q.13. Describe differentiating features of acute abscess and
with cutaneous lymphatic edema with cutaneous lymphatic edema cold abscess. (Jan 2017, 4 Marks)
is not seen is predominant feature Ans.
Acute abscess Cold abscess
Q.11. Write short note on erysipelas. (Dec 2015, 5 Marks)
No previous history of swelling Soft, cystic fluctuant swelling
Ans. Erysipelas is an acute spreading inflammation of skin and
subcutaneous tissues which is caused by Streptococcus Throbbing pain is characteristic Pain is not present
pyogenes and is associated with severe lymphangitis. Extremely tender No tenderness
Infection occurs through small scratch or abrasion and Local rise in temperature is No local rise in temperature
spread rapidly causing toxemia. present
Redness is present No redness is seen
Clinical Features
Pyogenic bacteria are non- Tuberculous bacteria
♦♦ It occurs commonly in children and old people. specific organisms
♦♦ Sites commonly affected are face, eyelids, scrotum in in- For drainage dependent incision Non-dependent incision is used
fants and umbilicus. is used
♦♦ There is presence of rose pink rash with raised edge and
Suturing of wound is not done Wound is sutured
its consistency is of button hole.
♦♦ Features of toxemia are commonly present. Drain is placed Drain is not placed.
♦♦ Redness of the lesion becomes brown and later on yellow It contains pus It contain cheesy caseating
with presence of vesicles. material
♦♦ Discharge from the lesion is serous.
Q.14. Define, describe clinical features and principles of
♦♦ There is presence of edema of face and orbit.
♦♦ When erysipelas occurs on the face involves pinna since treatment of cavernous sinus thrombosis.
it is a cuticular lymphangitis. This is called as Milian’s  (Jan 2017, 5 Marks)
ear sign positive. This sign differentiate erysipelas from Ans. In cavernous sinus thrombosis, the infection from
cellulitis. maxillary anterior region is carried via facial, angular
and nasofrontal veins to superior ophthalmic vein
♦♦ Tender, regional lymph nodes are palpable.
which enter cavernous sinus via superior orbital fissure.
Complications Veins of dangerous area of face are valveless and
allow retrograde infection in form of cavernous sinus
♦♦ Toxemia and septicemia. thrombosis.
♦♦ Gangrene of skin and subcutaneous tissue.
♦♦ Lymphoedema of face and eyelids occur due to lymphatic Clinical Features
obstruction. ♦♦ Patient had sinusitis or midfacial infection which was
♦♦ Abscess pneumonia and meningitis may develop. manipulated.
280   Mastering the BDS IIIrd Year  (Last 25 Years Solved Questions)

♦♦ Prior to ocular symptoms, there is presence of headache, - In case, if nerve and vessels are present below or
fever and malaise. surrounding abscess, incision parallel to nerve
♦♦ Patient complaints of orbital fullness and pain along with and vessel is given to avoid these structures.
visual disturbances. – Incision by Hilton method:
♦♦ If the complains remains untreated for long time symptoms - This method is used when there are plenty of
get spread to other eye and it can be fatal too. nerves and vessels around the abscess cavity,
♦♦ Signs of venous congestion are present, i.e. chemosis, which could be injured.
edema of eyelid, periorbital edema. - In this method, incision is given on most
♦♦ Signs of retrobulbar pressure are present such as exoph- prominent and most dependent part.
thalmos, ophthalmoplegia and loss of corneal reflex - A pair of artery forceps or sinus forcep is
♦♦ Meningeal signs are present, i.e. nuchal rigidity, Brudz- introduced and then the blades are open. It is
inski’s sign. swiped in the abscess cavity to break abscess
♦♦ Systemic signs are also present such as chills, fever, de- loculi.
lirium and shock. - After removal of forceps, finger is introduced
to explore the abscess cavity.
Principles of Treatment • Counter incision:
♦♦ Aggressive antibiotic therapy with broad-spectrum an- – When prominent part is not the most dependent
tibiotics. part, a counter incision is made on most dependent
♦♦ Anticoagulation is done by giving heparin. part facilitates pus drainage under gravity. In case of
♦♦ Steroids are given to reduce the inflammation. counter incision, sinus forcep is introduced on most
♦♦ Primary source of the infection should be eliminated. dependent part rather than most prominent part.
• Drainage:
Q.15. Describe etiopathogenesis and principle of manage­
– A corrugated rubber drain is usually used for
ment of acute pyogenic abscess.
drainage of pus.
 (Apr 2010, 15 Marks) (Mar 2001, 15 Marks)
– It is removed when pus stop coming out.
Ans. Pyogenic *abscess: It is the most common form of an • Follow-up: Proper antibiotics and analgesics are given.
abscess, subcutaneous, deep or it can occur within the
Q.16. Describe the clinical features and treatment of acute
viscera such as liver or kidney.
abscess. (Sep 2009, 5 Marks)
Etiopathogenesis Ans. For treatment refer to Ans 15 of same chapter.
♦♦ It is usually produced by staphylococcal infections. Clinical Features
♦♦ Organism enters in the soft tissue by an external wound, ♦♦ Fever with chills and rigors.
minor or major. ♦♦ Localized swelling which is smooth, soft and fluctuant.
♦♦ It can also be due to hematogenous spread from a distant ♦♦ Pus is visible
focus like tonsillitis or carious tooth, etc. ♦♦ Throbbing pain and pointing tenderness
♦♦ Pyogenic abscess can also be due to cellulitis. ♦♦ Brawny induration around
♦♦ Following an injury, there is inflammation of the part ♦♦ Redness and warmth with restricted movement around
brought about by the organisms. a joint
♦♦ The end result is production of pus, which is composed of ♦♦ Rubor (redness), Dolor (pain), calor (warmness), tumor
dead leukocytes, bacteria and necrotic tissue. (swelling) and functio laesa (loss of localized and adjacent
♦♦ The area around the abscess is encircled by fibrin products tissue/joint function) are quite obvious.
and it is infiltrated with leukocytes and bacteria. It is called
Q.17. Write short note on various types of abscesses and
pyogenic membrane.
Hilton’s method of drainage. (Jun 2010, 5 Marks)
Management Ans. An abscess is a pathological thick-walled tissue cavity
filled with necrotic tissue, bacteria and leukocytes caused
♦♦ When pus is not localized: Conservative treatment is needed.
by localized collection of purulent inflammatory tissue
• Proper antibiotics and anti-inflammatory agents.
and suppuration from infection in a buried tissue, organ
♦♦ When pus is localized: There is a golden rule that “Pus should
or confined space.
be drained”.
• Incision and drainage (I and D) under general anesthesia. Types of Abscess
–– General anesthesia is preferred because of the ♦♦ Pyogenic abscess: It is produced by staphylococcal bacteria.
presence of infection. Organism enter soft tissues by an external wound. It can
• Infection: be subcutaneous, deep or it can be an viscera.
–– Free or liberal incision: ♦♦ Pyemic abscess: It is due to pus-producing organisms in
- It is made on most prominent part to avoid circulation. It is the systemic effect of sepsis. It commonly
damage to healthy tissue. Incision should be occurs in diabetics and patients receiving chemotherapy
adequate for easy drainage of pus. and radiotherapy.
Q2. *Abscess= An abscess is a localized collection of pus
Section 2:  General Surgery  281

♦♦ Cold abscess: It means an abscess with no signs of inflam- ♦♦ Rehydration and maintaining optimum urine output.
mation. It is caused due to tuberculosis. ♦♦ Electrolyte management is done.
♦♦ In severe cases, amputation has to be done as a life-saving
Hilton’s Method of Drainage procedure-stump should never be closed (Guillotine
♦♦ Initially broad-spectrum antibiotics are started. amputation).
♦♦ Under general anesthesia or regional block anesthesia after ♦♦ Often ventilator support is required.
cleaning and draping, abscess is aspirated and presence ♦♦ Once a ward or peration theater is used for a patient with
of pus is confirmed. gas gangrene, it should be fumigated for 24–48 hours
♦♦ Skin is incised adequately in line parallel to neurovascular properly to prevent the risk of spread of infection to other
bundle. patients especially with open wounds.
♦♦ Pyogenic membrane is opened using sinus forceps and all ♦♦ Hypotension in gas gangrene is treated with whole blood
loculi are broken up. transfusion.
♦♦ Abscess cavity is cleared of pus and washed with saline. Q.2. Describe briefly gas gangrene. (Sep 2002, 10 Marks)
♦♦ A drain is placed Or
♦♦ Wound is not closed, wound is allowed to heal and Write short note on gas gangrene. (Apr 2017, 4 Marks)
granulate.  (Dec 2010, 5 Marks) (May/June 2009, 5 Marks)
Q.18. Write short answer on Hilton’s method of treatment. Or
Write in short about gas gangrene.
 (June 2018, 3 Marks)
 (July 2016, 5 Marks)
Ans. Hilton’s method of treatment is the surgical method of
draining the abscess. Or
Describe gas gangrene, etiology and management
For Hiton’s method in detail refer to Ans 17 of same chapter.
 (Jan 2018, 10 Marks)
Ans. Definition: This is a spreading infective gangrene of the
3. Specific Infections muscles characterized by collection of gas in the muscles
and subcutaneous tissue. As this condition is caused by
Clostridium infection, it is also called ‘Clostridium myone-
Q.1. Describe clinical features and treatment of gas gan­
crosis’.
grene. (Apr 2010, 5 Marks)
Ans. It is highly fatal spreading infection caused by Clostridium Etiology
organism, which result in “myone­crosis”.
Gas gangrene is caused by Clostridium perfringens which is the
• It is also known as clostridial myositis, clostridial
most common microorganism. Various other organisms causing
myonecrosis, and infective gangrene of muscles.
gas gangrene are Clostridium septicum, Clostridium oedematiens,
• The most common causing agent is Clostridium welchii.
Clostridium histolyticum.
Clinical Features
Predisposing Factors
♦♦ Presence of features of toxemia, fever, tachycardia and
pallor. ♦♦ Contaminated, manured or cultivated soil, intestines are
♦♦ Wound is under tension with foul smelling discharge. the sources. Fecal flora commonly contains clostridial
♦♦ Color of skin is khaki brown due to hemolysis. organisms enters the wound; in presence of calcium from
♦♦ Crepitus can be felt. blood clot or silica (silicic acid) of soil, it causes infection.
♦♦ Jaundice can be present and oligouria signifies renal ♦♦ It is common in crush wounds, following road traffic
failure. accidents, after amputations, ischemic limb, gunshot
♦♦ Most commonly the site affected are adductor region of wounds, war wounds. Injury or ischemia or necrosis of
lower limb and buttocks and subscapular region in upper the muscle due to trauma predisposes infection.
limb. ♦♦ Anaerobic environments in the wound-initial infection
with aerobic organism utilizes existing oxygen in tissues
Treatment creating anaerobic environment to cause clostridial sepsis.
♦♦ Injection benzyl penicillin 20 lacs 4 hourly + lnjection
metronidazole 500 mg 8 hourly + lnjection amino- Types
glycosides or third generation cephalosporins or
metronidazole. ♦♦ Subcutaneous type: This is a crepitant infection involving
♦♦ Fresh blood transfusion. necrotic tissue, but healthy muscles are not involved.
♦♦ Polyvalent antiserum 25,000 units given intravenously Cellulitis is characterized by foul smelling, seropurulent
after a test dose and repeated after 6 hours. infection of a wound.
♦♦ Hyperbaric oxygen is very useful. ♦♦ Single muscles type: The infection is limited to one muscle.
♦♦ Liberal incisions are given. All dead tissues are excised and ♦♦ Group type: The gas gangrene is limited to one group of
debridement is done until healthy tissue bleeds. muscles.
282   Mastering the BDS IIIrd Year  (Last 25 Years Solved Questions)

♦♦ Massive type: The gas gangrene involved almost the whole ♦♦ Often ventilator support is required.
muscle mass of one limb. ♦♦ Once a ward or Operation theatre is used for a patient
♦♦ Fulminating type: In this condition, the gas gangrene with gas gangrene, it should be fumigated for 24 - 48 hours
spreads very rapidly even beyond the limb and is often properly to prevent the risk of spread of infection to other
associated with intense toxemia with high fatal rate. patients especially with open wounds.
♦♦ Hypotension in gas gangrene is treated with whole blood
Clinical Features transfusion.
♦♦ Presence of features of toxaemia, fever, tachycardia and Q.3. Write short note on cancrum oris.
pallor.  (Sep 2006, 6 Marks) (Apr 2017, 5 Marks)
♦♦ Wound is under tension with foul-smelling discharge. Ans. It is also called as NOMA or gangrenous stomatitis.
♦♦ Color of skin is khaki brown due to hemolysis.
• It is a rapidly spreading gangrene of oral and facial
♦♦ Crepitus can be felt.
tissues occurring in deliberated or nutritionally
♦♦ Jaundice can be present and oligouria signifies renal
deficient person.
failure.
• The disease is caused by Borrelia vincentii and
♦♦ Most commonly the site affected are adductor region of
fusiformis bacteria.
lower limb and buttocks and subscapular region in up-
per limb. Predisposing Factors
Investigations ♦♦ Low socioeconomic status: It occurs in people of low socio-
♦♦ X-ray shows gas in muscle plane or under the skin. economic status or in poverty.
♦♦ Liver function tests, blood urea, serum creatinine, total ♦♦ Diseases: In debilitated diseases such as diphtheria,
count, PO2, PCO2 measles, pneumonia, scarlet fever, etc.
♦♦ CT scan of the part may be useful, especially in chest or ♦♦ Injury: In cases with mechanical injury.
abdominal wounds. ♦♦ Immunodeficiency state: In AIDS
♦♦ Gram stain shows gram-positive bacilli. ♦♦ Poor Oral hygiene: This leads to growth of bacteria causing
♦♦ Robertson’s cooked meat media is used which causes meat increase chances of infection.
to turn pink with sour smell and acid reaction. Clinical Features
♦♦ Clostridium welchii is grown in culture media containing
20% human serum in a plate. Antitoxin is placed in one— ♦♦ It is seen chiefly in children mainly in malnourished
half of the bacteria grown plate sparing the other half. Zone children.
of opacity will be seen in that half of the plate where there is ♦♦ The common sites are areas of stagnation around fixed
no antitoxin. In the other half part of the plate where there bridge or crown.
is antitoxin there is no opacity-Nagler reaction. ♦♦ The commencement of gangrene is denoted by blenching
of skin. Small ulcers of gingival mucosa spread rapidly
Complications and involve surrounding tissue of jaws, lips and cheeks
♦♦ Septicemia, toxemia by gangrenous necrosis.
♦♦ Renal failure, liver failure ♦♦ Odor is foul. Patient has high temperature during course
♦♦ Circulatory failure, Disseminated intravascular coagula- of disease, suffers secondary infection and may die from
tion, secondary infection. toxemia.
♦♦ Death occurs in critically ill patients. ♦♦ Overlying skin is inflamed, edematous and finally necrotic
which results in formation of line of demarcation between
Treatment healthy and dead tissue.
♦♦ Injection benzyl penicillin 20 lacs 4 hourly + lnjection met- ♦♦ In advanced stage, there is blue black discoloration of skin.
ronidazole 500 mg 8 hourly + lnjection aminoglycosides or ♦♦ As gangrenous process advances, slough appears and soon
third generation cephalosporins or metronidazole. separated, leaving perforating wound in involved area.
♦♦ Fresh blood transfusion. ♦♦ The large masses may be sloughed out leaving jaws exposed.
♦♦ Polyvalent antiserum 25,000 units given intravenously
Treatment
after a test dose and repeated after 6 hours.
♦♦ Hyperbaric oxygen is very useful. ♦♦ Systemic antibiotics should be given, i.e. high dose
♦♦ Liberal incisions are given. All dead tissues are excised and penicillin and metronidazole.
debridement is done until healthy tissue bleeds. ♦♦ Thorough nasogatric tube, high protein and vitamin rich
♦♦ Rehydration and maintaining optimum urine output. diet should be given.
♦♦ Electrolyte management is done. ♦♦ Blood transfusion is given.
♦♦ In severe cases, amputation has to be done as a life-saving ♦♦ Parenteral fluid is given.
procedure-stump should never be closed (Guillotine ♦♦ Wound irrigation and liberal excision of dead tissue is done.
amputation). ♦♦ Later on patient require flap to cover the defect.
Section 2:  General Surgery  283

Characteristic Primary syphilis Secondary syphilis Tertiary syphilis


Site Chancre occurs at the site of entry of Mucus patches are present on tongue, Gumma can occur anywhere in jaw but
treponema. buccal mucosa, pharyngeal region more frequent site is palate, mandible
It occurs on lip, oral mucosa, and and lips. and tongue.
lateral surface of tongue, soft palate Split papules develop at commissure
and gingiva. of lips.
Appearance It has narrow copper-colored slightly Mucus patches appears as slightly Gumma manifests as solitary, deep
raised borders with reddish brown base raised grayish white lesions surrounded punched out mucosal ulcer.
in centers. by erythematous base. The split
papules are cracked in middle giving
“Split pea appearance”.
Symptom Intraoral chancres are slightly painful Mucus patches are painless mild-to- In gumma, the breathing and swallowing
due to secondary infection and are moderately painful. difficulty may be encountered by the
covered with grayish white film. patient
Signs White sloughy material is present Snail tract ulcers and raw bleeding Perforation of palatal vault is present.
surfaces are present.
Tongue Tongue lesion may be commonly seen Tongue gets fissured. Numerous small healed gumma in
on lateral surface of anterior two- thirds- tongue results in series of nodules
or on dorsal surface and often there is or sparse in deeper area of organ
enlargement of folate papilla. giving tongue an upholstered or tufted
appearance.

Q.4. Describe various syphilitic lesion of lip and oral The most common organism is Actinomyces israelii, A.
cavity. (Sep 2002, 5 Marks) naeslundii, A. viscosus, A. odontolyticus and A. propionica.
Or
Write short note on syphilis  (Nov 2014, 3 Marks) Clinical types of Actinomycosis
Ans. For syphilitic lesions of lips and oral cavity, refer to the ♦♦ Faciocervical: It is the most common type. Infection spreads
table above. either from tonsil or from adjacent infected tooth. Initially,
an induration develops. Nodules form with involvement
Diagnosis
of skin of face and neck. It softens and bursts through
♦♦ By blood tests using treponemal or non – treponemal tests. the skin as sinuses which discharge pus- which contains
♦♦ Non-treponemal tests are used initially and are VDRL and sulphur granules (60%).
rapid plasma regain tests. ♦♦ Thorax: Lungs and pleura get infected by direct spread
♦♦ Positive confirmation is required by treponemal test such from pharynx or by aspiration. Empyema develops.
as Treponema pallidum particle agglutination or fluorescent Later chest wall nodules appear leading to sinuses with
treponemal antibody absorption test. discharge (20%).
♦♦ In right iliac fossa: It presents as a mass abdomen with
Treatment
discharging sinus.
♦♦ Primary and secondary syphilis are treated by injection ♦♦ Liver is infected through portal vein (Honeycomb liver).
procaine penicill in 10 lakh units IM for 14 days. ♦♦ Pelvic: Pelvic actinomycosis can occur due to intra—uterine
♦♦ In late syphilis, the above-mentioned treatment should be devices.
continued for 21 days.
Q.5. Write short note on actinomycosis. Pathogenesis
 (Feb 2013, 5 Marks) (Mar 1997, 5 Marks) Organism enters through deeper plane of the tissue,
 (Feb 2015, 5 Marks) (Mar 2006, 5 Marks) causes subacute inflammation with induration and nodule
 (Dec 2015, 5 Marks) formation. Eventually discharging sinus forms at the surface.
Or Pus collected in a swab or sterile tube will show sulphur
Write short note on actinomycosis of jaw. granules.
 (Aug 2012, 5 Marks)
Or
Predisposing Factors
Write brief answer on actinomycosis.
 (Apr 2017, 5 Marks) ♦♦ Trauma
Ans. It is a chronic granulomatous suppurative disease which ♦♦ Presence of carious tooth
is caused by anaerobic or microaerophilic Gram-positive ♦♦ Secondary bacterial invasion
non-acid-fast branched filamentous bacteria. ♦♦ Hypersensitivity reaction.
284   Mastering the BDS IIIrd Year  (Last 25 Years Solved Questions)

Clinical Features of sinus tracts and recalcitrant fibrotic lesions, decompres-


sion of closed space infections and the interventions which
Cervicofacial Actinomycosis / Actinomycosis are aimed for relieving the obstruction.
of Jaw / Faciomaxillary ♦♦ Welsh regimen: Injection amikacin l5 mg/kg IV daily for 2l
♦♦ Its occurrence is more common in males. days; such cycle is repeated 3 times at a gap of l5 days along
♦♦ Disease may remain localized to soft tissues or spread to with tablet trimethoprim (7 mg/kg)—sulfamethoxazole
involve salivary glands, bone (maxilla or mandible), skin (35 mg/kg) daily for 6 months.
of face and neck. Most commonly involve area is sub- Q.6. Write the clinical features of faciomaxillary Actino­
mandibular region. mycosis. (Sep 2000, 5 Marks)
♦♦ Presence of trismus is there before formation of pus. Ans. It is caused by the Actinomycosis israelii.
♦♦ The disease is characterized by presence of palpable mass
which is indurated and is painless. Skin surrounding the Clinical Features
lesion has wooden indurated area of fibrosis.
♦♦ Age: Commonly seen in adult male.
♦♦ Multiple subcutaneous nodules over bluish-colored skin
♦♦ Cause: Dental plaque, calculus, gingival debris, bad cari-
of jaw.
ous tooth.
♦♦ Nodules rupture resulting in multiple discharging sinuses.
♦♦ Location: Submandibular region is the most frequent site
Abdominal Actinomycosis of infection. Cheek, masseter region and parotid gland
may also involve.
♦♦ It is more severe form of disease.
♦♦ Symptoms: Trismus is a common feature, before pus
♦♦ Patient complains of fever with chills and vomiting.
formation.
♦♦ There is involvement of liver and spleen.
♦♦ On palpation, abdominal mass is felt which is the sign in Signs
diagnosis of disease.
♦♦ The first sign of infection is the presence of palpable mass.
Thoracic Actinomycosis ♦♦ Mass is painful and indurated.
♦♦ Multiple subcutaneous nodules over bluish-colored skin
♦♦ Patient gives history of aspiration. of jaw.
♦♦ Dry or productive cough, occasionally blood – streaked ♦♦ The nodules rupture resulting in multiple discharging
sputum, shortness of breadth and chest pain. sinuses.
♦♦ Sinus tracts are present with drainage from chest wall. ♦♦ The discharge contains sulphur granules.
Pelvic actinomycosis ♦♦ Lymph nodes are not involved.
♦♦ History of IUCD is present. Q.7. Write short note on candidal infection of oral cavity.
♦♦ Presence of lower abdominal discomfort, abnormal vaginal  (Sep 1997, 6 Marks)
bleeding or discharge. Ans. Oral Candidiasis is most commonly associated with
Candida albicans.
Investigations
Oral Candidal Infection
♦♦ Pus under microscopy shows branching filaments. ♦♦ Thrush (Pseudomembranous candidiasis):
♦♦ Gram ’s staining shows Gram-positive mycelia in centre • It is superficial infection of upper layer of mucous
with Gram-negative radiating peripheral filaments. These membrane.
clubs are due to host reaction which are lipoid material • Characterized as creamy white removable plaque on
(antigen-antibody complex). the oral mucosa and caused by overgrowth of fungal
♦♦ Cultured in brain heart infusion agar and thioglycolate hyphae mixed with desquamated epithelium and
media. inflammatory cells.
Differential Diagnosis • This type of candidiasis may involve any part of the
mouth or pharynx.
♦♦ Chronic pyogenic osteomyelitis. ♦♦ Acute atrophic candidiasis:
♦♦ Carcinomas at the site • It is also called as “Antibiotics sore mouth”.
♦♦ Tuberculous disease • It appears as flat, red patches of varying size.
• It commonly occurs on the palate and the dorsal
Management
surface of the tongue.
♦♦ Penicillins are the drug of choice and should be given ♦♦ Chronic hyperplastic candidiasis:
for longer period (6 to l2 weeks). If patient is allergic to • It is called as “candidal leukoplakia”.
penicillin doxycycline can be given. • It present as firm and adherent white patches in the
♦♦ Antifungals are often given because it is fungal-like bac- oral mucosa.
terium. • It occurs on cheek, lip and tongue.
♦♦ Surgical debridement is occasionally required. Surgical Symptoms of oral candidal infection: Including burning
therapy include incision and drainage of abscess, excision mouth, problem in eating spicy food and change in taste.
Section 2:  General Surgery  285

Treatment Differential Diagnosis


♦♦ Removal of the cause. ♦♦ Strychnine poisoning
♦♦ Topical treatment: Clotrimazole, nystatin, ketoconazole or ♦♦ Trismus due to various causes such as dental, oral, tonsillar
amphotericin in ointment or cream base, suspension. sepsis, oral malignancy.
♦♦ Systemic treatment: ♦♦ Meningitis
• Nystatin 250 mg TDS for 3 weeks. ♦♦ Hydrophobia
• Ketoconazole 200 mg once daily. ♦♦ Convulsive disorders
• Fluconazole 100 mg once daily. ♦♦ Epilepsy
• Itraconazole 100 mg may be used. ♦♦ Symptomatic hyperactivity.
Q.8. Discuss etiology, clinical features. Differential
diagnosis and treatment of tetanus. Treatment / Management
 (Sep 1999, 15 Marks) (Sep 2009, 15 Marks) ♦♦ Patient is admitted and isolated in a dark, quiet room.
Or ♦♦ Antitetanus globulin (ATG), 3,000 units IM single dose
Discuss the clinical features and management of is given.
tetanus (Dec 2010, 8 Marks) ♦♦ Anti-tetanus serum (ATS): When ATG is not available or
Or when patient cannot afford, after IV test dose (l ,000 units
Discuss etiology, clinical features and management of of ATS), full dose is given, i.e. l,00,000 units, half of it is
tetanus. (July 2016, 10 Marks) given IM and half of it is given IV.
Ans. Tetanus is also called as lockjaw. ♦♦ Wound debridement, drainage of pus, injection of ATG
It is a disease of nervous system characterized by 250-500 units locally to reduce the toxin effect.
intensive activity of motor neuron and resulting in severe ♦♦ Ryle’s tube has to be passed, initially to decompress, so as
muscle spasm. to prevent aspiration, but later for feeding purpose.
♦♦ Catheterization should be done.
Etiology ♦♦ IV fluids and electrolyte balance has to be maintained.
It is caused by anaerobic, gram-positive Bacillus clostridium ♦♦ Tetanus toxoid should be given as disease will not give
tetani. immunity against further infections. To start first dose,
second dose after one month, third dose after six months.
Clinical Features Aluminum phosphate absorbed tetanus toxoid 0.5 mL is
injected into deltoid muscle. Booster dose should be given
Symptoms every 4 years or after any significant trauma. In patients
♦♦ Trismus is common. who have not been immunized earlier it needs 30 days to
♦♦ Presence of jaw stiffness and pain. to develop antibody after tetanus toxoid injection.
♦♦ Sweating is present and patient is anxious. ♦♦ IV diazepam 20 mg 4th or 6th hourly. Dose is adjusted
♦♦ Presence of headache, delirium and sleeplessness. depending on severity and convulsions.
♦♦ Presence of dysphagia and dyspnea. ♦♦ IV phenobarbitone 30 mg 6th hourly.
♦♦ IV chlorpromazine 25 mg 6th hourly.
Signs ♦♦ Injection crystalline penicillin 20 lacs 6th hourly and
♦♦ Spasm and rigidity of all muscles. injection gentamicin and metronidazole to prevent
♦♦ Hyper-reflexia secondary infection.
♦♦ Respiratory changes due to laryngeal muscle spasm, ♦♦ Regular suction and clearance of respiratory tract.
infection, aspiration. ♦♦ Nasal oxygen is given.
♦♦ Tonic-clonic convulsions. ♦♦ In severe cases, patient is curarised and placed in ventilator.
♦♦ Abdominal wall rigidity often with hematoma formation. ♦♦ Endotracheal intubation or tracheostomy are often life-
♦♦ Severe convulsion may often lead to fractures, joint saving procedures.
dislocations and tendon ruptures. ♦♦ Good nursing care: Change of position, prevention of
♦♦ Fever and tachycardia. bedsores, prevention of DPT.
♦♦ Retention of urine (due to spasm of urinary sphincter), ♦♦ Chest (respiratory) physiotherapy during recovery period.
constipation (due to rectal spasm). ♦♦ Steroids are given when carditis is suspected.
♦♦ Rarely features of carditis are seen due to involvement of ♦♦ Cardiac pacemaker may be useful in refractory bradycardia
the cardiac muscle, which is dangerous, as it often leads and arrhythmias.
to cardiac arrest and death. ♦♦ Following treatment patient often gets spasm of different
♦♦ Symptoms will be aggravated by stimuli-like light and muscles (tics) for a long period which can be prevented by
noise. giving methocarbamol for 6 months to one year.
286   Mastering the BDS IIIrd Year  (Last 25 Years Solved Questions)

Q.9. Write short note on tetanus prophylaxis. Investigations


(Sep 2006, 6 Marks) (Mar 2001, 10 Marks) ♦♦ ELISA test is the screening test
Or ♦♦ Western blot is the confirmatory test
Discuss about prophylaxis of tetanus. ♦♦ Polymerase chain reaction
(Sep 2009, 5 Marks) (Mar 2008, 5 Marks)
♦♦ Anti-HIV detection
Or
♦♦ CD4+ count
Write short note on prophylaxis against tetanus.
• Normal value is >500/mm3
(Feb 2013, 5 Marks)
• Values between 200 to 500/mm3 is seen in Kaposi’s
Ans. sarcoma and candidiasis
Tetanus Prophylaxis • Values between 50 to 200/mm3 is seen in Pneumocystis
carinii and Toxoplasma infection.
♦♦ In adults in which fresh immunization to start second in
• Values <50/mm3 is seen in atypical mycobacteria,
one month, next in 6 month period. Tetanus toxoid 0.5 mL
cytomegalovirus, lymphomas.
IM Booster dose should be given once in every 4 years or
after any significant trauma. As patient is HIV infected, a time gap occurs for these tests
♦♦ Tetanus neonatorum can be prevented by immunization to become reactive. This time gap is known as window period.
During this period, person in infected.
of the mother with two tetanus toxoid injection, ½ mL IM
in third trimester of pregnancy. Treatment
♦♦ Infants and children are immunized with tetanus toxoid,
Diphtheria and Pertussis vaccine (DPT) three dose at 6, 10, ♦♦ Antiviral therapy or HAART therapy
14 weeks of age. This is called “Triple antigen”. A booster • Nucleoside reverse transcriptase inhibitor (NRTI):
dose is given at 18 months and once in five years, 1 mL of Zidovudine, didanosine, abacavir, lamivudine,
tetanus toxoid is given to achieve active immunity. stavudine.
♦♦ ATG- 500-1000 units IM given as prophylaxis in road acci- • Non-nucleoside reverse transcriptase inhibitor (NNRTI):
dent, severe burns, crush injuries, war wounds, penetrating Nevirapine, delavirdine.
wounds and wounds of head and face. • Protease inhibitors: Ritonavir, indinavir, amprenavir.
♦♦ Treatment of opportunistic infections.
Q.10. Write short note on Tetanus. (Mar 2006, 10 Marks) ♦♦ Treatment of tumors.
 (Mar 2007, 2.5 Marks) (Feb/Mar 2004, 5 Marks) ♦♦ Immunotherapy:
 (Dec 2015, 5 Marks) (Feb 2013, 5 Marks) • Alpha and gamma interferons.
Ans. Refer to Ans 8 of same chapter. • Interleukins.
Q.11. Write short note on AIDS. ♦♦ Bone marrow transplantation.
 (Nov 2008, 5 Marks) (Apr 2008, 5 Marks) ♦♦ Anti-CD3 or IL-2 after HAART (Highly Active Anti-
Ans. AIDS—Acquired immunodeficiency syndrome. Retroviral Therapy).
AIDS is the end stage of a progressive state of immunodeficiency. ♦♦ Psychotherapy
♦♦ Counseling of HIV patients and their families.
Causative organism: Human immunodeficiency virus (HIV).
♦♦ Life-expectancy after initial HIV infection is 8–l0 years.
Mode of Transmission
Prevention and Control
♦♦ Sexual intercourse
♦♦ Safe sexual contact (use of condom)
♦♦ Mother to fetus
♦♦ Prevent sharing of needles among drug abusers.
♦♦ Through contaminated needles
♦♦ Separate and sterilized needle should be used for each
♦♦ Through contaminated blood transfusion.
patient.
General Features ♦♦ Blood transfusion should be done after HIV testing.
♦♦ Health education.
♦♦ Weight loss more than l0%.
♦♦ Fever more than l month. Q.12. Describe etiology, epidemiology, pathology and
♦♦ Diarrhea more than l month. prevention of AIDS in dental practice. 
♦♦ Neuralgia, arthralgia, headache.  (Feb 2002, 10 Marks)
♦♦ Generalized lymphadenopathy. Ans. Etiology (in dental clinic)
♦♦ Cutaneous rashes, dermatitis, fungal, bacterial, viral • Due to infected instruments.
(herpes simplex l and 2) infection. • Uses of infected needles.
♦♦ Dental infection, gingivitis, candidiasis of oral cavity and • Infected blood transfusion during dental procedures.
esophagus. • Contaminated gloves and dressing materials.
♦♦ Varicella zoster infection.
Epidemiology
♦♦ Opportunistic infections
♦♦ Poor healing after surgery, trauma, infection with more ♦♦ AIDS was first describe in US the disease has now attained
complications. high proportions involving all continents.
Section 2:  General Surgery  287

♦♦ Africa constitutes 50% of all positive cases globally ♦♦ Staff who enter the theater wear overshoes, gloves and
♦♦ 1 in every 100 sexually active adult worldwide is infected disposable water-resistant gowns and eye protectors.
with HIV.
♦♦ In India epicenter of epidemic lies in states of Maharashtra Surgical Technique
or Tamil Nadu which together compro­mise about 50% HIV ♦♦ Avoid sharp injury.
♦♦ Avoid “needlestick” injuries
Pathology ♦♦ Proper autoclaving at the end of surgery.
It is largely related to depletion of CD4 + T-cells resulting in: AZT—Zidovudine, lamivudine and indinavir should be
given for the health workers following exposure of susceptible
Selective tropism and internalization area to infected material from AIDS patient.
Q.14. Write short note on prevention and precautions to be
Uncoating and proviral DNA integration
taken on treating a HIV positive patient.
 (Jan 2012, 5 Marks)
Budding and syncytia formation
Or
Cytopathic effects Write short note on precautions to be taken while treat­
ing surgical patient with HIV (Aug 2012, 5 Marks)
Effects on monocytes and macrophages Or
Write in brief on universal precaution for AIDS.
HIV infection of nervous system (Dec 2010, 5 Marks)
Or
B-cell dysfunction
Write briefly on universal precautions in HIV and
hepatitis. (Jan 2016, 2 Marks)
Prevention in Dental Clinic
Ans.
♦♦ Needle sharing among patients is avoided.
♦♦ Instruments are properly sterilized by autoclave or proper Preventions and Precautions to be Taken While Treating
use of chemical sterilization. a HIV Positive or Hepatitis B Patient
♦♦ Before commencement of surgery dentist should use
proper asepsis measures. ♦♦ Care in handling sharp objects such as needles, blades.
♦♦ Patient should be educated about AIDS. ♦♦ All cuts and abrasions in an HIV or hepatitis B patient
should be covered with a waterproof dressing
Q.13. Write in brief about the HIV and dental surgeon. ♦♦ Minimal parenteral injections
 (Feb 2013, 5 Marks) (Mar 2007, 5 Marks) ♦♦ Equipment and areas which are contaminated with secre-
Ans. The HIV is the virus which results in the causation of tions should be wiped with sodium hypochlorite solution
the AIDS. or 2% glutaraldehyde.
Prophylactic measures to be adopted by dental surgeon ♦♦ Contaminated gloves, cottons should be incinerated.
while treating AIDS patient. ♦♦ Equipments should be disinfected with glutaraldehyde.
♦♦ Disposable equipment (drapes, scalpels, etc.) should be
In OPD used, whenever possible.
♦♦ Any patient with open wound, gloves are worn when ♦♦ Walls and floor should be cleaned properly with soap water.
examining a patient. ♦♦ Separate operation theatre and staff to do surgeries to HIV
♦♦ During dental diagnostic procedure gloves should be or hepatitis B patients is justifiable
worn. ♦♦ Avoid shaving whenever possible before surgery in HIV
♦♦ Use disposable instruments. or hepatitis B patients.
♦♦ Reusable instruments are cleaned in soap and water and ♦♦ All people inside the theater should wear disposable
emerged in glutaraldehyde. gowns, plastic aprons, goggles, overshoes and gloves.
♦♦ No surgical procedure involving sharp instrument should ♦♦ Surgeons, assistants and scrub nurse should wear in ad-
be performed in OPD. dition double gloves.
♦♦ Suction bottle should be half-filled with freshly prepared
In Operating Room glutaraldehyde solution.
♦♦ Dental chair is covered with a single sheet of polythene. ♦♦ Soiled body fluids should be diluted with glutaraldehyde.
♦♦ The number of personnel in dental operating room should ♦♦ Accidental puncture area in surgeon or scrub nurse should
be reduced to minimum. immediately washed with soap and water thoroughly
♦♦ The staff with abrasion or lacerations on their hands is not ♦♦ Theater should be fumigated after surgery to HIV or
allowed inside the operating room. hepatitis B patient.
288   Mastering the BDS IIIrd Year  (Last 25 Years Solved Questions)

Q.15. Write briefly on necrotizing fasciitis or subdermal Clinical Features


gangrene. (Aug 2011, 5 Marks)
♦♦ Lesion occurs in limbs, lower abdomen, groin and
Ans. It is defined as rapidly progressing necrosis of sub- perineum.
cutaneous tissue and fascia usually sparing the muscles ♦♦ Presence of sudden swelling and pain in part with edema,
and accompanied by toxicity, high fever and apathy. discoloration, necrotic areas and ulceration.
Etiology ♦♦ Presence of foul-smell discharge
♦♦ Presence of high-grade fever with chills and hypotension
Bacteria such as Streptococcus pyogenes, anaerobes, coliforms, ♦♦ Oliguria with acute renal failure due to tubular necrosis.
Gram-negative organisms ♦♦ Jaundice.
♦♦ Rapid spread in short period (in few hours).
Types
♦♦ Features of multiple organ dysfunction syndrome with
♦♦ Type I: Polymicrobial (80%): Mixed infection; by non— drowsy, ill-patient.
group A streptococci with anaerobes or clostridial or ♦♦ Condition, if not treated properly may be life-threatening.
enterobacteriaceae (E coli, Pseudomonas). It is common in ♦♦ The subdermal spread of gangrene is always much more
perineum, trunk and postoperative wounds; common in extensive than appears from initial examination.
diabetics and immunosuppressed people.
♦♦ Type II: Monomicrobial: It is due to group A hemolytic Management
streptococci or methicillin-resistant Staphylococcus aureus
♦♦ IV fluids, fresh blood transfusion.
(MRSA). It is common in young individual; common in
♦♦ Antibiotics depend on culture and sensitivity or broad-
extremities without any comorbid status.
spectrum antibiotics. High-dose penicillins are very
♦♦ Type III: It is Gram-negative rod (Vibrio vulficus) after
a minor trauma; associated with chronic liver disease, effective. Clindamycin, third generation cephalosporins,
diabetes, steroid therapy, chronic kidney disease. It is rare. aminoglycosides are also often needed.
♦♦ Type IV: It is due to fungal infection commonly, Aspergillus ♦♦ Catheterization and monitoring of hourly urine output.
zygomycetes. it is also rare. ♦♦ Electrolyte management and monitoring.
♦♦ Control of diabetes, if patient is diabetic.
Predisposing Factors ♦♦ Oxygen, ventilator support, dopamine, dobutamine
♦♦ In old age supplements, whenever required.
♦♦ In smokers ♦♦ Radical wound excision of gangrenous skin and necrosed
♦♦ Diabetics tissues at repeated intervals.
♦♦ Immunosupressed individuals ♦♦ Vacuum-assisted dressing is given.
♦♦ Malnourished ♦♦ Once patient recovers and healthy granulation tissue
♦♦ Obesity appears split skin grafting is done. Mesh graft is needed.
♦♦ Patients on steroid therapy Q.16. Define and describe differentiating features of tetanus
♦♦ HIV patients. and gas gangrene. (Jan 2012, 5 Marks)

Ans.
Features Tetanus Gas gangrene
Definition It is an infective condition leading to reflex muscle spasm It is an infective gangrene caused by clostridial organisms
and is often associated with tonic clonic convulsions. involving mainly skeletal muscles as edematous myonecrosis.
Etiology Clostridium Tetani Clostridium welchii (perfringens)
Incubation period 7–10 days 1–2 days
Toxins Tetanospasmin and Tetanolysin Lecithinase (alpha toxin), beta-toxin, Epsilon toxin
Symptoms Trismus is most common Extensive necrosis of muscle with production of gas which
stains muscle brown
System affected Affect central nervous system Affects muscular system
Treatment Tracheostomy is the live-saving procedure Amputation of involved organ is the life saving procedure
Prophylaxis Inj. Antitetanus globulin Penicillin should be given as prophylactic antibiotic

Q.17. Discuss the management of tetanus. Ans. For management of tetanus in detail, refer to Ans 8 of
 (Dec 2012, 10 Marks) same chapter.
Section 2:  General Surgery  289

Q.18. Enumerate differences between virus of Hepatitis B and HIV. (June 2014, 2 Marks)
Ans.

Features Virus of Hepatitis B HIV


Diameter 42 nm 90-120 nm
Genome DNA RNA
Symmetry Icosahedral Spherical
Laboratory diagnosis HBsAg is present ELISA is positive and confirmation is done by western blot

Q.19. Enumerate differences between tetanus and strychnine poisoning. (June 2014, 2 Marks)
Ans.

Tetanus Strychnine poisoning


Tetanus is a bacterial infection caused by Clostridium tetani Strychnine poisoning is caused due to the overdosage of chemical
known as strychnine. Strychnine is used to kill rats
Predisposing Features Predisposing Features
• Absence of tetanus toxoid immunization • Release of strychnine into drinking water
• Chronic otitis media with perforation • Contamination of food with strychnine
• Improper sterilization in ward • Strychnine is absorbed through the membranes in the nose, eyes,
• Tattooing and rusted nails or mouth
• Strychnine could be smoked or snorted as a component of street
drugs
Mechanism of Action Mechanism of Action
Tetanus release exotoxin, i.e. Tetanospasmin and tetanolysin. Strychnine prevents the proper operation of the chemical that controls
Tetanospasmin through lymphatic and perineural sheath enters CNS nerve signals to the muscles. The chemical controlling nerve signals
and block cholinesterase enzyme at anterior horn cells. This leads to works like the body’s “off-switch” for muscles. When this “off-switch” does
hyperexcitability and reflex spasm of muscles often with tonic-clonic not work correctly, muscles throughout the body have severe, painful
convulsions spasms. Eventually, the muscles tire and the person can not breathe
Time between entry of spore and appearance of first symptom is 7–10 It acts within 10–20 min after exposure.
days

Q.20. Write short note on candidiasis. Predisposing Factors


 (Feb 2013, 5 Marks) ♦♦ Changes in oral microbial flora: Marked changes in oral mi-
Ans. Candidiasis is the fungal infection caused by yeast-like crobial flora can be seen during administration of systemic
fungus, i.e. Candida albicans.
antibiotics, due to chronic use of mouthrinses, xerostomia
Types of Candidiasis due to anti – cholinergic agent. These all lead to candidiasis.
♦♦ Local irritant: Local irritants such as denture, due to heavy
♦♦ Primary candidiasis: smoking, orthodontic appliances can lead to candidiasis.
• Acute form: ♦♦ Drug therapy: Administration of immunosuppressive
–– Pseudomembranous agents, corticosteroids, head and neck radiotherapy can
–– Erythematous cause candidiasis.
• Chronic form: ♦♦ Acute and chronic diseases: Various acute and chronic diseases
–– Hyperplastic such as leukemia, diabetes, tuberculosis can cause candidiasis.
–– Erythematous ♦♦ Malnutrition state: Low serum vitamin A, low iron level and
–– Pseudomembranous low pyridoxine levels may lead to candidiasis.
• Candida-associated lesion: ♦♦ Endocrinopathy: Endocrinopathies such as hyperparathy-
–– Denture stomatitis roidism, hypothyroidism and Addison’s disease can cause
–– Angular stomatitis candidiasis.
–– Median rhomboid glossitis
• Keratinized primary lesion super-infected with Candida:
Clinical Features
–– Leukoplakia
–– Lichen planus ♦♦ It is more common in females as compared to males.
–– Lupus erythematosus ♦♦ Commonly affected sites are roof of mouth, retromolar
♦♦ Secondary candidiasis area, mucobuccal fold and tongue.
290   Mastering the BDS IIIrd Year  (Last 25 Years Solved Questions)

♦♦ Patient complaints of bad taste and spicy food can cause • Nystatin oral pastilles can be given, i.e. one or two
discomfort. pastilles five times a day.
♦♦ Pearly white or bluish white plaques are seen on mucosa. • Amphotericin B 5–10 mL of oral solution used as rinse
Mucosa adjacent to these plaques appears red and mod- and then expectorated 3 to 4 times a day.
erately swollen. ♦♦ Systemic treatment:
♦♦ White patches are easily wiped off with wet guage which • Nystatin 250 mg TDS for 2 weeks followed by 1 troche
leaves normal or erythematous area. per day for third week.
♦♦ Candida in chronic form does not rub off by lateral pres- • Ketoconazole 200 mg tablet with food once daily.
sure. Lesions are slightly white to dense white with cracks • Fluconazole 100 mg tablet OD for 2 weeks
and fissures occasionally present. Borders are often vague, • Itraconazole 200 mg tablet OD for 2 weeks
which produces appearance of epithelial dysplasia. Q.21. Describe etiopathogenesis, clinical features and man­
Diagnosis agement of tetanus. (Nov 2014, 8 Marks)
Clinically, pseudomembranous lesion is scrapped off which is Ans. For clinical features and management, refer to Ans 8 of
diagnostic of candidiasis. same chapter.

Treatment Etiopathogenesis
♦♦ Removal of cause: Tetanus is caused by Gram-positive, anaerobic, motile, non-
• Any of the local irritant should be removed. capsulated, organism with peritrichous flagella with terminal
• Withdrawal of antibiotics is done. spores, i.e. bacillus Clostridium tetani. Spore is the infective agent
♦♦ Topical treatment: and is found in soil, dust, manure, etc. Spore enters the wound
• Cotrimazole, one troche 10mg is dissolved in mouth through prick injuries which result from road-traffic accidents,
for 5 times a day. penetrating injuries, foreign body, etc.
Section 2:  General Surgery  291

Q.22. Write short note on prophylaxis of hepatitis B. ♦♦ Chancre remains painless before get infected after infected
 (Apr 2015, 3 Marks) it becomes tender.
Ans. Following is the prophylaxis of hepatitis B: ♦♦ Chancre can disappear after 10 days without any therapy.
• Recombinant hepatitis B vaccine having HBsAg Q.24. Define and classify surgical infection. Describe the
capable of producing active immunization. surgical principles, and treatment of infection in dental
• Usually three injections of vaccine should be given
surgery. (Mar 1997, 15 Marks)
IM during current, first and sixth month. These
Ans.
vaccinations provide 90% of prophylaxis from
hepatitis B virus. Surgical Infections
• If patient is immunocompromised larger doses of
vaccination should be given. A disease caused by microorganisms especially those that
• Passive immunization is provided by I.M. injection release toxins or invade body tissue during or after surgical
of hyperimmune serum globulins which is given procedures.
within 24 hours or almost within a week of exposure Classification of Surgical Infections
to infected blood.
• Active along with passive immunization is provided ♦♦ According to depth of wound infection:
to the paramedicos who has undergone needle stick • Superficial incisional surgical site infection
injury, to newborn babies of hepatitis B positive • Deep incisional surgical site infection
mothers and to regular sexual partner of hepatitis • Organ space infection.
B positive patient. Dosage is 500 IU for adults and ♦♦ According to etiology:
200 IU for babies. • Primary infection
Following precautions are to be taken for prevention • Secondary infection.
from hepatitis B: ♦♦ According to the time:
• Avoid infected blood transfusion, body organs, • Early infection
sperms and other tissues. Blood should be screened • Intermediate infection
before transfusion. • Late infection.
• Strict sterilization process should be ensured in Surgical Principles in Dental Surgery
clinics.
• Presterilized needles and syringe should be used. ♦♦ Follow the shortest and most direct route to the accumula-
• Avoid injections unless they are absolutely tion of exudates or pus, but always preserving integrity of
necessary. anatomical structures.
• Carrier should be told not to share razors or tooth ♦♦ Performing incisions with esthetic criteria in areas of
brushes, use barrier methods of contraceptions, minimal impact as on the face.
avoid blood donation. ♦♦ Place the incisions in areas of healthy mucosa or skin,
avoiding areas with fluctuation and atrophic alterations.
Q.23. Write briefly on primary chancre. (Jan 2016, 2 Marks)
♦♦ Perform strictly cutaneous or mucosal incisions (with a
Or No. 11 blade)
Write short note on primary chancre. ♦♦ The incision is penetrated using hemostat or sinus for-
ceps in closed position, advanced into the pus locules,
 (Aug 2012, 5 Marks)
by blunt dissection in open position of the sinus forceps.
Ans.
The hemostat is withdrawn in the same position in open
♦♦ Primary chancre is the hallmark sign of primary syphilis state to avoid damage to anatomical structures such as
caused due to Treponema pallidum. nerves, vessels.
♦♦ Primary chancre is also known as Hunterian chancre or ♦♦ Choice of appropriate drainage material is according to the
hard chancre. site of infection. Avoid using gauze as drainage material,
♦♦ Primary chancre occurs on penis of males and cervix of since secretions would be retained and coagulate, thereby
females. It can also be seen over extragenital sites such as creating a tamponade that would cause the infection to
fingers, perianal region, nipples, lips tonsils, tongue and persist.
palate.
♦♦ Chancre is slightly raised, ulcerated, non-tender, non- Treatment of Infection in Dental Surgery
bleeding and firm plaque which is round, indurated along ♦♦ Excision of sinus:
with rolled raised edges. • In most of the cases, the abscess escapes the tissue
♦♦ It starts as a papule, then enlarges to various centimeters spaces spontaneously, through a sinus if left without
and converted to ulcer. any treatment for sufficient period of time.
♦♦ Intraorally chancre appears as narrow copper colored • Pus discharge through the skin in a location
with slight raised borders and in center with reddish- unfavorable for drainage follows and the resulting
brown base. scar is always puckered, thickened and depressed.
292   Mastering the BDS IIIrd Year  (Last 25 Years Solved Questions)

• Further, the sinus will become chronic unless the Q.25. Write short note on AIDS and surgeon.
original source of infection is removed, and it is  (Apr 2007, 5 Marks)
subjected to exacerbations and remissions with Or
attempts at healing during the quiescent phase.
Write in brief about HIV and surgeons.
• To treat this sinus, an elliptical incision is made around
its external orifice so that on closure the scar lies in  (Dec 2012, 5 Marks)
Langer’s line without puckering. Ans. The HIV is the virus which results in the causation of
• This is done with a scissors, using which the sinus tract the AIDS.
is followed to its source which is usually found on the Prophylactic measures to be adopted by surgeon
bony surface of the jaws. Then a deep soluble suture while treating AIDS patient.
is inserted to eliminate the dead space and the skin
In OPD
wounds are closed with careful eversion of the edges.
♦♦ Antibiotic therapy: ♦♦ Any patient with open wound, gloves are worn when
Since dental infections are caused by aerobic and anaerobic examining a patient.
bacteria following antibiotics are given: ♦♦ During proctoscopy or sigmoidoscopy, gloves should be
• Amoxycillin with clavulanic acid 2 g one hour prior to worn.
surgery followed by 2 g every 12 hours for 5 to 7 days. ♦♦ Hand gloves and eye protection during flexible endoscopy.
• As an alternative regime clindamycin 300 mg every 6 ♦♦ Use disposable instruments.
hourly for 5–7 days can be given. ♦♦ Reusable instruments are cleaned in soap and water and
• When the patient fails to respond to empirical emerged in glutaraldehyde.
antibiotic therapy and after treatment of the causes ♦♦ No surgical procedure involving sharp instruments is
within 48 hours. performed in OPD.
• When the infection is disseminated to other fascial
spaces despite initial treatment. In Operating Room
• In an immunosuppressed patient, or if he/she has prior ♦♦ Operating table is covered with a single sheet of polythene.
history of bacterial endocarditis and does not respond ♦♦ The number of personnel in operating room should be
to the initial antibiotic. reduced to minimum.
♦♦ Supportive therapy: ♦♦ The staff with abrasion or lacerations on their hands is not
• Apart from antibiotic therapy, patients with dental allowed inside the operating room.
infection may require complementary measures ♦♦ Staff who enter the theater wear over-shoes, gloves and
particularly in severe cases with considerable systemic disposable water-resistant gowns and eye protectors.
involvement or in life-threatening situation.
• Analgesics, nonsteroidal anti-inflammatory drugs Surgical Technique
(NSAIDs) and nutritional support are mandatory. ♦♦ Avoid sharp injury.
• Patients with infection and fever present a considerable ♦♦ Prefer scissors or diathermy to scalpel
loss of body fluids—250 mL for every degree ♦♦ Use of skin clips
(centigrade) temperature rise. ♦♦ Avoid “needle-stick” injuries
• Ambulatory patients must drink 8–10 glasses of water ♦♦ Proper autoclaving at the end of surgery.
or any other liquid. ♦♦ AZT—Zidovudine, lamivudine and indinavir should
• Intravenous fluids can be given to those patients who be given for the health workers following exposure of
are hospitalised to improve hydration. susceptible area to infected material from AIDS patient.
• The daily calorie requirement also increases by 13%
Q.26. Write short note on HIV.
for each degree (centigrade) above normal body
temperature.  (Feb 2015, 5 Marks) (Feb 2014, 3 Marks)
• Thermal agents should be used to aid the body defenses. Or
• Heat produces vasodilatation and increased
circulation, more rapid removal of tissue breakdown Write briefly on HIV. (June 2015, 2.5 Marks)
products and greater influx of defensive cells and Ans. HIV means human immunodeficiency virus.
antibodies. • HIV was discovered in 1983 and first case was
• A crucial aspect to be considered in these patients is detected in UK
the potential risk onset of respiratory impairment, • It was discovered by Barre-Sinoussi and Montagnier
requiring airway monitoring, perhaps even on an • HIV virus is classified under HTLV-III
emergency basis, by means of endotracheal intubation, • HIV is of two types, i.e. HIV-1 and HIV-2 which are
cricothyrotomy or tracheotomy. the retroviruses.
Section 2:  General Surgery  293

Mode of Transmission of HIV Non-nucleoside reverse transcriptase inhibitor (NNRTI): Nevi-


♦♦ Sexual intercourse either vaginal or anal. rapine, delavirdine.
♦♦ Needle pricks, i.e. using unsterilized needles for injections, Protease inhibitors: Ritonavir, indinavir, amprenavir.
In IV drug abusers, careless handling ♦♦ Treatment of opportunistic infections.
♦♦ Mother to child: During birth through vaginal secretion, ♦♦ Treatment of tumors.
transplacental, through breast milk ♦♦ Immunotherapy:
♦♦ Through blood transfusions and organ transplantations • Alpha and gamma interferons.
• Interleukins.
Pathogenesis ♦♦ Bone marrow transplantation.
♦♦ Anti-CD3 or IL-2 after HAART (Highly Active Anti- Ret-
Envelope glycoprotein of HIV binds with surface molecules
roviral Therapy).
of CD4 cells of T-lymphocytes, monocytes, macrophages,
♦ Psychotherapy
Langerhans cells and dendritic cells of all tissues.
♦ Counseling of HIV patients and their families.
CD4 of lymphocytes and T helper cells control normal ♦ Life-expectancy after initial HIV infection is 8–l0 years.
immune response. HIV suppresses immune response
completely suppressing B-cell. Finally, it dismantles and destroy Q.27. Describe the mode of spread of HIV. Describe treat­
immune system making the individual prone to opportunistic ment. (Jan 2017, 10 Marks)
infections. Ans.

Clinical Classification of HIV Mode of Spread of HIV


♦♦ Acute infections ♦♦ Sexual intercourse either vaginal or anal.
♦♦ Asymptomatic but positive HIV. ♦♦ Needle pricks, i.e. using unsterilized needles for injections,
♦♦ Persistent generalized lymphadenopathy In IV drug abusers, careless handling
♦♦ AIDS (HIV-related diseases): ♦♦ Mother to child: During birth through vaginal secretion,
• Constitutional diseases such as weight loss, fever, transplacental, through breast milk
diarrhea ♦♦ Through blood transfusions and organ transplantations
• Neurological diseases such as dementia, neuropathy,
myelopathy Treatment of HIV
• Opportunistic infections ♦♦ Antiviral therapy or HAART therapy:
• Malignancies: Kaposi’s sarcoma, non-hodgkin’s Nucleoside reverse transcriptase inhibitor (NRTI): Zidovudine,
lymphoma didanosine, abacavir, lamivudine, stavudine.
• Other diseases attributable to HIV infection. Non-nucleoside reverse transcriptase inhibitor (NNRTI): Nevi-
rapine, delavirdine.
Test for HIV
Protease inhibitors: Ritonavir, indinavir, amprenavir.
♦♦ ELISA test (Screening test) ♦ Treatment of opportunistic infections.
♦♦ Western blot (Diagnostic test) ♦ Treatment of tumors.
♦♦ Polymerase chain reaction ♦ Immunotherapy:
♦♦ Anti-HIV antibody reaction Alpha and gamma interferons.
♦♦ Viremia quantification—to start the treatment and to see Interleukins.
the response of anti-viral drugs. ♦ Bone marrow transplantation.
♦♦ CD4 cell count: ♦ Anti-CD3 or IL-2 after HAART (Highly Active Anti-
• Normal value > 500/mm3 Retroviral Therapy).
• Values between 200 to 500/mm3 is seen in Kaposi’s ♦ Psychotherapy
sarcoma, Candida infection and M. tuberculosis ♦ Counseling of HIV patients and their families.
• Values between 50 to 200/mm3 is seen in Pneumocystis ♦ Life-expectancy after initial HIV infection is 8–l0 years.
carinii and Toxoplasma infection
• Values< 50/mm3 is seen in atypical mycobacteria, Q.28. Describe the nosocomial infection. (Jan 2017, 10 Marks)
cytomegalovirus and lymphomas. Ans. It is an infection acquired due to hospital stay.
After the HIV infection, there is time gap for the patient
to become reactive to tests. This time is known as “Window Sources
period”. This period is variable and during this period an ♦♦ Contaminated infected wounds.
individual is infective. ♦♦ Urinary tract infections.
♦♦ Respiratory tract infections.
Treatment of HIV
♦♦ Opportunistic infections.
♦♦ Antiviral therapy or HAART therapy ♦♦ Abdominal wounds with severe sepsis.
Nucleoside reverse transcriptase inhibitor (NRTI): Zidovudine, ♦♦ Spread can occur from one patient to another, through
didanosine, abacavir, lamivudine, stavudine. nurses or hospital staff who fail to practice strict asepsis.
294   Mastering the BDS IIIrd Year  (Last 25 Years Solved Questions)

♦♦ It is more common in: Treatment (Only modalities)


• Diabetics ♦♦ Antibiotic therapy
• Immunosuppressed individuals ♦♦ Excision of sinuses
• Patients on steroid therapy and life-supporting ♦♦ Actinomycosis of right iliac fossa should undergo hemi-
machines colectomy
• Instrumentations (including catheter, IV cannula,
tracheostomy tube) Q.30. Name etiological agent, pathognomic diagnostic fea­
• Patients with artificial prosthesis ture and treatment (only modalities) of anthrax.
 (Jan 2017, 3 Marks)
Organisms Ans.
♦♦ Staphylococcus aureus is the most common organism
causing hospital-acquired wound infection. Others are Etiological Agent
Pseudomonas, Klebsiella, E. coli, Proteus. Anthrax is caused by Bacillus anthracis
♦♦ Streptococcus pneumoniae, Haemophilus, Herpes, Varicella,
Aspergillus, Pneumocystis carinii are the most common Pathognomic Diagnostic Feature
pathogens involved in hospital-acquired respiratory tract After 3 to 4 days of incubation an itching erythematous papule
infection which spreads through droplets. develop on exposed portion of body, i.e. hand, forearm, etc.
♦♦ Klebsiella is the most common pathogen involved in hospi- The papules suppurate and form black slough. This lesion is
tal acquired UTI which is highly resistant to drugs. surrounded by vesicles known as malignant pustule.
Management
Treatment (Only Modality)
Most of the time, organisms involved are multidrug resistant,
Antibiotic therapy is given.
virulent and hence, cause severe sepsis.
♦♦ Antibiotics. Q.31. Name etiological agent, pathognomic diagnostic fea­
♦♦ Isolation. ture and treatment (only modalities) of chancre.
♦♦ Blood, urine, pus for culture and sensitivity to isolate the  (Jan 2017, 3 Marks)
organisms. Ans.
♦♦ Blood transfusion, plasma or albumin therapy.
♦♦ Ventilator support. Etiological Agent
♦♦ Maintaining optimum urine output. It is caused by Treponema palladium
♦♦ Nutritional support.
Pathognomic Diagnostic Feature
Prevention
A shallow, painless, indurated, non-bleeding ulcer seen
♦♦ Isolation of patients with badly infected open wounds.
on genitalia, lips, breasts and anal region, this is known as
♦♦ Severe RTI/UTI.
Hunterian chancre. This is confirmed by dark-field microscopic
♦♦ Following strict aseptic measures in OT and in ward by
study of discharge for organism.
hospital attendants.
♦♦ Proper cleaning and use of disinfectant lotions and sprays Treatment (Modality only)
for bedpans, toilets and floor. Antibiotic therapy should be given.
♦♦ The precipitating causes have to be treated, along with car-
ing for proper nutrition and improving the anemic status Q.32. Write short note on septicemia. (Feb 2013, 5 Marks)
by blood transfusion.  (Apr 2007, 10 Marks) (Feb 2004, 5 Marks)
Ans. Presence of overwhelming and multiplying bacteria
Q.29. Name etiological agent, pathognomic diagnostic fea­ in blood with toxins causing systemic inflammatory
ture and treatment (only modalities) of actinomycosis. response syndrome or multiorgan dysfunction syndrome.
 (Jan 2017, 3 Marks)
Clinical Features
Ans.
♦♦ Intermittent high-grade pyrexia (fever)
Etiological Agent ♦♦ Rigors and chills.
♦♦ Jaundice due to liver damage.
It is caused by Actinomyces israelii
♦♦ Peripheral circulatory failure.
Pathognomic Diagnostic Feature ♦♦ Intravascular coagulation.
♦♦ Patient may go into septic shock.
Multiple discharging sinuses which discharge sulphur granules ♦♦ Septic shock is secondary to sepsis; it is characterized by
and there is no lymph node enlargement present. inadequate perfusion of tissue.
Section 2:  General Surgery  295

♦♦ The septic shock differs from all other forms of shock by ♦♦ Jaundice due to liver damage.
having hot stage before cold stage. ♦♦ Peripheral circulatory failure.
♦♦ Intravascular coagulation.
Types ♦♦ Patient may go into septic shock.
♦♦ Gram-positive septicemia: It is due to staphylococci, strepto- ♦♦ Septic shock is secondary to sepsis; it is characterized by
cocci, pneumococci, etc. It is common in children, old age. inadequate perfusion of tissue.
diabetics and after splenectomy. Common origin is skin, ♦♦ The septic shock differs from all other forms of shock by
respiratory infection having hot stage before cold stage.
♦♦ Gram-negative septicemia is common in acute abdomen such
Treatment
as peritonitis, abscess, urinary infections, biliary infections,
postoperative sepsis. It is commonly seen in malnutrition, ♦♦ Management of primary focus of infection.
old age, diabetics, immunosuppressed people. Common ♦♦ Broad-spectrum antibodies are given.
focus of infection is gram-negative infection is urinary ♦♦ Blood and fluid transfusion to correct septic shock.
infection, abscess or infected wounds, biliary sepsis, post- ♦♦ Injection of hydrocortisone in case of septic shock may
operative wounds. be useful.
Investigations Pyaemia
♦♦ Urine/pus/discharge culture Presence of multiplying bacteria in blood as emboli which
♦♦ Blood culture spread and lodge in different organs in the body like liver, lungs,
♦♦ Hematocrit kidneys, spleen, brain causing pyaemic abscess. This may lead
♦♦ Electrolyte assessment to multiorgan dysfunction syndrome (MODS). It may endanger
♦♦ PO2 and CO2 analysis life, if not treated properly.
♦♦ Blood urea, serum creatinine, liver function test
Clinical Features
Complications
♦♦ Fever with chills and rigors
♦♦ Disseminated intravascular coagulation ♦♦ Jaundice, oliguria, drowsiness
♦♦ Acute respiratory distress syndrome ♦♦ Hypotension, peripheral circulatory collapse and later
♦♦ Liver dysfunction coma with MODS
♦♦ Renal failure
♦♦ Bone marrow suppression-thrombocytopenia Common Causes
♦♦ Multiorgan failure ♦♦ Urinary infection (most common)
Treatment ♦♦ Biliary tract infection.
♦♦ Lower respiratory tract infection.
♦♦ Antibiotics such as cefoperazone, ceftazidime, cefotaxime, ♦♦ Abdominal sepsis of any cause.
amikacin, tobramycin, metronidazole. ♦♦ Sepsis in diabetics and immunosuppressed individuals
♦♦ Fresh blood transfusion. such as HIV, steroid therapy.
♦♦ Adequate hydration.
♦♦ Oxygen supplementation. Investigations
♦♦ Ventilatory support.
♦♦ Electrolyte management. ♦♦ Total leucocyte count, platelet count
♦♦ Parenteral nutrition ♦♦ Biliary tract infection
♦♦ CVP line for monitoring and perfusion. ♦♦ Pus, blood and urine culture depending on the need.
♦♦ Fresh-frozen plasma or platelets in disseminated intravas- ♦♦ Blood urea, serum creatinine
cular coagulation ♦♦ Liver function tests, prothrombin time
♦♦ Chest X-ray, USG abdomen
Q.33. Write short note on septicemia, toxemia and pyaemia. ♦♦ CT chest/abdomen/brain as needed
 (Jun 2010, 5 Marks) ♦♦ Arterial blood gas analysis, if needed
Ans. For septicemia, refer to Ans 32 of same chapter.
Treatment
Toxemia
Distribution throughout the body of poisonous product ♦♦ Monitoring of vital parametres
of bacteria growing in a focal or local site, thus producing ♦♦ Antibiotics mainly cephalosporins
generalized symptoms such as: fever, diarrhea, vomiting. ♦♦ IV fluids and maintenance of urine output.
♦♦ Hydrocortisone
Clinical Features ♦♦ Blood and plasma transfusion
♦♦ Intermittent high-grade pyrexia (fever) ♦♦ Nasal oxygen, ventilator support, monitoring of pulmo-
♦♦ Rigors and chills. nary function.
296   Mastering the BDS IIIrd Year  (Last 25 Years Solved Questions)

Q.34. Describe differentiating features of septicemia and Contd...


pyemia (Jan 2017, 4 Marks) Bacteraemia Septicemia
Ans. Following are the differentiating features of septicemia Caused by Staphylococcus, Staphylococci, are thought to
and pyemia: Streptococcus, Pseudomonas, cause more than 50% of cases
Septicemia Pyemia Haemophilus, E. coli, dental of sepsis. Other commonly
procedures, herpes (including implicated bacteria include
Septicemia is a disease Pyemia is the type of septicemia herpetic whitlow), urinary tract Streptococcus pyogenes,
In septicemia actual "putrid" In pyemia ,the pus cells were floating infections, peritonitis, Clostridium Escherichia coli, Pseudomonas
matter (of unknown character) with blood difficile colitis, intravenous drug aeruginosa, Klebsiella species
was supposed to be present in use, and colorectal cancer. and even Candida spp.
the blood
Q.36. Differentiate between Gram-negative and Gram-posi­
Presence of overwhelming and Presence of multiplying bacteria in tive septicemia and management. (Jan 2018, 10 Marks)
multiplying bacteria in blood blood as emboli which spread and Ans.
with toxins causing systemic lodge in different organs in the body
inflammatory response like liver, lungs, kidneys, spleen, Gram-positive septicemia Gram-negative septicemia
syndrome or multiorgan brain causing pyemic abscess. It is caused by staphylococci, It is caused by E. coli, Klebsiella,
dysfunction syndrome. streptococci, pneumococci, etc. Pseudomonas and Proteus
It is differentiated into two It is of various types, i.e. arterial, Overwhelming post-splenectomy It leads to urinary infection, biliary
types, i.e. Gram-positive and cryptogenic, metastatic, portal infection is the classical example.sepsis, peritonitis, abdominal
Gram negative infection, post-operative sepsis.
It is common in children, old age, I t i s c o m m o n l y s e e n i n
Gram positive or Gram- Mainly Gram-negative bacteria are diabetics and after splenectomy. malnourished, old age, diabetics
negative bacteria are the the etiological agents and immunocompromised
etiological agents. people.
Septicemia comprehends Pyemia in its present acceptance Common origin is skin and Commonly occur in urinary tract
systemic infection from various comprehends a focus of necrotic respiratory infection and wounds
causes; the original infection is tissue, caused by the peculiar Leads to gram-positive septic Leads to Gram-negative septic
not always to be determined, organisms of pus development with shock shock
and it is so often of a mixed pus cells present in greater or less Gram-positive septicemia is Gram-negative septicemia is
character that its real nature is quantities. caused by exotoxin caused by endotoxin
difficult of determination. In this fluid loss, hypotension Urinary/gastrointestinal/biliary
Q.35. Describe differentiating features of bacteremia and is common with normal cardiac and respiratory foci is common.
output
septicemia. (Jan 2017, 4 Marks)
Ans. Management of Gram-Positive and Gram-
Bacteremia Septicemia Negative septicemia
Bacteremia is the simple Septicemia is the presence and ♦♦ Correction of fluid and electrolyte by crystalloids, blood
presence of bacteria in the blood. multiplication of bacteria in the transfusion. Perfusion is very/most important.
blood. ♦♦ Appropriate antibiotics—third generation cephalosporins/
Bacteremia is not as dangerous Septicemia is a potentially life- aminoglycosides.
as septicemia. threatening infection. ♦♦ Treat the cause or focus—drainage of an abscess;
Less amount of bacteria are Large amounts of bacteria are laparotomy for peritonitis; resection of gangrenous bowel;
present in blood. present in the blood. wound excision.
This may occur through a wound It can arise from infections ♦♦ Pus/urine/discharge/bile/blood culture and sensitivity for
or infection, or through a surgical throughout the body, including antibiotics.
procedure or injection. infections in the lungs, abdomen, ♦♦ Critical care, oxygen, ventilator support, dobutamine/
and urinary tract. dopamine/noradrenaline to maintain blood pressure and
Toxins are not produced. Toxins may be produced by urine output.
bacteria. ♦♦ Activated C protein prevents the release of inflammatory
Bacteremia usually causes no It shows symptoms such as mediators and blocks the effects of these mediators on
symptoms or it may produce chills, fever, prostration, very fast cellular function.
mild fever. respiration and/or heart rate. ♦♦ Monitoring the patient by pulse oximetry, cardiac status,
It can resolve without treatment. Untreated septicemia can quickly urine output, arterial blood gas analysis.
progress to sepsis. ♦♦ Short-term (one or two doses) high-dose steroid therapy
to control and protect cells from effects of endotoxemia. It
R a p i d l y r e m o v e d f r o m t h e Antibiotics will be used to treat the improves cardiac, renal and lung functions. Single dose of
bloodstream by the immune bacterial infection that is causing
methylprednisolone or dexamethasone which often may
system. septicemia.
be repeated again after 4 hours is said to be effective in
Contd... Gram-negative septicemia.
Section 2:  General Surgery  297

Q.37. Write short answer on Australian antigen.


Non-healing ulcer is also known as chronic ulcer
 (Apr 2018, 3 Marks) depending on cause of ulcer.
Ans. A scientist named Blumberg and his coworkers in 1965 • Chronic ulcer has no tendency to heal by itself.
describe a protein antigen in serum of an Australian • The bases of ulcer are indurated (hard).
aborigine which gave positive precipitation reaction with • Floor of ulcer contain unhealthy granulation tissue
sera from two hemophiliacs who had received multiple and slough.
transfusions. This antigen was named as Australian
• Here edges depend on the cause, i.e. punched out,
Antigen.
undermined, rolled out and beaded.
♦♦ HBsAg is known as Australia Antigen and was established
• Regional draining lymph nodes may be enlarged
to be the surface component of hepatitis B virus.
but non-tender.
♦♦ Australian antigen consists of two different antigenic
determinants i.e. Causes of Chronic or Non-healing Ulcer
• A group specific antigenic determinant – a
• wo pairs of type – specific antigens – d – y and w – r. In Local Causes
these only one member of each pair is present at a time.
♦♦ Recurrent infection
♦♦ Australia antigen on the basis of type specific pairing is
♦♦ Trauma, presence of foreign body or sequestrum
divided in four types i.e.
♦♦ Absence of rest and immobilization
• adw is worldwide in distribution
♦♦ Poor blood supply, hypoxia
• adr in Asia
♦♦ Edema of the part
• ayw in Africa, India, Russia
• ayr in Africa, India, Russia ♦♦ Loss of sensation
♦♦ Various additional surface antigens i.e. q, x, f, t, j, n and g ♦♦ Periostitis or osteomyelitis of the underlying bone
are described, but their characterization is not done. ♦♦ Fibrosis of the surrounding soft tissues
• Electron microscopy of serum of hepatitis B patients ♦♦ Lymphatic diseases
show three types of particles, i.e. General/Specific Causes
–– Spherical particle: It is most abundant and is 22
nm in diameter. ♦♦ Anemia, hypoproteinemia
–– Tubular particle: It is of varying length and is 22 ♦♦ Vitamin deficiencies
nm in diameter. ♦♦ Tuberculosis, leprosy
–– Dane Particle: It is double sheilded spherical ♦♦ Diabetes mellitus, hypertension
structure which is 42 nm in diameter. This particle ♦♦ Chronic liver or kidney diseases
is complete hepatitis B virus. ♦♦ Steroid therapy locally or systemically
♦♦ The spherical and tubular particles are antigenically identi- ♦♦ Cytotoxic chemotherapy or radiotherapy
cal and are surface sub units of hepatitis B virus australia ♦♦ Malignancy
antigen (HBsAg). For Example
♦♦ Diabetic ulcer
♦♦ Malignant ulcer
♦♦ Carcinomatous ulcer
♦♦ Ulcer of squamous cell carcinoma
♦♦ Rodent ulcer.
Q.2. Write short note on differential diagnosis of ulcer on
the lateral border of tongue.
 (Sep 2004, 10 Marks)
Ans. The various types of ulcer on the lateral border of tongue
are as follows:
• Aphthous ulcer
Fig. 4:  Structure of australian antigen • Traumatic or dental ulcer
• Chronic non-specific ulcer
• Post-pertussis ulcer
4. Ulcer • Syphilitic ulcer
• Tuberculous ulcer
Q.1. Write note on non-healing ulcer. (Sep 1997, 6 Marks) • Carcinomatous ulcer
Ans. Non-healing ulcer is classified under clinical classification • Herpetic ulcer
of ulcers • Ulcer due to glossitis.
298   Mastering the BDS IIIrd Year  (Last 25 Years Solved Questions)

• Such ulcers are shallow, often multiple and grayish


yellow with slightly red undermining edges.
• These ulcers are seen at the margin (lateral border),
tip or dorsum of tongue.
• These ulcers are very painful with enlargement of
regional nodes.
♦♦ Carcinomatous ulcer:
• It is usually occur in elderly individual above the age
of 50 year.
• Common site is the margins of tongue.
• It may occur at the dorsum of tongue.
Fig. 5:  Ulcers on lateral border of tongue • It is usually single, but may be multiple.
• It is usually painless to start.
♦♦ Aphthous ulcer
• The ulcer is irregular in shape, has a raised and everted
• Minor aphthous ulcer:
edges with indurated base.
–– It is small painful ulcer seen on the tip, under­
• The floor of such ulcer is covered with necrotic debris.
surface and side of the tongue.
• It bleeds on touch.
–– The ulcer is small, superficial with white floor,
• It is fixed with underlying structures.
yellowish border and surrounded by a hyper­
emic zone. Q.3. Write short note on lesion on anterior 2/3rd of tongue.
–– It can occur at any age group, more common in  (Mar 2009, 5 Marks) (Sep 2006, 5 Marks)
females at the time of menstruation. Ans. Lesions on the anterior 2/3rd of tongue are:
• Major aphthous ulcer:
–– When they are larger, deeper, painful, they are Ulcers
called major aphthous ulcer. ♦♦ Aphthous ulcer
–– They subside within a few days. ♦♦ Tubercular ulcer
–– Temporary relief can be obtained by applying ♦♦ Traumatic ulcer
salicylate gel. ♦♦ Carcinomatous ulcer
–– Vitamin B complex is usually given. ♦♦ Gummatous ulcer.
♦♦ Traumatic or dental ulcer For details of above ulcers refer to ans 2 of same chapter.
• This ulcer occurs due to broken, sharp tooth, ill-fitting
Q.4. Classify ulcers of tongue. (Mar 2011, 3 Marks)
dentures, prosthesis, etc.
• These ulcers occur at the lateral border or under Ans. Classification of ulcers of tongue
surface of tongue. • Non-Specific:
• It often presents a slough at its base and surrounded – Traumatic or dental ulcer due to sharp tooth or
by a zone of erythema and induration. dentures
• This ulcer is quit painful. – Infective:
♦♦ Chronic non-specific ulcers - Simple ulcer due to glossitis
• Usually occur in the anterior two-thirds of the tongue. - Herpetic ulcer
• Its etiology is unknown. - Post-pertussis ulcer.
• It is moderately indurated and not very painful. – Aphthous ulcer.
♦♦ Post-pertussis ulcer • Specific:
• Occurs only in children with whooping cough. – Tubercular ulcer
• Usually seen on the upper part of frenum lingulae and – Syphilitic ulcer
in the undersurface of tongue. – Malignant ulcer.
♦♦ Syphilitic ulcer Q.5. Write short note on trophic ulcer.
• Primary ulcer, i.e. chancre found on the tip of the tongue.  (Aug 2011, 5 Marks)
• Mainly snail-track ulcers are seen in oral cavity in Ans. It is also called as decubitus ulcer or pressure sore.
second stage of syphilis. Trophic ulcer is tissue necrosis and ulceration due to
• In tertiary syphilis, the gummatous ulcers are found prolonged pressure. Blood flow to the skin stops once
on the midline in the anterior two-thirds of the tongue external pressure becomes more than 30 mm Hg and this
• The gumma found on the dorsum of tongue. causes tissue hypoxia, necrosis and ulceration. It is more
♦♦ Tuberculous ulcer: prominent between bony prominence and an external
• Young adults are usually involved. surface.
Section 2:  General Surgery  299

Etiology Ans.
It is due to: Features Curling Ulcer Cushing's Ulcer
♦♦ Impaired nutrition. Definition Acute ulcers which Acute ulcers which
♦♦ Defective blood supply. develop after major develop after cerebral
♦♦ Neurological deficit: Due to the presence of neurological burns trauma or after
deficit, trophic ulcer is also called as neurogenic ulcer/ neurological operations
neuropathic ulcer. Initially, it begins as callosity due to Type True stress ulcer Not the true stress ulcer
repeated trauma and pressure, under which suppuration Number Multiple single ulcer
occurs and gives way through a central hole which extends Symptoms Pain in epigastric region, No symptoms present
down into the deeper plane up to the underlying bone as vomiting, hematemasis
perforating ulcer. Area of Body and fundus Esophagus and
occurrence duodenum
Sites
Treatment It is conservative by Treatment is by IV
♦♦ Over ischial tuberosity giving IV ranitidine, IV ranitidine
♦♦ Sacrum pantoprazole 80 mg in
♦♦ In the heel 100 mL DNS—slow
♦♦ In relation to head of metatarsals Transfer to Commonly transferred Not commonly transferred
♦♦ Buttocks malignancy to malignancy to malignancy
♦♦ Over the shoulder
♦♦ Occiput Q.7. Write short note on ulcer. (Aug 2012, 5 Marks)
Ans. An ulcer is the break in the continuity of the covering
Clinical Features epithelium either skin or mucus membrane due to
♦♦ It occurs in 5% of hospitalized patients. molecular death.
♦♦ Ulcer is painless and is punched out.
Classification of Ulcer
♦♦ Ulcer is non-mobile and base of the ulcer is formed by bone.
Classification I (Clinical)
Management ♦♦ Spreading ulcer
♦♦ Cause should be treated. ♦♦ Healing ulcer
♦♦ Nutritional supplementation is given. ♦♦ Non-healing ulcer
♦♦ Rest, antibiotics, slough excision, regular dressings. ♦♦ Callous ulcer
♦♦ Vacuum-assisted closure: It is the creation of intermittent Classification II (Based on duration)
negative pressure of minus l25 mm Hg to promote ♦♦ Acute ulcer: Duration less than 2 weeks
formation of healthy granulation tissue. A perforated ♦♦ Chronic ulcer: Duration more than 2 weeks
drain is kept over the foam dressing covered over the
pressure sore. It is sealed with a transparent adhesive Classification III (Pathological)
sheet. Drain is connected to required vacuum apparatus. ♦♦ Specific ulcers:
Once ulcer granulates well. Flap cover or skin grafting • Tuberculous ulcer
is done. • Syphilitic ulcer
♦♦ Excision of the ulcer and skin grafting is done. • Actinomycosis
♦♦ Flaps: Local rotation or other flaps (transposition flaps). • Meleney’s ulcer
♦♦ Proper care: Change in position once in 2 hours; lifting ♦♦ Malignant ulcers:
the limb upwards for l0 seconds once in l0 minutes; nu- • Carcinomatous ulcer
trition; use of water bed/air bed/air-fluid floatation bed • Rodent ulcer
and pressure dispersion cushions to the affected area; uri- • Melanotic ulcer
nary and fecal care; hygiene; psychological counseling. ♦♦ Non-specific ulcers:
Regular skin observation; keeping skin clean and dry • Traumatic ulcers
(using regular use of talcum powder); oil massaging of • Arterial ulcer
the skin and soft tissues using clean, absorbent porous • Venous ulcer
clothing; control and prevention of sepsis helps in the • Trophic ulcer
management. • Infective ulcer
• Tropical ulcer
Q.6. Define and describe differentiating features of Curling • Ulcers due to chilblain and frostbite
ulcer and cushing ulcer. (Jan 2012, 5 Marks) • Martorell’s hypertensive ulcer
300   Mastering the BDS IIIrd Year  (Last 25 Years Solved Questions)

• Bazin’s ulcer • Punched out edge is seen in a gummatous (syphilitic)


• Diabetic ulcer ulcer and trophic ulcer. It is due to endarteritis.
• Ulcers due to leukemia, polycythemia, jaundice, • Raised and beaded edge (pearly white) is seen in a
collagen diseases, lymphoedema rodent ulcer (basal cell carcinoma). Beads are due to
• Cortisol ulcers proliferating active cells.
• Everted edge (rolled out edge): It is seen in a carcinomatous
ulcer due to spill of the proliferating malignant tissues
over the normal skin.
♦♦ Floor: lt is the one which is seen. Floor may contain dis-
charge, granulation tissue or slough.
♦♦ Base: Base is the one on which ulcer rests. It may be bone
or soft tissue.
Induration of ulcer: It is the clinical palpatory sign which
means a specific type of hardness in a diseased tissue. It is seen
in carcinomatous ulcers.
Fig. 6:  Parts of an ulcer
Investigations for Ulcer
Parts of an Ulcer ♦♦ Study of discharge: Culture and sensitivity, acid fast bacilli
♦♦ Margin: It may be regular or irregular. It may be rounded study and cytology.
or oval. ♦♦ Edge biopsy: Biopsy is taken from the edge because edge
♦♦ Edge: Edge is the one which connects floor of the ulcer to contains multiplying cells. Usually, two biopsies are taken.
the margin. Different edges are: Biopsy taken from the centre may be inadequate because
• Sloping edge: It is seen in a healing ulcer. Its inner part of central necrosis.
is red because of red, healthy granulation tissue. Its ♦♦ X-ray of the part to look for periostitis/osteomyelitis.
outer part is white due to scar/fibrous tissue. Its middle ♦♦ FNAC of the lymph node.
part is blue due to epithelial proliferation. ♦♦ Chest X-ray, Mantoux test in suspected case of tubercu-
lous ulcer.
♦♦ Hemoglobin, ESR, total WBC count, serum protein esti-
mation.
Management of An Ulcer
♦♦ Cause should be found and treated
♦♦ Correct the deficiencies like anemia, protein and vitamins
deficiencies
♦♦ Transfuse blood, if required
♦♦ Control the pain and infection
♦♦ Investigate properly
♦♦ Control the infection and give rest to the part
♦♦ Care of the ulcer by debridement, ulcer cleaning and dress-
ing is done daily or twice daily.
♦♦ Remove the exuberant granulation tissue
♦♦ Topical antibiotics for infected ulcers only like, silver
sulphadiazine, mupirocin.
♦♦ Antibiotics are not required once healthy granulation tis-
sues, if are formed
♦♦ Once granulates, defect is closed with secondary suturing,
skin graft, flaps
Q.8. Describe briefly diabetic ulcer.
 (Apr 2017, 5 Marks)
Fig. 7:  Different types of edges-ulcers Ans. Diabetic ulcer is most common in foot. It can cause
abscess, ulcer, osteomyelitis, gangrene, septicemia.
• Undermined edge is seen in a tuberculous ulcer. Disease Initially, patient undergo toe amputation but later
process advances in deeper plane (in subcutaneous eventually land with below knee or above knee
tissue) whereas (skin) epidermis proliferates inwards. amputation.
Section 2:  General Surgery  301

Causes Different types of ulcer


♦♦ Increased glucose in the tissue precipitates infection. Classification I (Clinical)
♦♦ Diabetic microangiopathy which affects microcirculation.
♦♦ Spreading ulcer: In this edge is inflamed, irregular and
♦♦ Increased glycosylated hemoglobin decreases the oxygen
edematous. It is an acute painful ulcer, floor consists of
dissociation.
profuse purulent discharge and slough. Surrounding area
♦♦ Increased glycosylated tissue protein decreases the oxygen
is red and edematous
utilization.
♦♦ Healing ulcer: Edge is sloping with healthy pink/red
♦♦ Diabetic neuropathy involving all sensory, motor and
granulation tissue with scanty/minimal serous discharge
autonomous components.
on the floor, slough is absent. Surrounding area does not
♦♦ Associated atherosclerosis.
show any signs of inflammation or induration. Base is not
Sites indurated. Three zones are seen innermost red zone of
healthy granulation tissue, middle bluish zone of growing
♦♦ Foot-plantar aspect is the most common site epithelium, outer white zone of fibrosis and scar formation.
♦♦ Leg ♦♦ Non-healing ulcer: In this ulcer, edge of the ulcer depend
♦♦ Upper limb, back, scrotum, perineum on the cause punched out (trophic), undermined (tuber-
♦♦ Diabetic ulcer may be associated with ischemia culosis), rolled out (carcinomatous ulcer), beaded (rodent
♦♦ Ulcer is usually spreading and deep ulcer); floor consists of unhealthy granulation tissue and
Investigations slough, and serosangguineous/purulent/bloody dis-
charge, regional draining lymph nodes are enlarged but
♦♦ Blood sugar both random and fasting. non – tender.
♦♦ Urine ketone bodies. ♦♦ Callous ulcer: This is a chronic non – healing ulcer, floor
♦♦ Discharge for culture and sensitivity. consists of pale, unhealthy, flabby, whitish yellow granula-
♦♦ X-ray of the part to see osteomyelitis. tion tissue and thin scanty serous discharge or rarely with
♦♦ Arterial Doppler of the limb; glycosylated hemoglobin copious serosanguinous discharge, with indurated non –
estimation. tender edge, base in indurated, non – tender and is often
fixed. Ulcer does not show any tendency to heal. It lasts for
Problems with Diabetic Ulcer
months to years. Induration and pigmentation can be seen.
♦♦ Neuropathy, in foot – clawing of toes, hammer toe (due to
intrinsic muscle paralysis) Classification II (Based on duration)
♦♦ Multiple deeper abscesses; osteomyelitis of deeper bones ♦♦ Acute ulcer: Duration less than 2 weeks
are common. ♦♦ Chronic ulcer: Duration more than 2 weeks
♦♦ Reduced leukocyte function; resistant infection; spread-
ing cellulitis Classification III (Pathological)
♦♦ Arterial Insufficiency ♦♦ Specific ulcers:
♦♦ Septicemia; diabetic ketoacidosis • Tuberculous ulcer: Ulcer can be single or multiple;
♦♦ Associated cardiac diseases like ischemic heart disease oval or rounded with undermined edge, painless with
caseous material on the floor. Ulcer is not deep
Treatment
• Syphilitic ulcer: It has punched out edge, deep with
♦♦ Control of diabetes by using insulin. wash leather slough in the floor and indurated base.
♦♦ Proper antibiotics should be started after culture and • Actinomycosis: In this initially an induration develops.
sensitivity report It softens and bursts via skin as sinuses which
♦♦ Nutritional supplements. discharge pus and have sulphur granules.
♦♦ Regular cleaning, debridement, dressing. • Meleney’s ulcer: It is seen in postoperstive wounds
♦♦ Once granulates, the ulcer is covered with skin graft or flap. in abdomen and chest wall. This is a acute rapidly
♦♦ Revascularization procedure is done by endarterectomy spreading ulcer with destruction and deep burrowing
or thrombectomy or balloon angioplasty or arterial bypass of subcutaneous tissues.
graft. But if distal vessels are involved, then success rate ♦♦ Malignant ulcers:
is less. • Carcinomatous ulcer: This ulcer arises from prickle
♦♦ Toe foot/leg amputation. cell layer of skin. It has rolled out/everted edge. Floor
♦♦ Microcellular rubber (MCR) shoes to prevent injuries; consists of necrotic content, unhealthy granulation
care of foot. tissue and blood. Ulcer bleeds on touch and is vascular
Q.9. Define ulcer. Describe different types of ulcer. or friable. Induration is felt at base and edge. It is
 (June 2018, 3 marks) circular or irregular in shape.
Ans. An ulcer is the break in the continuity of the covering • Rodent ulcer: It is seen in basal cell carcinoma. Ulcer
epithelium either skin or mucus membrane due to shows central area of dry scab with peripheral, raised
molecular death. active and beaded edge. Often floor is pigmented. It
302   Mastering the BDS IIIrd Year  (Last 25 Years Solved Questions)

erodes in deeper planes such as soft tissue, cartilage Or


and bones Write in brief about clinical features and manage­ment
• Melanotic ulcer: It is the ulcerative form of melanoma. of tuberculous lymphadenitis. (Mar 2001, 15 Marks)
Ulcer is pigmented often with halo around. Ulcer is
rapidly growing often with satellite nodules. Or
♦♦ Non-specific ulcers: Describe the clinical features and treatment of tubercular
• Traumatic ulcers: It occurs after trauma. Ulcer is sinus in neck. (Mar 2008, 10 Marks)
superficial, painful and tender.
Or
• Arterial ulcer: This ulcer occurs after trauma and
soon become non – healing. Ulcer is usually deep, Write in brief management of tubercular cervical
destruct deep fascia, exposing tendons, muscles and lymphadenopathy. (Sep 2007, 5 Marks)
underlying bone. Ulcer is very painful, tender and Ans.
often hyperaesthetic
• Venous ulcer: It is common around ankle due to Causes of Enlargement of Lymph Node in Neck
ambulatory chronic venous hypertension. Ulcer ♦♦ Inflammatory: due to microorganism.
is initially painful but once it become chronic, it is • Bacterial as: Streptococcus, Mycobacterium, Treponema
painless. It is vertically oval in shape. Floor is covered pallidum, Actinomycosis.
with pale or often without any granulation tissue. • Viral as: Lymphogranuloma venereum, Infectious
Edge is sloping. Induration and tenderness is seen mononucleosis, HIV
often at base of an ulcer. • Parasite as: Wuchereria bancrofti
• Trophic ulcer: It is the ulcer due to prolonged pressure. • Fungus as: Blastomycosis.
Blood flow to skin stops once external pressure ♦♦ Lymphatic leukemia.
becomes more than 30mm of Hg and this leads to ♦♦ Autoimmune disorders:
tissue hypoxia, necrosis and ulceration. • Systemic lupus erythematous
• Infective ulcer • Rheumatoid arthritis
• Tropical ulcer: It is an acute ulcerative lesion of • Sclerosis.
skin seen in tropical countries. Pustule formation ♦♦ Neoplasms: Malignant neoplasm involves lymph node like.
occur which bursts in three days with necrobiosis • Malignant lymphoma
and phagedena causing spreading painful ulcer • Hodgkin’s disease
with an undermined edge, brownish floor and • Lymphosarcoma
serosanguineous discharge • Malignant melanoma.
• Ulcers due to chilblain and frostbite: This is due to ♦♦ Secondaries in lymph nodes.
exposure of a part to wet cold below freezing point.
Ulcers here are deep Clinical Features of Tubercular Lymphadenitis
• Martorell’s hypertensive ulcer: It is seen in hypertensive ♦♦ Swelling in the neck is present, which is firm and matted.
patients often with atherosclerosis. In this necrosis of ♦♦ Cold abscess is soft, smooth, non-tender, fluctuant, without
calf skin occur with sloughing away and formation involvement of the skin. It is not warm.
of deep, punched out ulcers extending to deep fascia. ♦♦ As a result of increased pressure, cold abscess ruptures
• Bazin’s ulcer out of the deep fascia to form collar stud abscess which is
• Diabetic ulcer: Diabetic ulcer is most common in foot. adherent to the overlying skin.
It can cause abscess, ulcer, osteomyelitis, gangrene, ♦♦ Once collar stud abscess bursts open, discharging sinus is
septicemia. Initially, patient undergo toe amputation formed. It can be multiple, wide open mouth, often under-
but later eventually land with below knee or above mined, non-mobile with bluish color around the edge. It
knee amputation. is usually not indurated.
• Ulcers due to leukemia, polycythemia, jaundice, ♦♦ Tonsils may be studded with tubercles and so clinically
collagen diseases, lymphoedema should always be examined.
• Cortisol ulcers: They are due to long time application ♦♦ Associated pulmonary tuberculosis should also be looked
of cortisol creams to certain skin diseases. for. In 20% cases of tuberculous lymphadenitis, there
may be associated pulmonary tuberculosis or it may be
a primary focus.
5. Lymphatics and Lymph ♦♦ Cervical spine is examined for tuberculosis.
♦♦ Axillary nodes, when involved are due to retrograde lym-
Node Enlargement phatic spread from neck nodes or blood spread.
♦♦ Inguinal lymph nodes are involved occasionally through
Q.1. Enumerate the causes of lymph node enlargement in blood.
neck, describe the clinical features and manage­ment ♦♦ Bluish hyperpigmented involved overlying skin is called
of tubercular lymphadenitis. (Mar 1997, 15 Marks) as scrofuloderma.
Section 2:  General Surgery  303

♦♦ Tuberculous pus with caseating cheesy creamy material is Pathology


infective as it contains multiplying organisms.
When the tubercle bacilli are introduced into the tissue.
♦♦ Atypical mycobacterial tuberculosis can occur occasionally.
♦♦ Initial response of neutrophils
Such disease may be resistant to drug therapy.
♦♦ There is progressive infiltration by macrophages
♦♦ Sinus may persist due to fibrosis, calcification, secondary
infection inadequate reach of drug to maintain optimum ♦♦ Macrophages start phagocytosing the tubercle bacilli.
concentration in caseation ♦♦ Activated CD4 + T-cells develop.
♦♦ In 2 to 3 days, the macrophages undergo structural changes
Management of Tubercular Lymphadenitis and form epithelioid cells.
♦♦ Drugs: ♦♦ The epithelioid cells aggregated into tight clusters or
Antitubercular drugs have to be started: granulomas.
• Rifampicin 450 mg OD on empty stomach. It is ♦♦ Some macrophages form multinucleated giant cells.
bactericidal. ♦♦ Hard tubercles form by the mass of epithelioid cells and
• INH: 300 mg OD. It is bactericidal. the giant cells in a zone of lymphocytes, plasma cells and
• Ethambutol 800 mg OD. It is bacteriostatic. fibroblasts.
• Pyrazinamide 1500 mg OD (or 750 mg BD). It is ♦♦ Center of the cellular mass undergoes caseous necrosis,
bactericidal. characterized by cheesy appearance called as soft tubercle.
Duration of treatment is usually 6–9 months. Q.3. Write short note on tuberculous lymphadenitis. 
♦♦ Aspiration: When there is cold abscess, initially it is  (Mar 2006, 5 Marks) (Sep 2005, 8 Marks)
aspirated. Wide bore needle is introduced into the cold Or
abscess in a nondependent site along a “Z” track (in zig- Discuss briefly tubercular lymphadenitis. 
zag pathway) so as to prevent sinus formation.  (Aug 2012, 5 Marks)
♦♦ Incision and drainage: If cold abscess recurs, then it should Or
be drained. Drainage is done through a nondependent Write brief notes on tubercular lymphadenitis. 
incision. After draining the caseating material, wound is  (Apr 2017, 2 Marks)
closed without placing a drain. Or
♦♦ Surgical removal: Surgical removal of tubercular lymph Write in brief about tubercular cervical lympha­
nodes are indicated when denitis. (Mar 2007, 5 Marks)
• There is no local response to drugs Ans. Tubercular lymphadenitis is defined as chronic specific
• When sinus persists. granulomatous inflammation with caseation necrosis of
It is done by raising skin flaps and removing all caseating lymph node.
material and lymph nodes. Care is taken not to injure major
structures. Clinical features
Excision of the sinus tract is often essential when sinus Refer to Ans 1 of same chapter.
develops.
Staging
Q.2. Write in brief about etiology, pathology, clinical
features and treatment of cervical tuberculous
Stage of Lymphadenitis
lymphadenitis.
 (Feb 2013, 10 Marks) (Sep 2000, 15 Marks) ♦♦ Common in young adult between 20–30 years.
Or ♦♦ Upper anterior deep cervical nodes are enlarged.
Discuss etiology, pathology, clinical features and treat­ ♦♦ Non-tender, discrete, mobile, firm lymph node is palpable.
ment of tuberculous lymphadenitis.
Stage of Matting
 (Mar 2003, 15 Marks)
Ans. For clinical features and treatment of tuberculous ♦♦ Results due to involvement of capsule of lymph node.
lymphadenitis, refer to Ans 1 of same chapter. ♦♦ Nodes moves together
♦♦ Firm, non-tender.
Etiology Stage of Cold Abscess
♦♦ Tuberculous bacillus, i.e. Mycobacterium tuberculosis is the ♦♦ It occurs due to caseation necrosis of lymph node.
bacteria which leads to tuberculosis. ♦♦ No local rise in temperature.
♦♦ Disease is caused in people of low socioeconomic status, ♦♦ No tenderness.
unhygienic living conditions and malnutrition. ♦♦ No redness.
♦♦ In 20% of cases, lymph nodes are affected in posterior ♦♦ Soft, cystic and fluctuant swelling.
triangle due to involvement of adenoid. ♦♦ Transillumination is negative.
304   Mastering the BDS IIIrd Year  (Last 25 Years Solved Questions)

♦♦ On sternomastoid contraction test, it becomes less promi- ♦♦ Branchial cyst and lymphatic cyst mimic cold abscess
nent indicating that it is deep-to-deep fascia. ♦♦ HIV with lymph node involvement
Stage of Collar Stud Abscess Investigations
It results when a cold abscess rupture through the deep fascia and ♦♦ Hematocrit, ESR, peripheral smear.
form another swelling in the subcutaneous plane which is fluctuant. ♦♦ FNAC of lymph node and smear for AFB and culture: FNAC is
very useful but not as superior as open node biopsy. False
Stage of Sinus negative, false positive results and altering the node archi-
♦♦ Collar stud abscess burst and form tubercular sinus. tecture, and so eventual need of open biopsy are the prob-
♦♦ Common in young female. lems. Epithelioid cells (modified histiocytes/macrophages)
♦♦ It can be multiple. are diagnostic. Langhans giant cells, lymphocytes, plasma
♦♦ Resemble an ulcer with undermined edge. cells are other features.
♦♦ No indurations. ♦♦ Open biopsy: Open biopsy is more reliable for tuberculosis
♦♦ Skin surrounding the sinus shows pigmentation and (and also in lymphoma; but it is contraindicated in node
sometimes bluish in color. secondaries); entire node ideally two nodes if possible has
to be taken intact; one in formalin for pathology, other in
normal saline for microbiology (AFB).
♦♦ HIV test (ELISA and western blot).
♦♦ Lowenstein-Jensen media is used for culture which takes 6
weeks to give result; so selenite media is often used which
shows growth in 5 days.
♦♦ Mantoux test may be useful; but not very reliable.
♦♦ Chest X-ray to look for pulmonary tuberculosis.
♦♦ Polymerase chain reaction (PCR) is very useful method.

Management
Refer to Ans 1 of same chapter.
Q.4. Write short note on: Collar stud abscess. 
 (Dec 2010, 5 Marks)
 (Sep 2007, 5 Marks) (Sep 2005, 5 Marks)
Ans. Collar stud abscess is an acute suppurative infection of
a digit which present stud-like blister.
• It is bilocular abscess with one lobule deep to the
deep fascia and another lobule is superficial to fascia.
Both the locule intercommunicates with each other
through a small perforation in the deep fascia.
Such an abscess can occur anywhere in the body.

Types
♦♦ Pyogenic: When pyogenic abscess develops deep to
deep fascia and gradually pressure mouth so that the
deep fascia perforates, the pus comes out into superficial
fascia.
 So, pyogenic collar stud abscess is more commonly seen
in hand where deep fascia is in the palmer fascia.
♦♦ Tuberculous: This is more often seen in the neck from casea-
tion tubercular lymph node. The cold abscess beneath the
deep fascia ruptures and forms another swelling in subcu-
Fig. 8:  Tubercular lymphadenitis taneous plane which is fluctuant. It remains adhere to skin.

Differential Diagnosis Treatment


Pyogenic Collar Stud Abscess
♦♦ Non-specific lymphadenitis
♦♦ Secondaries in neck ♦♦ Drainage of terminal pulp space by an oblique deep incision.
♦♦ Lymphomas and chronic lymphatic leukemia ♦♦ Systemic antibiotics should be given.
Section 2:  General Surgery  305

Tuberculous Collar Stud Abscess Q.6. Write in brief cold abscess in neck. (Apr 2008, 5 Marks)
Or
It should be treated like as cold abscess, i.e.
Write short note on cold abscess in neck.
♦♦ Anti-tubercular treatment is given.
(Dec 2009, 5 Marks) (Nov 2008, 5 Marks)
♦♦ Zig-zag aspiration by wide bore needle in non-dependent
Or
area for prevention of sinus perforation.
Describe diagnostic features and treatment of cold
♦♦ Drainage can be done by using non-dependent incision and
abscess. (Jan 2012, 5 Marks)
later on closure of wound is done without placing a drain.
Ans. Cold abscess is common in neck.
Q.5. Describe the causes of generalized lymphadeno­pathy
and management of Hodgkin’s lymphoma. Etiology
 (Oct 2007, 15 Marks) ♦♦ Tuberculous lymphadenitis: Cold abscess is seen commonly
Ans. Enlargement of lymph gland is called as lymphadeno­ in anterior triangle of neck
pathy. ♦♦ Tuberculosis of cervical spine: Cold abscess is seen commonly
Causes of Generalized Lymphadenopathy in posterior triangle of neck.

♦♦ Acute infection process: Glandular fever. Clinical Features


♦♦ Tuberculosis ♦♦ It is commonly seen in young individuals but can occur
♦♦ Syphilis: In secondary stage in any age group.
♦♦ Disseminated lupus erythematosus ♦♦ Swelling in neck is smooth, non-tender, soft, fluctuating,
♦♦ Blood disease: Acute lymphatic leukemia, chronic lymphatic non-transilluminant, non-mobile and is not adherent to skin.
leukemia ♦♦ There is presence of neck pain, neck rigidity and restricted
♦♦ Neoplasm: Secondary carcinoma movements of cervical spine.
♦♦ HIV infection and AIDS ♦♦ Rust sign: With change in position and often when patient
♦♦ Sarcoidosis. is seated, he support his head with hands and forearm.
♦♦ There is presence of evening fever, loss of weight and
Management of Hodgkin’s Lymphoma
appetite.
♦♦ Stage I and II: ♦♦ Matted lymph nodes adjacent to cold abscess should be
• Mainly radiotherapy, i.e. external high cobalt palpable.
radiotherapy
• Above the diaphragm: Y-field therapy, covering cervical, Diagnostic Features
axillary, mediastinal lymph nodes. ♦♦ Oral cavity, chest and tonsils of the patient are thoroughly
• Below the diaphragm: Mantle or inverted-Y field examined.
therapy covering para-aortic and iliac nodes. ♦♦ ESR is raised
• Chemotherapy is also given. ♦♦ Mantoux test is positive. This test is useful but not reliable.
♦♦ Stage III and IV: ♦♦ Presence of anemia and lymphocytosis
• In this, mainly chemotherapy is given. ♦♦ Chest X-ray show pulmonary tuberculosis
• Regimens used are: ♦♦ FNAC of cold abscess under microscopic examination
show epithelioid cells.
MOPP Regimen ♦♦ Fluid obtained by FNAC should be stained by Ziehl-
♦♦ Mechlorethamine: 6 mg/sqm on 1st and 8th day Neelsen stain which reveals acid-fast bacilli
♦♦ Oncovine: 1.4 mg/sqm on 1st and 10th day ♦♦ X-ray neck is done in cervical spine tuberculosis to identify
♦♦ Procarbazine: 100 mg orally daily for 10 days reduced joint space, vertebral destruction, soft tissue
♦♦ Prednisolone: 45 mg orally daily for 10 days shadow.
♦♦ MRI of cervical spine, ultrasonography or CT scan neck
MOPP combination is given in 6 courses with no drugs given
are needed to confirm anatomical location and number
from 15th to 28th day.
of lesions.
ABVD Regimen
Treatment
♦♦ Adriamycin: 30 mg/sqm
♦♦ Bleomycin: 10 mg/sqm ♦♦ Anti-tubercular treatment is given to the patient.
♦♦ Vinblastine: 6 mg/sqm ♦♦ Non-dependent aspiration of cold abscess.
♦♦ Dacarbazine: 350 mg/sqm ♦♦ Diseased neck nodes should be excised.
The cycle is repeated on 20th day. ♦♦ Immobilization of cervical spine by plaster jacket/collar for
Splenectomy is done in many patients except with stage 4 months. Cervical spine fusion by open surgical method,
IV disease. if diseased spine is unstable.
306   Mastering the BDS IIIrd Year  (Last 25 Years Solved Questions)

Q.7. Write short note on lymphoedema. • Complex decongestive therapy should be given.
 (May/June 2009, 5 Marks) It occurs in two phases, i.e. intensive therapy and
Ans. Lymphoedema is the accumulation of lymph in maintenance therapy.
extracellular and extravascular fluid compartment ♦♦ Surgeries:
commonly in subcutaneous tissue. • Excision of lesion can be done by Charle’s or Homan’s
It is due to defective lymphatic drainage. operation.
• Physiological: Omentoplasty can be done.
Classification • Combination of Excision and physiological: Both excision
Primary without any identifiable lymphatic disease. and creation of communication between superficial
Secondary is acquired. and deep lymphatics. Sistrunk and Thompson’s
operation can be done.
Pathophysiology • Bypass procedures can be done
Decreased lymphatic contractility, lymphatic valvular • Limb reduction surgeries can be done.
insufficiency, lymphatic obliteration by infection, tumour or Q.8. Write short note on clinical features of TB and its manage-
surgery causes all effects and pathology of lymphoedema. This ment, especially “DOTS regime”.
leads to lymphatic hypertension and dilatation causing lymph  (Jun 2010, 5 Marks)
stasis, accumulation of proteins, glycosamines, growth factors, Ans. TB is a granulomatous disease caused by Mycobacterium
and bacteria. There is more collagen formation, deposition of tuberculosis.
proteins, fibroblasts, ground substance causing fibrosis in sub-
cutaneous and outside deep fascia. Muscles are normal without Clinical Features
any edema but may get hypertrophied. ♦♦ Seen commonly in middle aged and older individuals.
♦♦ Patient complains of episodes of fever with chills with
Clinical Features evening rise of temperature. Patient becomes tired early.
♦♦ Presence of gradual loss of weight.
♦♦ Most commonly lower limb is involved.
♦♦ Patient has persistent cough with or without hemoptysis.
♦♦ Presence of swelling in the foot which extends progres-
♦♦ Swelling on neck is present which is firm and tender on
sively in the neck and show tree-trunk pattern leg.
palpation. When abscess is formed swelling perforates
♦♦ Presence of buffalo hump in dorsum of foot.
and pus drains out.
♦♦ Athlete’s foot with joint pain and disability
♦♦ Scrofula is the condition in which there is marked
♦♦ Fever, malaise and headache
enlargement of cervical lymph nodes with caseation
♦♦ Initially pitting edema occurs which later on become
necrosis and frequent breakdown of gland.
non-pitting.
♦♦ As skin is involved by tuberculosis, this is known as lupus
♦♦ Stemmer’s sign is positive, i.e. skin over dorsum of foot
vulgaris.
cannot be pinched because of subcutaneous fibrosis.
♦♦ Tubercular involvement of spine is called as Pott’s disease.
Grading of Lymphoedema ♦♦ When tuberculosis spreads through bloodstream it
involves organs such as liver, kidney the disease is known
This is given by Brunner as miliary tuberculosis.
♦♦ Latent: No clinically apparent lymphoedema
♦♦ Grade I: Pitting edema which more or less disappears on Management—DOTS Regimen
elevation of the limb. ♦♦ Directly observed treatment strategy adopted by WHO is
♦♦ Grade II: Non-pitting edema occur which does not reduce followed all over the World including India.
on elevation ♦♦ Directly observed treatment means that an observer
♦♦ Grade III: Edema with irreversible skin changes like fibrosis, watches the patient swallowing their tablets.
papillae, fissuring. ♦♦ DOT ensures accountability of TB services and helps to
prevent emergence of drug resistance.
Treatment
Components of DOTS
♦♦ Conservative:
• Elevation of limb, exercise and weight reduction. ♦♦ Political and administrative commitment at all levels.
• Static isometric activities should be stopped such as ♦♦ Diagnosis through sputum microscopy
prolong standing or carrying heavy weight. Rhythmic ♦♦ Uninterrupted supply of short course chemotherapy
movements should be encouraged such as swimming, drugs.
massaging ♦♦ Direct observation of drug intake (DOTS)
• Daily wearing of below knee stockings ♦♦ Systematic monitoring, evaluation and supervision at all
• Trauma and infection should be avoided. levels.
Section 2:  General Surgery  307

DOTS Regimen

Category Features of TB case Treatment regimen


Intensive phase Continuation phase
I New smear positive primary tuberculosis 2 H3R3Z3E3 4 (HR)
Seriously ill sputum smear negative cases.
Seriously ill extrapulmonary involvement
II Relapse/Failure of treatment cases 2 S3H3R3Z3E3 followed by 1 H3R3Z3E3 5 (HRE)
Others
III New sputum smear negative primary tuberculosis 2 H3R3Z3 4 H 3R 3
New less severe form of extrapulmonary tuberculosis
H: Isoniazid (300 mg), R: Rifampicin (450 mg), Z: Pyrazinamide (1500 mg), E: Ethambutol (1200 mg), S: Streptomycin (750 mg)

Note: Initial numeral before each regimen indicates the duration of therapy of that regimen. Numeral in subscript refer to thrice
weekly schedule for the drugs.

Q.9. Give causes of cervical lymphadenopathy and describe Tubercular Adenitis


tubercular adenitis. (Aug 2011, 10 Marks)
For tubercular adenitis in detail, refer to Ans 2 and Ans 3 of
Ans. same chapter.
Causes of Cervical Lymphadenopathy Q.10. Enumerate causes of cervical lymphadenopathy. De­
♦♦ Inflammation and infection: scribe etiopathogenesis, clinical features and manage­
• Bacterial infections: ment of tubercular lymphadenopathy of neck.
–– Specific bacterial infection  (Aug 2012, 10 Marks)
- Tuberculosis Ans. For causes of cervical lymphadenopathy, refer to Ans 9
- Syphilis. of same chapter.
–– Non-specific bacterial infection
- Pericoronitis Etiopathogenesis
- Periodontal disease Mycobacterium tuberculosis is the bacteria which lead to tuber-
- Peri-apical infections. culosis. When the tubercle bacilli are introduced into the tissue.
• Viral infections: ♦♦ Initial response of neutrophils
–– Infection mononucleosis ♦♦ There is progressive infiltration by macrophages
–– AIDS ♦♦ Macrophages start phagocytosing the tubercle bacilli
–– Herpes simplex ♦♦ Activated CD4 + T cells develop
–– Cat scratch disease. ♦♦ In 2 to 3 days, the macrophages undergo structural changes
• Fungal infections: and form epithelioid cells
–– Oral candidiasis ♦♦ The epithelioid cells aggregated into tight clusters or
–– Histoplasmosis. granulomas
• Parasitic infections: ♦♦ Some macrophages form multinucleated giant cells
–– Rickettsial Infection. ♦♦ Hard tubercles form by the mass of epithelioid cells and
♦♦ Allergic conditions: the giant cells in a zone of lymphocytes, plasma cells and
• Serum sickness. fibroblasts
♦♦ Primary neoplasms: ♦♦ Center of the cellular mass undergoes caseous necrosis,
• Lymphoma. characterized by cheesy appearance called as soft tubercle.
♦♦ Metastatic tumors:
For clinical features and management, refer to Ans 1 of same
• Oral squamous cell carcinoma
chapter.
• Metastasis of carcinoma of breast.
♦♦ Miscellaneous conditions: Q.11. Write short note on lymphangitis.(Aug 2011, 5 Marks)
• Leukemia Ans. It is an acute non-suppurative infection and spreading
• Collagen diseases inflammation of lymphatics of skin and subcutaneous
• Sarcoidosis tissues due to beta hemolytic streptococci, staphylococci,
• Non-tender lymphoid hyperplasia. clostridial organisms.
308   Mastering the BDS IIIrd Year  (Last 25 Years Solved Questions)

• It is commonly associated with cellulitis. Management


• Erysipelas is a type of lymphangitis.
Acute Cervical Lymphadenopathy
• In endemic areas, filariasis is the most common cause
(coastal India). It is caused by Wulchereria bancrofti. ♦♦ In a situation, where a self-limited infection is thought to be
the etiology of acute cervical lymphadenitis, it is reasonable
Clinical Features to observe and provide reassurance.
♦♦ Streaky redness which is spreading is typical. ♦♦ Unilateral lymphadenopathy that has a history of a
♦♦ On pressure, area blanches; on release redness reappears. rapid onset and overlying erythema of the skin and is
♦♦ Edema of the part, palpable tender regional lymph nodes accompanied by a fever suggests a bacterial infection. In
are obvious. this case, oral antibiotic therapy for 10–14 days and close
♦♦ Fever, tachycardia, features of toxemia. follow-up is the appropriate management.
♦♦ Groin lymph nodes are enlarged and tender in lower limb ♦♦ An ultrasound may be indicated to evaluate for the
lymphangitis. presence and extent of suppuration. If confirmed surgical
♦♦ In upper limb, as lymphatics are mainly located on the drainage is required with the presence of a fluctuant mass,
dorsum of hand, edema and redness develops on the suggesting an abscess.
dorsum. Infection in thumb and index finger causes ♦♦ On follow-up of adenopathy, further evaluation should be
considered, if there is not a decrease in size after 2 weeks.
palpable tender axillary nodes; in little and ring finger
♦♦ Laboratory evaluation may include the following: CBC
causes first tender palpable epitrochlear nodes to appear;
with differential, ESR, titers for Ebstein-Barr virus/
infection in middle finger causes first deltopectoral nodes
cytomegalovirus/toxoplasmosis/cat-scratch disease,
to enlarge.
antistreptolysin O or anti-DNAase serologic tests, PPD,
♦♦ Toxemia, septicamia may occur.
and chest radiograph.
♦♦ Rapidity may be more in diabetics and immunosuppressed.
♦♦ Non-infectious causes of cervical lymphadenopathy
♦♦ Chronic lymphangitis occurs due to repeated attacks
include Kawasaki Disease, PFAPA (periodic fever,
of acute recurrent lymphangitis leading into acquired aphthous stomatitis, pharyngitis, and cervical adenitis), and
lymphoedema. Sarcoidosis. Cervical lymphadenopathy is accompanied by
Investigations a number of other signs and symptoms in each of these
non-infectious disorders, and evaluation and management
♦♦ Blood cell count is individualized based on the disease.
♦♦ Platelet count
♦♦ Renal and liver function tests Chronic Cervical Lymphadenopathy
♦♦ Peripheral smear and blood culture ♦♦ Laboratory evaluation includes CBC, ESR, PPD, B
henselae serology, and titers for Ebstein-Barr virus/
Management
cytomegalovirus/toxoplasma among other tests based on
♦♦ Antibiotics such as penicillin, cloxacillin. history and physical examination.
♦♦ Elevation, rest, glycerine magnesium sulpha dressing. ♦♦ Children who have chronic cervical lymphadenopathy
♦♦ Management of toxemia or septicemia with critical care. often undergo extensive diagnostic evaluation before an
Q.12. Enumerate differences between sinus of tuberculosis etiology is determined.
and actinomycosis. (June 2014, 2 Marks) ♦♦ Special attention should be given to the possibility of TB
and HIV disease; the hematologic and serologic testing
Ans.
noted previously can be helpful.
Sinus of tuberculosis Sinus of actinomycosis ♦♦ Urine antigen tests for Histoplasma capsulatum occasionally
Sinus of tuberculosis occurs when Nodules form with involvement
can be helpful.
collar stud abscess ruptures of skin. Nodule softens and burst ♦♦ If there is an increase in lymph node size over 2 weeks,
through the skin through the skin as sinuses lack of decrease in size over 4–6 weeks, lack of regression
Occurs at the neck Occurs in face and neck to normal within 8–12 weeks, or the presence of persistent
fever, weight loss, fatigue, night sweats, hard nodes, or
Induration is not present Induration is present
fixation of nodes to surrounding tissues, a biopsy should
Group of lymph nodes is palpable No lymph nodes are palpable be performed for further evaluation.
underneath the sinus
♦♦ Biopsy can be in the form of open biopsy or fine needle
Discharge from sinus contains Discharge from sinus contains aspiration to further investigate for certain pathology,
M. tuberculosis sulphur granules which are Gram-
including infectious and non-infectious causes, specifically
positive mycelia surrounded by
Gram-negative clubs malignancy. Because lymphadenitis caused by non-
tuberculous Mycobacterium (NTM) evolves to draining
Q.13. Enumerate causes of cervical lymphadenopathy. How skin fistulas associated with scarring, the safety of needle
will you manage this? (Feb 2014, 8 Marks) aspiration when this infection is suspected has been
Ans. For causes refer to Ans 9 of same chapter. questioned.
Section 2:  General Surgery  309

♦♦ Needle aspiration may not lead to increased risk for this Investigations
complication because the treatment of a node found to ♦♦ Radiograph: It shows moth-eaten appearance with loss of
be infected with NTM is surgical excision-a cure for skin lamina dura.
fistulas. ♦♦ Biopsy: Starry Sky appearance is characteristic for Burkitt’s
♦♦ NTM and Bartonella infection are diagnosed best using lymphoma.
material obtained from a suppurative lymph node, which ♦♦ USG: This done for abdomen so that kidneys should be
can be stained and cultured for acid-fast organisms and evaluated.
sent for polymerase chain reaction (PCR) examination to ♦♦ Blood urea and serum creatinine estimation is done.
detect B henselae infection. Importantly, PCR analysis for
Bartonella can be performed on material that was obtained Treatment
recently and preserved by freezing. It is sensible to freeze ♦♦ Radiotherapy.
extra material obtained by needle aspiration so PCR studies ♦♦ Chemotherapy, i.e. cyclophosphamide, methotrexate,
can be performed, if bacterial studies are unexpectedly orthomelphalan.
negative. ♦♦ Surgery is usually not indicated unless it is localized or in
Q.14. Write briefly on Burkitt’s tumor. (Feb 2013, 5 Marks) case of involvement of ovaries.
Or Q.15. Write briefly on Hodgkin’s lymphoma.
Write short on Burkitt’s jaw tumor.  (Nov 2014, 3 Marks)
 (Sep 2009, 5 Marks) Ans. Hodgkin’s lymphoma is a lymphoproliferative disorder
Ans. It is also known as African jaw lymphoma. which arises from lymph nodes and from lymphoid
components of various organs.
Etiology
Epstein-Barr virus is considered to be the etiological factor. Clinical Features
♦♦ The tumor is seen in young adults, i.e. during 20 to 30 years
Types or in elderly people during 5th decade of life.
♦♦ Endemic (African): Occurs commonly in jaw. ♦♦ Onset of the tumor is insidious with enlargement of one
♦♦ Non-endemic (Sporadic): Occurs commonly in abdomen group of superficial nodes.
♦♦ Aggressive lymphoma: In HIV patients. ♦♦ Associated lymph nodes are painless. There is presence
of generalized weakness, cough, dyspnea and anorexia.
Clinical Features ♦♦ Lymph nodes are discrete and rubbery in consistency with
overlying skin is purely mobile.
♦♦ It is commonly seen in children during the age of 6–9 years. ♦♦ Pel-Ebstein fever is present, i.e. presence of cyclic fever with
♦♦ Males are commonly affected as compared to females. generalized severe pruritus which is of unknown etiology.
Male to female ration is 2:1. ♦♦ In oral cavity, the lesion appears as an ulcer or swelling.
♦♦ Lesion is found most commonly in maxilla and spreads It can also be present as intrabony lesion which appears
towards floor of the orbit. as hard swelling.
♦♦ In African form jaw is more involved while in American
form abdominal involvement is common. Ann Arbor Clinical Staging
♦♦ Lesion has very fast growth and it doubles in size within
a day. Stage I I Involvement of a single lymph node region
♦♦ Patient complaints swelling of jaw, abdomen and paraplegia. (A or B) IE Involvement of a single extra-lymphatic organ or site
♦♦ Peripheral lymphadenopathy is commonly seen. Stage II II Involvement of two or more lymph node
♦♦ Renal involvement is present which can be bilateral. (A or B) regions on the same side of the diaphragm
♦♦ In females, ovaries are also affected. IIE (or) with localized contiguous involvement
of an extranodal organ of site.
Oral Manifestations Stage III III Involvement of lymph node regions on both sides of
(A or B) the diaphragm
♦♦ Tumor is rapidly growing and involves maxillary, ethmoi- IIIE (or) with localized contiguous involvement of
dal and sphenoidal sinus along with orbit. IIIS an extanodal organ or site.
♦♦ Presence of loosening and mobility of permanent teeth. IIIES (or) with involvement of spleen
♦♦ Presence of paresthesia of inferior alveolar canals and other (or) both features of IIIE and IIIs
sensory facial nerves. Stage IV IV Multiple or disseminated involvement of one
♦♦ Gingiva adjacent to affected teeth become swollen, ulcer- or
ated and necrotic. (A or B) More extra-lymphatic organs of tissues with
♦♦ There can be presence of large amount of mass in the or without lymphatic involvement
mouth on the surface of which present the rootless devel- A = Asymptomatic; B = Presence of constitutional symptoms;
oping permanent teeth. E = Extranodal involvement; S = Splenomegaly
310   Mastering the BDS IIIrd Year  (Last 25 Years Solved Questions)

Investigations ♦♦ B-cell neoplasms:


• Precursor B cell neoplasm: ALL, LBL.
♦♦ Blood test:
• Precursor B cell neoplasm: It includes all B-cell-related
• Non-specific anemia of chronic disease is common
non-Hodgkin’s lymphomas.
• Lymphopenia is present.
♦♦ T-Cell and putative NK cell neoplasms
• ESR is elevated.
• Precursor B-cell neoplasm: ALL, LBL T cell related.
♦♦ Lymph node biopsy:
• Precursor T-cell and NK cell neoplasm: It includes all T
Show presence of Reed-Sternberg cells.
cell related non-hodgkin’s lymphomas.
♦♦ Imaging:
♦♦ Hodgkin’s lymphoma:
• Chest X-ray to look for mediastinal lymph nodes and
• Predominant Hodgkin’s lymphoma: Nodular lymphocyte
pleural effusion
type.
• CT is valuable in detecting intrathoracic and abdominal
• Classical Hodgkin’s lymphoma:
lymphadenopathy.
–– Nodular sclerosis
♦♦ USG: To look for liver, spleen and abdominal lymph
–– Lymphocyte rich
node.
–– Mixed cellularity
Management –– Lymphocyte depletion.
♦♦ Precusor lymphoid neoplasms
♦♦ Stage I and II:
♦♦ Immunodeficiency associated lymphoproliferative dis-
• Mainly radiotherapy, i.e. external high cobalt
orders.
radiotherapy
• Above the diaphragm: Y field therapy, covering cervical, ALL is Acute lymphoblastic leukemia.
axillary, mediastinal lymph nodes. LBL is Lymphoblastic lymphoma.
• Below the diaphragm: Mantle or inverted Y field therapy
Etiology
covering para-aortic and iliac nodes.
• Chemotherapy is also given. ♦♦ Genetic predisposition
♦♦ Stage III and IV: ♦♦ Sjogren’s syndrome
• In this, mainly chemotherapy is given. ♦♦ HIV infection
Regimens used are: ♦♦ Virus etiology, i.e. Epstein-Barr virus infection
♦♦ Occupational causes: Hair dye workers, herbicide ex-
MOPP Regimen posure
♦♦ Mechlorethamine: 6 mg/sqm on 1st and 8th day. ♦♦ Ionising radiation
♦♦ Oncovine: 1.4 mg/sqm on 1st and 10th day. ♦♦ Celiac sprue—Intestinal T-cell lymphoma
♦♦ Procarbazine: 100 mg orally daily for 10 days. ♦♦ Bloom’s syndrome.
♦♦ Prednisolone: 45 mg orally daily for 10 days.
Types of Lymphoma
♦♦ MOPP combination is given in 6 courses with no drugs
given from 15th to 28th day. ♦♦ Hodgkin’s lymphoma
♦♦ Non-Hodgkin’s lymphoma
ABVD Regimen
Q.17. Discuss differential diagnosis of cervical lymphad­
♦♦ Adriamycin: 30 mg/sqm enopathy. (June 2015, 20 Marks)
♦♦ Bleomycin: 10 mg/sqm Ans. Following is the differential diagnosis of cervicofacial
♦♦ Vinblastine: 6 mg/sqm lymphadenopathy:
♦♦ Dacarbazine: 350 mg/sqm
The cycle is repeated on 20th day. Infective Causes
Splenectomy is done in many patients except with stage IV ♦♦ Acute lymphadenitis
disease. ♦♦ Chronic non-specific lymphadenitis
♦♦ Tuberculous lymphadenitis
Q.16. Write short note on lymphoma. (Apr 2015, 3 Marks)
♦♦ Infectious mononucleosis
Ans. Lymphomas are the progressive neoplastic condition of
♦♦ Toxoplasmosis
lymphoreticular system which arises from stem cells.
♦♦ Cat scratch fever.
Lymphoma is the third most common malignancy
among the children. Neoplastic Causes
♦♦ Metastatic lymph node enlargement
Classification of Lymphoma
♦♦ Lymphomas, i.e Hodgkin’s lymphoma, Non-Hodgkin’s
WHO modified REAL (Revised European American Lymphoma) lymphoma, Burkitt’s lymphoma
classification of lymphoma. ♦♦ Chronic lymphatic leukemia.
Section 2:  General Surgery  311

Other Causes Toxoplasmosis


♦♦ Autoimmune diseases, i.e. systemic lupus erythematosus, ♦♦ Caused by Toxoplasma gondii, protozoa through meat.
rheumatoid arthritis. ♦♦ Cervical lymphadenopathy is most common.
♦♦ HIV infection, Immunosuppression. ♦♦ Diagnosed by Sabin Feldman dye test.
Acute Suppurative Lymphadenitis
Cat Scratch Disease
♦♦ It is a bacterial infection leading to acute inflammation and
suppuration of lymph nodes caused by group A strepto- ♦♦ Caused by Bartonella henselae.
cocci or staphylococci. ♦♦ Lymphadenopathy is present.
♦♦ There is presence of tender, enlarged, firm or soft palpable ♦♦ Diagnosed by lymph node biopsy with Warthin-Starry
neck lymph nodes. staining.

Chronic Non-specific Lymphadenitis Lymphomas


♦♦ Infections such as chronic tonsillitis, recurrent dental ♦♦ In Hodgkin’s lymphoma, there is painless progressive
infection can lead to chronic non-specific lymphadenitis. enlargement of lymph nodes. Lymph nodes are smooth,
♦♦ There is presence of firm, non-tender, multiple, bilateral firm, non-tender. In this cervical lymph nodes, i.e. lower
lymph node enlargement in neck. deep cervical group in posterior triangle are commonly
♦♦ Tuberculous lymphadenitis enlarged. In this type, peripheral lymph node involvement
♦♦ It is caused by mycobacterium tuberculosis infection. is not common.
♦♦ Lymph nodes are matted, enlarged with cold abscess or ♦♦ In non-Hodgkin’s lymphoma any group of lymph node
sinus formation. is involved. In this type, peripheral lymph node involve-
ment is common.
Infectious Mononucleosis
Q.18. Write briefly on chemotherapy of tuberculosis.
♦♦ Caused by Epstein-barr virus.  (Jan 2016, 2 Marks)
♦♦ There is presence of generalized lymphadenopathy. Ans. Category-wise Alternative Treatment Regimens for
♦♦ Monospot test is positive. Tuberculosis (WHO-1997)

TB Patient type Intial phase (Daily 3 × Continuation phase Duration


category per-week)
I New sputum positive or new smear negative or new case 2 HRZE (S) 4HR/4H3R3 or 6HE 6 Months
with severe from of extrapulmonary tuberculosis 8 Months
II Smear positive failure or smear positive relapse or sputum 2 HRZES 5 HRE or 5H3R3E3 8 Months
positive treatment after default 8 Months
III Smear negative pulmonary TB with limited parenchymal 2 HRZ 4HR/4H3R3 or 6HE 6 Months
involvement or less severe from of extrapulmonary TB 8 Months
IV Chronic cases or suspected MDR-TB cases For H resistance for RZE ZE + S / Ethionamide + 12 Month
H + R resistance Ciprofloxacin / ofloxacin

Explanation of Standard Code Ans.


♦♦ Each antitubercular drug has standard abbreviation, i.e.
• Isoniazid (H) Etiological Agent
• Rifampicin (R)
• Pyrazinamide (Z) Since matted lymph nodes in neck are seen in tuberculosis, the
• Ethambutol (E) etiological agent is Mycobacterium tuberculosis.
• Streptomycin (S).
♦♦ Numerical before a phase is the duration of that phase Pathognomic Diagnostic Feature
in months.
Matting of lymph nodes in neck is itself a pathognomic
♦♦ Numerical in subscript is the number of doses of that drug
diagnostic feature of tuberculosis. Matting is due to involvement
per week. If there is no subscript numerical, then the drug of capsule. Nodes are firm and non-tender.
is given daily.
Q.19. Name etiological agent, pathognomic diagnostic fea­ Treatment (Only Modality)
ture and treatment (only modalities) of matted lymph
nodes in neck. (Jan 2017, 3 Marks) Anti-tubercular treatment should be given to the patient.
312   Mastering the BDS IIIrd Year  (Last 25 Years Solved Questions)

6. Skin Tumors ♦♦ Surgery: Wide excision (l cm clearance) with skin grafting,


primary suturing or flap (Z plasty, rhomboid flap, rotation
flap) is the procedure of choice.
Q.1. Write short note on basal cell carcinoma.
♦♦ Laser surgery, photodymamic therapy, 5-fluorouracil lo-
 (May/June 2009, 5 Marks) (Sep 2005, 5 Marks)
cal application.
 (Mar 2000, 10 Marks)
♦♦ Cryosurgery.
Ans. It is the most common malignant skin tumor. ♦♦ Microscopically Oriented Histographic Surgery (MOHS):
• It is a slow growing neoplasm. It is useful to get a clearance margin and in conditions
• It arises from basal cell of the pilosebaceous adnexa such as basal cell carcinoma close to eyes, nose or ear, to
and occurs only on the skin. preserve more tissues. MOHS is becoming popular in basal
Common Sites cell carcinoma. Procedure is done by dermatological sur-
geon along with a histotechnician/histologist. Under local
♦♦ Majority of lesions are found on the face. anesthesia, a saucerized excision of the primary tumour
♦♦ Inner canthus and outer canthus of eyes. is done and quadrants of the specimen are mapped with
♦♦ Eyelids. different colors. Specimen is sectioned by histotechnician
♦♦ Bridge of the nose. from margin and depth and it is stained using eosin and
♦♦ Around nasolabial fold. hematoxylin. lt is studied by MOHS surgeon or histologist.
♦♦ These sites are the area where the tears roll down; hence, Residual tumor from relevant mapped area is excised and
it is also called ‘Tear cancer’. procedure is repeated until clear margin and clear depth
♦♦ Nodular: Common on face. are achieved. Clearance must be complete and proper in
♦♦ Cystic/Nodulocystic basal cell carcinoma.
♦♦ Ulcerative Q.2. Write short note on epithelioma. (Mar 1997, 5 Marks)
♦♦ Multiple: Associated with syndromes and other malignan- Ans. It is also called “squamous cell carcinoma”.
cies.
• It is the second common malignant tumor of skin
♦♦ Pigmented basal cell carcinoma: It mimics melanoma
after basal cell carcinoma.
♦♦ Geographical or forest fire basal cell carcinoma: Involve
• It arises from prickle cell layer of the skin.
wide area with central scabbing and peripheral active
• It usually affects elderly males.
proliferating edge.
♦♦ Basio-squamous: Behave-like squamous cell carcinoma and Clinical Features
spread in lymph nodes
♦♦ It is an ulcerative or cauliflower-like lesion.
Etiology ♦♦ Edges are *everted and *indurated.
♦♦ Base is indurated and it may be subcutaneous tissue,
♦♦ Ultraviolet rays are the main factor for its develop­ment.
muscle or bone.
♦♦ Fair skin is vulnerable for development of it.
♦♦ Floor contains cancerous tissue, which look like granula-
♦♦ Prolonged administration of arsenic in the form of arseni-
tion tissue.
cal ointments.
♦♦ It is pale, *friable, bleed easily on touch.
Clinical Features ♦♦ Surrounding area is also indurated
♦♦ Mobility is usually restricted.
♦♦ It is common in males in middle aged and elderly
♦♦ The most common clinical presentation is an ulcer that Investigations
never heals.
♦♦ It is common on the region of face, i.e. above the line drawn ♦♦ Wedge biopsy from edge.
between angle of mouth and ear lobe. ♦♦ FNAC from lymph node
♦♦ It can also present as a painless, firm, nodule. ♦♦ USG/CT scan to identify the nodal disease
♦♦ It is pigmented with fine blood vessels on its surface. ♦♦ MRI to identify local extension.
♦♦ The ulcer has raised and *beaded edge, induration may be Treatment
present, and bleed on touch.
♦♦ Radiotherapy using radiation needles, moulds, etc. is
Spread
given.
♦♦ It spread by local invasion. ♦♦ Wide excision, 2 cm clearance followed by skin grafting
♦♦ It slowly penetrates deep inside, destroying the underlying or flaps. Wide excision should show clearance both at
tissue-like bone, cartilage or even eyeball, hence, the name margin as well as in the depth. If muscle, fascia, cartilage
“rodent ulcer”. are involved, it should be cleared. Reconstruction is usually
Treatment done by primary split skin grafting. Delayed skin grafting
♦♦ It is radiosensitive. lf lesion is away from vital structure can also be done once wound granulates well. Often flaps
then curative radiotherapy can be given. Radiotherapy is of different type are needed depending on the site of lesion.
not given, once it erodes cartilage or bone. ♦♦ Amputation with one joint above.
Section 2:  General Surgery  313

♦♦ For lymph nodes, block dissection of the regional lymph ♦♦ Lisch nodules are present which are translucent brown
nodes is done. pigmentation on iris.
♦♦ Curative radiotherapy is also useful in tumors which are ♦♦ Crowe’s sign is present, i.e. axillary freckling, brown spot
not adherent to deeper planes or cartilage as squamous cell on skin.
carcinoma is radiosensitive. It is also useful in recurrent
squamous cell carcinoma and in patients who are not fit Complications
for surgery. A dose of 6000 cGy units over 6 weeks; 200 ♦♦ Cystic degeneration is present.
units/day is used. Recurrence after radiotherapy is treated ♦♦ Spinal and cranial neurofibromas can cause neurological
by surgical wide excision. deficits.
♦♦ In advanced cases with fixed lymph nodes, palliative ♦♦ Spine dumb-bell tumor lead to compression of spinal cord
external radiotherapy is given to palliate pain, fungation and paralysis of limb.
and bleeding. ♦♦ Hemorrhage in tissues.
♦♦ Chemotherapy is given using methotrexate, vincristine,
bleomycin. Treatment
♦♦ Field therapy using cryoprobe or topical fluorouracil or Excision is done.
electrodessication.
Q.4. Describe features of mole turning into melanocarci­
Q.3. Write short note on neurofibroma.
noma. (Jan 2012, 5 Marks)
 (May/June 2009, 5 Marks) (Aug 2011, 5 Marks)
Ans. Neurofibroma is a benign tumor arising from connective Ans. Following are the Features
tissue of nerve containing ectodermal neural and • Lesion show superficial radial growth pattern.
mesodermal connective tissue components. • Lesion becomes ulcerated and growing day by day
in size.
Types of Neurofibroma • Fungating growth is present associated with the
♦♦ Nodular: bleeding.
• Single, smooth, firm, tender swelling which moves • It leads to destruction of the underlying bone
horizontally or perpendicular to direction of nerve. • Lesion is firm on palpation
♦♦ Plexiform: • Borders of lesion are erythematous.
• Occurs along distribution of trigeminal nerve in skin
Q.5. Write difference between squamous cell carcinoma and
of face.
• Attain enormous size with thickening of skin which basal cell carcinoma. (Mar 2016, 3 Marks)
hang downwards. Ans. Following are the differences between squamous cell
carcinoma and basal cell carcinoma:
♦♦ Von-Recklinghausen’s Disease:
• Inherited disease with multiple neurofibromas in Squamous cell carcinoma Basal cell carcinoma
body. It is also known as epithelioma It is also known as rodent ulcer
• It can be cranial, spinal or peripheral It spreads commonly It spreads rarely
• Associated with pigmented spots on skin, i.e. Café-
It can occur anywhere on skin or It is found on upper part of face
au-lait spots.
mucous membrane
♦♦ Elephantiatic:
• Origin is congenital and involve limbs. It arises from squamous cell layer It arises from basal cell layer of
of epidermis epidermis
• Skin of limb is thickened, dry and coarse.
♦♦ Cutaneous: Edge of the lesion is raised and Edge of the lesion is raised and
• Small, multiple, firm/hard nodules arising from everted rolled up
terminal ends of dermal nerves Base is indurated Base is non-indurated
• It can be pedunculated or sessile. Microscopically, it shows cell M i c r o s c o p i c a l l y, c e l l s a r e
nests or epithelial pearls arranged in palisaded pattern
Clinical Features
♦♦ Mild pain or painless swellings seen in subcutaneous
and cutaneous plane with tingling, numbness and
paresthesia. 7. Bleeding Disorders
♦♦ Most commonly affected sites are trunk, face and
extremities Q.1. Write short note on hemophilia. (Feb 2013, 5 Marks)
♦♦ Sessile or pedunculated elevated small nodules of various  (Feb 2015, 5 Marks) (Aug 2012, 5 Marks)
sizes  (Jan 2012, 5 Marks) (Dec 2015, 5 Marks)
♦♦ Majority of patients have asymmetric areas of pigmentation
Or
known as Café-au-lait spots. They are smooth edge dark
brown macules. Write briefly on hemophilia. (June 2015, 2.5 Marks)
314   Mastering the BDS IIIrd Year  (Last 25 Years Solved Questions)

Or Types of Hemorrhage
Answer briefly on hemophilia. (Mar 2016, 3 Marks) ♦♦ Depending upon the nature of the vessel involve:
Ans. It is a hereditary disorder of blood coagulation, • Arterial hemorrhage:
characterized by excessive hemorrhage due to prolonged –– Bright red in color, jet out.
bleeding time. –– Pulsation of the artery can be seen.
It is the X – linked genetic disorder of coagulation. –– It can be easily controlled as, it is visible.
• Venous hemorrhage:
Types –– Dark red in color.
–– It never jet out but oozes out.
Following are the types of hemophilia, i.e.
–– Difficult in control because vein gets retracted,
♦♦ Hemophilia A or Classic Hemophilia: Deficiency of factor
non pulsatile.
VIII or antihemophilic factor is the cause of hemophilia
• Capillary hemorrhage:
A. It is recessive X-linked. Females are always carrier and
–– Red color, never jet out, slowly oozes out.
males are sufferer.
–– It becomes significant, if there are bleeding ten-
♦♦ Hemophilia B or Christmas Disease: Deficiency of factor IX
dencies.
is the cause of hemophilia B.
♦♦ Depending upon the timing of hemorrhage:
Clinical Features • Primary hemorrhage: Occurs at the time of surgery.
• Reactionary hemorrhage: Occurs after 6 to 12 hours of
♦♦ The most common manifestation is hemorrhage into joint.
surgery. Hypertension in postoperative period, violent
♦♦ Because of repeated hemorrhage knee joint becomes non-
sneezing, coughing or retching are the usual causes.
functional hemarthroses takes place.
• Secondary hemorrhage: Occurs after 5 to 7 days of
♦♦ Bleeding in muscles take place.
surgery. It is due to infection which eats away the
♦♦ Bleeding from GIT occurs in form of esophageal varices.
suture material, causing sloughing of vessel wall.
♦♦ Bleeding from genitourinary tract occurs in form of he-
♦♦ Depending upon the duration of hemorrhage:
maturia.
• Acute hemorrhage: Occurs suddenly, e.g. esophageal
♦♦ A simple cut or injury may lead to profuse bleeding.
varices bleeding.
♦♦ First symptom is seen in form of large bruises and hema-
• Chronic hemorrhage: Occurs over a period of time, e.g.
tomas on hips which regresses as the child learn to walk.
hemorrhoids/piles.
Laboratory Diagnosis • Acute or chromic hemorrhage: It is more dangerous
as bleeding occur in individuals who are already
Bleeding time is normal but coagulation time is prolonged. hypoxic, which may get worsened faster.
♦♦ Depending upon the nature of bleeding:
Treatment
• External hemorrhage or reversal hemorrhage, e.g.
Hemophilia A epistaxis, haematemesis.
The main aim is to raise the factor VIII level, which can arrest • Internal hemorrhage or concealed hemorrhage, e.g.
bleeding. splenic rupture following injury.
♦♦ Based on the possible intervention
♦♦ Replacement therapy: Varies from of replacement therapy
• Surgical hemorrhage: Can be corrected by surgical
are available like plasma, cryoprecipitate and factor VIII
intervention
concentrates.
• Non-surgical hemorrhage: It is diffuse and ooze due
♦♦ Give 30% amount of factor VIII, because it is very expen-
to coagulation abnormalities and disseminated
sive.
intravascular coagulation.
♦♦ Any major operation or tooth extraction requires 100%
concentration of factor VIII. Management of Hemorrhage
♦♦ Hypovolemia, allergic reaction and development of factor
VIII antibodies are complication of factor VIII concentrate. ♦♦ General management:
To avoid this complication, cryoprecipitate from animal • Hospitalization
origin should be given in between human cryoprecipitate. • Care of critically ill patients start with A, B, C (Airways,
breathing, circulation).
Hemophilia B • Oxygen should be administered.
• Intravenous line: Urgent intravenous adminis­tration of
Fresh frozen plasma or Factor IX concentrates should be given
isotonic saline to restore the blood volume to normal.
to the patient.
• Colloids such as gelatins or hetastarch have also been
Q.2. Describe different types of hemorrhage and manage­ used.
ment of hemorrhage.  ♦♦ Specific measures:
 (Mar 2003, 15 Marks) (Mar 2001, 15 Marks) • Conservative
Ans. Hemorrhage is the escape of blood from a blood vessel. – By local pressure and packing:
Section 2:  General Surgery  315

- Pressure applied with tight dressing, applying Q.4. Write short note on reactionary hemorrhage. 
digital pressure, or a cloth pegs for epistaxis.  (Mar 1996, 5 Marks)
Use of double balloon in the esophagus and Ans. Reactionary hemorrhage may follow primary hemorrhage
stomach to control the bleeding from esopha- within 24 hours (usually 4–6 hours) and is mainly due
geal varices. to rolling (Slipping) of a ligature, dislodgment of a clot
- Packing by means of rolls of wide gauge is an or cessation of reflex vasospasm.
important stand by in operative surgery.
  The precipitating circumstances are the rise of
– Elevation of part: As in bleeding varicose ulcer.
blood pressure and the shifting of the venous system
- Rest: Absolute rest.
on recovery from shock, and restlessness, coughing
- Sedation: With diazepam.
and vomiting which raise the venous pressure, e.g.
- Treatment of shock: Treatment of hypo­volemia
reactionary venous hemorrhage within a few hours of
by restoration of blood volume by blood trans-
thyroidectomy.
fusion and or saline, dextrose, etc.
• Operative: Q.5. Give a brief account of bleeding disorders, their etiol­
–– Clamping the bleeding vessels with catgut, thread ogy, and management in dental practice. 
or silk.  (Sep 1999, 15 Marks)
–– Coagulation by thermocautery or diathermy. Ans. Bleeding disorders: Bleeding disorder or hemorrhagic
–– By local application of adrenaline swabs. diatheses are a group of disorders characterized by
–– Application of silver clips as in neurosurgery. defective hemostasis with abnormal bleeding.
–– Under running or transfusion of vessels by needle
and suture. Etiology of Bleeding Disorders
–– Application of crushed piece of muscle at the site
♦♦ Vascular Defects:
of bleeding.
Bleeding disorders caused by vascular defects may be
–– Bone wax or bismuth iodoform paraffin paste
caused by structural malformation of vessels. Hereditary
(BIPP) is used for oozing from bone.
disorders of connective tissue and acquired connective
–– Repair of vascular defect by patches of vein or
Dacron mash. tissue disorders. Vascular defects rarely cause serious
–– Excision of bleeding organ: Splenectomy. bleeding. Bleeding into skin or mucous membrane starts
• Restoration of blood volume after hemorrhage immediately alter trauma but ceases within 24 to 48
• Resuscitation from hemorrhage includes restoration of hours. The vascular defects are hereditary hemorrhagic
the circulating volume. So Ringer lactate is preferred telangectasia, Henoch-Schönlein purpura
over the normal saline. Isotonic crystalloid or colloid
♦♦ Platelet Disorder:
solutions can be used for volume replacement in
It can be of two types:
hemorrhage.
• Reduction in number: Thrombocytopenic purpura.
• Blood transfusion: For restoring the circulating volume
If the total number of circulating platelets falls
and replace coagulation factors and oxygen carrying
below 50,000 per mm3 of blood the patient can have
capacity many blood products are available. By
bleeding. In some cases the total platelet count is
crystalloid solution hypovolemia is corrected. Packed
reduced by unknown mechanism, this is called
RBCs restore intravascular volume and oxygen
primary or idiopathic thrombocytopenic purpura
carrying capacity. Platelet transfusions can be done
(ITP). Chemicals, radiation and various systemic
in significant thrombocytopenia and continued
disease, (e.g. leukemia) may have direct effect on
hemorrhage
the bone marrow and may result in secondary
Q.3. Enumerate difference between capillary, venous and thrombocytopenia.
arterial bleeding. (June 2014, 2 Marks) • Defect in quality:Non-thrombocytopenic purpura e.g.
Ans. von Willebrand’s disease, Bernard-Soulier disease,
Features Capillary Venous Arterial
Glanzmanns thrombasthenia.
bleeding bleeding bleeding Von-Willebrand’s disease (pseudohemophilia) is the
Color Intermediate red Dark red Bright red most common inherited bleeding disorder. Unlike
hemophilia it can occur in females. This is a disease of
Pulsating Capillary Negative Positive
character bleeding may be both coagulation factors and platelets. It is caused by
quite aggressive an inherited defect involving platelet adhesion. Platelet
Vigour of Oozing type Less rapid Increased adhesion is affected because of a deficiency of Von
flow flow Willebrand`s factor.
Spurt Ooze from raw It does not Blood will spurt with Various drugs such as carbamazepine, aspirin, methyl
surface spurt each heart beat from
cut end of artery
dopa, phenytoin can also lead to platelet disorders.
316   Mastering the BDS IIIrd Year  (Last 25 Years Solved Questions)

♦♦ Coagulation Defects ♦♦ When given prophylactically platelets should be given


Hemophilia A: It is the most common coagulation defect. half before surgery to control capillary bleeding and half
It is inherited as X-linked recessive trait. The hemostatic at the end of the operation to facilitate the placement of
abnormality in hemophilla A is caused by a deficiency/ adequate sutures.
defect of factor VIII. Until recently, factor VIII was thought ♦♦ Platelets should be used within 6–24 hours after collection
to be produced by endothelial cells and not by the liver as and suitable preparations include platelet-rich plasma
most of coagulation factors. The defective gene is located (PRP), which contains about 90% of the platelets from a
on the X chromosome. unit of fresh blood and platelet-rich concentrate (PRC),
Hemophilia B (Christmas disease): Factor IX is deficient or which contains about 50% of the platelets from a unit of
defective. It is inherited as X- linked recessive trait. Like fresh whole blood.
Hemophilia A, the disease primarily affects males and the ♦♦ PRC is thus the best source of platelets. Platelet infusions
clinical manifestations of the two are identical. carry the risk of isoimmunization, infection with blood-
borne viruses and, rarely, graft-versus-host disease.
Disseminated intravascular coagulation (DIC): It is a condition
♦♦ Where there is immune destruction of platelets (e.g. in
that results when the clotting system is activated in all or a
ITP), platelet infusions are less effective.
major part of vascular system. Despite wide spread fibrin ♦♦ The need for platelet transfusions can be reduced by
production, the major clinical problem is bleeding not local hemostatic measures and the use of desmopressin
thrombosis. DIC is associated with a number of disorders or tranexamic acid or topical administration of platelet
such as infection, obstetric complications, cancer and snake concentrates.
bite. ♦♦ Absorbable hemostatic agents such as oxidized regenerated
Management of Bleeding Disorders cellulose (Surgicel), synthetic collagen (lnstat) or
microcrystalline collagen (Avitene) may be put in the
I. Hereditary Hemorrhagic Telangiectasia socket to assist clotting in postextraction socket.
♦♦ Drugs that affect platelet function, such as gentamicin,
It is transmitted as autosomal dominant trait and is characterized antihistamines and aspirin should be avoided.
by bleeding from mucous membrane.
Major surgery
Management
For major surgery, platelet levels over 75,000 cells/mm3 are
♦♦ In patient having repeated attacks of epistaxis septal desirable.
dermoplasty should be done. In septal dermoplasty
involved mucosa get removed and skin grafting is done. III. Hemophilia
♦♦ If spontaneous hemorrhages are present or nasal bleeding
It is a hereditary disorder of blood coagulation characterized
is present, it is controlled by giving pressure packs.
by excessive hemorrhage due to increased coagulation time. It
♦♦ Sclerosing agents i.e. sodium tetradecyl sulphate, if injected
is of two types, i.e. Hemophilia A and hemophilia B.
intralesionally stop bleeding.
♦♦ Electrocautery is done. It helps in arresting bleeding. Management
II. Idiopathic Thrombocytopenic Purpua (ITP) ♦♦ Local anesthesia is contraindicated. So intrapulpal
Steroid Treatment Protocol anesthesia, intraligamentary anesthesia should be used.
Sedation with diazepam or NO2–O2 sedation can be given.
Initial steroid treatment protocol for ITP: Initial steroid treatment
♦♦ Endodontic procedures should be carried out and care is
protocol l mg/kg/day prednisolone, PO for 2–6 weeks.
taken not to do instrumentation beyond apex. If hemo-
subsequent steroid treatment protocol for ITP: Prednisolone
rrhage is present it should be controlled by 1:1000 aqueous
dose is individualized for every patient. Usually the dose of
epinephrine on paper point.
prednisolone is tapered to less than l0 mg per day for 3 months
and then withdrawn. Splenectomy is done, if discontinuation of ♦♦ Restorative treatments can be carried out by proper appli-
prednisolone causes a relapse. Follow the ’rule of twos’ for major cation of rubber dam for avoiding trauma to gingiva and
dental treatment and provide extra steroids prior to surgery, other soft tissues. In case, if rubber dam is not present an
if the patient is currently on steroids or has used steroids for epinephrine impregnated hemostatic cord is kept in gin-
2 weeks longer within the past 2 years. gival sulcus before preparation of crown or inlay margin.
♦♦ Complete dentures and removable partial dentures can
Minor surgery be given to hemophilic patients and are well tolerated by
♦♦ Hemostasis after minor surgery is usually adequate, if them. Patient has to take care for proper maintenance of
platelet levels are above 50,000 cells/mm3. hygiene of prosthesis.
♦♦ Platelets can be replaced or supplemented by platelet ♦♦ Conservative periodontal treatment should be done rather
transfusions; though sequestration of platelets occurs than attempting for periodontal surgeries.
rapidly. Platelet transfusion is indicated for established ♦♦ In case, if oral surgical procedures are to be done local
thrombocytopenic bleeding. hemostatic agents should be used, pressure surgical packs
Section 2:  General Surgery  317

should be employed, sutures, topical thrombin is used. extirpation. Sclerotherapy/Cryotherapy/CO2 snow therapy
After removal of tooth socket is packed with mechanical cause unpleasant scarring.
splint. Postoperative use of anti-fibrinolytic agent is used ♦♦ Preoperative embolization facilitates surgical excision and
to support clot maintenance. reduces the operative blood loss. When once embolization
♦♦ In cases of Hemophilia A, Human freeze-dried factor VIII done surgery should be done as early as possible otherwise
concentrate or new recombinant factor VIII is used. recurrence occurs and much more worried formation of
♦♦ In hemophilia B, human dried factor IX concentrate is sup- enlarged collaterals can occur.
plied as powder which is to be mixed with distill water ♦♦ Rapidly growing hemangioma may need systemic/oral
and administer IV and intra-lesional steroid therapy.
♦♦ Anti-angiogenic interferon 2a may be useful.
IV. Von Willebrand Disease ♦♦ Life-threatening platelet trapping may be controlled by
cyclophosphamide chemotherapy.
♦♦ It is the most common inherited bleeding disorder. It is ♦♦ Hemangioma with drug resistant CCF can be treated with
inherited as autosomal dominant but a severe form of radiotherapy.
disease may be inherited as a sex-linked recessive trait.
♦♦ It is caused due to the deficiency or defect in Von Wille- Treatment of Cavernous Hemangioma
brand Factor. ♦♦ Sclerosant therapy: It is the initial first line therapy. It causes
♦♦ Types of Von-Willebrand diseases are: Type I, Type II A aseptic thrombosis and fibrosis of the cavernous heman-
and II B, Type III. gioma with less vascularity and smaller size. It is directly
injected into the lesion. Sodium tetradecyl Sulphate hyper-
Management tonic saline are used. Often multiple injections are needed
♦♦ Surgical procedures can be performed in patients with to achieve complete required effect. Later excision of the
mild Von Willebrand disease by using DDAVP and EACA. lesion is done.
Patients with severe Von Willebrand disease requires ♦♦ Ligation of feeding artery and often at later stage excision
cryoprecipitate and Factor VIII concentrate. is done once hemangioma shrinks.
♦♦ Bleeding should be controlled by using local measures ♦♦ Therapeutic embolization.
such as pressure packs, gelfoam with thrombin, tranexamic ♦♦ lf small and located in accessible area, excision is the initial
acid etc. therapy.
♦♦ Aspirin and NSAIDs are avoided and acetaminophen can ♦♦ LASER ablation: Diode-pulsed LASER is becoming popu-
be given to patients. lar because of good control of bleeding.
♦♦ In majority of patients with Von Willebrand disease, Q.7. Describe hemorrhage—its types, causes, clinical
hemostatic defect is controlled with desmopressin via features and management. (Jan 2012, 10 Marks)
nasal spray. Or
♦♦ Type I Von Willebrand disease is treated with desmopres-
Write a long answer on cause of hemorrhage and its
sin while Type II A and B, and Type III require clotting
management. (June 2018, 5 Marks)
factor replacement.
Ans. Hemorrhage is the escape of blood from the blood vessel.
V. Disseminated Intravascular Coagulation For types and management refer to Ans 2 of same chapter.
♦♦ Correction of hemodynamic instability by fluid therapy,
Causes
transfusion of packed cells or whole blood.
♦♦ Factor replacement: This is the specific therapy, in this ♦♦ Hemorrhage occurs due to road accidents and injuries
fresh frozen plasma, cryoprecipitate, platelet concentrate ♦♦ Gun shot wounds
transfusions are essential. Fresh-frozen plasma is given at ♦♦ During surgeries such as thyroid surgery, circumcision
the dose of 15 mL/Kg. Platelet is transfused at the dosage and hydrocele surgery.
of 0.1 unit/Kg. ♦♦ Due to erosion of carotid artery by cancer.
Q.6. Write short note on treatment of hemangioma. ♦♦ During inguinal block dissection.
 (Sep 2009, 3 Marks) ♦♦ In bleeding disorders such as hemophilia.
Ans. Hemangioma is a benign tumor containing hyperplastic Clinical Features
endothelium with cellular proliferation with increased
♦♦ Patient becomes pallor and thirsty. At times, cyanosis is
mast cells.
also present.
♦♦ There is presence of tachycardia and tachypenia
Treatment of Capillary Hemangioma
♦♦ Patient feels air hunger.
♦♦ They are treated by wait and watch policy commonly ♦♦ Skin is cold and clammy due to vasoconstriction.
allowed for spontaneous regression. ♦♦ Patient has dry face, dry mouth and goose skin appearance.
♦♦ Diode laser, surgical excision and reconstruction may need ♦♦ There is presence of rapid thready pulse and hypotension
to be ligated after wide exposure before achieving complete ♦♦ Oliguria is present.
318   Mastering the BDS IIIrd Year  (Last 25 Years Solved Questions)

Signs of Hemorrhage Types


♦♦ Pulse > 100/min Refer to ans 2 of same chapter.
♦♦ Systolic blood pressure < 100 mm Hg
♦♦ Diastolic blood pressure drop on sitting or standing > 10 Causes
mm Hg ♦♦ Trauma
♦♦ Pallor/sweating ♦♦ Infections
♦♦ Shock index, i.e. ratio of pulse rate to blood pressure >1 ♦♦ Local irritants
Q.8. Discuss briefly hemangioma. (Nov 2008, 5 Marks) ♦♦ Congenital malformations
Ans. Hemangioma is a benign tumor containing hyperplastic ♦♦ Surgical (intraoperative/postoperative)
endothelium with cellular proliferation with increased ♦♦ Hemorrhage due to abnormalities in clotting factors:
mast cells. • Clotting factor deficiencies
– Hereditary: Hemophilia A, Haemophilia B,
Classification Thrombocytopenia.
♦♦ Capillary hemangioma: – Anticoagulant, antiplatelet or fibrinolytic therapy:
• Salmon patch Warfarin, coumarin, heparin, enoxaparin, aspirin,
• Strawberry hemangioma clopidogrel etc.
• Port-wine stain. – Liver disease.
♦♦ Cavernous hemangioma. • Dysfunction of clotting—multiple myeloma:
– Haemorrhage due to abnormalities in platelets
Clinical Features - Deficiencies:
♦♦ It is a most common tumor in children. * Idiopathic thrombocytopaenia purpura
♦♦ It has biphasic growth showing initial rapid growth with * Secondary thrombocytopaenia purpura
gradual involution over 5 to 7 years. * Leukemia.
♦♦ It is more common in girls. - Thrombocytosis
♦♦ It is commonly seen in skin and subcutaneous tissue but - Dysfunction: Thrombocytopenia.
can occur anywhere in the body like liver, brain, lungs – Hemorrhage due to systemic disease
- Viral infection
and other organs.
- Scurvy
♦♦ It grows rapidly in first year and 70% involutes in 7 years.
- Allergy.
♦♦ Early proliferative lesion is bright red, irregular; deep
lesion is bluish colored. Involution causes color fading, Investigations
softness, shrinkage leaving crepe paper like area.
Patients with the above mentioned causes may be advised
♦♦ Commonly it is central; common in head and neck region.
♦♦ Often large hemangiomas may be associated with visceral to be subjected to an investigation before any oral surgical
anomalies. Head and neck hemangioma is associated procedure.
with ocular and intracranial anomalies; sacral with spinal Investigations are:
dysraphism. ♦♦ Clotting time
♦♦ Multiple cutaneous hemangiomas may be associated with ♦♦ Bleeding time
hemangioma of liver causing hepatomegaly, cardiac failure ♦♦ Prothrombin time and International normalized ratio
(CCF), anemia. ♦♦ Activated partial thromboplastin time
♦♦ Ulceration, bleeding, airway block and visual disturbances ♦♦ Factors assay.
are common complications.
Investigations for Measuring the Blood Loss
♦♦ A definitive even though rare, but important life-threatening
complication is platelet trapping and severe throm- ♦♦ Hb% and PCV estimation
bocytopenia presenting as ecchymosis, petechiae, intracranial ♦♦ Blood volume estimation using radioiodine technique or
hemorrhage and massive gastrointestinal bleed. microhematocrit method
♦♦ Measurement of CVP or PCWP
Treatment ♦♦ Investigations specific for cause, i.e. ultrasonography of
abdomen, Doppler and often angiogram in vascular injury,
Refer to Ans 6 of same chapter. Chest X-ray in hemothorax, CT scan in major injuries, CT
Q.9. Define hemorrhage, its types, causes, investigations scan head in head injuries.
and measures to control hemorrhage. Measures to Control Hemorrhage
 (Jun 2010, 15 Marks)
Mechanical Methods
Ans. Hemorrhage is defined as the escape of blood from
cardiovascular system to the surface of body or into the ♦♦ Pressure: Firm pressure is applied over bleeding site for
body tissues or cavities. 5 minutes.
Section 2:  General Surgery  319

♦♦ Hemostat: It is applied at bleeding points and leads to direct Ans.


occlusion of bleeding vessel
♦♦ Sutures and ligation: For severed blood vessels ligature is Features Cirsoid aneurysm Strawberry hemangioma
done. For large pulsating artery suturing is done. Definition It is a rare variant of It may start at birth in
capillary hemangioma between one to three weeks
Chemical Methods occurring in skin beneath it appears as red mark
which abnormal artery which increases in size
♦♦ Adrenaline: It leads to vasoconstriction of bleeding capil- communicates with the within 3 months and form
laries distended veins strawberry hemangioma
♦♦ Thrombin: Converts fibrinogen into fibrous clot Type Variant of capillary True capillary hemangioma
♦♦ Surgicel: Acts by forming acid products from partial dis- hemangioma
solution which coagulate plasma protein Swelling Pulsatile swelling Cystic swelling
♦♦ Surgicel fibrillar: It is modified surgical and is used in ir-
Bony Involves bone Does not involve bone
regular surfaces and inaccessible areas involvement
♦♦ Oxycel: Platelet plug in its meshwork and forms the clot
♦♦ Gelatine sponge Or Gelfoam/surgifoam Treatment Ligation of feeding artery Wait and watch policy as
and excision of lesion the lesion disappears till
♦♦ Microfibrillar collagen (Avitene) age of 7–8 years
♦♦ Fibrous glue
♦♦ Styptics and astringents: They precipitate protein and Q.12. Enumerate differences between primary, reactionary
arrest bleeding and secondary hemorrhage. (Jun 2014, 2 Marks)
♦♦ Alginic acid Ans.
♦♦ Natural collagen sponge: It activates coagulation factors
XI and XIII and helps in clotting Primary Reactionary Secondary
♦♦ Fibrin sponge: It stimulates coagulation forming normal hemorrhage hemorrhage hemorrhage
clot and also act as temporary plug over small injured Occurs during Occurs after 6 to 12 Occur after 5 to 7 days
blood vessels surgery hours of surgery of surgery
♦♦ Bone wax: It is used in cases of bleeding from bone or Occurs due to Hypertension in post- Occurs due to infection
chipped edges of bone. It is softened with fingers and is accidental cut operative period, violent which eats away the
applied to bleeding site of vessel during sneezing, coughing or suture material causing
♦♦ Ostene: a new water—soluble bone hemostatic agent. surgery retching are the causes sloughing of vessel wall

Thermal Agents Q.13. Write short note on capillary hemangioma.


♦♦ Eletrocautery/surgical diathermy: Small capillaries and bleed-  (June 2015, 5 Marks)
ing vessels are coagulated by diathermy. Ans. Capillary hemangioma is of three types i.e.
♦♦ Monopolar diathermy • Salmon patch
♦♦ Bipolar diathermy • Strawberry hemangioma
♦♦ Cryosurgery • Port-wine stain.
♦♦ Lasers: Coagulate small blood vessels.
Salmon Patch
Q.10. Write short note on hemophilia—its types and
management. (Jun 2010, 5 Marks) ♦♦ It is present at the time of birth.
Ans. Types of Hemophilia ♦♦ It commonly occurs at the nape of neck, face, scalp and limbs.
♦♦ Lesion involves the wide area of skin.
Types of Another name Deficiency of clotting ♦♦ It is caused due to the area of persistent fetal dermal
hemophilia factor circulation.
Hemophilia A Classic hemophilia Factor VIII or plasma ♦♦ It regresses with the age and disappear completely.
thromboplastinogen
Hemophilia B Christmas disease Factor IX or plasma Strawberry Hemangioma
thromboplastin component
♦♦ Child is normal at birth, but between 1 to 3 weeks lesion
Hemophilia C -------- Factor XI or plasma
thromboplastin antecedent
appear as red mark which increases in size to 3 months.
♦♦ This is a true capillary hemangioma.
♦♦ Lesion is 20 times more common than port-wine stain.
Management
♦♦ Lesion is seen more commonly in white girls
For management, refer to Ans 5 of same chapter. ♦♦ Its male to female ratio is 1:3.
Q.11. Define and describe differentiating features of cirsoid ♦♦ Lesion occurs most commonly in head and neck region.
aneurysm and strawberry hemangioma. ♦♦ On palpation lesion is compressible and is warm with
 (Jan 2012, 5 Marks) bluish surface.
320   Mastering the BDS IIIrd Year  (Last 25 Years Solved Questions)

♦♦ Lesion involves skin, muscles and subcutaneous tissues. Clotting Occurs in Three Stages
♦♦ Lesion begins to disappear one year of age and it com- 1. Formation of prothrombin activator.
pletely regresses in 7 to 8 years. 2. Conversion of prothrombin into thrombin.
Port-wine Stain 3. Conversion of fibrinogen to fibrin.
♦♦ Formation of prothrombin activator: The prothrombin
♦♦ It occurs at birth and persists throughout the life. activator is formed into two ways:
♦♦ It presents as smooth, flat, reddish blue or purplish. ♦♦ Extrinsic pathway
♦♦ Lesion is common in head, neck and face. –– Factor III initiates this pathway after injury to
♦♦ Eventually surface of lesion become nodular and keratotic. damage tissues. After injury, these tissues release
♦♦ It results due to defect in maturation of sympathetic in- thromboplastin which contains protein, phospho-
nervations of skin causing localized vasodilatation of lipid and glycoprotein which act as proteolytic
intradermal capillaries. enzymes.
♦♦ It requires cosmetic coverage. Excision, grafting or LASER –– The glycoprotein and phospholipid component
ablation. of thromboplastin convert factor X into activated
factor X, in presence of factor VIII.
Treatment of Capillary Hemangioma –– Activated factor X reacts with factor V and phos-
pholipid content of tissue thromboplastin to form
♦♦ They are treated by wait and watch policy commonly prothrombin activator in presence of calcium.
allowed for spontaneous regression. –– Factor V is activated by thrombin formed from
♦♦ Diode laser, surgical excision and reconstruction may prothrombin. This factor V now accelerates forma-
need to be ligated after wide exposure before achieving tion of prothrombin activator.
complete extirpation. Sclerotherapy/Cryotherapy/CO2
snow therapy cause unpleasant scarring.
♦♦ Preoperative embolization facilitates surgical excision and
reduces the operative blood loss. When once embolisation
done surgery should be done as early as possible otherwise
recurrence occurs and much more worried formation of
enlarged collaterals can occur.
♦♦ Rapidly growing hemangioma may need systemic/oral
and intralesional steroid therapy.
♦♦ Antiangiogenic interferon 2a may be useful.
♦♦ Life-threatening platelet trapping may be controlled by
cyclophosphamide chemotherapy.
♦♦ Hemangioma with drug resistant CCF can be treated with
radiotherapy.
Q.14. Write short note on coagulation. (June 2015, 5 Marks)
Ans. It is the spontaneous arrest of the bleeding.
Following are the factors which are involved in the
mechanism of coagulation of blood:
• Factor I—Fibrinogen
• Factor II—Prothrombin
• Factor III—Thromboplastin (Tissue factor)
• Factor IV—Calcium ions
• Factor V—Labile factor
• Factor VI—Presence not approved
• Factor VII—Stable factor
• Factor VIII—Anti-hemophilic factor
• Factor IX—Christmas factor
• Factor X—Stuart-Prower factor
• Factor XI—Plasma thromboplastin antecedent
• Factor XII—Hageman factor
• Factor XIII—Fibrin-stabilizing factor
• Factor XIV—Prekallikrein
• Factor XV—Kallikrein
• Factor XVI—Platelet factor. Fig. 9:  Coagulation pathway
Section 2:  General Surgery  321

• Intrinsic pathway: It occurs in the following sequence: • Depending upon the duration of hemorrhage:
–– During injury, the blood vessel is ruptured, – Acute hemorrhage: Occurs suddenly, e.g.
endothelium is damaged and collagen beneath esophageal varices bleeding.
endothelium is exposed. – Chronic hemorrhage: Occurs over a period of
–– When factor XII comes in contact with collagen, time, e.g. hemorrhoids/ piles.
it is converted to active factor XII. • Depending upon the nature of bleeding:
–– The active factor XII converts inactive factor XI to – External hemorrhage or reversal hemorrhage,
active factor XI in presence of kininogen. e.g. epistaxis, haematemesis.
–– The activated factor XI activates factor IX in pres- – Internal hemorrhage or concealed hemorrhage,
ence of calcium ions. e.g. splenic rupture following injury.
–– Activated factor IX activates factor X in presence • Based on the possible intervention
of factor VIII and calcium. – Surgical hemorrhage: Can be corrected by surgical
–– When platelet comes in contact with collagen of intervention
damaged blood vessel, it releases phospholipids. – Non-surgical hemorrhage: It is diffuse and ooze due
–– Now, active factor X reacts with platelet phospho- to coagulation abnormalities and disseminated
lipid and factor V to form prothrombin activation intravascular coagulation.
in presence of calcium ions.
–– Factor V is activated by positive feedback method. Complications of Blood Transfusion
♦♦ Conversion of prothrombin into thrombin: Prothrombin
activator converts prothrombin into thrombin in presence Following are the complications of blood transfusion:
of calcium by positive feedback mechanism. This accelerates ♦♦ Febrile reactions: It is the most common complication due
formation of extrinsic and intrinsic prothrombin activator. to impurities like pyrogens in the blood or in infusion set.
♦♦ Conversion of fibrinogen to fibrin: During this, the Headaches, fever, chills and rigors, tachycardia, nausea
soluble fibrinogen is converted to fibrin by thrombin. The are the features. Transfusion is temporarily stopped or the
fibrinogen is converted to activated fibrinogen due to loss flow is slowed down with administration of antipyretic
of two pairs of polypeptides. The first form fibrin contains drug to reduce fever. Often transfusion of that unit needs
loosely arranged strands which are modified later into to be discontinued.
tight aggregate by factor XIII in presence of calcium ions. ♦♦ Allergic reaction: Urticaria and allergy to specific proteins
in the donor’s plasma can occur. Usually, it is mild and is
Q.15. Classify hemorrhages. Management of hypovolemic
treated with steroid and antihistaminics. In severe urticaria
shock. Complications of blood transfusion.
that unit of blood is discarded; new washed RBC’s and
 (Dec 2015, 10 Marks)
platelets are used.
Ans. Classification of Hemorrhage
♦♦ Acute hemolytic reactions: It is the most dangerous com-
• Depending upon the nature of the vessel involve: plication. It is due to ABO incompatibility. Usually it is
– Arterial hemorrhage: nonfatal but occasionally can be fatal. It is commonly due
- Bright red in color, jet out. to technical error at different levels. It amounts for criminal
- Pulsation of the artery can be seen. negligence in court of law.
- It can be easily controlled as, it is visible. ♦♦ Transfusion-related graft versus host disease: This very
– Venous hemorrhage: serious, very rare complication occurs due to recognition
- Dark red in color. and reaction against host tissues by infused donor lym-
- It never jet out but oozes out. phocytes. lt is common in immunosuppressed, lymphoma,
- D ifficult in control because vein gets leukemic patients. Any type of blood products including
retracted, non pulsatile. leukocyte reduced blood can cause the condition. Features
– Capillary hemorrhage: are pancytopaenia, toxic epidermal necrosis, liver dysfunc-
- Red color, never jet out, slowly oozes out. tion with more than 90% mortality. It is difficult to treat.
- It becomes significant if there are bleeding ♦♦ Congestive cardiac failure: It occurs if especially large
tendencies. quantities of whole blood are transfused in chronic severe
• Depending upon the timing of hemorrhage: anaemia, pregnancy, elderly patients, in patients who have
– Primary hemorrhage: Occurs at the time of cardiac problems.
surgery.
For management of hypovolemic shock refer to ans 2 of
– Reactionary hemorrhage: Occurs after 6 to 12
hours of surgery. Hypertension in postoperative chapter SHOCK.
period, violent sneezing, coughing or retching
Q.16. Write briefly on Rhesus (Rh) blood group.
are the usual causes.
 (Jan 2016, 2 Marks)
– Secondary hemorrhage: Occurs after 5 to 7 days of
surgery. It is due to infection which eats away the Ans. Rhesus blood group was discovered by Landsteiner and
suture material, causing sloughing of vessel wall. Weiner in 1940.
322   Mastering the BDS IIIrd Year  (Last 25 Years Solved Questions)

♦♦ Rh antigen system has three closely linked gene loci, cod- Ans.
ing for D antigen (there is no d antigen), C and/or c antigen Primary hemorrhage Reactionary hemorrhage
and E and/or e antigen. Thus, the antigens produced are It occur during the surgery It occurs after 6 to 12 hours of
C. D, E. c and e. surgery
♦♦ An individual may have similar or different sets of these It is due to the accidental cut Causes of reactionary hemor-
three Rh antigens on each chromosome; for example, CDE/ of vessel during surgery. This rhage are: Slipping away of
cde, cde/cde, or CdE/cdE (each person inherits one trio bleeding is more common in ligatures
gene from each parent). surgery on malignancy Dislodgement of clots
♦♦ Individuals who are positive for D antigen are considered Cessation of reflex vasospasm
Rh—positive (85% of the population) and those who lack normalization of blood pressure
it are Rh—negative. This should be resolved during As the causing factor is corrected
♦♦ Individuals with a weak variant of D antigen, called the the operation, with any major the hemorrhage stops
Du variant, are also considered Rh—positive. haemorrhages recorded in
the operative notes and the
♦♦ Alloimmunization, i.e. formation of an antibody against patient monitored closely post-
an antigen occurs if a person is exposed to an Rh antigen operatively.
that is not on the patients RBCs. Primary hemorrhage can be Reactionary hemorrhage can be
♦♦ The majority of clinically important antibodies that arterial, venous or capillary arterial or venous. Bleeding starts
produce a transfusion reaction are warm-reacting (IgG) when there is rise in arterial or
antibodies (eg, anti-D, anti-Kell) rather than cold-reacting venous pressure
(IgM) antibodies.
Rh incompatibility or erythroblastosis fetalis or
hemolytic disease of newborn: When a mother is Rh 8. Shock
negative and fetus is Rh positive the first child does not
undergo Rh incompatibility. This is because Rh antigen Q.1. Discuss shock. (Apr 2010, 15 Marks)
cannot pass from fetal blood into mother’s blood due to Or
placental barrier but during delivery of child Rh antigen Discuss briefly shock. (Aug 2012, 5 Marks)
from fetal blood may leak into mother’s blood due to Or
placental detachment and mother develop Rh antibody What is shock? Describe its classification, causes, clini­
in her blood. When mother conceives for second time and cal features and management. (Jan 2012, 10 Marks)
if fetus is Rh positive the Rh antibody crosses mother’s Or
placental barrier and enters fetal blood and cause fetal Write on classification, causes, clinical features and
agglutination of fetal RBCs which leads to hemolysis. management of shock. (Nov 2014, 8 Marks)
Due to excessive hemolysis child suffer from severe Ans. Shock is defined as an acute clinical syndrome
anemia, hydrops fetalis, kernicterus. characterized by a significant, systemic reduction
in tissue perfusion, resulting in decreased tissue
Treatment oxygen delivery and insufficient removal of cellular
metabolic products, resulting in tissue injury and severe
Exchange blood transfusion should be done soon after birth, dysfunction of vital organs.
i.e. small quantities of infant’s blood successively from intra
Classification of Shock
venous catheter and replacing an equal volume of compatible
Rh negative blood. So the infants Rh positive RBCs prone to Following is the classification of shock:
destruction are removed from circulation. ♦♦ Hypovolemic shock
♦♦ Cardiogenic shock
Prevention ♦♦ Distributive shock:
• Septic shock
Destruction of Rh positive fetal cells in maternal blood is • Anaphylactic shock
brought about by administering single dosage of anti-Rh • Neurogenic shock
antibody in form of Rh immunoglobulin soon after birth of child. ♦♦ Obstructive shock.
This prevent active antibody formation by mother.
Hypovolemic Shock
Q.17. Describe differentiating features of primary and reac­
♦♦ Hypovolemic shock occurs due to loss of blood plasma
tionary hemorrhage.
or body fluid and electrolytes, usually caused by massive
 (Jan 2017, 4 Marks) hemorrhage, vomiting, diarrhea, and dehydration.
Section 2:  General Surgery  323

♦♦ Hypovolemic shock is the most common type of shock ♦♦ Cardiac causes:


which is characterized by a loss in circulatory volume, • Acute myocardial infarction, acute carditis
which leads to decreased venous return, decreased fill- • Acute pulmonary embolism wherein embolus blocks
ing of the cardiac chambers, and so there is decrease in the pulmonary artery at bifurcation or one of the
cardiac output which leads to increase in the systemic major branches
vascular resistance. • Drug induced
• Toxemia of any causes
Cardiogenic Shock • Cardiac surgical conditions like valvular diseases,
♦♦ Cardiogenic shock occurs due to the dysfunction of one congenital heart diseases
ventricle or other. • Cardiac compression causes:
♦♦ This type of shock is seen in myocardial infarction, chronic –– Cardiac tamponade due to collection of blood,
congestive cardiac failure, cardiac arrhythmias, pulmonary pus, fluid in the pericardial space which prevents
embolism, etc. resulting in inability of the heart to pump the heart to expand leading to shock.
the adequate amount of blood into the lungs and decreased –– Trauma to heart.
cardiac output. ♦♦ Septic shock—is due to bacterial infections which release
♦♦ Myocardial infarction is the most common cause of car- toxins leading to shock.
diogenic shock. ♦♦ Neurogenic shock—due to sudden anxious or painful
stimuli causing severe splanchnic vessel vasodilatation.
Distributive Shock Here patient either goes for cardiac arrest and dies or recov-
Distributive shock occurs when the afterload is excessively ers fully spontaneously—spinal cord injury/ anaesthesia
reduced due to extensive vasodilatation and is associated not can cause neurogenic shock.
only with poor vascular tone in the peripheral circulation but ♦♦ Anaphylactic shock—is due to type I hypersensitivity
maldistribution of blood flow to organs within the body also. reaction
♦♦ Septic shock: This type of shock is mostly due to release of ♦♦ Respiratory causes:
endotoxins in blood, which causes wide spread vasodila- • Atelectasis (collapse) of lung
tion of blood vessels resulting in fall in the cardiac output. • Thoracic injuries
Bacteria responsible for release of endotoxins are E. coli, • Tension pneumothorax
Pseudomonas proteus, etc. It is most common shock among • Anesthetic complications.
all the distributive shocks. ♦♦ Other causes:
♦♦ Neurogenic shock: This type of distributive shock is • Acute adrenal insufficiency (Addison’s disease)
caused by the suppression or loss of sympathetic tone • Myxedema.
caused any disruption of the sympathetic nervous system
like spinal injury, spinal anesthesia, and drugs. Pathophysiology of Shock
♦♦ Anaphylactic shock: This type of shock is a result of type
I hypersensitivity reaction and is caused when the body’s Any cause of shock
antibody—antigen response is triggered by something the
person is allergic to drugs, like penicillin, cephalosporins, Low cardiac output
iodinated contrast media, serum, etc., are common causes
of this type of shock. Vasoconstriction occurs as a
compensation to perfuse vital organs
Obstructive Shock
like brain, heart, kidney, liver
♦♦ This type of shock is associated with physical obstruction
of the great vessels or the heart itself. Because of vasoconstriction and tachycardia
♦♦ Most commonly obstructive shock is due to cardiac
temponade, due to tension pneumothorax and pulmonary Dynamic circulation increases
embolus.
Causes of Shock Tachypnea occurs to increase in
♦♦ Hypovolemic shock—due to reduction in total blood volume. the oxygen saturation
It may be due to:
• Hemorrhage: Peripheral veins (capacitance vessels)
–– External from wounds, open fractures constrict diverting blood from splanchnic
–– Internal from injury to spleen, liver, mesentery system towards essential vital organs
or pelvis.
• Severe burns, which results in loss of plasma Decreased renal blood flow
• Peritonitis, intestinal obstruction reduces the GFR and urine output
• Vomiting and diarrhea of any cause
324   Mastering the BDS IIIrd Year  (Last 25 Years Solved Questions)

Renin angiotensin mechanism gets Stage 2: Stage of decompensatory shock—where there is


activated causing further vasoconstriction progressive shock causing persistent shock with severe hypotension
and aldosterone release (with mean arterial pressure <65 mm Hg); oliguria, tachycardia.
Stage 3: Stage of irreversible shock - with severe hypoxia and
Causes salt and water retention Multi organ dysfunction syndrome (MODS).
Clinical Features of Shock
ADH is released
♦♦ Presence of anxiety, restlessness, altered mental state due
Further concentration of urine occurs to decreased cerebral perfusion and subsequent hypoxia.
♦♦ Hypotension because of decrease in circulatory volume.
When shock persists cardiac output falls ♦♦ Due to decreased blood flow there is tachycardia and weak
further thready pulse.
♦♦ Cold and clammy skin due to vasoconstriction and
Hypotension and tachycardia occurs stimulation of vasoconstriction.
leading to poor perfusion of coronaries ♦♦ Rapid and shallow respirations (tachypnea) due to
sympathetic nervous system stimulation and acidosis.
Hypoxia—metabolic acidosis ♦♦ Hypothermia due to decreased perfusion and evaporation
of sweat.
Release of cardiac depressants ♦♦ Thirst and dry mouth due to fluid depletion.
♦♦ Fatigue due to inadequate oxygenation.
Cardiac (pump) failure ♦♦ Cold and mottled skin, especially extremities, due to insuf-
ficient perfusion of the skin.
Hypoxia
♦♦ Pallor is present
♦♦ Fainting
Anaerobic metabolism ♦♦ Oliguria/anuria due to decreased renal perfusion and af-
ferent arteriolar vasoconstriction.
Lactic acidosis
Investigations and Monitoring of Shock
Cell wall damage ♦♦ Regular monitoring with blood pressure, pulse, heart
rate, respiratory rate, urine output measurement (hourly)
should be done. Urine output should be more than 0.5 ml/
Sodium and calcium enter the cell kg/hour. Pulse oximetry should be used.
♦♦ Central venous pressure (CVP), pulmonary capillary
Potassium leaks out of the cell
wedge pressure (PCWP—an accurate assessment of left
ventricular/function) monitoring should be done. ICU care
is needed during monitor period. But both CVP and PCWP
Leads to hyperkalaemia, hyponatraemia
are not accurate method of assessing tissue perfusion.
and Hypocalcaemia
♦♦ Complete blood count, ESR, pH assessment, serum electro-
Intracellular lysosomes break down lyte estimation, chest X-ray (to rule out acute respiratory
releasing powerful enzymes which destroy distress syndrome/pulmonary problems).
own cell ♦♦ Pus/urine/blood/bile/sputum cultures depending on the
focus and need in sepsis.
Sick cell syndrome ♦♦ Serum lactate estimation is an important prognostic factor.
Level >2 mEq/L suggest tissue ischemia.
Platelets are activated forming small clots ♦♦ USG of a part, CT/MRI of the location of pathology of
in many places standard focus should be done; often may require repeti-
tion of these imaging to assess progress.
Disseminated intravascular coagulation ♦♦ Blood urea, serum creatinine, liver function tests, pro-
thrombin time (PT), activated partial thromboplastin time
Further bleeding (APTT), ECG monitoring are also should be done.
♦♦ All these tests including platelet count and arterial blood
gas (ABG) should be repeated at regular intervals.
Stages of Shock
Treatment of Shock
Stage l: Stage of compensatory shock—by neuroendocrine ♦♦ Treat the cause, e.g. arrest hemorrhage, drain pus.
response to maintain the perfusion of the vital organs like brain, ♦♦ Fluid replacement: Plasma, normal saline, dextrose, Ringer’s
heart, kidney, liver. lactate, plasma expander (haemaccel). Dosage is maximum
Section 2:  General Surgery  325

l liter can be given in 24 hours. Initially crystalloids then nificant change in heart rate, cardiac output and splanchnic
colloids are given. Blood transfusion is done whenever blood compensates for the same.
required. ♦♦ Overt compensated hypovolemia (Moderate 15–40%):
♦♦ Ionotropic agents: Dopamine, dobutamine, adrenaline Here patient has cold periphery, tachycardia, a wide arte-
infusions—mainly in distributive shock like septic shock. rial pressure, tachypnea, confusion, hyponatremia, meta-
♦♦ Correction of acid-base balance: Acidosis is corrected by using bolic acidosis, but systolic pressure is well—maintained
8.4% sodium bicarbonate intravenously. but postural hypotension.
♦♦ Steroid is often lifesaving. 500–l000 mg of hydrocortisone ♦♦ Decompensated hypovolemia (Severe >40%): Here all
can be given. It improves the perfusion, reduces the capil- features of hypovolemia are present like hypotension,
lary leakage and systemic inflammatory effects. tachycardia, sweating, tachypnea, oliguria, drowsiness,
♦♦ Antibiotics in patients with sepsis; proper control of blood eventually features of systemic inflammatory response
sugar and ketosis in diabetic patients. syndrome is seen and often if not treated on time leads to
♦♦ Catheterization to measure urine output (30–50 mL/hour multiorgan dysfunction syndrome, i.e. irreversible shock
or > 0.5 mL/kg/hour should be maintained).
♦♦ Nasal oxygen to improve oxygenation or ventilator sup-
port with intensive care unit monitoring has to be done.
♦♦ Central venous pressure line to perfuse adequately and to
monitor fluid balance. Total parentral nutrition is given
when required.
♦♦ Pulmonary capillary wedge pressure to monitor very
critical patient.
♦♦ Hemodialysis may be necessary when kidneys are not
functioning.
♦♦ Control pain—using morphine (4 mg IV).
♦♦ Ventilator and ICU/critical care management.
♦♦ Injection ranitidine IV or omeprazole IV or pantoprazole
IV.
♦♦ Activated protein even though costly is beneficial as it
prevents the release and action of inflammatory response.
♦♦ MAST(Military Anti-shock Trouser) provides circumferen-
tial external pressure of 40 mm Hg. lt is wrapped around
lower limbs and abdomen, and inflated with required pres-
sure. It redistributes the existing blood and fluid towards
center. It should be deflated carefully and gradually.
Q.2. Describe briefly hypovolemic shock.
 (Sep 1997, 8 Marks)
Or
Describe clinical feature and management of hypovo­ Fig. 10:  Hypovolemic shock
lemic shock. (Jan 2018, 20 Marks) Causes
Or
♦♦ Loss of extracellular fluid:
Write a short note on hypovolemic shock.
• Deviation of normal exchange pattern: As in vomiting,
 (Apr 2017, 4 Marks) diarrhea, intestinal obstruction, peritonitis, and acute
Ans. pancreatitis.
It occurs due to loss of blood plasma or body fluid and • Increased sweating without replacement in a non-
electrolytes, usually caused by massive hemorrhage, vomiting, acclimatized individual.
diarrhea and dehydration. • Third space shift to sodium from extracellular to
Hypovolemic shock is most common type of shock which intracellular compartment due to failure of sodium
is characterized by loss in circulatory volume which leads pump caused by hypoxia.
to decrease in venous return, decrease in filling of cardiac ♦♦ Plasma loss: Due to burn.
chambers, so there is decreased cardiac output which causes ♦♦ Hemorrhage: Due to whole blood loss like.
increase in systemic vascular resistance. • Surgical: During and following any major surgery
especially cardiopulmonary bypass, pelvic surgery
or major abdominal surgery.
Types of Hypovolemia
• Traumatic: As a result of any type of major accident,
♦♦ Covert compensated hypovolemia (Mild <15%): When warfare injuries, homicidal or following suicidal injury
blood volume is reduced by 10–15%, there will not be sig- as by knife, bullet, etc.
326   Mastering the BDS IIIrd Year  (Last 25 Years Solved Questions)

• GI bleeding: Bleeding from peptic ulcer, perforation of For management of hypovolemic shock refer to ans 2 of
intestine, bleeding from esophageal varices, etc. same chapter.
• Obstructive bleeding: Incomplete abortion, placenta Q.4. Write short note on vasovagal shock. 
previa, etc.  (Mar 1996, 6 Marks)
Clinical Features Ans. •  This is a response to sudden fear or severe pain and
♦♦ Anxiety, restless, excitation and disorientation. the effects from slight fainting fit to death.
♦♦ Pallor • This type of shock is also known as neurogenic or
♦♦ Thirst and hunger psychogenic shock.
♦♦ Cold and clammy skin • There is sudden pooling of blood in the capacitance
♦♦ Faint in upright position vessels of legs and splanchnic arterial bed. This
♦♦ Tachycardia with rapid, thready pulse causes reduced cardiac output and shock. It can be
♦♦ Hypotension life threatening due to hypoxia.
♦♦ Oligouria or anuria.
Pathophysiology
Management ♦♦ Nucleus tractus solitarius of the brainstem is activated
directly or indirectly by the triggering stimulus.
♦♦ Simultaneous enhancement of parasympathetic nervous
system, i.e. vagal tone and withdrawal of sympathetic
nervous system tone, which causes either cardioinhibitory
response or vasodepressor response
♦♦ Cardioinhibitory response is characterized by a drop in
heart rate, i.e. negative chronotropic effect and in contrac-
tility, i.e. negative ianotropic effect which causes decrease
in cardiac output.
♦♦ Unconsciousness or vasodepressor response is caused by
a drop in blood pressure as low as 80/20 without much
change in heart rate.
Clinical Features
♦♦ History of emotional stress or pain of a sudden nature.
♦♦ Bradycardia or pallor.
♦♦ Tachypnea
♦♦ Fainting
♦♦ Reflexes are usually intact.
Management
♦♦ Place the patient flat or in head low position.
♦♦ Ensure potency of airway
♦♦ IV atropine may be needed for persistent or increasing
bradycardia.
Q.5. Describe briefly anaphylactic shock. 
Q.3. Enumerate different types of shock and discuss man­  (Mar 1998, 5 Marks) (Apr 2008, 5 Marks)
agement of hypovolemic shock.  Ans. This type of shock is a result of type I hypersensitivity
 (Sep 1999, 20 Marks) (Feb 2002, 5 Marks) reaction.
Ans. • Anaphylactic shock can occur when a previously
sensitized individual is exposed to a specific antigen,
Enumeration of Types of Shock iv. Drug, especially penicillin, cephalosporins and
♦♦ Vasovagal shock iodinated contrast media are common offenders.
♦♦ Neurogenic shock
Pathophysiology
♦♦ Hypovolemic shock
♦♦ Cardiogenic shock Injections—penicillins, anesthetics, stings, venom, shellfish
♦♦ Cardiac compression shock may be having antigens which will combine with IgE of mast
♦♦ Septic shock cells and basophils, releasing histamine and large amount
♦♦ Anaphylactic shock. of SRS—A (Slow releasing substance of anaphylaxis). They
Section 2:  General Surgery  327

cause bronchospasm, laryngeal edema, respiratory distress, ♦♦ Correction of acid-base balance: Acidosis is corrected by using
hypotension and shock. Mortality is 10%. 8.4% sodium bicarbonate intravenously.
♦♦ Steroid is often lifesaving. 500–l000 mg of hydrocortisone
Clinical Features
can be given. It improves the perfusion, reduces the
♦♦ Due to reduced cerebral perfusion, there is change in capillary leakage and systemic inflammatory effects.
mental status. ♦♦ Antibiotics in patients with sepsis; proper control of blood
♦♦ Due to reduced preload and cardiac contractility, there is sugar and ketosis in diabetic patients.
hypotension. ♦♦ Catheterization to measure urine output (30–50 mL/hour
♦♦ Due to release of histamine and other chemical mediators or > 0.5 ml/kg/hour should be maintained).
there is urticaria. ♦♦ Nasal oxygen to improve oxygenation or ventilator
♦♦ Due to hypoxia the cyanosis is caused. support with intensive care unit monitoring has to be done.
♦♦ Due to anaerobic metabolism and hepatic dysfunction the ♦♦ Central venous pressure line to perfuse adequately and to
lactic acidosis is caused. monitor fluid balance. Total parentral nutrition is given
♦♦ Due to coronary ischemia other dysrhythmias are caused. when required.
♦♦ Pulmonary capillary wedge pressure to monitor very
Treatment critical patient.
♦♦ Summon ambulance ♦♦ Hemodialysis may be necessary when kidneys are not
♦♦ Always check whether respiratory distress is due to other functioning.
causes. ♦♦ Control pain-using morphine (4 mg IV).
♦♦ Assess the degree of cardiovascular collapse by checking ♦♦ Ventilator and ICU/critical care management.
pulse and blood pressure. ♦♦ Injection ranitidine IV or omeprazole IV or pantoprazole IV.
♦♦ Assess the degree of airway obstruction ♦♦ Activated C protein even though costly is beneficial as it
♦♦ Stop administration of drug prevents the release and action of inflammatory response.
♦♦ Patient should be kept supine ♦♦ MAST (Military Anti-shock Trouser) provides circumferential
♦♦ Assess breathing difficulty by checking for stridor, wheeze external pressure of`40 mm Hg. lt is wrapped around lower
♦♦ Administer oxygen to patient by face mask limbs and abdomen, and inflated with required pressure. It
♦♦ Give antihistamine chlorpheniramine maleate 10 mg redistributes the existing blood and fluid towards center. It
♦♦ Administer hydrocortisone 20 mg should be deflated carefully and gradually.
♦♦ Monitor consciousness, airway, breathing, circulation, Q.7. Discuss the etiopathology, clinical feature and manage­
pulse, blood pressure ment of hemorrhage shock.
♦♦ Raise legs if blood pressure is low  (Sep 2008, 5 Marks) (Sep 2004, 15 Marks)
♦♦ Adrenaline 1:1000, 0.5 mL IM is given immediately.
Or
♦♦ Repeat IM adrenaline every 5 minutes while waiting for
ambulance Describe hemorrhagic shock and management.
♦♦ Administer 100% oxygen.  (Jan 2018, 10 Marks)
♦♦ CPR if cardiac arrest occurs. Ans. Hemorrhagic Shock
♦♦ If BP fall is rapid, 1 :10,000 adrenalin may be infused IV Etiopathology: Due to whole blood loss like.
slowly. • Surgical: During and following any major surgery
Q.6. Describe management of a patient in state of shock. especially cardiopulmonary bypass, pelvic surgery
 (Sep 2010, 15 Marks) or major abdominal surgery.
Ans. Shock is defined as an acute clinical syndrome • Traumatic: As a result of any type of major accident,
characterized by a significant, systemic reduction warfare injuries, homicidal or following suicidal
in tissue perfusion, resulting in decreased tissue injury as by knife, bullet, etc.
oxygen delivery and insufficient removal of cellular • GI bleeding: Bleeding from peptic ulcer, perforation
metabolic products, resulting in tissue injury and severe of intestine, bleeding from esophageal varices, etc.
dysfunction of vital organs. • Obstructive bleeding: Incomplete abortion, placenta
previa, etc.
Management Clinical Features
♦♦ Treat the cause, e.g. arrest hemorrhage, drain pus. ♦♦ Anxiety, restless, excitation and disorientation.
♦♦ Fluid replacement: Plasma, normal saline, dextrose, Ringer’s ♦♦ Pallor
lactate, plasma expander (haemaccel). Dosage is maximum ♦♦ Thirst and hunger
l liter can be given in 24 hours. Initially crystalloids then ♦♦ Cold and clammy skin
colloids are given. Blood transfusion is done whenever ♦♦ Faint in upright position
required. ♦♦ Tachycardia with rapid, thready pulse
♦♦ Ionotropic agents: Dopamine, dobutamine, adrenaline ♦♦ Hypotension
infusions—mainly in distributive shock like septic shock. ♦♦ Oligouria or anuria.
328   Mastering the BDS IIIrd Year  (Last 25 Years Solved Questions)

Management of Hemorrhage Shock Clinical Features


♦♦ The primary treatment of hemorrhagic shock is to control ♦♦ Syncopal attack is sudden.
the source of bleeding as soon as possible and to replace ♦♦ Prodromal symptoms: Tingling or numbness in limbs, sud-
fluid. den darkness before eyes and patient may have a feeling
♦♦ In controlled hemorrhagic shock, where the source of blacking out.
bleeding has been occluded, fluid replacement is aimed ♦♦ Person is cold and sweating fall in ground suddenly be-
toward normalization of hemodynamic parameters. coming unconscious.
♦♦ In uncontrolled hemorrhagic shock, in which the bleeding
has temporarily stopped because of hypotension, vaso- Treatment
constriction, and clot formation, fluid treatment is aimed ♦♦ All the dental procedure or treatment is stopped.
at restoration of radial pulse or restoration of sensorium ♦♦ Remove instruments from oral cavity such as rubber dam,
or obtaining a blood pressure of 80 mm Hg by aliquots guaze, cotton, etc.
of 250 mL of lactated Ringer’s solution (hypotensive re- ♦♦ Patient is kept in Trendelenburg position, i.e. patient is
suscitation). kept in a head low and feet up position.
♦♦ When evacuation time is shorter than 1 hour (usually ur- ♦♦ Loose tighten clothing of patient.
ban trauma), immediate evacuation to a surgical facility is ♦♦ Aromatic fumes inhalation is given or sprinkle cold water
indicated after airway and breathing have been secured. on face of patient for reflex stimulation.
♦♦ When expected evacuation time exceeds 1 hour, an intra- ♦♦ If recovery is gained escort patient home.
venous line is introduced and fluid treatment is started ♦♦ If recovery is not gained Injection atropine 0.6 mg IM or
before evacuation. The resuscitation should occur before, IV is given.
or concurrently with, any diagnostic studies. ♦♦ If still recovery is not gained look for hypoglycemia and
♦♦ Crystalloid is the first fluid of choice for resuscitation. Addison’s crisis.
Immediately administer 2 L of isotonic sodium chloride ♦♦ Start basic life support
solution or lactated Ringer’s solution in response to shock ♦♦ Summon medical help.
from blood loss.
♦♦ Fluid administration should continue until the patient’s Q.9. How will you manage a patient in shock from road side
hemodynamics become stabilized. accident having fracture of mandible and maxilla?
♦♦ Because crystalloids quickly leak from the vascular space,  (Feb 2013, 10 Marks) (Mar/Apr 2003, 18 Marks)
each liter of fluid expands the blood volume by 20–30%; Ans. In such case, there following shock occurs:
therefore, 3 L of fluid need to be administered to raise the • Neurogenic or vasovagal shock
intravascular volume by 1 L. For management refer to ans 5 of same chapter.
♦♦ Alternatively, colloids restore volume in a 1:1 ratio. Cur- • Hypovolemic shock, i.e. hemorrhagic shock
rently available colloids include human albumin, hydroxy- For management refer to Ans 2 of same chapter.
ethyl starch products (mixed in either 0.9% isotonic sodium
chloride solution or lactated Ringer’s solution), or hyper- Q.10. Write short note on management of hemor­rhagic
tonic saline-dextran combinations. shock. (Apr 2007, 5 Marks)
♦♦ Packed red blood cells (PRBCs) should be transfused if Or
the patient remains unstable after 2000 mL of crystalloid
Write in brief about hemorrhagic shock. 
resuscitation. For acute situations, O-negative noncross
 (Sep 2007, 5 Marks)
matched blood should be administered. Administer 2 U
rapidly, and note the response. For patients with active Or
bleeding, several units of blood may be necessary. Write short note on treatment of hemorrhagic shock. 
Q.8. Write short note on syncope. (Sep 2002, 5 Marks)  (Sep 2008, 5 Marks)
Ans. Syncope may be defined as transient loss of consciousness Ans. Refer to Ans 7 of same chapter.
which comes suddenly, lasts for a short-time and is due
to diminished blood supply to the brain. Q.11. Write in short on septic shock. (Oct 2007, 5 Marks)
The symptoms are fainting and vasovagal attack. Or
Etiology Write short note on septic shock. (Apr 2015, 3 Marks)

♦♦ Simple faint Or
♦♦ Decreased cardiac output in various heart diseases. Write short note on septicemic shock.
♦♦ Fear and sudden anxiety.  (June 2015, 5 Marks)
♦♦ Trauma to the deep lying structures. Ans. Septic shock is a vasodilator shock wherein there is
♦♦ Hypoglycemia peripheral vasodilation causing hypotension which is
♦♦ Bouts of coughing, etc. resistant to vasopressors.
Section 2:  General Surgery  329

Etiology ♦♦ Monitoring the patient by pulse oximetry, cardiac status,


urine output, arterial blood gas analysis.
Septic shock may be due to Gram-positive organisms, Gram
♦♦ Short-term (one or two doses) high dose steroid therapy
negative organisms, fungi, viruses or protozoal origin.
to control and protect cells from effects of endotoxaemia.
Pathophysiology of Septic Shock It improves cardiac, renal and lung functions. Single dose
of methylprednisolone or dexamethasone which often
Toxins/endotoxins from organisms like E. Coli,
may be repeated again after 4 hours is said to be effective
Klebsiella, Pseudomonas, and Proteus
in endotoxic shock.
Inflammation, cellular activation of Q.12. Describe various causes of shock. Discuss in detail
macrophages, neutrophils, monocytes management of septic shock. (Nov 2008, 15 Marks)

Release of cytokines, free radicals


Ans. For various causes of shock refer to Ans 1 of same
chapter. For management of septic shock refer to Ans
11 of same chapter.
Chemotaxis of cells, endothelial injury, altered
coagulation cascade—systemic inflammatory Q.13. Enumerate the complications of blood transfusion and
response syndrome describe the management of hypovolemic shock.
(Dec 2010, 10 Marks)
Reversible hyperdynamic warm stage of septic Ans.
shock with fever, tachycardia, tachypnea
Enumeration of Complications of Blood Transfusion
Severe circulatory failure with multiple organ ♦♦ Congestive cardiac failure
dysfunction syndrome with disseminated ♦♦ Transfusion reactions HBV, HCV:
intravascular coagulation • Incompatibility. Major and minor reactions with fever,
rigors, pain, hypotension
Hypodynamic irreversible cold stage of septic
• Pyrexial reactions due to pyrogenic ingredients in
shock.
the blood.
• Allergic reactions
Clinical Features
• Sensitisation to leukocytes and platelets
Based on stages of septic shock. • Immunological sensitization.
♦♦ Hyperdynamic or warm shock: ♦♦ Infections:
• It is a reversible stage. • Serum hepatitis
• Patient has fever, tachycardia and tachypnea • HIV infection
• Pyogenic response is intact • Bacterial infection
♦♦ Hypodynamic or cold septic shock: • Malaria transmission
• Pyogenic response is lost. • Epstein-Barr virus infection
• Patient is in decompensated shock. • Cytomegalovirus infection
• Presence of anuria, cyanosis, jaundice, cardiac • SyphiIis,Yersinia
depression, pulmonary edema, hypoxia, drowsiness, • Babesia microti infection
coma and death. • Trypanosoma cruzi infection.
♦♦ Air embolism
Treatment / Management
♦♦ Thrombophlebitis
♦♦ Correction of fluid and electrolyte by crystalloids, blood ♦♦ Coagulation failure:
transfusion. Perfusion is very/most important. • Dilution of clotting factors
♦♦ Appropriate antibiotics-third generation cephalosporins • Disseminated intravascular coagulation
and aminoglycosides. • Dilutional thrombocytopenia occurs in patients with
♦♦ Treat the cause or focus—drainage of an abscess; lapa- massive blood transfusion.
rotomy for peritonitis; resection of gangrenous bowel; ♦♦ Circulatory overload causing heart failure
wound excision. ♦♦ Haemochromatosis in patients with CRF receiving re-
♦♦ Pus/urine/discharge/bile/blood culture and sensitivity for peated blood transfusions
antibiotics. ♦♦ Citrate intoxication causes bradycardia and hypocal-
♦♦ Critical care, oxygen, ventilator support, dobutamine cemia.
dopamine/noradrenaline to maintain blood pressure and ♦♦ Iron overload.
urine output.
♦♦ Activated C protein prevents the release of inflammatory Management
mediators and blocks the effects of these mediators on
cellular function. Refer to Ans 2 of same chapter.
330   Mastering the BDS IIIrd Year  (Last 25 Years Solved Questions)

Q.14. Define and classify shock. In your practice local Q.15. What are the different type of hemorrhages? Discuss
anesthetic is used, which type of shock it can produce. clinical features and management of hypovolaemic
How will you recognize and treat. What precautions shock. (Aug 2012, 10 Marks)
should be taken to prevent. (Jan 2012, 15 Marks) Ans. For different types of hemorrhages refer to ans 2 of
Ans. Shock is a clinical condition characterized by inadequate chapter BLEEDING DISORDERS.
tissue perfusion and hence cellular hypoxia. For clinical features and management of hypovolemic
shock refer to Ans 2 of same Chapter.
Classification of Shock
Q.16. Write in brief about complications of blood transfusion.
♦♦ Hypovolemic shock  (Dec 2012, 5 Marks)
♦♦ Cardiogenic shock
Ans. Following are the complications of blood transfusion:
♦♦ Distributive shock
• Febrile reactions: It is the most common complication
• Septic shock
due to impurities like pyrogens in the blood or in
• Anaphylactic shock
infusion set. Headaches, fever, chills and rigors,
• Neurogenic shock.
tachycardia, nausea are the features. Transfusion
♦♦ Obstructive shock.
is temporarily stopped or the flow is slowed down
In clinical practice local anesthetic leads to anaphylactic
with administration of antipyretic drug to reduce
shock.
fever. Often transfusion of that unit needs to be
discontinued.
Recognition of Anaphylactic Shock • Allergic reaction: Utrticaria and allergy to specific
♦♦ Patient has asthma like symptoms, i.e. sneezing and breath- proteins in the donor’s plasma can occur. Usually, it is
ing. mild and is treated with steroid and antihistaminics.
♦♦ Urticaria and angioedema are present. In severe urticaria that unit of blood is discarded;
♦♦ Presence of bronchospasm and tachycardia new washed RBC’s and platelets are used.
♦♦ Patient can undergo circulatory collapse. • Acute hemolytic reactions: It is the most dangerous
♦♦ Due to rapid fall in blood pressure cardiac arrest may occur. complication. It is due to ABO incompatibility.
Usually it is nonfatal but occasionally can be fatal. It
Treatment of Anaphylactic Shock is commonly due to technical error at different levels.
It amounts for criminal negligence in court of law.
♦♦ Summon ambulance • Transfusion related graft versus host disease:
♦♦ Always check whether respiratory distress is due to other This very serious, very rare complication occurs
causes. due to recognition and reaction against host tissues
♦♦ Assess the degree of cardiovascular collapse by checking by infused donor lymphocytes. lt is common in
pulse and blood pressure. immunosuppressed, lymphoma, leukemic patients.
♦♦ Assess the degree of airway obstruction Any type of blood products including leukocyte
♦♦ Stop administration of drug reduced blood can cause the condition. Features
♦♦ Patient should be kept supine are pancytopenia, toxic epidermal necrosis, liver
♦♦ Assess breathing difficulty by checking for stridor, wheeze dysfunction with more than 90% mortality. It is
♦♦ Administer oxygen to patient by face mask difficult to treat.
♦♦ Give antihistamine chlorpheniramine maleate 10 mg • Congestive cardiac failure: It occurs if especially
♦♦ Administer hydrocortisone 20 mg
large quantities of whole blood are transfused in
♦♦ Monitor consciousness, airway, breathing, circulation,
chronic severe anemia, pregnancy, elderly patients,
pulse, blood pressure
in patients who have cardiac problems.
♦♦ Raise legs if blood pressure is low
♦♦ Adrenaline 1:1000, 0.5 mL IM is given immediately. Q.17. Enumerate differences between hypovolemic and
♦♦ Repeat IM adrenaline every 5 minutes while waiting for septic shock. (June 2014, 2 Marks)
ambulance Or
♦♦ Administer 100% oxygen. Write difference between septic and hypovolemic
♦♦ CPR if cardiac arrest occurs. shock. (Jan 2016, 2 Marks)
♦♦ If BP fall is rapid, 1:10,000 adrenalin may be infused IV Ans.
slowly.
Hypovolemic shock Septic shock
Precautions Taken to Prevent the Anaphylactic Shock It is due to reduction in total blood It is due to bacterial infections which
volume release toxins leading to shock
♦♦ Intradermal test should be done before administering local
anesthetic solution. Causes are hemorrhage due to Causes are Gram-positive and
trauma, severe burns, peritonitis Gram-negative organisms, fungi,
♦♦ Proper medical history of the patient is taken, if patient
etc. viruses and protozoa
gives history of allergy from local anesthetic solution, drug
should not be administered. Fluid replacement should be done Antibiotics are to be given
Section 2:  General Surgery  331

Q.18. Describe etiopathogenesis and classification of shock. ♦♦ Because crystalloids quickly leak from the vascular space,
How will you manage hemorrhagic shock. each liter of fluid expands the blood volume by 20–30%;
 (Feb 2014, 8 Marks) therefore, 3 L of fluid need to be administered to raise the
Ans. Etiopathogenesis intravascular volume by 1 L.
♦♦ Alternatively, colloids restore volume in a 1:1 ratio.
Decreased effective circulatory blood volume
Currently available colloids include human albumin,
Decreased cardiac output hydroxyethyl starch products (mixed in either 0.9%
isotonic sodium chloride solution or lactated Ringer’s
Decreased blood flow solution), or hypertonic saline-dextran combinations.
♦♦ Packed red blood cells (PRBCs) should be transfused
Decreased supply of oxygen if the patient remains unstable after 2000 mL of
crystalloid resuscitation. For acute situations, O-negative
Anoxia noncrossmatched blood should be administered.
Administer 2 U rapidly, and note the response. For
Shock patients with active bleeding, several units of blood may
be necessary.
Classification Q.19. Describe different types of shock. How to manage
♦♦ Hypovolemic shock hemorrhagic shock? (Jan 2017, 10 Marks)
♦♦ Cardiogenic shock Or
♦♦ Distributive shock: Define shock, clinical feature and managment of vari­
• Septic shock ous type of shock? (Apr 2018, 5 Marks)
• Anaphylactic shock Ans. Shock is defined as an acute clinical syndrome
• Neurogenic shock. characterized by a significant, systemic reduction
♦♦ Obstructive shock. in tissue perfusion, resulting in decreased tissue
oxygen delivery and insufficient removal of cellular
Management of Hemorrhagic Shock metabolic products, resulting in tissue injury and severe
dysfunction of vital organs.
♦♦ The primary treatment of hemorrhagic shock is to control
the source of bleeding as soon as possible and to replace Description of Different Types of Shock
fluid.
Classification of Shock
♦♦ In controlled hemorrhagic shock, where the source
of bleeding has been occluded, fluid replacement ♦♦ Hypovolemic shock
is aimed toward normalization of hemodynamic ♦♦ Cardiogenic shock
parameters. ♦♦ Distributive shock:
♦♦ In uncontrolled hemorrhagic shock, in which the bleeding • Septic shock
has temporarily stopped because of hypotension, • Anaphylactic shock
vasoconstriction, and clot formation, fluid treatment • Neurogenic shock
is aimed at restoration of radial pulse or restoration ♦♦ Obstructive shock
of sensorium or obtaining a blood pressure of 80 mm
Hypovolemic shock
Hg by aliquots of 250 mL of lactated Ringer’s solution
(hypotensive resuscitation). ♦♦ It occurs due to loss of blood plasma or body fluid and
♦♦ When evacuation time is shorter than 1 hour (usually electrolytes, usually caused by massive hemorrhage,
urban trauma), immediate evacuation to a surgical vomiting, diarrhea and dehydration.
facility is indicated after airway and breathing have been ♦♦ Hypovolemic shock is most common type of shock which
secured. is characterized by loss in circulatory volume which leads
♦♦ When expected evacuation time exceeds 1 hour, an to decrease in venous return, decrease in filling of cardiac
intravenous line is introduced and fluid treatment is chambers, so there is decreased cardiac output which
started before evacuation. The resuscitation should causes increase in systemic vascular resistance.
occur before, or concurrently with, any diagnostic
Causes
studies.
♦♦ Crystalloid is the first fluid of choice for resuscitation. ♦♦ Loss of extracellular fluid:
Immediately administer 2 L of isotonic sodium chloride • Deviation of normal exchange pattern: As in vomiting,
solution or lactated Ringer’s solution in response to shock diarrhea, intestinal obstruction, peritonitis, and acute
from blood loss. pancreatitis.
♦♦ Fluid administration should continue until the patient’s • Increased sweating without replacement in a non-
hemodynamics become stabilized. acclimatized individual.
332   Mastering the BDS IIIrd Year  (Last 25 Years Solved Questions)

• Third space shift to sodium from extracellular to ♦♦ Cardiogenic shock occurs if more than 40% of left ventricle
intracellular compartment due to failure of sodium is involved in acute infection.
pump caused by hypoxia. ♦♦ Elevated cardiac chamber filling procedure is hallmark of
♦♦ Plasma loss: Due to burn. cardiogenic shock.
♦♦ Hemorrhage: Due to whole blood loss like.
• Surgical: During and following any major surgery Clinical Features of Cardiogenic Shock
especially cardiopulmonary bypass, pelvic surgery ♦♦ The primary problem is decrease in contractility of heart,
or major abdominal surgery. due to decrease contractility, there is decrease in stroke
• Traumatic: As a result of any type of major accident, volume.
warfare injuries, homicidal or following suicidal injury ♦♦ Patient present with tachycardia, low blood pressure and
as by knife, bullet, etc. decrease urinary output.
• GI bleeding: Bleeding from peptic ulcer, perforation of ♦♦ Jugular venous pressure may be raised.
intestine, bleeding from esophageal varices, etc. ♦♦ Peripheries are cold and patient may be confuse or
• Obstructive bleeding: Incomplete abortion, placenta *moribund.
previa, etc.
Treatment
Clinical Features
♦♦ Proper oxygenation with intubation, ventilator support,
♦♦ Anxiety, restless, excitation and disorientation.
cardiac version, pacing, antiarrhythmic drugs, correction
♦♦ Pallor
of electrolytes, avoiding fluid overload and prevention of
♦♦ Thirst and hunger
pulmonary edema as immediate measures.
♦♦ Cold and clammy skin
♦♦ Dobutamine is used to raise cardiac output provided there
♦♦ Faint in upright position
is adequate preload and intravascular volume. Dopamine
♦♦ Tachycardia with rapid, thready pulse
is preferred in patients with hypotension. But it may
♦♦ Hypotension
increase peripheral resistance and heart rate worsening
♦♦ Oligouria or anuria.
cardiac ischemia. Often both dopamine and dobutamine
Management combination may be required.
♦♦ Careful judicial use of epinephrine. norepinephrine,
phosphdiesterase inhibitors (amrinone, milrinone)
are often needed. Anticoagulants and aspirin are
given. Thrombolytics can be used β blockers, nitrates
(nitroglycerine causes coronary arterial dilatation). ACE
inhibitors are also used.
♦♦ Intra-aortic balloon pump: May need to be introduce;
transfemorally as a mechanical circulatory support to raise
cardiac output and coronary blood flow.
♦♦ Relief of pain, preserving of remaining myocardium and
its function, maintaining adequate preload, oxygenation,
minimizing sympathetic stimulation, correction of
electrolytes should be the priorities.
♦♦ Percutaneous transluminal coronary angioplasty (PTCA)
and coronary artery bypass graft (CABG) are the final
choices.

Distributive Shock
This occurs when the after load is excessively reduced.
Distributive shock occurs in following situations:
♦♦ Septic shock
♦♦ Anaphylactic shock
♦♦ Neurogenic shock
Septic shock
♦♦ This type of shock is mostly due to release of endotoxins
in blood, which causes wide spread vasodilation of blood
Cardiogenic shock vessels resulting in fall in the cardiac output. Fall in the
♦♦ Myocardial infection is the most common cause of cardiac output is not initial feature and vasoconstriction
cardiogenic shock. is not observed.
Section 2:  General Surgery  333

♦♦ Bacteria responsible for release of endotoxins are E. coli, ♦♦ Repeat IM adrenaline every 5 minutes while waiting for
Pseudomonas, Proteus, etc. ambulance
♦♦ Administer 100% oxygen
Clinical features
♦♦ CPR if cardiac arrest occurs
♦♦ Restlessness, anxiety ♦♦ If BP fall is rapid, 1:10,000 adrenalin may be infused IV
♦♦ Cyanosis slowly.
♦♦ Cold and clammy skin
♦♦ Tachycardia Neurogenic Shock
♦♦ Oligouria or anuria ♦♦ It occurs due to spinal cord injury which leads to dilatation
♦♦ Acidotic breathing. of splanchnic vessels
♦♦ There will be bradycardia, hypotension, arrhythmias and
Management
decreased cardiac output.
♦♦ Sedation with diazepam
♦♦ IV fluids Clinical Features
♦♦ Blood culture and sensitivity ♦♦ History of emotional stress or pain of a sudden nature.
♦♦ Antimicrobial agents: Combination of penicillin or ♦♦ Bradycardia or pallor
cephalosporins and aminoglycosides and metronidazole. ♦♦ Tachypenea
♦♦ Injection hydrocortisone ♦♦ Fainting
♦♦ Reflexes are usually intact.
Anaphylactic Shock
Treatment
Anaphylactic shock can occur when a previously sensitized
individual is exposed to a specific antigen, IV drug, specially ♦♦ Blood pressure should be controlled by giving
penicillin, cephalosporins and iodinated contrast media are vasoconstrictors.
common offenders. ♦♦ Oxygen is administered
♦♦ Hemodynamics should be maintained.
Clinical Features ♦♦ Airways are cleared.
♦♦ Due to reduced cerebral perfusion, there is change in ♦♦ Fluid therapy should be given
mental status. ♦♦ Intravenous methylprednisolone therapy is done.
♦♦ Due to reduced preload and cardiac contractility, there is ♦♦ Dopamine and phenylephrine can be used.
hypotension.
Obstructive Shock
♦♦ Due to release of histamine and other chemical mediators
there is urticaria. ♦♦ The obstructive shock is due to cardiac temponade, due to
♦♦ Due to hypoxia the cyanosis is caused. tension pneumothorax and pulmonary embolus.
♦♦ Due to anaerobic metabolism and hepatic dysfunction the ♦♦ In cardiac temponade, there is compression of all chambers
lactic acidosis is caused. of heart with reduce cardiac output. The filling pressure
♦♦ Due to coronary ischemia other dysrhythmias are caused. of left- and right-sided chambers equalizes. The central
venous pressure is high and the BP is low.
Treatment
Treatment
♦♦ Summon ambulance
♦♦ Always check whether respiratory distress is due to other ♦♦ To maintain preload with fluid or blood.
causes. ♦♦ Relief of obstructions, drain pericardial cavity as early as
♦♦ Assess the degree of cardiovascular collapse by checking possible.
pulse and blood pressure. For management of hemorrhagic shock refer to Ans 22 of same
♦♦ Assess the degree of airway obstruction chapter.
♦♦ Stop administration of drug
♦♦ Patient should be kept supine Q.20. Define shock. Name types of shock which can be met
♦♦ Assess breathing difficulty by checking for stridor, wheeze within your practice. How will you manage them.
♦♦ Administer oxygen to patient by face mask
 (Jan 2017, 20 Marks)
♦♦ Give antihistamine chlorpheniramine maleate 10 mg
Ans. Shock is defined as an acute clinical syndrome
♦♦ Administer hydrocortisone 20 mg characterized by a significant, systemic reduction
♦♦ Monitor consciousness, airway, breathing, circulation, in tissue perfusion, resulting in decreased tissue
pulse, blood pressure oxygen delivery and insufficient removal of cellular
♦♦ Raise legs if blood pressure is low metabolic products, resulting in tissue injury and severe
♦♦ Adrenaline 1:1000, 0.5 mL IM is given immediately. dysfunction of vital organs.
334   Mastering the BDS IIIrd Year  (Last 25 Years Solved Questions)

Types of shock which can be met in my dental practice and ♦♦ Very rapid transfusion
management of the same
Prevention
Following are the shocks which can be in my dental practice:
♦♦ Anaphylactic shock: It occurs due to the allergy caused by These reactions can be prevented by using sterilized plastic
local anesthetic agent. For details refer to Ans 14 of same disposable sets.
chapter
Treatment
♦♦ Hypovolemic shock: It occur during and following any major
dental surgery. For details refer to Ans 20 of same chapter. Transfusion should be stopped immediately though temporarily.
Antihistaminics and antipyretics should be given. After his
Q.21. Hazards of blood transfusion. (Jan 2017, 10 Marks)
condition returns to normal, blood transfusion can be restarted.
Ans. Following are the hazards of blood transfusion:
Transfusion Reactions—Incompatibility Allergic Reactions

Causes Usually, within few hours of transfusion, patients may get mild
urticaria, tachycardia, fever and dyspnea. He may even go into
♦♦ Mistake in crossmatching: This is a technical error, if the severe anaphylactic shock.
serum is old or labeling is wrong.
♦♦ Due to transfusion of blood which is already hemolysed Treatment
by warming, over freezing or shaking. Transfusion should be stopped. Antihistamines and cortisones
♦♦ Due to transfusion of blood after expiry date. should be given. Shock should be treated.
Clinical Features of Mismatched Blood Transfusion Transmission of Diseases
♦♦ Serum hepatitis: Hepatitis B is a common disease which can
♦♦ Presence of rigors and fever. Patient may complain of
be transmitted during blood transfusion. The symptoms
nausea, vomiting, headache, pain in the loins, tingling
usually appear within 3 months.
sensation in the extremities.
♦♦ AIDS: HIV can be transmitted from the donor’s blood to
♦♦ There can be chest pain and dyspnea.
the recipient.
♦♦ If the patient is already in shock, it may become more
♦♦ Bacterial infection: This occurs due to faulty storage
pronounced instead of curing it. Gradually, he will lose
technique. This should be treated with higher antibiotics
consciousness.
otherwise patient may go into septicemia.
♦♦ Urine output decreases and hemoglobinuria may occur
within 2 to 3 hours. Reaction Due to Massive Blood Transfusion
♦♦ Jaundice may appear within 24–36 hours, this is the
confirmatory evidence of mismatching. "Massive blood transfusion implies single transfusion of 8-10
♦♦ Ultimately renal failure sets in due to the blockage of renal units of blood in 24 hours."
tubules by hematin pigment. ♦♦ Acid-base imbalance results in significant metabolic
alkalosis.
Treatment ♦♦ Hyperkalemia
♦♦ Transfusion should be stopped immediately. ♦♦ Citrate toxicity: After massive blood transfusion, increased
♦♦ Fresh blood specimen of venous blood and urine from the citrate level consumes ionized calcium from patient’s body.
patient should be sent to laboratory for rechecking along The body compensates it by rapidly mobilizing calcium
with the rejected blood pack. from the bones. Rarely when hypocalcemia is recognized
♦♦ IV fluids should be started instead of blood. calcium can be infused.
♦♦ Alkalization of blood to be done by isotonic solution ♦♦ Hypothermia: During massive blood transfusion, cold blood
of sodium lactate and 10 mL of sodium bicarbonate to is rapidly infused from the refrigerator to the patient. His
facilitate precipitation of hematin pigments. temperature may drop by 3 to 4°C.
♦♦ Frusemide 80–120 mg IV should be given for forced diuresis. ♦♦ Failure of coagulation: After massive blood transfusion, the
This may be repeated, if urine output is increased to 30 mL/h. natural process of coagulation may fail due to dilution of
♦♦ Antihistamine and hydrocortisones should be given. platelets and various clotting factors. .
♦♦ In very severe cases. hemodialysis should be undertaken. ♦♦ Disseminated intravascular coagulation (DIC): This may occur
after a massive blood transfusion. Actually, it occurs after
Pyrexial Reactions mismatched blood transfusion.
It is common to see simple reactions like pyrexia, chills, rigors,
Complications of Over Transfusion
restlessness, headache, tachycardia, nausea and vomiting.
These complications may be seen in patients with chronic
Causes anaemia, in children and elderly patients. They should receive
♦♦ Improperly sterilized drip sets. packed cells rather whole blood. Transfusions should be given
♦♦ Presence of pyrogens in the donor set. slowly for 4–6 hours and after some intervals. Elderly patients
♦♦ Transfusion of infected blood. should be given packed cells with diuretics.
Section 2:  General Surgery  335

Complications of Intravenous Transfusions


Thrombophlebitis and air embolism.

Pulmonary Complications
Syndrome of transfusion—related acute lung injury is defined
as noncardiogenic pulmonary edema related to transfusion.

Q.22. Write short note on indications of blood transfusion.


 (Apr 2017, 4 marks)
Ans. Following are the indications of blood transfusion:
♦♦ Acute blood loss following trauma ≥15% of total body
volume in otherwise healthy individuals (liver, spleen,
kidney, GIT injuries, fractures, hemothorax, perineal
injuries).
♦♦ During major surgeries—abdominoperineal surgery,
thoracic surgery, hepatobiliary surgery.
♦♦ Following burns.
♦♦ In septicemia.
♦♦ As a prophylactic measure prior to surgery.
♦♦ Whole blood is given in acute blood loss.
♦♦ Packed cells are given in chronic anemia.
♦♦ Blood fractions are given in idiopathic thrombocytopenic
purpura, hemophilias. Percentage of burns in (A) Adults; (B) Children

Figs 11A and B:  Extent of burn

9. Burn The amount of fluid that should be ideally transfused in case


of burn is calculated by:
Q.1. What is Wallace’s rule of nine? Calculate the amount
Muir and Barclay Formula
of fluid that should be ideally transfused to an adult
weighing 60 kg and having 50% deep burns in the first % of burn × Body weight (kg)
24 hours. (Sep 2004, 15 Marks) 1 ration =
2
Ans. Wallace’s rule of nine is given by Wallace for early
3 ration in 12 hours, 2 ration in next 12 hours and 1 ration in
assessment of extent of burn in terms of body surface
next 12 hours.
area.
50% burns patient weighting 60 kg
Burn: A burn is a wound in which there is coagulative
necrosis of tissue. 50 × 60
1 ration = = 1500 mL
2
Rule of Nine (Wallace’s Rule of “9”) 1st 12 hours: 1500 × 3 = 4500 mL
2nd 12 hours: 1500 × 2 = 3000 mL
Adults Children Infants
Total 7500 mL fluid in 24 hours.
Head and neck 9% 18% 20%
Q.2. Describe the causes of burn and management of face
Front of chest and 9 × 2 = 18% 18% 10 × 2 = 20%
abdominal wall
and neck burn. (Mar 2007, 10 Marks)
Ans. Burn: A burn is a wound in which there is coagulative
Back of chest and 9 × 2 = 18% 18% 10 × 2 = 20%
necrosis of tissue.
abdominal wall
Lower limb 18 × 2 = 36% 13.5 × = 27% 10 × 2 = 20% Causes of Burn
Upper limb 9 × 2 = 18% 18% 10 × 2 = 20% ♦♦ Ordinary causes: Dry heat, fire, hot metal, aeroplane crash.
Perineum 01% 01% ♦♦ Due to moist heat, e.g. boiling liquid or lipids, it is called
as scald.
Total 100% 100% 100%
♦♦ Electric burn: Due to high voltage current.
336   Mastering the BDS IIIrd Year  (Last 25 Years Solved Questions)

♦♦ Chemical burns: Due to strong acid and alkali Local Management


♦♦ Radiation burn: Due to X-ray and radiation ♦♦ Dressing at regular intervals under general anesthesia
♦♦ Cold burn: It is caused by prolonged exposure to cold. using paraffin gauze, hydrocolloids, plastic films, vase-
Management of Face and Neck Burn line impregnated gauze or fenestrated silicone sheet or
biological dressings like amniotic membrane or synthetic
First Aid biobrane.
♦♦ Stop the burning process and keep patient away from ♦♦ Open method with application of silver sulfadiazine
burning area. without any dressings, used commonly in burns of face
♦♦ Cool the area with tap water by continuous irrigation for and neck.
20 minutes. ♦♦ Closed method is with dressings done to soothen and to
protect the wound, to reduce the pain, as an absorbent.
Definitive Treatment ♦♦ Tangential excision of burn wound with skin grafting can
♦♦ Patient should be admitted. be done within 48 hours in patients with less than 25%
♦♦ Airway, breathing and circulation is maintained. burns. It is usually done in deep dermal burn wherein
♦♦ Percentage, degree and type of burn is assessed. dead dermis is removed layer by layer until fresh bleeding
♦♦ Patient should be kept in clean environment. occurs. Later skin grafting is done.
♦♦ Patient is sedated and proper analgesia is given. ♦♦ In burns of head and neck region, exposure treatment is
Fluid Resuscitation advised.
♦♦ Slough excision is done regularly.
♦♦ Fluid replacement is done by calculating with various ♦♦ After cleaning with povidone iodine solution silver
formulas such as Parkland regime, Muir and Buckley sulfadiazine ointment is used. It is an antiseptic and
regime, etc. soothening agent.
♦♦ Fluids used are normal saline, Ringer lactate, Hartmanns
fluid. Ringer lactate is the choice of fluid. Wound Coverage
♦♦ Blood is transfused in later period after 48 hours.
♦♦ Better outcomes can be achieved if the non-healing areas
General Treatment are excised (likely to take 3 weeks or more) and then skin
♦♦ For first 24 hours only crystalloids should be given. grafted.
♦♦ After 24 hours up to 30–48 hours, colloids should be given ♦♦ The donor skin needs to be taken from area above the
to compensate plasma loss. Plasma, haemaccel (gelatin), nipple for the best color match. Many surgeons favor
dextrans, hetastarch are used. Usually at a rate of 0.35–0.5 the scalp skin but, alopecia and hair growth from trans-
mL/kg% burns is used in 24 hours. planted skin is of concern whereas the upper part of
♦♦ Urinary catheterization to monitor output; 30–50 mL/hour back has thick skin.
should be the urine output. ♦♦ Facial excision is carried out using Goulian knives or
♦♦ Tetanus toxoid. versajet water dissector. Exposed cartilage needs excision
♦♦ Monitoring the patient: Hourly pulse, BP, PO2, PCO2 elec- with closure of skin.
trolyte analysis, blood urea, nasal oxygen, often intubation ♦♦ Sheet of autografts are used as the meshed grafts are cos-
is required. Endotracheal intubation is secured in such a metically unacceptable.
♦♦ Epinephrine lysis is essential to limit hemorrhage.
way to minimize pressure necrosis of lip.
♦♦ A face mask should be placed to help immobilization of
♦♦ IV ranitidine 50 mg 8th hourly is given.
skin graft.
♦♦ Ryle’s tube insertion initially for aspiration purpose later
♦♦ Graft should be placed in such a fashion as to mimic the
for feeding.
esthetic units. Fibrin glue can be used to enhance the graft
♦♦ Antibiotics: Penicillins, aminoglycosides, cephalosporins,
adherence.
metronidazole should be given.
♦♦ Postoperative facial elastic mask compression helps in
♦♦ Culture of the discharge; total white cell count and platelet
avoiding hypertrophic scar.
count at regular intervals are essential to identify the sepsis
along with fever, tachycardia and tachypnea. Q.3. Describe briefly “rule of nine” and management of
♦♦ In burns of oral cavity tracheostomy may be required to burn patient. (Jun 2010, 5 Marks)
maintain the airway.
♦♦ Total parenteral nutrition (TPN) is required for faster Or
recovery, using carbohydrates, lipids, vitamins. Describe briefly Wallace's “rule of nine”
♦♦ Tracheostomy/intubation tube may be required in impend-  (Apr 2017, 5 Marks)
ing respiratory failure or upper airway block. Ans. For rule of nine, refer to Ans 1 and for management of
♦♦ Intensive nursing care. burn patient refer to Ans 2 of same chapter.
Section 2:  General Surgery  337

Q.4. Classify the types of burns and describe the manage­ A chronic recurring, inflammatory, vascular occlusive
ment of a case of 30% burns. (Mar 2006, 15 Marks) disease, chiefly of the peripheral arteries and veins of
Ans. the extremities.

Classification of Types of Burns Classification


Depending on Percentage of Burns ♦♦ Type I- Upper limb thromboangiitis obliterans.
♦♦ Type II- Involving leg and feet – crural/infrapopliteal
♦♦ Mild: ♦♦ Type III- Femoropopliteal
• Partial thickness burns < 15% in adult or < 10% in ♦♦ Type IV- Aortoiliofemoral
children ♦♦ Type V- Generalized.
• Full thickness burns less than 2%
• Can be treated on outpatient basis. Pathogenesis
♦♦ Moderate:
Smoke contains carbon monoxide and nicotinic acid
• Second degree of 15–25% burns
• Third degree between 2–10% burns Causes initially vasospasm and hyperplasia of intima
• Burns which do not involve eyes, ears, face, hand, feet.
♦♦ Major Thrombosis and so obliteration of vessels occur,
• Second degree burns more than 25% in adults and commonly medium sized vessels are involved.
more than 20% in children.
• All third degree burns of 10% or more Panarteritis is common
• Burns involving eyes, ears, feet, hands, perineum
• All inhalation and electric burns Usually involvement is segmental
• Burns with fractures or major mechanical trauma.
Eventually artery, vein and nerve are together involved
Depending on Thickness of Skin Involved
Nerve involvement causes rest pain
♦♦ First degree: Epidermis look red and painful, no blisters,
heal rapidly in 5 to 7 days by epithelialization without
Patient presents with features of ischemia in the limb
scaring.
♦♦ Second degree: Affected area is mottled, red, painful with Once blockage occurs, plenty of collaterals open up
blisters, heals by epithelialization in 14 to 21 days. depending on the site of blockage either around knee
• Superficial second degree burn heals causing joint or around buttock. Once collaterals open up,
pigmentation
through these collaterals blood supply is maintained to
• Deep second degree burn heals causing scarring and
the ischemic area
pigmentation.
♦♦ Third degree: Affected area is charred, parchment like,
painless and insensitive with thrombosis of superficial It is called as compensatory peripheral vascular disease.
vessels. It needs grafting. Charred, denatured, insensitive,
contracted full thickness burn is known as eschar. If patient continues to smoke disease progresses into the
♦♦ Fourth degree: It involve deeper structures, i.e. muscles collaterals blocking them eventually, leading to severe
and bones. ischemia and is called as decompensatory peripheral
vascular disease. It is presently called as critical limb
Depending on Thickness of Skin Involved ischemia. It causes rest pain, ulceration and gangrene.
♦♦ Partial thickness burns: It is a first or second degree burn
which is red and painful often with blisters. Clinical Features
♦♦ Full thickness burns: It is a third degree burn which is
charred, insensitive, deep involving all layers of skin. ♦♦ It is common in male smokers between 20 to 40 years of age.
♦♦ Intermittent claudication in foot and calf progressing to
Management of Case of 30% Burn rest pain, ulceration and gangrene.
Case of 30% burn comes under the major burn. ♦♦ It includes paresthesia of the foot or pain confined to one
For management in detail refer to ans 2 of same chapter. toe.
♦♦ Easy fatiguability and leg cramps. The leg fatigue quickly
especially during walking.
10. Arterial Diseases ♦♦ Ulceration or moist gangrene of hands and feet; amputa-
tion may be necessary.
Q.1. Write in short on Burger’s disease. ( Oct 2007, 10 Marks) ♦♦ Recurrent migratory superficial thrombophlebitis
Ans. It is also known as thromboangiitis obliterans or smoker’ ♦♦ Absence/Feeble pulses distal to proximal; dorsalis pedis,
disease. popliteal and femoral arteries.
338   Mastering the BDS IIIrd Year  (Last 25 Years Solved Questions)

Diagnosis ♦♦ Lumbar sympathectomy to increase the cutaneous perfu-


sion so as to promote ulcer healing. But it may divert blood
Criteria of Olin (2000) from muscles towards skin causing muscle more ischemic.
♦♦ Amputations are done at different levels depending on
♦♦ It occurs typically between 20 and 40 years of age in male,
site, severity and extent of vessel occlusion. Usually either
although females are also diagnosed.
below—knee or above-knee amputations are done.
♦♦ History of tobacco use.
♦♦ Ilzarov method of bone lengthening helps in improving
♦♦ Presence of distal extremity ischemia indicated by clau-
the rest pain and claudication by creating neo-osteogenesis
dication, pain at rest, ischemic ulcers or gangrene. This
and improving the overall blood supply to the limb.
is documented by noninvasive vascular testing, i.e. ul-
trasound. Q.2. Discuss briefly pressure sores.
♦♦ Exclusion of other autoimmune diseases such as hyperco-  (May/Jun 2009, 5 Marks)
agulable states and diabetes mellitus by laboratory tests. Or
♦♦ Exclusion of proximal source of emboli by echocardiogra- Discuss briefly bed (pressure) sores. (Aug 2012, 5 Marks)
phy and arteriography. Ans. It is also known as decubitus ulcer or pressure sores or
♦♦ Consistent arteriographic findings in clinically involved Bedsores.
and non-involved limbs.
No laboratory test should confirm the diagnosis of Burger’s Pathophysiology
disease. Main goal of various investigations is to exclude other
Pressure sore occurs due to compression of tissues by an external
diseases in differential diagnosis.
force. Blood flow to skin stops as external pressure becomes
Treatment more than 30mm of Hg which causes ischemia. Ischemia causes
inflammation and tissue anoxia. Tissue anoxia leads to cell
Smoking is strictly stopped. death, necrosis and ulceration.
Drugs Clinical Features
♦♦ Vasodilators, i.e. nifedipine should be given. ♦♦ It is a trophic ulcer with bone as a base.
♦♦ Pentoxifylline should be given since it increases flexibility ♦♦ It is non-mobile, deep, punched out ulcer.
of RBCs so they reach easily in microcirculation. ♦♦ It is commonly seen in old age bed ridden patients, teta-
♦♦ Aspirin 75 mg OD is given. nus, orthopedic patients, In diabetics and patients with
♦♦ Analgesics, often sedatives, antilipid drugs like atorvas- head injuries.
tatin may be needed.
Sites Affected
Care of the Limbs
♦♦ Over the ischial tuberosity
Buerger’s position and exercise: Regular graded exercises up to ♦♦ Sacrum
the point of claudication improves the collateral circulation. In ♦♦ In the heel
Buerger’s position, head end of bed is raised; foot end of bed ♦♦ In relation to head of metatarsala
is lowered to improve circulation. In Buerger’s exercise leg ♦♦ Buttocks
is elevated and lowered alternatively, each for 2 minutes for ♦♦ Over the shoulder
several times at time. ♦♦ Occiput
Care of feet: It is better to wear socks with footwear. Heel raise Factors Causing Pressure Sore
by raising the heels of shoes by 2 cm decreases the calf muscle
work to improve claudication. ♦♦ Normal stimulus to relieve pressure is absent in anesthe-
tized patient.
Chemical Sympathectomy ♦♦ Nutritional deficiencies worsen the necrosis
♦♦ Inadequate padding over bony prominences in malnour-
Sympathetic chain is blocked to achieve vasodilatation by ished patients
injecting local anesthetic agent (xylocaine 1%) paravertebrally ♦♦ Urinary incontinence in paraplegia patient causes skin
beside bodies of L 2, 3 and 4 vertebrae in front of lumbar soiling—maceration—infection—necrosis.
fascia, to achieve temporary benefit. Long time efficacy can be
achieved by using 5 mL phenol in water. It is done under C-Arm Staging of Pressure Sore
guidance. Feet will become warm immediately after injection.
♦♦ Non-blanching erythema—Early superficial ulcer
♦♦ Partial thickness skin loss—Late superficial ulcer
Surgery
♦♦ Full thickness skin loss extending into subcutaneous tissue
♦♦ Omentoplasty to revascularize the affected limb. but not through fascia—Early deep ulcer
♦♦ Profundaplasty is done for blockage in profunda femoris ♦♦ Full thickness skin loss with fascia and underlying struc-
artery so as to open more collaterals across the knee joint. tures like muscle, tendon, bone, etc—Late deep ulcer
Section 2:  General Surgery  339

Treatment Types of Gangrene


♦♦ Cause for pressure sore should be treated. Gangrene is of three types, i.e. dry, wet and gas gangrene.
♦♦ Adequate nutritional supplementation should be done. ♦♦ Dry gangrene: It is a condition which results when one or
♦♦ Proper rest, antibiotics, slough excision, regular dressings more arteries become obstructed. In this type of gangrene,
should be given. the tissue slowly dies because of inadequate or no blood
♦♦ Vacuum-assisted closure: It is the creation of intermittent supply. It happens mostly in the extremities and it devel-
negative pressure of minus 125 mm Hg to promote for- ops in people with diabetes or arteriosclerosis.
mation of healthy granulation tissue. Negative pressure ♦♦ Wet gangrene: It is a gangrene which develops as a compli-
reduces tissue edema, clears the interstitial fluid and im- cation of an untreated infected wound, caused by various
proves the perfusion, increases the cell proliferation and so bacterial infection. The tissue is infected by saprogenic
promotes the healing. A perforated drain is kept over the microorganisms, i.e. (C. perfringens, fusiformis, etc.) which
foam dressing covered over the pressure sore. It is sealed cause tissue to swell and emit a fetid smell.
with a transparent adhesive sheet. Drain is connected to ♦♦ Gas gangrene: It is is a bacterial infection that produces
required vacuum apparatus. gas within tissues. It is caused by bacterial exotoxins pro-
♦♦ Once ulcer granulates well, flap cover or skin grafting is duced by Clostridium perfringens which are mostly found in
done. soil and other anaerobes. Infection spreads rapidly as the
♦♦ Excision of the ulcer and skin grafting can also be done. gases produced by bacteria expand and infiltrate healthy
♦♦ Flaps—local rotation or other flaps (transposition flaps). tissue in the vicinity.
♦♦ Cultured muscle interposition should be done. ♦♦ Other gangrenes:
♦♦ Proper care should be given to the patient, i.e. • Noma or cancrum oris is a gangrene of the face.
♦♦ Change in position once in 2 hours; lifting the limb up- • Necrotizing fasciitis affects the deeper layers of the
wards for 10 seconds once in 10 minutes skin.
♦♦ Use of water bed/air bed/air-fluid floatation bed and pres- • Fournier gangrene usually affects the male genitals
sure dispersion cushions to the affected area. and groin.
♦♦ Urinary and fecal care
♦♦ Hygiene should be maintained Investigations
♦♦ Psychological counseling should be done.
♦♦ Regular skin observation; keeping skin clean and dry (using ♦♦ CBC, hemoglobin and blood sugar
regular use of talcum powder); oil massaging of the skin ♦♦ Arterial Doppler, angiogram
and soft tissues using clean, absorbent porous clothing; ♦♦ Ultrasonography of abdomen to find out status of aorta
control and prevention of sepsis helps in the management. Treatment of Gangrene
Q.3. Write short note on gangrene. Limb-Saving Method
 (Dec 2007, 3 Marks) (Apr 2007, 10 Marks) ♦♦ Drugs such as antibiotics, vasodilators, small dose of as
Ans. Gangrene is a form of necrosis of tissue with superadded pirin, praxilene are started.
putrefaction.
♦♦ Care of feet and toes: Part of the feet or toe should be kept dry.
Etiology • Any injury has to be avoided
♦♦ Trauma: Direct or indirect • Proper footwear is advised to patient
♦♦ Infection: Boil, carbuncle, gas gangrene, Fournier’s gan- • Pressure areas should be protected.
grene, cancrum oris • If pus is present, it has to be drained.
♦♦ Physical: Burns, scald, frostbite, chemicals, irradiation and • Limb should not be warmed.
electrical • Diabetes should be controlled.
♦♦ Venous gangrene • Surgeries such as omentoplasty, profundaplasty,
♦♦ Secondary to arterial occlusion like atherosclerosis, emboli, arterial bypass graft are done to improve limb
diabetes, Raynaud’s disease, ergot. perfusion.

Clinical Features Lifesaving Procedures


♦♦ In gangrene, the organ involved is pale, gray, purple, Amputations can be done as lifesaving procedure.
brownish black. This is due to disintegration of hemo- ♦♦ Below knee amputation is better option as fit of prosthesis
globin to sulfide is better and movements of knee joint are retained.
♦♦ Pulse is absent. ♦♦ In above knee amputation range of movement is less and
♦♦ There is loss of sensation and function third support is required to walk.
♦♦ There is presence of line of demarcation between vital
Q.4. Write short note on dry and wet gangrene.
and dead tissue. A band of hyperemia and hyperesthesia
is present along with developmental layer of granulation  (Jan 2012, 5 Marks)
tissue. Or
340   Mastering the BDS IIIrd Year  (Last 25 Years Solved Questions)

Write briefly on wet gangrene. (Jan 2016, 2 Marks) Gas Gangrene


Or ♦♦ Gas gangrene is a bacterial infection that produces gas
Write short note on dry wet and gas gangrene. within tissues. It is caused by bacterial exotoxins pro-
 (June 2010, 5 Marks) duced by Clostridium perfringens which are mostly found
Or in soil and other anaerobes (e.g. Bacteroides and anaerobic
Write short answer on dry, wet and gas gangrene. streptococci).
♦♦ Infection spreads rapidly as the gases produced by bacteria
 (June 2018, 3 Marks)
expand and infiltrate healthy tissue in the vicinity.
Ans.
♦♦ After an incubation period of one to four or five days,
Dry Gangrene the affected tissue is swollen, painful and cold. A watery,
brownish, foul-smelling fluid drains from the wound, and
♦♦ It is a condition which results when one or more arteries little bubbles of gas develop in the tissues.
become obstructed. ♦♦ Gas gangrene can cause necrosis, gas production, and sep-
♦♦ In this gangrene, the tissue slowly dies because of inad- sis. Progression to toxaemia and shock is often very rapid.
equate or no blood supply. ♦♦ Gas gangrene is often treated with the antitoxin for
♦♦ It happens mostly in the extremities and it occurs in people Clostridium. In a number of cases. amputation may have
with diabetes or arteriosclerosis. It can also develop after to be used to keep the infection under control.
prolonged exposure to freezing temperatures. ♦♦ Severe cases have been treated by keeping the patient in
♦♦ In dry gangrene tissue first becomes bluish and patient feel an oxygen-rich atmosphere, as in a hyperbaric chamber.
cold to the touch, As time progresses, a line of demarcation
appears between the healthy and devitalized tissue, which Q.5. Enumerate differences between dry and wet gangrene.
becomes dry and dark black.  (June 2014, 2 Marks)
♦♦ Eventually, there may be a separation of the dead tissue Ans.
from the living tissue; with spontaneous amputation (au-
Features Dry gangrene Wet gangrene
toamputation) of the involved extremity.
♦♦ Treatment of this type of gangrene is aimed at improving Cause Slow occlusion of arteries Sudden occlusion of
circulation, revascularization (i.e. restoration of blood arteries
to flow) to the affected area. This may be accomplished Involvement Small area is gangrenous Large area is affected
with drugs or through the surgical removal of the ob- of part due to presence of due to absence of
struction. collaterals collaterals
Local Dry, shrivelled and Wet, turgid, swollen,
Wet Gangrene findings mummified edematous
♦♦ Wet gangrene is gangrene which develops as a complica- Line of Present Absent
tion of an untreated infected wound, caused by certain demarcation
bacterial infection. Crepitus Absent May be present
♦♦ The tissue is infected by saprogenic microorganisms (C.
Odor Absent Foul odor
perfringens, fusiformis, etc.) which cause tissue to swell and
produces a fetid smell. Infection Not present Present
♦♦ This gangrene develops rapidly due to blockage of venous Diseases TAD: Atherosclerosis Emboli, crush injuries,
and/or arterial blood flow. etc
♦♦ In this affected part is saturated with stagnant blood, which Treatment Conservative amputation Major amputation is
promotes the rapid growth of bacteria. necessary

♦♦ Toxic products formed by bacteria are absorbed causing
systemic manifestation of septicemia and finally death. Q.6. Define gangrene. Describe various causes of gangrene
♦♦ Wet gangrene occurs in naturally moist tissue and organs and management of diabetic gangrene foot.
such as the mouth, bowel, lungs, cervix, and vulva.  (Nov 2008, 15 Marks)
♦♦ Bedsores occurring on body parts such as the sacrum, Ans. Gangrene is the death of body tissue associated with loss
buttocks, and heels—although not necessarily moist areas of vascular supply and followed by bacterial invasion
—are also categorized as wet gangrene infections. and putrefaction.
♦♦ The affected part is edematous, soft, putrid, rotten and
Causes
dark. The darkness in wet gangrene occurs due to the same
mechanism as in dry gangrene. ♦♦ Secondary to arterial occlusion like atherosclerosis, emboli,
♦♦ Administration of antibiotics and sometimes the surgi- diabetes, raynaud’s disease.
cal removal of the dead tissue to keep the infection from ♦♦ Infective: Boil, carbuncle, gas gangrene, Fournier’s gan-
spreading is the treatment. grene, cancrum oris
Section 2:  General Surgery  341

♦♦ Traumatic: Either direct trauma or indirect trauma motor neuropathy, dysfunction of muscles, arches of foot
♦♦ Physical: Burns, scalds, frostbite, chemicals, irradiation, and joints occur. And loss of reflexes of foot occurs caus-
electrical ing more prone to trauma and abscess. Due to autonomic
♦♦ Venous gangrene. neuropathy, skin will be dry causing defective skin barrier
and so more prone for infection.
Management of Diabetic Gangrene Foot
♦♦ Diabetic atherosclerosis: It itself reduces the blood supply
Foot can only be saved if good blood supply is present. and causes gangrene. Thrombosis can be precipitated by
♦♦ Antibiotics should be started based on culture and sen- infection causing infective gangrene. Blockage occurs at
sitivity test. plantar, tibial and dorsalis pedis vessels.
♦♦ Regular dressings should be given to patient ♦♦ Increased glycosylated hemoglobin in blood causes de-
♦♦ Drugs such as vasodilators, pentoxifylline, dipyridamole fective oxygen dissociation leading to more hypoxia. At
and low dose aspirin is given. tissue level, there will be increased glycosylated tissue
♦♦ Diabetes is controlled by insulin proteins, which prevents proper oxygen utilization and
♦♦ Diet should be controlled so aggravates hypoxia.
♦♦ Surgical debridement of the wound is carried out.
Clinical Features
Lifesaving Procedures
♦♦ Pain in the foot
Amputations of diabetic foot should be done occasionally. ♦♦ Ulceration
♦♦ Level of amputation is decided based on skin changes, ♦♦ Absence of sensation
temperature, line of demarcation and Doppler study. ♦♦ Absence of pulsations in the foot (Posterior tibia and dor-
♦♦ Below knee amputation is done, in this BK prosthesis is fit- salis pedis arteries).
ted better and also the movements of knee joint are retained. ♦♦ Loss of joint movements.
♦♦ In above knee amputation range of movements are less, ♦♦ Abscess formation
limb is present and require third support to walk. ♦♦ Changes in temperature and color when gangrene sets in.
♦♦ Patient may succumb to ketoacidosis, septicemia or
Care of Feet and Toes myocardial infarction.
♦♦ Part of the foot should be kept dry.
Investigations
♦♦ Injury to the foot and toes is avoided
♦♦ Proper footwear is advised to patient, i.e. microcellular ♦♦ Blood sugar, urine ketone bodies.
rubber footwear ♦♦ Blood urea and serum creatinine.
♦♦ Measures for pain relief are to be taken ♦♦ X-ray of a part to look for osteomyelitis.
♦♦ Nutritional supplementation is given ♦♦ Pus for culture and sensitivity.
♦♦ Hyperkeratosis is avoided ♦♦ Doppler study of lower limb to assess arterial patency.
♦♦ Localized pus should be drained ♦♦ Angiogram to look for proximal blockage.
♦♦ Limb should not be warmed. ♦♦ Ultrasound of abdomen to see the status of abdominal
aorta.
Q.7. Write short note on diabetic foot. (Jan 2018, 5 Marks)
♦♦ Glycosylated hemoglobin estimation.
Ans. Diabetic foot consists of callosities, ulceration, abscess,
cellulitis of foot, osteomyelitis of different bones, diabetic
gangrene and arthritis of joints. Treatment
♦♦ Foot can be saved if only there is good blood supply:
Meggitt's Classification of Diabetic Foot
• Antibiotics—decided by pus culture and sensitivity
♦♦ Grade 0: Foot symptoms like pain, only • Regular dressing
♦♦ Grade l: Superficial ulcers • Drugs: Vasodilators, pentoxifylline, dipyridamole,
♦♦ Grade 2: Deep ulcers low dose aspirin.
♦♦ Grade 3: Ulcer with bone involvement • Diabetes is controlled by insulin only.
♦♦ Grade 4: Forefoot gangrene • Diet control, control of obesity.
♦♦ Grade 5: Full foot gangrene. • Surgical debridement of wound.
• Amputations of the gangrenous area. Level of
Pathogenesis of Diabetic Foot amputation has to be decided by skin changes and
♦♦ High glucose level in tissues is a good culture media for temperature changes or Doppler study.
bacteria. So infection is common. • Care of feet in diabetic:
♦♦ Diabetic microangiopathy causes blockade of microcircula- ♦♦ Any injury has to be avoided.
tion leading to hypoxia. ♦♦ MCR footwears must be used (Microcellular rubber)
♦♦ Diabetic neuropathy: Due to sensory neuropathy, minor ♦♦ Feet has to be kept clean and dry, especially toes and clefts
injuries are not noticed and so infection occurs. Due to ♦♦ Hyperkeratosis should be avoided.
342   Mastering the BDS IIIrd Year  (Last 25 Years Solved Questions)

11. Venous Diseases day they are sufficiently large because of the venous en-
gorgement.
♦♦ Dragging pain in the leg or dull ache is due to heaviness.
Q.1. Write short note on complications of varicose veins ♦♦ Night cramps occur due to change in the diameter of
with reasons. (Mar 2008, 3 Marks) veins.
Ans. Varicose veins are dilated and tortuous, superficial veins ♦♦ Aching pain is relieved at night on taking rest or elevation
predominantly in lower limbs. Varicose veins are the of limbs.
price that man has to pay for erect posture. ♦♦ Sudden pain in the calf region with fever and edema of the
ankle region suggests deep vein thrombosis.
Complications ♦♦ Patients can present with ulceration, eczema, dermatitis,
bleeding, etc. Symptoms of pruritis/itching and skin
♦♦ Eczema and dermatitis: This occurs because of extravasa-
thickening are also seen.
tion and break down of RBCs in the lower leg. It leads to
itching which precipitates varicose ulcer. This condition Signs
is treated by application of zinc oxide cream or silver sul-
♦♦ Dilated veins are present in the medial aspect of leg and
fadiazine cream (stasis dermatitis).
the knee. Sometimes they, are visible in the thigh also.
♦♦ Lipodermatosclerosis: It refers to various skin changes in
♦♦ Single dilated varix at SF junction is called saphena varix.
the lower leg associated with varicose veins such as thick-
It is due to saccular dilatation of the upper end of long
ening of subcutaneous tissue, indurated feel like wood,
saphenous vein at the saphenous opening.
pigmentation, etc. It occurs because of increased venous
♦♦ Veins are tortuous and dilated
pressure resulting in capillary leakage with extravasation
♦♦ Ankle flare is a group of veins nearer the medial
of blood and fibrin into surrounding tissues. Blood is malleolus.
broken down and heme is released. This combines with
iron giving rise to hemosiderin which is responsible for
pigmentation. Classically, this affects gaiter area of leg just
above the malleoli.
♦♦ Hemorrhage: It occurs due to trauma or eczema. This is
controlled by elevation of the leg and crepe bandage.
♦♦ Thrombophlebitis: It refers to the inflammation of su-
perficial vein. In this vein is tender, hard and cord like.
Here skin is inflamed and pyrexia is usually present. It is
treated by bed rest, elevation, crepe bandage, antibiotics
and anti-inflammatory drugs.
♦♦ Venous ulcer: It is also called gravitational ulcer. Precipi-
tating factors are venous stasis and tissue anoxia. Deep
vein thrombosis is also an important cause of venous ulcer
where in valves is either destroyed or incompetent due to
damage. Sustained venous pressure results in extravasa-
tion of cells, activation of capillary endothelium resulting
in release of free radicals. These free radicals cause tissue
destruction and ulceration.
♦♦ Calcification: This is seen inside the walls of vein. Fig. 12:  Varicose vein of lower leg
♦♦ Periostitis: It may occur in tibia because of the ulcer pre-
Diagnosis
sent on the medial surface of leg. Due to involvement of
periosteum ulcer leads to severe pain. ♦♦ Venous Doppler: With the patient standing, Doppler probe is
♦♦ Equinovarus deformity: It occurs due to improper habit placed at saphenofemoral junction and later wherever re-
of walking on toes which leads to shortening of tendo quired. Basiaally by hearing the changes in sound, venous
Achilles. flow, venous patency, venous reflux can be well identified.
♦♦ Marjolin’s ulcer: It is the squamous cell carcinoma which ♦♦ Duplex scan: It is a ultrasonographic Doppler imaging
arises from healed varicose ulcer with scarring. technique which along with direct visualization of veins,
Q.2. Write short note on varicose veins of lower leg. gives functional or anatomical information and also the
(Sep 2008, 3 Marks) color map.
Ans. Dilated, tortuous and elongated superficial veins of limb ♦♦ Venography: Before introduction of venous Doppler, venog-
are called as varicose veins. raphy is the common method of diagnosis.
♦♦ Plethysmography: This is a non-invasive method which
Symptoms measures the volume change in leg.
♦♦ Majority of the patients come, with dilated veins in the ♦♦ Arm foot venous pressure: Foot pressure is not more than 4
leg. They are minimal to start with and at the end of the mm Hg above arm pressure.
Section 2:  General Surgery  343

♦♦ Varicography: In this non-ionic, iso-osmolar, non-throm- Treatment


bogenic contrast is injected directly in variceal vein to get ♦♦ Elevation of the limb.
detailed anatomical mapping of varicose veins. This is used ♦♦ Anti-inflammatory drugs and antibiotics should be given
in recurrent varicose veins. ♦♦ Application of crepe bandage—Compression therapy
Surgical Treatment ♦♦ Anticoagulation: Low molecular weight heparin for super-
ficial venous thrombosis >3 cm in length.
♦♦ Trendelenburg’s operation:
• An inguinal incision is made, long saphenous vein
identified and the 3 tributaries are ligated.
• Long saphenous vein is ligated close to the femoral 12. Cyst
vein juxta femoral flush ligation.
• An incision is given in front of the medial malleolus Q.1. Write short note on sebaceous cyst.
and long saphenous is isolated.  (Feb 2013, 5 Marks) (Aug 2012, 5 Marks)
• The lower end is ligated and the vein incised.  (Sep 2009, 5 Marks) (Nov 2008, 5 Marks)
• A long metallic stripper is introduced within the vein Or
and brought out from the long saphenous vein in the Write short answer on sebaceous cyst.
inguinal incision.  (Apr 2018, 5 Marks)
• A metallic head is connected to the stripper and the Ans. Sebaceous cyst is also called as epidermoid cyst.
vein is avulsed. • This occurs due to obstruction of sebaceous ducts,
• Tight crepe bandage is applied, inguinal incision is resulting in accumulation of sebaceous material.
sutured and the limb is elevated. • The sebaceous material becomes enlarged due to
♦♦ Subfascial ligation of Cockett and Dodd: retention of its sebum. So it is also called retention
• In this operation, perforators are identified deep to cyst.
deep fascia and they are ligated subfascially. This is • The cyst is lined by squamous epithelium material
indicated in cases of perforator incompetence with with an unpleasant smell.
saphenofemoral competence. This is also done by an • In the center of cyst a black spot is found, it is keratin
endoscope. filled punctum.
Q.3. Write short note on thrombophlebitis. • Site: Scalp, face, back, scrotum, etc. The sebaceous
 (May/Jun 2009, 5 Marks) cyst may occur anywhere in the body where there
Ans. Thrombophlebitis is a superficial vein thrombosis with are sebaceous glands.
inflammation.
Clinical Features
Types ♦♦ It is a painless swelling which is smooth, soft, non-tender,
♦♦ Acute: It is caused due to IV cannulation, trauma, minor freely mobile, adherent to skin specially over the summit,
infections and hypercoagulability. fluctuant, non-transilluminating with punctum over the
♦♦ Recurrent summit.
♦♦ Spontaneous: Polycythemia vera, polyarteritis, Buerger’s ♦♦ Cyst moulds on finger indentation.
disease ♦♦ Punctum is present over the summit in 70% of cases be-
♦♦ Thrombophlebitis migrans: It is spontaneous migrating cause sebaceous duct opens directly in skin which gets
thrombophlebitis seen in visceral malignancy like pan- blocked. Punctum is the black colored spot over summit
creas, stomach. of sebaceous cyst.
♦♦ Mondor’s disease ♦♦ Hair loss over the surface is common because of constant
pressure over the roots of hair follicle.
Clinical Features ♦♦ Unpleasant odor of sebum content is typical.
♦♦ Pain, redness, tenderness, cord like thickening of veins
♦♦ Fever is present
♦♦ It is seen in upper limb or lower limb.
Investigations
Duplex ultrasound Doppler of both limbs is must.
Complications
♦♦ Destruction of venous valves lead to varicose veins
♦♦ Embolism, infection. Fig. 13:  Sebaceous cyst
344   Mastering the BDS IIIrd Year  (Last 25 Years Solved Questions)

Complication ♦♦ Swelling may often attain large size presenting both


sublingually, intraorally and midline submentally on
♦♦ Infection and abscess formation
external side.
♦♦ Surface get ulcerated leading to formation of painful
♦♦ Occasionally it can lead to trismus, dysphagia, pain and
fungating mass with discharge known as Cock’s peculiar
odynophagia.
tumor which resembles epithelioma. Cock’s peculiar tu-
mor is not a tumor but a chronic granuloma on ulcerated
surface of sebaceous cyst.
♦♦ Sebaceous horn results from hardening of slowly dis-
charged sebum through the punctum.

Treatment
♦♦ Excision including skin adjacent to punctum using ellipti-
cal incision also known as dissection method.
♦♦ Incision and avulsion of cystic wall.
♦♦ If abscess is formed, then drainage initially and later exci- Fig. 14:  Sublingual dermoid cyst
sion is done.
♦♦ If capsule is not removed properly cyst will recur. Differential Diagnosis
Q.2. Describe briefly sublingual dermoid cyst. ♦♦ Ranula: When sublingual dermoid cyst is in midline at
 (Sep 2008, 3 Marks) floor of the mouth and above mylohyoid muscle ranula
Ans. It is a type of congenital sequestration dermoid cyst. is considered as differential diagnosis. Ranula is blue in
The cyst is formed by inclusion of the surface ectoderm color and is brilliantly translucent.
at the fusion line of two mandibular arches. ♦♦ Thyroglossal cyst: It is to be taken in differential diagnosis
when sublingual dermoid cyst is below mylohyoid mus-
Pathology cle. Thyroglossal cyst moves up with deglutition whereas
♦♦ The cyst is lined by squamous epithelium. sublingual dermoid cyst does not.
♦♦ The wall of cyst contains hair follicle, sweat and sebaceous
Treatment
glands.
♦♦ Cyst contains the cheesy material. Excision is done through intra-oral approach usually; large cyst ex-
♦♦ It never contains hair. tending under geniohyoid muscle may require external approach.
Types Q.3. Write short note on thyroglossal cyst. 
 (Feb 2013, 3 Marks) (Jan 2012, 5 Marks)
It may be:  (Dec 2010, 5 Marks) (June 2010, 5 Marks)
♦♦ Median variety: It is derived from epithelial cell rests at  (Mar 1998, 6 Marks)
the level of fusion of two mandibular arches. It may be Or
supramylohyoid or inframylohyoid. It is located between Describe briefly thyroglossal cyst. 
two genial muscles, in relation to mylohyoid muscle. It  (Nov 2008, 5 Marks)
is a midline swelling which is smooth, soft, cystic, non- Or
transilluminant. Write brief note on thyroglossal cyst. 
♦♦ Lateral variety: It develops in relation to submandibular  (Apr 2017, 2 Marks)
duct, lingual nerve and stylohyoid ligament. It is derived Or
from first branchial arch. It forms a swelling in the lateral Write in short about thyroglossal cyst. 
aspect of the floor of mouth.  (Jan 2018, 5 Marks)
It also may be: Or
♦♦ Supra mylohyoid type. Write about thyroglossal cyst.  (June 2018, 5 Marks)
♦♦ Infra mylohyoid type. Ans. It is a congenital tubular dermoid cyst.
Clinical Features • It arises from thyroglossal duct, which extends from
foramen cecum at the base of tongue to the isthmus
♦♦ It occurs in young children between the ages of 10 to 12 of the thyroid gland.
years. • It is lined by pseudostratified, ciliated and columnar
♦♦ Congenital, painless and bidigitally palpable swelling or squamous epithelium which produces desqua­
in the floor of mouth. mated epithelial cells or mucus at times.
♦♦ Swelling is soft and cystic
♦♦ Fluctuation test is positive. Sites
♦♦ Transillumination test is negative as it contains thick, ♦♦ Subhyoid is the most common site
cheesy, sebaceous material. ♦♦ At the level of thyroid cartilage
Section 2:  General Surgery  345

♦♦ Suprahyoid: Double chin appears ♦♦ Cyst with surrounding tissues is dissected up to the hyoid
♦♦ At the foramen caecum bone. Sternohyoid and thyrohyoid muscles are divided.
♦♦ At the level of cricoid cartilage ♦♦ Central part of the hyoid bone of l cm width is resected
♦♦ At the floor of the mouth. along with intact tract within it.
♦♦ Geniohyoid and mylohyoid muscles are divided off from
Clinical Features the hyoid.
♦♦ Swelling is present in midline towards the left side. ♦♦ Track with adjacent tissues is dissected above up to the
♦♦ Moves with deglutition as well as with the protrusion of foramen cecum.
tongue. ♦♦ Adjacent tissues also should be removed because of possi-
♦♦ Patient is asked to open the mouth and keep the lower jaw bility of multiple tracts which otherwise lead to recurrence
still. Examiner holds the cyst between the thumb and fore- or fistula formation.
finger. When the patient is asked to protrude the tongue, ♦♦ After this, anesthetist is asked to apply digital pressure over
a “tugging sensation” can be felt. the base of tongue near foramen cecum to facilitate the dis-
♦♦ Swelling is smooth, soft, fluctuant (cystic), non-tender, section and to confirm the reach up to the foramen cecum.
mobile, often transilluminant. ♦♦ Track is ligated at foramen caecum and removed.
♦♦ Thyroid fossa is empty if there is no thyroid in normal ♦♦ If there is no normal thyroid gland after the surgery main-
location. tenance dose of L-thyroxine 0.l mg OD is given lifelong.
♦♦ Thyroglossal cyst can get infected and may form an ab-
Complications
scess. Cyst wall contains lymphatic tissue and so infection
is common. ♦♦ Recurrent thyroglossal fistula formation
♦♦ Malignancy can develop in papillary carcinoma. ♦♦ Hemorrhage/hematoma formation
♦♦ Infection.
Q.4. Write short note on cystic hygroma.
 (June 2010, 5 Marks) (Nov 2008, 5 Marks)
 (Sep 2006, 5 Marks) (Mar 2006, 5 Marks)
Or
Describe briefly on cystic hygroma.
(Apr 2017, 2 Marks)
Or
Fig. 15:  Thyroglossal cyst
Write short answer on cystic hygroma.
 (June 2018, 3 Marks)
Differential Diagnosis Ans. It is also called as lymphangioma of neck or cavernous
♦♦ Subhyoid bursa lymphangioma or hydrocele of neck.
♦♦ Pretracheal lymph node It is a congenital cystic swelling, which contains multiple
♦♦ Dermoid cyst lobules of clear lymph.
♦♦ Solitary nodule of thyroid • It gets filled up with lymph in early week of
♦♦ Submental lymph node childhood and present as a large cyst in the lower
♦♦ Collar stud abscess. part of the neck.
Investigations • The cyst is not a single cavity but it is a collection of
numerous small cysts.
♦♦ Radioisotope study • Their lobules may intercommunicate with one
♦♦ Ultrasound of neck; T3, T4 and TSH estimation another.
♦♦ FNAC from the cyst. • Each lobule is lined by a single layer of endothelium.
• Cystic hygroma may infiltrate into the muscular
Treatment
plane.
Sistrunk Operation
Clinical Features
Excision of cyst and also full tract up to the foramen cecum is
♦♦ Swelling is present at birth in the posterior triangle of neck
done along with removal of central part of the hyoid bone as
causing obstructed labor.
the tract passes through it.
♦♦ Swelling is smooth, soft, fluctuant (cystic), partially com-
pressible, brilliantly transilluminant. It is not reducible
Technique
completely.
♦♦ Thorough transverse neck incision placed over the cyst, ♦♦ During crying swelling often increases in size.
skin flap is raised above along with platysma. Care should ♦♦ Disfigurement of face of child which is more worrying
be taken not to open the cyst. factor for the parents.
346   Mastering the BDS IIIrd Year  (Last 25 Years Solved Questions)

♦♦ Swelling may rapidly increase in size causing respiratory Types


obstruction which is a dangerous sign.
♦♦ It may get infected forming an abscess which is a tender, There are two types of ranulas, i.e.
warm, soft swelling. It may cause septicemia which may 1. Oral ranula
be life threatening. 2. Plunging ranula or cervical ranula.
♦♦ Rupture with lymph ooze can occur. Oral ranula occur secondary to mucus extravasation where
mucus pool superior to mylohyoid muscle which in plunging
Complication ranula mucus extravasation is along the facial planes of neck.
♦♦ Too much enlargement of cystic hygroma may cause
respiratory distress. Clinical Features
♦♦ Infection, abscess, septicemia
Oral Ranula
♦♦ Surgery itself may cause torrential hemorrhage
♦♦ Chylous fistula, chylothorax ♦♦ Seen in young children and adults
♦♦ Recurrence of cyst in 15% of cases. ♦♦ Swelling is typically located in the floor of the mouth to
one side of the midline.
Treatment ♦♦ Its surface is smooth with diffuse borders.
♦♦ Aspiration of the cyst is done. Later on sac and capsule is ♦♦ On palpation, it is soft, cystic, nontender and fluctuant
thickened by fibrous tissue and is excised. swelling which gives brilliant transillumination.
♦♦ Meticulous dissection is done across all planes including ♦♦ It is covered by thin mucosa containing clear, serous fluid.
deeper muscular one to clear entire cyst wall. If it is not Hence, it is bluish in color and resembles like a belly of frog.
done properly chances of recurrence are present. ♦♦ The lesion can cross midline when it is large, this makes
♦♦ If respiratory obstruction is present then tracheostomy offending salivary gland difficult to locate.
is done. ♦♦ Large oral ranulas displace the tongue and interfere with
♦♦ Under antibiotic coverage drainage of abscess is done and functions of tongue.
later on sac is excised. ♦♦ If the mass is located deep, it looses its bluish translucent
♦♦ Preoperative injection of sclerosants is given and later on color.
fibrosis develops then excision of entire aggregation of
cyst is done. In past days boiling water injection is given Plunging Ranula
at 7 days interval.
♦♦ Ranula often extends in submandibular region through
Q.5. Write briefly on ranula.
deep part of posterior margin of mylohyoid muscle and
 (Jan 2011, 5 Marks) (Dec 2010, 3 Marks)
is known as plunging ranula.
Or ♦♦ Plunging ranula is also an intra-oral ranula but with cervi-
Write short note on ranula. cal extension.
 (Feb 2013, 5 Marks) (Aug 2012, 5 Marks) ♦♦ It remains asymptomatic but enlarges continuously. Over-
 (Dec 2010, 5 Marks) (Dec 2009, 5 Marks) lying skin remains intact.
 (Apr 2008, 5 Marks) (Sep 2006, 5 Marks) ♦♦ The swelling is fluctuant, freely mobile and is non-tender.
 (Sep 2002, 5 Marks) (Mar 2000, 10 Marks) ♦♦ It is bidigitally palpable.
 (Apr 2017, 4 Marks) (Mar 2006, 5 Marks)
Or Investigations
Write in short about ranula.
 (Jan 2018, 5 Marks) Ultrasound of neck or MRI neck should be done.
Or
Treatment
Write short answer on ranula. (Apr 2018, 3 Marks)
Ans. Ranula is a cystic swelling occurs in the floor of the mouth Oral ranula
and involves mainly sublingual salivary glands duct.
• It also arises from accessory salivary glands. ♦♦ Marsupialization can be done initially, in marsupialization
• The word ranula is derived from the resemblance of major part of the cyst along with mucus membrane of floor
the swellings to the belly of frog. of mouth is excised. Cut edges of cyst wall are sutured to
cut mucus membrane and later as the wall of ranula is
Etiology thickened it should be excised completely.
♦♦ If ranula is small, it can be excised without marsupializa-
♦♦ Ranula occurs due to obstruction to the duct, secreting
tion.
mucosa.
♦♦ Laser ablation and cryosurgery alone or after completion
♦♦ Artesia (Obstruction of duct)
of marsupialization can be done for some patients of oral
♦♦ *Stricture of duct due to surgery
ranula.
♦♦ Perforation of duct.
Section 2:  General Surgery  347

Plunging ranula ♦♦ Phase of enlargement: Once a small cyst is formed, it


♦♦ Plunging ranula often requires approach from neck for enlarges gradually by the following mechanisms:
complete excision. Excision of submandibular and sub- • Higher osmotic tension of the cystic fluid causes
lingual salivary gland is often needed in plunging ranula. progressive increase in the amount of fluid inside its
♦♦ For small plunging ranula excision is done orally along lumen and this causes increased internal hydrostatic
with excision of sublingual salivary gland. tension within the cyst. The process results in cyst
expansion due to resorption of the surrounding
Complications bone.
• The epithelial cells of the cystic lining release some
Rupture for the cyst decreases the size but it can appear at a
bone resorbing factors like prostaglandins and
later date.
collagenase, etc. which destroy the bone and facilitate
When the swelling is big, the tongue is pushed upwards and expansion of the cyst.
may cause difficulty in speech and swallowing.
Q.6. Write short note on dental cyst. (Sep 2000, 10 Marks) Clinical Features
Or
Write in brief about dental cyst. (July 2016, 5 Marks) ♦♦ It is common in woman around third and fourth decade.
Ans. Dental cyst is also called as radicular cyst or periapical ♦♦ Upper anterior teeth are more affected
cyst. ♦♦ The involved tooth shows presence of caries, fracture or
discoloration.
It is the most common type of inflammatory cystic lesion,
♦♦ Slow enlarging bony hard swelling of jaw with expansion
which occurs in relation to the apex of non-vital tooth.
and distortion of cortical plates.
In this case, if the involved tooth is extracted the ♦♦ Cyst remain asymptomatic if uninfected
remaining cystic cavity within the bone is known as ♦♦ Severe bone destruction by the cystic lesion may produce
residual cyst. springiness.
♦♦ If the cyst is secondary infected, it leads to the formation
Pathogenesis
of abscess then it is called a cyst abscess.
The radicular cyst develops due to the proliferation and
subsequent cystic degeneration of the “epithelial cell rests of Radiological Features
Malassez”, in the periapical region of a non-vital tooth.
♦♦ It appears as a rounded or pear-shaped radiolucency at the
The process of development of this cyst occurs in various apex of nonsensitive tooth or with nonvital tooth.
stages: ♦♦ Radiolucency is more than 1.5 cm in diameter but usually
♦♦ Phase of initiation. less than 3 cm in diameter. It has got well-defined outline
♦♦ Phase of proliferation. with thin hyperostotic borders.
♦♦ Phase of cystification. ♦♦ Margins: In uncomplicated cases margins are smooth, cor-
♦♦ Phase of enlargement. ticated and cortex is usually well-defined, well-etched and
continuous, except in some cases, there may be window
♦♦ Phase of initiation: During this phase, the bacterial infec-
formation. There is also thin white line surrounding the
tion of the dental pulp or direct inflammatory effect of ne-
margins of bone cavity. This thin layer of cortical bone is
crotic pulpal tissue, in a non-vital tooth causes stimulation
almost always present unless suppuration supervenes in
of the “cell rest of Malassez” which are present within the
the cyst.
bone near the root apex of teeth.
♦♦ Image of radiopaque borders is continuous with lamina
♦♦ Phase of proliferation: The stimulation to the cell rests
dura around the associated tooth. Infection may cause the
of Malassez leads to excessive proliferation of these cells,
borders to become less distinct.
which leads to the formation of a large mass or island of
♦♦ Radicular cysts of long duration may cause resorption
immature proliferating epithelial cells at the periapical of roots.
region of the affected tooth. ♦♦ Adjacent teeth are usually displaced and rarely resorbed.
♦♦ Phase of cystification: Once a large bulk of the cell rest There is also buccal expansion and involves maxillary area
of Malassez is produced, its peripheral cells get adequate than displacement of antrum occurs.
nutritional supply but its centrally located cells are often
deprived of proper nutritional supply. As a result the cen- Differential Diagnosis
tral group of cells undergo ischemic liquefactive necrosis
while the peripheral group of cells survive. This eventu- ♦♦ Periapical granuloma: If radiolucency which appear on the
ally gives rise to the formation of a cavity that contains a radiograph is smaller than 1.5 cm, it is considered to be
hollow space or lumen inside the mass of the proliferating periapical granuloma.
cell rest of Malassez and a peripheral lining of epithelial ♦♦ Periapical scar: It is ruled out on basis of history and
cells around it. location.
348   Mastering the BDS IIIrd Year  (Last 25 Years Solved Questions)

♦♦ Lateral periodontal cyst: Radicular cyst originates from ♦♦ Blue domed cyst: When it contains blood then it is called as
maxillary lateral incisor and is positioned in between blue domed cyst.
root of lateral incisor and canine and is confused with the ♦♦ Dentigerous cyst has potential to become an aggressive
lateral periodontal cyst. In this case tooth vitality should lesion with expansion of bone and subsequent facial
be checked, tooth associated with lateral periodontal cyst asymmetry.
is vital and with radicular cyst is nonvital. ♦♦ There is extreme displacement of teeth, severe root resorp-
♦♦ Periapical cementoma: In case of radicular cyst tooth is non tion of adjacent teeth and pain.
vital while in case of periapical cementoma tooth is vital.

Treatment
♦♦ Nonvital teeth are associated with the cyst can either be
extracted or they can be retained by endodontic treatment,
i.e. apicoectomy.
♦♦ External sinus tracts should always be excised to prevent
epithelial ingrowth.
♦♦ Commonly employed surgical procedure for radicular
cyst is enucleation.
♦♦ Very small cysts can be removed through tooth socket.
♦♦ Large cysts that encroach upon maxillary sinus or
inferior alveolar neurovascular bundle may be treated by
marsupialization.
Fig. 16:  Dentigerous cyst
Q.7. Write short note on dentigerous cyst.
 (Feb 2013, 5 Marks) (Apr 2008, 5 Marks)
Radiographic Features
(Sep 2008, 5 Marks) (Nov 2008, 5 Marks)
(Sep 2001, 10 Marks) (Mar 2000, 10 Marks) ♦♦ It is a well-defined radiolucency usually associated with
hyperostotic borders unless it is secondarily infected.
 (Mar 2006, 5 Marks)
♦♦ Bony margins are well-defined and sharp.
Or ♦♦ It may involve the crown symmetrically; it may expand

Write brief note on dentigerous cyst. from the crown.
 (Apr 2017, 2 Marks) ♦♦ Large cysts are confined to mandible. There may be re-
Or sorption of roots.

Write in short about dentigerous cyst. Differential Diagnosis


 (Jan 2018, 5 Marks) ♦♦ Ameloblastoma or ameloblastic fibroma: They are multilocular
Ans. It is also known as follicular odontoma. and not associated with crown of an unerupted teeth.
It is a common odontogenic cyst of epithelial origin, ♦♦ Adenomatoid odontogenic tumor: They are rare and occur in
which encloses the crown of an impacted tooth at its maxillary anterior region.
neck portion. ♦♦ Calcifying odontogenic cyst: It occurs as pericoronal radio-
The cyst is lined by squamous epithelium, surrounded lucency and contains evidences of calcification.
by connective tissue. ♦♦ Developmental primordial and follicular primordial cyst: It
occurs in the crown of unerupted tooth and superimposition
Within the cyst, the tooth lies obliquely or sometimes
of image which may cause cyst like radiolucency to appear
embedded in the wall of the cyst.
as dentigerous cyst on radiograph.
As cyst grows, it displace the teeth deeper and deeper
and prevent from eruption. Treatment
Clinical Features ♦♦ Treatment via an intraoral approach or extraoral is decided
♦♦ Age and sex: It is usually found in children, equal in both by the size of cyst, adequate access and whether it desirable
the sex. to save the involved tooth.
♦♦ Site: Most common site is mandibular third molar and ♦♦ Marsupialization: It is indicated in children if the cyst is
maxillary canine which are most commonly impacted. very large in the size and involved tooth/teeth are to be
♦♦ Symptoms: Cyst remains asymptomatic, if uninfected. On maintained. Tooth may erupt in occlusion as defect heals
infection inflammatory signs are present. with normal bone or orthodontic forces may be used to
♦♦ Expansion of mandible: Since the inner table of mandible is bring tooth in occlusion.
strong the expansion mainly occurs in the outer aspect of ♦♦ Enucleation: Alternatively cyst can be enucleated together
the mandible. The bone gets thinned out resulting in egg with involved tooth in adults as possibility of tooth
shell cracking. eruption is low.
Section 2:  General Surgery  349

Q.8. Describe the cysts of jaw and their manage­ment. Management of Cysts
(Oct 2007, 10 Marks) (Sep 2005, 10 Marks) It is also known as partsch operation.
Ans. Cyst is defined as “A pathological cavity having fluid,
semifluid or gaseous contents and which is not created Principle
by accumulation of pus.” Kramer (1974) ♦♦ Marsupialization or decompression, refers to creating a
surgical window in the wall of the cyst and evaluation of
Classification of Cyst of Jaw by Mervin Shear the cystic contents.
♦♦ Cysts of the Jaws: ♦♦ This process decreases intracystic pressure and promotes
• Epithelial: shrinkage of the cyst and bone fill.
–– Developmental:
- Odontogenic Method
* Gingival cyst of infants ♦♦ Area is anesthetized with local anesthesia.
* Odontogenic keratocyst (neoplasm) ♦♦ Incision should be long enough to provide good exposure
* Dentigerous cyst (circular/oval).
* Eruption cyst ♦♦ In edentulous patient, incision is given along the crest of
* Lateral periodontal cyst ridge and in dentulous patient, the incision is given around
* Gingival cyst of adults the neck of teeth.
* Botryoid odontogenic cyst ♦♦ Incision is given bucally or lingually depending on loca-
* Glandular odontogenic cyst tion of cyst.
* Calcifying odontogenic cyst (neoplasm). ♦♦ Mucoperiosteal flap is raised.
- Non-odontogenic ♦♦ The character of underlying bone is determined. If this
* Naso palatine duct cyst layer of bone is present on cyst, it is carefully peeled off
* Naso labial cyst with periosteal elevator. If the bone over cyst is intact, a
* Midpalatal raphe cyst of infants window is made with bur/chisel in postage stamp method.
* Median palatine, median alveolar ♦♦ Window is enlarged with rongeurs.
* Median mandibular cyst ♦♦ An incision shaped like St Andrew and cross is made on
* Globulo maxillary cyst. cyst lining.
– Inflammatory: ♦♦ Fluid content of cyst is evaluated with suction.
- Radicular cyst, apical and lateral ♦♦ Four triangular flaps created are turned outwards and
- Residual cyst sutured with mucoperiosteum.
♦♦ Cavity is packed with gauge in iodoform or white heads
- Paradental cyst and mandibular infected buccal
varnish.
cyst
♦♦ If the cyst lining is friable as in infected cyst, be sutured
- Inflammatory collateral cyst.
with mucoperiosteum the gauge is used to hold the cyst
• Non epithelial: (Pseudocysts)
lining and mucoperiosteum together.
–– Solitary bone cyst
♦♦ One week later, the gauge is removed. Sutures also, by new
–– Aneurysmal bone cyst. union occur between lining and new periosteum.
♦♦ Cyst associated with maxillary antrum: ♦♦ Now a plug is made to maintain the opening of cavity
• Benign mucosal cyst of the maxillary antrum patent and prevent food from entering the cavity.
• Post operative maxillary cyst. ♦♦ The plug is worn continuous by but removed after meals.
♦♦ Cyst of the soft tissues of mouth, face and neck: ♦♦ The cavity is irrigated with syringe.
• Dermoid and epidermoid cyst ♦♦ The plug is never made of full cavity depth because it
• Lymphoepithelial cyst (Brachial cyst) interferes with bone regeneration of cavity.
• Thyroglossal duct cyst. ♦♦ The size of plug (depth) is decreased gradually as the cav-
• Anterior medial lingual cyst (Intra lingual cyst of ity is filled with bone.
foregut origin) ♦♦ Finally the mucoperiosteum is closed with sutures.
• Oral cyst with gastric or intestinal epithelium
• Cystic hygroma Enucleation
• Nasopharyngeal cyst
Method
• Thymic cyst
• Cyst of salivary glands: mucous extravasation cyst, ♦♦ Area is anesthetized by local anesthesia.
mucous retention cyst, ranula, polycystic disease of ♦♦ Incision should be long enough to provide good exposure
the parotid. and at the end of operation flap edge must rest on healthy
• Parasitic cyst: hydatid cyst, cysticercus cellulosae, bone.
trichinosis. ♦♦ In edentulous patient incision is given along the crest of
For description of various types of jaw refer to table in Ans ridge and in edentulous patient, it is given around the
10 of same chapter. neck of the teeth.
350   Mastering the BDS IIIrd Year  (Last 25 Years Solved Questions)

♦♦ Incision is given facially or lingually depending on location ♦♦ After few days, it again starts discharging mucus and
of cyst. closes by itself again.
♦♦ Mucoperiosteal flap is raised. ♦♦ The normal position of fistula or sinus remains in midline
♦♦ The character of underlying bone is determined. If thin layer in front of thyroid cartilage.
of bone is present on cyst, it is carefully peeled off with ♦♦ Its internal opening is on foramen cecum of tongue.
periosteal elevator. If the bone over a cyst is intact, a window ♦♦ If fistula becomes infected than surrounding skin becomes
is made with bur or chisel in postage stamp method. red, warm and tender and fistula secrete pus.
♦♦ The window is enlarged with rongeurs forcep to allow ♦♦ A fistulous opening in center of neck which is covered
complete enucleation. by hood of skin can occur due to increased growth of
♦♦ Care is taken not to puncture cyst wall because the intact neck when compared to that of fistula. This is known as
cyst is removed easily. semilunar sign or hood sign.
♦♦ The cyst lining is gently separated from cavity wall using
periosteal elevator or curette/mitchell trimmer/spoon Investigations
excavator. The convex surface of blade is kept facing the Radioisotope study and fistulogram.
lining of cyst.
♦♦ If cyst lining is difficult to separate from cavity wall or Treatment
nasal or antral lining use H2O2 gauge packing and then ♦♦ Infection is controlled with antibiotics
perform blunt dissection. ♦♦ Sistrunk’s operation is usually performed: For details, refer to
♦♦ A plane of cleavage is used to remove cystic sac in one piece. Ans 3 of same chapter.
♦♦ If any tooth or root is involved in cyst from it is either
extracted or
♦♦ The cavity is cleaned, debrided, irrigated and inspected.
♦♦ Bony margins are smoothened.
♦♦ Bleeding is checked and flap is re-positioned.
♦♦ Wound is closed with interrupted suture.
♦♦ Sutures are removed after 6 to 7 days.
♦♦ Routine analgesics and antibiotics are prescribed to prevent
postoperatory pain and infection.
Fig. 17:  Thyroglossal fistula
Decompression Followed by Enucleation
Q.10. What are different types of cysts in an oral cavity?
♦♦ Decompression of a cyst relieves the pressure within the Describe etiology, pathogenesis and management.
cyst and causes it to grow.  (Mar 2003, 15 Marks)
♦♦ It is performed by making a small opening in the cyst and Ans. Cyst is defined as “A pathological cavity having fluid,
keeping it open with drain. semifluid or gaseous contents and which is not created
♦♦ Cyst is kept open initially by medicated gauze pack and by accumulation of pus.” Kramer (1974).
an acrylic plug.
♦♦ Bone regeneration occur and cavity reduces in size Classification of Cyst in Oral Cavity by Mervin Shear
♦♦ This technique is not a definitive treatment but allows a ♦♦ Cysts of the Jaws:
second stage of enucleation to be undertaken on much • Epithelial:
smaller lesion which would have been impossible. –– Developmental:
Q.9. Write note on thyroglossal fistula. - Odontogenic
 (Sep 2010, 15 Marks) (Sep 1999, 10 Marks) * Gingival cyst of infants
Or * Odontogenic keratocyst (neoplasm)
Write short note on thyroglossal sinus. * Dentigerous cyst
 (Apr 2015, 3 Marks) * Eruption cyst
Ans. Thyroglossal fistula or sinus is never congenital. It is * Lateral periodontal cyst
* Gingival cyst of adults
always acquired due to following reasons:
* Botryoid odontogenic cyst
• Infected thyroglossal cyst rupturing into the skin.
* Glandular odontogenic cyst
• Inadequate drained infected thyroglossal cyst.
* Calcifying odontogenic cyst (neoplasm).
• Incompletely excised thyroglossal cyst.
- Non-odontogenic
Clinical Features * Nasopalatine duct cyst
♦♦ It occurs during 10 to 20 years. * Nasolabial cyst
♦♦ Patient gives history of previous swelling in neck (thyroglossal * Midpalatal raphe cyst of infants
cyst), which becomes infected, burst forms a fistula. * Median palatine, median alveolar
♦♦ The fistula or sinus discharges mucus and after sometime * Median mandibular cyst
fistula closes by itself. * Globulomaxillary cyst.
Section 2:  General Surgery  351

– Inflammatory: • Dermoid and epidermoid cyst


- Radicular cyst, apical and lateral • Lymphoepithelial cyst (Brachial cyst)
- Residual cyst • Thyroglossal duct cyst.
- Paradental cyst and mandibular infected buccal • Anterior medial lingual cyst (Intra lingual cyst of
cyst foregut origin)
- Inflammatory collateral cyst. • Oral cyst with gastric or intestinal epithelium
• Non epithelial: (Pseudocysts) • Cystic hygroma
–– Solitary bone cyst • Nasopharyngeal cyst
–– Aneurysmal bone cyst. • Thymic cyst
♦♦ Cyst associated with maxillary antrum: • Cyst of salivary glands: mucous extravasation cyst,
• Benign mucosal cyst of the maxillary antrum mucous retention cyst, ranula, polycystic disease of
• Postoperative maxillary cyst. the parotid.
♦♦ Cyst of the soft tissues of mouth, face and neck: • Parasitic cyst: hydatid cyst, cysticercus cellulosae,
trichinosis.

Etiology, Pathogenesis and Management of Various Cyst of Oral Cavity

Cyst Etiology Pathogenesis Management


Dentigerous cyst Impacted tooth, i.e. Intrafollicular theory: Dentigerous cyst is caused Small cyst by intraoral approach.
third molars or canines by fluid accumulation between reduced enamel Larger cyst involves surgical drainage and
epithelium and enamel surface which result in a marsupialization
cyst in which crown is located within the lumen.
Extrafollicular theory: Dentigerous cyst may
arise by proliferation and cystic transformation of
islands by odontogenic epithelium in connective
tissue wall of dental follicle or even outside dental
follicle and this transformed epithelium then unite
with lining follicular epithelium forming cystic cavity
around tooth crown
Eruption cyst Erupting deciduous or It develops as a result of separation of dental In some cases, cyst rupture spontaneously
permanent teeth follicle from around the crown of an erupting tooth which lead to tooth eruption.
which is within the soft tissues overlying bone If cyst does not rupture excision of roof of
cyst is done
Odontogenic –––––– Cyst arises from dental lamina which poses the Commonly OKCs are treated by enucleation
keratocyst growth potential with curettage
It also arises from proliferation from basal cells as Most of surgeons recommend peripheral
a residue/remanent oral epithelium osteotomy of bone cavity
Other group of surgeons undergo chemical
cauterization by Carnoy’s solution
Adenomatoid Impacted tooth Cystic hamartoma arising from odontogenic Enucleation with curettage
odontogenic cyst epithelium In some lesions marsupializatio is done.
Lateral Inflammation by pocket From proliferation of cell rest of malassez in PDL Conservative enucleation is done
periodontal cyst content near alveolar From proliferation and cystic transformation of
crest at lateral root rest of dental lamina
surface of tooth From reduced enamel epithelium
Developing tooth germ
Gingival cyst of Traumatic injury Cystic transformation of dental lamina Local surgical excision is done
adult Traumatic implantation of surface epithelium
From post functional rest of dental lamina
Gingival cyst of –––––– Proliferation of remnants of dental lamina after No management
newborn tooth formation
Palatal cyst of –––––– Due to entrapment of epithelium during formation No management is required. They are self-
new born of secondary palate healing
Epstein pearls occur along midpalatal raphae
and presumably arise from epithelium entrapped
along line of fusion
Bohn’s nodules are scattered over hard palate
near soft palate junction and are derived from
minor salivary glands
Glandular –––––– Arises from epithelial lining of dental lamina which Enucleation or curettage is carried out
odontogenic cyst has ability to induce formation of dental tissues.
Contd…
352   Mastering the BDS IIIrd Year  (Last 25 Years Solved Questions)

Contd…
Cyst Etiology Pathogenesis Management
Calcifying Arises from epithelial lining of dental lamina which Enucleation and curettage should be done
odontogenic cyst has ability to induce formation of dental tissues
Nasolabial cyst Development From non-odontogenic fissural cyst It is removed intraorally through surgical
Develop from remnants of nasolacrimal duct or approach
arising from epithelial lining of floor of mouth
Nasopalatine cyst •  Trauma From epithelial remnants of nasopalatine Enucleation is done
•  Bacterial infection duct which undergo proliferation and cystic
• B l o c k e d d u c t o f transformation
mucous gland
Median palatal –––––– Develops from epithelium entrapped along Surgical enucleation
cyst embryonic line of fusion of lateral palatal shelves
of maxilla
Anterior median –––––– Develops from epithelial entrapment between Incision and drainage
lingual cyst lateral tubercles of developing tongue
Periapical cyst Carious tooth Arises from epithelial residues in periodontal Nonsurgical treatment followed by
ligament as a result of inflammation. Pathogenesis apicocectomy
occur in: Enucleation or marsupialization if lesion is
•  Phase of initiation large
•  Phase of proliferation
•  Phase of cystification
•  Phase of enlargement
Residual cyst Incompletely removed Originates in residual epithelium of cell rests from Enucleation with primary closure
periapical granuloma. PDL of lost tooth
If impacted tooth or
carious tooth removed
and cystic lesion is not
recognized
Traumatic cyst Trauma It results due to osteosclerosis resulting from a Surgical exploration
disturbed circulation caused by trauma creating an
unequal balance of osteoclasts and repair of bone
Aneurysmal bone History of trauma It arises as an osseous arteriovenous fistula Surgical curettage or excision
cyst and thereby creates via its hemodynamic forces
secondary reactive lesion of bone
Mucocele Trauma to salivary duct Obstruction or trauma results in extravasation of Sharp and blunt dissection followed by
Obstruction to salivary mucus in connective tissue. Due to continuous excision of minor salivary gland
duct accumulation of mucus and pooling of saliva,
a cavity develops which has no epithelial lining
Ranula Trauma to excretory It occurs when the extra-vasated mucus passes Sublingual gland is removed by intraoral
ducts through mylohyoid muscle in submandibular approach
region
Thyroglossal duct It may arises from the residues of the duct at any Surgical excision along with cyst tract
cyst point of foramen cecum through neck of thyroid
gland
Cystic hygroma Developmental Arises as the lymphatic spaces communicates Surgical excision along with cyst tract
with each other to form large thin walled cyst
Dermoid and Trauma It arises from epithelial rests persisting in midline Intraoral lesions are removed by surgical
epidermoid cyst after fusion of mandible and hyoid brachial arches excision through a midline incision in free
edge of lingual frenum from behind the tip of
tongue down to its attachment in mandible.
Branchial cleft Arises from: Complete surgical excision by cervical or
cyst •  Epithelial remanent of branchial cleft intra-oral approach.
•  Residual cervical sinus epithelium
•  Cystic changes within cervical lymph nodes of
epithelial inclusion
Section 2:  General Surgery  353

Q.11. Write short note on branchial cyst.  Q.12. Write short note on plunging ranula. 
 (Feb 2002, 10 Marks)  (Oct 2007, 5 Marks)
Ans. This is a cystic neck swelling in the lateral aspect of Ans. When ranula extends in submandibular region through
the neck, which is a result of a persisting cervical sinus the deeper part of posterior margin of mylohyoid muscle,
formed by the second branchial cleft. it is known as plunging ranula.
• It is an intraoral ranula with cervical extension.
• It is cross fluctuant across mylohyoid.
• It can arise from both submandibular and sublingual
salivary gland as mucus retention cyst which reaches
neck by passing across the mylohyoid muscle
presenting as soft, fluctuant, non-tender, dumbbell
shaped swelling in the submandibular region.
• It is bidigitally palpable.
• Ultrasonography and/or MRI is diagnostic.
• It is treated by surgical excision through neck
approach along with excision of submandibular and
sublingual salivary glands.
• Small plunging ranula is often excised per orally
Fig. 18:  Branchial cyst along with excision of sublingual salivary gland.
Q.13. Write short note on cyst. (Nov 2008, 5 Marks)
Pathogenesis Ans. Cyst is defined as “A pathological cavity having fluid,
semifluid or gaseous contents and which is not created
It arises from by accumulation of pus.” Kramer (1974)
♦♦ Epithelial remnant of branchial cleft • True cysts are lined by epithelium while false cysts
♦♦ Residual cervical sinus epithelium are not lined by epithelium.
♦♦ Cystic changes within cervical lymph nodes of epithelial
inclusion General Examination of Cyst
Clinical Features ♦♦ Location: Most of the congenital cystic swellings have a
♦♦ Swelling is seen in the neck beneath anterior border of typical location wherein diagnosis can be made with fair
upper third of sternomastoid muscle. accuracy. A few examples are:
♦♦ Swelling is smooth, soft, fluctuant and transilluminant • Branchial cyst: Anterior triangle partly covered by
with the sensation of half-filled double hot water bottle. upper one-third of sternomastoid
♦♦ It is seen commonly in late adolescent and during third • Dermoid cyst: Midline, outer or inner canthus of the eye
decade of life. ♦♦ Shape: Majority of the cystic swellings are round or oval.
♦♦ Swelling is painless unless it is infected. • For example subhyoid bursitis: Transverse oval cystic
♦♦ Mobility of swelling is restricted because of adherence to swelling in the midline of the neck
sternomastoid muscle. • Thyroglossal cyst: Vertically placed oval swelling in the
♦♦ It contain cholesterol crystals which is from the lining midline of the neck
of mucus membrane which consists of sebaceous gland. • Sebaceous cyst: Hemispherical swelling
Cheesy toothpaste like material is typical. ♦♦ Surface: Almost all the cystic swellings in the skin and
subcutaneous tissue have smooth surface.
Treatment ♦♦ Consistency: Fluctuation is positive in all cystic swellings.
♦♦ Cyst should be excised under general anesthesia. However, depending on the contents the fluctuation may
♦♦ Branchial cyst is in relation to carotid, hypoglossal nerve, be different, which an experienced surgeon can diagnose.
glossopharyngeal nerve, spinal accessory nerve, posterior • Soft cystic: Thyroglossal cyst, meningocoele, lymph
belly of digastrics and pharyngeal wall. Medially, it is close cyst.
to posterior pillar of tonsils. During excision all the above • Tensely cystic: Ganglion.
structures should be taken care of. • Putty or tooth paste: Sebaceous cyst
♦♦ Sclerotherapy with OK–432 (Picibanil) is effective and is ♦♦ Transillumination Test: Cystic swelling which contains clear
done under ultrasonography guidance. fluid show positive transillumination.
♦♦ Mobility: Almost all cystic swellings in skin, subcutaneous
Complications
tissue or in deep plane are freely mobile. At times, this is
♦♦ Since the wall is rich in lymphatic tissue, it can undergo not true due to various anatomical factors:
secondary infection with pain and swelling. • Branchial cyst: Restricted mobility is due to its
♦♦ Recurrent infection. adherence to sternomastoid muscle.
354   Mastering the BDS IIIrd Year  (Last 25 Years Solved Questions)

• Thyroglossal cyst: Transverse mobility is absent because Clinical Features


cyst is tethered by remnant of thyroglossal duct. ♦♦ They are common and occur on the inner aspect of lower
♦♦ Sign of compressibility: Swelling which have communica- lip. They may also occur on palate, cheek, tongue and
tion with cavity or with tissue spaces give positive sign of floor of the mouth.
compressibility. ♦♦ They occur most frequently during third decade of life.
♦♦ Pulsations: Aneurysms are characterized by expansile pul- ♦♦ Patient complains of painless swelling. The swelling
sations and when the swelling pushes the vessel anteriorly may suddenly develops at meal time and may drain
transmission pulsation is obtained. simultaneously at interval.
Complications of Cyst ♦♦ The swelling is round, oval or smooth.
♦♦ The superficial cyst appears as bluish mass and if
♦♦ Infection: For example sebaceous cyst
inflammed it is fluctuant, soft, nodular and dome shaped
♦♦ Calcification: For example hematoma, hydatid cyst elevation.
♦♦ Pressure effects: Ovarian cyst pressing iliac veins
♦♦ Hemorrhage: In thyroid cyst. Treatment
Q.14. Write short note on mucous cyst of lower lip. Surgical removal is done by sharp and blunt dissection followed
(May/Jun 2009, 5 Marks) by excision of minor salivary gland.
Ans. It is also known as mucocele.
Mucous Retention Cyst
Etiology It is also known as salivary duct cyst. It is a true cyst.
♦♦ Due to obstruction of salivary duct
Etiology
♦♦ Trauma to secretory acini.
Obstruction to the minor salivary gland leads to retention of
Clinical Features saliva, continuous pressure due to retention leads to dilatation
♦♦ They are common and occur on the inner aspect of lower of duct and forms a cyst.
lip. They may also occur on palate, cheek, tongue and
Clinical Features
floor of the mouth.
♦♦ They occur most frequently during third decade of life. ♦♦ It more commonly occur in older individuals.
♦♦ The patient complains of painless swelling. The swell- ♦♦ Most common site is parotid gland. Intraorally, it is seen
ing may suddenly develops at meal time and may drain over buccal mucosa, lips and floor of mouth.
simultaneously at interval. ♦♦ It is a slow growing lesion, soft, fluctuant swelling which
♦♦ The swelling is round, oval or smooth. appear bluish in color.
♦♦ The superficial cyst appears as bluish mass and if in-
Treatment
flammed it is fluctuant, soft, nodular and dome-shaped
elevation. Surgical excision is treatment of choice.
Treatment Q.16. Write short note on types of cysts of oral cavity. De­
scribe ranula. (Jun 2010, 5 Marks)
Surgical removal is done by sharp and blunt dissection followed
Ans. For types of cysts of oral cavity refer to classification of
by excision of minor salivary gland.
cysts given in Ans 8 of same chapter. For ranula refer to
Q.15. Write short note on mucous cysts. (Dec 2009, 5 Marks) ans 5 of same chapter.
Ans. Mucous cysts are of two types, i.e. mucous extravasation
Q.17. Give classification, clinical features and complications
cyst and mucus retention cyst.
of cysts. (Aug 2011, 10 Marks)
Mucous Extravasation Cyst Ans. Cyst is defined as “A pathological cavity having fluid,
semifluid or gaseous contents and which is not created
It is the swelling caused by pooling of saliva at injured minor
by accumulation of pus.”  —Kramer (1974)
salivary gland.
Classification of Cysts
Etiology
♦♦ Congenital cyst
Trauma to minor salivary gland duct causes extravasation of • Dermoids: Sequestration dermoid cyst
mucus in connective tissue, due to this there is accumulation of • Tubulodermoids: Thyroglossal cyst, postanal dermoid,
mucous in connective tissue with continuous pooling of saliva. ependymal cyst, urachal cyst
Section 2:  General Surgery  355

• Cyst of embryonic remnants: cyst from paramesonephric ♦♦ Hemorrhage: In thyroid cyst


duct and mesonephric duct, cyst of urachus and vitel- ♦♦ Torsion: Ovarian dermoid
lointestinal duct cyst. ♦♦ Transformation into malignancy
♦♦ Acquired cyst ♦♦ Ovarian cachexia.
• Retention cysts: sebaceous cyst, bartholin’s cyst, parotid
Q.18. Define and describe differentiating features of dermoid
cyst, breast cyst.
cyst and sebaceous cyst. (Jan 2012, 5 Marks)
• Distention cyst: lymph cyst, ovarian cyst, colloid goiter
• Exudation cyst: Dursa, hydrocele, pancreatic pseudocyst Ans.
♦♦ Cystic tumors: Dermoid cyst of ovary, cystadenomas
Features Dermoid cyst Sebaceous cyst
♦♦ Traumatic cyst: Hematoma, implantation dermoid cyst
♦♦ Degenerative cyst: Tumor necrosis Definition It is a cyst which occurs It is a retention cyst
due to inclusion of which occur due to
♦♦ Parasitic cyst: Hydatid cyst, cysticercosis cellulosae, trichi-
epithelium beneath the blockage of duct of
nosis. surface which later get sebaceous gland
sequestered causing cystic swelling
Clinical Features of Cysts
Etiology It is congenital It is acquired
♦♦ Hemispherical swelling which is smooth, fluctuant, non- Location Midline of body, along Occurs anywhere
tender and well localized. the line of fusion except palm and sole
♦♦ Some cysts are transilluminant
Sign of indentation Uncommon Very common
♦♦ Presentation varies depending on anatomical location
and pathology Punctum Absent Present
♦♦ Cyst can be single or multiple. Skin fixation Absent Skin is fixed at the
♦♦ Cysts leads to the compression of adjacent structures site of punctum
♦♦ This leads to infection, sinus formation, hemorrhage, cal- Bony defect Present Absent
cification and cachexia. Mobility It is restricted Mobile

Complications of Cysts Intracranial Rare Absent


communication
♦♦ Infection: For example sebaceous cyst
Treatment Excision is done under Excision is done under
♦♦ Calcification: For example hematoma, hydatid cyst general anesthesia local anesthesia
♦♦ Pressure effects: Ovarian cyst pressing iliac veins

Q.19. Name the treatment modalities and differentiating features of ranula and sublingual dermoid.
 (Jan 2012, 5 Marks)
Or
Describe differentiating features sublingual dermoid and ranula. (Jan 2017, 3 Marks)
Ans.

Features Ranula Sub-lingual dermoid

Treatment modality Sublingual gland is removed by intraoral approach. Excision should be done

Definition It is an extravasation cyst arising from sublingual Sublingual dermoids develop in relation to sub-
glands or mucus glands of Nuhn or glands of Blandin mandibular duct, lingual nerve and stylohyoid
in floor of mouth ligament

Features Blue, smooth, soft, fluctuant brilliantly transilluminant It is smooth, soft, fluctuant, non-tender and non-
swelling transilluminant swelling

Position It is laterally placed It can be laterally or medially placed

Q.20. Name the treatment modalities and differentiating Or


features of dental cyst and dentigerous cyst.
Write difference between periapical cyst and dentiger­
 (Jan 2012, 5 Marks) ous cyst. (Feb 2014, 3 Marks)
Or
Write difference between dental cyst and dentigerous Ans.
cyst. (Mar 2016, 3 Marks)
356   Mastering the BDS IIIrd Year  (Last 25 Years Solved Questions)

Features Dental cyst/periapical Dentigerous cyst


Treatment modality Nonsurgical treatment followed by apicoectomy Small cyst by intraoral approach
Enucleation or marsupialization if lesion is large Larger cyst involves surgical drainage and marsupialization
Origin Originates due to fluid accumulation in reduced enamel Originates due to periradicular inflammatory changes which
epithelium leads to proliferation of epithelium
Association Associated with and occurs in relation to apex of nonvital Associated with and occurs in relation to impacted tooth
tooth
Type of cyst Inflammatory cyst Odontogenic cyst
Site Maxillary anterior teeth are commonly affected Mandibular third molars and maxillary canines are affected
Radiology Rounded radiolucency at apex of vital or nonvital tooth Well-defined radiolucency along with hyperostotic borders
and is visible around an unerupted tooth
Infection Common Not common
Complication Osteomyelitis Ameloblastoma

Q.21. Name the treatment modalities and differentiating features of thyroglossal cyst and subhyoid bursitis.
 (Jan 2012, 5 Marks)
Ans.
Features Thyroglossal cyst Subhyoid bursitis
Treatment modality Surgical excision along with cyst tract Excision under general anesthesia
Definition It is a swelling occurring in the neck in any part along the In subhyoid bursitis due to constant friction inflammatory
line of thyroglossal tract fluid collects in bursa leading to bursitis
Site Present beneath the foramen cecum, in floor of mouth, Horizontally placed midline swelling between lower part
suprahyoid, subhyoid and on thyroid cartilage of hyoid bone and thyrohyoid membrane
Features Swelling is smooth, soft, fluctuant, non-tender, mobile Swelling is smooth, soft, fluctuant, and non- transilluminant
and transilluminant
Movement of swelling Swelling move upward with deglutition but not while Swelling move upward with deglutition as well as while
protruding the tongue out protruding the tongue out

Q.22. Write short note on sequestration dermoid. swelling. This feature differentiate it from swelling arising
(Aug 2012, 5 Marks) from lacrimal gland. It can extend to orbital cavity also.
Ans. It occurs at the line of embryonic fusion, due to inclusion ♦♦ Internal angular dermoid: lt is a sequestration dermoid cyst
of epithelium beneath the surface which later gets in central position near the root of the nose. It occurs in
sequestered forming a cystic swelling in the deeper plane. frontonasal suture line. It mimics swelling from lacrimal
• It is congenital type. sac or mucocele of frontal sinus. Mucocele of frontal sinus
• Common sites are: is due to blockage of frontonasal duct.
– Forehead, neck, postauricular dermoid.
– External angular dermoid. Clinical Features
– Root of nose. ♦♦ Painless swelling in the line of embryonic fusion.
– Sublingual dermoid. ♦♦ Seen in second or third decades of life.
– Anywhere in midline or in the line of fusion. ♦♦ It is smooth, soft, non-tender and fluctuant and nontran-
• Dermoids occurring in the skull may extend into the silluminating
cranial cavity. ♦♦ There is presence of resorption and indentation of bone
• When it occurs as an external angular dermoid, it beneath.
extends into the orbital cavity, or it can extend into ♦♦ Impulse on coughing may be present only if there is intra-
any cavity in relation to its anatomical location (e.g. cranial extension.
thorax, abdomen).
Complications
Types of Angular Dermoid
♦♦ Infection
♦♦ External angular dermoid: lt is a sequestration dermoid if situ- ♦♦ Hemorrhage
ated over the external angular process of the frontal bone. ♦♦ Surface ulceration
Outer extremity of eyebrow extend over some part of ♦♦ Calcification.
Section 2:  General Surgery  357

Investigations • Smoked tobacco: Cigar, cigarette, beedi and pipe.


♦♦ X-ray skull or part Smoking tobacco is harmful as this smoke contains
♦♦ CT-scan head or part. polycyclic hydrocarbons, beta naphthylamine,
nitrosoamines, carbon monoxide, nicotine, all of which
Treatment acts as source of irritation. Heat also plays a major
Excision is done under general anesthesia. Often formal neuro- role. Heat induces alterations in tissues increasing
surgical approach is required by raising cranial osteocutaneous reddening and stippling of mucosal surface.
flaps. ♦♦ Alcohol: Whether the use of alcohol itself is an independ-
ent etiological factor in the development of leukoplakia is
Q.23. Enumerate midline swelling of neck. Describe thy­ still questionable. Its effect, at best may be synergistic to
roglossal cyst in brief. (Apr 2018, 5 marks) other well-known etiological factors (physical irritants).
Ans. Enumeration of midline swelling of neck Alcohol leads to irritation and burning sensation of oral
♦♦ Ludwig’s angina mucosa. Alcohol facilitates entry of carcinogen in exposed
♦♦ Enlarged sub-mental lymph node cells of oral mucosa and alters the oral epithelium as well
♦♦ Sub-lingual dermoid cyst as its metabolism.
♦♦ Thyroglossal cyst ♦♦ Chronic irritation: Continuous trauma or local irritation
♦♦ Sub-hyoid bursitis in the oral cavity leads to leukoplakia. Irritation or trauma
♦♦ Goiter of thyroid, isthmus and pyramidal lobe can be caused by malocclusion, ill-fitting dentures, sharp
♦♦ Enlarged lymph node and lipoma in substernal space of tooth or broken tooth, hot spicy food and root piece.
burns ♦♦ Candida albicans: As a possible etiological factor in leu-
♦♦ Retrosternal goiter koplakia and its possible role in malignant transformation
♦♦ Thymic swelling is still unclear. About 10% of oral leukoplakias satisfy the
♦♦ Bony swelling arising from the manubrium sterni. clinical and histological criteria for chronic hyperplastic
For thyroglossal cyst in brief refer to Ans 3 of same chapter. candidiasis.
♦♦ Viruses: The possible contributory role of viral agents such
as human papilloma virus 16 and 18 in the pathogenesis
of oral leukoplakia particularly with regard to exophytic
13. Oral Cavity, Lip and verrucous leukoplakia.
Palate ♦♦ Vitamin deficiency: Serum levels of vitamin A, B12, C,
beta-carotene and folic acid were significantly decreased
in patients with oral leukoplakia than compared to normal
Q.1. Write note on leukoplakia.  patients.
 (Dec 2010, 3 Marks) (Sep 1999, 5 Marks) ♦♦ Genetic factor: Relatively little is known yet with regard to
Or possible genetic factors in the development of leukoplakia.
Write briefly leukoplakia. (Dec 2010, 5 Marks) Clinical Features
Or
♦♦ Usually, the lesion occurs in 4th, 5th, 6th and 7th decades

Answer briefly on leukoplakia. (Mar 2016, 3 Marks) of life.
Or ♦♦ Buccal mucosa and commissural areas are most frequent
affected sites followed by alveolar ridge, tongue, lip, hard

Write in brief about leukoplakia.
and soft palate, etc.
 (July 2016, 5 Marks) ♦♦ Oral leukoplakia often present solitary or multiple white
Ans. Leukoplakia is defined as a white patch or plaque that patches.
cannot be characterized clinically or pathologically as ♦♦ The size of lesion may vary from small well localized patch
any other disease and is not associated with any physical measuring few millimeters in diameter.
or chemical causative agent except use of tobacco. First ♦♦ The surface of lesion may be smooth or finely wrinkled or
International Conference on Oral Leukoplakia Malmo, even rough on palpation and lesion cannot be removed
Sweden (1984) or modified WHO definition (1984). by scrapping.
♦♦ The lesion is whitish or grayish or in some cases, it is
Etiology brownish yellow in color due to heavy use of tobacco.
♦♦ In most of the cases, these lesion are asymptomatic, however
♦♦ Tobacco: Tobacco is widely used in two forms— in some cases, they may cause pain, feeling of thickness and
• Smokeless tobacco: Chewable tobacco and snuff. When burning sensation, etc.
tobacco is chewed, various chemical constituents leach
Differential Diagnosis
out such as nitrosonornicotine, nicotine, pyridine and
N picoline. Alkaline ph: 8.2–9.3 acts as local irritants ♦♦ Lichen planus: Shows Wickham’s straie
and leads to alterations of mucosa. ♦♦ Syphilis: Split papule or condyloma latum is seen
358   Mastering the BDS IIIrd Year  (Last 25 Years Solved Questions)

♦♦ Leukoedema: Shows faint milky appearance with folded and Although occasionally preceded by and/or associated
wrinkled pattern. Most commonly seen on buccal mucosa. with vesicle formation, it is always associated with
juxta-epithelial inflammatory reaction followed by
Treatment fibroelastic changes in lamina propria, with epithelial
Removal of etiological factors may lead to the reversal or atrophy leading to stiffness of oral mucosa and causing
elimination of disease. trismus and inability to eat.”  —Pindborg (1966)

Conservative Treatment Clinical Features


♦♦ Vitamin therapy is given to patient. Vitamin A should be ♦♦ It occurs during 20 to 40 years of age.
given to apply topically. ♦♦ Most commonly involved sites are buccal mucosa,
♦♦ Along with vitamin A, vitamin E should be given this leads retromolar area, uvula, soft palate, palatal fauces, tongue,
to inhibit metabolic degeneration. lips, pharynx and esophagus.
♦♦ Nystatin therapy is given to eliminate candidal infection. ♦♦ Onset of disease is insidious or develop over the period
♦♦ Vitamin B complex can be given as supplemental therapy. of 2 to 4 years.
Surgical Treatment ♦♦ Initially the patient complains of burning sensation in the
mouth, particularly during taking hot and spicy foods.
Conventional Surgery ♦♦ It is often accompanied or followed by the formation of
♦♦ Incision is made around the lesion including the safe multiple vesicles over the palate or ulcers or inflammatory
margins. reactions in other parts of oral mucosa.
♦♦ Incision is deep and wide. ♦♦ There can be either excessive salivation or deceased
♦♦ Affected area is undermined and is dissected from salivation (xerostomia) along with recurrent stomatitis.
underlying tissue. ♦♦ Patients also develop defective gustatory sensation.
♦♦ Sliding mucosal flap should be prepared for covering the ♦♦ In the initial phases of the disease, palpation of the mucosa
wound. elicits a “wet-leathery” feeling.
♦♦ Fine iris scissors and skin hook is used for decreasing ♦♦ Petechial spots may also be seen in the early stages of
trauma. the disease over the mucosal surfaces of tongue, lips and
♦♦ Extensive undermining of mucosal flap should be carried cheek, etc.
out so that when flap is advanced into its position, amount ♦♦ Oral mucous membrane is very painful upon palpation
of tension will be minimum. at this stage.
♦♦ As mobilization of mucosal flap is completed, it is ♦♦ One of the most important characteristic features of oral
advanced and free edges are approximated by multiple submucous fibrosis is the gradual stiffening of the oral
interrupted silk sutures. mucosa with progressive reduction in the mouth opening.
Fulguration ♦♦ Stiffness of the oral mucosa and the subsequent
trismus develops gradually within a few years after the
It is a technique in which there is destruction of tissues by high
development of the initial symptoms.
voltage electric current and the action is controlled by movable
♦♦ In the advanced stage of OSF, the oral mucosa losses its
electrode. This is done by electrocautery and electrosurgery.
resiliency to a great extent and it becomes blanched and
Laser stiff. Severe trismus develops at this stage.
Laser peel: It is used to remove the lesion which involves ♦♦ Because of stiffness of the lips and the tongue patients are
relatively large surface area. unable to blow whistles or even blow out a candle.
♦♦ Oral mucosa is symmetrically affected on both sides of the
Procedure mouth and it shows extreme pallor.
♦♦ Beam of laser is highly defocussed and should be kept at ♦♦ Oral submucous fibrosis often causes a blanched opaque
distant from the tissue. (white marble-like) appearance of the mucosa, on which,
♦♦ Initially, not any effect is seen on the tissue plane. there may be occasional presence of leukoplakic or
♦♦ Beam of laser should be gradually brought closer in focus, erythroplakic patches.
but remains in defocussed mode until tissue have white ♦♦ Palpation of the mucosa often reveals many vertical white
appearance and it begins to blister. fibrous bands on the inner aspect of the cheek.
♦♦ Blistering usually occur at the basement membrane. ♦♦ Patients of OSMF often develops difficulty in deglutition,
♦♦ The technique is extended over the rest of the lesion to referred pain in the ear or deafness and nasal intonation
be peeled. of voice.
♦♦ White area is then grafted with tissue forcep or hemostat. ♦♦ Depapillation of the tongue with recurrent or sometimes
Q.2. Describe clinical feature and treatment of submucous persistent glossitis occurs. Later on the tongue becomes
fibrosis. (Apr 2008, 5 Marks) (Oct 2007, 5 Marks) stiff and shows restricted movements.
Ans. OSMF is defined as “An insidious chronic disease ♦♦ Uvula become ’bud—like” or hockey stick shaped or
affecting any part of oral cavity and sometime pharynx. become shrunken.
Section 2:  General Surgery  359

Treatment ♦♦ Pregnancy epulis refer to this variety.


♦♦ It is a mass of granulation tissue around the carious tooth
Supportive Treatment
on the gum.
♦♦ Diet rich with the vitamins along with iron preparation ♦♦ It is a soft to firm fleshy mass and bleed on touch.
should be given to the patient.
♦♦ IM injection of Iodine B complex is given to the patient. Treatment
During early phase low doses are given and later on high ♦♦ Extraction of associated carious tooth.
doses are effective. ♦♦ Removal of local irritation as ill-fitting denture.
♦♦ Local injection of hydrocortisone in the lesional area is ♦♦ Excision of granulation tissue.
of value. ♦♦ Pregnancy epulis regress spontaneously after delivery.
♦♦ Intralesional injection of hyaluronidase can be given.
♦♦ Systemically 100 mg/day of hydrocortisone is effective in Fibrous Epulis
reducing the burning sensation.
It is the most common type of epulis.
♦♦ 2 mL injection of placentrex intralesional is effective.
♦♦ Lycopene as an antioxidant should be given to the patient. Clinical Features
♦♦ Vitamin E along with hyalurodinase and dexamethasone
is effective. ♦♦ Arises from periosteum at the neck of teeth.
♦♦ Intralesional injection of interferon gamma helps in ♦♦ It is localized inflammatory hyperplasia of gum.
increment in mouth opening. ♦♦ It occurs in response to local irritation.
♦♦ It is composed of fibrous tissue and blood vessels.
Surgical Treatment ♦♦ It is a firm polypoidal mass, slowly growing and non-tender.
♦♦ Conventional Surgery:
Treatment
• Tongue mucosa as a graft: The fibrous bands are
excisioned and tongue mucosa as a flap is used. Excision of growth and removal of local irritants.
• Implantation of fresh human placenta can be done
following the surgical excision of fibrous bands. Giant Cell Epulis / Myeloid Epulis
• Fibrotic bands are excisioned and nasolabial flap is Clinical Features
taken for reconstruction.
• Bilateral palatal flap is taken for generally covering ♦♦ It is less common, purple and pedunculated tumor.
of exposed area. ♦♦ It is an “osteoclastoma” arising in the jaw.
♦♦ Laser: ♦♦ It present as hyperemic vascular edematous, soft to firm
• Patient should be incorporated with general gums with indurated underlying mass due to expansion
anesthesia. CO2 LASER incise the buccal mucosa and of the bone.
♦♦ It may ulcerate and result in hemorrhage.
vaporizes mucosal connective tissue.
♦♦ Multinucleated giant cells are found in histology.
♦♦ Cryosurgery: It helps in providing the relief from the local
♦♦ It is more rapidly growing tumor than any other epulis.
lesions.
Q.3. Describe about etiology, clinical features and treatment Treatment
of epulis. (Sep 2001, 15 Marks) ♦♦ Small tumors are treated by curettage.
Ans. Epulis means upon gum. ♦♦ Large tumors are treated by radical excision.
It refers to solid swelling situated on the gum.
Localized enlargement of gingiva is mainly known as Carcinomatous Epulis
epulis.
Clinical Features
It arises from alveolar margin of jaw.
Epulis can originate from the bone, periosteum or the ♦♦ This is an “epithelioma” arising from mucous membrane
mucous membrane. of alveolar margin.
♦♦ It presents as a non-healing, painless ulcer.
Types of Epulis ♦♦ It slowly infiltrates the bone.
♦♦ Hard regional lymph nodes are due to metastasis.
Granulomatous Epulis / Pyogenic Granuloma
Treatment
♦♦ It is also called false epulis or pyogenic epulis.
♦♦ Precipitating factors are: Carious tooth, dentures, poor Treated by wide excision, which include removal of segment
oral hygiene. of the bone.

Clinical features Congenital Epulis


♦♦ The important point is that it does not contain pus while ♦♦ This is a benign condition seen in a newborn arising from
its name is pyogenic epulis. the gum pads.
360   Mastering the BDS IIIrd Year  (Last 25 Years Solved Questions)

♦♦ This is the variant of granular cell myeloblastoma originat- ♦♦ Cysts arising in relation to dental epithelium:
ing from the gums. • Dental cyst
♦♦ This is common in girls and more common in upper jaw • Dentigerous cyst
mainly in canine or premolar area. ♦♦ Swelling arising from the mandible or maxilla:
♦♦ Congetinal epulis is a well localized swelling from the gum • Osteoma and osteoblastoma
which bleeds on touch. • Torus palatinus and mandibularis
♦♦ Excision of congenital epulis should be done. • Fibrous dysplasia
• Osteoclastoma
Myelomatous Epulis • Osteosarcoma
♦♦ This occurs mainly in leukemic patients. • Secondaries
♦♦ Treatment of leukemia resolves this condition. • Giant cell reparative granuloma
♦♦ Surface tumors:
Tumors from the surface which extend into the jaw.
Giant Cell Epulis
• Ossifying fibroma
This is the osteoclastoma causing ulceration and hemorrhage • Osteofibrosis of maxilla
of gingiva. • Ivory osteoma of jaw
• Leontiasis ossea (diffuse osteitis)
Fibrosarcomatous Epulis • Carcinoma extending into the jaw
Also refer to Ans 3 of same chapter for epulis.
It is the fibrosarcoma arising from the fibrous tissue of gums.
Q.6. Write in short adamantinoma. (Sep 2002, 10 Marks)
Carcinomatous Epulis (Feb 2004, 5 Marks) (Dec 2010, 3 Marks)
Or
Squamous cell carcinoma of alveolus and gingiva, present Write short answer on adamantinoma.
localized, hard, indurated, swelling with ulceration.  (Apr 2018, 3 Marks)
Q.4. Write short note on epulis.  Ans. In 1885, Malassez coined the term adamantinoma. In
 (Feb 1999, 5 Marks) (Sep 2000, 10 Marks) 1934, Churchill replaced the term adamantinoma with
 (Sep 2002, 5 Marks) (Mar 2003, 10 Marks) ameloblastoma.
 (Feb 2004, 5 Marks) (Sep 2005, 8 Marks) Ameloblastoma is defined as “usually unicentric, non-
 (Mar 2008, 5 Marks) (Mar 2009, 5 Marks) functional, intermittent growth, anatomically benign and
 (Dec 2012, 5 Marks) (Aug 2012, 5 Marks) clinically persistent”  —By Robinson
 (Apr 2008, 5 Marks) (Feb 2015, 5 Marks)
 (Mar 2006, 5 Marks) (Nov 2008, 5 Marks) Pathogenesis
 (Dec 2009, 5 Marks) (Apr 2017, 4 Marks) Ameloblastoma originates from:
Ans. Refer to Ans 3 of same chapter. ♦♦ Epithelial rest of Malassez.
♦♦ Epithelium of odontogenic cysts
Q.5. Classify tumors of jaw. How will you treat a case of
♦♦ Disturbances in developing enamel organ
epulis? (Mar 2000, 15 Marks)
♦♦ Basal cells of surface epithelium
Ans. ♦♦ Heterotrophic epithelium.
Classification of Tumors of Jaw
♦♦ Swelling arising from the gums (Epulis) Clinical Features
• Congenital epulis ♦♦ It occurs during 2nd, 3rd and 4th decades of life.
• Fibrous epulis ♦♦ Predilection for males is seen.
• Pregnancy epulis ♦♦ It is seen in molar ramus area in mandible and in third molar
• Giant cell epulis area including maxillary sinus and floor of nose in maxilla.
• Myelomatous epulis ♦♦ Tumor start as a lesion of bone and later on expands the
• Sarcomatous epulis bone.
• Carcinomatous epulis ♦♦ Patient complains of asymmetry of face.
♦♦ Swelling arising from the dental epithelium (Odontomes): ♦♦ Teeth in the lesional area are displaced.
• Ameloblastoma ♦♦ Pain and paresthesia is present if lesion involves any of
• Compound odontome the nerve.
• Enameloma ♦♦ As the tumor enlarges palpation leads to crepitus also
• Cementoma known as egg shell crackling.
• Dentinoma
• Odontogenic fibroma and myxoma Investigations
• Radicular odontome ♦♦ Clinically: Presence of swelling in posterior mandible with
• Composite odontome expansion as well as egg shell crackling.
Section 2:  General Surgery  361

♦♦ Radiographically: Honey comb or soap bubble appearance The following are the rare varieties:
in posterior region of mandible. Labial and lingual plate • Connective tissue odontomes
expansion is also seen. –– Fibrous
♦♦ Biopsy: Biopsy of the lesion is needed for the confirmation –– Cementous
of diagnosis so that histological type of ameloblastoma is –– Sarcomatous.
diagnosed. • Composite odontomes (i.e. arising from both epithelial and
connective tissue elements):
Treatment –– Radicular
–– Compound follicular
♦♦ Ameloblastomas are generally slow growing but locally –– Composite complex.
invasive tumors and have a high recurrence rate after ♦♦ Osseous tumors: Any bone tumor can affect jaw
treatment. • Benign:
♦♦ Curettage of ameloblastomas, which was favored in the –– Fibro-osseous group
past, is now not advocated because of the high recurrence –– Paget’s disease
rate associated with it. –– Osteoclastoma.
♦♦ Ameloblastomas are best treated by resection of the lesion • Malignant tumors:
with a marginal clearance of 1.5–2 cm of normal bone to –– Osteosarcoma
prevent recurrence. –– Squamous cell carcinoma
♦♦ The lesion may be resected as block resection with or
–– Burkitt’s tumor
without continuity defect based on the integrity of inferior
–– Columnar cell carcinoma of maxillary antrum.
cortex.
♦♦ Inflammatory group:
♦♦ Radiologically a minimum of 1 cm residual mandible
• Alveolar abscess
inferior cortex is required postoperatively to prevent
• Osteomyelitis
pathologic fracture.
• Actinomycosis.
♦♦ Inferior alveolar nerve should be sacrificed if it lies within
the lesion. For investigation, clinical features and treatment of
♦♦ Maxillary ameloblastomas are particularly dangerous, adamantinoma refer to Ans 6 of same chapter.
partly because the bones are considerably thinner than Q.8. Write short note on Vincent’s angina.
those of the mandible and present less effective barriers to  (Sep 2005, 5 Marks) (Jan 2012, 5 Marks)
spread. Therefore, radical excision is essential, preferably Ans. It is an acute ulceromembranous stomatitis or acute
maxillectomy. ulcerative gingivitis.
♦♦ Peripheral ameloblastomas are treated by excision, as
This disease is caused by Vincent’s organism-Borrelia
usually there is no alveolar bone involvement. If prior
vincentii an anaerobic spirochete and fusiformis.
biopsy indicates involvement of bone, block resection with
continuity defect is the choice of treatment. Precipitating Factors
Q.7. Discuss the differential diagnosis of jaw swellings. ♦♦ Malnutrition, diabetes mellitus, carious tooth, war seasons,
Describe investigation, clinical features and treatment winter, etc.
of adamantinoma.  (Mar 1997, 15 Marks) ♦♦ The disease starts in the intergingival defects as a deep
Ans. penetrating ulcer, which results in a spon­taneous gingival
hemorrhage.
Differential Diagnosis of Jaw Swellings ♦♦ Once infections spread to tonsillar region, it is called as
Vincent’s angina. Very severe painful condition.
♦♦ Swelling arising from the mucoperiosteum, i.e. epulis. It has 5
varieties: Clinical Features
• Fibrous ♦♦ Common in children and young adults between 20 and
• Granulomatous 40 years of age.
• Myeloid ♦♦ It is a gangrenous devastating stomatitis which begins
• Sarcomatous in mucus membrane of corner of mouth or cheek which
• Carcinomatous. progress rapidly to involve entire thickness of lips or cheek
♦♦ Swellings arising from tooth germs, i.e. odontomes. The differ- or both with necrosis and sloughing of entire tissue which
ent varieties are: is observed in poorly nourished children and debilitated
• Epithelial odontomes adults.
–– Dental cyst ♦♦ It start from lips extend to gums, spread to cheeks, bone,
–– Dentigerous cyst soft tissues and skin leading to extensive tissue loss with
–– Adamantioma. severe toxemia.
362   Mastering the BDS IIIrd Year  (Last 25 Years Solved Questions)

♦♦ There is presence of excessive salivation, fetid odor with Q.9. Write short note on tumors of jaw.
destruction, discharge and toxic features.  (Sep 2006, 10 Marks)

Investigations Or
♦♦ Culture of Borrelia vincentii Write short note on swelling of jaw.
♦♦ X-ray of involved part shows bony destruction.  (Feb 2015, 5 Marks)
Treatment Or

♦♦ Systemic antibiotics, i.e. higher doses of penicillin and Discuss etiology, clinical features and treatment of
metronidazole should be given. tumor of jaw. (Sep 2007, 10 Marks)
♦♦ High protein and vitamin rich diet should be given through
nasogastric intubation. Or
♦♦ Wound irrigation is done and liberal excision of dead tissue Describe swelling of the jaw and its manage­ment. 
should be carried out.  (Apr 2007, 15 Marks)
♦♦ Blood transfusion is given.
Ans. For classification refer to Ans 5 of same chapter.
♦♦ Later on patient require flaps to cover the defect.

Tumor of jaw Etiology Clinical features Treatment/Management


Epulis • Carious tooth • In congenital epulis a well localized swelling Excision should be done
• Dentures is seen over the gum which is firm and
• Poor oral hygiene bleeds on touch
• In fibrous epulis swelling is red in color, firm/
hard, sessile or pedunculated. It is painless
Ameloblastoma • Irritation in posterior • It occurs during 2nd, 3rd and 4th decades • Ameloblastomas are generally slow growing
region of mandible of life but locally invasive tumors and have a high
• Infection • Predilection for males is seen recurrence rate after treatment
• Trauma • It is seen in molar ramus area in mandible • Curettage of ameloblastomas, which was
• Dietary deficiency and in third molar area including maxillary favored in the past, is now not advocated
• Virus sinus and floor of nose in maxilla because of the high recurrence rate
• The tumor start as a lesion of bone and later associated with it
on expands the bone • Ameloblastomas are best treated by resection
• Patient complains of asymmetry of face of the lesion with a marginal clearance of 1.5–
• Teeth in the lesional area are displaced 2 cm of normal bone to prevent recurrence.
• Pain and paresthesia is present if lesion • Lesion may be resected as block resection
involves any of the nerve with or without continuity defect based on the
• As the tumor enlarges palpation leads to integrity of inferior cortex
crepitus also known as egg shell crackling • Radiologically a minimum of 1 cm residual
mandible inferior cortex is required
postoperatively to prevent pathologic fracture
• Inferior alveolar nerve should be sacrificed if
it lies within the lesion
• Maxillary ameloblastomas are particularly
dangerous, partly because the bones are
considerably thinner than those of the
mandible and present less effective barriers
to spread. Therefore, radical excision is
essential, preferably maxillectomy
• Peripheral ameloblastomas are treated by
excision, as usually there is no alveolar
bone involvement. If prior biopsy indicates
involvement of bone, block resection with
continuity defect is the choice of treatment
Compound • Genetic transmission • Slow growing. Non-infiltrating lesion Enucleation
odontome • Local trauma • Occur commonly in maxilla in incisor-canine
• Infection region
• Seen in 2nd and 3rd decades
• Permanent teeth fail to erupt due to
interference from compound odontome

Contd…
Section 2:  General Surgery  363

Contd…

Tumor of jaw Etiology Clinical Features Treatment/Management

Enameloma Trauma • Commonly seen in bifurcation or trifurcation It should be removed if leading to periodontal
of roots diseases
• Common site is maxillary molar
• Appears as tiny globule of enamel adhering
to the root

Cementoma –––––– • Painless boney expansion of jaw is present. Extraction of associated tooth and complete
• Occur during 15-30 years enucleation of mass
• Mandible is more commonly affected and
the affected teeth are mandibular first molar,
second and third molars
• Slow growing lesion which expand and
resorb medial and lateral cortical plates

Dentinoma –––––– • Seen in 2nd and 3rd decades Surgical excision with curettage
• Seen in mandibular molar region and is
associated with impacted tooth
• Patient complains of swelling with pain

Odontogenic – Irritation associated Central Central


fibroma with overextended • Occurs during 2nd and 3rd decades Enucleation and curettage
margins of faulty res- • Female predilection is seen
toration and `due to • Maxilla is more commonly affected
deposition of calculus • Appear as asymptomatic, progressive
enlargement of jaw
• Mobility of associated teeth is seen
Peripheral Peripheral
• Lesion appear as pink, firm, sessile or Excision is done
pedunculated mass on attached gingiva
• Mandible is commonly affected

Osteoma –––––– • Seen in age group less than 30 years Excision or curettage
• Patient complaints of pain which worsen in
night
• Local soft tissue swelling is present
Fibrous • Genetic • Monostotic fibrous dysplasia • Surgical removal of the lesion should be
dysplasia • Developmental – It involves only one bone and pigmented done.
• Endocrine skin lesions are present • Osseous contouring is done to correct the
– The sites which affected are maxilla, deformities so that aesthetics of patient
mandible, ribs and femur should be improved.
– It occurs in children younger than 10
years and both the sexes are equally
affected
– Maxilla is more commonly affected than
the mandible and most of the changes
occur in posterior region. The most
common area involved is premolar area
– There is presence of unilateral facial
swelling which is slow growing with intact
overlying mucosa
– Swelling is painless but patient may feel
discomfort
– T h e r e i s p r e s e n c e o f e n l a r g i n g
deformities of alveolar process mainly
buccal and labial cortical plates
– In mandible, there is protrusion of inferior
border of mandible
• Polystotic fibrous dysplasia
– It is seen during first and second
decades of life
– Female predilection is seen with male to
female ratio of 1:3

Contd…
364   Mastering the BDS IIIrd Year  (Last 25 Years Solved Questions)

Contd…

Tumor of jaw Etiology Clinical features Treatment/Management


– Most common sites to be involved are skull, facial bones,
clavicle, thighs, shoulder, chest and neck
– Café-au-lait spots are seen over the skin which are
irregular in shape and are light brown melanotic spots
– Patient complains of recurrent bone pain
– There is expansion of jaws and asymmetry of facial bones
which leads to enlargement and deformity
Ossifying –––––– • Slow growing painless expansile lesion which replaces • The lesions have capsule and
fibroma normal bone as it enlarges. are enucleated by intraoral
• Patient complains of facial asymmetry. approach
• Seen between 2nd and 4th decades. • Large lesions which distort the
• Female predilection is seen. jaws requires local resection
with grafting of bone

Q.10. Name the treatment modalities and differentiating Pachyonychia congenita


features of adamantinoma and osteoblastoma. Porokeratosis
(Jan 2012, 5 Marks) Darier’s disease
Ans. Pseudoxanthoma elasticum.
• Non-infective Disease
Features Admantinoma Osteoclastoma – Vesicular:
Age Childhood or early adult life 25 to 30 years - Erythema multiforme
Rate of growth Slow growing tumor Rapidly growing tumor
- Pemphigus
- Bullous lichen planus.
Expansion Mostly the outer table of Both inner and outer
jaw bone table are expanded
– Non-vesicular:
Lichen planus.
Pain It rarely causes pain It is painless
– Collagen disorders:
Fungation Some fungates in very Usually not fungate
- Lupus erythematous
late case to the exterior
- Scleroderma.
X-ray Fine honey-comb Soap-bubble
– Degenerative
appearance appearance appearance with large
cysts and fine or ill-
Oral submucous fibrosis.
defined trabeculae – Pigmentation:
Histology Peripheral columnar cells Multinucleated giant
- Anemia
with central core of star cells are many in - Addison’s disease
cells with large vacuoles number - Racial pigmentation
in the cytoplasm OSMF is defined as “An insidious chronic disease
Radiosensitive No radiosensitive Radiosensitive affecting any part of oral cavity and sometime pharynx.
Malignancy It is locally malignant May turn into
Although occasionally preceded by and/or associated
tumor malignancy with vesicle formation, it is always associated with
juxta-epithelial inflammatory reaction followed by
Treatment Ameloblastomas are best Curettage and
fibroelastic changes in lamina propria, with epithelial
modalities treated by resection of conservative surgical
the lesion with a marginal excision is done. atrophy leading to stiffness of oral mucosa and causing
clearance of 1.5–2 cm of trismus and inability to eat.” —Pindborg (1966)
normal bone to prevent
recurrence Clinical Features
♦♦ It is caused during 20 to 40 years of age.
Q.11. Classify oral mucosal lesions. Describe the clinical ♦♦ Females are affected more than males.
features and histopathological features of submucosal ♦♦ In OSMF fibrotic changes are frequently seen in buccal
fibrosis. (Sep 2004, 20 Marks) mucosa, retromolar area, vulva, tongue, etc.
Ans. Classification of oral mucosal lesions ♦♦ Initially patient complains of burning sensation in the
• Genodermatous mouth, particularly during taking hot and spicy foods.
White sponge nevus ♦♦ There can be excessive salivation, decreased salivation and
Hereditary benign intraepithelial dyskeratosis defective gustatory sensation.
Section 2:  General Surgery  365

♦♦ In initial phase of disease palpation of mucosa elicits a ♦♦ The tuberculous lesions of oral cavity are tuberculous ulcers,
“wet leathery” feeling. tuberculous gingivitis and tuberculosis of salivary gland.
♦♦ In advanced stage, the oral mucosa losses its resilience and ♦♦ Tongue is most common location for the occurrence,
become blanched and stiff and thereby causing trismus. besides this palate, gingiva, lips, buccal mucosa, alveolar
♦♦ Palpation of mucosa often reveals vertical fibrous bands. ridge and vestibules may also be affected.
♦♦ Tongue leison: Tuberculous lesion of tongue develops on the
Histopathology lateral borders and appears as single or multiple ulcers which
Microscopically OSMF reveals following features: are well defined, painful, firm and yellowish gray in color.
♦♦ Overlying hyperkeratinized, atrophic epithelium often ♦♦ Lip lesions: The lesions produce small, non-tender, granu-
shows flattening and shortening of rete pegs. lating ulcer at mucocutaneous junction.
♦♦ There can be variable degree of cellular atypia or epithelial ♦♦ Gingival lesions: These lesions produce small granulating
dysplasia. ulcers with concomitant gingival hyperplasia.
♦♦ In OSMF dysplastic changes are found in epithelium ♦♦ Tuberculous lesion of jaw bone: Chronic osteo­myelitis of max-
which include nuclear pleomorphism, severe intercellular illa and mandible may occur and infection reaches to bone
edema, etc. via blood or root canal or extraction socket. Tuberculous
♦♦ Stromal blood vessels are dilated and congested and there osteomyelitis of jaw bone produces pain, swelling, sinus
can be areas of hemorrhage. or fistula formation.
♦♦ Underlying connective tissue stroma in advanced stage
of disease shows homogenization and hyalinization of Leprosy
collagen fibers.
Oral manifestations of leprosy
♦♦ Decreased number of fibroblastic cells and narrowing of
blood vessels due to perivascular fibrosis are present. ♦♦ In oral cavity, the disease produces tumor like lesions
♦♦ There can be presence of signet cells in some cases. called as “lepromas” which are found on lips, gingiva,
tongue and hard palate.
♦♦ Oral lesion appears as yellowish soft or hard sessile growth
which have tendency to breakdown and ulcerate.
♦♦ Ulceration, necrosis and perforation of palate.
♦♦ Fixation of palate with loss of uvula.
♦♦ Difficulty in swallowing and regurgitation.
♦♦ Cobblestone appearance of tongue with loss of taste
sensation.
♦♦ Chronic gingivitis, periodontitis and candidiasis are present.
♦♦ The enamel hypoplasia of teeth, pinkish discoloration of
teeth and tapering of teeth is present.

Herpes Zoster
Fig. 19:  Oral submucous fibrosis
(For color version see Plate 3)
Oral Manifestations of Herpes Zoster
Q.12. Write the oral manifestations of systemic disease. ♦♦ Herpes zoster may involve the face by infection of trigemi-
 (Apr 2010, 5 Marks) (Sep 2004, 5 Marks) nal nerve, mainly first branch.
Ans. ♦♦ There is usually involvement of skin and oral mucosa
supplied by trigeminal nerve.
Oral Manifestations of Systemic Disease ♦♦ Lesions of the oral mucosa are extremely painful vesicles
Syphilis which may be found on the buccal mucosa, tongue,
pharynx and larynx and uvula.
The oral manifestations of syphilis are given in the folowing ♦♦ This vesicle generally ruptures and leaves the area of
table. erosion.
Tuberculosis ♦♦ The erosive ulcers heal up in a few days without scar
formation.
Oral manifestations of tuberculosis
♦♦ In herpes zoster, neuralgic pain in oral cavity stimulates
♦♦ Tuberculous infection in oral cavity may produce nodules, tooth ache.
vesicles, fissures, plaque, granulomas or verrucal papil- ♦♦ The pain may persist long after the lesion heals up and the
lary lesions. condition is known as post-herpetic neuralgia.
366   Mastering the BDS IIIrd Year  (Last 25 Years Solved Questions)

Characteristic Primary syphilis Secondary syphilis Tertiary syphilis


Site Chancre occurs at the site on entry Mucus patches are present on tongue, Gumma can occur anywhere in jaw but
of Treponema. It occurs on lip, oral buccal mucosa, pharyngeal region and lips. more frequent site is palate, mandible
mucosa, and lateral surface of – Split papules develop at commissure of and tongue
tongue, soft palate and gingiva lips
Appearance It has narrow copper colored slightly Mucus patches appears as slightly Gumma manifests as solitary, deep
raised borders with reddish brown raised grayish white lesions surrounded punched out mucosal ulcer
base in centers. by erythematous base. Split papules
are cracked in middle giving “Split pea
appearance”
Symptom Intraoral chancres are slightly painful Mucus patches are painless mild-to-mod- In gumma the breathing and swallowing
due to secondary infection and are erately painful difficulty may be encountered by the
covered with grayish white film. patients
Signs White sloughy material is present Snail tract ulcers and raw bleeding surfaces Perforation of palatal vault is present.
are present
Tongue Tongue lesion may be commonly Tongue gets fissured. Numerous small healed gumma in
seen on lateral surface of anterior tongue results in series of nodules
two-thirds or on dorsal surface and or sparse in deeper area of organ
often there is enlargement of foliate giving tongue an upholstered or tufted
papilla appearance

Q.13. Write short note on lipoma. ♦♦ Hibernoma: Benign tumor arising from brown fat is called
 (Aug 2012, 5 Marks) (Sep 2008, 5 Marks) as hibernoma (reddish brown), which has got serpentine
 (Nov 2008, 5 Marks) (Sep 2007, 5 Marks) vascular elements.
Ans. Lipoma is a benign neoplasm arising from yellow fat. ♦♦ Fibrolipoma: Lipoma with fibrous component is called as
Often it can be hyperplasia or combination of neoplasm fibrolipoma.
and hyperplasia. ♦♦ Naevolipoma: Lipoma with telangiectasis is called as
♦♦ It is the most common benign tumor naevolipoma.
♦♦ It is also called as universal tumor (ubiquitous tumor) as ♦♦ Neurolipoma: (with nerve tissue and is painful), angi-
it can occur anywhere in the body except brain. olipoma (with vascular element), myolipoma, chondroid
lipoma, spindle cell lipoma, pleomorphic lipoma-are dif-
Types of Lipoma ferent types
♦♦ Localized (encapsulated): Localized lipoma is encapsulated ♦♦ Depending on the type of nonadipose component associ-
with yellowish orange. ated.
♦♦ Color. Clinical Features
♦♦ Diffuse (nonencapsulated): Diffuse lipomas are not encapsu-
lated and not localized. It is common in palm, sole, head, ♦♦ Localized swelling, which is lobular (surface), nontender.
neck. lt is seen in subcutaneous and intermuscular tissues. ♦♦ Often fluctuant like feel but actually not (because fat in body
♦♦ Superficial lipomas: They are more common; common in temperature remains soft). It is usually nontransilluminant.
subcutaneous plane. It is common in back, neck, proximal ♦♦ Mobile, with edges slipping between the palpating finger
extremities and abdomen. It is commonly less than 5 cm, (slip sign).
but can attain large size. ♦♦ Skin is free.
♦♦ Deep lipomas: They are commonly intramuscular, but often ♦♦ Lipomas may be pedunculated at times.
may be intermuscular; often both intra- and intermuscular ♦♦ It is rare in children.
(infiltrating lipoma). They are common in lower limbs ♦♦ Pain in lipoma may be due to neural element or compres-
(45%), trunk (l7%), shoulder and upper limb. They attain sion to nerves or adjacent structures. Angiolipoma was
large size compared to superficial lipomas. being highly vascular is commonly tender.
♦♦ Single lipoma: It is common. It is usually superficial in ♦♦ Trunk is the most common site; nape of neck and limbs
subcutaneous plane but can be deep also. are next common.
♦♦ Multiple lipomas: They are l5% common; common in males ♦♦ Clinically lipoma can be single, multiple or diffuse.
(6:1). Common in back, shoulder and upper arm; can be
Differential Diagnosis
symmetrical. It can be associated with many syndromes
like multiple endocrine neoplasia (MEN), Cowden’s, ♦♦ Neurofibroma: It moves horizontally but not longitudinally
Frohlich syndromes, etc. along the line of nerve. Neurofibroma is firmer.
Section 2:  General Surgery  367

♦♦ Cystic swellings like dermoid, sebaceous cyst. Secondaries in the sub mandibular lymph node
♦♦ Liposarcoma: All lipomas are benign. Large lipoma should arises from carcinoma of cheek, tongue, palate. The
be differentiated from liposarcoma. nodes are hard with or without fixity.
Non-Hodgkin’s Lymphoma: It can involve sub
Investigations
mandibular lymph nodes along with horizontal
♦♦ Ultrasound or CT or MR imaging is done in deep or large group of lymph nodes in neck. The nodes are firm
or intracavitary lipomas. and rubbery in consistency.
♦♦ FNAC or incision biopsy is needed in large or deep or Q.15. Describe different types of odontomes and their man­
intracavitary lipomas to confirm it as benign. agement. (Oct 2003, 15 Marks)
Complications Ans. Odontome is defined as “tumor formed by an overgrowth
of complete dental tissue”. —Broca
♦♦ Myxomatous changes—occurs in retroperitoneal lipoma.
In WHO classification of odontogenic tumors (2017)
♦♦ Saponification
odontomes are classified as benign mixed epithelial and
♦♦ Calcification—11% mineralization.
mesenchymal odontogenic tumor. Under this heading
♦♦ Submucosal lipoma can cause intussusception and so
two types of odontomas are there:
intestinal obstruction.
– Complex odontoma
Treatment – Compound odontoma.

♦♦ Excision should be done if lipomas are painful. Small Compound Odontome


lipoma is excised under local anesthesia and larger one It is defined as “A tumor like malformation (hamartoma) with
under general anesthesia. varying numbers of tooth like elements (odontoids)”. —WHO
♦♦ Percutaneous liposuction is relatively new treatment
option Pathogenesis
Q.14. Discuss differential diagnosis of submandibular It is produced by repeated divisions of tooth germ or by multiple
swelling. (Dec 2010, 8 Marks) budding off from dental lamina with formation of many tooth
Ans. Differential diagnosis of submandibular swellings. germs.
Submandibular swellings are of three types, i.e.
Clinical Features
• Acute
– Stones: Refer to ans 6 of chapter SALIVARY ♦♦ It is a painless slow growing lesion.
GLAND ♦♦ It occurs commonly in maxilla in incisor-canine region.
– Stenosis: Most commonly occur as complication ♦♦ Male and females are equally affected.
of ludwig’s angina ♦♦ It is seen during 2nd and 3rd decades of life.
– Sialectasia: It is an aseptic dilatation of salivary ♦♦ At times permanent teeth fails to erupt.
ductules causing grape like dilatation. Radiographic Features
It present as a smooth, soft, fluctuant, non-
transilluminating swelling which increases in Compound odontoma appears as collection of tooth like
size during mastication. It is tender initially. It structures surrounded by radiolucent zone.
last for many days being asymptomatic.
– Acute Lymphadenitis: Very often, poor oral Management
hygiene or a caries tooth produces painful, ♦♦ Surgical removal by enucleation is the treatment of choice.
tender, soft enlargement of these lymph nodes. ♦♦ Odontomas can be approached by intraoral mucosal incision
Extraction of tooth or with improvement of oral and removal of adequate overlying bone to expose lesion.
hygiene, lymph nodes regress. ♦♦ Removal of entire soft tissue portion is recommended to
• Recurrent acute prevent recurrence.
– Stones
– Stenosis Complex Odontome
– Sialectasis.
• Chronic It is defined as “a tumor like malformation (hamartoma) in
– Sialosis which enamel and dentin and sometimes cementum is present”.
– Autoimmune diseases  —WHO
– Neoplasms.
Pathogenesis
Chronic Tuberculous Lymphadenitis: Can affect these
nodes along with upper deep cervical lymph nodes. It develops from dental lamina or enamel organ in place of
These nodes are firm and matted. normal tooth.
368   Mastering the BDS IIIrd Year  (Last 25 Years Solved Questions)

Clinical Features Benign Tumors


♦♦ Painless slowly growing lesions. ♦♦ Fibroma: It is a painless, sessile, dome shaped or pe-
♦♦ They form hard masses. dunculated lesion with smooth contour. On tongue it is
♦♦ Mandible is more frequently affected. present as circumscribed nodule. Color of tumor is pink
♦♦ When the lesion becomes large it leads to expansion of and its surface is smooth consistency ranges from soft to
bone and facial symmetry. firm and elastic.
♦♦ Adjacent teeth may be displaced. ♦♦ Granular cell myoblastoma: It occur over the dorsum
of tongue. It is single, firm, submucosal nodule which is
Radiographic Features present within the substance of tongue.
It appear as irregular dense radiopaque mass surrounded by ♦♦ Glomus tumor: It is present over dorsum of tongue. Lesion
thin radiolucent area overlying displaced unerupted tooth. is small usually less than 1 cm. Color of lesion ranges from
red to purple. It is tender.
Management ♦♦ Leiomyoma: It is present in circumvallate papillae of
tongue. It is a slow growing painless lesion which is mostly
♦♦ Enucleation or curettage is done if odontoma is source of
pedunculated.
obstruction to erupting teeth.
♦♦ Rhabdomyoma: It is a well circumscribed tumor mass,
♦♦ Large complex odontomas should be cut in segments for
painless and growth is slow.
removal in order to conserve normal bone and prevent
♦♦ Neurofibroma: It has diffuse involvement over tongue.
jaw fracture.
Sessile or pedunculated elevated small nodules of various
Q.16. Write brief on odontomes. sizes are present. Due to its diffuse involvement it leads
 (Dec 2010, 5 Marks) (Dec 2009, 5 Marks) to macroglossia.
Ans. Refer to Ans 15 of same chapter. ♦♦ Keratoacanthoma: Lesion is tender, present as an elevated
umblicated or crateriform with depressed central core
Q.17. Write in brief tumors of tongue. (Dec 2009, 5 Marks)
which represent plug of keratin.
Ans. Following are the tumors of tongue:
♦♦ Neuroma: It presents as small swelling or nodule over
Benign the tongue.
♦♦ Papilloma: It is an exophytic lesion with cauliflower like
♦♦ Fibroma
surface. Projections are pointed or blunt. Appearance is
♦♦ Granular cell myoblastoma
due to presence of deep clefts.
♦♦ Glomus tumor
♦♦ Hemangioma: Part of tongue or entire tongue is in-
♦♦ Leiomyoma
volved. Loss of mobility of tongue. Color of tumor is
♦♦ Rhabdomyoma
bluish.
♦♦ Neurofibroma
♦♦ Lymphangioma: It involves dorsal and ventral surfaces
♦♦ Keratoacanthoma
of tongue. Presence of irregular nodularity over surface
♦♦ Traumatic neuroma
of tongue with grape like projections which are gray and
♦♦ Papilloma
pink in color.
♦♦ Pyogenic granuloma
♦♦ Adenoma Malignant Tumors
♦♦ Hemangioma
♦♦ Squamous cell carcinoma: Non-healing ulcer present
♦♦ Lymphangioma.
over the tongue. Edges of ulcer are everted and base is
indurated. Ulcer bleeds on touch.
Malignant ♦♦ Transitional cell carcinoma: Lesion arises from the base
of tongue. Lesion is ulcerated with granular eroded ap-
♦♦ Squamous cell carcinoma
pearance.
♦♦ Adenocarcinoma
♦♦ Malignant melanoma: A deep pigmented area is seen over
♦♦ Transitional cell carcinoma
the tongue which is ulcerated and hemorrhagic.
♦♦ Verrucous carcinoma
♦♦ Verrucous carcinoma: Pebbly surface is present which is
♦♦ Mucoepidermoid carcinoma
keratinized. Have rugae like folds with cleft in between.
♦♦ Reticular cell carcinoma
♦♦ Lymphosarcoma Q.18. Write short note on cavernous lymphangioma.
♦♦ Angiosarcoma (Dec 2010, 5 Marks)
♦♦ Kaposi’s sarcoma Ans. It is a congenital localized cluster of dilated lymph sacs in
♦♦ Melanoma the skin and sub-cutaneous tissue that has failed to join
♦♦ Rhabdomyosarcoma. the normal lymph system during development period.
Section 2:  General Surgery  369

Clinical Features Clinical Features


♦♦ It is present at birth. ♦♦ It begins with hoarseness of voice.
♦♦ It is common on face, mouth, lips and tongue. ♦♦ Pain is present while speaking or swallowing.
♦♦ Disfigurement is present over the child which is noticed ♦♦ Presence of dry cough
by his/her parents. ♦♦ Fever, malaise, restlessness and dyspnea
♦♦ Lesion is soft, lobulated and is fluctuant. ♦♦ Shortness of breadth
♦♦ It is a brilliant transilluminant larger lymphatic swelling ♦♦ Throat clearing
with multiple communicating lymphatic cysts. ♦♦ Laryngeal edema.
♦♦ It often extends in deeper plane like muscle. It is common
on face, mouth, lips and tongue Treatment

Differential Diagnosis ♦♦ Primary treatment involves resting of voice.


♦♦ For viral infection symptomatic care includes an analgesic
♦♦ Hemangioma: It occurs at posterior triangle of neck. Heman- and throat lozenges for pain relief.
gioma is soft, cystic and fluctuant. Its transillumination is ♦♦ Bacterial infections require antibiotic therapy, i.e. 250 mg
negative and sign of compressibility is positive. of cefuroxime twice daily.
♦♦ Lipoma: It is a soft lobular swelling with fluctuation. ♦♦ Severe acute laryngitis requires hospitalization.
While palpating edge of lipoma slips between palpating ♦♦ When laryngeal edema results in airway obstruction tra-
fingers. Both transillumination and compressibility tests cheostomy should be done.
are negative.
♦♦ Cold abscess: It is a soft, cystic, fluctuant swelling with nega- Q.21. Discuss briefly stridor. (May/Jun 2009, 5 Marks)
tive transillumination. It is located at the carotid triangle. Or
Write briefly on stridor. (Aug 2011, 5 Marks)
Management Ans. Stridor is an abnormal, high-pitched sound produced by
♦♦ Surgical excision of lesion is done for removing complete turbulent airflow through a partially obstructed airway
bulk of lesion. All the loculi or cysts should be removed. at the level of the supraglottis, glottis, subglottis, and/or
♦♦ As lymphangioma extends to the muscle plane, to avoid trachea.
recurrence careful examination should be done. • Stridor is a symptom, not a diagnosis or disease, and
the underlying cause must be determined.
Complications • Stridor may be inspiratory, expiratory, or biphasic
♦♦ As the size of lymphangioma is large, it can lead to dif- depending on its timing in the respiratory cycle.
ficulty in breathing in both neonates and infants. • Inspiratory stridor suggests a laryngeal obstruction,
♦♦ Secondary infection can occur in lymphangioma while expiratory stridor implies tracheobronchial
♦♦ In mediastinum it can lead to dyspnea, dysphagia because obstruction.
of compression of trachea. • Biphasic stridor suggests a subglottic or glottic
anomaly.
Q.19. Write short note on oral submucous fibrosis.
• In addition to a complete history and physical,
(Aug 2011, 5 Marks) as well as other possible additional studies, most
Or cases require flexible and/or rigid endoscopy to
adequately evaluate the etiology of stridor.
Write in brief on submucous fibrosis.
 (Feb 2013, 5 Marks) Pathophysiology
Ans. Refer to Ans 2 of same chapter.
Gases produce pressure equally in all directions; however,
Q.20. Write short note on acute laryngitis. (Aug 2011, 5 Marks) when a gas moves in a linear direction, it produces pressure in
Ans. Acute laryngitis may occur as an isolated infection or as a the forward vector and decreases the lateral pressure. When air
part of a generalized bacterial or viral upper respiratory passes through a narrowed flexible airway in a child, the lateral
tract infection. pressure that holds the airway open can drop precipitously (the
Bernoulli principle) and cause the tube to close. This process
Etiology obstructs airflow and produces stridor. Stridor may result from
lesions involving the CNS, the cardiovascular system, the GI
♦♦ Infection, i.e. viral system, and the respiratory tract.
♦♦ Excessive use of voice, i.e. in teaching, public speaking,
singing Etiology
♦♦ Leisure activities such as cheering at sport event Stridor may occur as a result of:
♦♦ Inhalation of smoke or fumes ♦♦ Foreign bodies (e.g., aspirated foreign body, aspirated
♦♦ Aspiration of caustic chemicals. food bolus);
370   Mastering the BDS IIIrd Year  (Last 25 Years Solved Questions)

♦♦ Tumor (e.g., laryngeal papillomatosis, squamous cell car- Developmental Lesions


cinoma of larynx, trachea or esophagus)
♦♦ Congenital lip pits
♦♦ Acute lymphatic leukemia (ALL) (T-cell ALL can present
♦♦ Commissural lip pits
with mediastinal mass that compresses the trachea and
♦♦ Double lip
causes inspiratory stridor)
♦♦ Cleft lip.
♦♦ Infections (e.g., epiglottitis, retropharyngeal abscess, croup)
♦♦ Subglottic stenosis (e.g., following prolonged intubation Inflammatory Lesions
or congenital)
♦♦ Glandular cheilitis
♦♦ Airway edema (e.g., following instrumentation of the air-
♦♦ Angular cheilitis
way, tracheal intubation, drug side effect, allergic reaction)
♦♦ Granulomatous cheilitis
♦♦ Subglottic hemangioma (rare) ♦♦ Contact cheilitis
♦♦ Vascular rings compressing the trachea ♦♦ Actinic cheilitis
♦♦ Many thyroiditis such as Riedel’s thyroiditis ♦♦ Eczematous cheilitis
♦♦ Vocal cord palsy ♦♦ Exfoliative cheilitis
♦♦ Tracheomalacia or tracheobronchomalacia (e.g., collapsed ♦♦ Plasma cell cheilitis
trachea) ♦♦ Cheilitis due to drugs.
♦♦ Congenital anomalies of the airway are present in 87% of
all cases of stridor in infants and children. Lip Carcinoma
♦♦ Vasculitis.
Miscellaneous
Treatment
♦♦ Actinic elastosis
Medical Care ♦♦ Lip ulcers.
♦♦ Immediate tracheal intubation should be done.
♦♦ Expectant management with full monitoring, oxygen by Developmental Lesions of Lip
facemask, and positioning the head on the bed for opti- Congenital Lip Pits
mum conditions (e.g., 45–90°).
♦♦ It is also known as congenital fistula.
♦♦ Use of nebulized racemic adrenaline epinephrine (0.5 to
♦♦ Pathogenesis: It occur due to failure of union of embryonic
0.75 mL of 2.25% racemic epinephrine added to 2.5 to 3 mL
sulcus of lip which leads to persistent lateral sulci on em-
of normal saline) in cases where airway edema may be the
bryonic mandibular arch.
cause of the stridor.
♦♦ Features: It more commonly occurs in females; vermilion
♦♦ Use of dexamethasone (Decadron) 4–8 mg IV, 8 - 12 hourly
border of lip is commonly involved. Lower lip is involved;
in cases where airway edema may be the cause of the
lesion is present in form of depression; on palpation mu-
stridor; note that some time (in the range of hours) may
cous secretion is seen from the base of lip pit.
be needed for dexamethasone to work fully. ♦♦ Treatment: Surgical excision is done.
♦♦ Use of inhaled Heliox (70% helium, 30% oxygen); the effect
is almost instantaneous. Helium, being a less dense gas Commissural Lip Pits
than nitrogen, reduces turbulent flow through the airways. ♦♦ They are mucosal invagination which arises at vermilion
Always ensure an open airway. border of lip.
♦♦ In obese patients elevation of the panniculus has shown ♦♦ Pathogenesis: Its occurrence is due to failure of normal fu-
to relieve symptoms by 80%. sion of embryonic maxillary and mandibular processes.
♦♦ Features: Males are commonly affected; it present as uni-
Surgical Care
lateral or bilateral pit at corners of mouth on vermilion
♦♦ Certain conditions, such as severe laryngomalacia, border. Its size ranges from a shallow depression to an
laryngeal stenosis, critical tracheal stenosis, laryngeal and open tract measuring 4 mm; On palpation less amount of
tracheal tumors and lesions (e.g., laryngeal papillomas, saliva ooze out.
hemangiomas, others), and foreign body aspiration, ♦♦ Treatment: Surgical excision is done.
require surgical correction.
Double Lip
♦♦ Occasionally, tracheotomy is done to protect the airway
to bypass laryngeal abnormalities and stent or bypass ♦♦ It is a fold of excessive tissue over inner mucosa of lip.
tracheal abnormalities. ♦♦ Pathogenesis: It arises during second week of gestation
♦♦ Other conditions, such as retropharyngeal and peritonsillar because of persistence of sulcus between pars glabrosa
abscess, may have to be dealt with on an emergent basis. and pars villosa of lip.
♦♦ Features: Inner aspect of lip is involved; at times when the
Q.22. Write briefly on lesions of lips. upper lip become tensed, double lip give appearance of
 (Dec 2009, 5 Marks) cupid bow.
Ans. Following are the lesions of lips: ♦♦ Treatment: Surgical excision is done.
Section 2:  General Surgery  371

Cleft Lip ♦♦ If tumor is more than 2 cm, wide excision is done. Exci-
sion of lower lip up to one-third can be sutured primarily,
♦♦ Cleft lip results from abnormal development of the median
in layers keeping vermilion border in proper apposition
nasal and maxillary process.
without causing any microstomia.
♦♦ Pathogenesis: This is due to imperfect fusion of maxillary
♦♦ Excision of more than one-third of the lip requires recon-
process with median nasal process which produce lateral
struction using different flaps.
cleft lip and due to failure of fusion of two median nasal
processes which produce central cleft lip. Q.23. Write short note on stridor and its emergency treatment.
♦♦ Features: Patient has difficulty in sucking; defective speech  (Aug 2012, 5 Marks)
is present, i.e. patient is unable to speak word such as B, Ans. Refer to Ans 21 of same chapter.
F, M, P and V. Q.24. Write short note on tongue tie. (Aug 2012, 5 Marks)
♦♦ Treatment: Millard’s criteria is use to undertake surgery for Ans. It is also known as ankyloglossia.
cleft lip, i.e. Rule of ten, i.e. 10 pound in weight; 10 weeks
Ankyloglossia is the condition which arises when the
old; 10 g% hemoglobin; Millard cleft repair by rotating
inferior frenulum attaches to the bottom of tongue and
local nasolabial flaps; proper postoperative management
subsequently restricts free movements of the tongue.
like control of infection, training for sucking, swallowing
and speech. Clinical Features
Inflammatory Lesions of Lip ♦♦ Males are affected more commonly than females
♦♦ Glandular cheilitis: Lower lip is mostly affected and the lip ♦♦ It can cause feeding problems in infants
become enlarged, become firm and get everted. It mainly ♦♦ Tongue movements become restricted.
occurs due to sun exposure. ♦♦ It causes speech defects specially articulation of the sounds
♦♦ Angular cheilitis: In it cracking of lips is seen from the cor- l,r,t,d,n,th,sh and z
ners. Main cause is nutritional deficiency. ♦♦ It leads to persistent gap between the mandibular incisors.
♦♦ Actinic cheilitis: It is a pre-malignant lesion. Occurs due to ♦♦ When attempt is made for sticking the tongue out a V
sun exposure. Lower lip is mostly affected. Lip become shaped notch is seen at the tip of tongue.
dry and scaly. If scales are removed bleeding points are ♦♦ As high frenal attachment is present and patient has
seen. periodontal problems.
♦♦ During protrusion, lateral margin and tip of tongue is
Lip Carcinoma everted with dorsal mid part heaping.
Vermilion border of lip and mucosa is the main site of carcinoma Management
of lip. It is common in the western elderly, white people,
especially those people exposed to sunlight. Tongue tie should be treated surgically under the local
anesthesia.
Clinical Features
Procedure
♦♦ Elderly males are affected in 90% of cases.
♦♦ Non-healing ulcer or growth is a common presentation. ♦♦ Local anesthesia should be given to the patient.
♦♦ Lesion appears in the form of white plaque of non-healing ♦♦ Retract the tongue and held the tongue by traction suture.
ulcers. This makes frenum taut and easily visible for surgical
♦♦ Edges are everted and indurated, this is characteristic of release.
♦♦ Take a sharp scissor and made a cut of 1 to 2 cm midway
carcinoma.
between the tip of tongue and lingual surface of mandible.
♦♦ Ulcer contain slough in floor.
Cut should be given in such a way that blade of scissor
♦♦ Bleeding may occur from ulcer.
should be parallel to floor of mouth.
♦♦ Pain and paresthesia may occur.
♦♦ Place a hemostat across frenal attachment at base of tongue
♦♦ Lesion may get fixed to the subcutaneous structure of lip.
and keep it clamped for 3 min. This provide bloodless
♦♦ Ulcer spreads and destroys the tissue of lip and chin.
field for surgery.
♦♦ Submental and submandibular lymph nodes are involved,
♦♦ As hemostat is removed, place an incision through area of
lymph node becomes hard and may be fixed.
previously closed hemostat.
Treatment ♦♦ Care should be taken not to injure submandibular duct,
papilla and blood vessels under floor of mouth.
♦♦ If lesion is less than 2 cm, then curative radiotherapy, ♦♦ Wound margins are carefully undermined should be
either brachytherapy or external beam radiotherapy. It approximated and closed without tension, i.e. in linear
gives a good cure. fashion.
372   Mastering the BDS IIIrd Year  (Last 25 Years Solved Questions)

Q.25. Describe diagnostic features and treatment of aphtous Complications


ulcers of tongue. (Jan 2012, 5 Marks)
♦♦ Ulceration
Ans. Diagnostic Features ♦♦ Serious hemorrhage.
Diagnosis of aphtous ulcers is made on basis of clinical
examination: Treatment
• Minor aphtous ulcers: Painful round ulcer with ♦♦ If swelling is small the treatment is curettage along with
yellow base and red margins. Size of the lesion is filling the cavity with cancellous bone chips.
0.4 to 1 cm. ♦♦ If tumor is large radical excision of bone is done along
• Major aphtous ulcers: Large and deep ulcers. Size with the grafting.
may reach to 5 cm. They restrict the mobility of
Q.27. Define, describe clinical features and principles of
tongue and uvula.
treatment of leukoplakia. (Jan 2017, 5 Marks)
• Herpetiform aphthous ulcers: These ulcer are
Ans. Leukoplakia is defined as a white patch or plaque that
numerous and very painful. They are small 1-2 mm
cannot be characterized clinically or pathologically
in diameter. They occur in crops.
as any other disease and is not associated with any
Treatment physical or chemical causative agent except use of
tobacco. First International Conference on oral
♦♦ Minor aphtous ulcers: Topical corticosteroid ointment is leukoplakia Malmo, Sweden (1984) or Modified WHO
applied over ulcer 4 to 5 times a day. Nutritional supple- Definition (1984)
ments should be given. Oral hygiene should be maintained.
♦♦ Major aphtous ulcer: Chlorhexidine gluconate should be Clinical Features
given. Triamcinolone acetate is locally applied, choline
♦♦ Usually the lesion occurs in 4th, 5th, 6th and 7th decades
salicylate gel is given.
of life.
♦♦ Herpetiform aphtous ulcer: Nutritional supplements
♦♦ Buccal mucosa and commissural areas are most frequent
should be given. Oral hygiene should be maintained.
affected sites followed by alveolar ridge, tongue, lip, hard
Chlorhexidine or tetracycline mouthwash will lead to
and soft palate, etc.
rapid heading.
♦♦ Oral leukoplakia often present solitary or multiple white
Q.26. Write short note on myeloid epulis. patches.
 (Apr 2015, 3 Marks) ♦♦ The size of lesion may vary from small well localized patch
Ans. It is also known as giant cell epulis. measuring few millimeters in diameter.
♦♦ The surface of lesion may be smooth or finely wrinkled or
Origin even rough on palpation and lesion cannot be removed
Two views are given over the origin of myeloid epulis: by scrapping.
1. It is an osteoclastoma arising from the peripheral part of ♦♦ The lesion is whitish or grayish or in some cases it is brown-
jaw and so present under the gums. ish yellow in color due to heavy use of tobacco.
2. It is an inflammatory hyperplasia. Granuloma extends in ♦♦ In most of the cases these lesion are asymptomatic, how-
the body of bone where it produces cyst like structure and ever in some cases they may cause pain, feeling of thickness
is surrounded by shell of bone giving the pseudo appear- and burning sensation, etc.
ance of osteoclastoma.
Principles of Treatment
Clinical Features ♦♦ Take thorough history and do proper clinical examination.
♦♦ Myeloid epulis is found on gingival margin between teeth ♦♦ Remove the causative factor
anterior to permanent molars. ♦♦ Excisional or incisional biopsy should be done.
♦♦ Swelling is round, soft and is maroon or purplish in color. ♦♦ Manage the lesion by oral medication or by eliminating
♦♦ Tumor is painless. the lesion
♦♦ Its consistency is firm and surface is smooth. ♦♦ Long term follow up should be done at every 3 months
interval.
♦♦ At each review submandibular and cervical lymph nodes
Investigations should be checked for signs of metastasis.
♦♦ After doing biopsy on microscopical examination there is Q.28. Name cysts and tumors of “lower jaw” with diagnostic
presence of fibrous tissue with rich vascularity and giant features and method to treat them.
cells of foreign body type.
♦♦ X-ray examination shows typical soap bubble appearance  (Jan 2017, 20 Marks)
of osteoclastoma. Ans. Cysts of lower jaw in see below Table.
Section 2:  General Surgery  373

Name of cyst in lower Diagnostic features Methods to treat


jaw
Dentigerous cyst –  Clinical diagnosis: Expansion of swelling in posterior area of Surgical removal: Small lesions are
mandible enucleated. Larger lesions undergo surgical
–  Radiological: Presence of well defined radiolucency associated drainage along with marsupialization.
with impacted teeth with hyperostotic border Decompression: Short section of rubber
–  Laboratory diagnosis: It is usually composed of thin connective is placed in preformed surgical opening in
tissue wall with a thin layer of stratified squamous epithelium cyst which remains open and allows the
lining the lumen. Rete peg formation is absent except in case drainage.
of secondarily infected cyst. Connective tissue wall is frequently
quite thickened and composed of very loose fibrous connective
tissue. Inflammatory cells commonly infiltrate connective tissue.
It also shows Rushton bodies within the lining epithelium which
are peculiar linear and often curved hyaline bodies
Odontogenic keratocyst Radiological diagnosis: Extension of radilucency in anteroposterior Enucleation: Enucleation is done with
direction along with the undulating border vigorous curettage
Laboratory diagnosis: Presence of corrugated or wrinkled parakeratin Peripheral osteotomy: It is done in bony
surface. Presence of prominent palisaded basal cell layer having cavity to reduce the chances of recurrence
picket fence or tombstone appearance. Connective tissue shows Chemical cauterization: Chemical
satellite or daughter cysts cauterization of bony cavity along with
intraluminal injection of carnoy’s solution is
done which leads to the freezing of cyst from
bony wall which allow easier removal of cyst
Decompression: This is done by polyethylene
drainage tube which is kept inside the bony
cavity
Paradental cyst –  Radiological features: Radiolucency is present which is distal to Surgical enucleation is done
the third molar with intact PDL space
–  Laboratory diagnosis: Proliferating, non-keratinized squamous
epithelium lines the cyst. Fibrous capsule consists of mixed or
chronic inflammation
Solitary bone cyst –  Radiological diagnosis: Presence of well-defined radiolucency Enucleation or curettage should be done.
with vital tooth and there is history of trauma. Intralesional steroid injection proved to be
–  Laboratory diagnosis: Fragments of fibrin with enmeshed RBCs successful
are seen. Hemorrhage and hemosiderin pigments are seen
Mandibular buccal –  Clinical diagnosis: Patient is young, site is mandibular molar, Enucleation of cyst is done without removing
infected cyst buccal periostitis, vital pulp is present the associated tooth
–  Radiological diagnosis: Lamina dura is intact with expansion of
buccal cortical plates
Lateral periodontal cyst –  Clinical diagnosis: Normal color swelling visible at canine or Surgical excision is done along with removal
premolar region of the offending tooth
–  Radiological: Oval shaped radiolucency present between roots
of teeth along with hyperostotic borders
–  Laboratory diagnosis: Cyst is lined by the stratified squamous
epithelium with connective tissue wall. Lumen of cyst shows focal
thickened plaque of proliferating lining cell
Gingival cyst of adult –  Clinical diagnosis: Presence of blue color swelling of gingiva. It Surgical excision of the lesion is done
is dome shaped.
–  Radiological: Bony erosion is evident
–  Laboratory diagnosis: Cyst is lined by the stratified squamous
epithelium with connective tissue wall. Lumen of cyst shows focal
thickened plaque of proliferating lining cell
Dental lamina cyst –  Clinical diagnosis: Soft tissue elevation is present on alveolar No treatment is needed
ridge in infant which is white in color
–  Laboratory diagnosis: Presence of thin epithelial lining, shows
lumen filled with desquamated keratin
Glandular odontogenic –  Radiological features: Presence of multilocular appearance with Enucleation or curettage should be done
cyst scalloped margin
–  Laboratory diagnosis: Histologically a cystic space is present
lined by non-keratinized epithelium. Mucus as well as cylindrical
cells form part of epithelial component with mucinous material
in cystic space
374   Mastering the BDS IIIrd Year  (Last 25 Years Solved Questions)

Tumors of lower jaw

Name of tumor in Diagnostic features Methods to treat


lower jaw
Ameloblastic fibroma Radiological diagnosis: CT scan can be helpful Aggressive surgical excision should be
Laboratory diagnosis: Presence of scattered islands of epithelial cell carried out
in a rosette form. Cells are present which resemble like ameloblasts.
Mesenchymal component is of primitive connective tissue
Calcifying epithelial Radiological diagnosis: Typical snow driven appearance is present Conservative surgical excision is done or
odontogenic tumor Laboratory diagnosis: Presence of polyhedral epithelial cells present in simple enucleation can also be done
large sheets. Calcification is present in form of Liesegang rings
Dentinoma Radiological diagnosis: Presence of radiopaque mass along with crown Surgical excision is done along with thorough
of tooth curettage
Laboratory diagnosis: Presence of irregular dentin in biopsy which is
known as dentinoid
Complex odontomea Radiological diagnosis: It shows disorganized mass of tooth structure. Local surgical excision is done
Laboratory diagnosis: Mature tubular dentin is seen in biopsy
Ameloblastoma Clinical diagnosis: Presence of swelling in the posterior mandible along
with bony expansion and egg shell crackling Ameloblastomas are best treated by
Radiological diagnosis: Presence of characteristic honeycomb resection of the lesion with a marginal
appearance, soap bubble appearance in posterior mandible along clearance of 1.5–2 cm of normal bone to
with bony expansion prevent recurrence.
Laboratory diagnosis: Histopathologically various types of The lesion may be resected as block
ameloblastomas are identified, i.e. follicular (Islands of tumor cells resection with or without continuity defect
with peripheral layer of cuboidal and columnar ameloblast like cells), based on the integrity of inferior cortex.
plexiform (Presence of network of interconnecting strands of cells, Radiologically a minimum of 1 cm residual
bounded by layer of columnar cells and between these layers stellate mandible inferior cortex is required
reticulum like cells are present), acanthomatous type, granular type, postoperatively to prevent pathologic
basal type and unicystic. fracture.
Inferior alveolar nerve should be sacrificed
if it lies within the lesion.

Squamous odontogenic Radiological diagnosis: Presence of semicircular radiolucency between Conservative enucleation along with
tumor the roots curettage is done
Laboratory diagnosis: Histopathology shows presence of islands of
squamous epithelium without peripheral palisaded polarized columnar
cells
Odontogenic myxoma Clinical diagnosis: Presence of missing tooth along with hard swelling Surgical excision is done along with resection
is suggestive of this disease of proper amount of bone
Radiological diagnosis: Tennis racquet pattern is characteristic.
Laboratory diagnosis: Presence of loosely arranged spindle shaped as
well as stellate cells majority of which have long fibrillar process that
tend to intermesh
Peripheral odontogenic Clinical diagnosis: Attachment of sessile mass to gingiva Surgical excision is done
fibroma Laboratory diagnosis: Histopathology shows cellular fibrous connective
tissue parenchyma along with the non – neoplastic islands, strands and
cords of columnar or cuboidal cells
Granular c e l l Radiological diagnosis: Inside the unilocular radiolucency calcification Simple curettage is done
odontogenic tumor is seen.
Laboratory diagnosis: Histopathology show large eosinophilic granular
cells. Small islands of odontogenic epithelium are scattered in area of
granular cells.
Periapical cemental Radiological diagnosis: In early stage there is presence of radiolucency Surgical excision of large lesions should be
dysplasia at apex of vital teeth with no loss of lamina dura. In mature stage carried out
radiopaque lesion is visible
Laboratory diagnosis: Histopathology shows periapical bone is replaced
by the mass of fibrous tissue, Various small round to oval calcifications
are seen in fibrous tissue
Contd...
Section 2:  General Surgery  375

Contd...

Name of tumor in lower jaw Diagnostic features Methods to treat


Benign cementoblastoma Radiological diagnosis: Surrounding of radiopaque lesion by Surgical excision of tumor along with the
radiolucent capsule attached to root surface. involved tooth
Laboratory diagnosis: In histopathology there is presence of Surgical excision of tumor mass along with
cementum like tissue which is deposited in globular pattern root amputation and endodontic treatment
is done

Q.29. Write long answer on types and causes of oral ulcer ♦♦ The ulcer appears crateriform (owing to its increased
and their treatment. (Apr 2018, 5 marks) depth), and it heals with scar formation in about 6 weeks
Ans. Aphthous ulcer is the most common type of non- time.
traumatic, ulcerative condition of the oral mucosa. ♦♦ Few lesions may look like malignant ulcers, moreover
sometimes these lesions occur in association with HIV
Types of oral ulcer infections.
Clinically, aphthous ulcers present three recognizable forms, ♦♦ Major aphthous ulcers often become secondarily infected
namely: and in such cases, the healing process is further delayed.
1. Minor aphthous ulcers
Herpetiform Ulcers
2. Major aphthous ulcers
3. Herpetiform ulcers. ♦♦ Herpetiform type of aphthous ulcers produce recurrent
crops of extremely painful, small ulcers in the oral mucosa,
Minor Apthous Ulcer
which resemble herpetic ulcers. However, these ulcers do
♦♦ It is the most common type of aphthous ulcer of the oral not develop following vesiculations and exhibit no virus
cavity and it appears episodically either as single lesion infected cells.
or in clusters of l to 5 lesions. ♦♦ Their numbers vary from few dozens to several hundreds
♦♦ The ulcers are very painful, shallow, round or elliptical in and each ulcer is surrounded by a wide zone of erythema.
shape and they measure about 0.5 cm in diameter with a ♦♦ Size of these ulcers ranges between l to 2 mm in diameter
crateriform margin. only. However on few occasions, small ulcers coalesce
♦♦ The lesion is usually surrounded by an erythematous together to form large irregular ulcers.
“halo” and is covered by a yellowish, fibrinous membrane. ♦♦ The ulcers last for several weeks or months.
♦♦ Minor aphthous ulcers mostly develop over the nonkerati- ♦♦ Children in their late teens often suffer from this disease
nized mucosa, e.g. lips, soft palate, anterior fauces,floor of and the lesions occur in both gland bearing mucosa as well
the mouth and ventral surface of the tongue (gland bearing as over keratinized mucosa.
mucosa), etc. ♦♦ The lesions usually heal up within l to 2 week time.
♦♦ The ulcer lasts for about 7–10 days and then heals up
without scarring but recurrence is common. Causes of Oral Ulcer
♦♦ New lesions may continue to appear during an attack for
about 3–4 weeks period. The exact etiology is not known and only the probable factors
♦♦ Few lesions may be present in the mouth almost continu- have been identified which are as follows:
ously. ♦♦ Genetic predisposition: The disease often affects several
members of the same family and moreover identical twins
Major Apthous Ulcers
are most frequently affected.
♦♦ Major aphthous ulcers are less common than the minor ♦♦ Exaggerated response to trauma: The ulcer develops in
form of the disease. those mucosal sites which are subjected to trauma in the
♦♦ These are larger, 0.5 cm in diameter and can be as big as past. e.g. tooth prick injury.
several centimeters in diameter. ♦♦ Immunological factors: The disease may occur due to some
♦♦ Major aphthous ulcers are more painful lesions than the autoimmune reactions, or in patients with immunosup-
minor variety; and they persist in the mouth for longer pression, e. g. AIDS. Some investigators believe, then it is
durations as they take more time to heal. an immune complex-mediated Type III or cell mediated
♦♦ These lesions are considered to be the most severe among type IV reaction.
all types of aphthae and they often make the patients ill. ♦♦ Microbiologic factors: The disease may be caused by herpes
♦♦ Only one or two lesions develop at a time and are mostly simplex virus Type I or S. sanguinis.
seen over the lips, soft palate and fauces, etc. Besides in- ♦♦ Nutritional factors: Deficiency of vitamin B12, folate and
volving the non-keratinzed mucosa, major aphthous ulcers iron, etc. often reported in patients with aphthous ulcer;
can involve the masticatory mucosa as well, such as the moreover supplementation of these elements may cause
dorsum of the tongue and gingiva, etc. rapid recovery.
376   Mastering the BDS IIIrd Year  (Last 25 Years Solved Questions)

♦♦ Systemic conditions: Behcet’s syndrome, Crohn’s disease ♦♦ Erythroplakia: It is a red velvety lesion which has incidence
and celiac disease are associated with increased incidences of malignancy till around 15%. It is 17 to 20 times more
of aphthous ulcer. potentially malignant than leukoplakia
♦♦ Hormonal imbalance: Hormonal change during menstrual ♦♦ Oral submucus fibrosis: 4.5 to 7.6% of oral submucus fi-
cycle may be associated with higher incidence of aphthous brosis cases turn into malignancy. For details refer to ans
ulcer. 2 of same chapter
♦♦ Non-smoking: The disease almost exclusively occurs in ♦♦ Syphilitic glossitis: Tertiary syphilis leads to chronic su-
nonsmokers or the people those who have given up smoke perficial glossitis which leads to carcinoma of tongue. This
♦♦ recently. is rare nowadays.
♦♦ Allergy and chronic asthma: Allergic manifestations to any ♦♦ Dyskeratosis congenita: Reticular atrophy, nail dystrophy
medicines or foods (e.g. Nuts and chocolates, etc.) may and leukoplakia in oral cavity.
lead to the development of aphthous ulcer. ♦♦ Sideropenic dysphagia: It is common in Scandinavian
♦♦ Miscellaneous factors: Stress and anxiety. females. It leads to atrophy of epithelium and become
potentially malignant. Proper iron therapy controls the
Treatment disease and reduces the risk.
Medicinal treatment ♦♦ Chronic hyperplastic candidiasis: It is common in com-
misures of moth and tongue. Dense plaque of leukopla-
♦♦ Topical corticosteroid i.e. 0.1% triamcinolone acetonide
kia is common with curdy white patches due to candida
QDS is effective on daily use.
albicans infection. It is treated by systemic anti – fungals
♦♦ Topical anesthetics i.e. 2% viscous lidocaine, benzocaine
or surgical excision or laser therapy.
and benzydamine hydrochloride can reduce pain. Topical
♦♦ Papilloma of tongue or cheek
protective emollient base can be given.
♦♦ Discoid Lupus erythematosus
♦♦ Topical application of sucralfate 4 times a day has soothing
♦♦ Oral lichen planus
effect on an ulcer.
♦♦ Topical tetracycline mouthwash QDS for 5 to 7 days pro-
vides good response.
♦♦ Beclomethasone spray is given in severe cases. 14. Cleft Lip and Palate
♦♦ In resistant cases systemic steroids can be given. Most com-
monly beclomethasone or prednisolone syrup in swish and Q.1. Write short note on cleft lip. (June 2015, 5 Marks)
swallow method is advised. In some cases prednisolone  (Jan 2012, 5 Marks) (May/June 2009, 5 Marks)
tabelet 20 to 30mg/day and beclomethasone 2 to 3 mg per  (Dec 2007, 3 Marks) (Apr 2007, 5 Marks)
day for 4 to 8 days can be given.  (Feb/Mar 2004, 5 Marks) (Mar 2003, 5 Marks)
 (Mar 2016, 5 Marks) (Sep 2001, 5 Marks)
Surgical treatment Or
Laser surgery Describe briefly cleft lip  (Apr 2017, 5 Marks)
Surgical removal of apthous ulcer should also be used. Laser Or
ablation shortens the duration and decreases the associated Write in short about cleft lip. (July 2016, 5 Marks)
symptoms. CO2 or Nd:YAG lasers are used. Laser treatment Ans. Cleft lip results from abnormal development of the
requires frequent visits. median nasal and maxillary process.
Local cauterization
Causes
Application of 0.5% hydrogen peroxide, 1 to 2% of silver nitrate
♦♦ Increase parenteral age.
or silver nitrate caustic stick represents several older therapeutic
♦♦ Infection during pregnancy.
methods which reduce duration of solitary oral ulcer.
♦♦ Smoking and steroid therapy during pregnancy.
Q.30. Write short answer on premalignant conditions of oral ♦♦ Trauma and psychological stress during pregnancy.
cavity. (Apr 2018, 3 Marks)
Ans. The premalignant condition is defined as “A generalized Types of Cleft Lip
state of body, which is associated with significantly
increased risk of cancer development”. ♦♦ Central: It is very rare and occurs due to failure of fusion
Following are the premalignant conditions of oral cavity: of two median nasal processes (Hare lip).
♦♦ Leukoplakia: Its incidence in those who smoke or chew ♦♦ Lateral: It is the most common variety, there is a cleft
pan is 20% while its incidence in non – smokers is 1%. between the frenum and the lateral part of the upper lip.
Its incidence in turning malignancy is 4 to 10% which in- This is due to imperfect fusion of maxillary process with
creases with age, duration of pan chewing and smoking. median nasal process. Lateral variety can be unilateral
For details refer to Ans 1 of same chapter or bilateral.
Section 2:  General Surgery  377

♦♦ Proper postoperative management like control of infec-


tion, training for sucking, swallowing and speech therapy
should be done.

Principles of Cleft Lip Repair

♦♦ Rule of 10 should be fulfilled.


♦♦ Before six months it should be operated.
♦♦ Infection should not be present.
Fig. 20:  Cleft lip ♦♦ Millard advancement flap is used for unilateral cleft lip
repair.
♦♦ Complete or incomplete: In case of complete variety, cleft lip
♦♦ Bilateral cleft lip repair can be done either in single or
extend to the nose. In case of incomplete variety the cleft
two stages.
does not extend up to the nostrils. ♦♦ One stage bilateral cleft lip repair is done using Veau III
♦♦ Simple or compound: Simple cleft lip is only cleft in the lip method.
while compound refers to cleft lip associated with a cleft ♦♦ Proper markings are made prior to surgery and incision
in the alveolus. should be over full thickness flap.
Clinical Features ♦♦ Often 1:2,00,000 adrenalin injection is used to achieve
hemostasis.
♦♦ Patient has difficulty in sucking. ♦♦ Three layer lip repair should be done, i.e. mucosa, muscle
♦♦ Defective speech is present, i.e. patient is unable to speak and skin.
word such as B, F, M, P and V ♦♦ Cupid’s bow should be horizontal.
♦♦ Presence of soft tissue mass in between the ends of bone ♦♦ Continuity of white line is maintained.
which unite tongue to lip. ♦♦ Vermilion notching should not be present.
♦♦ In hare lip cleft lies in middle of lip.
♦♦ In incomplete cleft lip extension is from nostril to some Q.2. Write short note on cleft palate. (Feb 2015, 5 Marks)
distance forward  (2012, 5 Marks) (June 2010, 5 Marks)
♦♦ In complete cleft lip extension is from nostril to palate.  (Apr 2008, 5 Marks) (Oct 2007, 5 Marks)
Ans. The palate is formed from the Y- shaped fusion of
Treatment premaxilla and two palatine processes.
Imperfect fusion of these processes or developmental
♦♦ Millard’s criteria is use to undertake surgery for cleft lip,
anomalies results in cleft palate.
i.e. Rule of ten, i.e. 10 pound in weight; 10 weeks old; 10
g% hemoglobin.
Types
♦♦ If cleft lip is bilateral and is extensive two surgeries should
be done to close the cleft. Surgery of one side should be ♦♦ Complete cleft palate
done first and later on after few week surgery of second • Failure of fusion of palatine process and premaxilla
side is done. result in complete cleft palate.
♦♦ Millard’s rotation advancement flap technique is commonly • The nasal cavity and the mouth are interconnected.
used. In this correction of both lip and nasal deformity is • This may be unilateral or bilateral.
done. Realigning of muscles and of lip and their correct ♦♦ Incomplete cleft palate
anatomical position is an important part of this repair. • When the fusion of three component of palate takes
In this method a tilted cupid bow is rotated downwards place.
following curved incision extending to columella. This • It starts from uvula and then backwards.
allows rotation of misplace cupid’s bow and philtral
♦♦ Various types
dimple in normal position. High rotation gap created is
• Bifid uvula
filled with triangular flap which consists of skin muscle
• The whole length of soft palate is bifid.
and mucosa from upper part of lateral side of cleft. The
• The whole length of soft palate and posterior part of
resultant scar is hidden inside the nose or follows natural
hard palate are involved.
line of philtrum.
♦♦ Hagedorn – LeMesurier repair: In this method medial lip
Effects of Cleft Palate
element should be lengthened by introducing the quad-
rilateral flap which is developed from lateral lip element. ♦♦ Interferers with swallowing and speech.
♦♦ Tennison – Randall repair: In this a cut is given on lower ♦♦ Unable to make the constant sound like B, P, D, K and T.
one-third of lip to correct upward tilt of cupid’s bow by ♦♦ Teeth: Upper incisors may be small maxilla tends to be
placing a Z-shape wire. Gap is filled with triangle of skin, smaller. Teeth are crowded.
muscle and mucosal flap from lower end of lateral lip ele- ♦♦ Nose: Oral organisms contaminate the upper respiratory
ment. In this resultant scar is Zigzag scar. mucous membrane.
378   Mastering the BDS IIIrd Year  (Last 25 Years Solved Questions)

♦♦ Hearing: Even with repair, acute and chronic hearing cleft are malposed, often severely rotated, and poorly
problem can occur. calcified. They certainly need to be orthodontia. If
extracted, specially the super­numerary one.
Management of Cleft Palate
Even with complete orthodontic treatment, there is
♦♦ Cleft palate is usually repaired in l2–l8 months. Early maxillomandibular *discrepancy, which may need
repair causes retarded maxillary growth. Late repair causes surgical correction in the form of maxillary advance­ment
speech defect. with or without mandibular pushback, with or without
♦♦ Both soft and hard palates are repaired. *genioplasty.
♦♦ Abnormal insertion of tensor palati is released. Mucoperi-
osteal flaps are raised in the palate which is sewed together. Q.4. Discuss about treatment of cleft palate. 
♦♦ If maxillary hypoplasia is present, then osteotomy of the  (Mar 2008, 5 Marks)
maxilla is done. With orthodontic teeth extraction and Ans. Refer to Ans 2 of same chapter.
alignment of dentition is done.
♦♦ Regular examination of ear, nose and throat during follow-
up period, i.e. postoperative speech therapy. Q.5. Write about pathogenesis, classification, structural and
♦♦ Whenever complicated problems are present, staged functional problems of cleft lip and palate. Write note
surgical procedure is done. on principles of management.
♦♦ Wardill-Kilner push back operation or V—Y pushback  (Feb 2013, 10 Marks)
palatoplasty by raising mucoperiosteum flaps based on Ans.
greater palatine vessels.
Pathogenesis of Cleft Lip and Cleft Palate
V—Y Pushback Palatoplasty Or Wardill – Kilner Push-
Back Palatoplasty ♦♦ Cleft lip results from abnormal development of medial
nasal process and maxillary process.
♦♦ In this palate is infiltrated by 1:2 adrenaline saline solution. ♦♦ Cleft palate occurs due to fusion of two palatine process.
♦♦ Both mucoperiosteal flaps are raised, one from either ♦♦ Defect in fusion of lines between premaxilla and palatine
side of palatal shelves and then nasal layers should be processes of maxilla one on each side.
mobilized. ♦♦ When premaxilla and both palatine processes do not fuse,
♦♦ Closure of palate is done in three layers, i.e. nasal layer, it leads to complete cleft palate.
muscle layer and oral layer. ♦♦ Incomplete fusion of all three components lead to incom-
♦♦ In this procedure palatal lengthening is achieved by V – Y plete cleft palate.
plasty. Hook of hamulus can be fractured to relieve tension
on suture line by relaxing tensor palate muscle. Classification of Cleft Lip and Cleft Palate
Refer to Ans 6 same chapter.
Secondary Management
♦♦ Hearing support is given using hearing aids if defect is Functional and Structural Problems of Cleft Lip and
present; control of otitis media. Cleft Palate
♦♦ Speech problems occur due to velopharyngeal incompe- ♦♦ Difficulty in sucking and swallowing
tence; articulation problems also can occur. Speech therapy ♦♦ Speech is defective especially in cleft palate, mainly to
is given. It is corrected by pharyngoplasty, veloplasty, phonate B,D,K,P,T and G
speech devices. ♦♦ Altered dentition or supernumerary teeth
♦♦ Dental problems like uneruption, unalignment are com- ♦♦ Recurrent upper respiratory tract infection
mon. They should be corrected by proper dentist opinion, ♦♦ Respiratory obstruction
and reconstructive surgery. ♦♦ Hypoplasia of maxilla
♦♦ Orthodontic management with alveolar bone graft, maxil- ♦♦ Cosmetic problem
lary osteotomy is done in 8–11 years. ♦♦ Chronic otitis media and middle ear problems.
♦♦ Veloplasty, dental implants, rhinoplasty, orthognathic
Management of Cleft Lip and Cleft Palate
surgeries etc.
For management of cleft lip refer to ans 1 and for management
Q.3. Write short note on orthodontic treatment of cleft lip
of cleft palate refer to Ans 2 of same chapter.
and palate. (Sep 2004, 5 Marks)
Ans. One of the clinical features common to cleft lip and Q.6. Write briefly on classification of cleft lip and palate
palate is constricted and distorted maxillary arch. The defects. (Dec 2009, 5 Marks)
severe the cleft, more severe the arch deformity due to Ans. Following are the classifications:
collapse. Orthodontic treatment is necessary to correct
Davis and Ritchie (1922)
the deformity.
Orthodontic treatment should be started during mixed Group I: Prealveolar cleft.
dentition, and continued through the permanent Group II: Post alveolar cleft.
dentition. Permanent teeth specially, those adjacent to Group III: Alveolar cleft.
Section 2:  General Surgery  379

extending through the alveolus at the position of the future


lateral incisor tooth
♦♦ Group IV: Complete bilateral cleft, resembling Group
III with two clefts extending from the incisive foramen
through the alveolus.

Kernahan and Stark (1958)


♦♦ Incomplete cleft of secondary palate
♦♦ Complete cleft of secondary palate
♦♦ Incomplete cleft of primary and secondary palate
♦♦ Unilateral complete cleft of primary and secondary palate.
♦♦ Bilateral complete cleft of the primary and secondary
palate.

Harkins and Associates (1962)


Fig. 21:  Davis and Ritchie classification
♦♦ Cleft or primary palate:
• Cleft lip
–– Unilateral: Right, left (Extent; one-third, two-
thirds, complete)
–– Bilateral: Right, left (Extent: one—third, two-
thirds, complete)
–– Median (Extent: one-third, two-third, complete)
–– Prolabium: Small, medium, large
–– Congenital scar: Right, left, median (one-third,
two-thirds, complete)
• Cleft of alveolar process
–– Unilateral: Right, left (Extent: one-third, two-
thirds, complete)
–– Bilateral: Right, left (Extent: one-third, two-thirds,
Fig. 22: Veau classification complete)
–– Median (Extent: one-third, two-thirds, complete)
–– Submucous: Right, left, median
–– Absent incisor tooth
♦♦ Cleft of palate:
• Soft palate
–– Posteroanterior: One-third, two-thirds, complete
–– Width: Maximum
–– Palatal shortness: None, slight, moderate, marked
–– Submucous cleft (Extent: one-third, two-thirds,
complete)
• Hard palate
–– Posteroanterior (Extent: one-third, two-thirds,
complete)
–– Width: Maximum (mm)
–– Vomer attachment: Right, left, absent
Fig. 23:  Kernahan and Stark classification –– Submucous cleft (Extent: one-third, two-thirds,
complete)
Veau (1931) ♦♦ Mandibular process clefts:
♦♦ Group I: Cleft of soft palate only • Lip extent: One-third, two-thirds, complete
♦♦ Group II: Cleft of the hard and soft palate extending no • Mandible (Extent: one-third, two-thirds, complete)
further than the incisive foramen, thus involving the sec- Lip pits: Congenital lip sinuses
ondary palate alone. ♦♦ Naso-ocular: Extending from the nasal region towards the
♦♦ Group III: Complete unilateral cleft, extending from uvula medial canthus
to the incisive foramen, thus the uvula to the incisive fora- ♦♦ Oro-ocular: Extending from angle of the mouth towards
men in the midline, then deviating to one side and usually the palpebral fissure
380   Mastering the BDS IIIrd Year  (Last 25 Years Solved Questions)

♦♦ Oro-aural: Extending from the angle of the mouth towards ♦♦ The mesodermal basis of the median part of the lip (called
the tragus of ear. philtrum) is formed from the frontonasal process. The
ectoderm of the maxillary process however, overgrows this
Q.7. Describe the development of face and various
mesoderm to meet that of the opposite maxillary process
congenital abnormalities of lip and palate.
in the midline. As a result, the skin of the entire upper lip
 (Dec 2010, 10 Marks)
is innervated by maxillary nerves.
Or The muscles of the face are derived from mesoderm of second
Write briefly on development of face. branchial arch and are therefore supplied by the facial nerve.
 (Feb 2013, 5 Marks)
Ans. Nose
Nose receives contributions from the frontonasal process,
Development of Face and from the medial and lateral nasal processes of the right
♦♦ Face is derived from the following structures that lie and left sides. External nares are formed when the nasal
around the stomatodeum, i.e. pits are cut-off from the stomatodeum by the fusion of the
• Frontonasal process maxillary process with the medial nasal process. External nares
• The first pharyngeal (or mandibular) arch of each side. gradually approach each other. Mesoderm becomes heaped
♦♦ At this stage each mandibular arch forms the lateral wall up in the median plane to form the prominence of the nose.
of the stomatodaeum. Simultaneously, groove appears between the regions of the nose
♦♦ This arch gives off a bud from its dorsal end. This bud and the bulging forebrain. As the nose becomes prominent the
is called the maxillary process. It grows ventro-medially external nares come to open downwards. The external form of
cranial to the main part of the arch which is now called the nose is thus established.
the mandibular process.
♦♦ The ectoderm overlying the frontonasal process soon Cheeks
shows bilateral localized thickenings, that are situated a After the formation of the upper and lower lips, the stomatodeum
little above the stomatodeum. These are called the nasal (which can now be called the mouth) is very broad. In its lateral
placodes. The formation of these placodes is induced by part, it is bounded above by the maxillary process and below by
the underlying forebrain. The placodes soon sink below the mandibular process. These processes undergo progressive
the surface to form nasal pits. fusion with each other to form the cheeks.
♦♦ The pits are continuous with the stomatodeum below. The
edges of each pit are raised above the surface. The medial Eye
raised edge is called the medial nasal process and the
The region of the eye is first seen as an ectodermal thickening,
lateral edge is called the lateral nasal process.
the lens placode, which appears on the ventro-lateral side of the
Lower Lip developing forebrain, lateral and cranial to the nasal placode.
The lens placode sinks below the surface and is eventually
Mandibular processes of the two sides grow towards each other
cut- off from the surface ectoderm. The developing eyeball
and fuse in the midline. They now form the lower margin of
produces a bulging in this situation. The bulging of the eyes
the stomatodeum. If it is remembered that the mouth develops
are at first directed laterally and lie in the angles between
from the stomatodeum, it will be readily understood that the
the maxillary processes and the lateral nasal processes. With
fused mandibular processes give rise to the lower lip and to
the narrowing of the frontonasal process they come to face
the lower jaw.
forwards. The eyelids are derived from folds of ectoderm
Upper Lip formed above and below the eyes, and by mesoderm enclosed
within the folds.
Each maxillary process now grows medially and fuses, first with
the lateral nasal process and then with the medial nasal process. External Ear
The median and lateral nasal processes also fuse with each
other. In this way the nasal pits are cut-off from stomatodeum. External ear is formed around the dorsal part of the first
ectodermal cleft. A series of mesodermal thickenings (often
The maxillary processes undergo considerable growth. At the
called tubercles or hillocks) appear on the mandibular and
same time the frontonasal process becomes much narrower
hyoid arches where they adjoin this cleft. The pinna (or auricle)
from side to side, with the result that the two external nares
is formed by fusion of these thickenings.
come close together.
The stomatodeum is now bounded above by the upper lip which Nasal Cavities
is derived as follows:
♦♦ The mesodermal basis of the lateral part of the lip is formed Nasal cavities are formed by extension of the nasal pits. These
from the maxillary process. The overlying skin is derived pits are at first in open communication with the stomatodeum.
from ectoderm covering this process. Soon the medial and lateral nasal processes fuse and form a
Section 2:  General Surgery  381

partition between the pit and the stomatodeum. This is called The nasal pits now deepen to form the nasal sacs which
the primitive palate and is derived from the frontonasal expand both dorsally and caudally.
process.

Fig. 24:  Development of face

The dorsal part of this sac is, at first, separated from the of this membrane it has a free lower edge. The nasal cavities
stomatodeum by a thin membrane called the bucconasal are separated from the mouth by the development of the
membrane (or nasal fin). This soon breaks down. palate.
The nasal sac now has a ventral orifice that opens on the face The lateral wall of the nose is derived, on each side, from the
(anterior or external nares) and a dorsal orifice that opens into lateral nasal process. The nasal conchae appear as elevations on
the stomatodeum (primitive posterior nasal aperture). the lateral wall of each nasal cavity.
The two nasal sacs are at first widely separated from one The original olfactory placodes form the olfactory epithelium
another by the frontonasal process. However, the frontonasal that lies in the roof, and adjoining parts of thin walls, of the nasal
process becomes progressively narrower. This narrowing of the cavity.
frontonasal process, and the enlargement of the nasal cavities
themselves, bring them closer together. Congenital Abnormalities of Lip and Palate
This intervening tissue becomes much thinned to form the Following are the congenital abnormalities of lip and palate:
nasal septum. ♦♦ Congenital lip pits
The ventral part of the nasal septum is attached below to ♦♦ Commissural lip pits
the primitive palate. More posteriorly the septum is at first ♦♦ Double lip
attached to the bucconasal membrane, but on disappearance ♦♦ Cleft lip and cleft palate.
382   Mastering the BDS IIIrd Year  (Last 25 Years Solved Questions)

Congenital Lip Pits Cleft Lip and Cleft Palate


It is also known as congenital fistula. For cleft lip refer to ans 1 and for cleft palate refer to Ans 2 of
same chapter.
Pathogenesis
Q.8. Write short note on management of a case of unilateral
It occur due to failure of union of embryonic sulcus of lip which
cleft lip and palate repair. (Apr 2015, 3 Marks)
leads to persistent lateral sulci on embryonic mandibular arch.
Ans.
Clinical Features
Management of a Case of Unilateral Cleft Lip Repair
♦♦ It more commonly occur in females.
♦♦ For unilateral cleft lip repair most commonly used methods
♦♦ Vermilion border of lip is commonly involved. Lower lip
are Millard’s rotation advancement flap and Tennison –
is involved. Randall triangular flap method.
♦♦ Lips appear swollen ♦♦ Millar’s cleft lip repair is done by rotating local nasolabial
♦♦ Lesion is present in form of depression. flaps.
♦♦ On palpation mucous secretion is seen from the base of ♦♦ Majority of surgeons follow Millard criteria or “Rule of 10”,
lip pit. i.e. at the time of repair hemoglobin should be more than
10 g%, age is 10 weeks, weight is more than 10 pound and
Treatment
total leukocyte count is less than 10,000/cu mm.
Surgical excision is done.
Millard Rotation Advancement Flap
Commissural Lip Pits In this a tilted cupid bow is rotated downwards following
They are mucosal invagination which arises at vermilion border curved incision extending to columella. This allows rotation of
of lip. misplace cupid’s bow and philtral dimple in normal position.
High rotation gap created is filled with triangular flap which
Pathogenesis consists of skin muscle and mucosa from upper part of lateral
Its occurrence is due to failure of normal fusion of embryonic side of cleft. The resultant scar is hidden inside the nose or
maxillary and mandibular processes. follows natural line of philtrum.

Clinical Features Tennisonrandall Triangular Flap Repair

♦♦ Males are commonly affected In this a cut is given on lower one third of lip to correct upward
♦♦ It present as unilateral or bilateral pit at corners of mouth tilt of cupid’s bow by placing a Z-shape wire. Gap is filled with
on vermilion border triangle of skin, muscle and mucosal flap from lower end of
lateral lip element. In this resultant scar is Zigzag scar.
♦♦ Its size ranges from a shallow depression to an open tract
measuring 4 mm Management of Cleft Palate Repair
♦♦ On palpation less amount of saliva ooze out.
For details refer to Ans 2 of same chapter.
Treatment
Q.9. Write brief notes on treatment of cleft lip.
Surgical excision is done.  (Apr 2017, 2 Marks)
Ans. Following is the treatment of cleft lip:
Double Lip
♦♦ Millard’s criterion is used to undertake surgery for cleft
It is a fold of excessive tissue over inner mucosa of lip. lip, i.e. rule of ten, 10 pound in weight; 10 weeks old; 10
g% hemoglobin.
Pathogenesis ♦♦ If cleft lip is bilateral and is extensive two surgeries should
It arises during second week of gestation because of persistence be done to close the cleft. Surgery of one side should be
of sulcus between pars glabrosa and pars villosa of lip. done first and later on after few week surgery of second
side is done.
Clinical Features ♦♦ Millard’s rotation advancement flap technique is common-
♦♦ Inner aspect of lip is involved. ly used. In this correction of both lip and nasal deformity
is done. Realigning of muscles and of lip and their correct
♦♦ At times when the upper lip become tensed, double lip
anatomical position is an important part of this repair. In
give appearance of cupid bow.
this technique medial lip element is rotated inferiorly and
Treatment the lateral lip element is advanced into resulting upper lip
Surgical excision is done. defect. The columellar flap is then used to create nasal sill.
Section 2:  General Surgery  383

♦♦ Hagedorn-LeMesurier repair: In this method medial lip ♦♦ Chronic infection:


element should be lengthened by introducing the quad- • Candidiasis
rilateral flap which is developed from lateral lip element. • Syphilis.
♦♦ Tenninson-Randall repair: In this method medial lip element ♦♦ Occupational hazards: Woollen textile workers.
is lengthened by introducing a triangular flap from inferior ♦♦ Genetic factors:
portion of lateral lip element. • Oncogenes
♦♦ Proper postoperative management like control of infec- • Tumor suppressor genes
tion, training for sucking, swallowing and speech therapy ♦♦ Pre-existing oral disease:
should be done. • Oral lichen planus
• Oral sub-mucous fibrosis
Q.10. Write classification of cleft lip and palate and its treat­
• Leukoplakia.
ment. (Apr 2018, 5 marks)
Ans. For classification of cleft lip and palate refer to Ans 6 of Clinical Features
same chapter.
♦♦ Male predilection is seen.
♦♦ For treatment of cleft lip refer to Ans 9 of same chapter.
♦♦ Carcinoma mostly occurs in the older age.
♦♦ For treatment of cleft palate refer to Ans 2 of same chapter
♦♦ Site: Most commonly involved are the posterior and lateral
borders of tongue and lower lip and less frequently the
floor of mouth, palate and buccal mucosa.
15. Oral Cancer ♦♦ Small lesion is asymptomatic.
♦♦ Large lesion may cause some pain or paresthesia and
Q.1. Describe the etiology, pathology, clinical features and swelling.
treatment of oral cancer. (Apr 2008, 15 Marks) ♦♦ Patients complain of persistent ulcer in the oral cavity.
Or ♦♦ Function of organ is impaired.
♦♦ Appearance: The clinical appearance of a carcinomatous
Describe the etiological factors, clinical features and
ulcer is that one of irregular shape induration and raised
management of oral carcinoma. (Feb 1999, 20 Marks)
everted edges.
Ans.
♦♦ Base: Usually have broad base and are dome like or nodular.
Etiology ♦♦ Surface: May range from granular to pebbly to deeply
creviced.
Etiological Factors of Oral Cancer ♦♦ Surface may be entirely necrotic and have ragged whitish
♦♦ Tobacco smoking: gray appearance.
• Cigarettes ♦♦ Color: It may be completely red or red surface may be
• Bidis sprinkled with white necrotic or keratin area.
• Pipes ♦♦ Lymph nodes: Superficial and deep cervical nodes are com-
• Cigars monly affected.
• Reverse smoking.
♦♦ Use of smoking tobacco: Pathology
• Snuff dipping
• Tobacco sachets ♦ Allelic imbalance (loss of heterozygosity [LOH]) has been
• Tobacco chewing: identified in tumor suppressor gene.
–– Betal chewing Damage to tumor suppressor gene may also involve
–– Chewing of areca nut damage to other genes involved in growth control, mainly
–– Consumption of alcohol those involve in cell signaling (oncogenes, especially some
–– Diet and nutrition: Vit. A, B complex and C defi- on chromosome 11 and chromo­some 16)
ciency. ♦ Changes in oncogenes can disrupt cell growth control,
♦♦ Dental factors: leading ultimately to uncontrolled growth of cancer.
• Chronic irritation from broken teeth.
• Ill-fitting or broken prosthesis.
♦♦ Ultraviolet radiation: Actinic radiation
♦♦ Viruses:
• Herpes simplex virus
• Human papilloma virus
• Human immunodeficiency virus (HIV)
• Epstein-Barr virus
♦♦ Immunosuppression:
• AIDS
• Organ transplantation
384   Mastering the BDS IIIrd Year  (Last 25 Years Solved Questions)

Management of Oral Cancer ♦♦ Pectoralis major myocutaneous flap (PMMF) based on


thoracoacromial artery.
Management should be curative or palliative: ♦♦ Free microvascular flaps may be from radial artery fore-
♦♦ Early growth without bone involvement arm flap.
• Curative radiotherapy using caesium needles or ♦♦ For small defects—tongue flap, buccal flap, palatal mu-
iridium wires, i.e. brachytherapy. coperiosteal flap.
• Other option is wide excision wire 1–2 cm clearance.
Chemotherapy
Often, the approach to the tumor is by raising the
cheek flap (outside). After the wide excision, the flap ♦♦ Drugs used are methotrexate, cisplatin, vincristine,
is placed back (Patterson operation). bleomycin, adriamycin. Often it is given intra-arterially
• Presently advanced technology in radiotherapy, through external carotid artery using arterial pump or by
facilitates the use of external radiotherapy also. increasing the height of the drip more than l3 feet, so as
The incidence of dreaded complication like to attain a pressure more than systolic pressure. Chemo-
osteoradionecrosis mandible has been reduced due therapy can also be given IV or orally postoperatively.
to better radiotherapy methods. ♦♦ Initial chemotherapy to downstage the tumor followed by
♦♦ Growth with mandible involvement: Here along with wide surgery and later again end with chemotherapy.
excision of the primary tumor hemimandibulectomy or ♦♦ Chemoradiotherapy is used in unresectable tumors as
segmental resection of the mandible or marginal man- consecutive therapies.
dibulectomy (using rotary electric saw) is done.
♦♦ Operable growth with mandible involvement and mobile Radiotherapy
lymph nodes on the same side (confirmed by FNAC): ♦♦ Early lesions are managed by radiotherapy.
Along with wide excision of the primary, hemimandibulec- ♦♦ Radiotherapy is of two types, i.e. external radiotherapy
tomy and radical neck lymph node dissection is done and interstitial radiotherapy.
(commando operation). ♦♦ In external radiotherapy large dose of 6000 to 8000 cGy
Wide excision of primary lesion, hemimandibulectomy units are given, i.e. 200cGy units/day
with radical neck node dissection is called as composite ♦♦ Interstitial radiotherapy is indicated in infiltrative small
resection. lesions. Caesium 137 or iridium wires are placed within
♦♦ Operable growth with mandible involvement; mobile the tumor. Minimal tissue resection is the basic advantage
lymph node on same side and opposite side: Along with of this procedure.
wide excision of the tumour, hemi-mandibulectomy, Q.2. Briefly discuss the etiology, epidemiology, classifica­
radical neck lymph node dissection on same side and tion and principles of treatment of oral cancer.
functional block dissection on opposite side are done,  (Feb 1999, 15 Marks)
retaining the internal jugular vein, sternomastoid and Ans. Etiology: Refers to Ans 1 of same chapter.
spinal accessory nerve.
♦♦ Operable primary tumor with mobile lymph nodes on Epidemiology
same side but without mandibular involvement: Wide
♦♦ Squamous cell carcinoma representing about 90% of all
excision of primary tumor and radical neck lymph node
oral cancer, for this reason, oral squamous cell carcinoma
dissection on same side are done. Mandible is not removed.
is often designated as “oral cancer.”
♦♦ Fixed primary tumor or advanced neck lymph node secondaries:
♦♦ On an average, oral squamous cell carcinoma represent
Only palliative external radiotherapy is given to palli-
about 3% of all cancer in males and about 2% of all cancer
ate pain fungation and to prevent anticipated torrential
in females.
hemorrhage.
♦♦ The incidence of oral squamous cell carcinoma increases
♦♦ Preoperative radiotherapy is often used in fixed lymph with age and most of the causes occur usually after the
node to downstage the disease to make it operative. age of 40 years.
♦♦ Postoperative radiotherapy is given in T3 and T4 tumors: N2
and N3 nodal status to reduce the recurrence and to im- Classification of Oral Cancer
prove the prognosis.
♦♦ Prophylactic block dissection has become popular in N0 ♦♦ Oral cancer of epithelial tissue origin:
diseases • Basal cell carcinoma
♦♦ If growth is extending to upper alveolus: Partial maxillectomy • Squamous cell carcinoma
or total maxillectomy is done. • Verrucous carcinoma
• Adenoid squamous cell carcinoma
Reconstruction After Surgery • Malignant melanoma
Flaps used for reconstruction after oral surgery: ♦♦ Oral cancer of mesenchymal tissue origin:
♦♦ Forehead flap based on superficial temporal artery. • Oral cancer of fibrous tissue
♦♦ Deltopectoral flap based on 1, 2 and 3 perforating vessels –– Fibrosarcoma
from internal mammary vessels. –– Malignant fibrous histiocystoma.
Section 2:  General Surgery  385

• Oral cancer of adipose tissue ♦♦ In fixed primary or secondary, radiotherapy with chemo-
–– Liposarcoma. therapy is used for palliation to relieve pain, fungation,
• Oral cancer of vascular tissue sepsis.
–– Hemangiopericytoma ♦♦ In advanced stage terminal events may be severe malnutri-
–– Hemangioendothelioma tion, bleeding, sepsis, and bronchopneumonia.
–– Angiosarcoma. ♦♦ Posterior lesions has got poor prognosis than anterior le-
• Oral cancer of lymphoid tissue sions. Lip carries best prognosis depends on anatomical
–– Hodgkin’s lymphoma location, grading, lymph node status, soft tissue involve-
–– Non-Hodgkin’s lymphoma ment and response of therapy.
–– Burkitt’s lymphoma Q.3. Write short note on *modalities of treatment of oral
–– Leukemia. cancer. (Sep 2004, 10 Marks)
• Oral cancer of bone
Ans. Oral cancer is treated by surgery, radiotherapy and
–– Osteosarcoma
chemotherapy or by combination of these. The other
–– Ewing’s sarcoma.
developing techniques in the treatment of oral cancer
• Oral cancer of neural tissue
are laser therapy, hormonal therapy, hyperthermia, etc.
–– Neurosarcoma
–– Neurofibrosarcoma Surgery
–– Neuroblastoma.
• Oral cancer of muscle ♦♦ The surgical treatment of oral cancer as a primary modality
–– Leiomyosarcoma is excisional in nature.
–– Malignant granular cell myoblastoma ♦♦ All clinical detectable tumor must be excised with adequate
• Tumor of salivary gland margins of adjacent normal tissue, to ensure that the re-
–– Mucoepidermoid carcinoma sidual element of the microscopic disease do not remain
–– Adenocarcinoma within the surgical field.
–– Acinic cell carcinoma. ♦♦ The surgical treatment of oral carcinoma includes three
important steps:
Principle of Treatment of Oral Cancer 1. Wide excision of primary tumor
2. Neck dissection, i.e. surgical removal of involved
Following are the principles of treatment of oral cancer:
lymph nodes present in the neck
♦♦ If only primary is present which is mucosal with size less
3. Reconstruction of the resected region in the oral cavity.
than 2 cm without nodal spread, then wide local exci-
sion with supraomohyoid block dissection of same side
Lesion in Mandible
is done (N0); primary may also be treated with curative
brachytherapy or external beam teletherapy. If nodes are Surgical procedures that can be done for the resection of tumors
histologically positive then radical neck dissection is done. in the mandible include:
♦♦ Larger mucosal primary with similar features are also ♦♦ Alveolectomy: This is an intraoral procedure in which
treated similarly, but postoperative radiotherapy or/and alveolus is removed alone. This is done for small alveolar
chemotherapy is added depending on grading of the lesions in the mandible.
tumor. ♦♦ Marginal mandibulectomy: It is the surgical procedure
♦♦ In all these types of lesions, if there are positive mobile which involves the removal of body of the mandible in
neck nodes which is confirmed by FNAC, then radical the involved area leaving lower border of the mandible
neck dissection should be done. intact. This procedure is done in cases where the tumor
♦♦ If primary lesion extends into adjacent soft tissue with involves body of the mandible but does not involve the
mandibular involvement then mandibular resection is lower border of the mandible.
needed. Part is reconstructed using plates or bone graft ♦♦ Segmental resection: It is the surgical procedure in which
taken from iliac crest or opposite 11th rib. 2.4 mm re- a segment of mandible including the lower border should
construction plate with pectoralis major myocutaneous be excised. Depending on the location of the tumor, this
flap (PMMF) or non-vascularized bone graft (iliac crest procedure is divided into two parts, i.e. anterior segmental
cancellous chips) or vascularized bone graft from fibula/ or posterior segmental resection. Since the lower border
iliac crest/ scapula are the present recommendations. Skin of the mandible is also resected, it causes discontinuity of
covering is done by split skin graft inside to mucosa or by the mandible.
appropriate flaps depending on the need and feasibility ♦♦ Hemimandibulectomy: In this procedure excision of
of donor area (PMMF/DP flap/forehead flap). Neck is ad- one half of the mandible is done. This is carried out in
dressed similarly. Postoperative EBRT and chemotherapy extensive tumors which involve the mandible. This could
is needed either concurrent or sequential. be for a tumor of the buccal mucosa or floor of the mouth
♦♦ If primary is advanced then chemotherapy with EBRT is infiltrating into the mandible. The soft tissues involved
used. If lesion reduces in size and becomes operable it is in the tumor are resected along with the mandible. The
then operated accordingly. condyle is usually spared in these cases.
386   Mastering the BDS IIIrd Year  (Last 25 Years Solved Questions)

♦♦ Disarticulating hemimandibulectomy: In this surgical - Uninvolved areas of patient should be pre-


procedure, one half of the mandible including the condyle vented by doing shielding.
is excised. - Host tissues of patient should be protected
from radiation by two methods, i.e. fractiona-
Lesion in Maxilla tion and multiple ports.
- In fractionation instead of giving maximum
The different surgical options in the maxilla depending upon
radiation patient is given radiation in small incre-
the size and extent of the tumor include:
ments for several weeks which provides time for
♦♦ Alveolectomy: This is an intraoral procedure which normal tissues for recovery between dosages.
involves removal of the involved part of the maxillary - In multiple ports multiple beams are used
alveolus. which provide radiation to tumor from differ-
♦♦ Subtotal maxillectomy: In this removal of maxilla exclud- ent angles. In this radiation delivery is on every
ing the floor of the orbit and infraorbital rim. 5th day a week.
♦♦ Total maxillectomy: Removal of maxilla including orbital
floor as well as rim. Chemotherapy
♦♦ Radical maxillectomy: It involves removal of orbital con-
♦♦ It is used in the treatment of malignant tumor. It selectively
tents along with the maxilla.
kill’s tumor cells by virtue of cell kinetic proliferation
♦♦ T1, T2 lesions involving only the maxillary alveolus should
character and cell biology.
be incised with an adequate margin of normal tissue in- ♦♦ Chemicals which interferes with rapid growth of tumor
traorally. Alveolectomy is done for these lesions. T3, T4 cells are used for treating oral cancer.
lesions with invasion into the maxillary antrum or nasal ♦♦ Vincristine, bleomycin and methotrexate in various com-
cavity require subtotal maxillectomy. binations are used.
♦♦ Larger lesions eroding the floor of the orbit require total ♦♦ Chemotherapy should be given intravenously but nowa-
maxillectomy. Invasion of the tumor into the orbit requires days its intra-arterial injections can be given
removal of orbital contents and is called a radical maxil- ♦♦ It produces only partial or temporary tumor regression.
lectomy. Alveolectomy can be done intraorally. ♦♦ It may be used in combination with radiotherapy or sur-
♦♦ For all other procedures, a wide exposure of the maxilla gery or as palliative treatment.
is required. This can be achieved by the Weber Ferguson ♦♦ Chemotherapy is most effective in the lesions which are
incision. This incision may have a subciliary or a brow confined to the soft tissues.
extension for a better exposure of the maxilla.
Q.4. Write about clinical features and treatment of carci­
Reconstruction noma of tongue. (Sep 2001, 15 Marks)
♦♦ Although cure rates have not changed much over the years, Or
better function and appearance of the patient have been Describe the causes, clinical features and management
made possible by reconstructive techniques. of carcinoma of tongue. (Apr 2007, 15 Marks)
♦♦ Various surgical advances have provided means for soft
tissue and hard tissue reconstruction of the excised region. Or
♦♦ Soft tissues used for reconstruction include: Write a short note on carcinoma of tongue.
• Deltopectoral flap  (June 2014, 5 Marks)
• Sternocleidomastoid flap Ans. Carcinoma of tongue is mostly epidermoid carcinoma.
• Pectoralis major flap
♦♦ These and various other flaps have revolutionized the Responsible Features or Causes
reconstructive procedures.
♦♦ Poor oral hygiene
♦♦ Pipe smokers
Radiotherapy ♦♦ Chronic alcoholic
♦♦ Radiotherapy is the treatment of the disease with ionizing ♦♦ Chewing of betel nut
or non-ionizing radiation. ♦♦ Chronic irritation by sharp tooth
♦♦ Following methods are to be followed for radiotherapy: ♦♦ Syphilis
• X-ray therapy: ♦♦ Leukoplakia
–– Superficial X-ray therapy 45 – 100 kV ♦♦ Erythroplakia.
–– Kilovoltage X-ray therapy 300 kV.
• Electron therapy Pathological Types
• Surface applicator. ♦♦ Non-healing ulcer
–– Interstitial implantation ♦♦ Proliferative growth
- Radiation is given externally by the use of X-ray ♦♦ Frozen tongue or indurated plaque
generators. ♦♦ Fissure variety.
Section 2:  General Surgery  387

Clinical Features ♦♦ Bilateral mobile lymph nodes are dealt with one side
radical block and other side junctional block dissection
♦♦ A bleeding ulcer is seen over the tongue.
with essentially retaining internal jugular vein (on opposite
♦♦ Pain in the tongue is due to involvement of lingual nerve.
side) to maintain the cerebral venous blood flow. Other
Pain can refer to the ear and lower temporal region.
option is doing same side radical neck dissection and on
♦♦ Disarticulation—difficulty in talking is due to dis­ability of opposite side supraomohyoid block dissection.
the tongue to move freely. ♦♦ Wide excision is done when growth is in the tip of the tongue.
♦♦ Dysphagia is a common presentation from the carcinoma ♦♦ Posterior third growth can be approached by lip split
of posterior one-third. and mandible resection, so as to have total glossectomy-
♦♦ Fetor oris is due to infected necrotic growth. Kochers approach. It is not done commonly as it carries
♦♦ Ankyloglossia restricted mobility of the tongue. It is due significant morbidity and mortality due to difficulty in
to infiltration of the mouth or mandible. speech, swallowing, aspiration, sepsis.
♦♦ Bilateral massive enlargement of lower deep cervical ♦♦ When mandible is involved hemimandibulectomy is
nodes in an elderly patient is suggestive of carcinoma of done.
posterior one-third. ♦♦ The procedure that involves wide excision or hemiglos-
♦♦ Painless ulcer or swelling is present on the tongue which sectomy, hemimandibulectomy and radical neck dissection
later becomes painful. together is called as Commando Operation.
♦♦ Excessive salivation is present and saliva is blood tinged. ♦♦ Reconstruction of tongue and other area after surgery: By
♦♦ Visible ulcer can be seen on anterior two-third of tongue. deltopectoral flap, forehead flap, pectoralis major muscle
Ulcer can bleed on touch; edge, base and surrounding areas flap, skin grafting.
are indurated. Often indurated area is more extensive than ♦♦ Prophylactic block dissection is becoming popular at present.
primary tumor. Edges are everted. Ulcer may cross the
midline and extend to the floor or mouth/alveolus/mandible. Radiotherapy
♦♦ Features of bronchopneumonia due to aspiration during ♦♦ In small primary tumor-brachytherapy using caesium or
lying down sleeping mainly to lower segment of lung. iridium needles.
Investigations ♦♦ In large primary tumor initial radiotherapy is given to
reduce the tumor size so that resection will be better later.
♦♦ Wedge biopsy ♦♦ Advanced primary as well as secondaries in neck are
♦♦ FNAC of lymph nodes controlled by palliative external radiotherapy.
♦♦ Indirect and direct laryngoscopy to see posterior third ♦♦ Postoperative radiotherapy is given in large tumors to
growth reduce the chances of relapse.
♦♦ CT scan to see the extension of posterior third growth or to ♦♦ In case of growth in posterior one-third of tongue
see status of lymph node secondaries. radiotherapy is of curative as well as palliative mode.
♦♦ MRI to assess extent of primary tumor
♦♦ Chest X-ray to see bronchopneumonia Chemotherapy
♦♦ Orthopantomogram ♦♦ It is given in post-operative period and also for pal-
Management liation.
♦♦ Price-Hill regimen is commonly used. Drugs are metho-
Following is the management of the carcinoma of tongue: trexate, vincristine, adriamycin, bleomycin and mercap-
♦♦ Wide excision with l cm clearance in margin and depth is topurine.
done in tumor less than l cm in size or in carcinoma in situ. ♦♦ It is either given intra-arterially, as regional chemotherapy
Laser (CO2/diode) can be used. through extemal carotid artery using arterial pump or
♦♦ Tumor between 1–2 cm in size, partial glossectomy is done through IV. It can be given orally also.
with 2 cm clearance from the margin with removal of l/3rd
of anterior two-third of the tongue.
♦♦ Tumor larger than 2 cm, hemiglossectomy is done with
removal of anterior 2/3rd of tongue on one side up to
sulcus terminalis.
♦♦ Raw area in these procedures can be left alone when area is
wide allowing it to granulate and heal by epithelialization.
lf area is small like in wide excision it can be closed by
primary suturing. Wide raw area can also be covered with
PMMF or quilted split-skin graft.
♦♦ Larger primary tumor can be given preoperative
radiotherapy then later hemiglossectomy is done.
♦♦ Same side palpable mobile lymph nodes are removed by Figs 25A and B:  Glossectomy; (A) Partial glossectomy
radical neck block dissection. (B) Hemiglossectomy
388   Mastering the BDS IIIrd Year  (Last 25 Years Solved Questions)

Q.5. Discuss management of anterior two-third tongue ♦♦ In large primary tumor initial radiotherapy is given to
carcinoma. (Feb/Mar 2004, 10 Marks) reduce the tumor size so that resection will be better later.
♦♦ Advanced primary as well as secondaries in neck are
Ans. Carcinoma of the tongue is a common lesion.
controlled by palliative external radiotherapy.
Carcinoma of the tongue occurs on the anterior 2/3, 50%
♦♦ Postoperative radiotherapy is given in large tumors to
of carcinoma is seen in this region.
reduce the chances of relapse.
Management Chemotherapy
Following is the management of the carcinoma of anterior two- ♦♦ It is given in postoperative period and for palliation.
third of tongue: ♦♦ Price – Hill regimen is commonly used. Drugs are methotrex-
ate, Vincristine, adriamycin, bleomycin and mercaptopurine.
Investigations ♦♦ It is either given intra-arterially as regional chemotherapy
♦♦ Wedge biopsy through external carotid artery using arterial pump or
♦♦ FNAC of lymph nodes through IV. It can be given orally also.
♦♦ CT-scan to see the status of lymph node secondaries. Q.6. Write short note on Ca lip. (Mar 2001, 10 Marks)
♦♦ MRI to assess extent of primary tumor Ans. Ca lip: Carcinoma of lip.
♦♦ Chest X-ray to see bronchopneumonia Vermilion border of lip and mucosa is the main site of
♦♦ Orthopantomogram carcinoma of lip.
Treatment It is common in the western elderly, white people,
specially those people exposed to sunlight.
Surgery The lesions known as countryman’s lip because it occurs
♦♦ Wide excision with l cm clearance in margin and depth is commonly in agriculturists.
done in tumour less than l cm in size or in carcinoma in
Etiology
situ. Laser (CO2/diode) can be used.
♦♦ Tumor between 1–2 cm in size, partial glossectomy is done ♦♦ Excessive use of tobacco
with 2 cm clearance from the margin with removal of l/3rd ♦♦ Leukoplakia and syphilis
of anterior two-third of the tongue. ♦♦ Placing khaini between lower lip and gum
♦♦ Tumor larger than 2 cm, hemiglossectomy is done with ♦♦ Heavy consumption of alcohol
removal of anterior 2/3rd of tongue on one side up to ♦♦ Radiation to the lip.
sulcus terminalis. Clinical Features
♦♦ Raw area in these procedures can be left alone when area
♦♦ Elderly males are affected in 90% of cases.
is wide allowing it to granulate and heal by epithelializa-
♦♦ Non-healing ulcer or growth is a common presentation.
tion. lf area is small like in wide excision it can be closed ♦♦ Lesion appears in the form of white plaque of non-healing
by primary suturing. Wide raw area can also be covered ulcers.
with PMMF or quilted split-skin graft. ♦♦ Edges are everted and indurated, this is characteristic of
♦♦ Larger primary tumor can be given preoperative radio- carcinoma.
therapy then later hemiglossectomy is done. ♦♦ Ulcer contain slough in floor.
♦♦ Same side palpable mobile lymph nodes are removed by ♦♦ Bleeding may occur from ulcer.
radical neck block dissection. ♦♦ Pain and paresthesia may occur.
♦♦ Bilateral mobile lymph nodes are dealt with one side radi- ♦♦ Lesion may get fixed to the subcutaneous structure of lip.
cal block and other side junctional block dissection with ♦♦ Ulcer spreads and destroys the tissue of lip and chin.
essentially retaining internal jugular vein (on opposite ♦♦ Sub-mental and submandibular lymph nodes are involved,
side) to maintain the cerebral venous blood flow. Other lymph node becomes hard and may be fixed.
option is doing same side radical neck dissection and on
opposite side supraomohyoid block dissection. Differential Diagnosis
♦♦ Wide excision is done when growth is in the tip of the ♦♦ Leukoplakia
tongue. ♦♦ Syphilitic chancre
♦♦ Reconstruction of tongue and other area after surgery: By ♦♦ Keratocanthoma
deltopectoral flap, forehead flap, pectoralis major muscle ♦♦ Ectopic salivary gland tumor
flap, skin grafting. ♦♦ Pyogenic granuloma.
♦♦ Prophylactic block dissection is becoming popular at Treatment
present. ♦♦ If lesion is less than 2 cm, then curative radiotherapy,
Radiotherapy either brachytherapy or external beam radiotherapy. It
gives a good cure.
♦♦ In small primary tumor—brachytherapy using caesium ♦♦ If tumor is more than 2 cm, wide excision is done. Excision
or iridium needles. of lower lip up to one-third can be sutured primarily, in
Section 2:  General Surgery  389

layers keeping vermilion border in proper apposition Etiology


without causing any microstomia.
♦♦ All the ‘S’, i.e. smoking, spirit, syphilis, sharp tooth and
♦♦ Excision of more than one-third of the lip requires recon-
spices.
struction using different flaps.
♦♦ Premalignant lesions and conditions, i.e. leukoplakia,
Methods erythroplakia, OSMF, candidiasis, etc.
♦♦ Placing the quid of betel nut and tobacco in cheek mucosa.
♦♦ Abbe-Estlander‘s rotation flap used for either upper or
lower lip lesions located at the angle based on labial artery. Pathological Types
Here base at a later stage need not be disconnected unlike ♦♦ A non-healing ulcer
in Abbe lip. ♦♦ An exophytic growth or verrucous carcinoma
♦♦ Fries modified bernard facial flap: Reconstruction using lateral ♦♦ An infiltrative lesion, which involves adjacent structures
facial flaps. It is used when defect is more than half of lip like tongue, mandible, floor of mouth.
and midline.
Clinical Features
♦♦ Gillies fan flap: It is a cheek flap usually bilateral but can
be unilateral. Incision is full thickness around commissure ♦♦ Ulcer (painless to begin with) in the cheek which gradu-
extending into nasolabial fold and upper lip up to upper ally increases in size in a patient with history of chewing
lip vermilion border. Flap which is based on labial vessels pan and smoking is the most common presentation and
advanced towards the defect. Vermilion is reconstructed initially it is painless.
with tongue mucosal flap which is divided in 3 weeks. ♦♦ Pain occurs when it involves the skin, bone or if secondar-
♦♦ Karapandzic flap: It is the modified version of Gillie’s flap ily infected. Referred pain to the ear signifies involvement
of lingual nerve.
used for lower lip defect with less angulations towards
♦♦ Halitosis which is bad odour breath is common.
upper lip. Reverse Karapandzic flap is used for upper lip.
♦♦ Involvement of retromolar trigone indicates that it is an
♦♦ Microvascular flaps.
advanced disease, as the lymphatics here communicate
♦♦ Nasolabial flap: It is used when defect is more than half of freely with the pharyngeal lymphatics.
lip laterally or defect is in the floor of mouth. ♦♦ Everted edge, induration are the typical features of the
♦♦ Cheek flap. ulcer.
♦♦ Free radial artery flap ♦♦ Mandible is examined bidigitally, for thickening, tender-
♦♦ Abbe flap: It is used for upper or lower lip lesions at the ness, irregularity and sites of fracture. Mandible may get
middle or the site other than angle based on labial artery. involved by direct extension, through mandibular canal
Here at the later second stage base of the flap should be or through periodontal membrane. Loss of central part of
released once as flaps takes up. mandible due to destruction by tumor will cause pout-
♦♦ ‘W’ flap plasty: It is done for lower lip middle tumor which ing of lower lip with drooling of saliva, i.e. Andy Gump
is less than one-third of the lip. deformity.
♦♦ Johansen stepladder procedure is used for extensive car- ♦♦ Mandibular canal is close to occlusive alveolar surface in
cinoma of lower lip. elderly and edentulous patients to cause early mandibular
♦♦ Other regular flaps like forehead flap, deltopectoral flap spread in carcinoma.
can also be used. ♦♦ Trismus and dysphagia signify involvement of pterygoids
♦♦ Lymph nodes are dealt with by radical neck dissection on or posterior extension.
one side and functional block or supraomohyoid block ♦♦ Occasionally it may extend into the upper alveolus and to
dissection on other side. For central tumor N0 disease, the maxilla causing swelling, pain and tenderness.
bilateral elective (prophylactic) supraomohyoid dissection ♦♦ Once involvement of soft tissue occurs, it may come out
is done. For lateral tumour N0 disease, elective ipsilateral through skin as fungating lesion often with orocutaneous
fistulas with saliva dribbling through fistula.
supraomohyoid dissection is done.
♦♦ Submandibular lymph nodes and upper deep cervical
♦♦ Postoperative radiotherapy is given if tumor is large or if
nodes are involved which are hard and nodular; initially
lymph nodes are involved.
mobile and later get fixed to each other and then to deeper
♦♦ When mandible is involved, segmental resection is done. structure.
Q.7. Write about etiology, clinical features and treatment Features of Advanced Carcinoma Cheek
of carcinoma of cheek. (Mar 2000, 15 Marks)
♦♦ Involvement of retromolar trigone.
Or
♦♦ Extension into the base of skull and pharynx
Write short note on etiology and treatment of carcinoma ♦♦ Fixed neck lymph nodes
of cheek. (Feb 2004, 6 Marks) ♦♦ Extension to the opposite side
Ans. The common carcinoma of cheek is squamous cell
carcinoma. Spread
This is also called verrucous carcinoma or tobacco ♦♦ Local spread: Result in orosubcutaneous fistula and man-
chewers carcinoma of cheek. dibular sinus.
390   Mastering the BDS IIIrd Year  (Last 25 Years Solved Questions)

♦♦ Lymphatic spread: Enlargement of submandibular lymph ♦♦ Operable primary tumor with mobile lymph nodes on same
nodes. side but without mandibular involvement: Wide excision of
♦♦ Hematogenous spread: Very rare. primary tumor and radical neck lymph node dissection on
same side are done. Mandible is not removed.
Investigation ♦♦ Fixed primary tumor or advanced neck lymph node secondaries:
♦♦ Wedge biopsy, usually taken from two sites. Biopsy has Only palliative external radiotherapy is given to palli-
to be taken from the edge as it contains active cells; not ate pain fungation and to prevent anticipated to rrential
from the center as it is the area of necrosis. Malignant hemorrhage.
squamous cells with epithelial pearls (keratin pearls) are ♦♦ Preoperative radiotherapy is often used in fixed lymph
the histological features. node to downstage the disease to make it operative.
♦♦ FNAC from lymph nodes. ♦♦ Postoperative radiotherapy is given in T3 and T4 tumors:
♦♦ CT scan is used to assess the extent of tumor into mandi- N2 and N3 nodal status to reduce the recurrence and to
ble, pterygoid region, in patient with trismus, with neck improve the prognosis.
lymph nodes, with carotid involvement by lymph nodes. ♦♦ Prophylactic block dissection has become popular in N0
♦♦ MRI is very useful in assessing the soft tissues, base of diseases
skull and perineural spread. ♦♦ If growth is extending to upper alveolus: Partial maxillectomy
♦♦ Orthopantomogram to look for the involvement of mandi- or total maxillectomy is done.
ble—destruction and fracture sites. Symphysis menti and Reconstruction after Surgery
lingual plate are not clearly appreciated. So often OPG may
be supported with dental occlusion and intraoral X-rays. Flaps used for reconstruction after oral surgery:
• Forehead flap based on superficial temporal artery.
Treatment • Deltopectoral flap based on 1, 2 and 3 perforating vessels
from internal mammary vessels.
Treatment should be curative or palliative • Pectoralis major myocutaneous flap (PMMF) based on
♦♦ Early growth without bone involvement thoracoacromial artery.
• Curative radiotherapy using caesium needles or • Free microvascular flaps may be from radial artery forearm
iridium wires, i.e. brachytherapy. flap.
• Other option is wide excision wire 1–2 cm clearance. • For small defects—tongue flap, buccal flap, palatal
Often, the approach to the tumor is by raising the mucoperiosteal flap.
cheek flap (outside). After the wide excision, the flap
is placed back (Patterson operation). Chemotherapy
• Presently advanced technology in radiotherapy,
facilitates the use of external radiotherapy also. ♦♦ Drugs used are methotrexate, cisplatin, vincristine,
The incidence of dreaded complication like bleomycin, adriamycin. Often it is given intra-arterially
osteoradionecrosis mandible has been reduced due through external carotid artery using arterial pump or by
to better radiotherapy methods. increasing the height of the drip more than l3 feet, so as
♦♦ Growth with mandible involvement: Here along with wide to attain a pressure more than systolic pressure. Chemo-
excision of the primary tumor hemimandibulectomy or therapy can also be given IV or orally postoperatively.
segmental resection of the mandible or marginal man- ♦♦ Initial chemotherapy to downstage the tumor followed by
dibulectomy (using rotary electric saw) is done. surgery and later again end with chemotherapy.
♦♦ Operable growth with mandible involvement and mobile lymph ♦♦ Chemoradiotherapy is used in unresectable tumors as
nodes on the same side (confirmed by FNAC): Along with consecutive therapies.
wide excision of the primary, hemimandibulectomy and
radical neck lymph node dissection is done (commando Radiotherapy
operation).
Wide excision of primary lesion, hemimandibulectomy ♦♦ Early lesions are managed by radiotherapy.
with radical neck node dissection is called as composite ♦♦ Radiotherapy is of two types, i.e. external radiotherapy
resection. and interstitial radiotherapy.
♦♦ Operable growth with mandible involvement; mobile lymph ♦♦ In external radiotherapy large dose of 6000 to 8000 cGy
node on same side and opposite side: Along with wide exci- units are given, i.e. 200cGy units/day
sion of the tumor, hemi-mandibulectomy, radical neck ♦♦ Interstitial radiotherapy is indicated in infiltrative small
lymph node dissection on same side and functional block lesions. Caesium 137 or iridium wires are placed within
dissection on opposite side are done, retaining the internal the tumor. Minimal tissue resection is the basic advantage
jugular vein, sternomastoid and spinal accessory nerve. of this procedure.
Section 2:  General Surgery  391

Q.8. Describe the pathology, clinical features and principles Stage Grouping of Oral Cancer
of treatment of head and neck cancer.
(Feb 2002, 20 Marks) Stage 0 Tis N0 M0
Ans. For pathology, refer to Ans 1 of same chapter. Stage I T1 N0 M0
For clinical features, refer to Ans 1 of same chapter.
Stage II T2 N0 M0
For principles of treatment, refer to Ans 2 of same chapter.
Stage III T1 N1 M0
Q.9. Write short note on TNM classification of malignant T2 N1 M0
tumors. (Sep 1999, 10 Marks) T3 N0, N1 M0
Ans. TNM classification was given by American Joint Stage IVa T1,T2,T3 N2 M0
Committee on Cancer (AJCC): T4a N0, N1, N2 M0
T is suggestive of primary tumor
Stage IVb Any T N3 M0
N is suggestive of regional lymph nodes T4b Any N M0
M is suggestive of distant metastasis Stage IVc Any T Any N M1
T – primary tumor
TX – Primary tumor cannot be assessed Q.10. Difference between benign and malignant tumors.
 (Mar 2001, 10 Marks) (Feb 1999, 10 Marks)
T0 – No evidence of primary tumor
Tis – Carcinoma in situ Or
T1 – Tumor 2 cm of less in greatest dimension
Enumerate difference between benign and malignant
T2 – Tumor more than 2 cm but not more than 4 cm in neoplasia. (June 2014, 2 Marks)
greatest dimension Or
T3 – Tumor more than 4 cm in greatest dimension Describe differentiating features of benign and malig­
T4a (Lip) – Tumor invades through cortical bone, inferior nant tumors. (Jan 2017, 3 Marks)
alveolar nerve, floor of mouth or skin (chin or nose) Ans.
T4a (Oral cavity) – Tumor invades through cortical bone,
into deep/extrinsic muscle of tongue (genioglossus, Characters Benign tumor Malignant tumor
hyoglossus, palatoglossus and styloglossus), maxillary Age May occur at any age Usually after 40 years
sinus or skin of face. Size Usually small Usually large
T4b (lip and oral cavity) – Tumor invades masticatory Growth i. Slow growing i. Rapidly growing
space, pterygoid plates or skull base or encases internal
ii. Expansive type of ii. Invasive type of
carotid artery growth growth
N–Regional lymph nodes Local The surrounding Involvement of
NX–Regional lymph nodes cannot be assessed infiltration structures are not surrounding structure
N0–No regional lymph node metastasis involved is a characteristic
feature
N1–Metastasis in a single ipsilateral lymph node, 3 cm
Fixity Usually not fixed to the Fix to the surrounding
or less in greatest dimension
surrounding structures structure due to local
N2a–Metastasis in a single ipsilateral lymph node, more invasion
than 3 cm but not more than 6 cm in greatest dimension H i s t o l o g i c a l • Well differentiated • Less differentiated
N2b–Metastasis in multiple ipsilateral lymph nodes, not features • Well formed stromal • Stroma is poorly
more than 6 cm in greatest dimension cells formed
N2c–Metastasis in bilateral or contralateral lymph nodes, • Few mitosis • Mitosis numerous
not more than 6 cm in greatest dimension H e m o r r h a g e Usually does not occur More tendency
and ulceration towards hemorrhage
N3–Metastasis in a lymph node more than 6 cm in
tendency and ulceration
greatest dimension
Metastasis Never occur Metastasis frequent
M – Distant metastasis
Fetal to life Usually not fatal to life, if Almost fatal, if
MX – Distant metastasis cannot be assessed death occur it is due to untreated. Cause of
M0 – No distant metastasis mechanical pressure and death is mechanical
M1 – Distant metastasis. obstructive effects and destructive effect.
392   Mastering the BDS IIIrd Year  (Last 25 Years Solved Questions)

Q.11. Describe the clinical features and treatment of anterior • Carcinoma in situ
2/3 of tongue. (Sep 2008, 10 Marks) • Bowen disease
Ans. • Actinic keratosis, actinic chelitis and actinic elastosis.

Clinical Features Clinical Features


♦♦ Painless ulcer or swelling on the tongue which later on may ♦♦ Painless ulcer or swelling on the tongue which later on may
become painful. Pain is present in the tongue due to infec- become painful. Pain is present in the tongue due to infec-
tion or ulceration or due to involvement of lingual nerve. tion or ulceration or due to involvement of lingual nerve.
♦♦ Salivation is excessive and is often blood stained. ♦♦ Salivation is excessive and is often blood stained.
♦♦ Visible ulcer can be seen on anterior two-third of tongue. ♦♦ Visible ulcer can be seen on anterior two-third of tongue.
Ulcer can bleed on touch; edge, base and surrounding ar- Ulcer can bleed on touch; edge, base and surrounding ar-
eas are indurated. Often indurated area is more extensive eas are indurated. Often indurated area is more extensive
than primary tumor. Edges are everted. Ulcer may cross than primary tumor. Edges are everted. Ulcer may cross
the midline and extend to the floor or mouth/alveolus/ the midline and extend to the floor or mouth/alveolus/
mandible. mandible.
♦♦ Fetor oris is due to infected necrotic growth. ♦♦ Fetor oris is due to infected necrotic growth.
♦♦ Ankyloglossia restricted mobility of the tongue. It is due ♦♦ Ankyloglossia restricted mobility of the tongue. It is due
to infiltration of the mouth or mandible. to infiltration of the mouth or mandible.
♦♦ Disarticulation—difficulty in talking is due to disability of ♦♦ Disarticulation—difficulty in talking is due to disability of
the tongue to move freely. the tongue to move freely.
♦♦ Presence of palpable lymph nodes in the neck which are ♦♦ Presence of palpable lymph nodes in the neck which are
hard, nodular and get fixed to underlying tissues in ad- hard, nodular and get fixed to underlying tissues in ad-
vanced stages. vanced stages.
For treatment of anterior two-third of tongue ,refer to Ans 5
Investigations
of same chapter.
♦♦ Wedge biopsy
Q.12. Discuss the etiology, clinical features and management
♦♦ FNAC of lymph nodes
of carcinoma of alveolus. (Jan 2011, 10 Marks)
♦♦ CT scan to see the status of lymph node secondaries.
Ans. For etiology refer to Ans 1 of same chapter.
♦♦ MRI to assess extent of primary tumor
Clinical Features ♦♦ Chest X-ray to see bronchopneumonia
♦♦ Orthopantomogram
♦♦ Early invasion of bone takes place mostly via the PDL and
this causes extensive mobility and premature loss of the Management
regional teeth. Refer to Ans 5 of same chapter.
♦♦ Extraction of tooth often leads to early bone invasion,
which causes non-healing or delayed healing of the Q.14. Write short note on incisional and excisional biopsy.
extraction socket.  (Jan 2012, 5 Marks)
♦♦ Mandibular lesions often extend to the adjoining Or
structures, e. g. labial mucosa, tongue, bone, floor of the Write short answer on incisional biopsy.
mouth and the retromolar areas.  (Apr 2018, 3 Marks)
♦♦ Metastasis occurs often to the submandibular and deep Or
cervical lymph nodes.
Write short answer on incisional and excisional biopsy.
♦♦ Involvement of inferior alveolar nerve can lead to pares-
 (June 2018, 3 Marks)
thesia.
Ans.
Management
Incisional Biopsy
For management, refer to Ans 3 of same chapter.
♦♦ This is the excision of a portion of lesion for microscopic
Q.13. Enlist premalignant lesions of oral cavity. Discuss examination.
clinical features, investigations and management of ♦♦ This method is employed on large, diffuse lesions which
carcinoma of anterior two-third of tongue. has the size of 2 cm in its greatest dimension.
(Feb 2013, 10 Marks) ♦♦ This method can also be dome on lesions suspected for
Ans. Premalignant Lesions of Oral Cavity malignancy.
• Leukoplakia ♦♦ Aim of this method is to remove a portion of lesional tis-
• Erythroplakia sue in question along with the sample of normal adjacent
• Mucosal changes associated with smoking habits tissue for comparison.
Section 2:  General Surgery  393

Types ♦♦ Handling of tissues: Tissue which has to be removed


should be handled carefully so that histopathological ex-
♦♦ Punch biopsy: This is done by using a surgical punch of
amination can be performed. A non-toothed tissue holding
diameter 4, 8 or 10 mm. This incisional biopsy is done in
forceps is used and care is taken not to crush the tissues.
mass screening programmes.
♦♦ Care of specimen: After removal of the tissues, the speci-
♦♦ Wedge biopsy: It is done by making the wedge shaped
men is transferred to a bottle containing 10% formalin
incision which begins 2 to 3 mm from normal tissue and
which should be at least 20 times the volume of the speci-
penetrates in the region surrounding abnormal tissue. Tis-
men obtained.
sue should always be incised narrow and deep.
♦♦ Surgical closure of wound: Primary closure is possible in
Exisional Biopsy most cases. Where it is not possible, the tissues are under-
mined to facilitate closure.
♦♦ This procedure should be done for the small lesions which
are clinically benign. Q.15. Write short note on hemimandibulectomy.
♦♦ In this complete lesion should be removed for examina-  (Feb 2013, 5 Marks)
tion and diagnosis. So it is both diagnostic and curative. Ans. Hemimandibulectomy means half of the mandible is
removed to excise the lesion which is involving that
Application region.
♦♦ This procedure is performed on the lesions which need
complete removal for diagnostic and therapeutic purposes. Types of Hemimandibulectomy
♦♦ It is indicated in lesions which are diagnosed as benign
♦♦ Condyle sparing hemimandibulectomy: This method is done
and need complete removal
for extensive lesions which involve both inferior and
Advantages posterior borders of the mandible. The condyle can be
spared if it is not involved. Sparing of the condyle allows
♦♦ It allows histopathological examination of an entire lesion.
the reconstruction procedure to be simpler. This is because
♦♦ Amount of tissue which is removed from one biopsy site,
there will be a segment for attachment of the reconstruction
ensure adequate sample for various studies such as cul-
plate or bone graft.
ture, histopathology, immunofluorescence and electron
♦♦ Disarticulating hemimandibulectomy: This procedure is done
microscopy.
for extensive lesions which involve inferior border, poste-
Disadvantages rior border and condyle. Condyle removal makes recon-
struction a little more difficult as the condylar prosthesis
♦♦ If the tumor is highly infiltrative margin of excision can- has to be placed very carefully in condylar fossa without
not be exactly elecited, further surgery should be needed. applying pressure inside the fossa. Reconstruction plates
♦♦ Cancerous cells actively multiply at tumor margins, debulk- are available with the condylar prosthesis for reconstruc-
ing of the mass results in residual cancer cells left behind. tion of hemimandibulectomy defects.
♦♦ Excision needs greater precision and skill of surgeon.
Basic Procedure for Hemimandibulectomy
Procedure of Incisional or Excisional biopsy
♦♦ Depending on the extent of involvement a partial or hemi-
♦♦ Anesthesia: Give a block to anesthetize the region where mandibulectomy is done.
specimen is to be obtained. Local infiltration and injections ♦♦ Once the inferior border of the mandible is exposed mas-
into the tissue which should be biopsied is avoided as it seter and medial pterygoid are reflected of from the buccal
leads to the artifacts in the specimen. If a block is not effec- ramus of the mandible.
tive give local infiltration atleast l cm away from the lesion. ♦♦ Similarly temporalis muscle is reflected off the coronoid
♦♦ Stabilisation of tissue: Soft tissue biopsies are done over process and the mylohyoid muscle from the lingual surface
the movable tissues of oral cavity i.e. tongue, lips etc. Den- of the mandible.
tal assistant stabilizes the tissue by stretching it. ♦♦ A bone cut is made anterior to the lesion extending till
♦♦ Hemostasis: Gauze pieces are the best means for compress- the inferior border of the mandible using either a Gigli
ing the tissue and achieving hemostasis. Gauze piece can saw or bur.
also be placed to cover the mouth of suction tip and is ♦♦ Once the cut is made the segment is rotated laterally, the
used to prevent the specimen from being sucked inside. inferior alveolar bundle entering the lingula is identified
♦♦ Incisions: Use a sharp scalpel. Provide two incisions which and ligated.
form an elliptical incision and converge to form a V at the ♦♦ The condyle is then freed from the lateral pterygoid muscle
base, this provides a good specimen and a wound which and the mandible is disarticulated.
is easy to close. Alternatively, a triangular-shaped inci- ♦♦ Hemostasis is achieved and closure is accomplished by ap-
sion can be made which converges in the form of a tip of proximating the buccal and lingual mucoperiosteal flaps.
a pyramid at the base. Incisions should be given parallel ♦♦ Similarly the lip and sub mandibular incisions are ap-
to the nerves and vessels in that region to avoid damage. proximated and closed in layers.
394   Mastering the BDS IIIrd Year  (Last 25 Years Solved Questions)

♦♦ A drain may be inserted to avoid collection of fluid in ♦♦ Floor of the mouth lesions often cause fixation of the
dead space. tongue to the underlying structures with difficulty in
♦♦ Pressure dressing should be applied. speech and inability to open the mouth.
Q.16. Discuss etiology, pathology and treatment of carcinoma ♦♦ When malignant tumor cells invade into the alveolar bone
alveolar margin. (Oct 2003, 15 Marks) of either maxilla or mandible, they usually cause mobility
Ans. For etiology and pathology and treatment refer to ans 1 or exfoliation of regional teeth.
of same chapter. ♦♦ Involvement of inferior alveolar nerve often causes
paresthesia of the lower teeth and the lower lip.
Q.17. Discuss briefly squamous cell carcinoma. ♦♦ Regional lymph nodes are often enlarged, tendered and
 (Nov 2008, 5 Marks) fixed; some of these nodes can be stony hard in consistency.
Ans. Squamous cell carcinoma is histological terminology for ♦♦ Untreated lesions may sometimes destroy the oral tissues
cancer arising from stratified squamous epithelium. and extend into the skin on the outer surface of the face to
Squamous cell carcinoma of oral cavity i.e. oral squamous produce a nodular or lobulated growth on the facial skin,
cell carcinoma is the most common malignant tumor of which appears as an extraoral discharging sinus.
oral cavity. ♦♦ Pathological fracture of the jaw bone may sometimes occur
in untreated cases due to extensive destruction of the bone
Etiology by the tumor.
Following are the etiological factors which lead to oral
Histological Grading
squamous cell carcinoma:
Tobacco smoking: Cigarettes, bidis, pipes, and cigars. Squamous cell carcinoma is divided in following categories by
Broader also known as Broader’s classification
Reverse Smoking ♦♦ Well-differentiated.
♦♦ Use of smokeless tobacco: Snuff dipping, gutkha, tobacco ♦♦ Moderately differentiated.
chewing, tobacco as a toothpaste. ♦♦ Poorly differentiated.
♦♦ Alcohol: Drinking spirits, drinking wines, drinking beers Well-Differentiated Squamous Cell Carcinoma
♦♦ Diet anal nutrition: Vitamin A, B-complex and C deficiency,
Most of the squamous cell carcinomas histologically belong to
♦♦ Nutritional deficiency with alcoholism.
the well-differentiated category.
♦♦ Dental factors: Chronic irritation from broken teeth, Ill-
fitting or broken prosthesis. ♦♦ In this lesion, the tumor epithelial cells to a large extent
♦♦ Radiations: Actinic radiation, X-ray radiation resemble the cells of the squamous epithelium both
♦♦ Viral infections: Herpes simplex virus (HSV), human structurally and functionally.
♦♦ Tumor cells produce large amount of keratin in the form
papilloma virus (HPV), human immunodeficiency virus
of “keratin pearls”.
(HIV), Epstein-Barr virus (EBV)
♦♦ Tumor cells invade into the underlying connective tissue,
♦♦ Chronic infections: Candidiasis, syphilis
where the cells proliferate further and give rise to the
♦♦ Genetic factors: Oncogenes, tumor suppressor genes
formation of many epithelial islands within the connective
♦♦ Pre-existing Oral diseases: Lichen planus, Plummer-Vinson
tissue stroma.
Syndrome, DLE, OSMF
♦♦ Tumor cells often exhibit dysplastic features like cellular
Clinical Features pleomorphism, nuclear hyperchromatism, individual cell
keratinization and altered nuclear-cytoplasmic ratio, loss
♦♦ Carcinomas mostly occur in the 4th to 7th decades of life. of cohesion, etc.
♦♦ Males are more commonly affected ♦♦ Prognosis is better.
♦♦ Lower lip is the most common site, the second most
common site is the lateral border of the tongue. Among all Moderately Differentiated Squamous Cell Carcinoma
intraoral sites, dorsum of the tongue and hard palate are
♦♦ The tumor cells are usually more severely dysplastic than
the least common sites for oral squamous cell carcinoma.
that of the well-differentiated type.
♦♦ The initial lesion may be asymptomatic or can be presented
♦♦ Tumor cells produce little or no keratin and these cells
as white or red nodule or fissure over the oral mucosa.
exhibit greater number of mitotic cell divisions.
♦♦ Initially the lesion is usually painless. ♦♦ There is formation of epithelial islands or cell nests, etc. are
♦♦ More advanced lesions present either as a fast enlarging, diminished since these tumor cells do not differentiate or
exophytic or invasive ulcer or sometimes as a large tumor mature as much as the well-differentiated type of cells do.
mass or a verrucous growth. ♦♦ This tumor also carries a reasonably good prognosis.
♦♦ Ulcerated lesion often shows persistent induration around
the periphery with an elevated and everted margin. Poorly Differentiated Squamous Cell Carcinoma
♦♦ The lesion can be painful either due to secondary infection
or due to involvement of the peripheral nerves by the ♦♦ In poorly differentiated squamous cell carcinoma, the
tumor cells. The lesion can also bleed easily. malignant tumor cells produce no keratin.
Section 2:  General Surgery  395

♦♦ The tumor exhibits extensive cellular abnormalities with Treatment


lack of normal architectural pattern and loss of intercellular
Refer to Ans 6 of same chapter in detail.
bridges between the tumor cells.
♦♦ Mitotic cell division is extremely high and because of this, Q.20. Describe features to benign swelling turning malig­
the neoplastic cells are often very immature and primitive nant. (Jan 2012, 5 Marks)
looking and it is often very difficult even to recognize them Ans. Following are the features of benign swelling turning
as squamous epithelial cells. malignant:
♦♦ Prognosis is poor. • In benign swelling boundaries are encapsulated or
well circumscribed but when it become malignant
Treatment boundaries are poorly circumscribed and irregular.
Surgical excision is the treatment of choice. • In benign swelling surrounding tissue is often
compressed but when it turns malignant surrounding
For more details refer to Ans 1 of same chapter. tissue is invaded.
Q.18. What are the causes of secondaries in neck. Describe • In benign swelling size is small but when it turns
carcinoma of tongue in detail. malignant size is large.
 (May/Jun 2009, 15 Marks) • In benign swelling secondary changes occur less
Ans. Following are the causes of secondaries in neck: often but when it turns malignant secondary changes
• Sub-mental lymph nodes: Infections and metastasis occur more often.
• Submandibular lymph nodes: Infections and carcinoma • In benign swelling growth rate is slow but when it
turns malignant growth rate is rapid.
• Parotid: Eyelid tumors, parotid tumors and
• Benign swelling often compresses the surrounding
tuberculosis
tissues without invading or infiltrating them but
• Prelaryngeal: Laryngeal carcinoma
when it turns malignant it infiltrate and invades
• Pretracheal and paratracheal: Papillary carcinoma of
adjacent tissues.
thyroid and tuberculosis • In benign swelling metastasis is absent but when it
• Upper anterior deep (Jugulo-digastric): Tonsillitis, turns malignant metastasis is frequently present.
carcinoma of posterior one-third of tongue,
oropharyngeal carcinoma and tuberculosis Q.21. Describe the pathology, diagnosis and management of
• Upper posterior deep: Tuberculosis, nasopharyngeal carcinoma of tongue. (Aug 2012, 15 Marks)
carcinoma Ans.
• Middle group: Papillary carcinoma of thyroid Pathology
• Lower anterior (Jugulo-omohyoid): Carcinoma of
tongue and carcinoma of thyroid For pathology refer to Ans 1 of same chapter.
• Lower posterior (Supraclavicular ): Bronchogenic
Diagnosis
carcinoma, intra-abdominal malignancy and
lymphoma ♦♦ Wedge biopsy is the golden rule for confirmation of
For carcinoma of tongue in detail refer to Ans 4 of same diagnosis
chapter. ♦♦ FNAC of lymph nodes is done to rule out invasion of
cancer in lymph nodes.
Q.19. Describe incidence, spread, differential diagnosis and ♦♦ Indirect and direct laryngoscopy is done to see posterior
treatment of carcinoma of lips. (Dec 2009, 15 Marks) one-third growth.
Ans. Incidence ♦♦ CT scan is done to see extension of posterior one-third
Incidence of carcinoma of lip is 15% of head and neck growth or to see status of advanced secondaries.
cancers and 1% of all cancers. ♦♦ MRI can also be done to assess the extent of primary tumor.
♦♦ Chest X-ray is done to see bronchopneumonia
Spread ♦♦ Orthopantomogram (OPG) is done to assess the bony
involvement.
Carcinoma of lip spreads to sub-mental and sub-mandibular
♦♦ Staging should be done by TNM classification
lymph nodes (Level I) and later to other neck nodes bilaterally.
For management refer to Ans 4 of same chapter.
Differential Diagnosis Q.22. a. Enlist predisposing factors, premalignant lesions
♦♦ Basal cell carcinoma: It occurs only in upper lip of oral cavity.
♦♦ Pyogenic granuloma: On palpation it is soft and bleeds b. Discuss clinical features, investigations and man­
easily. agement of carcinoma of tongue (anterior two-
♦♦ Verrucous carcinoma: Surface is papillomatous and white. third) stage II. (Aug 2012, 10 Marks)
♦♦ Necrotizing sialometaplasia: Ulcers in this are painful with no Ans. a. Pre-malignant lesions are defined as “A morphologically
raised borders, no hardening and characteristic histology. altered tissue in which cancer is more likely to occur
♦♦ Keratoacanthoma than its apparently normal counter part”.
♦♦ Malignant melanoma. Following are the premalignant lesions:
396   Mastering the BDS IIIrd Year  (Last 25 Years Solved Questions)

• Leukoplakia ♦♦ Tumor between 1–2 cm in size, partial glossectomy is done


• Erythroplakia with 2 cm clearance from the margin with removal of l/3rd
• Mucosal changes associated with smoking habits of anterior two-third of the tongue.
• Carcinoma in situ ♦♦ Tumor larger than 2 cm, hemiglossectomy is done with
• Bowen disease removal of anterior 2/3rd of tongue on one side up to
• Actinic keratosis, chelitis and elastosis. sulcus terminalis.
   Refer to ans 1 etiology part for predisposing ♦♦ Raw area in these procedures can be left alone when area is
features of premalignant lesions of oral cavity. wide allowing it to granulate and heal by epithelialization.
lf area is small like in wide excision it can be closed by
b. Carcinoma of Tongue (anterior two-third) Stage II.
primary suturing. Wide raw area can also be covered with
Clinical Features PMMF or quilted split-skin graft.
♦♦ Larger primary tumor can be given preoperative
♦♦ Painless ulcer or swelling on the tongue which later on
radiotherapy then later hemiglossectomy is done.
may become painful. Pain is present in the tongue due to
♦♦ Same side palpable mobile lymph nodes are removed by
infection or ulceration or due to involvement of lingual
radical neck block dissection.
nerve.
♦♦ Bilateral mobile lymph nodes are dealt with one side
♦♦ Salivation is excessive and is often blood stained.
radical block and other side junctional block dissection
♦♦ Visible ulcer can be seen on anterior two-third of tongue.
with essentially retaining internal jugular vein (on opposite
Ulcer can bleed on touch; edge, base and surrounding
side) to maintain the cerebral venous blood flow. Other
areas are indurated. Often indurated area is more extensive
option is doing same side radical neck dissection and on
than primary tumor. Edges are everted. Ulcer may cross
opposite side supraomohyoid block dissection.
the midline and extend to the floor or mouth/alveolus/
♦♦ Wide excision is done when growth is in the tip of the
mandible.
tongue.
♦♦ Fetor oris is due to infected necrotic growth.
♦♦ Reconstruction of tongue and other area after surgery: By
♦♦ Ankyloglossia restricted mobility of the tongue. It is due
deltopectoral flap, forehead flap, pectoralis major muscle
to infiltration of the mouth or mandible.
flap, skin grafting.
♦♦ Disarticulation—difficulty in talking is due to disability of
♦♦ Prophylactic block dissection is becoming popular at
the tongue to move freely.
present.
♦♦ Presence of palpable lymph nodes in the neck which
are hard, nodular and get fixed to underlying tissues in
Radiotherapy
advanced stages.
♦♦ In small primary tumor—brachytherapy using caesium
Investigations or iridium needles.
♦♦ Biopsy is the golden standard to identify the carcinoma ♦♦ In large primary tumor initial radiotherapy is given to
of tongue. Biopsy of the lesional margin is done and reduce the tumor size so that resection will be better later.
histopathological evaluation is done. Broader classification ♦♦ Advanced primary as well as secondaries in neck are
histologically divides oral cancer in various stages which controlled by palliative external radiotherapy.
are: ♦♦ Postoperative radiotherapy is given in large tumors to
• Well-differentiated squamous cell carcinoma reduce the chances of relapse.
• Moderately differentiated squamous cell carcinoma Chemotherapy
• Poorly differentiated squamous cell carcinoma
♦♦ It is given in postoperative period and for palliation.
♦♦ FNAC of lymph nodes
♦♦ Price-Hill regimen is commonly used. Drugs are
♦♦ CT scan to see the status of lymph node secondaries.
methotrexate, Vincristine, adriamycin, bleomycin and
♦♦ MRI to assess extent of primary tumor
mercaptopurine.
♦♦ Chest X-ray to see bronchopneumonia
♦♦ It is either given intra – arterially as regional chemotherapy
♦♦ Orthopantomogram
through external carotid artery using arterial pump or
Treatment through IV. It can be given orally also.
Q.23. Write short note on management of lower lip carci­
Stage II tumor means T2N0M0, i.e. Tumor more than 2 cm
noma. (Mar 2006, 5 Marks)
but not more than 4 cm in its greatest dimensions. There is no
Ans. Following is the management of lower lip carcinoma:
regional lymph node metastasis and no distant metastasis. So
• If lesion is less than 2 cm, then curative radiotherapy,
following treatment can be done.
either brachytherapy or external beam radiotherapy.
Surgery It gives a good cure.
• Tumor is more than 2 cm, wide excision is done.
♦♦ Wide excision with l cm clearance in margin and depth is Excision of lower lip up to one-third can be sutured
done in tumor less than l cm in size or in carcinoma in situ. primarily, in layers keeping vermilion border in
Laser (CO2/diode) can be used. proper apposition without causing any microstomia.
Section 2:  General Surgery  397

• Excision of more than one-third of the lip requires • Melanotic neuroectodermal tumor of infancy.
reconstruction using different flaps. ♦♦ Muscles
Methods • Leiomyoma
• Rhabdomyoma
♦♦ Abbe-Estlander’s rotation flap: It is done in lower lip lesions
• Granular cell myoblastoma.
which located at the angle based on labial artery.
♦♦ Giant cell tumor
♦♦ Abbe flap: Done in lower lip lesions at the middle or at the
• Central giant cell tumor
site other than angle based on labial artery. In the later
• Peripheral giant cell tumor
second stage base of the flap should be released once the
• Giant cell granuloma
flap takes up.
♦♦ W flap plasty: It is done for the lower lip middle tumor • Giant cell tumor of hyperthyroidism.
which is less than one-third of the lip. ♦♦ Teratoma.
♦♦ Karapandzic flap: It is done in lower lip defect with less Q.25. Describe etiopathology, clinical feature and manage­
angulation towards upper lip. ment of Ca of tongue. (Apr 2017, 10 Marks)
Q.24. Enumerate benign tumors around oral cavity. Ans.
 (June 2015, 5 Marks)
Ans. Following are the benign tumors around oral cavity: Etiopathology of Carcinoma of Tongue
♦♦ Epithelial tissue Benzopyrenes and nitrosamines in cigarette smoke and
• Papilloma tobacco products, arecoline in areca nut are the carcinogenic
• Keratoacanthoma agents; alterations in activity of genes on 3p, 9p and 17; E6
• Squamous acanthoma and E7 proteins of human papilloma virus inactivate p53
• Nevus. and Retinoblastoma tumor suppressor gene later leading to
♦♦ Fibrous connective tissue overexpression of p16 presence of which is correlated with HPV
• Fibroma associated carcinoma.
• Fibrous hyperplasia For clinical feature and management of carcinoma of tongue,
• Fibrous epulis refer to Ans 4 of same chapter.
• Giant cell fibroma
• Fibrous histiocytoma
• Desmoplastic fibroma
• Myxoma
• Myxofibroma.
♦♦ Cartilage tissue
• Chondroma
• Chondroblastoma
• Chondromyxoid fibroma.
♦♦ Adipose tissue
• Lipoma
• Angiolipoma. Q.26. Discuss etiology, clinical features and management of
♦♦ Bone carcinoma alveolar margin. (Jan 2018, 20 Marks)
• Osteoma Ans. For etiology, refer to Ans 1 of same chapter
• Osteoid osteoma
For clinical features, refer to Ans 12 of same chapter
• Osteoblastoma
For management, refer to Ans 3 of same chapter
• Torus palatines or torus mandibularis
• Osteomatosis.
♦♦ Vascular tissue
• Hemangioma 16. Neck Swelling
• Lymphangioma
Q.1. Describe clinical features and treatment of carotid body
• Arteriovenous fistula
tumor. (Sep 1999, 4 Marks)
• Glomus tumor.
Ans. It is also called as chemodectoma or potato tumor.
♦♦ Neural tissue
• Neurofibroma Definition: It is a non-chromaffin paraganglioma.
• Neurilemmoma It most commonly arises near the bifurcation of common
• Ganglioneuroma carotid artery.
• Traumatic neuroma It is a benign tumor.
398   Mastering the BDS IIIrd Year  (Last 25 Years Solved Questions)

Clinical Features Features


♦♦ It is usually unilateral. ♦♦ Restricted neck movements
♦♦ More common in middle age. ♦♦ Chin pointing towards opposite side
♦♦ Swelling (75%) in the carotid region of the neck which is ♦♦ Presence of squint
smooth, firm, pulsatile and moves only side to side but
not in vertical direction. Treatment
♦♦ It can often compress over esophagus and larynx. Botulinus toxins have been used to inhibit the spastic contraction
♦♦ Headache, neck pain, dysphagia, syncope are other of affected muscle.
presentations.
♦♦ It can present with unilateral vocal cord palsy; can cause
Horner’s syndrome.
♦♦ Features of transient ischemic attacks due to compression
over the carotids, “carotid body syncope. “
♦♦ Thrill may be felt and bruit may be heard.
♦♦ It is located at the level of hyoid bone deep to anterior edge
of the sternomastoid muscle in anterior triangle, vertically
placed, round, firm ‘potato’ like swelling.
♦♦ Often tumor may extend into the cranial cavity along with
internal carotid artery as dumbbell tumor.

Treatment
♦♦ If it is small, it can be excised easily as the tumor is situ
ated in adventitia.
♦♦ When it is large, as commonly observed, complete exci-
sion has to be done followed by placing a vascular graft.
♦♦ During resection a temporary shunt is placed between
common carotid below and internal carotid above to
safeguard cerebral perfusion; external carotid artery is
ligated. Venous or prosthetic graft is placed between
common carotid and internal carotid arteries. Fig. 26: Torticollis
Q.2. Write in short torticollis. (Sep 2002, 5 Marks)
Q.3. Discuss the differential diagnosis of swelling in the
Ans. Torticollis or wryneck is a deformity in which the head lateral aspect of neck. (Sep 2001, 15 Marks)
is bent to one side with the chin point to the outer side. Ans. It is classified according to their location in three triangles
In long standing cases there may be atrophy of the face of the neck:
on the affected side. 1. Submandibular or digastric triangle
The different varieties of wry neck are: • Enlarged lymph node
• Congenital: • Enlarged submandibular salivary gland
– The diagnosis is made by a history of difficult – Calculus
labor, followed by the appearance of a sterno- – Chronic sialadenitis
mastoid tumor. – Cancer
– The affected muscle feels firm and rigid. – Chronic diseases-autoimmune.
2. Carotid triangle
• Traumatic: Fracture dislocation of the cervical spine.
• Aneurysm of carotid artery
• Rheumatic: Sudden appearance of wryneck after an
• Carotid body tumor
exposure to cold or draught is suggestive.
• Branchial cyst
• Inflammatory: For example, from inflammed cervical
• Neurofibroma vagus
lymph node. • Enlargement of thyroid gland
• Spasmodic: When the sternomastoid of the affected • Lymph node swelling (Cold abscess)
side and the posterior cervical muscle of the opposite • Laryngocele.
side are found in a state of spasm. • Sternomastoid tumor.
• Compensatory: For example, from scoliosis, defect in 3. In posterior triangle
sight (ocular torticollis) • Solid swellings:
• From Pott’s disease of the cervical spine. – Metastasis in lymph node
• From contracture: For example, after burns, ulcer, etc. – Tuberculosis
Section 2:  General Surgery  399

– Lymphoma borders are distinct. Both superior and inferior borders are
– Lipoma continuous with the swelling.
– Cervical rib ♦♦ Laryngocele: It is a smooth, oval, boggy swelling which
– Pancoast tumor. moves upwards on swallowing. Expansile cough impulse
• Cystic swellings: is present.
• Lymphangioma ♦♦ Neurofibroma of vagus nerve: It produces swelling in
• Hemangioma carotid triangle in region of thyroid swelling. It is a verti-
• Cold abscess. cally placed oval swelling. On pressure over the swelling
• Pulsatile swellings: dry cough occurs and in some cases bradycardia can occur.
– Subclavian artery aneurysm
– Vertebral artery aneurysm. Posterior Triangle
Solid Swellings
Sub-mandibular or Digastric Triangle
♦♦ Metastasis in lymph nodes: Lymph nodes become en-
Enlarged Sub-mandibular Lymph Node larged and become fixed to the underlying structures.
They form a nodular swelling which is deep to deep fascia. They They become immobile and are stoney hard in consistency.
are palpable only in the neck. The nodes can get enlarged due ♦♦ Tuberculosis: Lymph nodes become enlarged mainly
to following conditions: cervical. The nodes are firm and matted.
♦♦ Acute lymphadenitis: Very often, poor oral hygiene or a ♦♦ Lymphoma: It can involve submandibular lymph nodes
along with horizontal group of nodes in the neck. The
caries tooth produces painful, tender, soft enlargement
nodes are firm or rubbery in consistency.
of these lymph nodes. Extraction of the tooth or with im-
♦♦ Lipoma: It is a localized swelling with lobular surface,
provement of oral hygiene, lymph nodes regress.
non-tender. It is semifluctuant and non-transilluminant. It
♦♦ Chronic tuberculous lymphadenitis can affect these nodes
is mobile with edges slipping between palpating fingers.
along with upper deep cervical nodes. The nodes are firm ♦♦ Cervical rib: It is an extra, rib present in the neck. A hard
and matted. mass is visible or palpated in root of neck.
♦♦ Secondaries in the submandibular lymph nodes arise from ♦♦ Pancoast tumor: It is a tumor felt in the lower part of
carcinoma of the cheek, tongue, palate. The nodes are hard posterior triangle. It is hard in consistency, fixed, irregular
with or without fixity. and sometimes tender. Lower border of mass cannot be
♦♦ Non-Hodgkin’s lymphoma can involve submandibular appreciated.
lymph nodes along with horizontal group of nodes in the
neck. The nodes are firm or rubbery in consistency. Cystic Swellings
♦♦ Lymphangioma: Skin vesicles contain watery or yellow
Submandibular Salivary Gland Enlargement
fluid. Bleeding in vesicle turn into brown or black. Area
The common causes are chronic sialadenitis with or without is soft, spongy, often fluctuant with fluid thrill and trans-
a stone, tumors of the salivary gland or enlargement due to lucency. It is non-compressible. Vesicles will not fade on
autoimmune diseases. They form irregular or nodular swelling. pressure.
The diagnosis is confirmed by bidigital palpation of the gland. ♦♦ Hemangioma: Swelling is warm and bluish in color, non-
Enlarged submandibular gland is bidigitally palpable because pulsatile, soft, fluctuant, transillumination negative. Com-
the deep lobe is deep to mylohyoid muscle. pressibility is present. When the swelling is compressed
between fingers blood diffuses under vascular spaces and
Carotid Triangle when pressure is released it slowly fills up.
♦♦ Branchial cyst: It is located in anterior triangle of neck. ♦♦ Cold abscess: Patient present with history of tuberculosis.
It is soft, cystic, fluctuant and transillumination negative. Lymph nodes are firm and matted. Signs of inflammation
♦♦ Lymph node swelling (Cold abscess): Patient present with are absent.
history of tuberculosis. Lymph nodes are firm and matted.
Q.4. Write short note on swelling midline of neck. 
Signs of inflammation are absent.
 (Sep 2006, 10 Marks)
♦♦ Aneurysm of carotid artery: It is firm, fluctuant and transil-
lumination negative swelling with presence of expansile Or
pulsations. Bruit/thrill can be heard. Write briefly on midline swellings of neck.
♦♦ Carotid body tumor: It is has a typical location, i.e. located  (Dec 2009, 5 Marks)
at level of hyoid bone in upper part of anterior triangle
of neck beneath anterior edge of sternomastoid muscle. Ans. The midline swellings of neck are:
On palpation it moves in transverse direction. Surface is • Ludwig’s angina
smooth or lobulated, borders are round, oval in shape, • Enlarged sub-mental lymph node
vertically placed swelling. • Sub-lingual dermoid cyst
♦♦ Sternomastoid tumor: Swelling is present in infants or • Thyroglossal cyst
children. It is tender, mobile sideways, medial and lateral • Sub-hyoid bursitis
400   Mastering the BDS IIIrd Year  (Last 25 Years Solved Questions)

• Goiter of thyroid, isthmus and pyramidal lobe 3. Swelling move sideways but not vertically as it is
• Enlarged lymph node and lipoma in substernal space tethered by the thyroglossal duct.
of burns
• Retrosternal goiter Enlarged Isthmus of Thyroid Gland
• Thymic swelling Almost all the diseases of the thyroid gland result in enlargement
• Bony swelling arising from the manubrium sterni. of the isthmus. However, a solitary nodule and cysts can occur
in relation to isthmus. The swelling moves with deglutition.
Ludwig’s Angina
However, it does not move on protrusion of the tongue.
♦♦ This is an inflammatory edema of the floor of the mouth. It
spreads to the submandibular region and sub-mental region. Pretracheal and Prelaryngeal Lymph Nodes
♦♦ Tense, tender, browny edematous swelling in the sub- These lymph nodes produce nodular swelling in the midline.
mental region with putrid halitosis is characteristic of One or two discrete nodes are palpable. They can enlarge due
this condition. to following conditions:
Enlarged Submental Lymph Nodes ♦♦ Acute laryngitis: The nodes are tender, soft.
♦♦ Papillary carcinoma of thyroid: The nodes are firm without
The three important causes of enlargement: matting, with or without evidence of thyroid nodule.
1. Tuberculosis: Matted submental nodes, firm in consistency, ♦♦ Carcinoma of the larynx: The nodes are hard in consistency.
with enlarged upper deep cervical lymph nodes, with or
without evening rise of temperature are suggestive of Thymic Swelling
tuberculosis. ♦♦ It is caused by an aneurysm of innominate or subclavian
2. Non-Hodgkin’s lymphoma can present with sub-mental artery.
nodes along with other lymph nodes in the horizontal ♦♦ It is a pulsatile swelling.
group of nodes such as submandibular, upper deep
cervical, pre-auricular, post-auricular and occipital lymph Lipoma in Suprasternal Space of Burns
nodes (external Waldeyer’s ring). Nodes are firm or ♦♦ It is soft and lobular.
rubbery, discrete without matting. ♦♦ Edge of lipoma slips under the palpating finger.
3. Secondaries in the submental lymph nodes can arise from
carcinoma of the tip of the tongue, floor of the mouth, Q.5. Write short note on pulsatile swellings in neck.
central portion of the lower lip. The nodes are hard in  (Dec 2009, 5 Marks)
consistency and sometimes, fixed. Ans. Following are the pulsatile swellings in neck.
• Carotid body tumor: Refer to ans 1 of same chapter in
Sublingual Dermoid Cyst detail
♦♦ It is a type of sequestration dermoid cyst which occurs • Aneurysm
due to sequestration of the surface ectoderm at the site of 1. Arterial Hemangioma:
fusion of the two mandibular arches. Hence, such a cyst • An abnormal communication between artery
occurs in the midline, in the floor of the mouth. and vein results in AV fistula.
♦♦ When they arise from 2nd branchial cleft, they are found • It is a soft, cystic, fluctuant, transillumination
lateral to the midline. Hence, lateral variety. negative, pulsatile swelling.
• A continuous bruit/murmur is characteristic.
Subhyoid Bursitis • On compressing the feeding artery, venous
♦♦ Accumulation of inflammatory fluid in the subhyoid bursa return to heart diminishes which leads to fall in
results in a swelling and is described as subhyoid bursitis. pulse rate and pulse pressure.
♦♦ Bursa is located below hyoid bone and in front of 2. Cirsoid aneurysms:
thyrohyoid membrane. • It is a rare variant of capillary hemangioma
♦♦ It is the swelling in front of neck in midline below hyoid occurring in skin beneath which abnormal artery
bone. communicates with the distended veins.
♦♦ Swelling moves up with deglutition and is tender. • Variant of capillary hemangioma
• Pulsatile swelling
Thyroglossal Cyst • Involves bone
• Treatment is ligation of feeding artery and
♦♦ It arises from thyroglossal tract or duct which extends from
excision of lesion.
foramen cecum at base of tongue to isthumus of thyroid.
♦♦ It is common in females and is painless midline swelling. Q.6. Describe various triangles of neck and their boundaries.
Swelling is deviated to left side. Discuss the differential diagnosis of neck swelling.
♦♦ Thyroglossal cyst exhibits three types of mobilities, i.e.  (Jun 2010, 15 Marks)
1. It moves upwards with deglutition Ans. Each side of neck is the quadrilateral space which is
2. Cyst moves with protrusion of tongue subdivided by sternocleidomastoid into anterior triangle
Section 2:  General Surgery  401

and posterior triangle. These triangles are further Digastric Triangle


subdivided into
Boundaries
• Anterior Triangle
• Posterior Triangle ♦♦ Anteroinferior: Anterior belly of digastric
♦♦ Posteroinferior: Posterior belly of digastric
♦♦ Base: Base of the mandible and an imaginary line joining
the angle of mandible to the mastoid process
♦♦ Apex: Intermediate tendon of diagastric muscle bound
down to hyoid bone by a facial sling.
♦♦ Floor: Is formed by mylohyoid muscle (anteriorly), hyoglos-
sus muscle and small part of middle constrictor (posteriorly).
♦♦ Roof: Is formed by the investing layer of deep cervical
fascia which splits to enclose the submandibular salivary
gland.
Carotid Triangle
Boundaries
♦♦ Superior: Posterior belly of digastric and stylohyoid
♦♦ Anteroinferior: Superior belly of omohyoid
♦♦ Posterior: Anterior border of sternocleidomastoid
♦♦ Roof: Is formed by investing layer of deep cervical fascia
♦♦ Floor: Is formed by four muscles
1. Thyrohyoid
Fig. 27:  Triangles of neck
2. Hyoglossus
3. Middle constrictor of pharynx
Anterior Triangle 4. Inferior constrictor of pharynx.
Boundaries Muscular Triangle
♦♦ Anterior: Anterior midline of the neck extending from Boundaries
symphysis menti above to the middle of suprasternal
notch below. ♦♦ Anterior: Anterior midline of the neck
♦♦ Posterior: Anterior border of sternocleidomastoid ♦♦ Antero-superior: Superior belly of the omohyoid
♦♦ Base: Lower border of the body of mandible and line joining ♦♦ Posteroinferior: Anterior border of sternocleidomastoid.
the angle of mandible with the mastoid process Posterior Triangle
♦♦ Apex: Suprasternal notch, at the meeting point between an- Boundaries
terior border of sternocleidomastoid and anterior midline.
♦♦ Anterior: Posterior border of sternocleidomastoid
Subdivisions of Anterior Triangle ♦♦ Posterior: Anterior border of trapezius
The anterior triangle in subdivided by the diagastric muscle ♦♦ Base: Middle third of the clavicle
and superior belly of omohyoid into following: ♦♦ Apex: Meeting point of sternocleidomastoid and trapezius
on the superior nuchal line.
♦♦ Submental triangle
♦♦ Roof: Is formed by investing layer of deep cervical fascia
♦♦ Digastric triangle
stretching between sternomastoid and trapezius muscles
♦♦ Carotid triangle
♦♦ Floor: Is muscular and is formed by following muscles
♦♦ Muscular triangle.
From Above Downwards
Submental Triangle
♦♦ Semispinalis capitis
This triangle is complete only when the neck is seen from the ♦♦ Splenius capitis
front. Each half of the triangle is visible when viewed from side. ♦♦ Levator scapulae
♦♦ Scalenus posterior
Boundaries
♦♦ Scalenus medius
♦♦ On each side: Anterior belly of diagastric ♦♦ Outer border of 1st rib.
♦♦ Base: Body of hyoid bone
♦♦ Apex: Chin or symphysis menti Sub-divisions of Posterior Triangle
♦♦ Floor: Oral diaphragm formed by the mylohyoid ♦♦ Occipital triangle
muscles. ♦♦ Supraclavicular triangle
402   Mastering the BDS IIIrd Year  (Last 25 Years Solved Questions)

Occipital Triangle (From Above Downwards) ♦♦ Superficial layer is dense and tough while the deep layer
is thin and weak.
♦♦ Occipital artery at apex
♦♦ Stylomandibular ligament which lies between the styloid
♦♦ Spinal part of accessory nerve
process and the angle of the mandible is derived from
♦♦ Four cutaneous branches of cervical plexus of nerves
1. Lesser occipital the deep layer. The ligament is tough, and separates the
2. Great auricular parotid from the submandibular gland.
3. Transverse cervical ♦♦ Communication of parotid space is medial with the lat-
4. Supra clavicular. eral pharyngeal space and with the posterior area of the
♦♦ Muscular branches of C3 and C4 nerves masticator space.
♦♦ Dorsal scapular nerve. ♦♦ Posterior area of the masticator space consists of masseter
muscle, pterygoid muscles, the small pterygomandibular
Supraclavicular Triangle space and the space of the body of the mandible.
♦♦ Trunks of brachial plexus of nerves with their branches ♦♦ As many intraparotid anatomic structures radiate from
• Dorsal scapular parotid gland, surgeon should be familiar with all those
• Long thoracic structures, especially those which should not be sacrificed.
• Nerve to subclavius. Bed of the Parotid Gland
♦♦ Subclavian artery — 3rd part
♦♦ Subclavian vein Complete removal of the parotid gland reveals the following
♦♦ External jugular vein structures:
♦♦ Supraclavicular lymph nodes. ♦♦ Internal jugular vein, i.e. one vein
For differential diagnosis of neck swellings refer to ans 3 and ♦♦ Carotis externa artery and carotid interna artery, i.e. two
ans 4 of same chapter. arteries
♦♦ IX, X, XI, and XII cranial nerves, i.e. four nerves

17. Salivary Gland


Q.1. Describe surgical anatomy of parotid gland. Describe
signs, symptoms and management of pleomorphic
adenoma of parotid gland. 
 (Sep 2008, 5 Marks) (Sep 2005, 15 Marks)
Ans. Parotid gland is a major salivary gland.

Surgical Anatomy of Parotid Gland


♦♦ Parotid gland lies beneath the skin, in front and below
the ear.
♦♦ Parotid gland is contained in the investing layer of the
deep fascia of the neck which is known as parotid fascia.
♦♦ Parotid gland is separated from submandibular gland by
a fascial thickening known as stylomandibular ligament. Fig. 28:  Surgical anatomy of parotid gland
♦♦ Parotid space is occupied by the parotid gland.
♦♦ From anterolateral edge of the gland, parotid duct or
Signs and Symptoms (Clinical Features)
Stensen’s duct passes lateral to the masseter muscle.
Parotid duct turns medial at the anterior margin of the ♦♦ Middle aged women around 40 years are commonly
muscle, where it is related to the buccal fat pad or "boule affected.
de Bichat". ♦♦ Swelling is painless.
♦♦ Location of buccal pad is medial to the parotid duct, be- ♦♦ Parotid swelling has following classical features:
tween the masseter and buccinator muscles. • It present as a swelling in front, below and behind
♦♦ Stenson’s duct pierces the buccinator muscle and enters the ear.
the oral cavity at the level of the upper second molar tooth. • Raises ear lobule.
♦♦ At times accessory parotid tissue may extend along the • Retromandibular groove is obliterated.
stenson’s duct. A short accessory duct may enter the ♦♦ It is rubbery or firm
main duct. ♦♦ Soft area indicates necrosis
♦♦ In long standing cases it can be hard
Parotid Fascia ♦♦ Surface can be nodular
♦♦ Parotid fascia is mainly the splitting of general investing ♦♦ Skin is stretched and becomes shiny
layer which envelops both parotid and submandibular ♦♦ Being a benign tumor it is neither adherent to the skin
glands forming the superficial and deep layers. nor to masseter.
Section 2:  General Surgery  403

♦♦ After few years pleomorphic adenoma shows features of Ans. Pleomorphic adenoma is also known as mixed parotid
transformation into malignancy. tumor or mixed salivary tumor.
♦♦ This is the most common tumor of the major salivary gland.
Management
♦♦ Pleomorphic adenoma is benign epithelial tumor.
♦♦ Surgery is the first line management. ♦♦ Epithelial cells proliferate in strands or may be arranged
♦♦ If only superficial lobe is involved, then superficial pa- in form of acini or cords.
rotidectomy is done wherein parotid superficial to facial Pleomorphic adenoma is mixed tumor because of the presence
nerve is removed. of epithelial cells, myoepithelial cells, mucoid material, pseudo-
♦♦ Various steps in superficial parotidectomy: cartilage and lymphoid tissue.
• Give incision, incision should start in front of tragus,
vertically descend downwards, curve round the ear Etiology
lobule till mastoid process and is carried till the neck. ♦♦ Dardick’s theory: A neoplastically altered epithelial cell
• Facial nerve should be recognized which lies 1cm with potential for multidirectional differentiation can be
inferomedial to pointed end of tragal cartilage of responsible for pleomorphic adenoma.
external ear. Trace posterior belly of digatric till ♦♦ Differentiation of the ductal reserve cells: Intercalated
mastoid process. Facial nerve lies between muscle ductal reserve cells may be differentiated into ductal and
and tympanic plate. myoepithelial cells and later on these cells undergo Mes-
• Both facial nerve and retromandibular vein divides the enchymal metaplasia as they inherently consist of smooth
parotid gland into deep and superficial lobes. Benign muscle like properties.
tumors do not invade faciovenous plane of Patey.
• Gentle handling, good suction and nice hemostasis Clinical Features
provide visibility to nerve. ♦♦ It occurs in middle aged women around 40 years are com-
• Tumor along with lobe should be removed in Toto to monly affected.
avoid spillage. ♦♦ It is usually unilateral, present as single painless, smooth,
♦♦ If both the lobes are involved then total conservative pa- firm, lobulated mobile swelling in front of parotid with
rotidectomy is done. In this, tumor along with the normal positive Curtain sign, i.e. as the deep fascia is attached
lobe is removed by retaining facial nerve. Avoid rupture above to zygomatic bone, it acts as curtain, not allowing
of gland. parotid swelling to move above the level of zygomatic
Q.2. Describe in short mixed parotid tumor.  bone. This is curtain sign.
 (Sep 1999, 10 Marks) ♦♦ Obliteration of retromandibular groove is common.
Or ♦♦ Ear lobule is raised or lifted.
Write short note on mixed parotid tumor.  ♦♦ Swelling is rubbery or firm. Soft area indicates necrosis. In
 (June 2014, 5 Marks) (Aug 2012, 5 Marks) long standing cases it can be hard, surface can be nodular.
(Apr 2007, 5 Marks) (Sep 2006, 6 Marks) Skin is stretched and becomes shiny.
(Sep 2000, 5 Marks) (Dec 2010, 3 Marks) ♦♦ When deep lobe is involved, swelling is located in lateral
(Nov 2014, 5 Marks) (Dec 2015, 5 Marks) wall of pharynx, posterior pillar and over soft palate. Deep
Or lobe tumor passes through Patey’s submandibular tunnel
Write in short about mixed parotid tumor.  pushing tonsils, pharynx, soft palate often without any
 (July 2016, 5 Marks) visible swelling or only small swelling when only deep
lobe tumor is present.
Or
♦♦ Being a benign tumor it is neither adherent to the skin

Write briefly on clinical features and treatment of nor to masseter.
pleomorphic salivary adenoma. (Mar 1998, 6 Marks) ♦♦ After few years pleomorphic adenoma shows features of
Or transformation into malignancy.
Write in short about etiology, clinical features and Investigations
management of mixed parotid tumor. 
♦♦ Fine needle aspiration cytology: It is done to confirm diagnosis
 (Sep 2000, 15 Marks)
and rule out the malignancy.
Or ♦♦ CT scan: This is to be done when tumor arises from deep
Discuss about pleomorphic adenoma. lobe. It defines the extraglandular spread and extent of
(Sep 2008, 5 Marks) parapharyngeal disease as well as cervical lymph nodes.
Or ♦♦ FNAC of lymph nodes: Palpable lymph nodes in neck are to
Write short note on pleomorphic adenoma. be examined for malignancy.
(Jan 2012, 5 Marks) (June 2010, 5 Marks) ♦♦ X-ray of bones: For seeing the bony resorption.
Or ♦♦ MRI is the better method compared to CT scan. MRI pro-
Write short answer on pleomorphic adenoma.  vides better soft tissue delineation, i.e. superior perineural
 (June 2018, 5 Marks) invasion.
404   Mastering the BDS IIIrd Year  (Last 25 Years Solved Questions)

Complications Clinical Features


♦♦ Recurrence in 5 to 50% of cases. ♦♦ Pain and swelling on one side of face
♦♦ Malignancy is seen in 3 to 5% of tumors ♦♦ Browny edematous swelling over the parotid region with
♦♦ Malignancy is seen in 10% of tumors in long duration. all signs of inflammation.
♦♦ Cellulitis of overlying skin
Treatment ♦♦ Pus comes out on pressing the parotid gland.
♦♦ Surgery is the first line treatment.
Investigations
♦♦ If only superficial lobe is involved, then superficial pa-
rotidectomy is done wherein parotid superficial to facial ♦♦ Ultrasonography of parotid region should be done.
nerve is removed. ♦♦ Pus collected from duct orifice should be sent for culture
♦♦ Various steps in superficial parotidectomy: and sensitivity
• Give incision, incision should start in front of tragus, ♦♦ Needle aspiration from the abscess is done to confirm
vertically descend downwards, curve round the ear formation of pus.
lobule till mastoid process and is carried till the neck.
Treatment
• Facial nerve should be recognized which lies 1cm
inferomedial to pointed end of tragal cartilage of ♦♦ Conservative line of management:
external ear. Trace posterior belly of digatric till • It is indicative in stage of cellulitis with no evidence
mastoid process. Facial nerve lies between muscle of abscess.
and tympanic plate. • Maintain good oral hygiene.
• Both facial nerve and retromandibular vein divides the • Proper antibiotic mainly cloxacillin 500 mg 6 hourly
parotid gland into deep and superficial lobes. Benign along with metronidazole 400 mg 8 hourly.
tumors do not invade faciovenous plane of Patey. ♦♦ Surgical treatment:
• Gentle handling, good suction and nice hemostasis • Incision and drainage should be done under general
provide visibility to nerve. anesthesia.
• Tumor along with lobe should be removed In Toto to • Incise the skin in front of tragus vertically and then
avoid spillage. parotid sheath is opened horizontally. Pus is drained
♦♦ If both the lobes are involved then total conservative pa- by using the sinus forcep. This is known as blair’s
rotidectomy is done. In this, tumor along with the normal incision. Antibiotics should be continued.
lobe is removed by retaining facial nerve. Avoid rupture • Proper hydration, mouthwash using povidone iodine
of gland. or potassium permagnate solutions.
Q.3. Write short note on acute parotitis.  Q.4. Discuss clinical features, investigation and mana­
 (Aug 2012, 5 Marks) gement of submandibular salivary calculus. 
Ans. Acute inflammation of parotid can occur due to bacterial  (Sep 2001, 15 Marks)
or non-bacteria causes. This may be unilateral and
Or
bilateral.
Describe etiology, clinical features, investigations and
Acute Non-suppurative Parotitis (Mumps Parotitis) management of submandibular gland stone.
(Feb 2013, 10 Marks)
It is an acute generalized viral disease with painful enlargement Ans. This is a pathological condition characterized by
of salivary gland chiefly parotid. formation of calcified mass (sialolith) within the salivary
Clinical Features gland or its duct.

Fever, headache, muscular pain are usually found, both parotids Etiology
are enlarged with pain and temperature.
The exact cause for sialolith or calculus formation is not clear
Treatment but factors which contribute to its formation are:
♦♦ Stagnation of saliva
Only symptomatic treatment as analgesic and anti-inflammatory ♦♦ Focus for sialolith formation resulting from ductal epithe-
drugs. lial inflammation and injury.
Acute Suppurative Parotitis ♦♦ Poorly understood biological factors favoring precipitation
of calcium salts.
It is and acute inflammation of parotid gland caused by Hilus is the most common site for the formation of sialolith
Staphylococcus aureus. but it can arise anywhere throughout the ductal system. 80%
Streptococcus viridans and pneumococci may be involved. of all salivary duct stones develop in the submandibular or
Pathogenesis: The bacterium reaches to the salivary gland Warthin’s duct. Predisposition of sialolith formation for the
through the stenson’s duct. This is called as retrograde infection. submandibular gland can be due to:
Section 2:  General Surgery  405

♦♦ Composition of secretion of submandibular salivary gland • Incision: It should be a skin crease incision over the
is more alkaline and viscous. lower pole of the gland.
♦♦ Submandibular gland consists of higher concentration of • Mobilization of the gland: Division of the facial artery
calcium and phosphorus ions as compared to other major twice, once in deeper plane on the posterolateral aspect
salivary glands. and another at the superolateral aspect close to the
♦♦ Both submandibular gland and duct are placed in such an lower border of the mandible which gives mobilization
anatomically dependent position that the flow of saliva of the gland.
is against gravity which gives more chances for stasis of • Excision of the gland: It is done by ligating and dividing
saliva inside the ducts. the submandibular duct.
♦♦ Stagnation of secretions in Warthin’s duct can also due to Q.5. Describe the etiology, clinical features and treatment
angulation of duct as it courses around the mylohyoid mus- of submandibular sialolithiasis. (Feb 1999, 8 Marks)
cle and the vertical orientation of the distal duct segment. Ans. Sialolithiasis is the formation of hardened intraluminal
Clinical Features deposits in ductal system of salivary gland which
obstructs the normal flow of saliva.
♦♦ Patient complains of periodic painful swelling when eating,
interspersed with periods of remission. Etiology
♦♦ Occasionally patients report spontaneous extrusion of The exact cause for sialolith or calculus formation is not clear
small calculi from the ducts. but factors which contribute to its formation are:
♦♦ Most common finding on examination is point tenderness
♦♦ Stagnation of saliva
in the region of the hilum or, near Wharton’s duct of the
♦♦ Focus for sialolith formation resulting from ductal epithe-
submandibular gland.
lial inflammation and injury.
♦♦ Salivary secretion may be affected slightly.
♦♦ Poorly understood biological factors favoring precipitation
♦♦ A gelatinous, cloudy, mucopurulent material is seen in
of calcium salts.
basically clear and adequate saliva. This mucopurulent
Hilus is the most common site for the formation of sialolith
material is derived from the inflammatory ductal changes
but it can arise anywhere throughout the ductal system. 80%
caused by calculus blockage and salivary stagnation.
of all salivary duct stones develop in the submandibular or
♦♦ If treatment is not instituted early pronounced exacerba-
Warthin’s duct. Predisposition of sialolith formation for the
tions are seen, characterized by an acute suppurative
submandibular gland can be due to:
process with attendant systemic manifestations.
♦♦ Pus may exude from the duct orifice. ♦♦ Composition of secretion of sub mandibular salivary gland
♦♦ Mucosa around the duct is inflamed, particularly in the is more alkaline and viscous.
floor of the mouth where swelling, redness and tenderness ♦♦ Submandibular gland consists of higher concentration of
are present along the course of Wharton’s duct. calcium and phosphorus ions as compared to other major
♦♦ Glands are enlarged, tender and tense. Palpation of the salivary glands.
gland and the duct causes pain and a flow of pus. ♦♦ Both submandibular gland and duct are placed in such an
anatomically dependent position that the flow of saliva
Investigations is against gravity which gives more chances for stasis of
♦♦ Occlusal radiograph saliva inside the ducts.
♦♦ Intraoral periapical radiograph: Submandibular stones are ♦♦ Stagnation of secretions in Warthin’s duct can also due
mostly radioopaque. to angulation of duct as it courses around the mylohy-
♦♦ Ultrasound (Excellent non-invasive method) oid muscle and the vertical orientation of the distal duct
♦♦ Sialography: A retrograde injection of a radiopaque dye segment.
as neohydriole into the duct of salivary gland. (Occluded Clinical Features
submandibular salivary duct can be best observed by
simple palpitation. ♦♦ It is usually seen in patients in the 5th to 8th decade of life.
♦♦ Recurrent swelling of the gland region is seen at the meal
Treatment time.
♦♦ Stone in the submandibular duct: ♦♦ Recurrent episodes of acute, subacute or suppurative
• Small stone in the distal part of the duct is removed sialadenitis are present.
manually. ♦♦ Swelling is sometimes seen as hard lump present in the
• Stone in deeper parts require operation. This can be floor of the mouth or cheek.
removed by incising the mucosa over the floor of the ♦♦ Submandibular salivary gland becomes tense and
mouth, after stabilizing the stone. tender.
♦♦ If the stone is present inside the gland: ♦♦ Swelling and tenderness subside only to recur again dur-
This requires excision of submandibular salivary gland. ing meal time.
Three steps of dissection of gland includes incision, mo- ♦♦ Large submandibular calculi can be seen as a swelling in
bilization and excision the floor of the mouth.
406   Mastering the BDS IIIrd Year  (Last 25 Years Solved Questions)

♦♦ Stone may be palpable during bimanual palpation and ♦♦ Inorganic materials: Calcium phosphate, calcium carbonate,
may be movable up and down the duct. combined with other salts such as Mg, Zn, etc.
♦♦ As in chronic infection and obstruction, the gland un- ♦♦ Organic materials, i.e. glycoproteins, mucopolysaccharides
dergoes atrophy rarely, becomes indurated and when and cellular debris.
operated it is seen to be adherent to adjacent structures. ♦♦ Aggregations of bacteria, clumps of epithelial cells, mucus,
blood clots following trauma, are all suggested to form foci.
Treatment
Q.7. Write short note on Warthin’s tumor.
Treatment is surgical. (Oct 2007, 5 Marks)
Ans.
Removal of Submandibular Calculi ♦♦ Warthin’s tumor is also called adenolymphoma.
(Transoral sialolithotomy) ♦♦ It is a benign parotid tumor, it constitute about 10% of
♦♦ Place the patient in sitting position and give local anes- parotid tumors.
thesia. ♦♦ Origin of adenolymphoma during development some
♦♦ Locate the stone accurately by using radiographs and parotid tissue gets included within lymph nodes which
palpation. are present within the parotid sheath.
♦♦ Pass a suture behind the stone as well as below the duct ♦♦ It involves only superfical lobe of parotid gland.
to prevent stone from sliding backwards during removal.
Etiology
♦♦ Retract the tongue for proper visualization.
♦♦ Palpate submandibular gland extraorally in submandibu- Smoking and radiation exposure can be the cause.
lar region and is pushed upwards toward floor of mouth
to fix intraoral tissues under tension. During this take care Clinical Features
of lingual nerve and sublingual gland. ♦♦ Middle aged or elderly males are commonly affected
♦♦ If the sialolith is present posteriorly, incision should be usually they are smokers.
placed slightly medially to avoid injury to the lingual ♦♦ Can be bilateral, in some cases.
nerve. ♦♦ It has smooth surface, round border with soft, cystic fluctu-
♦♦ Place a superficial incision through mucosa alone and give ant swelling in lower pole often bilateral and is non-tender.
blunt dissection to reach the duct for preventing injury to ♦♦ Classically, situated at the lower pole of parotid elevating
the lingual nerve. the ear lobule.
♦♦ If stone is more anteriorly placed, incision is given medial ♦♦ May be multicentric.
to plica sublingualis or else there are chances of injury to ♦♦ This tumor affects only parotid gland.
sublingual gland.
Investigations
♦♦ Duct should be located at place where stone is lodged. As
duct is located, a longitudinal incision is given directly ♦♦ Adenolymphoma produces “hot spot” in 99Technetium
over the duct where stone is located. pertechnetate scan which is diagnostic.
♦♦ Transverse incision should not be given as it retracts and ♦♦ FNAC can be done
gets divided completely and a salivary fistula may be ♦♦ Biopsy is done and histology reveals:
formed. • Cyst is lined by a bilayered oncocytic epithelium,
♦♦ Incision given should reveal the stone and is of sufficient the inner cells of which are tall columnar with fine
length to be removed easily. Stone can usually be removed granular and eosinophilic cytoplasm and slightly
easily with a forceps or a larger stone may need to be hyperchromatic nuclei. The outer layer consists of
crushed into smaller pieces and removed. basaloid cells.
♦♦ A probe is then passed from the caruncle to the region of • An eosinophilic coagulum is present within the cystic
stone to ensure patency of the duct in the anterior region. spaces.
♦♦ Incision on the duct need not be sutured. Incision in the • The numerous lymphocytic components may
floor of the mouth should be sutured with interrupted represent normal lymphoid tissue within which tumor
sutures. is developed.
Q.6. Write short note on Sialolithiasis. 
Treatment
 (May/June 2009, 5 Marks) (Sep 2007, 5 Marks)
Ans. Sialolithiasis is the formation of hardened intraluminal It is best treated by superficial parotidectomy which spares the
deposits in ductal system of salivary gland which facial nerve
obstructs the normal flow of saliva.
For etiology, clinical features and management, refer to Q.8. Write short note on parotid fistula.
Ans 5 of same chapter  (Jan 2011, 5 Marks)
Ans. It is an uncommon condition, which occurs after the
Sialolith Composition surgery on the parotid gland.
Sialolith is made up of: It may arise from parotid duct or gland.
Section 2:  General Surgery  407

Types • It is an ascending bacterial parotitis, due to reduced


♦♦ Duct fistula forms after superficial parotidectomy. It is pro- salivary flow, dehydration, starvation, sepsis, after
fuse and often persisting. So duct should be ligated using major surgery, radiotherapy for oral malignancies
non-absorbable suture as far as possible, anteriorly to allow and poor oral hygiene.
normal saliva drainage from deep lobe. If common duct • Parotid fascia is densely thick and tough and so
is ligated deep lobe atrophies without causing any fistula. parotid abscess does not show any fluctuation until
♦♦ Gland fistula occurs from the raw surface after superfi- very late stage.
cial parotidectomy. It is mild and symptom subsides in • Causative organism are Staphylococcus aureus
a month with anti-cholinergic drugs. Jacobsom tympanic (commonest), Streptococcus viridans and often others
neurectomy completely stops the secretion from the fistula like gram-negative and anaerobic organisms.
in this type. Clinical Features
♦♦ Pyrexia. malaise, pain and trismus.
Etiology ♦♦ Red, tender, warm, well-localized, firm swelling is seen in
the parotid region (brawny induration).
♦♦ Rupture or bursting of parotid abscess.
♦♦ Tender lymph nodes are palpable in the neck.
♦♦ Inadvertent incision for drainage of parotid gland ♦♦ Features of bacteremia are present in severe cases.
♦♦ Penetrating injury to parotid gland ♦♦ Pus or cloudy turbid saliva may be expressed from the
♦♦ As a complication after the superficial parotidectomy. parotid duct opening.
Clinical Features Investigations
♦♦ Ultrasonography of parotid region should be done.
♦♦ The chief complaint is an opening on the cheek, which
♦♦ Pus collected from duct orifice should be sent for culture
discharges during meal.
and sensitivity
♦♦ *Excoriation of adjacent structure takes place.
♦♦ Needle aspiration from the abscess is done to confirm
♦♦ Tenderness and induration formation of pus.
♦♦ Trismus.
Treatment
Diagnosis
♦♦ Antibiotics are started depending on culture report.
♦♦ Sialography is done to find out origin of fistula whether ♦♦ When it is severely tender and localized, incision and
from parotid gland or duct or ductules. drainage is done under general anesthesia. Skin is incised
♦♦ Fistulogram or CT fistulogram should be done. in front of tragus vertically and then parotid sheath is
♦♦ Discharge study opened horizontally. Pus is drained using sinus forceps
♦♦ MRI and is sent for culture. Antibiotics should be continued.
♦♦ Proper hydration, mouth wash using povidone iodine;
Treatment potaium permanganate solutions, nutrition. Often patient
♦♦ Anticholinergics: Hyoscine bromide (probanthine). with parotid infection needs admission and treatment.
♦♦ Radiotherapy. Q.10. Write short note on sialadenitis. (Mar 2008, 3 Marks)
♦♦ Often exploration of fistula is required. Ans. Sialadenitis is defined as the inflammation of the salivary
♦♦ Repair or reinsertion of the duct into the mucosa. gland.
♦♦ Newman Seabrook operation—A probe is passed into the
parotid duct through the opening in mouth. Another probe Types
is passed through the fistula. Duct and fistula are dissected
♦♦ Acute bacterial sialadenitis
over the probe. After removal of the fistula tract severed
♦♦ Chronic bacterial sialadenitis
duct ends are identified; and ends are trimmed. Probes are
♦♦ Chronic sclerosing sialadenitis/ Kuttner’s disease
removed. A tantalum wire is passed into the duct across
♦♦ Allergic sialadenitis.
the severed ends and duct is sutured over it using 4 zero
vicryl. Tantalum stent is removed after 3 weeks. Etiology
♦♦ If still persists, auriculotemporal nerve which supplies ♦♦ Microorganisms, i.e. S. aureus, S. viridans
secretomotor component of parotid is cut. ♦♦ Predisposing factor, i.e. dehydration, malnutrition, cancer
♦♦ If there is stenosis at the orifice of the Stenson’s duct, papil- and surgical infections
lotomy at the orifice may help. ♦♦ Poor oral hygiene
♦♦ Total conservative parotidectomy is done in failed cases. ♦♦ Drugs: Anti-Parkinson, diuretics and antihis­taminics.
Q.9. Write short note on acute parotid abscess. 
 (Sep 1997, 6 Marks) Symptoms
Ans. It is a result of an acute bacterial sialadenitis of the ♦♦ Fever
parotid gland. ♦♦ Sudden onset of pain at the angle of jaw.
408   Mastering the BDS IIIrd Year  (Last 25 Years Solved Questions)

Sign ♦♦ High dose of parentral antibiotic


♦♦ IV saline is given.
♦♦ Unilateral involvement of parotid gland is common
♦♦ Parotid gland is tender, enlarged and the overlying skin Surgical Treatment
is warm and red
♦♦ Swelling causes elevation of the ear lobule. If improvement does not occur surgical drainage of the affected
♦♦ Intraorally, parotid papilla may be inflamed gland should be performed.
♦♦ Cervical lymphadenopathy
Q.11. Name the treatment modalities and differentiating
Management
features of parotid abscess and periodontal abscess.
♦♦ Meticulous oral hygiene should be practiced (Jan 2012, 5 Marks)
♦♦ Soft diet should be given Ans.

Features Parotid abscess Periodontal abscess


Treatment modality Antibiotics should be started and if it is tender •  Antibiotics should be started. Incision and drainage is given.
localized incision and drainage is done under •  Debridement of root surface is done
general anesthesia •  If roots are denuded beyond apical third extraction of tooth is done
Pathogenesis It is an ascending bacterial parotitis due to It occurs in pre-existing periodontal pocket. Bacteria inside the pocket
reduced salivary flow, dehydration, starvation, multiply and cause sufficient irritation
sepsis, major surgery, radiotherapy for
malignancies and poor oral hygiene
Location Parotid gland Gingiva
Features Red, tender, warm, well localized firm swelling •  Presence of swelling of soft tissues over the surface of involved root
in parotid region •  Tooth becomes mobile and tender and pus drains from gingival crevice

Q.12. Write in brief on Sjögren’s syndrome. Investigations


 (Feb 2013, 5 Marks) ♦♦ Sialography: Presence of snowstorm and branchless fruit
Or laden tree appearance.
Write short note on Sjögren’s syndrome. ♦♦ Rose bengal staining test: Keratoconjunctivitis sicca is char-
 (Sep 2007, 3 Marks) (Mar 2008, 3 Marks) acterized by corneal keratotic lesion which stain pink when
‘rose bengal’ dye is used.
Ans. It is a chronic inflammatory autoimmune disorder that ♦♦ Schirmer test: Lacrimal flow rate is measured by this test. In
affects salivary, lacrimal and other exocrine gland. this test a strip of filter paper is placed in between eye and
eyelid for determining degree of tears which is measured in
Types
millimeter. When flow is reduced to 5 mm in 5 minute sam-
♦♦ Primary Sjögren’s syndrome: It is also known as Sicca syn- ple, patient is considered positive for Sjögren’s syndrome.
drome. It consists of dry eyes, i.e. xerophthalmia and dry ♦♦ Sialometry: In this salivary flow rate estimation is carried
mouth, i.e. Xerostomia. out. Stimulated flow rate in symptomatic primary and
♦♦ Secondary Sjögren’s syndrome: It consists of dry eyes, i.e. xeroph- secondary sjögren’s syndrome is below 0.5 to 1 mL/min.
thalmia, dry mouth, i.e. xerostomia and collagen diso-rders, i.e. ♦♦ Sialochemistry: In Sjögren’s syndrome saliva of parotid
rheumatoid arthritis or systemic lupus erythematous. gland is has twice total lipid content and high phospho-
lipids and glycolipids.
Clinical Features ♦♦ Blood investigations should be done.
♦♦ Xerostomia is present with unpleasant taste, soreness and
difficulty in eating dry fruits. Treatment
♦♦ Patient also complains of xerophthalmia and arthralgia ♦♦ Ocular lubricants, i.e. artificial tears should be used and
♦♦ Severe tiredness is present. provide relief.
♦♦ There is cobble stone appearance of tongue. ♦♦ Xerostomia is managed by saliva substitutes.
♦♦ There is often secondary acute bacterial sialadenitis and ♦♦ Frequent drinking of water is mandatory.
rapid progressive dental caries. ♦♦ Maintenance of oral hygiene is mandatory.
♦♦ Burning sensation present in the eyes. ♦♦ Fluoride application should be done.
♦♦ Parotid gland is predominantly affected, sometimes sub- ♦♦ Various saliva stimulants, i.e. pilocarpine, bromhexine and
mandibular and minor glands can also be affected. cevimeline are used.
Section 2:  General Surgery  409

♦♦ If salivary gland is enlarged to the extent that it is giving ♦♦ Advanced tumor with the fixed nodes in neck may require
discomfort to the patient, then surgery is carried out. radiotherapy but the response rate is poor.
Q.13. Classify tumors of parotid gland. Discuss clinical
Postoperative Radiotherapy
features and management of malignant parotid tumor.
 (Jan 2018, 20 Marks) ♦♦ This is useful to reduce the chances of relapse.
Ans. Classification of tumors of parotid gland ♦♦ Mainly external radiotherapy should be given.

International Classification of Parotid Tumors Chemotherapy


♦♦ It is also given.
Epithelial ♦♦ Drugs given here depend on the tumor type.
♦♦ Adenomas ♦♦ Intra – arterial chemotherapy is beneficial
• Pleomorphic adenoma ♦♦ Overall efficacy of chemotherapy is less as compared to
• Monomorphic adenoma radiotherapy.
–– Adenolymphoma ♦♦ 5FU, Cisplatin, Doxorubicin, epirubicin, cetuximab are
–– Oxiphilic used.
–– Other type
♦♦ Mucoepidermoid—low grade malignancy
♦♦ Acinic cell tumor 18. Thyroid and
♦♦ Carcinoma
• Adenoid cystic carcinoma Parathyroid Gland
• Adenocarcinoma
• Epidermoid carcinoma Q.1. Write short note on thyroid crisis.(Sep 2009, 6 Marks)
• Undifferentiated carcinoma Or
• Carcinoma Ex pleomorphic adenoma
Write short note on thyroid storm/crisis.
Non-epithelial  (Dec 2012, 5 Marks)
Ans. It is also known as thyroid crisis or thyroid storm.
♦♦ Hemangioma
♦♦ Lymphangioma • It occurs in a thyrotoxic patient inadequately
♦♦ Neurofibroma prepared for thyroidectomy.
• Other causes are infection, trauma, pre-eclampsia,
Metastatic diabetic ketosis, emergency surgery and stress.
♦♦ Epidermoid carcinoma • It is an acute life threatening metabolic state which
♦♦ Malignant melanoma is induced by excess release of thyroid hormones in
individuals with thyrotoxicosis.
Malignant Parotid Tumor
Etiology
It is also known as carcinoma ex pleomorphic adenoma
♦♦ Previously thyroid storm was a common complication of
Clinical Features of Malignant Parotid Tumor toxic goitre surgery during intraoperative and postopera-
♦♦ It occurs from 2nd to 9th decade of life. But is common tive stages.
between 5th to 6th decades. ♦♦ In modern era thyroid storm is seen in a thyrotoxic patient
♦♦ Pain is very commonly present. with intercurrent illness or surgical emergency.
♦♦ Size of the tumor is very large and tumor can be associated ♦♦ Most common cause of thyroid storm is intercurrent ill-
with ulceration. ness or infection.
♦♦ These tumors can also lead to facial nerve palsy. ♦♦ Various other causes which rapidly increase the thyroid
♦♦ Tumor gets fixed to underlying structures as well as to hormone levels are radioiodine therapy, withdrawal of
overlying mucosa or skin. antithyroid drug therapy, vigorous thyroid palpation,
iodinated contrast dye, ingestion of thyroid hormone and
Management of Malignant Parotid Tumor sepsis or infection.
Radical Parotidectomy
Clinical Features
♦♦ It is the removal of both the lobes of parotid, facial nerve,
parotid duct, fiber of masseter, buccinators, pterygoids ♦♦ Thyroid crisis is present in l2–24 hours after surgery; with
and radical block dissection of the neck. severe dehydration, circulatory collapse, hypotension,
♦♦ If there is no involvement of the facial nerve, it should hyperpyrexia, tachypnea, hyperventilation, palpitation,
be preserved. But, if facial nerve is removed it should be restlessness, tremor, delirium, diarrhea, vomiting and
reconstructed by the greater auricular nerve or sural nerve cardiac failure; later coma.
410   Mastering the BDS IIIrd Year  (Last 25 Years Solved Questions)

♦♦ Bayley’s symptom complex of thyroid storm are insomnia, ♦♦ Carcinoma


anorexia, diarrhea, vomiting, sweating, emotional ♦♦ Cyst.
instability, fever, tachycardia, aggravated toxic features, Clinical Features
multiorgan dysfunction.
♦♦ Common in female, seen in age group 20-40 year.
Treatment ♦♦ Long duration of swelling in front of neck, dyspnea,
♦♦ Patient should be sedated immediately with morphine dysphagia.
or pethidine. ♦♦ Various obstructive signs are present i.e. stridor, tracheal
♦♦ Hyper pyrexia should be controlled by ice bag, rapid deviation, neck vein engorgement.
sponging, hypothermic blanket, and rectal ice irrigation. ♦♦ Single nodule is present.
♦♦ Oxygen is administered and IV glucose saline solution ♦♦ Hard area may suggest calcification and soft area necrosis.
should be combat dehydration. ♦♦ Sudden increase in size may occur due to hemorrhage.
♦♦ Potassium may be added to control tachycardia. ♦♦ Solitary nodule has more tendencies to change in
♦♦ Hydrocortisone is often highly effective. malignancy then MNG (multi-nodular goiter).
♦♦ Lugol’s iodine should be given IV Complication
♦♦ Propranolol should be used 20–40 mg 6 hourly.
♦♦ Calcification in long standing.
♦♦ For atrial fibrillation, digitalis may be cautiously admin-
♦♦ It may change into MNG (multi-nodular goitre).
istered.
♦♦ Sudden hemorrhage causes sudden enlargement of gland
♦♦ Large dose of Propyl thiouracil orally, rectally or through
and even causes dyspnea.
Ryle’s tube.
♦♦ Patient may develop secondary thyrotoxicosis.
♦♦ Saturated solution of potassium iodide.
Q.2. Write short note on solitary thyroid nodule.  Investigations
 (Mar 2006, 5 Marks) ♦♦ Thyroid scan: This is basically a radioactive scan which
Ans. makes out a hot or cold nodule. Hot nodules are not ma-
♦♦ It is the end stage result of diffuse goiter. lignant and are toxic while cold nodules are malignant but
♦♦ Almost all the thyroid swellings initially present as soli- they can be a simple cyst.
tary nodule. ♦♦ Thyroid function tests: T3, T4 and TSH are not of use as most
♦♦ Solitary thyroid nodule is a discrete lesion/nodule within of the nodules are euthyroid.
the thyroid gland and or radiologically distinct from sur- ♦♦ Ultrasound: It shows either cystic or solid nodule. Solid
rounding thyroid parenchyma. swellings can be edema or carcinoma.
♦♦ FNAC: It shows benign or indeterminate or malignant and
Types at times it is undiagnostic also.
♦♦ Toxic solitary nodule—3—5% of solitary nodules of ♦♦ Power Doppler can be done to know vascularity of the
thyroid gland. Vascularity is described in resistive index.
♦♦ Nontoxic solitary nodule. ♦♦ Serum calcitonin estimation is done if FNAC confirms
medullary carcinoma.
Based on Radioisotope Study ♦♦ CT scan or MRI can only be done in selective cases.
♦♦ Hot: Means autonomous toxic nodule. Normal surround- ♦♦ X-ray of neck is done to see tracheal deviation.
ing thyroid tissue is inactive and so will not take up isotope.
Management
Nodule is overactive. It is 5% common of which only 5%
can be malignant. ♦♦ Non toxic benign nodule is treated with observation
♦♦ Warm: Normally functioning nodule. Nodule and sur- without any therapy. There is no role of any hormone
rounding normal thyroid will take up the isotope (active). therapy (L thyroxine). Annual clinical examination and
It is l0% common of which l0% can be malignant. ultrasound neck is needed during essential follow-
♦♦ Cold: Nonfunctioning nodule; may be malignant (need not up—any nodule of 20% increase in size or more than 2
be always). Nodule will not take up isotope (underactive). mm increase in diameter warrants a repeat FNAC and
It is 80% common of which 20% are malignant. hemithyroidectomy may be considered. Compressive
symptoms and cosmesis are the indications for surgery,
Etiopathogenesis i.e. hemithyroidectomy.
♦♦ Puberty or pregnancy nodule ♦♦ Solitary toxic nodule needs initial antithyroid drugs and
♦♦ Iodine deficiency nodule then radioactive iodine therapy (5 m curie); occasionally
♦♦ Adenoma surgery is done, i.e. hemithyroidectomy.
Section 2:  General Surgery  411

♦♦ During thyroid surgery complete thyroid gland should


be explored properly. Care to be taken not to miss any
similar nodule in any other part of gland. If there are no
nodules and only solitary nodule is found it is resected
with normal surrounding thyroid tissue, i.e. resection
enucleation.
♦♦ If nodule is situated at junction of isthmus and the lobe,
hemithyroidectomy is done. Histopathology of excised
nodule is done. In histopathology report if there is presence
of any evidence of malignancy immediate total thyroid-
ectomy is done. Fig. 29:  Carpopedal spasm
Q.3. Discuss difference in physiological, colloidal and
nodular goiter. (Sep 1999, 3 Marks)
Ans.

Physiological Goiter
♦♦ It is called sporadic goiter
♦♦ Goiter is soft and diffuse
♦♦ Puberty, pregnancy goiter.

Colloidal Goiter
♦♦ It is the late stage of diffuse hyperplasia. TSH level have
gone down and many follicles are inactive and full of
colloid
♦♦ May be due to iodine deficiency.

Nodular Goiter Fig. 30:  Chvostek’s sign

♦♦ The formation of nodules takes place due to fluctuating


♦♦ Carpopedal spasm or Trousseau’s sign:
TSH stimulation and its level in circulation
• It occurs in extreme cases and latent tetany.
♦♦ The nodule may be solid or cellular
• Arm is flexed at elbow, wrist, and meta­carpophalangeal
♦♦ It may occur due to adenoma or carcinoma.
joints but the inter-phalangeal joints are extended.
Q.4. Write short note on tetany. (Feb 2013, 5 Marks) • Thumb means towards the palm.
Ans. Tetany is a condition where is hyperexcitability of ♦♦ Chvostek’s sign:
peripheral nerves. • It also occurs in latent tetany.
• It indicates facial hyperexcitability.
Etiology • If a tap is given to facial nerve infront of ear, *twitching
♦♦ It occur due the decrease in calcium level in blood. of eyelids, corners of mouth takes place.
♦♦ After thyroidectomy there is decreased level of para- ♦♦ Laryngeal spasm:
thormone in the blood which leads to hypocalcemia. It is • Increased excitability of the laryngeal muscles
temporary and lasts for 4 to 6 weeks. produces laryngeal spasm.
♦♦ Other causes are neck dissection, hemochromatosis, Wil- • This leads to blockage of respiratory passage and
son's disease, DiGeorge syndrome. death may occur.
♦♦ Severe vomiting, hyperventilation associated with respira- ♦♦ Convulsions can occur in infants.
tory alkalosis.
Diagnosis
♦♦ Metabolic alkalosis.
♦♦ Rickets, osteomalacia. It is confirmed by estimating serum calcium level which is less
♦♦ Chronic renal failure. than 7 mg%.
♦♦ Acute pancreatitis
Management
Clinical Features ♦♦ Serum calcium estimation is done. It will be less than
♦♦ The first symptoms of tetany are tingling and numbness 7mg%.
in the face, fingers, and toes. ♦♦ IV calcium gluconate 10% 10 mL 6 to 8 hourly is given.
♦♦ Cramps are present in hand and feet. ♦♦ Later oral calcium (1 g TDS) with vitamin D supplementa-
♦♦ Stridor is the dangerous complication of severe tetany due tion (1–3 µg daily).
to spasm of muscles of respiration. ♦♦ Follow-up at regular intervals by doing serum calcium
♦♦ Spasm of intra-ocular muscles lead to blurring of vision. level
412   Mastering the BDS IIIrd Year  (Last 25 Years Solved Questions)

♦♦ Magnesium sulphate supportive therapy is also often


needed—l0 mL 10% magnesium sulphate intravenously.
Q.5. Describe briefly parathyroid tumor.
 (Sep 2002, 5 Marks)
Ans. Parathyroid adenoma and carcinoma are the neoplasm
found in parathyroid gland.

Parathyroid Adenoma
♦♦ The most common tumor of the parathyroid gland is an
adenoma.
♦♦ It may occur at any age and in either sex but is found more
frequently in adult life.
♦♦ Most adenomas are first brought to attention because
of excessive secretion of parathyroid hormones causing
features of hyperthyroidism.
♦♦ Parathyroid adenoma is small, encapsulated, yellowish
brown, ovoid nodular and weighing up to 5 g or more.

Parathyroid Carcinoma
♦♦ Carcinoma of parathyroid is rare and produces manifesta-
tion of hyperthyroidism
♦♦ Carcinoma tends to be irregular in shape and is adherent
to the adjacent tissue.

Treatment
Fig. 31  Thyrotoxicosis
♦♦ Single adenoma: Excision of the gland.
♦♦ Diffuse hyperplasia: 3½ or 3¾ parathyroid are removed.
♦♦ Presence of tremors, i.e. involuntary movement of body
♦♦ Carcinoma: All four glands should be removed with thy-
parts is present.
roid tissue.
♦♦ Cardiac arrhythmias, i.e. atrial fibrillation and atrial tachy-
Q.6. Describe clinical symptoms, signs and treatment of cardia develops.
primary thyrotoxicosis. ♦♦ Diaphoresis is present, i.e. excessive sweating is present.
 (Oct 2007, 15 Marks) (Sep 2006, 15 Marks) ♦♦ There is presence of powerful wide pulse pressure and
Ans. good bounding pulse is present.
♦♦ Exaggerated deep tendon reflexes are seen.
Symptoms ♦♦ There is protrusion of eyes, i.e. exophthalmus with star-
ing look.
♦♦ Hyperactivity, excitability irritability and dysphoria
♦♦ Pretibial myxedema: Thickening of skin due to mucin depo-
♦♦ Heat intolerance and swelling
sition over tibia.
♦♦ Palpitations
♦♦ Hyperreflexia, muscle wasting proximal myopothy with- Treatment
out fasciculations.
♦♦ Fatigue and weakness ♦♦ General:
♦♦ Weight loss with increased appetite • Allow the patient to take mental and physical rest.
♦♦ Diarrhea and polyuria. • Maintain nutrition of patient by giving nutritious diet.
♦♦ Oligomenorrhea and loss of libido. • If patient is anxious alprazolam 0.25 to 0.5 mg BD is
♦♦ Profuse sweating. given.
♦♦ Goitre or enlarged thyroid gland. • Most of the patients of thyrotoxicosis are fearful
because of increased sympathetic activity. In such type
Signs of patient, propranolol 40-80 mg BD is given.
♦♦ Drug therapy:
♦♦ Tachycardia is main sign which is present due to activation • Carbimazole is commonly used drug. It is started with
of thyrocardiac component 30 mg/day, adjustment of doses is made when patient
♦♦ Systolic hypertension is present come under control and maintenance dose is given,
♦♦ Palms are hot and moist. i.e. 10 to 20 mg/day.
Section 2:  General Surgery  413

• Potassium per chlorate 800 mg/day in divided doses ♦♦ Skin and sub-cutaneous tissue: Coarse dry skin, puffiness of
is given and the dose is reduced with improvement face with malar flush, baggy eyelids with swollen edema-
in patient’s condition. tous appearance of supraclavicular regions, neck and lacks
• Iodides are given, i.e. sodium or potassium iodide 6 of hand and feet.
to 10 mg/day. ♦♦ Cardiovascular and respiratory features: Bradycardia, an-
♦♦ Surgery: gina, cardiac failure, pericardial effusion and pleural
The commonly performed surgery is subtotal thyroidec- effusion.
tomy, prior surgery anti-thyroids are given to make patient ♦♦ Neuromuscular features: Aches and pains, cerebellar syn-
euthyroid. Potassium iodide 50 to 100 mg/day is given for drome with slurred speech and ataxia, muscle cramps
10 to 14 days. and stiffness.
♦♦ Radioiodine treatment: ♦♦ Gastrointestinal features: Constipation and ascites
I131 is used and average effective dose is 8 to 10 millicuries. ♦♦ Developmental: Growth and mental retardation
Q.7. Write short note on hypothyroidism.  ♦♦ Reproductive system: Infertility, menorrhagia, hyperprol-
 (Dec 2012, 5 Marks) (Apr 2007, 10 Marks) actinemia and galactorrhea.
Or
Management
Write short answer on management of hypothyroidism.
 (Apr 2018, 3 Marks) ♦♦ In patient of myxedema adequate ventilation is maintained
Ans. Inadequate release of thyroid hormone and its defective along with electrolyte balance and slow warming.
synthesis give rise to clinical synthesis of hypothyroidism. ♦♦ Principle of therapy is replacement of deficient thyroid
hormones.
Etiology ♦♦ Replacement with L – thyroxine 100 to 150µg/day. In
♦♦ After doing thyroidectomy old patients with ischemic heart disease initial therapy
♦♦ Agenesis or dysgenesis is with 25 to 50 µg/day and then gradually increased till
♦♦ Enzyme deficiency the required dose. Drug may take at least a week to act.
♦♦ Iodine deficiency It is better to give the drug in morning hours to obviate
♦♦ Hashimoto’s thyroiditis sleeplessness.
♦♦ Antithyroid drugs ♦♦ Initial rapid response is achieved by giving L iodothyro-
♦♦ Radioiodine nine 20µg TID.
♦♦ Drugs: Lithium, amiodarone Q.8. Etiology, clinical picture and treatment of Grave’s
Forms of Hypothyroidism disease. (Sep 2008, 8 Marks) (Mar 2008, 8 Marks)
Ans.
♦♦ Cretinism
♦♦ Myxedema. Etiology
Cretinism Grave’s disease is an autoimmune disease caused by production
of autoantibodies that stimulate thyroid stimulating hormone
Congenital absence of thyroid hormone leads to condition
receptor.
called as cretinism.
♦♦ A cretin has retarded physical and mental growth. Clinical Features
♦♦ Child is obese with pads of fat in supraclavicular region,
coarse features, limbs which are stumpy, thick lips and ♦♦ There is presence of diffused goiter which is with or
tongue, protuberant abdomen, small eyes, coarse hairs without bruit.
and dry skin. ♦♦ Fever, anxiety and restlessness are present.
♦♦ TSH will be raised; T3 and T4 will be low. ♦♦ There is weight loss, fatigue, sweating and heat intolerance.
♦♦ Cretinism will be treated by L thyroxine once in a day in ♦♦ Cardiovascular features:
morning orally. • Tachycardia is present which persists during sleep.
• Large pulse pressure is present with raised systolic
Myxedema blood pressure.
• Cardiac arrhythmias are present.
Myxedema is a clinical condition resulting from decreased
• Capillary pulsations may be seen.
circulating levels of T 3 and T 4 . It is characterized by
♦♦ Ocular manifestations:
deposition of mucinous material causing swelling of skin and
• Primary manifestations: Proptosis, exophthalmos and
subcutaneous tissue.
ophthalmoplegia.
• Secondary manifestations: Optic nerve compression,
Clinical Features
impaired convergence and exposure keratitis.
♦♦ General: There is tiredness, somnolence (Prolong drowsiness ♦♦ Gastrointestinal: Weight loss, diarrhea and vomiting.
or sleepiness), weight gain, cold intolerance and goiter. ♦♦ Reproductive system: Oligomenorrhea and infertility.
414   Mastering the BDS IIIrd Year  (Last 25 Years Solved Questions)

Management Treatment
♦♦ General ♦♦ L-thyroxine 0.2 mg/day is given as a supplementary dose.
• Allow the patient to take mental and physical rest. ♦♦ lf there is compression on the trachea, isthmusectomy is
• Maintain nutrition of patient by giving nutritious diet. done to relieve compression.
• If patient is anxious alprazolam 0.25 to 0.5 mg BD is given. ♦♦ lf the goitre is big and causing discomfort, subtotal thy-
• Most of the patients of thyrotoxicosis are fearful roidectomy can also be done.
because of increased sympathetic activity. In such type ♦♦ Steroid therapy often is helpful.
of patient propranolol 40–80 mg BD is given.
♦♦ Drug therapy: Complications of Hashimoto’s Thyroiditis
• Carbimazole is commonly used drug. It is started with ♦♦ Permanent hypothyroidism
30 mg/day, adjustment of doses is made when patient ♦♦ Papillary carcinoma of the thyroid
come under control and maintenance dose is given, ♦♦ Malignant lymphoma.
i.e. 10–20 mg/day.
Q.10. Discuss briefly myxedema. (May/Jun 2009, 5 Marks)
• Potassium per chlorate 800 mg/day in divided doses
is given and the dose is reduced with improvement Ans. Myxedema is a clinical condition resulting from decreased
in patient’s condition. circulating levels of T3 and T4. It is characterized by
• Iodides are given, i.e. sodium or potassium iodide deposition of mucinous material causing swelling of
6 to 10 mg/day. skin and subcutaneous tissue.
♦♦ Surgery: The commonly performed surgery is subtotal
thyroidectomy, prior surgery anti-thyroids are given to Clinical Features
make patient euthyroid. Potassium iodide 50–100 mg/day ♦♦ General: There is tiredness, somnolence, weight gain, cold
is given for 10–14 days. intolerance and goiter.
♦♦ Radioiodine treatment ♦♦ Skin and subcutaneous tissue: Coarse dry skin, puffiness of
I131 is used and average effective dose is 8–10 millicuries. face with malar flush, baggy eyelids with swollen edema-
Q.9. Write short note on hashimoto disease of thyroid. tous appearance of supraclavicular regions, neck and lacks
(Dec 2007, 3 Marks) of hand and feet.
♦♦ Cardiovascular and respiratory features: Bradycardia, angina,
Ans. It is an autoimmune disease
cardiac failure, pericardial effusion and pleural effusion.
It is also called as Hashimoto’s thyroiditis or diffuse ♦♦ Neuromuscular features: Aches and pains, cerebellar syn-
non-goitrous thyroiditis or struma lymphomatosa. drome with slurred speech and ataxia, muscle camps and
stiffness.
Clinical Features ♦♦ Gastrointestinal features: Constipation and ascites
♦♦ It is very common in women. Most common in perimeno- ♦♦ Developmental: Growth and mental retardation
pausal females. ♦♦ Reproductive system: Infertility, menorrhagia, hyperprol-
♦♦ There is painful, diffuse enlargement of both the lobes of actinemia and galactorrhea.
thyroid gland which is firm, rubbery, tender and smooth.
♦♦ Initially both lobes of thyroid are present with toxic features Investigation
but later they manifest with the features of hypothyroidism.
♦♦ Hepatosplenomegaly can be present. ♦♦ Thyroid function test: There is reduction in T3 and T4 levels
♦♦ The condition can predispose to papillary carcinoma of and rise in serum thyroid stimulating hormone which
thyroid. indicates primary hypothyroidism. Reduction in T3 and
T4 levels with TSH level below normal range is secondary
Histology hypothyroidism.
♦♦ Serum cholesterol: It is raised in primary thyroid failure.
♦♦ Histology is characterized by extensive lymphocytic in-
♦♦ The fall in serum level is more than 50 mg/100 mL.
filtration resulting in destruction of thyroid follicles with
♦♦ Tendon reflex duration is prolonged.
variable degree of fibrosis.
♦♦ In ECG bradycardia, low voltage complexes and flattened
♦♦ The thyroid follicles are destroyed by significant fibrosis.
or inverted T waves are present.
♦♦ The deep eosinophilic staining thyroid follicular cell
Askanazy cell, is characteristic.
Management
Investigations ♦♦ In patient of myxedema adequate ventilation is maintained
♦♦ FNAC can be done along with electrolyte balance and slow warming.
♦♦ Assessment of T3, T4 and TSH levels ♦♦ Principle of therapy is replacement of deficient thyroid
♦♦ Thyroid antibodies assay: Significant rise is observed in 85% hormones.
of cases. ♦♦ Replacement with L – thyroxine 100 to 150µg/day. In old
♦♦ ESR is very high, i.e. over 90 mm/hour. patients with ischemic heart disease initial therapy is with
Section 2:  General Surgery  415

25 to 50 µg/day and then gradually increased till the ♦♦ Neoplastic


required dose. Drug may take at least a week to act. It • Benign Adenomas: follicular, Hurthle cell.
is better to give the drug in morning hours to obviate • Malignant:
sleeplessness. –– Carcinomas: Papillary, follicular, medullary,
♦♦ Initial rapid response is achieved by giving L iodothyro- anaplastic.
nine 20µg TID. –– Lymphomas.
♦♦ Thyroiditis
Q.11. Write classification of thyroid swelling. Discuss treat­
• Hashimoto’s autoimmune thyroiditis.
ment of simple goiter. (Sep 2009, 10 Marks)
• De-Quervain’s autoimmune thyroiditis.
Ans. Classification of Thyroid Swelling • Riedel’s thyroiditis.
♦♦ Simple nontoxic ♦♦ Rare causes: Bacterial (suppurative), amyloid.
• Diffuse hyperplastic:
–– Physiological Treatment of Simple Goiter
- Puberty
- Pregnancy. ♦♦ When entire gland is diseased total thyroidectomy is done.
–– Primary iodine deficiency ♦♦ Subtotal thyroidectomy is done depending on the amount
–– Secondary iodine deficiency: of gland involved, amount of normal gland existing and
- Goitrogens of Brassica family, e.g. cabbage, location of nodules. It is a commonly done procedure
soyabean in multinodular goitre. Eight grams of thyroid tissue is
- Excess dietary fluoride. retained in each lateral lobe.
- Drugs: PAS, lithium, phenylbutazone, thiocy- ♦♦ Often partial thyroidectomy or Hartley Dunhill operation
anates, potassium perchlorate, anti-thyroid (isthumus + one entire lateral lobe and opposite side
drugs, radioactive iodine. subtotal or partial) is also done depending on the amount
- Dyshormonogenetic goitre. of diseased gland and normal tissues behind. Partial
• Colloid goitre. thyroidectomy is not well approved now.
• Nodular goitre (Multinodular). ♦♦ Postoperative L thyroxine is often given to prevent any
• Solitary nontoxic nodule. fluctuation in TSH level which may cause recurrent nodule
• Recurrent nontoxic nodule. formation.
♦♦ Toxic Q.12. Write differential diagnosis between follicular carci­
• Diffuse (Primary)—Grave’s Disease. noma and medullary carcinoma of thyroid regarding
• Multinodular (Secondary)—Plummer’s disease. etiopathology, clinical picture, metastasis and manage­
• Toxic nodule (solitary) (Tertiary). ment. (Jan 2011, 8 Marks)
• Recurrent toxicosis. Ans. See the table below

Features of differential Follicular carcinoma Medullary carcinoma


diagnosis
Etiopathology It arises in a multinodular goiter mainly in cases of endemic It is sporadic or familial. It arises from parafollicular C
goiter. Tumor cells line the blood vessels and get dislodged cells which are derived from ultimobranchial body. It
into systemic circulation producing secondaries in bones contains amyloid stroma in which malignant cells are
dispersed. Spreads mainly to lymph nodes
Clinical picture •  Occurs during 30 to 50 years of age. •  Occurs in middle age.
• Presence of swelling in neck which is firm or hard and • Thyroid swelling with enlargement of neck lymph
nodular. nodes.
• If lung secondaries are present dyspnea, chest pain and •  Presence of diarrhea and flushing
hemoptysis are present. •  Associated with MEN syndrome.
•  Pulsatile secondaries in skull and bones. • Paraneoplastic syndrome like Cushing’s and
carcinoids
Metastasis It spreads mainly through blood in bone, lungs and liver It spreads both by lymphatics and blood
Management • Total thyroidectomy is done along with central lymph node Total thyroidectomy is done along with central lymph node
compartment dissection (Level VI) dissection (Level VI) even if there are no nodes in neck.
• Functional neck node dissection is done with preservation Neck lymph nodes block dissection if lymph nodes
of sternocleidomastoid, spinal accessory nerve and are involved.
internal juglar vein if nodes are present. External beam radiotherapy for residual tumor tissue
416   Mastering the BDS IIIrd Year  (Last 25 Years Solved Questions)

Q.13. Write short note on hyperthyroidism. Classification of Goiter


(Aug 2011, 5 Marks) ♦♦ Simple goitre:
Ans. Hyperthyroidism is the condition resulting from the effect • Puberty goitre
of excessive amounts of thyroid hormones on body. In • Colloid goitre
hyperthyroidism pathology is in thyroid gland itself. • Iodine-deficiency goitre
• Multinodular goitre.
Etiology
♦♦ Toxic goitre:
♦♦ Exophthalmic goiter leads to hyperthyroidism. The condi- • Graves disease
tion is characterized by hyperplasia of thyroid gland and • Secondary thyrotoxicosis in multinodular goitre
eye involvement. • Solitary nodule
♦♦ Pituitary diseases which occurs in or involves anterior lobe • Other causes.
of pituitary gland. ♦♦ Neoplastic goitre:
♦♦ Toxic adenoma • Benign adenoma (Follicular adenoma)
♦♦ Multi-nodular goiter • Malignant tumors; They are further classified into
A. Primary
♦♦ Ectopic thyroid tissue.
• Well-differentiated carcinoma
Clinical Features –– Papillary carcinoma
♦♦ It occurs at the age of 20 to 40 years. –– Follicular carcinoma.
♦♦ Female predilection is present. • Poorly differentiated carcinoma
♦♦ Thyroid become enlarged, nodular, smooth and –– Anaplastic carcinoma
asymmetrical. • Arising from parafollicular cells
♦♦ Gastrointestinal features: Weight loss and diarrhea –– Medullary carcinoma.
♦♦ Cardiovascular features: Palpitation, shortness of breadth at • Arising from lymphatic tissue
rest, angina, irregularity in heart rate. –– Malignant lymphoma.
♦♦ Neuromuscular features: Undue fatigue and muscle B. SECONDARY (Metastasis)
weakness, tremors • Malignant melanoma, renal cell carcinoma, breast
♦♦ Skeletal features: Increase in linear growth in children carcinoma produce secondaries in the thyroid, due
♦♦ Genitourinary system: Oligomenorrhea or amenorrhea, to blood spread.
occasional urinary frequency ♦♦ Thyroiditis
♦♦ Integumentary system: Hair loss, gynecomastia, pruritus, • Granulomatous thyroiditis
palmer erythema. • Autoimmune thyroiditis
• Riedel’s thyroiditis.
Treatment ♦♦ Other rare causes of goitre
• Acute bacterial thyroiditis
Anti-thyroid Drugs
• Thyroid cyst
Drugs such as carbimazole 5–10 mg 8 hourly is given for 12-16 • Thyroid abscess
months; Methimazole can also be given; Propyl thiouracil is • Amyloid goitres.
given 200 mg 8 hourly.
Differentiating Features between Primary and Secondary
Surgery Thyrotoxicosis
♦♦ Before doing thyroid surgeries patient should become Features Primary Secondary
euthyroid. thyrotoxicosis thyrotoxicosis
♦♦ Subtotal thyroidectomy: Both lobes with isthmus are re- Age Occur in15 to 25 years Occur in 25 to 40
moved and a tissue equivalent to pulp of finger is retained years
at lower pole of gland bilaterally. S y m p t o m s a n d It appear simultaneously Duration is long
signs
Radioiodine Therapy S k i n o v e r t h e It is warm Not warm
♦♦ It destroys the cells and causes the complete ablation of swelling
thyroid gland. It should be given after the age of 45 years. Consistency Soft and firm Firm and hard
♦♦ Dosage is 5–10 millicurie. Surface Smooth Nodular
Auscultation B r u i t i s c o m m o n l y Bruit is uncommon
Q.14. Define and classify goiters. Describe differentiating heared
features between primary and secondary thyrotoxicosis Eye signs Found commonly Found rarely
along with medical treatment to make patient Predominant Of central nervous Of Cardiovascular
euthyroid. (Jan 2012, 15 Marks) symptoms system system
Ans. Diffuse enlargement of thyroid gland is described as Pretibial Can be found Never found
goiter. myxedema
Section 2:  General Surgery  417

Medical Treatment to Make Patient Euthyroid ♦♦ Inferior thyroid artery is a branch of thyrocervical trunk
♦♦ Carbimazole 10 mg 6 to 8 hourly intervals daily, till the and enters the posterior aspect of the gland. It supplies
patient is euthyroid, after 8 to 12 weeks dosage may be the gland by dividing into 4 to 5 branches which enter the
reduced to 5 mg 8 hourly. Last dose is given in the evening gland at various levels (not truly lower pole).
before surgery. ♦♦ Thyroidea ima artery is a branch of either brachiocephalic
♦♦ Propyl thiouracil is given as 200 mg 8 hourly. trunk or direct branch of arch of aorta and enters the lower
♦♦ Lugol’s iodine, i.e. 5% iodine in 10% potassium iodide part of the isthmus in about 2 to 3% of the cases.
solution: 10 drops TDS for 2 weeks before operation to Venous Drainage
reduce vascularity.
♦♦ Thyroxine 0.1 mg daily to prevent TSH stimulation which ♦♦ Superior thyroid vein drains the upper pole and enters the
may increase size and vascularity of the gland. internal jugular vein. The vein follows the artery.
♦♦ Middle thyroid vein is single, short and wide and drains
Q.15. Enlist midline neck swellings. Describe surgical into internal jugular vein.
anatomy of thyroid gland with reference to embryology, ♦♦ Inferior thyroid veins form a plexus which drain into in-
blood supply relationship and nerves related to thyroid nominate vein. They do not accompany the artery.
gland. (Aug 2012, 10 Marks) ♦♦ Kocher’s vein is rarely found (vein in between middle and
Ans. inferior thyroid vein).
Enlisting of Midline Swellings of Neck Reference to Nerve Supply
The midline swellings of neck are: ♦♦ External laryngeal nerve: Vagus gives rise to superior
♦♦ Ludwig’s angina laryngeal nerve, which separates from vagus at skull
♦♦ Enlarged sub-mental lymph node base and divides into two branches. The large, internal
♦♦ Sub-lingual dermoid cyst laryngeal nerve is sensory to the larynx. The small external
♦♦ Thyroglossal cyst laryngeal nerve runs close to the superior thyroid vessels
♦♦ Sub-hyoid bursitis and supplies cricothyroid muscle (tensor of the vocal cord)
♦♦ Goiter of thyroid, isthmus and pyramidal lobe and is sensory to upper half of the larynx. This nerve is
♦♦ Enlarged lymph node and lipoma in substernal space of away from the vessels near the upper pole. Hence, in
burns thyroidectomy, the upper pedicle should be ligated as
♦♦ Retrosternal goiter close to the thyroid as possible.
♦♦ Thymic swelling ♦♦ Recurrent laryngeal nerve: It is a branch of vagus, hooks
♦♦ Bony swelling arising from the manubrium sterni. around ligamentum arteriosum on the left and subclavian
artery on the right, runs in tracheo, esophageal groove
Surgical Anatomy of Thyroid Gland
near the posteromedial surface. Close to the gland, the
Reference to Embryology nerve lies in between (anterior or posterior) the branches
of inferior thyroid artery. Hence, inferior thyroid
♦♦ It develops from median down growth (midline
artery should be ligated away from the gland, to avoid
diverticulum) of a column of cells from the pharyngeal
damage to recurrent laryngeal nerve. On right side it is
floor between first and second pharyngeal pouches.
1 cm within the trache-oesophageal groove. The nerve
♦♦ By 6 weeks of time the central column, which becomes
traverses through the gland in about 5–8% of cases. The
thyroglossal duct, gets reabsorbed.
nerve may be very closely adherent to the posteromedial
♦♦ The duct bifurcates to form thyroid lobes.
aspect of the gland. Nerve not seen may be far away in
♦♦ Pyramidal lobe is formed by a portion of the duct.
the tracheoesophageal groove.
Reference to Blood Supply ♦♦ Non-recurrent laryngeal nerve is found in about l in 1,000
cases. Nerve has a horizontal course. In 25% of the cases it
Artery Supply
is within the ligament of Berry.
♦♦ Superior thyroid artery is a branch of external carotid
artery, enters the upper pole of the gland, divides into Q.16. Write the differential diagnosis of malignant tumors
anterior and posterior branches and anastomoses with of thyroid with special reference to their management.
ascending branch of inferior thyroid artery. Upper pole is  (Feb 2014, 8 Marks)
narrow, hence ligation is easy. Ans.
418   Mastering the BDS IIIrd Year  (Last 25 Years Solved Questions)

Features Papillary Follicular Anaplastic Medullary


Etiology Irradiation Endemic Unknown Sporadic or familial
Age 20-40 30–50 50 and above Middle age
Diagnosis Thyroid swelling with lymph node Thyroid swelling, Thyroid swelling, local fixity, Difficult to diagnose clinically
metastasis-bone stridor
Histology Orphan Annie-eyed nuclei, Angioinvasion, capsular Poorly differentiated cells Amyloid stroma like carcinoid
psammoma bodies invasion
Spread Lymphatic Blood Local infiltration Lymphatic, blood
Investigation FNAC Frozen section FNAC, biopsy FNAC, calcitotin
Management • Total or near thyroidectomy • Total thyroidectomy • External radiotherapy is • Total thyroidectomy is
with central node comparment is done along done done along with central
dissection (Level IV) with central lymph • Tracheostomy and isthem- lymph node dissection
• S u p p r e s s i v e d o s e o f node compartment ectomy relieve respiratory (Level VI) even if there
L-thyroxine 0.3 mg OD life long dissection (Level VI) obstruction temprorily are no nodes in neck
• Modified radical neck dissec- • Functional neck node • A driamycin as chemo- • Neck lymph nodes block
tion type III is required if lymph dissection is done with therapy dissection if lymph nodes
nodes are involved preservation of sterno- are involved
• If small lymph nodes are pres- cleidomastoid, spinal • External beam radio-
ent ‘Berry pickling may be accessory nerve and therapy for residual tumor
done internal juglar vein if tissue
nodes are present
TSH dependence Yes Yes No No
Hormone production Very rare Very rare No Calcitonin, 5HT, ACTH
Prognosis Excellent Good Worst Bad

Q.17. Write on classification of thyroiditis with clinical pic­ • Chronic autoimmune thyroiditis Or Hashimoto’s
ture, etiology and management. (Nov 2014, 10 Marks) disease
Ans. Classification • Post partum and silent thyroiditis
Revised American Thyroid Association Classification • Riedel’s thyroiditis.
• Acute thyroiditis Classification of Thyroiditis in Relation to Inflammation
• Sub-acute thyroiditis (de Quervain’s) Response and Clinical Course

Predominant Name of thyroiditis Synonyms Subcategories Clinical course


inflammatory cell
Neutrophil Acute thyroiditis • Acute suppurative thyroiditis • Bacterial thyroiditis Acute
• Infectious thyroiditis • Mycobacterial thyroiditis
• Fungal thyroiditis
• Parasitic thyroiditis
Macrophage/ Subacute granulomatous • De Quervain thyroiditis Sub-acute
histiocyte • Subacute thyroiditis

Thyroiditis • Painful subacute thyroiditis


• Post viral thyroiditis
• Giant cell thyroiditis
• Sub-acute nonsuppurative
thyroiditis
• Pseudotuberculous thyroiditis
• Struma granulomatosa
Infectious granulomatous • Infectious thyroiditis • Tuberculosis Sub-acute to chronic
thyroiditis • Fungal thyroiditis

• Sarcoidosis Sub-acute to chronic to


• Granulomatous vasculitis sub-clinical

Other granulomatous Multifocal granulomatous • Reaction to hemorrhage Sub-clinical


reactions folliculitis • Reaction to surgery
• Foreign body reaction
Palpation thyroiditis

Contd...
Section 2:  General Surgery  419

Contd...
Predominant Name of thyroiditis Synonyms Subcategories Clinical course
inflammatory cell
Lymphocyte Chronic lymphocytic thy- • Hashimoto's thyroiditis • Classic Chronic
roiditis • Autoimmune thyroiditis • Fibrous variant
• Struma lymphomatosa • Atrophic or fibrous atrophy
Silent thyroiditis • Sporadic thyroiditis variant Sub-acute
• Painless thyroiditis • Juvenile variant
• Painless sporadic thyroiditis • Hashitoxicosis variant
• Painless thyroiditis with hyper-
thyroidism
• Silent thyrotoxic thyroiditis
• Sub-acute lymphocytic thy-
roiditis
• Atypical subacute thyroiditis
• Spontaneously resolving thy-
roiditis
• Lymphocytic thyroiditis with
spontaneously resolving hy-
perthyroidism

Postpartum thyroiditis Painless postpartum thyroiditis Sub-clinical


• Non-specific thyroiditis
• Focal autoimmune thyroiditis
• Focal lymphocytic • Riedel thyroiditis
thyroiditis • Fibrosing thyroiditis Chronic
• Invasive fibrous thyroiditis • Sclerosing thyroiditis

Etiology, Clinical Picture and Management of Thyroiditis

Type of thyroiditis Etiology Clinical picture Management


Investigation Treatment
Granulomatous Or Virus, i.e. • Occur in young individuals • T3 and T4 levels are high Conservative treatment is
Sub-acute Or de either mumps • Patient complaints of fever, bodyache • FNAC is useful given in form of analge-
Quervain’s Thyroiditis or coxsackie and painful enlargement of thyroid sics and short course of
virus gland prednisolone
• Gland is enlarged, tender, soft to firm
with few symptoms of hyperthyroidism
occur initially
Autoimmune Autoimmune • It commonly occur in women at • ESR is very high, i.e. • Thyroxine 0.2 mg/day
or Hashimoto’s perimenopause. about 90 mm/hour. is given
thyroiditis or struma • Initially symptoms of mild • Thyroid antibody assay • If there is presence
lymphomatosa hyperthyroidism, i.e. hashitoxicosis is done of compression
can be present • If goiter is big and lead over trachea,
• Later on there is presence of to discomfort, subtotal isthmusectomy is done
permanent hypothyroidism thyroidectomy can be
• Thyroid gland is firm to hard and at done
times rubbery in consistency, smooth
or irregular and can involve a lobe or
entire gland
Riedel’s thyroiditis Collagen • It is common in males • FNAC is mandatory to • Treatment with
disorder • There is presence of swelling with rule out carcinoma thyroxine is given to
irregular surface • T3 and T4 can be low treat hypothyroidism
• Involvement of trachea, esophagus, • Radioisotope will not • Isthmectomy is done
internal jugular vein, etc, causes show any uptake to relieve compression
dysphagia and dyspnea on airway
• Consistency is stony hard
• There is presence of stridor
• Berry’s sign is positive
• Goiter is small
420   Mastering the BDS IIIrd Year  (Last 25 Years Solved Questions)

Q.18. Describe classification, clinical features and manage­ ♦♦ Malignant (Dunhill classification)
ment of thyroid neoplasia. (Feb 2015, 10 Marks) • Differentiated:
Or –– Papillary carcinoma
Write about clinical features of papillary and follicular –– Follicular carcinoma
carcinoma of thyroid and management –– Papillo-follicular carcinoma behaves like papillary
 (Jan2018, 15 Marks) carcinoma of thyroid
Ans. –– Hurthle cell carcinoma behaves like follicular
carcinoma.
Classification of Thyroid Neoplasia • Undifferentiated:
♦♦ Benign –– Anaplastic carcinoma
• Follicular adenoma—Colloid, embryonal, fetal • Medullary carcinoma
• Hurthle cell adenoma • Malignant lymphoma
• Colloid adenoma • Secondaries in thyroid from colon, kidney, melanoma
• Papillary adenoma. and breast.

Clinical Features and Management of Thyroid Neoplasias


Name of Clinical features Management
thyroid
neoplasia Investigation Treatment
Papillary • It occurs commonly in young females • FNAC of thyroid nodule • Total or near total thyroidectomy, with
carcinoma • It presents as soft or hard or firm, solid or demonstrate colloid filled central node compartment dissection
cystic, solitary or multinodular thyroid swelling follicles with papillary pro- (Level IV)
• Often discrete lymph nodes are palpable in cess • Suppressive dose of L – thyroxine is 0.3
lower deep cervical region and thyroid may or • Radioisotope shows cold mg OD life long
may not be palpable. If thyroid is not palpable it nodule • Modified radical neck dissection type III
is known as occult • TSH level in blood is high is required if lymph nodes are involved
• Some of the patients who reported late have • Plain X ray of neck shows • Radioactive iodine therapy (I131) is in-
fixed nodes in neck and fixed thyroid to tra- fine calcification dicated if tumor is more than 4 cm or
chea with or without recurrent laryngeal nerve • CT scan in neck to identify if there is extra thyroid involvement or
paralysis non-palpable nodes in neck lymph node involvement or multicentric
Follicular • It occurs commonly in females and peak age • Frozen section biopsy is • Total thyroidectomy, with central node
carcinoma group is around 40 years done compartment dissection (Level IV)
• There is presence of swelling in the neck • Thyroid scan demonstrate • Functional neck node dissection is done
which is firm or hard and nodular cold nodule with preservation of sternocleidomas-
• There is presence of tracheal compression • Alkaline phosphatase level toid, spinal accessory nerve and internal
and stridor is checked if high, scan- jugular vein if nodes are present clini-
• If lung secondaries are present there is pres- ning of bone is done cally. Or by imaging
ence of dyspnea, hemoptysis and chest pain • Plain X-ray of bone shows • L thyroxine 0.1 mg OD is given life long
• Involvement of recurrent laryngeal nerve osteolytic lesion • If secondaries are detected therapeutic
leads to hoarsness of voice dose radiation I131 is given. L – thyrox-
• Pulsatile secondaries in skull and long bones ine should be stopped 7 days before ra-
diation therapy
• Secondaries in bone are treated by ex-
ternal radiotherapy
• Internal fixation should be done when-
ever there is presence of pathological
fracture
Anaplastic • It is common in elderly women which are of 60 • FNAC is diagnostic • Tracheostomy and isthmectomy provide
carcinoma to 70 years of age temporary relieve in obstruction
• Patients are present with the rapidly growing • External radiotherapy is done
thyroid swelling of short duration • Adriamycin is used in chemotherapy
• Surface of swelling is irregular and consist-
ency is hard
• There is presence of stridor and hoarsness of
voice due to tracheal obstruction
• Dysphagia is also present
• Tumor is fixed to the skin
• Positive Berry’s sign, i.e. involvement of carot-
id sheath leads to absence of carotid pulsation
• Isthumus is also involved along with lateral
lobes
Contd...
Section 2:  General Surgery  421

Contd...

Name of thyroid Clinical features Management


neoplasia
Investigation Treatment
Medullary carcinoma • There is presence of thyroid swelling • FNAC shows amyloid • Total thyroidectomy with central node
with neck lymphadenopathy deposition with dispersed dissection in all patients along with main-
• Presence of diarrhea and flushing malignant cells and C cell tainence dose of L-thyroxine
• There is presence of hypertension, hyperplasia • Neck lymph node block dissection when
pheochromocytoma and mucosal neu- • Level of calcitonin will be lymph nodes are involved
romas when associated with MEN II high. Unstimulated calcium • External beam radiotherapy is given for
syndrome serum more than 100 pg/ residual tumor disease
• Sporadic and familial type of cases are mL is suggestive of medul- • Adriamycin is used in chemotherapy
seen in adults while cases with MEN lary carcinoma of thyroid • All the members of patient’s family are
II syndrome occur in young age group • CT of neck and chest is evaluated for serum calcitonin and if the
done to identify nodes levels are high, they should undergo pro-
• 111 indium octreotide phylactic total thyroidectomy
scanning is useful for de-
tection of medullary carci-
noma

Q.19. Etiopathology, clinical picture and management of Management


a case of secondary thyrotoxicosis. Write indication, Antithyroid Drugs
contraindication and complication of methods applied
for the treatment. (Apr 2015, 8 Marks) Carbimazole and propylthiouracil should be given. Treatment
Ans. Secondary hyperthyroidism is the result of abnormal, should be started 48 hours later and should be continued till
excessive thyroid stimulating hormone (TSH) release and radioiodine has had its effect till 6 weeks. Carbimazole should
stimulation of the thyroid resulting in excessive T4 release. be given in dose of 40 to 60 mg, treatment should be continued
for 12 to 18 months.
Etiopathology of Secondary Thyrotoxicosis
Indication
♦♦ Autoimmune: Here the thyroid IgG antibodies stimulate
thyroid to produce more hormone. This mechanism causes ♦♦ In all patients preoperatively.
diffuse enlargement of thyroid along with the hyperfunc- ♦♦ In patients not willing for surgery
♦♦ In recurrence after surgery.
tion. The antibodies which are directed specifically against
TSH receptors are known as thyroid receptor antibodies. Contraindication
There is presence of circulating antibodies and lymphocytic
♦♦ In cases with hypersensitivity
infiltration of thyroid tissue.
♦♦ Anemia or leukemia
♦♦ Genetics: Association of HLA B8 DR3 and DR4 indicates
♦♦ Kidney or liver disease.
the genetic susceptibility to environmental factors such as
viruses and bacteria which may produce antibodies which Complications
cross react with TSH receptors and causes thyrotoxicosis
♦♦ Agranulocytosis
Clinical Picture of Secondary Thyrotoxicosis ♦♦ Arthralgia
♦♦ It occurs during 25 to 40 years of age. ♦♦ Skin rashes
♦♦ Swelling is present for the long time while symptoms ♦♦ Fever.
remain for shorter duration.
Radioactive iodine
♦♦ Skin over the swelling is not warm.
♦♦ Consistency of the swelling is firm to hard and surface is
nodular. I131 should be given 150 microcuries per gram orally.
♦♦ On auscultation bruit is commonly heard. Indication
♦♦ Cardiovascular symptoms are predominant, i.e. Tachy-
cardia—may be atrial fibrillation, presence of wide pulse ♦♦ In patients over the age of 40 years.
pressure, extrasystole and/or heart failure. ♦♦ Toxicity recurring after previous subtotal thyroidectomy.
♦♦ CNS and GIT manifestations are less predominant. ♦♦ In very nervous patients who have fear of surgery.
♦♦ Internodular tissues of gland are overactive. ♦♦ In patients with severe thyrotoxic heart disease.
422   Mastering the BDS IIIrd Year  (Last 25 Years Solved Questions)

Contraindications Q.21. Classify thyroid swellings. Describe pathogenesis,


clinical features and management of multinodular
♦♦ In patients under 45 years of age because of high incidence
goiter. (Dec 2015, 10 Marks)
of hypothyroidism.
♦♦ During pregnancy and lactation. Ans. For classification of thyroid swelling refer to Ans 11 of
same chapter.
Complication
Multinodular Goiter
♦♦ Hypothyroidism
♦♦ Worsening ophalmopathy Multinodular growth is a discordant growth with structurally
♦♦ Risk for cancer and birth defects in long standing cases. and functionally altered thyroid follicles which present multiple
nodules in thyroid.
Subtotal thyroidectomy
Pathogenesis
♦♦ If cardiac symptoms are controlled well and risk of anes-
thesia is acceptable, subtotal thyroidectomy is done. Following is the pathogenesis of multinodular goiter.
♦♦ Before surgery, it should be remembered that patient
should be euthyroid. Antithyroid drugs should be stopped
two weeks before surgery and is replaced by potassium
iodate 170 mg daily orally.
♦♦ In the surgery, both lobes with isthumus are removed and
a tissue equivalent to the pulp of finger is retained at lower
pole of gland on both sides.
Indications
♦♦ In recurrent thyrotoxicosis after 12 to 18 month course of
antithyroid drugs under the age of 40 years.
♦♦ In cases with sensitivity reactions to antithyroid compounds.
♦♦ In severe thyrotoxicosis not responding to medical treat-
ment.
♦♦ In tracheal compression.
Contraindications
In patients showing evidence of marked exophthalmos.

Complications
♦♦ Hemorrhage
♦♦ Hematoma formation
♦♦ Edema of glottis
♦♦ Injury to recurrent laryngeal nerve
♦♦ Tetany.
Q.20. Write short note on parathormone.(Feb 2015, 5 Marks)
Ans. Parathormone is secreted by chief cells whenever serum Clinical Features
calcium falls. ♦♦ Multinodular goiter is more common in females. Female
Parathormone stimulate osteoclast cells for bone to male ratio is 10:1
resorption, kidney for promoting calcium reabsorption ♦♦ It occurs during the age of 20 to 40 years.
and production of 1, 25 dihydroxy vitamin D and ♦♦ It is a slowly progressive disease.
gastrointestinal tract to promote absorption of calcium ♦♦ There is presence of multiple nodules of various sizes which
and phosphorus. are present in both lobes and in isthmus which are firm,
Half-life of parathormone is 4 minutes. nodular, non-tender and does not move with deglutition.
In nomal persons the parathormone is balanced by ♦♦ Swelling remain in front of neck, dyspnea is present due
calcitonin. to tracheomalacia.
♦♦ Dysphagia is also present.
Actions of Parathormone ♦♦ Hard areas suggest calcification and soft areas are sug-
♦♦ It increases absorption of calcium from gut. gestive of necrosis.
♦♦ It mobilizes calcium from the bone.
Management
♦♦ It increases calcium reabsorption from renal tubules and
promote excretion of phosphate. Management part consists of both investigations and treatment:
Section 2:  General Surgery  423

Investigations ♦♦ Postoperatively L-thyroxine is given to prevent any fluctua-


tion in TSH which can lead to recurrent nodule formation.
♦♦ T3, T4, TSH and ultrasound neck and FNAC. FNAC should
be done from the most dominant and suspected nodule. Prevention
Ultrasound-guided FNAC is more reliable. This method ♦♦ When patient develop goiter in puberty he/she should be
identifies impalpable nodules, their number, nature of supplemented with 0.1 to 0.2 mg of L – thyroxine.
nodule and vascularity of nodule. ♦♦ Patient should be given iodine rich diet.
♦♦ X-ray neck: It shows rim of calcification which shows dis- ♦♦ Goitrogenic diet, i.e. cabbage and goitrogenic drugs should
placement and compression of trachea. be avoided.
♦♦ Indirect laryngoscopy: It shows the mobility of vocal Q.22. Describe classification, clinical features, treatment and
cords. complications of thyroid malignancy.
♦♦ Radioisotope iodine scan: It should be done in selected pa-  (Jan 2016, 10 Marks)
tients only. Ans. Classification of thyroid malignancy Or Dunhill
♦♦ Complete blood picture, routine urine examination, fasting Classification
and post prandial blood sugar, serum calcium estimation • Differentiated (80%)
should be done. – Papillary carcinoma (60%)
– Follicular carcinoma (17%)
Treatment
– Papillo-follicular carcinoma
Treatment for multinodular goiter is surgical. – Hurthle cell carcinoma.
♦♦ When complete gland is affected total thyroidectomy • Undifferentiated—20%
should be done. – Anaplastic carcinoma (13%).
♦♦ Subtotal thyroidectomy is carried out depending on • Medullary carcinoma (6%)
amount of gland involved and amount of normal gland • Malignant lymphoma (4%)
left along with location of nodules. In this 8 g of thyroid • Secondaries in thyroid—from colon, kidney,
tissue is retained in each lateral lobe. melanoma, breast.

Clinical Features and Treatment of Thyroid Malignancy

Name of thyroid Clinical features Treatment


malignancy
Papillary carcinoma • It occurs commonly in young females • Total or near total thyroidectomy, with central node
• It presents as soft or hard or firm, solid or cystic, solitary or compartment dissection (Level IV)
multinodular thyroid swelling • Suppressive dose of L-thyroxine is 0.3 mg OD life
• Often discrete lymph nodes are palpable in lower deep long
cervical region and thyroid may or may not be palpable. If • Modified radical neck dissection type III is required
thyroid is not palpable it is known as occult if lymph nodes are involved
• Some of the patients who reported late have fixed nodes in • Radioactive iodine therapy (I131) is indicated if
neck and fixed thyroid to trachea with or without recurrent tumor is more than 4 cm or if there is extrathyroid
laryngeal nerve paralysis involvement or lymph node involvement or multi-
centric
Follicular carcinoma • It occurs commonly in females and peak age group is • Total thyroidectomy, with central node compart-
around 40 years ment dissection (Level IV)
• There is presence of swelling in the neck which is firm or • Functional neck node dissection is done with pres-
hard and nodular ervation of sternocleidomastoid, spinal accessory
• There is presence of tracheal compression and stridor nerve and internal jugular vein if nodes are present
• If lung secondaries are present there is presence of clinically. Or by imaging
dyspnea, hemoptysis and chest pain • L thyroxine 0.1 mg OD is given life long
• Involvement of recurrent laryngeal nerve leads to • If secondaries are detected therapeutic dose radi-
hoarsness of voice ation I131 is given. L-thyroxine should be stopped
• Pulsatile secondaries in skull and long bones 7 days before radiation therapy
• Secondaries in bone are treated by external radio-
therapy
• Internal fixation should be done whenever there is
presence of pathological fracture

Contd...
424   Mastering the BDS IIIrd Year  (Last 25 Years Solved Questions)

Contd...

Name of thyroid Clinical features Treatment


malignancy
Anaplastic carcinoma • It is common in elderly women which are of 60 to 70 years • Tracheostomy and isthmectomy provide tempo-
of age rary relieve in obstruction
• Patients are present with the rapidly growing thyroid • External radiotherapy is done
swelling of short duration • Adriamycin is used in chemotherapy
• Surface of swelling is irregular and consistency is hard
• There is presence of stridor and hoarsness of voice due to
tracheal obstruction
• Dysphagia is also present
• Tumor is fixed to the skin.
• Positive Berry’s sign, i.e. involvement of carotid sheath
leads to absence of carotid pulsation
• Isthumus is also involved along with lateral lobes
Medullary carcinoma • There is presence of thyroid swelling with neck • Total thyroidectomy with central node dissection
lymphadenopathy in all patients along with maintence dose of
• Presence of diarrhea and flushing L-thyroxine
• There is presence of hypertension, pheochromocytoma • Neck lymph node block dissection when lymph
and mucosal neuromas when associated with MEN II nodes are involved
syndrome • External beam radiotherapy is given for residual
• Sporadic and familial type of cases are seen in adults while tumor disease
cases with MEN II syndrome occur in yound age group • Adriamycin is used in chemotherapy
• All the members of patient’s family are evaluated
for serum calcitonin and if the levels are high, they
should undergo prophylactic total thyroidectomy

Complications of Thyroid Malignancy Injury to External Laryngeal Nerve


Complications of thyroid malignancy are associated with It leads to weakening of cricothyroid muscle which causes
the thyroidectomy procedure followed as surgical treatment alteration in pitch of voice, voice fatigue, breathy voice and
in most of thyroid malignancies. So following are the frequent throat clearing.
complications:
Hypothyroidism
Hemorrhage
It reveal clinically after 6 months.
♦♦ Chances of reactionary hemorrhage are present which is
more dangerous and occurs within 6 to 8 hours after sur- Stitch Granuloma
gery. It occurs due to slipping of ligature, coughing, etc.
It can occur with or without sinus formation and is seen after
♦♦ Tension hematoma develop deep-to-deep fascia it com-
the use of non-absorbable suture material.
presses the larynx.
Respiratory Obstruction Q.23. Describe differentiating features of primary and sec­
ondary thyrotoxicosis. (Jan 2017, 3 Marks)
It occur due to hematoma or laryngeal edema, due to
Ans. Following are the differences between primary and
tracheomalacia or bilateral recurrent laryngeal nerve palsy. secondary thyrotoxicosis:
Recurrent Laryngeal Nerve Palsy Features Primary Secondary
thyrotoxicosis thyrotoxicosis
It can be transient or permanent. Transient recover in 3 weeks Age Occur in 15 to 25 years Occur in 25 to 40 years
to 3 months. It presents with hoarseness of voice, aphonia, Symptoms and It appear simultane- Duration is long
aspiration and ineffective cough. signs ously
Skin over the It is warm Not warm
Hypoparathyroidism swelling
Consistency Soft and firm Firm and hard
It is temporary and due to vascular spasm of parathyroid glands. It Surface Smooth Nodular
occurs on 2nd and 5th post-operative day. It presents with weakness, Auscultation Bruit is commonly heard Bruit is uncommon
positive Chvostek’s sign, carpopedal spasm and convulsion. Eye signs Found commonly Found rarely
Predominant Of central nervous Of Cardiovascular
Thyrotoxic Crisis symptoms system system
It occurs in thyrotoxic patient which is inadequately prepared Pretibial Can be found Never found
for thyroidectomy. myxedema
Section 2:  General Surgery  425

Q.24. Write short note on complication of thyroid surgery. Clinical Features


 (Jan 2017, 5 Marks)
Ans. Complications of thyroid surgery can be divided into General Features
three parts i.e. minor, rare, or major. ♦♦ Presence of high-grade fever with chills and rigors.
♦♦ Minor complications: Postoperative surgical site seromas, ♦♦ Malaise and bodyache and headache
poor scar formation. ♦♦ Nausea and constipation.
♦♦ Rare complications: Damage to the sympathetic trunk.
♦♦ Major complications: Local Features
• Bleeding: Present with neck swelling, neck pain, and/ ♦♦ Severe unilateral throat pain
or signs and symptoms of airway obstruction (e.g. ♦♦ Odynophagia leading to dehydration
dyspnea, stridor, hypoxia). ♦♦ Hot potato speech
• Injury to recurrent laryngeal nerve: Present with post- ♦♦ Foul breath
operative hoarseness or breathlessness. ♦♦ Poor orodental hygiene
• Hypoparathyroidism: Signs of hypocalcemia, i.e. ♦♦ Ipsilateral earache
circumoral paresthesias, mental status changes, ♦♦ Trismus.
tetany, carpopedal spasm, laryngospasm, seizures,
QT prolongation on ECG, and cardiac arrest. Examination
• Thyrotoxic storm ♦♦ Tonsils, pillars, soft palate: Congestion the involved side.
• Injury to superior laryngeal nerve: Mild hoarseness or ♦♦ Uvula: Swollen and pushed to the opposite side.
decreased vocal stamina. ♦♦ Region of soft palate and anterior pillar above the tonsil
• Infection: Manifests as superficial cellulitis or as an appears bulged.
abscess. ♦♦ Mucopus covering the tonsil.
Q.25. Classify goiter. Discuss clinical features, investi­ ♦♦ Cervical lymph nodes enlarged, tender.
gations and management of multinodular goiter. ♦♦ Torticollis: Head tilted to the involved side.
 (Apr 2017, 15 Marks) Treatment
Ans. For classification of goiter refer to Ans 14 of same chapter.
Conservative treatment
For clinical features, investigations and management
of multinodular goiter refer to Ans 21 of same chapter. ♦♦ Hospitalization is done and IV fluids should be given.
♦♦ IV antibiotics for aerobic and anaerobic organisms should
be given
19. Tonsils ♦♦
♦♦
Analgesics are started
Oral hygiene: Hydrogen peroxide and warm saline mouth
gargles are started.
Q.1. Write short note on peritonsillar abscess. 
 (Sep 2000, 10 Marks) Surgical treatment
Ans. Peritonsillar abscess is also called as “Quinsy”. ♦♦ Incision and drainage:
Collection of pus in peritonsillar space, i.e. between • If no response to conservative management and frank
capsule of tonsil and superior constrictor muscle is pus is present, incision and drainage is done.
known as peritonsillar abscess or quinsy. • Incision site: Point of maximum bulge above the upper
Etiology pole of tonsil, just lateral to the meeting point of lines
drawn at the base of uvula and along the anterior
♦♦ As a sequelae of acute tonsillitis. pillar. Once the incision is made, sinus forceps is
♦♦ De novo. inserted into the incision to open up the abscess.
♦♦ Interval tonsillectomy:
Causative Organisms
• Interval tonsillectomy is done 4 to 6 weeks following
♦♦ Streptococcus pyogenes quinsy.
♦♦ Staphylococcus aureus • Abscess or hot tonsillectomy done in the presence of the
♦♦ Anaerobic organisms abscess has the following disadvantages:
–– Risk of rupture during anesthesia
Mechanism –– Increased bleeding.
One of the crypts or crypta magna is filled with pus and is sealed off Complications
Intratonsillar abscess formation ♦♦ Parapharyngeal abscess
♦♦ Laryngeal edema
Abscess bursts through capsule
♦♦ Septicemia
Presence of peritonsillitis ♦♦ Pneumonitis/lung abscess
♦♦ Endocarditis
Formation of peritonsillar abscess ♦♦ Nephritis
426   Mastering the BDS IIIrd Year  (Last 25 Years Solved Questions)

♦♦ Brain abscess ♦♦ Type IV proximal gastric ulcer is difficult to manage. It is


♦♦ Jugular vein thrombosis. treated by subtotal gastrectomy. Often distal gastrectomy
Q.2. Write short note on quinsy. (Sep 2005, 5 Marks) with selective sleeve like extension cut along the lesser
curve to remove the ulcer is done—Pauchet’s procedure.
Ans. Refer to Ans 1 of same chapter.
♦♦ Other surgical procedures:
Q.3. Write short answer on Singer’s nodule. • de Miguel’s antrectomy: Distal antrectomy, pylorec-
 (Apr 2018, 3 Marks) tomy with excision of ulcer along with gastroduodenal
Ans. Vocal nodules are fibrous thickenings of vocal folds at anastomosis is done. It preserves gastric reservoir
the junction of middle and anterior third and are the function, shows less recurrence rate and less opera-
result of vocal abuse. They are known as Singer’s nodule tive morbidity.
in adults and screamer’s nodule in children. • Maki’s pylorus preserving gastrectomy: Hemigas-
In these speech therapy is the preferred treatment. trectomy with excision of pyloric ulcer but retaining
Some of the lesions will resolve spontaneously in most 2 cm prepyloric stomach. It is only used in type l gas-
of the cases. tric ulcer. Even though it has got fewer incidences of
Occasionally nodules will need to be surgically removed postoperative diarrhea and dumping, it has got high
using modern microlaryngoscopic dissection or laser recurrence rate.
techniques, but speech therapy will be required for post ♦♦ HSV with excision of ulcer
operative voice rehabilitation. ♦♦ Kelling Madlener procedure: It is antrectomy and excision of
proximal gastric ulcer Type IV.
♦♦ Csendes procedure: It is subtotal gastrectomy with sleeve
20. STOMACH extended resection along the lesser curve for type IV
proximal gastric ulcer.
Q.1. How we differentiate between malignant and benign
ulcer and management. (Jan 2018, 10 Marks)
Ans. Benign ulcer and malignant ulcer are the types of gastric 21. Operative Surgery
ulcer.
Features Malignant ulcer Benign ulcer Q.1. Describe briefly tracheostomy. (Jan 2011, 5 Marks)
 (Sep 2010, 10 Marks) (Mar 1997, 10 Marks)
Mucosal folds Effacing mucosal folds Converging mucosal
folds till margin Or
Site Greater curvature 95% less curvature Write short note on tracheostomy.
Margin Irregular margin Regular margin  (Nov 2015, 5 Marks) (Mar 2006, 10 Marks)
 (Dec 2009, 5 Marks) (Apr 2008, 5 Marks)
Floor Necrotic slough on floor Granulation tissue on  (Oct 2007, 5 Marks) (Oct 2003, 10 Marks)
floor
 (Sep 2001, 10 Marks) (Sep 1999, 5 Marks)
Edge Everted edge Punching or sloping
edge Or
S u r r o u n d i n g S u r r o u n d i n g a r e a s Surrounding area and Write briefly on tracheostomy. (Jan 2016, 5 Marks)
area show nodules, ulcer rugae are normal Or
and irregularities
Write in briefly about tracheostomy.
Size and extent Large and deep Small, deep up to part
of muscle  (July 2016, 5 Marks)
Bleeding Present Absent Or
Luminal Endoluminal Exoluminal Discuss indication, operative procedure and complica­
tion of tracheostomy. (Sep 2005, 18 Marks)
Management of Benign Ulcer
Or
Drugs like H2 blockers, proton-pump inhibitors, carbenoxolone
Describe indication, procedure and complication of
(biogastrone, sucralfate, prostaglandins which coats the
tracheostomy. (Feb 2015, 10 Marks)
ulcer and so creates a mucosal barrier) helps in reducing or
Ans. Tracheostomy refers to a surgical entry into trachea
eliminating the symptoms of benign ulcer.
through anterior wall to secure airway for oxygenation.
Management of Malignant Ulcer
Classification / Types
♦♦ But asymptomatic ulcer may exist silently and may turn
into malignancy. So surgery is the preferred line of treat- ♦♦ Depending on the method of initial insertion:
ment. Partial gastrectomy and Billroth I gastroduodenal • Surgical
anastamosis is done. • Percutaneous (needle).
Section 2:  General Surgery  427

♦♦ Depending on the intended duration: ♦♦ Step VIII: Tube is placed and is secured to neck. Tube is
• Temporary tracheostomy inserted vertically downward into the trachea avoiding
• Permanent tracheostomy. damage to the tracheal mucosa of posterior wall. The tube
♦♦ Depending on situation demanding the procedure: is secured by suturing the flanges to the neck skin. This is
• Emergent tracheostomy followed by tying the flanges of tube with thread encircling
• Urgent tracheostomy the neck taking care to avoid strangulation.
• Elective tracheostomy.
Indications
♦♦ To secure and clear the airway in upper respiratory tract
obstruction (actual or potential).
♦♦ To secure and maintain a safe airway in patients with
injuries to the face, head or neck and following certain
types of surgery to the head and neck in unstable cervical
spine fracture.
♦♦ To facilitate the removal of bronchial secretions where
there is poor cough effort with sputum retention.
♦♦ To protect the airway of patients who are at high risk of
aspiration, that is patients with incompetent laryngeal
and tongue movement on swallowing, e.g. neuromuscular
disorders, unconsciousness, head injuries, stroke, etc.
♦♦ To enable long-term mechanical ventilation of patients, Fig. 32:  Tracheostomy tube
either in an acute ICU setting or sometimes chronically
in hospitals. Complications
♦♦ To facilitate weaning from artificial ventilation in acute
respiratory failure and prolonged ventilation. Complications are as follows:
♦♦ Immediate complications (Perioperative period)
Contraindications • Hemorrhage
♦♦ Children under 5 years of age. • Misplacement of tube
♦♦ Preexisting pathology of larynx, e.g. carcinoma • Pneumothorax
♦♦ Lack of experience and knowledge of cricothyroidotomy. • Tube occlusion.
♦♦ Cervical trauma. • Surgical emphysema.
• Loss of the upper airway.
Procedure
♦♦ Delayed complications (Post-operative period; less than
♦♦ Step I: Skin from the chin to below the clavicles is sterilely 7 days)
prepared. • Tube blockage with secretions or blood.
♦♦ Step II: Local anesthesia with vasoconstrictor is infiltrated • Partial or complete tube displacement.
in skin and deeper tissues. • Infection of the stoma site.
♦♦ Step III: Skin of the neck over second tracheal ring is identi- • Infection of the bronchial tree (pneumonia).
fied and an incision is placed horizontally along natural • Ulceration and/or necrosis of trachea.
cervical skin crease. • Mucosal ulceration by tube migration
♦♦ Step IV: Sharp dissection following the skin incision is done • Risk of occlusion of the tracheostomy tube in obese
to cut across platysma muscle. or fatigued patients who have difficulty extending
♦♦ Step V: Blunt dissection is given parallel to long axis of their neck.
trachea for separating submuscular tissues until isthmus • Tracheoesophageal fistula formation.
is identified. • Hemorrhage.
♦♦ Step VI: A cricoids hook engages the space between cricoids ♦♦ Late complications (Late postoperative period; more
and first tracheal ring pull trachea upward. Blunt dissec- than 7 days)
tion is continued longitudinally through pretracheal fascia. • Granulomata of the trachea may cause respiratory
♦♦ Step VII: Entrance in trachea difficulty when the tracheostomy tube is removed.
• A linear incision is made through inter-space between • Tracheal dilation, stenosis, persistent sinus or collapse
second and third tracheal rings. (tracheomalacia)
• Mid-portion of third or fourth tracheal ring is removed • Scar formation requiring revision.
for creating tracheal window. • Blocked tubes may occur at any time, especially if
• An inferiorly placed U-shaped flap also known as Bjork secretions become thick, the secretions are not managed
flap incorporates the ring below the tracheal incision is appropriately (suction) and humidification is not used.
raised and sutured to the skin at inferior margin. • Hemorrhage.
428   Mastering the BDS IIIrd Year  (Last 25 Years Solved Questions)

Q.2. Discuss emergency tracheostomy.  ♦♦ For laparoscopic surgical procedures.


 (Feb/Mar 2004, 10 Marks) ♦♦ To remove small cutaneous lesions.
Ans. Emergency tracheostomy is performed within 2 to 3
minutes in an emergency situation when patient is anoxic Disadvantages
and requires immediate oxygenation and verification to ♦♦ It leads to infection.
avoid cerebral hypoxia. ♦♦ For the cauterization of normal tissues.
♦♦ It has problem of explosion.
Procedure ♦♦ It can cause diathermy burn to the patient at the site where
diathermy plate is used.
♦♦ Step I and II: Preparation of skin and local anesthesia is not
♦♦ It can cause burn injury or electrical shock to surgeon and
performed in emergency tracheostomy.
assisting personnel.
♦♦ Step III: Skin of the neck over second tracheal ring is identi-
fied and an incision is placed horizontally along natural Q.4. Classify suture material. Add a small note on catgut.
cervical skin crease.  (Feb 2013, 5 Marks)
♦♦ Step IV: Sharp dissection following the skin incision is done Ans.
to cut across platysma muscle.
Classification of Suture Material
♦♦ Step V: Blunt dissection is given parallel to long axis of
trachea for separating submuscular tissues until isthmus Classification I
is identified. ♦♦ Absorbable suture material:
♦♦ Step VI: A cricoids hook engages the space between cricoids • Plain catgut
and first tracheal ring pull trachea upward. Blunt dissec- • Chromic catgut
tion is continued longitudinally through pretracheal fascia. • Vicryl
♦♦ Step VII: Entrance in trachea • Dexon
• A linear incision is made through inter-space between • Maxon
second and third tracheal rings. • Poly dioxanone suture material
• Mid-portion of third or fourth tracheal ring is removed • Monocryl
for creating tracheal window. • Biosyn.
• An inferiorly placed U-shaped flap also known as Bjork ♦♦ Non-absorbable suture material
flap incorporates the ring below the tracheal incision • Silk
is raised and sutured to the skin at inferior margin. • Polypropylene
♦♦ Step VIII: Tube is placed and is secured to neck. Tube is • Polyethylene
inserted vertically downward into the trachea avoiding • Cotton
damage to the tracheal mucosa of posterior wall. The tube • Linen
is secured by suturing the flanges to the neck skin. This is • Steel, polyester, polyamide, nylon.
followed by tying the flanges of tube with thread encircling
the neck taking care to avoid strangulation. Classification II
Q.3. Write short note on diathermy. ♦♦ Natural:
 (Jan 2011, 3 Marks) (Nov 2014, 5 Marks) • Catgut
Ans. It is also known as electrocautery • Silk
• Cotton
It is the method to control bleeding or to cut the tissues
• Linen.
during surgery
♦♦ Synthetic:
Types • Vicryl, dexon, PDS, maxon
• Polypropylene, polyethylene, polyester, polyamide.
♦♦ Based on type of current used:
• Unipolar cautery Classification III
• Bipolar cautery. ♦♦ Braided: Polyester, polyamide, vicryl, dexon, silk
♦♦ Based on type of action: ♦♦ Twisted: Cotton, linen.
• Coagulation cautery
Classification IV
• Cutting cautery
• Blended current: Combination both coagulation and ♦♦ Monofilament: Polypropylene, polyethylene, PDS, catgut,
cutting cautery. steel
♦♦ Multifilament: Polyester, polyamide, vicryl, dexon, silk,
Uses cotton.
♦♦ For coagulation of bleeding during surgery to achieve Classification V
hemostasis. ♦♦ Coated
♦♦ For cutting muscles, fascia, etc. ♦♦ Uncoated.
Section 2:  General Surgery  429

Catgut ♦♦ Resorption gets completed after 90 days.


♦♦ Chromic gut produces tissue reaction which is less as
Catgut was the first absorbable suture material available.
compared to plain gut.
♦♦ Original word was kitgut, i.e. Kit means fiddle and present
♦♦ It is used for suturing muscle. Fascia, ligating pedicles, etc.
form arise thorough confusion with kit. = cat.
♦♦ Another explanation of origin of cat in catgut is it is an Fast Absorbing Gut
abbreviation of cattle which originally denoted not only ♦♦ When plain surgical gut is treated by heat to allow rapid
the cows but all types of livestock. resorption it is known as fast absorbing gut.
♦♦ Catgut is derived from natural source which is purified ♦♦ This is designed for use on skin.
collagen tissue derived from serous layer of cow’s intestine ♦♦ Its tensile strength lost from 5 to 7 days and get resorbed
or submucous fibrous layer of sheep intestine. by 2 to 4 weeks.
♦♦ Catgut is pseudofilamentous in nature, i.e. microscopically
it is made up of multiple filaments which are processed Q.5. Enlist indications, types of tracheostomy. Add a note
in such a way that they are twisted in ground together on steps of tracheostomy and enlist its postoperative
and polished to produce the appearance of monofilament complications. (Feb 2013, 10 Marks)
suture material. Ans. For indications, types, steps of tracheostomy and post-
♦♦ As it is composed of collagen fibers, chances of degradation operative complications refer to Ans 1 of same chapter.
are present, i.e. the material should be kept moist. Q.6. Write short note on indications of tracheostomy.
♦♦ Commercial supply of this material is as the package  (June 2010, 5 Marks) (May/Jun 2009, 5 Marks)
soaked in isopropyl alcohol which act as preservative. Or
♦♦ Resorption of catgut is via enzymatic digestion through Write briefly on indications of tracheostomy.
proteolytic enzymes and phagocytosis. (Dec 2010, 5 Marks)
♦♦ When catgut is placed inside the tissues, it looses its tensile Or
strength under 10 to 15 days, this is resorbed under 2 to
Write short answer on indications of tracheostomy.
3 months.
 (June 2018, 3 Marks)
Types of Catgut Ans. Refer to Ans 1 of same chapter.
It is of three types, i.e. Q.7. Describe briefly suture materials. (Apr 2017, 5 Marks)
♦♦ Plane gut Ans. Suture materials are used to hold the several tissues in
♦♦ Chromic gut close approximation.
♦♦ Fast absorbing surgical gut
Properties of an Ideal Suturing Material
Plane Gut
♦♦ At the time of suturing, it should be dry or else it become ♦♦ It should produce less tissue reaction.
stiff and difficult to handle. ♦♦ There should be adequate strength of suture material to
♦♦ It breaks easily as it consists of weak areas along its length withstand stress.
due to manufacturing process. ♦♦ It should be easily sterilized.
♦♦ It is easy to handle within the tissues.
♦♦ It is easily degraded by enzymatic action and inflammatory
♦♦ It should have good knot tying properties.
reaction is present during this procedure.
♦♦ It should have less capillary action.
♦♦ Bacterial adhesion is more as compared to nylon and
polypropylene.
♦♦ Tensile strength of this material is poor. Tensile strength Classification of Suture Materials
lost in 7 to 10 days ♦♦ Based on the degradation of material within the tissues:
♦♦ This is used to suture subcutaneous tissue, muscle and • Absorbable
suturing circumcision in children. • Non-absorbable
Chromic Gut ♦♦ Based on the source of materials:
• Natural, e.g. silk
♦♦ It is the plain surgical gut which is tanned with the chro- • Synthetic, e.g. polyglycolic acid
mic salts. • Metallic, e.g. stainless steel
♦♦ This is done to decrease reactivity of tissue and increase ♦♦ Based on the number of filaments in the suture material:
in tensile strength. • Monofilament
♦♦ Chromic gut has better knot security as compared to • Multifilament
plane gut. • Pseudomonofilament
♦♦ Coating by chromic salts increases its resistance to re- ♦♦ Based on the diameter of the thread in cross section:
sorption. As plain gut looses its strength by 10 to 15 days, • 1 – 0 to 10 – 0. With an increase in number of zeros,
chromic gut takes the double of this time, i.e. 3 to 4 weeks. diameter of material reduces
430   Mastering the BDS IIIrd Year  (Last 25 Years Solved Questions)

♦♦ Based on the coating applied on the material: • 4 – 0, 5 – 0: used for suturing in extremities
• Teflon coated • 3 – 0: For scalp sutures
• Chromic coated, etc. • 3 – 0, 4 – 0: commonly used in most of the oral surgical
procedures.
Description of Suturing Materials
Q.8. Write brief answer on types of sutures.
Absorbable Materials
 (Apr 2017, 5 Marks)
♦♦ They loose their strength into the tissues and degrades un- Ans. Following are the types of sutures:
der 60 days, e.g. catgut, polyglycolic acid, polyglactin 910
(Vicryl), polydiaxanone, polyglecaprone, polytrimethylene Interrupted sutures
carbonate, Polyglytone 6211, etc. Interrupted sutures are most common type of suture and are
♦♦ These sutures undergo enzymatic degradation by natural universally used.
enzymes present in the body. Used to close oral mucosal incisions and skin wounds.
♦♦ They are used in deep layer suturing and suturing of
wounds in patient who do not come for removal of sutures. Technique
Non-absorbable Suture Materials ♦♦ Needle should be held two-thirds the distance from tip of
needle with needle holder.
♦♦ They are not degraded by the body.
♦♦ Needle is passed via one side of the flap perpendicular to
♦♦ Suture removal has to be done after end of healing phase.
the tissues and brought out along the curvature of needle.
♦♦ Examples are silk, nylon and polyester.
♦♦ Needle is now passed via other flap at same distance from
Monofilament Suture Material edge of the flap and also at the same depth.
♦♦ It is brought out of the flap along with suture material till
♦♦ It is known as monofilament as they are made up of single
3 to 4 cm of free end of suture material is left.
strand.
♦♦ Now needle should be held in the left hand and wound
♦♦ They produce the advantage of least capillary effect.
around the needle holder once or twice depending on the
♦♦ They decrease the chances of infection.
type of knot.
♦♦ Examples: In monofilament absorbable is monocryl and
♦♦ Free end of suture material is grasped with beaks of needle
non – absorbable is polyamide, polyester, etc.
holder.
♦♦ They have major disadvantage of memory effect due to
♦♦ Material which is wound around the needle holder is
which material come to its original position and this leads
made to slip over the beaks by slow pulling on needle end
to loosening of knot.
of suture material. Free end of suture material should be
Multifilament Suture Material pulled minimally.
♦♦ Knot is stabilized in the manner that it comes to one side
♦♦ It is known as multifilament as it is made up of multiple
of the flap. It should not rest along edges of wound.
thin strands of suture material which are either rolled,
♦♦ For completion of knot, hold needle in left hand and roll
twisted or braided together to form uniform strand of
suture material around the beaks of needle holder in
thread.
opposite direction. Now again grasp free end of suture
♦♦ They are easy to handle and have good knot tying prop-
material and slide suture material over free end to sta-
erties.
bilize the knot.
♦♦ Knot placed cannot get slipped.
♦♦ Hold both the free ends and needle end of suture mate-
♦♦ They are used at places where good strength is needed to
rial taut for assistant to trim them with scissors leaving 3
hold the wound edges together.
to 4 mm.
♦♦ It has more capillary action and can act as source of infection.
♦♦ Example is black braided silk. Mattress Sutures
Pseudomonofilament Horizontal Mattress Sutures
♦♦ This suture material is made up of numerous strands of They are used in areas where there is underlying bony defect.
fiber which are processed by twisting, grinding and polish-
ing to produce a monofilamentous appearance. Technique
♦♦ Example is catgut. ♦♦ Needle is first passed via one flap and at same vertical
level through other flap similar to placing of an interrupted
Based on Diameter of Thread in Cross-section
suture but the knot is not placed.
♦♦ Suture materials are labeled from 1 – 0 to 10 – 0. ♦♦ Needle is now passed at the distance 3 to 4 mm parallel
♦♦ With an increase in number of zeros, diameter of material horizontally where the needle was passed through the
reduces. second flap.
• 10 – 0: is used for microsurgery repair ♦♦ Now it is passed via the first flap at same vertical level
• 5 – 0, 6 – 0: suturing for skin on face as last bite.
Section 2:  General Surgery  431

♦♦ In this way, the needle comes back via the same flap where Technique
it is started at a distance of 3 to 4 mm from entry point. ♦♦ For this type of suturing technique non-absorbable mono-
♦♦ Now the knot is placed and is stabilized over that side. filament suture is used.
♦♦ First the suture is passed via skin at one end of the wound
Vertical Mattress Suture
such that the needle is brought in the wound.
They are used to close skin wounds mainly in areas where edges ♦♦ Needle is now passed alternately through opposite wound
of skin tend to invert. edges completely in subcuticular layer without piercing the
skin and without placing the knot anywhere in the wound.
Technique ♦♦ At the end, needle should be brought again through edge
of the wound through skin.
♦♦ In this needle is first passed far away from wound edges
♦♦ Pull the suture material tightly to get good approximation
and then nearer or at a more superficial level.
of wound edges without any bunching.
♦♦ Needle is passed via one wound edge taking a deep bite
♦♦ Trim the suture materials to leave long ends and taped on
of tissue almost 4 to 8 mm from wound edge.
both ends to secure from slipping.
♦♦ It is now passed through other edge at same depth and
brought out. Knot is still not placed. Q.9. Write brief answer on endotracheal intubation.
♦♦ Needle is now turned around and passes backward  (Apr 2017, 5 Marks)
through second flap at a level closer to wound edges. Ans. Endotracheal intubation is the most basic skill which is
♦♦ Needle is now passed via one flap at same superficial level acquired by an anesthesiologist. Endotracheal intubation
and is brought out. In this way, both edges suture material consists of introduction of a tube inside trachea for
are on the same side. maintaining the patency and protecting the airway as
♦♦ Knot is now placed and stabilized on side where the suture well as to ensure proper oxygenation and ventilation.
first began. Endotracheal intubation is the definitive way of maintaining
the airway in patients who need muscle paralysis as well
Continuous Sutures without Locking as intermittent positive pressure ventilation.
They are used to suture large wounds. Intra-orally, they are Whenever general anesthesia is given and needs to be
used when full quadrant alveoloplasty is done. maintained for long periods, endotracheal intubation is
done.
Technique
Indications
♦♦ At first time place the interrupted suture.
♦♦ While cutting the suture ends, cut the free ends leaving ♦♦ For inducing general anesthesia for long time, i.e more
the suture material with needle behind. than l to 2 hours.
♦♦ Needle is now passed via flaps of wound alternately to get ♦♦ To maintain patency of the airway in unconscious patients.
continuous oblique sutures all along the length of wound. ♦♦ For protecting lungs from aspiration of regurgitated gastric
♦♦ At the end of wound, knot is placed by loop of the suture contents.
and the needle end of suture material. ♦♦ For ensuring proper delivery of adequate tidal volumes
to the lungs.
Continuous Sutures with Locking ♦♦ For clearing excessive as well as retained secretions from
Technique the lungs.

♦♦ First of all an interrupted suture is placed. Contraindications


♦♦ Now the needle is passed through the loop made by suture ♦♦ In cases where upper airway integrity is lost, i.e.
material. • In extensive maxillofacial injury with bilateral
♦♦ Assistant should be made to follow the suture by holding fractures of mandible and maxillae.
suture material close to the tissues where needle was last • Injuries to the neck along with laryngeal rupture
passed via the loop. • Large tumors of upper airway.
♦♦ Each time needle pass through the flap and under the In above conditions, endotracheal intubation may be
suture loop, assistant should hold suture material tightly extremely difficult and even dangerous. In above situations,
close to the tissue to prevent suture material slipping and tracheostomy may be a better choice.
becoming loose.
♦♦ At end of suture line the knot is made with suture loop Technique
and the needle end of suture material.
♦♦ A pillow of 7 to 10 cm should be positioned under patient's
head which enables mild flexion at the cervical spine.
Subcuticular Sutures
♦♦ Head is then extended at the atlanto-occipital joint. This is
They are used for closing the cosmetic wounds. known as intubating position or "sniffing position".
432   Mastering the BDS IIIrd Year  (Last 25 Years Solved Questions)

Procedure ♦♦ As incision is given and pus comes out of the swelling, it


should be sent for culture and sensitivity.
♦♦ Endotracheal intubation can be done in multiple ways,
♦♦ A sinus forceps or finger is introduced now inside the
i.e. can be done either orally or nasally; can be done either
abscess cavity and all the loculi present inside the cavity
under direct vision or indirectly by fiberoptic scope. It can
should be broken. As fresh blood oozes out, this indicates
be done blindly when visualization of the glottis by direct
the completion of procedure.
means is not possible and fibreoptic scope is not available.
♦♦ Cavity should be irrigated with hydrogen peroxide solu-
In such cases, if the regular antegrade technique, i.e. mouth
tion or iodine solution.
or nose to larynx or a retrograde intubation, i.e. larynx to
♦♦ If size of the cavity is large, it should be packed by roller
mouth can be tried.
gauze which is soaked in iodine and it is removed after
♦♦ In the retrograde technique, pass a guide wire from
24 to 48 hours. Packing helps in controlling the bleeding
the cricothyroid membrane upward inside the mouth
and due to the pack the opening of abscess cavity does
or nose and an endotracheal tube is guided over it into
not close. By 7 to 10 days, the cavity collapses spontane-
the larynx.
ously and granulation tissue fills up the cavity and healing
♦♦ Procedure of endotracheal intubation is done with the
takes place.
patient anesthetized but can also be carried out with the
♦♦ An abscess should not be closed, as it contains bacteria,
patient awake after administering local anesthesia to up-
etc. It should be drained out.
per airway when a difficult intubation is anticipated.
♦♦ Many of the airway adjuncts are available for use when
a difficult airway is encountered, especially when it is Postoperative Management
unanticipated. These include oropharyngeal airway,
nasopharyngeal airway, laryngeal mask airway and ♦♦ Proper antibiotics should be given to the patient.
Combitube®. ♦♦ Control of diabetes should be done, if patient is diabetic.
♦♦ Regular dressing of wound is done by anti-septic
Complications solution.
Following are the complications of endotracheal intubation:
Q.11. Write short note on laparoscopic surgery or minimal
Immediate
access surgery. (Jan 2017, 10 Marks)
♦♦ Trauma to the teeth, lips, tongue, pharynx or larynx. Ans.
♦♦ Hemodynamic changes such as tachycardia, hypertension,
myocardial ischemia. Advantages of Laparoscopic Surgery
♦♦ Misplaced tube, i.e. accidental extubation and esophageal
♦♦ Relatively less painful compared to open surgery. Trauma
intubation
of access is very less.
Delayed ♦♦ Shorter hospital stay and early return to work.
♦♦ Faster postoperative recovery.
Laryngeal granuloma, laryngeal or subglottic stenosis ♦♦ Better visualization of the anatomy, i.e. better approach for
Q.10. Write brief notes on incision and drainage. dissection and visualization of other parts of abdomen for
 (Apr 2017, 2 Marks) any other pathology.
Ans. Incision and drainage is the surgical procedure carried ♦♦ Instrumental access to different abdominal locations is
out to relieve the pus. many times better compared to open method.
♦♦ Mainly incision and drainage is done in pyogenic abscess. ♦♦ Minimal scar on the abdomen.
♦♦ Incision and drainage is contraindicated in cold abscess.
Instruments Used
♦♦ During incision and drainage position of the patient should
be supine, prone or lateral depending upon site of abscess. ♦♦ Zero degree laparoscope is commonly used. Side viewing
scopes are also used to have better visualization 30°.
Procedure ♦♦ Cold light source either halogen lamp or xenon lamp is
used. Halogen lamp is used commonly and is cheaper.
♦♦ For this procedure, preferred anesthesia is general anes- Xenon lamp gives high visualization.
thesia because abscess is multiloculated and infiltration ♦♦ Camera: 3 chip camera is commonly used with high reso-
of lignocaine into the abscess cavity does not act because lution
of the acidic pH of the pus. However, a superficial ab- ♦♦ Video—monitor to display images.
scess which is pointing can be managed without general ♦♦ CO2 insufflator.
anesthesia. ♦♦ Long fine dissectors like in open surgical techniques.
♦♦ Provide a stab incision over the most prominent part of ♦♦ Hooks and spatulas are used along with cautery for dis-
the swelling, i.e. part which is red, skin is thinned out over section.
it and is pointed. ♦♦ Clip applicators
Section 2:  General Surgery  433

♦♦ Needle holders ♦♦ Wound infection.


♦♦ Endostaplers ♦♦ Mortality—0.5%.
♦♦ Veress needle
♦♦ Suction—irrigation apparatus. Relative Contraindications
♦♦ Trocars of different sizes—10 mm, 5 mm. ♦♦ Patients with compromised cardiac status
♦♦ Reducers to negotiate smaller instruments through larger ♦♦ Peritonitis
ports. ♦♦ Previous abdominal surgeries
♦♦ Bleeding disorders
Preparation ♦♦ Morbid obesity
Always general anesthesia. Other preparations are same as for ♦♦ Third trimester of pregnancy
open method. ♦♦ Portal hypertension.

Technique Basic Laparoscopic Surgeries


♦♦ Pressure bandages are applied to both legs to improve the ♦♦ Laparoscopic cholecystectomy: This is indicated in gallstones,
venous return and to decrease the stasis. cholecystitis, biliary colic
♦♦ Head end of the table is lowered to have easier insertion ♦♦ Laparoscopic appendicectomy: This is indicated in acute ap-
of veress needle and scope. pendicitis.
♦♦ Ryle’s tube and Foley’s catheter are essential before inser-
Q.12. Write short note on robotic surgery.
tion of the trocars.
♦♦ Pneumoperitoneum is created using veress needle through  (Jan 2018, 10 Marks)
umbilical incision. Access can be achieved by open method Ans. Robotic surgery is a type of minimally invasive surgery.
through an umbilical incision. Carbon dioxide is com- “Minimally invasive” means that instead of operating
monly used to create pneumoperitoneum. on patients through large incisions, we use miniaturized
♦♦ Pneumoperitoneum is created up to a pressure of 15 mm surgical instruments that fit through a series of quarter-
Hg which distends the abdominal cavity adequately to inch incisions.
have proper visualization of the abdominal contents. ♦ When performing surgery with the da Vinci Si—the
♦♦ Laparoscope is inserted through the umbilical port (10 mm). world’s most advanced surgical robot—these miniatur-
♦♦ Abdomen is evaluated for any pathology. Liver, gallblad- ized instruments are mounted on three separate robotic
der, pelvic organs are visualized. arms, allowing the surgeon maximum range of motion and
♦♦ Additional ports (3–4) through trocars are placed depend- precision. The da Vinci’s fourth arm contains a magnified
ing on the procedure to be done. It may be either 5 mm high-definition 3-D camera that guides the surgeon during
port or 10 mm port. These ports are placed in such a way to the procedure.
have a proper triangulation of instruments for dissection. ♦♦ The surgeon controls these instruments and the camera
♦♦ To use clip applicator 10 mm port is required. from a console located in the operating room. Placing his
fingers into the master controls, he is able to operate all
Physiologic Changes due to Pneumoperitoneum four arms of the da Vinci simultaneously while looking
♦♦ Carbon dioxide causes hypercapnia, acidosis and hypoxia. through a stereoscopic high-definition monitor that liter-
♦♦ Pneumoperitoneum exerts pressure on the IVC, decreases ally places him inside the patient, giving him a better, more
the venous return and so the cardiac output. detailed 3-D view of the operating site than the human
♦♦ It increases the arterial pressure also. eye can provide.
♦♦ It compromises the respiratory function by compressing ♦♦ Every movement he makes with the master controls is
over the diaphragm impairing the pulmonary compliance. replicated precisely by the robot.
Complications ♦♦ When necessary, the surgeon can even change the scale of
♦♦ Carbon dioxide narcosis and hypoxia. the robot’s movements: If he selects a three-to-one scale,
♦♦ Sepsis—subphrenic abscess, pelvic abscess, septicemia. the tip of the robot’s arm will move just one inch for every
♦♦ IVC compression three inches the surgeon’s hand moves. And because of
♦♦ Bleeding the console’s design, the surgeon’s eyes and hands are
♦♦ Leak from the site, e.g. bile leak. always perfectly aligned with his view of the surgical site,
♦♦ Organ injury during insertion of ports, e.g. major vessels, minimizing surgeon fatigue.
bowel, mesentery, liver. ♦♦ The ultimate effect is to give the surgeon unprecedented
♦♦ Subcutaneous emphysema and pneumomediastinum. control in a minimally invasive environment. Utilizing
♦♦ Gas emboli, though is rare but fatal. this advanced technology, surgeons are able to perform
♦♦ Postoperative shoulder pain due to irritation of diaphragm. a growing number of complex urological, gynecological,
♦♦ Cardiac dysfunction due to decreased venous return. cardiothoracic and general surgical procedures. Since these
♦♦ Injury to the abdominal wall vessels and nerves. procedures can now be performed through very small inci-
♦♦ Cautery burn to abdominal structures. sions, patients experience a number of benefits compared
♦♦ Abdominal wall hernias. to open surgery, including:
434   Mastering the BDS IIIrd Year  (Last 25 Years Solved Questions)

• Less trauma on the body ♦♦ Pain attacks are precipitated by touching some “trigger
• Minimal scarring, and zone” on the face, i.e. vermilion border of lip, the ala of
• Faster recovery time. nose, the cheek and around the eyes. Patient usually avoid
touching of skin over these areas. Patient has unshaven
face, avoid brushing, undergo dental extraction.
22. Neurological Injuries ♦♦ Objective signs or sensory loss are demonstrated on
examination.
Q.1. Write short note on trigeminal neuralgia. ♦♦ Onset of pain is sudden and tends to persist for weeks or
 (Feb 2013, 5 Marks) (June 2015, 5 Marks) months before remitting spontaneously.
 (Mar 2006, 5 Marks) (Jan 2016, 5 Marks)
 (Jan 2012, 5 Marks) (Aug 2011, 5 Marks)
 (June 2010, 5 Marks) (Sep 2009, 5 Marks)
 (Apr 2008, 5 Marks) (Oct 2007, 5 Marks)
 (Mar 2007, 5 Marks) (Sep 2005, 8 Marks)
 (Mar 2003, 10 Marks) (Mar 1998, 6 Marks)
Or
Answer briefly on trigeminal neuralgia.
 (Mar 2016, 3 Marks)
Or
Write in short about trigeminal neuralgia.
 (July 2016, 5 Marks)
Or Fig. 33:  Trigeminal neuralgia
Discuss the treatment of trigeminal neuralgia.
(Sep 2010, 10 Marks) Treatment
Or Medical Treatment
Write short answer on trigeminal neuralgia.
♦♦ Carbamazepine (tegretol) has a special effect on the parox-
 (Apr 2018, 3 Marks)
ysmal pain. This is considered to be the best conservative
Ans. It is also called as Tic Douloureux, trifacial neuralgia or treatment for trigeminal neuralgia. As an initial dose, 100
Fothergill’s disease. mg twice daily till relief is established.
Trigeminal neuralgia is an extremely painful condition ♦♦ Dilantin: Diphenylhydantoin, an anticonvulsant drug has
along the distribution of any branches of trigeminal been recommended, effective when given orally, 300 to
nerve. 400 mg/day.
♦♦ Recently, baclofen an antispastic drug is also being used.
Etiology ♦♦ A combination of dilantin and carbamazepine may also
be given.
♦♦ Dental pathosis at times leads to trigeminal neuralgia.
♦♦ Clonazepam an antiepileptic found to be useful.
♦♦ Due to excessive traction divisions of trigeminal nerve are
♦♦ Anti-inflammatory agents like indomethacin and short
affected which leads to trigeminal neuralgia.
course of steroids have been found to be useful.
♦♦ Allergic and hypersensitivity reaction may lead to trigemi-
nal neuralgia. Surgical Treatment
♦♦ Mechanical factors such as pressure caused by aneurysms
♦♦ Infection of the nerve with anesthetic solution: Local anesthetics
of internal carotid artery.
of various types injected near the peripheral branches of
♦♦ Secondary lesions such as carcinomas of maxillary antrum,
the trigeminal nerve to serve to provide temporary relief
carcinoma of nasopharynx leads to trigeminal neuralgia.
from pain.
♦♦ Injection of the nerve with alcohol:
Clinical Features
• The most popular material, alcohol, can be placed
♦♦ It occurs in middle age and older people. directly into the area where a nerve exits from the
♦♦ Female predilection is seen. skull or more peripherally.
♦♦ Pain is paroxysmal in nature, last for few minutes and is • When alcohol contracts the nerve, neurolysis occurs
of extreme intensity. distal to the injection site.
♦♦ Pain is usually limited to the distribution of trigeminal • Generally, 95% alcohol is used or procaine or
nerve and is unilateral. At times pain is bilateral too. monocaine 2%, chloroform 5%, absolute alcohol 70%,
♦♦ Pain is provoked by obvious stimuli to the face. A touch, a Ringer’s solution 23% can also be used.
draft of air, any movement of face as in talking, chewing, ♦♦ Nerve sectioning and nerve evolution (peripheral neurectomy):
yawning or swallowing. This procedure is more lasting and effective than an injec-
Section 2:  General Surgery  435

tion with alcohol. Peripheral neurectomy results in high ♦♦ Presence of wide palpebral fissure
degree of success in elimination of pain. ♦♦ Presence of epiphora.
♦♦ Electrocoagulation of gasserian ganglion: Diathermy ap-
paratus is placed in the gasserian ganglion to coagulate Treatment
and destroy it. Medicinal Treatment
♦♦ Percutaneous radiofrequency trigeminal neurolysis: It is per-
♦♦ Patients with Bell’s palsy have excellent prognosis.
formed by insertion of temperature monitoring electrode
Treatment of Bell’s palsy is controversial as spontaneous
through foramen ovale into trigeminal ganglion. Advan-
recovery is present.
tages include decreased mortality and morbidity and
♦♦ Treatment is given to the patients who have onset of pa-
permanent cure.
ralysis under 1–4 days of an initial visit.
♦♦ Rhizotomy: Actual cutting of trigeminal sensory root results
♦♦ Corticosteroids, i.e. prednisolone 1 mg/kg or 60 mg per
in permanent anesthesia in most patients.
♦♦ Bulbar trigeminal tractotomy: The descending tract of the day for 6 days followed by the taper of 10 days.
trigeminal nerve may be cut in the area of medulla ob- ♦♦ Anti-viral drugs can be given in the patients in which bell’s
longata to induce loss of pain and temperature sensation. palsy is associated with herpes infection.
♦♦ In Bell’s palsy eye of the patient is at risk for drying which
Q.2. Describe in brief Bell’s palsy. (Sep 2002, 5 Marks) cause corneal abrasion and corneal ulcer. Eye care consists
Or of inducing the artificial tears in daytime along with eye-
Write short note on bell’s palsy. glasses. At night eye lubricant can be used.
 (Nov 2014, 3 Marks) (Dec 2015, 5 Marks) Surgical treatment
(Feb 2013, 5 Marks) (Aug 2012, 5 Marks) ♦♦ Surgical treatment for Bell’s palsy is surgical decompres-
Ans. It is also called as 7th nerve paraplegia or facial palsy. sion and anastomosis of the nerve. In this facial and hy-
poglossal nerve get anastomosed which helps in restoring
Etiology
the partial function.
♦♦ Cold: It occurs after exposure to cold. ♦♦ Nerve grafting using greater auricular nerve, sural nerve,
♦♦ Trauma: Extraction of teeth or injection of local anesthetic lateral cutaneous nerve of thigh or hypoglossal nerve.
may damage the nerve and causes subsequent paralysis. ♦♦ Suspension of angle of mouth to zygomatic bone using
♦♦ Surgical procedure: Such as removal of parotid gland tumor temporal fascia sling.
in which the facial nerve is sectioned can also cause facial ♦♦ Lateral tarsorrhaphy: This prevent corneal ulceration
paralysis. ♦♦ Medial canthus reconstruction: To decrease epiphora
♦♦ Tumors: Tumors of the cranial base, parapharyngeal space ♦♦ Cross facial nerve transplantation from opposite side using
and infratemporal fossa cause facial nerve palsy. its insignificant branches.
♦♦ Familial: Familial and hereditary occurrence is also ♦♦ Dynamic neurovascular muscle graft
reported in case of Bell’s palsy. ♦♦ Upper lid gold weights to protect cornea.
♦♦ Facial canal and middle ear neoplasms may lead to Bell’s
palsy. Q.3. Discuss etiology and management of injury of facial
nerve. (Feb/Mar 2004, 10 Marks)
Clinical Features Ans. Injuries of facial nerve causes facial palsy or Bell’s palsy.
♦♦ It is in older age group.
♦♦ Female predilection is most common. Etiology of Facial Paralysis
♦♦ There is presence of paralysis over one side of the face. ♦♦ Congenital: Mobius syndrome (Uncommon and poorly
♦♦ Drooping of angle of mouth is present over the affected understood)
side and saliva is drooling from the affected side. ♦♦ Traumatic:
♦♦ Patient is unable to raise the eyebrow of the affected side. • Birth injury
♦♦ Patient is unable to close the eye over affected side and • Iatrogenic injury at the time of surgeries, e.g. surgery
tears continuously roll down. for parotid gland surgery etc.
♦♦ Eyeball roll upward when attempted to close the eye, this • Blunt or penetrating trauma to nerve.
is known as Bell’s phenomenon. • Fracture of the temporal bone can lead to facial nerve
♦♦ Over the affected side, patient has lost the taste sensation. injury.
♦♦ Patient complaints of pain in or behind the ear. ♦♦ Infections
♦♦ Presence of numbness over the affected side of face. • Virally mediated diseases, i.e. herpes zoster, mumps,
♦♦ Patient has mask like appearance when he/she tries to coxsackie virus and mononucleosis.
smile. • Bacterial infections, i.e. sequelae to otitis media, Lyme
♦♦ There is difficulty in blowing or clenching. disease, mastoiditis.
♦♦ Obliteration of nasolabial fold ♦♦ Inflammatory conditions: Sarcoidosis
436   Mastering the BDS IIIrd Year  (Last 25 Years Solved Questions)

♦♦ Neoplastic: Tumors of the parotid gland (typically malig- and if a mismatch is seen, then one end may be trimmed
nant tumors) in a beveled fashion to obtain a better surface area.
• Facial nerve schwannomas
Cable Nerve Grafting
• Acoustic neuromas
• Neoplasms of the brain such as brainstem tumors. If sufficient length is not present to approximate the nerve
♦♦ Idiopathic: primarily, cable grafting is done.
• Bell`s palsy is most common type of facial paralysis. Patient must be informed about the operation of the donor site.
It occurs due to virally induced inflammation of the Commonly used nerves are greater auricular nerve, sural nerve,
nerve resulting in compromise of the function of nerve, lateral femoral cutaneous nerve, etc.
swelling and vascular supply to the nerve.
• Melkersson—Rosenthal syndrome Technique
• Myasthenia gravis.
♦♦ Great auricular nerve is located by drawing a line between
Management angle of jaw and mastoid tip. Bisect this line at a right angle
by great auricular nerve as it passes around the posterior
Facial nerve repair can be done by following surgical methods, border of the sternocleidomastoid muscle just behind the
i.e. external jugular vein.
♦♦ Direct repair ♦♦ Sural nerve may be located between lateral malleolus and
♦♦ Cable nerve grafting Achilles tendon. It lies deep or posterior to the saphenous
♦♦ Nerve substitution techniques. vein. Sural nerve then runs superiorly up to back of lower
Direct Repair leg in a subcutaneous plane until it descends between
the two heads of the gastrocnemius toward the popliteal
Preoperative Details fossa and its origin off the tibial nerve. Sural nerve can be
• Direct repair of the facial nerve is the best method harvested either by giving a single long incision from the
to rehabilitate the paralyzed face. By this method, ankle to popliteal fossa (depending on the length of nerve
there are chances of restoring spontaneous, emotional required) or a series of shorter transverse incisions. The
expression to the face. nerve may be dissected under direct vision with the single
• Direct repair is indicated in cases where the length of incision, or by employing a fascia stripper and making the
nerve is adequate and reapproximation of the nerve stepwise incisions. Nerve grafting should be done same
is possible without tension along the nerve. as for primary repair. In cable grafting, it may be helpful
• It involves restoring the continuity of the both ends to obtain enough nerve graft length to allow the graft to
of the nerve directly by using sutures. have some redundancy between the ends of facial nerve.
• For the successful repair of nerve, it is mandatory This provides a C or S shape and ensures tension-free
that functional motor unit be available to receive coaptation.
the innervations, i.e. facial musculature should not ♦♦ Graft should lie in healthy and vascularized recipient site
have undergone excessive atrophy and there should which is free of scar tissue.
be no fibrosis at the motor end plate which prevent ♦♦ Epineurium should be intact and 10-0 sutures may be used
reinnervation. to repair the nerve without tension.
♦♦ A soft silicone tube can be used which surround the anas-
Technique of Direct Repair tomotic site to prevent in growth of scar tissue inside the
♦♦ Parotidectomy incision is given and the nerve is exposed. surgical site and also to keep cut axons approximated.
Identify the nerve and its branches. If the intratemporal
Nerve Substitution Techniques
portion of the nerve is injured, this area is exposed with
the help of a mastoidectomy. There are two types of nerve substitution techniques:
♦♦ As identification is over, the nerve is followed distally as 1. Hypoglossal facial anastomosis
required. 2. Cross face grafting
♦♦ Handle the nerve as atraumatically as possible. Surgery
should be performed by using a surgical microscope. This Hypoglossal Facial Nerve Anastomosis
allows precise alignment of the nerve ends.
♦♦ 2 to 3 sutures should be placed by using 8-0 to l0-0 fine It is the most standard procedure to reanimate the face when
monofilament sutures. Sutures are usually placed through proximal end of the facial nerve is not present or undergone
the epineurium. degeneration and the peripheral aspect of nerve is still viable.
♦♦ Perineurial, endoneurial and interfascicular suturing This procedure can be done as a primary procedure, i.e it can be
should also be done. done along with the surgical procedure that lead to the sacrifice
♦♦ Most important relationship is the size match of endoneu- of the facial nerve or as a secondary procedure, i.e. when facial
rial surfaces. This must be inspected with magnification, nerve paralysis is noticed postoperatively.
Section 2:  General Surgery  437

Technique Q.4. Discuss the management of head injury.


♦♦ Parotidectomy incision is given and facial nerve is exposed.  (Mar 2003, 15 Marks)
Identify the nerve as it exits the stylomastoid foramen. It Ans. Initial assessment of head injuries must follow advanced
trauma and life support (ATLS) guidelines with an
is sharply transected here in this region.
initial primary survey, then *resuscitation followed by
♦♦ Identify the hypoglossal nerve in the neck by following secondary survey then definite management like: airway,
the posterior belly of the digastric muscle to the hyoid breathing, circulation, disability and exposure.
bone. Nerve is followed distally to gain length for the
anastomosis.
♦♦ Transect descendens hypoglossi to gain length and mobi-
lization for the anastomosis.
♦♦ Transect hypoglossal nerve distally and approximate it to
the facial nerve passing medial or lateral to the digastrics
muscle. The nerve ends are then grafted together.
Cross Face Grafting
♦♦ This procedure attempts to connect the branches of para-
lyzed facial nerve to the corresponding branches on the
normal side.
♦♦ It is done in cases where the proximal end of the nerve is
not available for repair but the motor end plates on the
paralyzed side should be functional for the success of
this procedure. Fig. 34:  Malunion of fracture of bone
♦♦ This procedure is not possible in cases in which the nerve
has been paralyzed for over a year. Important Histories of Head Injury
Technique ♦♦ Period of loss of unconsciousness
♦♦ Period of post-traumatic amnesia
♦♦ Very commonly sural nerve is used to carry nerve to
♦♦ Cause and circumstances of injury
opposite side as it consists of adequate length to thread
♦♦ Presence of headache and vomiting.
across from one end of the face to another. It is indicated
in cases where multiple branches are to be anastomosed. Physical Examination Includes
Great auricular nerve may be used in cases when a single
♦♦ Thorough general examination.
nerve is grafted.
♦♦ Dissect affected side first and identify all the branches. If ♦♦ Local examination for evidence of injury, skull fractures.
multiple branches should be grafted they are identified ♦♦ Determination of conscious level by Glasgow coma scale.
and exposed a little beyond the parotid gland.
♦♦ Expose the normal facial nerve and the donors should be Investigations
taken from distal border of the parotid gland. ♦♦ X-ray skull: To look for fracture, relative position of calcified
♦♦ Now identify sural nerve and the branches should be pineal gland, presence of intracranial air.
tunneled across the face. First suture the nerve graft to ♦♦ Serum electrolyte measurement is done
the normal side and then to the chosen branch on the ♦♦ Blood grouping and cross matching of blood is done.
paralyzed side. ♦♦ CT scan: Plain (not contrast) to look for cerebral edema,
♦♦ For good healing meticulous hemostasis and the use of hematoma, midline shift, fractures, ventricles, brainstem
drains to prevent the formation of hematoma is essential. injury.
♦♦ Some iatrogenic weakness is expected on the donor side ♦♦ Carotid arteriography
after this procedure and the patient must be warned of this ♦♦ Investigations for other injuries such as ultrasound of
before going for procedure. abdomen.
Postoperative Care of the Patient ♦♦ Monitoring of intracranial pressure.

♦♦ As return of function takes few months with each of these Glasgow Coma Scale
procedures, attention to patient care should be given such
I. Eye Opening
as eye protection.
♦♦ Various adjunctive procedures, i.e. gold weight eyelid ♦♦ Spontaneous 4
implants or brow lift may be considered. ♦♦ To speech 3
♦♦ Reexploration and revision should be done if improvement ♦♦ To pain 2
is not seen in one year following grafting. ♦♦ Nil 1
438   Mastering the BDS IIIrd Year  (Last 25 Years Solved Questions)

II. Best Motor Response ♦♦ After evacuation of hematoma patient should be admitted
♦♦ Obeys command 6 to ICU and ventilated to a Pco 2 of 4 to 4.5 Kpa.
♦♦ Localizes pain 5 ♦♦ A central line, arterial line and urinary catheter should
♦♦ Withdrawal to pain 4 be inserted.
♦♦ Flexion to pain 3 ♦♦ Head of bed should be positioned 40 degrees up and pa-
♦♦ Extension to pain 2 tient is given analgesia (Fentanyl)
♦♦ Nil 1 ♦♦ IV fluids are administered should be isotonic. It is
administered till nasogastric tube is inserted for feeding.
III. Verbal Response ♦♦ ICP monitor should be inserted intra­parenchymally to
♦♦ Oriented 5 measure ICP and CPP (ideally should be < 25 mm Hg and
♦♦ Confused 4 CPP should be about 70 mm Hg) If CPP is low, ionotropic
♦♦ Inappropriate words 3 agent should be used.
♦♦ Incomprehensible 2 ♦♦ Mannitol or frusemide could lower the ICP if it is not
♦♦ Nil 1 controlled by these agents then EEG burst suppression
Total score is 15 therapy with a barbiturates, ventriculator or lumber CSF
Mild head injury score 13 to 15 drainage to be considered.
Moderate head injury score 9 to 12 ♦♦ Antibiotics like penicillin, ampicillin are given to prevent
Severe head injury score less than 8 (3 to 8) risk of meningitis.
Any patient who has a coma score of 7 or less than 7 is said to Surgical Management
be in coma.
♦♦ Burr hole is made and hematoma is evacuated.
Pupillary response should be elicitated to determine whether
♦♦ Surgery is done in case when:
there is incipient transtentorial habitat, with Oculomotor palsy
• Consciousness is decreasing continuously.
and responses recorded.
• Pupil becomes fixed or dilated.
General Management • Pulse rate becomes less than 60/minute.
♦♦ Management of head injuries includes ventilation, surgery, Surgical Management
ICU management of intracranial pressure, cerebral perfu-
sion pressure and oxygenation. ♦♦ Craniotomy is done and cranial flap is raised. Clot is evacu-
♦♦ Management of patients having head injury is based on ated applying hitch stitches between dural layer and scalp.
Glasgow coma scale following the resuscitation.
Q.5. Write short note on danger area of face. 
♦♦ Patients with mild injury, i.e. having Glasgow comma scale
 (Sep 2005, 5 Marks)
of 14 to 15 should be admitted to the ward where thorough
neurological examinations are performed. Ans. Dangerous area of face extends from corner of mouth to
♦♦ If patient with mild head injury subsequently deteriorate bridge of nose. It is basically the area of upper lip and
neurologically, CT scan of patient’s head should be done lower part of nose.
and local neurosurgical unit should be contacted. ♦♦ Infection from this area spreads through deep facial vein
♦♦ Mild head injury patients should remain under observation to pterygoid plexus and from pterygoid plexus to commu-
until complete neurological recovery occur. Such patients nicating vein and from communicating vein to cavernous
are discharged to the responsible adult which can take sinus which can lead to life-threatening cavernous sinus
good care of such patient at home for few days. thrombosis.
♦♦ Patients with Glasgow comma scale of 13 or less should ♦♦ Mainly due to the special nature of blood supply to hu-
undergo for CT scan of their head. If there is presence man nose and the surrounding area there is possibility for
of acute lesion on CT scan or there is presence of diffuse retrograde infections from nose to spread to brain.
cerebral edema should referred to local neurosurgical unit. ♦♦ One of the misconception is present that veins of head does
CT scan should also be sent to the unit. A provisional ra- not have one way valves as it is present in other veins of
diography report should also be sent to referring hospital. circulatory system. Reality is venous valves are not absent
♦♦ If there is presence of compound depressed skull fracture, but there is communication between facial vein and cav-
severely depressed fracture, CSF otorrhea and rhinorrhea ernous sinus and the direction of blood flow which leads
patient should be referred to neurosurgical unit. to spread of infection from face.
♦♦ Airways are protected by using mouth gag, endotracheal ♦♦ Boils and pimples present in the area of upper lip and
intubation or tracheostomy whenever required. lower part of nose should never be squeezed and pricked
♦♦ Throat suction, bladder and bowel care as well as good by a needle, by doing so the infection spreads to cavernous
nursing is essential. sinus causing cavernous sinus thrombosis.
Section 2:  General Surgery  439

♦♦ Patient is unable to raise the eyebrow of the affected side.


♦♦ Patient is unable to close the eye over affected side and
tears continuously roll down.
♦♦ Eyeball rolls upward when attempted to close the eye, this
is known as Bell’s phenomenon.
♦♦ Over the affected side, patient has lost the taste sensation.
♦♦ Patient complaints of pain in or behind the ear.
♦♦ Presence of numbness over the affected side of face.
♦♦ Patient has mask-like appearance when he/she tries to
smile.
♦♦ There is difficulty in blowing or clenching
♦♦ Obliteration of nasolabial fold
♦♦ Presence of wide palpebral fissure
Fig. 35:  Danger area of face ♦♦ Presence of Epiphora.

Q.6. Write short note on facial palsy. Treatment


 (May/June 2009, 5 Marks) (Mar 2007, 5 Marks) Medicinal Treatment
Or ♦♦ Patients with Bell’s palsy have excellent prognosis.
Write brief note on facial palsy. (Apr 2017, 2 Marks) Treatment of Bell’s palsy is controversial as spontaneous
Ans. Facial palsy refers to the paralysis of facial muscles. recovery is present.
It is of two types: ♦♦ Treatment is given to the patients who have onset of pa-
i. Upper motor neuron palsy ralysis under 1 to 4 days of an initial visit.
ii. Lower motor neuron palsy or Bell’s palsy ♦♦ Corticosteroids, i.e. prednisolone 1 mg/kg or 60 mg per
day for 6 days followed by the taper of 10 days.
Upper Motor Neuron Palsy ♦♦ Antiviral drugs can be given in the patients in which bell’s
♦♦ It affects mainly muscles of lower part of face and is never palsy is associated with herpes infection.
complete. ♦♦ In Bell’s palsy eye of the patient is at risk for drying which
♦♦ It is seldom isolated palsy. cause corneal abrasion and corneal ulcer. Eye care consists
♦♦ The emotional movements are preserved. of inducing the artificial tears in day-time along with eye-
♦♦ There is no muscle contracture. glasses. At night, eye lubricant can be used.
♦♦ There is no reaction of degeneration.
Surgical Treatment
♦♦ Electromyography and nerve conduction is normal.

Lower Motor Neuron Palsy or Bell’s Palsy ♦♦ Surgical treatment for Bell’s palsy is surgical decompres-
sion and anastomosis of the nerve. In this facial and hy-
It is also called as 7th nerve paraplegia or facial palsy. poglossal nerve get anastomosed which helps in restoring
Etiology the partial function.
♦♦ Nerve grafting using greater auricular nerve, sural nerve,
♦♦ Cold: It occurs after exposure to cold. lateral cutaneous nerve of thigh or hypoglossal nerve.
♦♦ Trauma: Extraction of teeth or injection of local anesthetic ♦♦ Suspension of angle of mouth to zygomatic bone using
may damage the nerve and causes subsequent paralysis. temporal fascia sling.
♦♦ Surgical procedure: Such as removal of parotid gland tumor ♦♦ Lateral tarsorrhaphy: This prevents corneal ulceration
in which the facial nerve is sectioned can also cause facial ♦♦ Medial canthus reconstruction: To decrease epiphora
paralysis. ♦♦ Cross facial nerve transplantation from opposite side using
♦♦ Tumors: Tumors of the cranial base, Parapharyngeal space its insignificant branches
and infratemporal fossa cause facial nerve palsy. ♦♦ Dynamic neurovascular muscle graft
♦♦ Familial: Familial and hereditary occurrence is also re- ♦♦ Upper lid gold weights to protect cornea
ported in case of Bell’s palsy.
Facial canal and middle ear neoplasms may lead to Bell’s Q.7. Classification of peripheral nerve injuries with meth­
palsy. ods of treatment of cut injured nerve. 
 (Jan 2011, 8 Marks)
Clinical Features Ans.
♦♦ It is in older age group.
Classification of Peripheral Nerve Injuries
♦♦ Female predilection is most common.
♦♦ There is presence of paralysis over one side of the face. ♦♦ Seddon’s Classification
♦♦ Drooping of angle of mouth is present over the affected • Neurapraxia: It is a temporary physiological paralysis
side and saliva is drooling from the affected side. of nerve conduction. Here recovery is complete.
440   Mastering the BDS IIIrd Year  (Last 25 Years Solved Questions)

• Axonotmesis: Division of nerve fibers or axons with • Often guide sutures of silk may be present which were
intact nerve sheath. Reaction of degeneration is placed earlier during exploration of trauma.
distally with near complete recovery. • Once nerve ends are clean, it is sutured alike primary
• Neurotmesis: Here complete division of nerve fibers suturing with stay sutures, with proper alignment of
with sheath occurs. Degeneration occurs proximally fascicles, followed by epineural suturing.
up to the first node of Ranvier as well as distal to • Here as epineurium is thicker, suturing is easier.
the injury. Recovery is incomplete even after nerve Q.8. Describe briefly Glasgow coma scale.
suturing. There is complete loss of motor and sensory  (Jun 2010, 5 Marks) (Dec 2010, 5 Marks)
functions with loss of reflexes. lf the nerve is mixed Ans. Glasgow coma scale is used in assessment of conscious
type other than pure motor or sensory recovery is level.
still poorer.
• It provides a grading of coma by using a numerical
• Neuromas:
scale which allows serial comparison and prognostic
–– True neuroma or false neuroma
information.
–– End neuroma or side neuroma.
• It relates clinical observation under three headings,
♦♦ Sunderland’s classification
i.e. motor response, verbal response and eye opening.
• Conduction block: Temporary neuronal block
• Following is the Glasgow coma scale
• Axonotmesis but endoneurium is preserved.
• Axonotmesis with disruption of endoneurium, but Glasgow Coma Scale
perineurium is preserved
♦♦ Eye Opening (E):
• Here disruption of endo and perineurium has
• Spontaneous 4
occurred but endoneurium is intact.
• To speech 3
• Neurotmesis with disruption of endoneurium,
• To pain 2
perineurium and epineurium has occurred.
• Nil 1.
Treatment of Cut Injured Nerve ♦♦ Motor Response (M):
• Obeys command 6
Usually, microscope or loup is used for nerve suturing. • Localizes pain 5
♦♦ Epineurorrhaphy: Only epineurium is sutured using inter- • Withdrawal to pain 4
rupted sutures. • Flexion to pain 3
♦♦ Epi-perineurorrhaphy: Initially, perineural sheath and then • Extension to pain 2
epineurium is sutured. • Nil 1.
• If nerve is lacerated marker stitches are placed at cut ♦♦ Verbal Response (V):
end site to identify the nerve for suturing at later • Oriented 5
period. • Confused 4
• If nerve suturing fails or it cannot be done, tendon • Inappropriate words 3
transfer is done at later period after 4 to 6 months. • Incomprehensible 2
Nerve suturing can be: • Nil 1.
♦♦ Primary repair:
• It is done immediately after injury. Coma Score = E + M + V
• Nerve ends are minimally trimmed very close using Total score is 15
a blade. Mild head injury score 13 to 15
• Fascicles of the nerve are oriented correctly.
Moderate head injury score 9 to 12
• Two stay sutures are placed to keep the orientation
properly. Severe head injury less than 8 (3 to 8).
• Usually, epineural suturing is done using 8 zero Q.9. Write short note on peripheral nerve injuries.
polypropylene interrupted sutures.  (Jan 2016, 5 Marks) (Dec 2010, 5 Marks)
• 6-8 sutures are placed for large peripheral nerve like  (Mar 2016, 5 Marks)
median or ulnar nerve. Ans. For classification, refer to Ans 7 of same chapter.
• For or small nerve like digital nerve, only 2-3 sutures
are placed. Etiology
♦♦ Secondary repair: ♦♦ Traumatic: Either closed or open injury
• It is done at a later period. ♦♦ Inflammatory: Leprosy, diphtheria, herpes zoster
• It is in a pre-existing scar tissue. ♦♦ Lead and arsenical poisoning
• Here first nerve ends, but proximal and distal are ♦♦ Alcoholism
identified, carefully dissects; adequately. ♦♦ Diabetes mellitus
• Proximal neuroma and distal glioma trimmed for l cm ♦♦ Vitamin B1 deficiency
to expose the normal fascicles of nerve ends. ♦♦ Porphyria
Section 2:  General Surgery  441

♦♦ Neurofibroma and other neural tumors • Unconsciousness tends to last longer for people with
♦♦ Idiopathic. injuries on the left side of the rain than for those with
Clinical Features injuries on the right.
• With mild traumatic brain injury, the patient may
♦♦ Loss of sensory, motor, autonomous and reflex functions.
remain conscious or may lose consciousness for a
♦♦ Secondary changes in the skin and joint.
few seconds or minutes.
Management • Other symptoms of mild traumatic brain injury
♦♦ Medicinal include headache, vomiting, nausea, lack of motor
• Steroids: They reduce the edema around nerve and coordination, dizziness, difficulty in balancing,
is useful in neurapraxia. Prednisolone 5 to 10 mg is lightheadedness, blurred vision or tired eyes,
effective. ringing in the ears, bad taste in the mouth, fatigue
• Nerve tonics: Vitamin B1, B6, B12, they are supposed to or lethargy, and changes in sleep patterns.
facilitate nerve fiber regeneration and are useful in • Cognitive and emotional symptoms include
cases of neuropraxia and axonotmesis. behavioral or mood changes, confusion, and
• In the cases with acute neuralgic pain, drugs like trouble with memory, concentration, attention, or
carbamazepine or gabapentin can be prescribed. It is thinking.
purely symptomatic treatment. • A person with a moderate or severe traumatic
• Physiotherapy: In the form of electrical nerve stimulation brain injury may have a headache that does not go
(TENS) and in cases of motor nerve exercises and away, repeated vomiting or nausea, convulsions,
massage therapy can be given. an inability to awaken, dilation of one or both
♦♦ Surgical pupils, slurred speech, aphasia (word-finding
• Decompression: It is used, if nerve compression occurs difficulties), dysarthria (muscle weakness that causes
resulting into neuropraxia. It is usually done when
disordered speech), weakness or numbness in the
nerve due to bone deposition in the nerve canal; there
limbs, loss of coordination, confusion, restlessness,
is pressure on the nerve leading to neuropraxia. Here,
or agitation.
enlargement of the canal boundaries is done to relieve
the pressure on the nerve. • Common long-term symptoms of moderate-
• Anastomosis: It is microsurgical repair of the severed to-severe traumatic brain injury are changes in
ends of the nerve. It is useful, when there is no loss appropriate social behavior, deficits in social
of nerve tissue as in accidental clean surgical by judgment, and cognitive changes, especially
transection of the nerve. problems with sustained attention, processing
• Cross innervation: It is useful when there is motor nerve speed, and executive functioning.
deficit due to a lesion in the course of the nerve. In this • Alexithymia, a deficiency in identifying,
repair, a nerve is grafted to connect the affected nerve understanding, processing, and describing emotions
to the normal functional nerve on the other side of the occurs in 60.9% of individuals with traumatic brain
body using microsurgical repair. injury.
• Nerve grafts: It is use of a nerve segment from one part • Cognitive and social deficits have long-term
of the body to reconstruct and repair an affected nerve consequences for the daily lives of people with
in some other part using microsurgical technique. moderate-to-severe traumatic brain injury, but can
• Glassgow coma scale gives clear idea about neuronal be improved with appropriate rehabilitation.
injury.
• When the pressure within the skull (intracranial
• Autonomic disturbances with bradycardia, systolic
hypertension, deep and slow respiration, Cheyne pressure) rises too high, it can be deadly. Signs of
stokes ventilation. increased intra cranial pressure include decreasing
• Cushing’s triad of raised intracranial pressure is level of consciousness, paralysis or weakness on
obvious i.e. bradycardia, hypertension and respiratory one side of the body, and a blown pupil, one that
irregularity. fails to constrict in response to light or is slow to
• Features such as restlessness, irritability, headache, do so.
vomiting and progressive deterioration are common. • Cushing’s triad, a slow heart rate with high blood
Q.10. Write briefly on clinical features of head injury. pressure and respiratory depression is a classic
 (Aug 2011, 5 Marks) manifestation of significantly raised intracranial
Ans. Following are the clinical features of head injury: pressure.
• Unequal pupil size is potentially a sign of a serious • Anisocoria, unequal pupil size, is another sign of
brain injury. serious traumatic bone injury.
• Symptoms are dependent on the type of traumatic • Abnormal posturing, a characteristic positioning
brain injury (diffuse or focal) and the part of the of the limbs caused by severe diffuse injury or high
brain that is affected. intra cranial pressure, is an ominous sign.
442   Mastering the BDS IIIrd Year  (Last 25 Years Solved Questions)

Q.11. Describe features to extradural hematoma. Chronic Inflammatory Demyelinating Polyneuropathy


 (Jan 2012, 5 Marks) ♦♦ Chronic inflammatory demyelinating polyneuropathy
Ans. Following are the features to extradural hematoma (CIDP) typically manifests as a symmetric demyelinat-
• Patient soon regain consciousness and again after 6 ing disease.
to 12 hour start deteriorating (Lucid interval). ♦♦ Both motor and sensory abnormalities are common, such
• Later the patient presents with confusion, irritability, as difficulty in walking, weakness, numbness, and pain or
drowsiness, hemiparesis on same side of the injury. tingling sensations.
Initially pupillary constriction and later pupillary ♦♦ CIDP is immune-mediated and occurs at increased fre-
dilatation occurs on the same side, finally becomes quency in patients with other immune disorders, such as
totally unconscious—Hutchinson Pupils systemic lupus erythematosus and HIV infection.
• Death can occur, if immediate surgical intervention ♦♦ CIDP follows a chronic, relapsing-remitting or progres-
is not done. sive course.
• Features of raised intracranial pressure such as high ♦♦ The peripheral nerves show segments of demyelination
blood pressure, bradycardia, vomiting is also seen. and remyelination.
Occasionally, convulsions may be present. ♦♦ In long-standing cases, chronically regenerating Schwann
• Wound and hematoma in the temporal region of cells may concentrically wrap around axons in multiple
scalp may be seen. layers in an onion-skin pattern.
• Glassgow coma scale gives clear idea about neuronal Toxic and Vasculitic Forms of Peripheral Neuropathy
injury.
♦♦ Drugs and environmental toxins that interfere with axonal
• Autonomic disturbances with bradycardia, systolic
transport or cytoskeletal function often produce peripheral
hypertension, deep and slow respiration, Cheyne-
neuropathies. The longest axons are most susceptible, so
Stokes ventilation. the resulting clinical presentation is often most pronounced
• Cushing’s triad of raised intracranial pressure in the distal extremities.
is obvious, i.e. bradycardia, hypertension and ♦♦ Peripheral nerves are often damaged in many different
respiratory irregularity. forms of systemic vasculitis including polyarteritis nodosa,
• Features such as restlessness, irritability, headache, Churg-Strauss syndrome, and Wegener granulomatosis.
vomiting and progressive deterioration are common. Overall, peripheral nerve damage is seen in about a third
Q.12. Describe the types, pathology, clinical features and of all patients with vasculitis at the time of presentation.
management of peripheral nerve injuries. The most common clinical picture is that of mononeuritis
 (June 2014, 10 Marks) multiplex with a painful asymmetric mixed sensory and
Ans. For types, refer to classification part of Ans 7 of same motor peripheral neuropathy. Patchy involvement also is
chapter. apparent at the microscopic level, as single nerves may
For clinical features and management, refer to Ans 9 of show considerable interfascicular variation in the degree
same chapter. of axonal damage.
Leprosy
Pathology
♦♦ There is peripheral nerve involvement in both lepromatous
and tuberculoid leprosy.
Guillain-Barré Syndrome
♦♦ In lepromatous leprosy, Schwann cells are often invaded
♦♦ Guillain-Barré syndrome is one of the most common life- by Mycobacterium leprae, which proliferate and eventually
threatening diseases of the peripheral nervous system. infect other cells. There is evidence of segmental
♦♦ It is a rapidly progressive acute demyelinating disorder demyelination and remyelination and loss of both
affecting motor axons that results in ascending weakness myelinated and unmyelinated axons. As the infection
that may lead to death from failure of respiratory muscles advances, endoneurial fibrosis and multilayered thickening
over a period of only several days. of the perineurial sheaths occur. Clinically, these patients
♦♦ It appears to be triggered by an infection or a vaccine that develop a symmetric polyneuropathy that prominently
breaks down self-tolerance, thereby leading to an autoi- involves pain fibers; the loss of sensation that results
mmune response. contributes to the tissue injury of the disease.
♦♦ Associated infectious agents include Campylobacter jejuni, ♦♦ Tuberculoid leprosy shows evidence of active cell-
Epstein-Barr virus, cytomegalovirus, and human immu- mediated immune response to M. leprae, with nodular
nodeficiency virus. granulomatous inflammation situated in the dermis.
♦♦ The injury is most extensive in the nerve roots and proxi- ♦♦ The inflammation injures cutaneous nerves in the vicinity;
mal nerve segments and is associated with mononuclear axons, Schwann cells, and myelin are lost, and there is fibrosis
cell infiltrates rich in macrophages. of the perineurium and endoneurium. With this form of
♦♦ Both humoral and cellular immune responses are believed leprosy, patients have much more localized nerve involvement
to play a role in the disease process. but do develop areas of abnormal sensation from the injury.
Section 2:  General Surgery  443

Diphtheria ♦♦ VII Facial (Taste, tears, saliva, facial expressions)


♦♦ Peripheral nerve involvement results from the effects ♦♦ VIII Vestibulocochlear (Auditory)
of the diphtheria exotoxin and begins with paresthesias ♦♦ IX Glossopharyngeal (Swallowing, saliva, taste)
and weakness; early loss of proprioception and vibratory ♦♦ X Vagus (Control of PNS, e.g. smooth muscles of GI tract)
sensation is common. ♦♦ XI Accessory (Moving head and shoulders, swallowing)
♦♦ The earliest changes are seen in the sensory ganglia, ♦♦ XII Hypoglossal (Tongue muscles—speech and swallowing)
where the incomplete blood-nerve barrier allows entry Q.15. Describe differentiating features of Bell’s palsy and
of the toxin. trigeminal neuralgia. (Jan 2017, 3 Marks)
♦♦ There is selective demyelination of axons that extends into Ans.
adjacent anterior and posterior roots as well as into the
Bell’s palsy Trigeminal neuralgia
mixed sensorimotor nerve.
It is also known as idiopathic It is also known as Tic Douloureux,
Varicella-Zoster Virus facial paralysis Trifacial neuralgia or Fothergill’s
disease
♦♦ This virus is one of the few that produce lesions in the
peripheral nervous system. It affects facial nerve It affects trigeminal nerve
♦♦ Latent infection of neurons in the sensory ganglia of It is caused due to cold, trauma, It is caused due to various
the spinal cord and brain stem follows chickenpox, and during surgical procedures, factors such as dental pathosis,
reactivation leads to a painful, vesicular skin eruption in ischemia, familial, tumors and excessive traction, allergies,
due to facial canal and middle mechanical factors, anomalies
the distribution of sensory dermatomes (shingles), most
ear neoplasms of superior cerebellar artery and
frequently thoracic or trigeminal. due to secondary lesions
♦♦ The virus may be transported along the sensory nerves
It begins abruptly as paralysis of It occurs more commonly over
to the skin, where it establishes an active infection of epi-
facial musculature, unilaterally. the right side and lower portion of
dermal cells. In a small proportion of patients, weakness face is more commonly affected.
is also apparent in the same distribution. Although the
Pain is present over side of face P a i n i s p r e s e n t w h i c h i s
factors giving rise to reactivation are not fully understood,
which is involved. Intensity of paroxysmal and last for few
decreased cell-mediated immunity is of major importance pain is low. seconds to few minutes and is of
in many cases. extreme intensity.
♦♦ Affected ganglia show neuronal destruction and loss,
Pain is particularly present within Pain is confined to trigeminal
usually accompanied by abundant mononuclear inflam- the ear, temple, mastoid area and zone.
matory infiltrates. Regional necrosis with hemorrhage angle of jaw.
may also be found.
Trigger zones are absent. Trigger zones are present which
♦♦ Peripheral nerve shows axonal degeneration after the death precipitate an attack when
of the sensory neurons. Focal destruction of the large mo- touched. Trigger zones are at
tor neurons of the anterior horns or cranial nerve motor vermilion border of lip, ala of
nuclei may be seen at the corresponding levels. Intranu- nose, cheeks, around eyes
clear inclusions generally are not found in the peripheral Eye of the affected side cannot be Eye of the affected side can be
nervous system. closed and wrinkles are absent closed easily and wrinkles are
on that side. Watering of eye is present on that side
Q.13. Write on type of nerve injuries with methods of repair
present as patient is unable to
of cut nerves. (Apr 2015, 7 Marks) close his/her eye.
Ans. For types of nerve injuries refer to Ans 7 of same chapter.
Due to muscular paralysis, there No drooling of saliva present.
and for methods of repair of cut nerves, refer to Ans 9 of is dropping of corner of mouth,
same chapter. from which saliva may dribble.
Q.14 Enumerate the cranial nerves. (Jan 2017, 5 Marks) Patient save his face easily, brush Patient avoid shaping, avoid
easily. brushing of teeth and undergo
Ans. Following are the cranial nerves: indiscriminate dental extraction
♦♦ I Olfactory (Smell)
♦♦ II Optic (Sight) Q.16. Enumerate branches of facial nerve.
♦♦ III Oculomotor (Moves eyelid and eyeball and adjusts the  (June 2018, 3 Marks)
pupil and lens of the eye) Ans. Following are the branches of facial nerve:
♦♦ IV Trochlear (Moves eyeballs) ♦♦ Within the facial canal
♦♦ V Trigeminal (Facial muscles including chewing; Facial • Greater petrosal nerve
sensations) • Nerve to stapedius
♦♦ VI Abducens (Moves eyeballs) • Chorda tympani nerve
444   Mastering the BDS IIIrd Year  (Last 25 Years Solved Questions)

♦♦ At exit from stylomastoid foramen


• Posterior auricular
• Digastric
• Stylohyoid
♦♦ Terminal branches within the parotid gland
• Temporal
• Zygomatic
• Buccal
• Marginal mandibular
• Cervical
♦♦ Communicating branches with adjacent cranial and
spinal nerves

23. Fractures of Bone


Q.1. Describe different types of mandibular fractures their
clinical presentation and treatment. Fig. 36:  Different types of mandibular fractures
 (Jan 2012, 10 Marks)
Clinical Presentation/ Clinical Features
Or
Describe various types of mandibular fractures, their Fracture at Angle
clinical presentation and treatment. ♦♦ Swelling at the angle externally and there may be obvious
 (Mar 2016, 7 Marks) deformity.
Or ♦♦ Laceration of skin or mucosa.
♦♦ Step deformity behind the last molar tooth may be vis-
Classify fractured mandible and discuss clinical fea­ ible which is more apparent, if no teeth are present in the
tures and management of fracture mandible. molar region.
 (July 2016, 10 Marks) ♦♦ Undisplaced fractures are usually revealed by the presence
Or of a small hematoma adjacent to the angle on either the
Write short answer on types of fractures mandible with lingual or buccal side, or both.
diagram  (June 2018, 3 Marks) ♦♦ Anesthesia or paraesthesia of the lower lip may present
Ans. on the side of the fracture.
♦♦ Inability to close the jaw causing premature dental contact.
Types of Mandibular Fracture ♦♦ Occlusion is often deranged. Movements of the mandible
are painful and range of movements are reduced.
Dingman and Natvig Anatomic Classification
♦♦ Trismus to some degree is usually present.
♦♦ Midline: Fractures between central incisors. ♦♦ Anterior open bite is seen in bilateral angle fracture.
♦♦ Symphysis: Fractures occurring within the area of sym- ♦♦ Ipsilateral open bite is seen in unilateral angle fracture.
physis.
♦♦ Parasymphysis: Bounded by vertical lines distal to canine Fracture of Body
teeth. ♦♦ Physical signs and symptoms such as swelling and bone
♦♦ Body: From distal symphysis to a line coinciding with the tenderness similar to that as seen in fracture of angle of
alveolar border of the masseter muscle usually including mandible.
third molar. ♦♦ Even slight displacement of the fracture causes derange-
♦♦ Angle: Triangular region bounded by the anterior border ment of the occlusion.
of masseter muscle to posterosuperior attachment of the ♦♦ Premature contact occurs on the distal fragment because
masseter muscle. of the displacing action of muscles attached to the ramus.
♦♦ Ramus: Bounded by superior aspect of the angle to two ♦♦ Fractures between adjacent teeth tend to cause gingival tears.
lines forming an apex at the sigmoid notch. ♦♦ When there is gross displacement, inferior dental artery
♦♦ Condylar process: Area of condylar process superior to the may be torn and this can give rise to severe intraoral
ramus region. hemorrhage.
♦♦ Coronoid process: Includes coronoid process of the mandible ♦♦ Sublingual hematoma or ecchymosis in floor of mouth.
superior to ramus region. ♦♦ Flattened appearance of lateral aspect of face.
♦♦ Dentoalveolar process: Region that would normally contain ♦♦ Inability to open or close the jaw.
the teeth ♦♦ Crepitation on palpation.
Section 2:  General Surgery  445

Fracture of Symphysis and Parasymphysis Write briefly on fracture of mandible


♦♦ These fractures are commonly associated with fractures (Aug 2011, 5 Marks)
of one or both the condyles. Or
♦♦ This fracture may be missed, if occlusion is undisturbed Write short note on types of mandibular fractures with
locally. diagrams (June 2010, 5 Marks)
♦♦ Presence of bone tenderness and a small lingual hematoma Ans. Refer to Ans 1 of same chapter.
may be the only physical signs. Q.3. Discuss in brief management of a case of fracture of
♦♦ Sublingual hematoma or ecchymosis in floor of mouth. mandible. (Sep 2009, 5 Marks) (Mar 2000, 15 Marks)
♦♦ Posterior open bite or unilateral open bite is seen in Or
parasymphysis fracture. Posterior crossbite can result from Discuss management of fracture of mandible.
midline symphysis fractures. (Jan 2011, 10 Marks) (Feb/Mar 2004, 10 Marks)
♦♦ Crepitation on palpation is noted in symphyseal fracture. Or
♦♦ An inability to close the jaw causing premature dental Write brief answer on management of fractured man­
contact. dible. (Apr 2017, 7, 5 Marks)
♦♦ A retruded chin can be caused by bilateral parasymphy- Ans.
seal fracture.
♦♦ Fracture line is often oblique which allows over-riding of Management of Mandibular Fracture
the fragments with lingual inversion of the occlusion on ♦♦ Reduction
each side. • Open reduction
Fracture of Ramus • Closed reduction.
♦♦ Fixation.
♦♦ They are uncommon. ♦♦ Immobilization.
♦♦ Flattened appearance of the lateral aspect of face. ♦♦ Prevention of infection and rehabilitation.
♦♦ Inability to open or close the jaw.
♦♦ Swelling and ecchymosis usually noted both extraorally Reduction of Mandibular Fracture
and intraorally.
Reduction of fracture is the process to bring the fractured
♦♦ Tenderness over the ramus and movements produce pain
fragments into alignment.
over the same area.
♦♦ Severe trismus is present. Reduction is of two types, i.e. closed and open reduction.

Fracture of Coronoid Process Closed Reduction

♦♦ The fracture can be caused by direct trauma to the ramus In closed reduction method the fractured fragments are brought
but is rarely in isolation. It is usually considered to result into an alignment without actually exposing the fractured bone
from reflex contracture of the powerful anterior fibers of ends, occlusion is the key to reduction.
the temporalis muscle.
Reduction by Manipulation
♦♦ This fracture is difficult to diagnose clinically.
♦♦ Tenderness over anterior part of the ramus. It is done in fresh cases of fracture where displacement of the
♦♦ Painful limitation of movement, especially during protru- fractured fragments is due to muscle spasm:
sion of the mandible may be found. ♦♦ Carried out under local anesthesia.
♦♦ After anesthesia is given, the dislocated fragments of the
Treatment
mandible are held between the finger and thumbs on each
Refer to Ans 3 of same chapter. side of the fractured line.
♦♦ It depends upon the line of fracture. ♦♦ The fragments are shaken up and down to disengage them
♦♦ Oblique fracture shows bad prognosis. or to break the fibrous union.
♦♦ Spiral fracture shows good prognosis due to increased ♦♦ When normal occlusion is achieved the fragments are
surface area. fixed in position.
♦♦ Various methods used for close reduction of mandibular
Q.2. Write short note on fracture mandible. fracture are:
(Feb 2015, 5 Marks) (Sep 2005, 8 Marks) • Arch bar fixation
Or • Bridle wire
Write note on mandibular fracture. • Figure of 8 wiring
(Feb 1999, 10 Marks) • Gilmer’s direct wiring
Or • Eyelet wiring
Write short note on fracture of mandible. • Essig’s wiring
 (Mar 2006, 5 Marks) • Stout’s continuous loop wiring
Or • Risdon’s wiring.
446   Mastering the BDS IIIrd Year  (Last 25 Years Solved Questions)

Reduction by Traction ♦♦ A fixation device is given to stabilize the reduced frag-


ments into their normal anatomical position till clinical
Traction is a procedure by which the fractured dislocated
bony union takes place.
fragments are subjected to a continuous gradual pull by elastic
♦♦ For mandibular fracture, period of immobilization should
bands.
be 4 to 6 weeks.
Open Reduction ♦♦ If mandibular condyle is fractured, then period of immo-
bilization is for 2 to 3 weeks.
♦♦ It is a procedure by which we expose the fractured bone
ends and bring them into alignment under direct vision. Prevention of Infection and Rehabilitation
♦♦ The fractured bone can exposed by intraoral or extraoral ♦♦ Proper antibiotic regimen should be given to the patient to
approach. prevent intra-operative and post-operative complications.
♦♦ The extraoral approach is preferred to avoid contamina- ♦♦ If there is presence of gap between the bony ends, proper
tion. bone graft should be given.
♦♦ As depending on the areas to accessed various surgical ♦♦ After 4 to 6 weeks as fracture is healed, jaw regain some
approaches are used in open reduction. of its strength, patient should be encouraged to be on a
♦♦ Symphysis, parasymphysis and body of the mandible normal diet and use the jaw as before he/she use it prior
can be approached via intraoral incisions given in labial to the trauma.
mucosa. Extra-oral incision can be used to approach these Q.4. Describe briefly cause of malunion of fracture of
regions. Symphysis can be approached via an incision bone. (Mar 1998, 5 Marks)
placed in submental region in skin crease. Body of man-
Ans. The malunion means union of fragments in a defective
dible can be approached by sub-mandibular approach.
position.
♦♦ Angle of mandible should be approached via intra-oral
The most common deformity is angulations, beside
incision placed on third molar region which extends to
this there may be overlapping with shortening and
anterior border of ramus. Extraorally the angle of mandible
malrotation.
can be approached via Risdon’s incision.
♦♦ Ramus of mandible can be approached via retromandibu- The causes of malunion are:
lar incision. • Fracture was not reduced properly.
♦♦ Coronoid process can be approached via intra – oral inci- • After reduction replacement occurs within the
sion placed high on anterior border of ramus of mandible. plaster for this a check X-ray after a week is advisable
♦♦ Condyle of the mandible can be approached via various in certain fractures anticipating redisplacement, e.g.
incisions, i.e. preauricular, retromandibular and subman- Fracture of both bones of the forearm.
dibular, etc. • Growth disturbance due to injury to the epiphyseal
♦♦ Aligned bone is held in position with the help of wire, cartilage may lead to malunion; fracture separation
screws or bone plates as per requirement. of an epiphysis does not lead to growth disturbance
as the fracture occurs through the metaphyseal plate
Fixation of Mandibular Fracture keeping the epiphyseal cartilage intact.
♦♦ In this phase, the fractured fragments are fixed, in their  Site of malunion are those where the bone is
normal anatomical relationship to prevent displacement cancellous so union occurs as a rule, but malunion
and achieve proper approximate fixation. complicates due to imperfect position of the
♦♦ If a closed reduction is done, the fractured fragments are bone ends. These sites are fracture neck and the
fixed and immobilized in anatomically reduced position supracondylar fracture of humerus, collars fracture,
by means of wire which are placed around teeth, i.e. inter- fracture through the, condyles of the tibia, etc.
dental wiring. Various wiring techniques are used and each Q.5. Describe briefly General treatment of fracture of
of them are used in various types of mandibular fractures. bone. (Dec 2010, 7 Marks)
Wiring techniques used are Bridle wire, Figure-of-8 wiring, Ans. General Management of Fracture of Bone
Gilmer’s direct wiring, eyelet wiring, Essig’s wiring, stout’s • Early treatment (Neurovascular problems)
continuous loop wiring and Risdon’s wiring. • The principle of management of fracture is to deal
♦♦ If an open reduction is done the fixation can be: with life-saving problems first
• Non-rigid fixation: By use of transosseous wires or wire • This means paying attention to ABC (airway, breathing
osteosynthesis and circulation) and to the neurovascular status
• Semi-rigid fixation: By use of monocortical miniplates For reduction, immobilization and rehabilitation, refer
or lag screws to Ans 8 of same chapter.
• Rigid fixation: By use of dynamic compression plates
Q.6. Describe briefly causes of delayed union of fracture of
Immobilization bone. (Sep 2002, 5 Marks)
♦♦ In this phase, the reduced and fixed fragments of bone are Ans. When the procedure of healing a fracture is slower than
immobilized for certain period for healing to occur. the normal speed, then it is called delayed union.
Section 2:  General Surgery  447

Causes • Spiral fracture shows good prognosis due to increased


♦♦ Severe initial injury: This is most probably due to associated surface area.
soft tissue damage which allows diffusion of hematoma
General Principles of Treatment of Fracture
and also due to the blood supply to the bone fragments.
♦♦ Infection of fracture hematoma: Healing fails because of the The three fundamental principles of fracture treatment—
cellular element which is required for the production of reduction, immobilization and preservation of function, i.e.
bone at the production of pus. rehabilitation.
♦♦ Soft tissue interposition between the fracture fragments
may so separate them that it is physically impossible for
them to unite.
♦♦ A poor blood supply at the fracture site.
♦♦ Inadequate immobilization: Excessive movement at the frac-
ture site during the healing phase may produce delayed
union because the fracture site is constantly ‘refractured’
by the movement.
♦♦ Compound fracture (when more than two segments) or
open fracture.
Q.7. Describe different types of fracture of mandible and
their management.  Figs 37A to D:  Types of fracture; (A) Transverse fracture;
 (Mar 2007, 10 Marks) (Feb 2002, 15 Marks) (B) Oblique fracture; (C) Spiral fracture; (D) Comminuted fracture
Ans. For different types of fracture of mandible refer to ans 1
and for management, refer to Ans 3 of same chapter. Reduction

Q.8. Discuss the classification, general principles of This first principle must be qualified by the words ‘if necessary’.
treatment and healing of fracture. In many fractures reduction is unnecessary, either because there
 (Feb/Mar 2004, 15 Marks) is no displacement or because the displacement is immaterial
to the final result.
Or
If it is judged that perfect function can be restored without

Define and classify fractures. Discuss general princi­ undue loss of time, despite some uncorrected displacement of
ples of management of fractures. (Sep 2008, 8 Marks) the fragments, there is clearly no object in striving for perfect
Or anatomical reduction. Indeed, meddlesome intervention may
Describe fracture and give principles of fracture sometimes be detrimental, especially if it entails open operation.
management and healing. (May/Jun 2009, 15 Marks) Methods of Reduction
Or When reduction is decided upon it may be carried out in three
Describe fracture. Give principles of fracture ways:
management and healing. (June 2018, 5 Marks) ♦♦ By closed manipulation
Ans. Fracture is defined as break in the continuity of lamellar ♦♦ By mechanical traction with or without manipulation
pattern of bone. ♦♦ By open operation.

Classification of Fracture of Bone Manipulative Reduction


♦♦ Closed or simple fracture: When the outer skin is not injured Closed manipulation is the standard initial method of reducing
due to fracture. Fracture side dose not communicate to the most common fractures. It is usually carried out under general
outer surface. anesthesia, but local or regional anesthesia is sometimes
♦♦ Open or compound fracture: In this type, outer tissue also appropriate. The technique is simply to grasp the fragments
broken down and fracture site communicate with outer through the soft tissues, to disimpact them if necessary, and
surface. then to adjust them as nearly as possible to their correct position.
♦♦ Comminuted fracture: When the bone is broken down into
more than two segments. It show bad prognosis. Reduction by Mechanical Traction
♦♦ Impacted fracture: When one fragment enters into another When the contraction of large muscles exerts a strong displacing
segment. force, some mechanical aid may be necessary to draw the
♦♦ Green stick fracture: Only the bone is bent-like a green stick. fragments out to the normal length of the bone. This particularly
Fracture is incomplete and occurs in children. applies to fractures of the shaft of the femur, and to certain
♦♦ Transverse, oblique and spiral fracture. types of fracture or displacement of the cervical spine. Traction
• It depends upon the line of fracture. may be applied either by weights or by a screw device, and the
• Oblique fracture shows bad prognosis. aim may be to gain full reduction rapidly at one sitting with
448   Mastering the BDS IIIrd Year  (Last 25 Years Solved Questions)

anesthesia, or to rely upon gradual reduction by prolonged strips of aluminum, from wire, or from heat-moldable plastic
traction without anesthesia. materials such as polyethylene foam. Rarely, a halo-thoracic
splint is used for an unstable fracture of the cervical spine. This
Operative Reduction consists of a metal ‘halo’ or ring that is screwed to the skull and
When an acceptable reduction cannot be obtained, or joined by bars to a plaster or plastic splint enclosing the chest.
maintained, by these conservative methods, the fragments are Cast bracing (functional bracing): A brace has come to be
reduced under direct vision at open operation. Open reduction understood as a supportive device that allows continued
may also be required for some fractures involving articular function of the part. Cast bracing, or functional fracture bracing
surfaces, or when the fracture is complicated by damage to (to use a better term), is a technique in which a fractured
a nerve or artery. When operative reduction is resorted to, long bone is supported externally by plaster of Paris or by a
the opportunity should always be taken to fix the fragments mouldable plastic material in such a way that function of the
internally to ensure that the position is maintained. adjacent joints is preserved and use of the limb for its normal
purposes can be resumed. Functional bracing is used mainly
Immobilization for fractures of the shaft of femur or tibia.
Like reduction, this second great principle of fracture treatment
must be qualified by the words ‘if necessary’. Whereas some Immobilization by Sustained Traction
fractures must be splinted rigidly, many do not require In some fractures—notably, those of the shaft of the femur and
immobilization to ensure union, and excessive immobilisation certain fractures of the shaft of the tibia or of the distal shaft
is actually harmful in some. of the humerus—it may be difficult or impossible to hold the
fragments in proper position by a plaster or external splint
Indications for Immobilization alone. This is particularly so when the plane of the fracture is
There are only three reasons for immobilizing a fracture: oblique or spiral, because the elastic pull of the muscles, then
1. To prevent displacement or angulation of the fragments tends to draw the distal fragment proximally so that it overlaps
2. To prevent movement that might interfere with union the proximal fragment. In such a case, the pull of the muscles
3. To relieve pain. must be balanced by sustained traction upon the distal fragment,
If in a given fracture, none of these indications applies, then either by a weight or by some other mechanical device.
there is no need for immobilization. Immobilisation by External Fixation
Methods of Immobilization Strictly, immobilization in plaster or in a splint might be
When immobilization is deemed necessary, there are four regarded as external fixation. By convention, however, the
methods by which it may be effected: term external fixation is used to imply anchorage of the bone
fragments to an external device such as a metal bar through the
♦♦ By a plaster of Paris cast or other external splint
medium of pins inserted into the proximal and distal fragments
♦♦ By continuous traction
of a long bone fracture. In its simplest form, external fixation
♦♦ By external fixation
may be provided by transfixing each fragment with a Steinmann
♦♦ By internal fixation.
pin and incorporating the protruding ends of the pins in a
Immobilization by plaster, splint or brace. plaster of Paris splint. This simple method is now seldom used,
For most fractures, the standard method of immobilization is and fixation is now by means of rigid bars or a frame.
by a plaster of Paris cast.
Methods of internal fixation: The following methods are
currently in general use:
Plaster Technique
♦♦ Metal plate held by screws or locking plate (with screws
The plaster bandages are applied in two forms: round-and- fixed to the plate by threaded holes)
round bandages and longitudinal strips or ‘slabs’ to reinforce ♦♦ Intramedullary nail, with or without cross-screw fixation
a particular area. Round-and-round bandages must be applied for locking
smoothly without tension, the material being drawn out to ♦♦ Dynamic compression screw-plate
its full width at each turn. Slabs are prepared by unrolling a ♦♦ Condylar screw-plate
bandage to and fro upon a table: an average slab consists of ♦♦ Tension band wiring
about 12 thicknesses. ♦♦ Transfixion screws.
The slabs are placed at points of weakness or stress and are The choice of method depends upon the site and pattern of
held in place by further turns of plaster bandage. A plaster is the fracture.
best dried simply by exposure to the air: artificial heating is Plate and screws: This method is applicable to long bones.
unnecessary. Usually, a single six-hole plate suffices, but an eight-hole plate
Other external splints: Apart from plaster of Paris, splints that may be preferred for larger bones.
are in general use are mostly those for the thigh and leg and for Locking plate: A newer concept is the ‘locking plate’, that uses
the fingers. Individual splints may also be made from malleable screws with heads that are threaded and when tightened lock into
Section 2:  General Surgery  449

matching threads in the holes of the plate. This produces a more Active Exercises
rigid fixation in terms of length and angle, which is particularly These comprise exercises for the muscles and joints. They should
valuable in comminuted fractures in osteoporotic bone. It can be encouraged from an early stage. While a limb is immobilized
also be inserted with less stripping of soft tissue that preserves in a plaster or splint, exercises must be directed mainly to
bone vascularity, particularly in the metaphyseal region. the preservation of muscle function by static contractions.
Intramedullary nail: This technique is excellent for many The ability to contract a muscle without moving a joint is
fractures of the long bones, especially when the fracture is near soon acquired under proper supervision. When restrictive
the middle of the shaft. It is used regularly for fractures of the splints are no longer required, exercises should be directed to
femur and tibia, and less commonly in the humerus. mobilizing the joints and building up the power of the muscles.
Compression screw-plate: The compression screw-plate Finally, when the fracture is soundly united, treatment may be
(dynamic hip screw) is a standard method of fixation for intensified, movements being carried out against gradually
fractures of the neck of the femur and for trochanteric fractures. increased resistance until normal power is regained.
The screw component, which grips the femoral head, slides Healing of Fracture
telescopically in the barrel to allow the bone fragments to be
Healing of fracture takes place in three steps:
compressed together across the fracture. This compression effect
is brought about by tightening a screw in the base of the barrel. ♦♦ Hematoma and granulation tissue formation
• After fracture a hematoma is formed due to rupture
Transfixion screws: The use of a transfixion screw has wide of blood vessels
application in the fixation of small detached fragments—for • Inflammatory changes start at the site of fracture
instance the capitulum of the humerus, the olecranon process • Hematoma is gradually resorbed
of the ulna or the medial malleolus of the tibia. • The necrotic debris is removed by neutrophils and
Kirschner wire fixation: These thin flexible wires with macrophages
sharpened ends are available in a number of diameters and • Collagen fibers are laid down at the site of union of
provide a useful alternative to transfixion screws for the fixation fracture
of small bony fragments or for fractures of the small bones in • Capillaries and collagen fiber form granulation tissue
the hand and foot. which covers the fracture site, it takes about 15 days
Tension band wiring: This technique of fixation is most • Bone ends are now united at fracture site, by weak
commonly used in the patella and olecranon, but can be applied fibrous band.
to other small metaphyseal fragments such as the medial ♦♦ Callous formation
malleolus. It uses the mechanical principle of converting the • Calcification of newly formed fibrous tissue takes
tensile stresses of the muscles acting on the bone fragment, into place after 3 weeks
a compressive force at the fracture site. • Matrix becomes ossified and is called ‘Callous’
• Callous is distributed along the fractured site
Rehabilitation • Callous provides a firm and rigid bridge at the fracture
Improved results in the treatment of fractures owe much to segments.
rehabilitation, perhaps the most important of the three great ♦♦ Mature bone formation
principles of fracture treatment. Reduction is often unnecessary; • Callus is now replaced by a lamellar bone. The
immobilization is often unnecessary; rehabilitation is always bone undergoes remodeling and comes in normal
essential. Rehabilitation should begin as soon as the fracture morphology
comes under definitive treatment. Its purpose is twofold: first, to • Haversian system develops in bone
preserve function so far as possible while the fracture is uniting and • Bony union takes place in about 2 to 3 months.
second, to restore function to normal when the fracture is united. Q.9. How will you manage a case of ankylosis of TMJ? 
This purpose is achieved not so much by any passive treatment  (Sep 2000, 15 Marks)
as by encouraging patients to help themselves. The two essential Ans. Ankylosis is a Greek term means “stiff joint”.
methods of rehabilitation are active use and active exercises. • TMJ ankylosis: There is immobility of joint, the jaw
Active Use function get affected. Hypomobility to immobility
of the joint can lead to inability to open the mouth
This implies that the patient must continue to use the injured
from partial to complete
part as naturally as possible within the limitations imposed
• Two main factors predisposing to the ankylosis are
by necessary treatment. The degree of function that can be
trauma and infection, in or around the joint region.
retained depends upon the nature of the fracture, the risk of
redisplacement of the fragments, and the extent of any necessary Management of TMJ Ankylosis
splintage. Although in some injuries rest may be necessary in The treatment of TMJ ankylosis is always surgical.
the early days or weeks, there should be a graduated return to The internationally accepted protocol for the management
activity as soon as it can be allowed without risk. of TMJ ankylosis
450   Mastering the BDS IIIrd Year  (Last 25 Years Solved Questions)

♦♦ Early surface intervention ♦♦ Now two horizontal bony cuts are given in superior aspect
♦♦ Aggressive resection: A gap of at least 1 to 1.5 cm should of ramus and the wedge of bone between the two cuts is
be created. removed. Take care while removing the bone from medial
♦♦ Ipsilateral coronoidectomy and temporalis myotomy. aspect as it is close to maxillary artery and carotid canal.
♦♦ Contralateral coronoidectomy and temporalis myotomy. Remove the bone by using large round bur till medial
♦♦ Lining of the glenoid fossa region with temporalis fascia. bone gets thinned out completely to remove by osteotome.
♦♦ Reconstruction of the ramus with costochondral graft. ♦♦ There is recommendation of leaving gap of about 1 to 1.5
♦♦ Early mobilization and aggression physiotherapy for at cm laterally and medially for preventing the reankylosis.
least 6 month postoperatively.
♦♦ Regular long-term follow-up. Interpositional Arthroplasty
♦♦ To carry out cosmetic surgery at the later date when the
♦♦ In cases of gap arthroplasty chances of reankylosis are
growth of the patient is complete.
present, if bony cuts come in contact.
Principles of Management ♦♦ So to avoid this an interpositional material is inserted in
between the two cut ends which avoid contact between
♦♦ Removal of the ankylosed mass of bone to mobile the jaw.
them and decreases the chances of reankylosis.
♦♦ Reconstruction of joint and maintenance of vertical height
♦♦ Various materials are used which can by autogenous or
of ramus.
alloplastic.
♦♦ Prevention of recurrence.
♦♦ Autogenous materials used for interpositioning are carti-
♦♦ Restoration of occlusion and maintain function.
laginous graft, temporal muscle, temporal fascia, fascia,
♦♦ Correction of secondary facial deformity.
dermis while alloplastic materials used are metallic and
Step-by-Step Treatment of Ankylosis non-metallic. In metallic ones there are tantalum plate,
stainless steel, titanium, gold and in non-metallic ones
♦♦ Use of brisement force: Forced opening of the jaw by mouth there are Teflon, acrylic, ceramic and elastic.
gag under general anesthesia. It is used in case of fibrous
ankylosis. Q.10. Write on management of maxillofacial injuries.
♦♦ Surgical lysis: Opening of joint by preauricular approach  (Dec 2009, 15 Marks)
and destroy union of condyle and glenoid fossa. Ans. Management of maxillofacial injuries
Condylectomy
Prehospital Care
♦♦ This procedure is done in cases of fibrous or partial
ankylosis where the anatomical features of joint are not ♦♦ General airway: Administer oxygen and maintain a patent
completely changed. airway. Maintain an immobilized cervical spine at all times.
♦♦ Condylectomy procedure should be started by giving the Clear the mouth of any foreign body or debris, and suction
preauricular incision. any blood present.
♦♦ Now a horizontal osteotomy cut is given by the help of ♦♦ Intubation: Intubate, if indicated. Have the
bur at the level of condylar neck. cricothyroidotomy and tracheotomy tray set up prior
♦♦ Section the head of the condyle till the level of neck and to an initial attempt at intubation. Consider conscious
separate it from superior attachment. sedation intubation, if distortions of the mandible and
♦♦ Stump of the condyle at neck is smoothened and wound maxilla exist because a tight seal with the mask may not be
is closed in layers. possible when bagging. Consider nasotracheal intubation,
♦♦ As unilateral condyle leads to deviation of mandible at if massive oropharyngeal edema is present. Consider
operated side on mouth opening and in cases of bilateral orotracheal intubation, if midface or upper face trauma is
present. If unable to intubate the patient nasotracheally or
ankylosis open bite is present due to loss of vertical rami.
endotracheally, cricothyroidotomy is the next procedure
So when condylectomy is done, after recontouring an
of choice
alloplastic material is used for maintain space, provide
♦♦ Breathing: Assess breath sounds. Check tube placement.
proper occlusion as well as joint movements.
♦♦ Circulation: Do not remove impaled foreign bodies that
Gap Arthroplasty can result in worsening of damage and bleeding. Control
hemorrhage with direct pressure. Obtain large-bore intra-
♦♦ Gap arthroplasty is the procedure which involves the
venous access bilaterally.
creation of an anatomical gap in an ankylosed segment to
♦♦ Disability: Assess the patient using the Glasgow coma scale.
form artificial joint space.
Perform a brief neurologic examination. Note any change
♦♦ In patients with complete bony ankylosis anatomical in mental status.
features of joint are very difficult to appreciate as they get ♦♦ Exposure: Expose patients, but keep them warm. Remove
covered by the bone. So in complete bony ankylosis cases a all clothing and accessories. Recover all avulsed hard and
gap in the bone is made for separating ramus of mandible soft tissue, and transport them in damp gauze with no ice
from ankylosed mass in glenoid fossa. and very little manual manipulation.
Section 2:  General Surgery  451

Medical and Surgical Therapy lateral canthotomy. The subciliary approach has most
complications (e.g., ectropion) and the transconjunctival
♦♦ General medical therapy: Administer oxygen and, isotonic crys- approach the least complications.
talloid fluids. Administer packed red blood cells if the patient ♦♦ However, when major surgical exposure is necessary, a
is bleeding excessively. Tetanus prophylaxis is indicated. transconjunctival approach with or without a lateral can-
♦♦ Antibiotics: For facial lacerations, use cefazolin (Ancef, thotomy incision is recommended.
Kefzol). For oral cavity lacerations, use clindamycin or ♦♦ Orbital floor repair via subtarsal approach.
penicillin. For fractures communicating with the sinus,
use amoxicillin. For fractures with dural tears or CSF leaks, Nasal Fractures
use vancomycin and a third-generation cephalosporin. ♦♦ Nasal fractures should be managed between days 2–10.
♦♦ Pain management: Use oral medications for minor injuries This allows time for resolution of the edema and therefore
and parenteral medications if the patient cannot take oral assists in obtaining the best reduction possible.
medications (i.e., nothing by mouth [NPO]). For anti- ♦♦ After 10 days, achieving good closed reduction results may
inflammatory control, use ibuprofen, naproxen, or be difficult and it may be necessary to wait for as long as
ketorolac (Toradol). For central control, use narcotics 6 months to obtain satisfactory good results via an open
(e.g., codeine, oxycodone, hydrocodone, meperidine, reduction technique.
morphine).
Nasoethmoidal (NOE) Fractures
Frontal Bone Fractures
♦♦ Fractures with suspected or detected dural tears require
♦♦ Of great concern is the patency of the nasofrontal duct. If consultation with a neurosurgeon, and the patients should
this duct is blocked, surgery is indicated. be admitted for observation and intravenous antibiotics.
♦♦ Blockage may result in mucopyocele or abscess. ♦♦ An ophthalmologist should be consulted for repair of the
♦♦ Non-displaced anterior sinus wall fractures are treated lacrimal apparatus, if disrupted.
by observation. ♦♦ An oral and maxillofacial surgeon, plastic surgeon, or
♦♦ Displaced anterior sinus wall fractures with severe otolaryngologist should be consulted for repair of nasal
comminution and mucosal injury require neurosurgery, bones, medial canthus, and the nasofrontal duct.
oral and maxillofacial surgery, otolaryngology, or plastic
surgery for bone grafting and frontal sinus obliteration. Zygomatic Arch Fractures
♦♦ Treatment of posterior sinus wall fractures is controversial ♦♦ Patients with isolated minimally displaced fractures to the
and variable. zygomatic arch usually do not require treatment, unless it
♦♦ Posterior sinus wall fractures are examined for displace- caused a facial asymmetry.
ment, dural tears, and cerebrospinal fluid leakage. ♦♦ Marked displacement and/or impingement of the coronoid
♦♦ Non-displaced fractures with a cerebrospinal fluid leak process of the mandible, preventing the patient from open-
may be observed for 5–7 days while undergoing treatment ing their mouth, requires admission and an open reduction
with intravenous antibiotics. Frontal sinus obliteration is via transoral (Keen) or temporal (Gillies) approach.
indicated, if a cerebrospinal fluid leak persists. ♦♦ In cases of a severe comminuted fracture, an open reduc-
♦♦ Surgical treatment of displaced fractures with no cerebro- tion with internal fixation (ORIF) may be required.
spinal fluid leak is based on the severity of comminution.
♦♦ Mild comminution requires an osteoblastic flap and sinus Zygomaticomaxillary Complex (ZMC) Fractures
obliteration. ♦♦ When the impact is sufficient to sustain a fracture of the
♦♦ Comminution of greater than 30% of the posterior sinus ZMC consultation with an ophthalmologist is warranted
wall require the neurosurgeon to remove the posterior to rule out ocular injury. Like the zygomatic arch fracture,
table allowing the brain to expand into the frontal sinus, surgical treatment of a ZMC fracture is indicated when a
this is known as cranialization. cosmetic deformity or functional loss is noted.
♦♦ Displaced sinus wall fractures with a cerebrospinal fluid ♦♦ Waiting 4–5 days for the edema to be reduced is helpful
leak and minimal-to-mild comminution requires sinus
to properly assess the situation.
obliteration.
♦♦ The standard of care is open reduction and internal fixation
♦♦ Moderate-to-severe comminution requires sinus crani-
with miniplates and screws. The orbital floor is frequently
alization.
explored and repaired, if necessary.
Orbital Floor Fractures
Maxillary Fractures
♦♦ Blow-out fractures of the orbital floor require consultation
with an ophthalmologist and maxillofacial trauma special- ♦♦ When the impact is severe enough to cause mobility of the
ist (e.g., oral and maxillofacial surgeon, otolaryngologist maxilla or to a part of it, the patient should be placed in
or plastic surgeon). intermaxillary fixation and open reduction with internal
♦♦ Several approaches are available including subciliary, fixation should be performed at the piriform rim and zy-
subtarsal, transconjunctival, and transconjunctival with gomaticomaxillary buttress.
452   Mastering the BDS IIIrd Year  (Last 25 Years Solved Questions)

♦♦ Patients with a maxillary fracture should be placed on sinus Classification


precautions, and if they have subcutaneous emphysema,
they should be placed on antibiotics because some of the ♦♦ Fracture of lower third which comprises of mandible
bacterial flora could have been forced by the air into the ♦♦ Fracture of middle third which comprises of maxilla,
zygoma and nose
subcutaneous planes.
♦♦ Fracture of upper third of face involving part of orbit and
Mandibular Fractures frontal bones.
♦♦ Management is provided by an oral and maxillofacial Soft Tissue Injuries
surgeon, otolaryngologist or plastic surgeon.
♦♦ Lacerations, contusion, cut wounds, etc.
♦♦ Temporary stabilization in the emergency department can
♦♦ Eyelid injuries with black eyes
be addressed with the application of a Barton bandage.
♦♦ Facial nerve injury: Primary repair is required
♦♦ Bring the teeth into occlusion and wrap the bandage
♦♦ Parotid duct injury: Primary Anastomosis of injured duct is
around the crown of the head and jaw. This stabilizes the
done, with fine polyethylene cannula kept as stent inside
jaw and greatly reduces pain and hemorrhage.
the duct which is removed in 14 days.
♦♦ A symphysis or body fracture can be reduced temporar-
♦♦ Lacrimal apparatus injuries: Here the duct is sutured with
ily with a bridal wire (a 24-gauge wire wrapped around
fine nylon thread in canaliculus which is kept for three
2 teeth on either side of the fracture). This greatly reduces
months.
hemorrhage, pain and infection.
♦♦ Nondisplaced mandibular fractures may be treated by Injuries to Facial Bones
closed reduction and intermaxillary fixation for 5–6 weeks.
♦♦ Fracture nose: Nasal bones are most commonly injured
However many patients do not want to be closed down for
bones in face. Patient presents with pain and swelling in
that length of time and prefer open reduction.
the nose with deviation and displacement. Here reduc-
♦♦ Initially, the fracture is stabilized with intermaxillary fixa-
tion of the fractured nasal bones and nasal septum under
tion followed by open reduction and rigid fixation using
general anesthesia is done. Later position is maintained by
titanium miniplates, mandibular plates, or reconstruction
nasal packs from inside (which is removed in 7 days) and
plates, depending on where the fracture is located.
by a nasal plaster from outside (which will be kept for 14
♦♦ Nondisplaced fractures of the condyle require intermaxil-
days). Procedure is done using Walsham and Asch forceps.
lary fixation for 10 days, followed by physiotherapy to help
♦♦ Injuries to the maxilla
restore improved function.
♦♦ Zygomatic bone injuries.
♦♦ Ankylosis of the joint is extremely rare and is believed
♦♦ Mandibular bone fracture and mandibular dislocation.
to be caused by an untreated intracapsular injury or
♦♦ Orbital bone fracture: Presents with diplopia, enophthal-
fracture.
mous, sensory loss in the area of infraorbital nerve.
Panfacial Fractures ♦♦ Infraorbital ecchymosis of the orbit is called Panda sign.

♦♦ At the time of surgery, tracheostomy or submandibular Clinical Features


intubation is required. ♦♦ Localized swelling due to hematoma
♦♦ A submandibular intubation, which avoids a tracheos- ♦♦ Facial edema
tomy, is performed by first intubating orally, and then ♦♦ Bleeding with open wounds
surgically bringing the tube out through the subman- ♦♦ Asymmetry which is clinically confirmed by observing
dibular space. supraorbital ridges, nasal bridge
♦♦ Nasoendotracheal intubation is definitely contraindicated. ♦♦ Localized tenderness
♦♦ Facial bones are repositioned beginning at the cranium. ♦♦ Step deformity
After the occlusion is established by intermaxillary fixa- ♦♦ Trismus
tion, the remaining facial bones are repaired with open ♦♦ Diplopia
reduction and internal fixation. ♦♦ Features of associated injuries such as intracranial, abdomi-
nal or thoracic injuries.
Q.11 Write short note on types of mandibular fracture with
diagrams. (Jun 2010, 5 Marks) Investigations
Ans. For types of mandibular fractures, refer to Ans 1 of same ♦♦ X-ray face
chapter. ♦♦ CT scan of head and jaw

Q.12 Write short note on maxillofacial injuries. Management


 (Dec 2010, 5 Marks) ♦♦ As the initial assessment, evaluation and management of
Ans. Maxillofacial injuries are due to road-traffic accidents, life-threatening injury get completed compound fracture
assaults, bullet injuries or sport injuries. should be treated in following manner, i.e.
Section 2:  General Surgery  453

• Hemostasis should be achieved. • Alteration of musculature:


• For Type I and Type II compound fractures cephazolin –– By closed condylectomy: Gigli saw is used to
or clindamycin are the choice of drugs while for Type intraorally bisect the condylar neck.
III compound fractures aminoglycoside is given. –– Ligation of coronoid process to zygomatic arch
• Tetanus vaccination should be given. anterior to articular tubercle
• Irrigation as well as debridement of the wound should • Alteration of bony structure:
be carried out immediately. –– By condylectomy
• In cases with Type II and Type III compound fractures –– Eminectomy, i.e. reduction of height of eminences.
serial irrigation and debridement is recommended Q.14. Describe etiology, pathology and treatment of acute
for every 24 to 48 hours till clean surgical wound is osteomyelitis. (Aug 2012, 15 Marks)
confirmed. Close the wound when it get clean fully. Ans. Osteomyelitis is the inflammation of medullary portion
• Management of open fracture depends on its type of bone.
and its site. Later on wound is stabilized temporarily
or definitively. Etiology
• If coverage of soft tissue after injury is not proper soft
♦♦ Direct spread of infection from dental pulp into the man-
tissue transfer or free flap is given to the patient when dible.
fracture is treated. ♦♦ Spread of infection in the mandible from presenting sup-
Q.13. Describe diagnostic features and treatment of tempo­ purative odontogenic infections.
romandibular joint dislocation. (Jan 2012, 5 Marks) ♦♦ Spread of infection following removable of tooth without
Ans. Temporomandibular joint dislocation occurs when proper asepsis and antibiotic coverage.
condyle is displaced forcefully anteriorly out of the ♦♦ Compound fracture of mandible with exposure of bone
articular fossa but lie in the capsule of joint. outside the mucosa.
♦♦ Postradiation secondary infection.
Diagnostic Features ♦♦ Infection to the pre-existing bony lesions, e.g. Paget’s
♦♦ Clinical diagnosis: Patient complains of difficulty in swal- disease of bone and fibrous dysplasia.
lowing. Saliva is seen drooling over chin. Severe pain is
Pathology
present in area over temporal fossa. The place where the
condylar head is normally placed, at that area depression
is seen. There is also presence of anterior open bite along
with gagging of molar teeth.
♦♦ Radiographical diagnosis: Due to dislocation of the condyle
articular fossa space appears to be empty.
Treatment
♦♦ Non-surgical treatment:
• Acute dislocation: It is done within 72 hours. Manual
reduction can be done. It can be done with or without
use of anesthesia. Patient should be sit upright on the
chair. Clinician should wear the gloves. Thumbs of
the clinician are positioned over lower molar teeth
bilaterally. Index fingers are placed under inferior
border of mandible. Posterior aspect of mandible is
depressed inferiorly to depress the condyle, while
the chin is elevated anteriorly and entire mandible
is pushed backwards with palm. Mandible is moved
downwards, backward and upward, manipulating the
condyle back in position.
• Chronic dislocation: Manual reduction is done under
general anesthesia.
♦♦ Surgical treatment:
• Alteration of ligaments:
–– By injecting the sclerosing agent in capsular space
of TMJ.
–– Strengthening ligaments by surgically exposing
temporal fascia and suturing flap of fascia on
capsular ligament.
454   Mastering the BDS IIIrd Year  (Last 25 Years Solved Questions)

Treatment   Osteomyelitis more frequently occurs in mandible


than in maxilla.
The treatment include:
♦♦ Conservative treatment. Classification of Osteomyelitis
♦♦ Surgical treatment.
• The goal of management is: ♦♦ Acute Osteomyelitis
–– Attenuate and eradicate proliferating pathologi- • Acute suppurative osteomyelitis
cal organism. • Acute subperiosteal osteomyelitis
–– Promote healing. • Acute periostitis.
–– Re-establish vascular permeability. ♦♦ Chronic Osteomyelitis
• Nonspecific type:
Successful Treatment Based on –– Chronic intramedullary osteomyelitis
♦♦ Early diagnosis. –– Chronic focal sclerosing osteomyelitis
♦♦ Bacterial culture and sensitivity test. –– Chronic diffuse sclerosing osteomyelitis
♦♦ Adequate and prompt antibiotic therapy. –– Chronic osteomyelitis with proliferative periostitis
♦♦ Adequate pain control. –– Chronic subperiosteal osteomyelitis
♦♦ Proper surgical intervention. –– Chronic periostitis.
♦♦ Reconstruction (if indicated). • Specific type:
–– Tuberculous osteomyelitis
Conservative Treatment –– Syphilitic osteomyelitis
♦♦ Complete bed rest. –– Actinomycotic osteomyelitis.
♦♦ Supportive therapy: It includes nutritional support in form ♦♦ Radiation-induced osteomyelitis.
of high protein and high caloric diet and adequate mul- ♦♦ Idiopathic osteomyelitis.
tivitamins.
♦♦ Dehydration control: Hydration orally or through IV fluid. Pathogenesis
♦♦ Blood transfusion: When RBC and Hb is low.
♦♦ Control of pain: Analgesic and sedation.
Periapical infection/Other foci
♦♦ Intravenous antimicrobial agents: Penicillin.

Surgical Treatment Enter the bone marrow and from there it extend to
Surgical intervention is done under antibiotic cover, started at cancellous bony spaces
least 1 to 2 days prior to the procedure.
Occlusion of the nutrients vessels of the living bone by thrombus
Incision and Drainage
formation (thrombus consists of dead or viable neutrophils,
♦♦ Done as soon as possible. microorganisms and necrotic tissue debris, etc.)
♦♦ It relives pain and pressure caused by pus accumulation.
♦♦ Extraction of loose or offending teeth.
Thrombosis of the nutrient vessels and excessive pressure from
♦♦ Debridement: Followed by incision and drainage thorough
inflammatory exudates against the rigid and confined spaces in
debridement of affected area should be carried out.
♦♦ The area may be irrigated with hydrogen peroxide and the bone
saline thrice a day.
♦♦ Any foreign body, necrotic tissue or small sequestrum Disturbed nutrition supply to the bone cells
should be removed.
♦♦ Drainage for the body and angle of the mandible area is es- Death of the cancellous bony trabeculae
tablished through extraoral sub mandibular skin incision.
♦♦ Intraoral drainage can be established for the body of
mandible. Sequestrum formation
♦♦ Sequestrum formed should be removed, if it can be gently
picked up (sequestrectomy). Spread of infection to the Volkmann’s canal in the cortical plates to
Q.15. Write short note on osteomyelitis.(Apr 2015, 3 Marks) reach external surface of bone below periosteum
Or
Write short note on osteomyelitis of jaw. Inflammation of periosteum causing periostitis
 (Dec 2009, 5 Marks)
Ans. Osteomyelitis may be defined as an inflammatory Accumulation of exudates and pus
condition of bone that begin as an infection of medullary
cavity and haversian system of the cortex and extends
to involve the periosteum of the affected side. Separation of the periosteum from the cortical plate
Section 2:  General Surgery  455

Surgical Treatment
Necrosis of the cortical bone Surgical intervention is done under antibiotic cover, started at
least 1 to 2 days prior to the procedure.
Single or multiple sinus tracts communicating the external Incision and Drainage
surface of the skin and mucous membrane
♦♦ Done as soon as possible.
♦♦ It relieves pain and pressure caused by pus accumulation.
Enveloping of sequestrum by a layer of bone formed by the ♦♦ Extraction of loose or offending teeth.
few survived bone forming cells (involucrum) ♦♦ Debridement: Followed by incision and drainage thorough
debridment of affected area should be carried out.
Pus discharge from the involucrum can reach the skin ♦♦ The area may be irrigated with hydrogen peroxide and
through the sinus openings called cloacae saline thrice a day.
♦♦ Any foreign body, necrotic tissue or small sequestrum
Clinical Features should be removed.
♦♦ Drainage for the body and angle of the mandible area is es-
♦♦ Pain and tenderness is minimal. tablished through extra oral sub mandibular skin incision.
♦♦ Non-healing bony and overlying soft tissue wounds with ♦♦ Intraoral drainage can be established for the body of
indurations of soft tissues. mandible.
♦♦ Thickened or wooden character of bone. ♦♦ Sequestrum formed should be removed, if it can be gently
♦♦ History of chronic discharge in oral cavity or on face picked up (sequestrectomy).
through the sinus present. B. Treatment of Chronic Osteomyelitis
♦♦ Pathological fracture may occur.
Done by sequestrectomy and saucerization (removal of bony
♦♦ Teeth in area tend to become loose and sensitive to palpa-
cavity).
tion and percussion.
♦♦ Lymphadenopathy is present. Sequestrectomy
♦♦ Removal of sequestrum.
Treatment
♦♦ It may be carried out under proper cover of antibiotics.
A. Treatment of Acute Osteomyelitis ♦♦ Small sequestra are removed under local anesthesia.
The treatment include: ♦♦ General anesthesia may be given for the removal of bigger
♦♦ Conservative treatment. sequestrum.
♦♦ Surgical treatment. ♦♦ Sequestra from lower border of mandible are best removed
• The goal of management is by extraoral approach using Risdon’s incision.
–– Attenuate and eradicate proliferating pathologi- ♦♦ From ramus, it may be removed by retromandibular incision.
cal organism. ♦♦ From condyle by preauricular incision.
–– Promote healing. ♦♦ Coronoid process is best approached by or intra oral incision
–– Re-establish vascular permeability. given along the anterior border of ramus of the mandible.
♦♦ After removal of the sequestrum the residual granulation
Successful Treatment Based on tissue is curetted till white shining bone appears.
♦♦ Early diagnosis. ♦♦ Bleeding is controlled by pressure pack.
♦♦ Bacterial culture and sensitivity test. ♦♦ When complete infection has been eradicated the wound
♦♦ Adequate and prompt antibiotic therapy. can be closed by primary closure.
♦♦ When the elimination of the infection is doubtful, a glove
♦♦ Adequate pain control.
drain or a rubber drain is kept in place and is changed
♦♦ Proper surgical intervention.
every 24 hours till no discharge from the bone, is seen.
♦♦ Reconstruction (if indicated).

Conservative Treatment Saucerization


♦♦ Complete bed rest. ♦♦ Removal of bony cavity.
♦♦ Supportive therapy: It includes nutritional support in form ♦♦ It consists of elimination for bony cavity in the jaw bone to
of high protein and high caloric diet and adequate mul- avoid collection of blood and formation of large hematoma
tivitamins. which is liable to get infected.
♦♦ Dehydration control: Hydration orally or through IV fluid. ♦♦ It is simple procedure for eliminating the dead space in bone.
♦♦ Blood transfusion: When RBC and Hb is low. ♦♦ Saucerization is carried out by existing the wall of the bony
♦♦ Control of pain: Analgesic and sedation. cavity by means of Rongeur bone-cutting forceps or burs.
♦♦ Intravenous antimicrobial agents: Penicillin. ♦♦ The bone is smoothen by file.
456   Mastering the BDS IIIrd Year  (Last 25 Years Solved Questions)

♦♦ The wound can be partially or completely closed depend- ♦♦ Use when all teeth are present.
ing upon the amount of suppuration. ♦♦ A 26 gauge, 25 cm long wire is passed around the neck
Q.16. Write the close methods of reduction and immobiliza­ of the 2nd molar on each side so that both ends of wire
tion of fracture of long bone and fracture body and extend to buccal side. Than the ends of both wires twisted
angle of mandible. (Dec 2012, 10 Marks) together for their entire length. So that the strong base
Ans. For close methods of reduction and immobilization of wires is formed on either side, coming towards midline
fracture of long bone refer to Ans 8 of same chapter. from each second molar.
♦♦ The excess wire is cut and the ends are checked in inter
Close Method of Reduction of Fracture Body and Angle dental space.
of Mandible ♦♦ The base wire is secured to individual tooth by using ad-
Following are the closed methods: ditional inter dental wires.
♦♦ Direct wiring: ♦♦ The one of small wire is passed from the distal surface of
• Direct interdental wiring the tooth below the base wire and brought out towards the
• Eyelet lingual side and then brought out on the buccal surface
• Continuous or multiple loop wiring from mesial interdental space above the base wire.
• Risdon’s wiring. ♦♦ Both ends are again grasped together and twisted, cut and
♦♦ Arch bars: finished in interdental space.
• Erich ♦♦ Each tooth is engaged in the same manner to the base wire,
• German silver so that the base wire is fully secured to the dental arch.
• Jelenko. ♦♦ Two types of horizontal wiring after strong fixation and
prevent supraeruption.
Methods of Immobilization of Fracture Body and
Angle of Mandible
Arch Bars
♦♦ Intermaxillary fixation:
• Dental wiring: Arch bar are used for immobilization of jaw during management
–– Direct Interdental of fractured jaw.
–– Eyelet ♦♦ There are many types of arch bars available.
–– Continuous or multiple loop wiring ♦♦ Rigid types are made by half round stainless steel wire of
–– Risdon’s wiring. 18 and 21 gauge.
• Arch bars: ♦♦ The read-made arch bars are available (Erich arch bar)
–– Erich These are consider better as these are soft, easy to adapt
–– German silver and have hooks.
–– Jelenko. ♦♦ The arch bars are indicated when there are not enough
• Cap splints. teeth in the arch for conventional Risdon’s wiring or when
♦♦ Intermaxillary fixation with osteosynthesis: all teeth in arch cannot be secured due to poor periodontal
• Transosseous wiring condition of teeth.
• Circumferential wiring ♦♦ The arch bar should be perfectly adapted to the teeth in
• External pin fixation the arch because if the bar is not fitted, it can cause ortho-
• Bone clamps. dontic movement.
♦♦ Osteosynthesis without intermaxillary fixation: ♦♦ The arch is adapted by starting from distal most point in
• Non-compression small plates the arch.
• Compression plates ♦♦ A sharp bend is given at the edge of wire to be pushed into
• Mini plates. the interdental space between 2nd and 3rd molar to avoid
slippage of arch bar.
Direct Interdental Wiring ♦♦ It is adapted progressing to midline and finishing on
In this a pre-stretched soft stainless steel wire of 0.35 mm other end.
thickness and 15 cm in length is taken. Middle portion of the ♦♦ Arch bar should not cross the fracture line.
wire should be twisted around the tooth. Free ends of the wire ♦♦ The bar should be cut and adapted to each fragment
are twisted together and form a plaited tail which is 3 cm in separating.
length. In this way, all other teeth are attached by the wires ♦♦ The arch bar is secured in place by using ligature wire
and then twisted. passed around each individual tooth.
♦♦ One end of the wire coming below the arch bar and above
Risdon’s Wiring it in the buccal side and then finished.
♦♦ Care is taken that hooks on the arch bar are directed up-
Risdon’s wiring is a commonly utilized method of horizontal
ward in the maxilla and downward in mandible.
wire fixation.
Section 2:  General Surgery  457

Transosseous Wiring Q.17. Write short note on methods of immobilization of


fracture body of mandible. (Feb 2014, 5 Marks)
The direct wiring of the fractured ends after exposing and Ans. For details of methods of immobilization of fracture body
reducing the fractured fragment. of mandible, refer to Ans 16 of same chapter.
♦♦ There are two method of wiring: The upper border wiring
and lower border wiring, in case of angle and body of the Q.18. Write short note on compound fracture.
mandible.  (Dec 2015, 5 Marks)
♦♦ The upper border wiring is done intraorally just below Ans. Compound fracture is also known as open fracture.
the alveolus and lower border wiring is done by extra-   If the bone breaks in such a way that bone fragments
oral approach. stick out through the skin or a wound penetrates down
♦♦ The edges of bony fragments are cleaned and reduced with to the broken bone, the fracture is called as compound
the help of “Bone-holding” device. fracture. For example, when a pedestrian is struck by
♦♦ Holes are drilled in the bone using an electric or “hand the bumper of a moving car, the broken shinbone may
drill”. protrude through a tear in the skin and other soft tissues.
♦♦ Drilling is done under constant jet of normal saline solu-   Since compound fractures often involve more damage
tion. to the surrounding muscles, tendons, and ligaments,
♦♦ The holes should be drilled at a distance of atleast 5 mm they have a higher risk for complications and take a
from the fractured site. longer time to heal.
♦♦ A no 26-gauge wire is passed through the holes across   This type of fracture is particularly serious because
the fracture line and tightened by trusting the two ends. once the skin is broken, infection in both the wound and
the bone can occur.
Miniplates
Classification of Compound Fractures
♦♦ In multiple mandibular fractures, accurate reduction and (Gustilo et al and Anderson’s)
establishment of normal occlusion is tuf, so it is advisable
to secure the normal occlusion through interdental wiring, ♦♦ Type I: Wound is smaller than 1 cm, clean and generally
before the miniplate osteosynthesis is performed. caused by fractured fragment which pierces the skin. This
♦♦ Following reduction, the osteosynthesis lines should be is a low energy injury.
established. The adaptation of the bone plate is done by ♦♦ Type II: Wound is longer than 1 cm, not contaminated and
bending pliers. is without major soft tissue damage or defect. It is also a
♦♦ Adapted bone plate should lie passively over the contour low energy injury.
of external cortex, and it is confirmed that there should not ♦♦ Type III: Wound is longer than 1 cm with significant soft tis-
be any gap between the plate and the bone. sue disruption. Mechanism involves high energy trauma,
♦♦ The miniplate is fixed in its specific position with the resulting in severe unstable fracture with varying degrees
screws. of fragmentation.
♦♦ In fractures of the angle of the mandible, plate is located ♦♦ Type IIIA: Wound has sufficient soft tissue to cover the bone
on the posterior fragment, medial to the external oblique without need for local or distant flap coverage.
line, this is done so that it can be bent over the surface and ♦♦ Type IIIB: Disruption of soft tissue is extensive such as
the posterior screws are placed in sagittal direction. local or distant flap coverage is necessary to cover the
♦♦ In cases with simultaneous fractures of the alveolar pro- bone. Wound may be contaminated, serial irrigation and
cess or if impacted third molar teeth are present, the plate debridement procedures are necessary to ensure clean
may be fixed to the outer surface of the mandible which surgical wound.
correspond to the position in the course of line of tension. ♦♦ Type III C: Any open fracture associated with an arterial
♦♦ When fracture is present between the canine and pre- injury requires repair is considered as Type IIIC.
molars, mental nerve may be damaged by applying the
Etiology
plate. In such cases, it is recommended to place concave
section of plate between screw holes precisely at the exit ♦♦ Compound fractures are caused by high-energy trauma,
point of the nerve. most commonly from a direct blow, such as from a fall or
♦♦ In some of the very exceptional cases, transposition of motor vehicle collision.
nerve to lower level may be indicated. To reduce the effect ♦♦ These fractures can also occur indirectly, such as a high-
of torsional forces in symphysis region between mental energy twisting type of injury.
foramina, it is mandatory to use two parallel plates.
Management
♦♦ In the cases of comminuted fractures or in cases where
there are detached triangular pieces of bone, longer plates Management of compound fracture is divided into two parts,
with six or more screws should be used. i.e. investigations and treatment.
458   Mastering the BDS IIIrd Year  (Last 25 Years Solved Questions)

be held together by inserting rods down through the marrow


space in the center of the bone. These methods of treatment can
reposition the fracture fragments very exactly. Because open
fractures may include tissue damage and be accompanied by
additional injuries, it may take time before internal fixation
surgery can be safely performed.
External fixation: Depending on the injury, surgeon may
use external fixation to hold bones in general alignment. In
external fixation, pins or screws are placed into the broken
bone above and below the fracture site. Then the orthopedic
surgeon repositions the bone fragments. The pins or screws are
connected to a metal bar or bars outside the skin.
♦♦ As fixation gets completed the skin should be sutured.
♦♦ In complex wounds, flaps should be place to cover the
injury.
Local flap. The muscle tissue from the involved limb is rotated
to cover the fracture. A patch of skin taken from another area
Fig. 38:  Compound fracture of the body (graft) is placed over this.
Free flap. Some wounds may require a complete transfer of
Investigations tissue. This tissue is often taken from the back or abdomen.
♦♦ X-rays will show how complex the fracture is. A free flap coverage procedure requires the assistance of a
♦♦ Routine blood and urine tests provide information about microvascular surgeon to ensure the blood vessels connect and
the general health. circulation is established.
Complications
Treatment
Compound fractures are serious injuries and, therefore, serious
♦♦ Antibiotics are started as soon as possible in the emergency complications are associated with them:
room. Severity of injury determines which antibiotics are ♦♦ Infection is the most common complication of open frac-
given. Tetanus shot should be given. Cefazolin /clindamy- tures. Infection can occur early, during the healing phase of
cin is given the Type I and Type II open fractures while for the fracture, or even later. In general, the greater the extent
type III fractures aminoglycoside is given. of soft tissue damage, the greater the risk for infection. If
♦♦ Injury will be covered with a sterile dressing. Doctor will an infection becomes chronic (osteomyelitis), it may lead
gently put the bones back into alignment to prevent the to further surgeries and amputation.
fragments from causing further damage to soft tissues. ♦♦ Compound fractures may have difficulty healing. If your
Then splint should be applied to injured limb to protect it fracture is failing to heal, further surgery may be required.
and keep it from moving until patient is taken to surgery. Surgery to promote healing usually includes placing a
♦♦ Débridement: During this procedure, surgeon will remove bone graft over the fracture, as well as new internal fixa-
all dirt and foreign bodes, as well as any contaminated and tion components.
unhealthy skin, muscle, and other soft tissues. The bone ♦♦ Acute compartment syndrome may develop. This is a pain-
is also cleaned of all dirt and other foreign material. Any ful condition that occurs when pressure within the muscles
unattached pieces of bone are removed. Severely contami- builds to dangerous levels. Unless the pressure is relieved
nated bone fragments are also discarded. This bone loss quickly, permanent disability and tissue death may
can be corrected later with additional surgeries. result.
♦♦ Irrigation. After débridement, the wound is cleansed and Q.19. Describe etiology, pathology, clinical features and
irrigated with several liters of saline. treatment of acute osteomyelitis. What is the sequel
♦♦ It is important to stabilize the broken bones as soon as of poorly treated osteomyelitis. (Jan 2016, 10 Marks)
possible to prevent further soft tissue damage. The broken Ans. Acute osteomyelitis is a rapidly destructive, pus
bones in an open fracture are typically held in place using producing infection which is capable of destructing the
external or internal fixation methods. These methods bone.
require surgery.
For etiology, pathology and treatment of acute
♦♦ For type II and type III compound fractures, serial
osteomyelitis refer to Ans 14 of same chapter.
irrigation and debridement is done till clean surgical
wound is achieved. Clinical Features
Internal fixation: During the operation, the bone fragments are ♦♦ There is presence of deep intense pain.
first repositioned (reduced) into their normal alignment, and ♦♦ Patient complaints of intermittent fever.
then held together with special screws or by attaching metal ♦♦ Signs of acute infection are present, i.e. bodyache, malaise,
plates to the outer surface of the bone. The fragments may also leucocytosis, raised ESR, etc.
Section 2:  General Surgery  459

♦♦ Presence of paresthesia or anesthesia of lower lip as inferior ♦♦ Based on etiology of fracture:


alveolar nerve is involved. • Traumatic fractures
♦♦ Acute infection is present in the tooth and pus discharge • Pathological fractures
is seen from gingival crevice. • Stress fractures
♦♦ Presence of mobility of involved teeth. ♦♦ Based on condition of soft tissue:
• Close or simple fracture: This fracture lacks
Sequel of Poorly Treated Osteomyelitis communication between site of fracture and exterior
♦♦ If the infection is left untreated, an abscess may develop in of body
the bone and surrounding tissue. In time, this may burst on • Open or compound fracture: In this, fracture a wound is
to the skin and leave a tract i.e. sinus between the infected present through the adjacent or overlying soft tissue
bone and surface of the skin. communicates outside the body.
♦♦ Blood infection, i.e. septicemia develop which cause seri- ♦♦ Special fractures:
ous illness. • Depressed fracture: It is common in skull bone
♦♦ If the infection follows a fracture, then there is a chance that • Fracture-dislocation: Anterior dislocation of shoulder
the fracture will not heal, i.e. there is non-union of fracture. along with fracture neck of humerus.
♦♦ Compression of other structures occur next to the infection. • Fracture involving a joint
♦♦ Some bone infections are caused by methicillin-resistant • Complex fracture: These fractures involve major nerves
S. aureus (MRSA) which is difficult to clear with antibiotics. and vessels.
♦♦ Persistent infection of the bone, i.e. chronic osteomyelitis
sometimes develops and can be difficult to clear. Clinical Features of Compound Fracture
Q.20. Classify fractures. Describe etiology, clinical features ♦♦ Swelling or bruising over a bone.
and management of compound fractures. ♦♦ Deformity of an arm or leg.
 (Mar 2016, 10 Marks) ♦♦ Pain in the injured area that gets worse when the area is
moved or pressure is applied.
Ans. Classification of Fractures
♦♦ Loss of function in the injured area.
♦♦ Based on plane of fracture surface:
♦♦ Bone appears protruding from the skin.
• Linear fractures: Fracture which run parallel to long
♦♦ Bleeding is present at the fracture site.
axis of bone.
♦♦ Presence of contamination of fracture site by mud, dirt,
• Transverse fracture: Fracture in right angle with axis
etc.
of bone
♦♦ Victim heard a bone break or snap or heard a grating
• Oblique fracture: Fracture with oblique fracture line
sensation.
• Spiral fracture: Fracture with spiral fracture line
For etiology and management of compound fractures, refer
• Comminuted fracture: Fracture with splintered or
to Ans 18 of same chapter.
crushed bone with more than two fragments.
• Segmental fracture: Fracture where a part of bone is Q.21. Describe differentiating features of Le Fort Type I, II
completely separated from are not bone with same and III fractures (Jan 2017, 3 Marks)
diameter. Ans.

Differentiating features of Le Fort type I,II,and III fractures in Table.


Features Le Fort Type I Le Fort Type II Le Fort Type III
Synonyms It is also known as Guerin’s fracture or It is also known as pyramidal fracture It is also known as craniofacial disjunction or
floating fracture or horizontal fracture high level fracture or suprazygomatic fracture
of maxilla or low level fracture
Fracture Fracture line runs horizontally above Fracture line run from nasal bone at Fracture line runs parallel to skull base. It
line the floor of nasal cavity involving top most, fracture run laterally towards passes through the nasal bone, lacrimal bone,
lower third septum, palate, alveolar lacrimal bones, medial wall of orbit, ethmoid bone, optic foramen, inferior orbital
process of maxilla and lower third of infraorbital margin, through medial to fissure, pterygomaxillary fissure and lateral
pterygoid plates of maxilla infraorbital foramen and backward below orbital wall with frontozygomatic suture. With
the zygomaticomaxillary area through zygomatic arch.
lateral wall of maxillary sinus and pterygoid
plates. Zygoma is intact with skull base.
Etiology Occur due to strong blow with sharp Occur from strong force applied in central Occur due to strong blow to face at level of orbit
object at the level above tooth bearing region at the level of nasal bones. either from lateral or frontal direction
region
Contd...
460   Mastering the BDS IIIrd Year  (Last 25 Years Solved Questions)

Contd...
Features Le Fort Type I Le Fort Type II Le Fort Type III
Features Bleeding from nose Edema of middle third of face Lengthening of face
Posterior gagging of occlusion Bilateral circumorbital and subconjunctival Enophthalmos, ocular level depression
Upper lip swelling ecchymoses Hooding of eyes, occlusal plane tilting
Palatal ecchymosis Nasal bleeding/obstruction/deformity Entire facial skeleton moves as a single block
Occlusion derangement Dish face deformity of face Tenderness and separation of suture line
Floating maxilla Diplopia Trismus
Retroposition of maxilla with posterior Teeth malalignment
gagging Guerin sign: Hematoma at greater palatine
Limitation of ocular movements foramen is present.
CSF rhinorrhea
Tenderness and separation of infraorbital
margin

24. Advanced Methods of ♦♦


♦♦
More accurate, sensitive and specific.
Small lesions are detected.
Surgery and Radiology
Disadvantages
Q.1. Write short note on CT scan. (Apr 2008, 5 Marks)
♦♦ Interpretation by experienced radiologist is done
Or
♦♦ Artifacts can be present
Write briefly on CT scan. (Dec 2010, 5 Marks)
♦♦ Cost is high.
Ans. Computed tomography scan was invented by Godfrey
Hounsfield in 1963. Q.2. Write short note on biopsy.
 (Feb 2013, 5 Marks) (Mar 1997, 5 Marks)
Working of CT Scan Ans. Biopsy is the removal of tissue from the living organism
for the purpose of microscopic examina­t ion and
Narrow X-ray beams are passed from rotating X-ray generator
diagnosis.
through the gantry where patient is placed. When X-ray pass
through the tissues, some of the X-rays get absorb and some Types of Biopsy
pass through, depending on the tissue density, different grades
of absorption in different tissues are detected through sensitive ♦♦ Excisional biopsy: If a lesion is totally excised for histologi-
detectors which are translated to a Gray scale image by a computer. cal evaluation it is called as excisional biopsy.
Density of tissues is numbered as Hounsfield Number (HN). ♦♦ Incisional biopsy: When only a small section of tissue is
removed from the lesion for the purpose of histological
Contrast Agents evaluation it is called as incisional biopsy.
♦♦ Fine needle aspiration cytology (FNAC): It is done by
♦♦ Ionic: Water soluble iodide dyes such as sodium dia-
aspirating tissue material inside a lesion which is later on
trizoate, meglumine iothalamate. They are cheaper but
often toxic and cause anaphylaxis. diagnosed microscopically after preparing a smear.
♦♦ Non-ionic are safer but expensive like Iohexol, Iopamiro. ♦♦ Frozen section biopsy: It is performed in order to get an
immediate histological report of a lesion. The tissue is
Indications obtained from the lesion and is kept in deep freeze and
than frozen tissue is sectioned and stained to get a prompt
♦♦ Trauma-like head injury, chest injury. Abdomen trauma.
In trauma only plain CT scan is taken. diagnosis.
♦♦ Neoplasms: To see the exact location, size, vascularity, Procedure of Biopsy
extent and operability.
♦♦ Inflammatory conditions: In various sites such as in pseu- ♦♦ Anesthesia: Give a block to anesthetize the region where
docyst of pancreas. specimen is to be obtained. Local infiltration and injections
into the tissue which should be biopsied is avoided as it
leads to the artifacts in the specimen. If a block is not effec-
Contraindications
tive give local infiltration atleast l cm away from the lesion.
♦♦ In pregnancy. ♦♦ Stabilization of tissue: Soft tissue biopsies are done over
♦♦ In restless patient. the movable tissues of oral cavity, i.e. tongue, lips, etc.
Dental assistant stabilizes the tissue by stretching it.
Advantages ♦♦ Hemostasis: Gauze pieces are the best means for compress-
ing the tissue and achieving hemostasis. Gauze piece can
♦♦ 1 to 2 mm sized sections are possible. also be placed to cover the mouth of suction tip and is
♦♦ Amount of exposure to radiation is less used to prevent the specimen from being sucked inside.
Section 2:  General Surgery  461

♦♦ Incisions: Use a sharp scalpel. Provide two incisions ♦♦ It is less time consuming.
which form an elliptical incision and converge to form a ♦♦ It do not spread the tumors, disrupt the field for surgical
V at the base, this provides a good specimen and a wound dissection.
which is easy to close. Alternatively, a triangular-shaped Q.4. Write briefly on Laser. (Dec 2010, 5 Marks)
incision can be made which converges in the form of a
tip of a pyramid at the base. Incisions should be given Or
parallel to the nerves and vessels in that region to avoid Write short note on Laser in surgery.
damage.  (Dec 2015, 5 Marks) (June 2014, 5 Marks)
♦♦ Handling of tissues: Tissue which has to be removed Ans. Molecules are placed in a compact area and power is
should be handled carefully so that histopathological passed through this so as to activate the molecules.
examination can be performed. A non-toothed tissue Molecules get activated at different periods and move
holding forceps is used and care is taken not to crush the in different directions, which they hit to each other
tissues. releasing energy. This energy is allowed to act through
♦♦ Care of specimen: After removal of the tissues, the speci- optical system to the area, wherever required. It is named
men is transferred to a bottle containing 10% formalin depending on the molecules used as:
which should be atleast 20 times the volume of the speci- • Argon Laser.
men obtained. • Neodymium Yttrium Aluminum Garnet Laser (Nd:
♦♦ Surgical closure of wound: Primary closure is possible in YAG Laser)
most cases. Where it is not possible, the tissues are under- • CO2 Laser.
mined to facilitate closure. • Neon Laser.
Q.3. Write short note on FNAC. (Nov 2008, 5 Marks) • Holmium Laser.
• Erbium Laser.
Or
Write short note on fine needle aspiration cytology. Uses of Laser
 (Aug 2011, 5 Marks) ♦♦ In cranial surgery in children.
Ans. FNAC is also called as fine needle aspiration cytology. ♦♦ In ENT it is used to treat vocal cord lesions, laryngeal
• FNAC is the microscopic examination of aspirate lesions.
which is obtained by penetrating a fine needle in ♦♦ In ophthalmology, it is very useful in retinal surgery
the lesion. ♦♦ In general surgery:
• FNAC is a procedure for rapid diagnosis. • In bleeding duodenal ulcer.
• For palliative decoring of tumors in carcinoma
Procedure esophagus.
♦♦ Position the needle over the target area in the lesion. • In carcinoma of rectum.
♦♦ As the needle is penetrated and is positioned a plunger is • In treatment of hemorrhoides (lst and 2nd degree).
pulled to apply negative pressure. • In resection of bladder tumor.
♦♦ Negative pressure should be released as needle remains • In cervical cancer.
in target tissue. • To achieve bloodless field.
♦♦ Needle is withdrawn and defumed air is withdrawn outside. • Often in making incisions in abdomen and other places.
♦♦ Drop of aspirate is taken on the slide and is thoroughly
spreaded. Precautions
♦♦ Fix the slide by keeping it in 95% of alcohol for one hour. ♦♦ All reflecting instruments should be avoided otherwise
♦♦ Staining with PAP stain should be done. laser gets reflected and injure normal tissues or the work-
ing team in the OT
Indications ♦♦ All should wear protective spectacles to their eyes.
♦♦ Help in diagnosis of swellings of head and neck region.
Advantages
♦♦ Part of initial diagnostic work-up of lymphadenopathy,
metastatic lesion or lymphomas. ♦♦ Provide bloodless field
♦♦ Helps in distinguishing benign from malignant and cystic ♦♦ It is fast
lesions from inflammatory lesions. ♦♦ Small lesions can be removed easily and completely.
♦♦ Aids in diagnosis for salivary gland pathologies.
♦♦ Helps to identify the cause for enlarged lymph nodes. Disadvantage

Advantages Availability and cost factors.


♦♦ It is minimally invasive. Q.5. Write short note on cryosurgery or laser in surgery.
♦♦ It is safe, fast and cost effective method.  (Dec 2012, 5 Marks)
462   Mastering the BDS IIIrd Year  (Last 25 Years Solved Questions)

Or Disadvantages
Write short note on cryosurgery. (Feb 2015, 5 Marks) ♦♦ Technique is difficult.
Ans. ♦♦ Availability of facility.
♦♦ Produces local complications such as displacement/erosion.
Cryosurgery
Q.7. Describe briefly different types of biopsy.
♦♦ It is the method of destruction of tissues by controlled
cooling.  (Apr 2017, 5 Marks)
♦♦ It consists of an automatic defrosting device with cryo- Ans. Following are the different types of biopsies:
probe. ♦♦ Incision biopsy—wedge biopsy
♦♦ Commonly nitrous oxide is used as it is easily available, ♦♦ Excision biopsy
cheaper and have optimum temperature required for other ♦♦ Trucut biopsy
procedures. ♦♦ Pap smear
♦♦ Other gases used are CO2, Liquid nitrogen, Freon. ♦♦ FNAC
♦♦ Frozen section biopsy
Mode of Action ♦♦ Punch biopsy
♦♦ It produces intracellular crystallization, dehydration and ♦♦ Ultrasound guided biopsy
denaturation of proteins and cell death. ♦♦ Brush biopsy
♦♦ It causes the obliteration of microcirculation and so cell death. ♦♦ Laparoscopic biopsy
♦♦ CT-guided biopsy
Indications ♦♦ Thoracoscopic biopsy
♦♦ To remove warts and lesions in the skin. ♦♦ Endoscopic biopsy (gastroscopic or colonoscopic or
♦♦ Cryotherapy for piles. through ERCP or through cystoscopy)
♦♦ For chronic cervicitis. ♦♦ Proctoscopic biopsy
♦♦ Open biopsy either laparotomy or thoracotomy or
Advantages craniotomy using Dancly’s brain cannula
♦♦ Relatively bloodless and painless.
Incision Biopsy
♦♦ Adequate control of extent and depth in freezing
♦♦ Equally effective. ♦♦ This is the excision of a portion of lesion for microscopic
examination.
Disadvantages ♦♦ This method is employed on large, diffuse lesions which
♦♦ Can cause infection. has the size of 2 cm in its greatest dimension.
♦♦ Discharge from the site. ♦♦ This method can also be dome on lesions suspected for
For LASER in surgery refer to Ans 4 of same chapter. malignancy.
♦♦ Aim of this method is to remove a portion of lesional tissue
Q.6. Write short note on brachytherapy. (Aug 2012, 5 Marks)
in question along with the sample of normal adjacent tissue
Ans. It is radiation given with source close to the tumor. for comparison.
• It is given using iridium192 caesium137.
• It is curative radiotherapy. Types of Incision Biopsy
• It is used in carcinoma of oral cavity, penis, breast, ♦♦ Punch biopsy: This is done by using a surgical punch of
cervix and bladder. diameter 4, 8 or 10 mm. This incisional biopsy is done in
• Radiation material placed in the cavity is called mass screening programs.
intracavitary radiotherapy. ♦♦ Wedge biopsy: It is done by making the wedge-shaped
• Radiation material is inserted into the tissues— incision which begins 2 to 3 mm from normal tissue and
interstitial radiotherapy. penetrates in the region surrounding abnormal tissue.
• Implants can be kept permanently or temporarily. Tissue should always be incised narrow and deep.
• Radioactive material is placed into the cavity/tissue
through applicators under general anesthesia. Excision Biopsy
• Intraoperative radiotherapy is also becoming ♦♦ This procedure should be done for the small lesions which
popular. It has only localised effect with adjacent are clinically benign.
tissue being spared. ♦♦ In this, complete lesion should be removed for examination
and diagnosis. So it is both diagnostic and curative.
Advantages
♦♦ High, localized, continuous dose of radiotherapy. Trucut Biopsy
♦♦ Deeper and adjacent tissues get spared ♦♦ It is done using a specialized device wherein gun with
♦♦ Dose rate is high in short time. truecut tip is inserted into the surface tissue/organ and
♦♦ Side effects are less gun is fired to close the punching needle of to catch and
♦♦ It is curative and effective in early cancers. cut the adequate tissue. It is done in prostrate, breast and
♦♦ Surgery is avoided and part is retained. surface tumors.
Section 2:  General Surgery  463

FNAC 1. Heat method: Dry heat or moist heat


♦♦ This is the cytological study of tumor cells to find out the 2. Ionizing radiation
disease and also confirm whether it is malignant or not. 3. Ethylene oxide gas.
♦♦ It is done by using 23 or 24 gauge needle fixed to specialized
syringes which create negative pressure for aspiration and Heat Method
contents are smeared on slides.
a. Dry heat: The most common apparatus, which works on
♦♦ It is contraindicated in testicular tumor.
the principles of dry heat, is hot air oven. The hot air oven,
Frozen Section Biopsy which has controlled cycles,160°C for one hour are suitable
for killing bacteria on materials, which are not penetrated
♦♦ It is done when biopsy report is needed at earliest.
♦♦ It is done in pathology set-up existing adjacent to operation by steam. Dry heat oxidize the bacterial cytoplasm, e.g.
theater. glassware, oils, petroleum jelly.
♦♦ In this an unfixed fresh tissue is frozen (using carbon b. Moist heat: Steam kills bacteria by coagulating cytoplasm
dioxide) in a metal and sections are made and stained. and is an extremely efficient sterilizer when used under
♦♦ Advantage of this technique is that, it is quick and surgeon increased pressure, when the temperature exceeds 100°.
can decide further steps in surgery. Autoclave is the apparatus used which causes sterilization
by moist heat within 30 min. The autoclave work at
Ultrasound Guided or CT-guided temperature of 121°C at 15 Ibs pressure per square inch
♦♦ This procedure is conducted with large needle with for 15 to 20 minutes.
assisted CT scan equipment.
Ionizing Radiation
♦♦ Simultaneous CT scan allow identification and visualization
of exact size of tumor on computer screen. Gamma rays are lethal, non-charged, ultrashort, wavelength
♦♦ It enables operator to guide the needle in tumor and obtain rays with great penetrating power from a radioactive isotope
several samples of tissue. such as cobalt-60, disposable plastic syringes, sutures and
♦♦ Tissues then later examined by histopathologist. rubber glass are examples.
Brush Biopsy Ethylene Oxide Gas
♦♦ In this, a brush biopsy kit is supplied by the manufacturer
It is a highly toxic inflammable gas, which kills all types of
which consists of brush biopsy instrument, bar-coded glass
microbes including bacterial spores. CO2 is mixed with ethylene
slide, alcohol-based fixative and protective plastic case for
oxide gas to reduce likelihood of explosion. The gas diffuses
mailing and instruction sheet.
♦♦ In this, nylon brush is designed to collect cells from well through items such as plastic materials, swab and paper.
all layers of epithelium including basal cell layer of Q.2. Discuss the asepsis and antiseptic measures in oro-
epithelium. dental surgery patient. (Dec 2007, 8 Marks)
♦♦ Procedure includes application of firm pressure on Ans. Asepsis means precaution taken before any surgical
lesion rotating brush 5–10 times. After this nylon brush procedure against development of infection.
is manipulated on glass slide so more cells are spreaded
over the slide. Antisepsis means all surgical procedures done after
♦♦ Slide is analyzed by computer program designed for taking precautions.
pathological review. The concept of asepsis can be applied in any clinical setting.
♦♦ Results are interpreted as negative, positive or typical. The element requiring careful attention is equipment or
supplies. Medical and dental equipment can be sterilized
by chemical treatment, radiation, gas, or heat. Personnel
25. Sterilization can take steps to ensure sterility by assessing that sterile
packages are dry and intact and checking sterility
indicators such as dates or colored tape that changes
Q.1. Write briefly on sterilization. (Mar 1998, 5 Marks)
color when sterile.
Or Besides overall attention to the clinical environment
Write short note on sterilization in surgery. and equipment, clinicians need to be attentive to their
 (Feb 2013, 5 Marks) (Sep 2006, 5 Marks) own practices and those of their peers in order to avoid
Or inadvertent contamination.
Write short note on sterilization. (Apr 2008, 5 Marks) Aseptic technique is most strictly applied in the operating
 (Mar 2016, 5 Marks) room because of the direct and often extensive disruption
Ans. Sterilization is the complete removal of all types of of skin and underlying tissue. Aseptic technique helps to
microbes. It can be attempted by one of the following prevent or minimize postoperative infection. The patient
methods of sterilization. is prepared or prepped by shaving hair from the surgical
464   Mastering the BDS IIIrd Year  (Last 25 Years Solved Questions)

site, cleansing with a disinfectant such as iodine, and Asepsis in the operating room or for other invasive
applying sterile drapes. procedures is also maintained by creating sterile surgical
In all clinical settings, handwashing is an important step fields with drapes. Sterile drapes are sterilized linens
in asepsis. In general settings, hands are to be washed placed on the patient or around the field to delineate
when visibly soiled, before and after contact with the sterile areas. Drapes or wrapped kits of equipment are
patient, after contact with other potential sources of opened in such a way that the contents do not touch non-
sterile items or surfaces. Aspects of this method include
microorganisms, before invasive procedures, and after
opening the furthest areas of a package first, avoiding
removal of gloves. Patients and visitors should also be
leaning over the contents, and preventing opened flaps
encouraged to wash their hands. Proper handwashing
from falling back onto contents.
for most clinical settings involves removal of jewelry,
avoidance of clothing contact with the sink, and a Other principles that are applied to maintain asepsis
minimum of 10–15 seconds scrubbing hands with soap, include:
warm water, and vigorous friction. • All items in a sterile field must be sterile.
• Sterile packages or fields are opened or created as
A surgical scrub requires use of a long-acting, powerful, close as possible to time of actual use.
antimicrobial soap, careful scrubbing of the fingernails, • Moist areas are not considered sterile.
and a longer period of time for scrubbing. Institutional • Contaminated items must be removed immediately
policy usually designates an acceptable minimum length from the sterile field.
of time required. Thorough drying is essential, as moist • Only areas that can be seen by the clinician are
surfaces invite the presence of pathogens. Contact considered sterile, i.e. the back of the clinician is not
after handwashing with the faucet or other potential sterile.
contaminants should be avoided. The faucet can be • Gowns are considered sterile only in the front, from
turned off with a dry paper towel, or, in many cases, chest to waist and from the hands to slightly above
through use of foot pedals. Despite this careful scrub, the elbow.
bare hands are always considered potential sources of • Tables are considered sterile only at or above the
infection. An important principle of aseptic technique is level of the table.
that fluid (a potential mode of pathogen transmission) • Nonsterile items should not cross above a sterile
flows in the direction of gravity. With this in mind, hands field.
are held below elbows during the surgical scrub and • There should be no talking, laughing, coughing, or
above elbows following the surgical scrub. sneezing across a sterile field.
Sterile surgical clothing or protective devices such as • Personnel with colds should avoid working while
ill or apply a double mask.
gloves, facemasks, goggles, and transparent eye/face
• Edges of sterile areas or fields (generally the outer
shields serve as a barrier against microorganisms and are
inch) are not considered sterile.
donned to maintain asepsis in the operating room. This
• When in doubt about sterility, discard the potentially
practice includes covering facial hair, tucking hair out
contaminated item and begin again.
of sight, and removing jewelry or other dangling objects
• A safe space or margin of safety is maintained
that may harbor unwanted organisms. This garb must
between sterile and nonsterile objects and areas.
be done with deliberate care to avoid touching external,
• When pouring fluids, only the lip and inner cap
sterile surfaces with nonsterile objects including the skin. of the pouring container is considered sterile. The
This ensures that potentially contaminated items such as pouring container should not touch the receiving
hands and clothing remain behind protective barriers, container, and splashing should be avoided.
thus prohibiting inadvertent entry of microorganisms • Tears in barriers are considered breaks in sterility.
into sterile areas. Personnel assist the surgeon to wear Q.3. Write short note on autoclave. (Aug 2012, 5 Marks)
gloves and garb and arrange equipment to minimize the  (Apr 2017, 4 Marks)
risk of contamination.
Ans. It is the means of moist heat sterilization.
Donning sterile gloves requires specific technique so • It is the method of choice for sterilization of
that the outer glove is not touched by the hand. A large instrument as it eliminates even resistant spore
cuff exposing the inner glove is created so that the glove forming microorganisms, fungi, viruses along with
may be grasped during donning. It is essential to avoid vegetative microorganisms.
touching nonsterile items once sterile gloves are applied; • It works on principle of steam under pressure.
the hands may be kept interlaced to avoid inadvertent • It has two pressure cycles i.e.
contamination. Any break in the glove or touching the A. 15 psi pressure at 121°C for 30 minutes.
glove to a nonsterile surface requires immediate removal B. 30 psi pressure at 134°C for 3 to 5 minutes also
and application of new gloves. known as Flash method.
Section 2:  General Surgery  465

Mechanism of Action • Autoclaving is the most accepted method of


sterilization of surgical instruments as it eliminates
Steam is the mixture of heat and water vapor. As it comes
bacteria, viruses, fungi and spores. It works on the
in contact with any cool surface it get condensed and heat is
principle of steam under pressure of 15 lb at l2l°C for
released from water. This heat is taken by surface it comes in
20 min or 30 lb at l34°C for 3 minutes. It has excellent
contact with. Heat goes on penetrating deeper layers of object.
penetration, facilitating exposure of all instrument
Steam and air move in vertical direction and the movement is
surfaces to the steam.
quicker penetration of steam into the material, it is also better,
• Dry heat ovens or the unsaturated chemical vapor
if articles are placed vertically in autoclave.
sterilizers are the other means of sterilisation.
Advantages Hot air ovens require 160°C and 2 hours for
sterilization
♦♦ It is economical.
• Ultraviolet light may kill microorganisms that are
♦♦ Penetration is good.
directly exposed to the light; however, the light
♦♦ Cycle time is short.
may not reach all the surface of an instrument. A
♦♦ It is monitored easily.
temperature of l60°C–l70°C maintained for l hour
♦♦ No exhaust or special chemicals are to be required.
is capable of sterilization.
Disadvantages • Glass bead sterilizers are used to sterilize endodontic
files and burs in beads. They require 10 seconds for
♦♦ Carbon steel gets damaged. sterilization.
♦♦ Moisture retention is present.
Q.4. Write short note on methods of sterilization of dental Q.5. Write briefly on chemical sterilization.
instrument. (Aug 2012, 5 Marks)  (Jan 2016, 2 Marks)
Ans. Following are the methods for sterilization of a dental Ans. Chemical sterilization is also known as cold
instrument: sterilization.
• Placing the instrument presoaking solutions, i.e.
Following are the various chemicals used in chemical
phenolic compounds prevents drying of debris and
sterilization:
also helps in microbial killing.

Name of chemical Target Mechanism of action Materials sterilized

Glutaraldehyde Cell wall or outer • In Gram-positive bacteria and fungi crosslinking of Corrugated rubber, anesthetic tubes,
membrane of protein occur plastic endotracheal tubes, metal
bacteria • In Gram-negative bacteria, there is removal of magnesium instruments and polythene tubing
and there is release of some phospholipids
• In virus, there is inhibition of DNA synthesis
Formaldehyde DNA of bacteria It reacts with carboxyl and sulfhydriy groups. Hydroxyl group Operation theaters
also reacts with nucleic acid and inhibit RNA and DNA
synthesis by crossinking proteins
Chlorhexidine and Cytoplasmic It causes membrane damage which involves phospholipid For preparation of skin for surgery
quaternary ammonium membrane of bilayers. Low concentration of this affect membrane integrity
compounds bacteria and high concentration leads to congealing of cytoplasm
Phenols Cytoplasmic It leads to leakage of potassium and other intercellular For disinfecting the surfaces
membrane of constituents. It also causes uncoupling of oxidative
bacteria phosphorylation which leads to irreversible cell damage.

Halogens, i.e. chlorine DNA of bacteria It causes DNA synthesis For preparation of skin for surgery
and iodine

Q.6. Write difference between sterilization and disinfection. Examples of methods of Examples of disinfectants are
 (Apr 2017, 2 Marks) sterilization are sunlight, phenols, halogens, aldehydes,
Ans. drying, heating, radiation and oxidizing agents, etc.
filtration
Sterilization Disinfection
It is sporicidal It is not sporicidal
Sterilization is a process by It is the process of removal of
It includes both physical and It includes mainly chemical methods
which articles are freed of pathogenic microorganisms to the
chemical methods and some of radiation methods
all microorganisms both in level where it can no more cause
vegetative and spore state. disease. It is a well-defined process It does not require any strict protocol
strictly done under proper
Not applicable to living tissue Applicable to inanimate objects
quality control
Agents used for sterilization Agents used for disinfection are
This method guarantee total It never assures complete prevention
are known as sterilants known as disinfectants
control in preventing infection from acquiring the infection
466   Mastering the BDS IIIrd Year  (Last 25 Years Solved Questions)

26. Upper Limb Ischemia nant features of median nerve involvement, more so in
menopausal women gives a clue to the diagnosis.
♦♦ Raynaud’s phenomenon.
Q.1. Write short note on cervical rib. (Sep 2009, 5 Marks)
Treatment
Ans. This is an extra rib present in the neck in about l–2% of
the population. ♦♦ Conservative: Patients with mild neurological symptoms
Commonly, unilateral and in some cases, it is bilateral. are managed by shoulder girdle exercises or correction
of faulty posture.
It is more frequently encountered on the right side.
♦♦ Surgery:
lt is the anterior tubercle of the transverse process • Excision of cervical rib including periosteum: This is
of the 7th cervical vertebra which attains excessive called extraperiosteal excision of cervical rib. This is
development and results in cervical rib. included with cervical sympathectomy, if vascular
symptoms are predominant.
Types of Cervical Rib
• If there is a thrombus in the subclavian artery, it is
♦♦ Type I: Free end of the cervical rib is expanded into a hard, explored and thrombus is removed and the artery is
bony mass which can be felt in the neck. repaired.
♦♦ Type II: Complete cervical rib extends from C7 vertebra • At exploration, if cervical rib is not found, divide
posteriorly to the manubrium anteriorly. scalenus anterior muscle. This is called scalenotomy
♦♦ Type III: Incomplet e cervical rib, which is partly bony, • If hyperabduction syndrome is diagnosed, pectoralis
partly fibrous. minor is divided from its insertion into the coracoid
♦♦ Type IV: A complete fibrous band which gives rise to process.
symptoms but cannot be diagnosed by X-ray.
Clinical Features
♦♦ It is common in young females. 27. Rectum and Anal Canal
♦♦ Dull aching pain in the neck is caused by expanded bony
end of cervical rib. Q.1. Write short note on Goodsall’s rule.(Nov 2008, 5 Marks)
♦♦ Upper limb ischemia is usually present. Ans. It is also known as Goodsall’s (1900) rule.
♦♦ Ulnar nerve paralysis or weakness manifest as paralysis,
of interosseous muscles. Goodsall’s Rule
♦♦ A hard mass may be movable or visible or palpable in neck. ♦♦ Fistulas with an external opening in relation to the anterior
♦♦ On palpation a thrill and on auscultation, a bruit can be half of the anus is of direct type.
heard in cases of poststenotic dilatation. ♦♦ Fistulas with external openings in relation to posterior
half of the anus, has a curved track may be of horseshoe
type, opens in the midline posteriorly and may present
with multiple external opening all connected to a single
internal opening.
♦♦ Per rectal (P/R) examination shows indurated internal
opening usually in the midline posteriorly.
♦♦ Most of the fistulas are on posterior half of anus.
♦♦ Probing in the ward and fistulogram in the ward before
surgery using Lipiodol is not advisable as it may cause
recrudescence of inflammation. It can be done with
adequate precaution. Probing is done under general
anesthesia gently with care without creating extensions.

28. Miscellaneous
Fig. 39:  Types of cervical rib
Q.1 Write short note on Sushruta. (Feb 2015, 5 Marks)
Differential Diagnosis Ans. Sushruta, one of the earliest surgeons of the recorded
♦♦ Cervical spondylosis: This should be considered as a pos- history (600 BC) is believed to be the first individual to
sibility in patients above the age of 40 years. describe plastic surgery.
♦♦ Cervical disc protrusion and spinal cord tumors: May mimic • Sushruta who lived nearly 150 years before
cervical rib with predominant neurological feature. Hippocrates vividly described the basic principles
♦♦ Carpal tunnel syndrome can occur due to various causes of plastic surgery in his famous ancient treatise
such as myxoedema, rheumatoid arthritis, etc. Predomi- Sushruta Samhita.
Section 2:  General Surgery  467

• He is dubbed as the “Founding Father of Surgery” – Classification of burns into four degrees and
and the Sushruta Samhita is identified as one of explaining the effect of heat stroke, frostbite, and
the best and outstanding commentary on Medical lightening injuries.
Sciences of Surgery. – Fourteen types of bandaging capable of
• He is said to have been a physician originally of covering almost all the regions of the body and
South India active in Varanasi. different methods of dressings with various
medicaments.
– Use of wine to dull the pain of surgical incisions.
– Described 20 varieties of sharp instruments
(sastra) and 101 types of blunt instruments
(yantra) and their handling techniques.
– Systematic dissection of cadavers.
– Advocated the practice of mock operations on
inanimate objects such as watermelons, clay
plots and reeds.
– Use of leeches to keep wounds free of blood clots.
– A code of ethics for teachers as well as students.
Q.2. Write short note on Surveyor. (Apr 2015, 3 Marks)
Ans. Surveyor is the one of the member of trauma team.
Surveyor are of two types, i.e. primary surveyor and
secondary surveyor.
Fig. 40: Sushruta
Primary Surveyor (Surgical Resident)
• His period is usually placed between the period of
♦♦ Performs the primary survey, relaying all pertinent find-
1200-600 BC.
ings to the team.
• One of the earliest known mention of the name is
♦♦ May perform the secondary survey, relaying all pertinent
from the Bower Manuscript (4th or 5th century),
findings to the team.
where Sushruta is listed as one of the ten sages
♦♦ Performs or assists in the performance of any life-saving
residing in the Himalayas.
procedures at the direction of the team leader
• Sushruta has described surgery under eight heads
Chedya (excision), Lekhya (scarification), Vedhya Secondary Surveyor (Surgical Resident or Intern)
(puncturing), Esya (exploration), Ahrya (extraction),
♦♦ Assists with the “exposure" aspect of the primary survey
Vsraya (evacuation) and Sivya (Suturing).
and applies warm blankets.
• The Sushruta Samhita, in its extent form, in 184
♦♦ May perform the secondary survey, relaying all pertinent
chapters contains descriptions of 1,120 illnesses,
findings to the team
700 medicinal plants, 64 preparations from mineral
♦♦ Performs or assists in the performance of any life-saving
sources and 57 preparations based on animal
procedures at the direction of the team leader
sources.
• The text discusses surgical techniques of making
incisions, probing, extraction of foreign bodies,
alkali and thermal cauterization, tooth extraction,
excisions. etc.
• The Sushruta’s contribution in the field of Plastic
Surgery can be enumerated as follows:
– Rhinoplasty (cheek)
– Classification of mutilated earlobe defects
and techniques for repair of torn earlobes (15
different types of otoplasties)
– Cheek flap for reconstruction of absent ear lobe.
– Repair of accidental lip injuries and congenital
cleft lip.
– Piercing children’s ear lobe with a needle or awl.
– Use of suture materials of bark, tendon, hair and
silk.
– Needles of bronze or bone (circular, two finger-
breadths wide and straight, triangular bodied, Fig. 41: Various members which comprises of trauma team and their
three finger - breadths wide) position with respect to traumatically injured patient
468   Mastering the BDS IIIrd Year  (Last 25 Years Solved Questions)

Surveying in Maxillofacial Injuries germ theory of disease and its application in clinical
Primary Survey medicine.
• He is best known to the general public for his
♦♦ Identify the airway compromise from fracture or hemor- invention of the technique of treating milk and wine
rhage. to stop bacterial contamination, a process now called
♦♦ Bilateral anterior fractures of mandible have risk of falling pasteurization.
back of tongue. • He is regarded as one of the three main founders
♦♦ Orotracheal intubation should be needed. of bacteriology, together with Ferdinand Cohn and
♦♦ By help of mouth props and epistaxis balloons hemorrhage Robert Koch, and is popularly known as the “Father
should be controlled. of Microbiology”
♦♦ Anterior and posterior nasal packing can be needed.
Q.4. Write briefly on types of skin grafts.
Secondary Survey  (Jan 2016, 2 Marks)
♦♦ Orbital rim, zygomatic arches and mandible should be Ans. Skin grafts are of two types, i.e.
palpated to identify fractures. 1. Partial thickness graft
♦♦ Eyes should be examined: Restricted eye movement is 2. Full thickness graft.
suggestive of orbital fracture.
Partial Thickness Graft
♦♦ Subconjunctival hemorrhage can be suggestive of fracture
of skull. ♦♦ It is also known as Thiersch graft or split thickness skin
♦♦ Proptosis and ophthalmoplegia can be suggestive of ret- graft
robulbar hemorrhage. ♦♦ In it, there is removal of full epidermis and part of dermis
♦♦ Sensation should be assessed in maxillary branch of from donor area.
trigeminal nerve. ♦♦ It can be thin, intermediate and thick.
♦♦ Check intracanthal distance which should be 30–35mm. If ♦♦ It is indicated in well-granulated ulcer, clean wound or
distance is more than mentioned range, it is suggestive of defect which is not apposed and after the surgery to close
nasoethmoid fracture. and cover the created defect.
♦♦ Intraoral examination is necessary. Assess the occlusion ♦♦ It should not be done over bone, tendon, cartilage and joint.
and intraoral hematomas.
Full Thickness Graft
Q.3. Write briefly on Louis Pasteur. (Jan 2016, 2 Marks)
Ans. Louis Pasteur was a French chemist and microbiologist ♦♦ It includes both epidermis and full dermis.
renowned for his discoveries of the principles of ♦♦ It should be done over face, eyelid, hand, fingers and over
vaccination, microbial fermentation and pasteurization. the joints.
♦♦ It is used only for small areas.
♦♦ Its functional and cosmetic results are excellent.
Q.5. Write short note on local anesthesia.
 (June 2014, 5 Marks)
Ans. Local anesthesia is defined as a condition of loss of pain
sensation over a portion of the anatomy without loss of
consciousness.

Properties of Local Anestheisa


The ideal local anesthetic should possess the following
properties:
♦♦ Its action must be reversible.
♦♦ It must be non-irritating to the tissues and produces no
secondary local reaction.
♦♦ It should have a low degree of systemic toxicity.
Fig. 42:  Louis Pasteur ♦♦ It should have a rapid onset and be of sufficient duration.
• He is remembered for his remarkable breakthroughs ♦♦ It should have potency sufficient to give complete anes-
in the causes and preventions of diseases, and his thesia without the use of harmful concentration solution.
discoveries have saved countless lives ever since. ♦♦ It should have sufficient penetrating properties to be
• He reduced mortality from puerperal fever, and effective as a topical anesthetic.
created the first vaccines for rabies and anthrax. His ♦♦ It should be relatively free from producing allergic reac-
medical discoveries provided direct support for the tions.
Section 2:  General Surgery  469

♦♦ It should be stable in solution and undergo biotransforma- ♦♦ In abdominal surgery: Epidural or spinal anesthesia is com-
tion readily within the body. bined with the general anesthesia.
♦♦ It should be either sterile or capable of being sterilized by ♦♦ In gynecological and urological procedures: Spinal or epidural
heat without deterioration. anesthesia is given.
♦♦ In skin and peripheral vascular surgery: Topical anesthesia,
Techniques of Local Anesthesia
field block, peripheral nerve block or epidural anesthesia
♦♦ Surface anesthesia: Local anesthetic spray is applied by can be given.
spray, solution or cream to skin or the mucus membrane.
Complications of Local Anesthesia
♦♦ Infiltration anesthesia: Local anesthetic is injected in tissue
which is to be anesthetized. Both surface as well as infiltra- ♦♦ Local complications
tion anesthesia makes up topical anesthesia. • Needle breakage
♦♦ Field block: Subcutaneous injection of local anesthetic • Paresthesia
agent is given in area which borders the field to be anes- • Facial nerve paralysis
thetized. • Trismus
• Hematoma
♦♦ Peripheral nerve block: Local anesthesia is given in vi-
• Burning on giving injection
cinity of peripheral nerve to anesthetize nerve’s area, of
• Infection
innervations. • Edema
♦♦ Plexus anesthesia: Local anesthesia is given in vicinity • Soft tissue injury
of plexus of nerve, often inside the tissue compartment • Sloughing of tissues
which limits diffusion of drug away from site of injection. • Postanesthetic intraoral lesion
♦♦ Epidural anesthesia: In this Local anesthetic solution is ♦♦ Systemic complications:
injected in the epidural space where it acts primarily on • Toxicity
spinal nerve roots. Depending on site of anesthesia and • Idiosyncrasy
volume injected, anesthetized area varies from limited • Allergy
areas of abdomen or chest to large regions of body. • Anaphylactic reactions.
♦♦ Spinal anesthesia: It is given at lumbar spine in CSF where
Q.6. Write short answer on sequelae of inflammation.
it acts on spinal nerve root and at the part of spinal cord.
Anesthesia occurs from legs to abdomen or chest.  (Apr 2018, 3 Marks)
Ans. The inflammatory process can culminate into one of the
♦♦ Regional anesthesia: In this method, circulation of blood
following outcomes;
over a limb is interrupted by tourniquet, than large volume
♦♦ Resolution: It means complete return to normal tissue
of local anesthetic is injected into the peripheral vein. Now following acute inflammation. This occurs when tissue
the drug fills the venous system of limb and diffuses in changes are slight and the cellular changes are reversible
the tissues where peripheral nerves and nerve endings e. g. resolution in lobar pneumonia.
anesthetize. Effect of anesthesia is limited to the area ♦♦ Healing. Healing by fibrosis takes place when the tissue
which gets excluded from blood circulation and resolves destruction in acute inflammation is extensive so that there
as circulation is restored. is no tissue regeneration. But when tissue loss is superficial,
it is restored by regeneration.
Uses of Local Anesthesia ♦♦ Suppuration: When the pyogenic bacteria causing acute
♦♦ In dentistry: Surface or infiltration anesthesia is used dur- inflammation result in severe tissue necrosis, the process
ing restorative procedures or extractions. Regional nerve progresses to suppuration. Initially, there is intense neu-
trophilic infiltration. Subsequently, mixture of neutrophils,
blocks are used in oral surgical procedures.
bacteria, fragments of necrotic tissue, cell debris and fibrin
♦♦ Ophthalmic surgeries: Surface anesthesia and topical
comprise pus which is contained in a cavity to form an
anesthetics are used. In eyes, retrobulbar block can also abscess. The abscess, if not drained, may get organized by
be given. dense fibrous tissue, and in time, get calcified.
♦♦ In head and neck surgery: Field block, peripheral nerve block ♦♦ Chronic inflammation. Persisting or recurrent acute inflam-
and plexus anesthesia is used. mation may progress to chronic inflammation in which
♦♦ During heart and lung surgeries: Epidural anesthesia is the processes of inflammation and healing proceed side
combined with the general anesthesia. by side.
470   Mastering the BDS IIIrd Year  (Last 25 Years Solved Questions)

MULTIPLE CHOICE QUESTIONS


As per Dci and Examination Papers 1 Mark Each
of Various Universities

1. Adamantinoma or ameloblastoma may be seen in all c. 6 years to 8 years of age


except: d. At puberty
a. Odontogenic epithelium 8. Sjögren’s syndrome is characterized by all except:
b. Femur Bone a. Xerostomia
c. Tibia Bone b. Keratoconjunctivitis
d. Stalk of pituitary c. Rheumatic arthritis
2. The most common tumor in the minor salivary gland, d. Enlargement of salivary gland
which spreads through neural sheath is: 9. The X-ray of Jaw shows “Honeycomb” appearence, the
a. Acinar cell carcinoma diagnosis is:
b. Adenoid cystic carcinoma a. Granulomatous epulis
c. Mucoepidermoid carcinoma b. Osteoclastoma
d. Pleomorphic adenoma c. Ameloblastoma
3. In submandibular gland surgery, the nerve least likely d. Dentigerous cyst
to be injured is: 10. The most common cause of thyroiditis is:
a. Hypoglossal nerve a. Riedle’s thyroiditis
b. Lingual nerve b. Hashimoto’s thyroiditis
c. Inferior alveolar nerve c. Subacute thyroiditis
d. Mandibular branch of facial nerve d. Cervicofacial actinomycosis
4. Hyperparathyroidism is characterized by the following 11. The skin grafting is not done in infection with:
except:
a. Pseudomonas
a. Generalized osteoporosis
b. E. coli
b. Osteosclerosis
c. β-hemolyticus streptococci
c. Hypercalcemia
d. Staphylococci
d. Renal calculus
12. The most common presenting complaint of medullary
5. True about Warthin tumor of salivary gland is: carcinoma of thyroid is:
a. Malignant neoplasm a. Diarrhea
b. Rapidly growing tumor b. Dysphagia
c. Gives a “Hot” pertechnetate scan c. Flushing
d. Gives a “Cold” pertechnetate scan d. Hoarseness of voice
6. In fracture, mandible osteosynthesis or rigid fixation 13. What is the advantage of chromic catgut over plain
is done by all except: catgut suture in deep wounds?
a. Bone grafting a. Greater strength
b. Miniplates b. Greater ease to use
c. Leg screws c. Delayed resorption
d. Arch bar d. Less tissue reaction
7. The cleft lip is best repaired at the age of: 14. An allograft is transfer of tissue between:
a. 5 months a. Sister to brother
b. One year of age b. Unrelated donor

Answers: 1. b 2. b 3. c 4. b
5. c 6. a 7. a 8. d
9. c 10. b 11. c 12. a
13. d 14. a
Section 2:  General Surgery  471

c. Same individual 22. Tissue is best


d. Monozygotic twins a. Antibiotic
15. Hyperparathyroidism is characterized by following b. Antiseptic
except: c. Meat
a. Generalized osteoporosis d. Culture
b. Hypercalcemia 23. Rh-incompatibility in pregnant woman causes:
a. Neonatal jaundice
c. Osteosclerosis
b. Placenta previa
d. Renal stone formation
c. Hydrops fetalis
16. Ameloblastoma may be seen in all except:
d. Acute renal failure
a. Jaws 24. Hemophilia is due to congenital deficiency of:
b. Pituitary stalk a. Factor III
c. Tibia b. Factor VII
d. Hip bone c. Factor IX
17. The papillary carcinoma of thyroid having all except: d. Factor VIII
a. Most common thyroid cancer 25. Hepatitis B virus is:
b. Psammoma bodies seen a. DNA virus
c. Encapsulated b. RNA virus
d. Blood borne metastasis is common c. Flavi virus
18. In cleft lip operation, all the stitches are removed on: d. Incomplete RNA virus
a. 2nd day 26. Edge of a malignant ulcer is:
a. Raised
b. 4th to 5th day
b. Undermined
c. 10th day
c. Sloping
d. 14th day d. Everted
19. A patient presents with neck swelling and respiratory 27. Most rapid diagnosis of tuberculosis can be done with:
distress few hours after thyroidectomy, the next man­
a. Polymerase chain reaction
agement would be:
b. ZN staining
a. Open immediately
c. Bacterial culture
b. Tracheostomy d. Tuberculin test
c. Wait and watch 28. Most suitable antibiotic in gas gangrene is:
d. Oxygen by mask a. Cloxacillin
20. In facial injury with voluntary control lost over tongue, b. Penicillin
the best emergency treatment to prevent tongue from c. Gentamicin
falling back is by: d. 3rd generation cephalosporin
a. Definitive treatment 29. Most ancient plastic surgery was performed in
b. Oropharyngeal airway a. Greece
c. Towel clipping of tongue b. Rome
d. Deep traction suture of tongue c. Egypt
21. Skin is best: d. India
a. Dressing 30. HIV-1 virus is considerably less infective than
b. Cosmetic Hepatitis-B:
a. True
c. Layer
b. False
d. Cover

Answers: 15. c 16. d 17. d 18. b


19. b 20. c 21. d 22. b
23. c 24. c 25. a 26. d
27. b 28. b 29. d 30. a
472   Mastering the BDS IIIrd Year  (Last 25 Years Solved Questions)

31. FNAC is useful in all thyroid carcinomas except: 38. The most common site of thyroglossal cyst is:
a. Papillary a. Suprahyoid
b. Follicular b. Subhyoid
c. Medullary c. Floor of mouth
d. Anaplastic d. At the level of cricoid cartilage
32. The open methods of reduction and immobilization 39. Common type of cleft lip is:
of fracture body of mandible are all except: a. Mid line
a. Miniplates b. Unilateral
b. Transosseous wiring c. Bilateral
c. Lag screws
d. Cleft lip combined with cleft palate
d. Arch bar
40. Immediate management of a patient with multiple
33. Hormones produced by medullary carcinoma of fracture and fluid loss includes the infusion:
thyroid are:
a. Blood
a. Calcitonin
b. Dextran
b. Prostaglandins
c. Normal saline
c. Serotonin
d. Ringer lactate
d. All a, b and c
41. What is cognizable medicolegal situation?
e. None
a. Fracture
34. A patient with a fistula and chronic pus discharge from
lower face and mandible is most commonly suffering b. Abrasion
from: c. Ecchymosis
a. Dental cyst 42. Surgical incision heal by:
b. Vincent angina a. First intension
c. Ludwig angina b. Second intension
d. Actinomycosis 43. Sulphur granules are seen in pus of:
35. Axonotmesis is: a. Carbuncle
a. Rupture of nerve fibers in an intact sheath b. Abscess
b. Rupture of nerve sheath only c. Actinomycosis
c. Rupture of nerve fibers and nerve sheath d. Erysipelas
d. Physiological paralysis of nerve fibers 44. Torniquet is the best treatment for bleeding venous
36. ‘Tetany’ is caused by all except: ulcer in lower limb:
a. Hypoparathyroidism a. True
b. Rickets b. False
c. Osteomalacia 45. Normally, cyst is diagnosed by eliciting fluctuation
d. Hyperparathyroidism test:
37. The most common tumor of minor salivary gland is: a. True
a. Mucoepidermoid b. False
b. Acinic cell carcinoma 46. Hypertrophied scar usually is result of healing by:
c. Adenoid cystic carcinoma a. Second intension
d. Pleomorphic adenocarcinoma b. First intension

Answers: 31. b 32. b 33. a 34. d


35. c 36. d 37. a 38. b
39. b 40. d 41. a 42. a
43. c 44. b 45. a 46. a
Section 2:  General Surgery  473

47. Hemophilia is transferred genetically by: c. Ameloblastoma


a. Autosomes d. Dentigerous cyst
b. Sex chromosomes 55. Skin grafting is absolutely contraindicated in which
c. Mitochondria infection:
d. Golgi complex a. Staphyloccoccus
48. Involvement of lymph nodes by secondaries from a b. Pseudomonas
cancer is: c. Proteus
a. Stoney hard d. Streptococcus hemolyticus
b. Soft
56. The structure preserved in functional block dissection
c. Firm in papillary carcinoma thyroid are all except:
d. Fluctuant a. Sternomastoid muscle
49. The edge of tubercular ulcer is:
b. Internal jugular vein
a. Everted
c. Spinal accessory nerve
b. Raised
d. Enlarged lymph nodes
c. Undermined
57. The swelling in neck moves with deglutition are all
d. Sloping
except:
50. Universal blood donor belongs to blood group:
a. Thyroglossal cyst
a. A
b. Subhyoid bursitis
b. B
c. Pretracheal lymph nodes
c. AB
d. Branchial cyst
d. O
58. Highly transilluminated, cystic and compressible
51. Unilateral cleft lip is more common on:
swelling in posterior triangle of neck is:
a. Right side
a. Cystic hygroma
b. Left side
b. Branchial cyst
c. Median
c. Thyroglossal cyst
d. None of the above
52. Which type of thyroid cancer has the best prognosis? d. Dermoid cyst
a. Papillary 59. Recurrent laryngeal nerve is in close relation or associ­
ated with:
b. Follicular
c. Medullary a. Superior thyroid artery
d. Anaplastic b. Inferior thyroid artery
53. The most common form of actinomycosis is: c. Superior thyroid vein
a. Thoracic d. Inferior thyroid vein
b. Fasciocervical 60. Advantages of vicryl suture material over chromic
c. Liver catgut are all except:
d. Right iliac fossa a. Delayed resorption
54. Radiologically, the ‘Honeycomb’ appearance is seen in: b. Less tissue reaction
a. Myeloid epulis c. Greater strength
b. Granulomatous epulis d. Cheap then chromic

Answers: 47. b 48. a 49. c 50. d


51. b 52. a 53. b 54. c
55. d 56. d 57. d 58. a
59. b 60. a
474   Mastering the BDS IIIrd Year  (Last 25 Years Solved Questions)

61. Open methods of treatment of fracture mandible are 68. All are the pulsating swellings except:
all except: a. Plexiform hemangioma
a. Miniplates b. Carotid aneurysm
b. Transosseous wiring c. Primary thyrotoxicosis
c. Bone plating d. Sternomastoid tumors
d. Arch bar 69. Early multiple painful ulcers on tongue are seen in all
62. The cysts containing cholesterol crystals are all except: except:
a. Periapical cyst a. Aphthous ulcers
b. Dentigerous cyst b. Carcinomatous ulcer
c. Branchial cyst c. Tubercular
d. Dermoid cyst d. Herpes
63. A patient has lacerated untidy wound in leg and at­ 70. Incisional or open biopsy should not be taken from all
tended the casualty after two hours. The wound should except:
be:
a. Malignant melanoma
a. Sutured immediately
b. Parotid gland
b. Debrided and sutured immediately
c. Tongue and cheek
c. Debrided and sutured secondarily
d. Thyroid gland
d. Clean and dress only
71. Admantinoma or ameloblastoma may be seen in all
64. A 80-year-old edentulous patient with midline tumor
except:
of lower jaw involving alveolar margin the treatment
of choice is: a. Odontogenic epithelium
a. Hemimandibulectomy b. Femur bone
b. Marginal mandibulectomy c. Tibia bone
c. Commando’s operation d. Stalk of pituitary
d. Segmental mandibulectomy 72. The X-ray of jaw shows ‘honeycomb’ appearance:
65. Not a feature of De Quervains disease: a. Granulomatous epulis
a. Autoimmune etiology b. Osteoclastoma
b. Increased ESR c. Ameloblastoma
c. Tends to regress spontaneously d. Dentigerous cyst
d. Painful and associated with enlargement of thyroid 73. In fracture mandible osteosynthesis or rigid fixation
66. Highest chances of malignancy in oral cavity is due to: is done by all except:
a. Leukoplakia a. Bone grafting
b. Lichen planus b. Miniplates
c. Erythroplakia c. Leg screw
d. Aphthous ulcers d. Arch bar
67. Which of the following causes maximum bleeding? 74. The most common site of thyroglossal cyst is:
a. Partial arterial tear a. Suprahyoid
b. Complete arterial tear b. Subhyoid
c. Artery caught between fractured ends of bone c. Floor of the mouth
d. Intimal tear d. At the level of cricoids cartilage

Answers: 61. d 62. a 63. b 64. d


65. a 66. c 67. a 68. d
69. b 70. c 71. d 72. c
73. a 74. b
Section 2:  General Surgery  475

75. The papillary carcinoma thyroid is having all except: 82. Biopsy can be taken with as needle:
a. Most common thyroid cancer a. True
b. Psammoma bodies are seen b. False
c. Encapsulated 83. Abscess should always be sutured closed after drain­
d. Blood-borne metastasis is common age:
76. In cleft lip operation all stitches after operation are a. True
removed on: b. False
a. 3rd day 84. Skin should not be sutured by non-absorbable
b. 4th to 5th day sutures:
c. 10th day a. True
d. 14th day b. False
77. A 14-years-old child presented with progressive cer­
85. Heat sensitive surgical items can be best sterilized in
vical lymph node enlargement since 3 month. The
diagnosis can be achieved by: a well-equipped OT by:
a. Formalin chambers
a. X-ray soft tissues of neck
b. CBP ESR b. 2% glutaraldehyde
c. Lymph node biopsy c. Ethylene oxide
d. CT scan of neck d. Ionizing radiation
78. Structure preserved in modified radical neck dissection 86. One of the following is an example of painless midline
in: neck swelling:
a. Internal jugular vein a. Branchial cyst
b. Sternomastoid muscle b. Thyroglossal cyst
c. XI nerve c. Cystic hygroma
d. X nerve d. Carotid body tumor
79. Advantages of vicryl suture material over chromic
87. Jaw tumors associated with unerupted tooth is:
catgut are all except:
a. Dental cyst
a. Delayed resorption
b. Dentigerous cyst
b. Less tissue reaction
c. Admantinoma
c. Greater strength
d. Giant cell granuloma
d. Cheaper than chromic
80. The swelling in neck moves with deglutition are all 88. Hyperparathyroidism is characterized by the following
except: except:
a. Generalized osteoporosis
a. Thyroglossal cyst
b. Osteosclerosis
b. Subhyoid bursa
c. Hypercalcemia
c. Pretracheal lymph nodes
d. Renal calculus
d. Branchial cyst
81. Hand of surgeon can be rendered aseptic by 89. The cleft lip is repaired at the age of:
washing: a. 5 months
a. True b. 1 year ago
b. False c. 6–8 years
d. At puberty

Answers: 75. d 76. b 77. c 78. c


79. a 80. c 81. b 82. a
83. a 84. b 85. c 86. b
87. b 88. b 89. a
476   Mastering the BDS IIIrd Year  (Last 25 Years Solved Questions)

90. Most common cause of thyroiditis is: b. LMA


a. Riedel’s thyroiditis c. Nasotracheal tube with cuff
b. Hashimoto’s thyroiditis d. Endotracheal tube
c. Subacute thyroiditis 98. Knife used for harvesting split thickness skin graft is:
d. Viral thyroiditis a. Husson’s knife
91. FNAC is useful in all thyroid carcinoma except: b. Bard Parker knife
a. Papillary c. Humby’s knife
b. Follicular d. Foley’s knife
c. Medullary 99. Rapid infusion of blood causes:
d. Anaplastic a. Acute left heart failure
92. Immediate management of patient with multiple frac­ b. Pulmonary edema
ture and loss of blood with shock is the infusion of: c. Ankle edema
a. Dextrosaline d. Respiratory distress
b. Dextran
100. The best vein for total parenteral nutrition is:
c. Blood a. Subclavian vein
d. Ringer’s lactate b. Femoral
93. Bone metastasis is common in which type of thyroid c. Brachial vein
cancer: d. Saphenous
a. Medullary
101. In the blood bank platelets are stored at:
b. Follicular
a. 180C for 1 year
c. Papillary
b. 2 to 40C for 35 days
d. Anaplastic
c. 20 to 240C for 3 to 5 days
94. The most common type of cleft lip is: d. 20 to 250C for 35 days
a. Unilateral
102. The best dressing for open wounds is:
b. Bilateral
a. Skin
c. Midline
b. Amnion
95. A patient has wide eyes, nervousness, raised systolic c. Opsite
BP and weight loss. Most probable diagnosis is:
d. Tulle grass
a. Hypothyroidism
103. Reactionary hemorrhage occurs:
b. Hyperthyroidism
a. Within 24 hours of surgery
c. Hyperparathyroidism
b. After 48 hours of surgery
d. Hypoparathyroidism
c. After 72 hours of surgery
96. Collar stud abscess occurs in:
d. After 7 to 14 days of surgery
a. Cervical TB lymphadenitis
104. The treatment of choice for stage I cancer larynx:
b. Peritonsillar abscess
a. Radical surgery
c. Pyogenic lymphadenitis
b. Chemotherapy
d. Retropharyngeal abscess
c. Radiotherapy
97. In long surgical procedure intubation method:
d. Surgery followed by radiotherapy
a. Nasotracheal tube with cuff

Answers: 90. b 91. b 92. d 93. c


94. d 95. b 96. a 97. d
98. c 99. b 100. a 101 c
102. a 103. a 104. c
Section 2:  General Surgery  477

105. In an emergency tracheostomy incision is: b. Sebaceous cyst


a. Vertical c. Branchial cyst
b. Oblique d. Thyroglossal cyst
c. Horizontal 113. Ranula is a condition which occurs in:
d. Paramedian a. Preauricular area
106. Punched out edge is characteristic of ulcer: b. Nasolabial angle
a. Tuberculosis c. Medial angle of orbit
b. Rodent ulcer d. Floor of the mouth
c. Syphilitic 114. Commonest premalignant lesion of oral cancer:
d. Non – specific a. Leukoplakia
107. Loss of differentiation of tumor cells is called as: b. Syphilitic ulcer
a. Metaplasia c. Apthous ulcer
b. Dysplasia d. Erythroplakia
c. Anaplasia 115. Undermined edge is the characteristic of:
d. Hyperplasia a. Rodent
108. The muscle which is resected in classical neck dissec­ b. Syphilitic
tion is: c. Tuberculosis
a. Sternohyoid d. Squamous cell carcinoma
b. Sternomastoid
116. Duct of parotid gland is known as:
c. Sternothyroid a. Stenson’s duct
d. Sternocricoid b. Wharton’s duct
109. Painless ulcer of tongue is due to: c. Nasolacrimal duct
a. Dysplasia d. Bartholin’s duct
b. Syphilis
117. Largest number of lymph nodes are present in which
c. Tuberculosis area:
d. None of the above a. Axilla
110. A patient having tumor of maxillary sinus manifest b. Groin
with excessive rolling tears from his eye, likely due to c. Neck
obstruction of: d. Abdomen
a. Nasolacrimal duct
118. Marjolin’s ulcer is:
b. Conjunctival sac
a. Wet gangrene
c. Lacrimal gland
b. Dry gangrene
d. None of the above
c. Premalignant
111. Basal cell carcinomas: d. Malignant
a. Usually metastatize to regional lymph nodes
119. Which of the following is congenital:
b. Common on the face and neck
a. Sebaceous cyst
c. Common in females
b. Hypothyroidism
d. Radioresistant
c. Cystic hygroma
112. All the following are congenital cysts except: d. Vincent’s angina
a. External angular dermoid cyst

Answers:
105. c 106. c 107. c 108. b
109. a 110. a 111. b 112. b
113. d 114. a 115. c 116. a
117. c 118. d 119. c
478   Mastering the BDS IIIrd Year  (Last 25 Years Solved Questions)

120. Bell’s palsy occurs due to injury to: d. Anaplastic type


a. Recurrent laryngeal nerve 128. There was sudden increase in the size of thyroid swell­
b. Facial nerve ing along with pain. Most likely cause is:
c. Trigeminal nerve a. Malignant change
d. Ulnar nerve b. Nodular goiter
121. Most common cyst for thyroglossal cyst is: c. Hemorrhage within the thyroid cyst
a. Suprahyoid d. Colloid goiter
b. Subhyoid 129. Best dressing for open wound is:
c. Level of cricoids a. Skin
d. Floor of mouth b. Amnion
122. Which neck swelling moves with swallowing: c. Opsite
a. Submandibular cyst d. Tulle grass
b. Thyroid cyst 130. Cystic compressible swelling in the posterior triangle
c. Supraclavicular lymph node of neck which is translucent is:
d. Sternomastoid tumor a. Cystic hygroma
123. All are true about cystic hygroma except: b. Branchial cyst
a. Pulsatile c. Thyroglossal cyst
b. May cause respiratory obstruction d. Dermoid cyst
c. Common in neck 131. Sistrunk’s operation is done in:
d. Brilliant translucent a. Parotid tumor
124. In management of thyroglossal cyst: b. Thyroglossal fistula
a. Central portion of hyoid excised c. Branchial cyst
b. Sternothyroid muscle dissected d. Cystic hygroma
c. Isthmusectomy 132. Complications of total thyroidectomy include all ex­
d. Strap muscles excised cept:
a. Airway obstruction
125. FNAC is useful in all the following tumors of thyroid
except: b. Hoarsness
a. Papillary carcinoma c. Hemorrhage
b. Anaplastic carcinoma d. Hypercalcemia
c. Thyroiditis 133. Hyperparathyroidism is characterized by the following
d. Follicular carcinoma except:
a. Generalized osteoporosis
126. Everted edge is characteristic of………….ulcer:
a. TB b. Renal calculi
b. Malignant c. Hypercalcemia
c. Syphilitic d. Osteosclerosis
d. Healing 134. After thyroidectomy patient developed stridor within
two hours. All are likely cause of stridor except:
127. Most common histological type of thyroid carcinoma
a. Hypocalcemia
is:
a. Medullary type b. Recurrent laryngeal nerve palsy
b. Follicular type c. Laryngomalacia
c. Papillary type d. Wound hematoma

Answers:
120. b 121. b 122. b 123. a
124. a 125. d 126. b 127. c
128. a 129. a 130. a 131. b
132. d 133. d 134 a
Section 2:  General Surgery  479

FILL IN THE BLANKS


As per Dci and Examination Papers 1 Mark Each
of Various Universities

1. Duct of parotid gland is known as …………….... 17. The organ most sensitive to hypovolemic shock is
Ans. Stensen’s duct ……………....
2. Duct of submandibular gland is known as …………….... Ans. Brain, heart and lung
Ans. Wharton’s duct 18. Mandible is most commonly fractured at ……………....
3. Branches of facial nerve are (any three) ……………., Ans. Condyles
…………………. And ………………. 19. Undermined edge is seen in …………….... ulcer.
Ans. Temporal, zygomatic and buccal Ans. Tuberculous ulcer
4. Two premalignant lesions are …………….... and 20. Submandibular gland lies on …………….... muscle.
…………….... Ans. Hyoglossus
Ans. Leukoplakia and erythroplakia 21. The most commonly used absorbable suture material
5. The most Common site of thyroglossal cyst is for closure of intra-oral wound is ……………....
…………….... Ans. Catgut
Ans. Subhyoid 22. The most common site of enlargement of lymph nodes
6. FNAC cannot distinguish between a benign ……………. in Hodgkin’s lymphoma is ……………....
and …………….... carcinoma of thyroid. Ans. Cervical region
Ans. Follicular adenoma and follicular 23. The nerve most likely to get injured in parotid gland
7. The triad of thyroid swelling (Goiter), thyrotoxicosis surgery is ……………....
and exopthalmosis is seen in …………….... Ans. Facial nerve
Ans. Grave’s disease 24. Father of Antiseptic surgery is ……………....
8. The swelling which moves on deglutition and also a Ans. Joseph Lister
protrusion of tongue is …………….... 25. Commonest site for carcinoma tongue is ……………....
Ans. Thyroglossal cyst Ans. Anterior two third part
9. Infection in floor of mouth along with inflammatory 26. The most common indication for doing tracheostomy
swelling in the neck is called as …………….... is ……………....
Ans. Ludwig’s angina Ans. In head, neck and facial injuries
10. The most commonly used local anesthetic agent is 27. Father of Plastic Surgery is ……………....
…………….... Ans. Sir Harold Gillies
Ans. Lignocaine 28. True or False: Antisepsis is better than asepsis
11. The term ‘Ubiquitous tumor’ is usually referred to ……………....
…………….... Ans. False
Ans. Fibrous tumor 29. Most common method of instrument sterilization is
12. The line of demarcation is usually seen between a liv­ ……………....
ing zone and dead zone is …………….... Ans. Autoclave
Ans. Gangrene 30. True/False: Cancrum oris is often associated with mal­
13. Wash leather slough appearance of floor of ulcer is seen nutrition
in …………….... ulcer. Ans. True
Ans. Syphilitic 31. Tissue is best antiseptic. Yes or No
14. The abbreviation FNAC stands for …………….... Ans. Yes
Ans. Fine Needle Aspiration Cytology 32. Metabolism of Ca 2+ and PO 4- is regulated by……
15. CSF rhinorrhea is seen in …………….... fossa fractures. ……………….. hormone
Ans. Anterior Ans. Parathyroid
16. A patient with multiple discharging sinuses along with 33. Biopsy and histopathological examination achieves
faciocervical involvement is most probably suffering final diagnosis of a lump. True/False
from …………….... Ans. True
Ans. Cervicofacial actinomycosis
480   Mastering the BDS IIIrd Year  (Last 25 Years Solved Questions)

34. Medullary thyroid cancer can cause diarrhea. True/ 53. Tetanus has both tonic and clonic convulsions. (True
False or false)
Ans. True Ans. True
35. Cacinomatous masses are often hard. True/False 54. Metastatic lymph nodes have …………….... consistency.
Ans. True Ans. Stony hard
36. Most common salivary gland tumor is …………….... 55. Aphthous ulcers are painful and multiple. (True or false)
Ans. Pleomorphic adenoma Ans. True
37. Universal blood donor group is …………….... and 56. Primary healing achieves better function than second­
universal blood recipient group is …………….... ary healing. (True or false)
Ans. O negative, AB positive Ans. True
38. Most common bacteria involved in abscess formation 57. Skin grafting can spoil the cosmetic appearance. (True
is …………….... or false)
Ans. Staphylococcus aureus Ans. True
39. The difference between sinus and fistula is ………… 58. Carbuncles are seen commonly in nondiabetics. (True
Ans. Fistula a pathway that leads from an internal cavity or false)
or organ to the surface of the body. A sinus tract is an Ans. False
abnormal channel that originates or ends in one opening. 59. Mothers are carriers and fathers are sufferer in …………
40. Consistency of a malignant tumor on palpation is Ans. Hemophilia
…………….... 60. Universal tumor is the name given to …………
Ans. Hard Ans. Lipoma
41. Catgut is manufactured from …………….... 61. Neurotmesis involves rupture of both…………….... and
Ans. Submucosa of jejunum of sheep ……………....
42. Most commonly employed method of sterilization in Ans. Neuron sheath and axons
OT is …………….... 62. Line of demarcation is the feature of ……………....
Ans. UV radiation or formaldehyde gangrene
43. Edges of malignant ulcer is …………….... Ans. Wet
Ans. Everted 63. Discoverer of system of human blood groups …………
44. ‘Slipping sign’ is observed in …………….... Ans. Landsteiner
Ans. Lipoma 64. Edge of tubercular ulcers is……………....
45. A line of demarcation is usually seen between living Ans. Undermined
and dead zone in …………….... 65. Consistency of metastatic lymph nodes is …………
Ans. Gangrene Ans. Hard
46. …………….... discovered germ theory of diseases. 66. Plastic surgery was invented in …………....(country).
Ans. Louis Pasteur Ans. India
47. …………….... performed first rhinoplasty. 67. Venous bleeding is ………….... in color and
Ans. Joseph Constantine Carpue flows……………....
48. …………….... pioneered aseptic surgery Ans. Dark in color and flows continuously
Ans. Joseph Lister 68. In lymphoma lymph nodes are …………….... in consist­
49. Rodent ulcer edge is…………….... ency.
Ans. Beaded Ans. Rubbery
50. The first sign in hypovolemic shock is…………….... in 69. Sharp instruments should not be sterilize by
pulse. ……………....
Ans. Increase Ans. Boiling
51. FNAC achieves better diagnosis than biopsy. (True or 70. ……………... suffer from hemophilia while ……………...
false) transmit it.
Ans. False Ans. Males suffer from hemophilia while females transmit it
52. Cancrum oris is generally associated with malnutrition. 71. Calcium and phosphate in blood and bones are regu­
(True or false) lated by ……………....
Ans. True Ans. Calcitonin
Section 2:  General Surgery  481

72. …………………. is the most common site for keloid. 86. Tuberculous ulcer has a punched out edge. True/False
Ans. Nape of the Neck Ans. False
73. …………………..is known as hydrocele of neck. 88. Thyroglossal cyst moves on deglutition. True/False
Ans. Serous cyst Ans. True
74. Duct of parotid gland is known as…………….. 89. Consistency of tuberculous lymph node is hard. True/
Ans. Stensön’s duct False
Ans. False
75. Branches of facial nerve are …………….. and…………..
(any two). 90. Granulomatous epulis is……………….in consistency.
Ans. Temporal and mandibular Ans. Soft
76 Hilton method is used for the treatment of 91. 5 cardinal signs of inflammation…………….
…………………. Ans. Rubor, Calor, Tumor, Dolor, Functio laesa
Ans. Abscess 92. Gas gangrene is caused by……………………(Name of
77. Boil is also called as furuncle. True/False organism)
Ans. True Ans. Clostridium perfringens
78. Clean-incised surgical wound is healed by primary 93. ……………..cranial nerves are affected in plexiform
intension. True/False neurofibromatosis.
Ans. True Ans. 5th, 9th and 10th
79. Cold abscess is caused by tuberculosis. True/False 94. Punched out edges are seen in………………….ulcer.
Ans. True Ans. Syphilitic
80. Abscess of peritonsillar space is called as………………. 95. Contusion is a type of closed wound. (True/False)
Ans. Quinsy Ans. True
81. Cystic hygroma is a brilliantly transilluminant swelling. 96. Keratocanthoma arising from………………
True/False Ans. Pilosebaceous glands
Ans. True 97. Ranula is a mucus retention cyst……………..(True/
82. Edge of squamous cell carcinoma is………………… False)
Ans. Everted Ans. True
83 Repair of cleft lip should be done at the age of 98. In autologous transfusion, patient’s own blood is used
………………… ……………….(True/False)
Ans. 3 to 6 months Ans. True
84. Cellulitis is commonly caused by the microorganism 99. Crushed wounds are closed after debridement. True/
………………… False
Ans. Streptococcus pyogenes Ans. False
85. Hemophilia is caused by deficiency of……………. 100. PMMC flap is supplied by…………………
Ans. Factor VIII Ans. Thoracoacromial artery and its venae comitantes
482   Mastering the BDS IIIrd Year  (Last 25 Years Solved Questions)

Viva-Voce Questions for


Practical Examination

1. What is an abscess? 16. What do you mean by risus sardonicus.


Ans. It is a cavity which is filled with pus and is lined by a Ans. It is the anxious expression of the patient suffering from
pyogenic membrane. the tetanus.
2. What is pyemic abscess? 17. Which is the most common form of actinomycosis?
Ans. In this, multiple abscesses are formed from the infected Ans. Cervicofacial actinomycosis
emboli in pyemia. 18. In which disease, Leonine facies are seen.
3. What is pyemia? Ans. Lepromatous leprosy
Ans. It is the condition where there is formation of secondary 19. Name the peripheral nerve which is commonly affected
foci of suppuration in various body parts. in leprosy.
4. What is bacteremia? Ans. Ulnar nerve
Ans. It is the condition where the bacteria circulate in blood 20. What is carbuncle.
stream. Ans. Carbuncle is the infective gangrene of skin and
5. What is septicaemia? subcutaneous tissue.
Ans. It is the condition where multiple clinical manifestations 21. What is cellulitis?
arise as toxins are liberated by the bacteria in the blood Ans. It is the condition in which there is nonsuppurative
stream. spreading inflammation of subcutaneous and fascial
6. What is toxemia? planes.
Ans. It is the condition where toxins either derived from 22. Name the microorganism which leads to cellulitis.
bacteria or from chemical circulate in the bloodstream. Ans. Streptococcus pyogenes
7. What is the another name of gas gangrene? 23. What is erysipelas?
Ans. Clostridial myonecrosis Ans. It is the spreading inflammation of the skin and
8. Gas gangrene comes under which type of gangrene? subcutaneous tissues due to the infection caused by
Ans. Moist gangrene Streptococcus pyogenes.
9. What is the most characteristic feature of gas gangrene? 24. Which is the characteristic sign of erysipelas?
Ans. There is presence of profuse discharge of brown, foul Ans. Milian’s ear sign.
smelling fluid in between sutures and crepitus is also 25. Define cyst.
present. Ans. A pathological cavity having fluid, semifluid or gaseous
10. Name various organisms which leads to gas gangrene. contents and which is not created by accumulation of
Ans. Clostridium welchii, Clostridium septicum, Clostridium pus.
histolyticum, Clostridium oedematiens 26. What is a fracture?
11. Name the exotoxins released by Clostridium tetani? Ans. It is the loss of continuity of bone.
Ans. Tetanospasmin and Tetanolysin 27. Name the fracture occurring in the children.
12. How much is the incubation period of tetanus. Ans. Greenstick fracture
Ans. It is from 3 days to 3 weeks. 28. What is callus?
13. What is Tetanus neonatorum? Ans. It is the new bone formed at the site of fracture.
Ans. It is the contamination of the cut surface of umbilical cord 29. What is neuropraxia?
of neonate which causes spasm of respiratory muscles. Ans. It is the temporary physiological paralysis of nerve
14. What is latent tetanus? conduction.
Ans. At times when wound are ignored Clostridium tetani 30. What is axonotmesis?
rest in them for months and years but when suitable Ans. It is the division of nerve fibers or axons with intact nerve
conditions come the bacteria multiply and tetanus sheath.
develops. 31. What is neurotmesis?
15. Which is the earliest symptom of tetanus? Ans. In this, there is complete division of nerve fibers with
Ans. Trismus sheath occurs.
Section 2:  General Surgery  483

32. What is an ulcer? 52. How are the lymph nodes in Hodgkin’s lymphoma?
Ans. It is the break in continuity of the epithelium. Ans. Rubbery and elastic
33. Name the parts of an ulcer. 53. How is the edge of ulcer in syphilis and aphthous ulcer.
Ans. Edge, margin, floor and base. Ans. Punched out
34. Which disease shows rolled out ulcer? 54. Name type of ameloblastoma which has maximum
Ans. Squamous cell carcinoma and adenocarcinoma chances of turning into malignancy.
35. Name the disease which consists of undermined ulcer. Ans. Granular cell ameloblastoma
Ans. Tuberculosis 55. Name the disease in which tracheostomy is mandatory.
36. What is cleft palate? Ans. Ludwig’s angina
Ans. Cleft palate occur due to defect in fusion of lines between 56. While performing tracheostomy, which is the site of
premaxilla and palatine processes of maxilla one on each entrance inside the trachea.
side.
Ans. Second and third tracheal ring is the site of entrance
37. What is cleft lip?
57. Name the disease in which Trottler’s triad is seen.
Ans. Cleft lip occurs due to defect in fusion of median nasal
process along with maxillary process. Ans. Nasopharyngeal carcinoma
38. Name the surgical treatment which is used for unilat­ 58. Name the most common complication which arises
eral cleft lip repair. after doing the tracheal intubation.
Ans. Millard advancement flap Ans. Sore throat is the most common complication.
39. At what age cleft lip repair is done? 59. Which is the major advantage of tracheostomy?
Ans. Between 3 to 6 months Ans. The procedure increases the dead space upto 50%.
40. What is rule of ten. 60. What is the another name of Ringer’s lactate solution?
Ans. When baby is of 10 lbs weight, 10 week old and has 10 Ans. Hartmann’s solution.
gm% of hemoglobin cleft lip repair is done. 61. At how much temperature does the blood is stored in
41. Name the prosthetic device which covers palatal defects the blood bank.
in patients having cleft palate. Ans. 4°C ± 2°C
Ans. Obturator
62. How much is the shelf life of stored blood in blood
42. At what age cleft palate repair has to be done. banks.
Ans. Between 12 to 15 months of age. Ans. 3 weeks
43. Name the classification which is represented symboli­ 63. At how much temperature, does whole blood is stored
cally as Y for cleft lip and cleft palate.
in the blood bank.
Ans. Kernahan classification
Ans. 4 to 8°C
44. Which is the tumor known as universal tumor or ubiq­
64. At what temperature does fresh frozen plasma is stored.
uitous tumor?
Ans. Lipoma Ans. –40°C to –50°C
45. Which is the most common site of carcinoma of lip? 65. Name the method which is commonly used to stop the
bleeding.
Ans. Vermilion border of lip
Ans. Applying the pressure over the injured area.
46. Name the carcinoma which does not metastatize.
Ans. Basal cell carcinoma 66. In how much time after the surgery reactionary hemor­
rhage occur.
47. Which is the most common site for occurrence of car­
cinoma of tongue? Ans. In 24 hours of surgery.
Ans. Lateral border of tongue 67. Name the derivatives of plasma.
48. What is commando’s operation? Ans. Plasma, platelet rich plasma, fibrinogen, albumin,
Ans. In this, there is hemiglossectomy with the block cryoprecipitate.
dissection of the lymph nodes. 68. What are the synthetically prepared solutions?
49. Bedsore represents which type of an ulcer. Ans. Fluorocarbons, gelatin, dextran.
Ans. Trophic ulcer. 69. How much is the blood transfusion rate?
50. Which condition is known as quinsy? Ans. It is 1 unit for 4 to 6 hours
Ans. Peritonsillar abscess 70. Name the solution which is given to the patient who
51. How are the lymph nodes in oral carcinoma? get burn.
Ans. Hard and fixed lymph nodes Ans. Human albumin 4.5%
484   Mastering the BDS IIIrd Year  (Last 25 Years Solved Questions)

71. Which is the most common cause of the death in burn Ans. Stensen’s duct
patient. 88. What is the duct of submandibular duct known as.
Ans. Oligemic shock Ans. Wharton’s duct
72. Name some of the topical agents used in cases of burn. 89. Name the most common parotid gland tumor.
Ans. Silver nitrate, silver sulphadiazine, cerium nitrate Ans. Pleomorphic adenoma
73. What are plasma expanders? 90. What is an adenoma?
Ans. They are the high molecular weight substances which Ans. All the benign tumors which arises from the glandular
exert colloidal osmotic pressure and when they are epithelium are known as adenomas.
infused intravenously, they retain fluid in the vascular 91. Where is pleomorphic adenoma located in parotid
compartment, e.g. Dextran and human albumin gland?
74. What is shock? Ans. At the tail of parotid gland
Ans. Shock is a state of poor perfusion with impaired cellular 92. Name the salivary gland tumor which gets spread sur­
metabolism manifesting with severe pathophysiological rounding the nerves.
abnormalities. It is due to circulatory collapse and tissue
Ans. Adenoid cystic carcinoma
hypoxia.
93. What is the feature in diagnosis except biopsy which
75. Which type of graft is given in deep skin burns?
gives conformational diagnosis of Warthin’s tumor in
Ans. Split thickness graft. comparison to other salivary gland tumors.
76. What is an allograft? Ans. Warthin’s tumor shows hot spot in Scintiscan while other
Ans. It is the graft which is given from individual of same salivary tumors show cold spot.
species. 94. Name the nerve which cannot get preserved, while
77. What is a heterograft? doing the parotid surgery.
Ans. It is the graft taken from one specie to another species. Ans. Facial nerve
78. What is keloid? 95. Name the tumors which metastasize to the parotid
Ans. Keloid is the defect in maturation and stabilization of gland.
collage fibrils. In this normal collagen bundles are absent. Ans. Malignant melanoma and epidermoid carcinoma of skin
79. What is Buerger’s disease? 96. What is Sjögren’s syndrome?
Ans. Buerger’s disease is the inflammation and thrombosis Ans. It is the triad of xerostomia, Keratoconjunctivitis sicca
in small and medium-sized blood vessels, typically in and rheumatoid arthritis.
the legs and leading to gangrene. It has been associated 97. Name the route by which papillary carcinoma of thy­
with smoking roid spread and follicular carcinoma of thyroid spread.
80. Name the most common ulcer which occur in the leg. Ans. Papillary carcinoma spread via lymphatic route while
Ans. Venous ulcer follicular carcinoma spreads via hematogenous route.
81. Name the most common cause which leads to hypo­ 98. What is thyroid storm?
thyroidism. Ans. It is the thyrotoxic reaction which occurs under 3–4 days
Ans. Autoimmune thyroiditis after thyroid surgery. This occur, if the thyrotoxic patient
82. Name the cyst which moves while protruding the has not been brought down to euthyroid state before the
tongue. thyroid surgery.
Ans. Thyroglossal cyst 99. What is patey’s operation?
83. Where is thyroglossal cyst located? Ans. It is the superficial parotidectomy
Ans. Just near to the hyoid bone 100. What is goiter.
84. Name the thyroid malignancy which occur most com­ Ans. Goiter is the enlargement of the thyroid gland.
monly. 101. Name the goiter which has maximum chances of un­
Ans. Papillary carcinoma of thyroid. dergoing malignancy.
85. What is ranula? Ans. Nodular goiter
Ans. It is the cystic swelling which is present at the floor of 102. Name some of the goitrogens.
mouth and is the retention cyst of sublingual gland Ans. Cabbage, cauliflower, turnip, kale, sprouts. Antithyoid
86. Name the salivary gland in which there is maximum drugs, thiocyanate, etc.
chances of formation of stones. 103. In bilateral recurrent laryngeal nerve paralysis, what
Ans. Submandibular salivary gland. is the life-saving measure?
87. What is the duct of parotid gland? Ans. Immediate tracheostomy.
Section 2:  General Surgery  485

Additional Information

Thyroid Swellings on Scintiscanning Preference for hot --------- -5


Hot nodule It is overactive and takes up isotope but surrounding Signs Present Absent
gland does not. This also actively secrete excess Palpable thyroid +3 -3
thyroid hormone
Exopthalmos +2 ---------
Warm This is active, both swelling as well as normal thyroid
tissue takes up an isotope Lid retraction +2 ---------

Cold This is underactive and it does not take an isotope Finger tremor +1 ---------
Bruit over thyroid +2 -2

Various Surgical Procedures and Anatomical Structures Atrial fibrillation +4 ---------


Injured during the Procedures Pulse rate (>90/min) +3 ---------
Pulse rate (>80/min) ------- -3
Surgical procedure Anatomical structure injured
Parotidectomy Facial nerve
Brachial cyst excision Hypoglossal nerve and accessory
Various Signs of Thyrotoxicosis
nerve
Name of sign Description
Ranula excision Submandibular duct
Dalrymple’s sign Upper sclera is seen because of retraction of
Submandibular gland excision Lingual nerve, Hypoglossal nerve, upper eye lid
Marginal mandibular branch of
Joffroy’s sign As patient is asked to look upward with head
facial nerve
fixed, there is absence of wrinkling on forehead
Moebius sign Eye balls fail to converge
Various Hemorrhage and Arteries Involved in them Stellwag’s sign There is absence of normal wrinkling
Name of hemorrhage Artery involved Von Graffe’s sign As patient is asked to look down, his upper
eyelid fails to follow rotation of an eyeball and
Extradural Middle meningeal artery
so lags behind
Intracerebral bleed Lenticulostriate arteries
Subarachnoid Rupture of berry aneurysm
Types of Laryngeal Nerve Paralysis
Subdural Communicating veins or cortical bridging
veins Type of laryngeal Description
nerve paralysis
Unilateral It produces whispering voice. No airway
Various Thyroid Carcinomas and their Spread
obstruction present
Name of thyroid carcinoma Spread Bilateral It leads to dyspnea. Tracheostomy should
be done
Papillary carcinoma Lymphatic route
Superior It leads to paralysis of cricothyroid muscle.
Follicular carcinoma Hematogenous route
It leads to weak and husky voice
Anaplastic carcinoma Direct route
Medullar carcinoma Lymphatic route
Various Staphylococcal Infections with Their Description

Wayne’s Clinical Diagnostic Index for Thyrotoxicosis Staphyloccocal skin Description


infection
Symptoms Present Absent Boil Hair follicle infection which proceed to
Palpitations +2 ------ suppuration and central necrosis. It is
common on face, neck and head
Excessive sweating +3 ------
Carbuncle It is the infective gangrene of subcutaneous
Appetite increased +3 ------ tissue which occurs at nape of neck. There
Appetite decreased ------ -3 is presence of skin sloughing and discharge
of pus
Weight increased ------ -3
Impetigo It is a bulla which soon ruptures, erodes and
Weight decreased +3 --------- forms a crust. It is an intradermal infection
Preference for cold +5 --------- Stye It is an infection of an eyelash follicle
486   Mastering the BDS IIIrd Year  (Last 25 Years Solved Questions)

Various Clostridium Species and Infections Produced by Important Points About Cleft Lip and Cleft Palate
them
Negros have less incidence of cleft lip/palate
Name of clostridium species Infection Mongolians have high incidence of cleft lip/palate
C. tetani Tetanus In males cleft lip is common
C. perfringens Gas gangrene In females cleft palate is common
C. difficile Pseudomembranous colitis Most common are unilateral clefts to occur i.e. 80%
C. botulinum Food poisoning Least common are bilateral clefts to occur i.e. 20%
Unilateral clefts occur more commonly on left side
Various types of Tetanus

Type of tetanus Description Name of Cleft Lip and Cleft Palate Repair Techniques
Acute tetanus Incubation period is less than 10 days Unilateral cleft lip repair • Millards rotation advancement
Chronic tetanus Incubation period is till one month technique technique
• Mirault blair – brown triangular flap
Delayed or latent tetanus Organism remain latent in wound for
technique
months or years
• Le – Mesurier triangular flap
Local tetanus Leads to local contracture of muscles techniques
• Tenninson – Randall Z – shape
Cephalic tetanus Irritation and paralysis of cranial nerves.
incision technique
More often facial nerve is affected
Bilateral cleft lip repair • Millards rotation advancement
Bulbar tetanus Extensive spasm of muscles of
technique technique
deglutition and respiration
• Mirault blair – brown triangular flap
technique
• Le – Mesurier triangular flap
Types of Gas Gangrene
techniques
Type of gas gangrene Description • Tenninson – Randall Z – shape
incision technique
Clostridial cellulitis Crepitant infection which involve necrotic • V – Y flap techniques
tissue, but there is no involvement of
healthy muscle. It is characterized by C l e f t p a l a t e r e p a i r • Von Langen back technique
foul smelling and seropurulent infection techniques • Furlow
of wound • The three flap or V – Y technique
• Vomer flap technique
Fulminating type Spread is very fast. Associated with
intense toxaemia
Group type This is limited to one group of muscles Bone Grafting in Cleft Palate at Recommended Ages
Massive type It involve whole muscle mass of one limb Type of bone grafting Age recommended
Single muscle type This is limited to one muscle only Primary bone grafting Less than two years
Early secondary bone grafting 2 to 4 years
Various Streptococcal Species and Infections Produced Secondary bone grafting 6 to 15 years
by them Late secondary bone grafting In adults
Name of streptococcal Infection
species
Various Tumors and Their Location
S. pyogenes Cellulitis, Erysipelas,
glomerulonephritis, necrotizing fasciitis, Name of the tumor Location
scarlet fever, rheumatic fever, tonsillitis Carotid body tumor or Potato tumor Beneath anterior edge of
S. viridians Septicaemia and Endocarditis or Chemodectoma sternomastoid
Anaerobic streptococci Liver abscess Ulcerated sebaceous cyst or Cock’s Scalp
peculiar tumor
Elements of primary survey Lipoma or Ubiquitous tumor Can occur anywhere
♦♦ A refers to airway management Strenomatoid tumor or Congenital Middle of sternomastoid
♦♦ B refers to breathing and ventilation torticollis muscle
♦♦ C refers to circulation and hemorrhage control Extradural abscess or Pott’s puffy Skull
♦♦ D refers to dysfunction of CNS tumor
♦♦ E refers to exposure in controlled environment
Section 2:  General Surgery  487

Parts of an Ulcer Levels of Axillary Lymph Nodes


♦♦ Margin: This is the junction between normal epithelium Levels of Axillary Description
and an ulcer lymph node
♦♦ Edge: This is the area between margin and floor of an ulcer
Level I or Low axilla Lymph nodes lateral to lateral border of
♦♦ Floor: This is an exposed surface of an ulcer pectoralis minor muscle
♦♦ Base: This is an area on which ulcer rest. This is better felt
Level II or Mid axilla Lymph nodes deep or posterior to
than seen
pectoralis minor muscle and interpectoral
Various Ulcers and Their Description lymph nodes
Level III or High axilla Apical lymph nodes and those medial to
Name of an ulcer Description medial margin of pectoralis minor muscle,
Bazin’ s ulcer It is seen in young adult females excluding those which are designated as
subclavicular or infraclavicular
Cryopathic ulcer It occur because of cold injury and chilblain
Marjolins ulcer This is the malignant ulcer occur due to scar
or burn Trotter’s Triad
Martorell ulcer It is basically a hypertensive ulcer ♦♦ Ipsilateral temporoparietal neuralgia (Due to Trigeminal
Rodent ulcer It is also known as basal cell carcinoma nerve)
♦♦ Palatal paralysis (Due to vagus nerve)
Trophic ulcer It is also known as bed sore ulcer
♦♦ Conducive deafness (Due to Eustachian tube blockade)
Tropical ulcer It occur because of Vincent’s infection
Various Parenteral Fluids with their Composition and
Various Types of Ulcers and their Edges indications
Name of an ulcer Type of an edge Name of the Composition Indications
Apthous ulcer Regular punched out parenteral fluid
Healing ulcer Sloping edge Plasma, albumin 4.5% • Albumin In severe burns
• Sodium
Rodent ulcer Rolled out edge
• Potassium
Squamous cell carcinoma Edge is raised, everted and indurated • Chloride
Syphilitic ulcer Regular punched out ulcer • Bicarbonate

Traumatic ulcer Irregular margins 5% dextrose Low sodium In postoperative period


and potassium when there is decrease
Tuberculous ulcer Undermined
concentration in sodium excretion
Non – specific ulcer Shelving edge
0.9% Isotonic saline High concentration In Vomiting
Type of Lymph Nodes in Various Diseases o f s o d i u m a n d During gastric or
chloride duodenal aspiration
Type of a lymph node Disease
Ringer’s lactate Or Sodium In Hypovolemic shock
Elastic and rubbery Hodgkin’s lymphoma Hartmann’s solution Potassium
Fixed and hard Carcinoma Chloride in
concentration
Firm, discrete and shotty Syphilis
similar to plasma
Matted Tuberculosis

Various Terminologies Various Blood Substitutes


Terminology Meaning Name of blood substitute Contents
Hypertrophy Increase in size of cells and increase in size of Plasma and its derivatives • Albumin
an organ • Cryoprecipitate
Hyperplasia Increase in number of cells • Fibrinogen
• Plasma
Metaplasia Reversible change of one epithelial cell to another • Platelet rich plasma
Dysplasia It refers to disordered cellular development Various synthetically prepared solutions • Dextran
Anaplasia This is the loss of differentiation of tumor cells from • Fluorocarbons
where they arise • Gelatin
• Hydroxyethyl starch
Carcinoma in It resemble like a cancer but without invasion
situ across basement membrane
488   Mastering the BDS IIIrd Year  (Last 25 Years Solved Questions)

Various Replacements and their Conditions Various Blood Products and their Storages

Name of replacement Condition Name of blood Method of storage


product
Cryoprecipitate Hemophilia
Autologous blood It can be stored till three weeks
Fibrinogen Disseminated intravascular
coagulation Fresh frozen plasma At -30°C with 2 years of shelf life
4.5% human albumin Burns Packed red cells These are stored in saline adenine glucose
– mannitol (SAG – M) solution to increase
Packed red cells Anemia, elders and small children
shelf life till 5 weeks at 2 to 6°C
Platelet concentrate and Thrombocytopenia
Platelets At 20 to 24°C with shelf life of 5 days
Platelet rich plasma

Various Formulas used in Burn Various Types of Thyroplasty


♦♦ Type I: There is medial displacement of vocal cord
Timing Moore’s formula Evan’s formula Brooke’s ♦♦ Type II: There is lateral displacement of vocal cord to
formula
improve airway
1st 24 • Ringer’s lactate: • Normal • Ringer’s ♦♦ Type III: To shorten or relax the vocal cord
hours 1000 to 4000ml saline is 1ml/ lactate ♦♦ Type IV: To lengthen or tighten the vocal cord.
• Normal saline is Kg/%burn is 1ml/
1000ml • 5% dextrose Kg/%burn Various Markers of Cancer
• 5% dextrose is is 2000ml • Colloid
1500 to 5000ml • Colloid containing Name of the marker Name of Cancer
• Colloid containing containing fluid is
fluid: 7.5% of fluid is 1ml/ 1ml/ Oncofetal antigens
body weight Kg/%burn Kg/%burn • Alpha – foeto protein (AFP) • Hepatocellular carcinoma,
• 5% • Carcinoembryonic antigen Non – seminomatous germ
dextrose is (CEA) cell tumor of testis
2000ml • Breast cancer, Bowel cancer,
Bladder cancer
2nd 24 • Ringer’s lactate: • Normal • Ringer’s
hours 1000 to 4000ml saline is lactate and Enzymes
• Normal saline is 0.5ml/ Colloid • Prostrate acid phosphatase • Carcinoma of prostrate
1000ml Kg/%burn containing (PAP) • Oat cell carcinoma of lung
• 5% dextrose is • 5% dextrose fluid is • Neuron specific enolase and Neuroblastoma
1500 to 5000ml is 2000ml ½ to ¾ (NSE)
• Colloid containing • Colloid of above Secreted cancer antigens
fluid: 2.5% of containing mentioned
• CA – 125 • Carcinoma of ovary
body weight fluid is 0.5ml/ amount
• CA – 15 – 3 • Carcinoma of breast
Kg/%burn • 5%
dextrose is Hormones
2000ml • Calcitonin • Medullary carcinoma of
• Catecholamines and vanillyl thyroid
mandelic acid (VMA) • Pheochromocytoma and
Types of Grafts neuroblastoma

Type of graft Description Cytoplasmic proteins


• Immunoglobulins • Multiple myeloma and other
Allograft or homograft Transfer of graft from one individual to • Prostrate specific antigens gammopathies
another of same species • Carcinoma of prostrate
Heterograft Transfer of graft from one individual to
another individual of different species
ASA (American Society of Anesthesiologists)
ISO or Synergistic graft This is the graft transplant between
identical twins Classification of Physical Status
Hetrotropic graft Transplant which is positioned at Type of ASA Description
different site
Type I Normal healthy patient
Orthotropic graft Transplant which is positioned at its
Type II Patient with mild to moderate systemic disease
anatomical site
Type III Patient with severe systemic disease that limit
Autogenous graft This is the graft taken and received by
activity but is non incapacitating
same individual
Section 2:  General Surgery  489

Type IV Patient with severe systemic disease that limit • Thyroid enlargement
activity and is a constant threat to life ♦♦ Acute swellings in neck
• Acute lymphadenitis
Type V Moribund patient not expected to survive 24 hours
with or without operation • Boil
• Carbuncle
Type VI Clinically dead patient being maintained for
• Ludwig’s angina
harvesting of organs
♦♦ Cysts demonstrating cholesterol crystals
• Branchial cyst
Swellings of Head and Neck • Cystic hygroma
• Dental cyst
♦♦ Swellings in midline
• Dentigerous cyst
• Lipoma
• Old hydrocele
• Sublingual dermoid
• Thyroglossal cyst
• Submental lymph nodes
♦♦ Swellings that move with deglutition
• Thyroid gland enlargement
• Enlarged pretracheal lymph nodes which are fixed
• Thyroglossal cyst
to trachea
♦♦ Lateral swellings in neck
• Laryngocele
• Branchial cyst
• Subhyoid bursal cyst or Subhyoid bursitis
• Lipoma
• Thyroid swelling
• Lymph node swelling
• Thyroglossal cyst
• Salivary gland enlargement
3
sECTION

Oral Pathology

Section I: Disturbances of Development and 13. Physical and Chemical Injuries of the Oral
Growth Cavity
1. Developmental Disturbances of Oral and Para 14. Regressive Alterations of the Teeth
Oral Structures 15. Healing of Oral Wounds
2. Benign and Malignant Tumors of Oral Cavity Section IV:  Disturbance of the Metabolism
3. Tumors of Salivary Glands 16. Oral Aspects of Metabolic Diseases
4. Odontogenic Tumors Section V:  Diseases of Specific System
5. Cysts of Oral Cavity 17. Diseases of Bone and Joints
Section II:  Disturbances of Microbial Origin 18. Diseases of Blood and Blood Forming Organs
6. Bacterial Infections of Oral Cavity 19. Diseases of Skin
7. Viral Infections of Oral cavity 20. Diseases of Nerves and Muscles
8. Fungal Infections of Oral Cavity Section VI:  Forensic Odontology
9. Diseases of Periodontium 21. Forensic Odontology
10. Dental Caries Multiple Choice Questions as per DCI and Examina-
11. Diseases of the Pulp and Periapical Tissues tion Papers of Various Universities
Section III:  Injuries and Repair Viva-voce Questions for Practical Examination
12. Spread of Oral Infection Additional Information
Oral Pathology
I Disturbances of Development and Growth

Q.2. Write notes on microdontia. (Mar 2000, 5 Marks)


1. Developmental Distur- Ans. Microdontia is the term used to describe the teeth which
bances of Oral and Para are smaller than the normal.
♦♦ There are three types of microdontia:
oral Structures 1. True generalized microdontia
2. Relative generalized microdontia
Q.1. Write notes on xerostomia. (Sep 2011, 3 Marks) 3. Microdontia involving the single tooth
(Feb 2013, 5 Marks) 1. True generalized microdontia: In true generalized mi­
crodontia all the teeth are smaller than the normal.
Write short notes on xerostomia. (Apr 2017, 5 Marks)
Its occurrence is reported in the cases of pituitary
Ans. Xerostomia or dryness of mouth is not a disease but a dwarfism. The teeth are well formed and are small.
symptom of different diseases. . Relative generalized microdontia: In relative general­
Xerostomia produces serious negative effects on patient’s ized microdontia the normal or slightly smaller
life affecting the dietary habits, nutritional status, speech, than normal teeth are present in the jaws that are
taste and increase susceptibility to dental caries.
somewhat larger than the normal and there is an
Etiology illusion of true microdontia.
The causes of xerostomia are classified into two categories: 3. Microdontia involving the single tooth: It is a com­
1. Temporary causes mon condition which more commonly affects
2. Permanent causes maxillary lateral incisor and third molar. Lateral
incisor becomes peg shaped.
Temporary Causes
♦♦ Psychological: Anxiety and depression are well recognized
as causes of reduced salivary flow.
♦♦ Duct calculi: Blockages of a duct of major salivary gland
mainly sub mandibular produces dryness of affected side.
♦♦ Sialoadenitis: Inflammation of salivary glands can cause
reduced secretions.
♦♦ Drug therapy: A wide variety of drugs, i.e. anticholinergic,
tricyclic antidepressants, bronchodilators and histamines
may cause xerostomia.

Permanent Causes
♦♦ Salivary gland aplasia: The congenital absence of salivary
glands leads to xerostomia.
Fig. 1: Microdontia
♦♦ Sjogrens syndrome: This is the combination of dry mouth,
(For color version, see Plate 1)
dry eyes and often rheumatoid arthritis.
♦♦ Radiotherapy: The distressing cause of xerostomia is thera­ Q.3. Write notes on geographic tongue.(Feb 1999, 5 Marks)
peutic radiography for head and neck tumors. Or
Write short note on benign migratory glossitis.
Clinical Features  (Dec 2010, 3 Marks) (Aug 2011, 5 Marks)
♦♦ Unilateral dryness with pain or discomfort and swelling Or
in affected gland. Write short note on geographic tongue.
♦♦ Drying and burning sensations are present but mucosa  (Jan 2018, 4 Marks)
appears normal. Ans. Geographic tongue is a multifocal, patchy irregular area
♦♦ Due to lack of saliva mucosa will appear dry, atrophic and of depapillation of tongue characterized by frequent
more often pale and translucent. remissions and recurrences.
♦♦ Soreness, burning and pain of mucous membrane and
tongue are common symptoms. Clinical Features
♦♦ It is seen in children as well as in adults.
Treatment
♦♦ Geographic tongue clinically presents multiple, irregular,
It is advisable to promote salivary stimulation by using sugar well demarcated, patchy erythematous areas on dorsum
free chewing gum which is affected and convenient. of tongue with desquamation of filiform papilla.
494   Mastering the BDS IIIrd Year  (Last 25 Years Solved Questions)

♦♦ Although filiform papilla are absent in desquamated zone, ♦♦ The curve or the bend may occur anywhere along the
the fungiform papillae remain present which appear as length of the root, sometimes at the cervical portion, at
few red dots projecting on the surface. other times midway along the root or even just at the apex
♦♦ Geographic tongue is a painless, asymptomatic condition, of the root depending on the amount of root formed when
however on few occasions it may produce soreness or injury occurs.
burning sensations. ♦♦ Dilacerated teeth cause problems at the time of extraction,
♦♦ Remissions of initial lesions always followed by fresh if operator is unaware of condition, so preoperative roent­
recurrent lesions which involves new areas of tongue genograms before any surgical procedure are carried out.
surface. Q.5. Write short note on Dens Invaginatus.
Histopathology  (Mar 2003, 5 Marks)

♦♦ The condition show hyperparakeratinization of covering Or


epithelium of the tongue with loss of filiform papillae. Write short note on Dens in dente. (July 2016, 5 Marks)
♦♦ Intercellular edema and accumulation of neutral Ans. Oehlers describe dens invaginatus.
polymorphs is often seen in layers of epithelium. ♦♦ Dens in dente or dens invaginatus is the deep surface in­
♦♦ Mild inflammatory cell infiltration is present in underlying vagination of crown or root which is lined by the enamel.
connective tissue. ♦♦ Dens invaginatus is a developmental variation which is
thought to arise as the result of invagination in the surface
of tooth crown before calcification has occurred.
♦♦ The causes of the condition are increased localized exter­
nal pressure, focal growth, retardation and focal growth
stimulation in certain areas of tooth bud.
♦♦ The permanent maxillary lateral incisors are the teeth most
commonly involved.

Types
♦♦ Coronal dens invaginatus
♦♦ Radicular dens invaginatus

Coronal Dens Invaginatus


♦♦ This is seen more frequently as compared to radicular
Fig. 2:  Geographic tongue dens invaginatus.
(For color version, see Plate 1) ♦♦ In their decreasing order, teeth affected by this anomaly
are permanent lateral incisors, central incisors, premolars,
Q.4. Write short note on dilaceration.
canines and molars.
 (Mar 2003, 5 Marks) (Jan 2012, 5 Marks)
♦♦ This is commonly seen in maxillary teeth.
Ans. Dilaceration refers to an angulation or a sharp bend or
♦♦ Variation in the depth of invagination is seen from slight
a curve in the root and crown of the formed tooth.
enlargement of cingulum pit to deep infolding which
extend to apex.
♦♦ Coronal dens invaginatus is of three types viz:
• Type I: Exhibit invagination which is confined to the
crown.
• Type II: Extends below CEJ and ends in a blind sac
that may or may not communicate with adjacent
dental pulp.
• Type III: It extends through the root and perforates in
apical or lateral radicular area without any immediate
communication with the pulp.
♦♦ Occasionally invagination can be rather large and resem­
ble a tooth inside a tooth that’s why it is known as dens
in dente.
♦♦ In some of the other cases invagination can be dilated and
Fig. 3: Dilaceration disturb the formation of tooth which lead to anomalous
It occurs due to the mechanical trauma to calcified portion development of tooth known as dilated odontome.
of partially formed teeth which lead to the displacement of ♦♦ Roentgenographically, it is recognized as a pear shaped
calcified portion in different direction. invagination of enamel and dentin with a narrow constric­
Section 3: Oral Pathology  495

tion at the opening on the surface of the tooth and closely Q.6. Write a note on anodontia. (Feb 1999, 5 Marks)
approximating the pulp in its depth. (Mar 2001, 5 Marks) (Sep 2008, 3 Marks)
Ans. Anodontia is defined as the condition in which there is
congenital absence of teeth in oral cavity.

Etiology of Anodontia
The causes of anodontia are:
♦♦ Hereditary factor
♦♦ Environmental factor
♦♦ Familial factor
♦♦ Syndrome associated
♦♦ Radiation injury to the developing tooth germ.

Fig. 4: Coronal dens invaginatus

Radicular Dens Invaginatus


♦♦ This condition is rare.
♦♦ This condition arises secondary to proliferation of
Hertwig’s epithelial root sheath, with formation of strip
of enamel that extends along surface of root.
♦♦ Roentgenographically, affected tooth shows enlargement
of root. On close examination there is presence of dilated
invagination which is lined by the enamel with opening of Fig. 6: Anodontia
invagination situated along lateral aspect of root. (For color version, see Plate 1)
Anodontia is of two types:
1. Complete anodontia: There is congenital absence of all the
teeth.
2. Partial anodontia: Congenital absence of one or few teeth.
• Anodontia can also be divided into following types:
– True anodontia: It occurs due to failure of develop­
ment or formation of tooth in jaw bone.
– Pseudoanodontia: It refers to the condition in which
teeth are present within the jaw bone but are not
clinically visible in the mouth, as they have not
erupted, e.g. impacted teeth.
– False anodontia: It is the condition in which teeth
are missing in the oral cavity because of their
previous extraction.

Complete Anodontia
Fig. 5: Radicular dens invaginatus
♦♦ It is the condition in which there is neither any deciduous
Treatment tooth nor any permanent tooth present in the oral cavity.
♦♦ Type I invagination, opening should be restored after the ♦♦ It is usually seen in association with hereditary ectodermal
eruption to prevent development of caries and pulpal dysplasia. A complete anodontia is a common feature of
inflammation. hereditary ectodermal dysplasia however in many cases
♦♦ In large invagination content of lumen and carious den­ cuspids are present in this disease.
tine is removed and then calcium hydroxide base may be ♦♦ Complete anodontia occurs among children those who
placed. have received high doses of radiation to the jaws as infants
♦♦ Type III invaginations require endodontic therapy. for therapeutic extraction.
496   Mastering the BDS IIIrd Year  (Last 25 Years Solved Questions)

Partial Anodontia Clinical Features


♦♦ It is a common phenomenon and is characterized by con­ ♦♦ Color of teeth is mostly chalky white but sometimes it can
genital absence of one or few teeth. be yellow or even dark brown.
♦♦ In partial anodontia any tooth can be congenitally missing. ♦♦ Contact points in proximal surfaces are mostly open
♦♦ Third molars are most frequently observed congenitally while the occlusal surfaces and incisal edges are severely
missing teeth. abraded.
♦♦ Mandibular first molar and the mandibular lateral incisor ♦♦ Enamel may have a cheesy consistency which is easily
are least likely to be missing. removable from tooth surface.
Q.7. Write note on amelogenesis imperfecta.  ♦♦ Amelogenesis imperfecta does not increase the susceptibil­
 (Dec 2010, 8 Marks) (Feb 1999, 5 Marks) ity of teeth to dental caries.
Ans. Amelogenesis imperfecta is also known as hereditary ♦♦ In amelogenesis imperfecta the enamel is of near normal
enamel dysplasia or hereditary brown enamel. hardness and has some white opaque flecks at incisal areas of
teeth. These types of teeth are known as snow capped teeth.

Treatment
There is no definite treatment of amelogenesis imperfecta.
Composite veneering can be done to improve esthetics of teeth.
Q.8. Write note on mottled enamel. (Feb 1999, 5 Marks)
Ans. Mottled enamel is a type of hypoplasia due to fluoride
and was first described by GV Black and Frederick
S Mckay.

Fig. 7:  Amelogenesis imperfecta


(For color version, see Plate 1)
• Amelogenesis imperfecta is a heterogeneous group
of hereditary disorders of enamel formation affecting
both deciduous or permanent teeth.
• Disease involves only the ectodermal part of the
tooth, i.e. enamel.
• It is of three types:
1. Hypoplastic type Fig. 8:  Mottled enamel
2. Hypocalcified type (For color version, see Plate 1)
3. Hypomature type Etiology
1. Hypoplastic type: In this the enamel thickness is
usually far below the normal since disease affects It is caused due to ingestion of fluoride containing water during
the stage of matrix formation. In this stage the the time of tooth formation which may result in mottled enamel.
teeth exhibit complete absence of enamel from Pathogenesis
crown surface or there may be a very thin layer
of enamel in some focal areas. Mottled enamel is due to the disturbance of ameloblasts during
2. Hypocalcification type: This is due to distur­bance formative stage of tooth development.
in the process of early mineralization of enamel. There is histological evidence of cell development which
In this stage the enamel is soft and can easily be is likely that the cell product, enamel matrix is defective or
removed with blunt instrument. deficient. With higher level of fluoride, there is interference
3. Hypomaturation type: This occurs due to interrup­ with calcification process of matrix.
tion in the process of maturation of enamel. Here
the enamel is of normal thickness but it does not Clinical Features
have normal hardness and is translucent. The ♦♦ There is white flecking or spotting of enamel.
enamel can be pierced with an explorer tip with ♦♦ Mild changes are manifested by white opaque areas in­
firm pressure. cluding tooth areas.
Section 3: Oral Pathology  497

♦♦ Moderate and severe changes showing pitting and brown­ ♦♦ Environmental enamel hypoplasia
ish staining of teeth. ♦♦ Dentinogenesis imperfecta
♦♦ Corroded appearance of teeth. ♦♦ Dentin dysplasia.
♦♦ Those teeth which are moderately or severely affected may ♦♦ Regional odontodysplasia or odontodysplasia or odonto­
show a tendency of wear and even fracture of enamel. genic dysplasia, odontogenesis imperfecta or ghost teeth
♦♦ Dentin hypocalcification
Treatment
Factors Associated with Acquired Enamel Hypoplasia
Mottled enamel frequently becomes stained an unsightly brown
color. For cosmetic reasons it has become the practice to bleach There are two types of factors which are associated with
the affected teeth with hydrogen peroxide. acquired enamel hypoplasia:
Q.9. Enumerate various anomalies of teeth affecting num- 1. Local factor
ber, size and shape. Describe one condition from each. 2. Environmental/systemic factor
 (Sep 1999, 15 Marks) 1. Local factors: The local factors are infection, trauma, radio­
Ans. therapy and idiopathic factors.
• When local infection or trauma causes damage to
Enumeration of Anomalies Affecting Number of Teeth ameloblasts cells during odontogenesis, it may
♦♦ Anodontia result in defect in enamel formation in isolated
♦♦ Hypodontia permanent tooth and this is known as focal enamel
♦♦ Hyperdontia, i.e. natal teeth, neonatal teeth, premature hypoplasia.
eruption. The focal enamel hypoplasia is caused due to
See anodontia in description. For anodontia refer to Ans 6 periapical spread of infection from a carious
of same chapter. deciduous tooth or trauma to the deciduous tooth,
the tooth affected in this process is commonly
Enumeration of Anomalies Affecting Shape of Teeth known as turner’s tooth.
♦♦ Gemination 2. Environmental or systemic factors: The systemic or environ­
♦♦ Twinning mental disturbances in the functioning of ameloblasts at
♦♦ Fusion specific period of time during odontogenesis often mani­
♦♦ Concrescence fest as horizontal line of small pits or grooves or enamel
♦♦ Dilaceration surfaces. This line on tooth surface indicates zone of enamel
♦♦ Talon cusp hypoplasia and corresponds to time of development and
♦♦ Dens invaginatus duration of insult. The factors in following stages:
♦♦ Dens evaginatus • Prenatal period: The prenatal infections are rubella,
♦♦ Taurodontism syphilis.
♦♦ Hypercementosis –– There is presence of internal disease
♦♦ Enamel pearl –– There are excess fluoride ions
♦♦ Mulberry molar • Neonatal period: During this period enamel hypoplasia
♦♦ Globodontia is caused due to:
♦♦ Hutchinson’s incisor –– Hemolytic disease of newborn
See dens invaginatus in description. For dens invaginatus, –– Birth injury
refer to Ans 5 of same chapter. –– Premature delivery
–– Prolong labor
Enumeration of Anomalies Affecting Size of Teeth –– Low birth weight
♦♦ Microdontia • Postnatal period: During this period enamel hypoplasia
♦♦ Macrodontia is due to:
–– Sever childhood infection.
See microdontia in description. For microdontia refer to Ans
–– Prolong fever due to infectious disease in child­
2 of same chapter.
hood
Q.10. Enumerate the developmental anomalies associated –– Nutritional deficiency
with the teeth. Describe factors associated with ac- –– Hypocalcaemia
quired enamel hypoplasia. (Sep 1999, 15 Marks) –– Rickets
Ans. Anomalies Affecting Hard Tissues of Oral Cavity –– Celiac disease.
The anomalies which affect the hard tissues of oral cavity are: For anomalies affecting number of teeth, affecting shape
♦♦ Amelogenesis imperfecta or hereditary enamel dysplasia of teeth and affecting size of teeth, refer to Ans 9 of same
or hereditary brown enamel or hereditary opalescent teeth. chapter.
498   Mastering the BDS IIIrd Year  (Last 25 Years Solved Questions)

Q.11. Write notes on gemination and fusion. Q.12. Write notes on taurodontism.(Aug/Sep 1998, 5 Marks)
 (Mar 2001, 5 Marks) Or
Ans. Gemination Write short note on taurodontism. (Sep 2005, 5 Marks)
• Gemination is a developmental anomaly which re­ (Jan 2017, 5 Marks) (Dec 2007, 3 Marks)
fers to partial development of two teeth from single Ans. Taurodontism is a dental anomaly in which the body of
tooth bud following incomplete division. tooth is enlarged at the expense of the roots. Actually
• The structure is usually one with two completely or taurodontism meAns the bull shape appearance of the
incompletely separated crowns that have a single tooth.
root and a root canal.
• The condition is seen in both deciduous and perma­ Clinical Features
nent dentition. ♦♦ Affected tooth in taurodontism has elongated pulp cham­
• It appears to exhibit hereditary tendency. ber with the rudimentary root formation.
♦♦ Tooth becomes rectangular in shape with minimum con­
striction at the cervical area.
♦♦ Taurodontism commonly affects multirooted permanent
molar teeth and sometimes the premolar teeth.
♦♦ The bifurcation or trifurcation may be a few millimeter
above the apices of the roots.
♦♦ Patient with hypodontia may have taurodontism.

Fig. 9: Gemination

Fusion

Figs 11A and B: Taurodontism

Treatment
No specific treatment is required.
Q.13. Write short note on supernumerary teeth.
(Mar 1998, 5 Marks)
Ans. Presence of any extra tooth in dental arch in addition to
normal series of teeth is known as supernumerary teeth.
Fig. 10: Fusion (For color version, see Plate 2)
Classification
♦♦ Fusion is defined as union of two adjacent normal tooth
germs at level of dentin during development. ♦♦ According to the morphology
♦♦ One of the most important criteria for fusion is that fused • Conical: This small peg shaped conical tooth is
tooth must exhibit confluent dentin. supernumerary tooth.
♦♦ Both permanent and deciduous dentition are affecting in • Tuberculate: This type of supernumerary tooth
case of fusion, although it is more common in deciduous possesses more than one cusp or tubercle. It is of barrel
teeth. shaped and may be invaginated.
♦♦ Fusion can be complete or incomplete and its extent will • Supplemental: It refers to the duplication of the teeth in
depend on stage of odontogenesis at which fusion takes normal series. The most common tooth is permanent
place. maxillary lateral incisor.
♦♦ The incisor teeth are more frequently affected in both the • Odontome: This represents the hamartomatous
dentitions during the condition. malformation.
Section 3: Oral Pathology  499

♦♦ According to location • Dentinogenesis imperfecta1: Dentinogenesis


• Mesiodens: They are located between two upper central imperfecta without osteogenesis imperfecta (opalescent
incisor. dentine). This corresponds to dentinogenesis imperfect
• Distomolars: They are located on the distal aspect of Type II of Sheild’s classification.
regular molar teeth in dental arch. • Dentinogenesis imperfecta 2: Brandywine type
• Paramolar: They are located either in buccal or lingual dentinogenesis imperfecta: This corresponds
aspect of normal molars. to dentinogenesis imperfect Type II of Sheild’s
• Extra-lateral incisors: They are more common in classification.
maxillary arch. There is no substitute in present classification for the category
which is designated as Dentinogenesis imperfect Type I of
Clinical Features
Sheild’s classification.
♦♦ Supernumerary teeth cause crowding or malocclusion and
give rise to cosmetic problem. Etiology
♦♦ These extra teeth are responsible for the increase caries ♦♦ Gene affected is present on chromosome 4 and it codes for
incidence and periodontal problem. DSPP (Dentine sialoprotein and phosphoprotein)
♦♦ Multiple supernumerary teeth can occur in association
with the conditions like Gardener’s syndrome. Clinical Features
♦♦ Dentigerous cyst may sometimes develop from an im­
♦♦ On eruption the teeth exhibit a normal contour and have
pacted supernumerary tooth.
opalescent amber like appearance.
Treatment ♦♦ Few days after the eruption the teeth may achieve the
normal color. Finally the teeth become gray or brownish
♦♦ Supernumerary teeth should be extracted.
in color with bluish reflection from enamel.
♦♦ Impacted supernumerary teeth should also be removed surgi­
♦♦ In some cases the affected teeth may exhibit hypomineral­
cally since they can interfere with normal tooth alignment.
ized areas on surface of enamel.
♦♦ Teeth are not particularly sensitive even when most of the
surface enamel is lost.
♦♦ The dentin is soft and easily penetrable in dentinogenesis
imperfecta, these teeth are not caries prone.

Histopathology
♦♦ Histopathologically the enamel appears normal in
Dentinogenesis imperfecta. Mantle dentin is also nearly
normal.
♦♦ Dentinal tubules are less in number per square unit area
of dentin as compared to normal dentin. The tubules are
often distorted, irregular in shape, widely spaced and
larger in size.
Fig. 12:  Supernumerary teeth
(For color version, see Plate 2)
♦♦ Pulp chamber and root canal are often obliterated by the
abnormal dentin deposition.
Q.14. Write short note on dentinogenesis imperfecta. ♦♦ DEJ appears smooth or flattened instead of being scalloped.
(Mar 2006, 5 Marks) (Mar 1997, 5 Marks) ♦♦ Large area of a tubular dentin is present.
Ans. Dentinogenesis imperfecta is an inherited disorder
of dentin formation which affects deciduous and
permanent dentition.

Classification
♦♦ Sheild’s Classification
• Dentinogenesis imperfecta Type I: Dentinogenesis
imperfecta without osteogenesis imperfecta.
• Dentinogenesis imperfecta Type II: Dentinogenesis
imperfecta with osteogenesis imperfect
• Dentinogenesis imperfecta Type III: It is a racial isolate
in Maryland and is known as Brandywine type.
♦♦ Extensive studies have shown that dentinogenesis imper-
fecta is clearly a disorder from osteogenesis imperfect, so
Fig. 13:  Dentinogenesis imperfecta
the following revised classification is given:
(For color version, see Plate 2)
500   Mastering the BDS IIIrd Year  (Last 25 Years Solved Questions)

Q.15. Write note on median rhomboid glossitis. Clinical Features


(Mar 1998, 5 Marks) ♦♦ It is seen in females more commonly.
Or ♦♦ In tongue thyroid tissue appears as nodular exophytic
Write short note on median rhomboid glossitis. mass measuring 2 to 3 cm in diameter and is located to
(Dec 2009, 5 Marks) (Feb 2002, 5 Marks) foramen cecum.
 (Feb 2015, 5 Marks) (Jan 2016, 5 Marks) ♦♦ It can be present as smooth cystic swelling.
Or ♦♦ Symptoms may vary which include change of voice,
Write short answer on median rhomboid glossitis. bleeding, pain, dysphagia, dyspnea and feeling of tightness
(May 2018, 3 Marks) in throat.
Ans. It is an asymptomatic, elongated, erythematous patch Histopathology
of atrophic mucosa on mid-dorsal surface of tongue.
♦♦ Normal mature thyroid tissue, although embryonic or fetal
Clinical Features thyroid gland tissue may also be seen.
♦♦ Thyroid nodules may exhibit colloid degeneration or
♦♦ Condition is seen mostly in the young adults.
goiter.
♦♦ It is most common among the males.
♦♦ Microscopic examination of human tongue removes at
♦♦ Lesion is located immediately anterior to the foramen
autopsy reveals remnant of thyroid tissue within tongue.
cecum and circumvallate papillae in midline of dorsum
of tongue. Treatment
♦♦ It starts as narrow, mildly erythematous area located along
the median fissure of the tongue. Surgical excision should be done.
♦♦ Fully developed lesion of the median rhomboid glossitis
appears diamond or lozenge shaped area devoid of papilla.
♦♦ Color of lesion varies from pale pink to bright red.
♦♦ It is usually asymptomatic but occasionally causes slight
soreness or burning sensation.

Histopathology
♦♦ Parakeratosis of surface epithelium
♦♦ Loss of papilla
♦♦ Thinning of supra papillary epithelium
♦♦ Presence of acanthosis with elongation of rete ridges.
♦♦ Superficial layer of epithelium shows neutrophilic infiltra­
tion and there is presence of candida hyphae.
♦♦ Underlying connective tissue is vascular and is infiltrated
by chronic inflammatory cells.
♦♦ The epithelium may show features of dysplasia. Fig. 15:  Lingual thyroid
(For color version, see Plate 2)

Q.17. Enumerate the developmental anomalies of tongue and


describe any two. (Mar 2007, 2.5 Marks)
Ans.

Enumeration of Developmental Anomalies of Tongue


♦♦ Aglossia
♦♦ Microglossia
♦♦ Macroglossia
♦♦ Ankyloglossia or tongue tie
♦♦ Cleft tongue
♦♦ Fissured tongue
♦♦ Median rhomboid glossitis
♦♦ Geographic tongue
Fig. 14:  Median rhomboid glossitis ♦♦ Hairy tongue
(For color version, see Plate 2) ♦♦ Lingual thyroid nodule
Q.16. Write note on lingual thyroid.(Aug/Sep 1998, 5 Marks) ♦♦ Lingual varices
Ans. Accessory accumulation of functional thyroid gland For detail, refer to median rhomboid glossitis (Ans 15) and
tissue within the body of tongue is called lingual thyroid. lingual thyroid (Ans 16) of same chapter.
Section 3: Oral Pathology  501

Treatment
No specific treatment.

Type II: Coronal Dentin Dysplasia


It is an inherited disorder of dentin, which affects the coronal
dentin.

Clinical Features
♦♦ Both deciduous and permanent teeth are affected.
♦♦ Permanent teeth are of normal color whereas deciduous
teeth are amber gray color.
Histopathology
♦♦ Pulp chambers in permanent teeth are abnormally large
and have a flame shaped.
♦♦ Pulp chamber contains many denticles.
♦♦ Root canal may be partially obliterated in apical third region.
Fig. 16: Perleche Q.19. Write short note on concrescence.
(For color version, see Plate 2)
(Sep 2007, 2.5 Marks)
Q.18. Write short note on dentin dysplasia. Ans. Concrescence is defined as union of roots of two or more
(Sep 2006, 5 Marks) (Jan 2012, 5 Marks) adjoining teeth due to deposition of cementum.
Or It is the type of fusion which is limited only to the roots
of teeth and it occurs after the root formation of involved
Write notes on dentin dysplasia. (Feb 2013, 8 Marks) teeth is completed.
Ans. It is the autosomal dominant inherited disorder
characterized by the defective dentin formation and Etiology
abnormal pulp morphology. ♦♦ Traumatic injury
• The condition is also known as “rootless teeth”. ♦♦ Crowding of teeth
• It is classified into two types ♦♦ Hypercementosis
– Type I: Radicular dentin dysplasia
– Type II: Coronal dentin dysplasia Important Features

Type I: Radicular Dentin Dysplasia ♦♦ Concrescence represents an acquired defect and it can
occur in both erupted or unerupted.
It represents a peculiar disturbance in the development of ♦♦ In concrescence beside cementum union or fusion does not
radicular dentin. occur between the enamel, dentin or pulp of involved teeth.
♦♦ Permanent maxillary molar are usually affected by this
Clinical Features
anomaly.
♦♦ It affects both deciduous and permanent dentition.
♦♦ Root of teeth is defective and the crown portion of teeth is
normal both structurally and morphologically.
♦♦ In some cases the crown of teeth reveals slight bluish or
brownish tendency.
♦♦ Because of presence of functionally unstable short roots,
the affected teeth exhibit mobility.
Histopathology
♦♦ Enamel and mantle dentin are normal.
♦♦ Remaining coronal and radicular dentin appear fused nod­
ular mass comprising of tubular dentin and osteo dentin.
♦♦ Histological appearance of such defective mass of dentinal
tissue often reveals what is called as “series of sand dunes”
or “lava flowing around boulders”.
♦♦ Normal and abnormal dentin is well demarcated and
later reveals an abnormal distribution and orien­tation of Fig. 17: Concrescence
dentinal tubules with whorled appearance. (For color version, see Plate 3)
502   Mastering the BDS IIIrd Year  (Last 25 Years Solved Questions)

♦♦ Concrescence can occur between normal molar and a ♦♦ Ankyloglossia or tongue tie
supernumerary molar. ♦♦ Cleft tongue
♦♦ Condition is seen in those areas of dental arch where roots ♦♦ Fissured tongue
of neighboring teeth lie close to one another. ♦♦ Median rhomboid glossitis
Q.20. Write short note on Fordyce’s granules. ♦♦ Geographic tongue
(Jan 2016, 5 Marks) (Sep 2009, 3 Marks) ♦♦ Hairy tongue
Ans. It is a developmental anomaly characterized by ♦♦ Lingual thyroid nodule
heterotrophic collection of sebaceous glands at various ♦♦ Lingual varices
sites in oral cavity. For median rhomboid glossitis refer to Ans 15 of same chapter
For Lingual thyroid refer to Ans 16 of same chapter.
Pathogenesis For Geographic tongue refer to Ans 3 of same chapter.
It is suggested that occurrence of sebaceous gland in the mouth
may result from inclusion in the oral cavity of ectoderm having Aglossia
some of the potentialities of skin in the course of development It is characterized by the complete absence of tongue
of the maxillary and mandibular process during embryonic life.
Microglossia
Clinical Features
This is in reality a microglossia with extreme glossoptosis. In
♦♦ They appear as yellow spots either separated from each such cases a rudimentary small tongue is observed. Due to the
other and remain in groups. lack of muscular stimulus between alveolar arches they do not
♦♦ They are seen bilaterally on the mucosa of cheeks opposite develop transversely and mandible does not grow in anterior
to molar tooth, on the inner surfaces of lips, retromolar direction.
area.
♦♦ They are also seen in the esophagus, female genitalia, Macroglossia
male genitalia, nipples, palm and soles, parotid gland,
Macroglossia is an uncommon condition characterized by
larynx and orbit.
enlargement of tongue.
Histological Features
Type of Macroglossia
♦♦ They are unassociated with the hair follicles.
♦♦ Glands are superficial and may consist of only few or more There are two broad categories under the heading of
tubules, all grouped around one or more ducts which open Macroglossia, i.e.
in the mucosa. ♦♦ True Macroglossia
♦♦ Ducts may show keratin plugging. ♦♦ Pseudomacroglossia

Treatment Causes
♦♦ Congenital and hereditary:
No treatment is required.
• Vascular malformations
Q.21. Define anomaly. Describe in detail developmental • Lymphangioma
anomalies of the tongue. (Sep 2011, 8 Marks) • Hemangioma
Or • Hemihyperplasia
• Cretinism
Enumerate and describe the developmental distur-
• Beckwith-Wiedemann syndrome
bances affecting tongue. (Aug 2012, 15 Marks)
• Down syndrome
Or • Mucopolysaccharidosis
Describe in detail developmental disturbances • Neurofibromatosis
affecting tongue. (Apr 2015, 8 Marks) ♦♦ Acquired:
• Edentulous patients
Or
• Amyloidosis
Give an account of developmental anomalies of • Myxedema
tongue. (Mar 2016, 8 Marks) • Acromegaly
Ans. Anomaly is defined as the irregularity or deviation • Angioedema
compared to the normal structure. • Carcinoma and other tumors
Enumeration of Developmental Anomalies of Tongue Clinical Features
♦♦ Aglossia ♦♦ It most commonly occur in children.
♦♦ Microglossia ♦♦ In infants it is manifested by noisy breathing, drooling and
♦♦ Macroglossia difficulty in eating.
Section 3: Oral Pathology  503

♦♦ Macroglossia result in lisping speech. ♦♦ Hyperplasia of the retepegs and increased thickness of
♦♦ Pressure of tongue against mandible and teeth produces the lamina propria.
lateral crenated border of tongue. ♦♦ Mixed inflammatory cell infiltration in the connective
♦♦ Presence of open bite and mandibular prognathism is seen. tissue stroma.
♦♦ Children with macroglossia often develop tongue thrust­
Treatment
ing habits, which may lead to malocclusion, open bite and
diastema formation, etc. No treatment is required.
♦♦ Macroglossia developing in adult people (as in acromegaly
or in tumors, etc.) may produce spacing of teeth and distor­ Hairy Tongue
tion of the mandibular arch. ♦♦ Hairy tongue is an unusual condition, which occurs due
♦♦ Blockage of the pharyngeal airway due to macroglossia to hypertrophy of the filiform papilla of tongue along with
may result in a condition called obstruction sleep apnea. loss of normal desquamation process.
♦♦ Abnormal hair-like growth of the papilla eventually leads
Ankyloglossia
to formation of a pigmented, thick, matted layer on the
It is also known as tongue tie tongue surface often heavily coated with bacteria and fungi.
Ankyloglossia is the condition which arises when the inferior
Etiology
frenulum attaches to the bottom of tongue and subsequently
restricts free movements of the tongue. ♦♦ Poor oral hygiene
♦♦ Fungal infections
Clinical Features ♦♦ Prolonged use of antibiotics
♦♦ Males are affected more commonly than females ♦♦ Heavy smoking
♦♦ It can cause feeding problems in infants ♦♦ Excessive use of antiseptic mouth washes
♦♦ It causes speech defects specially articulation of the sounds ♦♦ Chronic illness
l, r, t, d, n, th, sh and z ♦♦ Lack of tooth brushing and consumption of soft foods with
♦♦ It leads to persistent gap between the mandibular incisors. little or no roughage.

Treatment Pathogenesis

Frenulectomy is the treatment of choice. ♦♦ Normally the keratinized surface layers of the tongue
papillae are continuously desquamated through friction
Cleft Tongue of the tongue with food, rough surface of the palate
and the upper anterior teeth. Following desquamation,
♦♦ It is a rare condition which arises due to lack of merging
tongue papillae are replaced by newer epithelial cells
of lateral lingual swellings of this organ.
from below.
♦♦ A partial cleft tongue is more commonly seen.
♦♦ Lack of tongue movements due to local or systemic disease
♦♦ Partial cleft occurs because of incomplete merging and
disturbs the regular desquamation process of the tongue
failure of groove obliteration by underlying mesen­chymal papilla; especially the filiform papillae, which lengthens
proliferation. considerably and produces a hairy appearance on the
Fissured Tongue tongue surface.
♦♦ Such hypertrophied papillae are often coated with micro­
It is also known as scrotal tongue. organisms and become discolored.
Clinical Features Clinical Features
♦♦ More common among males. ♦♦ Hairy tongue commonly affects the mid dorsum of the
♦♦ No clinical symptom is seen in fissured tongue but collec­ tongue.
tion of food debris and microorganisms in the fissures or ♦♦ Hypertrophy of the filiform papillae produces a thick mat­
grooves sometimes cause discomfort. ted layer on the dorsal surface.
♦♦ Fissures or grooves often radiate freely in central groove ♦♦ Hypertrophied filiform papillae may grow up to half a
on the dorsal surface in oblique direction. centimeter long, which often brushes the soft palate and
♦♦ Large and deep fissures may be inter-connected and they produces gagging sensations.
separate the dorsum of the tongue into multiple lobules. ♦♦ There can be irritation to the tongue due to accumulation
of food debris and microorganisms.
Histological Features
♦♦ There will be loss of filiform papillae from the surface Treatment
mucosa. ♦♦ Removal of etiological factor.
♦♦ Neutrophilic microabscess formation within the epithelium. ♦♦ Proper cleaning of tongue.
504   Mastering the BDS IIIrd Year  (Last 25 Years Solved Questions)

Q.22. Describe environmental enamel hypoplasia. initially in the oral mucosa and extending to the degenera­
(Mar 2011, 8 Marks) tion of the epithelial-derived ameloblasts, which results in a
Ans. Environmental enamel hypoplasia may be defined as an hypoplastic enamel matrix. If vitamin A deficiency is severe,
incomplete or defective formation of the organic enamel ameloblast cells will become completely atrophied, which re­
matrix of teeth. sults in an absence of enamel formation. In less severe cases, the
Enamel hypoplasia (EH) is a quantitative defect associated columnar ameloblasts apparently shorten, and adjacent enamel
with a reduced localized thickness of enamel, following exhibits hypoplasia. If vitamin A deficiency is relieved during
disruption of the secretory phase of amelogenesis. subsequent tooth development, normal enamel is produced,
The enamel may be translucent or opaque, with single or although defective tissue is not repaired.
multiple pits or grooves and partial or complete absence Enamel Hypoplasia Due to Vitamin D Deficiency
of enamel over significant areas of dentin. The enamel
hypoplasia defects tend to occur in the incisal or cuspal Vitamin D is essential for deposition of calcium and phosphorus
one-third of the crown. in hard tissues. Its presence increases the absorption of
• Defect which is caused by environmental factors, ei­ dietary calcium and maintains proper levels of calcium and
ther dentition may be involved and sometimes only phosphorus in the blood. Primary deficiency of vitamin D
a single tooth; both enamel and dentin are usually results from insufficient exposure to the sun and insufficient
affected, at least to some degree. dietary intake. Secondary deficiencies result from abnormal
• It is known that a number of different factors, each intestinal resorption. Secondary deficiencies may be overcome
capable of producing injury to the ameloblasts, may by alteration of dietary intake of calcium and phosphorus. A
give rise to the condition, including: severe vitamin D deficiency in children results in rickets, a
– Nutritional deficiency (vitamins A and D) condition characterized by insufficient deposition of calcium
– Exanthematous diseases (e.g. measles, chicken­ salts in bony tissue. Dental features of rickets include enamel
pox, scarlet fever) hypoplasia due to failure of tooth mineralization. Enamel
– Congenital syphilis hypoplasia is of pitted type.
– Hypocalcemia
Enamel Hypoplasia Due to Exanthematous Diseases
– Birth injury, Prematurity and Rh hemolytic
disease Some studies have indicated that exanthematous diseases,
– Local infection or trauma including measles, chickenpox and scarlet fever, are etiologic
– Ingestion of chemicals (chiefly fluoride) factors. In general, it might be stated that any serious
– Idiopathic nutritional deficiency or systemic disease is potentially
• In mild environmental hypoplasia, there may be only capable of producing enamel hypoplasia, since ameloblasts
a few small grooves, pits, or fissures on the enamel are one of the most sensitive groups of cells in body in terms
surface. If the condition is more severe, enamel may of metabolic function. The type of hypoplasia occurring from
exhibit rows of deep pits arranged horizontally these deficiency or disease states is usually of the pitting
across the surface of the tooth. There may be only a variety. Since the pits tend to stain, the clinical appearance
single row of such pits or several rows indicating a of the teeth may be very unsightly. Clinical studies indicate
series of injuries. that most cases of enamel hypoplasia involve those teeth
• In most severe cases, a considerable portion of that form within the first year after birth, although teeth
enamel may be absent, suggesting a prolonged that form somewhat later may be affected. Thus the teeth
disturbance in the function of the ameloblasts. most frequently involved are the central and lateral incisors,
• Hypoplasia results only if the injury occurs at the cuspids and first molars.
time teeth are developing, or more specifically dur­
Enamel Hypoplasia Due to Congenital Syphilis
ing the formative stage of enamel development.
Once the enamel has calcified, no such defect can Congenital syphilis arises from transplacental fetal infection
be produced. Thus, knowing the chronologic de­ with Treponema pallidium acquired during pregnancy from an
velopment of the deciduous and permanent teeth, untreated mother. The disease is divided into an early stage that
it is possible to determine from the location of the usually occur before 3 months, but may be seen up to 2 years,
defect on the teeth the approximate time at which and late stage disease that occurs after 2 years. Late congenital
the injury occurred. syphilis affect the amelogenesis of the molars and incisors.
Both Hutchinson teeth and mulberry molars are seen in about
Enamel Hypoplasia Due to Vitamin A Deficiency 65% of patients. These characteristic teeth present at around 6
Although tissues of ectodermal origin i.e., the epidermis is years; they are centrally notched, widely spaced, peg-shaped
primarily affected in vitamin A deficiency, teeth also record upper permanent central incisors. Patients with congenital
this deficiency. Avitaminosis A is evidenced by marked meta­ syphilis may also have mulberry molars, which are first molars
plasia of the enamel organ, which results in defective enamel dwarfed by a small occlusal surface, and are characterized by
formation. This view originates from histological changes seen roughened lobulated hypoplastic enamel leading to caries. The
Section 3: Oral Pathology  505

surface has numerous poorly formed cusps which overcome to Pathogenesis


form a dome-shaped tooth, which is considerably narrower at
Cleft in lower lip usually occurs either due to failure of the
the grinding surface than at its base.
copula to form the mandibular arch or due to persistence of
Enamel Hypoplasia Due to Hypocalcemia the central groove of the mandibular process.
Cleft of the upper lip and premaxilla occur due to failure of
Tetany induced by a decreased level of calcium in the blood, may
mesodermal penetration and subsequent obliteration of the
result from several conditions, the most common being vitamin ectodermal grooves between the median nasal process, lateral
D deficiency and parathyroid deficiency (para­thyroprivic nasal process and the maxillary process, which occurs during
tetany). In tetany the serum calcium level may fall as low as 6–8 the seventh week of intrauterine life.
mg per l00 mL, and at this level enamel hypoplasia is frequently
produced in teeth developing concomitantly This type of enamel Clinical Features
hypoplasia is usually of the pitting variety and thus does not
♦♦ Occur most commonly in males.
differ from that resulting from a nutritional disturbance or
♦♦ Most common type of isolated cleft lip only is unilateral
exanthematous disease.
complete type.
Enamel Hypoplasia Due to Birth Injuries ♦♦ Breastfeeding is impossible to babies having cleft lip, as
they cannot generate sufficient suction.
The neonatal line or ring, described by Schour in 1936 and ♦♦ There is difficulty in correct phonation and articulation
present in deciduous teeth and first permanent molars, may be of speech.
thought of as a type of hypoplasia because there is a disturbance ♦♦ Mental trauma to the child due to the unusual appearance.
produced in the enamel and dentin, which is indicative of
trauma or change of environment at the time of birth. Treatment
♦♦ Cosmetic repair of face and lips
Enamel Hypoplasia Due to Local Infection or Trauma
♦♦ Proper development of speech
A type of hypoplasia occasionally seen is unusual in that only a ♦♦ Prevention of maxillary arch collapse
single tooth is involved, most commonly one of the permanent Q.24. Write short note on fluorosis. (Feb 2013, 5 Marks)
maxillary incisors or a maxillary or mandibular premolar. There Ans. Fluorosis is a toxic manifestation of chronic fluoride
may be any degree of hypoplasia, ranging from a mild, brownish intake.
discoloration of the enamel to a severe pitting and irregularity • It can be described as a diffuse symmetric hypomin­
of the tooth crown. Another frequent pattern of enamel defects eralization disorder of ameloblasts.
seen in permanent teeth is caused by periapical inflammatory • Fluorosis is irreversible and only occurs with expo­
disease of the overlying deciduous tooth. The altered tooth is sure to fluoride when enamel is developing.
called Turner’s tooth. • Instead of being a normal creamy-white translucent
Q.23. Write short note on cleft lip. (Jan 2012, 5 Marks) color, fluorosed enamel is porous and opaque.
Ans. Cleft lip is a developmental anomaly characterized by • Teeth can resemble a ghostly white chalk colored
wedge shaped defect in the lip which results from failure (light refractivity is greatly reduced because the
of two parts of the lip to fuse together at the time of enamel’s prism structure is defective).
development. • Cloudy striated (lines of demarcation) enamel, white
specks or blotches, ‘snow-capping’, yellowish-brown
spots, or brown pits on teeth are all characteristic of
Etiology
fluorosis.
♦♦ Nutritional factors such as deficiency of or excess of vita­ • In its more severe form, fluorosed enamel is structur­
min A and deficiency of riboflavin. ally weak (brittle) and prone to erosion and break­
♦♦ Maternal smoking (during pregnancy) is a very high-risk age, especially when drilled and filled.
factor. • Even in the milder forms, there is increased enamel
♦♦ Psychogenic, emotional or traumatic stress in pregnant attrition.
mothers. • To prevent fluorosis from occurring in the most
♦♦ Relative ischemia to the area due to defective vascular prominent and/or most susceptible teeth, the most
supply. critical time to avoid fluoride exposure is the first
♦♦ Mechanical obstruction by enlarged tongue. three to six years of a child’s life.
♦♦ High dose of steroid therapy during pregnancy. Fejerskov et al. (1977) stated that the effect of fluoride on enamel
♦♦ Localized mucopolysaccharide metabolism defect in the area. formation can follow several possible pathogenic pathways:
♦♦ Infections. ♦♦ Effect on ameloblasts:
♦♦ Substances such as alcohol, drugs or toxins in the • Secretory phase:
circulation. –– Diminished matrix production
506   Mastering the BDS IIIrd Year  (Last 25 Years Solved Questions)

–– Change of matrix composition ♦♦ Type II–Hypomaturation


–– Change in ion transport mechanism ♦♦ Type IIA–Hypomaturation, pigmented autosomal recessive
• Maturation phase: Diminished withdrawal of protein ♦♦ Type IIB–Hypomaturation, X-linked recessive
and water ♦♦ Type IIC–Hypomaturation, snow-capped teeth, X-linked
♦♦ Effect on nucleation and crystal growth in all stages of ♦♦ Type IID–Hypomaturation, snow-capped teeth, autosomal
enamel formation dominant?
♦♦ Effect on calcium homeostasis generally with dental fluo­ ♦♦ Type IIIA–Autosomal dominant
rosis as an indirect result ♦♦ Type IIIB–Autosomal recessive
–– After the tooth erupts and calcification has been ♦♦ Type IV–Hypomaturation-hypoplastic with taurodontism
completed, ingested fluoride does not have ad­ ♦♦ Type IVA–Hypomaturation-hypoplastic with tauro­
verse dental consequences. dontism, autosomal dominant
–– Fluorosis is seen to affect mainly permanent denti­ ♦♦ Type IVB–Hypoplastic-hypomaturation with tauro­
tion and very high fluoride levels (>10 ppm) are dontism, autosomal dominant
required in drinking water for it to cross placental For types and clinical features, refer to Ans 7 of same chapter.
barrier and affect primary dentition.
Also refer to Ans 8 of same chapter. Histopathology
Q.25. Enumerate the developmental disturbances affecting ♦♦ There is a disturbance in the differentiation or viability of
the structures of teeth and discuss in detail amelogenesis ameloblasts in the hypoplastic type, and this is reflected
imperfecta. (Feb 2013, 10 Marks) in defect in matrix formation up to and including total
Or absence of matrix.
♦♦ In the hypocalcification type there are defects of matrix
Enumerate developmental disturbances affecting structure and of mineral deposition.
structures of teeth. Write in detail about amelogenesis ♦♦ In the hypomaturation type there are alterations in enamel
imperfecta. (July 2016, 10 Marks) rod and rod sheath structures.
Ans. ♦♦ Ground section of the teeth involved showed very thin
Enumeration of Developmental Disturbances Affecting enamel, composed of laminations of irregularly arranged
Structure of Teeth enamel prisms.

♦♦ Amelogenesis imperfecta Radiographic Features


♦♦ Dentinogenesis imperfecta ♦♦ The enamel may appear totally absent on the radiograph,
♦♦ Enamel hypoplasia
or when present may appear as a very thin layer chiefly
♦♦ Dentin dysplasia
over the tips of cusps and on the inter-proximal surfaces.
♦♦ Regional odontodysplasia
♦♦ In other cases the calcification of the enamel may be so
♦♦ Dentin hypocalcification
affected that it appears to have the same approximate ra­
Amelogenesis Imperfecta diodensity as the dentin, making differentiation, between
the two difficult.
Amelogenesis imperfecta is a group of hereditary disorders
characterized by alteration of the quantity and quality of enamel Treatment
in humAns and is frequently associated with a significant dental
♦♦ There is no definite treatment of amelogenesis imperfecta.
disease. Witkop and Sauk (1976)
♦♦ Composite veneering can be done to improve esthetics
Classification of Amelogenesis Imperfecta of teeth.

Witkop, 1988 Four major categories based primarily on Q.26. Describe histologic features with diagram of ghost
phenotype (hypoplastic, hypomaturation, hypocalcified, teeth. (Nov 2008, 5 Marks)
hypomaturation-hypoplastic with taurodontism) subdivided
Ans. It is also known as regional odontodysplasia.
into 15 subtypes by phenotype and secondarily by mode of
inheritance. Following are the histologic features of ghost teeth:
♦♦ Type I–Hypoplastic ♦♦ In ground section enamel thickness varies.
♦♦ Type IA–Hypoplastic, pitted autosomal dominant ♦♦ Prism structure of enamel is irregular and it lacks lami­
♦♦ Type IB–Hypoplastic, local autosomal dominant nated appearance.
♦♦ Type IC–Hypoplastic, local autosomal recessive ♦♦ Dentin show clefts which are scattered through mixture of
♦♦ Type ID–Hypoplastic, smooth autosomal dominant interglobular dentin and amorphous material.
♦♦ Type IE–Hypoplastic, smooth X-linked dominant ♦♦ Reduction in amount of dentin is seen.
♦♦ Type IF–Hypoplastic, rough autosomal dominant ♦♦ Widening of predentin layer is present.
♦♦ Type IG–Enamel agenesis, autosomal recessive ♦♦ Large areas of interglobular dentin are seen.
Section 3: Oral Pathology  507

♦♦ Pulp contains free or attached pulp stones which exhibit Clinical Features
tubules or have laminated calcification. ♦♦ Hypodontia is very uncommon in deciduous dentition.
♦♦ Follicular tissue surrounding the crown is enlarged and ♦♦ When hypodontia is present it frequently involves lateral
exhibit collections of basophilic enamel like calcifications incisors.
called as enameloid conglomerates. ♦♦ In permanent dentition third molars are more commonly
affected and after these second premolars and lateral inci­
sors are absent.
♦♦ Hypodontia is also associated with microdontia.

Treatment
♦♦ Fixed prosthesis should be given to patient in form of
bridges.
♦♦ Removable partial dentures can also be given.
Q.31. Discuss in detail about etiology, clinical features and
treatment of cleft lip and palate. Add a note on syn-
dromes associated. (Aug 2012, 10 Marks)
Ans. For etiology, clinical features of cleft lip refer to Ans 23
Fig. 18:  Ghost teeth of same chapter.
(For color version, see Plate 3)
Cleft Palate
Q.27. Write in detail enamel hypoplasia.
(May/Jun 2009, 10 Marks) Etiology of cleft lip and cleft palate is similar so for etiology,
Ans. For enamel hypoplasia, refer to Ans 22 of same chapter. refer to Ans 23 of same chapter.

Q.28. Classify enamel hypoplasia. Describe in detail about Clinical Features


environmental factors causing enamel hypoplasia. ♦♦ It is seen more common in females as compared to males.
(Jan 2012, 15 Marks) ♦♦ Cleft is seen over the hard and soft palate and it can also
Ans. be seen in cleft of soft palate alone.
♦♦ Extension of cleft palate varies, i.e. it involves uvula or
Classification of Enamel Hypoplasia
soft palate, at times it extend over the complete palate,
♦♦ Mild: Presence of few small grooves, pits and fissures on it also sometimes involve alveolar ridge unilaterally or
enamel bilaterally.
♦♦ Moderate: Presence of rows of deep pits arranged horizon­ ♦♦ Patient feels difficulty in drinking and eating since liquid
tally over surface. and food regurgitates through the nose.
♦♦ Severe: Portion of enamel may be absent ♦♦ Patient have problem in speaking.
For environmental factors causing enamel hypoplasia, refer to ♦♦ Upper lateral incisors of patient are short or may be absent.
Ans 22 of same chapter. ♦♦ Crossbite is present.
Q.29. Write in brief on macroglossia. (Jun 2010, 5 Marks)
Or Treatment of Cleft Lip and Cleft Palate
Write short note on macroglossia. (Jun 2017, 5 Marks)
♦♦ Cleft palate is usually repaired in l2–l8 months. Early re­
Ans. Refer to Ans 21 of same chapter. pair causes retarded maxillary growth. Late repair causes
Q.30. Write short note on hypodontia. (Aug 2012, 5 Marks) speech defect.
Ans. It is the developmental alteration in number of teeth. ♦♦ Both soft and hard palates are repaired.
• Hypodontia denotes the lack of development of one ♦♦ Abnormal insertion of tensor palati is released. Mucoperi­
or more teeth. osteal flaps are raised in the palate which is sewed together.
• Oligodontia is a type of hypodontia which indicates ♦♦ If maxillary hypoplasia is present, then osteotomy of the
lack of development of six or more teeth. maxilla is done. With orthodontic teeth extraction and
alignment of dentition is done.
Etiology ♦♦ Regular examination of ear, nose and throat during follow-
♦♦ Genetic: Various syndromes are associated such as he­ up period, i.e. postoperative speech therapy.
reditary ectodermal dysplasia, craniofacial dysostosis etc. ♦♦ Whenever complicated problems are present, staged surgi­
♦♦ At present scenario evolution is there towards some teeth. cal procedure is done.
♦♦ X-ray radiation causing damage to developing tooth bud ♦♦ Wardil-Kilner push back operation-by raising mucoperi­
results in absence of teeth. osteum flaps based on greater palatine vessels.
508   Mastering the BDS IIIrd Year  (Last 25 Years Solved Questions)

♦♦ Hearing support is given using hearing aids if defect is ♦♦ Accessory teeth may be present at or shortly after birth.
present; control of otitis media. ♦♦ Teeth present in newborn baby are known as natal teeth
♦♦ Speech problems occur due to velopharyngeal incompe­ while teeth arising in first 30 days of life are known as
tence; articulation problems also can occur speech therapy neonatal teeth.
is given. It is corrected by pharyngoplasty, veloplasty, ♦♦ These teeth may represent pre-deciduous supernumer­
speech devices. ary teeth.
♦♦ Dental problems like uneruption, unalignment are com­ Q.33. Enumerate the different developmental disturbances
mon. They should be corrected by proper dentist opinion, of teeth on basis of size, number and shape. Describe
and reconstructive surgery. in detail the developmental disturbances in structure
♦♦ Orthodontic management with alveolar bone graft, maxil­ of tooth. (Mar 2013, 8 Marks)
lary osteotomy is done in 8–11 years.
Ans. For enumeration of developmental disturbances of teeth
Syndromes Associated with Cleft Lip and Cleft Palate on basis of size, number and shape, refer to Ans 9 of same
chapter.
There are multiple syndromes associated but most common
syndrome associated is Pierre-Robin syndrome. This syndrome Developmental Disturbances in Structure of Tooth
is characterized by cleft palate, micrognathia and glossoptosis.
Following are the developmental disturbances in structure of
Other syndromes associated are as follows: tooth:
1. Goldenhar syndrome: Cleft palate, microstomia, hypoplastic ♦♦ Amelogenesis imperfecta or hereditary enamel dysplasia
zygomatic arch. or hereditary brown enamel or hereditary opalescent teeth.
♦♦ Marfan’s syndrome: Cleft palate, skeletal defects, ocular ♦♦ Environmental enamel hypoplasia.
lens defect. ♦♦ Dentinogenesis imperfecta.
♦♦ Down’s syndrome: Cleft palate, teeth anomalies, fissured ♦♦ Dentin dysplasia.
tongue, malocclusion. ♦♦ Regional odontodysplasia or odontodysplasia or odonto­
♦♦ Patau syndrome: Cleft lip and cleft palate, polydactyly and genic dysplasia, odontogenesis imperfecta or ghost teeth
heart anomalies. ♦♦ Dentin hypocalcification.
♦♦ Orofacial digital syndrome: Cleft lip, digital malformation, For amelogenesis imperfecta, refer to Ans 7 and Ans 25 of same
deformed facial features. chapter.
♦♦ Treacher Collin syndrome: Cleft lip and cleft palate, facial
For environmental enamel hypoplasia, refer to Ans 22 of same
deformities.
chapter.
♦♦ Median cleft face syndrome: Cleft palate, nasal cleft and
frontonasal dysplasia. For dentinogenesis imperfecta, refer to Ans 14 of same chapter.
♦♦ Otopalatodigital syndrome: Cleft palate, mandibular micro­ For dentin dysplasia, refer to Ans 18 of same chapter.
gnathia, facial deformity.
♦♦ Blepharocheilodontic syndrome: Eye anomalies, cleft lip and Regional Odontodysplasia
palate, microdontia. It is also known as odontodysplasia or ghost teeth.
Q.32. Write short note on developmental disturbances in ♦♦ In this both ectodermal and mesodermal tooth components
number of teeth. (Dec 2012, 3 Marks) are affected.
♦♦ In this tooth is hypocalcified and hypoplastic.
Or
Describe in detail developmental disturbances af- Etiology
fecting number of teeth. (Nov 2014, 8 Marks) ♦♦ Due to abnormal migration of neural crest cells
Ans. Developmental disturbances in number of teeth ♦♦ Due to medication taken during pregnancy
• Anodontia ♦♦ In local trauma or infection
• Hypodontia ♦♦ In cases of radiation therapy
• Hyperdontia ♦♦ Due to somatic mutations
For anodontia in detail refer to Ans 6 of same chapter. ♦♦ Local circulatory deficiency
For hypodontia in detail refer to Ans 30 of same chapter.
Clinical Features
Hyperdontia ♦♦ Both the dentitions, i.e. deciduous and permanent
♦♦ Hyperdontia is the development of an increased number dentitions are affected.
of teeth. ♦♦ Slight female predilection is present.
♦♦ The additional teeth are known as supernumerary teeth. ♦♦ Maxillary teeth are more commonly involved as compared
♦♦ For supernumerary teeth in detail refer to Ans 13 of same to mandibular teeth. More often involved are maxillary
chapter. anterior teeth.
Section 3: Oral Pathology  509

♦♦ Many of the affected teeth are failed to erupt. ♦♦ Gemination: Refer to Ans 11 of same chapter
♦♦ Erupted teeth have small irregular crowns that are yellow ♦♦ Twinning
to brown with rough surface. ♦♦ Fusion: Refer to Ans 11 of same chapter
♦♦ Presence of caries and periapical inflammatory lesions ♦♦ Concrescence: Refer to Ans 19 of same chapter
are common. ♦♦ Dilaceration: Refer to Ans 4 of same chapter
♦♦ Gingival swelling is present adjacent to the affected tooth. ♦♦ Talon cusp
♦♦ Dens invaginatus: Refer to Ans 5 of same chapter
Histopathology ♦♦ Dens evaginatus
For histopathology, refer to Ans 26 of same chapter. ♦♦ Taurodontism: Refer to Ans 12 of same chapter
♦♦ Hypercementosis: Refer to Ans 6 of chapter regressive
Treatment alterations of teeth
♦♦ Prosthetic replacement of teeth should be done. ♦♦ Enamel pearl
♦♦ In some of teeth root canal should be done followed by ♦♦ Mulberry molar
capping of teeth. ♦♦ Globodontia
♦♦ Hutchinson’s incisor.
Q.34. Enumerate the developmental disturbances affecting the
structure of teeth and discuss in detail dentinogenesis Twinning
imperfecta. (June 2014, 10 Marks)
Cleavage of tooth germ leads to formation of supernumerary
Ans. For enumeration of developmental disturbances affecting tooth which is duplicate image of tooth from which it is
the structure of teeth, refer to Ans 25 of same chapter. developed.
For dentinogenesis imperfecta in detail, refer to Ans 14
of same chapter. Talon Cusp
Q.35. Write short note on Turner’s tooth. (June 2014, 5 Marks) ♦♦ It resembles as an eagle’s talon which projects lingually
Ans. Turner’s tooth is also known as Turner’s hypoplasia. from cingulum areas of a maxillary or mandibular per­
It is a localized enamel hypoplasia caused due to local manent incisor.
infection or trauma and tooth affected is known as ♦♦ Talon’s cusp is more prevalent in person’s with Rubinstein-
Turner’s tooth. Taybi syndrome.
♦♦ It blends smoothly with the tooth except that there is deep
Pathogenesis developmental groove where cusp blends with sloping
♦♦ If a deciduous tooth is affected by dental caries when lingual tooth surface.
crown of succeeding permanent tooth is formed, bacterial ♦♦ Talons’s cusp consists of normal enamel and dentin and
infection involving periapical tissues may occur and this consists of horn of pulp tissue.
disturbs the ameloblastic layer of permanent tooth bud ♦♦ Major significance of talon’s cusp is that there is occlusal
which leads to hypoplastic crown. interference and there is also high incidence of caries.
♦♦ During trauma when deciduous tooth get lodged in
Dens Evaginatus
alveolus and disturb the budding permanent tooth bud,
this results in yellowish or brownish stain or pigmentation ♦♦ It is also known as Leong’s premolar, occlusal tuberculated
of enamel on labial surface, at times pitting of enamel and premolar and occlusal enamel pearl.
irregularity of tooth crown is also present. ♦♦ It is a developmental condition which appears clinically
as an accessory cusp or a globule of enamel on occlusal
Clinical Features surface between buccal and lingual cusps of premolar.
♦♦ The enamel defects vary from focal areas of white, yellow, ♦♦ Dens evaginatus occur due to proliferation and evagination
or brown discoloration to extensive hypoplasia, which can of an area of the inner enamel epithelium and subjacent
involve the entire crown. odontogenic mesenchyme into the dental organ during
♦♦ Anterior teeth are involved less frequently because crown early development of tooth.
formation is usually complete before the development of ♦♦ Clinical significance is that the extra cusp may contribute
any apical inflammatory disease in the relatively caries- to incomplete eruption, displacement of teeth and pulp
resistant anterior deciduous dentition. exposure with subsequent infection following occlusal
♦♦ Maxillary central incisors are affected in the majority of wear or fracture.
the cases; the maxillary lateral incisors are altered less
frequently. Enamel Pearl
Q.36. Describe developmental disturbances affecting shape ♦♦ It is also known as enameloma or ectopic enamel.
of tooth. (Feb 2014, 8 Marks) ♦♦ Enamel pearl is a nodule of enamel which is 1 to 2 mm in
Ans. Following are the developmental disturbances affecting diameter which form on the root, at root bifurcation, at
shape of tooth: root trifurcation.
510   Mastering the BDS IIIrd Year  (Last 25 Years Solved Questions)

♦♦ Enamel pearl originates due to activity of remnants of peri and para vascular aggregation of lymphocytes,
Hertwig’s epithelal before it get reduced to cell rest of plasma cells and histiocytes. At places non-caseating
Malasez. granuloma formation is seen along with epithelioid
♦♦ Clinical significance is that these pearls give rise to peri­ cells and LanghAns type giant cells.
odontal problems. • Six patterns of fissured tongue are associated with
the Melkersson, Rosenthal syndrome, i.e. central lon­
Mulberry Molar
gitudinal fissuring, transverse fissuring originating
♦♦ It occurs in syphilis. from central fissure, plication, lateral longitudinal
♦♦ Posterior tooth is involved. fissuring, transverse fissuring with central fissure
♦♦ In this hyperplastic enamel occur with the spherical ag­ and lateral longitudinal fissuring.
gregates or the globules over surface of dentin. Q.39. Write short note on talon’s cusp. (Feb 2015, 5 Marks)
Globodontia Ans. Talon’s cusp is a well delineated accessory cusp which
resembles as eagle’s talon which project lingually from
♦♦ Globodontia is the enlarged bulbous malformed posterior
cingulum areas of maxillary or mandibular permanent
tooth with no discernable cusps or grooves. In this teeth
incisor.
have a clover leaf appearance.
• Talon’s cusp consists of normal enamel and dentin
♦♦ In globodontia relation between cusps and groove get
and has a horn of pulp tissue.
eliminated.
• Talon’s cusp blends smoothly with the tooth except
♦♦ This condition occur both in deciduous or permanent
that there is deep developmental groove where the
dentition.
cusp blends with sloping lingual tooth surface.
Hutchinson’s Incisor • Due to presence of Talon’s cusp patient’s face the
problem with esthetics. There is also presence of
♦♦ It occurs in syphilis.
occlusal interference and high incidence of caries is
♦♦ In this incisal edge show notching.
♦♦ Tooth has shape of screw driver. also noticed.
• Talon’s cusp is associated with Rubinstein – Taybi’s
Q.37. Enumerate causes of enamel hypoplasia and describe syndrome as well as Sturge-Weber syndrome.
amelogenesis imperfecta in detail.
 (Feb 2013, 16 Marks) Q.40. Write short note on facial hemiatrophy.
 (Dec 2015, 3 Marks)
Ans.
Ans. It is also known as Parry – Romberg syndrome.
Enumeration of Causes of Enamel Hypoplasia It is the slowly progressive atrophy of soft tissue mainly half of
Following are the causes of enamel hypoplasia: the face and is also characterized by wasting of subcutaneous
♦♦ Nutritional deficiency (Vitamins A, C and D) fat which sometimes accompanied by atrophy of skin, cartilage,
♦♦ Exanthematous diseases, i.e. measles, chicken pox and bone and muscle.
scarlet fever Clinical Features
♦♦ Congenital syphilis
♦♦ Hypocalcemia ♦♦ It occurs during first and second decades of life.
♦♦ Birth injury, prematurity, Rh hemolytic disease ♦♦ Females are more commonly affected as compared to
♦♦ Local infection or trauma males.
♦♦ Ingestion of chemicals such as fluoride ♦♦ It occurs more commonly over the left side of face.
♦♦ Idiopathic causes ♦♦ Onset of facial hemiatrophy is marked by white line fur­
For amelogenesis imperfecta in detail refer to Ans 25 of same row.
chapter. ♦♦ Earliest sign is the presence of painless cleft.
♦♦ Patients can show sharp line of demarcation which resem­
Q.38. Write short note on Melkersson–Rosenthal syndrome. ble as large scar in between normal and abnormal skin.
 (Feb 2015, 5 Marks) This is known as Coup de sabre.
Ans. This syndrome is the triad of chelitis granulomatosa, ♦♦ Overlying the atrophic fat a bluish hue may appear on
fissured tongue, facial palsy. the skin.
• Melkersson–Rosenthal syndrome is occasionally a
manifestation of Crohn disease or orofacial granu­ Oral Manifestations
lomatosis. ♦♦ Presence of severe facial hemiatrophy which leads to facial
• A genetic predisposition may exist in Melkersson– deformation and mastication.
Rosenthal syndrome. ♦♦ Presence of hemiatrophy of lips and tongue.
• Histologically the swellings of syndrome consist of ♦♦ Delayed eruption is present which leads to malocclusion.
chronic inflammatory cell infiltration which shows ♦♦ Presence of incomplete root formation.
Section 3: Oral Pathology  511

♦♦ During opening the mouth jaw is deviated towards af­ ♦♦ Mesiodens is a small tooth consisting of cone shaped crown
fected side. and short root.
♦♦ Mesiodens can lead to retarded eruption, displacement or
Differential Diagnosis resorption of adjacent root.
♦♦ Post traumatic atrophy: Brief history from patient is im­ ♦♦ Mesiodens can frequently cause improper alignment.
portant. ♦♦ It usually results in oral problems such as malocclusion,
♦♦ Goldenhar syndrome: It is non-progressive and is congenital. food impaction, poor esthetics, and cyst formation.
♦♦ Mandibulofacial dysostosis: It is hereditary and cleft palate ♦♦ Immediate removal of mesiodens is usually indicated in
is present. the following situations; inhibition or delay of eruption,
displacement of the adjacent tooth, interference with or­
Treatment thodontic appliances, presence of pathologic condition, or
♦♦ Surgical reconstruction is done. spontaneous eruption of the supernumerary tooth.
♦♦ Malocclusion should be corrected by orthodontic treat­
ment. Q.42. List developmental disturbances affecting the tooth.
Discuss taurodontism and dilacerations.
Q.41. Write short note on mesiodens. (Apr 2017, 5 Marks)
 (May 2018, 5 Marks)
Ans. Mesiodens is a supernumerary tooth in maxillary
anterior incisor region. Ans. Developmental disturbances affecting the tooth

Developmental Disturbances affecting Structure of tooth


♦♦ Amelogenesis imperfecta
♦♦ Dentinogenesis imperfecta
♦♦ Enamel hypoplasia
♦♦ Dentin dysplasia
♦♦ Regional odontodysplasia
♦♦ Dentin hypocalcification

Developmental disturbances affecting number of tooth


♦♦ Anodontia
♦♦ Hypodontia
♦♦ Hyperdontia

Developmental disturbances affecting shape of tooth


♦♦ Gemination
♦♦ Twinning
Fig. 19: Mesiodens (For color version, see Plate 3) ♦♦ Fusion
♦♦ Mesiodens represents one of the more common supernu­ ♦♦ Concrescence
merary teeth and can erupt spontaneously. ♦♦ Dilaceration
♦♦ Mesiodens supernumerary tooth was classified according ♦♦ Talon cusp
to its location, i.e. it is located at or near to midline in incisal ♦♦ Dens invaginatus
region of maxilla between the central incisors. ♦♦ Dens evaginatus
♦♦ According to the shape and size, two subclasses are consid­ ♦♦ Taurodontism
ered in the classification of mesiodens; namely, eumorphic ♦♦ Hypercementosis
and dysmorphic. The eumorphic subclass is usually similar ♦♦ Enamel pearl
to a normal-sized central incisor, whereas the dysmorphic ♦♦ Mulberry molar
teeth have different shapes and sizes and are categorized ♦♦ Globodontia
into conical, tuberculate, supplemental and odontomes. ♦♦ Hutchinson’s incisor
Out of these conical form is most common type. Enumeration of Anomalies Affecting Size of Tooth
♦♦ Mesiodens may be seen as an isolated finding or as part
of a syndrome, specially cleft lip and palate, cleidocranial ♦♦ Microdontia
dysostosis and Gardner’s syndrome. ♦♦ Macrodontia
♦♦ Mesiodens can occur either as single tooth or it is paired. For taurodontism in detail refer to Ans 12 of same chapter.
It can be impacted or inverted. For dilacerations in detail refer to Ans 4 of same chapter.
512   Mastering the BDS IIIrd Year  (Last 25 Years Solved Questions)

2. Benign and malignant development of leukoplakia. It is believed that during


smoking a large amount of tobacco end products are
tumors of Oral cavity produced in oral cavity. The products in association with
heat cause severe irritation to oral mucus membrane and
Q.1. Enumerate white lesions of orofacial region. Describe finally result in development of leukoplakia.
etiology, histopathology and clinical features of leu- ♦♦ Alcohol: Alcohol leads to the entry of carcinogen into
koplakia. (Sep 1999, 15 Marks) exposed cells and thus alters oral epithelium as well as
its metabolism.
Or ♦♦ Candidiasis: Chronic candidal infections are associated
Enumerate white lesions of oral cavity. Describe leu- with leukoplakia.
koplakia in detail. (June 2015, 10 Marks) ♦♦ Dietary deficiency: Deficiency of vitamin A causes meta­
Ans. The white lesions of oral cavity/orofacial region are as plasia and hyperkeratinization of epithelium which may
follows: result in development of leukoplakia.
• Hereditary condition: ♦♦ Syphilis: Syphilitic infections play minor role in causation
– Leukoedema of leukoplakia.
– White sponge nevus ♦♦ Hormonal imbalance: Imbalance or dysfunction of both
– Hereditary benign intraepithelial dyskeratosis male and female sex hormones causes keratogenic changes
– Keratosis follicularis in oral epithelium. These changes lead to the development
– Ptylosis syndrome. of leukoplakia.
• Leukoplakia and malignancies: Modified Classification of Leukoplakia
– Chronic cheek biting
– Friction or trauma associated leukoplakia L1 Size of leukoplakia <2 cm
– Tobacco associated leukoplakia L2 Size of leukoplakia 2 to 4 cm
– Carcinoma in situ L3 Size of leukoplakia >4 cm
– Squamous cell carcinoma
Lx Size not specified
– Verrucous carcinoma.
• Dermatosis: P0 No epithelial dysplasia
– Lichen planus P1 Distinct epithelial dysplasia
– Lupus erythematous. Px Dysplasia not specified in pathology report
• Inflammation:
– Mucous patches of syphilis Oral leukoplakia staging system
– Candidiasis Stage 1 – L1P0
– Koplik spots of measles.
Stage 2 – L2P0
• Miscellaneous conditions:
Stage 3 – L3P0 Or L1L2P1
– Oral submucous fibrosis
– Papilloma Stage 4 – L3P1
– Lipoma
Histopathology
– Hairy tongue
– Geographic tongue During leukoplakia variety of histologic changes are present
– Fordyce’s granules. which are related to:
Leukoplakia is defined as “a white patch or plaque that cannot ♦♦ Keratinization pattern
be characterized, clinically or pathologically as any other disease ♦♦ Changes in cellular layer
and is not associated with any other physical or chemical ♦♦ Thickness of epithelium
causative agent except the use of tobacco”. Axell et al, 1984 ♦♦ Alteration in underlying connective tissue stroma.
Leukoplakia is defined as a predominantly white lesion of the
oral mucosa that cannot be characterized as any other definable Keratinization Pattern
lesion. WHO (1997)
Leukoplakia generally presents hyperorthokeratinization or
Leukoplakia should be used to recognize white plaques hyperparakeratinization or both with or without the presence
of questionable risk having excluded (other ) known of epithelial dysplasia.
diseases or disorders that carry no increased risk for cancer.
♦♦ In case of leukoplakia an abnormal increase in the thickness
Warnakulasuriya et al (2008).
of orthokeratin layer is seen in area of epithelium which
Etiology of Leukoplakia is usually keratinized.
The common predisposing factors of leukoplakia are: ♦♦ An important histological criterion of leukoplakia is
♦♦ Tobacco: It is used by large number of people in various presence of hyperkeratinization of normally keratinized
forms such as smoking of cigarette, cigar, biddies and epithelium or some amount of parakeratin deposition in
pipes. All these types of tobacco habits are important for area of epithelium which are usually not keratinized.
Section 3: Oral Pathology  513

♦♦ Epithelium dysplasia is more frequently associated with


hyperkeratinized lesion.

Changes in Cellular Layer


Epithelial dysplasia is the hallmark of histologic changes seen
in epithelium in case of leukoplakia. The criteria for epithelial
dysplasia are:

Architecture and Cytologic Criteria for Grading Epithelial


Dysplasia Given by WHO (2005)
Architecture Criteria
♦♦ Irregular epithelial stratification
♦♦ Loss of polarity of basal cells
♦♦ Basal cell hyperplasia
♦♦ Drop shaped rete pegs
♦♦ Increased number of mitotic figures
A.  Mild dysplasia
♦♦ Abnormally superficial mitosis
♦♦ Dyskeratosis i.e. premature keratinization in the cell
♦♦ Keratin pearls within rete ridges
Cytologic Criteria
♦♦ Anisonucleosis: Abnormal variation in nuclear size
♦♦ Nuclear pleomorphism: Abnormal variation in nuclear shape
♦♦ Anisocytosis: Abnormal variation in cell size
♦♦ Cellular pleomorphism: Abnormal variation in cell shape
♦♦ Increased nuclear cytoplasmic ratio
♦♦ Increased nuclear size
♦♦ Atypical mitotic figures
♦♦ Increase in the number and size of nucleoli
♦♦ Hyperchromatism
Histopathology
♦♦ Classification of epithelial dysplasia is done on basis of its
severity which is:
• Mild epithelial dysplasia: It refers to the alteration B.  Moderate dysplasia
which is limited to basal and parabasal cell layers.
Figs 20A and B: Leukoplakia (For color version, see Plate 3)
• Moderate epithelial dysplasia: It shows involvement
from basal layer to mid – portion of spinous cell layer. Clinical Features
• Severe epithelial dysplasia: It shows alterations
from basal layer to the level above mid – portion of ♦♦ Usually the lesion occurs in 4th, 5th, 6th and 7th decades
epithelium. of life.
• When complete thickness of epithelium, term ♦♦ Buccal mucosa and commissural areas are most frequent
carcinoma in situ is used. affected sites followed by alveolar ridge, tongue, hard and
♦♦ Histopathological report of leukoplakia should in­ soft palate, etc.
clude a statement on absence or presence of epithelial ♦♦ Oral leukoplakia often present solitary or multiple white
dysplasia.. patches.
♦♦ Size of lesion may vary from small well localized patch
Thickness of the Epithelium measuring few millimeters in diameter.
♦♦ The surface of lesion may be smooth or finely wrinkled or
In leukoplakia the thickness of epithelium is altered and it occurs
even rough on palpation and lesion cannot be removed
in epithelial atrophy or acanthosis.
by scrapping.
♦♦ Lesion is whitish or greyish or in some cases it is brownish
Alteration in Underlying Connective Tissue
yellow in color due to heavy use of tobacco.
In leukoplakia there is often variable degree of destruction of ♦♦ In most of the cases these lesion are asymptomatic, how­
collagen fibers and moreover chronic inflammatory cell infiltrate ever in some cases they may cause pain, feeling of thickness
is also present in underlying connective tissue stroma. and burning sensation, etc.
514   Mastering the BDS IIIrd Year  (Last 25 Years Solved Questions)

Q.2. Define the premalignant lesions and conditions. De- ♦♦ The lesion often appears as white patch and is character­
scribe etiology, histopathology and clinical features of ized by an irregular surface, exhibiting numerous linear
leukoplakia. (Sep 1999, 15 Marks) vertical folds or projections, sometimes so marked to as
Or resemble “Hairs”.
♦♦ The lesions are always colonized by Candida albicans.
Define and enumerate precancerous lesions. Describe ♦♦ Hairy leukoplakia probably occurs as an opportunistic
in detail leukoplakia. (Dep 2011, 8 Marks) infection caused by Epstein-Barr virus.
Or ♦♦ Hairy leukoplakias are asymptomatic lesions and when­
Define and enumerate precancerous lesions and ever they occur they occur on buccal mucosa, the lesions
conditions. Write in detail about histopathological are smooth and homogeneous with straitened margin.
features of leukoplakia. (Aug 2011, 15 Marks) Histopathology
Or ♦♦ A very characteristic finding in hairy leukoplakia is pres­
Define precancerous lesions and conditions of oral ence of subcorneal upper spinous layer zone made up of
cavity with examples. Describe leukoplakia in detail. cytopathologically altered keratinocytes.
 (Jan 2018, 10 Marks) ♦♦ Parakeratin layer is thick often colonized by candidal
Ans. Premalignant Lesions or Precancerous Lesions hyphae.
Premalignant lesions are defined as ”A morphologically ♦♦ The submucosa does not exhibit many inflammatory cell
altered tissue in which cancer is more likely to occur then infiltrate.
in its apparently normal counter part”.
Differential Diagnosis
The examples are:
• Leukoplakia ♦♦ Lichen planus
• Erythroplakia ♦♦ Verrucous leukoplakia
• Mucosal changes associated with smoking habits ♦♦ Chronic tongue biting habits.
• Actinic cheilitis, actinic keratosis and actinic elastosis
• Bowen’s disease
• Carcinoma in situ

Premalignant Condition Or Precancerous Condition


The premalignant condition is defined as “A generalized state
of body, which is associated with significantly increased risk
of cancer development”.
The examples are:
♦♦ Oral submucus fibrosis
♦♦ Syphilis
♦♦ Lichen planus
♦♦ Sideropenic dysphagia
♦♦ Lupus erythematosus
♦♦ Dyskeratosis congenita
♦♦ Xeroderma pigmentosum Fig. 21:  Hairy leukoplakia (For color version, see Plate 3)
♦♦ Epidermolysis bullosa. Q.4. Write notes on oral submucus fibrosis.
For etiology, histopathology and clinical features of leukoplakia, (Sep 2003, 5 Marks) (Apr 2007, 10 Marks)
refer to Ans 1 of same chapter. (Dec 2007, 3 Marks)
Ans. OSMF is defined as “An insidious chronic disease
Q.3. Write notes on hairy leukoplakia.
affecting any part of oral cavity and sometime pharynx.
 (Mar 2003, 5 Marks) Although occasionally preceded by and/or associated
Ans. Hairy leukoplakia is HIV associated mucosal disorder, with vesicle formation, it is always associated with
which often involves lateral and ventral surfaces of juxtaepithelial inflammatory reaction followed by
tongue. fibroelastic changes in lamina propria, with epithelial
Homosexual man with HIV infection may develop white atrophy leading to stiffness of oral mucosa and causing
patchy lesion in oral cavity. trismus and inability to eat.” Pindborg (1966)

Clinical Features Etiology


♦♦ Clinically hairy leukoplakia occurs more frequently on the The OSMF is caused due to:
lateral border of the tongue however it can also occur on ♦♦ Excessive consumption of red chillies.
floor of the mouth, buccal mucosa, etc. ♦♦ Excessive “areca nut” chewing.
Section 3: Oral Pathology  515

♦♦ Nutritive deficiency Treatment


♦♦ Immunological factors ♦♦ Stoppage of all causing habits.
♦♦ Genetic factors ♦♦ Definitive treatment of OSMF includes intralesional injection
♦♦ Protracted tobacco use of collagenase, corticosteroids and fibrinolysin, etc.
♦♦ Patient with deficiency of micronutrients. ♦♦ Systemic administration of steroids in severe cases.
♦♦ Use of lime with areca nut
Q.5. Describe in brief histopathology of intraepithelial
Clinical Features
carcinoma. (Mar 1994, 8 Marks)
♦♦ It is caused during 20 to 40 years of age. Ans. This is also called as carcinoma in situ
♦♦ Females are affected more than males. • Intraepithelial carcinoma is a condition which
♦♦ In OSMF fibrotic changes are frequently seen in buccal arises frequently on skin, but occurs also on mucus
mucosa, retromolar area, uvula, tongue, etc. membrane including those of oral cavity.
♦♦ Initially patient complains of burning sensation in the • Metastasis is impossible in intra epithelial carcinoma.
mouth, particularly during taking hot and spicy foods. • Bowen’s disease is a special form of intraepithelial
♦♦ There can be excessive salivation, decreased salivation and carcinoma occurring with some frequency on skin,
defective gustatory sensation. particularly in patients who have had arsenic
♦♦ In initial phase of disease palpation of mucosa elicits a therapy and is often associated with development
“wet leathery” feeling. of internal or extra cutaneous cancer.
♦♦ In advanced stage the oral mucosa loses its resilience and
becomes blanched and stiff and thereby causing trismus. Histological Features
♦♦ Palpation of mucosa often reveals vertical fibrous bands. ♦♦ Keratin may or may not be found on the surface of lesion
Histopathology but, if present, is more apt to be parakeratin rather than
orthokeratin.
Microscopically OSMF reveals following features:
♦♦ Overlying hyper keratinized, atrophic epithelium often
shows flattening and shortening of rete pegs.
♦♦ There can be variable degrees of cellular atypia or epithelial
dysplasia.
♦♦ In OSMF dysplastic changes are found in epithelium which
include nuclear pleomorphism, sever inter-cellular edema, etc.
♦♦ Stromal blood vessels are dilated and congested and there
can be areas of hemorrhage.
♦♦ Underlying connective tissue stroma in advanced stage
of disease shows homogenization and hyalinization of
collagen fibers.
♦♦ Decreased number of fibroblastic cells and narrowing
of blood vessels due to perivascular fibrosis are present.
♦♦ There can be presence of signet cells in some cases.

Fig. 23:  Intraepithelial carcinoma (For color version, see Plate 4)


♦♦ In some instances, there appears to be hyperplasia of the
altered epithelium while in others there is atrophy.
♦♦ An increased nuclear/cytoplasmic ratio and nuclear
hyperchromatism is seen.
♦♦ Cellular pleomorphism is common
♦♦ There is loss of orientation of cells and loss of their normal
polarity.
♦♦ Sometimes a sharp line of division between normal or
altered epithelium extends from surface down to the con­
nective tissue rather then a blending of epithelial changes.

Treatment
Fig. 22:  Oral submucous fibrosis (For color version, see Plate 4) Treatment is done by surgery, radiotherapy or electro­cautery.
516   Mastering the BDS IIIrd Year  (Last 25 Years Solved Questions)

Q.6. Describe etiology, histopathology and clinical features


of carcinoma of tongue.
(Feb 1999, 15 Marks) (Sep 2009, 6 Marks)
(Dec 2009, 15 Marks)
Or
Describe in detail the etiology, clinical features and
histopathology of squamous cell carcinoma of tongue.
 (Dec 2015, 8 Marks)
Ans.

Etiology
♦♦ Syphilis either an active case or past history of it coex­
istent with carcinoma of tongue. Syphilis is accepted as
Fig. 24:  Carcinoma of tongue (For color version, see Plate 4)
having strong association with development of dorsal
tongue carcinoma. Arsenicals and heavy metals used Q.7. Write short note on peripheral giant cell granuloma.
to treat syphilis before advent of modern antibiotics (Sep 2006, 5 Marks)
have carcinogenic properties themselves and are re­ Or
sponsible for some of earlier carcinoma development
Describe histologic features with diagram of peripheral
in this disease.
giant cell granuloma. (Nov 2008, 5 Marks)
♦♦ Leukoplakia is the common lesion of tongue which leads
Ans. Peripheral giant cell granuloma is the most common of
to carcinoma of tongue.
giant cell lesions which arises from tooth bearing areas
♦♦ Carcinoma of tongue is due to poor oral hygiene, chronic
of jaw and appears as a purplish red nodule.
trauma and use of alcohol and tobacco.
♦♦ Carcinoma of tongue is due to source of chronic irritation Clinical Features
such as carious or broken tooth or an ill fitting denture.
♦♦ Lesion usually arises during mixed dentition or during
Histopathology third and fourth decade of life.
♦♦ It is most common in males.
♦♦ Cells are generally large and show distinct cell membrane.
♦♦ Its site in dentulous patient is interdental papilla. Mandible
♦♦ Nuclei of neoplastic cells are large. is more frequently affected than maxilla.
♦♦ Carcinomas of tongue ranges from well differentiated ♦♦ Peripheral giant cell granuloma appears as a small, exophytic,
keratinizing carcinoma to highly differentiated neoplasm. well circumscribed, pedunculated lesion on gingival surface.
♦♦ Changes of epithelial dysplasia are present. ♦♦ Color of lesion varies from purplish red to darkish red.
♦♦ There is increased number of mitotic figure per field. ♦♦ There can be bleeding from the surface either spontane­
♦♦ For histopathologic gradation, refer to Ans 15 and Ans 23 ously or on provocation from instrument.
of same chapter. ♦♦ Sometimes the peripheral cell granuloma can be aggressive
in nature and such lesion may attain very large size and
Clinical Features
involves some teeth.
♦♦ Most common presenting signs of carcinoma of tongue is ♦♦ In some cases the lesion may develop with an ‘hour-glass
painless mass or ulcer. The lesion becomes painful when shape.’
it is secondarily affected.
♦♦ Tumor begins as superficially indurated ulcer with slightly Histopathology
raised borders and may proceed either to develop a fungat­
♦♦ Peripheral giant cell granuloma present following histo­
ing, exophytic mass or to infiltrate deep layers of tongue,
logical features.
producing fixation and induration without much surface ♦♦ Overlying covering epithelium is ulcerated with areas of
change. hemorrhage.
♦♦ If carcinoma is present on the dorsum of tongue, ♦♦ Underlying connective tissue stroma reveals numerous
then patient has past or present history of syphilitic proliferating fibroblasts, blood capillaries and multinucle­
glossitis. ated giant cells, which are scattered throughout the lesion.
♦♦ Paresthesia of tongue frequently occurs due to invasion of ♦♦ Fibroblasts present in hypercellular stroma are spindle
lingual nerve by tumor cells. shaped and have oval shaped nuclei.
♦♦ The common site where the lesion develops is lateral bor­ ♦♦ Giant cells are large in size and contain more number of
der of tongue and ventral surface of tongue. nuclei as compared to true giant cell tumor.
♦♦ Initial lesions often appear as erythematous macules or ♦♦ Areas of hemorrhage and hemosiderin pigment are present
nodules or fissured areas over the tongue. within connective tissue stroma.
Section 3: Oral Pathology  517

♦♦ Endothelial cells are spindle shaped.


♦♦ Capillaries are well formed and are present throughout
the lesion.
♦♦ Fibrous connective tissue stroma is not well formed and
is loosely arranged.

Fig. 25:  Peripheral giant cell granuloma


(For color version, see Plate 4)

Treatment
Surgical excision with curettage. Fig. 26:  Capillary hemangioma (For color version, see Plate 4)

Q.8. Write short note in hemangioma. (Feb 2013, 5 Marks) Histopathology of Cavernous Hemangioma
Ans. Hemangiomas are lesions which are not present at birth but ♦♦ They are histologically characterized by large, irregularly
they manifest within the first month of life, exhibit a rapid shaped, dilated, endothelialized sinuses which contain
proliferative phase and slowly involute to non-existent. large aggregates of erythrocytes.
Classification of Hemangiomas by Watson and McCarthy ♦♦ A single layer of flatted endothelial cell lines each sinus.
♦♦ Capillary hemangioma ♦♦ Sinus lacks muscular coat on their walls.
♦♦ Cavernous hemangioma ♦♦ Large area of hemorrhage and hemosiderin pigments is
♦♦ Angioblastic hemangioma often seen within cavernous hemangioma lesions.
♦♦ Racemose hemangioma
♦♦ Diffuse systemic hemangioma
♦♦ Metastaizing hemangioma
♦♦ Port-Wine stain
♦♦ Hereditary hemorrhagic telangiectasia
Clinical Features
♦♦ Occur most commonly in infants and children.
♦♦ Peak incidence of central hemangiomas is during 2nd
decade of life.
♦♦ More common in females
♦♦ Most commonly affected bones are facial bones, i.e. man­
dible, maxilla and nasal bones.
♦♦ Lesion appears as a flat or raised lesion of mucosa which
is deep red or bluish red and is circumscribed.
♦♦ Lesion is compressible and filled slowly when released.
♦♦ Intra-orally commonly affected sites are lip, tongue, buccal
mucosa and palate. Fig. 27:  Cavernous hemangioma (For color version, see Plate 4)

Histopathology Treatment
♦♦ There are several histopathologic types of heman­gioma found Surgical excision is treatment of choice.
in oral cavity, among them two very common types are:
• Capillary hemangioma Q.9. Enumerate oral precancerous lesions and conditions.
• Cavernous hemangioma. Describe in detail leukoplakia.
(Aug/Sep 1998, 15 Marks)
Capillary Hemangioma
♦♦ They are histologically characterized by numerous, small, Or
endothelial lined capillaries in lesion which are densely Enumerate premalignant lesions and conditions de-
packed by erythrocytes. scribe in detail about leukoplakia.
♦♦ Cells of endothelial lining are single layered. (June 2010, 15 Marks)
518   Mastering the BDS IIIrd Year  (Last 25 Years Solved Questions)

Ans. For enumeration of oral precancerous lesions and ♦♦ All bulbous rete ridges of the epithelium projects into the
conditions, refer to Ans 2 of same chapter and for connective tissue and is known as pushing margin.
leukoplakia in detail, refer to Ans 1 of same chapter. ♦♦ The basement membrane is intact and underlying connec­
tive tissue shows inflammatory cell infiltration.
Q.10. Describe precancerous lesion and condition. Describe
♦♦ Formation of epithelial pearls and microcytes are seen.
in detail about oral submucous fibrosis.
(Sep 2004, 15 Marks)
Ans. For description of precancerous lesion and condition
refer to Ans 2 of same chapter in detail.
For OSMF, refer to Ans 4 of same chapter.
Q.11. Write notes on verrucous carcinoma.
(Mar 1998, 5 Marks) (Dec 2007, 4 Marks)
(Aug 2012, 5 Marks) (Aug 2011, 10 Marks)
 (Feb 2015, 5 Marks)
Or
Discuss about verrucous carcinoma. (Sep 2004, 8 Marks)
Or
Describe in detail clinical features, histopathology and
treatment of verrucous carcinoma. (Mar 2011, 8 Marks) Fig. 28:  Verrucous carcinoma (For color version, see Plate 5)
Or
Treatment
Write short note on verrucous carcinoma.

♦♦ Surgical excision or laser therapy is done.
 (Jan 2016, 5 Marks) ♦♦ Prognosis should be good.
Or
Q.12. Enumerate giant cell lesions and describe peri­pheral
Write short answer on verrucous carcinoma. giant cell granuloma. (Feb 2002, 15 Marks)
 (May 2018, 3 Marks) Ans. For enumeration of giant cell lesions refer to Ans 19 of
Ans. Verrucous carcinoma is a diffused papillary, non- same chapter. For peripheral giant cell granuloma refer
metastasizing well differentiated malignant neoplasm to Ans 7 of same chapter.
of oral epithelium.
Q.13. Discuss about central giant cell granuloma.
It is also known as Ackerman’s tumor.
(Sep 2005, 8 Marks)
Clinical Features Ans. Central giant cell granuloma is relatively a common
benign intraosseous destructive giant cell lesion which
♦♦ Tumor occurs during 60 years of life and males are more
affects anterior part of jaw bone.
commonly affected.
♦♦ Common locations for verrucous carcinoma are gingiva, Clinical Features
alveolar mucosa and buccal mucosa.
♦♦ Verrucous carcinoma presents as slow growing, exophytic, ♦♦ Lesion occurs in young children and female predilection
papillary growth having white pebbly surface. is present.
♦♦ Carcinoma occurs either as single entity or there can be ♦♦ Central giant cell granuloma affects the mandible more
multiple lesions involving different parts of oral cavity. often than maxilla and occurs anterior to first molar.
♦♦ Lesions on buccal mucosa are sometimes very extensive ♦♦ Central giant cell granuloma is a small, slow enlarging
and often cause pain, tenderness and difficulty in taking bony hard swelling of jaw which is painful on palpitation.
the food. ♦♦ Lesion causes expansion and distortion of cortical plates
♦♦ Regional lymph nodes are often enlarged and tender. and there is presence of displacement or mobility of
regional teeth.
Histopathology ♦♦ Central giant cell granuloma follow an aggressive course
♦♦ Hyperplastic epithelium often exhibits a papillary surface and in such cases they produce fast enlarging, large,
painful swelling in the jaw.
being covered by the thick layer of parakeratin.
♦♦ Massively enlarged bulb like acanthotic rete ridges are seen
which invaginated into underlying connective tissue stroma. Histopathology
♦♦ Many deep cleft like spaces lined by thick layer of par­ ♦♦ Central giant cell granuloma exhibits fibro vascular
akeratin, these extend from the surface of epithelium and connective tissue stroma, consisting of numerous stromal
project deep into the center of bulbous rete ridges. This is cell which are plum and spindle shaped and undergo
known as parakeratin plugging. frequent mitosis.
Section 3: Oral Pathology  519

♦♦ Several areas of hemorrhage and hemosiderin


pigmentations are also evident.
♦♦ Multiple multinucleated giant cells of varying size are
dispersed throughout fibrous tissue stroma. Giant cells
are found around the blood capillaries or near the area of
hemorrhage and giant cells consist of 5 to 20 nuclei.
♦♦ Small foci of osteoids are often found near periphery of
lesion.
♦♦ Little amount of chronic inflammatory cell infiltration in
connective tissue stroma.

Fig. 30:  Moderately differentiated squamous cell carcinoma


(For color version, see Plate 5)
Q.16. Write notes on giant cell. (Mar 2000, 5 Marks)
 (Aug 2012, 5 Marks)
Ans. Giant cells are multinucleated cells that commonly occur
at sites of chronic inflammation mostly complicated by
granuloma formation. Besides giant cells occurring in
various lesions, there also exists physiological giant
cells by virtue of the presence of multinucleated nature,
Fig. 29:  Central giant cell granuloma (For color version, see Plate 5)
present as a part of healthy tissues.
Multinucleated giant cells are highly stimulated cells
Radiological Features of the monocyte-macrophage lineage at a terminal
stage of maturation and studies have demonstrated
♦♦ Radiographically the lesion produces multicellular that their multinucleated appearance is produced by
radiolucent area in jaw with soap bubble appearance. cell–cell fusion rather than by nuclear division without
♦♦ Margin of lesion is scalloped and well demarcated. cytokinesis.
♦♦ Resorption of roots of nearby teeth or divergence of roots
is common feature. Classification of Giant Cells

Treatment Giant cell can be classified based according to their occurrence


in the body:
Surgical excision is done with curettage. ♦♦ Physiological giant cells:
Q.14. Define epithelial dysplasia. Describe verrucous • Osteoclast
carcinoma. (Mar 2007, 7.5 Marks) • Striated muscle cells
Ans. Epithelial dysplasia comprises a loss in the uniformity • Megakaryocytes
of individual cells or cytological atypia as well as a loss • Syncytiotrophoblast
in their architectural orientation. ♦♦ Pathological giant cells:
For verrucous carcinoma refer to Ans 11 of same • Langhan’s giant cells
chapter. • Foreign body giant cells
• Touton giant cells
Q.15. Describe the histopathology of moderately differen­
• Tumor giant cells
tiated squamous cell carcinoma. (Sep 2006, 5 Marks)
• Warthin–finkeldey giant cells
Ans. Histopathology
• In it the tumor cells are usually more severely dys­ Osteoclast
plastic than of well differentiated type.
• Malignant epithelial cells produce little or no Osteoclasts are highly specialized for resorption of bone mineral
keratin and they exhibit greater number of mitotic and matrix through the coordinated secretion of hydrogen
division. ions and proteolytic enzymes. Although most osteoclasts are
• Formation of epithelial islands is diminished since large multinucleated cells, there are reports of mononuclear
tumor cells do not mature properly. osteoclasts. Situated on the bone surface, it occupies a concavity
• Malignant tumor cells are recognized as stratified surface of small bone spicules. An enlarged surface area
squamous epithelial cells. created by plasma membrane infoldings, the ruffled border,
520   Mastering the BDS IIIrd Year  (Last 25 Years Solved Questions)

characterizes the secretory or apical surface directed toward protoplasmic mass without intercellular boundaries. From this
the bone. In routine histologic sections, the ruffled border mass emerge finger like projections, which penetrate through
appears striated and lightly stained. The presence of a ruffled the endometrial epithelium into the endometrial stroma.
border is an indication that the osteoclast is actively engaged in
bone resorption. Each ruffled border is surrounded by a clear Langhans Giant Cell
zone (or sealing zone), a cytoplasmic area rich in cytoplasmic
actin filaments and devoid of major cytoplasmic organelles. They are characterized by location of the nuclei at the
Through close adaptation of the cell surface to the bone matrix, periphery of the cell in an acute configuration. They are
the clear zone establishes a seal between the bone resorption seen in lesions such as tuberculosis, sarcoidosis, leprosy and
compartment and the interstitial fluid. vasculitis. These are special, more highly organized forms
than are ‘foreign body’ multinucleated giant cells. These
Striated Muscle giant cells may attain diameters of 40 to 50 µm. They have a
They are the muscles in which the cells exhibit cross striations large mass of cytoplasm containing 20 or more small nuclei
at the light microscopic level. arranged peripherally.
It is further sub-classified based on its location:
Foreign Body Giant Cell
♦♦ Skeletal muscle
♦♦ Visceral striated muscle Formation of foreign body giant cell is hallmark of the foreign
♦♦ Cardiac muscle body reaction and is harmful to implanted biomaterials because
Skeletal Muscle Cell it contributes to the degradation of the bio-material and leads to
stress cracking, tissue fibrosis and a chronic response. Foreign
A skeletal muscle cell is a multinucleated syncytium. In skeletal body giant cells are thought to be a source of chemokines that
muscle, each muscle fiber is a multinucleated syncytium. A mediate the neutrophils and lymphocytes.
muscle fiber is formed during development by the fusion of
small, individual muscle cells called myoblasts. When viewed Touton Giant Cell
in cross section, the mature multinucleated muscle fibers reveal
a polygonal shape with a diameter of 10 to 100 µm. Touton giant cells are characterized by a ring of nuclei
surrounding central eosinophilic zone and surrounded by a
Visceral Striated Muscle zone of pallor extending to the periphery of cell. These giant cells
It is morphologically identical to skeletal muscle but is restricted are seen in lesions with high lipid content such as xanthoma,
to the soft tissues, namely the tongue, pharynx, lumbar part of xanthogranuloma, fat necrosis. The characteristic appearance
diaphragm and upper part of esophagus. These play an essential of ‘Xanthelasmatic giant cell’ of Touton is determined merely
role in speech, breathing and swallowing. by the presence of demonstrable lipid in the cytoplasm.
Cardiac Muscle
Tumor Giant Cell
Cardiac muscle has the same type and arrangement of the
contractile filament as skeletal muscle. In addition, cardiac A feature of anaplasia is the formation of tumor giant cells,
muscle fibers exhibit densely staining cross bands called some possessing only a single huge polymorphic nucleus
intercalated disks, that cross the fibers in a linear fashion or and others having two or more nuclei. These giant cells are
frequently in a way that resembles the rosters of a stairway. not to be confused with inflammatory LanghAns or foreign
The intercalated disks represent a highly specialized body giant cells, which are derived from macrophages
attachment sites between adjacent cells. The linear cell—cell and contain many small normal-appearing nuclei. In the
attachment of the cardiac muscle cells results in ‘fibers’ of cancer giant cell, the nuclei are hyperchromatic and large
variable length. in relation to the cell.

Megakaryocytes
Warthin-Finkeldey Giant Cell
Platelets are derived from large polypoid cells (cells whose
nuclei contain multiple sets of chromosomes) in bone marrow The Warthin Finkeldey cells have up to 100 nuclei and contain
are called megakaryocytes. spherical eosinophileic intra-cytoplasmic and intranuclear
inclusions. They are present in viral infections like measles.
Syncytiotrophoblast Tompkins and Macaulay reported Warthin-Finkeldey giant cells
The syncytiotrophoblast is a continuous, normally unin­ in nasal secretions before the appearance of other clinical signs
terrupted layer that extends over the surfaces of all villous of measles such as Koplik’spots and skin rash. These cells are
trees as well as over pAns of the inner surfaces chorionic and found throughout the reticuloendothelial system and contain
basal plates. The syncytiotrophoblast is a multinucleated up to 100 nuclei.
Section 3: Oral Pathology  521

♦♦ It is a special stain which is used for connective tissue ele­


ments. It is used for differentiating between connective
tissue fibers and muscle fibers.
♦♦ Epithelium (cell and cytoplasm): It takes greenish yellow
stain.
♦♦ Collagen fibers are red
♦♦ Muscle fibers are yellow
♦♦ Nuclei of cells are blue black.

Mallory Stain
♦♦ It is a special stain for keratin that stains deep orange. It is
used in hyperkeratotic lesions.
♦♦ The epithelium is royal blue.
♦♦ Collagen fibers are royal blue.
A ♦♦ Muscle fibers are royal blue.
♦♦ Keratin layers are orange.
♦♦ Nucleus is blue black.

B
Figs 31A and B:  Giant cell (For color version, see Plate 5)
Q.17. Write notes on special stains. (Mar 2000, 5 Marks) Fig. 33:  Mallory stain (For color version, see Plate 6)
Ans. The special stains are:
Periodic Acid Schiff’s Stain or PAS Stain
van Gieson’s Stain

Fig. 32:  van Gieson’s stain (For color version, see Plate 5)
Fig. 34:  PAS stain (For color version, see Plate 6)
522   Mastering the BDS IIIrd Year  (Last 25 Years Solved Questions)

♦♦ It is a special stain for mucopolysaccharide granules. These Q.19. Enumerate giant cell lesions. Describe in detail central
are prominently seen in basement membrane, inter­cellular giant cell granuloma. (Sep 2007, 7.5 Marks)
spaces and keratin layer. It is used to detect continuity of Ans. Enumeration of giant cell lesions
basement membrane in intra- epithelial carcinoma and Giant cell lesions of oral cavity
squamous cell carcinoma.
♦♦ Epithelium and connective tissue is pink. According to the Nature of Different Pathologic
♦♦ Collagen fibers are pink. Conditions
♦♦ Muscle fibers are pink.
♦♦ Infections
♦♦ Nucleus is blue black.
• Bacterial
♦♦ Granules are of magenta color.
• Viral
Masson’s Trichrome Stain • Fungal
• Protozoal
It is a special stain used to differentiate between collagen fibers • Parasitic
and muscle fibers. It demonstrates connective tissue disorders ♦♦ Fibro-osseous lesions and osteodystrophies
like leiomyosarcoma and rhabdomyosarcoma. • Immunologic
♦♦ Epithelium is red. • Idiopathic
♦♦ Muscle fibers are bluish violet. • Orofacial granulomatosis
♦♦ Collagen fibers and blood vessels are blue. • Reaction to materials
• Benign and malignant tumors

In this Classification Giant Cells are Categorized into


Three Categories
1. Giant cells are the main cause for the pathology
2. Giant cells characterize these lesions
3. Lesions that may be associated with giant cells

Main Cause for the Pathology


♦♦ Giant cell granuloma
• Peripheral
• Central
♦♦ Giant cell tumors
♦♦ Giant cell fibroma
♦♦ Hyperparathyroidism

Fig. 35:  Masson’s trichrome stain


Giant Cells That Characterize Lesions
(For color version, see Plate 6) ♦♦ Infections
• Tuberculosis
Q.18. Define premalignant lesion and condition. Describe • Hansen’s disease
etiology, clinical features and histo­pathology of oral • Syphilis
submucous fibrosis. (Sep 2007, 7.5 Marks) • Measles
Or ♦♦ Granulomatous lesions
• Wegener’s granulomatosis
Define precancerous condition and precancerous le- • Orofacial granulomatosis
sion. Discuss about etiology and histopathology of • Pulse granuloma
OSMF. (Mar 2016, 8 Marks) • Sarcoidosis
Ans. The premalignant lesions are defined as morphologically ♦♦ Lesions in bone
altered tissue in which cancer is more likely to occur then • Aneurysmal bone cyst
in its apparently normal counter part, e.g. leukoplakia, • Cherubism
erythroplakia, nicotiana palati, stomatitis, dyskeratosis • Paget’s disease
congenita, etc. ♦♦ Foreign body lesions
• The premalignant condition is defined as generalized • Silicosis
state of body, which is associated with significantly • Berylliosis
increased risk of cancer, e.g. oral sub- mucus fibrosis, ♦♦ Malignancies
syphilis, lichen planus, etc. • Lymphoma- hodgkin’s disease
• For etiology, clinical features and histopathology of • Bronchogenic carcinoma
OSMF refer to Ans 4 of same chapter. • Carcinoma of thyroid
Section 3: Oral Pathology  523

♦♦ Miscellaneous Cartilage Tissue


• Xanthoma
Benign tumors
• Giant cell arteritis
♦♦ Chondroma
Lesions that may be Associated with Giant Cell ♦♦ Chondroblastoma
♦♦ Chondromyxoid fibroma.
♦♦ Malignancies
Malignant tumors
• Multiple myeloma
• Ewing’s sarcoma ♦♦ Chondrosarcoma
• Fibrosarcoma Adipose Tissue
• Chondrosarcoma
♦♦ Fibro-osseous lesions Benign tumors
• Osteoblastoma ♦♦ Lipoma
• Fibrous dysplasia ♦♦ Angiolipoma.
• Cemento-ossifying fibroma Malignant tumors
• Radicular cyst ♦♦ Liposarcoma

Based on Their Origin Bone


♦♦ Epithelially derived, i.e. Warthin-finkeldey giant cells, Benign tumors
tumor giant cells ♦♦ Osteoma
♦♦ Stromally derived, i.e. Reed-sternberg giant cells. ♦♦ Osteoid osteoma
For central giant cell granuloma, refer to Ans 13 of same chapter. ♦♦ Osteoblastoma
Q.20. Enumerate and classify non odontogenic tumors of Malignant tumors
oral cavity. Write briefly about malignant mela­noma. ♦♦ Osteosarcoma
(Sep 2011, 8 Marks) ♦♦ Osteochondrosarcoma
Ans. Vascular Tissue
Non-odontogenic Tumors of Oral Cavity Benign tumors
♦♦ Hemangioma
Epithelial Tissue ♦♦ Hereditary hemorrhagic telangiectasia
Benign tumors ♦♦ Lymphangioma
♦♦ Papilloma Malignant tumors
♦♦ Keratoacanthoma ♦♦ Hemangioendothelioma
♦♦ Squamous acanthoma Neural Tissue
♦♦ Nevus
Malignant tumors Benign tumors
♦♦ Squamous cell carcinoma ♦♦ Neurofibroma
♦♦ Mucoepidermoid carcinoma ♦♦ Neurilemmoma
♦♦ Adenocarcinoma ♦♦ Schwannoma
♦♦ Basal cell carcinoma Malignant tumors
♦♦ Transitional cell carcinoma ♦♦ Neurosarcoma
♦♦ Melanoma ♦♦ Neurofibrosarcoma
♦♦ Verrucous carcinoma
♦♦ Intraepidermoid carcinoma Muscles

Fibrous Connective Tissue Benign tumors


♦♦ Leiomyoma
Benign tumors ♦♦ Rhabdomyoma
♦♦ Fibroma Malignant tumors
♦♦ Fibrous hyperplasia ♦♦ Leiomyosarcoma
♦♦ Fibrous epulis ♦♦ Rhabdomyosarcoma
♦♦ Giant cell fibroma
♦♦ Myxoma Giant Cell Tumor
♦♦ Myxofibroma.
♦♦ Central and peripheral giant cell tumor
Malignant tumors ♦♦ Giant cell granuloma
♦♦ Fibrosarcoma ♦♦ Giant cell tumor of hyperthyroidism.
524   Mastering the BDS IIIrd Year  (Last 25 Years Solved Questions)

Teratoma ♦♦ Lentigo maligna melanoma: Exists in a radial-growth


phase. The lesion occur as macular lesion on malar skin
Salivary Gland Tumor of Caucasians.
♦♦ Acral lentiginous melanoma: Melanoma developing on
Benign tumors
the palms and soles as well as toe and fingers. It is char­
♦♦ Adenoma
acterized by macular lentiginous pigmented area around
♦♦ Warthin’s tumor
nodule.
♦♦ Pleomorphic adenoma.
♦♦ Mucosal lentiginous melanoma: Develops from mucosal
Malignant tumors epithelium that lines respiratory, gastrointestinal and
♦♦ Mucoepidermoid carcinoma genitourinary systems. It is more aggressive.
♦♦ Adenocystic carcinoma ♦♦ Amelanotic melanoma: It is an erythematous or pink
♦♦ Adenocarcinoma sometimes eroded nodule.
♦♦ Acinic cell carcinoma
♦♦ Malignant change in pleomorphic adenoma Clinical Features

Lymphoid Tissue ♦♦ Oral melanomas initiate as macular pigmented focal


lesions.
Malignant tumors ♦♦ Most of the lesions are pigmented excepting few non-pig­
♦♦ Hodgkin’s and non-hodgkin’s lymphoma mented lesions which referred to as “amelanotic melano­
♦♦ Lymphosarcoma mas”, which appear as “slightly” inflamed looking areas.
♦♦ Reticular cell sarcoma ♦♦ Pigmented lesions are often dark-brown, bluish-black or
♦♦ Ewing’s sarcoma simply black in appearance.
♦♦ Burkitt’s lymphoma ♦♦ Initial macular lesions grow very rapidly and often result
♦♦ Multiple myeloma in a large, painful, diffuse mass.
♦♦ Leukemia. ♦♦ Surface ulceration is very common and beside; this, hem­
orrhage, paresthesia and superficial fungal infections are
Malignant Melanoma
often present.
Malignant melanoma is a neoplasm of epidermal melanocytes. ♦♦ As the tumor continues to grow, small satellite lesions can
It is the third most common cancer of the skin. develop at the margin of the primary tumor.
♦♦ Like other epithelial malignant tumors, melanomas exhibit
Etiology little or no induration at the periphery.
♦♦ Sun exposure: Persons who are exposed to the excess of ♦♦ Oral melanomas often cause rapid invasion and extensive
sunlight develops malignant melanoma. destruction of bone. This often results in loosening and
♦♦ Artifical UV source: PUVA thearpy has been reportedly exfoliation of the regional teeth in the jaw.
associated with risk of melanoma. ♦♦ Widespread dissemination of the tumor cells occurs fre­
♦♦ Socioeconomic status: It is seen in high socioeconomic status quently in the lymph nodes as well as in the distant sites,
since people of high socioeconomic status go for holidays. e.g. the lung, liver, bone and brain, etc.
♦♦ Fare skin, freckles, red hair: These characteristics increased
Histopathology
the risk of melanoma.
♦♦ Melanotic nevi are the strong risk factors. ♦♦ Microscopically the malignant cells lie in nest or cluster of
♦♦ Genetic factors: Familial melanoma and xeroderma pig­ groups in an organoid fashion.
mentosum are considered to be strong genetic factors for ♦♦ Melanoma cells have large nuclei, often with prominent
development of malignant melanoma. nucleoli and show nuclear pseudoinclusion.
♦♦ Cytoplasm of the cell is abundantly eosinophilic or opti­
Clinical Types of Malignant Melanoma cally clear.
♦♦ There is presence of large, epithelioid melanocytes distrib­
♦♦ Superficial spreading melanoma: Exists in a radial growth uted in pagetoid manner.
phase. Lesion present as tan, brown or black admixed le­ ♦♦ When melanocytes penetrate the basement membrane a
sion on sun exposed skin. Radial growth phase may last florid host cell response of lymphocytes develop.
for several months to years. ♦♦ Macrophages or melanophages may be present.
♦♦ Nodular melanoma: It exists in a vertical growth phase. It ♦♦ Vertical growth phase is characterized by the proliferation
present sharply delineated nodule with varying degrees of malignant epithelioid melanocytes in the underlying
of pigmentation. They may be pink or black. connective tissues.
Section 3: Oral Pathology  525

♦♦ These malignant melanocytes often exhibit extensive cel­ Treatment


lular pleomorphism and nuclear hyperchromatism.
Surgical excision of tumor should be done.
♦♦ However, in some lesions melanin production by the tu­
mor cells can be very little and on few occasions there can Q.22. Write in detail about Hodgkin’s lymphoma.
be virtually no melanin production. (Aug 2011, 10 Marks)
Ans. It is also known as hodgkin’s disease.
• Epstein-Barr virus is considered to be the major cause.
• Patients with HIV infection have higher incidence
of Hodgkin’s disease.
Clinical Features
♦♦ It is most commonly seen in young adults and older in­
dividuals.
♦♦ More common in males as compared to females.
♦♦ Clinical signs and symptoms of the disease are protean
♦♦ There is painless enlargement of one or more cervical
lymph nodes.
♦♦ Palpable painless cervical lymphadenopathy occurs in
cervical area, axilla and less commonly in inguinal area
Fig. 36:  Malignant melanoma (For color version, see Plate 6) and Waldeyers ring and occipital nodes.
♦♦ Lymph nodes are firm and rubbery in consistency and
Treatment overlying skin is normal.
♦♦ Symptoms are of unexplained weight loss, fever and night
Radical surgery with prophylactic neck dissection is often sweats.
advised.
Histopathology
Q.21. Describe in brief teratoma.
♦♦ Nodular sclerosis hodgkin’s disease
 (Aug 2011, 5 Marks) • Morphology show nodular pattern.
Ans. A true teratoma is a developmental tumor composed of • Broad bands of fibers divide node into nodules
tissues from all three germ layers, i.e. ectoderm, meso­ • Characteristic cell is lacunar type Reed-Sternberg Cell
derm and endoderm: which has monolobated, multilobated nucleus and a
• Such tumors are believed to derive from germ cells or small nucleolus with abundant and pale cytoplasm.
entrapped totipotent blastomeres which can produce ♦♦ Mixed cellularity hodgkin’s disease
derivatives of all three germ layers. • Infiltrate is usually diffuse
• Teratomas are more common in ovaries and or testis • Reed-Sternberg cells are of classic type i.e. large with
and can be benign or malignant. bilobate, double or multiple nuclei and a large eosino­
philic inclusion like nucleolus.
Clinical Features ♦♦ Lymphocyte depleted hodgkin’s disease
• Infiltrate is diffuse and often appears hypocellular.
♦♦ It is most commonly seen in infants.
• Large number of Reed-Sternberg cells and bizarre
♦♦ Orally it involves soft palate and hard palate. Systemically sarcomatous variants are present.
it involves testis, ovary, abdominal viscera and pineal • It is associated with older age and HIV positivity
region. ♦♦ Lymphocyte rich classic hodgkin’s disease
♦♦ It consist of teeth, sebaceous material and some hairs. • Reed-Sternberg cells of the classic or lacunar
type are observed with background infiltrate of
Histopathology lymphocytes.
♦♦ Nodular lymphocyte-predominant hodgkin’s disease
♦♦ H and E stained section show stratified squamous epithe­ • In this typical Reed-Sternberg Cell is not seen, instead
lium with epithelial appendages consisting of sebaceous a variant of Reed-Sternberg Cell the lymphocytic and
glands, sweat glands, hairs, salivary glands, teeth. histiocytic cells or popcorn cells are seen within the
♦♦ At times thyroid gland tissue and pancreatic tissue can background of inflammatory cells which are predo­
be seen. minantly benign lymphocytes.
526   Mastering the BDS IIIrd Year  (Last 25 Years Solved Questions)

Fig. 37:  Hodgkin’s lymphoma (For color version, see Plate 6)

Treatment ♦♦ Alcohol: Drinking spirits, Drinking wines, drinking beers


Combination of radiotherapy and chemotherapy help in curing ♦♦ Diet anal nutrition: Vitamin A, B-complex and C deficiency,
the disease. Nutritional deficiency with alcoholism.
♦♦ Dental factors: Chronic irritation from broken teeth, Ill-
Q.23. Write in detail on squamous cell carcinoma. fitting or broken prosthesis.
(May/June 2009, 10 Marks) ♦♦ Radiations: Actinic radiation, X-ray radiation
Enumerate all malignant epithelial neoplasms. De- ♦♦ Viral infections: Herpes simplex virus (HSV), Human papil­
scribe in detail histopathology of oral squamous cell loma virus (HPV), Human immunodeficiency virus (HIV),
Epstein-Barr virus (EBV).
carcinoma. (Jan 2012, 10 Marks)
♦♦ Chronic infections: Candidiasis, syphilis
Or
♦♦ Genetic factors: Oncogenes, Tumor suppressor genes
Enumerate malignant tumors of epithelial origin. De- ♦♦ Pre-existing oral diseases: Lichen planus, Plummer-Vinson
scribe in detail about squamous cell carcinoma. Syndrome, discoid lupus erythematosus, OSMF.
(Feb 2013, 10 Marks)
Ans. Enumeration of malignant epithelial neoplasms/Tumors: Clinical Features
• Basal cell carcinoma
♦♦ Carcinomas mostly occur in the 4th to 7th decades of life.
• Squamous cell carcinoma
♦♦ Males are more commonly affected
• Verrucous carcinoma
♦♦ Lower lip is the most common site, the second most com­
• Adenoid squamous cell carcinoma
mon site is the lateral border of the tongue. Among all
• Malignant melanoma
intraoral sites, dorsum of the tongue and hard palate are
• Spindle cell carcinoma the least common sites for oral squamous cell carcinoma.
• Primary intra-alveolar carcinoma ♦♦ Initial lesion may be asymptomatic or can be presented
• Multicentric oral carcinoma. as white or red nodule or fissure over the oral mucosa.
Squamous Cell Carcinoma ♦♦ Initially the lesion is usually painless.
♦♦ More advanced lesions present either as a fast enlarging,
♦♦ Squamous cell carcinoma is defined as a malignant exophytic or invasive ulcer or sometimes as a large tumor
epithelial neoplasm exhibit squamous differentiation as mass or a verrucous growth.
characterized by formation of keratin and/or presence of ♦♦ Ulcerated lesion often shows persistent induration around
intercellular bridges. (Pindborg et al, 1997) the periphery with an elevated and everted margin.
♦♦ Squamous cell carcinoma is the commonest malignant epi­ ♦♦ Lesion can be painful either due to secondary infection or
thelial tissue neoplasm of oral cavity. It is mostly derived due to involvement of the peripheral nerves by the tumor
from stratified squamous epithelium. cells. Lesion can also bleed easily.
Etiology ♦♦ Lesions of floor of mouth often cause fixation of the tongue
to the underlying structures with difficulty in speech and
Following are the etiological factors which lead to squamous inability to open the mouth.
cell carcinoma: ♦♦ When malignant tumor cells invade into the alveolar bone
♦♦ Tobacco smoking: Cigarettes, bidis, pipes, cigars and reverse of either maxilla or mandible, they usually cause mobility
smoking. or exfoliation of regional teeth.
♦♦ Use of smokeless tobacco: Snuff dipping, gutkha, tobacco ♦♦ Involvement of inferior alveolar nerve often causes pares­
chewing, tobacco as a toothpaste. thesia of the lower teeth and the lower lip.
Section 3: Oral Pathology  527

♦♦ Regional lymph nodes are often enlarged, tendered and ♦♦ There is formation of epithelial islands or cell nests, etc. are
fixed; some of these nodes can be stony hard in consistency. diminished since these tumor cells do not differentiate or
♦♦ Untreated lesions may sometimes destroy the oral tissues mature as much as the well differentiated type of cells do.
and extend into the skin on the outer surface of the face to ♦♦ This tumor also carries a reasonably good prognosis.
produce a nodular or lobulated growth on the facial skin, Also refer to Ans 15 of same chapter.
which appears as an extraoral discharging sinus.
♦♦ Pathological fracture of the jaw bone may sometimes oc­ Poorly-differentiated Squamous Cell Carcinoma
cur in untreated cases due to extensive destruction of the ♦♦ In poorly-differentiated squamous cell carcinoma, the
bone by the tumor. malignant tumor cells produce no keratin .
♦♦ The tumor exhibits extensive cellular abnormalities with
Histological Features lack of normal architectural pattern and loss of intercellular
Histological finding as given by Broder’s grading for squamous bridges between the tumor cells.
cell carcinoma. ♦♦ Mitotic cell division is extremely high and because of this,
the neoplastic cells are often very immature and primitive
looking and it is often very difficult even to recognize them
as squamous epithelial cells.
♦♦ Prognosis is poor.

Fig. 38:  Well-differentiated squamous cell carcinoma


(For color version, see Plate 7)

Well-differentiated Squamous Cell Carcinoma


Fig. 39: Poorly-differentiated squamous cell carcinoma
(For color version, see Plate 7)
♦♦ Most of the squamous cell carcinomas histologically belong
to the well-differentiated category. Investigations
♦♦ In this lesion, the tumor epithelial cells to a large extent
resemble the cells of the squamous epithelium both struc­ ♦♦ Biopsy of the involved area should be done for con­
turally and functionally. firmation of diagnosis.
♦♦ Tumor cells produce large amount of keratin in the form ♦♦ Toluidine blue test should be done.
of “keratin pearls”. ♦♦ Tumor markers like cytokeratins, epithlial membrane
♦♦ Tumor cells invade into the underlying connective tissue, antigen, carcinoembryonic antigen (CEA) and alpha-
where the cells proliferate further and give rise to the for­ fetoprotein should be used for detection of cancer.
mation of many epithelial islands within the connective Treatment
tissue stroma.
♦♦ Tumor cells often exhibit dysplastic features like cellular Surgical excision of the involved site is the treatment of choice.
pleomorphism, nuclear hyperchromatism, individual cell Q.24. Write short note on leukoplakia. (Feb 2013, 5 Marks)
keratinization and altered nuclear-cytoplasmic ratio, loss Ans. Refer to Ans 1 of same chapter.
of cohesion, etc.
♦♦ Prognosis is better. Q.25. Define neoplasm. Describe in brief clinical features,
X-ray details and histopathology of osteogenic
Moderately-differentiated Squamous Cell Carcinoma
sarcoma. (Mar 2001, 15 Marks) (Sep 2008, 8 Marks)
♦♦ Tumor cells are usually more severely dysplastic than that Ans. "A neoplasm is an abnormal mass of tissue, the growth of
of the well-differentiated type. which exceeds and is uncoordinated with that of the normal
♦♦ Growth rate of individual cells is more rapid and this is tissues and persists in the same excessive manner after
reflected in greater number of mitotic figures. cessation of the stimuli which evoked the change." Willis
528   Mastering the BDS IIIrd Year  (Last 25 Years Solved Questions)

Osteogenic Sarcoma
It is a common malignant neoplasm arising from the bone and
beside plasma and myeloma.

Clinical Features
♦♦ It occurs during 10–25 years of age in the jaw and males
are more commonly affected.
♦♦ The tumor involves maxilla more often than mandible.
♦♦ A very fast enlarging, painful swelling of jaw, causes
expansion and distortion of cortical plates.
♦♦ Severe facial deformity and difficulty in taking the food
due to restricted jaw movement.
♦♦ Displacement and loosening of regional teeth.
Fig. 40: Osteosarcoma (For color version, see Plate 7)
♦♦ Ulceration, hemorrhage and pathological fracture of bone
are commonly associated. Q.26. Write short note on osteogenic carcinoma.
(Mar 1997, 5 Marks)
X-ray Details
Or
The X-ray details of osteosarcoma are divided into following Write short essay on osteosarcoma. ( Jan 2012, 5 Marks)
stages, i.e. Or
♦♦ Osteolytic Stage
Write short note on osteosarcoma. (Aug 2012, 5 Marks)
• It reveals moth eaten appearance.
• Border of the lesion at this stage are ill-defined.  (Mar 2016, 3 Marks)
• There is perforation and expansion of cortical plates. Ans. Refer to Ans 25 of same chapter.
• Lamina dura is absent, i.e. it get destroyed. Q.27. Define oral submucous fibrosis. Describe in detail
• Pathological fracture may be present. etiopathogenesis and histopathology of same.
• Root resorption is present. (Apr 2008, 15 Marks)
♦♦ Mixed Stage Ans. OSMF is defined as “An insidious chronic disease
• It is called as mixed because this stage show formation affecting any part of oral cavity and sometime pharynx.
and destruction of bone. Although occasionally preceded by and/or associated
• It reveals honeycomb appearance. with vesicle formation, it is always associated with
• Margins of lesion are ill defined. juxta-epithelial inflammatory reaction followed by
♦♦ Osteoblastic Stage fibroelastic changes in lamina propria, with epithelial
• It reveals sun ray appearance. atrophy leading to stiffness of oral mucosa and causing
• At times subperiosteal bone laid down in layers which trismus and inability to eat.” —Pindborg (1966)
results in onion peel appearance.
• In osteosarcoma periosteum is elevated over the Etiopathogenesis
expanding tumor mass in a tent like fashion. At point
on the bone where the periosteum begin to merge an ♦♦ Betel nut: Tannic acid and arecoline present in betel nut
acute angle between periosteum and bone is created affect the vascular supply of oral mucosa leading to a
which is known as Codman’s Triangle. neurotropic disorder. Nitrosation of arecoline causes
formation of nitrosoguvacine and nitrosoguvacoline as
well as 3 methyl nitrosamino propionitrile which leads
Histopathology
to alkylation of DNA. Metabolism of nitrosoguvacine
♦♦ There will be presence of numerous, actively proliferating, and nitrosoguvacoline as well as 3 methyl nitrosamino
spindle shaped, oval or angular, malignant osteoblast cells propionitrile causes formation of cyanoethyl. Cyanoethyl
within cellular stroma. interacts with O’ methyl guanine in DNA. Constant ir­
♦♦ The malignant osteoblast cells exhibit cellular ritation by this leads to OSMF and exposure for long time
pleomorphism, abnormal increased mitosis and nuclear leads to malignant transformation.
hyperchromatism Recent concept: Arecoline in betel nut stimulates
♦♦ Multiple areas of newly formed bone or osteoid tissue are fibroblasts. Fibroblasts proliferates and produce
often present within fibrous stroma. collagen. Flavonoids, i.e. catechin and tannin in betel
♦♦ In chondroblastic variations the malignant tumor cells nut stabilizes collagen fibers which make them resistant
produce large amount of cartilagenous tissue within to degradation by collagenase enzyme. Trismus is
tumor. caused due to juxta-epithelial hyalinization as well
Section 3: Oral Pathology  529

as involvement of the secondary muscles. Increased ♦♦ Female predilection is seen.


muscle activity leads to glycogen depletion and reduced ♦♦ It can occur anywhere in the oral cavity but most
blood supply due to connective tissue changes leads to commonly it is seen on buccal mucosa along plane of
degeneration of muscle and also cause fibrosis. occlusion. It also affects gingiva, tongue, buccal mucosa,
♦♦ Tobacco and lime: Tobacco act as a local irritant which lips and palate.
leads to OSMF. Lime causes local irritation as well as ♦♦ Lesion appears as elevated nodule of normal color with
vesicle and ulcer formation in the oral mucosa. smooth surface and a sessile or at times pedunculated
♦♦ Chillies: Capsaicin is an active ingredient present in red base.
chillies. It is the irritant in chillies which leads to oral ♦♦ Its size can ranges from several millimeters to centimeters.
submucous fibrosis. ♦♦ Lesion if traumatized it become painful.
♦♦ Hereditary: Presence of genetic susceptibility is present. ♦♦ Color of the lesion is pink and texture is smooth.
Familial occurrence of OSMF is seen. ♦♦ Consistency of lesion can be soft or firm or can be elastic.
♦♦ Immunological Studies: It was found that human leukocyte ♦♦ At times lesion is traumatized and become inflamed and
antigen (HLA) A10, B7 and DR3 occur significantly more show ulceration or hyperkeratosis.
frequently in oral submucous fibrosis.
For histopathology, refer to Ans 4 of same chapter. Histopathology

Q.28. Enumerate premalignant lesions and conditions. ♦♦ Lesion consists of stratified squamous epithelium which
show shortening and flattening of rete pegs.
Describe in detail about etiopathogenesis, clinical fea-
♦♦ Underlying connective tissue stroma show bundles of
tures, histopathology and treatment of oral submucous
interlacing collagen fibers which are interspersed with
fibrosis. (Jan 2012, 15 Marks) numerous fibroblasts.
Or ♦♦ There is presence of chronic inflammatory cell infiltrate
consisting of lymphocytes and plasma cells.
Describe in detail about etiopathogenesis, clinical fea-
♦♦ Areas of calcification and ossification can also be seen.
tures, histopathology and treatment of oral submucous
fibrosis. (Aug 2012, 10 Marks)
Or
Enumerate premalignant lesions and conditions in
oral cavity. Describe in detail about oral submucous
fibrosis (OSMF).
 (Feb 2015 10 Marks) (Jun 2014, 10 Marks)
Or
List premalignant lesions and conditions. Discuss
etiopathogenesis, clinical features and histopathology
of oral submucous fibrosis. (May 2018, 15 Marks)
Ans. For enumeration of premalignant lesions and condition,
refer to Ans 2 of same chapter.
For etiopathogenesis, refer to Ans 27 of same chapter.
For clinical features, histopathology and treatment, refer Fig. 41: Fibroma
(For color version, see Plate 7)
to Ans 4 of same chapter.
Q.29. Classify the non-odontogenic tumors of oral cavity and Treatment
describe fibroma. (Dec 2010, 16 Marks)
Or Excision of the lesion should be done.
Write short answer on fibroma. (May 2018, 3 Marks)
Q.30. Write short note on fibroma H/P (histopathology).
Ans. For classification of non-odontogenic tumors of oral
(Mar 2013, 3 Marks)
cavity refer to Ans 20 of same chapter.
Ans. For H/P (histopathology) of fibroma refer to Ans 29 of
Fibroma same chapter.
It is a benign tumor of connective tissue origin.
Q.31. Enumerate non-odontogenic connective tissue malig-
Clinical Features nant tumors. Discuss in detail about clinical features,
etiology and histopathology of fibrosarcoma.
♦♦ It is a slow growing lesion and can be seen during 3rd, 4th
(Mar 2013, 8 Marks)
and 5th decades of life.
530   Mastering the BDS IIIrd Year  (Last 25 Years Solved Questions)

Ans. Enumeration of non-odontogenic connective tissue


malignant tumors
• Fibrous connective tissue: Fibrosarcoma
• Adipose Tissue: Liposarcoma
• Cartilage: Chondrosarcoma
• Bone
– Osteosarcoma
– Osteochondrosarcoma
• Vascular:
– Hemangioendothelioma
– Angiosarcoma
– Kaposi sarcoma
• Neural tissue: Neurosarcoma or Neurofibrosar­
coma
Fig. 42: Fibrosarcoma
• Muscle (For color version, see Plate 7)
– Leiomyosarcoma
– Rhabdomyosarcoma Well-differentiated
• Lymphoid tissue ♦♦ In this multiple plump-shaped fibroblasts with pale eosino­
– Hodgkin and Non-Hodgkin lymphoma philic cytoplasm, hyperchromatic spindle-shaped nuclei
– Lymphosarcoma with tapered ends is seen.
– Reticular cell sarcoma ♦♦ Malignant fibroblasts are dispersed in rich collagen area.
– Ewing’s sarcoma ♦♦ Few mitotic figures are evident.
– Burkitt’s lymphoma
– Multiple myeloma Intermediate Grade
– Leukemia. ♦♦ In this tumor show Herring bone pattern, i.e. parallel sheets
of cells arranged in intertwining whorls.
Fibrosarcoma ♦♦ Cellularity is high.
It is the malignant fibrous connective tissue tumor and is the ♦♦ Cellular pleomorphism is evident.
malignant tumor of fibroblasts. ♦♦ Areas of hyalinization can be appreciated.

High Grade
Clinical Features
♦♦ Marked cellular atypia and mitotic activity is evident.
♦♦ It arises at any age but mean age is 40 years. ♦♦ This grade is highly anaplastic and pleomorphic with
♦♦ Male predilection is seen. bizarre nuclei.
♦♦ It is most commonly seen in lower extremities i.e. femur
and tibia. Q.32. Write short note on keratoacanthoma.
♦♦ In oral cavity tumor involves mandible, maxilla, maxillary (June 2014, 5 Marks)
sinus, lip and palate. Ans. It is also known as molluscum sebaceum, self-healing
♦♦ Tumor is generally a large painless mass which lies deep carcinoma or pseudocarcinoma.
to fascia and has ill-defined margin. It clinically and histologically resembles epidermoid
♦♦ Associated teeth become mobile. carcinoma so it is mistaken as oral carcinoma.
♦♦ Tumors in starting show benign growth and later on they
spread rapidly producing large tumor with ulceration Etiology
and hemorrhage. ♦♦ Hereditary predisposition is present.
♦♦ They can also cause pathological fracture. ♦♦ Human papilloma virus (HPV) 26 or 37 can lead to ker­
atoacanthoma.
Etiology ♦♦ Sun exposure
♦♦ Chemical agents such as coal tar and minerals
♦♦ Most of the fibrosarcomas arise from preexisting lesions
such as Paget’s disease, fibrous dysplasia chronic osteo­ Clinical Features
myelitis, bone infarcts and in previously irradiated areas ♦♦ Its occurrence is at the age of 50 to 70 years. Male predilec­
of bone. tion is present.
♦♦ Congenital fibrosarcomas are thought to arise from genetic ♦♦ Intraorally, it is most commonly found on lips.
mutations. ♦♦ Lesion is painful and regional lymphadenopathy is pre­
sent.
Histopathology ♦♦ Lesion is elevated, umblicated with depressed central
core with presence of plug of keratin. Lesion appears as
Various histological grading of fibrosarcoma are: dome shaped.
Section 3: Oral Pathology  531

♦♦ Margins of lesion are sharply delineated. Classic Type


♦♦ Lesion begins as small nodule which increases in size from ♦♦ Development of cutaneous multifocal blue red nodules on
4 to 6 weeks. Later on it undergoes spontaneous regression the lower extremities.
from 6 to 8 weeks with scar formation. ♦♦ Lesion slowly increases in size and number with some
of the lesions extinguishing and new ones forming on
Histopathological Features
adjacent or distinct skin.
♦♦ Orally, soft bluish nodules occur on palatal mucosa or
gingiva.
Lymphadenopathic
♦♦ Present in young African children.
♦♦ There is generalized or localized enlargement of lymph
node chain which includes cervical lymph nodes.
♦♦ Disease follow fulminant course with visceral involvement
and minimum skin or mucous membrane involvement.
Immunosuppression Associated
♦♦ It is usually seen in renal transplant patients.
♦♦ It occurs 1 to 2 years after transplantation
♦♦ Progression of disease is directly proportional to loss of
cellular immunity of host.

Fig. 43: Keratoacanthoma (For color version, see Plate 7)


AIDS Related
♦♦ Homosexual AIDS patients have maximum chances of
♦♦ Hyperplastic squamous epithelium growing into underly­ developing Kaposi’s sarcoma.
ing connective tissue. ♦♦ Lesions occur on cutaneous lesions i.e. along lines of cleav­
♦♦ Epithelial surface is covered by parakeratin or orthokeratin age and tip of nose.
with parakeratin plugging. ♦♦ Oral lesions can occur anywhere in oral cavity, but predi­
♦♦ Pseudocarcinomatous infiltration typically present lection is for palatal mucosa and gingiva.
smooth, regular, well demarcated front which does not ♦♦ Early oral mucosal lesions are flat and slight blue, red or
extend beyond the level of sweat gland. purple plaques, either focal or diffuse and may be com­
♦♦ Connective tissue show chronic inflammatory cell infil­ pletely asymptomatic.
tration. ♦♦ Later on these lesions may be more deeply discolored
♦♦ Most characteristic feature of lesion is seen at the and there is development of surface papules and nodules
margins where normal adjacent epithelium is elevated which may become exophytic and ulcerated. These lesions
towards central portion of crater, later on an abrupt can also bleed.
change in normal epithelium occurs as hyperplastic ♦♦ Cervical lymphadenopathy and salivary gland enlarge­
acanthotic epithelium is reached. ment is seen.
Treatment Histopathology
Surgical excision. Early lesion or Patch stage
♦♦ There is proliferation of small veins and capillaries around
Q.33. Write short note on Kaposi’s sarcoma. one or more preexisting dilated vessels.
(Feb 2014, 3 Marks) ♦♦ Slit like vessels are seen near periphery of preexisting
Ans. It is also known as multiple idiopathic hemorrhagic blood vessel, skin adnexa and in between collagen fibers.
sarcoma of Kaposi. ♦♦ Slit like vessels are lined by plump, mild atypical endothe­
• Kaposi’s sarcoma is the multicentric proliferation of lial cells.
vascular or spindle cell components. ♦♦ There is presence of mononuclear inflammatory cell infil­
• Kaposi’s sarcoma is currently associated with HIV/ trate consisting of mast cells, scattered erythrocytes and
AIDS. hemosiderin deposit.
Advance Lesion or Plaque Stage
Clinical Features
♦♦ There is presence of increased numbers of increased capil­
Clinically, Kaposi’s sarcoma is of four types, i.e. laries and dilated vascular channels which are interspersed
♦♦ Classic (Chronic) with proliferating sheets of sarcomatous cells with large
♦♦ Endemic (Lymphadenopathic; African) number of extravasated erythrocytes.
♦♦ Immunosuppression associated (Transplant) ♦♦ Slit like vessels without visible endothelial lining are in­
♦♦ AIDS related. terspersed with spindle cells.
532   Mastering the BDS IIIrd Year  (Last 25 Years Solved Questions)

♦♦ Lesional cells show enlarged, hyperchromatic nuclei with Treatment


mild-to-moderate pleomorphism.
Surgical Excision.
Nodular Stage
Q.36. Enumerate malignant epithelial tumors of oral cavity.
In all features are more prominent than plaque stage. Discuss about etiology, clinical features and histopa-
Q.34. Write short note on tori. (Feb 2014, 3 Marks) thology of verrucous carcinoma. (Feb 2014, 8 Marks)
Ans. Tori are of two types, i.e. torus palatinus and torus Ans. Enumeration of malignant epithelial tumors of oral cavity
mandibularis • Squamous cell carcinoma
• Intraepidermoid carcinoma
Torus Palatinus
• Adenocarcinoma
It is a slow growing, flat based boney protuberance which occurs • Metastatic carcinoma
in midline of hard palate. • Basal cell carcinoma
Clinical Features • Transitional cell carcinoma
• Malignant melanoma
♦♦ Women are affected more commonly.
• Verrucous carcinoma
♦♦ Torus palatinus can occur at any age. It reaches at its peak
• Intraepidermoid carcinoma
incidence at age of 30 years.
• Spindle cell carcinoma
♦♦ It is an outgrowth in the midline of palate.
• Primary intra-alveolar carcinoma
♦♦ It is spindle shaped, nodular or lobular.
• Adenoid squamous cell carcinoma.
♦♦ Mucosa overlying torus is intact and occasionally it ap­
pears blanched. For clinical features and histopathology of verrucous carcinoma
♦♦ It may become ulcerated if traumatized. refer to Ans 11 of same chapter.
♦♦ Torus itself may be composed of dense compact bone or
Etiology
dense compact bone with center of cancellous bone.
Tobacco: Verrucous carcinoma develops in tobacco chewers. It
Torus Mandibularis usually occurs in the buccal sulcus area where the person holds
It is an exostosis or outgrowth of mandible found on the lingual the tobacco most.
surface. Q.37. Describe histopathology of well-differentiated squa-
mous cell carcinoma. (Mar 2006, 5 Marks)
Clinical Features
Ans. For histopathology of well differentiated squamous cell
♦♦ Growth on lingual surface of mandible occurs at mylohy­ carcinoma refer to Ans 23 of same chapter.
oid line which is usually opposite to cuspid teeth.
Q.38. Write in brief lipoma. (Feb 2013, 8 Marks)
♦♦ Mandibular tori are bilateral.
Ans. Lipoma is a rare intra-oral tumor.
♦♦ Bilateral overgrowths are globular or they are multiple.
It is a benign, slow growing tumor of mature fat cells.
Treatment
Classification
For both tori treatment is surgical excision.
Q.35. Write short note on congenital epulis. On the basis of morphology
(Feb 2014, 3 Marks) Superficial: It is a single, yellow, lobulated, painless lesion with
Ans. It is also known as congenital epulis of newborn or sessile or pedunculated base.
Neumann’s tumor. Diffuse: It occurs in deeper tissues and produces a slight surface
elevation.
It is benign in nature and mostly occurs as single tumor.
Encapsulated: It is surrounded by a capsule.
Clinical Features
♦♦ Tumor is present at birth. Pathogenesis
♦♦ It is located over maxillary or mandibular gingiva. But ♦♦ HMGI-C gene which is mapped to 12q15 is a member of
more common on maxillary. high mobility group protein gene family, play a role in
♦♦ Lesion is pedunculated and is found on incisor region development of lipoma.
arising from crest of alveolar ridge or alveolar process. ♦♦ Cells of lipoma are different metabolically as compared to
♦♦ Lesion varies from few centimeters to millimeters. normal fat cells. Precursors of fatty acid should be incorpo­
rated at faster rate in fat of lipoma as compared to normal
Histology fat and there is reduction in lipoprotein lipase activity.
♦♦ Presence of sheets of large closely packed cells showing
fine, granular, eosinophilic cytoplasm which comprises Clinical Features
the tumor mass. ♦♦ Lipomas commonly occur in adults, usually around 3rd
♦♦ Numerous capillaries are present. and 4th decades of life or in older individuals too.
Section 3: Oral Pathology  533

♦♦ Lipomas are soft, smooth surface and nodular masses • Borders of cell are ill defined.
which can be sessile or pedunculated. • Cells also show mitotic figures.
♦♦ It is asymptomatic for months or years. • Cells of the sarcoma are arranged in filigree pattern.
♦♦ Buccal mucosa and buccal vestibule are the most common • Small vascular channels are also evident.
sites which are affected. Other sites involved are tongue, • Hemorrhage along with vascular lakes or sinuses
floor of mouth and lips. are appreciated.
♦♦ The lesion is 3 cm in size but with time, its size increases • Perivascular sparing and geographical necrosis is
up to 5 to 6 cm. very common in Ewing’s sarcoma.
Histopathology

Fig. 45:  Ewing’s sarcoma (For color version, see Plate 8)

Fig. 44: Lipoma (For color version, see Plate 8) Q.40. Write short note on epithelial dysplasia.
 (Nov 2014, 3 Marks)
♦♦ Lesion consists of mature fat cells and can demonstrate a Ans. Epithelial dysplasia comprises a loss in the uniformity
thin fibrous capsule. of individual cells or cytological atypia as well as a loss
♦♦ Lesion shows the lobular arrangement of fat cells. in their architectural orientation.
♦♦ Various microscopic variants of lipoma are appreciated
Dysplasia is a histopathological diagnosis which is made on the
histologically, i.e.
basis of presence of certain histological and cytological features
Fibrolipoma: Fibrous component intermixed with lobules of fat in premalignant lesions and conditions.
cells.
Angiolipoma: Admixture of mature adipocytes and multiple Architecture and Cytologic Criteria for Grading Epithelial
small blood vessels. Dysplasia Given by WHO (2005)
Spindle cell lipoma: Variable amount of spindle cell in conjunction
with lipomatous component. Architecture Criteria
Pleomorphic lipoma: Presence of spindle cells along with bizarre, ♦♦ Irregular epithelial stratification
hyperchromatic giant cells. ♦♦ Loss of polarity of basal cells
Myolipoma: When spindle cells are of smooth muscle origin ♦♦ Basal cell hyperplasia
Intramuscular lipomas: Infiltrative growth of mature adipocytes ♦♦ Drop-shaped rete pegs
which extend between skeletal muscle bundles. ♦♦ Increased number of mitotic figures
♦♦ Abnormally superficial mitosis
Myxoid lipoma: If stromal background is myxoid.
♦♦ Dyskeratosis, i.e. premature keratinization in the cell
Treatment ♦♦ Keratin pearls within rete ridges.
Conservative local excision is the treatment of choice. Cytologic Criteria
Q.39. Write short note on Ewing’s sarcoma—histopathology. ♦♦ Anisonucleosis: Abnormal variation in nuclear size
 (Nov 2014, 3 Marks) ♦♦ Nuclear pleomorphism: Abnormal variation in nuclear
Ans. Following is the histopathology of Ewing’s sarcoma: shape
• The lesion consists of solid sheets of small round ♦♦ Anisocytosis: Abnormal variation in cell size
cells with very minimal stroma but at a few places ♦♦ Cellular pleomorphism: Abnormal variation in cell shape
connective tissue septae are present. ♦♦ Increased nuclear cytoplasmic ratio
• Cells are round and small in shape along with scanty ♦♦ Increased nuclear size
cytoplasm, nuclei of the cell is large round to oval in ♦♦ Atypical mitotic figures
shape along with dispersed chromatin and hyper­ ♦♦ Increase in the number and size of nucleoli
chromasia. ♦♦ Hyperchromatism.
534   Mastering the BDS IIIrd Year  (Last 25 Years Solved Questions)

Grading of Epithelial Dysplasia ♦♦ Antoni A: This pattern is made up of the cells which have
spindle shape or elongated nuclei which are aligned to
♦♦ Histopathological interpretation of potentially malignant
form a characteristic palisaded pattern and the intercel­
disorders shows presence of epithelial dysplasia.
lular fibers are arranged in parallel fashion between rows
♦♦ There are numerous prognostic molecular markers but
of nuclei. In some areas, these fibers show arrangement in
the epithelial dysplasia is considered to be the strongest
shape of whorls or swirls. In the palisaded arrangement of
predictor of future malignant transformation in potentially
cells around central acellular area eosinophilic areas are
malignant disorders.
seen known as Verocay bodies.
♦♦ Grading of epithelial dysplasia is divided into three cat­
♦♦ Antoni B: In this pattern, there is disordered arrange­
egories:
ment of spindle cells and fibers in loose myxomatous
Mild: Cellular atypia and architectural disturbances limited to
stroma with areas of edema fluid and formation of
basal and parabasal layers.
microcysts.
Moderate: Cellular atypia and architectural disturbances limited
from basal to midportion of spinous cell layer.
Severe: Cellular atypia and architectural disturbances from basal
layer to a level above midpoint of epithelium.
Carcinoma in situ: Theoretical concept of carcinoma in situ is
that malignant transformation has occurred but invasion is
absent. This is the most severe form of epithelial dysplasia and
involves entire thickness of epithelium. This is cytologically
similar to squamous cell carcinoma but architecturally the
epithelial basement membrane remains intact and no invasion
in connective tissue has occurred.
Q.41. Write short note on neurilemmoma. Fig. 46: Neurilemmoma (For color version, see Plate 8)
 (Feb 2014, 5 Marks)
Ans. It is also known as schwannoma.
Treatment
It is a benign neural neoplasm of Schwann cell origin.
Surgical excision is the treatment of choice.
Clinical Features
Q.42. Enumerate the carcinomas of oral cavity. Describe
♦♦ The lesion occurs most commonly in young and middle staging and grading of squamous cell carcinoma.
age adults.  (June 2015, 10 Marks)
♦♦ Neurilemmoma is a slow growing encapsulated tumor Ans. Enumeration of carcinomas of oral cavity
which arises in association with nerve trunk.
• Squamous cell carcinoma
♦♦ Usually the tumor mass is asymptomatic, but tenderness
• Verrucous carcinoma
or pain is present if tumor is causing pressure in associ­
• Basaloid squamous cell carcinoma
ate nerves.
• Adenoid squamous cell carcinoma
♦♦ Tumor may range from few millimeters to several cen­
• Spindle cell carcinoma
timeters in size.
• Adenosquamous carcinoma
• Undifferentiated carcinoma.
Oral Manifestations
♦♦ It occurs commonly in oral cavity and the most common Staging of Squamous Cell Carcinoma
site of occurrence is tongue. Staging is defined as extent of spread of tumor within the body.
♦♦ The lesion is a single, circumscribed nodule of varying size Staging of squamous cell carcinoma is done by TNM
which presents no pathognomonic features.
classification which was given by American Joint Committee
♦♦ Neurilemmoma also occur as central lesion in the mandible
on Cancer (AJCC)
and arises from mandibular nerve.
♦♦ Centrally occurring lesion may lead to destruction of bone T is suggestive of primary tumor
with expansion of cortical plates. N is suggestive of regional lymph nodes
♦♦ Pain and paresthesia is common in centrally occurring M is suggestive of distant metastasis
lesions.
T – primary Tumor
Histopathology TX – Primary tumor cannot be assessed.
♦♦ Usually, the lesion presents two types of histologic pat­ T0 – No evidence of primary tumor
terns, i.e. Antoni A and Antoni B pattern. Tis – Carcinoma in situ
Section 3: Oral Pathology  535

T1 – Tumor 2 cm of less in greatest dimension Broader’s Classification


T2 – Tumor more than 2 cm but not more than 4 cm in greatest ♦♦ Grade I – Well differentiated, - <25% undifferentiated cells
dimension ♦♦ Grade II – Moderately differentiated - <50% undifferenti­
T3 – Tumor more than 4 cm in greatest dimension ated cells
T4a (Lip) – Tumor invades through cortical bone, inferior ♦♦ Grade III – Poorly differentiated - <75% undifferentiated
alveolar nerve, floor of mouth or skin (chin or nose). cells
T4a (Oral Cavity) – Tumor invades through cortical bone, into ♦♦ Grade IV – Anaplastic/Pleomorphic - >75% undifferenti­
deep/extrinsic muscle of tongue (genioglossus, hyoglossus, ated cells
palatoglossus and styloglossus), maxillary sinus or skin of face.
Q.43. Write short note on radiological features of osteosar-
T4b (lip and oral cavity) – Tumor invades masticatory space, coma. (Jan 2016, 5 Marks)
pterygoid plates or skull base or encases internal carotid Ans. The radiographic features of osteosarcoma are divided
artery into following stages, i.e.
N – Regional lymph nodes ♦♦ Osteolytic Stage
NX – Regional lymph nodes cannot be assessed • It reveals moth eaten appearance.
N0 – No regional lymph node metastasis • Border of the lesion at this stage are ill defined.
N1- Metastasis in a single ipsilateral lymph node, 3 cm or less • There is perforation and expansion of cortical plates.
in greatest dimension. • Lamina dura is absent, i.e. it get destroyed.
N2a – Metastasis in a single ipsilateral lymph node, more than • Pathological fracture may be present.
3 cm but not more than 6 cm in greatest dimension. • Root resorption is present.
N2b – Metastasis in multiple ipsilateral lymph nodes, not more ♦♦ Mixed Stage
than 6 cm in greatest dimension. • It is called as mixed because this stage shows formation
N2c – Metastasis in bilateral or contralateral lymph nodes, not and destruction of bone.
more than 6 cm in greatest dimension. • It reveals honeycomb appearance.
N3 – Metastasis in a lymph node more than 6 cm in greatest • Margins of lesion are ill defined.
dimension. ♦♦ Osteoblastic Stage
M – Distant Metastasis • It reveals sun ray appearance.
MX – Distant metastasis cannot be assessed • At times subperiosteal bone laid down in layers which
M0 – No distant metastasis results in onion peel appearance.
M1 – Distant metastasis. • In osteosarcoma periosteum is elevated over the
expanding tumor mass in a tent like fashion. At point on
Stage Grouping of Oral Cancer the bone where the periosteum begin to merge an acute
angle between periosteum and bone is created which is
Stage 0 Tis N0 M0 known as Codman’s Triangle.
Stage I T1 N0 M0 Q.44. Write short note on papilloma. (Mar 2016, 3 Marks)
Stage II T2 N0 M0 Ans. Papilloma is the benign proliferation of stratified
Stage III T1, N1 M0
squamous epithelium which result in papillary and
verruciform mass.
T2 N1 M0
T3 N0, N1 M0 Etiology
Stage Iva T1,T2,T3 N2 M0 Papilloma is caused by human papilloma virus. Viral subtypes
T4a N0, N1, N2 M0 6 and 11 are identified in 50% of oral papillomas.
Stage IVb Any T N3 M0
T4b Any N M0
Mode of Transmission
Stage IVc Any T Any N M1 Exact mode of transmission is unknown but transmission is by
sexual and non-sexual person to person contact, contaminated
objects, saliva or breast milk.
Grading of Squamous Cell Carcinoma
Clinical Features
Grading is defined as macroscopic and microscopic degree of
♦♦ Papilloma can be diagnosed at any stage but more com­
differentiation of a tumor.
monly it is seen in persons of 30 to 50 years of age.
Squamous cell carcinoma is divided in following categories by ♦♦ Papilloma occurs more commonly on tongue, lip and soft
Broder also known as Broder’s classification: palate but it can involve any of the surface.
536   Mastering the BDS IIIrd Year  (Last 25 Years Solved Questions)

♦♦ Papilloma is a soft, painless usually pedunculated, exo­ ♦♦ Appearance of jaw swelling is rapid and intra-oral mass
phytic nodule with numerous finger like projections which become ulcerated.
impart cauliflower or wartlike appearance. ♦♦ Tumor commonly penetrates the cortex resulting in soft
♦♦ Lesion can be white, slightly red or normal in color depend tissue mass which overlie affected area of bone.
on surface keratinization.
♦♦ Papilloma remains solitary and enlarges to the maximum Radiographic Features
size of 0.5 cm but lesions of 3 cm are also reported. ♦♦ Most common finding is formation of layers of new sub­
Histopathology periosteal bone which produces onion skin appearance
of film.
♦♦ Papilloma is characterized by proliferation of keratinized ♦♦ There is presence of irregular lytic bone destruction with
stratified squamous epithelium which is arranged in finger ill-defined margins.
like projections along with the fibrovasccular cores.
♦♦ Its hallmark feature is proliferation of spinous cells in
papillary pattern.
♦♦ Connective tissue core can show inflammatory changes.
Presence of chronic inflammatory cells can be variably
noted in connective tissue.
♦♦ Koilocytes, i.e. virus altered epithelial clear cells with small
dark or pyknotic nuclei with perinuclear clear spaces are
sometimes seen in prickle cell layer.

Fig. 48:  Ewing’s sarcoma (H and E stain) (For color version, see Plate 8)

Histopathology
♦♦ Ewing’s sarcoma is highly cellular which consists of solid
sheets or mass of small round cells with little stroma, few
connective tissue septa can also be seen.
♦♦ Cells are small and round in shape having scanty cyto­
plasm. Nuclei of cell is large round to oval in shape with
dispersed chromatin and hyperchromasia. Borders of cell
Fig. 47:  Papilloma (H and E Stain) (For color version, see Plate 8)
are indistinct.
Treatment ♦♦ Cells are arranged in filigree pattern.
♦♦ Mitotic figures are commonly seen.
Surgical excision should be done including the base of lesion. ♦♦ Multiple small vascular channels are also present.
♦♦ Hemorrhage with vascular lakes or sinuses can be seen.
Q.45. Write short note on Ewing’s sarcoma.
♦♦ There is also presence of geographic necrosis with perivas­
 (July 2016, 5 Marks) cular sparing.
Ans. Ewing’s sarcoma is a small round cell tumor. It arises ♦♦ Necrosis can be seen on the opposite side of fragment of bone.
inside the bone.
Histological Differential Diagnosis
Clinical Features
♦♦ Small cell osteosarcoma
♦♦ It occurs predominantly in children and young adults ♦♦ Peripheral neuroectodermal tumor of infancy
between 5 to 25 years with median age of 13 years. ♦♦ Metastatic neuroblastoma
♦♦ It is more common in males as compared to females. ♦♦ Mesenchymal chondrosarcoma
♦♦ Whites develop this tumor more commonly but in blacks ♦♦ Malignant lymphoma
not even single case is reported. ♦♦ Embryonal rhabdomyosarcoma.
♦♦ Pain in Ewing’s sarcoma is intermittent in nature.
♦♦ Swelling of the bone is earliest clinical sign. Treatment
♦♦ Long bones of extremities are more commonly affected
besides these skull, clavicle, ribs, shoulder are also affected. ♦♦ Radical surgical excision should be done alone or coupled
♦♦ Ewing’s sarcoma in occur more commonly in mandible as with X-ray radiation.
compared to maxilla ♦♦ Current treatment consists of combined surgery, radio­
♦♦ Paresthesia and loosening of teeth are common findings therapy and multidrug chemotherapy which led to 40 to
in Ewing’s sarcoma of jaw. 80% of survival rates.
Section 3: Oral Pathology  537

Q.46. Write short note on features of epithelial dysplasia. ♦♦ Death occurs due to renal failure as there is accumulation
 (July 2016, 5 Marks) of abnormal proteins in renal tissue.
Ans. Following are the features of epithelial dysplasia:
Oral Manifestations
Architecture and Cytologic Criteria for Grading Epithelial ♦♦ Mandible is commonly involved as compared to maxilla.
Dysplasia Given by WHO (2005) Angle of mandible is commonly involved.
♦♦ Patient complaints of pain, numbness and swelling of jaw.
Architecture Criteria ♦♦ Intra-oral swelling is present which tends to be ulcerated,
♦♦ Irregular epithelial stratification rounded and bluish red.
♦♦ Loss of polarity of basal cells ♦♦ Tongue may show diffuse enlargement and firmness or
♦♦ Basal cell hyperplasia may have nodular appearance. Sometimes nodules are
♦♦ Drop-shaped rete pegs ulcerated.
♦♦ Increased number of mitotic figures
Radiographic Features
♦♦ Abnormally superficial mitosis
♦♦ Dyskeratosis, i.e. premature keratinization in the cell Presence of multiple well defined, punched out radiolucency or
♦♦ Keratin pearls within rete ridges. ragged radiolucent lesions. This is evident on skull radiograph.

Cytologic Criteria Laboratory Investigations


♦♦ Anisonucleosis: Abnormal variation in nuclear size ♦♦ Urinary examination: Light chain products found in urine
♦♦ Nuclear pleomorphism: Abnormal variation in nuclear of 30 to 50% of multiple myeloma patients are known as
shape Bence Jones proteins.
♦♦ Anisocytosis: Abnormal variation in cell size ♦♦ On serum protein electrophoresis most patients suffer­
♦♦ Cellular pleomorphism: Abnormal variation in cell shape ing from multiple myeloma have decreased quantity of
♦♦ Increased nuclear cytoplasmic ratio normal immunoglobulin and an abnormal monoclonal
♦♦ Increased nuclear size immunoglobulin protein peak, known as M spike. The
♦♦ Atypical mitotic figures immunoglobulin is usually of IgG or IgA class with
♦♦ Increase in the number and size of nucleoli monoclonal light chain component.
♦♦ Hyperchromatism. ♦♦ Alkaline phosphatase levels are raised.
Q.47. Write short note on multiple myeloma.
Histopathology
 (July 2016, 5 Marks)
Ans. Multiple myeloma is a relatively uncommon malignancy ♦♦ There is presence of sheets of closely packed cells resem­
of plasma cell origin which often has multicentric origin bling plasma cells. Cells are round to oval in shape with
within the bone. eccentrically placed nucleus exhibiting chromatin clump­
ing in cartwheel or checkerboard pattern.
Pathogenesis ♦♦ Perinuclear halo can also be seen.
Mutation of terminally differentiated B cells or early committed ♦♦ Russell bodies are seen in chronic inflammatory lesions
B cells lead to more differentiated plasma cells. Abnormal with numerous typical plasma cells.
plasma cells probably arising from single malignant precursor ♦♦ Mitotic activity can be seen along with some frequency.
which had undergone uncontrolled mitotic division and spreads ♦♦ If amyloid is present, it appears as homogenous, eosino­
throughout the body. Since the neoplasm develops from the philic and relatively acellular.
single cell, all daughter cells which comprises of the lesional
tissue have same genetic makeup and produce same proteins.

Clinical Features
♦♦ It occurs between 40 to 70 years of age.
♦♦ Male to female ratio is 4:1.
♦♦ Bone pain is present particularly in the lumbar spine.
♦♦ Pathological fractures are also present.
♦♦ Petechial hemorrhages of skin can be seen.
♦♦ Swelling over the areas of bony involvement can be seen.
♦♦ Metastatic calcification may involve the soft tissue and
is due to hypercalcemia secondary to tumor related os­
teolysis.
♦♦ Amyloid deposits occur at periorbital region appearing
Fig. 49:  Multiple myeloma (For color version, see Plate 9)
waxy, firm, plaque like lesions.
538   Mastering the BDS IIIrd Year  (Last 25 Years Solved Questions)

Treatment Collagen Production Pathway as Regulated by TGF – β


♦♦ Multiple myeloma is treated by chemotherapeutic agents Transforming growth factor-β (TGF–β) is a growth factor which
such as melphalan and cyclophosphamide along with regulate the collagen production pathway, it has autocrine
prednisolone. activity. TGF–β activate procollagen genes which lead to
♦♦ Oral bicarbonates, high fluid intake and corticosteroid production of more procollagen. TGF–β also increases secretion
decreases calcium levels. of procollagen – C – proteinase (PCP) and procollagen – N –
♦♦ Local radiation therapy decreases painful bony lesions. proteinase (PNP) both of these are needed for conversion of
procollagen to collagen fibrils.
Q.48. Write short note on pathogenesis of oral submucous
fibrosis. (July 2016, 5 Marks)
Ans. Basically pathogenesis of OSMF consists of disturbance
in collagen metabolism via TGF – β. The whole procedure
is described as:

Initial Event of Disease Process


According to P.Rajalitha, S.Vali (2005) initial event of the
disease process is, oral mucosa which is in direct contact with
betel quid (tobacco + areca nut + slaked lime + catechu and
condominents) because of the habit which is the meAns of
constant irritation. Major areca nut alkaloids are arecoline,
arecaidine, arecolidine, guacoline and guacine. Out of all
these arecoline is most abundant. These alkaloids undergo
nitrosation and give rise to N – nitrosamines which have
cytotoxic effects on cells. Important flavonoid components
of areca nut are tannins and catechins. So all these alkaloids,
flavonoids and microtrauma produced by friction of coarse
fibers of areca nut leads to chronic inflammatory process
which is characterized by presence of inflammatory cells, i.e.
T cells and macrophages, these cells release and/or stimulate
synthesis of various cytokines and growth factors. Increased
susceptibility among individuals who are anemic due to iron 
or vitamin B12 deficiencies. It is because of increased fragility
of mucosa by which there is more betel quid absorption. Fig. 51:  Collagen production pathway as regulated by TGF-β

In oral submucous fibrosis, there is increase in cross


linking of collagen which results to increase in insoluble form.
This is facilitated by increased activity and production of key
enzyme Lysyl oxidase (LOX). PCP and bone morphogenetic
protein 1 and increased copper in betel quid stimulate the
activity of LOX which produces increase collagen which is
insoluble. Flavonoids increases the cross linking of collagen
fibers. These all steps lead to increase in collagen production.
TGF – β strongly promote the expression of LOX both at
mRNA and protein levels in various cell lines. LOX activity
is important in formation of insoluble collagen due to cross
linking. Process of cross linking provides tensile strength
and mechanical properties to fibers as well as makes collagen
fibers resistant to proteolysis.
Collagen Degradation Pathway as Regulated by TGF–β
There are also two main events modulated by TGF–β which also
decreases collagen degradation, i.e. activation of tissue inhibitor
of matrix metalloproteinase gene (TIMPs) and activation of
Fig. 50:  Initial event of disease process plasminogen activator inhibitor (PAI) gene.
Section 3: Oral Pathology  539

Q.49. Write short note on histopathology of oral submucous


fibrosis. (Jan 2017, 5 Marks)
Ans. Following is the common histopathological features of
oral submucous fibrosis:
♦♦ Overlying hyperkeratinized or atrophic epithelium often
shows flattening and shortening of rete pegs.
♦♦ There can be variable degrees of cellular atypia or epithelial
dysplasia.
♦♦ In OSMF dysplastic changes are found in epithelium
which include nuclear pleomorphism, sever intercellular
edema, etc.
♦♦ Stromal blood vessels are dilated and congested and there
can be areas of hemorrhage.
♦♦ Underlying connective tissue stroma in advanced stage
of disease shows homogenization and hyalinization of
collagen fibers.
♦♦ Decreased number of fibroblastic cells and narrowing
of blood vessels due to perivascular fibrosis are present.
♦♦ There can be presence of signet cells in some cases.

Fig. 52:  Collagen degradation pathway as regulated by TGF-β


Collagen degradation pathway as regulated by TGF-β
activate genes for TIMPs and so more TIMP is formed.
Now this inhibit activated collagenase enzyme which is
necessary for degradation of collagen. It also activate gene
for PAI which is the inhibitor of plasminogen activator,
so there is no plasmin formation. Plasmin is needed
for the conversion of procollagenase to active form of
collagenase and absence of plasmin lead to absence of
active collagenase. Flavonoids inhibit collagenase activity. Fig. 54:  Oral submucous fibrosis
Reduction in activity and levels of collagenase result in
Histopathology Based on Various types of OSMF
decrease in collagen degradation.
Khanna JN and Andrade NN in 1995 given the group
Overall Effect of TGF–β pathway
classification system for OSMF which consists of various groups
So, overall effect of activated TGF–β pathway is that there is of OSMF and their clinical and histological aspect. So following
an increase in collagen production and cross-linking (insoluble are the histological aspects as per the OSMF group:
form) along with decrease in collagen degradation. This Group of OSMF Histological aspect
produces an increased collagen deposition in subepithelial
Group I or very - Presence of fine fibrillar collagen network
connective tissue layer of oral mucosa causing oral submucous early cases interspersed with marked edema
fibrosis. - Blood vessels are dilated and congested
- Large aggregate of plump, young fibroblast
present with abundant cytoplasm
- Inflammatory cells mainly consist of
polymorphonuclear leukocytes with few
eosinophils
- Epithelium is normal
Group II or early - Juxta-epithelial hyalinization is present
cases - Collagen present as thickened but separate
bundles
- Blood vessels are dilated and congested
- Young fibroblast seen in moderate number
- Inflammatory cells mainly consist of
polymorphonuclear leukocytes with few
eosinophils and occasional plasma cells
- Flattening or shortening of epithelial rete
Fig. 53:  Overall effect of TGF-β pathway pegs evident with degree of keratinization
Contd...
540   Mastering the BDS IIIrd Year  (Last 25 Years Solved Questions)

Contd... ♦♦ Tongue involvement results in increase in size of tongue,


Group of OSMF Histological aspect i.e. macroglossia. Anterior dorsal part of tongue is com­
Group III or - Juxtaepithelial hyalinization present monly affected.
moderately - Thickened collagen bundles faintly ♦♦ Lip involvement can lead to macrocheilia.
advanced cases discernible, separated but very slight, ♦♦ Small lymphangiomas are less than 1 cm in size and occur
residual edema on alveolar ridge of black neonates.
- Blood vessels, mostly constricted
- Mature fibroblasts with scanty cytoplasm and Histopathology
spindle-shaped nuclei
- Inflammatory exudates consist mainly of
♦♦ Lesion shows multiple, intertwining lymph vessels in loose
lymphocytes and plasma cells fibrovascular stroma.
- Epithelium markedly atrophic with loss of ♦♦ Cavernous type show many dilated lymphatics with single
rete pegs layer of endothelial cells with flat nuclei and having lymph.
- Muscle fibers seen interspersed with ♦♦ Vessels which lie beneath surface epithelium replace
thickened and dense collagen fibers connective tissue papillae and produce papillary surface
Group IV A– - Collagen hyalinized as smooth sheet change. This appears as translucent vesicle like clinical
Advanced cases - Extensive fibrosis obliterated the mucosal appearance.
blood vessels and eliminated the
Group IV B– ♦♦ At times channels can be filled with blood which is known
melanocytes
Advanced cases - Fibroblasts were markedly absent within the as hemangiolymphangioma.
with premalignant hyalinized zones ♦♦ At times channels demonstrate proliferation of lymphatic
and malignant - Total loss of epithelial rete pegs channels with smooth muscle cells known as lymphangi­
changes - Mild-to-moderate atypia present omyoma.
- Extensive degeneration of muscle fibers ♦♦ Cystic hygroma shows cyst like structures.
evident

Q.50. Write short note on lymphangioma. (Jan 2018, 5 Marks)


Ans. Lymphangioma is a benign hamartomatous tumor of
lymphatic vessels.
Lymphangioma is thought to be developmental malformation of
vessels which have poor communication with normal lymphatic
system.

Types
Following are the types of lymphangiomas:
♦♦ Lymphangioma simplex or capillary lymphangioma: It consists
of small thin walled capillaries.
♦♦ Cavernous lymphangioma: It consists of dilated lymphatic Fig. 55: Lymphangioma (For color version, see Plate 9)
vessels with surrounding adventitia.
♦♦ Cystic lymphangioma or cystic hygroma: It consists of large, Treatment
macroscopic cystic spaces with surrounding fibrovascular ♦♦ Surgical removal should be done.
tissue and smooth muscle. ♦♦ Complete removal is impossible.
♦♦ Benign lymphangioendothelioma: In this lymphatic channels ♦♦ Surgical debulking is the typical treatment provided.
appear dissecting through dense collagen bundles.

Clinical Features
♦♦ Lymphangioma occur more commonly on head and neck.
3. Tumors of Salivary
♦♦ Cervical lymphangiomas are common in posterior triangle Glands
and are soft as well as fluctuant masses.

Oral Manifestations Q.1. Describe the clinical features, histology, patho­genesis


and differential diagnosis of pleomorphic adenoma.
♦♦ Most commonly lesion occurs on tongue and also seen on
(Mar 2003, 15 Marks)
palate, buccal mucosa, gingiva and lips.
♦♦ Superficial lesions occur as papillary lesions which are of Or
same color to surrounding mucosa or can be of red hue. Classify salivary gland tumors. Describe in detail the
♦♦ Deep lesions appear as diffuse nodules or masses. clinical features, histolopathology and treatment of
♦♦ Secondary hemorrhage in lymphatic spaces can cause some
pleomorphic adenoma. (Apr 2017, 10 Marks)
of vesicles to appear purple.
Section 3: Oral Pathology  541

Ans. Clinical Features


Salivary Gland Tumor Classification by WHO (2017)
♦♦ Malignant tumors ♦♦ Pleomorphic adenoma develops in 5th and 6th decade
• Mucoepidermoid carcinoma 8430/3 of life.
• Adenoid cystic carcinoma 8200/3 ♦♦ It produces slow growing, well delineated exophytic
• Acinic cell carcinoma 8550/3 growth of salivary gland.
• Polymorphous adenocarcinoma 8525/3 ♦♦ Surface of lesion is smooth and lobulated and generally
• Clear cell carcinoma 8310/3 there is no pain.
• Basal cell adenocarcinoma 8147/3 ♦♦ Neoplasm is usually soft or rubbery in consistency and is
• Intraductal carcinoma 8500/2 freely movable.
• Adenocarcinoma, NOS 8140/3 ♦♦ Parotid gland lesion is usually superficial and often arises
• Salivary duct carcinoma 8500/3 in superficial lobe as a small mass overlying angle of man­
• Myoepithelial carcinoma 8982/3 dible or anterior to external ear.
• Epithelial – myoepithelial carcinoma 8562/3 ♦♦ Sometimes, lesion can be multinodular and can assume an
• Carcinoma ex pleomorphic adenoma 8941/3 enormous size especially in long standing lesions.
• Secretory carcinoma 8502/3* ♦♦ In buccal mucosa or lip pleomorphic adenoma presents
• Sebaceous adenocarcinoma 8410/3 small, painless, well defined, movable nodular lesion with
• Carcinosarcoma 8980/3 intact overlying mucosa.
• Poorly differentiated carcinoma
–– Undifferentiated carcinoma 8020/3 Histopathology
–– Large cell neuroendocrine carcinoma 8013/3
–– Small cell neuroendocrine carcinoma 8041/3 ♦♦ Neoplasm often exhibits proliferation of glandular epithe­
• Lymphoepithelial carcinoma 8082/3 lial cells in form of diffuse sheet or clusters.
• Squamous cell carcinoma 8070/3 ♦♦ Neoplastic cells are polygonal, spindle or stellate shape
• Oncocytic carcinoma 8290/3 and have tendency to form duct like structures.
Uncertain malignant potential ♦♦ Duct like structures are of varying size, shape, number and
• Sialoblastoma 8974/1 are widely distributed within lesion.
♦♦ Benign tumors ♦♦ Histologically, each duct like structure exhibits an inner
• Pleomorphic adenoma 8940/0 row of cuboidal or columnar cells and outer row of spindle
• Myoepithelioma  8982/0 shape myoepithelial cells.
• Basal cell adenoma 8147/0 ♦♦ Epithelial cells show “squamous metaplasia” and some­
• Warthin tumor 8561/0 times, there may be formation of keratin pearls by meta­
• Oncocytoma 8290/0 plastic epithelial cells.
• Lymphadenoma8563/0* ♦♦ Connective tissue undergoes hyalinization to form struc­
• Cystadenoma 8440/0 tureless homogeneous material.
• Sialadenoma papilliferum  8406/0 ♦♦ Mucoid materials in myochondroid are composed of
• Ductal papillomas 8503/0 glycosaminoglycAns and consist mainly of chondroitin
• Sebaceous adenoma 8410/0 sulfate.
• Canalicular adenoma and other ductal ♦♦ Complete capsule is never present.
adenomas 8149/0
♦♦ Non-neoplastic epithelial lesions
• Sclerosing polycystic adenosis
• Nodular oncocytic hyperplasia
• Lymphoepithelial sialadenitis
• Intercalated duct hyperplasia
♦♦ Benign soft tissue lesions
• Haemangioma 9120/0
• Lipoma/sialolipoma 8850/0
• Nodular fasciitis 8828/0
♦♦ Haematolymphoid tumors
• Extranodal marginal zone lymphoma of
mucosa-associated lymphoid tissue
(MALT lymphoma) 9699/3
*These new codes were approved by the IARC/WHO Committee for ICD-0.
Pleomorphic adenoma or benign mixed tumor is most
common neoplasm of salivary glands. The parotid gland is
Fig. 56:  Pleomorphic adenoma (For color version, see Plate 9)
mostly affected by the tumor.
542   Mastering the BDS IIIrd Year  (Last 25 Years Solved Questions)

Pathogenesis Q.2. Write notes on pleomorphic adenoma.


♦♦ There is presence of myoepithelial cells and reserve cells (Feb 1999, 5 Marks) (Sep 1999, 5 Marks)
arranged in intercalated duct. Ans. Refer to Ans 1 of same chapter.
♦♦ The intercalated duct reserve cells can differentiate into Q.3. Write short note on histopathology of pleomorphic
ductal and myoepithelial cells and the later can then un­ adenoma. (Mar 2013, 3 Marks)
dergo mesenchymal metaplasia.
♦♦ Dardick’s theory: It is the most accepted theory. They state Or
that a neoplas­tically altered epithelial cell with the poten­ Describe in brief histopathological features of pleo-
tial for multid­irectional differentiation may be histologi­ morphic adenoma. (Aug 2011, 5 Marks)
cally responsible for the pleomorphic adenoma.
Or
Differential Diagnosis Describe histopathology of pleomorphic adenoma.
♦♦ Polymorphous low-grade adenocarcinoma: It shows perineural  (Mar 2006, 5 Marks)
growth and is infiltrative into periglandular tissue. It form Ans. Refer to Ans 1 of same chapter.
small tubular structures or single cord of cells at periphery.
♦♦ Carcinoma Ex pleomorphic adenoma: Features of malignancy Q.4. Classify salivary gland diseases. Write about clinical
are seen in background of mixed tumor. features and histopathology of mucoe­pidermoid car-
♦♦ Epithelial myoepithelial carcinoma: Two families of cells cinoma. (Sep 2007, 6 Marks)
are seen, larger epithelial cells are arranged in tubular or Or
acinar crystals and small myoepithelial cells seen mainly
Classify salivary gland neoplasm. Discuss in detail
as “bipolar” spindly nuclei.
mucoepidermoid carcinoma. (Sep 2005, 14 Marks)
♦♦ Warthin’s tumor: It consists of bilayered oncocytic epithe­
lium, the inner cells of which are tall columnar with fine Or
granular and eosinophilic cytoplasm and slightly hyper­ Classify salivary gland disorders describe mucoepi-
chromatic nuclei. The outer layer consists of basaloid cells. dermoid carcinoma. (Aug 2012, 15 Marks)
♦♦ Oncocytoma: It consists of sheets of large polyhedral cells
Or
with abundant granular eosinophilic cytoplasm. These
cells have centrally located nuclei which can vary from Classify salivary gland tumors. Write in detail about
small and hyperchromatic to large and vesicular. mucoepidermoid carcinoma. (July 2016, 10 Marks)
♦♦ Mucoepidermoid carcinoma: It consists of mixture of mucus Ans. Enumeration of salivary gland tumors.
producing cells ad squamous (epidermoid) cells. Mucus
cells vary in shape and consist of foamy cytoplasm while Salivary Gland Tumor Classification by WHO (2017)
epidermoid cells have squamoid features, polygonal shape, ♦♦ Malignant tumors
intercellular bridges and rarely keratinization. It is of three • Mucoepidermoid carcinoma 8430/3
types, i.e. low grade, high grade and intermediate grade. • Adenoid cystic carcinoma 8200/3
♦♦ Adenoid cystic carcinoma: It is a malignant tumor. It con­ • Acinic cell carcinoma 8550/3
sists of mixture of myoepithelial cells and ductal cells • Polymorphous adenocarcinoma 8525/3
which have varied arrangement. Three major patterns • Clear cell carcinoma 8310/3
are cribriform, tubular and solid. It also shows increased • Basal cell adenocarcinoma 8147/3
nuclear atypia. • Intraductal carcinoma 8500/2
♦♦ Various mesenchymal tumors such as nerve sheath tumor, • Adenocarcinoma, NOS 8140/3
smooth muscle tumor. • Salivary duct carcinoma 8500/3
Treatment • Myoepithelial carcinoma 8982/3
• Epithelial – myoepithelial carcinoma 8562/3
♦♦ Pleomorphic adenomas are best treated by surgical exci­ • Carcinoma ex pleomorphic adenoma 8941/3
sion. • Secretory carcinoma 8502/3*
♦♦ Pleomorphic adenoma present in superficial lobe of parotid • Sebaceous adenocarcinoma 8410/3
gland undergoes superficial parotidectomy with identifica­
• Carcinosarcoma 8980/3
tion and preservation of facial nerve.
• Poorly differentiated carcinoma
♦♦ For tumors of deep lobe of parotid gland, total parot­
–– Undifferentiated carcinoma 8020/3
idectomy is necessary with preservation of facial nerve
–– Large cell neuroendocrine carcinoma 8013/3
if possible.
–– Small cell neuroendocrine carcinoma 8041/3
♦♦ Pleomorphic adenoma in submandibular gland is treated
• Lymphoepithelial carcinoma 8082/3
by total removal of the gland along with tumor.
• Squamous cell carcinoma 8070/3
♦♦ Pleomorphic adenoma of hard palate is excised down to
• Oncocytic carcinoma 8290/3
periosteum, including overlying mucosa.
♦♦ At other oral sites lesion enucleates easily through the Uncertain malignant potential
incision site. • Sialoblastoma 8974/1
Section 3: Oral Pathology  543

♦♦ Benign tumors ♦♦ Facial nerve palsy and pain is present at times.


• Pleomorphic adenoma 8940/0 ♦♦ Low-grade mucoepidermoid carcinomas is a slowly
• Myoepithelioma  8982/0 enlarging painless mass which rarely exceeds 5 cm in
• Basal cell adenoma 8147/0 diameter.
• Warthin tumor 8561/0 ♦♦ High-grade mucoepidermoid carcinomas grow rapidly
• Oncocytoma 8290/0 and cause pain. It also infiltrates the surrounding tissues
• Lymphadenoma8563/0* and metastatize to regional lymph nodes.
• Cystadenoma 8440/0
• Sialadenoma papilliferum  8406/0 Histopathology
• Ductal papillomas 8503/0 ♦♦ Tumor is encapsulated and consists of three types of cells:
• Sebaceous adenoma 8410/0 1. Mucus secreting cells
• Canalicular adenoma and other ductal 2. Epidermoid cells
adenomas 8149/0 3. Intermediate type of cells.
♦♦ Non-neoplastic epithelial lesions ♦♦ According to distribution of these cells the muco­
• Sclerosing polycystic adenosis epidermoid cells are divided into three grades:
• Nodular oncocytic hyperplasia 1. Well differentiated tumor or low-grade tumor
• Lymphoepithelial sialadenitis 2. Poorly differentiated tumor or high-grade tumor
• Intercalated duct hyperplasia 3. Intermediate-grade tumor
♦♦ Benign soft tissue lesions • Low-grade tumor: They show well formed glandu­
• Haemangioma 9120/0 lar structures and prominent mucin filled with
• Lipoma/sialolipoma 8850/0 cystic space, minimal cellular atypia and high
• Nodular fasciitis 8828/0 proportion of mucus cells.
♦♦ Haematolymphoid tumors • Intermediate grade tumor: They have solid areas
• Extranodal marginal zone lymphoma of of epidermoid cells or squamous cells with
mucosa-associated lymphoid tissue interme­diate basaloid cells. Cyst formation is
(MALT lymphoma) 9699/3
seen but is less prominent than observed in low-
*These new codes were approved by the IARC/WHO Committee for grade tumor.
ICD-0. • High-grade tumor: They consist of cells present as
solid nests and cords of intermediate basaloid cells
and epidermoid cells, prominent nuclear pleomo­
Mucoepidermal Tumor/Mucoepidermoid Carcinoma
rphism and mitotic activity is noted. Necrosis and
Mucoepidermal tumor is an unusual type of malignant salivary perineural invasion may be present.
gland neoplasm with varying degree of aggressiveness.

Pathogenesis
♦♦ It can occur due to entrapment of retromolar mucous
glands in the mandible which undergo malignant trans­
formation.
♦♦ Developmentally induced embryonic remnants of sub-
maxillary gland within the mandible.
♦♦ Mucous secreting cells which are commonly found in
pluripotential epithelial lining of dentigerous cyst as­
sociated with impacted third molars undergo neoplastic
transformation.
♦♦ Neoplastic transformation as well as invasion from lining
of maxillary sinus.
Fig. 57:  Mucoepidermoid tumor (For color version, see Plate 9)
Clinical Features
♦♦ Tumor occurs at the age of 30 to 50 years. Differential Diagnosis
♦♦ Tumor involves the parotid and minor salivary glands of ♦♦ Pleomorphic adenoma
palate, lips, buccal mucosa, tongue and retromolar areas. ♦♦ Squamous cell carcinoma
♦♦ Tumor occurs as slowly enlarging painless mass which ♦♦ Metastatic carcinoma
leads to the stimulation of pleomorphic adenoma. ♦♦ Adenocarcinoma.
544   Mastering the BDS IIIrd Year  (Last 25 Years Solved Questions)

Treatment
Conservative excision with the preservation of facial nerve.
The affected submandibular gland should be removed entirely.
Treatment for minor gland is surgical.
Q.5. Enumerate the disease of salivary gland and describe
in detail adenoid cystic carcinoma.
(Feb 2006, 6.5 Marks)
Or
Classify various diseases affecting salivary glands.
Describe in detail adenoid cystic carcinoma.
(Apr 2008, 15 Marks)
Or
Classify salivary gland tumors. Describe in detail
cylindroma. (Feb 2015, 10 Marks)
Ans. For enumeration of diseases, refer to Ans 4 of same
chapter. Fig. 58:  Adenoid cystic carcinoma (For color version, see Plate 10)

Adenoid Cystic Carcinoma/Cylindroma


Treatment
♦♦ It is a malignant neoplasm arising from glandular epithe­
lium of either major or minor glands. By wide surgical excision.
♦♦ It is also known as cylindroma. Q.6. Write note on Sjogren’s syndrome.
Clinical Features (Sep 2006, 5 Marks) (Dec 2007, 4 Marks)

♦♦ Tumor arises at the age of 50–70 years and is more com­ Or


mon in females. Write short note on Sjogren’s syndrome.
♦♦ It frequently affects parotid and other common sites are (Mar 2006, 5 Marks) (Aug 2012, 5 Marks)
minor glands of palate, tongue, lacrimal glands, breast,
Or
prostrate, etc.
♦♦ Lesion produces, slow enlarging growth with frequent Write in brief on Sjogren’s syndrome.
surface ulceration. (Jan 2012, 5 Marks)
♦♦ Pain is common feature and neurological signs are anes­ Ans. It is a chronic inflammatory autoimmune disorder that
thesia, paresthesia or palsy frequently develops. affects salivary, lacrimal and other exocrine gland.
♦♦ Often there is fixation of tumor to underlying structures
along with the local invasion. Types
♦♦ It has marked tendency to spread through perineural ♦♦ Primary Sjögren’s syndrome: It is also known as Sicca syn­
spaces and usually invades well beyond the clinically drome. It consists of dry eyes, i.e. xerophthalmia and dry
apparent borders. mouth, i.e. xerostomia.
Histopathology ♦♦ Secondary Sjögren’s syndrome: It consists of dry eyes, i.e.
xerophthalmia, dry mouth, i.e. xerostomia and collagen
♦♦ It is characterized by the presence of numerous, small, disorders, i.e. rheumatoid arthritis or systemic lupus
darkly stained polygonal or cuboidal cells. Cells often erythematous.
resemble basal cells of oral epithelium and have hyper­
chromatic nuclei and minimum mitotic activity. Clinical Features
♦♦ Double layer of tumor cells are arranged in duct like
pattern and contains eosinophilic coagulum at center, ♦♦ Xerostomia is present with unpleasant taste, soreness and
because of this lesion typically produces “Swiss Cheese difficulty in eating dry fruits.
appearance”. ♦♦ Patient also complains of xerophthalmia and arth­ralgia
♦♦ Stroma of connective tissue tumor is hyalinized which ♦♦ Severe tiredness is present.
surrounds tumor cells by forming structural pattern of ♦♦ There is cobble stone appearance of tongue.
many cylinders. ♦♦ There is often secondary acute bacterial sialadenitis and
♦♦ Most striking feature of adenoid cystic carcinoma is spread rapid progressive dental caries.
of tumor cells via perineural or intraneural spaces. This is ♦♦ Burning sensation present in the eyes.
known as neurotrophism. This accounts for high rates in ♦♦ Parotid gland is predominantly affected, sometimes sub­
the recurrence of tumors. mandibular and minor glands can also be affected.
Section 3: Oral Pathology  545

Histopathology Histopathological Features


♦♦ Initially infiltration of lymphocytes in intralobular ducts
of involved salivary gland.
♦♦ There is atrophy of salivary gland acini and proliferation
of ductal epithelial cells.
♦♦ Hyperplasia of ductal epithelium obliterates ductal lumen
and there is formation of myoepithelial islands.
♦♦ In fully-developed lesions entire glandular tissue is re­
placed by myoepithelial islands which are surrounded by
proliferating lymphoid tissue.

Treatment
♦♦ Use of systemic steroids
♦♦ Antibiotic eye drops
♦♦ Antifungal drugs
♦♦ Maintenance of oral hygiene.
Q.7. Write notes on mucoepidermoid carcinoma.
(Mar 2006, 2.5 Marks) (Mar 2007, 2.5 Marks) A.  Mucous retention cyst

Or
Write short note on mucoepidermoid carcinoma.
 (June 2015 5 Marks)
Ans. Refer to Ans 4 of same chapter.

Q.8. Describe the histopathology of mucoepidermoid car-


cinoma. (Sep 2006, 5 Marks) (Jan 2010, 5 Marks)
Ans. Refer to Ans 4 of same chapter.

Q.9. Write short note on mucocele. (Mar 2006, 5 Marks)


 (Apr 2007, 5 Marks) (Jan 2016, 5 Marks)
Ans. It is also called as mucus extravasation phenomenon or
mucus escape reaction.
Mucocele is defined as swelling caused by pooling of B.  Mucous extravasation cyst
saliva at the site of injured minor salivary gland. Figs 59A and B: Mucocele (For color version, see Plate 9)
Types ♦♦ In poorly defined cyst, it consists of irregularly shaped
poorly defined pools which contain eosinophilic mucinous
♦♦ Mucus extravasation cyst: In this mucus is extravasated
material, numerous vacuolated macro­phages which are
into the connective tissue and is devoid of epithelial lining.
known as mucinophage.
It is also known as pseudocyst.
♦♦ In well-defined cysts the periphery consists of granulation
♦♦ Mucus retention cyst: In this mucin is retained in the di­
tissue or condensed fibrous tissue or both and is infiltrated
lated salivary excretory duct and is lined by epithelium.
by vacuolated macrophages, lymphocytes, polymorpho
This is known as true cyst.
nuclear leukocytes including eosinophils.
♦♦ Lumen of cyst like cavity is filled with eosinophilic
Clinical Features coagulum.
♦♦ They are common and occur on the inner aspect of lower Treatment
lip. They may also occur on palate, cheek, tongue and
floor of the mouth. ♦♦ Complete excision of cyst under local anesthesia should
♦♦ They occur most frequently during third decade of life. be done.
♦♦ Patient complains of painless swelling. The swelling may Q.10. Classify salivary gland neoplasms. Write about clinical
suddenly develops at meal time and may drain simultane­ features and histopathology of benign pleomorphic
ously at interval. adenoma. (Sep 2008, 6 Marks)
♦♦ Swelling is round, oval or smooth. Ans. Refer to Ans 4 for classification of salivary gland neoplasm
♦♦ Superficial cyst appears as bluish mass and if inflammed and Ans 1 for clinical features and histopathology of
it is fluctuant, soft, nodular and dome shaped elevation. benign pleomorphic adenoma of same chapter.
546   Mastering the BDS IIIrd Year  (Last 25 Years Solved Questions)

Q.11. Write short note on clinical features and histopathology ♦♦ Low-grade fever and trismus may be present.
of cylindroma. (Sep 2011, 3 Marks) ♦♦ A purulent discharge is observed from the duct orifice.
Ans. Cylindroma is also known as adenoid cystic carcinoma.
Histopathology
For clinical features and histopathology refer to Ans 5
of same chapter. ♦♦ Accumulation of neutrophils is observed within the ductal
system and acini.
Q.12. Write short note on adenoid cystic carcinoma. ♦♦ Chronic sialadenitis is characterized by scattered or patchy
 (Mar 2016, 3 Marks) (Jan 2012, 5 Marks) infiltration of parenchyma by lymphocytes and plasma cells.
Ans. Refer to Ans 5 of same chapter. ♦♦ Atrophy of acini is common.
Q.13. Classify salivary gland tumor. Write briefly about Sialographic Features
etiopathogenesis, clinical features, histopathology and
radiographic findings of mucoepidermoid carcinoma. ♦♦ It demonstrates ductal dilatation proximal to the area of
(Sep 2011, 8 Marks) obstruction.
Or ♦♦ In acute sialedenitis sialography is contraindicated.
♦♦ In chronic sialedenitis stenson’s duct may show a
Write about clinical features, etiopathogenesis and
characteristic sialographic pattern known as sausaging
histopathology of mucoepidermoid carcinoma.
(Mar 2011, 5 Marks) which reflects a combination of dilatation plus ductal
strictures from scar formation.
Ans. For classification refer to Ans 4 of same chapter.
For clinical features and histopathology refer to Ans 4 Treatment
of same chapter. ♦♦ Antibiotics should be given to patient
Etiopathogenesis ♦♦ Proper hydration should be maintained
♦♦ If necessary go for surgical intervention.
♦♦ As per the multicellular theory mucoepidermoid tumor
arise from the excretory duct cells of salivary gland. Q.15. Classify salivary gland pathologies. Discuss in detail
♦♦ During development the retromolar mucous glands get warthin’s tumor. (Feb 2013, 10 Marks)
entrapped in the mandible which undergo neoplastic Or
transformation and leads to formation of intraosseous
Write in detail on Warthin’s tumor.
mucoepidermoid carcinoma.
(Jan 2010, 10 Marks)
For clinical features and histopathology refer to Ans 4 of same
chapter. Or
Write short note on Warthin’s tumor.
Radiographic Findings
 (Apr 2017, 5 Marks) (Feb 2013, 8 Marks)
♦♦ Mucoepidermoid carcinoma appear as a unilocular or Ans. For classification of salivary gland pathologies refer to
multilocular expanding mass. Ans 4 of same chapter.
♦♦ Margins are corticated, well defined.
♦♦ At times lesion show honeycomb or soap bubble Warthin’s Tumor
appearance.
♦♦ Lamina dura of associated teeth is lost. ♦♦ It is also known as papillary cystadenoma lympho­
♦♦ Inferior border of mandible, buccal and cortical plates are matosum or adenolymphoma.
displaced and thinned. ♦♦ It is the second most common tumor of salivary glands.

Q.14 Write short note on sialadenitis. (Sep 2011, 3 Marks) Pathogenesis


Ans. Inflammation of salivary gland is known as sialadenitis. ♦♦ According to the most accepted theory, the tumor arises
in salivary gland tissue entrapped within paraparotid or
Etiology intraparotid lymph nodes during embryogenesis.
♦♦ According to Allegra Warthin’s tumor is a delayed hypersen­
♦♦ Viral Infections: Mumps, CMV virus, Coxsackie virus and
parainfluenza. sitivity disease, the lymphocytes being an immune reaction to
♦♦ Bacterial Infections the salivary ducts which undergo oncocytic change.
♦♦ Blockage of duct by sialolithiasis Etiology
♦♦ Congenital strictures
♦♦ Compression by adjacent tumor ♦♦ Smoking
♦♦ Recent surgeries of salivary glands. ♦♦ Epstein Barr virus infection.

Clinical Features Clinical Features

♦♦ It occours most commonly in parotid gland. ♦♦ It occurs during 6th and 7th decades of life with average
♦♦ Affected gland is swollen and painful and overlying skin age of 62 years.
is warm and erythematous. ♦♦ Men are most commonly affected
Section 3: Oral Pathology  547

♦♦ Tumor is superficial lying beneath the parotid capsule or ♦♦ Adenoid cystic carcinoma is derived from neoplastic
protruding towards it. transformation of salivary acinar type cells and myoepi­
♦♦ The tumor appear as a slow growing painless nodular mass thelial cells.
over the angle of jaw. Lesion can be bilateral too. For clinical and histological features of adenoid cystic carcinoma
♦♦ Tumor is 1 to 3 cm in diameter and is spherical in shape. refer to Ans 5 of same chapter.
Surface of the lesion is smooth
Q.18. Enumerate benign and malignant tumors of salivary
♦♦ On palpation lesion is firm and is non-tender.
gland. Discuss in detail adenoid cystic carcinoma
Histopathology (ACC). (June 2014, 10 Marks)
♦♦ Tumor is composed of epithelial and lymphoid tissue. Ans. For benign and malignant tumors of salivary gland refer
♦♦ Lesion is an adenoma undergoing cyst formation with to Ans 4 of same chapter.
papillary projections in cystic spaces. For adenoid cystic carcinoma refer to Ans 5 and Ans
♦♦ Lymphoid matrix exhibit germinal centers. 17 of same chapter.
♦♦ Cyst is lined by a bilayered oncocytic epithelium, the in­ Q.19. Enumerate diseases of salivary gland. Write histopa-
ner cells of which are tall columnar with fine granular and thology and etiology of mucoepidermoid carcinoma.
eosinophilic cytoplasm and slightly hyperchromatic nuclei.
 (Nov 2014, 8 Marks)
The outer layer consist of basaloid cells.
Ans. For enumeration of diseases of salivary gland and
♦♦ An eosinophilic coagulum is present within the cystic spaces.
histopathology of mucoepidermoid carcinoma refer to
♦♦ The numerous lymphocytic component may represent
Ans 4 of same chapter.
normal lymphoid tissue within which tumor is developed.
Etiology of Mucoepidermoid Carcinoma
Mucoepidermoid carcinoma is most commonly occurs due to
radiation.
Q.20. Write short note on sialolithiasis. (Apr 2015, 3 Marks)
Ans. Sialolithiasis is also known as salivary duct calculi or
salivary duct stone.
Sialolith are the calcified structures which develop within the
salivary ductal system.

Pathogenesis
Salivary duct stone are formed by deposition of calcium salts
around a central nidus which consists of altered salivary
Fig. 60:  Warthin’s tumor (For color version, see Plate 10) mucins, desquamated epithelial cells, bacteria, foreign bodies
or products of bacterial decomposition.
Treatment
Surgical excision is treatment of choice. Clinical Features

Q.16. Write short note on Warthin’s tumor. ♦♦ Sialoliths occur during 2nd to 4th decades of life.
(Dec 2015, 3 Marks)(Nov 2008, 5 Marks) ♦♦ Submandibular gland ductal system is the most common
site for occurrence of sialolith.
Or ♦♦ Sialoliths cause episodic pain or swelling of the affected
Describe in brief Warthin’s tumor. gland during meal or thought of the meal.
(May/Jun 2009, 5 Marks) (Feb 2014, 3 Marks) ♦♦ If it is not treated for longer time exacerbation of the le­
Ans. Refer to Ans 15 of same chapter. sion occur and systemic symptoms occur such as malaise
and fever.
Q.17. Classify salivary gland tumors and write in detail etiol- ♦♦ Pus can exude from the ductal orifice and surrounding
ogy, clinical features and histopathological features of soft tissues are inflamed.
adenoid cystic carcinoma. (Dec 2012, 8 Marks) ♦♦ At times ulceration of the overlying mucosa is evident
Ans. For classification refer to Ans 4 of same chapter. which allows calculus to extend in floor of oral cavity.
♦♦ Saliva can never be seen oozing out from the orifice of the
Etiology affected duct.
Histopathology
♦♦ Some evidence supports an association with mutation on
chromosome 6 and 12 and deletion of genetic material ♦♦ Calcified mass exhibit concentric lamellation which sur­
from chromosome 19. rounds the nidus of amorphous deposits.
548   Mastering the BDS IIIrd Year  (Last 25 Years Solved Questions)

♦♦ Associated ductal epithelium shows squamous, oncocytic Classification of ameloblastoma WHO 2017
or mucus cell metaplasia. ♦♦ Ameloblastoma
♦♦ Periductal inflammation is also present. • Ameloblastoma, unicystic type
♦♦ Ductal obstruction is frequently associated with acute or • Ameloblastoma, extraosseous/peripheral type
chronic sialadenitis of feeding gland. • Metastasizing ameloblastoma
Treatment For ameloblastoma in detail refer to ans1 of same chapter in
detail.
♦♦ Small calculi are removed by manipulation or increasing
the salivation.
♦♦ Larger calculi require surgical exposure for removal.
Q.21. Classify salivary gland tumors. Describe in detail about
4. Odontogenic tumors
pleomorphic adenoma. (Jan 2016, 10 Marks)
Q.1. Classify odontogenic tumors. Describe patho­genesis,
Ans. For classification of salivary gland tumor refer to Ans 4 histopathology, clinical features and radiographic ap-
of same chapter. pearance of amelo­blastoma.
For pleomorphic adenoma in detail refer to Ans 1 of (Feb 1999, 15 Marks)
same chapter.
Or
Q.22. List benign epithelial tumors of salivary gland. Discuss Classify odontogenic tumors describe in detail amelo-
in detail pleomorphic adenoma. (May 2018, 5 Marks) blastoma of mandible. (Nov 2008, 15 Marks)
Ans. Salivary Gland Tumor Classification by WHO (2017)
Or
Benign tumors Describe in detail ameloblastoma.
♦♦ Pleomorphic adenoma 8940/0 (Jan 2010, 10 Marks)
♦♦ Myoepithelioma 8982/0 Or
♦♦ Basal cell adenoma 8147/0 Describe in detail about clinical, radiographic, histo-
♦♦ Warthin tumor 8561/0 pathological features of ameloblastoma.
♦♦ Oncocytoma 8290/0 (Jan 2012, 15 Marks)
♦♦ Lymphadenoma 8563/0* Or
♦♦ Cystadenoma 8440/0
Classify odontogenic tumors and write in detail about
♦♦ Sialadenoma papilliferum 8406/0
ameloblastoma. (Jan 2016, 10 Marks)
♦♦ Ductal papillomas 8503/0
(Aug 2011, 15 Marks) (Dec 2012, 8 Marks)
♦♦ Sebaceous adenoma 8410/0
♦♦ Canalicular adenoma and other ductal adenomas 8149/0 Or
• Non-neoplastic epithelial lesions Classify odontogenic tumor and describe the clinical
♦♦ Sclerosing polycystic adenosis features, radiographic features and histological fea-
♦♦ Nodular oncocytic hyperplasia tures of ameloblastoma. (Feb 2013, 10 Marks)
♦♦ Lymphoepithelial sialadenitis Or
♦♦ Intercalated duct hyperplasia
• Benign soft tissue lesions Classify odontogenic tumor. Describe in detail the
♦♦ Haemangioma 9120/0 clinical features, histopathology and treatment of
♦♦ Lipoma/sialolipoma 8850/0 ameloblastoma. (Apr 2017, 10 Marks)
♦♦ Nodular fasciitis 8828/0 Ans.
• Haematolymphoid tumors Classification of Odontogenic Tumors by WHO (2017)
♦♦ Extranodal marginal zone lymphoma of mucosa-associat­ ♦♦ Odontogenic carcinomas
ed lymphoid tissue (MALT lymphoma) 9699/3 • Ameloblastic carcinoma
*These new codes were approved by the IARC/WHO • Primary intraosseous carcinoma, NOS
Committee for ICD-0. • Sclerosing odontogenic carcinoma
For pleomorphic adenoma in detail refer to ans1 of same • Clear cell odontogenic carcinoma
chapter. • Ghost cell odontogenic carcinoma
♦♦ Odontogenic carcinosarcoma
Q.14. Classify and discuss ameloblastoma. ♦♦ Odontogenic sarcomas
♦♦ Benign epithelial odontogenic tumors
 (May 2018, 5 Marks)
• Ameloblastoma
Ans. Ameloblastoma is classified, according to WHO and
the International Agency for Research on Cancer, 2017, –– Ameloblastoma, unicystic type
as a benign epithelial odontogenic tumor. It is further –– Ameloblastoma, extraosseous/peripheral type
classified as: –– Metastasizing ameloblastoma
Section 3: Oral Pathology  549

• Squamous odontogenic tumor ♦♦ Neoplastic odontogenic epithelial cells proliferate in form


• Calcifying epithelial odontogenic tumor of multiple discrete follicles and islands within fibrous
• Adenomatoid odontogenic tumor connective tissue stroma.
♦♦ Benign mixed epithelial & mesenchymal odontogenic ♦♦ Each follicle like structure is bordered on the periphery by
tumors the single layer of tall columnar cells resembling amelo­
• Ameloblastic fibroma blasts like cells.
• Primordial odontogenic tumor ♦♦ Cells located at the center of follicle are loosely arranged
• Odontoma and resemble stellate reticulum cells.
–– Odontoma, compound type ♦♦ Microcyst formation is often observed inside these follicles.
–– Odontoma, complex type
Plexiform Type
• Dentinogenic ghost cell tumor
♦♦ Benign mesenchymal odontogenic tumors ♦♦ Peripheral layer of cells are tall columnar in nature and
• Odontogenic fibroma often resemble ameloblast like cells.
• Odontogenic myxoma/myxofibroma ♦♦ Cells situated at center portion of strands resemble stel­
• Cementoblastoma late reticulum.
• Cemento-ossifying fibroma ♦♦ Intervening connective tissue stroma is thin with minimum
cellularity and shows multiple areas of cystification which
AMELOBLASTOMA may be either large or small in number.
Pathogenesis of Ameloblastoma
The tumor may derive from the:
♦♦ Cell rest of enamel organ, either remnants of dental lamina
or remnants of Hertwig’s Sheath, the epithelial rest cells
of Malassez.
♦♦ Epithelium of odontogenic cyst.
♦♦ Disturbance of the developing enamel organ
♦♦ Basal cell of the surface epithelium of the jaw.

Histopathology
Histologically, the ameloblastoma shows neoplastic proliferation
of odontogenic epithelial cells mostly in four distinct patterns:
♦♦ Follicular type Fig. 62:  Plexiform ameloblastoma (For color version see Plate 10)
♦♦ Plexiform type
Acanthomatous Type
♦♦ Acanthomatous type
♦♦ Granular type ♦♦ Cells occupying the position of the stellate reticulum
undergo squamous metaplasia, sometimes with keratin
Follicular Type formation in the anterior portion of tumor islands.
♦♦ Occasionally, epithelial or keratin pearls may be observed.
♦♦ Areas of calcification may be found in the metaplastic
squamous epithelium.
♦♦ It may be confused with squamous cell carcinoma.

Fig. 61:  Follicular ameloblastoma (For color version, see Plate 10) Fig. 63:  Acanthomatous ameloblastoma (For color version, see Plate 10)
550   Mastering the BDS IIIrd Year  (Last 25 Years Solved Questions)

Granular Type ♦♦ Some surgeons adopt conservative approach by planning


surgery after careful CT scan evaluation. Removal of tumor
♦♦ There is marked transformation of the cytoplasm, usually
is done followed by peripheral ostectomy.
of the stellate reticulum like cells that it takes a very coarse
♦♦ Other surgeons advocate that margin of resection should
granular eosinophilic appearance.
be 1 to 1.5 cm past radiographic limits of tumor.
Q.2. Describe clinical features, histopathology and inves-
tigations associated with ameloblastoma.
(Mar 2003, 15 Marks)
Ans. For clinical features and histopathology, refer to Ans 1
of same chapter.
For investigations refer to radiographical features of Ans
1 of same chapter.
Q.3. Describe histopathology of ameloblastoma.
(Sep 2006, 5 Marks)
Or
Write short note on histological features of ameloblas-
Fig. 64:  Granular cell ameloblastoma (For color version, see Plate 10)
toma. (Sep 2005, 5 Marks)
For more histopathology refer to Ans 9 of same chapter. Ans. Refer to Ans 1 and Ans 9 of same chapter.
Clinical Features Q.4. Classify the odontogenic tumor and describe the etiol-
♦♦ Ameloblastoma occurs in 2nd, 3rd, 4th and 5th decade ogy, clinical features and histopathology of ameloblas-
of life. toma. (Sep 2002, 15 Marks)
♦♦ Mean age of occurrence is 32 years. Ans. Refer to Ans 1 of same chapter.
♦♦ Males are affected more commonly than females. Q.5. Write short note on adenomatoid odontogenic tumor.
♦♦ Ameloblastoma in most of the cases involves mandible in (Aug 2012, 5 Marks) (April 2007, 5 Marks)
molar ramus area.
Ans. Adenomatoid Odontogenic Tumor
♦♦ Clinically ameloblastoma presents slow enlarging, pain­
It is also called adenoameloblastoma or ameloblastic
less, ovoid and fusiform bony hard swelling of the jaw.
adenomatoid tumor.
♦♦ Pain, paresthesia and mobility of regional teeth is present
in some cases. Adenomatoid odontogenic tumor is uncommon, well
circumscribed, odontogenic neoplasm characterized
♦♦ Pathological fractures may occur in many affected bones.
by the formation of multiple duct like structures by
Radiographical Appearance neoplastic epithelial cells.

♦♦ During the early stage area of bone destruction is well Clinical Features
defined and have hyperostotic borders.
♦♦ Tumor usually occurs in younger age.
♦♦ Outline of margins of lesion is smooth, well defined, scal­
♦♦ Females are more commonly affected
loped and is corticated.
♦♦ Lesion most typically appears in maxillary anterior
♦♦ Ameloblastomas are mostly multilocular but at times they region.
are unilocular too. ♦♦ Tumor presents a slow enlarging, small, bony hard swell­
♦♦ It reveals honeycomb appearance because of arrangement ing in maxillary anterior region.
of septae. It also reveal soap bubble appearance if larger ♦♦ Sometimes, it occurs in premolar region
compartments are present. ♦♦ There is displacement of regional teeth, mild pain and
♦♦ Presence of bony expansion and thinning of cortical plates expansion of cortical bones.
which leaves thin egg shell of bone which is known as egg
shell crackling.
Histopathology
♦♦ Extensive root resorption is seen in lesional area.
♦♦ Microscopically, adenoid odontogenic tumor reveals
Treatment neoplastic odontogenic epithelial cells, proliferating in
multiple “duct like” patterns.
♦♦ Patients with conventional solid or multicystic intra- ♦♦ Presence of these duct like structures often give glandular
osseous ameloblastoma are treated by simple enucleation lesion.
to curettage to en bloc resection. ♦♦ Each duct like structure is bordered on periphery
♦♦ Marginal resection is the most widely used treatment. But by a single layer of tall columnar cells resembles
recurrence rate up to 15% are evident. ameloblastoma.
Section 3: Oral Pathology  551

♦♦ Lumen of duct like structures are filled with the Clinical Features
homogenous eosinophilic coagulum. ♦♦ Tumor occur in middle age persons.
♦♦ Small foci of calcifications are often seen, which are ♦♦ Mandible is involved more often than maxilla.
scattered throughout the lesion. ♦♦ Molar region is more common site of occurrence followed
♦♦ Droplets of amorphous (PAS positive) eosinophilic by premolar region.
material are found between neoplastic cells. ♦♦ Tumor presents a slow enlarging, painless swelling of jaw
with expansion and distortion of cortical plates.
♦♦ Swelling is bony hard and clinically, it is well defined or
diffused.
♦♦ Pain, paresthesia may develop on rare occasions and few
lesions may be completely asymptomatic.
Histopathology
♦♦ Tumor reveals sheet of closely packed, polyhedral cells in
noninflamed connective tissue stroma.
♦♦ Tumor cells contain oval-shaped nuclei and homo­genous
eosinophilic cytoplasm.
♦♦ Prominent intra cellular bridges and distinct cell bounda­
ries are often found in the lesions.
♦♦ Some amount of homogenous, hyaline material is often
deposited in between tumor cells called amyloid material.
♦♦ One of the most important histological characteristics of
CEOT is the presence of several calcified masses in and
Fig. 65:  Adenomatoid odontogenic tumor around the tumor cells.
(For color version, see Plate 11)
♦♦ Some Liesegang rings are also found.

Differential Diagnosis

♦♦ Dentigerous cyst
♦♦ Odontomes
♦♦ Unicystic ameloblastoma
♦♦ CEOT
♦♦ CEOC.

Treatment
The treatment is surgical enucleation.
Q.6. Write short note on Pindborg tumor.
(Feb 2015, 5 Marks) (Sep 1999, 5 Marks)
Or
Describe in brief Pindborg tumor. Fig. 66:  Pindborg’s tumor (For color version, see plate 11)
(May/June 2009, 5 Marks) Treatment
Or
Surgical enucleation is done.
Write short note on CEOT. (Dec 2015, 3 Marks)
Ans. Pindborg tumor is locally aggressive neoplasm, which Q.7. Write note on odontoma. (Aug/Sep 1998, 5 Marks)
is also known as calcifying epithelial odontogenic (Sep 2004, 5 Marks) (Feb 2002, 6 Marks)
tumor.
(June 2015, 5 Marks) (Mar 2013, 3 Marks)
Pathogenesis Or
♦♦ Some investigators suggest that the Pindborg tumor arises Write short note on odontome. (Jun 2014, 5 Marks)
from remnant of cells in stratum intermedium layer of the Ans. Odontomas are hamartoma that contain both epithelial
enamel organ in tooth development. Some hypo­thesize and mesenchymal dental tissue components.
that the Pindborg’s tumor arises from the remnants of the • Generally there are two types of odontoma:
primitive dental lamina. 1. Complex odontoma
♦♦ Definite etiology of neoplasm still remains enigmatic. 2. Compound odontoma.
552   Mastering the BDS IIIrd Year  (Last 25 Years Solved Questions)

Complex Odontoma
It is always benign and contains enamel, dentin and cementum
which are not differentiated, so that structure of actual tooth
is not identifiable.

Compound Odontoma
It is also benign. In compound odontomas, the enamel and
dentin are laid down in such a fashion that the structure bears
considerable anatomic resemblance to normal teeth, except that
they are often smaller than typical teeth.

Clinical Features
♦♦ Lesion occurs among children or young adults.
Fig. 68:  Compound composite odontome
♦♦ Both sexes are equally affected or slight male predomi­
(For color version, see Plate 11)
nance is present.
♦♦ Maxilla is commonly affected. Odontomas are commonly Treatment
seen in pericoronal area of permanent teeth.
♦♦ Odontoma produces large, bony, hard swellings of jaw, Surgical Enucleation.
with expansion of cortical plates and displacement of Q.8. Draw a well-labeled histopathological diagram of
regional teeth. plexiform ameloblastoma. (Dec 2007, 3 Marks)
♦♦ If odontoma is located high in alveolus, they may tend to Ans. Refer to Ans 1 of same chapter.
erupt in oral cavity by resorbing overlying bone and as a
result there may be pain, inflammation, ulceration, etc. Q.9. Write short note on histopathological variants of
ameloblastoma. (Sep 2011, 3 Marks)
Histopathology
Or
Write in detail on histopathological types/variants of
ameloblastoma. (April 2008, 10 Marks)
Ans. Following are the histopathological variants of
ameloblastoma
• Follicular type: Refer to Ans 1 of same chapter
• Plexiform type: Refer to Ans 1 of same chapter
• Acanthomatous Type: Refer to Ans 1 of same chapter
• Granular cell type: Refer to Ans 1 of same chapter
• Desmoplastic type: There is presence of marked hyali­
nization of connective tissue stroma.
• Basal cell type: Tumor islands show basaloid pattern
of cells.
• Clear cell type: Solid multicystic ameloblastoma may
Fig. 67:  Complex composite odontome contain clear cells which are localized to the stellate
(For color version, see Plate 11)
reti­culum like areas of follicular ameloblastoma.
♦♦ Fully developed compound odontoma reveals the presence
of encapsulated mass of separate denticles, embedded in • Keratoameloblastoma and papilliferous kerato­
fibrous tissue stroma. ameloblastoma: Ameloblastoma consisting partly of
♦♦ The fully developed complex odontoma reveals an irregu­ keratinizing cysts and partly of tumor islands with
larly arranged but well-formed mass of enamel, dentin and papilliferous arrangement
cementum which is surrounded by fibrous tissue capsule. • Mucous cell differentiation type: Follicular ameloblas­
♦♦ The dentinal tissues lie in the direct contact with connective toma showing focal mucous cell differentiation.
tissue that resembles the dental pulp. • Hemangiomatous ameloblastoma: It is an ameloblas­
♦♦ Most of enamel tissues are fully calcified and appear as toma in which part of the tumor contain spaces filled
small empty space. with blood or large endothelial lined capillaries.
Section 3: Oral Pathology  553

Q.10. Classify odontogenic tumors. Describe in detail ♦♦ Margins of the lesion are scalloped and are well defined.
etiopathogenesis, clinical features, radiology, his- ♦♦ Tumor can displace the developing tooth.
topathologic features and differential diagnosis of ♦♦ Cortical plate expansion is also appreciated bucally, lin­
adenomatoid odontogenic tumor. (Jan 2012, 10 Marks) gually and in vertical dimension.
Or Q.13. Classify odontogenic tumors and describe in detail
Give classification of odontogenic tumors. Discuss Pindborg tumor. (Feb 2013, 18 Marks)
about clinical features and histopathology of AOT. Ans. For classification, refer to Ans 1 of same chapter.
 (Nov 2014, 8 Marks) For Pindborg tumor in detail, refer to Ans 6 and Ans 12 of same
Or chapter.
Classify odontogenic tumors. Write in detail about ad-
enomatoid odontogenic tumor.  (July 2016, 10 Marks)
Ans. For classification, refer to Ans 1 of same chapter. 5. Cysts of Oral Cavity
Etiopathogenesis
Q.1. Classify cysts of oral cavity. Describe patho­genesis,
♦♦ Since adenomatoid odontogenic tumor occurs within histopathology and clinical features and roentge­no­
tooth-bearing areas of ja, it arises from reduced enamel graphic appearance of dentigerous cyst.
epithelium during the presecretory phase of enamel organ (Mar 1998, 15 Marks)
development.
♦♦ Some category of authors also believes that it arises from Or
dental lamina or from preexisting dentigerous cyst.
Define and classify cyst. Write in detail about dentiger-
ous cyst.
Radiology
 (Mar 2006, 15 Marks) (Feb 2013, 18 Marks)
♦♦ Radiographically tumor presents a well-circumscribed
Or
unilocular radiolucent area which consisted of impacted
tooth or odontome and exhibits a smooth corticated border. Classify odontogenic cyst. Describe patho­genesis,
♦♦ Multilocular radiolucencies are also seen rarely with scal­ clinical radiological and histopathologic features of
loped borders. dentigerous cyst. (June 2015, 10 Marks)
♦♦ At times radio-opaque foci is detectable within radiolucent Or
lesion. Classify odontogenic cysts and describe clinical fea-
♦♦ As lesion progresses divergence of roots and displacement tures, radiographic and histopathologic features of
of teeth is seen.
dentigerous cyst. (Feb 2015, 10 Marks)
For clinical features, histopathology and differential diagnosis
refer to Ans 5 of same chapter. Or
Classify odontogenic cyst. Describe patho­genesis, clini-
Q.11. Write short note on histopathology of adenomatoid
cal, radiological features along with histopathology of
odontogenic tumor. (Aug 2012, 5 Marks)
dentigerous cyst. (Jan 2018, 10 Marks)
Ans. Refer to Ans 5 of same chapter.
Ans. Cyst is defined as “A pathological cavity having fluid,
Q.12. Classify odontogenic tumors. Write in detail about semifluid or gaseous contents and which is not created
the clinical, radiological and histological features of by accumulation of pus.” —Kramer (1974)
calcifying epithelial odontogenic tumor. Classification of Cyst of Oral Cavity by Mervin Shear
 (Mar 2006, 15 Marks)
Ans. For classification of odontogenic tumor refer to Ans 1 of I. Cysts of the Jaws
same chapter.
♦♦ Epithelial:
For clinical and histological features of calcifying epithelial • Developmental:
odontogenic tumor refer to Ans 6 of same chapter. –– Odontogenic:
- Gingival cyst of infants
Radiological Features
- Odontogenic keratocyst (Neoplasm)
♦♦ Radiographically tumor consists of either a unilocular or - Dentigerous cyst
multilocular radiolucent defect with radiolucent to ra­ - Eruption cyst
diopaque area surrounding the crown of impacted tooth. - Lateral periodontal cyst
♦♦ Radiopacity is of proteinaceous material released by the - Gingival cyst of adults
tumor cells. The radiopaque structures are of varying size - Botryoid odontogenic cyst
and density and are scattered all over which provides the - Glandular odontogenic cyst
lesion a driven snow appearance - Calcifying odontogenic cyst (neoplasm)
554   Mastering the BDS IIIrd Year  (Last 25 Years Solved Questions)

– Non-odontogenic: Pathogenesis
- Naso palatine duct cyst
♦♦ Intrafollicular theory: Dentigerous cyst is caused by fluid
- Naso labial cyst
accumulation between reduced enamel epithelium and
- Midpalatal raphe cyst of infants
enamel surface which result in a cyst in which crown is
- Median palatine, median alveolar
located within the lumen.
- Median mandibular cyst
♦♦ Extrafollicular theory: Dentigerous cyst may arise by
- Globulo maxillary cyst
proliferation and cystic transformation of islands by
• Inflammatory
odontogenic epithelium in connective tissue wall of dental
–– Radicular cyst, apical and lateral
follicle or even outside dental follicle and this transformed
–– Residual cyst
epithelium then unite with lining follicular epithelium
–– Paradental cyst and mandibular infected buccal
forming cystic cavity around tooth crown.
cyst
–– Inflammatory collateral cyst Clinical Features
♦♦ Non epithelial (pseudo cysts)
♦♦ It is usually found in the children.
• Solitary bone cyst
♦♦ Most lesions are present in the 2nd and 3rd decades with
• Aneurysmal bone cyst
male predilection.
II. Cyst Associated With Maxillary Antrum ♦♦ Most common site of the cyst are the mandibular and
♦♦ Benign mucosal cyst of the maxillary antrum maxillary third molar and maxillary cuspid areas, since
♦♦ Post operative maxillary cyst these are most commonly impacted teeth.
♦♦ Generally, it is painless but may be painful if it is infected.
III. Cyst of the Soft Tissues Of Mouth, Face And Neck ♦♦ Dentigerous cyst has potential to become an aggressive
♦♦ Dermoid and epidermoid cyst lesion with expansion of bone and subsequent facial
♦♦ Lymphoepithelial cyst (branchial cyst) asymmetry.
♦♦ Thyroglossal duct cyst ♦♦ There is extreme displacement of teeth, severe root resorp­
♦♦ Anterior medial lingual cyst (intra lingual cyst of foregut tion of adjacent teeth and pain.
origin) Histopathological Features
♦♦ Oral cyst with gastric or intestinal epithelium
♦♦ Cystic hygroma
♦♦ Nasopharyngeal cyst
♦♦ Thymic cyst
♦♦ Cyst of salivary glands- mucous extravasation cyst
♦♦ Mucous retention cyst, ranula, polycystic disease of the
parotid.
♦♦ Parasitic cyst - hydatid cyst, cysticercus cellulosae, trichi­
nosis.

Classification of Odontogenic Cyst


WHO (2017) Classification of Odontogenic Cysts
♦♦ Odontogenic cysts of inflammatory origin Fig. 69:  Dentigerous cyst (For color version, see Plate 11)
• Radicular cyst ♦♦ The epithelial lining consists of two to four layers of flat­
• Inflammatory collateral cysts
tened non – keratinizing cells and the epithelium and
♦♦ Odontogenic developmental cysts
connective tissue interface is flat.
• Dentigerous cyst
♦♦ It is usually composed of thin connective tissue wall with a
• Odontogenic keratocyst
thin layer of stratified squamous epithelium lining the lumen.
• Lateral periodontal cyst and botryoid odontogenic cyst
♦♦ Rete pegs formation is absent except in case of secondarily
• Gingival cysts
infected cyst.
• Glandular odontogenic cyst
• Calcifying odontogenic cyst ♦♦ Connective tissue wall is frequently quite thickened and
• Orthokeratinized odontogenic cyst. composed of very loose fibrous connective tissue.
♦♦ Inflammatory cells commonly infiltrate connective tissue
Dentigerous Cyst ♦♦ It also shows Rushton bodies within the lining epithelium
♦♦ It is also called as follicular cyst or pericoronal cyst which are peculiar linear and often curved hyaline bodies.
♦♦ It is the odontogenic cyst that surrounds the crown of the ♦♦ Content of cystic lumen is usually thin watery yellow fluid
impacted tooth. and is occasionally blood tinged.
Section 3: Oral Pathology  555

Roentgenographic/Radiological Features Or
♦♦ There is presence of well-defined radiolucency having Write in detail on odontogenic keratocyst.
hyperostotic borders. (June 2010, 10 Marks)
♦♦ An unerupted tooth is also seen around the radiolucency Or
♦♦ Cyst is unilocular but at times it appears multilocular. Classify odontogenic cysts of oral cavity describe clini-
♦♦ Bony margins of the cyst are well defined as well as sharp. cal features and histopathology of keratocyst.
If infection persists margins are ill defined. (Dec 2010, 18 Marks)
♦♦ Cyst can envelop the crown symmetrically, but it can Or
expand laterally from the crown of tooth. Tooth can also Classify odontogenic cysts and describe the clincial and
be displaced away in any direction. histopathological features of odontogenic keratocyst
♦♦ Resorption of roots of adjacent teeth can also be seen. (OKC) in detail. (June 2014, 10 Marks)
♦♦ Floor of maxillary sinus gets displaced with the expan­ Or
sion of cyst. Classify odontogenic cysts. Describe pathogenesis,
clinical, radiological and histopathological features
Q.2. Classify cysts of jaw. Describe pathogenesis, histo­ of odontogenic keratocyst. (Jan 2016, 10 Marks)
pathology and clinical features and malignant potential Or
of dentigerous cyst. (Mar 2000, 15 Marks) Classify odontogenic cysts. Discuss clinical features,
Ans. Refer to Ans 1 of same chapter. radiological features and histopathology of odonto-
genic keratocyst. (Mar 2016, 8 Marks)
Malignant Potential of Dentigerous Cyst
Ans. For classification refer to Ans 1 of same chapter.
♦♦ Malignant potential of the epithelium of dentigerous cysts
to ameloblastoma, epidermoid carcinoma and mucoepi­ Odontogenic Keratocyst
dermoid carcinoma. Odontogenic keratocyst is a common cystic lesion of the jaw,
♦♦ Development of an ameloblastoma is from rest of odonto­ which arises from the remnants of dental lamina.
genic epithelium or from lining epithelium of cyst is known ♦♦ It is named as keratocyst because the cyst epithelium pro­
as mural ameloblastoma. duces so much keratin that it fills the cyst lumen.
♦♦ Mucoepidermoid carcinoma is the malignancy of sali­ ♦♦ Odontogenic cysts have more aggressive course than any
vary gland. The mucoepidermoid carcinoma is usually other cystic lesion of jaw and for this reason these are
associated with lining epithelium of dentigerous cyst or sometimes known as benign cystic neoplasms.
dentigerous cyst which consist of mucous secreting cell.
♦♦ Epidermoid carcinoma can also develop from dentiger­ Recent concept of Odontogenic Keratocyst
ous cyst from rest of odontogenic epithelium or lining of ♦♦ Keratocystic odontogenic tumor is now listed as ‘odon­
epithelium of cyst. togenic keratocyst (OKC)’ in the 2017 classification of
Q.3. Write notes on dentigerous cyst. (Feb 2006, 3 Marks) developmental odontogenic cysts.
♦♦ WHO 2005 classification reclassified this unique lesion as
Or a neoplasm and renamed it as ‘keratocystic odontogenic
Write short note on dentigerous cyst. tumor’ because of the high recurrence rate, aggressive clini­
cal behavior, association with nevoid basal cell carcinoma
(Aug 2012, 5 Marks) (Apr 2008, 5 Marks)
syndrome, and mutations in the PTCH tumor suppressor
Or gene. The WHO 2017 classification reverted back to the
original and well accepted terminology of ‘odontogenic
Write short essay on dentigerous cyst.
keratocyst’ because many papers showed that the PTCH
(Jan 2012, 5 Marks) gene mutation could be found in non-neoplastic lesions,
Ans. Refer to Ans 1 of same chapter. including dentigerous cysts. It has also been reported that
Q.4. Classify cysts of jaw. Describe pathogenesis, histo­ marsupialisation is an effective treatment for the odonto­
pathology and clinical features of odontogenic kerato- genic keratocyst and may be associated with reversion of
cyst. (Feb 2002, 16 Marks) the epithelium to normal, and with lower recurrence rates,
Or these features are not normally associated with neoplasia.
So after considering all the available data, the WHO con­
Describe the pathogenesis, histopathology and clinical sensus group concluded that further research is needed,
features of odontogenic keratocyst. but at the present time, there was insufficient evidence to
(Aug/Sep 1998, 15 Marks) support a neoplastic origin of the odontogenic keratocyst. It
Or was decided therefore that odontogenic keratocyst remains
Classify odontogenic cysts of oral cavity. Write about the most appropriate name for this lesion, and keratocystic
clinical features and histopathology of odontogenic odontogenic tumor was removed from the WHO 2017
keratocyst. (Feb 2013, 10 Marks) classification of odontogenic cysts.
556   Mastering the BDS IIIrd Year  (Last 25 Years Solved Questions)

Pathogenesis Radiological Features


Odontogenic keratocyst mainly arises from the: ♦♦ Odontogenic keratocyst is oval in shape and it extends to
♦♦ Dental lamina or its remnants. the body of mandible with mediolateral expansion.
♦♦ Primordium of developing tooth germ or enamel organ. ♦♦ It is very small in size or it can exceed the diameter of 5 cm.
♦♦ Sometimes from basal cell layer of oral epithelium. ♦♦ Margins of the cyst are hyperostotic.
♦♦ Mostly odontogenic keratocyst is unilocular and have
Histopathological Features smooth borders while some of the cysts show irregular
♦♦ A parakeratin surface which is usually corrugated rippled borders too.
or wrinkled. ♦♦ Radiolucency is seen in the cystic part which appears to
♦♦ Uniformity of thickness of epithelium is generally between be hazy if keratin is present in the cavity.
6 to 10 cells in depth. ♦♦ Radiolucency is surrounded by thin sclerotic rim.
♦♦ Prominent palisaded, polarized basal cell layer often ♦♦ In some of the cases perforation of lingual and buccal
described as having a “picket fence” or “tombstone” cortical plates is seen.
appea­rance. ♦♦ Displacement of inferior alveolar canal is seen downwards.
♦♦ Occasionally orthokeratin is found but if present, par­ Q.5. Classify cysts of oral region. Discuss radicular cyst in
akeratin is evident.
detail. (Sep 2005, 16 Marks)
♦♦ Connective tissue shows “Daughter cells” or “Satellite cysts”.
♦♦ Lumen of keratocyst may be filled with thin straw colored Or
fluid or with thick creamy material. Write note on radicular cyst.
♦♦ Sometimes a lumen contains a great deal of keratin while  (Mar 2001, 5 Marks) (Feb 2013, 5 Marks)
at other times it has little cholesterol as well as hyaline
bodies at the site of inflammation. Or
Write short note on radicular cyst. (Aug 2012, 5 Marks)
 (Apr 2017, 5 Marks)
Or

Classify odontogenic cyst. Describe the pathogenesis,


clinical, radiological and histopathogical features of
radicular cyst. (Jan 2017, 10 Marks)
Ans. For classification refer to Ans 1 of same chapter.

Radicular Cyst
♦♦ It is also called as apical periodontal cyst or periapical cyst
or dental root end cyst.
Fig. 70:  Odontogenic keratocyst (For color version, see Plate 11) ♦♦ It is an inflammatory odontogenic epithelial cyst.
♦♦ It is a common sequelae in progressive changes associated
Clinical Features with bacterial invasion and death of the dental pulp.
♦♦ It most commonly occurs at the apices of the teeth.
♦♦ Peak incidence is between 2nd and 3rd decades of life.
♦♦ It is found more frequently in males as compared to Pathogenesis of Radicular Cyst
females. Radicular cyst develops due to the proliferation and subsequent
♦♦ Mandible is affected more commonly than maxilla.
cystic degeneration of the “epithelial cell rests of Malassez”, in
♦♦ In mandible, the majority of cysts occurs in ramus third
the periapical region of a non-vital tooth.
molar area, followed by first and second molar area and
then the anterior mandible. The process of development of this cyst occurs in various stages:
♦♦ It is asymptomatic unless they become secondarily infected I. Phase of initiation
in which case patient complains of pain, soft tissue swell­ II. Phase of proliferation
ing and drainage. III. Phase of cystification
♦♦ Occasionally, they experience paresthesia of lower lip IV. Phase of enlargement
and teeth. I. Phase of initiation: During this phase, the bacterial
♦♦ There is often one tooth missing from the dental arch. infection of the dental pulp or direct inflammatory effect
♦♦ Expansion and thinning of bone may result in pathologi­ of necrotic pulpal tissue, in a non-vital tooth causes
cal fracture. stimulation of the “cell rest of Malassez” which are present
♦♦ Maxillary odontogenic keratocyst tends to be secondarily within the bone near the root apex of teeth.
infected with greater frequency than the mandibular ones, II. Phase of proliferation: The stimulation to the cell rests
due to its vicinity to maxillary sinus. of Malassez leads to excessive proliferation of these cells,
Section 3: Oral Pathology  557

which leads to the formation of a large mass or island of ♦♦ Radicular cyst is lined by the stratified squamous epithe­
immature proliferating epithelial cells at the periapical lium.
region of the affected tooth. ♦♦ It is lined by pseudostratified, columnar or respiratory
III. Phase of cystification: Once a large bulk of the cell rest type of epithelium.
of Malassez is produced, its peripheral cells get adequate ♦♦ Hyaline bodies or Rushton bodies often found in the great
nutritional supply but its centrally located cells are often numbers in the epithelium of apical periodontal cyst.
deprived of proper nutritional supply. As a result the cen­ ♦♦ Collagenous connective tissue makes the wall of radicular
tral group of cells undergo ischemic liquefactive necrosis cyst.
while the peripheral group of cells survive. This eventu­ ♦♦ Abundant fibroblast can be identified within the cystic
ally gives rise to the formation of a cavity that contains a wall.
hollow space or lumen inside the mass of the proliferating ♦♦ Cystic wall presents inflammatory infiltrate which contains
cell rest of Malassez and a peripheral lining of epithelial lymphocytes, plasma cells.
cells around it. ♦♦ Other histological findings within the cyst wall are eryth­
IV. Phase of enlargement: Once a small cyst is formed, it rocytes, area of hemorrhage, occasional spicules of dys­
enlarges gradually by the following mechanisms: trophic bone, multinucleated giant cells and cholesterol
• Higher osmotic tension of the cystic fluid causes crystals.
progressive increase in the amount of fluid inside its
lumen and this causes increased internal hydrostatic Radiological Features
tension within the cyst. The process results in cyst It is round or oval radiolucency of variable size which is
expansion due to resorption of the surrounding bone. generally well delineated and is most likely with marked
• The epithelial cells of the cystic lining release some radiopaque rim.
bone resorbing factors like prostaglandins and
collagenase, etc. which destroy the bone and facilitate Treatment
expansion of the cyst. ♦♦ Root canal treatment
♦♦ Extraction of involved tooth
Clinical Features ♦♦ Enucleation or marsupialization of large lesion is done.
♦♦ It most commonly occurs on 3rd, 4th and 5th decades of Q.6. Write note on dermoid cyst.
the life.
♦♦ It is more common among the males.  (Oct/Nov 1992, 6 Marks)
♦♦ Maxillary anteriors are most commonly affected. Ans. It is also known as dermoid cystic tumor or cystic
♦♦ Majority of cases are asymptomatic. teratoma.
♦♦ It is associated with the nonvital tooth. • It is a developmental cyst derived from the remnants
♦♦ Small cystic lesions are asymptomatic. of the embryonic skin.
♦♦ Large lesions often produce a slow enlarging bony hard • It is hamartomatous tumor containing multiple
swelling of the jaw with expansion of cortical plates. sebaceous glands and almost all skin adnexa, this
♦♦ Severe bone destruction produces “sprin­giness” of jaw may consist of substances such as nails and dental
bone. cartilage like and bone like structures.
♦♦ If the cyst is secondarily infected it leads to the formation
of the abscess, which is called “cyst abscess”. Clinical Features

Histological Features ♦♦ It occurs in young children.


♦♦ Dermoid cyst occurs most commonly on face, neck or scalp.
♦♦ In addition to skin dermoid cyst can be intracranial, in­
traspinal or perispinal.
♦♦ It is a painless swelling having the dough consistency.
♦♦ A cyst which develops above the mylohyoid muscle and
presents the sublingual swelling in the midline.
♦♦ Cyst below the mylohyoid muscle presents submental and
submandibular swelling in the midline.

Histological Features
♦♦ Dermoid cysts in the skin are lined by the orthokeratinized
stratified squamous epithelium which exhibit hair follicles,
sebaceous glands and erector pili muscles.
♦♦ Cavity lumen is often filled with the sebum, keratin and
hair shafts.
♦♦ Cyst capsule is composed of narrow zone of compres­sed
Fig. 71:  Radicular cyst (For color version, see Plate 12) connective tissue.
558   Mastering the BDS IIIrd Year  (Last 25 Years Solved Questions)

Treatment • It was also theorized that some of these lesions may


arise from inflammation of the reduced enamel
Surgical excision is done.
epithelium at the time of eruption of the teeth.
Q.7. Write note on epidermoid cyst. (Sep 1999, 5 Marks) • In some cases cysts in the globulomaxillary area may
Ans. It is also called epidermoid inclusion cyst or epidermoid be lined by pseudostratified, ciliated, columnar epi­
cyst or keratin cyst or sebaceous cyst. thelium. Such cases provide credence to the fissural
• Epidermoid inclusion cysts are the result of implan­ theory of origin.
tation of epidermoid elements and its subsequent • Since fissural cyst in this region probably does not
cystic transformation. exist, the term globulomaxillary cyst should no
longer be used.
Clinical Features • When a radiolucency between the maxillary lat­
♦♦ It is most common in 3rd and 4th decades of life. eral incisor and canine is encountered, the clinician
♦♦ It is twice as common in men as in women. should first consider an odontogenic origin for the
♦♦ Common sites are face, trunk, neck extremities and scalp. lesion.
♦♦ Epidermoid cyst appears as firm, round, mobile, flesh Q.9. Write notes on benign cervical lymphoepithelial cyst.
colored to yellow or white subcutaneous nodules of vari­ (Mar 1994, 6 Marks)
able size. In some patients, it contains melanin pigments. Ans. Benign cervical lymphoepithelial cyst is also known as
♦♦ Discharge of foul smelling cheese like material is a com­ branchial cleft cyst.
mon complaint.
♦♦ Sometimes cyst becomes inflamed and infected resulting Pathogenesis
in pain and tenderness.
It originates from cystic transformation of gland epithelium
♦♦ When located orally, it can cause difficulty in feeding,
entrapped in the oral lymphoid aggregates during embryogenesis.
swallowing and even speaking.

Histological Features Clinical Features


♦♦ Cyst is lined by the stratified squamous epithelium with ♦♦ It is located superficially in the neck, close to the angle of
glandular differentiation. mandible, anterior to sternocleidomastoid muscle.
♦♦ Cyst is filled with desquamated keratin disposed in a ♦♦ It occurs at all the ages with a fairly equal distribution
lamellar pattern. from 1st to 6th decades.
♦♦ Dystrophic calcification and reactive foreign body reaction ♦♦ Neck lesions vary in size from small to very large.
are seen associated with the cystic capsule. ♦♦ There is swelling, which may be progressive or inter­mittent
and pain may also be a feature.
♦♦ Pigmented epidermoid cyst may demonstrate melanin
♦♦ Lymphoepithelial cyst generally present as an asympto­
pigment in the wall and a keratin may present.
matic, circumscribed, movable swelling on lateral aspect
♦♦ A surrounding infiltrate of melanocytes and macrophages
of the neck.
may be observed.
♦♦ It may be unilateral or bilateral.
Treatment
Histopathology
Surgical excision is done.
♦♦ Cystic cavity is lined by thin stratified squamous epithe­
Q.8. Write note on globulomaxillary cyst. lium.
(Sep 1993, 6 Marks) ♦♦ Cyst is generally embedded in a circumscribed mass of
Ans. As per the older concept globulomaxillary cyst was lymphoid tissue.
a fissural cyst that arose from epithelium entrapped ♦♦ Capsule of the cyst also present variable amount of con­
during fusion of the globular portion of the medial nasal nective tissue, being infiltrated by lymphocytes, plasma
process with the maxillary process. cells, macrophages and some multinucleated giant cells.
• As per the recent concept globular portion of the ♦♦ Cystic lumen is filled up with either a thin watery fluid or
medial nasal process is primarily united with the a thick gelatinous material.
maxillary process and fusion does not occur. There­
fore, epithelial entrapment should not occur during Treatment
embryologic development of this area. It is removed by local surgical incision.
• All cysts in the globulomaxillary region, i.e. between
the lateral incisor and canine teeth can be explained
on an odontogenic basis. Q.10. Write notes on odontogenic keratocyst.
• Many are lined by inflamed stratified squamous (Mar 2007, 2.5 Marks) (Mar 2007, 2.5 Marks)
epithelium and are consistent with periapical cysts Ans. Refer to Ans 4 of same chapter.
Section 3: Oral Pathology  559

Q.11. Describe the histopathology of odontogenic kerato- According to WHO 2007


cyst. (Sep 2006, 5 Marks) (Sep 2011, 3 Marks) ♦♦ Nasopalatine duct cyst.
(Dec 2010, 3 Marks) For histopathology of odontogenic keratocyst refer to Ans 4 of
Ans. Refer to Ans 4 of same chapter. same chapter.

Q.12. Define and classify cyst. Describe the clinical, radio- Q.16. Write short note on pathogenesis of radicular cyst.
logical and histological features of radicular cyst. (Dec 2007, 3 Marks) (Sep 2011, 3 Marks)
(Sep 2006, 15 Marks) Ans. Refer to Ans 5 of same chapter.
Ans. For classification refer to Ans 1 of same chapter.
Q.17. Classify cysts of jaws. Describe in detail etiopa­
For clinical, radiological and histological features of thogenesis, clinical features, radiographic features and
radicular cyst refer to Ans 5 of same chapter.
histopathologic features of periapical cyst.
Q.13. Give the classification of oral cysts. (Jan 2012, 10 Marks)
(Mar 2007, 4 Marks) Ans. For classification of cysts refer to Ans 1 of same chapter.
Ans. Refer to Ans 1 of same chapter. For etiopathogenesis, clinical features, radiographic
features and histologic features refer to Ans 5 of same
Q.14. Classify odontogenic cyst. Write in detail on radicular chapter.
cyst. (Apr 2007, 15 Marks)
Q.18. Write note on eruption cyst. (Mar 2011, 3 Marks)
Ans.
Ans. It is defined as odontogenic cyst with histologic features
Classification of Odontogenic Cyst of a dentigerous cyst and that surrounds a tooth crown
which has erupted through bone but not soft tissue and
WHO (2017) Classification of Odontogenic Cysts is clinically visible as a soft fluctuant mass on alveolar
ridges.
♦♦ Odontogenic cysts of inflammatory origin
• Radicular cyst Clinical Features
• Inflammatory collateral cysts
♦♦ Odontogenic developmental cysts ♦♦ Most commonly found in children.
• Dentigerous cyst ♦♦ It arises most frequently anteriorly to first permanent
• Odontogenic keratocyst molar.
• Lateral periodontal cyst and botryoid odontogenic ♦♦ Lesion appears as circumscribed, fluctuant, often translu­
cyst cent swelling of alveolar ridge at site of erupting tooth.
• Gingival cysts ♦♦ When the cystic cavity contains blood, swelling appears
• Glandular odontogenic cyst deep blue or purple so it is known as eruption hematoma.
• Calcifying odontogenic cyst ♦♦ Swelling is painless since it is infected.
• Orthokeratinized odontogenic cyst
Histological Features
Q.15. Enumerate nonodontogenic cysts of oral cavity. Write
♦♦ On microscopic examination stratified squamous
about histopathology of odontogenic keratocyst.
epithelium of overlying gingiva is seen.
(Sep 2007, 5 Marks)
♦♦ Epithelium is separated from cyst by a strip of dense
Ans. Enumeration of nonodontogenic cysts of oral cavity connective tissue.
According to Shear ♦♦ There is presence of mild chronic inflammatory cell
infiltrate.
♦♦ Nasopalatine duct cyst ♦♦ In non-inflamed areas epithelial lining of cyst is of reduced
♦♦ Median palatine, median alveolar and median mandibular
enamel epithelium.
cyst
♦♦ Globulomaxillary cyst Treatment
♦♦ Nasolabial cyst.
No treatment is necessary as cyst often ruptures spontaneously.
According to Gorlin
Q.19. Describe in brief reasons for recurrence of odontogenic
♦♦ Globulomaxillary cyst
keratocyst. (May/Jun 2009, 5 Marks)
♦♦ Nasoalveolar cyst
♦♦ Nasopalatine cyst Ans. Following are the reasons for recurrence of odontogenic
♦♦ Median mandibular cyst keratocyst:
♦♦ Anterior lingual cyst • Odontogenic keratocyst multiply in some patients
♦♦ Dermoid and epidermoid cyst including the occurrence of satellite cysts which
♦♦ Palatal cyst of newborn infants. may be retained during an enucleation procedure.
560   Mastering the BDS IIIrd Year  (Last 25 Years Solved Questions)

If enucleation procedures are incomplete may be may be blurred. They are seen singly and also present
new cyst arise from retained satellite microcysts or in groups. Few ghost cells also show nuclear remnants.
retained mural cell islands. Ghost cells have abnormal type of keratinization and have
• Linings of odontogenic keratocyst are thin and frag­ affinity to calcify.
ile and are difficult to enucleate. Portions of lining ♦♦ Ghost cells also may remain in contact to connective tissue
may remain left behind and lead to recurrence. wall of cyst where they lead to foreign body reaction with
• When OKC is enucleated in a single piece chances formation of multinucleated giant cells.
of recurrence are low while when it is removed in ♦♦ An atubular dentinoid is also seen in the wall close to epi­
multiple pieces chances of recurrence is high. This thelial lining and also in relation to epithelial proliferations.
is because in multiple pieces at times remnants of ♦♦ Dentinoid found particularly in contact with masses of
cyst remain which lead to recurrence. ghost cells.
• OKCs may also arise from proliferation of basal
cells of oral mucosa also known as basal cell hamar­ Q.21. Classify cysts of oral region. Give a detailed account
tias in third molar region and ascending ramus of of clinical, radiological and histopathological features
mandible. It was observed that there is perforation of odontogenic keratocyst. (Mar 2013, 8 Marks)
of overlying bone and firm adhesion of cyst to Or
overlying mucosa. So when cysts were surgically
removed overlying mucosa should be excised with Classify cyst of jaw. Write in detail clinical radiolog-
them to prevent recurrence from residual basal cell ical and histopathological features of odontogenic
proliferations. If overlying mucosa is not excised or keratocyst. (Apr 2015, 8 Marks)
it remains it leads to recurrence. Ans. For classification, refer to Ans 1 of same chapter.
Q.20. Write short note on histopathology of COC. For clinical and histological features, refer to Ans 4 of
(Dec 2012, 3 Marks) same chapter.
Ans. It is also known as calcifying odontogenic cyst or
Calcifying epithelial odontogenic cyst. Radiological Features
♦♦ Odontogenic keratocyst is oval in shape and it extends to
Histopathology the body of mandible with mediolateral expansion.
♦♦ Epithelial lining of COC shows prominent basal cell layer ♦♦ It is very small in size or it can exceed the diameter of 5cm.
consisting of palisaded columnar or cuboidal cells and ♦♦ Margins of the cyst are hyperostotic.
hyperchromatic nuclei which are polarized away from ♦♦ Mostly odontogenic keratocyst is unilocular and have
basement membrane. smooth borders while some of the cysts show irregular
♦♦ Epithelium is 6–8 cell layer thick. borders too.
♦♦ Budding from the basal cell layer into adjacent connective ♦♦ Radiolucency is seen in the cystic part which appears to
tissue and epithelial proliferations into lumen are seen. be hazy if keratin is present in the cavity.
♦♦ Radiolucency is surrounded by thin sclerotic rim.
♦♦ In some of the cases perforation of lingual and buccal
cortical plates is seen.
♦♦ Displacement of inferior alveolar canal is seen downwards.
Q.22. Give classification of osteomyelitis. Discuss in detail
clinical features, etiology, histopathology of odonto-
genic keratocyst. (Mar 2013, 8 Marks)
Ans. Classification of osteomyelitis
• Acute osteomyelitis
– Acute suppurative osteomyelitis
– Acute subperiosteal osteomyelitis
– Acute periostitis
• Chronic osteomyelitis:
A. Non-specific type:
– Chronic intramedullary osteomyelitis
Fig. 72:  Calcifying odontogenic cyst (For color version, see Plate 12)
– Chronic focal sclerosing osteomyelitis
♦♦ Ghost cells: They are enlarged, ballooned shape, ovoid or – Chronic diffuse sclerosing osteomyelitis
elongated elliptoid epithelial cells. They are eosinophilic. – Chronic osteomyelitis with proliferative perios­
They are found in thick areas of epithelial lining. Cell titis
outlines of these cells are well defined and at times they – Chronic subperiosteal osteomyelitis
Section 3: Oral Pathology  561

– Chronic periostitis • Rushton body is a hyaline body is found in great


B. Specific type: numbers in epithelium of radicular cyst or residual
– Tuberculous osteomyelitis cyst.
– Syphilitic osteomyelitis • Rushton bodies are linear, straight or curved or of
hair pain shaped.
– Actinomycotic osteomyelitis
• Rushton bodies measure upto 0.1 mm
• Radiation-induced osteomyelitis
• Rushton bodies appear amorphous in structure,
• Idiopathic osteomyelitis.
eosinophilic in reaction and are brittle in nature.
For clinical features and histopathology, refer to Ans 4 of
• They are sometimes concentrically laminated and
same chapter.
frequently fractured.
Etiology of Odontogenic Keratocyst Origin
The cyst arises from: Related to origin of rushton bodies two views were put
♦♦ Dental lamina or its remnants. forward i.e.
♦♦ Primordium of developing tooth germ or enamel organ ♦♦ Rushton bodies were of odontogenic epithelial origin and
♦♦ Basal cell layer of oral epithelium. are probably a form of keratin. —By Shear
Q.23. Write short note on radicular cyst histopathology. ♦♦ Hyaline bodies has hematogenous origin and they
 (Feb 2013, 8 Marks) were derived from thrombi in venules of connective
tissue that had become varicose and strangled by the
Ans. For details, refer to Ans 5 of same chapter.
cuffs of epithelium which encircle it and these bodies
Q.24. Write short note on OKC-etiology and histopathology. reacted histochemically as hemoglobin. According to
 (Nov 2014, 3 Marks) the authors who had given this view also suggested
Ans. For etiology of OKC, refer to Ans 22 of same chapter. that thrombi shrinks centrifugally and undergo split­
ting or calcify.
For histopathology of OKC, refer to Ans 4 of same chapter.
Q.26. Discuss histology and reasons for recurrence of odon-
Q.25. Write short note on rushton bodies. togenic keratocyst. (May 2018, 3 Marks)
 (Feb 2015, 5 Marks) Ans. For histology of odontogenic keratocyst refer to ans4 of
same chapter.
Ans. Rushton bodies are seen in the histological picture of For recurrence of odontogenic keratocyst refer to ans19
radicular cyst or residual cyst. of same chapter.
562   Mastering the BDS IIIrd Year  (Last 25 Years Solved Questions)

II Disturbances of Microbial Origin

6. Bacterial Infections of Q.2. Describe oral manifestations of leprosy, tuber­culosis


and syphilis. (Mar 1998, 16 Marks)
oral Cavity Ans.
Oral Manifestations of Leprosy
Q.1. Write notes on oral manifestations of syphilis.
♦♦ In oral cavity, the disease produces tumor like lesions
 (Mar 2007, 2.5 Marks)
called “lepromas” which are found on lips, gingiva, tongue
Ans. Oral manifestations of syphilis are given below: and hard palate.
Charac- Primary syphi- Secondary Tertiary syphilis ♦♦ Oral lesion appears as yellowish soft or hard sessile growth
teristic lis syphilis which have tendency to breakdown and ulcerate.
Site Chancre occurs Mucus patches Gumma can oc- ♦♦ Ulceration, necrosis and perforation of palate.
at the site on en- are present on cur anywhere in ♦♦ Fixation of palate with loss of uvula.
try of treponema. tongue, buccal jaw but more fre- ♦♦ Difficulty in swallowing and regurgitation.
It occurs on lip, mucosa, pha- quent site is pal-
oral mucosa, and ryngeal region ate, mandible and
♦♦ Cobble stone appearance of tongue with loss of taste
lateral surface of and lips. Split tongue sensation.
tongue, soft pal- papules develop ♦♦ Chronic gingivitis, periodontitis and candidiasis are
ate and gingiva at commissure present.
of lips
♦♦ Enamel hypoplasia of teeth, pinkish discoloration of teeth
A p p e a r - It has narrow Mucus patches Gumma manifests and tapering of teeth is present.
ance copper colored appear as slightly as solitary, deep
slightly raised raised grayish punched out mu-
Oral Manifestations of Tuberculosis
borders with red- white lesions sur- cosal ulcer
dish brown base rounded by ery-
♦♦ Tuberculous infection in oral cavity may produce nodules,
in centers thematous base.
Split papules are vesicles, fissures, plaque, granulomas or verrucal papillary
cracked in middle lesions.
giving “Split pea ♦♦ Tuberculous lesions of oral cavity are tuberculous ulcers,
appearance” tuberculous gingivitis and tuberculosis of salivary gland.
Symptom Intraoral chanc- Mucus patches In gumma the ♦♦ Tongue is most common location for the occurrence,
ers are slightly are painless mild breathing and besides this palate, gingiva, lips, buccal mucosa, alveolar
painful due to in- to moderately swallowing diffi- ridge and vestibules may also be affected.
fection and are painful culty may be en-
covered with sec- countered by the
♦♦ Tongue lesion: Tuberculous lesion of tongue develops
ondary grayish patients on the lateral borders and appears as single or multiple
white film ulcers which are well defined, painful, firm and yellowish
gray in color.
Signs White sloughy Snail tract ulcers P e r f o r a t i o n o f
material is pres- and raw bleed- palatal vault is ♦♦ Lip lesions: Lesions produce small, nontender, granulating
ent ing surfaces are present ulcer at mucocutaneous junction.
present ♦♦ Gingival lesions: These lesions produce small granulating
Tongue Tongue Tongue gets fis- Numerous small ulcers with concomitant gingival hyperplasia.
lesion may sured healed gumma ♦♦ Tuberculous lesion of jaw bone: Chronic osteomyelitis
be commonly in tongue results of maxilla and mandible may occur and infection reaches
seen on lateral in series of nod- to bone via blood or root canal or extraction socket.
surface of ules or sparse
Tuberculous osteomyelitis of jaw bone produces pain,
anterior two- in deeper area
thirds or on of organ giving swelling, sinus or fistula formation.
dorsal surface tongue an uphol-
and often there stered or tufted Oral Manifestations of Syphilis
is enlargement appearance
of folate papilla Refer to Ans 1 of same chapter.
Section 3: Oral Pathology  563

Q.3. Write note on congenital syphilis. (Mar 2000, 5 Marks) Treatment


Or ♦♦ Reconstructive surgery should be done along with pallia­
Write short note on congenital syphilis. tive treatment.
(Dec 2015, 3 Marks) (Sep 2006, 5 Marks)
 (June 2015, 5 Marks) (Apr 2017, 4 Marks) Q.5. Describe histopathology of actinomycosis.
Ans. Congenital syphilis is a rare entity that occurs in children (Sep 2006, 5 Marks)
born of an infected mother. Or
• This condition occurs due to transplacental infection
Describe histologic features with diagram of actino-
with T. palladium during fetal development.
mycosis (Nov 2008, 5 Marks)
• Deciduous teeth are less frequently affected as com­
pared to the permanent teeth. Ans. Actinomycosis is a chronic granulomatous suppurative
and fibrosing infection. It is caused by filamentous,
Features of Congenital Syphilis Gram- positive and anaerobic actinomycosis group
♦♦ Mulberry molars and screw driven shaped incisors occur of infections, i.e. Actinomycoses israeli, Actinomycosis
due to involvement of developing tooth germs. viscosus, etc.
♦♦ Rhagades, i.e. fissuring and scaring at the corners of the
mouth. Histopathology
♦♦ Saddle nose or bull dog appearance is seen.
♦♦ Actinomycosis under microscope shows numerous ab­
♦♦ There is mandible prognathism and increased inter dental
scesses whose centres are occupied by bacterial colonies.
spaces.
♦♦ Bone tissue often exhibits extensive necrosis with multiple
♦♦ Delayed eruption of teeth is present.
areas of granuloma formation.
♦♦ Hypodontia and enamel hypoplasia is seen.
♦♦ Bacterial colony consists of dense, eosinophilic masses of
♦♦ There is occurrence of Hutchinson’s triad which consists of:
Gram-negative filaments.
• Hypoplasia of incisors and molar teeth
• 8th nerve deafness
• Interstitial keratitis of eyes.

Treatment
High doses of penicillin are given.
Q.4. Write note on NOMA. (Mar 2000, 5 Marks)
Ans. It is also called as gangrenous stomatitis.
It is rapidly spreading gangrene of oral and facial tissues
occurring usually in debilitated or nutritionally deficient
patients.

Predisposing Factors
♦♦ Occurs in undernourished persons.
♦♦ Debilitated from infections such as diphtheria, measles,
pneumonia, scarlet fever, TB and blood dyscrasias.
♦♦ Excessive mechanical injury. Fig. 73: Actinomycosis (For color version, see Plate 12)
♦♦ It is a specific infection by Vincent’s organism.

Clinical Features ♦♦ Periphery of each colony shows club-shaped swellings and


produces a “Ray fungus” like appearance.
♦♦ It is seen chiefly in malnourished children. ♦♦ Colonies are surrounded by polymorphonuclear neutro­
♦♦ Common sites are areas of stagnation around the fixed phils followed by lymphocytes, plasma cells and multi­
bridge or crown. nucleated giant cells.
♦♦ Commencement of gangrene is denoted by blackening of ♦♦ Colonies are surrounded by the fibrous tissue wall at
skin. Small ulcers of gingival mucosa spread rapidly and outer margin.
involves surrounding tissue of jaw, lips and cheeks by
gangrenous necrosis. Q.6. Write note on tuberculosis.
♦♦ Odor is foul and patient have high tempe­rature. (Mar 2011, 5 Marks) (Feb 2013, 5 Marks)
♦♦ Overlying skin is inflammed, edematous and finally Ans. Tuberculosis is a infectious granulomatous disease
necrotic. caused by mycobacterium tuberculosis.
564   Mastering the BDS IIIrd Year  (Last 25 Years Solved Questions)

Pathogenesis

Interaction of bacilli and host by droplet nuclei

Host bacterial interaction

Host macrophages control bacteria or bacteria grow


and kill macrophages

Monocytes from blood stream ingest bacilli released


from lysed macrophages

Tissue damaging occurs

Due to specific immunity and accumulation of Fig. 74: Tuberculosis (For color version, see Plate 12)
activated macrophages tubercles are formed
Treatment
Local macrophages are activated and lymphokines Multidrug therapy (MDT) is recommended.
are released
Q.7. Write oral manifestations of tuberculosis.
(Dec 2010, 8 Marks)
Neutralization of bacilli and prevention of further
destruction Or
Write short note on oral manifestations of tuberculo-
Caseous necrosis occur sis. (Jan 2017, 5 Marks)
Ans. Following are the oral manifestations of tuberculosis:
Formation of ranne complex, i.e. calcification occurs Tuberculous infection in oral cavity may produce
nodules, vesicles, fissures, plaque, granulomas or
Clinical Features verrucal papillary lesions.
• Tuberculous lesions of oral cavity are tuberculous
♦♦ Patient suffer from episodic fever and chills, easy fatigabil­
ulcers, tuberculous gingivitis and tuberculosis of
ity and malaise.
salivary gland.
♦♦ There is gradual loss of weight with persistent cough with
• Tongue is most common location for the occurrence,
or without hemoptysis.
besides this palate, gingiva, lips, buccal mucosa,
♦♦ Choroid tubercles are seen in children.
alveolar ridge and vestibules may also be affected.
♦♦ Lupus vulgaris may occur in children.
• Tongue lesion: Tuberculous lesion of tongue devel­
Oral Manifestation ops on the lateral borders and appears as single or
multiple ulcers which are well defined, painful and
♦♦ Tongue is the most common site involved followed by firm in consistency. Ulcers are deep and are painful.
palate, lips, buccal mucosa, gingiva and frenula. Margins of ulcer are undermined with no induration.
♦♦ There is presence of irregular, superficial or deep ulcer Area surrounding the ulcer remains inflamed and
which is painful and will increase in size. edematous. Base of the ulcer is yellowish.
♦♦ There is presence of diffuse hyperemic, nodular or papil­ • Lip lesions: Lesions produce small, nontender, granu­
lary proliferation of gingival tissues. lating ulcer at mucocutaneous junction.
♦♦ Tuberculous osteomyelitis occurs in the later stages of • Gingival lesions: These lesions produce small granu­
disease.
lating ulcers. Gingiva appear diffuse, hyperemic and
Histological Features nodular papillary proliferation is seen.
• Tuberculous lesion of jaw bone: Chronic Osteomyelitis
There is formation of granuloma exhibiting, foci of caseous of maxilla and mandible may occur and infection
necrosis surrounded by epithelioid cells, lymphocyte and reaches to bone via blood or root canal or extraction
occasionally multinucleated giant cells. socket. Tuberculous osteomyelitis of jaw bone pro­
♦♦ Epithelioid cells are morphologically altered macrophages duces pain, swelling, sinus or fistula formation. As
and appear like epithelial cells. jaw bone is involved patient complains of swelling
♦♦ Multinucleated giant cells are of LanghAns type. and difficulty in eating.
♦♦ Area of caseous necrosis appears eosinophilic. • Miliary lesion of oral mucosa in military tuberculosis
♦♦ Granuloma is surrounded by fibrous tissue and lympho­ is a small grey colored tubercle which breakdown
cytes. At times dystrophic calcification is seen. and ulcerate.
Section 3: Oral Pathology  565

• Periapical tissue can also be involved by the tubercle. Histopathology


Tooth socket is filled with tuberculous granulation
Refer to Ans 5 of same chapter.
tissue and has small pink and red elevations.
Q.8. Write short note on actinomycosis. Treatment
 (Feb 2015, 5 Marks) (Dec 2009, 5 Marks) Patient should be kept on high antibiotic therapy such as
 (Mar 2006, 5 Marks) (Aug 2012, 5 Marks) penicillin, cephalosporin, clindamycin, etc.
Or
Q.9. Write short essay on pyogenic granuloma.
Write short essay on actinomycosis. (Jan 2012, 5 Marks)
(Jan 2012, 5 Marks)
Or
Ans. It is a chronic granulomatous suppurative disease which
is caused by anaerobic or microaerophilic gram positive Write short note on pyogenic granuloma.
non-acid fast branched filamentous bacteria.  (Apr 2015, 3 Marks) (Apr 2017, 5 Marks)
• The most common organism is Actinomyces israelii, Ans. Pyogenic granuloma is considered as an exaggerated
A. naeslundii, A. viscosus, A. odontolyticus and A. conditioned response to minor trauma.
propionica. Pyogenic granuloma is a misnomer since condition is
not associated with pus formation.
Classification of Actinomycosis Based on Location
♦♦ Cervicofacial: When there is involvement of face and Etiology
cervical area
♦♦ It is caused by microorganisms such as streptococci and
♦♦ Abdominal: When there is involvement of abdomen staphylococci.
♦♦ Pulmonary: When there is involvement of pleural cavity.
♦♦ If there is minor trauma to the tissue it provides the path­
♦♦ Cutaneous: When there is involvement of skin
way for the non-specific microorganisms which can cause
♦♦ Central: When there is involvement of bone
pyogenic granuloma.
♦♦ Periphery: When there is involvement of soft tissue.
♦♦ Hormonal imbalance can lead to pyogenic granuloma.
Pathogenesis ♦♦ Sulfhydryl molecule is the agent which lead to pyogenic
granuloma.
Disease originates when there is disruption of mucosal barrier
which leads to invasion of bacteria. There is occurrence of initial Clinical Features
acute inflammation which is followed by chronic indolent phase.
Lesions appear as single or multiple indurations.
 ♦♦ It occurs at the age of 10 to 40 years.
♦♦ Female predilection is present.
Clinical Features ♦♦ Most affected sits are lip, gingiva, tongue, palate, vestibule.
Lesion is more common in maxillary anterior region.
Cervicofacial Actinomycosis ♦♦ Lesion is elevated, pedunculated or sessile mass with a
♦♦ Its occurrence is more common in males. smooth, lobulated or warty surface which is ulcerated.
♦♦ Disease may remain localized to soft tissues or spread to ♦♦ On manipulation the ulcer bleeds.
involve salivary glands, bone or skin of face and neck. ♦♦ Lesion is pink to red to purple in color depending on age of
Most commonly involve area is submandibular region. the lesion. It is usually painless and is soft in consistency.
♦♦ Presence of trismus is there before formation of pus. ♦♦ Size of the lesion ranges from 1 mm to centimeters.
♦♦ The disease is characterized by presence of palpable mass
which is indurated and is painless. Skin surrounding the Histopathology
lesion has wooden indurated area of fibrosis.
♦♦ Overlying epithelium is thin and atrophic. At times, it is
Abdominal Actinomycosis
hyperplastic too.
♦♦ It is more severe form of disease. ♦♦ Surface of the epithelium is usually ulcerated and is re­
♦♦ Patient complains of fever with chills and vomiting. placed by thick fibrinopurulent membrane.
♦♦ There is involvement of liver and spleen. ♦♦ Underlying connective tissue has number of endothelial
♦♦ On palpation abdominal mass is felt which is the sign in lined vascular spaces engorged with RBCs and extreme
diagnosis of disease. proliferation of fibroblasts and budding endothelial
Pulmonary actinomycosis cells.
♦♦ There is presence of moderate infiltration of PMN leuko­
♦♦ Patient complains of fever with chills, cough and presence cytes, lymphocytes and plasma cells.
of pain in pleural cavity. ♦♦ Areas of hemorrhage and hemosiderin pigmentation is
♦♦ Empyema is present and there is formation of sinus. seen in connective tissue stroma.
566   Mastering the BDS IIIrd Year  (Last 25 Years Solved Questions)

cavity. Spontaneous healing of the cavity occurs either by


fibrosis or collapse.
♦♦ Calcification of the cavities may occur in which bacteria
persist.
♦♦ In early stages, the spread of infection is mainly by
macrophages to lymph nodes, other tissues, and organs.
However, in children with poor immunity hematogenous
spread results in fatal miliary TB or tuberculous meningitis.
For flowchart of pathogenesis of tuberculosis refer to Ans 6 of
same chapter.
Q.12. Enumerate bacterial lesion involving oral cavity. De-
scribe clinical features, investigations, histopathology
Fig. 75:  Pyogenic granuloma (For color version, see Plate 12) and management of tuberculosis.
 (Jan 2018, 10 Marks)
Treatment Ans.
Surgical excision of the lesion is done.
Enumeration of Bacterial Lesions Involving Oral Cavity
Q.10. Write short note on histopathology of pyogenic
♦♦ Syphilis
granuloma. (Aug 2012, 5 Marks)
♦♦ Non-venereal treponematoses
Ans. For histopathology refer to Ans 9 of same chapter. ♦♦ Gonorrhea
Q.11. Write short note on pathogenesis of tuberculosis. ♦♦ Streptococcal tonsillitis and pharyngitis
 (Jan 2016, 5 Marks) ♦♦ Leprosy
Ans. Following is the pathogenesis of tuberculosis: ♦♦ Tuberculosis
♦♦ Actinomycosis
Pathogenesis of Tuberculosis ♦♦ Noma
♦♦ The interaction of the bacilli and the host begins when ♦♦ Scarlet fever
droplet nuclei from infectious patients are inhaled. ♦♦ Diphtheria
♦♦ The majority of the bacilli are trapped and exhaled by ♦♦ Cat scratch disease
ciliary action and a fraction less than 10% enters alveoli. ♦♦ Tularemia
♦♦ In the initial stage of the host-bacterial interaction, either ♦♦ Tetanus
host’s macrophages control the multiplication of the bac­ ♦♦ Rhinoscleroma
teria or the bacteria grow and kill the macrophages. ♦♦ Botryomycosis
♦♦ Non-activated monocytes attracted from blood stream to ♦♦ Melioidosis
the site by various chemotactic factors ingest the bacilli ♦♦ Granuloma Inguinale
released from the lysed macrophages. ♦♦ Lymphogranuloma venereum
♦♦ Initial stages are asymptomatic; about 2-4 weeks after ♦♦ Myiasis.
infection tissue damaging and macrophage activating For clinical features and histopathology refer to Ans 6 of
responses develop. same chapter.
♦♦ With the development of specific immunity and accumulation Investigations
of a large number of activated macrophages at the site of pri­
mary lesion, granulomatous reaction or tubercles are formed. ♦♦ Serology: In this ELISA technique is used which helpful
in diagnosis of tuberculosis in children. PCR technique
♦♦ The hard tubercle consists of epithelioid cells, LanghAns
is more specific and sensitive serological test than ELISA,
giant cells, plasma cells, and fibroblasts. These lesions
but PCR is less used due to its high cost.
develop when host resistance is high.
♦♦ Chest X-ray: Presence of multiple nodular infiltrations or
♦♦ Due to cell-mediated immunity in the majority of individu­
ill defined opacities in one of upper lobes is characteristic
als, local macrophages are activated and lymphokines are
for pulmonary tuberculosis. An area of translucency in
released, which neutralize the bacilli and prevent further
radiopacities is indicative of cavitation. Presence of cavity
tissue destruction. is indicative of an active lesion. In some of the patients,
♦♦ The central part of the lesion contains caseous, soft, and multiple thick-walled cavities can be seen. At the time
cheesy necrotic material (caseous necrosis). This necrotic of fibrosis, trachea and mediastinum shift to same side.
material may undergo calcification at a later stage called Fibrosis can also cause calcification.
Ranne complex, in the lung parenchyma and hilar lymph ♦♦ Pathological tests:
nodes in few cases. • Blood examination: Peripheral blood examination shows
♦♦ Caseous necrotic material under goes liquefaction and monocytosis, i.e. 8 to 12%
discharges into the lungs leading to the formation of a • ESR is elevated.
Section 3: Oral Pathology  567

• Tuberculin test: It is a test to recognize prior tubercular Contd...


infection, and is done by injecting one unit of purified
Indication Intensive phase Continuation
protein derivative (PPD) on the forearm and readings phase
taken after 48 hours. Induration of more than l5 mm
Drugs Duration Drug Duration
indicates a positive test. The younger is the patient,
greater is significance of positive test. A negative test
Isoniazid For 1 Iso- 5 months
rifampicin, months niazid
does not always exclude tubercular infection since it pyrazin- rifampi-
may be negative in patients of blood malignancies, amide cin and
malnourishment and those on immunosuppressive and, eth- pyrazin-
therapy. Tuberculin test is non-specific and only ambutol amind
indicates prior infection. Its sensitivity wanes with age.
Second line anti-tuberculous Drugs
Management
Drug Dose
Chemotherapy.
Streptomycin 20–40 mg/kg/day od im
Drugs for Primary Chemotherapy Kanamycin 15–30 mg/kg/day od im
(First Line Anti-Tubercular Drugs) Amikacin 15–22.5 mg/kg/day od im
Capreomycin 15–30 mg/kg/day od im
Drug Daily dose (Adult) Thrice weekly dose Ofloxacin 15–20 mg/kg/day bd orally
Rifampicin >50 kg 600 mg. (10 mg/kg) levofloxacin 7.5–10 mg/kg/day od orally
<50 kg 450 mg (10 mg/kg) Moxifloxacin 7.5–10 mg/kg/day orally
Isoniazid 200–300 mg (5 mg/kg) Ethionamide 15–20 mg/kg/day bd orally
(in miliary TB 10 mg/kg) (10 mg/kg) Prothionamide 15–20 mg/kg/day bd orally
Pyrazinamide >50 kg 2 g (35 mg/kg, max 3 g) Cycloserine* 10–20 mg/kg/day od or bd orally
(25 mg/kg max 2 g) PAS' 150 mg/kg/day bd or Ids orally
<50 kg 1.5 g "Cycloserine and PAS are bacteriostatic others are bactericidal.
Ethambutol 25 mg/kg in initial phase, (30 mg/kg)
15 mg/kg in continuation Treatment Regimen under RNTCP for MDR-TB (Multi- drug
phase resistant TB) and XDR-TB (Extensively drug resistant TB)

Under DOTs following treatment regimen is used ♦♦ For MDR-TB:


• Six drugs in intensive phase for 6–9 months: Kanamycin,
Indication Intensive phase Continuation levofloxacin, ethionamide, cycloserine, pyrazinamide
phase and ethambutol.
Drugs Duration Drug Duration • Four drugs in continuation phase for 18 months:
Category New Isoniazid Thrice Isonizid Thrice Levofloxacin, ethionamide, cycloserine and
I sputum rifampicin, weekly and weekly for ethambutol.
smear- pyrazin- for 2 rifampi- 4 months • Reserve drug is p-aminosalicylic acid.
postive amide months cin
New and eth-
♦♦ For XDR-TB:
sputum ambutol • Seven drugs in intensive phase for 6-12 months:
smear- Capreomycin, p-aminosalicylic acid, moxifloxacin,
negative high dose isoniazid, clofazimine, linezolid, Amoxicillin
New and clavulanic acid.
extrapul- • Six drugs in continuation phase for 18 months:
monary
p-aminosalicylic acid, moxifloxacin, high dose
tubercu-
losis
lsoniazid, clofazimine, linezolid, amoxicillin and
clavulanic acid.
Category Sputum Isoniazid Thrice
II smear- rifampicin, weekly • Reserve drugs: Clarithromycin, thiacetazone
positive pyrazin- for 2 ♦♦ Corticosteroids: They are to be given in the severe cases to
Sputum amide, months enable them to survive till anti-tubercular drugs become
smear- etham- followed effective. Oral prednisolone is given in doses of 20 mg
positive butol and by orally for 6 to 8 weeks. Steroids produce euphoria and
failure strepto- increase appetite in the patients.
Sputum mycin
♦♦ Surgery: Surgical resection of infected lobe is feasible.
smear-
♦♦ Symptomatic treatment:
positive
treatment • Cough: If it is irritative, linctus codeine is given.
after Smoking should be stopped
default • Laryngitis: Rest is given to the voice. If pain is present
Contd... anesthetic powders, spray and lozenges are given.
568   Mastering the BDS IIIrd Year  (Last 25 Years Solved Questions)

7. Viral Infections of Oral Write notes on herpes simplex.


(Mar 1998, 5 Marks) (Mar 2000, 5 Marks)
cavity  (Feb 2002, 6 Marks)

Q.1. Write short note on clinical features of AIDS. Or


(Sep 2004, 5 Marks) Write short note on herpes simplex.
Ans. AIDS is predominantly lethal type of viral infection (Sep 2007, 3 Marks)
caused by the HIV and is characterized by the depletion
of T4 lymphocytes in the body. Or
Clinical staging of AIDS is done by WHO in 1990 and Write short note on herpes simplex viral infection.
is revised on 2007. So based on the clinical staging (Nov 2008, 5 Marks)
following are the clinical features of AIDS.
Ans.
In Primary HIV Infection
Classifications of Viral Lesions in Oral Cavity
♦♦ Patient is asymptomatic
The classification depends on the presence of the major viruses.
♦♦ There is presence of acute retroviral syndrome, i.e. pres­
ence of sore throat, fever, maculopapular rash, headache, I. RNA Virus
myalgia, arthralgia, photophobia.
♦♦ Orthomyxovirus, i.e. influenza
Clinical Stage I ♦♦ Paramyxovirus, i.e. measles and mumps
♦♦ Rhabdovirus, i.e. rabies
♦♦ Patient remains asymptomatic
♦♦ Arena virus, i.e. lassa fever
♦♦ Presence of generalized lymphadenopathy
♦♦ Calicivirus, i.e. upper respiratory tract infection
Clinical Stage II ♦♦ Coronavirus
♦♦ Bunyavirus
♦♦ Presence of unexplained weight loss, i.e. less than 10% of
♦♦ Picornavirus
body weight.
♦♦ Reovirus
♦♦ Presence of recurrent respiratory infections
♦♦ Togavirus
♦♦ Herpes zoster or shingles
♦♦ Retrovirus
♦♦ Presence of minor mucocutaneous diseases, i.e. apthous
ulcers, pruritic eruptions etc. II. DNA Virus
Clinical Stage III ♦♦ Herpes virus:
• Herpes simplex
♦♦ Presence of severe weight loss, i.e. greater than 10% of
• Herpes zoster
body weight.
• Epstein-Barr virus
♦♦ Presence of diarrhoea more than a month.
♦♦ Pox virus:
♦♦ Presence of pyrexia of unknown origin for more than one
• Small pox
month.
• Monkey pox
♦♦ Presence of oral candidiasis which is persistant
• Adeno virus, i.e. pharyngo conjunctival fever
♦♦ Pulmonary tuberculosis since last two years
• Parvo virus
♦♦ Multiple bacterial infections
• Irido virus
♦♦ Presence of acute necrotizing ulcerative gingivitis and
• Papova virus, i.e. papillomas.
acute necrotizing ulcerative periodontitis.
Primary Herpes Simplex Infection
Clinical stage IV
♦♦ There is presence of HIV wasting syndrome It is also known as acute herpetic gingivostomatitis or infectious
♦♦ Pneumocystis pneumonia. stomatitis or herpes labialis.
♦♦ Presence of herpes simplex infection for more than a Pathogenesis
month.
♦♦ Candidiasis extends to esophagus. As the herpes virus enters in sensory autonomic nerve endings
♦♦ Presence of extrapulmonary tuberculosis and remain latent in ganglia. Replication of virus occurs and
♦♦ Kaposi’s sarcoma it spread to the mucosal as well as skin surfaces by centrifugal
♦♦ HIV encephalopathy. distribution of virions through peripheral nerves.
Q.2. Classify viral lesions of oral cavity. Describe in detail Etiology
herpes simplex. (Feb 2006, 6.5 Marks)
♦♦ It occurs during close personal contact, i.e. by sexual
Or contact or kissing.
Section 3: Oral Pathology  569

♦♦ Primary infection of newborn is caused by vaginal secre­ Recurrent or Secondary Herpes Simplex Infection
tion during birth.
It is of two types:
♦♦ Dentist may experience primary lesion of finger from
contact with the lesion of mouth and saliva of patient who 1. Recurrent herpes labialis.
are asymptomatic carrier of HSV also known as herpetic 2. Recurrent Intraoral herpes.
whitlow. Pathogenesis
♦♦ In cadence varies according to socioeconomic group.
As primary infection is over, virus can no longer recover from
Clinical Features the ganglion and viral DNA is seen in ganglionic cells. Humoral
♦♦ It is seen in children as well as young adults. and cell mediated immunity is responsible for the recurrence
♦♦ Incubation period of virus lies from 5–7 days. of the disease.
♦♦ Patient complains of irritability, fever, headache, nausea,
Etiology/Trigger Factors
vomiting and pain in oral cavity.
♦♦ Cervical and submandibular lymph nodes are enlarged. ♦♦ Any surgery which involves trigeminal ganglion leads to
recurrent herpes.
Oral Manifestations ♦♦ Low immunity leads to recurrent herpes.
♦♦ In oral cavity hard palate, tongue and gingiva are ♦♦ Trauma as in exodontias can precipitate recurrent herpes.
commonly involved. ♦♦ Upper respiratory infection can lead to recurrent herpes.
♦♦ Oral symptoms are flattening of mucosa, followed by ♦♦ Other factors which lead to recurrent herpes are fatigue,
vesicle formation over keratinized mucosa. pregnancy, fever, menstruation, etc.
♦♦ The vesicle consists of clear fluid and rupture to leave
multiple, small and shallow painful ulcers these are Clinical Features
followed by diffuse, large, whitish ulcers which are ♦♦ Recurrent herpes occur at varying intervals.
surrounded by red ring of inflammation. ♦♦ Patient complains of tingling and burning sensation, feeling
♦♦ Ulcer is about 2 to 5 mm in diameter. Base of the ulcer is of toughness and soreness before development of vesicle.
coated with greyish white or yellowish membrane. ♦♦ Edema is seen at the site of lesion and is followed by
♦♦ Excoriation of lip is seen with hemorrhage. development of multiple clusters of vesicles.
♦♦ Speech of patient become painful and difficult.
♦♦ Generalized acute marginal gingivitis is present. Gingiva Oral Manifestations
become edematous as well as swollen. ♦♦ In recurrent herpes labialis grayish or whitish vesicles
♦♦ Small ulcers are seen over gingiva. are seen which rupture and leave small red ulcers. There
♦♦ Inflammation is present over posterior part of pharynx is presence of very slight erythematous halo over the lip
and patient feels difficulty in swallowing. which is covered by brown-colored crustation. Size of crust
♦♦ Patient also suffers from myalgia or muscle soreness and is 1 to 4 mm in diameter.
difficulty in mastication. ♦♦ In intraoral type vesicles are seen which rupture and leave
Histopathology small red ulcers with very slight erythematous halo.

♦♦ Infected cells are swollen have pale eosinophilic cyto­ Treatment


plasm and large vesicular nuclei known as ballooning ♦♦ Trigger factors should be suppressed or removed.
degeneration. ♦♦ Antiviral therapy should be given.
♦♦ Acantholysis is seen and acantholytic cells are known as
Tzanck cells. Q.3. Describe etiology, histopathology and clinical features
♦♦ Some of the cells contain intranuclear inclusion bodies of acute herpetic gingivostomatitis.
known as Lipschutz bodies. (Mar 2000, 15 Marks)
♦♦ Lipschutz bodies are eosinophilic, ovoid, homogenous Or
structures in the nucleus which displaces nucleolus and Write short note on primary herpetic gingivos­
nuclear chromatin peripherally. tomatitis. (Sep 2005, 5 Marks)
♦♦ Multinucleated epithelial cells are formed when fusion
Ans. For etiology, histopathology and clinical features refer
occur between adjacent cells.
to Ans 2 of same chapter.
♦♦ Intercellular edema is present which leads to the
development of intraepithelial vesicle. Q.4. Write note on herpes zoster.  (Aug/Sep 1998, 5 Marks)
♦♦ Underlying connective tissue stroma shows chronic Or
inflammatory infiltrate. Write short note on herpes zoster. ( Jan 2018, 5 Marks)
Ans. It is also called as shingles or zona.
Treatment
• It is an acute infectious viral disease of extremely
♦♦ Symptomatic treatment is done. painful and incapacitating nature, characterized by
♦♦ Patient should be kept on antiviral therapy. inflammation of dorsal root ganglion.
570   Mastering the BDS IIIrd Year  (Last 25 Years Solved Questions)

Clinical Features Etiology


♦♦ It affects males and females with same frequency. ♦♦ Bacterial infection: Alpha hemolytic Streptococcus and
♦♦ There is prodromal period of 2 to 4 days in which shooting S. sanguis has implicated as causative agent of disease.
pain, paresthesia, burning and tenderness appears along ♦♦ Immunological abnormalities: IgG and IgM binding of
the course of affected nerve. epithelial cells of spinous layer of oral mucosa is seen in
♦♦ It may be found on buccal mucosa, tongue, uvula, pharynx patients suffering from recurrent aphthous ulcer.
and larynx. ♦♦ Iron deficiency or folic acid deficiency.
♦♦ Hematological deficiencies, serum iron or vitamin B12
♦♦ Trigeminal herpes zoster occur during tooth formation
deficiency.
causes pulpal necrosis and internal root resorption.
♦♦ Allergic factors: Patients may have history of asthma, hay
♦♦ First branch of trigeminal is most commonly affected. fever and food or drug allergy.
Oral Manifestations Clinical Features
♦♦ Herpes zoster may involve the face by infection of ♦♦ It occurs between second and third decades of life and
trigeminal nerve, mainly first branch. females are more commonly infected.
♦♦ There is usually involvement of skin and oral mucosa ♦♦ Clinically aphthous ulcer occur in three types, i.e.
supplied by trigeminal nerve. 1. Minor aphthous ulcer:
♦♦ Lesions of the oral mucosa are extremely painful vesicles • It is a single lesion or in cluster of 1 to 5 lesions.
which may be found on the buccal mucosa, tongue, • Ulcers are painful, shallow, round or elliptical in shape
pharynx, larynx and uvula. and measure 0.5 cm in diameter.
♦♦ This vesicle generally ruptures and leaves the area of • They develop over lips, soft palate, anterior fauces,
erosion. floor of mouth and ventral surface of tongue.
♦♦ Erosive ulcers heal up in a few days without scar forma­ • Lesion is usually surrounded by an erythematous and
tion. is covered by yellowish, fibrous membrane.
♦♦ In herpes zoster, neuralgic pain in oral cavity stimulates 2. Major aphthous ulcer:
tooth ache. • They are large and much painful lesions and measure
♦♦ Pain may persist long after the lesion heals up and the 1 to 5 cm in diameter.
condition is known as postherpetic neuralgia. • They are most severe and often makes patient ill.
• Only one and two lesions develops at the time and
Histopathology mostly seen over lips, soft palate and fauces.
♦♦ Herpes zoster is histologically characterized by swelling • Lesions heal slowly and leaves scars, which result in
of infected epithelial cell cytoplasm due to intercellular decreased mobility of uvula and tongue.
edema (Ballooning degeneration). 3. Herpetic ulcer:
♦♦ Margination of nuclear chromatin and formation of intra • They are multiple, small, shallow ulcers around 100
nuclear inclusion bodies. in number.
♦♦ Reticular degeneration of epithelial cells along with the • Lesions are painful than would be suspected by their
presence of multiple multinucleated giant cells. size.
• They are found on any intraoral mucosal surface.
Treatment
• They are present continuously for 1 to 3 years, with
Antiviral drugs such as acyclovir is given along with antibiotics relatively short remissions.
to prevent secondary infection.
Histopathology
Q.5. Write short note on aphthous ulcers.
♦♦ Fibrinopurulent membrane covers the ulcerated area.
(Apr 2015, 3 Marks) (Apr 2007, 5 Marks)
♦♦ Intense inflammatory cell infiltrate is present in connec­
Or tive tissue beneath the ulcer with considerable necrosis of
Write note on aphthous stomatitis. tissue near the surface of lesion.
 (Feb 2013 , 5 Marks) ♦♦ Neutrophils are predominant below the ulcer.
♦♦ Epithelial proliferation along the margins of lesion.
Or
♦♦ Cells are present with elongated nuclei containing linear
Write short note on aphthous stomatitis. bar of chromatin with radiating process of chromatin ex­
 (June 2015, 5 Marks) tending towards nuclear membrane known as Anitschkow
Ans. Aphthous ulcer or aphthous stomatitis is a common cells.
disease characterized by painful, recurrent, solitary or
Treatment
multiple ulcerations of oral mucosa with no other signs
of any other disease. Topical and systemic administration of steroids is beneficial.
Section 3: Oral Pathology  571

Q.6. Enumerate ulcerative lesions of oral cavity. Describe ♦♦ Erythema multiforme


etiology, histopathology and clinical features of aph- ♦♦ Erosive lichen planus (Secondary ulcer)
thous stomatitis. (Feb 1999, 15 Marks) ♦♦ Lupus erythematosus
Or ♦♦ Sarcoidosis
Enumerate ulcerative lesions of oral cavity, describe ♦♦ Cyclic neutropenia
histopathology and clinical features of aphthous sto- ♦♦ Ulcerative colitis
matitis. (Dec 2009, 15 Marks) ♦♦ HIV
Or ♦♦ Pemphigus
Describe etiology, histopathology and clinical features ♦♦ Epidermolysis bullosa
of acute aphthous stomatitis. (Mar 2001, 15 Marks)
V. Metabolic Ulcers
Ans.
♦♦ Diabetes
Classification of Ulcerative Lesions ♦♦ Uremia
I. Microbial Origin ♦♦ Neutropenia
♦♦ Sickle cell anemia
♦♦ Bacterial:
♦♦ Agranulocytosis
• Streptococcal
♦♦ Crohn’s disease.
• Tuberculosis
• Syphilis VI. Non-specific Ulcers
• Scarlet fever
♦♦ HIV ulcers
• Diphtheria
♦♦ Graft Vs host reaction
• Typhoid
♦♦ Necrotizing sialometaplasia
• Noma
♦♦ Reynaud’s phenomenon
♦♦ Fungal:
♦♦ Bacterial angiomatosis
• Histoplasmosis
• Blastomycosis VII. Neoplastic
• Paracoccidioidomycosis
♦♦ Squamous cell carcinoma
• Coccidioidomycosis
For etiology, histopathology and clinical features of aph­
• Cryptococcus
thous stomatitis refer to Ans 5 of same chapter.
• Zygomycosis
• Aspergillosis Q.7. Write short note on mumps. (Mar 2001, 5 Marks)
♦♦ Viral: Ans. It is an acute contagious viral infection, charac­terized by
• Herpes unilateral or bilateral swelling of salivary glands but also
• HIV affects testis, meninges, pancreas, heart and mammary
• Poxvirus glands.
♦♦ Protozoal:
• Entamoeba histolytica Etiology
• Leishmaniasis ♦♦ It is caused by paramyxovirus.
• Toxoplasmosis. ♦♦ It spreads from human reservoir by air-borne infection of
infected saliva and possibly urine.
II. Physical Origin
♦♦ Cheek bite (Morsicatio Buccarum) Clinical Features
♦♦ Traumatic ♦♦ It is seen between age of 5 to 15 years.
♦♦ Thermal ♦♦ It is most common in boys.
♦♦ Electrical ♦♦ It is preceded by onset of headache, chills, moderate fever,
♦♦ Osteoradionecrosis vomiting and pain below ear which lasts for one week.
♦♦ Anesthetic. ♦♦ Parotid gland is most common site.
♦♦ Elongation of parotid gland causes elevation of ear lobule
III. Chemical Origin
and produces pain on mastication especially while eating
♦♦ Phenol sore food.
♦♦ Silver nitrate ♦♦ Papilla on opening of parotid duct is puffy and reddened.
♦♦ Hydrogen peroxide
♦♦ Aspirin. Histopathology
♦♦ Microscopic section of diseased tissue reveals presence of
IV. Immunological degenerative changes in ductal epithelium of salivary gland.
♦♦ Behcet’s syndrome ♦♦ Interstitial infiltration of lymphocyte and mononuclear
♦♦ Reiter’s syndrome cells in glandular lobes and in few acinar atrophy occur.
572   Mastering the BDS IIIrd Year  (Last 25 Years Solved Questions)

Treatment ♦♦ Viral infections: Herpes simplex, human papilloma virus,


varicella—zoster
Only symptomatic treatment is done.
Group 3: Seen in HIV infection
Q.8. Discuss in detail about clinical features, histopathology ♦♦ Bacterial infections: Actinomyces israelii, Escherichia coli,
and laboratory investigations of oral herpes simplex Klebsiella, pneumonia.
infection. (Jun 2010, 15 Marks) ♦♦ Cat-scratch disease (Bartonella henselae).
Ans. For clinical features and histopathology, refer to Ans 2 ♦♦ Epithelioid (bacillary) angiomatosis (Bartonella henselae)
of same chapter. ♦♦ Drug reactions: Ulcerative, erythema multiforme, lichenoid,
Laboratory Investigations toxic epidermolysis.
♦♦ Fungal infections other than candidiasis: Cryptococcus neofor-
♦♦ Tzanck smear: It is a rapid, sensitive and inexpensive mans, Geotrichum candidum, Histoplasma capsulatum, Muc-
diagnostic method. Smears are prepared from the lesions, oraceae (Mucormycosis/Zygomycosis), Aspergillus flavus.
from the base of vesicles and is stained with 1% solution of ♦♦ Neurologic disturbances: Facial palsy, trigeminal neuralgia
toluidine blue for 15 seconds. Smear shows multinucleated ♦♦ Recurrent aphthous stomatitis.
giant cells with faceted nuclei and ground glass chromatin, ♦♦ Viral infections: Cytomegalovirus, Molluscum contagiosum.
i.e. Tzanck cells.
♦♦ In giemsa stained smears Type A inclusion bodies are seen. Description of Oral Manifestations
♦♦ Electron microscopy show virus particle. ♦♦ Candidiasis is the most common oral manifestation of
♦♦ Fluorescent antibody techniques show herpes virus anti­ HIV infection. All the three types, i.e. erythematous,
gen in smears and sections. pseudomembranous and hyperplastic forms are seen. Ery­
♦♦ ELISA is a serological test which helps in the detection of thematous candidiasis is seen when the CD4 count drops
rise in titre of antibodies. It helps in detection in primary below 400 cells/mm3 and pseudomembranous develop
infection. when CD4 count drop below 200 cells/mm3.
♦♦ Virus Isolation: The isolation of herpes virus can be done ♦♦ Hairy leukoplakia: Presence of soft painless plaque on the
and can be identified by various systems. lateral border of tongue with corrugated surface.
Q.9. Enumerate viral lesions occurring in oral cavity. Dis- ♦♦ Kaposi’s sarcoma: Single or multiple bluish swellings are
cuss oral manifestation of herpes simplex infection. seen with or without ulceration over gingiva and palate.
(Aug 2012, 15 Marks) ♦♦ Angular cheilitis: Linear fissures or linear ulcers are seen
Ans. For enumeration of viral lesions and oral manifestations at the angle of mouth.
of herpes simplex infection, refer to Ans 2 of same ♦♦ Linear gingival erythema: It is fiery red band along the gin­
chapter. gival margin and attached gingiva with profuse bleeding.
♦♦ Necrotizing ulcerative gingivitis: Destruction of interdental
Q.10. Describe in brief oral manifestations of AIDS.
papillae is seen.
(June 2015, 5 Marks) (Apr 2008, 10 Marks)
♦♦ Necrotizing ulcerative periodontitis: There is advanced ne­
Or
crotic destruction of periodontium, rapid bone loss, loss
Write short answer on oral manifestations of HIV of periodontal ligament and sequestration.
diseases. (May 2018, 3 Marks) ♦♦ Oral ulcerations: Single or multiple major recurrent
Ans. Following are the oral manifestations of AIDS: aphthous ulcers are seen with white pseudomembrane
Classification of Oral Manifestations by EC-Clearing house surrounding the erythematous halo.
♦♦ Non-Hodgkin’s lymphoma: It is the malignancy of HIV infect­
Group 1: Strongly associated with HIV infection
ed individuals. It occurs in extranodal locations and CNS
♦♦ Candidiasis: Erythematous, pseudomembranous, angular is the common site. Intraosseous involvement is also seen.
cheilitis ♦♦ Mycobacterial infection: Mycobacterial infection in form of
♦♦ Hairy leukoplakia tuberculosis is seen. When present tongue is affected most
♦♦ Kaposi’s sarcoma commonly. Affected areas show common ulcerations.
♦♦ Non-Hodgkin’s lymphoma ♦♦ Herpes simplex virus: Recurrent or secondary herpes simplex
♦♦ Periodontal diseases: Linear gingival erythema, necrotizing infection is seen in the patients. Herpes simplex lesions
gingivitis, necrotizing periodontitis increase when CD4 cell count drops below 50 cells/mm3.
Group 2: Less commonly associated with HIV infection ♦♦ Herpes zoster: It is common in HIV infected individuals.
♦♦ Bacterial infections: Mycobacterium avium—intracellular, Orally, involvement is severe and leads to sequestration
Mycobacterium tuberculosis of bone as well as loss of teeth.
♦♦ Melanotic hyperpigmentation ♦♦ Histoplasmosis: It is the fungal infection caused by histo­
♦♦ Necrotizing ulcerative stomatitis plasma capsulatum. Sign and symptoms of disease are
♦♦ Salivary gland disease: Dry mouth, unilateral or bilateral fever, weight loss, splenomegaly and pulmonary infiltrate.
swelling of major salivary glands. ♦♦ Molluscum contagiosum: It is caused by poxvirus. Lesions
♦♦ Thrombocytopenia purpura are small, waxy, dome-shaped papules which demonstrate
♦♦ Oral ulcerations NOS (not otherwise specified) central depressed crater.
Section 3: Oral Pathology  573

Q.11. Write short note on herpes stomatitis. Or


(June 2014, 5 Marks)
Write short note on oral candidiasis.
Ans. This is a contagious viral infection, which produces
(Mar 2001, 5 Marks)
ulceration and inflammation of mouth and gums.
It is a contagious viral illness caused by herpes simplex Ans. Enumeration of fungal lesion of oral cavity
virus (HSV-1). Fungal lesions affecting oral tissues are:
This condition is probably a child’s first exposure to the • Candidiasis
herpes virus. • Coccidioidomycosis
• Histoplasmosis
Clinical Features • Blastomycosis
♦♦ Occur in children of 2 to 4 years. • Paracoccidioidomycosis
♦♦ Irritability and refusal of food due to difficulty in swal­ • Sporotrichosis
lowing. • Chromomycosis and phaeomycotic abscess
♦♦ High fever • Aspergillosis
♦♦ Vesicles on the tongue, buccal mucosa, gums and skin • Cryptococcosis
around the mouth. The ulcers are very painful. • Zygomycosis
♦♦ Mucosa becomes red, swollen and bleeds easily. • Mycetoma.
♦♦ Vesicles breakdown to form ulcers. –– Candidiasis is the disease caused by the fungus
♦♦ Secondary bacterial infection may occur with enlarged called as Candida albicans.
lymph nodes and difficulty in swallowing. –– Oral involvement is probably most common
♦♦ It is self-limiting and lasts between 7 to 10 days. manifestation.
Etiology
Prevention
♦♦ Hormonal disturbances
Approximately 80% of the population carry the HSV which ♦♦ Local or systemic steroid therapy
makes it difficult to prevent children contacting the virus. ♦♦ Xerostomia
Parents should avoid kissing their children when they have a ♦♦ Poor oral hygiene
cold sore. Also avoid sharing glasses, food and utensils. ♦♦ Denture wearing
♦♦ Heavy smoking
Complication
♦♦ Prolong antibiotic therapy.
A secondary herpes infection of the eye may occur, i.e. herpetic ♦♦ Nutritional deficiency, e.g. vitamin A and vitamin B6.
keratoconjunctivitis.
Clinical Features
Treatment ♦♦ Common sites are roof of mouth, retromolar area and
♦♦ Acyclovir, which fights the virus causing the infection. mucobuccal fold.
♦♦ A mostly liquid diet of cool/cold nonacidic drinks. ♦♦ It is more common in women.
♦♦ Numbing medicine (viscous lidocaine) applied to the ♦♦ Prodromal symptom is rapid onset of bad taste. Spicy food
mouth if there is severe pain. causes discomfort.
♦♦ There is presence of inflammation, erythema and painful
eroded areas may be associated with this disease.
8. Fungal Infections of ♦♦ White patches of candidiasis are easily wiped out with wet
gauge which leaves erythematous area or atrophic area.
Oral Cavity ♦♦ Deeper invasion by the organism leaves an ulcerative le­
sion upon removal of patch.
Q.1. Enumerate fungal lesions of oral cavity. Describe clini- 1. Acute Pseudomembranous Candidiasis
cal features, histopathology and investigation of oral • Common sites are roof of mouth, retromolar area and
candidiasis. (Dec 2010, 18 Marks) mucobuccal fold.
Or • It is more common in women.
Write note on moniliasis. (Sep 1999, 5 Marks) • Prodromal symptom is rapid onset of bad taste. Spicy
food causes discomfort.
Or
• There is presence of inflammation, erythema and
Describe in brief moniliasis. painful eroded areas may be associated with this
(May/June 2009, 5 Marks) disease.
Or • White patches of candidiasis are easily wiped out
with wet gauge which leaves erythematous area or
Write short note on candidiasis. (Jan 2016, 5 Marks) atrophic area.
574   Mastering the BDS IIIrd Year  (Last 25 Years Solved Questions)

• Deeper invasion by the organism leaves an ulcerative Ans. It is commonly known as “oral thrush”.
lesion upon removal of patch. • Acute pseudomembranous candidiasis appears as
2. Acute Atrophic Candidiasis a smooth, thick, creamy-white or allow, soft and
• It can be seen anywhere in the oral cavity but most friable plaque on the oral mucosa.
commonly site involved are tongue as well as the • Plaque can be easily wiped off by gentle scraping,
tissue underlying prosthesis. which leaves an erythematous, raw, bleeding surface
• It appears as an erythematous area. in the underlying area.
• Patient complains of burning sensation in lesional area • Lesions may occur at any mucosal site and vary in
along with vague pain. size ranging from small areas to confluent plaques.
3. Chronic Hyperplastic Candidiasis • Plaque consists of fungal organisms, keratotic debris,
• Male predilection is seen. inflammatory cells, desquamated epithelial cells and
• Most common in heavy smokers. fibrin, etc.
• Oral sites involved are tongue, cheek and lips. • Oral thrush commonly occurs among children,
• There is presence of firm and white leathery plaques. debilitated elderly persons and AIDS patients.
• Lesion cannot be rubbed with the lateral pressure.
Histopathology
Lesion is whitish or creamy whitish in color. Borders
of the lesion are vague. ♦♦ Hyperplastic epithelium with superficial necrotic and
desquamating parakeratinized layer.
Histopathology ♦♦ Hyperplastic epithelium is infiltrated by candidal hyphae
♦♦ Epithelium show increase thickness of parakeratin at and yeast cells along with PMNs.
lesional area in conjunction with elongation of rete ridges. ♦♦ Often there is separation between the superficial pseu­
♦♦ Small collection of neutrophils, i.e. microabscess is seen in domembrane and the deeper layers of epithelium.
parakeratin layer and superficial spinous layer. ♦♦ Candidal hyphae often appear as a weakly basophilic
♦♦ Hyphae or mycelia and yeast cells are seen in parakeratin thread like structure.
layer of epithelium. ♦♦ Lamina propria is infiltrated by chronic inflammatory cells,
♦♦ There is presence of chronic inflammatory infiltrate i.e. lymphocytes and plasma cells.
cells such as lymphocytes and plasma cells immediately
subjacent to infected epithelium. Treatment
♦♦ Anti-fungal drugs, i.e. nystatin, amphotericin-B should
be given.
♦♦ Proper oral hygiene should be maintained.
Q.3. Write short note on burning mouth syndrome.
(Dec 2010, 4 Marks)
Ans.
• Burning mouth syndrome (BMS) is a painful, frus­
trating condition often described as a scalding sen­
sation in the tongue, lips, palate, or throughout the
mouth. Although BMS can affect anyone, it occurs
most commonly in middle-aged or older women.
• BMS often occurs with a range of medical and dental
conditions, from nutritional deficiencies and meno­
Fig. 76:  Candida albicans (For color version, see Plate 12)
pause to dry mouth and allergies. But their connec­
Treatment tion is unclear, and the exact cause of burning mouth
syndrome cannot always be identified with certainty.
♦♦ Topical and systemic administration of nystatin is done.
♦♦ In immunosuppressed patients systemic administra­tion
Signs and Symptoms
of amphotericin B and fluconazole is given.
♦♦ Improvement of oral hygiene is essential. Moderate to severe burning in the mouth is the main symptom
For investigations, refer to Ans 4 of same chapter. of BMS and can persist for months or years. For many people,
Q.2. Write short note on acute pseudomembranous candidi- the burning sensation begins in late morning, builds to a peak
asis. (Sep 2011, 3 Marks) by evening, and often subsides at night. Some feel constant
pain; for others, pain comes and goes. Anxiety and depression
Or are common in people with burning mouth syndrome and may

Write short note on oral thrush. (July 2016, 3 Marks) result from their chronic pain.
Section 3: Oral Pathology  575

Other symptoms of burning mouth syndrome include: • Chronic form:


–– Hyperplastic candidiasis
♦♦ Tingling or numbness on the tip of the tongue or in the mouth
–– Erythematous candidiasis
♦♦ Bitter or metallic changes in taste
–– Pseudomembranous candidiasis
♦♦ Dry or sore mouth.
• Candida associated lesion:
Causes –– Denture stomatitis
–– Angular stomatitis
There are a number of possible causes of burning mouth
–– Median rhomboid glossitis
syndrome, including:
• Keratinized primary lesion super-infected with candida:
♦♦ Damage to nerves that control pain and taste –– Leukoplakia
♦♦ Hormonal changes –– Lichen planus
♦♦ Dry mouth, which can be caused by many medicines and –– Lupus erythematosus
disorders such as Sjögren’s syndrome or diabetes ♦♦ Secondary candidiasis:
♦♦ Nutritional deficiencies • Candidal endocrinopathy syndrome.
♦♦ Oral candidiasis, a fungal infection in the mouth
♦♦ Acid reflux Laboratory Test for Candidiasis
♦♦ Poorly-fitting dentures or allergies to denture materials
♦♦ Fragments of plaque material are smeared on a microscopic
♦♦ Anxiety and depression.
slide, macerated with 20% potassium hydroxide and ex­
Diagnosis amined for typical hyphae.
A review of your medical history, a thorough oral examination, ♦♦ Gram stained smears from lesions or exudates show bud­
and a general medical examination may help identify the source ding Gram positive cells.
of your burning mouth. Tests may include: ♦♦ Sample can also be cultured on Sabouraud’s broth and or­
♦♦ Blood work to look for infection, nutritional deficiencies, dinary bacteriological culture. Colonies appear as creamy
and disorders associated with BMS such as diabetes or white, smooth and with yeasty odor.
thyroid problems ♦♦ Candida albicAns alone forms chlamydospores on cornmeal
♦♦ Oral swab to check for oral candidiasis agar culture at 20°C.
♦♦ Allergy testing for denture materials, certain foods, or other ♦♦ A rapid method of identifying Candida albicAns is based
substances that may be causing your symptoms. on its ability to form germ tubes within 2 hours when
incubated in human serum at 37°C.
Treatment
♦♦ By PAS method of staining candidal hyphae and yeasts
Treatment should be tailored to your individual needs. should be recognized. PAS method stains the carbohy­
Depending on the cause of your BMS symptoms, possible drates of fungal cell walls and organisms are identified by
treatments may include: bright magenta color. Hyphae are 2 µm in diameter, vary
♦♦ Adjusting or replacing irritating dentures in length and may show branching.
♦♦ Treating existing disorders such as diabetes, Sjögren’s
Q.5. Write short note on chronic atrophic candidiasis.
syndrome, or a thyroid problem to improve burning
mouth symptoms. (June 2014, 5 Marks)
♦♦ Recommending supplements for nutritional deficiencies Ans. It is also known as denture induced stomatitis.
♦♦ Switching medicine, where possible, if a drug you are It is a clinical manifestation of erythematous candidiasis.
taking is causing your burning mouth. Candida albicans is present in the lesion.
♦♦ Prescribing medications to:
• Relieve dry mouth Clinical Features
• Treat oral candidiasis ♦♦ Lesion is present under complete or partial denture.
• Help control pain from nerve damage ♦♦ Most commonly seen in females in palate.
• Relieve anxiety and depression. ♦♦ Patient complains of soreness and dryness of mouth.
When no underlying cause can be found, treatment is aimed at ♦♦ Lesion shows patchy distribution and is associated with
the symptoms to try to reduce the pain associated with burning speckled curd like white lesion.
mouth syndrome. ♦♦ Tissue of palate becomes bright red, edematous and
granular.
Q.4. Give classification of candidiasis and enumerate its ♦♦ Redness of mucosa is limited till the area covered by
laboratory tests. (Sep 2011, 5 Marks) denture.
Ans.
Investigations
Classification of Candidiasis by Axell Et Al 1997
Refer to Ans 4 of same chapter.
♦♦ Primary oral candidiasis:
• Acute form: Treatment
–– Pseudomembranous candidiasis Clotrimazole and nystatin ointments are applied after meal and
–– Erythematous candidiasis at bed time 3 to 4 times.
576   Mastering the BDS IIIrd Year  (Last 25 Years Solved Questions)

Q.6. Classify candidiasis. Describe in detail etiology, patho- Localized Predisposing Factors
genesis, clinical features, histopathology and treatment ♦♦ Marginal gingivitis
of acute pseudomembranous candidiasis. ♦♦ Poor oral hygiene
 (Dec 2015, 8 Marks) ♦♦ Faulty dental restorations
Ans. For classification of candidiasis refer to Ans 4 of same ♦♦ Deep periodontal pockets
chapter. ♦♦ Tobacco smoke
For etiology refer to Ans 1 of same chapter.
Clinical Features
For clinical features of acute pseudomembranous
candidiasis refer to Ans 1 of same chapter. ♦♦ Acute necrotizing ulcerative gingivitis usually occurs
For histopathology and treatment of acute pseudomembranous among young and middle aged adults, between the ages of
candidiasis refer to Ans 2 of same chapter. 16 and 30 years and males suffer more often than females.
♦♦ Stressed professionals like army recruits tend to suffer
Pathogenesis of Acute Pseudomembranous Candidiasis more from the disease.
♦♦ Moreover, young children suffering from malnutrition are
Overgrowth of yeast occurs on oral mucosa causing also prone to the disease.
desquamation of epithelial cells. This overgrowth also leads to ♦♦ Initially the gingiva becomes red, edematous, hemorrhagic
accumulation of bacteria, keratin, necrotic tissue and debris and and painful.
form a pseudomembrane which can closely adhere to mucosa. ♦♦ Later, on, a sharply demarcated “punched- out” crater-like
erosion of the inter-dental papillae occurs.
♦♦ Gingiva is often covered by a gray “pseudomembrane”
9. Diseases of with accumulation of necrotic tissue debris.
♦♦ Patient have pronounced spontaneous bleeding tendency,
periodontium exquisite pain and an extremely unpleasant fetid odor in
the mouth.
Q.1. Write note on acute necrotizing ulcerative gin­givitis. ♦♦ Patients often develop headache, fever, malaise and
(Mar 2003, 5 Marks) lymphadenopathy of the affected area.
Or ♦♦ Often there is difficulty in taking food due to increased
Write short note on acute necrotizing ulcerative gingi- salivation and a metallic taste in the mouth.
vitis. (Mar 2007, 2.5 Marks) (Sep 2011, 3 Marks) ♦♦ When the necrotizing process leads to the development
 (Mar 2016, 3 Marks) of periodontitis with loss of epithelial attachment the
condition is called necrotizing ulcerative periodontitis.
Or
♦♦ When the necrotizing process of ANUG extends further
Write short note on ANUG. (Nov 2014, 3 Marks)
through the oral mucosa and reaches to the extraoral skin
 (June 2014, 5 Marks) (Apr 2015, 3 Marks)
surface, the condition is called ‘noma’ or cancrum oris.
Ans. It is an endogenous oral infection which is characterized
by the necrosis of gingiva. Histopathology
It is also called as Trench mouth, Vincent’s infection,
acute ulceromembranous gingivitis and acute ulcerative ♦♦ It involves both stratified squamous epithelium and
gingivitis. underlying connective tissue.
♦♦ Surface epithelium is destroyed and is replaced by pseudo-
Etiology membranous meshwork of fibrin, necrotic epithelial cells,
polymorphonuclear neutrophils and various microorgan­
It is caused by the fusiform bacilli and spirochete. isms.
Predisposing Factors ♦♦ Underlying connective tissue is hyperemic with numerous
engorged capillaries and dense infiltration of polymorpho­
Systemic Predisposing Factors nuclear neutrophils.
♦♦ Numerous plasma cells may appear in periphery of in­
♦♦ Malnutrition: This may lead to acute necrotizing ulcerative filtrate.
gingivitis
Treatment
♦♦ Nutritional deficiency: Deficiency of Vitamin C, Vitamin B1,
Vitamin B2 leads to the exaggeration of pathologic changes ♦♦ Conservative treatment is employed, i.e. superficial
caused by fusospirochetal bacteria. cleaning of oral cavity by chlorhexidine, diluted hydrogen
♦♦ Psychosomatic conditions: Disease is associated with the peroxide or warm salt water. This is followed by scaling
stress and with increment in adrenocortical secretion. and polishing. Topical anesthetics are used to produce
♦♦ Diseases like leukemia, syphilis, AIDS and gastrointestinal pain during procedure.
disturbances also leads to ANUG. ♦♦ Use of antibiotics is coupled with local treatment.
Section 3: Oral Pathology  577

Q.2. Write short note on etiology of gingivitis. ♦♦ Psychosomatic conditions: Disease is associated with the
(Sep 2005, 5 Marks) stress and with increment in adrenocortical secretion.
Ans. Gingivitis is the inflammation of gingiva and occurs in ♦♦ Diseases like leukemia, syphilis, AIDS and gastrointestinal
acute, sub acute and chronic form. disturbances also leads to ANUG.

Etiology Localized Predisposing Factors


♦♦ Local Factors ♦♦ Marginal gingivitis
• Microorganisms: Plaque associated with gingivitis is ♦♦ Poor oral hygiene
complex and heterogeneous. Actinomyces group of ♦♦ Faulty dental restorations
organisms are dominant in supragingival plaque. ♦♦ Deep periodontal pockets
• Calculus: Supragingival or subgingival or both causes
♦♦ Tobacco smoke
irritation of contacting gingival tissue. Irritation is
produced by the mechanical friction resulting form Clinical Features
hard, rough surface of calculus.
• Food impaction: Impaction of food and accumu­lation ♦♦ Acute necrotizing ulcerative gingivitis usually occurs
of debris on teeth due to oral negligence results in among young and middle aged adults, between the ages of
gingivitis. 16 and 30 years and males suffer more often than females.
• Faulty and irritating restorations or appliance may ♦♦ Stressed professionals like army recruits tend to suffer
act as irritant to gingival tissue and induce gingivitis. more from the disease.
• Mouth breathing: It leads to drying of oral mucus ♦♦ Moreover, young children suffering from malnutrition are
membrane which results in gingival irritation with also prone to the disease.
accompanying irritation. ♦♦ Initially the gingiva becomes red, edematous, hemorrhagic
♦♦ Systemic factors and painful.
• Nutritional disturbances: The nutritional imbalance if ♦♦ Later, on, a sharply demarcated “punched- out” crater-like
frequently manifested leads to changes in gingiva. erosion of the inter-dental papillae occurs.
• Pregnancy: During pregnancy gingiva undergoes ♦♦ Gingiva is often covered by a gray “pseudomembrane”
changes causing pregnancy gingivitis. with accumulation of necrotic tissue debris.
• Diabetes mellitus ♦♦ Patient have pronounced spontaneous bleeding tendency,
• Hormonal changes: Puberty and menstruation may exquisite pain and an extremely unpleasant fetid odor in
result in gingival inflammation. the mouth.
Q.3. Describe in detail necrotizing ulcerative gingivitis. ♦♦ Patients often develop headache, fever, malaise and lym­
(Sep 2011, 8 Marks) phadenopathy of the affected area.
Ans. It is an endogenous oral infection characterized by ♦♦ Often there is difficulty in taking food due to increased
necrosis of gingiva. It is also called as trench mouth. salivation and a metallic taste in the mouth.
♦♦ When the necrotizing process leads to the development of
Etiopathogenesis periodontitis with loss of epithelial attachment the condi­
Proliferation of anaerobic fusiform bacteria and spirochaetes tion is called necrotizing ulcerative periodontitis.
results in ANUG. Infection mostly occurs in presence of ♦♦ When the necrotizing process of ANUG extends further
pschycological stress. Stress related corticosteroid hormones through the oral mucosa and reaches to the extraoral
are thought to alter T4/T8 lymphocyte ratios and may cause skin surface, the condition is called ‘noma’ or cancrum
the decreased neutrophilic chemotaxis and phagocytic response oris.
seen in patients with ANUG. Stress related epinephrine may
Investigations
result in localized ischemia which predisposes gingiva to
ANUG. Investigations are associated with testing of fusiform bacilli
Following are the predisposing factors for acute necrotizing and spirochaetes:
ulcerative gingivitis: ♦♦ Spirochetes and fusiform bacilli are demonstrated in
♦♦ Systemic predisposing factors stained smears of exudates from the lesion on microscopic
♦♦ Local predisposing factors examination. The spirochete T.pallidum is visible on dark
ground microscopy.
Systemic Predisposing Factors
♦♦ Direct fluorescent antibody test is done for detection of
♦♦ Malnutrition: This may lead to acute necrotizing ulcerative spirochete T. pallidum
gingivitis ♦♦ Enzyme immunoassays are also done to detect for spiro­
♦♦ Nutritional deficiency: Deficiency of Vitamin C, Vitamin B1, chetes present in ANUG.
Vitamin B2 leads to the exaggeration of pathologic changes ♦♦ Antibody tests, i.e. detection of specific IgM antibody is
caused by fusospirochetal bacteria. helpful in detection of Treponema.
578   Mastering the BDS IIIrd Year  (Last 25 Years Solved Questions)

Differential Diagnosis Histopathology


♦♦ Benign mucous pemphigoid: In this as compared to ANUG, ♦♦ Affected gingiva show inflammation, ulceration and
it is seen in elderly people and no necrosis is evident. necrosis.
♦♦ Pemphigoid lichen planus: It does not show any acute course, ♦♦ Gingival stratified squamous epithelium is replaced by
no foul breadth is observed. thick fibrinopurulent pseudomembrane.
♦♦ Pemphigus: It present a very regular histology. Seen in ♦♦ Pseudomembrane shows PMNs and necrotic debris.
older people. ♦♦ In unaffected areas of gingiva keratinization is absent.
♦♦ Syphilitic gingivitis: Lesion is seen on gingiva and it does ♦♦ Underlying connective tissue show hyperemia and acute
not get spread to adjacent gingiva. inflammatory cells i.e. PMNs.
♦♦ Streptococcal gingivostomatitis: Diffuse erythema is seen Q.5. Classify gingival hyperplasia. Describe clinical
on posterior parts of oral mucosal lining and no necrosis features and histopathology of idiopathic gingival
is evident. hyperplasia. (Dec 2009, 15 Marks)
♦♦ Gingivostomatitis by candida: Covering of white membrane Ans.
is seen which can be removed by scrapping. On laboratory
investigation the membrane reveals of candida. Classification of Gingival Hyperplasia

Complications On basis of etiological factors and pathologic changes


♦♦ Inflammatory enlargement
Patient affected by ANUG develop systemic complications • Chronic
such as: • Acute
♦♦ Pulse rate get increased ♦♦ Drug induced enlargement
♦♦ High fever ♦♦ Enlargement associated with systemic disease
♦♦ Loss of appetite • Conditional enlargement
♦♦ Generalized lassitude. –– Pregnancy
–– Puberty
Treatment –– Vitamin C deficiency
♦♦ The involved areas are isolated with cotton rolls and dried. –– Plasma cell gingivitis
A topical anesthetic is applied and after 2 to 3 minutes, –– Non-specific conditioned enlargement (Pyogenic
areas are gently swabbed with a cotton pellet to remove granuloma)
pseudomembrane. After the area is cleaned with warm • Systemic diseases causing gingival enlargement
water, superficial calculus is removed. –– Leukemia
♦♦ Patient is told to rinse the mouth every 2 hours with glass­ –– Granulomatous disease (E.g. Wegener’s Granu­
ful of an equal mixture of warm water and 3% hydrogen lomatosis, sarcoidosis)
peroxide. Twice daily rinsing with 0.12% chlorhexidine • Neoplastic enlargement
are effective. –– Benign tumors
♦♦ Penicillin V 250 or 500 mg, 6 hourly or erythromycin 250 –– Malignant tumors
or 500 mg, 6 hourly are given. • False enlargement
♦♦ Scaling is performed, if sensitivity permits. After disease Using the criteria of location and distribution gingival
process is diminished, complete gingival curettage and enlargement is designated as follows:
root planing is done. – Localized: Gingival enlargement limited to one or
♦♦ Supportive treatment consists of copious fluid consu­ more teeth.
mption and administration of nutritional supplements. – Generalized: Involving the gingiva throughout the
mouth.
Q.4. Describe in detail the clinical features, etiology and – Marginal: Confined to marginal gingiva.
histopathology of necrotizing ulcerative gingivitis. – Papillary: Confined to interdental papilla.
(May/Jun 2009, 15 Marks) – Diffuse: Involving the marginal and attached
Ans. For clinical features refer to Ans 3 of same chapter. papillae.
– Discrete: Isolated sessile or pedunculated tumor like
Etiology enlargement.
♦♦ It is caused by fusiform bacilli and by a spirochete known On basis of degree of gingival enlargement
as borrelia vincentii. Grade 0: No sign of gingival enlargement.
♦♦ Mainly precipitating factors are there which lead to Grade I: Enlargement confirmed to interdental papilla
necrotizing ulcerative gingivitis. For precipitating factors Grade II: Enlargement involves papilla and marginal
in detail refer to Ans 3 of same chapter. gingiva.
Section 3: Oral Pathology  579

Grade III: Enlargement covers three quarters or more of


Histology
the crown.
♦♦ Surface epithelium exhibits long, thin rete ridges that
Idiopathic Gingival Hyperplasia extend deep into underlying fibrous connective tissue.
♦♦ Connective tissue is dense hypocellular and hypovascular.
♦♦ Idiopathic gingival enlargement is a rare condition of
♦♦ Collagen fibers are present in form of bundles which are
undetermined cause.
interspersed with fibroblasts.
♦♦ It is also known as gingivomatosis or elephantiasis
gingivae or idiopathic fibromatosis or hereditary gingival ♦♦ Inflammation is mild to absent.
hyperplasia or congenital fibromatosis. ♦♦ At times scattered islands of odontogenic epithelium, foci
of dystrophic calcification or areas of osseous metaplasia
Clinical Features may be seen.
♦♦ Enlargement affects attached gingiva, gingival margin and Treatment
interdental gingiva.
♦♦ Facial and lingual surfaces of mandible and maxilla are Gingivectomy should be done.
affected. Q.7. Write short note on gingival enlargement.
♦♦ Involvement may be limited to one jaw.  (June 2015, 5 Marks)
♦♦ Enlarged gingiva is pink, firm and leathery in consistency
Or
and has minutely pebbled surface.
♦♦ In severe cases teeth are completely covered by enlarged Write short note on gingival hyperplasia.
gingiva.  (Jan 2017, 5 Marks)
Ans. Gingival enlargement is the increase in size of gingiva.
Histopathology For classification of gingival enlargement refer to Ans 5
♦♦ Epithelium is hyperplastic with elongation of rete ridges. of same chapter.
♦♦ Mild hyperkeratosis is also seen.
♦♦ Underlying connective tissue stroma consists of dense Gingival Hyperplasia Associated with Vitamin C Deficiency
bundles of collagen and numerous fibroblasts. Vitamin C deficiency leads to scurvy and the disease has
♦♦ At times presence of chronic inflammatory infiltrate is seen. following manifestations:
Q.6. Write short note on gingival fibromatosis. ♦♦ Gingiva becomes tender, edematous and swollen.
(Feb 2014, 3 Marks) ♦♦ Frequent bleeding is present from gingiva.
Ans. It is also known as hereditary gingival fibromatosis or ♦♦ Crest of the inter-dental papillae appear reddish or purple
elephantiasis gingivae. in color.
Gingival fibromatosis is the condition characterized by ♦♦ Ulceration and necrosis of gingiva occur.
the diffuse gingival enlargement and it also covers major ♦♦ Foul smell is present from the mouth.
parts of total tooth surfaces. Gingival Hyperplasia Associated With Leukemia
It is the slowly progressive gingival enlargement
caused by collagenous overgrowth of gingival fibrous ♦♦ Gingiva becomes soft, edematous and swollen.
connective tissue. ♦♦ Gingiva is painful and has purplish and has glossy ap­
It may be familial or idiopathic. pearance.
♦♦ Surrounding mucosa is pale and petechiae is seen.
Clinical Features ♦♦ Ulceration and severe hemorrhage occur on gingiva.
♦♦ Gingival enlargement starts before age of 20 years and it
is correlated with eruption of deciduous and permanent Gingival Hyperplasia Associated With Endocrine Imbalance
teeth. Hormonal imbalance mostly increases the proliferative potential
♦♦ It occurs as diffuse or nodular growth of gingiva over of gingival tissue in response to irritation caused by plaque,
maxillary or mandibular arch. Maxilla is affected more bacteria and local irritants. Following manifestations develop
frequently. clinically:
♦♦ Surface of gingiva is pebbled.
♦♦ Gingiva becomes red, edematous and swollen.
♦♦ Gingiva is pink or pale in color.
♦♦ Frequent bleeding is present.
♦♦ Consistency of gingiva is firm and leathery.
♦♦ Gingiva is non-tender and does not bleed. ♦♦ At times localized tumor growth develops on gingiva
♦♦ Gingival swelling leads to spacing between the teeth. during pregnancy.
♦♦ Extension of gingiva is so severe that it can cover the crown ♦♦ Gingiva becomes soft, edematous and swollen.
of the erupted tooth. ♦♦ Gingiva becomes soft, edematous and swollen.
580   Mastering the BDS IIIrd Year  (Last 25 Years Solved Questions)

Gingival Hyperplasia Associated With Dilantin Sodium face. Anterior and facial segments are more frequently
Therapy involved areas. Enlargement is painless.
♦♦ In extensive cases of hyperplasia gingiva cover almost
It causes gingival hyperplasia as its side effect. Manifestations
complete crown surface.
associated are:
♦♦ If there is extension of gingiva lingually and occlusally this
♦♦ Presence of painless enlargement of interdental papilla. can interfere with speech and mastication.
♦♦ Swelling is rough, lobulated and has a pebbled surface. ♦♦ Edentulous areas are usually not affected but hyperplasia is
♦♦ Gingiva is normal in color and increased stippling is seen under poorly maintained dentures as well as around
present. the implants.
♦♦ Gingiva is firm and tender and there is no tendency to ♦♦ Non-gingival soft tissue growths which look like pyogenic
bleed. granulomas are seen in allogenic bone marrow transplant
♦♦ As drug therapy is stopped gingival growth ceases. recipients who are taking cyclosporine for graft versus
host disease.
Idiopathic Gingival Enlargement ♦♦ Enlarged gingiva is normal in color and is firm, it has
smooth, stippled or granular surface.
For idiopathic gingival enlargement refer to Ans 5 of same
♦♦ If inflammation occurs affected gingiva is dark red and
chapter.
edematous, surface become friable, bleeds easily and oc­
Q.8. Classify gingival enlargement. Give a detailed account casionally ulcerated.
of enlargement caused by drugs. (Jan 2018, 10 Marks)
Ans. For classification of gingival enlargement refer to ans5 Histopathology
of same chapter. ♦♦ Gingival overgrowth caused by phenytoin shows redun­
dant tissue of normal composition.
Gingival Enlargement Caused by Drugs ♦♦ Gingival overgrowth caused by cyclosporine shows in­
♦♦ Drug related gingival enlargement is an abnormal growth crease amount of collagen per unit volume with normal
of gingival tissue which is secondary to the use of systemic density of fibroblast.
medication. ♦♦ Overlying surface epithelium show elongation of rete
♦♦ Drug related gingival enlargement is a misnomer because ridges with long extension in underlying lamina propria.
neither the epithelium and nor the cells inside the connec­ ♦♦ Patients with secondary inflammation show increased vas­
tive tissue undergo hyperplasia or hypertrophy. Increase cularity and increased chronic inflammatory cell infiltrate
in the size of gingiva is because of increased amount of composed of lymphocytes and plasma cells.
extracellular matrix mainly collagen.
♦♦ Strong association of drug induced gingival enlargement Treatment
is related to three drugs, i.e. cyclosporine, phenytoin and ♦♦ Discontinue the offending medication which leads to
nifedipine. All the drugs causing gingival enlargement lead regression in size of gingiva.
to calcium dysregulation which disrupt normal collagen ♦♦ If drug substitution can be done than cyclosporine is re­
phagocytosis and remodeling process. placed with tacrolimus; phenytoin with carbamazepine,
♦♦ Prevalence related to use of phenytoin is 50% while cyclo­ lamotrigine, gabapentin etc and nifedipine with atenolol.
sporine and nifedipine produce changes in 25%. ♦♦ Professional cleaning, frequent re-evaluation and home
♦♦ Degree of drug induced gingival enlargement is related plaque control is done.
significantly to patient’s susceptibility and level of oral ♦♦ Systemic or topical folic acid is beneficial in some cases.
hygiene. ♦♦ Cyclosporine induced gingival hyperplasia resolve after
♦♦ Degree of drug induced gingival hyperplasia is higher in short course of metronidazole or azithromycin.
smokers. ♦♦ When objectionable alterations are present and all other
♦♦ Dilantin sodium was the first drug reported to cause gin­ interventions fail, gingivectomy is the treatment of choice.
gival enlargement.
Clinical Features
♦♦ Phenytoin is commonly given in young patients so it in­ 10. Dental Caries
duces a problem in people younger than 25 years of age.
Patients taking calcium channel blockers are of middle age
and cyclosporine has a broad age range. Q.1. Describe in brief etiology of dental caries.
♦♦ As any of the drug is used for 1 to 3 months enlargement (Sep 2008, 3 Marks)
occur in interdental papillae and spread to the tooth sur­ Ans. Following is the etiology of dental caries:
Section 3: Oral Pathology  581

Host factors Components Urea increases the neutralizing power of saliva. Enzyme
salivary amylase leads to the degradation of starch and
Tooth • Composition
makes it more soluble in this way starch is washed away
• Morphologic characteristics
• Position from tooth surface.
Saliva • Composition pH
• pH
• Quantity ♦♦ Critical pH is the pH at which saliva appears to be satu­
• Viscosity rated with the calcium and phosphorus ions. Value of
• Anti-bacterial factors critical pH is 5.5, below the critical pH inorganic portion of
Diet • Physical factors tooth starts dissolving. As there is increase in concentration
• Local factors of hydrogen ion in cariogenic plaque, this leads to the loss
Systemic conditions of more phosphate ions from the tooth.
♦♦ Buffering property of saliva leads to diffusion of bicarbo­
Tooth nate ions in dental plaque and neutralizes the acid during
caries process.
Composition
♦♦ Structure and composition of the teeth influence initiation Quantity
and rate of progression of dental caries. ♦♦ Quantity of saliva is inversely proportional to dental car­
♦♦ Surface enamel is more resistant to caries as compared
ies activity.
to subsurface enamel. Surface enamel consists of more
♦♦ More is the salivary flow less is the caries index.
quantity of fluoride, zinc, lead and iron. Concentration
of carbonate, magnesium and sodium is lower in surface ♦♦ Hyposalivation occurs due to the conditions like diabetes
layer. Level of carbon dioxide is also lower in the surface mellitus, uremia and usage of anti-sialogogues.
layer which causes dissolution of surface layer by acids in Viscosity
lower rate and it consists of less organic and water content.
♦♦ Age changes in enamel such as decrease in the density and If saliva is thick mucinous there is presence of high caries
permeability and increase in nitrogen and fluoride content incidence.
causes teeth to become more caries resistant.
Anti-bacterial Factors
Morphologic Characteristic
♦♦ Saliva consists of many anti-bacterial products such as
♦♦ Deep and narrow occlusal fissures or buccal and lingual lysozyme, salivary peroxidase and immunoglobulins.
pits lead to development of dental caries. ♦♦ Lysozyme under the presence of sodium lauryl sulphate
♦♦ As age increases attrition of teeth occur and this leads to can lyse cariogenic streptococci.
less accumulation of food in fissures, and there is less oc­ ♦♦ Salivary peroxidase inactivates bacterial enzymes of gly­
currence of caries. colytic pathway and inhibits their growth. This is more
Position effective against lactobacillus bacteria.
♦♦ IgA immunoglobulin inhibits S. mutAns in saliva.
Malaligned tooth or rotated tooth has more chances of
predisposition of caries as it tends to accumulate more food Diet
debris, cariogenic plaque and bacteria. In all these teeth cleaning
cannot be done. Physical Factors

Saliva ♦♦ Raw unrefined food consists of roughage which cleAns


Composition the teeth but presence of soil and sand leads to attrition
of occlusal and proximal surfaces of teeth and reduces
♦♦ Inorganic components of saliva: In normal aspect saliva is dental caries.
supersaturated with calcium and phosphate ions. This
♦♦ Soft and refined foods stick to the teeth and causes in­
causes prevention of dissolution of enamel and also pre­
creased accumulation of debris which causes increased
cipitates apatite crystals in surface of enamel of carious
risk of dental caries.
lesion which helps in partial repair of tooth damaged by
dental caries. During caries saliva is unsaturated with Local Factors
calcium and phosphate ions which lead to dissolution of
enamel. Fluoride has also got excellent role in reduction ♦♦ Carbohydrates, i.e. starch, sucrose, lactose, glucose, fruc­
of the dental caries. tose or maltose play important role in process of dental
♦♦ Organic components of saliva: High concentration of am­ caries. Synthesis of extracellular polysaccharides, glucAns
monia retards plaque formation and neutralizes acid. and levan helps in adherence of bacteria to teeth.
582   Mastering the BDS IIIrd Year  (Last 25 Years Solved Questions)

♦♦ In lipids medium chain fatty acids and their salts have 2. Microorganisms
anti-bacterial properties at low pH. 3. Acids
♦♦ Deficiency of Vitamin A and D can lead to enamel hypo­ 4. Dental plaque.
plasia which can lead to dental caries in affected teeth.
i. Role of Carbohydrates
Systemic Conditions ♦♦ Fermentable dietary carbohydrates play an important role
♦♦ Hereditary: There is possibility of dental caries which leads in the causation of caries, e.g. glucose, fructose and sucrose.
to the inheritance of tooth form or structure which predis­ ♦♦ Among them sucrose is more potent.
poses to dental caries. ♦♦ These sugars are easily and rapidly fermented by cariogenic
♦♦ Pregnancy: In later stages of pregnancy because of lack of bacteria in the oral cavity to produce acid at or near the
oral hygiene there is increased risk of dental caries. tooth surface and causes dissolution of the hydroxyapatite
crystal of the enamel followed by the dentin.
Q.2. Describe the theories of etiology of dental caries.
♦♦ Risk of caries incidence increases greatly if the dietary
(Mar 2003, 15 Marks) sugar is sticky in nature which remains adheres to the tooth
Or surface for long time after taking the meal.
Define dental caries and theories associated with dental ♦♦ Following the ingestion of these sugars the pH of the
Plaque falls to 4.5 to 5 within 1 to 3 minutes and neutra­
caries. (Sep 2009, 8 Marks)
lization occurs after 10 to 30 minutes.
Or ♦♦ Glucose, sucrose and fructose, etc. are rapidly defused into
Describe the theories of dental caries. the plaque due to their low molecular weight.
(Aug/Sep 1998, 15 Marks) Role of Microorganisms
Ans. “Dental caries is an irreversible progressive microbial ♦♦ A large number of microorganisms play individual role
disease of the calcified tissues of the teeth, characterized in dental caries production and among them the most
by the demineralization of the inorganic portion and important one is Streptococcus mutans.
distortion of the organic substances of the tooth, which • It readily ferments the dietary carbohydrate to
often leads to cavitation”. produce acid, which causes tooth destruction.
• Etiology of dental caries is a very complex process, • It synthesizes dextran from sucrose, which helps in
which is often explained with the help of some adhering the plaque bacteria as well as the acid on to
theories. the tooth surface.
– Acidogenic theory • S. mutAns has the ability to adhere and to grow on
– Proteolytic theory hard and smooth surface of the teeth.
– Proteolytic chelation theory ♦♦ Actinomycosis group, e.g. Actinomycosis israelii, Actino-
– Sucrose chelation theory mycosis viscosus, etc. are the important organisms to cause
– Autoimmune theory. root caries.
♦♦ Lactobacillus acidophilus is important organism for the
Acidogenic Theory progress of dental caries.
♦♦ This theory is also known as Miller’s chemico- parasitic Role of Acids
theory.
♦♦ During the process of caries formation, a large variety of
♦♦ It proposes that acid formed due to the fermentation of
acids are produced in the oral cavity due to the bacterial
dietary carbohydrates by oral bacteria leads to progressive
fermentation of dietary carbohydrate.
decalcification of the tooth. Structures with subsequent ♦♦ These acids are lactic acid, aspartic acid, acetic acid, butyric
degeneration of the organic matrix. acid, glutamic acid.
♦♦ Acidogenic theory states that the process of dental caries ♦♦ They can cause demineralization of enamel and dentin
involves two stages. and causes the tooth decay.
1. Initial stage: Production of organic acid occurs as a
result of fermentation of the carbohydrates by the Role of Bacterial Plaque
Plaque bacteria. ♦♦ Plaque is a thin, transparent film produced on tooth surface
2. Later stage: Acid causes decalcification of enamel and it consists of microorganisms suspended in salivary
followed by dentin and thereby causes total destruction mucin, also contain desquamated epithelial cells, leuko­
of these two along with dissolution of their softened cytes and food debris, etc.
residues. Final result is cavity formation. ♦♦ The dental plaque helps in initiation of dental caries by:
♦♦ According to Miller there are four important factors, • It harbors the cariogenic bacteria on the tooth surface.
which can influence the process of tooth destruction in • It holds the acids on the tooth surface for long duration
dental caries. • It protects the acids from getting neutralized by
1. Dietary carbohydrates buffering action of saliva.
Section 3: Oral Pathology  583

Proteolytic Theory Histopathogical Features of Caries in Enamel


According to this theory, the proteolytic enzymes liberated Early Enamel Caries
by cariogenic bacteria causes destruction of organic matrix of
♦♦ There will be loss of inter prismatic or inter rod substances
enamel, as a result of which the inorganic crystals of the enamel
with increase in prominence of these enamel rods.
become detached form one another and finally the whole
♦♦ Dark line often appears at right angles of the enamel rods,
structure collapse, leading to a cavity formation.
suggesting segments.
This theory cannot explain the role of sucrose, pH, fluoride, ♦♦ Accentuation of the incremental striae of retzius often
etc. in dental caries. occurs.
Proteolysis-chelation Theory Advanced Enamel Caries
♦♦ According to this theory during caries, first of all pro­ ♦♦ It presents several zones in the tissues, out of which four
teolytic breakdown of the organic portion of the enamel zones are clearly visible, starting from the inner advancing
matrix takes place. front of the lesion the zones are:
♦♦ Following this a chelating agent is formed by the combi­
nation of the proteolytic breakdown products, acquired
pellicle and food debris, etc.
♦♦ Chelating agent (negatively charged) release the calcium
ion (positively charged) from enamel and dentin, this
process is called as chelation and eventually results in
tooth decay.

Sucrose Chelation Theory


♦♦ This theory propose that if there is very high concentra­
tion of sucrose in the mouth, there can be formation of
complex substances like calcium saccharide and calcium
complexing intermediaries.
♦♦ These complexes causes release of calcium and phosphorus
ions from the enamel and thereby results in tooth decay.
Fig. 77:  Dentinal caries H and E (DS) (For color version, see Plate 13)
Autoimmune Theory
Zone I: Translucent Zone
This theory suggests that few odontoblast cells at some specific
♦♦ It is the deepest zone lies at the advancing front of the
sites, within the pulp of few specific teeth are damaged by
enamel lesion.
autoimmune mechanisms.
♦♦ This zone is more porous than normal enamel.
♦♦ Pores are larger than the normal enamel.
Current Concept of Caries Etiology
♦♦ Pore volume is 1%.
♦♦ Dental caries is a multifactorial disease in which there is ♦♦ This zone appears structureless.
inter play of three primary factors—the host, microflora ♦♦ This zone contains more fluoride than normal enamel.
and the substrate. In addition a fourth factor, i.e. time.
Zone II: Dark Zone
♦♦ Caries formation requires a host, a cariogenic flora and
a suitable substrate that must be present for a sufficient ♦♦ Dark zone is located just superficial to the translucent zone
length of time. and its dark appearance is due to the excessive deminer­
alization of the enamel.
Q.3. Define dental caries describe histopathology of caries ♦♦ Zone is narrower in rapidly advancing caries and it is wider
in enamel and dentin. in slowly advancing lesion.
(Dec 2009, 15 Marks) (Feb 1999, 15 Marks) ♦♦ Zone contains 2 to 4% pore volume.
Or ♦♦ Pores are larger than normal but smaller than those of
translucent zone.
Describe in brief histopathology of dental caries.
♦♦ This zone reveals some degree of remineralization of
(June 2010, 5 Marks)
carious lesion.
Ans. “ D en t a l c a r i e s i s a n i r r e ve r s i b l e p r ogre s s ive
microbial disease of the calcified tissues of the Zone III: Body of Leison
teeth, characterized by the demineralization of ♦♦ Zone is situated between the dark zone and the surface
the inorganic portion and distortion of the organic layer of enamel.
substances of the tooth, which often leads to ♦♦ It represents the area of greatest demineralization.
cavitation”. ♦♦ Pore volume is 5 to 25%.
584   Mastering the BDS IIIrd Year  (Last 25 Years Solved Questions)

♦♦ This zone contains appetite crystals larger than those of ♦♦ In the process, the entire dentinal structures become
the normal enamel. destroyed and cavitation begins from dentino enamel
♦♦ Large crystals results from the reprecipitation of minerals junction.
dissolved from the deeper zone. Q.4. Describe the histopathology of caries in dentin. 
Zone IV: Surface Zone  (Feb 2002, 8 Marks)
Or
♦♦ Surface zone when examined by the polarizing light ap­
pears relatively unaffected, it may be due to the surface Describe histopathology of dentinal caries.
remineralization by the salivary mineral ions.  (Mar 2006, 5 Marks)
Or
Histological Features of Caries in Dentin/Dentinal Caries Write short note on histopathology of dentinal caries.
Dentinal caries histologically presents five zones in the tissue,  (Jan 2017, 5 Marks)
which are: Ans. Refer to Ans 3 of same chapter.
Q.5. Describe dental caries and describe chemical and his-
Zone I: Normal Dentin
topathological characteristics of caries in enamel.
♦♦ This zone represents the inner most layer of the carious (Sep 2004, 15 Marks)
dentin, here the dentinal tubules appears normal. Ans. “Dental caries is an irreversible progressive microbial
♦♦ There is evidence of fatty degeneration of the Tome’s disease of the calcified tissues of the teeth, characterized
process. by the demineralization of the inorganic portion and
♦♦ No crystals in the lumen of the tubules. distortion of the organic substances of the tooth, which
♦♦ No bacteria in the tubules. often leads to cavitation”.
♦♦ Inter tubular dentin has normal cross banded collagen and
normal dense appetite crystals. Chemical Characteristics
Zone II: Sub-transparent Dentin Refer to acidogenic theory in Ans 2 of same chapter
♦♦ This is the zone of dentinal sclerosis and is characterized For histopathological characteristics refer to Ans 3 of same
by the deposition of very fine crystal structures within the chapter.
dentinal tubules. Q.6. Write note on caries in dentin. (Sep 1999, 5 Marks)
♦♦ Superficial layer shows area of demineralization and dam­ Or
age of the odontoblastic processes.
Write short note on dentinal caries.
♦♦ No bacteria in the tubules.
(Dec 2010, 3 Marks)
♦♦ Dentin is capable of remineralization.
Or
Zone III: Transparent Dentin Write note on dentinal caries.
♦♦ This zone appears transparent and this is because of de­ (March 2007, 2.5 Marks)
calcification of dentin. Ans. Caries of the dentin begins with the natural spread of
♦♦ It is softer than normal dentin. process along the DEJ and the rapid involvement of
♦♦ No bacteria in tubules. the great number of dentinal tubules, each of which
♦♦ Cross banded inter tubular collagen is still intact. act as a tract leading to dentinal pulp along which the
♦♦ This zone is capable of self repair and remineraliza­tion. microorganisms may travel at a variable rate of spread.
Also refer to Ans 3 of same chapter.
Zone IV: Turbid Dentin
Q.7. Write short essay on caries activity test.
♦♦ This zone is marked by widening and distortion of dentinal
(Jan 2012, 5 Marks)
tubules, which are packed with micro­organisms.
Or
♦♦ There is very little amount of minerals in dentin, denatura­
Write short note on caries activity test.
tion of collagen fibers also takes place.
 (Dec 2015, 5 Marks)
♦♦ Zone cannot undergo self repair or remineraliza­tion.
♦♦ This zone must be removed before restoration. Ans. A number of caries activity test have been evolved which
are as follows:
Zone V: Infected Dentin
Snyder Test
♦♦ This is the outermost zone of the carious dentin.
♦♦ It is characterized by complete destruction of dentinal ♦♦ This test measures the ability of salivary micro­organisms
tubules. to produce organic acids from carbo­hydrate metabolism.
♦♦ In this zone the area of decomposition of dentin, which ♦♦ Glucose agar media containing an indicator dye, i.e. Bro­
occur along the direction of dentinal tubules are called mocresol green is useful.
“Liquefaction foci of Miller”, which occur perpendicular ♦♦ The indicator dye changes from green to yellow in range
to dentinal tubules are called “Transverse clefts”. of Ph between 5.4 to 3.8
Section 3: Oral Pathology  585

♦♦ Paraffin stimulated saliva is added into the medium, Fosdick Calcium Dissolution Test
change of the medium from green to yellow is indicative
25 mL of gum stimulated saliva is collected. Part of this is
of degree of caries activity.
analyzed for calcium content. The rest is placed in an 8 inch
Salivary Reductase Test sterile test tube with about 0.1g of powdered human enamel. The
tube is sealed and shaken for four hours at body temperature
♦♦ It measures the activity of Reductase enzyme present in
after which it is again analyzed for calcium content. If paraffin is
salivary bacteria.
used, a concentration of about 5% glucose is added. The amount
♦♦ Paraffin stimulated saliva is collected in the plastic con­
of enamel dissolution increases as the caries activity increases.
tainer and an indicator dye “Diazoresorcinol” is added to
it which colors the saliva blue. Dewar Test
♦♦ The reductase enzyme liberated by the cariogenic bacteria
causes color changes in the medium from blue to other This test is similar to the Fosdick calcium dissolution test except
colors, which indicates the caries “conductiveness” of the that the final pH after four hours is measured instead of the
patients. calcium dissolved.

Microbiological Test Swab Test


♦♦ Microbiological test helps to measure the number of Strep- The swab test involves sampling of the oral flora by swabbing
tococcus mutAns and Lactobacillus acidophilus. buccal tooth surfaces and placing it in the Snyder media. This
♦♦ Two samples of paraffin stimulated saliva is collected and is incubated for 48 hours and the pH changes are read and
diluted 10 times and each is cultivated in two different correlated with the caries activity.
special media.
♦♦ Rogosa’s SL agar medium for Lactobacillus. Interpretation
♦♦ Mit’s Salivarius agar medium for S. mutans. pH 4.1 and less than 4.1—Marked caries activity
♦♦ After incubation the numbers of colonies that develop in
pH 4.2–4.6—Active
two separate media are counted and then are multiplied
by 10 to estimate the number of bacteria in 1 ml of saliva. pH 4.5–4.6—Slightly active
♦♦ Result: If count is more than 10, 00,000 S. mutAns and more pH greater than 4.6—Caries active.
than 1,00,000 L. acidophilus, than the caries susceptibility
Q.8. Define dental caries. Enumerate the theories asso­
of individual is very high.
ciated with dental caries. Explain different measures
Alban’s Test in preventing and controlling dental caries.
It is the modification of synder test. It uses less agar, i.e. 5ml per (Mar 2000, 15 Marks)
tube. The saliva is drooled directly into the tubes and the tubes Ans. “Dental caries is an irreversible progressive microbial
are incubated for 4 days at 37°C. The color change is noted from disease of the calcified tissues of the teeth, characterized
bluish green to yellow and the depth to which the change has by the demineralization of the inorganic portion and
occurred is noted. distortion of the organic substances of the tooth, which
often leads to cavitation”.
Strip Test for S.mutAns Level in Saliva For theories refer to Ans 1 of same chapter.
Saliva/plaque samples are obtained by using tongue blades and
toothpicks (after air drying the tooth for plaque samples) and are Preventive and Controlling Methods of Caries
transferred to the S. mutAns strip which is incubated in M.S.B.
agar (Mitis Salivarius Bacitracin agar). The number of S. mutAns ♦♦ Chemical measures
colonies are used to estimate the caries activity and more than ♦♦ Nutritive measures
105 colonies per mL of saliva is indicative of high caries activity. ♦♦ Mechanical measures

Buffer Capacity Test Chemical Measures


10 mL of stimulated saliva is collected at least one after eating ♦♦ Substances which alter the tooth surfaces or tooth struc­
and stored under paraffin oil to prevent the loss of volatile tures.
bicarbonate ions, 4 mL of this is measured in beaker. After • Fluorine:
correcting the pH meter to room temperature the pH of the –– It has cariostatic activity
saliva is adjusted to 7.0 by addition of acid or base. The level –– Fluorine makes the teeth more resistant to acid
of lactic acid in the graduated cylinder is then again recorded. attack into oral cavity.
Lactic acid is then added to the sample until a pH of 6.0 is –– It decreases microbial acid production and en­
reached. The amount of lactic acid needed to reduce pH from hance the remineralization of underlying enamel.
7.0 to 6.0 is the measure of the buffer capacity. –– It is given in the form of:
586   Mastering the BDS IIIrd Year  (Last 25 Years Solved Questions)

- Communal water fluoridation • Systemic factors: Hereditary, pregnancy and lacta­


- Fluoride containing dentifrices tion factors have been suggested as etiological fac­
- Fluoride mouth rinses tors for dental caries.
- Dietary fluoride supplement. • Host factors: Poor oral hygiene and improper brush­
• Bis biguanides: Chlorhexidine and alexidine. They ing technique may lead to dental caries.
are antiplaque agents • Immunological factors: The functional role of
• Silver nitrate circulating antibodies as protective agents against
• Zinc chloride and ferrocyanide tooth decay has been demonstrated in nonhuman
♦♦ Substance which interfere with carbohydrate degradation primates.
through enzymatic alteration: Q.10. Write notes on nursing bottle syndrome.
• Vitamin K: It prevents acid formation in mixture of (Feb 2006, 2.5 Marks)
glucose into saliva. Ans. It is also called as nursing caries or baby bottle syndrome
• Sarcoside. and bottle mouth syndrome.
♦♦ Substance which interfere with bacteria growth and me­ This is a type of rampant caries affecting deciduous
tabolism: dentition.
• Urea and ammonium compounds
• Chlorophyll Etiology
• Nitrofurans It occurs due to prolong use of:
• Penicillin.
♦♦ A nursing bottle containing milk or milk like formula, fruit
Nutritive Measures juice or sweetened water.
♦♦ Breastfeeding
♦♦ Groups of patients whose diet is high in fat, low in carbo­ ♦♦ Sugar or honey sweetened pacifiers.
hydrate and practically free from sugar have low caries
activity. Clinical Features
♦♦ Phosphates diet causes significant reduction in indication ♦♦ The disease presents clinically as widespread carious de­
of caries. struction of deciduous tooth maxillary the fourth maxillary
anterior followed by Ist molar and then the cuspid if the
Mechanical Measures
habit is prolonged.
♦♦ Tooth brushing: Tooth brushing reduces the number of ♦♦ It has been emphasized that it is the absence of caries in
oral micro­organisms. If the teeth are brushed after each mandibular incisors which distinguishes this disease from
meal. ordinary rampant caries.
♦♦ Mouth rinsing: The use of mouthwash looses the food ♦♦ The carious process in affected teeth may be so sever that
debris from the teeth and prevents the caries. only root stumps remain
♦♦ Dental floss: It removes plaque from an area of gingiva to ♦♦ The mandibular incisors usually escape because they are
contact area on proximal surfaces of teeth, an area impos­ covered and protected by the tongue.
sible to reach with the toothbrush. Q.11. Write note on microbial plaque.
♦♦ Detergent foods: Fibrous foods prevent lodging of food (Mar 2000, 5 Marks) (Sep 2002, 6 Marks)
in the pit and fissures of the teeth and in addition acts as Ans. Microbial plaque is also called as dental plaque or
detergent. bacterial plaque.
♦♦ Pit and fissure sealants: The pits and fissure of occlusal
• It was demonstrated first time by William in 1897.
surface are most difficult areas on teeth to keep clean, so
• Microbial plaque is the soft, non-mineralized bacte­
pits and fissure sealants are generally used.
rial deposit which forms on teeth and dental prosthe­
Q.9. Write short note on contributing factors in dental car- sis that are not adequately cleaned. It is composed
ies. (Sep 2005, 5 Marks) of 80% water and 20% solids.
• Bacterial and salivary proteins comprise one half of
Ans. The following are the contributing factors in the dental dry weight of plaque. Plaque has high concentra­
caries. tion of protein, carbohydrates and lipids. Inorganic
• Dietary factors: Carbohydrates with types like components of plaque are calcium and phosphate.
monosaccharides, disaccharidase or polysaccharides • It forms on the tooth surfaces which are not properly
and the amount consumed and whether it is between cleaned.
the meals. • Dental pellicle which is a glycoprotein and is derived
• Microorganisms: Acidogenic Streptococcus mutAns from the saliva is adsorbed over the tooth surfaces
and Actinomycosis viscosus. and serve as nutrient for plaque microorganism.
Section 3: Oral Pathology  587

• Plaque is classified into two types based on its


anatomical location, i.e. supragingival plaque and
subgingival plaque.
• Supragingival plaque play important role in origin of
dental caries while subgingival plaque is responsible
for the causation of periodontal diseases.
Q.12. Describe the histopathology of enamel caries.
(May/June 2009, 5 Marks) (Sep 2006, 5 Marks)
Or
Write short note on histopathology of enamel caries.
(Apr 2017, 4 Marks) (Jan 2018, 4 Marks)
Ans. Refer to Ans 3 of same chapter.
Fig. 78:  Pit and fissure caries (GS) (For color version, see Plate 13)
Q.13. Define and classify and write in detail on etiopa­tho­­
genesis on dental caries. (Apr 2007, 15 Marks) Q.15. Write short note on streptococcus mutans in dental
Ans. “Dental caries is an irreversible progressive microbial caries. (Jan 2012, 5 Marks)
disease of the calcified tissues of the teeth, characterized Ans. It was first isolated by Clarke in 1924 and is termed as
by the demineralization of the inorganic portion and Streptococcus mutans.
distortion of the organic substances of the tooth, which • It is a catalyse negative Gram-positive cocci forming
often leads to cavitation”. short to medium chains.
• S. mutans synthesizes insoluble polysaccharides from
Classification of Dental Caries sucrose and is homofermentative.
♦♦ Based on location of the lesion • Most important substrate for involvement of
• Pit and fissure caries S. mutans in caries process is disaccharide sucrose.
–– Occlusal • S. mutans produces acid from fermentation of
–– Buccal or lingual pit sucrose, glucose, lactose, mannitol, etc. This leads
• Smooth surface caries to demineralization of tooth structure and finally
–– Proximal caries.
–– Buccal or lingual surface • S. mutans can survive at pH as low as 4.2 and can
• Root caries demineralize the tooth causing dental caries.
♦♦ Based on tissue involved • S. mutans synthesizes dextran which helps in adher­
• Enamel caries ing plaque bacteria to tooth surface which leads to
• Dentinal caries more decay.
• Cementum caries • S. mutans adhere to acquired pellicle and helps in
♦♦ Based on virginity of the lesion plaque formation which finally cause caries.
• Primary caries • S. mutans has ability to adhere on hard and smooth
• Secondary caries surfaces of tooth structure.
♦♦ Based on progression of lesion
• Progressive caries
–– Rapidly progressive like nursing caries and radia­
tion caries
–– Slowly progressing
• Arrested caries:
For etiopathogenesis refer to miller's chemico-parasitic theory
in Ans 2 of same chapter.

Q.14. Draw diagram of histopathology of pit and fissure


caries and dentinal caries. (Sep 2007, 5 Marks)
Ans. Pit and fissure caries:
• For dentinal caries, refer to Ans 3 of same chapter. Fig. 79:  Streptococcus mutans (For color version, see Plate 13)
588   Mastering the BDS IIIrd Year  (Last 25 Years Solved Questions)

Q.16. Write notes on Miller’s acidogenic theory. Streptococcus Sobrinus


(Apr 2008, 5 Marks) S. sobrinus has been implicated in caries development
Or particularly in instances where caries development appears
to be independent of S. mutans. It is interesting that S. sobrinus
Write in brief on Miller's acidogenic theory.
displays higher acid production and acid tolerance compared
 (Feb 2013, 8 Marks)
to S. mutans.
Ans. Refer to Ans 2 of same chapter.
Lactobacilli
Q.17. Define dental caries. Describe microbiology and factors
promoting dental caries. (Nov 2008, 15 Marks) Among the Lactobacillus rods in the oral cavity occur: L.
Ans. For definition refer to Ans 2 of same chapter. acidophilus, L. casei, L. fermentum, L. delbrueckii, L. plantarum,
L. jensenii, L. brevis, L. salivarius and L. gasseri. Lactobacilli are
Microbiology of Dental Caries divided into two main groups: homofermentative which in the
fermentation process of glucose produce mainly lactic acid, e.g.
♦♦ In the pathogenesis of dental caries an important role Lactobacillus casei, Lactobacillus acidophilus, heterofermentative
play cariogenic bacteria, i.e. oral streptococci, especially of which in addition to lactic acid produce acetate, ethanol and
group mutAns and lactic acid bacteria (Lactobacillus spp.). carbon dioxide, e.g. Lactobacillus fermentum.
♦♦ It is believed that bacteria of the species Streptococcus Lactobacilli are isolated from deep caries lesions but rarely just
mutans is the main factor that initiates caries and very before the development of dental caries and in the early tooth
important factor of enamel decay. decay. It is believed that they are pioneering microorganisms
♦♦ The bacteria of the genus Lactobacillus are important in in the caries progress, especially in dentin.
further caries development, especially in the dentin.
♦♦ Streptococcus mutans and lactobacilli are characterized by Veillonella
the ability to grow in an acid environment and the prop­
It is a Gram-negative cocci which is commonly found in plaque.
erty of rapid metabolism of sugars supplied in the diet to
It utilizes lactic acid by converting it to propionic acid and other
organic acids, including lactic acid.
weak acids.
♦♦ The microbial community from dentinal lesions is di­
verse and contains many facultatively and obligately Factors Promoting Dental Caries
anaerobic bacteria belonging to the genera Actinomyces,
Tooth Factors
Bifidobacterium, Eubacterium, Lactobacillus, Parvimonas
and Rothia. ♦♦ Tooth factors such as composition of tooth, structure of
♦♦ Streptococci are recovered less frequently. Caries can also enamel, morphologic characteristics of tooth, position of
be caused by other bacteria, including members of the tooth are the caries promoting factors.
mitis, anginosus and salivarius groups of streptococci, ♦♦ In composition of tooth surface enamel is more resistant
Propionibacterium, Enterococcus faecalis and Scardovia. to caries than subsurface enamel. This is because surface
enamel is highly mineralized.
Streptococcus Mutans ♦♦ Presence of deep, narrow occlusal fissures, buccal and
lingual pits leads to the development of dental caries.
Streptococcus mutans are the most cariogenic pathogens as ♦♦ Teeth which are malaligned, out of position, rotated or
they are highly acidogenic, producing short-chain acids otherwise not normally situated are difficult to clean
which dissolve hard tissues of teeth. They metabolize sucrose and favor accumulation of food and debris which lead to
to synthesize insoluble extracellular polysaccharides, dental caries.
which enhance their adherence to the tooth surface and
encourage biofilm formation. The reactions are catalyzed Salivary Factors
by three isozymes of glycosyltransferases. The most ♦♦ Salivary factors such as composition of saliva, pH, quan­
important mutans streptococci isolated from tooth caries tity, viscosity and antibacterial factors in saliva play role
samples are S. mutans and S. sobrinus. S. mutans is more in promotion of dental caries.
cariogenic than S. sobrinus because specific cell-surface ♦♦ When salivary flow rate is normal it leads to cleaning of
proteins, which aid in its primary attachment to the tooth. bacteria from tooth surface and reduces the chances of
S. sobrinus lacks such proteins. dental caries while in xerostomia incidence of caries is high.
S. mutans is able to metabolize a number of sugars and ♦♦ As viscosity of saliva is increased deposition of plaque
glycosides such as glucose, fructose, sucrose, lactose, galactose, increases because thick saliva does not produce proper
mannose, etc. In the presence of extracellular glucose and sucrose, cleaning action. If viscosity of saliva is decreased normal
S. mutans synthesizes intracellular glycogen-like polysaccharides contents of mineral is less and saliva does not produce
(IPSs). S. mutans produces also mutacins (bacteriocins), what anticaries functions.
is considered to be an important factor in the colonization and ♦♦ As the buffering action of saliva is decreased acid dem­
establishment of S. mutans in the dental biofilm. ineralization of tooth by dental caries become high this is
Section 3: Oral Pathology  589

because low concentration of salivary bicarbonate does For caries activity test, refer to Ans 9 of same chapter.
not cause neutralization of acids which is produced by
Q.19. What is dental caries. Enumerate different theories of
cariogenic bacteria.
etiology of caries. Describe in detail miller’s chemico-
• Salivary enzymes such as amylase leads to breakdown
parasitic theory. (Feb 2014, 8 Marks)
of starch (which is a residual carbohydrate) from
Ans. Refer to Ans 2 of same chapter.
tooth surface which is washed easily from mouth. As
if levels of salivary amylase are too low this will lead Q.20. Write short note on zones of dentinal caries.
to dental caries. (Jun 2014, 5 Marks)
• Certain antibacterial agents are found in saliva such Ans. Refer to Ans 3 of same chapter.
as lysozyme, thiocyanate, etc. These agents leads to
Q.21. Write short note on sequel of dental caries.
the destruction of cariogenic bacteria by anti-bacterial
action and reduces caries incidence. As if deficiency of (Dec 2012, 3 Marks)
such agents is present this will lead to the promotion Ans. Following is the sequel of dental caries
of dental caries.
• Salivary immunoglobulins such as IgA and IgG inhibit
S. mutAns by facilitating destruction process through
phagocytosis and lead to decrease in dental caries. If
salivary immunoglobulin levels are decreased this
leads to the increase in dental caries.

Diet Factors
♦♦ Diet factors such as physical nature, carbohydrates, vita­
mins and fluoride content play important role.
♦♦ More and more intake of soft and sticky food increases
possibility of dental caries.
♦♦ Foods rich in carbohydrates lead to the dental caries.
♦♦ Physical nature of diet is important as soft refined foods
cling to the teeth and are not removed because of lack of
roughage. This collection of debris is due to reduction
in mastication because of softness of diet. This leads to
dental caries.
♦♦ Fluoride ions limit rate of carbohydrate metabolism by
cariogenic bacteria and reduce acid attacks on tooth. Lag­
ging of fluoride in diet leads to the increase incidence of
dental caries.

Dental Plaque
♦♦ Dental plaque is the soft, non-mineralized bacterial deposit
which forms on teeth and dental prosthesis that are not
Q.22. Write short answer on role of carbohydrates in dental
adequately cleaned.
♦♦ Plaque harbors cariogenic bacteria on tooth surface. caries. (May 2018, 3 Marks)
♦♦ Rapid production of high amount of acids in plaque oc­ Ans. Fermentable dietary carbohydrates play an important
role in the causation of caries, e.g. glucose, fructose and
cur through fermentation of carbohydrates by cariogenic
sucrose. Among them sucrose is more potent.
bacteria.
♦♦ Cariogenic carbohydrates are dietary in origin, as uncon­
♦♦ Plaque hold the acids on tooth surfaces for longer duration.
taminated human saliva has negligible amount of carbo­
♦♦ Increased thickness of plaque does not allow salivary
hydrates regardless of blood sugar level.
buffers to enter into neutralize the acids produced by the
♦♦ Salivary carbohydrates get bound to proteins as well as
cariogenic bacteria.
other compounds. They are not readily available for mi­
Q.18. Define dental caries. Discuss in detail about theories crobial degradation.
and histopathology of dental caries. Add a note on ♦♦ Cariogenicity of dietary carbohydrates varies along with
caries activity tests. (Aug 2012, 10 Marks) frequency of ingestion, physical form and the chemical
Ans. For definition and theories of dental caries, refer to Ans composition, route of administration and presence of other
1 of same chapter. food constituents.
♦♦ Sticky solid carbohydrates are more cariogenic as com­
For histopathology of dental caries, refer to Ans 3 of same
pared to their liquid form.
chapter.
590   Mastering the BDS IIIrd Year  (Last 25 Years Solved Questions)

♦♦ Carbohydrates which get readily cleared from oral cavity ♦♦ Focal of complete destruction of odontoblast cells at pulp
via saliva and swallowing is less conducive to caries than dentin border.
to those which get slowly cleared. ♦♦ Many micro abscess formations in pulp characterized
♦♦ Polysaccharide gets less easily fermented by plaque bacte­ by the area of liquefaction degeneration in pulp being
ria as compared to monosaccharides and diasaccharides. surrounded by dense band of neutrophils and micro-
Organisms of plaque release less acid from sugar alcohols, organisms.
sorbitol, xylitol, mannitol. This implies that all these car­
bohydrates are non – cariogenic.
♦♦ Carbohydrates which are fed through stomach tube or
intravenously do not contribute to decay as they are una­
vailable for microbial breakdown.
♦♦ Food consisting of high fat, carbohydrate, protein or salt
decreases oral retentiveness of carbohydrates.
♦♦ Refined pure carbohydrates are more carious as compared
to crude carbohydrates which is complexed with other
food elements which are capable of reducing enamel solu­
bility or possessing antibacterial properties.

11. Diseases of the Pulp


and Periapical Tissues
Fig. 80:  Acute pulpitis (For color version, see Plate 13)
Q.1. Describe the etiology, histopathology and clinical
♦♦ There may be complete liquefaction and necrosis of pulp
features of acute pulpitis. with total destruction of odontoblastic cell layer. This is
(Sep 2008, 16 Marks) (Dec 2010, 8 Marks) known as acute suppurative pulpitis.
Or ♦♦ Death of pulp is due to tissue dehydration. This is known
as “dry gangrene of pulp”.
Describe etiology, histopathology, clinical features and
sequelae of acute pulpitis. (Sep 2002, 16 Marks) Clinical Features
Ans. Acute pulpitis is an irreversible condition characterized
by acute, intense inflammatory reaction in pulp tissue. ♦♦ Tooth is sensitive to cold and hot stimuli.
♦♦ Application of hot or cold stimuli causes an increase in
Etiology intensity to pain and such pain persists for longer duration
even after the stimuli is removed.
♦♦ Pulp exposure due to faulty cavity preparation.
♦♦ Intensity of pain increases during the sleep and occurs
♦♦ Caries progressing beyond the dentinal barriers and reach­
ing the pulp. due to increase in local blood pressure in head and
♦♦ Chemical irritation to pulp neck region.
♦♦ Cracked tooth syndrome ♦♦ As entrance of pulp is not wide, acute pulpitis helps in
♦♦ Metallic restoration in a tooth without proper thermal spread of inflammation throughout pulp with subsequent
insulation. necrosis.
♦♦ Blow to tooth with subsequent damage to pulp. ♦♦ Acute pulpitis is often associated with microabscess forma­
♦♦ Recurrent caries around the pre-existing restoration. tion in pulp along with liquefaction degenera­tion.
♦♦ Galvanic current produced due to dissimilar metal­ ♦♦ Pain subsides when drainage is established or when pulp
lic restoration may transmitted to pulp and causing undergo complete necrosis.
pulpitis. ♦♦ Tooth is nontender to percussion unless the pulpal inflam­
mation has spread beyond the root apex into peria­pical
Histopathology region.
♦♦ Severe edema in the pulp with vasodilatation. ♦♦ When intrapulpal pressure becomes very high during
♦♦ Moderate to dense infiltration of polymorphonuclear acute inflammation it cause collapse of apical blood ves­
leukocytes. sels. This is known as “pulp strangulation”.
Section 3: Oral Pathology  591

Sequelae of Acute Pulpitis ♦♦ Affected tooth has a large open carious cavity, which is
present for long duration.
♦♦ Lesion bleeds profusely on provocation.
♦♦ Involved tooth is painless and is sensitive to thermal
stimuli.

Histopathology

Q.2. What is pulpitis? Explain different causes of pulpitis.


Describe clinical features and histo­patho­logical picture Fig. 81:  Pulp polyp/chronic hyperplastic pulpitis
(For color version, see Plate 13)
of chronic open hyper­plastic pulpitis.
(Mar 2000, 15 Marks) ♦♦ Hyperplastic pulp tissue lesion presents the feature of
Ans. Pulpitis refers to the inflammation of dental pulp within granulation tissue mass, consisting of numerous proliferat­
the tooth. ing fibroblasts and young blood capillaries.
♦♦ Inflammatory cell infiltration by lymphocytes, plasma
Causes of Pulpitis cells and sometimes polymorphonuclear neutrophils in
♦♦ Reversible pulpitis: It is caused by an agent capable of tissue are common.
injuring the pulp like trauma, disturbed occlusal relation­ ♦♦ Stratified squamous epithelium is present on the surface
ship and thermal shock. of hyperplastic pulpitis which resembles oral epithelium.
♦♦ Irreversible pulpitis: It is caused by the bacterial in­ ♦♦ Epithelium surface show well formed rete peg formation.
volvement of pulp through caries, chemical or thermal or ♦♦ Epithelial cells on surface are believed to be desquamated
mechanical injury. epithelial cells which came either from buccal mucosa or
• Acute pulpitis: Refer to Ans 1 of same chapter from salivary gland ducts.
• Chronic pulpitis: Refer to Ans 3 of same chapter.
Q.3. Write short note on pulpitis. (Mar 2003, 5 Marks)
Chronic Open Hyperplastic Pulpitis Ans. Refer to Ans 1 and Ans 2 of same chapter.
♦♦ It is also known as chronic hyperplastic pulpitis or pulp
polyp or pulpitis aperta. Chronic Pulpitis
♦♦ It is a productive pulpal inflammation due to an extensive It is a condition characterized by the low grade fever often
carious exposure of young pulp. It is characterized by persistent inflammatory reaction in pulpal tissue with little or
development of granulation tissue covered by epithelium no constitutional symptoms.
and resulting from long standing low grade infection.
Etiology
Clinical Features
Refer to Ans 1 of same chapter.
♦♦ Pulp polyp appears as small, pinkish, red lobulated mass,
which protrudes from pulp chamber and fills up the cari­ Clinical Features
ous cavity. ♦♦ The tooth with chronic pulpitis may remain asymptomatic
♦♦ Condition is seen in young adults and children. It com­ for long time.
monly develops in deciduous molar and first permanent ♦♦ There may be an intermittent dull and throbbing pain in
molars. the tooth.
592   Mastering the BDS IIIrd Year  (Last 25 Years Solved Questions)

♦♦ Tooth is less sensitive to hot and cold stimuli. Q.5. Describe etiology, histopathology and clinical feature
♦♦ Tooth responds to a higher level of current when electric of periapical granuloma. (Sep 1999, 15 Marks)
pulp tester is used. Ans. It is also called as chronic apical periodontitis.
♦♦ Exposed pulp tissue may be manipulated by small instru­ Periapical granuloma is a localized mass of granulation
ment but bleeding can occur. tissue around the root apex of non vital tooth which
develop in relation to infection and inflammation.
Histopathology
Etiology
♦♦ Extension of pulpal inflammation
♦♦ Occlusal trauma
♦♦ Orthodontic tooth movements with excessive uncontrolled
force
♦♦ Acute trauma due to blows on tooth.
♦♦ Spread of periodontal infection in root apex.
♦♦ Perforation of root apex into endodontic therapy.

Clinical Features
♦♦ Tooth involves produce sensitivity to percussion which
occurs due to edema, hyperemia and inflammation of
apical periodontal ligament.
♦♦ Mild pain and discomfort in tooth during chewing solid foods.
Fig. 82:  Chronic pulpitis (For color version, see Plate 14) ♦♦ Involved tooth is slightly elongated from the socket.
♦♦ Chronic inflammatory response in the pulp is character­ ♦♦ Periapical granuloma may be asymptomatic in many cases.
ized by cellular infiltration by the lympho­cytes, plasma ♦♦ Tooth may be vital or partially vital in initial stages of
cells and macrophages. development of lesion but in fully developed periapical
♦♦ Prolong chronic inflammation may encourage fibroblastic granuloma the affected tooth is nonvital.
activity in pulp with formation of collagen bundle and in
some cases leads to internal resorption of the tooth. Histopathology
♦♦ Blood capillaries are prominent and few micro­organisms
are found in the pulpal tissue.
♦♦ Persisting chronic pulpitis may cause diffuse or solitary
areas of calcification in the pulp.

Treatment
♦♦ Extraction of tooth
♦♦ Root canal therapy
Q.4. Write short note on pulp polyp. (Mar 2013, 3 Marks)
(Dec 2009, 5 Marks) (Feb 2002, 5 Marks)
(Apr 2007, 5 Marks) (Sep 2007, 2.5 Marks)
(Aug 2011, 10 Marks) (June 2014, 5 Marks)
 (Nov 2014, 5 Marks)
Or
Write short note on chronic hyperplastic pulpitis.
(Mar 2011, 3 Marks) Fig. 83:  Periapical granuloma (For color version, see Plate 14)
Or
♦♦ Lesion appears as granulation tissue mass consisting of
Write short answer on pulp polyp.(Mar 2018, 3 Marks) proliferating fibroblasts, endothelial cells and numerous
Ans. Refer to Ans 2 of same chapter. immature blood capillaries.
♦♦ Chronic inflammatory cells, i.e. macrophages, lymphocytes
Treatment
and plasma cells are present in the lesion.
♦♦ Elimination of polypoid tissue, following the extirpation ♦♦ There is presence of epithelial islands, cholesterol clefts
of the pulp. and foam cells.
♦♦ After removing hyperplastic pulp tissue bleeding can be ♦♦ Plasma cells often produce immunoglobulin there is also
stopped by pressure. present of T lymphocytes in the lesion.
♦♦ Extraction of tooth or root canal treatment. ♦♦ Epithelial rest cell of malassez, proliferate in response to
Section 3: Oral Pathology  593

chronic inflammation and these proliferating cells undergo ♦♦ Adjacent tissue surrounding the bone has many dilated
cystification. blood vessels and infiltration with the neutrophils.
♦♦ Bony tissue at the periphery of lesion is lined by the os­ ♦♦ Inflammatory changes are observed in the PDL and adjoin­
teoclast cells with area of bone resorption. ing bone marrow.
♦♦ Few bacterias are present in the lesions which are not af­ ♦♦ Bony trabeculae in peri apical region may show empty
fected by the cellular immune mechanism. lacunae, which results from death of osteocytes.
♦♦ Occasionally Russell bodies are also found.
♦♦ Resorption of cementum and dentin often occurs as a result Q.8. Describe the etiology, histopathology and clinical
of chronic inflammation. In some areas along root, cemento­ features of acute suppurative osteo­myelitis in adult
blastic activity predominates leading to hypercementosis. patient mandible. (Aug/Sep 1998, 15 Marks)

Q.6. Write notes on periapical granuloma. Or


 (Sep 1999, 5 Marks) Describe etiology, histopathology and clinical features
Ans. Refer to Ans 5 of same chapter. and complications of acute suppurative osteomyelitis
in adult patient mandible. (Mar 2000, 15 Marks)
Treatment
Ans. Acute suppurative osteomyelitis is serious sequelae of
Extraction of involved tooth or under certain conditions root periapical infection, there is diffuse spread of infection
canal therapy with apical curettage. throughout medullary spaces with subsequent necrosis
of variable amount of bone.
Q.7. Describe etiology, histopathology and clinical features
of acute alveolar abscess. (Mar 1998, 15 Marks) Etiology
Ans. It is also known as dentoalveolar abscess. ♦♦ Direct spread of infection from dental pulp into the man­
It is defined as acute suppurative infection in periapical dible.
region of tooth. ♦♦ Spread of infection in the mandible from presenting sup­
purative odontogenic infections.
Etiology
♦♦ Spread of infection following removable of tooth without
♦♦ Extension of pulpal infection in periapical tissue. proper asepsis and antibiotic coverage.
♦♦ Fracture of tooth with pulp exposure. ♦♦ Compound fracture of mandible with exposure of bone
♦♦ Accidental perforation of apical foramen during root canal outside the mucosa.
treatment ♦♦ Post-radiation secondary infection.
♦♦ Extension of periodontal infection in periapical tissues. ♦♦ Infection to the preexisting bony lesions, e.g. Paget’s dis­
♦♦ Anachoretic infection of periapical tissues. ease of bone and fibrous dysplasia.
Clinical Features Clinical Features
♦♦ It is common odontogenic infection and constitutes 2% of
♦♦ It occurs after the 50 years of age and males are more
apical radiolucencies.
commonly affected.
♦♦ Due to acute abscess there is pain in the affected tooth.
♦♦ Mandibular lesions are diffuse in nature.
♦♦ Localized swelling and an erythematous change in overly­
♦♦ Acute suppurative osteomyelitis of mandible in young
ing mucosa is present.
adult causes severe pain, diffuse and enlarged swelling
♦♦ Affected area of jaw may be tendered during pal­pitation.
of mandible.
♦♦ Pain aggravates during percussion and when pressure is ♦♦ There is loosening and soreness of the regional teeth with
applied with the opposing tooth. difficulty in food intake.
♦♦ Application of heat intensifies pain, whereas application ♦♦ Multiple intraoral and extraoral pus discharging sinuses
of cold relieves pain temporarily. often develops and moreover discharge of pus is seen from
♦♦ Pus discharging sinus often develops on alveolar mucosa gingival cervice of the affected teeth.
over the affected root apex and sometimes on skin over­ ♦♦ Paresthesia of lip is common.
lying the jaw bone. ♦♦ Patient is slightly febrile and general symptoms include
♦♦ Infection from acute periapical abscess often spread to fever, malaise, anorexia and vomiting.
facial spaces, leading to space infections.

Histopathology Histopathology

♦♦ Lesion appears as zone of liquefaction necrosis, which is ♦♦ In acute suppurative osteomyelitis bone marrow un­
made up of proteinaceous exudates, necrotic tissue and dergoes liquefaction and purulent exudates occupy the
large number of dead neutrophils marrow space.
594   Mastering the BDS IIIrd Year  (Last 25 Years Solved Questions)

♦♦ A large number of acute inflammatory cells infiltrations –– Chronic diffuse sclerosing osteomyelitis
are present which shows PMNs with occasional presence –– Chronic osteomyelitis with proliferative peri­
of lymphocytes and plasma cells. ostitis
♦♦ Some areas of affected bone undergo necrosis with genera­ –– Chronic subperiosteal osteomyelitis
tion of osteoblast and osteocytes cells and therefore results –– Chronic periostitis
in development of sequestrum (a piece of dead bone). • Specific type
♦♦ When acute phase of infection subsides in new shell of –– Tuberculous osteomyelitis
bone called “involucrum” is formed over inflammatory –– Syphilitic osteomyelitis
focus. –– Actinomycotic osteomyelitis
• Radiation induced osteomyelitis
• Idiopathic osteomyelitis

Pathogenesis
Periapical infection/other foci

Enter the bone marrow and from there it extends to cancellous


bony spaces

Occlusion of the nutrients vessles of the living bone by


thrombus formation (thrombus consists of dead or viable
neutrophils, microorganisms and necrotic tissue debris, etc.)

Thrombosis of the nutrient vessels and excessive pressure


from inflammatory exudates against the rigid and confined
spaces in the bone
Fig. 84: Osteomyelitis (For color version, see Plate 14)
Disturbed nutrition supply to the bone cells
Complications
Death of the cancellous bony trabeculae
♦♦ Periostitis
♦♦ Cellulitis
Sequestrum formation
♦♦ Abscess.
Q.9. Define and classify osteomyelitis. Describe in detail Spread of infection to the Volkmann’s canal in the cortical
chronic osteomyelitis. (Sep 2008, 8 Marks) plates to reach external surface of bone below periosteum
Or
Inflammation of periosteum causing periostitis
Write short note on chronic osteomyelitis.
 (Jan 2017, 5 Marks)(June 2015, 5 Marks) Accumulation of exudates and pus
Ans. Chronic osteomyelitis is the persistent abscess of the
bone characterized by the complex inflammatory process Separation of the periosteum from the cortical plate
including necrosis of mineralized and marrow tissues,
suppuration, resorption, sclerosis and hyperplasia. Necrosis of the cortical bone
Osteomyelitis is defined as an inflammatory condition of bone
that begins as an infection of medullary cavity and haversian Single or multiple sinus tracts communicating the external
systems of the cortex and extends to involve the periosteum of surface of the skin and mucous membrane
the affected area.
Enveloping of sequestrum by a layer of bone formed by the
Classification of Osteomyelitis few survived bone forming cells (involucrum)
♦♦ Acute osteomyelitis
• Acute suppurative osteomyelitis Pus discharge from the involucrum can reach the skin through
• Acute subperiosteal osteomyelitis the sinus openings called cloacae
• Acute periostitis
♦♦ Chronic osteomyelitis Clinical Features
• Non- Specific Type ♦♦ Molar area of mandible is more frequently affected.
–– Chronic intramedullary osteomyelitis ♦♦ Pain is usually mild and insidious and is not related to the
–– Chronic focal sclerosing osteomyelitis real severity of disease.
Section 3: Oral Pathology  595

♦♦ Jaw swelling is common feature but mobility of teeth and Sequelae of Pulpitis
sinus tract formation are rare.
♦♦ Anesthesia and paresthesia of lip is uncommon.
♦♦ Regional lymphadenopathy is common.
♦♦ There is thickened, woodened feeling of bone and slow
increase in jaw size.

Histopathology
♦♦ Chronic inflammatory reaction of bone with accumulation
of exudate and pus within medullary spaces.
♦♦ Lymphocytes, macrophages and plasma cells predominate
among the inflammatory cells.
♦♦ Osteoblastic and osteoclastic cavity occurs partially with
formation of irregular bony trabeculae having reversal
lines.
♦♦ Sequestrum may develop in later stages of the disease.
♦♦ Colonies of bacteria are also seen within the inflamed
tissue.
Q.10. Classify pulpitis and write its sequelae. Write in short
etiology, clinical features, roentgeno­graphic features,
histology with treatment and prognosis of Garre’s
osteomyelitis. (Feb 2006, 15 Marks)
Ans. Inflammation of pulp is called as pulpitis.

Classification of Pulpitis
Clinical Features
Reversible
♦♦ It is common in young children and adults.
♦♦ Symptomatic (acute)
♦♦ Mandible is commonly involved in the posterior part.
♦♦ Asymptomatic (chronic).
♦♦ The involved jaw bone presents a carious non-vital tooth.
Irreversible ♦♦ There is a slight tenderness or a vague pain may be felt in
the affected area of the bone.
— Acute — Abnormally responsive to cold ♦♦ Slight pyrexia and leukocytosis may be present but ESR
is normal.
— Abnormally responsive to heat
— Chronic — Asymptomatic with pulp exposure Roentgenographic Features
— Hyperplastic pulpitis ♦♦ There is presence of a shadow of thin convex shell of bone
— Internal resorption. over cortex.
♦♦ As the infection proceeds cortex become thick and
Garre’s Osteomyelitis
laminated with alternating radiolucent and radiopaque
♦♦ It is also called as chronic osteomyelitis with proliferative layers. This is also known as onion skin appearance.
periostitis or periostitis ossificAns or Garre’s chronic non ♦♦ Cancellous bone adjacent to the lesion can be normal, become
suppurative sclerosing ostitis. sclerotic or it can show some areas of osteolytic changes.
♦♦ Garre’s osteomyelitis represents a reactive periosteal ♦♦ In the new bone osteolytic radiolucencies, i.e. small
osteogenesis in response to low grade infection or sequestra are seen.
trauma.
Histology
Etiology ♦♦ There is presence of newly formed bone consisting of
multiple osteoids and primitive bony tissue in sub peri­
♦♦ Mild infection osteal region.
♦♦ Chronic periapical abscess ♦♦ Osteoblastic as well as osteoclastic activities are observed
♦♦ Infected periapical cyst in central part of the bone.
♦♦ Mechanical irritation in the jaw from dentures ♦♦ Marrow space contains fibrous tissues showing patchy
♦♦ Chronic trauma in the jaw bone. areas of chronic inflammatory cell infiltration.
596   Mastering the BDS IIIrd Year  (Last 25 Years Solved Questions)

♦♦ Trabeculae are oriented perpendicular to the cortex with For clinical features and histopathology of acute
trabeculae arranged, parallel to each other shows “retiform osteomyelitis refer to Ans 8 of same chapter.
pattern”. For clinical features and histopathology of acute osteomyelitis
♦♦ Connective tissue between the bony trabeculae shows a dif­ refer to Ans 9 of same chapter.
fuse or patchy sprinkling of lymphocytes and plasma cells.
Q.17. Describe in brief etiology of pulpitis.
Treatment and Prognosis (Apr 2008, 5 Marks)
♦♦ Elimination of causative agent Ans. Following is the etiology of pulpitis
♦♦ Extraction of carious infected tooth and antibiotic therapy • Dental caries which extend beyond the dentinal
♦♦ Prognosis is good so no any additional surgical interven­ barrier and reaches pulp lead to pulpitis.
tion is required. • During cavity preparation if pulp exposure occurs
this will lead to pulpitis.
Q.11. Describe pathogenesis, histopathology and clinical
• When blow to the tooth occur which lead to damage
picture of osteomyelitis. (Mar 2003, 15 Marks)
of pulp.
Ans. Osteomyelitis is defined as the inflammation of bone and • If cavity preparation is done without using the water
bone marrow along with the surrounding periosteum. spray this lead to the excessive heat production to
• Pathogenesis: Refer to Ans 9 of same chapter. tooth which lead to pulpitis.
• Histopathology: Refer to Ans 8 and Ans 9 of same • Chemical irritation to pulp
chapter • Cracked tooth syndrome
• Clinical Features: Refer to Ans 8 and 9 of same chapter. • If metallic restoration is given in the tooth without
For pathogenesis of osteomyelitis, refer to Ans 9 of same chapter. providing proper thermal insulation this will lead
For clinical features and histopathology of acute osteomyelitis to pulpitis.
refer to Ans 8 of same chapter. Q.18. Write short note on periapical abscess.
For clinical features and histopathology of chronic osteomyelitis (Dec 2009, 5 Marks)
refer to Ans 9 of same chapter. Ans. Periapical abscess is an acute or chronic suppurative
Q.12. Write short note on Garrey’s osteomyelitis. process of dental periapical region.
 (Sep 2004, 5 Marks) (Aug 2011, 5 Marks) It is also known as dentoalveolar abscess or alveolar
 (Mar 2013, 3 Marks) abscess.
Acute exacerbation of chronic periapical lesion is called
Or
as phoenix abscess.
Give descriptive note on Garrey’s osteomyelitis.
 (Mar 2006, 5 Marks) Types of Periapical Abscess
Ans. Refer to Ans 10 of same chapter. ♦♦ Acute: It is associated with severe pain in tooth.
♦♦ Chronic: It is long standing and symptoms are of low grade.
Q.13. Define osteomyelitis and give an account of acute sup-
purative osteomyelitis in detail.
Clinical Features
 (Mar 2007, 6.5 Marks)
Ans. Osteomyelitis is defined as the inflammation of bone and Acute Periapical Abscess
bone marrow along with the surrounding periosteum.
♦♦ Patient complains of severe pain which is of throbbing
For acute suppurative osteomyelitis refer to Ans 8 of variety.
same chapter. ♦♦ There is also presence of swelling in the associated area.
Q.14. Draw well labeled histopathological diagram of ♦♦ Mucosa surrounding the swelling becomes tough and
chronic hyperplastic pulpitis. (Dec 2007, 3 Marks) inflamed.
Ans. Refer to Ans 2 of same chapter. ♦♦ Slight fever is present.
♦♦ Regional lymphadenitis is present.
Q.15. Classify pulpitis. Describe etiology, clinical features ♦♦ Patient feels sensitivity with the affected tooth.
and sequelae of acute pulpitis. (Sep 2009, 7 Marks) ♦♦ Tooth is tender to palpation and is mobile.
Ans. For classification of pulpitis refer to Ans 10 of same
Chronic Periapical Abscess
chapter.
For etiology, clinical features and sequelae of acute ♦♦ Pain is present from a longer time. Nature of pain
pulpitis refer to Ans 1 of same chapter. is dull.
♦♦ Sinus formation is seen either intraorally and ex­
Q.16. Write short note on osteomyelitis. traorally.
(Feb 2013, 5 Marks) ♦♦ At opening of sinus mass of inflamed granulation tissue
Ans. For definition, classification and pathogenesis refer to is present known as parulis.
Ans 9 of same chapter. ♦♦ Lymphadenopathy is present.
Section 3: Oral Pathology  597

Histopathology Phoenix abscess is also defined as an acute inflammatory


♦♦ Area of suppuration consists of central area of disintegrat­ reaction superimposed on existing chronic lesion such as cyst
ing PMN leukocytes surrounded by lymphocytes, cellular or granuloma.
debris, necrotic material and the bacterial colonies.
Etiology
♦♦ Dilated blood vessels are seen in PDL and adjacent mar­
row spaces of bone. Chronic periradicular lesions such as granulomas are in
♦♦ Inflammatory cell infiltrate is seen in marrow spaces. state of equilibrium during which they can be completely
♦♦ In chronic periapical abscess chronic inflammatory infil­ asymptomatic. But, sometimes, influx of necrotic products
trate consist of lymphocytes, plasma cells, macrophages. from diseased pulp or bacteria and their toxins can cause
Treatment the dormant lesion to react and this leads to initiation of
acute inflammatory response. Lowered body defenses and
Drainage of abscess is done followed by opening of root canal mechanical irritation during root canal treatment also trigger
or extraction of tooth. an acute inflammatory response.
Q.19. Describe etiopathogenesis of periapical granuloma.
Clinical features
 (Feb 2013, 8 Marks)
Ans. Following is the etiopathogenesis of periapical ♦♦ First symptom is that, there is tenderness on percussion.
granuloma: Tooth is slightly extruded from socket and is mobile.
• Periapical granuloma is caused as a response to pro­ ♦♦ Patient can or cannot have swelling. Swelling, if present
longed irritation from infected root canals which leads is localized and if left untreated may become diffuse (cel­
to the extension of chronic apical periodontitis in PDL. lulitis), leading to asymmetry of the patients face. In case
• Since pulp is infected it presents the release of of upper canines, it may even extend to the eyelids.
inflammatory mediators such as prostaglandins, ♦♦ Patient can be present with fever, malaise and lymphad­
enopathy.
kinins and endotoxins. Elevated levels of IgG are
♦♦ Tissue at the surface of swelling appears taut and inflamed;
seen in pulpoperiapical lesion.
pus starts to form beneath it.
• Inflammation as well as increase in the vascular
♦♦ As the liquefaction continues, tissue ruptures due to the
pressure leads to abscess formation and resorption
pressure to form a sinus tract which opens on the labial/
of bone in affected area which is replaced by the
buccal mucosa. This process is the beginning of chronic
granulation tissue.
alveolar abscess.
Q.20. Classify osteomyelitis. Write in detail the clinical,
radiological and histopathological features of Garre’s
osteomyelitis. Histopathology
 (Apr 2015, 8 Marks) There is presence of areas of liquefaction necrosis with
Ans. For classification of osteomyelitis refer to Ans 9 of same disintegrated polymorphonuclear leukocytes and cellular debris
chapter. surrounded by macrophages. lymphocytes, plasma cells in
For clinical, radiological and histopathological features periradicular tissues.
of Garre’s osteomyelitis refer to Ans 10 of same chapter.
Treatment
Q.12. Write short note on phoenix abscess. (Jan 2017, 5 Marks)
Ans. Acute exacerbation of chronic periapical lesion is called ♦♦ Establishment of drainage is done.
as phoenix abscess. ♦♦ As symptoms subsides root canal therapy should be
Phoenix abscess is also known as recrudescent abscess. done.
598   Mastering the BDS IIIrd Year  (Last 25 Years Solved Questions)

III Injuries and Repair


Etiology
12. Spread of Oral ♦♦ Dentoalveolar abscess in relation to mandibular 2nd or
infection 3rd molar.
♦♦ Deep periodontal abscess or pocket.
Q.1. Write note on cellulitis. (Apr 2007, 10 Marks) ♦♦ Pericoronal infection in relation to mandibular 3rd molar.
♦♦ Osteomyelitis.
Or ♦♦ Injection of contaminated needle.
Write short note on cellulitis. (Apr 2017, 4 Marks) ♦♦ Traumatic injuries: Fracture of mandible if infected or
Ans. It is also known as phlegmon. contaminated.
Cellulitis is defined as an acute edematous purulent ♦♦ Deep laceration or penetrating injury.
inflammatory process which spreads diffusely through
different tissue spaces or facial planes. Rare Causes
♦♦ Sublingual sialadenitis
Clinical Features ♦♦ Submandibular sialadenitis.
♦♦ There is wide spread swelling, redness and pain without ♦♦ Purulent tonsillitis.
definite localization.
Bacteriology
♦♦ Soft tissue swelling is firm and browny.
♦♦ When cellulitis involves the superficial tissue spaces, the ♦♦ Main microorganism is streptococci
overlying skin appears purplish. ♦♦ Others are staphylococci, gram-negative (E.coli and Pseu-
♦♦ Fever, chills leukocytosis are often present which make domonas), anaerobes, bacteriods and peptostreptococcus.
the patient slight ill.
Clinical Features
♦♦ Regional lymphadenopathy often develops.
♦♦ Some lesions resolve completely however in other cases ♦♦ It produces a board like swelling of floor of mouth and
pus discharging intraoral or extraoral sinuses may develop. consequent elevation of tongue.
♦♦ If maxillary tooth is involved there may be redness of eye. ♦♦ The swelling is firm, painful and diffuse showing no evi­
dence of localization.
Histopathology ♦♦ There is difficulty in eating, swallowing and breathing.
♦♦ There is collection of large amount of fibrin and serum ♦♦ Patient has high fever, rapid pulse and fast respiration.
fluid as well as tissue. ♦♦ Moderate leukocytosis is present.
♦♦ Acute inflammatory cell infiltration by PMNs and occa­ ♦♦ The swelling may spread to the neck with development
sionally lymphocytes. of edema glottis.
♦♦ Formation of sinus tracts over skin or mucosal surfaces.
Laboratory Findings
♦♦ Pus may develop in the later stages of the disease.
♦♦ Streptococci are invariably seen.
Treatment ♦♦ Fusiform bacilli and spiral forms, various staphylococci,
diphtheroids and other microorganisms are cultured.
♦♦ Bacteriological examination of exudates or pus. ♦♦ It is a nonspecific mixed infection.
♦♦ Drainage of pus is carried out.
♦♦ Antibiotic therapy Complication
♦♦ Elimination of primary source of inflammation. ♦♦ Asphyxia: Due to edema of glottis.
♦♦ Septicemia and cavernous sinus thrombosis.
Q.2. Write short note on Ludwig’s angina.
♦♦ Mediastinitis.
(Sep 2006, 6 Marks) (Mar 2001, 5 Marks)
♦♦ Meningitis
(Jan 2016, 5 Marks) (Feb 2013, 5 Marks)
♦♦ Brain abscess.
Or ♦♦ Death.
Describe in brief Ludwig’s angina. (The most common cause of death is as asphyxia).
(Apr 2008, 10 Marks)
Treatment
Ans. Ludwig’s angina is an acute toxic cellulitis begin
in submandibular space and secondarily involving ♦♦ Early diagnosis.
sublingual and submental spaces as well. ♦♦ Maintenance of patient airway.
Section 3: Oral Pathology  599

♦♦ Intense and prolonged antibiotic therapy. ♦♦ Subacute bacterial endocarditis (infective endocarditis): Major­
♦♦ Extraction of offending teeth. ity of these cases are related to oral infection and occur
♦♦ Surgical drainage of facial space. following tooth extraction. The close similarity between
Q.3. Write short note on focus of infection. the causative agent of subacute bacterial endocarditis and
(Dec 2012, 3 Marks) (Mar 2013, 3 Marks) the Streptococcus of viridAns group in the oral cavity, in
the dental pulp and periapical lesions, and frequent oc­
Or currence of transient streptococcal bacteraemia following
Write short note on focus of infection and focal tooth extraction are the indication that oral foci are the
infection. (July 2016, 5 Marks) cause of this disease.
Ans. Focus of infection refers to “the circumscribed area of ♦♦ Gastrointestinal diseases: It has been reported that constant
tissue which is infected with exogeneous pathogenic swallowing of streptococci from mouth may lead to a
microorganisms and is located near mucous or variety of gastrointestinal diseases. Gastric and duodenal
cutaneous surface”. By Billings ulcers are related to oral foci of infection.
• As there is local or general infection which is caused ♦♦ Ocular diseases: In many ocular diseases, such as iritis, cy­
by dissemination of microorganisms or toxic products clitis, choroiditis, uveitis, etc. microbes associated with the
from focus of infection this lead to focal infection. teeth, oral cavity, tonsils, sinuses, etc. have been considered
• Foci may be primary or secondary. Primary foci as primary foci of infection.
usually are located in tissues communicating with ♦♦ Skin diseases: Such as some form of eczema and possibly
a mucous or cutaneous surface. urticaria can be related to oral foci of infection.
• Secondary foci are the direct result of infections from ♦♦ Renal diseases: Streptococci, particularly S. hemolyticus pre­
other foci through contiguous tissues, or at a distance sent in dental root canals or periapical or gingival areas
through the blood stream or lymph channels. appears to have some relation with certain type of renal
diseases and may play a role in causing renal diseases.
Mechanism of Focal Infection Q.4. Describe in detail on diseases of maxillary sinus.
Two of the accepted mechanisms are: (June 2010, 10 Marks)
♦♦ Metastasis of microorganisms from an infected focus by Ans. Following are the diseases of maxillary sinus:
hematogenous or lymphogenous spread.
Classification
♦♦ Toxins or toxic products may be carried through the blood
or lymphatic channels froma focus to a distant site where ♦♦ Developmental:
they may incite a hypersensitive reaction in the tissues. • Crouzon's syndrome
• Treacher Collins syndrome
Oral Foci of Infection • Binder syndrome
Various infections of the oral cavity may act as sources of infection ♦♦ Inflammatory:
and may be responsible for spread of microbes to a distant site • Maxillary sinusitis
causing metastases. The potential foci in the oral cavity include: • Mucositis
♦♦ Infected periapical lesions, particularly the chronic lesions, • Empyema
such as the periapical abscess., granuloma and cyst. ♦♦ Cyst:
♦♦ Teeth with infected root canals are the potential sources of • Non-dental
dissemination of microbes as well as their toxins. –– Mucocele
♦♦ Periodontal disease following tooth extraction or dental –– Benign mucosal cyst of maxillary antrum
manipulation is also significant focus of infection; par­ –– Surgical ciliated cyst
ticularly the tooth extraction is an important cause of • Dental:
bacteremia. –– Radicular cyst
–– Globulomaxillary cyst
Impedance of Oral Foci of Infection –– Dentigerous cyst
Oral foci of infection either cause or aggravate many systemic –– Odontogenic keratocyst
♦♦ Benign tumor:
diseases. Most frequently encountered systemic diseases are:
• Osteoma
♦♦ Arthritis: Arthritis of the rheumatoid type and rheumatic
• Ameloblastoma
fever type. Which are manifested because of the occurrence
• Antral polyp
of streptococcal infection in mouth. The causal microbe
• Antral papilloma
(Group A streptococci) may not be cultured from the joints
♦♦ Malignant tumor:
and blood but the patients have a high titer of antibodies
• Squamous cell carcinoma
against these microbes. The presence of these antibodies
• Metastatic carcinoma of maxillary sinus
suggests that tissue hypersensitivity reaction is the cause
• Local malignant tumor invades maxillary sinus
of inflammatory reactions that occur.
600   Mastering the BDS IIIrd Year  (Last 25 Years Solved Questions)

♦♦ Due to trauma: Clinical Features


• Fractured root
Acute
• Sinus contusion
• Blow out fracture ♦♦ Patient complains of pain in eyeball, cheek or frontal areas
• Isolated injury which is severe and is constant. Pain gets exaggerated by
• Complex fracture downward positioning of head.
• Oroantral fistula ♦♦ Nasal discharge is present which is watery in beginning
• Foreign bodies of disease and later on becomes mucopurulent.
♦♦ Calcification: Antrolith ♦♦ There is presence of erythematous color and inflammation
♦♦ Miscellaneous of mucosa of anterior nares.
• Fibrous dysplasia ♦♦ Presence of tenderness on palpation.
• Pseudo tumor
Sub-acute
Crouzon Syndrome
♦♦ Symptoms are diminished.
It is also known as craniofacial dysostosis. ♦♦ Presence of purulent discharge. Nasal voice is present.
Following are the features ♦♦ Patient is unable to sleep because of presence of cough.
♦♦ Frontal defect is present and cranium is brachycephalic. Chronic
♦♦ Maxilla and maxillary sinus remains hypoplastic.
♦♦ High arch palate is seen. ♦♦ Nasal obstruction and headache are the constant features.
♦♦ Dental arch is V shaped ♦♦ Low fever and tiredness is present.
♦♦ Partial anodontia is present. Histopathology
Treacher Collins Syndrome ♦♦ Maxillary sinus lining have acute inflammatory infiltrate.
It is also known as mandibulofacial dysostosis. ♦♦ Edema of connective tissue is present.
♦♦ At times hemorrhage is also seen.
♦♦ Maxillary sinus and malar bones are underdeveloped.
♦♦ Cheek bones are underdeveloped. Treatment
♦♦ Facial cleft is present.
♦♦ Palate is high arched ♦♦ Removal of the cause.
♦♦ Malformation of external ear is seen. ♦♦ Antibiotics, analgesics and nasal decongestants should
be given.
Binder Syndrome
Mucositis
♦♦ It leads to hypoplasia of middle third of face. ♦♦ Inflammation of mucosa of maxillary sinus is known as
♦♦ Retrognathic maxilla mucositis.
♦♦ Presence of maxillonasal dysplasia
♦♦ It is caused by periodontal or periapical infection.
♦♦ Hypoplastic maxillary and frontal sinus
♦♦ Disease is asymptomatic.
Maxillary Sinusitis Benign Mucosal Cyst of Maxillary Antrum
♦♦ Inflammation of mucosa of maxillary sinus is known as ♦♦ Occurs during 2nd and 3rd decades of life.
maxillary sinusitis. ♦♦ Male predilection is present.
♦♦ It can be acute, sub-acute and chronic. ♦♦ Localized dull pain is present over antrum.
♦♦ Yellow fluid discharge is seen from nose.
Etiology ♦♦ Histology reveals presence of chronic inflammatory cells
in connective tissue wall.
♦♦ It is caused due to periapical infection of teeth.
♦♦ Due to oro-antral fistula Postoperative Maxillary Cyst
♦♦ By deep pocket
♦♦ It occurs due to the entrapped epithelial lining of maxil­
♦♦ Due to trauma of facial bones. lary sinus in wound closure during Caldwell-Luc surgery.
♦♦ Due to overpreparation of root canal which leads to gutta- ♦♦ Occurs from 2nd to 7th decades of life.
percha filling in sinus. ♦♦ Pain and swelling is present over cheek or palate or face
♦♦ If implant is placed too deep or alveolus.
Section 3: Oral Pathology  601

♦♦ Pus discharge is present. ♦♦ Foul smelling of mucopurulent discharge.


♦♦ Histopathology reveals presence of squamous metaplasia ♦♦ Mild tenderness over infraorbital region.
in pseudo-stratified columnar epithelium. ♦♦ Closure of opening is done by surgical methods.
♦♦ Enucleation should be done.
Antrolith
Osteoma
♦♦ It is the calcified mass present in the maxillary sinus.
♦♦ It occurs from 2nd to 4th decade of life. ♦♦ It can occur at any age
♦♦ Male predilection is seen. ♦♦ Lesion is asymptomatic and at times nasal discharge is
♦♦ Mainly asymptomatic but obstruct the ostium of sinus. present which is blood stained and nasal obstruction too
♦♦ It can expand to maxillary sinus and leads to swelling of is seen.
hard palate. ♦♦ Removal is done if lesion is symptomatic.
♦♦ Surgical excision is done.

Antral Polyp
♦♦ Mucosa of the chronically inflamed sinus leads to the
13. Physical and Chemical
formation of irregular folds known as polyp. injuries of the oral cavity
♦♦ Seen in young adults.
♦♦ Pain in the nose and nasal obstruction is present. Q.1. Write notes on osteoradionecrosis.
♦♦ Saint’s triad is present, i.e. asthma, nasal and antral polyp
(Mar 2001, 5 Marks)
and aspirin sensitivity.
♦♦ Excision of polyp is done. Or

Squamous Cell Carcinoma Write short note on osteoradionecrosis.


(Sep 2007, 3 Marks) (Sep 2011, 3 Marks)
♦♦ Squamous cell carcinoma of maxillary sinus occurs from
metaplastic epithelium of sinus lining. Ans. Osteoradionecrosis is a radiation induced pathologic
♦♦ Occurs during 6th decade of life and male predilection process characterized by the chronic and painful
is seen. infection and necrosis is accompanied by the late
♦♦ Patient complains of pain over face, nasal obstruction and sequestration and sometimes permanent deformity.
swelling. This is one of the most serious complications of radiation
to head and neck seen frequently today because of better
♦♦ Ulceration is present over the hard palate.
treatment modalities and prevention.
♦♦ Both medial and lateral walls of the sinus are involved.
♦♦ Involvement of floor of the sinus leads to expansion of Factors Leading to Osteoradionecrosis
boney plates, swelling, mobility of teeth and severe pain.
♦♦ Irradiation of an area of previous surgery before adequate
♦♦ Histopathology reveals features of squamous cell carci­
healing had taken place.
noma.
♦♦ Irradiation of lesion in close proximity to bone.
For details refer to Ans 23 of chapter BENIGN AND ♦♦ Prolong oral hygiene and continued use of irritants.
MALIGNANT TUMORS OF ORAL CAVITY ♦♦ Poor patient’s corporation in managing irradiated tissues.
♦♦ Surgery in irradiated area.
Oroantral Fistula
♦♦ Failure to prevent trauma to irradiated bony areas.
♦♦ It is an epithelialized pathological communication between
oral cavity and maxillary sinus. Clinical Features
♦♦ It is caused by trauma to sinus, malignancies and osteo­ ♦♦ Osteoradionecrosis is the result of nonhealing dead bone.
myelitis ♦♦ Mandible is affected more commonly than maxilla.
♦♦ There is presence of tenderness over the maxilla especially
in infraorbital region. Histology
♦♦ Mild edema to cheek in infraorbital soft tissue.
♦♦ Ear ache-referred pain from antrum. ♦♦ There is destruction of osteocytes, absence of osteoblasts
♦♦ On nose-red, shiny and swollen, mucous membrane, and lack of new bone or osteoid formation.
around osteum. ♦♦ Walls of regional blood vessels are thickened by fibrous
♦♦ Presence of pus or mucopurulent discharge in middle connective tissue.
meatus. ♦♦ Radiation causes proliferation of intima of blood vessels
♦♦ Oropharynx—mucopurulent discharge. leading to thrombosis of arteries which results in non
♦♦ Impairment of sense of smell. vital bone.
602   Mastering the BDS IIIrd Year  (Last 25 Years Solved Questions)

Treatment ♦♦ Following are the materials in detail which lead to chemical


injuries of oral cavity:
♦♦ Debridement of necrotic tissue should be done along with
removal of sequestrum. Chemical Injuries by Materials used Locally in Dentistry
♦♦ Administration of intravenous antibiotic and hyperbaric
oxygen therapy are essential Aspirin
♦♦ Maintenance of oral hygiene is necessary. ♦♦ Aspirin tablets or powder are mainly used mistakenly in
Q.2. Write short note on lead poisoning. oral cavity by patients as local obtundent mainly for relief
(Aug 2012, 5 Marks) of toothache.
♦♦ Initially there is burning sensation present in oral
Ans. Lead poisoning leads to plumbism.
mucosa.
Etiology ♦♦ Affected surface appear blanched as well as white in ap­
pearance.
♦♦ Paints consisting of lead: Workers and children are most ♦♦ Epithelial separation and sloughing of epithelium along
commonly affected. with frequent bleeding is seen.
♦♦ Illicit alcohol consisting of lead can cause lead poisoning. ♦♦ Healing take place under 1 to 2 weeks.
♦♦ Gasolining consisting of lead.
♦♦ Lead from automobile smoke can lead to occupational Endodontic Materials
exposure. ♦♦ Some of endodontic materials lead to soft tissue damage
Clinical Features causing deep spread of inflammation and necrosis.
♦♦ Paraformaldehyde is used to devitalize inflamed pulp. It
♦♦ Due to lead poisoning there is axon degeneration and can leak from pulp chamber in surrounding tissue and
demyelination. lead to necrosis of gingiva and bone.
♦♦ In more severe cases there is presence of encephalopathy, ♦♦ Sodium hypochlorite produces same effect as paraform­
seizures, mental retardation and cerebral palsy. aldehyde when it leaks in surrounding supporting tissue
♦♦ Patient often complains of nausea, vomiting and constipation. or injected beyond the apex.
♦♦ Due to lead poisoning bone deposition and resorption is ♦♦ Sodium hypochlorite when come in contact with vital tis­
disturbed. sue it leads to hemolysis and ulceration.
♦♦ Microscopically sodium hypochlorite inhibits neutrophil
Oral Manifestations
migration and damage endothelial and fibroblast cells.
♦♦ There is presence of excessive salivation, metallic taste
and dysphagia. Hydrogen Peroxide
♦♦ Burtonian line is seen in cases of lead poisoning with poor ♦♦ This is a caustic agent.
oral hygiene. A gray black line is seen along marginal ♦♦ As it comes under the contact with tissues it lead to burn­
gingiva which is known as burtonian line. ing of tissues and release toxic free radicals, perhydroxyl
♦♦ Characteristic signs are pale lips, Ashen colored face, ion or both.
muscle tone is poor. ♦♦ 30 to 35% of hydrogen peroxide is used with heat for
♦♦ Bilateral parotid gland hypertrophy is evident. bleaching teeth. This thermocatalytic process dam­
ages the tooth by causing irritation to cementum and
Treatment
periodontal ligament which also causes cervical root
Chelating agents such as EDTA should be given to treat the patient. resorption.
Q.3. Write short note on chemical injuries of oral cavity. Phenol
 (Jan 2018, 5 Marks)
Ans. Oral cavity manifests serious reaction to the wide variety ♦♦ It is cavity sterilizing and cauterizing agent.
of drugs and chemicals. ♦♦ This is used in treatment of aphthous ulcers.
♦♦ Tissue injury can be due to local response to a severe ir­ ♦♦ As extensive necrosis is seen from medicaments consisting
ritant or due to administration of systemic drugs. of 0.5% phenol, this product should be used with atmost
♦♦ Aspirin, sodium perborate, hydrogen peroxide, gaso­ care.
line, turpentine, rubbing alcohol and battery acid is the
examples. Silver Nitrate
♦♦ Various patients many children under psychiatric care ♦♦ It is useful for treatment of aphthous ulcers as chemical
hold medication under their mouth rather swallowing cautery leads to pain relief by destroying the nerve
them. Such medications are potentially caustic when held endings.
in mouth for long duration. ♦♦ Its over usage leads to the painful burn of oral cavity.
Section 3: Oral Pathology  603

Histopathology of Locally Acting Agents Leading to


Chemical Injuries
14. Regressive Altera-
♦♦ White slough removed from mucosal chemical burns tions of the teeth
shows coagulative necrosis of epithelium. Outline of epi­
thelial cells and nuclei is visible. Q.1. Write note on abrasion. (Mar 2000, 5 Marks)
♦♦ Necrosis starts over the surface and moves basally. Or
♦♦ Underlying connective tissue consists of mixture of acute Write short note on abrasion. (Sep 2002, 5 Marks)
and chronic inflammatory cells.
Or
Chemical Injuries by Materials used Systemically in Give descriptive note on abrasion.
Dentistry  (Mar 2006, 5 Marks)
Lead Ans. Abrasion is a pathological wearing of dental tissues
by friction with the foreign substances independent of
♦♦ Lead poisoning or plumbism is an occupational hazard.
occlusion.
♦♦ Lead line or Burtonians line is a grey or blue black line of
sulphide pigmentation present on gingiva. Etiology
♦♦ Ulcerative stomatitis is seen.
♦♦ Excessive salivation and metallic taste are commonly ♦♦ Tooth brush abrasion
present. ♦♦ Habitual abrasion, e.g. pipe smokers, tooth pick and dental
floss causes abrasion on proximal surfaces of teeth.
Mercury ♦♦ Occupation abrasion, e.g. hair dressers, carpenters and
shoemakers.
♦♦ Mercury poisoning occur when it is used therapeutically.
♦♦ Abrasion by prosthetic appliances
♦♦ In this tongue and salivary glands are swollen.
♦♦ Ritual abrasions.
♦♦ Metallic taste in mouth is present.
♦♦ Salivary flow is increased. Clinical Features
♦♦ Ulcerations are present on gingiva, palate and tongue.
♦♦ Exfoliation of teeth is also present. ♦♦ In abrasion of tooth the type and severity of surface will
depend upon the duration and type of faulty habit adapted
♦♦ Acrodynia occurs due to chronic mercury exposure in
by the person.
infants and children.
♦♦ Toothbrush abrasion commonly occurs in cervical region
of labial surface of incisors, canines and premolars.
Silver
♦♦ Maxillary teeth are more commonly affected than the
♦♦ Subepithelial deposition of silver in mucus membrane mandibular teeth.
leads to diffuse grayish discoloration. ♦♦ The produces a ‘V” shaped or wedge-shaped groove
♦♦ A blue silver line occurs at gingival margin due to second­ on the tooth with sharp angles and highly polished
ary deposition of metallic silver. dentin surface.
♦♦ Amalgam tattoo is the most common finding. In this par­ ♦♦ Toothbrush abrasion may cause gingival recession.
ticles enter via lacerations which occur during removal of ♦♦ Occupational abrasion often produces a small, deep, well
old amalgam restorations. It appears as raised blue, black polished ditch on an incisal edge of teeth.
or grey lesion. ♦♦ Sever abrasion may cause opening of dentinal tubules and
hence the patient experiences sensitivity in affected teeth
Bismuth due to hot and cold substances.
♦♦ It is used by oral surgeons in surgical packs.
♦♦ Pigmentation of bismuth is seen in gingiva and buccal
mucosa.
♦♦ Bismuth line, i.e. blue black line is present at marginal
gingiva.

Tetracycline
♦♦ It lead to the discoloration of permanent or deciduous teeth
due to deposition of tetracycline during prophylactic or
therapeutic regimens in pregnant female or postpartum
in infant.
♦♦ Affected teeth are yellowish or show brown gray discol­
oration.
♦♦ Dentine is more stained than enamel. Fig. 85: Abrasion (For color version see Plate 14)
604   Mastering the BDS IIIrd Year  (Last 25 Years Solved Questions)

Treatment Fibroblasts in aging pulp exhibit less perinuclear cytoplasm and


♦♦ Adaptance of normal brushing habits prevents abrasion. long thin cytoplasmic processes.
♦♦ Restorative treatment helps to keep the tooth surface intact Fibrosis
and prevents further tooth wear.
Q.2. Write short note on sclerotic dentin.
(Sep 2005, 5 Marks)
Ans. It is also known as transparent dentin
• Dentinal sclerosis is the condition characterized by
calcification of dentinal tubules of the tooth.
• It is the regressive alteration in the tooth surface.

Etiology
♦♦ Injury to dentin by caries
♦♦ Aging process
♦♦ Abrasion or erosion of the tooth.
♦♦ Sclerotic dentin is found under the slowly progressing caries
♦♦ It reduces the permeability of dentin and prolongs pulp
vitality.
Fig. 87:  Pulp fibrosis (For color version, see Plate 15)
♦♦ Dentinal sclerosis presents a translucent zone in the teeth
which is seen in the tooth by the transmitted light. ♦♦ In aging pulp accumulation of diffuse fibrillar components
♦♦ Sclerosis often decreases the conductivity of the tubules. and bundles of collagen fibers appears.
♦♦ Fiber bundles appear arranged longitudinally in bundles
in radicular pulp and more diffuse arrangement on coro­
nal area.
♦♦ Increase in fiber in pulp organ is generalized throughout
the organ.
♦♦ Vascular changes occur in aging pulp organ.
♦♦ Atherosclerotic plaque may appear in pulpal vessels.
♦♦ Calcifications in the wall of blood vessels are found more
often in region near the apical foramen.
♦♦ Outer diameter of vessel walls becomes greater, as collagen
fibers increase in the medial and adventitial layer.
Pulp Stones/Denticles
♦♦ Pulp stones and denticles are nodular calcified masses
Fig. 86:  Sclerotic dentin (For color version, see Plate 14) appearing on coronal portions of the pulp organs.
♦♦ They are asymptomatic unless they impinge nerves on
Q.3. Describe regressive changes in pulp. blood vessels.
(Mar 2001, 15 Marks) ♦♦ Pulp stones are classified according to their structures as
Or true and false denticles.
Write short note on age changes in pulp. ♦♦ They are also classified as free, attached and embedded
(Dec 2012, 3 Marks) depending on the relation to dentin.
Ans. Regressive changes of pulp mean aging of pulp. ♦♦ Pulp stones may eventually fill substantial part of pulp
chamber.
Cell Changes Pathogenesis of True Pulp Stone
♦♦ Cells are characterized by decrease in size and number of ♦♦ Development of true denticles is caused by inclusion of
cytoplasmic organelles. remnants of epithelial root sheath within pulp. These
♦♦ Pulpal fibrocytes or fibroblasts have abundant rough sur­ epithelial remnants induce cells of pulp to differentiate
face, cytoplasmic reticulum, Golgi bodies and numerous into odontoblasts which form dentin masses known as
mitochondria with well developed cristae. true pulp stone.
Section 3: Oral Pathology  605

Fig. 90:  Diffuse calcification (For color version, see Plate 15)
Fig. 86:  True pulp stones (For color version, see Plate 15)
Q.4. Write short note on pink tooth. (Feb 2002, 5 Marks)
Pathogenesis of False Pulp Stone  (Sep 2006, 5 Marks) (Mar 2016, 3 Marks)
Ans. It is also called as chronic perforating hyperplasia of the
pulp or internal granuloma or odontoclastoma or pink
tooth of Mummery or internal resorption of teeth.
Pathological resorption of tooth which is starting from
the pulpal surface is called as internal resorp­tion.

Pathogenesis
Sudden trauma or Injury to tooth

Intrapulpal hemorrhage

Organization of clot

Formation of hyperplastic granulation tissue

Compression of odontoblast cells

Cessation of dentin formation

Stimulation of dentinoclast cells

Resorption of dentin

Invasion of hyperplastic pulpal tissue into


resorbed spaces of dentin

Fig. 89:  False pulp stones (For color version, see Plate 15)
Clinical Features
Diffuse Calcification
♦♦ Internal resorption may involve either the crown portion
♦♦ They appear as irregular calcific deposit in pulp tissue of the tooth or the root portion.
following collagenous fibrous bundles in blood vessels. ♦♦ Any tooth may be involved and usually only a single
♦♦ They persists as fine calcified spicules and sometimes tooth is affected.
develop into larger masses. ♦♦ Pink appearance of tooth occur in advanced stage when
♦♦ Diffuse calcifications are found in root canal. the coronal dentin is involved.
♦♦ Diffuse calcifications may surround the blood vessels. ♦♦ When internal resorption affects the root of the tooth no
♦♦ Diffuse calcification is also termed as “calcific degeneration.” color change is found
606   Mastering the BDS IIIrd Year  (Last 25 Years Solved Questions)

♦♦ Affected tooth remains vital unless there is pulp necrosis Histological Features
due to fracture of tooth or due to its perforation.

Histopathology
♦♦ Multiple irregular or smooth areas of resorption in pulpal
surfaces of dentin.
♦♦ A hyperplastic, highly vascular pulp tissue is projecting
into the spaces of dentin which are created by resorption.
♦♦ Multiple multinucleated dentinoclasts are found near the
resorpting front of the dentin.

Fig. 92: Hypercementosis (For color version, see Plate 16)

♦♦ There is excessive amount of cellular cementum found


deposited directly over the layer of acellular cementum.
♦♦ Area involved may be the entire root and the apical region.
♦♦ Cementum is arranged in the concentric layers around
the root.
♦♦ Cementum shows numerous resting lines indicated by
deeply staining hematoxyphilic line parallel to root surface.

Roentgenographic Features
Radiograph shows excessive cemental thickening with typical
Fig. 91:  Internal resorption of teeth or pink tooth bulbous appearance of roots.
(For color version, see Plate 15)
Treatment
Treatment
No treatment is indicated for teeth exhibiting hyper­cementosis
♦♦ Extirpation (complete removal of part) of pulp tissue and since the condition is itself innocuous.
conventional endodontic therapy
♦♦ When the tooth is perforated extraction is only the treatment. Q.6. Write short note on attrition. (Apr 2007, 5 Marks)
Ans. Attrition is a constant form of retrogressive change in
Q.5. Write notes on hypercementosis. (Sep 1999, 5 Marks) teeth, characterized by the wear of tooth structure or
Or restoration as a result of tooth to tooth contact during
Describe in brief hypercementosis. mastication.
(Aug 2011, 5 Marks) Types
Ans. Hypercementosis is also known as cementum hyperplasia.
♦♦ Physiological attrition: Tooth loss is proportionate to age
It increases the thickness of cementum on root surfaces of
the tooth due to excessive cemento­genesis and is called of individual.
as hypercemen­tosis. ♦♦ Pathological attrition: It occurs due to certain abnor­malities
in occlusion, chewing pattern or due to some structural
Etiology defects.
♦♦ Periapical inflammation
Causes
♦♦ Mechanical stimulation
♦♦ Functionless or unerupted tooth ♦♦ Abnormal occlusion, e.g. crowding of teeth or malposed
♦♦ Paget’s disease of bone teeth.
♦♦ Tooth repair. ♦♦ Abnormal chewing habit, e.g. bruxism
♦♦ Structural defects in teeth, e.g. amelogenesis imperfecta,
Clinical Features dentinogenesis imperfecta.
♦♦ Involved teeth are completely asymptomatic.
♦♦ There is no increase or decrease in tooth sensitivity, no sensi­ Clinical Features
tivity to percussion unless periapical inflammation is present.
♦♦ When the tooth with hypercementosis is extracted the ♦♦ Attrition of tooth is manifested by formation of well pol­
roots appear larger in diameter than normal and present ished facete on tip of cusps, incisal edges and on proximal
rounded apices. contact area of teeth.
Section 3: Oral Pathology  607

♦♦ In advanced cases attrition may lead to severe reduction ♦♦ There is presence of cupping of occlusal surfaces with
in cuspal height with complete wearing of enamel and dentin exposure.
flattening of occlusal surface. ♦♦ Presence of increased incisal translucency
♦♦ When dentin is exposed it becomes discolored brown. ♦♦ Wearing away of nonoccluding surfaces.
♦♦ Attrition in proximal surface of teeth causes trans­formation ♦♦ Amalgam restorations in erosive teeth get raised.
of proximal contact point to relatively bordered contact areas. ♦♦ Hypersensitivity is seen with the affected tooth.
♦♦ Exposure of dentinal tubules in severe cases of attrition ♦♦ Pulp exposure is seen in deciduous teeth.
leads to hypersensitivity. Q.8. Enumerate the diseases of cementum and describe the
♦♦ Attrition may also lead to pulp exposure. regressive alterations of cementum.
(Nov 2008, 15 Marks)
Ans.

Enumeration of Diseases of Cementum


Based on clinical radiographic and histological features boney
lesions of cementum were classified by Pindborg et al (1971).
♦♦ Periapical cemental dysplasia (Cementoma)
♦♦ Benign cementoblastoma
♦♦ Cementifying fibroma
♦♦ Gigantiform cementoma
♦♦ Cemento-osseous dysplasia

Regressive Alterations of Cementum


Fig. 93: Attrition (For color version, see Plate 16) ♦♦ With aging, surface of cementum become more irregular.
♦♦ As thickness of cementum increases due to hypercemen­
Treatment
tosis permeability of cementum decreases.
♦♦ Correction of developmental abnormalities. ♦♦ Under light microscope only the surface layer of cemen­
♦♦ Correction of parafunctional chewing habits. tocytes appear viable. All other lacunae become empty.
♦♦ Protection of tooth by metal or metal ceramic crowns where ♦♦ Greater amount of cementum appear in apical zone, mid­
structural defects are present. dle third of the root and furcation areas.
Q.7. Write short note on attrition, abrasion, abfraction and ♦♦ With age as a result of functional influences on teeth the
erosion. (Jan 2012, 5 Marks) (July 2016, 5 Marks) location and shape of apical foramina undergo changes.
Ans. For attrition refer to Ans 6 of same chapter. ♦♦ Due to aging cementum can resorb or deposit creating
reversal lines.
For abrasion refer to Ans 1 of same chapter.
♦♦ Hypercementosis: Refer to Ans 5 of same chapter.
Abfraction ♦♦ Cementicles: Calcifying spherical bodies composed
of cementum either lying free within PDL, attached to
Abfraction is the loss of tooth surface at the cervical areas of cementum or within it. They are mostly 0.5 mm in size.
teeth caused by the tensile and compressive forces during tooth They usually are ovoid or round with similar appearance
flexure. to denticles and are classified as free, attached or embed­
ded. Cementicles are a response to either local trauma or
Clinical Features hyperactivity and appears in increasing numbers in an
♦♦ Affects buccal/labial cervical areas of teeth. aging person. Found in 35% of human roots.
♦♦ Deep, narrow V-shaped notch.
♦♦ Commonly affect single teeth with excursive interferences Pathogenesis or Formation of Cementicles
or eccentric occlusal loads Calcification of nest of epithelial cells

Erosion
Enlargement by further deposition of calcium
It is defined as progressive loss of hard dental tissue by chemical salts in adjacent surrounding connective tissue
process not involving bacterial action.

Clinical Features Continued peripheral calcification of connective tissue

♦♦ Broad concavities are present on tooth with smooth sur­


Pattern of calcification give rise to cementicle
face enamel.
608   Mastering the BDS IIIrd Year  (Last 25 Years Solved Questions)

♦♦ Most commonly affected teeth are facial surfaces of bicus­


pids and molars.
♦♦ There is presence of a narrow V-shaped notch.
♦♦ Single tooth is affected most commonly with excessive
interferences or eccentric occlusal loads.
♦♦ Tooth becomes hypersensitive and there is wearing of
occlusal surface.
♦♦ Since lateral forces damage the cervical area of tooth,
repeated restoration failures are appreciated.

Treatment
♦♦ Etiological agent should be removed.
♦♦ Progression of the problem is stopped by bite guard.
♦♦ Areas of V-shaped notching should be restored by tooth
colored restorative material.
Q.11. Write short note on erosion. (Jan 2016, 5 Marks)
Fig. 94: Cementicle Ans. Erosion is defined as progressive loss of hard dental
tissue by chemical process not involving bacterial action.
Q.9. Define attrition, abrasion and erosion.
(Feb 2014, 3 Marks) Etiology
Ans. Attrition: Attrition is defined as physiologic wearing
away of a tooth as a result of tooth to tooth contact as in Dissolution of mineralized part of tooth occurs due to intrinsic
mastication. causes and extrinsic causes.
Abrasion: Abrasion is the pathologic wearing away of tooth Extrinsic causes are acidic beverages and citrus fruits.
substance through some abnormal mechanical process. Intrinsic causes are gastroesophageal reflux and vomiting
Erosion: Erosion is defined as irreversible loss of dental
Extrinsic Causes
hard tissue by a chemical process that does not involve
bacteria. ♦♦ It consists of acidic foods or due to iatrogenic exposure.
Q.10. Write short note on abfraction. (June 2015, 5 Marks) ♦♦ Extrinsic causes consist of acidic beverages and foods,
 (Jan 2017, 5 Marks) (Jan 2018, 5 Marks) dietary acids, medication, environmental acids, sport
Ans. Abfraction is defined as the loss of tooth surface at the drink, fruit juices, etc.
cervical areas of teeth caused by tensile and compressive ♦♦ Erosion is commonly seen in professional swimmers and
forces during tooth flexure. occupational wine tasters.

Pathogenesis Intrinsic Causes

When occlusal forces are applied eccentrically to a tooth the ♦♦ Gastric acids which regurgitate in esophagus and mouth
tensile stress is concentrated at cervical fulcrum which causes because of acid reflux or due to excessive vomiting.
flexure that leads to disruption in the chemical bonds of enamel ♦♦ Anorexia bulimia can also cause erosion of teeth, mostly
crystals in cervical areas. Once damaged, cracked enamel can the palatal surfaces of maxillary anterior teeth are involved.
be lost or more easily removed by erosion or abrasion. Clinical Features
♦♦ Broad concavities are present on tooth with smooth sur­
Etiology face enamel.
♦♦ Static forces: These forces are produced during swallowing, ♦♦ There is presence of cupping of occlusal surfaces with
dentin exposure.
clenching and tongue thrusting.
♦♦ Presence of increased incisal translucency
♦♦ Cyclic forces: These forces are produced at the time of
♦♦ Wearing away of non-occluding surfaces.
chewing.
♦♦ Amalgam restorations in erosive teeth get raised.
♦♦ People with open bite or deep class I cavity are prone for
♦♦ Hypersensitivity is seen with the affected tooth.
abfraction.
♦♦ Pulp exposure is seen in deciduous teeth.
Clinical Features Treatment
♦♦ It can occur at any age. ♦♦ Proper etiology should be ruled out.
♦♦ It affects buccal or labial cervical third or gingival third ♦♦ Cases of acid reflux should be sent to physician for proper
area of teeth. treatment.
Section 3: Oral Pathology  609

♦♦ In cases with hyposalivation chewing gums are used to


enhance the salivary flow.
Q.12. Write short note on pulp stones. (Mar 2016, 3 Marks)
Ans. Pulp stones are nodular calcified masses appearing on
coronal portions of the pulp organs.
♦♦ They are asymptomatic unless they impinge nerves on
blood vessels.
♦♦ Pulp stones are classified according to their structures as
true and false denticles.
♦♦ They are also classified as free, attached and embedded
depending on the relation to dentin.
♦♦ Pulp stones develop around central nidus of pulp tissue
♦♦ Pulp stones may eventually fill substantial part of pulp
chamber.
♦♦ Pulp stones may arise as a part of age-related changes or
local pathologic changes. Fig. 96: False pulp stones (For color version, see Plate 15)
Pathogenesis of True Pulp Stone Histopathology
♦♦ Development of true denticles is caused by inclusion of ♦♦ Pulp stones show central amorphous mass of irregular
remnants of epithelial root sheath within pulp. These calcification which is surrounded by concentric lamellar
epithelial remnants induce cells of pulp to differentiate rings of regular calcified material.
into odontoblasts which form dentin masses known as ♦♦ Occasionally peripheral layer of tubular dentin is applied
true pulp stone. by odontoblasts which arise from surrounding pulp tissue
in response to pulp stone present.
♦♦ Fibrillar irregular calcified material can also be seen on
periphery of pulp stone.

Clinical Significance
♦♦ Pulp stones are cause of pain, varying from mild pulp neu­
ralgia to severe excruciating pain resembling of trigeminal
neuralgia.
♦♦ Difficulty is encountered in extirpating the pulp during
root canal therapy if pulp stones are present.

Treatment
No treatment is required.

Q.13. Write short note on pulp calcification.


Fig. 95: True pulp stones (For color version, see Plate 15)  (Apr 2017, 4 Marks)
Ans. Pulp calcifications are of three types, i.e. denticles, pulp
Pathogenesis of False Pulp Stone stones and diffuse calcifications.
All of the pulp calcifications start as free bodies in pulp
tissue but many of them get attached or embedded in
dentinal walls of the pulp.

Denticles

♦♦ They occur due to the epitheliomesenchymal interaction


inside the developing pulp.
♦♦ Epithelial strands which originate from the root sheath
or cervical extensions in pulp chamber adjacent to
furcation, induce differentiation of osteoblasts of sur­
rounding mesenchyme of dental papilla forming the
core of denticle.
610   Mastering the BDS IIIrd Year  (Last 25 Years Solved Questions)

♦♦ Odontoblasts deposit tubular dentin as they migrate away Clinical Significance of Pulp Calcifications
from central epithelium and produce thimble-shaped ♦♦ Pulp stones are cause of pain, varying from mild pulp neu­
structures which surround the epithelium. ralgia to severe excruciating pain resembling of trigeminal
♦♦ Denticles are formed at the time of root development and neuralgia.
occur in root canal and pulp chamber adjacent to furcation ♦♦ Difficulty is encountered in extirpating the pulp during
areas of multirooted teeth. root canal therapy if pulp stones are present.
♦♦ Most of the denticles remain attach or embedded in dentin. ♦♦ Pulp calcifications can also interfere with formation of
Histopathology root causing early periodontal destruction and tooth loss.
♦♦ Prominent pulp calcifications are associated with some dis­
♦♦ Denticles consist of tubular dentin which surround central eases such as dentin dysplasia, pulp dysplasia, Calcinosis
nest of epithelium. As time progresses this central epithe­ universalis and Ehlers Danlos syndrome.
lium degenerates and tubules undergo sclerosis making
their detection difficult. Treatment of Pulp Calcifications
♦♦ Mainly denticles are free or embedded but those remain
No treatment is required.
free in pulp at times develop outer layers of irregular
fibrillar calcification or lamellated layers of calcification. Q.14. Discuss attrition and abrasion. (May 2018, 3 Marks)
Ans. For attrition in detail refer to Ans 6 of same chapter.
Pulp Stones For abrasion in detail refer to Ans 1 of same chapter.
♦♦ Pulp stones are nodular calcified masses appearing on
coronal portions of the pulp organs.
♦♦ They are asymptomatic unless they impinge nerves on
blood vessels.
15. Healing of Oral
♦♦ Pulp stones are classified according to their structures as Wounds
true and false denticles.
♦♦ They are also classified as free, attached and embedded Q.1. Enumerate various factors which promotes the healing
depending on the relation to dentin. process. Describe the healing of an extraction socket.
♦♦ Pulp stones develop around central nidus of pulp tissue
(Sep 2004, 15 Marks)
♦♦ Pulp stones may eventually fill substantial part of pulp
chamber. Ans. Healing is defined as restoration to a normal, mental or
physical condition especially of an inflammation and
♦♦ Pulp stones may arise as a part of age related changes or
wound.
local pathologic changes.

Histopathology Factors which Promotes the Healing Process

♦♦ Pulp stones show central amorphous mass of irregular ♦♦ Localization of wound:


calcification which is surrounded by concentric lamellar Wounds in the area in which there is good vascular bed
heal helps in rapid healing of wound.
rings of regular calcified material.
♦♦ Physical factors:
♦♦ Occasionally peripheral layer of tubular dentin is applied
• Mild traumatic injury favors the healing process.
by odontoblasts which arise from surrounding pulp tissue
• Local temperature in the area of wound influences the
in response to pulp stone present.
rate of healing by its effect on local circulation and cell
♦♦ Fibrillar irregular calcified material can also be seen on
multiplication.
periphery of pulp stone.
♦♦ Circulating factors:
Diffuse Calcifications Good blood supply of wound tissues promotes healing
process.
♦♦ Diffuse calcification is also termed as “calcific degenera­
♦♦ Nutritional factors:
tion.”
• Presence of protein enhances the speed of wound
♦♦ They are areas of fine, fibrillar, irregular calcification which
healing.
are parallel to vasculature.
• Presence of vitamin C accelerates the rate of wound
♦♦ They persist as fine calcified spicules and sometimes de­ healing process.
velop into larger masses. • Vitamin A and D accelerates the wound healing.
♦♦ Diffuse calcifications are found in pulp chamber or root • Vitamin B complex promotes wound healing.
canal. ♦♦ Age of patient:
♦♦ Diffuse calcifications may surround the blood vessels. Wounds in younger patients heal rapidly due to increased
Histopathology circulatory insufficiency and presence of protein synthesis.
♦♦ Infection:
♦♦ Its pattern is amorphous, unorganized linear strands or A wound which is exposed to mild physical irritation or
columns paralleling blood vessels and nerves of pulp. expose to bacteria heals quickly.
Section 3: Oral Pathology  611

Healing Process of an Extracted Socket ♦♦ Pigmentory changes: These are common in healing of
wounds on skin and appear as hypopigmented or hyper­
Immediate reaction following an extraction:
pigmented areas, e.g. lichen planus and lichenoid reactions.
After the removal of a tooth, blood which fills in the socket ♦♦ Cicatrization: It refers to late reduction in the size of the
coagulates, red blood cells get entrapped in the fibrin meshwork scar in contrast to immediate wound contraction. It is
and the ends of blood vessels in periodontal ligament are complication due to burns of skin.
sealed off. ♦♦ Implantation cyst: Epithelial cells may slide or get en­
trapped later in the wound and later may proliferate to
Healing in First Week
form implantation cyst.
♦♦ There is proliferation of fibroblasts from connective tissue ♦♦ Healing after pulpal diseases: It depends upon the degree
and these fibroblasts grow into a clot. of infection, inflammation, amount of the pulpal tissue
♦♦ There is endothelial proliferation which shows capillary involved and age of the patient.
growth. ♦♦ Healing after periapical diseases: It may result in the
♦♦ During, this period blood clot begins to undergo organiza­ formation of fibrosis in the involved area.
tion by ingrowth of fibroblasts and occasionally by small
Q.3. Write short note on dry socket. (Nov 2014, 3 Marks)
capillaries from residual periodontal ligament.
♦♦ Crest of alveolar bone shows beginning of osteoclastic (Sep 2005, 5 Marks) (Apr 2007, 5 Marks) (June 2014, 5
activity. Marks) (Dec 2010, 3 Marks) (July 2016, 5 Marks)
Or
Second Week Wound Healing
Write note on dry socket. (Feb 2006, 2.5 Marks)
♦♦ During this period, remnants of PDL gradually undergo Or
degeneration.
Write short answer on dry socket.(May 2018, 3 Marks)
♦♦ Wall of bony socket appears slightly frayed.
Ans. It is also known as “Alveolitis Sicca Dolorosa” or
♦♦ Margins of alveolar socket exhibits prominent osteoclastic
alveolalgia or postoperative osteitis or localized acute
resorption and fragments of necrotic bone are seen in the
alveolar osteomyelitis or alveolar osteitis.
process of resorption or sequestration.
• Dry socket is the common complication in healing
Third Week Wound Healing of the extraction wound.
• It commonly occurs in the mandibular or molar
♦♦ The clot is completely organized by maturation of granu­ areas.
lation tissue. • Dry socket is a very painful condition and the patient
♦♦ New uncalcified bone is formed around the periphery of
often has a foul breath.
wound from the socket wall.
• Clinical examination reveals a socket devoid of clot
♦♦ Original cortical bone of alveolar socket undergoes re­
and bony walls of socket are bare and visible.
modeling.
• Histological section of socket bone reveals formation
♦♦ Crest of alveolar bone is rounded off by osteoclastic re­
of necrotic bone containing empty lacunae.
sorption.
• There is balance inflammatory reaction in surround­
Fourth Week Wound Healing ing bone.
• Zinc oxide eugenol pack is given in the socket for
♦♦ There is continuous deposition, remodeling and resorption
palliative reaction.
of the bone filling in alveolar socket.
♦♦ Due to absorption of alveolar crest bone filling of the socket Q.4. Describe the healing of fracture wound and complica-
does not extend beyond the alveolar crest. tion in healing of the fracture. (Mar 1998, 15 Marks)
Ans. Healing of fracture wound
Q.2. Describe the healing of an extraction socket wound
1. Immediate effect of a fracture
and complications of wound healing.
• After fracture haversian vessels of bone are torn
(Mar 1997, 15 Marks)
at the fracture site so the vessels periosteum and
Ans. For healing of extraction socket refer to Ans 1 of same marrow cavity cross the fracture line.
chapter. • There is considerable extravasation of blood in
fractured area but at the same time, there is lack
Complications of Wound Healing
of circulation and loss of blood supply.
♦♦ Infection: Wounds may provide a portal entry to microso­ • Due to disruption of blood supply and tearing
mal infections which delay the healing process. of blood vessels there is death of bone marrow
♦♦ Keloid and hypertrophic scar formation: Keloids are the adjacent to fracture line.
overgrowth scar tissues with no tendency for resolution. • Blood clot which forms plays an important role
They occur in wounds which heal without any complication. in healing of fracture through replacement by
Hypertrophic scars occur in wounds where healing is granulation tissue and its subsequent replace­
delayed. ment of bone.
612   Mastering the BDS IIIrd Year  (Last 25 Years Solved Questions)

2. Callus formation Q.6. Write note on biopsy.


• It is the structure which unites the fracture end  (Dec 2010, 8 Marks) (Feb 1999, 5 Marks)
of bone and consists of fibrous tissue, cartilage  (Sep 2002, 6 Marks) (Sep 2011, 3 Marks)
and bone. Or
• There are two types of callus: Write short note on biopsy. (Dec 2015, 3 Marks)
1. External callus: It consists of new tissue  (July 2016, 3 Marks)
which forms around the outside of two frag­ Ans. Biopsy is the removal of tissue from the living organism
ments of bone. for the purpose of microscopic examina­tion and
2. Internal callus: It consists of new tissue aris­ diagnosis.
ing from the marrow cavity.
• Periosteum is an important structure in callus Types of Biopsy
formation and ultimate healing of the fracture. ♦♦ Excisional biopsy: If a lesion is totally excised for histological
• Cells of the periosteum torn at the fracture line evaluation it is called as excisional biopsy.
usually die but peripheral to the area there is a ♦♦ Incisional biopsy: When only a small section of tissue is
flurry cellular activity within hours of injury. removed from the lesion for the purpose of histological
• The fibrous layer of periosteum is inert and lifted evaluation it is called incisional biopsy.
away from the surface of bone by proliferation ♦♦ Fine needle aspiration cytology (FNAC): It is done by aspirat­
of cell in osteogenic layer of periosteum which ing tissue material inside a lesion which is later on diag­
assumes features of osteoblasts which, in turn, nosed microscopically after preparing a smear.
begins the formation of small amount of new ♦♦ Frozen section biopsy: It is performed in order to get an
bone at some distance from the fracture site. immediate histological report of a lesion. The tissue is
• There is continuous proliferation of osteogenic obtained from the lesion and is kept in deep freeze and
cells forming collar of callus over the surface of then frozen tissue is sectioned and stained to get a prompt
fracture. diagnosis.
• New bone which begins to form an external cal­ ♦♦ Punch biopsy: A small cylindrical punch is applied to the
lus consists of irregular trabeculae. lesion through full thickness of skin and a plug of tissue
• This differentiation of cells into osteoblast and is removed.
subsequent formation of bone occurs in deepest
part of callus collar. Biopsy Procedure
• In rapidly growing area of collar, osteogenic ♦♦ Area of wound from where the biopsy is taken and cleaned
layer differentiates into chondroblasts rather first.
than osteoblast and forms cartilage. ♦♦ Area is anesthetized.
• The cartilage fuses with bone and begins to ♦♦ Most representative site of wound is identified.
calcify by endochondral bone formation. The ♦♦ A section of tissue from the identified site of wound is
calcified cartilage is gradually resorbed and removed.
replaced by the bone. ♦♦ Tissue is cleaned and put into 10% formalin solution for
• Shortly after the fracture endosteum proliferates fixation.
within the week to form new bone which unites ♦♦ Biopsy site is sutured after achieving hemostasis.
and establish the continuity of bone. ♦♦ Biopsy specimen is sent to the histopathologist for diag­
• After this bone external and internal callus re­ nosis after labeling it properly.
model to form indistinguishable bone. Also refer to Ans 8 of same chapter.

Complications Q.7. Write note on exfoliative cytology.


 (June 2015, 5 Marks) (Sep 1999, 5 Marks)
♦♦ Delayed union and non-union of the fragments of bones  (Jan 2012, 5 Marks) (Mar 2006, 5 Marks)
are the complication of the healing process. They result  (Aug 2012, 5 Marks)
when the calluses of osteogenic tissue over each of two
Or
fragments fail to meet and fuse.
Write in brief about exfoliative cytology
♦♦ Local infection and presence of foreign bodies
♦♦ Fibrous union in fracture arises usually as a result of lack (Sep 1999, 5 Marks) (Jan 2012, 5 Marks)
of immobilization of damaged bone. Ans. Exfoliative cytology is the microscopic study of cells
♦♦ Lack of calcification of newly formed bone in the callus obtained from the surface of an organ or lesion after
may occur. suitable staining.

Q.5. Write about healing of oral tissues. (Mar 2007, 3 Marks) Procedure of Exfoliative Cytology
Ans. For healing of extraction socket of oral cavity refer to ♦♦ Surface of lesion is cleaned by removing all debris and
Ans 1 of same chapter. mucins.
For healing of fracture of jaw bone refer to Ans 4 of same ♦♦ Gentle scrapping is done on the surface of lesion with metal
chapter. cement spatula or a moistened tongue blade for several times.
Section 3: Oral Pathology  613

♦♦ Thus material present on the surface of lesion are adhered Healing by Primary Intention
or collected at the border of instrument. ♦♦ In healing by primary intension wound edges are approxi­
♦♦ The collected material is then evenly spread over a micro­ mated by surgical sutures.
scopic slide and is immediately fixed with 95% of alcohol. ♦♦ In the initial phase, there will be formation of blood clot,
♦♦ The slide is then air dried and is stained by a special stain which helps to hold the parts of the wound together.
called as PAP stain (Papanicolaou stain). ♦♦ The tissue becomes edematous and an inflammatory
Findings in Exfoliative Cytology process starts, with the infiltration of polymorphonuclear
neutrophils (PMN) and lymphocytes into the area.
Class I (Normal): It indicates that only normal cells are present ♦♦ The tissue debris collected in the wound are cleared either
in the smear. by the process of phagocytosis or by their lysis with the
Class II (Atypical): It indicates the presence of minor cellular help of proteolytic enzymes, liberated by the inflamma­
atypia. tory cells.
Class III (Intermediate): This is an in between cytology that ♦♦ Once the tissue debris are cleared, granulation tissue
separates cancer from non-cancer diagnosis, the cells which forms that replaces the blood clot in the wound, and it
display wider atypia are suggestive of cancer. usually consists of young blood capillaries, proliferating
Biopsy is recommended for further diagnosis. fibroblasts, PMN and other leukocytes.
♦♦ The epithelium at the edge of the wound starts to prolifer­
Class IV (Suggestive cancer): It indicates that in the lesion there
ate and gradually it covers the entire wound surface.
is presence of few cells with malignant characteristic. Biopsy
♦♦ Finally, the healing process is complete with progressive
is mandatory.
increase in the amount of dense collagen bundles and
Class V (Positive of cancer): The cells exhibit definite features of decrease in the number of inflammatory cells in the area.
malignancy. Biopsy is mandatory.
Healing by Secondary Intention
Indication of Exfoliative Cytology ♦♦ When the opposing margins of the wound cannot be approxi­
The exfoliative cytology is establishing in diagnosis of following mated together by suturing, the wound fills in from the base
oral lesion: with the formation of a larger amount of granulation tissue,
♦♦ Herpes simplex and herpes zoster such type of healing of the open wound is known as healing
♦♦ Pemphigus vulgaris by “secondary intention” or “secondary healing”.
♦♦ Pemphigoid ♦♦ Secondary healing occurs essentially by the same pro­
♦♦ Squamous cell carcinoma cess as seen in the primary healing, the only difference
♦♦ Aphthous ulcer. is that a more severe inflammatory reaction and an
exuberant fibroblastic and endothelial cell proliferation
Q.8. Define biopsy. Enumerate indications and contraindi-
occur in the later.
cations of biopsy. (Sep 2011, 3 Marks)
♦♦ In secondary healing, once the blood clot is removed, the
Ans. Biopsy is the removal of tissue from the living organism
granulation tissue fills up the entire area and the epithe­
for the purpose of microscopic examination and
lium begins to grow over it, until the wound surface is
diagnosis.
completely epithelized.
Indications ♦♦ Later on, the inflammatory exudates disappear slowly and
the fibroblasts produce large amounts of collagen.
♦♦ Lesions that cannot be diagnosed by clinical and radiologi­ ♦♦ Most of the healing processes occurring due to second­
cal examination. ary intention, result in scar formation at the healing site.
♦♦ Lesions which does not respond to treatment However, in the oral cavity these are rare.
♦♦ For confirmation of precancerous conditions and lesions
♦♦ Lesions which exhibit rapid growth, paresthesia or loss Q.10. Enumerate the different techniques for biopsy taking.
of function. Describe in detail exfoliative cytology.
 (Feb 2013, 16 Marks)
Contraindications Ans.
♦♦ Acute inflammatory condition such as cellulitis Enumeration of Different Techniques for Biopsy Taking
♦♦ Normal anatomical variations such as linea alba ♦♦ Excisional biopsy
♦♦ Patients with bleeding disorders such as hemophilia ♦♦ Incisional biopsy
♦♦ Vascular lesions such as hemangioma ♦♦ Intraosseous biopsy
♦♦ Patients who are on anticoagulant therapy. ♦♦ Punch biopsy
Q.9. Write about healing of oral cavity injuries. ♦♦ Frozen section biopsy
(Mar 2011, 5 Marks) (Sep 2011, 5 Marks) ♦♦ Oral brush biopsy or Oral CDX test
Ans. When the cut surfaces of the oral cavity injuries be ♦♦ Fine needle aspiration cytology
approximated or closely sutured, the wound heals up by For details of exfoliative cytology refer to Ans 7 of same
primary intention. The process occurs in the following way: chapter.
614   Mastering the BDS IIIrd Year  (Last 25 Years Solved Questions)

Q.11. Write short note on factors affecting healing of wound. • Vitamin A and D accelerates the wound healing.
 (Jan 2016, 5 Marks) • Vitamin B complex promotes wound healing.
Ans. Following are the factors affecting wound healing: ♦♦ Age of patient
♦♦ Localization of wound Wounds in younger patients heal rapidly due to increased
Wounds in the area in which there is good vascular bed circulatory insufficiency and presence of protein synthesis.
heal helps in rapid healing of wound. ♦♦ Infection
♦♦ Physical factors A wound which is exposed to mild physical irritation or
• Mild traumatic injury favors the healing process. expose to bacteria heals quickly.
• Local temperature in the area of wound influences the ♦♦ Radiation
rate of healing by its effect on local circulation and cell Low dosages of ionizing radiation stimulate the healing
multiplication. while large doses suppress the healing. UV radiation fa­
♦♦ Circulating factors cilitates the healing.
Good blood supply of wound tissues promotes healing ♦♦ Hormonal factors
process. Adrenocorticotropic hormone and cortisone interfere with the
♦♦ Nutritional factors healing process. Patients receiving these hormones the growth
• Presence of protein enhances the speed of wound of granulation tissue is inhibited due to inhibition of fibroblast
healing. proliferation, angiogenesis and decreasing of inflammatory
• Presence of vitamin C accelerates the rate of wound reaction. Diabetes mellitus is the most common disease in which
healing process. healing is hindered after surgical procedures.
Section 3: Oral Pathology  615

IV Disturbance of the Metabolism


Clinical Features
16. Oral Aspects of Meta- ♦♦ Lassitude, anorexia, painful limbs and enlargement of
bolic Diseases costochondral junction.
♦♦ Hair follicle rises above the skin and there is perifollicular
Q.1. Write short note on pathologic calcification. hemorrhage.
(Feb 2006, 6 Marks) ♦♦ Hemorrhage may occur in the joint in the nerve sheath
Ans. Pathologic calcification is abnormal deposition of the under the nails or conjunctiva.
calcium in various tumors and orgAns of the body. ♦♦ Scorbutic child usually assumes the frog like position.
They are of three types:
1. Dystrophic calcification Oral Manifestations
2. Metastatic calcification ♦♦ It occurs chiefly in gingival and periodontal region.
3. Calcinosis ♦♦ Interdental and marginal gingiva is bright red, swollen,
smooth, shiny surface producing scurvy bud.
Dystrophic Calcification
♦♦ There is presence of typical fetid breath of the patient with
♦♦ It is a type of pathologic calcification in which calcium fusospirochetal stomatitis.
salts are deposited in the dead or degenerating tissue of ♦♦ In severe cases, hemorrhage and swelling of periodontal
the body. ligament membrane occurs followed by loss of bone and
♦♦ It is not associated with increased level of serum calcium loosening of teeth which are exfoliated.
and is related to change in local environment.
♦♦ This is the most frequent type of pathological calcification Histopathology
found in wide variety of tissues. For details refer to Ans 5 of same chapter.
♦♦ In oral cavity dystrophic calcification is found in gingiva,
tongue, cheek and pulp. Treatment
♦♦ One of the most common intraoral dystrophic calcification
found in the pulp of the teeth, i.e. “Pulp Stone.” Vitamin C 250 mg TDS daily is given.
Q.3. Write short note on hyperpituitarism.
Metastatic Calcification
(Sep 2004, 5 Marks)
♦♦ Abnormal deposition of calcium in the tissue due to in­ Ans. It results from hyperfunction of anterior lobe of pituitary
crease in amount of serum calcium. gland, most significantly if increases production of
♦♦ It occurs particularly in diseases like hyperparathy­roidism growth hormone.
which depletes the bone calcium and causes high level of
blood calcium. Types
♦♦ Metastatic calcification also occurs in hyper­vitaminosis D.
1. Gigantism: If the increase occur before the epiphysis of long
In this type of calcification, deposit of calcium occurs in
bones are closed, gigantism results.
kidney, lung, gastric mucosa and media of blood vessels.
2. Acromegaly: If the increase occur later in the life after epi­
Calcinosis physis, closure results in acromegaly.
♦♦ Abnormal deposition of calcium under the skin is also Oral Manifestations of Hyperpituitarism
known as calcinosis.
♦♦ Teeth in gigantism are proportional to size of jaw and
♦♦ There are two forms of calcinosis:
the rest of the body and the root may be longer than the
1. Calcinosis circumscripta: It is circumscribed form.
normal.
2. Calcinosis universalis: It is generalized form and is
♦♦ Mandibular condylar growth is very important, over­
associated with scleroderma and dermato­myositis.
growth of mandible leads to prognathism.
Q.2. Write short note on scurvy. (Sep 2006, 5 Marks) ♦♦ Mandible may be of extraordinary proportions creating a
(Sep 2007, 2.5 Marks) (Sep 2008, 3 Marks) major discrepancy between the upper and lower jaw and
(Dec 2009, 5 Marks) (Dec 2010, 8 Marks) class III malocclusion.
Ans. Scurvy is prolonged deficiency of vitamin C and is ♦♦ The palatal vault is flattened and tongue increases in the
characterized by: size and causes crenation on its lateral border.
• Microvessels having least muscular support. ♦♦ In edentulous patients, enlargement of alveolus may pre­
• Defective synthesis of osteoids. vent the comfortable fit of complete denture.
• Impaired wound healing. ♦♦ The lip becomes thick and Negroid.
616   Mastering the BDS IIIrd Year  (Last 25 Years Solved Questions)

Treatment Q.4. Write short note on vitamins. (Feb 2013, 5 Marks)


♦♦ Transsphenoidal surgery should be done. Ans. Each vitamin is typically used in multiple reactions, and,
♦♦ Octreotide lowers the growth hormone. therefore, most have multiple functions.
♦♦ Dopamine antagonists are used.

Vitamin Vitamin chemical Solu- Recom- Deficiency disease Overdose disease Food sources
generic names(s) bility mended
descriptor dietary al-
name lowances
Vitamin A Retinol Fat 900 mg Night-blindness, Hypervitaminosis A Orange, ripe
hyperkeratosis and yellow fruits, leafy
keratomalacia Vegetables, carrots,
pumpkin, squash,
spinach, liver, soy
milk, milk
Vitamin B1 Thiamine Water 1.2 mg Beriberi, Wernicke- Drowsiness or muscle Pork oatmeal, brown
Korsakoff syndrome relaxation with large rice, vegetables,
doses potatoes, liver, eggs
Vitamin B12 Cyanocobalamin, Water 2.4 µg Megaloblastic anemia Acne-like rash Meat and other
hydroxy-cobalamin animal products
methylcobalamin
Vitamin B2 Riboflavin Water 1.3 mg Ariboflavinosis Dairy products,
bananas, popcorn,
green beans,
asparagus
Vitamin B3 Niacin, niacinamide Water 16.0 mg Pellagra Liver damage Meat, fish, eggs,
many vegetables,
mushrooms tree nuts
Vitamin B5 Pantothenic acid Water 5.0 mg Paresthesia Diarrhea; possibly Meat, broccoli,
nausea and heartburn avocados
Vitamin B6 Pyridoxine, Water 1.3–1.7 mg Anemia peripheral Impairment of Meat, vegetables,
pyridoxamine, neuropathy proprioception, nerve tree nuts, bananas
pyridoxal damage (doses >100
mg/day)
Vitamin B7 Biotin Water 30.0 µg Dermatitis, enteritis Raw egg yolk, liver,
peanuts, certain
vegetables
Vitamin B9 Folic acid, folinic acid Water 400 µg Megaloblast and May mask symptoms Leafy vegetables,
deficiency during of vitamin B12 pasta, bread cereal,
pregnancy is associated deficiency; other liver
with birth defects, such effects
as neural tube defects
Vitamin C Ascorbic acid Water 90.0 mg Scurvy Vitamin C Many fruits and
megadosage vegetables, liver
Vitamin D Cholecalciferol Fat 10 µg Rickets and Hypervitaminosis D Fish, eggs, liver,
Osteomalacia mushrooms
Vitamin E Tocopherols, Fat 15.0 mg Deficiency is very rare; Increased congestive Many fruits and
tocotrienols mild hemolytic anemia in heart failure seen in one vegetables, nuts and
newborn infants large randomized study. seeds
Vitamin K Phylloquinone, Fat 120 µg Bleeding diathesis Increases coagulation Leafy green
menaquinones in patients taking vegetables such as
warfarin egg yolks spinach, liver
Section 3: Oral Pathology  617

Q.5. Describe histologic features with diagram of scurvy. calcium absorption from gut which results in low serum
(Nov 2008, 5 Marks) calcium levels.
Ans. Following are the histologic features of scurvy: ♦♦ Tertiary hyperparathyroidism: As secondary hyperparathy­
• In scurvy osteoblasts fail to form osteoid on spicules roidism remains for longer time it is known as tertiary
of calcified cartilage matrix. hyperparathyroidism.
• Cartilage cells of epiphyseal plate proliferate
normally and salts are deposited in matrix between Clinical Features
column of cartilage cells. ♦♦ It occurs mainly from 3rd to 6th decade of life.
• A wide zone of calcified but non-ossified matrix ♦♦ Female predilection is seen with male to female ratio of 1:3
known as scorbutic lattice develop in metaphysis. ♦♦ Patient has classic triad of kidney stones, resorption of
• As scorbutic lattice increases in width more fragile bone and duodenal ulcers.
zone develops which leads to complete fracture of ♦♦ Patient usually complains of back pain and blood in urination.
spicules with separation and deformity of cartilage ♦♦ Patient also suffers from emotional unstability.
shaft junction. Fracture of calcified matrix material ♦♦ Presence of gastrointestinal problems is present, i.e.
lead to the classic picture of scurvy known as nausea, vomiting, anorexia.
Trummerfeld zone. ♦♦ In severe cases, there is presence of headache, bone pain,
• Area beneath trummerfeld zone is free of hemat­ pathological fractures and comma.
opoietic cells and is formed of connective tissue cells
known as Gemest-mark. Oral Lesions
♦♦ A tumor like swelling is present either intraorally or extra-
orally which is known as Brown’s tumor.
♦♦ Mandible is affected more commonly.
♦♦ Presence of jaw bone fractures is present.
♦♦ There is presence of drifting, loosening and exfoliation
of the teeth.
♦♦ Fetid odor or halitosis is present.
♦♦ Malocclusion is present because of drifting and spacing
of the teeth.
Histopathology
♦♦ There is presence of osteoclastic resorption of multiple
boney trabeculae and there is also formation of new bone
by osteoblast cells.
Fig. 97: Scurvy (For color version, see Plate 16) ♦♦ Areas of excessive hemorrhage and hemosiderin pigmen­
Q.6. Write short note on hyperparathyroidism. tation are present.
(Dec 2010, 8 Marks) (Jan 2018, 4 Marks) ♦♦ Multiple multinucleated osteoclast type of giant cells are
Ans. Hyperparathyroidism is an endocrine disorder in which often seen in tumor.
there is secretion of excess of circulating parathyroid ♦♦ At places bone marrow is replaced by fibrous connective
hormone. tissue.
• Excess of parathyroid hormone stimulate osteoclast ♦♦ As disease progresses osteoclastomas develop which are
and mobilize calcium from bone which causes characterized by masses of fibroblasts growing in loose
hypercalcemia. syncytium.

Types of Hyperparathyroidism Treatment


♦♦ Primary hyperparathyroidism: In this excess secretion of ♦♦ Administration of vitamin D and dietary phosphate
parathyroid hormone is present because of parathyroid supplements.
adenomas. ♦♦ Parathyroidectomy should be done if patient does not
♦♦ Secondary hyperparathyroidism: It occurs when parathyroid respond to the medicinal treatment.
continuously produced in response to low levels of serum
calcium. This is related to chronic renal diseases. Kidney Q.7. Write short essay on oral lesions of hyperparathy-
utilizes vitamin D which is important for calcium absorp­
roidism. (Jan 2012, 5 Marks)
tion from gut. So in patient with chronic renal disease
active form of vitamin D is not produced and there is less Ans. Refer to Ans 6 of same chapter.
618   Mastering the BDS IIIrd Year  (Last 25 Years Solved Questions)

V Diseases of specific System

17. Diseases of Bone ♦♦ There is occurrence of pale blue sclera which is thin and
pigmented choroids shows through and produces blue color.
and Joints ♦♦ There is deafness due to osteosclerosis, laxity of ligaments
and peculiar shape of skull.
♦♦ Increase tendency for capillary bleeding.
Q.1. Write short note on cleidocranial dysplasia
(June 2014, 5 Marks) Oral Manifestations
Ans. It is also called as cleidocraniodysostosis or “Marie and
♦♦ Osteogenic imperfecta is associated with dentino­genesis
Santon disease.”
imperfecta.
Etiology ♦♦ There is hypoplasia of teeth.
♦♦ Deciduous teeth are poorly calcified and semi-translucent
♦♦ It appears as true dominant Mendelian characteristic. or waxy.
♦♦ It is transmitted as an autosomal dominant trait with com­ ♦♦ Teeth appear as faintly dirty pink, half normal size, with
plete penetrance and variable expressivity.
globular crown and relative short roots, in proportion to
Oral Manifestation other dimensions.
♦♦ Maxilla and paranasal air sinuses are under­developed Histopathological Features
resulting in maxillary micrognathia.
♦♦ Osteoblastic activity appears as retarded and imperfect.
♦♦ Maxilla is underdeveloped in relation to mandible.
♦♦ Failure of fetus to be transformed with mature collagen.
♦♦ There is prolonged retention of primary dentition.
♦♦ The trabeculae of cancellous bone are delicate and often
♦♦ There is complete absence of cementum.
♦♦ Disorganization of developing permanent dentition. show fracture.
♦♦ There is presence of supernumerary teeth usually in Treatment
anterior region.
♦♦ High narrow arched palate and cleft palate is common. No known treatment.
♦♦ Roots of teeth are often short and thinner than the normal. Q.3. Enumerate fibro-osseous lesion. Describe in detail
♦♦ The crown may be pitted as a result of enamel hypoplasia. monostotic fibrous dysplasia. (Sep 2005, 14 Marks)
Clinical Features Ans. In fibrous dysplasia, normal bone is replaced by the benign
fibrous tissue showing varying amount of mineralization.
♦♦ There is complete absence of clavicle.
♦♦ It primarily affects skull, clavicle and dentition. Classification of Fibro-osseous Lesions
Treatment ♦♦ Developmental:
• Solitary bone cyst
♦♦ Not specific
• Gigantiform cementoma
♦♦ Dental care should be taken.
• Cherubism
Q.2. Write note on osteogenesis imperfecta. ♦♦ Reactive/Reparative:
(Feb 2015, 5 Marks) (Mar 1996, 6 Marks) • Aneurysmal bone cyst
Ans. It is also called brittle bone or lobstein disease or fragilitas • Central giant cell granuloma
ossium or osteopsathyrosis. • Garre’s osteomyelitis
This is an autosomal dominant condition affecting bone • Osseous dysplasia
formation. –– Florid osseous dysplasia
It presents a hereditary autosomal dominant trait. –– Cemental osseous dysplasia
–– Focal osseous dysplasia or sclerosing osteomyelitis
Types • Osseous keloid
♦♦ Congenital type • Traumatic periostitis
♦♦ Lobstein type. ♦♦ Neoplasms:
• Benign cementoblastoma
Clinical Features • Ossifying fibroma
♦♦ It usually occur in infants. –– Conventional
♦♦ There is extreme fragility or porosities of bone with prone­ –– Juvenile trabecular
ness of fracture. –– Juvenile psammomatoid
Section 3: Oral Pathology  619

• Osteoma Or
• Osteoid osteoma Write short note on fibrous dysplasia.
• Osteoblastoma
 (Jan 2016, 5 Marks (Nov 2014, 3 Marks)
♦♦ Endocrinal/Metabolic: Brown tumor of hyperparathyroidism
 (Dec 2009, 5 Marks)
♦♦ Idiopathic:
• Fibrous dysplasia Ans. Fibrous dysplasia is an idiopathic condition in which
• Paget’s disease. an area of normal bone is gradually replaced by
abnormal fibrous connective tissue which then again
Monostotic Fibrous Dysplasia undergoes osseous metaplasia and eventually the bone
is transformed into dense lamellar bone.
Monostotic fibrous dysplasia is a condition in which single bone
is involved and is replaced by abnormal fibrous connective Types of Fibrous Dysplasia
tissue which undergoes osseous metaplasia and the bone is
♦♦ Monostotic fibrous dysplasia: Only one of the bone is in­
transformed into dense lamellar bone.
volved.
Clinical Features ♦♦ Polyostotic fibrous dysplasia: More than one bone is involved.
• Jaffe's type: Variable number of bones are involved
♦♦ It involves only one bone and pigmented skin lesions are
accompanied by pigmented lesion of skin or cafe-au-
present.
lait spots.
♦♦ The sites which affected are maxilla, mandible, ribs and
• Albright syndrome: It is a severe form of fibrous
femur.
dysplasia involving all the bones in the body,
♦♦ It occurs in children younger than 10 years and both the
accompanied by pigmented lesions of the skin and
sexes are equally affected.
endocrine disturbances of various types.
Oral Manifestations
Clinical Features
♦♦ Maxilla is more commonly affected than the mandible and
♦♦ It is seen during the first and second decades of life.
most of the changes occur in posterior region. The most
♦♦ Disease is more common among the females
common area involved is premolar area.
♦♦ Polyostotic fibrous dysplasia commonly involve skull,
♦♦ There is presence of unilateral facial swelling which is slow
facial bone, clavicle, pelvic bone, etc. and monostotic
growing with intact overlying mucosa.
fibrous dysplasia involve maxilla and mandible.
♦♦ Swelling is painless but patient may feel discomfort.
♦♦ Monostotic form of disease can never be transformed into
♦♦ There is presence of enlarging deformities of alveolar
polyostotic form.
process mainly buccal and labial cortical plates.
♦♦ In mandible, there is protrusion of inferior border of For more clinical features and oral manifestations refer to Ans
mandible. 3 of same chapter.
♦♦ Teeth present in the affected area are malaligned and Histopathology
tipped or displaced.
♦♦ Supernumerary teeth are often impacted and affect the ♦♦ Fibrous dysplasia reveals presence of highly cellular,
eruption of the teeth. proliferating well vascularized fibrillar connective tissue
which replaces the normal bone.
Histopathology ♦♦ Within fibrous tissue the fibroblasts cells are arranged in
♦♦ Lesion is essentially a fibrous bone made up of proliferating “Whorled pattern”.
fibroblasts in compact stroma of inter lacing collagen fibers. ♦♦ Trabeculae represent the Chinese letter pattern.
♦♦ Irregular trabeculae of bone are scattered throughout the ♦♦ Osteoblastic and osteoclastic activity may be present in
lesion. relation to some trabeculae of bone.
♦♦ Some of the trabeculae are C-shaped and described as
Chinese character shaped.
♦♦ There is permanent maturation arrest in woven bone stage.

Treatment
♦♦ Surgical removal of lesion
♦♦ Osseous countering is necessary for characterizing
deformity for aesthetic or pre esthetic purposes.
Q.4. Write short note on monostotic fibrous dysplasia.
(Sep 2004, 5 Marks)
Ans. Refer to Ans 3 of same chapter.
Q.5. Write note on fibrous dysplasia. (Feb 2002, 6 Marks)
 (Mar 1998, 5 Marks) Fig. 98:  Fibrous dysplasia (For color version, see Plate 16)
620   Mastering the BDS IIIrd Year  (Last 25 Years Solved Questions)

Treatment ♦♦ Newly formed bone may again resorbed by osteoclast


causing loss of normal architecture of bone.
Surgical removal of lesion.
♦♦ Chronic inflammatory cells and dilated blood capillaries
Q.6. Write note on Paget’s disease of bone. are present within the bone.
(Sep 2006, 5 Marks) (Sep 1999, 6 Marks) ♦♦ Bone resorption and deposition results in irregular
(Apr 2007, 10 Marks) (Aug 1998, 5 Marks) fragments of bone formation characterized by prominent
basophilic reversal and resting lines, and they produce
Or
mosaic pattern in bone.
Write notes on histopathology of Paget’s disease.
(Feb 2006, 2.5 Marks)
Or
Write in detail on Paget’s disease. (Apr 2008, 10 Marks)
(Feb 2006, 2.5 Marks)
Or
Write short note on Paget’s disease.
 (Apr 2015, 3 Marks) (June 2015, 5 Marks)
Ans. Paget’s disease is a relatively uncommon bony disorder
which is characterized by the excessive uncoordinated
phases of resorption and deposition of osseous tissue in
single or multiple bones.
It is also known as “Osteitis Deformans.” Fig. 99:  Paget’s disease of bone (For color version, see Plate 16)

Etiology Treatment
♦♦ Inflammatory ♦♦ Bisphosphonate therapy should be given.
♦♦ Circulatory disturbance ♦♦ Calcitonin is administered.
♦♦ Genetic and environmental factors ♦♦ Surgical correction for bone deformities, fractures and
♦♦ Others: Vasculitis, trauma, hormonal balance and degenera­ severe degenerative arthritis
tive neurological disorders. Q.7. Write note on cementoma. (Mar 1998, 5 Marks)
Clinical Features Or
♦♦ It occurs during 5th, 6th and 7th decades of life. Write short note on cementoma. (Mar 2003, 5 Marks)
♦♦ Males are more commonly affected. Ans. It is a true neoplasm of functional cementoblasts which
♦♦ It is prone to occur in axial skeleton especially in skull, forms large masses of cementum like tissue or tooth root.
femur, sacrum and pelvis.
♦♦ Most of the patient complain initially of the deep and aching Recent Concept
bone pain with bilaterally symmetrical swelling of bone. Cementoma these days is known as periapical cemento osseous
♦♦ Enlargement of skull, bowing of legs and curvature of dysplasia, this is because cementoma is an unorganized
spine are commonly seen. productivity of bone, PDL membrane cementum complex.
Oral Manifestations Clinical Features
♦♦ Maxilla is more commonly involved than the mandible. ♦♦ It occurs most frequently under the age of 25 years.
♦♦ There is movement and migration of affected teeth and ♦♦ Mandible is affected more commonly than maxilla.
malocclusion. ♦♦ Mandibular first molar is most commonly affected tooth.
♦♦ As the disease progresses, the mouth remains open ♦♦ It produces a slow enlarging bony hard swelling of jaw
exposing the teeth. which causes expansion of jaw and displacement of
♦♦ Extraction site heals slowly and incidence of osteomyelitis regional teeth.
is higher. ♦♦ Expansion of buccal and lingual cortical plates.
♦♦ Low-grade intermittent pain is present which felt more
Histopathology
when area is palpated.
♦♦ Osteoclastic bone resorption occurs and the bone is ♦♦ A dull sound is produced when tooth is percussed.
replaced by highly vascularized cellular connective tissue.
♦♦ Osteoclasts are usually larger and may contain over 100 Histopathology
nuclei. ♦♦ It presents a large mass of amorphous cemental tissue
♦♦ Deposition of new lamellar or woven bone within the which shows the presence of multiple reversal line.
connective tissue by osteoblast cells occur and fatty ♦♦ Intervening connective tissue stroma contains many
hemopoietic bone marrow is replaced by the fibrous stroma. cementoblasts and cementoblasts cells.
Section 3: Oral Pathology  621

♦♦ PDL adjacent to normal cementum becomes integrated Q.10. Write short note on cherubism.
with capsule and separate the neoplasm from surrounding (Jan 2012, 5 Marks) (Dec 2009, 5 Marks)
bone.
Or
♦♦ Multinucleated cells are present in the large number in
central area and are associated with active resorption. Describe in brief cherubism (Jan 2010, 5 Marks)
♦♦ Root of the involved tooth extends up to the center of
Or
lesion and neoplastic cemental tissue is continuous with
normal cemental layer. Write short essay on cherubism. (Jan 2012, 5 Marks)
Ans. Cherubism is an autosomal dominant fibro-osseous
Treatment lesion of the jaw which involves more than one quadrant
Surgical excision is done. that stabilizes after growth period leading to facial
deformity and malocclusion.
Q.8. Give classification of bone disorders of face and
jaw. Write shortly about the clinical features and Clinical Features
histopathology of fibrous dysplasia. ♦♦ Children of age 1 to 3 years are more commonly affected.
 (Mar 2007, 8 Marks) Males are more commonly affected than females.
Ans. Bone disorders of face and jaw: ♦♦ Affected children are normal at birth but as soon the child’s
1. Developmental defects of bone formation of face growth take place self-limited bone growth begins to slow
and jaw: down till patient reaches 5 years of age and stop at the age
• Agnathia of 12–15 years.
• Micrognathia ♦♦ There is deforming mandibular and maxillary overgrowth
• Macrognathia with respiratory obstruction and impairment of vision
• Facial hemiatrophy and hearing.
• Facial hemihypertrophy ♦♦ Enlargement of cervical lymph nodes contributes to
• Cleft palate. patient’s full faced appearance.
2. Benign and malignant lesions of bone: ♦♦ A rim of sclera is may be beneath the iris, giving classic
• Osteoma eye to heaven appearance.
• Osteosarcoma Oral Manifestations
• Ewing’s sarcoma.
3. Fibrosseous lesions ♦♦ Agenesis of 2nd and 3rd molars of mandible.
• Fibrous dysplasia of bone ♦♦ Displacement of the teeth
• Ossifying fibroma ♦♦ Premature exfoliation of primary teeth
• Cementifying fibroma ♦♦ Delayed eruption of permanent teeth
♦♦ Transposition and rotation of teeth
• Paget’s disease of bone
♦♦ In severe cases tooth resorption occurs.
• Cherubism
• Osteogenesis imperfecta Histopathology
• Cleidocranial dysplasia
♦♦ Lesion presents a highly cellular and vascular connective
• Hurler’s syndrome
tissue stroma, which is often arranged in a “whorled
• Garre’s osteomyelitis
pattern”.
• Jaw lesions in hyperparathyroidism
♦♦ Numerous proliferating fibroblasts and variable numbers
• Aneurysmal bone cyst.
of multinucleated giant cells are also found within the
For clinical features and histopathology of fibrous stroma.
dysplasia refer to Ans 5 of same chapter. ♦♦ Giant cells are relatively smaller in size and they often
Q.9. Classify fibro-osseous lesions of jaw. Give histopathol- aggregate around the thin-walled blood capillaries.
ogy of Paget’s disease. (Sep 2008, 5 Marks) ♦♦ A hallmark of the disease is the presence of an “eosinophilic
perivascular cuffing” of collagen fibers, which often
Or
surrounds the blood capillaries.
Classify fibro-osseous lesions of jaw. Write about clini- ♦♦ Within the connective tissue extravasated blood and
cal features, histopathology of Paget’s disease. deposits of hemosiderin pigments are sometimes seen.
(Mar 2011, 6 Marks) ♦♦ Lymph nodes exhibit reactive hyperplasia, fibrosis and
Ans. Refer to Ans 3 and 6 of same chapter. chronic inflammatory cell infiltration.
622   Mastering the BDS IIIrd Year  (Last 25 Years Solved Questions)

Or
Classify fibro-osseous lesions. Write in detail about
fibrous dysplasia. (July 2016, 10 Marks)
Ans. For classification of fibro-osseous lesions refer to Ans 3
of same chapter.
Fibrous dysplasia is a skeletal developmental anomaly
of the bone forming mesenchyme which manifest as a
defect osteoblastic differentiation and maturation.

Etiology/Etiopathogenesis
♦♦ Genetic: Fibrous dysplasia is caused by postzygotic muta­
tion in GNAS1 gene which encodes a G-protein which
Fig. 100: Cherubism (For color version, see Plate 17) stimulates production of cAMP. Mutation leads to exces­
sive secretion of cAMP which leads to hyperfunction of
Treatment endocrine organs, increase proliferation of melanocytes
No treatment is required as cherubism is a self-limiting disease. and affect differentiation of osteoblasts. If mutation occur
in early preembryonic life polystotic fibrous dysplasia
Q.11. Write short note on histopathology of Paget’s disease occur. If mutation occur in early postembryonic life mon­
and cherubism. (Sep 2011, 3 Marks) ostotic fibrous dysplasia occur.
Ans. For histopathology of paget’s disease refer to Ans 6 of ♦♦ Developmental: As the disease begins in early life, it was
same chapter. considered to be developmental.
For histopathology of cherubism refer to Ans 12 of same ♦♦ Endocrine: Some of the investigators thought that it occurs
chapter. due to local endocrinal disorders.
Q.12. Classify fibro-osseous lesions of oral cavity. Discuss Clinical Features
clinical features, radiographic differential diagnosis
♦♦ Monostotic Fibrous Dysplasia
and management of fibrous dysplasia.
• It involves only one bone and pigmented skin lesions
(Nov 2008, 20 Marks)
are present.
Or
• The sites which affected are maxilla, mandible, ribs
Classify fibro-osseous lesions of jaw. Discuss the clini-
and femur.
cal and histopathological features of fibrous dysplasia.
• It occurs in children younger than 10 years and both
(Dec 2007, 8 Marks)
the sexes are equally affected.
Or
• Maxilla is more commonly affected than the mandible
Classify fibro-osseous lesions of jaws. Describe in
and most of the changes occur in posterior region. The
detail fibrous dysplasia. (May/Jun 2009, 15 Marks)
most common area involved is premolar area.
Or
• There is presence of unilateral facial swelling which is
Classify fibro-osseous lesions. Discuss in detail about
slow growing with intact overlying mucosa.
the etiology, clinical, radiographic and histopathologic • Swelling is painless but patient may feel discomfort.
features of fibrous dysplasia. (Aug 2012, 10 Marks) • There is presence of enlarging deformities of alveolar
Or process mainly buccal and labial cortical plates.
Enumerate fibro-osseous lesions of jaws and de- • In mandible, there is protrusion of inferior border of
scribe fibrous dysplasia in detail.(Feb 2015, 10 Marks) mandible.
Or • The teeth present in the affected area are malaligned
Enumerate fibro-osseous lesions. Describe in detail the and tipped or displaced.
clinical radiological and histopathological features of • Supernumerary teeth are often impacted and affected
fibrous dysplasia. (Dec 2015, 8 Marks) the eruption of teeth.
Or ♦♦ Polyostotic Fibrous Dysplasia
Classify fibro-osseous lesions affecting the jaws.Write • It is seen during first and second decades of life.
in detail about the etiopathogenesis, clinical features, • Female predilection is seen with male to female ratio
radiographic features of fibrous dysplasia. of 1:3.
 (Apr 2017, 10 Marks) • Most common sites to be involved are skull, facial
Or bones, clavicle, thighs, shoulder, chest and neck.
Enumerate the fibro-osseous lesions involving orofa- • Café-au-lait spots are seen over the skin which are
cial region. Describe fibrous dysplasia in detail with irregular in shape and are light brown melanotic spots.
diagram. (Jan 2017, 10 Marks) • Patient complains of recurrent bone pain.
Section 3: Oral Pathology  623

• There is expansion of jaws and asymmetry of facial Mature Stage of Fibrous Dysplasia
bones which leads to enlargement and deformity. ♦♦ Paget’s disease: Ground glass appearance is present in both
• Teeth eruption is not proper. Paget’s disease and fibrous dysplasia. But in Paget’s disease
complete effect is rarefaction.
Radiographic Features
Histopathology
Lesions Representing Dominancy of Fibrous Tissue Or
Osteolytic Stage of Fibrous Dysplasia Refer to Ans 5 of same chapter.
♦♦ Such lesions are generally radiolucent with ill defined
borders. Defect in bone is unilocular but at times bony Management
septa presents a picture of multilocular activity. ♦♦ Surgical removal of the lesion should be done.
♦♦ Margins of lesion are well defined. ♦♦ Osseous contouring is done to correct the deformities so
♦♦ Loss of lamina dura is evident. that esthetics of patient should be improved.
♦♦ Resorption of roots is evident.
Lesions presenting mixed radiolucent and radiopaque picture Q.13. Describe in detail the serum investigations done in
Or Mixed stage of fibrous dysplasia bone disorders and give detailed account of fibrous
dysplasia. (Dec 2012, 8 Marks)
♦♦ The lesion got both the fibrous and osseous tissue so, i.e.
the lesion presents both mixed radiopaque and radiolu­ Ans. Serum investigations in bone disorders.
cent picture. Following are the serum investigations done in bone
♦♦ Newly formed bone represents multiple small opacities disorders:
of poor density. As they increases in size they appear
granular. Serum Calcium
Mature Lesions with Radiopacity or Mature Stage of Fibrous ♦♦ In circulation, the calcium is present in three forms namely
Dysplasia ionized calcium (48–50%) protein-bound fraction (40%)
♦♦ In this stage, bone is prominent, i.e. they are referred to as and rest as complex calcium.
mature lesions with radiopacity. ♦♦ The estimated normal value of calcium is around 9 to 10.6
♦♦ Radiograph reveals orange peel appearance. mg/dL and because of diurnal variation it reaches its peak
♦♦ In the affected area; teeth are bodily displaced and tilted. value in mid-day and lowest in early morning. It is essential
♦♦ Bony expansion is seen on both distal and buccal aspect. to correct serum calcium values to serum albumin levels
♦♦ As mandible get involved, vertical depth of bone is in­ as per formula.
creased. Inferior border of mandible appears ribbon like
cortex. In various cases, area over cortex is lost, there is a Free Serum Calcium
curved downward projection of inferior margins of mandi­ ♦♦ Serum calcium (mg/dL) + [4–serum albumin (g%) × 0.8].
ble. This appearance is called as thumb print appearance. It is advisable to avoid tourniquet while drawing blood
♦♦ It also shows ground glass appearance. for serum calcium estimation irrespective of fasting or
non-fasting state.
Radiographic Differential Diagnosis
Ionized Calcium
Osteolytic Stage of Fibrous Dysplasia ♦♦ It comprises 48 to 50% of total serum calcium, and is pri­
♦♦ Central Giant cell granuloma: Trabeculae in central giant cell marily responsible for physiological functions like muscle
granuloma are faint while in fibrous dysplasia internal contraction, coagulation and bone mineralization. The
calcifications are seen which are stippled and granular. normal value of ionized calcium is around 4.5 to 5.2 mg/dL
♦♦ Traumatic bone cyst: Cortical bulging as well as displace­ in fasting state (1.l2 to 1.3 micromoles/L) with maximum at
ment of teeth is present in traumatic bone cyst while it is 10:00 hours and minimum at 18:00 to 20:00 hours.
absent in fibrous dysplasia.
♦♦ Serum calcium decreases in osteomalacia, hypoparathy­
Mixed Stage of Fibrous Dysplasia roidism, secondary hyperparathyroidism and increases in
♦♦ Osteosarcoma: It presents its typical features as sunburst primary hyperparathyroidism.
pattern and Codman’s triangle while fibrous dysplasia
show granular appearance. Serum Phosphate
♦♦ Lymphoma of bone: Lymphoma of bone appears irregular ♦♦ Out of total phosphorus in plasma, Inorganic phosphate
as well as bizarre in radiograph while fibrous dysplasia is or phosphorus constitutes 1/3 fraction measurement of
smooth and has well contoured boney margins. which is clinically useful for assessment of metabolic bone
♦♦ Cemento ossifying fibroma: Shape of cemento ossifying fi­ disorders.
broma is rounded while of fibrous dysplasia is rectangular. ♦♦ The normal range of inorganic phosphate is 2.5–4.5 mg/dL
Margins in cemento ossifying fibroma are sharply defined with higher values in elderly male and postmenopausal
while in fibrous dysplasia they are indistinct. women. The inorganic phosphate is 20% protein bound
624   Mastering the BDS IIIrd Year  (Last 25 Years Solved Questions)

and rest is free ionic form. The serum phosphorus level age before that lesion regresses slowly. Face of the
is higher in infants and children. Phosphate is essential patient becomes normal in 4th and 5th decades of
to most biological systems. High levels are found in life.
renal failure and hypoparathyroidism, while low levels • Since the disease occur in childhood some of the
are associated with primary hyperparathyroidism, primary teeth at time are absent. This leads to gap­
hypophosphatemic rickets and osteomalacia. ping between the erupted deciduous teeth.
• Premature loss of primary teeth too is seen.
Serum Magnesium • Permanent dentition is often defective with absence
Magnesium is present mostly in ionized form (55%), protein of numerous teeth and displacement and lack of
bound (30%) and rest in complex form. The normal value of eruption of those present.
serum magnesium is 1.7 to 2.6 mg/dL. The hypomagnesemia • Malocclusion is also present.
is observed in hypoparathyroidism. Increase in serum Q.15. Write short note on trisomy 21. (Aug 2012, 5 Marks)
magnesium level is seen in hemolysis. Severe and prolonged Or
hypomagnesemia inhibits parathyroid hormone (PTH) release
Write short note on Down’s syndrome.
and induces resistance to PTH action on bones.
 (Jan 2018, 4 Marks)
Serum 25-hydroxyvitamin D Ans. It is also called as Down’s syndrome or Mongolism
• It occurs due to excessive chromosomal material
Vitamin D status is best assessed using serum 25-(OH)D3, as
involving a part of chromosome 21 or complete
1,25(OH)D3 has a short half-life and does not accurately reflect
chromosome 21.
true vitamin D status. Levels are only measured if disorders
• It is the form of a mental retardation which is associ­
of vitamin D metabolism are suspected. Whilst rickets and
ated with morphological features and many somatic
osteomalacia occur with vitamin D deficiency. The deficiency
abnormalities due to number of chromosomal aber­
is suspected at levels below <25 nmol/L (10 mg/mL) and
rations.
insufficiency is suspected at <75 nmol/L (30 mg/mL).
• Trisomy 21 is one of the cytologic variants of Down’s
Serum Alkaline Phosphatase syndrome and occurs in 95% of patients with Down’s
syndrome.
♦♦ The normal value in adults is 40–150 IU/L and in children • There is typical trisomy 21 with 47 chromosomes.
11-306 IU/L. The bone specific serum alkaline phosphatase
(ALP) a marker of osteoblastic activity is raised in Paget’s Clinical Features
disease of bones, metastatic bone disease, osteomalacia,
osteoporosis. However, it is important to exclude ♦♦ It occurs in children.
hepatobiliary disease where alkaline phosphatase level is ♦♦ Mental retardation is present.
also high, especially in patients with cholestasis. ♦♦ Head appears small, i.e brachycephaly.
♦♦ The bone specific alkaline phosphatase fraction has better ♦♦ Flat facies are present with hypertelorism.
predictive value in evaluating bone disorders. ♦♦ Nasal bridge is depressed and there is presence of broad,
For fibrous dysplasia in detail refer to Ans 12 of same chapter. short neck.
♦♦ Presence of narrow, upward and outward slanting of
Q.14. Write oral manifestations of cherubism.
palpebral fissures.
(Dec 2010, 8 Marks)
♦♦ Ocular abnormalities are seen such as strabismus, cataract
Ans. Following are the oral manifestations of cherubism: and retinal detachment.
• The most commonly affected site is angle of ♦♦ Skeletal abnormalities such as short stature, broad and
mandible bilaterally. short hands, feet as well as digits; dysplasia of pelvis; wide
• Patient complains of problems in speech, mastication, gap between first and second toes.
deglutition and have restricted jaw movements. ♦♦ Protuberant abdomen, hypogenitalism and delayed and
• Presence of enlargement of mandible bilaterally incomplete puberty.
which leads to the rounded appearance of the lower
face. This is followed by bilateral enlargement of
Oral Manifestations
maxilla.
• Alveolar process become wide, this is because to ♦♦ Macroglossia is present, scrotal tongue is seen.
occupy whole roof of mouth. ♦♦ Maxilla is hypoplastic.
• Palate becomes narrow fissure like between alveolar
♦♦ Tooth eruption is delayed. Partial anodontia and enamel
process.
hypoplasia can also be seen.
• Rapid expansion of mandible is seen at 7–8 years of
♦♦ High-arched palate is present.
age. After this age, lesion progresses very slowly till
♦♦ Cleft lip or cleft palate can be seen.
puberty. As puberty is over maxillary lesions regress.
Mandibular lesions start regressing after 20 years of ♦♦ Juvenile periodontitis is present.
Section 3: Oral Pathology  625

♦♦ There is fissuring and thickening of lips and angular Laboratory Findings


cheilitis is present.

18. Diseases of blood and


blood forming organs
Q.1. Write short note on pernicious anemia.
 (June 2015, 5 Marks) (Sep 2004, 5 Marks)
(Feb 2013, 5 Marks)
Ans. Pernicious anemia is a type of a chronic progressive,
megaloblastic anemia of adults and is caused by
deficiency of intrinsic factors in stomach.

Etiology
♦♦ Due to impaired absorption of vitamin B12: This occurs due
to the atrophy of gastric mucosa which results in lack of
Fig. 101:  Peripheral smear showing pernicious anemia
secretion of intrinsic factor. (For color version, see Plate 17)
♦♦ As autoimmune reaction to gastric parietal cells or intrinsic ♦♦ Macrocytosis is commonly seen.
factor. ♦♦ Red blood cell count is decreased, at times it is 10 lakh/
♦♦ Strict vegetariAns suffer from vitamin B12 deficiency as it cubic mm of blood.
is present only in eggs, meat and milk. ♦♦ Mild to moderate thrombocytopenia is also seen.
♦♦ Malabsorption of vitamin B12 due to inadequate gastric ♦♦ MCV, MCH, MCHC concentration is normal
production or defective functioning of intrinsic factor. ♦♦ Howell jolly bodies and cabot rings are present.
♦♦ Various other causes which lead to the deficiency of ♦♦ Neutrophils are hypersegmented.
vitamin B12 are gastrectomy, celiac disease, Crohn’s ♦♦ Buccal scrapings show nuclear abnormalities
disease, alcoholism, prolong usage of drugs, i.e. proton ♦♦ Schilling test show vitamin B12 deficiency.
pump inhibitors, colchicines. ♦♦ Serum lactate dehydrogenase is markedly increased while
serum potassium, cholesterol and skeletal alkaline phos­
Clinical Features phatase are often decreased.
♦♦ Bone marrow is hypercellular and show trilineage differen­
♦♦ Generalized weakness, palpitation, nausea, vomiting, tiation. Erythroid precursors are large and oval. Nucleus is
anorexia, diarrhea and dyspnea. large and contains coarse motley chromatin clumps giving
♦♦ Patients have smooth, dry and yellow skin. checkboard appearance.
♦♦ Neurological manifestation includes tingling sensation in
Treatment
hand and feet, paresthesia of extremities due to peripheral
nerve degeneration. IM injection of vitamin B12 is given.
♦♦ Glossitis, glossodynia (painful tongue) and glossopyrosis Q.2. Write notes on aplastic anemia.
(itching and burning tongue).  (Mar 1997, 5 Marks)
♦♦ Tongue appears beefy red in color. Ans. It is a rare disorder characterized by the peripheral
♦♦ Sometimes loss of papilla produces a bald appearance of blood pancytopenia (anemia, leukopenia and thrombo­
tongue. cytopenia) associated with bone marrow suppression.
♦♦ Sometimes hyperpigmentation occurs in mucosa.
Etiology
♦♦ Inflammation and burning sensation surround entire oral
mucosa. ♦♦ Drug and chemicals: Common drugs which can cause
aplastic anemia are benzene derivatives, chloramphenicol,
Histopathology penicillin, sulfonamides and anticancer drugs.
♦♦ Infections: Patients with bacterial disease such as tuber­
♦♦ Oral epithelial cells become enlarged and show hyperchro­ culosis and viral infections like hepatitis and infection
matic nuclei with prominent nucleoli along with serrated mononucleosis can cause pancytopenia.
nuclear membrane. ♦♦ Radiation: Long-term exposure to continuous radiation has
♦♦ Epithelium becomes atrophic. lead to development of aplastic anemia.
♦♦ There is presence of subepithelial chronic inflammatory ♦♦ Other causes: Pregnancy and thymoma can cause aplastic
cell infiltration. anemia.
626   Mastering the BDS IIIrd Year  (Last 25 Years Solved Questions)

Clinical Features ♦♦ Bleeding spots on skin or mucosal surface which do not


♦♦ It is more common in young adults and elderly individuals. blanch on pressure.
♦♦ Marked pallor of skin and petechiae. ♦♦ Women may have heavy menses or bleeding between
♦♦ Numbness and tingling of extremities. periods.
♦♦ Generalized edema of body (anasarca). ♦♦ Spontaneous gingival bleeding is present.
♦♦ Fever and infections occur due to neutropenia. ♦♦ Bleeding into TMJ results in pain and trismus.
♦♦ Spontaneous gingival bleeding and epistaxis. Treatment
♦♦ Multiple areas of ulceration in oral mucosa.
♦♦ Steroid therapy and repeated blood transfusions.
Treatment ♦♦ Splenectomy and immunosuppressive drug therapy is
Blood transfusion, splenectomy and bone marrow transplant. required.
Q.5. Write short note on hemophilia. (Sep 2005, 5 Marks)
Q.3. Write notes on sickle cell anemia.
 (Aug/Sep 1998, 5 Marks) Ans. Hemophilia is a potentially fatal inherited bleeding
Or disorder characterized by the profuse hemorrhage due
to deficiency of clotting factors.
Write short note on sickle cell anemia.
 (Mar 2007, 2.5 Marks) (Jan 2018, 4 Marks) Types
Ans. It is an autosomal dominant disorder.
♦♦ Hemophilia A or classic hemophilia: In this factor VIII
• In sickle cell anemia the erythrocytes assume
deficiency is present.
crescent shape and undergo lysis.
♦♦ Hemophilia B or christmas disease: In this factor IX defi­
Clinical Features ciency is present.
♦♦ von Willebrand’s disease: In this factor VIII deficiency is
♦♦ It is more common in females. present along with defective platelet function.
♦♦ Malaise, weakness and jaundice with yellow sclera.
♦♦ Pallor, loss of appetite and muscular rigidity. Clinical Features
♦♦ Fever, pain in abdomen and back in joints.
♦♦ Disease is mostly characterized by the easy bruising and
♦♦ Swelling in joints, hand and feet.
prolongs bleeding particularly often accidental, surgical
♦♦ There may be delayed eruption and hypoplasia of dentition.
and dental trauma.
♦♦ There is presence of mongoloid faces with high cheek
♦♦ Spontaneous bleeding into subcutaneous tissue or internal
bones and bimaxillary prognathism.
orgAns leads to recurrent soft tissue hematoma formation.
Radiographic Features ♦♦ Gastric hemorrhage may occur in case of gastric ulcer.
♦♦ Joint problems occur due to degenerative changes in joint
♦♦ Skull radiographs reveals multiple small spicules (small
structures, osteoporosis and muscle atrophy.
needle shaped body) across calvarium, which produces
♦♦ Severe hemorrhage from gingival tissue after dental
“Hair-on-end” appearance.
extraction.
♦♦ IOPA reveals “Step ladder” like trabeculae between con­
♦♦ Slight trauma may lead to hematoma formation in tongue,
tagious and posterior teeth.
lips and palate.
♦♦ Increased osteoporosis and hyperplasia of bone marrow.
♦♦ Hemophilic patient has high caries index and severe
Treatment periodontal disease.
No specific treatment. Treatment
Oxygen and blood transfusion in serious situation.
♦♦ Immediate transfusion of factor VIII or IX concentrate is
Q.4. Write notes on purpura. (Mar 1996, 5 Marks) primary treatment.
Ans. It is defined as purpulish discoloration of skin and ♦♦ Patient need transfusion in every 12 hours until bleeding
mucous membrane due to subcutaneous and submucus stops.
extravasation of blood. ♦♦ Analgesic and corticosteroids reduce joint pain and
swelling.
Clinical Features ♦♦ In mild hemophilia, use of IV desmopressin is done.
♦♦ Purpura commonly occurs among adults below Q.6. Write short note on oral manifestations of leukemia.
40 years and females are more commonly affected. (Mar 2011, 5 Marks)
♦♦ Sudden spontaneous occurrence of petechiae (small pin Ans. Following are the oral manifestations of leukemia
point hemorrhage under skin or mucosa), ecchymoses or • Patient has got gingival hyperplasia which is the
hematomas in skin and mucous membrane. constant feature of leukemia.
Section 3: Oral Pathology  627

• Gingiva become boggy, edematous and deep red Classification of Anemia


and bleed easily.
Etiological classification of anemia (By Lea and Febiger 1981)
• Gingival swelling is present due to leukemic infiltra­
♦♦ Loss of blood
tion.
• Acute posthemorrhagic anemia
• Purpuric lesions of oral mucosa analogus to cutane­
• Chronic posthemorrhagic anemia.
ous ecchymosis may also be seen.
♦♦ Excessive destruction of red blood corpuscles
• Loosening of teeth due to necrosis of PDL is present.
• Extracorpuscular causes
• Destruction of alveolar bone is seen.
–– Antibodies
• Crusting of lips is also seen. –– Infections like malaria
Q.7. Write short note on Plummer-Vinson syndrome. –– Splenic sequestration and destruction
(Dec 2010, 8 Marks) (Sep 2011, 3 Marks) –– Associated diseases like lymphomas
Ans. • Plummer-Vinson syndrome is also known as –– Drugs, chemical and physical agents
Paterson-Brown-Kelly syndrome. –– Trauma to RBC.
• Plummer-Vinson syndrome consists of following • Intracorpuscular hemolytic diseases
features i.e. dysphagia, iron deficiency anemia, –– Hereditary
koilonychia and glossitis. - Disorders of glycolysis
- Faulty synthesis or maintenance of reduced
Clinical Features glutathione.
- Qualitative or quantitative abnormalities in the
♦♦ It is seen in middle aged women.
synthesis of globulin
♦♦ Patient has characteristic asthenic appearance.
- Abnormalities in RBC membrane
♦♦ Esophageal webs are present which leads to dysphagia.
- Erythropoietic porphyria
♦♦ Tongue is smooth and red.
–– Acquired
♦♦ Angular cheilitis is also seen.
- Paroxysmal nocturnal hemoglobinuria
♦♦ Nails of the patient are spoon shaped. - Lead poisoning.
Diagnosis ♦♦ Impaired blood production resulting from deficiency of
substances essential for erythropoiesis
♦♦ Clinical diagnosis reveals all the above mentioned fea­ • Iron deficiency
tures. • Deficiency of various B vitamins: Vitamin B12 and folic
♦♦ During blood examination picture of iron deficiency ane­ acid (pernicious anemia and megaloblastic anemia);
mia is present. pyridoxine responsive anemia
♦♦ Biopsy reveals atrophy of epithelium, lamina propria as • Protein deficiency
well as muscles. • Possibly ascorbic acid deficiency
♦♦ Inadequate production of mature erythrocytes
Management
• Deficiency of erythroblast
♦♦ Iron deficiency anemia should be corrected by giving –– Atrophy of bone marrow: Aplastic anemia
hematinics to the patient. - Chemical or physical agents
♦♦ Esophageal dilatation should be done. - Hereditary
- Idiopathic.
Q.8. Define and classify anemia. Describe iron deficiency
–– Isolated erythroblastopenia
anemia. (Aug 2012, 15 Marks) - Thymoma
Or - Chemical agents
- Antibodies.
Write in detail on iron deficiency anemia.
• Infiltration of bone marrow
(May/Jun 2009, 10 Marks) –– Leukemia, lymphomas
Or –– Multiple myeloma
–– Carcinoma, Sarcoma
Write short note on iron deficiency anemia.
–– Myelofibrosis.
 (Feb 2015, 5 Marks) (Apr 2017, 4 Marks) • Endocrine abnormalities
Or –– Myxedema
–– Addison’s disease
Define anemia. Describe iron deficiency anemia in
–– Pituitary insufficiency
detail. (Jan 2017, 10 Marks) –– Sometimes hyperthyroidism.
Ans. Anemia is defined as abnormal reduction in number • Chronic renal failure
of circulating RBCs, quantity of hemoglobin and the • Chronic inflammatory disease
volume of packed red cells in a given unit of blood. –– Infectious
628   Mastering the BDS IIIrd Year  (Last 25 Years Solved Questions)

––
Non-infectious including granulomatous and ♦♦ Pigmentation can be seen over the dorsum of hand and
collagen disease. metacarpophalangeal joint which is also known as knuckle
• Cirrhosis of liver. pigmentation.
♦♦ Plummer-Vinson syndrome is associated with iron defi­
Morphological Classification of Anemia ciency anemia and other features are dysphagia, koilo­
Type of anemia Description Common causes nychia and glossitis.
Macrocytic Increased MCV, Lack of erythrocyte- Oral Manifestations
MCH and normal maturating factors
MCHC. (intrinsic and extrinsic ♦♦ Presence of pallor of oral mucosa as well as gingiva.
factors). ♦♦ Atrophy of oral mucosa is present. This is seen on tongue
and buccal mucosa.
Normocytic Reduction only Hemorrhage, hemolysis,
in RBC number, lack of blood formation ♦♦ Glossodynia, i.e. pain in the tongue and glossopyrosis, i.e.
Normal MCV, MCH and dilution of blood with reddening of tongue is present.
MCHC. fluid. ♦♦ Atrophy of filliform and fungiform papillae is present
Simple microcytic Reduced MCV, Associated with infec-
giving tongue a bald or smooth or glistening appearance.
MCH and normal tions and inflammatory ♦♦ Presence of cracking and fissuring is seen at corner of
MCHC. diseases. mouth suggestive of angular cheilitis.
♦♦ At times aphthous ulcers are also seen.
Hypochromic Reduced MCV, Iron deficiency.
microcytic MCH and MCHC. Laboratory Diagnosis
MCV = Mean corpuscular volume ♦♦ Examination of peripheral blood picture shows:
MCH = Mean corpuscular hemoglobin • Chromicity: Hypochromia of RBC
MCHC = Mean corpuscular hemoglobin concentration. –– Central pallor increased
–– Anisochromia present
Iron Deficiency Anemia • Size: Microcytic anisocytotic
Iron deficiency anemia results due to deficiency of iron in the • Shape: Poikilocytosis often present, pear shaped tailed
body. variety of RBC, elliptical form common.
• Target cell: Present
Etiology • Reticulocytes: Present
• Osmotic fragility: Slightly decreased
♦♦ Exogenous Cause: It is due to dietary deficiency
• ESR: Seldom elevated
♦♦ Endogenous Cause:
♦♦ RBC count is between 3,000,000 to 4,000,000 cells per cubic
• Absorption Defect as occurs in.
mm of blood.
–– Histamine, i.e. fast achlorhydria.
♦♦ Presence of low serum iron and ferritin with an elevated
–– Gastric operations, i.e. total gastrectomy and
total iron binding capacity.
partial gastrectomy.
♦♦ Hemoglobin level is at 4 mg/dL or below it.
• Enterogeneous:
♦♦ MCV, MCH and MCHC are decreased.
–– Unusual hurry in the passage of chyme.
–– Abnormality in absorbing mucosa.
• Transport Defect, i.e. atransferrinemia.
• Loss in Iron from Body:
–– Loss due to hemorrhage in peptic ulcer, hiatus
hernia, Ca stomach, Ca colon and hookworm
manifestation.
–– Excessive menstrual bleeding.
–– Excessive excretion.
• Increased Requirement of Iron in pregnancy and
lactation.
Clinical Features
♦♦ Females are more commonly affected during 4th and 5th Fig. 102:  Peripheral smear showing iron deficiency anemia
decades. (For color version, see Plate 17)
♦♦ Patient complains of lack of concentration, tiredness,
Treatment
headache, presence of tingling sensations over extremities.
♦♦ Nails of the patient become brittle and nail bed become ♦♦ Patient should be kept on oral iron therapy. Ferrous fu­
spoon shaped which is also known as koilonychia. marate or ferrous sulphate should be given to the patient.
♦♦ Patients feel difficulty in swallowing. This is also termed 300 mg tablet should be given 3 to 4 times a day for six
as dysphagia. months duration.
Section 3: Oral Pathology  629

Q.9. Describe in brief oral manifestations of anemia. Etiology


(Apr 2008, 5 Marks) (Jun 2010, 5 Marks)
♦♦ Chromosomal abnormality: Presence of an abnormal
Ans. For oral manifestations refer to Ans 8 of same chapter. chromosome, e.g. Philadelphia chromosome.
Q.10. Classify gingival hyperplasia and describe leukemia. ♦♦ Exposure to high doses of radiation therapy.
(Dec 2010, 18 Marks) ♦♦ Exposure to certain chemicals, e.g. phenyl butazone and
Ans. benzene.
♦♦ Myeloproliferative disorders like polycythemia vera.
Classification of Gingival Hyperplasia ♦♦ Congenital or genetic abnormalities.
On basis of etiological factors and pathologic changes ♦♦ Presence of primary immune deficiency
♦♦ Inflammatory enlargement ♦♦ Infection with human leukocyte virus.
• Chronic
Acute Leukemia
• Acute
♦♦ Drug Induced Enlargement Classification of Acute Leukemia (FAB Classification)
♦♦ Enlargement associated with systemic disease
1. Acute myeloblastic leukemia (AMD)
• Conditional enlargement
• M0: Minimally differentiated: Myeloblasts lack
–– Pregnancy
definite cytologic and cytochemical features but have
–– Puberty
–– Vitamin C deficiency myeloid lineage antigens.
–– Plasma cell gingivitis • M1: AML without maturation: Myeloblasts
–– Non-specific conditioned enlargement (Pyogenic predominate with distant nucleoli, few granules or
granuloma) Auer rods are present.
• Systemic diseases causing gingival enlargement • M2: AML with maturation: Myeloblasts with
–– Leukemia promyelocytes predominate and Auer rods may be
–– Granulomatous disease (E.g. Wegner’s Granulo­ present.
matosis, sarcoidosis) • M3: Acute promyelocytic leukemia: Hypergranular
♦♦ Neoplastic Enlargement promyelocytes often with multiple Auer rods are seen.
• Benign tumors • M4: Acute myelomonocytic leukemia: Mature cells
• Malignant tumors. of both myeloid and monocytic series in peripheral
♦♦ False Enlargement. blood; myeloid cells resemble M2
Using the criteria of location and distribution gingival • M5: Acute monocytic leukemia: Promonocytes or
enlargement is designated as follows: undifferentiated blast.
♦♦ Localized: Gingival enlargement limited to one or more • M6: Acute erythroleukemia: Erythroblast predominate;
teeth. myeloblasts and promyelocytes also increased
♦♦ Generalized: Involving the gingiva throughout the mouth. • M7: Acute megakaryocytic leukemia: Pleomorphic
undifferentiated blast cells predominate and react
♦♦ Marginal: Confined to marginal gingiva.
with anti-platelet antibodies.
♦♦ Papillary: Confined to interdental papilla.
II. Acute lymphoblastic leukemia
♦♦ Diffuse: Involving the marginal and attached papillae.
• L1: Acute lymphoblastic (Seen in children):
♦♦ Discrete: Isolated sessile or pedunculated tumor like en­ Homogeneous small lymphoblasts; scanty cytoplasm,
largement. regular round nuclei, inconspicuous nucleoli.
On basis of degree of gingival enlargement • L 2 : A c u t e l y m p h o b l a s t i c ( S e e n i n a d u l t s ) :
Heterogeneous lymphoblasts; variable amount of
Grade 0: No sign of gingival enlargement. cytoplasm, irregular or cleft nuclei, large nucleoli.
Grade I: Enlargement confirmed to interdental papilla. • L3: Burkitt’s type (Uncommon): Large homogenus
Grade II: Enlargement involves papilla and marginal gingiva. lymphoblasts; nuclei are round to oval, prominent
nucleoli, cytoplasmic vacuolation.
Grade III: Enlargement covers three quarters or more of the crown.
Clinical Features
Leukemia
Acute leukemias are more common in children and adults from
Leukemia is a disease which is characterized by overproduction of 1st to 4th decades of life.
WBCs which are present in circulating blood in an immature form. Clinical features are due to:
♦♦ Bone marrow failure
Types of Leukemia • Anemia is seen leading to pallor, lethargy and
♦♦ Acute dyspnea.
♦♦ Chronic • Pyrexia is present.
630   Mastering the BDS IIIrd Year  (Last 25 Years Solved Questions)

• Bleeding manifestations are present, such as


spontaneous bruises, petechiae, bleeding from gingiva.
• Infections of multiple orgAns are present.
♦♦ Organ infiltration
• Presence of pain and tenderness in bones.
• Enlargement of tonsils is present.
• Lymphadenopathy is seen.
• Hepatomegaly and splenomegaly are common
findings.
• Chloroma or granulocytic sarcoma is a localized
tumor mass occurring over the skin or orbit by local
infiltration of tissues by leukemic cells.
Fig. 104:  Acute lymphoblastic leukemia (For color version, see Plate 17)
Oral Manifestations
Chronic Myeloid Leukemia
♦♦ Bleeding from gingiva is present. Gingiva becomes boggy,
edematous and red in color. It consists of large leukemic cells and differentiated WBCs in
♦♦ Presence of paresthesia of lower lip. bone marrow.
♦♦ Crustation over lips is seen.
Clinical Features
♦♦ Mobility of permanent teeth is present.
♦♦ Oral mucosa appears pale with ulceration along with ♦♦ It occurs during 3rd and 4th decades of life.
petechiae and ecchymosis. ♦♦ Splenomegaly is present.
♦♦ Patient complains of loss of weight, abdomen prominence.
Laboratory Findings ♦♦ Presence of anemia leads to weakness, fatigue and dyspnea.
♦♦ Anemia: It is normochromic in type, moderate reticulocytes ♦♦ Petechiae and ecchymosis is seen.
are seen, few nucleated red cells.
Laboratory Findings
♦♦ Thrombocytopenia: Platelet count is below 50,000/µl of blood.
♦♦ WBC count: It ranges from sub-normal to markedly elevat­ ♦♦ Anemia is normocytic and normochromic type.
ed. It is 1,00,000/µl of blood in advanced cases. Leucocytes ♦♦ Marked leucocytosis is present, i.e. 2,00,000 cells/µl.
in peripheral blood are blast cells. ♦♦ There is increased proportion of basophils.
♦♦ Bone marrow examination shows hypercellularity. Bone ♦♦ Bone marrow shows hypercellularity with total or partial
marrow is packed with leukemic blast cells. Erythropoietic replacement of fat spaces by proliferating myeloid cells.
cells are reduced. Megaloblastic features and ring sidero­
blasts are present.

Fig. 105:  Chronic myeloid leukemia (For color version, see Plate 17)
Chronic Lymphocytic Leukemia
It is the malignancy of mature B cells.
Fig. 103:  Acute myeloid leukemia (For color version, see Plate 17)
Clinical Features
Chronic Leukemia ♦♦ It occurs during 4th decade of life.
These are hematologic malignancies in which predominant ♦♦ Male predilection is seen.
leukemic cells are initially well differentiated. They are of two ♦♦ Features of anemia are seen, i.e. weakness, fatigue and
types: dyspnea.
♦♦ Lymphadenopathy is commonly present.
1. Chronic myeloid leukemia ♦♦ Hepatomegaly and splenomegaly is commonly seen.
2. Chronic lymphocytic leukemia ♦♦ Hemorrhagic manifestations are common.
Section 3: Oral Pathology  631

Oral Manifestations Clinical Features


♦♦ Gingival hypertrophy is present. Ulceration of gingiva ♦♦ Features depend on the severity of hemolysis, if it is severe
with necrosis is present. child is stillborn or dead.
♦♦ Tongue is dark and swollen. ♦♦ Alive born child suffer from anemia with pallor, jaun­
♦♦ Presence of mobility of teeth is seen. dice, compensatory erythropoiesis, i.e. medullary and
♦♦ Necrosis of PDL is seen. extramedullary and edema.
♦♦ Alveolar bone destruction is also present.
Oral Manifestations
Laboratory Findings
♦♦ Deposition of blood pigments in enamel and dentin of
♦♦ Anemia is mild to moderate and is normocytic normo­
chromic type. neonate which produces green, brown or blue hue.
♦♦ Marked leucocytosis is present. ♦♦ Enamel hypoplasia can also be present.
♦♦ Leucocytes are mature small lymphocytes. ♦♦ A ring like defect is present at incisal edges of deciduous
♦♦ Platelet count is normal or moderately reduced. anterior teeth and middle portion of deciduous cuspid and
first molar crown known as Rh hump.

Histopathology
Ground section of the affected teeth shows positive test for
bilirubin.

Laboratory Findings
♦♦ RBC count at birth is from 1000000 cells/cu mm to normal
level.
♦♦ Peripheral smear shows normoblasts or nucleated red cells.
♦♦ High bilirubin level.
♦♦ Direct Coombs test on cord blood is positive.
Fig. 106:  Chronic lymphocytic leukemia (For color version, see Plate 18)
Treatment
Treatment No treatment is needed as the condition affects only deciduous
Chemotherapeutic drugs, radiation therapy and corticosteroids teeth and not the permanent teeth.
which leads to prolong remission and cures in some forms of
disease.

Q.11. Define anemia. Give morphological classification of


19. Diseases of Skin
anemia. Describe in detail pernicious anemia.
(Apr 2015, 8 Marks) Q.1. Name the vesiculobullous lesions of oral cavity.
Ans. For definition of anemia and morphological classification Describe etiology, histopathology and clinical features
of anemia refer to Ans 8 of same chapter. of oral lichen planus. (Dec 2010, 8 Marks)
For pernicious anemia in detail refer to Ans 1 of same chapter. Or
Write short answer on clinical features and histology
Q.12. Write short note on erythroblastosis fetalis.
of lichen planus (May 2018, 3 Marks)
 (July 2016, 5 Marks)
Ans. Erythroblastosis fetalis is a hemolytic anemia of newborn Ans.
secondary to blood incompatibility mainly Rh factor
Vesiculobullous Lesions
between the mother and fetus.
Currently this disorder is uncommon due to use of anti Fitzpatrick Classification
antigen gamma globulin at delivery in mothers with Rh
negative blood. ♦♦ According to Anatomical Plane:
• lntraepidermal Blister Granular Layer
Pathogenesis –– Pemphigus foliaceus
–– Frictional blisters
It occurs due to the inheritance by the fetus of a blood factor
from Rh positive father which act as foreign antigen to Rh –– Staphylococcus scalded syndrome.
negative mother. • Spinous Layer
Transplacental leak of RBCs from fetus to mother leads to –– Eczematous dermatitis
immunization of mother and formation of antibodies which –– Secondary to heat and cold
are transferred back to fetus transplacentally, this leads to fetal –– Herpes virus infection
hemolysis. –– Familial benign pemphigus
632   Mastering the BDS IIIrd Year  (Last 25 Years Solved Questions)

• Suprabasal ♦♦ Genetic factors: Lichen planus is reported in families, twins,


–– Pemphigus vulgaris husband and wife.
–– Pemphigus vegetans ♦♦ Infectious: By spirochete.
–– Darier’s disease ♦♦ Drugs and chemicals: It is responsible for the lichenoid
• Basal layer reaction.
♦♦ Psychogenic factors: Stress results, nervousness with emo­
–– Erythema multiforme
tional upset, overwork and some form of mental strain.
–– Toxic epidermal necrolysis
♦♦ Habit: Chewing of tobacco, betel nut and smoking.
–– Lupus erythematosis
–– Lichen planus Clinical Features
–– Epidermolysis bullosa simplex ♦♦ It occurs among the middle aged and elderly people.
♦♦ Dermal-Epidermal Junction Zone:
♦♦ There is slight predilection for the females.
• Lamina lucida
♦♦ Lichen planus can involve several areas of oral cavity.
–– Bullous pemphigoid
Oral lesion: Mucosal surface of buccal mucosa, vestibule,
–– Cicatrical pemphigoid
tongue, lips, floor of mouth, palate and gingiva.
–– Epidermolysis bullosa junctional
♦♦ Patient may report with burning sensation of oral mucosa.
• Below basal lamina
♦♦ Oral lesion is characterized by radiating white and gray
–– Erythema multiforme
velvety thread like papules in linear, angular or retiform
–– Epidermolysis bullosa dystrophica
arrangement, tiny white elevated dots are present at the
Vesiculobullous Lesions intersection of white lines known as “Wickhm’s striae”.
♦♦ Primary Blistering: Histopathology
• Pemphigus
• Bullous pemphigoid ♦♦ Overlying surface epithelium exhibits hyperortho­
• Cicatrical pemphigoid keratinization or hyperparakeratinization or both.
• Epidermolysis bullosa acqusita. ♦♦ Acanthosis of spinal cell layer is present.
♦♦ Secondary blistering: ♦♦ Shortened and pointed rete pegs of epithelium which
• Contact produces “Saw tooth” appearance.
• Erythema mutltiforme ♦♦ Intercellular edema in spinous cell layer is present.
• Toxic epidermal necrolysis. ♦♦ There is presence of necrosis or liquefaction degeneration
♦♦ Infection: of basal cell layer of epithelium.
• Varicella zoster ♦♦ Few rounded or ovoid, amorphous eosinophilic bodies are
• Herpes simplex present which are known as “Civatte bodies”.
• Bullous impetigo. ♦♦ These civatte bodies represent dead keratinocytes or other
♦♦ Systemic disease: necrotic epithelial components which are transported to
• Infection- cutaneous emboli connective tissue for phagocytosis.
• Metabolic ♦♦ Chronic inflammatory cell infiltration is present in juxta
–– Diabetic with bullae epithelial lesion.
–– Porphyria cutanea tarda.

Oral Lichen Planus

Lichen planus is also known as lichen ruber planus.


♦♦ It is a precancerous condition
♦♦ Lichen planus is a common mucocutaneous disease which
arises due to an abnormal immunological reaction and
the disease has some tendency to undergo malignant
transformation.

Etiology
♦♦ Immunology:
• Due to cell mediated immune response
• Due to autoimmunity
• Immunodeficiency. Fig. 107:  Lichen planus (For color version, see Plate 18)
Section 3: Oral Pathology  633

Q.2. Write short note on lichen planus.


(Feb 1999, 5 Marks) (Apr 2007, 10 Marks)
Or
Write note on oral lichen planus.
 (Mar 2001, 5 Marks) (Mar 2006, 5 Marks)
Or
Describe histopathology of lichen planus.
(Feb 2002, 5 Marks)
Or
Write in brief on lichen planus. (June 2010, 5 Marks)
Or
Write short note on oral lichen planus.
 (Jan 2018, 5 Marks)
Or
Write short note on histopathology of oral lichen Fig. 108: Pemphigus (For color version, see Plate 18)
planus. (July 2016, 5 Marks)
Ans. Refer to Ans 1 of same chapter. Treatment
Q.3. Enumerate vesiculobulbous lesions and describe pem- ♦♦ High dose of steroids
phigus. (Sep 2002, 15 Marks) ♦♦ Immunosuppressive agents
Ans. For enumeration of vesiculobulbous lesion refer to ans 1 ♦♦ Antibiotics to prevent secondary infection.
of same chapter. ♦♦ Fluid and electrolyte balance must be strictly main­
tained.
Pemphigus
Pemphigus is a group of vesiculobulbous lesion of skin and Q.4. Write note on pemphigus vulgaris.
mucous membrane which is characterized by formation (Aug 1998, 5 Marks) (Feb 1999, 5 Marks)
of intraepithelial vesicles or bulla causing separation of Or
epithelium.
Discuss about pemphigus. (Sep 2005, 5 Marks)
Clinical Features Or
♦♦ It occurs during 4th, 5th and 6th decades of life and is more Describe histopathology of pemphigus.
prevalent among the females. (Apr 2008, 5 Marks)
♦♦ Rapidly developing vesicle or bulla on several areas of skin Ans. Refer to Ans 3 of same chapter.
and mucous membrane which contain clear fluid initially
but later on there is formation of pus. Q.5. Write short note on erythema multiforme. 
♦♦ Vesicle ruptures very soon and leaves painful, erythema­  (Feb 2006, 2.5 Marks) (Mar 2007, 2.5 Marks)
tous ulcers that bleed profusely.  (Nov 2008, 5 Marks)
♦♦ Gentle traction or oblique pressure on and affected area Ans. Erythema multiforme is an acute inflammatory
around the lesion causes stripping of the normal skin or dermatological disorder that involves skin, mucous
mucous membrane which is known as “Nikolsky’s Sign”. membrane and sometimes internal organs.
♦♦ Patient may die due to dehydration and septicemia.

Histopathology

♦♦ Formation of vesicle or bulla within the epithelium that


result in supra basilar split.
♦♦ Following the suprabasilar split the basal cell layer remains
attached to lamina propria and appears as row of “Tomb
Stone”.
♦♦ Loss of intracellular bridges and collection of edema fluid
results in acantholysis within spinous cell layer which
causes disruption of prickle cell layer.
♦♦ As result of acantholysis, clumps of large hyperchromatic
epithelial cells lying free within the vesicular fluid, these
desquamated cells are round and smooth in appearance
and are known as Tzanck cells.
♦♦ Small number of PMNs and lymphocytes may be found. Fig. 109:  Erythema multiforme (For color version, see Plate 18)
634   Mastering the BDS IIIrd Year  (Last 25 Years Solved Questions)

Etiology Enumeration of Premalignant Lesion


♦♦ Infection: TB, herpes simplex. Premalignant lesion is defined as "A morphologically altered
♦♦ Drug hypersensitivity: Barbiturates, sulphonamides, tissue in which cancer is more likely to occur than its apparently
salicylates. normal counterparts". For example:
♦♦ Hyperimmune reaction: Due to formation of antigen ♦♦ Leukoplakia
antibody complex against submucosal and derma blood ♦♦ Erythroplakia
vessels. ♦♦ Mucosal changes associated with smoking habits
Types ♦♦ Carcinoma in situ
♦♦ Bowen disease
♦♦ EM minor or erythema multiforme minor. ♦♦ Actinic keratosis, Actinic cheilitis and Actinic elastosis.
It represents the localized eruption of skin with mild or
no mucosal involvement. Enumeration of Premalignant Condition
♦♦ EM major or erythema multiforme major or Steven Johnson Premalignant condition is defined as "A generalized state or
syndrome (SJS)
condition associated with significantly increased risk for cancer
It is more severe mucosal and skin disease and is poten­
development". For example:
tially life-threatening disorder.
♦♦ Syphilis
Clinical Features ♦♦ OSMF
♦♦ Oral lichen planus
♦♦ It occurs between the age of 15 to 40 years and males are ♦♦ Sideropenic dysplasia
more commonly affected. ♦♦ Dyskeratosis congenita
♦♦ Rapidly developing erythematous macules, papules, vesi­ ♦♦ Lupus erythematosus.
cle or bulla appear symmetrically over hand and arm, legs For lichen planus in detail refer to Ans 1 of same chapter.
and feet, face and neck.
♦♦ Classical dermal lesions of erythema multiforme which Q.8. Write short note on oral manifestations of hereditary
often appear on extremities are called “Bull’s eye”. ectodermal dysplasia. (Sep 2009, 3 Marks)
♦♦ Vesicles of mucosal surface are short lived and become Ans. Ectodermal dysplasia is a hereditary disorder
eroded or ulcerated and bleed profusely. characterized by defective formation of ectodermal
structures, i.e. teeth, nail, sweat glands, sebaceous glands
♦♦ Patient also develops tracheobronchial ulceration and
and hair follicles.
pneumonia.
Oral Manifestations
Histopathology
♦♦ Anodontia and oligodontia
♦♦ It usually consists of acanthosis, intercellular or intracel­ ♦♦ Frequent malformation of any teeth in both deciduous and
lular edema and necrosis of epithelium. permanent teeth. Incisors appear to be conical, tapered or
♦♦ Vesicle may form within epithelium or at epithelial con­ pointed while molars look narrow.
nective tissue junction. ♦♦ Salivary glands including intraoral accessory glands are
♦♦ Subepithelial connective tissue shows edema and perivas­ sometimes hypoplastic in this disease which leads to
cular infiltration of lymphocytes and macrophages. xerostomia.
♦♦ Patient have pharyngitis with dysphagia and there is
Treatment hoarseness of voice.
Topical and systemic steroid therapy coupled with antibiotic. ♦♦ Alveolar process does not develop in absence of teeth, there
is a reduction from normal vertical dimension resulting in
Q.6. Define premalignant lesion and condition with exam­ protuberant lips
ples and write in detail lichen planus. ♦♦ Palatal arch is high and at times patient exhibits cleft palate.
(Sep 2006, 15 Marks) Q.9. Define vesicle and bulla. Describe in detail etiopatho-
Ans. For premalignant lesions and conditions refer to Ans 7 genesis, clinical features, histopathology and investiga-
of same chapter. tions of pemphigus vulgaris. (Sep 2011, 8 Marks)
For lichen planus refer to Ans 1 of same chapter. Ans. Vesicle is defined as a superficial blister which is 5 mm
or less in diameter and is usually filled with clear fluid.
Q.7. Define premalignancy. Enumerate the prema­lignant
Bulla is defined as a larger blister which is greater than
lesions and conditions. Describe in detail lichen planus. 5 mm in diameter.
 (Dec 2007, 7 Marks) Pemphigus vulgaris is an autoimmune, intraepithelial
Ans. Premalignancy is an altered state of tissue which often blistering disease affecting the skin and mucous
has but not always has high potential to undergo membrane and is mediated by circulating autoantibodies
malignant transformation. directed against keratinocyte cell surfaces.
Section 3: Oral Pathology  635

Etiopathogenesis Mucocutaneous Lesions of Oral Cavity


Pemphigus antibody binds to keratinocyte cell surface Genodermatosis
molecules desmoglein 1 and desmoglein 3 ♦♦ Darier’s disease
♦♦ White sponge nevus
Patients have circulating and tissue bound antibodies of IgG1 ♦♦ Hereditary benign intraepithelial dyskeratosis
and IgG4 subclass ♦♦ Peutz-Jeghers syndrome
♦♦ Pachyonychia congenita
Pemphigus antibody fixes components of complement to
♦♦ Dyskeratosis congenita
surface of epidermal cells ♦♦ Pseudoxanthoma elasticum
Non-infective Disease
Antibody binding may active complement with release of ♦♦ Vesicular
inflammatory mediators and recruitment of activated T cells
• Bullous pemphigoid
• Benign mucous membrane pemphigoid
Protease Theory
• Pemphigus
According to protease theory deposition of autoantibody within • Erythema multiforme
the epithelium induces the proteolytic activity by activating • Lichen planus
tissue plasminogen. This inturn generate proteolytic enzyme • Epidermolysis bullosa
called plasmin which destroy desmosomes. ♦♦ Non-Vesicular
For clinical features and histopathology refer to Ans 3 of same • Geographic Tongue
chapter. • Lichen planus
♦♦ Collagen Disorders
Investigations • Wegener's granulomatosis
♦♦ Incisional/punch biopsy of the involved area should be • Midline lethal granuloma
done and is histopathologically examined. • Polyarteritis nodosa
♦♦ Immunofluorescent testing: Direct immunofluorescence • Scleroderma
testing is done to demonstrate the presence of immuno­ • Lupus erythematosus
globulins, predominantly IgG but sometimes in combina­ • Vasculitis
tion with C3, IgA and IgM, in the intercellular spaces or ♦♦ Degenerative Disorder
intercellular substance in either oral epithelium or clini­ • OSMF
cally normal epithelium adjacent to lesion. • Amyloidosis
♦♦ Indirect immunofluorescence: This is accomplished basi­ • Solar elastosis
cally by incubating normal animal or human mucosa with ♦♦ Pigmentation
serum from the patient suspected of having the disease and • Anemia
adding the fluorescein-conjugated human antiglobulin. A • Albert syndrome
positive reaction in the tissue indicates presence of circulat­ • Addison’s disease
ing immunoglobulin antibodies. • Racial pigmentation
• Endocrinopathy
Q.10. Write short note on pemphigus vulgaris. For pemphigus in detail refer to Ans 3 and Ans 9 of same
(Jan 2016, 5 Marks) (Aug 2011, 10 Marks) chapter.
Ans. For clinical features and histopathology refer to Ans 3 Q.12. Write short note on Steven-Johnson syndrome.
of same chapter.
(Mar 2011, 3 Marks)
For etiopathogenesis and investigations refer to Ans 9 Ans. Steven-Johnson syndrome is a severe bullous form of
of same chapter. erythema multiforme with widespread involvement
Q.11. Enumerate the mucocutaneous lesions of oral cavity. typically including skin, oral cavity, eyes and genitalia.
Describe in detail pemphigus. (Jan 2012, 10 Marks) • Skin lesions: The cutaneous lesions in this mucocu­
taneous ocular disease are those of erythema mul­
Or tiforme although they are commonly hemorrhagic
Enumerate mucocutaneous lesions. Describe in detail and are often vesicular or bullous.
pemphigus. (Mar 2016, 8 Marks) • Oral mucous membrane lesions: Lesions are
Ans. extremely painful and severe. Mucosal vesicles
636   Mastering the BDS IIIrd Year  (Last 25 Years Solved Questions)

or bullae occur which rupture and leave surfaces


covered with white or yellow exudates. Erosions of
pharynx are common. Lips may exhibit ulceration
with bloody crusting and are painful.
• Eye lesions: It consist of photophobia, conjunctivitis,
corneal ulceration and panophthalmitis. Blindness
may result.
• Genital lesions: There is non-specific urethritis,
balanitis, vaginal ulcers.
Q.13. Write notes on discoid lupus erythematosus.
(Apr 2008, 5 Marks)
Ans. Discoid lupus erythematosus remain confined to the
mucosa and skin.

Etiology
♦♦ Genetic susceptibility
♦♦ Autoimmune: Patient develops antibodies to their own cells. Fig. 110:  Lupus erythematosus (For color version, see Plate 18)
♦♦ Endocrine: Common in pregnant ladies.
Treatment
Clinical Features Corticosteroids along with immunosuppressant drugs is the
♦♦ It is seen during 3rd and 4th decades of life. choice.
♦♦ Female predilection is present with ratio of 5:1. Q.14. Describe in brief Tzanck cells.
♦♦ Lesion appears as circumscribed slightly elevated white
(May/June 2009, 5 Marks)
patch surrounded by erythematous halo.
Ans. Tzanck cell is a term given to a free floating epithelial
♦♦ Skin lesions are slightly elevated. They are purple or red
cell in any intraepithelial vesicle.
macules and are covered by gray or yellow scales.
♦♦ As removal of scale is done it result in carpet track Tzanck cells are seen in mainly two diseases, i.e.
extension. 1. Pemphigus vulgaris
♦♦ There is presence of butterfly shaped distribution over 2. Herpes simplex infection
malar region at bridge of nose.
Tzanck Cells in Pemphigus Vulgaris
Oral Manifestations Pemphigus consists of vesicle or bulla formation in spinous
♦♦ Tongue, buccal mucosa, vermilion border of lip are most layer of epithelium just above basal cell layer which leads to
common involved oral sites. suprabasilar split. Prevesicular edema weaken intercellular
♦♦ Patient complains of burning and tenderness in the junctions and intercellular bridges between epithelial cells
involved area. break and the epithelial cells fell apart which is known as
♦♦ Lesion begin as erythematous area with induration and acantholysis, due to this clumps of epithelial cells found free
white spots. At times pain is felt in ulcerated area with on the vesicular space. These loose cells are characterized by
crusting and bleeding. Scale formation is absent. degenerative changes such as hyperchromatic staining and
♦♦ Fine white striae are seen radiating from the margins. swelling of nuclei, cells become round in shape, these cells are
♦♦ Lips show erythematous area which is surrounded by known as tzanck cells.
keratotic border.
Tzanck Cells in Herpes Simplex Infection
Histopathology Herpes simplex virus mainly affects the epithelial cells. Infected
epithelial cells undergo ballooning degeneration and show
♦♦ It is characterized by hyperkeratosis with keratotic plug­ following features such as acantholysis, nuclear clearing and
ging. enlargement of nucleus. These acantholytic epithelial cells are
♦♦ There is presence of atrophy of rete pegs. known as tzanck cells.
♦♦ Presence of liquefaction degeneration of basal layer of cells.
♦♦ Perivascular infiltration of lymphocytes is present and their Q.15. Write in detail on genodermatosis.
collection at about dermal appendages. (May/June 2009, 10 Marks)
♦♦ Basophilic degeneration of collagen and elastic fibers is Ans. Genodermatoses are genetically determined skin
present. conditions.
♦♦ Hyalinization is also seen. • Some of the genodermatosis are characterized
♦♦ Edema and fibrinoid change is present beneath the particularly by alteration in normal keratinization
epithelium. process and are known as genokeratosis.
Section 3: Oral Pathology  637

Following are the genodermatosis: Hereditary Benign Intraepithelial Dyskeratosis


• Darier’s disease ♦♦ It is also known as Witkop-Von Sallmann syndrome.
• White sponge nevus
• Hereditary benign intraepithelial dyskeratosis Clinical Features
• Peutz-Jeghers syndrome ♦♦ It occurs commonly in children.
• Pachyonychia congenita ♦♦ Most commonly it involves eyes. In oral cavity it is seen in
• Pseudoxanthoma elasticum floor of mouth, buccal mucosa, tongue and palate.
• Porokeratosis ♦♦ It appears as spongy, white macerated area.
• Warty dyskeratoma.
Histopathology
Darier’s Disease
♦♦ Histologically thick epithelium is seen with hydropic
It is also known as keratosis follicularis. degeneration.
♦♦ Dyskeratotic cells are seen, i.e. round, eosinophilic, waxy
Clinical Features
appearing cells are seen.
♦♦ It occurs during childhood or in adults.
♦♦ Initially there is presence of papules over the skin which Peutz-Jeghers Syndrome
is red in color later on they become purple brown or gray It is also known as hereditary intestinal polyposis syndrome.
in color.
♦♦ Changes in nails are also seen such as splintering, longi­ Features
tudinal streaking and fissuring.
♦♦ In it there is pigmentation of face and oral cavity.
♦♦ White papule occurs in oral cavity over hard and soft pal­ ♦♦ Adenocarcinoma can occur in gastric, duodenum and
ate, tongue and gingiva. It is rough on palpation. colon.
Histopathology Histopathology
♦♦ Histopathology reveals hyperkeratosis, papillomatosis, Histologically acanthosis of epithelium is seen with elongation
acanthosis and benign dyskeratosis. of rete ridges.
♦♦ Benign dyskeratosis show typical cells, i.e. corps ronds
and grains Pachyonychia Congenita
♦♦ Corps ronds: These cells are larger than normal squamous
It is also known as Jadassohn-Lewandowsky syndrome.
cells and have round homogeneous basophilic nucleus
with dark eosinophilic cytoplasm and distinct cell mem­ Clinical Features
brane. They are seen in granular layer and superficial
spinous cell layer. ♦♦ It occur in neonates.
♦♦ Grains: Small, elongated, parakeratotic cells present in ♦♦ It is seen on nails, palms, hands and feet.
keratin layer. ♦♦ There is thickening of nail present, thickening increase
toward free border of nail bed and get filled with yellow
White Sponge Nevus keratotic debris which lead to the projection of nail upward
♦♦ It is also known as Cannon’s disease. at free border.
♦♦ Hairs are sparse and corneal dyskeratosis is seen.
Clinical Features ♦♦ Bullae are seen over the feet.
♦♦ It occurs most commonly at birth and its intensity increases Histopathology
till puberty.
♦♦ It is seen on buccal mucosa, gingiva and palate. Histopathology of skin and mucous membrane showing
♦♦ It appears as thick, corrugated, soft textured, white opal­ acanthosis, parakeratosis and hyperkeratosis.
escent line.
Pseudoxanthoma Elasticum
♦♦ White or keratotic area could be removed by rubbing and
normal looking epithelium is seen. It leads to the degeneration of elastic fibers and make them
susceptible for calcium.
Histopathology
Clinical Features
♦♦ Histopathologically epithelium shows hyperparakeratosis
and acanthosis. ♦♦ It occurs at age of 13 years.
♦♦ Cells of spinous layer show intracellular edema. ♦♦ Yellow papules are seen over the skin near neck, mouth, axilla.
♦♦ Parakeratin plugging is seen which is projecting deep in ♦♦ Skin surrounding the mouth give hound dog appearance.
♦♦ Lower lip is affected inside oral cavity.
spinous cell layer.
638   Mastering the BDS IIIrd Year  (Last 25 Years Solved Questions)

Porokeratosis Clinical Features


It is also known as Mibelli’s disease. ♦♦ Male predominance is seen.
♦♦ Patient is characterized by hypotrichosis, hypohydrosis
Clinical Features
and at times anhydrosis.
♦♦ This disease occurs in childhood. ♦♦ Patient has characteristic saddle nose.
♦♦ Male predilection is present. ♦♦ Fine and sparse hair are seen on eyebrows and eyelashes
♦♦ It is seen commonly in face, neck, extremities and genitalia.
♦♦ Periocular skin has fine wrinkles with hyperpigmentation.
♦♦ Lesion appear as crateriform keratotic papule which later
on form elevated plaque. ♦♦ Frontal bossing is commonly present.
♦♦ Plaque from it margins is surrounded by raised border ♦♦ Nails appear dystrophic and brittle.
of epidermis. ♦♦ Skin of the patient remains dry and there is absence or
♦♦ Nails of patient get thick and ridged. partial presence of sweat glands. Due to this patient cannot
♦♦ In oral cavity lesion is seen in upper lip and palate. perspire and his heat regulation is disturbed which lead
to increased temperature of patient.
Histopathology
♦♦ Histologically elevated horny margin of the lesion show Oral Manifestations
hyperkeratosis and acanthosis with deep groove filled with
Refer to Ans 8 of same chapter.
parakeratin and absence of underlying granular layer. This
forms cornoid lamella.
Histopathology
♦♦ Central portion of the lesion show epithelial atrophy.
Histopathology of skin reveals following features:
Warty Dyskeratoma
♦♦ There is reduction in number of sweat glands, hair follicles
It is also known as isolated Darier’s disease. and sebaceous glands.
Clinical Features ♦♦ Epidermis is thin and flattened.
♦♦ Eccrine sweat glands are few or poorly developed.
♦♦ The disease occur in older age.
♦♦ In oral histopathology salivary glands show ectasia of
♦♦ Male predilection is present.
♦♦ Lesions are seen on scalp, neck, face and over the upper ducts and inflammatory changes.
part of chest.
Treatment
♦♦ Lesion present as elevated nodule which is umblicated.
♦♦ Borders of lesion are raised. ♦♦ Condition is genetic so it is non-curable.
♦♦ Color of lesion is not specific and it varies from brown or ♦♦ Prosthetic rehabilitation of patient should be done by
yellow to black or grey. complete dentures, RPDs or fixed dentures.
♦♦ Orally lesion occurs very rarely and if present it can be seen ♦♦ Dental implants can also be considered.
as whitish area with central depression over it.
Q.17. Write short note on CREST syndrome.
Histopathology (Aug 2012, 5 Marks)
Ans. It is considered to be a mild variant of systemic sclerosis.
♦♦ Histopathologically intra-oral lesions show central or­
thokeratin or parakeratin core below which epithelium The term CREST is an acronym for:
show suprabasilar separation which result in cleft like C- Calcinosis Cutis
space having acantholytic and dyskeratotic cells. R- Raynaud’s phenomenon
♦♦ Connective tissue papillae have single layer of basal cells. E- Esophageal dysfunction
♦♦ Underlying connective tissue show non-specific chronic S- Sclerodactyly
inflammatory infiltrate.
T- Telangiectasia.
Q.16. Write in brief on ectodermal dysplasia.
(Dec 2010, 5 Marks) (Jan 2012, 5 Marks) Clinical Features
Ans. Term ectodermal dysplasia is a group of inherited
diseases in which two or more ectodermal structures ♦♦ Female predilection is present.
fails to develop. ♦♦ It most commonly occurs during sixth or seventh decades
• Disease can be autosomal dominant or autosomal of life.
recessive or it can be X-linked too. ♦♦ Signs of the disease develop from months to years.
• Most common ectodermal dysplasia is hereditary ♦♦ Calcinosis cutis: It is characterized by deposition of
hypohidrotic ectodermal dysplasia. calcium beneath the skin in form of nodules which are
Section 3: Oral Pathology  639

0.5 to 2 cm in size and are movable, multiple and are ♦♦ Apply fluorescent conjugated anti-human Ig antibodies.
non-tender. ♦♦ Wash off excess antibody.
♦♦ Raynaud’s phenomenon: It is seen when patient’s hand ♦♦ Section is viewed under ultraviolet microscope.
or feet are exposed to cold. Characteristic clinical sign
is blanching of digits which appear whitish in color
because of vasospasm. After few minutes extremity
become blue in color because of venous stasis. As area
get warm it gives dusky-red hue which indicates of
return of hyperemic blood flow. With all this presence
of throbbing pain is there.
♦♦ Esophageal dysfunction: It is present because of deposition
of abnormal collagen in esophageal submucosa.
♦♦ Sclerodactyly: In this fingers get stiff, skin becomes smooth
and shiny. Flexure of fingers occurs resulting in claw
deformity.
♦♦ Telangiectasia: In this bleeding from superficial dilated
vessels is seen.
Q.18. Classify mucocutaneous lesions and describe in detail
etiology, clinical features and histopathological fea- Fig. 111:  Indirect immunofluorescence
tures of oral lichen planus. (Dec 2012, 8 Marks)
Ans. For classification of mucocutaneous lesions refer to Ans
11 of same chapter.
For etiology, clinical features and histopathological
features of oral lichen planus refer to ans1 of same
chapter.
Q.19. Write short note on direct and indirect immunofluo-
rescence. (Dec 2012, 3 Marks)
Ans. Direct and indirect immunofluorescence are the
techniques which are used to detect immune mediated
or immunobullous diseases.

Direct Immunofluorescence
It is the diagnostic method which is used for detection of
autoantibodies which are bound to patient’s tissue.
Fig. 112:  Direct immunofluorescence
Method
♦♦ Frozen section of patient’s tissue is placed over a slide. Q.20. Describe etiopathogenesis of lichen planus.
♦♦ Tissue is incubated with fluorescein-conjugated goat  (Feb 2013, 8 Marks)
antihuman antibodies.
♦♦ Antibodies bind to the human immunoglobulin site. Ans. Following are the etiopathogenesis of lichen planus:
♦♦ Excess of antibodies are washed with buffered normal • Oral lichen planus is a T cell mediated autoimmune
saline. disease in which cytotoxic CD8 + T cells trigger the
apoptosis of oral epithelial cells. The CD8 + lesional
♦♦ Section is viewed under ultraviolet microscope.
T cells may recognize the antigen associated with
major histocompatibility complex (MHC) class I
Indirect Immunofluorescence on keratinocytes. After antigen recognition and
It is the diagnostic method which is used for detection of activation, CD8 + cytotoxic T cells may trigger
antibodies which are present in blood. keratinocyte apoptosis. Activated CD8 + T Cells
may release cytokines that attract additional
Method lymphocytes.
♦♦ Frozen section of monkey’s esophagus is placed over a • As per the recent studies in psychoneuro-immunology
slide and is incubated with patient’s serum. pshycosomatic stress results in the autoimmunity
♦♦ Excess serum is washed off. reactions and this leads to lichen planus.
640   Mastering the BDS IIIrd Year  (Last 25 Years Solved Questions)

Q.21. What are vesiculobullous lesions. Describe in detail Q.22. Enumerate dermal lesions with oral manifestation.
pemphigus. (Nov 2014, 8 Marks) Describe systemic lupus erythematosus in detail.
Ans. Vesicles are the superficial blisters which are 5 mm or  (June 2015, 10 Marks)
less in diameter and are usually filled with clear fluid. Or
Bulla is a large blister which is greater than 5 mm in diameter.
Enumerate the dermal diseases with oral manifestation.
The lesions which consist of vesicles or bullae are known as Describe the etiology, clinical and histopathological
vesiculobullous lesions. features of systemic lupus erythematosus in detail with
For classification of vesiculobullous lesions refer to Ans 1 of diagram. Give its treatment plan.
same chapter.  (Jan 2017, 10 Marks)
For pemphigus in detail refer to Ans 2 and Ans 9 of same Ans. Enumeration of dermal lesions with oral manifestations
chapter.

Dermal lesion Oral manifestation


Ectodermal dysplasia • Oligodontia or anodontia.
• Presence of midfacial hypoplasia.
Oral lichen planus • Presence of burning sensation in oral mucosa.
• Presence of white and grey velvety thread-like papules in linear, angular and retiform arrangement.
• Presence of wickham’s striae.
Psoriasis • Lesions appear as plaques, silvery, scaly lesions with erythematous base.
• Multiple popular eruptions are seen which can undergo ulceration.
Pityriasis rosea • Lesions appear as erythematous macules with or without central area of grayish desquamation.
Erythema multiforme • Lesions appear as bullae over the erythematous base and break in multiple major ulcers.
• Lesions are larger, irregular, deep and bleed freely.
Pachyonychia congenita • Presence of focal or generalized white opaque thickening of mucosa involving buccal mucosa, tongue and lips.
• Angular cheilosis is commonly present.
• Natal teeth are also present.
Keratosis follicularis • Oral lesions appear as minute white papules which are rough on palpation.
• Lesions occur commonly on gingiva, tongue, hard palate, soft palate, buccal mucosa and pharynx.
Incontinentia pigmenti • Presence of delayed eruption of tooth.
• Tooth crowns are peg or cone shaped.
• Congenitally missing teeth are seen.
Dyskeratosis congenita • Presence of mucosal leukoplakia on the buccal mucosa, tongue.
• Stenosis and constriction occur due to dysphagia, dysuria, phimosis and epiphora.
• Increased incidence of malignant neoplasms.
White sponge nevus • Oral mucosa appears thick and corrugated with soft or spongy texture and a opalescent white hue.
Hereditary benign in- • Lesions are white, spongy and macerated associated with buccal mucosa, with or without folds. Lesions also
traepithelial dyskeratosis appear as soft plaques with pin point elevation.
Acanthosis nigricans • Tongue and lips are frequently involved.
• Presence of hypertrophy of filiform papillae which produces a shaggy, papillomatous surface of dorsum of tongue.
Pemphigus • Presence of ill defined, irregularly shaped gingival, buccal or palatine erosions which are painful and slow to heal.
Pemphigoid • Gingiva is commonly involved.
• Vesicles or bullae develop on attached gingiva. As the vesicle rupture there is presence of raw bleeding surface.
• There may be a formation of ulcer surrounded by zone of erythema.
Epidermolysis bullosa • Bullae are present which are painful and when they rupture there is desquamation of epithelium.
• Lesion heals with the formation of scar which results in obliteration of sulci and restriction of tongue.
Lupus erythematosus • Intraoral lesion consists of central depressed red atrophic area which is surrounded by 2 to 3 mm elevated
keratotic zone which merge in white lines.
Systemic sclerosis • Presence of atrophy and induration of mucosal as well as muscular tissues.
• Tongue become stiff and board like.
• Gingiva become pale and firm.
• Lips become thin, rigid and partially fixed.
• Reduced opening of mouth.
• Fixation of jaw.
Section 3: Oral Pathology  641

Systemic Lupus Erythematosus Histopathology


It is an autoimmune disease which is characterized by ♦♦ In systemic lupus erythematosus, areas of epithelial atro­
autoantibodies, immune complex formation as well as phy are present with absence of keratinization.
dysregulation of immune system which causes damage to any ♦♦ There is presence of liquefactive degeneration of basal
organ of body. cell layer.
♦♦ There is presence of edema of subepithelial connective
Etiology tissue with dilatation of vessels.
Genetic predisposition: Higher incidence of auto-antibodies is ♦♦ In systemic lupus erythematosus degenerative areas and
seen in blood relations of patient. collagen disturbances are more prominent.
Viral infection ♦♦ Inflammatory features are less common.
Hormones, i.e. increase estrogen level in pregnancy. For diagram refer to Ans 13 of same chapter.
Autoimmune: Antibodies are develops towards one’s own body Laboratory Findings
cells.
♦♦ LE cell inclusion phenomenon is used in which there is
Pathogenesis addition of blood serum from a person who is suspected
to the buffy coat of normal blood. If patient is suffering
Antibodies are produced in reaction to exposure of normally from systemic lupus erythematosus, typical LE cells will
unexposed self antigens. Dysregulation of immune system leads appear. The test consists of rosette of neutrophils surround­
to excessive production of antibodies against DNA, ribosomes, ing pale nuclear mass.
other nuclear antigens, platelet, erythrocytes, leucocytes and ♦♦ There is also presence of anemia, leucopenia, thrombo­
various tissue specific antigens which causes tissue damage. cytopenia and elevated ESR and serum gamma globulin
level with positive Coombs test.
Clinical Features ♦♦ Lupus band test is positive, i.e. there is deposition of IgG,
♦♦ It occurs during 30 years of age in females and 40 years IgM or complement component at epidermal – dermal
of age in males. junction or basement membrane zone of skin.
♦♦ Female predilection is seen. Female to male ratio is 2:1
♦♦ Most common sites affected are face, neck, upper arm,
shoulders. Disease is characterized by repeated remission
and exacerbations over these sites.
♦♦ Patient complaints of pain and fever in joints and muscles.
Itching or burning sensation is also present along with the
areas of hyperpigmentation. Symptoms aggravates under
exposure to sunlight.
♦♦ The characteristic sign of the disease is presence of erythe­
matous patches over the face which coalesce to form roughly
symmetrical pattern over the cheeks and across the bridge
of nose, this is known as butterfly distribution.
In the kidney
fibrinoid thickening of glomerular capillaries produces char­ Fig. 113:  LE cell inclusion phenomenon (For color version, see Plate 19)
acteristic wire loops, this leads to renal insufficiency.
Treatment
♦♦ In the heart there is presence of typical endocarditis involv­
ing valves along with fibrinoid degeneration of epicardium ♦♦ Exposure to sunlight should be avoided.
and myocardium. ♦♦ Patient should be kept on systemic corticosteroid therapy.
♦♦ NSAIDs should also be given to combat the symptoms.
Oral Manifestations Q.23 Define genodermatosis and genokeratosis. Classify
vesiculobullous lesions and describe in detail
♦♦ Buccal mucosa, lip and palate are most commonly affected.
etiopathogenesis, clinical features and histopathology
♦♦ Patient complaints of presence of burning sensation in the
mouth. Xerostomia is also seen. of pemphigus. (Dec 2015, 8 Marks)
♦♦ Lesions have very much similarity to lesions of discoid Ans. Genodermatosis: Genodermatoses are genetically
lupus except that they are hyperemic, edematous and determined skin conditions.
extension of lesion is pronounced. Tendency for bleeding Genokeratosis: Some of the genodermatoses are
and petechiae is more as well as superficial ulcerations characterized particularly by alteration in normal
surrounded by red halo are also present. keratinization process and they are referred to as
♦♦ Intraoral lesion consists of central depressed red atrophic genokeratosis.
area which is surrounded by 2 to 3 mm elevated keratotic For classification vesiculobullous lesions refer to Ans 1
zone which merge in white lines. of same chapter.
642   Mastering the BDS IIIrd Year  (Last 25 Years Solved Questions)

For etiopathogenesis of pemphigus refer to Ans 9 of same ♦♦ Tumors—Tumors of the cranial base, parapharyngeal
chapter. space and infratemporal fossa cause 7th nerve palsy.
For clinical features and histopathology of pemphigus refer to ♦♦ Familial—Familial and hereditary occurrence is also re­
Ans 3 of same chapter. ported in case of Bell’s palsy.
♦♦ Facial canal and middle ear neoplasm.
Q.24. Describe pathogenesis, clinical manifestations and ♦♦ Herpes simplex—viral infection.
histopathological features of oral lichen planus.
Clinical Features
 (Jan 2016, 10 Marks)
♦♦ Symptoms
Ans. Pathogenesis
• Sudden following exposure to chill or without any
• Oral lichen planus is a T cell mediated autoimmune apparent precipitating causing maximum paralysis
disease in which cytotoxic CD8 + T cells trigger the in 24 hours.
apoptosis of oral epithelial cells. The CD8 + lesional • Post auricular pain is common.
T cells may recognize the antigen associated with • Spontaneous complaints of loss of sense of taste,
major histocompatibility complex (MHC) class I on hyperacusis (progressive loss of hearing and watering
keratinocytes. After antigen recognition and activation, of the eye).
CD8 + cytotoxic T cells may trigger keratinocyte • Sweating is less on the affected side.
apoptosis. Activated CD8 + T Cells may release cytokines ♦♦ Signs
that attract additional lymphocytes. • Forehead is not wrinkled and frowning is lost.
• As per the recent studies in pschyoneuro-immunology, • Eye of the affected side is not closed and on attempting
pshycosomatic stress results in the autoimmunity closure eyeball turns upwards and outwards.
reactions and this leads to lichen planus. • On showing teeth the lips do not separate on the
For clinical manifestations and histopathological features of affected side.
oral lichen planus refer to Ans 1 of same chapter. • Cheeks puff out with the expiration because of
buccinator paralysis and food collects between the
teeth and paralyzed cheek.
• Base of tongue is lowered.
20. Diseases of nerves • Deafness may result.
Management
and muscles ♦♦ Local heat: Infrared or moist heat over the face or parotid
region or both if there is tenderness of nerve trunk.
Q.1. Write notes on Bell’s palsy. (Sep 2008, 3 Marks) ♦♦ Local treatment of muscles: Patient should massage the facial
muscles with bland oil for twice a day for 5 min.
(Aug 2011, 5 Marks) (Dec 2010, 3 Marks)
♦♦ Protection of eye: It is done with dark glass or eye patch.
Or Mild zinc boric solution is used to wash the eye to prevent
Write short note on Bell’s Palsy. (Mar 2006, 5 Marks) conjunctivitis.
 (Nov 2014, 3 Marks) ♦♦ Corticosteroids: Prednisolone 60 mg/day along with amoxi­
cillin 250 mg 8 hourly help in reducing edema round about
Or the nerve.
Write in brief on Bell’s palsy. (June 2010, 5 Marks) ♦♦ Heavy doses of vitamin B12 1000 µ gm per day IM is given.
♦♦ Galvanism: It is given two weeks after the onset of paralysis
Or
three times in a week.
Write short essay on Bell’s palsy. ♦♦ Surgery: Plastic surgery is preferred.
 (Jan 2012, 5 Marks) Q.2. Write note on trigeminal neuralgia. 
Ans. Bell’s palsy is an acute apparently isolated, lower motor  (Sep 2007, 2.5 Marks) (Jan 2012, 5 Marks)
neuron facial palsy.  (Feb 2013, 8 Marks)
Or
Etiology Write short note on trigeminal neuralgia
 (Dec 2009, 5 Marks) (Apr 2017, 5 Marks)
♦♦ Cold—It usually occurs after exposure to cold. Ans. Trigeminal neuralgia is also called as Tic Douloureux.
♦♦ Trauma—Extraction of teeth or injection of local anesthetic A disorder characterized by the paroxysmal (occurring
may damage to the nerve and subsequent paralysis. repeatedly without warning) attacks of neuralgic pain
♦♦ Surgical procedure—Such as removal of parotid gland with affection of one or more division of trigeminal
tumor in which the facial nerve is sectioned can also cause nerve. The pain involves the first and second divisions
facial paralysis. equally and rarely the first.
Section 3: Oral Pathology  643

Clinical Features Management


♦♦ Pain is unilateral and is confined to one of the three divi­ ♦♦ Elimination of all possible sources of infection.
sions of nerve. Pain is sharp and onset is sudden. The pain ♦♦ Drugs.
is only of a few seconds. • Analgesics: Potent analgesics must be used with
♦♦ During attacks there is flushing of face, i.e. redness of the face. caution because of danger of habituation.
♦♦ Dilatation of pupil is present. • Carbamazepine: 100-200 mg BD a day and increasing
♦♦ There is excessive lacrimation. the dose to 600-800 mg per day.
♦♦ After repeated attacks skin becomes shiny and hair in the • Phenytoin sodium: 0.1 gm TDS when carbamazepine
area become gray. is not tolerated.
♦♦ Sometimes secretion of nasal mucus and saliva may occur • Vitamin B12: 1000 micrograms IM daily for two
in the side of pain. weeks.
♦♦ Injection of alcohol: It is given in affected nerve, or gas­
Etiology serian ganglion. If more than one division is affected inject
10 minims of 90% alcohol after local anesthesia with 2 to
Trigeminal neuralgia is spontaneous and following exposure 3 drops of procaine.
to cold wind, blow on face, or chewing or eating, drinking hot ♦♦ Surgery: Selective or complete preganglionic selection of
or cold fluid and washing the face. trigeminal root.
644   Mastering the BDS IIIrd Year  (Last 25 Years Solved Questions)

VI Forensic Odontology

21. FORENSIC ODONTOLOGY ♦♦ To simplify recording lips are divided into quadrants
similar to dentition, i.e. a horizontal line dividing the up­
per and lower lip and a vertical line dividing right and left
Q.1. Write short note on sex differences in tooth morphology sides. By noting the type of groove in each quadrant the
and jaw anatomy. (Sep 2011, 3 Marks) individual’s lip print pattern may be recorded.
Ans.

Sex Differences in Tooth Morphology


♦♦ Amongst teeth, mandibular canines show greatest
dimensional difference with larger teeth in males than
in females.
♦♦ Mesiodistal width of mandibular canines was significantly
greater in males than in females.
♦♦ Girls are more prone to caries as compared to boys because
tooth erupt at an early age.

Sex Differences in Jaw Anatomy


♦♦ In males jaw bone is large, having broad ascending ramus
while in females it is small, narrow ascending ramus.
♦♦ In males condyles of jaw bone are large while in females
they are small.
♦♦ In males shape of chin is square while in females it is
rounded/pointed.
♦♦ Gonial angle is less obtuse and flares in males while it is
more obtuse and does not flare in females.
♦♦ Body height of jaw bone in males is high while it is low
in females.
Q.2. Write short note on chelioscopy.
 (Feb 2013, 5 Marks)
Or
Write short note on lip prints. (Apr 2017, 4 Marks) Fig. 114:  Lip patterns (For color version, see Plate 19)
Ans. External surface of the lip has many elevations and
depressions forming a characteristic pattern called Q.3. Write short note on Bite marks. (Aug 2012, 5 Marks)
lip prints, examination of which is referred to as  (Apr 2015, 3 Marks) (Mar 2013, 3 Marks)
cheiloscopy.  (Feb 2015, 5 Marks) (June 2014, 5 Marks)
• This is unique for individuals like the fingerprints.
• Lip prints can constitute material evidence left at a Ans. Bite marks are defined as “A mark caused by the teeth
crime scene much like fingerprints. either alone or in combination with other mouth parts”.
• Lip prints provide direct link to the suspect.  —MacDonald.
• Use of lipstick was essential to leave behind color
traces of lip prints. Classification of Bite Marks
MacDonald’s Classification
Classification of Lip Prints
He had given an etiological classification
By Tsuchihashi ♦♦ Tooth pressure marks: Marks on tissue due to ‘direct appli­
♦♦ Type I- Clear cut vertical grooves that run across entire lip cation of pressure by teeth’. Incisal and occlusal surfaces
♦♦ Type I’- Similar to Type I but do not cover entire lip produce these marks.
♦♦ Type II- Branched grooves ♦♦ Tongue pressure marks: If sufficient amount of tissue is taken
♦♦ Type III- Intersected grooves in mouth tongue presses the tissue against rigid areas, i.e.
♦♦ Type IV- Reticular grooves lingual surface of teeth as well as palatal rugae. Marks left
♦♦ Type V- Grooves that cannot be morphologically over skin are called as suckling.
differentiated ♦♦ Tooth scrape marks: They occur due to scraping of teeth over
♦♦ A combination of these grooves may be found on any bitten material. They are caused by anterior teeth
given set of lips.
Section 3: Oral Pathology  645

Importance of Bite Marks of DNA fragments that result from digesting a DNA
♦♦ They provide accurate identification since alignment of sample with a special kind of restriction enzyme
teeth is specific in each individual. called “restriction endonuclease” which sections
♦♦ Bite marks are contaminated by saliva and consist DNA at a specific sequence pattern known as a
of amylin, ptyalin and blood group which help in restriction endonuclease recognition site. RFLP
determination of individual in criminal cases. requires relatively large amounts of DNA. Hence,
cannot be performed with the samples degraded by
Bite Mark Collection environmental factors and also takes longer time to
Collection of Bite Mark from Victim get the results.
• STRs typing: These are described as short stretches
Various methods are: of DNA that are repeated at various locations
♦♦ Case demographics: All the information related to case is throughout the human genome and this technology
selected such as name, age, address, etc. is used to evaluate specific regions (loci) within
♦♦ Visual examination: Visually examine the shape, size, nuclear DNA. Each person has some STRs that
color, contour, texture and other features of bite marks were inherited from father and some from mother,
and document all of them.
but however no person has STRs that are identical
♦♦ Photography: Photographs should be taken as quickly as
to those of either parent. The uniqueness of
possible. They provide permanent record of bite marks.
an individual’s STRs provides the scientific
♦♦ Impressions: Impressions of bitten area is made by vinyl
marker of identity and hence is helpful in forensic
polysiloxane.
♦♦ Saliva swab: Saliva act as a source of DNA. It should be identification and paternity testing. STR can be used
collected carefully and is preserved to match the DNA for identification of bodies in the mass disasters and
with suspect. old skeletal remains.
• Mitochondrial DNA (mtDNA) analysis: Long
Collection of Bite Mark from Suspect intervals between the time of death and examination
♦♦ Photographs of suspect’s teeth should be taken. of tissues complicate the genetic identification
♦♦ Impressions of maxillary and mandibular arch should with nuclear DNA and sometimes only bone and
be taken. teeth may be available for analysis. Teeth provide
♦♦ Saliva swabs from buccal vestibule should be taken. an excellent source for high molecular weight
mtDNA that offer several unique advantages for
Bite Marks Analysis the identification of human remains. mtDNA is
It is done by: a powerful tool for forensic identification as it
♦♦ Metric analysis in conjunction with pattern association. possesses high copy number, maternal inheritance,
♦♦ Direct method: In which suspect’s models are directly placed and high degree of sequence variability.
over bite mark. • Y-chromosome analysis: DNA-polymorphisms on
♦♦ Indirect method: Incisal and occlusal edges of suspect’s teeth the human Y chromosome are valuable tools for
are traced on clear acetate and superimposed on bite mark understanding human evolution, migration and for
photographs. tracing relationships among males. Majority of the
♦♦ Adobe photoshop software. length of the human Y chromosome is inherited as a
♦♦ 3D/CAD supported photogrammetry. single block in linkage from father to male offspring
Conclusion of Bite Marks Analysis as a haploid entity. Hence, Y chromosomal DNA
variation has been mainly used for investigations
♦♦ Definite biter: Presence of reasonable medical certainty which on human evolution and for forensic purposes or
indicate that bite mark is produced by suspect’s dentition. paternity analysis.
♦♦ Probable biter: There is some degree of specificity to sus­ • X-chromosome STR: Chromosome X specific STR
pect’s teeth by sufficient number of matching points. is used in the identification and the genomic stud­
♦♦ Possible biter: Suspect’s teeth could make the bite mark and ies of various ethnic groups in the world. Since the
there are no characteristic matches for certainty. Similarity size of X-chromosome STR alleles is small, generally
of class characteristics is seen. including 100–350 nucleotides, it is relatively easy
♦♦ Not the biter: Bite marks and suspect’s dentition is not to be amplified and detected with high sensitivity.
consistent. X-chromosome STR (X-STR) markers are a power­
Q.4. Write short note on dental DNA methods. ful complimentary system especially in deficiency
(Feb 2014, 3 Marks) paternity testing. Tooth-related jaw bone diseases
can be divided in cysts and odontogenic tumors.
Ans. Following are the dental DNA methods: Reactive bone diseases, fibro-osseous lesions, giant
• Restriction fragment length polymorphism (RFLP) cell lesions, and bone tumors are taken together as
Typing: It is used for analyzing the variable lengths the main second group.
646   Mastering the BDS IIIrd Year  (Last 25 Years Solved Questions)

Q.5. Write short note on forensic odontology. legal cases. These records make the foundation on which]
 (June 2015, 5 Marks) (Jan 2018, 5 Marks) ♦♦ Identification of bite marks: It is involved in identification of
Ans. Forensic odontology is that branch of dentistry which bite marks in criminal cases.
in the interest of justice deals with the proper handling ♦♦ Child abuse: For detection of child abuse or human abuse
and examination of dental evidence and with the too.
proper evaluation and presentation of dental findings. ♦♦ Lip print: For comparison and identification of suspect, lip
Federation Dentaire Internationale (FDI) prints are examined.
♦♦ Legal aspect: In cases with dental traumatology.
Scope of Forensic Odontology
♦♦ Identification: It helps in personal identification, i.e. Parameters to be Compared in Forensic Dentistry
individually or in mass disasters. This is done also through
comparison of antemortem and postmortem dental ♦♦ Teeth
information. ♦♦ Prosthetic appliances, i.e. bridges, partials, crowns, false
♦♦ Age assessment: This is done for assessing the age of person. teeth
♦♦ Record preparation: Correct handling as well as examination ♦♦ Shape, form (morphological) peculiarities
of presentation of dental evidence in civil and criminal ♦♦ Genetic anomalies.
Section 3: Oral Pathology  647

MULTIPLE CHOICE QUESTIONS


As per Dci and Examination Papers 1 Mark Each
of Various Universities

1. Fordyce’s granules actually are collection of: c. Herpetic glossitis


a. Salivary gland tissue d. Pemphigus
b. Sweat gland 8. Ramsay Hunt’s syndrome is associated with:
c. Sebaceous gland a. Herpes simplex
d. Fatty tissue b. Herpes zoster
2. In the lesion of median rhomboid glossitis, there is c. Mumps
absence of: d. Measles
a. Filiform and foliate papillae
9. Most common cells affected in HIV patients are:
b. Fungiform and filiform papillae
a. CD 1 and 2
c. Circumvallate and fungiform papillae
b. CD 4 and 8
d. Circumvallate papillae
c. CD 5 and 12
3. Enzyme considered to be of special importance in d. CD 2 and 4
establishment of S. mutans in dental plaque is:
10. Hansen’s disease is a:
a. Glucosyltransferase
a. Syphilis
b. Fructosyltransferase
b. Candidiasis
c. Dextranase
c. Tuberculosis
d. lnvertase
d. Leprosy
4. “Dead tracts” in ground section of teeth, are manifested
11. In xerostomia, the salivary pH is:
as:
a. Unaffected
a. White zone in transmitted light and black zone in
b. Low
reflected light.
c. High
b. Black zone in transmitted light and white zone in
d. Increased in morning and decreases in day
reflected light.
c. White zone in transmitted light and reflected light 12. Organisms involved in cellulitis is:
d. Black zone in transmitted light and reflected light. a. S. mutants
b. S. pyogenes
5. Which of the following drugs may cause gingival en-
c. Pneumococci
largement:
d. Kiebsieiia
a. Nifedipine
b. Cyclosporine 13. Which is the following are a traid of the sign and
c. Phenytoin sodium symptoms of osteogenesis imperfecta:
d. All of the above a. Blue sclera, sparse hair, anhydrosis
6. Apoptosis of basal cell layer is a feature of: b. Enlarged hand, feet, maxilla, mandible
a. Psoriasis c. Blue sclera, brittle bones, opalescent dentin
b. Pemphigus d. Blue sclera, arachnodactyly, brittle bone.
c. Lichen planus 14. Starry sky appearance is seen in:
d. Erythema multiforme a. Paget’s disease
7. White strawberry tongue is a feature of: b. Cherubism
a. Scarlet fever c. Garry’s osteomyelitis
b. Syphilis d. Burkitt’s lymphoma

Answers: 1. c 2. b 3. c 4. b
5. d 6. c 7. a 8. b
9. b 10. d 11. b 12. b
13. c 14. d
648   Mastering the BDS IIIrd Year  (Last 25 Years Solved Questions)

15. Acanthosis is: 24. The microscopic features of leukoedema consists of:
a. Increase in mitotic division a. Increase in thickness of epithelium
b. Increase in thickness of superficial layer b. Intracellular edema of spinous or malphigian layer
c. Increase in thickness of spinous layer c. Broad rete ridges
d. Disruption of basal lamina d. All of the above
16. Areca nut chewing is aetiological factor in: 25. Greenspan syndrome is associated with:
a. Leukoedema a. Dyskeratosis congenita
b. Oral submucous fibrosis b. Psoriasis
c. Erythema multiforme c. Leukoplakia
d. Oral lichen planus d. Lichen planus
17. Ameloblastoma most frequently occurs in: 26. Carpet track extensions are seen in:
a. Mandibular molar region a. Sarcoidosis
b. Maxillary molar region b. Systemic sclerosis
c. Mandibular premolar region c. Discoid lupus erythematosus
d. Maxillary premolar region d. Erythema multiforme
18. One of them is not a true cyst:
27. Serum alkaline phosphatase levels are seen in:
a. Hemorrhagic cyst
a. Osteoarthritis
b. Median palatine
b. Dentinogenesis imperfecta
c. Globulomaxillary
c. Paget’s disease
d. Nasolabial
d. Rheumatoid arthritis
19. Three stages in progression of acute odontogenic infec-
tion are: 28. Strawberry tongue is associated with:
a. Periapical osteitis, cellulitis, abscess a. Syphilis
b. Abscess, cellulitis, periapical osteitis b. Measles
c. Cellulitis, abscess, periapical osteitis c. Scarlet fever
d. Periapical osteitis, abscess, cellulitis d. Typhoid
20. Cyst with high recurrence rate: 29. Which of the following cyst develops in place of tooth:
a. Keratocyst a. Primordial cyst
b. Primordial b. Dentigerous cyst
c. Lateral Periodontal c. Keratocyst
d. Radicular cyst d. Radicular cyst
21. An inability to absorb adequate amount of vitamin B12 30. Cyst having high recurrence rate is:
from digestive tract may result in: a. Dentigerous cyst
a. Thalassemia b. Primordial cyst
b. Pernicious anemia c. Odontogenic keratocyst
c. Aplastic anemia d. Radicular cyst
d. None of these
31. The most common form of actinomycosis is:
22. Howell-Jolly bodies are seen in: a. Cervicofacial
a. Malaria b. Abdominal
b. Pernicious anemia c. Pulmonary
c. Iron deficiency anemia d. Any of the above
d. Leukemia
32. Virus which may cause Burkitt’s lymphoma is:
23. On stretching, the cheeks lesion disappears in:
a. HSV Type-II
a. Leukoplakia
b. Cytomegalovirus
b. Focal hyperkeratosis
c. Epstein-Barr virus
c. Leukoedema
d. Varicella Zoster virus
d. Typhoid

Answers: 15. c 16. b 17. a 18. a


19. a 20. a 21. b 22. b
23. a 24. b 25. d 26. c
27. c 28. c 29. a 30. c
31. a 32. c
Section 3: Oral Pathology  649

33. Radiographic appearance of osteosarcoma is: c. Cancrum oris


a. Sunray appearance d. Diphtheria
b. Onion-peel appearance
42. Tuberculosis of lymph nodes is known as:
c. Honey comb appearance
a. Lupus vulgaris
d. Ground glass appearance
b. Phthisis
34. Presence of Bence-Jones protein in urine is character- c. Scrofula
istic of: d. Miliary tuberculosis
a. Multiple myeloma
b. Hodgkin’s lymphoma 43. Ghost teeth is seen in which of the following:
c. Burkitt’s lymphoma a. Dens in dente
d. Hemangioma b. Regional odontodysplasia
c. Dentin dysplasia
35. A union of roots of adjacent teeth through the cemen- d. None of the above
tum is referred to as:
a. Concrescence 44. In Ramsay Hunt syndrome the crania nerve involved
b. Fusion is:
c. Gemination a. Trigeminal
d. None of the above b. Facial
36. Teeth that erupt in 30 days of birth are called as: c. Glossopharyngeal
a. Natal teeth d. Oculomotor
b. Neonatal teeth 45. Cherubism is associated with:
c. Primary teeth a. Down’s syndrome
d. Prenatal teeth b. Edward syndrome
37. Talon’s cusp is characteristic of which syndrome: c. Patau syndrome
a. Edward’s syndrome d. Noonan’s syndrome
b. Kinefelter’s syndrome 46. Which of the following is not a benign tumor of salivary
c. Rubinstein-Taybi syndrome
glands:
d. Down’s syndrome
a. Pleomorphic adenoma
38. Desquamative gingivitis may be seen in all of the fol- b. Myoepithelioma
lowing except: c. Sialadenosis
a. Pemphigus vulgaris d. Cystadenoma
b. Recurrent aphthae
c. Erythema multiforme 47. All of the following are true about pulp polyp except:
d. Cicatricial pemphigoid a. Excessive, exuberant proliferation of chronically
inflamed dental pulp tissues
39. Organism in etiology of ANUG: b. Generally occur in children and young adults
a. Cocci and bacilli
c. Lesion is tender
b. Bacilli and bacteriophage
d. Teeth most commonly involved are deciduous and
c. Spirochete and bacilli
permanent first molar
d. Bacteriophage and cocci
40. Areca nut chewing is an etiological factor in: 48. Squamous papilloma is associated with:
a. Leukoedema a. EBV
b. Oral submucous fibrosis b. HIV
c. Erythema multiforme c. HPV
d. Lichen planus d. HSV
41. Which of the following disease is characterized 49. Basal cell nevus syndrome is associated with:
by fever, headache, sore throat and formation of a. Odontogenic keratocyst
pseudomembrane in pharynx: b. Dentigerous cyst
a. Scarlet fever c. Radicular cyst
b. Tuberculosis d. Nasopalatine cyst

Answers: 33. a 34. a 35. a. 36. b


37. c 38. b 39. c 40. b
41. d 42. c 43. b 44. b
45. d 46. c 47. c 48. c
49. a
650   Mastering the BDS IIIrd Year  (Last 25 Years Solved Questions)

50. Carcinoma usually metastatize by which route: 59. Gingiva is affected mainly by deficiency of vitamin:
a. Hematogenous spread a. A
b. Local spread b. B complex
c. Lymphatic spread c. C
d. Mechanical spread d. D
51. Neurofibroma is associated with: 60. Silver tattoo is deposition of amalgam in:
a. MEN syndrome a. Bone
b. Bechet syndrome b. Dewtin
c. Von-Recklinghausen’s disease c. Enamel
d. Cannon’s disease d. Mucosa
52. Malignant melanoma is a neoplasm of: 61. Brown tumor is seen in:
a. Epidermal melanocytes a. Hyperthyroidism
b. Spinous cells b. Hyperparathyroidism
c. Basal cells c. Diabetes mellitus
d. Keratin d. Acromegaly
53. Most common site for compound odontoma is: 62. Rodent ulcers are seen in:
a. Anterior maxilla a. Basal cell carcinoma
b. Posterior maxilla b. Gangrene
c. Anterior mandible c. Leprosy
d. Posterior mandible d. Syphilis
54. Listed below are all methods to represent bite mark 63. Most common supernumerary tooth is:
except: a. Mesiodens
a. Xeroradiography b. Paramolar
b. Transillumination c. Peridens
c. Swab test d. Lateral incisor
d. Ultraviolet photography 64. Most common microbial disease of oral cavity is:
55. Civatte bodies are also called as: a. Candidiasis
a. Colloid bodies b. Dental caries
b. Cytoid bodies c. Carcinoma
c. Hyaline bodies d. Aphthous ulcer
d. All of above 65. Chronic hyperplastic pulpitis is also called as:
56. Which of the following is not type of oral lichen planus: a. Pulp polyp
a. Atrophic b. Acute pulpitis
b. Hypertrophic c. Chronic pulpitis
c. Verrucous d. Periapical abscess
d. Erosion 66. Screening test is done to determine saliva from secretor
57. Tzanck cells are characteristic of: is done with:
a. Pemphigus a. Antigen-A
b. Pemphigoid b. Antigen-B
c. Lichen planus c. Antigen-H
d. SLE d. Radioimmunoassay
58. A fluid-filled elevated lesion of skin in called as: 67. All are types of acquired Nevi except:
a. Papule a. Junctional nevi
b. Macule b. Compound nevi
c. Vesicle c. Blue nevi
d. Nodule d. Garment nevi

Answers: 50. c 51. c. 52. a 53. a


54. d 55. d 56. c 57. a
58. c 59. c 60. d 61. b
62. a 63. a 64. b 65. a
66. d 67. d
Section 3: Oral Pathology  651

68. Pregnancy tumor is histologically identical with: 77. A flat circumscribed discoloration of skin is called as:
a. Aphthous ulcer a. Macule
b. Pyogenic granuloma b. Ulcer
c. Ameloblastoma c. Papule
d. Traumatic ulcer d. Nodule
69. Most common site for compound odontoma is: 78. Bull’s eye lesion of hand is seen in:
a. Anterior maxilla a. Pemphigus
b. Posterior maxilla b. Erythema multiforme
c. Anterior mandible c. Pemphigoid
d. Posterior mandible d. Lichen planus
70. Which is not the odontogenic cyst: 79. Acanthosis is a feature of:
a. Lateral periodontal cyst a. Pemphigus
b. Gorlin cyst b. Erythroplakia
c. Traumatic bone cyst
c. Leukoplakia
d. Dentigerous cyst
d. OSMF
71. ‘Cafe aulait’ spots are present in:
80. Blue sclera is seen:
a. Monostotic fibrous dysplasia
a. Marfan’s syndrome
b. Polyostotic fibrous dysplasia
b. Cherubism
c. Paget’s disease
c. Osteogenesis imperfecta
d. Cherubism
d. Vitamin C deficiency
72. “Rathke’s Pouch tumor” is another name for:
81. Turner’s tooth is associated with:
a. Adamantinoma of lung bone
b. Pituitary ameloblastoma a. Enamel hypoplasia
b. Syphilis
c. Malignant ameloblastoma
d. Ameloblastic carcinoma c. Multiple caries
d. Cyst
73. Indentation of human premolar bite mark is:
82. Rushton bodies are seen in:
a. Oval in shape
a. Lichen planus
b. Spherical in shape
b. SLE
c. Dual triangular or deep triangular in shape
d. Square in shape c. Radicular cyst
d. Dental caries
74. Type of ameloblastoma is most aggressive and high
83. ‘Paul-Bunnell’ test is positive in:
recurrence rate:
a. Infectious mononucleosis
a. Follicular
b. Hodgkin’s syndrome
b. Plexiform
c. Leukemia
c. Granular
d. Glandular fever
d. Unicystic
84. Fixative commonly used in histopathology techniques
75. Most common odontogenic cyst is:
is:
a. Dentigerous cyst
a. 10% formalin
b. Radicular cyst
b. Dry heat
c. Keratocyst
c. 70% alcohol
d. CEOC
d. Saliva
76. Greenspan syndrome includes all except:
85. Caféau lait spots are found in all except:
a. Lichen planus
a. Peutz-Jeghers syndrome
b. Diabetes mellitus
c. Hypertension b. Cherubism
c. Von-Recklinghausen’s disease
d. Pemphigus
d. Fibrous dysplasia

Answers: 68. b 69. a 70. c 71. b


72. b 73. c 74. a 75. b
76. d 77. a 78. b 79. a
80. c 81. a 82. c 83. a
84. a 85. d
652   Mastering the BDS IIIrd Year  (Last 25 Years Solved Questions)

86. Which condition will produce a negative pulp vitality 94. Which of the following method is used to extract dental
test: DNA:
a. Acute reversible pulpitis a. Cryogenic method
b. Apical periodontitis b. PCR
c. Chronic irreversible pulpitis c. Cheiloscopy
d. Chronic hyperplastic pulpitis d. DNA finger printing
87. Which is not a feature of Greenspan’s syndrome: 95. Which of the following deficiencies are associated
a. Oral lichen planus with the disorders of hyperplasia of salivary glands
b. Diabetes mellitus and keratinization of salivary glands:
c. Leukoplakia a. Vitamin A
d. Hypertension b. Vitamin B
88. Which of the following cannot be used to investigate c. Vitamin C
bite marks: d. Vitamin K
a. Photography 96. Talon’s cusp is characteristics of which syndrome:
b. Saliva swap a. Edward syndrome
c. Impression b. Klinefelter syndrome
d. Bite detector c. Rubenstein Taybi syndrome
89. Congenital absence of salivary gland duct is also d. Down’s syndrome
known as: 97. Koilocytes are:
a. Aberrancy a. Cytomegalovirus altered epithelial cells
b. Aplasia
b. Epstein-Barr virus altered cells
c. Atresia
c. Human papilloma virus altered cells
d. Xerostomia
d. RNA virus altered cells
90. ……….. is not a feature of Sjogren’s syndrome:
98. Denture sore mouth is caused by:
a. Rheumatic arthritis
a. Actinomycosis
b. Enlargement of salivary gland
b. Candida albicans
c. Xerostomia
c. Blastomycosis
d. Keratoconjunctivitis
d. None of the above
91. Ghost cells are found in:
a. Calcifying epithelial odontogenic cyst 99. Hemophilia B is due to deficiency of:
b. Odontogenic cyst a. Factor VII
c. Residual cyst b. Factor IX
d. Paradental cyst c. Platelet
d. Vitamin C
92. Codman’s triangle is a characteristic feature of:
a. Chondrosarcoma 100. Widely accepted theory of dental caries is:
b. Ameloblastoma a. Proteolytic theory
c. Osteosarcoma b. Proteolytic chelation theory
d. Ewing’s sarcoma c. Acidogenic theory
d. Autoimmune theory
93. Reed-Sternberg cells are characteristic features of:
a. Non-Hodgkin’s Lymphoma 101. Number of zones seen in dental caries of dentin are:
b. Burkitt’s lymphoma a. One
c. Hodgkin’s lymphoma b. Two
d. Adenolymphoma c. Four
d. Five

Answers: 86. b 87. c 88. d 89. c


90. b 91. a 92. c 93. c
94. b 95. a 96. c 97. c
98. b 99. b 100. c 101. d
Section 3: Oral Pathology  653

102. Lateral spread of caries is facilitated by: b. Onion-Peel appearance


a. Enamel spindles c. Driven-Snow appearance
b. Dentinoenamel junction d. Cherry-Blossom appearance
c. Enamel lamellae 111. Rodent ulcer refers to:
d. Striae of retzius a. Squamous cell carcinoma
103. All of the following may cause gingival enlargement b. Verrucous carcinoma
except: c. Basal cell carcinoma
a. Vitamin C deficiency d. Both a and b
b. Fibromatosis gingivae 112. Abtropfung effect is seen in:
c. Monocytic leukemia a. Junctional nevus
d. Desquamative gingivitis b. Aphthous ulcer
104. Odontoma is: c. Erythema multiforme
a. Hamartoma d. Pemphigus
b. Teratoma 113. Civatte bodies are seen in:
c. Choristoma a. Bowen’s disease
d. None of the above b. Leukoplakia
105. Self-healing carcinoma refers to: c. Lichen planus
a. Verrucous xanthoma d. OSMF
b. Keratoacanthoma 114. Racquet cell, strap cells and ribbon cells are typically
c. Nevus seen in:
d. Fibroma a. Neuroblastoma
106. A patient has filmy white opalescence bilaterally on b. Rhabdomyosarcoma
buccal mucosa, the lesion fades on stretching. The most c. Leiomyosarcoma
likely diagnosis is: d. Ewing’s sarcoma
a. White sponge nevus 115. Van der Woude syndrome shows all except:
b. Leukoplakia a. Bilateral lip pits
c. Lichen planus b. Cleft lip
d. Leukoedema c. Cleft palate
107. “Eye raised to heaven” look is clinical feature of: d. Microdontia
a. Marfan’s syndrome 116. Fordyce’s granules are ectopic:
b. Fibrous syndrome a. Sebaceous glands
c. Albright’s syndrome b. Sweat glands
d. Cherubism c. Lacrimal glands
108. The most common gland involved in sialolithiasis is: d. Salivary glands
a. Parotid gland 117. Hansen’s disease is the another name of:
b. Submandibular gland a. Tuberculosis
c. Sublingual gland b. Leprosy
d. Both (a) and (b) c. Actinomycosis
109. Multiple OKC are found in the following syndrome: d. Diphtheria
a. Gorlin-Goltz syndrome 118. In primary stage of syphilis the lesion is called as:
b. Ectodermal dysplasia a. Mucocutaneous patch
c. Noonan’s syndrome b. Gumma
d. Down’s syndrome c. Chancre
110. Radiographic finding in pindborg tumor is: d. Crust
a. Sunburst appearance

Answers: 102. b 103. d 104. a 105. b


106. d 107. d 108. b 109. a
110. c 111. c 112. a 113. c
114. b 115. d 116. a 117. b
118. c
654   Mastering the BDS IIIrd Year  (Last 25 Years Solved Questions)

119. Herpes virus infects all tissues except: a. Radicular cyst


a. Skin b. Gorlin’s cyst
b. Eyes c. Paradental cyst
c. CNS d. Nasopalatine cyst
d. Salivary glands 128. Numbness in the palate, or area of looseness in palate
120. Civatte bodies are seen in: are clinical features of:
a. Radicular cyst a. Squamous cell carcinoma
b. Lichen planus b. Necrotizing sialometaplasia
c. Pemphigus c. Salivary duct carcinoma
d. Pemphigoid d. None of the above
121. Iris lesions are seen in: 129. Duct like spaces lined by single cells and rosette pat-
a. Psoriasis tern seen in:
b. Erythema multiforme a. AOT
c. Pemphigus b. Ameloblastoma
d. Lupus erythematosus c. Odontogenic fibroma
122. Abrasion is wearing away of tooth due to: d. Odontogenic myxoma
a. Mechanical forces 130. “Café au lait” spots are seen in:
b. Chemical a. Ganglioneuroma
c. Mastication b. Fibroma
d. None of the above c. Rhabdomyoma
123. Anitschkow cell is seen in: d. Neurofibroma
a. Herpes infection 131. Wickham’s striae are seen in:
b. Recurrent aphthous ulcers a. Leukoplakia
c. Tuberculosis b. Erythroplakia
d. None of the above c. Lichen planus
124. Costen’s syndrome has all except: d. Both a and b
a. Impairment of hearing 132. Most common malignancy in AIDS:
b. Tinnitus a. Osteosarcoma
c. Otalgia b. Fibrosarcoma
d. Blurring of vision c. Ewing’s sarcoma
125. Parakeratin plugging is seen in: d. Kaposi’s sarcoma
a. Verrucous carcinoma 133. Which of the following is an example of non-
b. Liposarcoma odontogenic cyst:
c. Osteosarcoma a. Eruption cyst
d. Fibrosarcoma b. Nasolabial cyst
126. Jig saw puzzle or Mosaic pattern of bone is seen in: c. Primordial cyst
a. Cherubism d. Glandular odontogenic cyst
b. Paget’s disease 134. Reed-Sternberg cells are characteristically seen in:
c. Fibrous dysplasia a. Thalassemia
d. Achondroplasia b. Glandular fever
127. Arcading pattern of rete pegs and rushton bodies are c. Hansen’s disease
seen in: d. Hodgkin’s disease

Answers: 119. d 120. b 121. b 122. a


123. b 124. b 125. a 126. b
127. a 128. b 129. a 130. d
131. c 132. d 133. b 134. d
Section 3: Oral Pathology  655

135. Cyst associated with root apex of a non vital tooth is 142. Most common tumor of parotid gland is:
most likely to be: a. Pleomorphic adenoma
a. Odontogenic keratocyst b. Adenoid cystic carcinoma
b. Dentigerous cyst c. Mucoepidermoid carcinoma
c. Radicular cyst d. Acinic cell carcinoma
d. Glandular odontogenic cyst 143. Punched out ulcers on interdental papillae are seen in:
136 Microscopic appearance of “swiss cheese” pattern is a. ANUG
seen in: b. Desquamative gingivitis
a. Pleomorphic adenoma c. Pericoronitis
b. Adenoid cystic carcinoma d. Crohn’s disease
c. Mucoepidermoid carcinoma 144. Suprabasilar split in the epithelium is a feature of:
d. Acinic cell carcinoma a. Lichen planus
137. Greatest demineralization is seen in the following zone b. Pemphigus
of enamel caries: c. Erythema multiforme
a. Translucent zone d. Pemphigoid
b. Dark zone 145. Café – au – lait spots can be seen in:
c. Body of the lesion a. McCune Albright Syndrome
d. Surface zone b. Neurilemmoma
138. Mucous extravasation phenomenon most commonly c. Neurofibromatosis
occurs on: d. Both a and c
a. Upper lip 146. Multiple impacted supernumerary teeth are seen in
b. Lower lip the following:
c. Tongue a. Gardner syndrome
d. Buccal mucosa b. Cowden’s syndrome
139. Bleeding gums can be seen in patient with deficiency c. Klinefelter’s syndrome
of: d. Peutz – Jeghers syndrome
a. Vitamin A 147. Commonest opportunistic fungal disease of the oral
b. Vitamin E cavity is:
c. Vitamin D a. Histoplasmosis
d. Vitamin C b. Mucormycosis
140. Clinical finding of “pinkish discoloration” of tooth c. Candidiasis
indicates: d. Blastomycosis
a. Internal resorption 148. Nikolsky’s sign is positive in:
b. External resorption a. Lichen planus
c. Hypercementosis b. Pemphigus vulgaris
d. Tooth ankylosis c. Systemic sclerosis
141. “Parakeratin plugging” is the hallmark feature for: d. Lupus erythematosus
a. Squamous cell carcinoma 149. Which of the following is not the feature of epithelial
b. Basal cell carcinoma dysplasia:
c. Malignant melanoma a. Loss of basal cell polarity
d. Verrucous carcinoma b. Nuclear and cellular pleomorphism
c. Individual cell keratinization
d. Koilocytic change

Answers: 135. c 136. b 137. c 138. b


139. d 140. a 141. d 142. a
143. a 144. b 145. d 146. a
147. c 148. b 149. d
656   Mastering the BDS IIIrd Year  (Last 25 Years Solved Questions)

150. Self healing carcinoma is another name for: b. Dilaceration


a. Squamous papilloma c. Macrodontia
b. Keratoacanthoma d. Anodontia
c. Verruca vulgaris
153. Malignant tumor of smooth muscle origin is:
d. Squamous cell carcinoma
a. Rhabdomyoma
151. “Driven snow” appearance on radiograph is seen in: b. Rhabdomyosarcoma
a. Adenomatoid odontogenic tumor c. Leiomyoma
b. Ameloblastoma d. Leiomyosarcoma
c. Calcifying epithelial odontogenic tumor
154. Ghost cells are seen:
d. Squamous odontogenic tumor
a. Odontogenic keratocyst
152. Which of the following condition can cause difficulty b. Dentigerous cyst
in tooth extraction: c. Radicular cyst
a. Microdontia d. Calcifying odontogenic cyst

Answers: 150. b 151. c 152. b 153. d


154. d
Section 3: Oral Pathology  657

Fill in the blanks


1 Mark Each
1. Complication of healing of extraction socket is called 21. The chemical disintegration of enamel is referred to
as …………… as ……………
Ans. Dry socket Ans. Erosion
2. ANUG is caused by …………… microorganism. 22. Koplik’s spots are an early intraoral manifestation of
Ans. P. intermedia, Borrelia and Treponema ……………
3. The other name of pindborg tumor is …………… Ans. Measles
Ans. Calcifying epithelial odontogenic tumor 23. Tumors of minor salivary glands are more frequently
seen in ……………
4. Papilloma is caused by …………… Virus.
Ans. HPV Ans. Palate

5. “Pink tooth” is caused due to …………… 24. A fluid filled elevated lesion of skin is called as
……………
Ans. Internal resorption
Ans. Bulla
6. Dens in dente is called ……………
25. The most common sequel of pulpitis is ……………
Ans. Dens invaginatus
Ans. Periapical granuloma
7. Syphilis is caused by ……………
26. Anitschkow cells are found in ……………
Ans. Treponema pallidium
Ans. Aphthous ulcers
8. Pseudocyst of jaws are ……………
Ans. Solitary bone cyst and aneurysmal bone cyst 27. Verocay bodies are the histologic feature of ……………
Ans. Neurilemmoma
9. Kaposi’s sarcoma is tumor of ……………
Ans. Human herpes virus-8 28. The term submerged tooth is used for ……………
Ans. Deciduous molar
10. Warthin’s tumor is also called as ……………
Ans. Papillary cystadenoma lymphomatosum 29. Reed-Sternberg cells are seen in ……………
Ans. Hodgkin’s lymphoma
11. Bence-Jones protein is seen in ……………
Ans. Plasmacytoma 30. Examination of lip prints is known as ……………
Ans. Cheiloscopy
12. Café au lait spots are seen in ……………
Ans. Neurofibroma 31. Cotton wool appearance radiographically is seen in
……………
13. “Liesgang’s rings” are characteristic are features of Ans. Paget’s disease
……………
Ans. Calcifying epithelial odontogenic tumor 32. Sunray appearance radiographically is seen in
……………
14. Phlegmon is also known as ……………
Ans. Osteosarcoma
Ans. Cellulitis
33. Wickham striae are characteristic features of ……………
15. Hyaline or rushton bodies is seen in ……………
Ans. Lichen planus
Ans. Radicular cyst
34. Reed-Sternberg cells are seen in ……………
16. Mulberry molars are seen in ……………
Ans. Hodgkin’s lymphoma
Ans. Syphilis
35. Codman’s triangle is seen in ……………
17. Shell teeth is associated with ……………
Ans. Osteosarcoma
Ans. Dentinogenesis imperfecta- Type III
36. Chicken wire pattern is seen in ……………
18. Virus implicated in squamous cell papilloma is
Ans. Pemphigus vulgaris
……………
Ans. Human papilloma virus (HPV) 6 and 11 37. Nikolsky’s sign is a characteristic feature of ……………
Ans. Pemphigus vulgaris
19. Self healing carcinoma is also known as ……………
Ans. Keratoacanthoma 38. Hutchinson’s triad is seen in ……………
Ans. Syphilis
20. Abtropfung or dropping off effect is seen in ……………
Ans. Junctional nevus and compound nevus
658   Mastering the BDS IIIrd Year  (Last 25 Years Solved Questions)

39. Ghost teeth appearance radiographically is seen in 57. Define neuralgia ……………
…………… Ans. Neuralgia is defined as the pain along the pathway of
Ans. Regional odontodysplasia nerve
40. Parakeratin plugging is seen in …………… 58. Other name of botryoid cyst ……………
Ans. Verrucous carcinoma Ans. Lateral periodontal cyst
41. Swiss cheese pattern is seen in …………… 59. Subepithelial split is seen in ……………
Ans. Cylindroma Ans. Bullous pemphigoid
42. Screw driven-shaped incisors are seen in …………… 60. HPV altered epithelial cells with perinuclear clear
Ans. Syphilis spaces and nuclear pyknosis found in squamous papil-
loma are ……………
43. Reed-Sternberg cells are feature of ……………
Ans. Koilocytes
Ans. Hodgkin’s lymphoma
61. Scattered macrophages with an abundant clear cyto-
44. Grinspan syndrome is associated with ……………
plasm often containing phagocytic cellular debris, a
mucocutaeous lesion
characteristic histopathological pictures seen in African
Ans. Lichen planus Jaw Lymphoma is ……………
45. Soap bubble appearance in X-ray is seen in …………… Ans. Ans. Starry Sky appearance
Ans. Ameloblastoma, aneurysmal bone cyst, central 62. Characterisitic malignant cells of Hodgkin’s disease
hemangioma are large cells known as ……………
46. Ghost’s teeth are seen in …………… Ans. Reed-Sternberg cells
Ans. Regional odontodysplasia 63. Cells with elongated nuclei containing a linear bar
47. Chronic hyperplastic pulpitis is also called as of chromatin with radiating processes of chromatin
…………… with extension towards nuclear membrane found in
Ans. Pulp polyp aphthous stomatitis are called as ……………
Ans. Anitschkow cells
48. Most-accepted theory in etiopathogenesis of dental
caries is …………… 64. Softened bone at the base of skull seen in Paget’s dis-
Ans. Acidogenic theory ease is called as ……………
Ans. Platybasia
49. Nikolsky’s sign is a feature of ……………
Ans. Pemphigus vulgaris 65. Syndrome consisting of a triad of persistant or recur-
ring lip or facial swelling, intermittent seventh (facial)
50. Keratin pearls are seen in ……………
nerve paralysis (Bell’s palsy) and fissured tongue is
Ans. Squamous cell carcinoma ……………
51. Syndrome associated with cherubism …………… Ans. Melkersson-Rosenthal syndrome
Ans. Noonan syndrome 66. Herpes simplex infection occurring in fingers due to
51. Monroe’s abscess is seen in …………… autoinoculation is called as ……………
Ans. Psoriasis Ans. Herpetic Whitlow
52. Shell teeth is also called as …………… 67. Degenerative cells showing swelling of the nuclei and
Ans. Brandywine type hyperchromatic staining seen in pemphigus vulgaris
is ……………
53. Virus infected cells are called as ……………
Ans. Tzanck cells
Ans. Koilocytes
68. Pronounced hyperextensibility in patients with Ehler-
54. Candida grows in …………… media. Danlos syndrome give them appearance called as
Ans. Sabouraud's broth ……………
55. Two lesion where we find keratin pearls …………… Ans. Rubber man
Ans. Squamous cell carcinoma and pleomorphic adenoma 69. Hunter’s glossitis or Moeller’s glossitis is characteristic
56. Hair pin cells are seen in …………… feature of ……………
Ans. Radicular cyst Ans. Deficiency of vitamin B12
Section 3: Oral Pathology  659

viva-voce questions for


practical examination
1. Name the condition in which teeth get united by the 17. What does ankylosed deciduous tooth known as?
cementum. Ans. Submerged teeth
Ans. Concrescence 18. Name the condition in which there is presence of heart-
2. Name the condition in which in attempt for the divi- shaped radiolucency and is bilateral, it is also lined by
sion of a single tooth germ by an invagination which pseudostratified ciliated epithelium.
causes incomplete formation of two teeth. Ans. Nasopalatine duct cyst
Ans. Gemination 19. What is the another name of klestadt cyst?
3. Name the condition in which there is sharp bent or Ans. Nasolabial cyst
curvature present in the root. 20. What is parulis?
Ans. Dilaceration Ans. It is the inflammatory enlargement which is seen at the
4. What are the other names of geographic tongue? terminus of fistula or the sinus tract
Ans. Wandering rash or erythema migrans 21. In which condition does Epstein pearls and Bohn’s
5. Name the papillae affected in geographic tongue. nodules are seen?
Ans. Filiform papillae Ans. Palatal cyst of neonate

6. Name the condition characterized by heterotrophic 22. Name the cellular layer which proliferates in papil-
collection of sebaceous glands at various sites in oral loma.
cavity. Ans. Spinous cell layer
Ans. Fordyce’s granules 23. Which is the most common benign soft tissue neoplasm
of oral cavity?
7. In hairy tongue which papillae of tongue get hyper-
trophied? Ans. Fibroma
Ans. Filiform papillae 24. Name the carcinoma which show no tendency for the
metastasis.
8. What is another name of dens evaginatus?
Ans. Basal cell carcinoma
Ans. Leong's premolar
25. Name the route by which oral carcinoma metastatize.
9. Which is the most common supernumerary teeth?
Ans. Lymphatic route
Ans. Mesiodens
26. Name the lymph nodes which commonly involved in
10. Which is second most common supernumerary tooth?
metastasis of oral cancer.
Ans. Distomolar Ans. Submaxillary and cervical lymph nodes
11. By which structure predeciduous teeth are formed of? 27. Name the disease which is known as self-healing
Ans. Hornified epithelial structures carcinoma.
12. During which stage of tooth development, injury occur Ans. Keratoacanthoma
which result in enamel hypoplasia. 28. Name the nevus which is clinically benign but histo-
Ans. Formative stage logically malignant.
13. In which condition screw driven-shaped incisors and Ans. Spindle cell nevus
moon molars are seen? 29. Which nevus show abtropfung or dropping off effect.
Ans. Syphilis Ans. Junctional nevus
14. Which condition consists of ghost teeth? 30. Which is the site of highest risk in leukoplakia?
Ans. Regional odontodysplasia Ans. Floor of the mouth
15. Name the condition in which there is single tooth 31. Which is the most common precancerous lesion and
hypoplasia due to trauma or infection. malignant precancerous lesion?
Ans. Turner’s hypoplasia Ans. Leukoplakia and erythroplakia
16. Name the teeth which erupt prematurely in first thirty 32. Which is the most common malignancy in males and
days of life. females?
Ans. Neonatal teeth Ans. In males it is lung cancer and in females it is breast cancer.
660   Mastering the BDS IIIrd Year  (Last 25 Years Solved Questions)

33. Which is the most common malignancy in males and 52. Name the most common malignant salivary gland
females in India? neoplasm of children.
Ans. Oral cancer in males and breast cancer in females Ans. Mucoepidermoid carcinoma
34. What is the hallmark of verrucous carcinoma? 53. Which is the most common etiology for necrotizing
Ans. Parakeratin plugging sialometaplasia?
Ans. Ischemia
35. What is the another name of lane tumor?
Ans. Spindle cell carcinoma 54. Name the cyst consisting of rushton bodies.
36. In which lesion epithelial melanocytes are distributed Ans. Radicular cyst
in pagetoid manner? 55. Which cyst consists of satellite cysts or daughter cysts?
Ans. Malignant melanoma Ans. Odontogenic keratocyst
37. Name the most common benign soft tissue tumor of 56. What do you mean by satellite or daughter cyst?
oral cavity. Ans. They represents the end of folds of lining epithelium of
Ans. Fibroma main cytic cavity which are cut in cross section
38. In which disease foam cells are evident? 57. Which tumor is known as pindborg tumor?
Ans. Verruciform xanthoma Ans. Calcifying epithelial odontogenic tumor
39. Which is the most common site for occurrence of lym- 58. In which disease gastric acid decalcification of teeth is
phangioma? present?
Ans. Tongue Ans. Anorexia nervosa
40. Which disease is known as Codman’s tumor? 59. What is the another name of pink tooth of Mummery?
Ans. Benign chondroblastoma Ans. Internal resorption
41. In which disease codman’s triangle is seen? 60. Which disease is known as Hansen’s disease.
Ans. Osteosarcoma Ans. Leprosy
42. Which tumor show starry sky effect on cytoplasm? 61. Where does pyogenic granuloma is seen commonly in
Ans. Burkitt’s lymphoma oral cavity.
43. In which disease Reed-Sternberg cell is seen? Ans. Gingiva
Ans. Hodgkin’s lymphoma 62. In which disease ballooning degeneration and
Lipschutz’s bodies are seen?
44. In which disease cartwheel or checker pattern histo-
pathologic appearance is seen? Ans. Herpes
Ans. Multiple myeloma 63. Which disease is known as ray fungus?
45. In which disease Antoni A and Antoni B cells are seen? Ans. Actinomycosis
Ans. Neurilemmoma 64. Which disease is known as Lues?
46. Name the salivary gland neoplasm in which predilec- Ans. Syphilis
tion for men is seen? 65. What is hutchinson’s triad?
Ans. Warthin’s tumor Ans. It is hypoplasia of incisor and molar teeth, eighth nerve
47. What is the histological similarity between mickulicz deafness and interstitial glossitis
disease and Sjogren’s syndrome? 66. Name the cells which are seen in cytologic margin of
Ans. Epimyoepithelial islands aphthous ulcer.
48. What is the another name of cylindroma. Ans. Anitischkow cells
Ans. Adenoid cystic carcinoma 67. Name the virus which causes hand, foot and mouth
disease and herpangina.
49. Name the salivary gland neoplasm which spread along
perineural spaces. Ans. Coxsackie virus
Ans. Adenoid cystic carcinoma 68. In which disease Koplik’s spots are seen?
50. Which is the most common salivary gland neoplasm. Ans. Measles
Ans. Pleomorphic adenoma 69. Which disease shows Henderson Paterson bodies?
51. Which is the second most common salivary gland Ans. Molluscum contagiosum
tumor? 70. Name the syndrome with which herpes zoster is
Ans. Warthin’s tumor associated.
Section 3: Oral Pathology  661

Ans. James Ramsay Hunt syndrome 89. Name the disease in which high predilection for os-
teomyelitis is seen after dental extraction.
71. Name the fungal lesion which closely mimics tuber-
culosis. Ans. Osteomyelitis
Ans. Histoplasmosis 90. Name the most common complication of Paget’s dis-
ease.
72. Name the most common opportunistic infection of the
Ans. It leads to pathologic fracture
world.
Ans. Candidiasis 91. Which is the most common site for monostotic fibrous
dysplasia.
73. Name the type of candidiasis which leads to pain.
Ans. Ribs
Ans. Acute atrophic form
92. Name the most common cause for the ankylosis of
74. Who had given acidogenic theory. temporomandibular joint.
Ans. WD Miller Ans. Trauma
75. Name the bacteria which cause dental caries. 93. In which disease Hunter glossitis is seen.
Ans. S. mutans Ans. Pernicious anemia
76. Name the tooth which is least susceptible to dental 94. Name the constant feature of pernicious anemia.
caries. Ans. Achlorhydria
Ans. Mandibular central incisors
95. Name the anemia in which there is lack of resistance
77. Name the bacteria which leads to root caries. to infection.
Ans. A. viscosus Ans. Aplastic anemia
78. Name the condition in which bacteria circulating in 96. Which type of leukemia is most common in children.
blood leads to pulpal inflammation. Ans. Acute lymphocytic leukemia
Ans. Anachoretic pulpitis 97. Which type of leukemia is most common in elders.
79. Which type of hypersensitivity is caused by periapical Ans. Chronic lymphocytic leukemia
granuloma. 98. Name the microorganisms causing ANUG.
Ans. Delayed hypersensitivity Ans. Borrelia vincentii and fusiform bacilli
80. Name the cyst which develops in maxilla after Caldwell 99. Name the disease in which saw tooth rete pegs with
Luc operation. Civatte bodies are seen.
Ans. Surgical ciliated cyst of maxilla Ans. Lichen planus
81. Name the disease in which pseudoepitheliomatous 100. Name the disease which show Auspitz sign, Monroe’s
hyperplasia and plasma pooling is seen. abscess.
Ans. Epulis fissuratum Ans. Psoriasis
82. Name the mucocele which occurs in the floor of mouth. 101. What is Grinspan syndrome.
Ans. Ranula Ans. Lichen planus + Diabetes mellitus + Vascular hypertension
83. In which disease does test tube rete pegs are seen. 102. Corps rods and Grains are seen in which disease.
Ans. Dilantin sodium induced gingival hyperplasia Ans. Darier’s disease.
84. Name the compound which causes acrodynia or pink 103. Name the sign in which there is loss of epithelium on
disease. normal rubbing.
Ans. Mercury Ans. Nikolsky’s sign
85. Name the microorganism which leads to dry socket. 104. Name the disease which show suprabasilar split and
Ans. Treponema denticola Tzanck Cells.
Ans. Pemphigus
86. Deficiency of which ions causes tetany.
Ans. Calcium and magnesium 105. Name the disease which shows histopathological
appearance of rosettes of neutrophils which surrounds
87. Which disease show punched out lesions of bone, the lymphocytes.
exophthalmos and diabetes insipidus. Ans. Systemic lupus erythematosus
Ans. Hand Schuller Christian Disease
106. Name the disease which show hypermobility of TMJ
88. Deficiency of which vitamin shows Trummerfeld zone. and and increase tendency to form pulp stones.
Ans. Vitamin C Ans. Ehler Danlos syndrome
662   Mastering the BDS IIIrd Year  (Last 25 Years Solved Questions)

107. Name the neuralgia which show alarm clock headache Ans. It is the condition in which there is pigmentary
with no trigger zone. changes in associated with the pregnancy or taking the
Ans. Sphenopalatine neuralgia contraceptive pills.
108. Name the condition in which there is shooting pain 112. What do you mean by perimolysis.
with trigger zone in tonsillar fossa. Ans. Perimolysis refers to the intrinsic dental erosion of teeth.
Ans. Glossopharyngeal neuralgia This occurs in patients, in whom the gastric acid from
stomach comes in contact with the teeth.
109. Name the condition in which there is severe pain which
arise after sectioning of peripheral sensory nerve. 113. What do you mean by ameloglyphics.
Ans. Causalgia Ans. These are tooth print patterns formed by enamel rods
end at crown surface of tooth and are recorded by acetate
110. Name the disease in which there is sorrowful peel technique.
appearance of patient with histological appearance of
114. Name the solution which preserves the bite marks on
lymphorrhage.
apples presented at crime scene.
Ans. Myasthenia gravis
Ans. Campden solution Or 5% acetic acid in 40% aqueous
111. What is melasma. formaldehyde.
Section 3: Oral Pathology  663

ADDITIONAL INFORMATION

Various Signs
Signs Meaning Seen in disease
Auspitz’s sign On removing the deep scales single or multiple tiny bleeding spots Psoriasis
are seen
Battle’s sign There is presence of ecchymosis in post auricular region over In sub condylar fractures and
mastoid process fracture of base of skull
Chvostek’s sign While tapping at the angle of mandible there is stimulation of facial Tetany
nerve which leads to twitching of muscles of face over the same side
Cluster of Jewel or Rosette sign or There is presence of new bullae over the old ones which show cluster Chronic bullous disease of
string of pearls of jewel appearance. childhood
Crowe’s sign Presence of axillary freckling Von Recklinghausen’s
neurofibromatosis
Dubois sign Shortening of little finger Congenital syphilis
Flag sign Presence of horizontal alternating bands of discoloration over the Kwashiorkor or Ulcerative colitis
hair shafts which correspond to the period of normal and abnormal
hair growth
Forchheimer sign Exanthem of red macules are seen over the soft palate Rubella
Gorlin’s sign Patient touch the tip of nose with extended tongue Ehler Danlos syndrome
Guiren’s sign Presence of ecchymosis near greater palatine foramen Le Fort I fracture
Higoumenakis sign Presence of irregular thickening of sternoclavicular portion of Congenital syphilis
clavicle
Hoagland’s sign Presence of early and transient bilateral upper lid edema Infectious mononucleosis
Hutchinson’s nose sign Vesicles are seen over the tip of nose Herpes zoster
Jellinek’s sign Pigmentation of eyelid which is occasionally seen Hyperthyroidism
Nikolsky’s sign Epithelium become lost on rubbing and there is presence of raw or Pemphigus, Recessive form of
sensitive surface epidermolysis bullosa
Raccoon sign Present as periorbital ecchymosis due to subconjunctival hemorrhage Basilar skull fracture
Shawl sign Presence of confluent, symmetric, macular violaceous erythema Dermatomyositis
over the shoulder and neck in posterior parts
Tin tack or Carpet track sign Horney track at the undersurface of scale removed from the lesion Discoid lupus erythematosus
Trousseau’s sign Presence of carpal spasm after application of pressure on arm by Hypopart athyroidism
inflating sphygmomanometer cuff

Various Types of Cells and Their Associated Diseases


Various types of cells Disease or pathology associated
Acantholytic cells Pemphigus
Angulate body cells Granular cell myoblastoma
Anitschkow cells Sickle cell anemia, Aphthous ulcer, Iron deficiency anemia, Rheumatic heart disease
Arbiskov cells Myeloblastoma
Benign dyskeratotic cells Or Corps, Ronds and Grains Dyskeratosis follicularis
Centrocytes Non-Hodgkins lymphoma (Follicular variety)
Downey cells Infectious mononucleosis
Foam cells Periapical granuloma
Contd…
664   Mastering the BDS IIIrd Year  (Last 25 Years Solved Questions)

Contd…

Various types of cells Disease or pathology associated


Gaucher cells Gaucher’s disease
Ghost cells Craniopharyngioma, calcifying odontogenic cyst, ameloblastic fibroma and odontoma
Glycogen free clear cell Mucoepidermoid carcinoma and acinic cell carcinoma
Glycogen rich clear cell Gingival cyst of adult and lateral periodontal cyst
Hurler cell or Gargoyle cell Hurler syndrome
Hyaline cell Pleomorphic adenoma
Koilocytes Papilloma
Lacunar cells Hodgkin’s lymphoma (Nodular variety)
LE cell Systemic lupus erythematosus
Lepra cells Lepromatous leprosy
Nevus cells Pigmented mole
Pale cells Odontogenic myxoma
Pericyte of Zimmerman Glomus tumor
Popcorn cells Hodgkin’s lymphoma (Nodular variety)
Raquet or ribbon cells Rhabdomyosarcoma
Reed – Sternberg cells Hodgkin’s lymphoma
Safety pin cells Thalassemia
Touton type multinucleated giant cell Fibrous histiocytoma
Tzanck cells Herpes and pemphigus
Warthin Finkeldey giant cells Measles

Various Spots Associated with Various Pathologies

Name of spots Presentation of spot Pathology


Bitot’s spot Present as white plaque in conjunctiva of children Vitamin A deficiency
Café au lait spot Presence of brown pigmentation on the skin Fibrous dysplasia, neurofibromatosis, Peutz-Jeghers
syndrome
Herald spot Primary lesion of skin Pityriasis Rosea
Koplik spot They are characterized as clustered, white lesions on the Measles
buccal mucosa opposite the lower 1st and 2nd molars
Pink spot Color of tooth become pink Internal resorption
Roth spot Retinal hemorrhages with white or pale centers Subacute bacterial endocarditis and typhoid fever
Sore spot Presence of traumatic ulcers Denture irritation

Various Inclusion Bodies Associated with Different Diseases

Inclusion body Disease associated


Asteroid bodies Sporotrichosis
Civatte or Colloid or hyaline or cytoid bodies Lichen planus
Cow dry Type A Herpes simplex
Cow dry Type B Poliomyelitis
Dohle bodies Chédiak – Higashi syndrome
Fessas bodies Thalassemia
Guarnieri bodies Small pox virus and Vaccinia virus
Heinz bodies In G6PD deficiency individuals
Contd…
Section 3: Oral Pathology  665

Contd…

Howell – Jolly body Pernicious anemia


Lipschutz bodies Primary herpetic stomatitis
Molluscum bodies or Henderson – Paterson inclusion Molluscum contagiosum
Negri bodies Rabies
Pappenhiemer bodies Sideroblastic anemia
Psammoma bodies Papillary carcinoma of thyroid
Reilly bodies Hurler syndrome
Rushton bodies Inflammed dentigerous cyst, periapical cyst, gingival cyst of neonate
Russell bodies Periapical granuloma, inflammatory conditions and multiple myeloma
Verocay bodies Neurilemmoma

Various Specific Epithelial Rete Pegs Associated with Some Specific Diseases

Type of rete pegs Disease associated


Absence of rete pegs Oral submucous fibrosis
Saw tooth rete pegs Lichen planus
Test tube rete pegs Dilantin hyperplasia

Various Important Diagnostic Tests for Various Diseases

Diagnostic test Disease


Dick test Scarlet fever
Elisa and western blot test Acquired Immunodeficiency syndrome (AIDS)
Figlus excretion test Folic acid absorption
Freis test Lymphogranuloma venereum
Gordons biological test Hodgkin’s lymphoma
Vdrl and fta abs Syphilis
Kveims test Sarcoidosis
Mantoux test Tuberculosis
Paul bunnel test Infectious mononucleosis
Prick skin or scratch skin test For allergic skin reactions
Rose-Waller test Rheumatoid arthritis
Schiller’s test Cervical carcinoma
Schilling’s test Vitamin B12 deficiency
Schimmer’s test or rose bengal test Sjögren’s syndrome
Schick test Diphtheria
Sweat test Cystic fibrosis
Tzanck test Pemphigus and Herpes simplex
Weil-felix test Rickettsial manifestations

Incubation Periods of Various Diseases

Various diseases Incubation periods


Chicken pox 2 to 3 weeks
Rubella 2 to 3 weeks
Contd…
666   Mastering the BDS IIIrd Year  (Last 25 Years Solved Questions)

Contd…

Mumps 2 to 3 weeks
Diphtheria 1 to 5 days
Typhoid 1 to 5 days
Cholera 1 to 5 days
Hepatitis A 2 to 7 weeks
Hepatitis B 7 to 23 weeks
Influenza 1 to 3 days
Measles 10 days
Tetanus 3 to 21 days

Characteristic Features of Tongue in Various Conditions

Appearance or changes in tongue Condition


Atrophic glossitis Iron deficiency anemia and pernicious anemia
Bald tongue of sandwith Pellagra
Bifid tongue or cleft tongue Orofacial digital syndrome
Black hairy tongue Chronic antibiotic therapy
Cerebriform tongue Pemphigus vegetans
Hunter’s glossitis Pernicious anemia
Luetic glossitis Syphilis
Magenta colored tongue Riboflavin deficiency
Migratory glossitis Geographic tongue
Strawberry tongue Scarlet fever
Tetanus 3 to 21 days

Various Theories of Dental Caries and Their Authors

Theory of dental caries Author


Acidogenic theory or chemico-parasitic theory Miller
Acidic theory Robertson
Parasitic theory Dubos
Proteolytic theory Gottileb
Proteolysis chelation theory Schartz and Martin
Sulfatase theory Pincus
Sucrose chelation theory Burch and Jackson

Important Terminologies with their Meanings in Oral Pathology

Terminology Meaning
Macule It is the focal area of color change which is not elevated or depressed in relation to the surroundings
Papule It is a solid and raised lesion which is less than 5 mm in diameter
Nodule It is a solid and raised lesion which is more than 5 mm in diameter
Vesicle It is a superficial blister which is 5 mm or less in diameter and is filled by the clear fluid
Bulla It is a large blister which is more than 5 mm in diameter
Sessile It is a tumor or the growth whose base is the widest part of lesion
Contd…
Section 3: Oral Pathology  667

Contd…

Pedunculated It is a tumor or the growth whose base is narrow and is widest part of lesion
Papillary It is a tumor or growth having numerous projections
Verrucous It is a tumor or growth having rough and warty surface
Pustule It is a blister which is filled with purulent exudates
Ulcer Ulcer is characterized by loss of epithelium along with underlying connective tissue
Erosion It is characterized by the partial or total loss of surface epithelium
Fissure It is a narrow slit like ulceration or groove
Plaque It is a lesion which is slightly elevated and is flat on its surface
Petechiae It is pinpoint area of hemorrhage
Ecchymosis It is a non-elevated area of hemorrhage which is larger than petechiae

Various Skin Diseases and Their Fluorescent Appearance

Skin disease Fluorescent appearance


Pemphigus Granular intercellular space fluorescence
Lichen planus Fluorscence is seen at basement membrane zone with multiple extension in lamina propria
Cicatricial pemphigoid Presence of patchy linear pattern at basement membrane zone
Bullous pemphigoid Presence of patchy linear pattern at basement membrane zone
Erythema multiforme Presence of patchy linear pattern
Discoid lupus erythematosus Presence of speckeled or particulate pattern at basement membrane zone
Systemic lupus erythematosus Fluorescence is seen at basement membrane zone with multiple extension in superficial lamina propria

Various Microorganisms and Their Associated Pathologies

Microorganism Associated pathology


Streptococcus mutans They causes initiation of smooth surface caries
Streptococcus sanguis They causes dental caries and subacute bacterial endocarditis
Streptococcus salivarius They causes subacute bacterial endocarditis
Streptococcus pyogenes They leads to pharyngitis, Ludwig’s angina, cellulitis
Staphylococci They leads to skin infections, osteomyelitis, sialedenitis
Staphylococcus aureus They lead to sialedenitis, toxic shock syndrome
Lactobacillus They lead to the progression of dental caries
Actinomyces viscosus They causes multiple abscess
Prevotella intermedia It causes pregnancy gingivitis
Capnocytophaga It causes puberty gingivitis
Porphyromonas gingivalis It causes generalized juvenile periodontitis
A. Actinomycetmcomitans It causes localized juvenile periodontitis
Clostridium tetani Tetanus
Fusospirochete Acute necrotizing ulcerative gingivitis
Treponema pallidum Syphilis
Clostridium perfringens Gas gangrene
Treponema denticola Dry socket
TOGA virus German measles
Human papilloma virus Papilloma or wart
Paramyxovirus Mumps and measles
668   Mastering the BDS IIIrd Year  (Last 25 Years Solved Questions)

Various Teeth Which are Associated with Specific Pathologies

Pathology Teeth Associated

Natal teeth which is commonly seen Deciduous mandibular central incisor

Most common ankylosed or submerged tooth Deciduous mandibular second molar and permanent first and second
molars

Most common permanent tooth which show variation in size and shape Maxillary lateral incisor

Which tooth is affected commonly by dens evaginatus Premolars

Teeth most commonly affected in concrescence Permanent maxillary molars mainly the third molar

Teeth commonly affected by microdontia Permanent maxillary lateral incisors and third molars

Teeth known as ghost teeth Permanent maxillary and mandibular anterior teeth

Teeth which get impacted commonly Third molars and maxillary canines

Teeth most commonly show supernumerary roots Mandibular canine and the premolars

Name the permanent teeth which are most commonly missing Third molars, permanent maxillary lateral incisor and permanent
mandibular second molar

Name the deciduous teeth which are most commonly missing Deciduous maxillary lateral incisor and deciduous mandibular lateral
incisor

Which are the teeth which get commonly extracted for orthodontic Maxillary and mandibular first premolars
purpose

Name the tooth which most commonly shows variation in its eruption Mandibular second premolar
timing

Which tooth is known as taurodont or exhibit taurodontism Deciduous or permanent molars

Name the tooth which is commonly affected by benign cementoblastoma Permanent mandibular first molar
and condensing osteitis

Name the tooth affected by pulp polyp Deciduous molars and maxillary or mandibular permanent first molars

Name the teeth affected most commonly by nursing bottle caries Deciduous maxillary incisors

Name the teeth which get prevented in nursing bottle caries Deciduous mandibular incisors

Various Jaw Abnormalities and Their Association with The Diseases

Jaw abnormalities Disease associated

Decrease in size of mandible or underdeveloped mandible • Pierre Robinson Syndrome


• Treacher Collin syndrome

Decrease in size of maxilla or underdeveloped maxilla • Cleidocranial dysostosis


• Osteogenesis imperfect
• Achondroplasia
• Down syndrome

Decrease in size of both maxilla and mandible or underdeveloped Hypopituitarism


maxilla and mandible

Increase in size of maxilla Monostotic fibrous dysplasia

Increase in size of mandible Acromegaly

Increase in size of both maxilla and mandible Paget’s disease


Section 3: Oral Pathology  669

Lesion/ Condition Radiological appearance


Acute osteomyelitis Moth eaten radiolucency
Aneurysmal bone cyst Honeycomb or soap bubble appearance
Apert’s syndrome Skull radiograph “Beaten metal” pattern
Calcifying epithelial odontogenic tumor Driven snow appearance
Cemento-osseous dysplasia Cotton wool radiopacities
Central hemangioma Honey combed or sunburst or cotton wool appearance
Cherubism Ground glass appearance; floating teeth syndrome
Chronic diffuse sclerosing osteomyelitis Cotton wool appearance/mosaic pattern
Coronal  dentin dysplasia Thistle tube appearance
Crouzon syndrome Skull  radiograph “Beaten metal” pattern
Dentinogenesis imperfecta – 2 Shell teeth
Ewing’ s sarcoma Onion skin appearance; sunray appearance rarely
Fibrous dysplasia Ground glass appearance; Rind sign
Garre’s osteomyelitis Onion peel appearance
Gaucher’s disease Erlenmeyer flask deformity of distal femur
Gigantiform cementoma Cotton wool radio-opacities
Globulomaxillary cyst Pear shaped radiolucency between maxillary lateral incisor and canine
Gardner syndrome Cotton wool radio-opacities
Hemangioma Hair on end or crew cut appearance
Hyperparathyroidism Partial loss of lamina dura; ground glass appearance of bone
Hypophosphatasemia Metaphyses of long bone show spotty or streaky or irregular ossifications
Meningioma Hair on end or crew cut appearance
Multiple myeloma Punched out areas of radiolucency
Nasopalatine duct cyst Heart shaped radiolucency between roots of maxillary central incisors
Odontogenic myxoma Honey comb / mottled appearance
Osteomalacia Looser’s zone pseudofracture line
Osteopetrosis Vertebrae- Rugger-Jersey pattern; Ribs - Endo bone (Bone with in bone) pattern; roots are not
easily distinguishable from adjacent bone
Osteosarcoma Intramedullary parts appears as cumulus cloud densities; sunray/ sun burst pattern; widening
of periodontal ligament; Codman’s triangle (periosteum raised like tent)
Paget’s disease Osteoporosis circumscripta; cotton wool appearance; hypercementosis; loss of lamina dura
Plasmacytoma Hair on end or crew cut appearance
Primary intraosseous carcinoma Moth eaten radiolucency
Pseudohypoparathyroidism Chevron pulp
Psoriatic arthritis Pencil cup appearance; opera glass deformity in joints
Regional odontodysplasia Ghost teeth
Rickets Muller’s line (Widened space @ the site of zone of preparatory calcification)
Rachitic rosary- costochondral prominence
Scleroderma Widening of lamina dura; bone resorption at angle, condyle or coronoid area
Scurvy and Chronic Vitamin C deficiency Increased density @ end of long bones as white lines- “line of Frenkel”;
Signet ring appearance of epiphyses;
Zone of rarefaction around white lines- represents “Trummerfield zone”
Sclerotic ring around epiphyses – “Winberger’ line”
Metaphyseal corner fracture-  “Pelkan spur”
Contd…
670   Mastering the BDS IIIrd Year  (Last 25 Years Solved Questions)

Contd…
Severe iron deficiency in child hood Hair on end or crew cut appearance
Sialdenosis (sialography) Leafless tree
Sickle cell anemia Hair on end or crew cut appearance
Sjögren’s syndrome (Sialography) Branchless fruit laden tree or cherry blossom appearance
Synovial sarcoma Spotty calcification; snow storm appearance
Synovial sarcoma Snow storm appearance
Thalassemia Rib with in a rib appearance of rib; hair on end or crew cut appearance; salt and pepper effect; 
thin lamina dura
IMPORTANT CLASSIFICATIONS

Classification of Supernumerary Teeth Type IVB: Hypoplastic-hypomaturation with taurodont­


ism, autosomal dominant
According to the morphology
♦♦ Conical: This small peg shaped conical tooth is supernu­ Classification of White Lesions of Oral Cavity
merary tooth. ♦♦ Hereditary condition:
♦♦ Tuberculate: This type of supernumerary tooth possesses • Leukoedema
more than one cusp or tubercle. It is of barrel shaped and • White sponge nevus
may be invaginated. • Hereditary benign intraepithelial dyskeratosis
♦♦ Supplemental: It refers to the duplication of the teeth • Keratosis follicularis
in normal series. The most common tooth is permanent
• Ptylosis syndrome.
maxillary lateral incisor.
♦♦ Leukoplakia and malignancies:
♦♦ Odontome: This represents the hamartomatous malfor­
• Chronic cheek biting
mation.
• Friction or trauma associated leukoplakia
According to Location • Tobacco associated leukoplakia
• Carcinoma in situ
♦♦ Mesiodens: They are located between two upper central
• Squamous cell carcinoma
incisor.
• Verrucous carcinoma.
♦♦ Distomolars: They are located on the distal aspect of
♦♦ Dermatosis:
regular molar teeth in dental arch.
• Lichen planus
♦♦ Paramolar: They are located either in buccal or lingual
• Lupus erythematous.
aspect of normal molars.
♦♦ Inflammation:
♦♦ Extra lateral incisors: They are more common in maxil­
• Mucous patches of syphilis
lary arch.
• Candidiasis
Classification of Amelogenesis Imperfecta • Koplik spots of measles.
♦♦ Miscellaneous conditions:
Witkop, 1988 Four major categories based primarily on • Oral submucous fibrosis
phenotype (hypoplastic, hypomaturation, hypocalcified, • Papilloma
hypomaturation-hypoplastic with taurodontism) subdivided • Lipoma
into 15 subtypes by phenotype and secondarily by mode of • Hairy tongue
inheritance. • Geographic tongue
Type I: Hypoplastic • Fordyce’s granules.
Type IA: Hypoplastic, pitted autosomal dominant
Type IB: Hypoplastic, local autosomal dominant Classification of Hemangiomas by Watson and McCarthy
Type IC: Hypoplastic, local autosomal recessive ♦♦ Capillary hemangioma
Type ID: Hypoplastic, smooth autosomal dominant ♦♦ Cavernous hemangioma
Type IE: Hypoplastic, smooth X-linked dominant ♦♦ Angioblastic hemangioma
Type IF: Hypoplastic, rough autosomal dominant ♦♦ Racemose hemangioma
Type IG: Enamel agenesis, autosomal recessive ♦♦ Diffuse systemic hemangioma
Type II: Hypomaturation ♦♦ Metastatizing hemangioma
Type IIA: Hypomaturation, pigmented autosomal ♦♦ Port-Wine stain
recessive ♦♦ Hereditary hemorrhagic telangiectasia
Type IIB: Hypomaturation, X-linked recessive
Classification of Giant Cell Lesions of Oral Cavity
Type IIC: Hypomaturation, snow-capped teeth, X-linked
♦♦ According to the nature of different pathologic conditions
Type IID: Hypomaturation, snow-capped teeth, auto­
• infections
somal dominant?
–– Bacterial
Type II: Hypocalcified –– Viral
Type IIIA: Autosomal dominant –– Fungal
Type IIIB: Autosomal recessive –– Protozoal
Type IV: Hypomaturation-hypoplastic with taurodontism –– Parasitic
Type IVA: Hypomaturation-hypoplastic with taurodont­ • Fibro-osseous lesions and osteodystrophies
ism, autosomal dominant –– Immunologic
672   Mastering the BDS IIIrd Year  (Last 25 Years Solved Questions)

–– Idiopathic Enumeration of premalignant lesion


–– Orofacial granulomatosis ♦♦ Leukoplakia
–– Reaction to materials ♦♦ Erythroplakia
–– Benign and malignant tumors ♦♦ Mucosal changes associated with smoking habits
♦♦ Giant cells are categorized into three categories ♦♦ Carcinoma in situ
• Giant cells are the main cause for the pathology ♦♦ Bowen disease
• Giant cells characterize these lesions ♦♦ Actinic keratosis, actinic cheilitis and actinic elastosis.
• Lesions that may be associated with giant cells Enumeration of premalignant condition
Main cause for the pathology ♦♦ Syphilis
♦♦ Giant cell granuloma ♦♦ OSMF
• Peripheral ♦♦ Oral lichen planus
• Central ♦♦ Sideropenic dysplasia
♦♦ Giant cell tumors ♦♦ Dyskeratosis congenita
♦♦ Giant cell fibroma ♦♦ Lupus erythematosus
♦♦ Hyperparathyroidism Unusual varients of squamous cell carcinoma
Giant cells that characterize lesions ♦♦ Verrucous carcinoma
♦♦ Infections ♦♦ Spindle cell carcinoma
• Tuberculosis ♦♦ Basaloid squamous cell carcinoma
• Hansen’s disease ♦♦ Adenoid/acantholytic squamous cell carcinoma
• Syphilis ♦♦ Papillary squamous cell carcinoma
• Measles ♦♦ Carcinoma cuniculatum
♦♦ Granulomatous lesions
• Wegener’s granulomatosis Classification of Odontogenic Tumors by WHO (2005)
• Orofacial granulomatosis Benign Tumors
• Pulse granuloma
• Sarcoidosis ♦♦ Odontogenic Epithelium with Mature Fibrous Stroma
♦♦ Lesions in bone without Odontogenic Mesenchyme:
• Aneurysmal bone cyst • Ameloblastoma (Solid, Multicystic, Extraosseous,
• Cherubism • Peripheral, Desmoplastic, Unicystic)
• Paget’s disease • Squamous Odontogenic Tumor
♦♦ Foreign body lesions • Calcifying Epithelial Odontogenic Tumor
• Silicosis • Adenomatoid Odontogenic Tumor
• Berylliosis • Keratocystic Odontogenic Tumor
♦♦ Malignancies ♦♦ Odontogenic Epithelium with Odontogenic Ectomesen­
• Lymphoma-hodgkin’s disease chyme with or without hard tissue formation:
• Bronchogenic carcinoma • Ameloblastic Fibroma
• Carcinoma of thyroid • Ameloblastic Fibro-Dentinoma
♦♦ Miscellaneous • Ameloblastic Fibro-Odontoma
• Xanthoma • Odontoma
• Giant cell arteritis –– Complex
–– Compound
Lesions that may be associated with giant cell
• Odonto-ameloblastoma
♦♦ Malignancies
• Calcifying Cystic Odontogenic Tumor
• Multiple myeloma
• Dentinogenic Ghost Cell Tumor
• Ewing’s sarcoma
♦♦ Mesenchyme and/or Odontogenic Ectomesenchyme with
• Fibrosarcoma
or without Odontogenic Epithelium:
• Chondrosarcoma
• Odontogenic Fibroma
♦♦ Fibro-osseous lesions
• Odontogenic Myxoma
• Osteoblastoma
• Cementoblastoma
• Fibrous dysplasia
• Cemento-ossifying fibroma Malignant Tumors
• Radicular cyst
♦♦ Odontogenic Carcinomas:
Based on their origin • Malignant Ameloblastoma
• Ameloblastic Carcinoma
♦♦ Epithelially derived, i.e. Warthin-Finkeldey Giant Cells,
• Primary Intraosseous Carcinoma
Tumor Giant Cells
• Clear Cell Odontogenic Carcinoma
♦♦ Stromally derived, i.e. Reed-Sternberg Giant Cells.
• Ghost Cell Odontogenic Carcinoma
Section 3: Oral Pathology  673

♦♦ Odontogenic Sarcoma: Adipose Tissue


• Ameloblastic Fibro-Sarcoma
• Ameloblastic Fibro-Dentino Sarcoma Benign Tumors
• Ameloblastic Fibro-Odonto-Sarcoma. ♦♦ Lipoma
Bone Related Lesions ♦♦ Angiolipoma

♦♦ Ossifying Fibroma Malignant Tumors


♦♦ Fibrous Dysplasia Liposarcoma
♦♦ Osseous Dysplasia
♦♦ Central Giant Cell Lesions Bone
♦♦ Cherubism
♦♦ Aneurysmal Bone Cyst Benign Tumors
♦♦ Simple Bone Cyst ♦♦ Osteoma
♦♦ Osteoid osteoma
Melanotic Neuroectodermal Tumor of Infancy.
♦♦ Osteoblastoma
Classification of Non-Odontogenic Tumors of Oral Cavity
Malignant Tumors
Epithelial Tissue
♦♦ Osteosarcoma
Benign Tumors ♦♦ Osteochondrosarcoma
♦♦ Papilloma
Vascular Tissue
♦♦ Keratoacanthoma
♦♦ Squamous acanthoma Benign Tumors
♦♦ Nevus
♦♦ Hemangioma
Malignant Tumors ♦♦ Hereditary hemorrhagic telangiectasia
♦♦ Lymphangioma
♦♦ Squamous cell carcinoma
♦♦ Mucoepidermoid carcinoma Malignant Tumors
♦♦ Adenocarcinoma
Hemangioendothelioma
♦♦ Basal cell carcinoma
♦♦ Transitional cell carcinoma
♦♦ Melanoma Neural Tissue
♦♦ Verrucous carcinoma Benign Tumors
♦♦ Intraepidermoid carcinoma
♦♦ Neurofibroma
Fibrous Connective Tissue ♦♦ Neurilemmoma
♦♦ Schwannoma
Benign tumors
♦♦ Fibroma Malignant Tumors
♦♦ Fibrous hyperplasia ♦♦ Neurosarcoma
♦♦ Fibrous epulis ♦♦ Neurofibrosarcoma
♦♦ Giant cell fibroma
♦♦ Myxoma Muscles
♦♦ Myxofibroma
Benign Tumors
Malignant Tumors ♦♦ Leiomyoma
Fibrosarcoma ♦♦ Rhabdomyoma

Cartilage Tissue Malignant Tumors


Benign Tumors ♦♦ Leiomyosarcoma
♦♦ Rhabdomyosarcoma
♦♦ Chondroma
♦♦ Chondroblastoma Giant Cell Tumor
♦♦ Chondromyxoid fibroma
Central and peripheral giant cell tumor
Malignant Tumors Giant cell granuloma
Chondrosarcoma Giant cell tumor of hyperthyroidism.
674   Mastering the BDS IIIrd Year  (Last 25 Years Solved Questions)

Teratoma ♦♦ Oncocytic carcinoma (8290/3)


♦♦ Salivary duct carcinoma
Salivary gland tumor
♦♦ Adenocarcinoma NOS (8140/3)
Benign Tumors ♦♦ Myoepithelial carcinoma (8982/3)
♦♦ Adenoma ♦♦ Carcinoma in pleomorphic adenoma (8941/3)
♦♦ Warthin’s tumor ♦♦ Carcinosarcoma (8980/3)
♦♦ Pleomorphic adenoma. ♦♦ Metastasizing pleomorphic adenoma (8940/1)
♦♦ Squamous cell carcinoma (8070/3)
Malignant Tumors ♦♦ Small cell carcinoma (8041/3)
♦♦ Mucoepidermoid Carcinoma ♦♦ Large cell carcinoma (8012/3)
♦♦ Adenocystic carcinoma ♦♦ Lymphoepithelial carcinoma (8082/3)
♦♦ Adenocarcinoma ♦♦ Sialoblastoma (8974/1)
♦♦ Acinic cell carcinoma
♦♦ Malignant change in pleomorphic adenoma Soft tissue tumors
♦♦ Haemangioma (9120/0)
Lymphoid Tissue
Haematolymphoma tumors
Malignant Tumors
♦♦ Hodgkin’s and non-hodgkin’s lymphoma ♦♦ Hodgkin lymphoma
♦♦ Lymphosarcoma ♦♦ Diffuse large B cell lymphoma
♦♦ Reticular cell sarcoma ♦♦ Extranodal marginal zone B cell lymphoma
♦♦ Ewing’s sarcoma Secondary tumors
♦♦ Burkitt’s lymphoma
♦♦ Multiple myeloma Classification of Oral Submucous Fibrosis
♦♦ Leukemia
Khanna JN and Andrade N (1995) develop classification of
Salivary Gland Tumor Classification by WHO (2005) OSMF based on clinical features and histopathological features.
Benign Epithelial Tumors
Group I: Very Early Cases
♦♦ Pleomorphic adenoma (8940/0)
♦♦ Common symptom is burning sensation in the mouth
♦♦ Myoepithelioma (8982/0)
♦♦ Acute ulceration and recurrent stomatitis
♦♦ Basal cell adenoma (8147/0)
♦♦ Not associated with mouth opening limitation.
♦♦ Warthin tumor (adenolymphoma) (8561/0)
♦♦ Oncocytoma (oncocytic adenoma) (8290/0) Histology
♦♦ Canalicular adenoma (8149/0)
♦♦ Sebaceous adenoma (8410/0) ♦♦ Fine fibrillar collagen network interspersed with marked
♦♦ Lymphadenoma (8410/0) edema
• Sebaceous ♦♦ Blood vessels are dilated and congested
• Non-sebaceus ♦♦ Largest aggregate of plump, young fibroblasts present in
♦♦ Ductal papilloma (8503/0) abundant cytoplasm
• Inverted ductal papilloma ♦♦ Inflammatory cells mainly consists of PMN leukocytes
• Intraductal papilloma with few eosinophils
• Sialadenoma papilliferum ♦♦ Epithelium is normal.
♦♦ Cystadenoma (8440/0)
Group II: Early Cases
Malignant Epithelial Tumors
♦♦ Buccal mucosa appears mottled and marble like
♦♦ Acinic cell carcinoma (8550/3)
♦♦ Widespread sheets of fibrosis palpable
♦♦ Mucoepidermoid carcinoma (8430/3)
♦♦ Patients with interincisal distance of 26 to 35 mm.
♦♦ Adenoid cystic carcinoma (8200/3)
♦♦ Polymorphous low-grade adenocarcinoma Histology
♦♦ Epithelial-myoepithelial carcinoma (8562/3)
♦♦ Clear cell carcinoma, not otherwise specified (8310/3) ♦♦ Juxtaepithelial hyalinization present
♦♦ Basal cell adenocarcinoma (8147/3) ♦♦ Collagen present as thickened but separate bundles
♦♦ Sebaceous carcinoma (8410/3) ♦♦ Blood vessels are dilated and congested
♦♦ Sebaceous lymphadenocarcinoma (8410/3) ♦♦ Young fibroblasts are seen in moderate number
♦♦ Cystadenocarcinoma (8440/3) ♦♦ Inflammatory cells mainly consists of polymorphonuclear
♦♦ Low grade cribriform cystadenocarcinoma leukocytes with few eosinophils and occasional plasma
♦♦ Mucinous adenocarcinoma (8480/3) cells
Section 3: Oral Pathology  675

♦♦ Flattening or shortening of epithelial rete pegs evident • Odontoma


with varying degree of keratinization. • Adenomatoid odontogenic tumor
• Squamous odontogenic tumor
Group III: Moderately Advanced Cases • Dental lamina cyst of newborn
♦♦ Trismus evident with interincisal distance of 15 to 25 mm ♦♦ Non-odontogenic origin
♦♦ Buccal mucosa appears pale and firmly attached to un­ • Vascular origin
derlying tissues –– Hemangioma
♦♦ Atrophy of vermilion border –– Lymphangioma
♦♦ Vertical fibrous bands are palpable at soft palate, ptery­ –– Glomus tumor
gomandibular raphae and anterior faucial pillars. • Osseous origin
–– Fibrous dysplasia
Histology • Oral and labial melanotic macule
♦♦ Juxtaepithelial hyalinization present • Pigmented cellular nevus
♦♦ Thickened collagen bundles faintly discernable separated ♦♦ Neural origin
Neurofibromatosis
by very light residual edema
♦♦ Syndrome
♦♦ Blood vessels are mostly constricted
Multiple hamartoma syndrome
♦♦ Mature fibroblasts with scanty cytoplasm and spindle
♦♦ Salivary gland
shape nuclei
Adenomatoid hyperplasia of mucous glands
♦♦ Inflammatory exudates consists mainly of lymphocytes
and plasma cells Enumeration of Round Cell Tumors
♦♦ Epithelium is markedly atrophic with loss of rete pegs
♦♦ Lymphoma
♦♦ Muscle fibers seen interspersed with thickened and dense
♦♦ Rhabdomyosarcoma
collagen fibers.
♦♦ Melanoma
Group IV A: Advanced Cases ♦♦ Ewing’s sarcoma
♦♦ Glomus tumor
♦♦ Trismus is severe with interincisal distance less than
♦♦ Esthesioneuroblastoma
15 mm
♦♦ Merkel cell tumor
♦♦ The fauces are thickened, shortened and firm on
♦♦ Adenocarcinoma
palpation ♦♦ Squamous cell carcinoma (poorly differentiated)
♦♦ Uvula is shrunken and appears as small, fibrous bud ♦♦ Plasmacytoma
♦♦ Tongue movements get limited ♦♦ Multiple myeloma
♦♦ On palpation of lips circular bands are felt at around ♦♦ Langerhan’s cell disease
entire mouth. ♦♦ Peripheral neuroectodermal tumor
Group IV B: Advanced Cases with Premalignant and
Enumeration of Lesions Associated with Impacted Tooth
Malignant Changes
♦♦ Cysts
Hyperkeratosis, leukoplakia or squamous cell carcinoma can
• Dentigerous cyst
be seen.
• Keratocystic odontogenic tumor
Histology • Calcifying odontogenic cyst
♦♦ Salivary gland
♦♦ Collagen hyalinized as small sheet
• Mucoepidermoid carcinoma (central variety)
♦♦ Extensive fibrosis obliterates the mucosal blood vessel and
♦♦ Odontogenic tumors
eliminates melanocytes
• Central odontogenic fibroma
♦♦ Fibroblasts were markedly absent within hyalinized zones • Ameloblastic fibroma
♦♦ Total loss of epithelial rete pegs • Ameloblastic fibroodontome
♦♦ Mild to moderate atypia present • Odontoma
♦♦ Extensive degeneration of muscle fibers evident. • Adenomatoid odontogenic tumor
• Squamous odontogenic tumor
Classification of Oral Hamartomas
• Unicystic ameloblastoma
♦♦ Those involving teeth
• Dens invaginatus Enumeration of Clear Cell Lesions
• Dens evaginatus
• Talon cusp ♦♦ Clear cell carcinoma
♦♦ Those not involving teeth ♦♦ Clear cell ameloblastic carcinoma
• Peripheral odontogenic fibroma ♦♦ Clear cell odontogenic tumor
• Ameloblastic fibro-odontoma ♦♦ Mucoepidermoid carcinoma
676   Mastering the BDS IIIrd Year  (Last 25 Years Solved Questions)

♦♦ Oncocytoma TNM Classification by American Joint Committee on


♦♦ Epimyoepithelial carcinoma Cancer (AJCC)
♦♦ Acinic cell adenocarcinoma
T is suggestive of primary tumor
♦♦ Clear cell ameloblastoma
♦♦ Clear cell chondrosarcoma N is suggestive of regional lymph nodes
M is suggestive of distant metastasis
Enumeration of Granular Cell Diseases
T – Primary Tumor
♦♦ Granular cell in salivary gland diseases
TX – Primary tumor cannot be assessed
• Oncocytoma
• Acinic cell carcinoma T0 – No evidence of primary tumor
• Warthin’s tumor Tis – Carcinoma in situ
• Oncocytic carcinoma T1 – Tumor 2 cm of less in greatest dimension
• Oncocytosis T2 – Tumor more than 2 cm but not more than 4 cm in greatest
• Oncocytic variant of mucoepidermoid carcinoma dimension
• Sclerosing polycystic adenosis T3 – Tumor more than 4 cm in greatest dimension
• Pleomorphic adenoma
T4a (Lip) – Tumor invades through cortical bone, inferior
♦♦ Granular cell in odontogenic tumors
alveolar nerve, floor of mouth or skin (chin or nose)
• Granular cell ameloblastoma
• Granular cell ameloblastic fibroma T4a (Oral cavity) – Tumor invades through cortical bone, into
• Granular cell odontogenic tumor deep/extrinsic muscle of tongue (genioglossus, hyoglossus,
♦♦ Granular cells in soft tissue tumors palatoglossus and styloglossus), maxillary sinus or skin of face.
• Granular cell tumor (Myoblastoma, Schwanoma) T4b (lip and oral cavity) – Tumor invades masticatory space,
• Congenital epulis pterygoid plates or skull base or encases internal carotid artery
• Rhabdomyoma N – Regional lymph nodes
• Rhabdomyosarcoma
NX – Regional lymph nodes cannot be assessed
• Alveolar soft part sarcoma
N0 – No regional lymph node metastasis
• Verruciform xanthoma
• Fibrous histiocytoma N1– Metastasis in a single ipsilateral lymph node, 3 cm or less
• LangerhAns cell histiocytosis in greatest dimension
♦♦ Granular cells in viral, fungal and bacterial infections N2a – Metastasis in a single ipsilateral lymph node, more than
• Molluscum contagiosum 3 cm but not more than 6 cm in greatest dimension
• Botyromycosis N2b – Metastasis in multiple ipsilateral lymph nodes, not more
• Histoplasmosis than 6 cm in greatest dimension
N2c – Metastasis in bilateral or contralateral lymph nodes, not
Staging of Hodgkin's Disease more than 6 cm in greatest dimension
N3 – Metastasis in a lymph node more than 6 cm in greatest
Ann Arbor Staging Classification of Hodgkin's Disease dimension.
Stage I I Involvement of a single lymph node region M – Distant Metastasis
(A or B) IE MX – Distant metastasis cannot be assessed
Stage II II Involvement of two or more lymph node M0 – No distant metastasis
(A or B) IIE regions on the same side of the diaphragm M1 – Distant metastasis
(or) with localized contiguous involvement of
an extranodal organ of site
Stage Grouping of Oral Cancer
Stage III III Involvement of lymph node regions on both
(A or B) sides of the diaphragm Stage 0 Tis N0 M0
IIIE (or) with localized contiguous involvement of Stage I T1 N0 M0
an extranodal organ or site
Stage II T2 N0 M0
IIIS (or) with involvement of spleen
(or) both features of IIIE and IIIS Stage III T1 N1 M0
IIIES
T2 N1 M0
Stage IV IV Multiple or disseminated involvement of one T3 N0, N1 M0
(A or B) or Stage Iva T1,T2,T3 N2 M0
More extra-lymphatic orgAns of tissues with T4a N0, N1, N2 M0
or without lymphatic involvement Stage IVb Any T N3 M0
A = asymptomatic; B = presence of constitutional symptoms; T4b Any N M0
E = extranodal involvement; S = splenomegaly Stage IVc Any T Any N M1
Section 3: Oral Pathology  677

Grading of Squamous Cell Carcinoma M-Category


Grading is defined as macroscopic and microscopic degree of M (Metastases)
differentiation of a tumor.
M0 – No distant metastasis
Squamous cell carcinoma is divided in following categories by
M1 – Clinical evidence of distant metastasis without definite
Broader also known as Broader’s classification.
histological and/ or radiographic confirmation
Broader’s Classification M2 – Proven evidence of metastasis beyond the regional nodes.
♦♦ Grade I – Well differentiated, - <25% undifferentiated cells P-Category
♦♦ Grade II – Moderately differentiated - <50% undifferenti­
ated cells P (Pathology)
♦♦ Grade III – Poorly differentiated - <75% undifferentiated P0 – Hyperkeratotic lesion showing atypia
cells P1 – Carcinoma in situ
♦♦ Grade IV – Anaplastic/Pleomorphic - >75% undifferenti­
P2 – Basal cell carcinoma
ated cells
P3a – Verrucous carcinoma
STNMP Classification of Oral Squamous Cell Carcinoma P3b – Well-differentiated squamous cell carcinoma
P3c – Moderately differentiated squamous cell carcinoma
S-Category
P3d – Poorly differentiated squamous cell carcinoma
S (Site)
STNMP Weighting and Stages
S1 lip – Skin
S2 lip – Mucous membrane S T N M P
S3 tongue S1 – 4 T1 – 0 N0 – 0 M0 – 0 P0 – 0
S4 cheek S2 – 6 T2 – 10 N1 – 10 M1 – 30 P1 – 5
S5 palate S3 – 8 T3 – 20 N2 – 20 M2 – 40 P2 – 5
S6 floor of the mouth S4 – 10 T4 – 35 N3 – 30 P3a – 5
S5 – 12 N4 – 40 P3b – 10
S7 alveolar process
S6 – 14 N5 – 40 P3c – 15
S8 antrum S7 – 16 P3d – 20
S9 central carcinoma of bone S8 – 18
S9 – 20
T-Category
For convenience and to enable direct comparison with TNM
T (Tumor)
staging, the scores are banded to form similar stages
T1 - Tumor less than 20 mm in diameter Stage I – 0 to 30
T2 – Tumor between 20 mm and 40 mm in diameter Stage II – 31 to 50
T3 – Tumor between 40 mm and 60mm in diameter and/or Stage III – 51 to 70
extend beyond the primary region and/or through adjacent Stage IV – 70 to 155
periosteum.
T4 – Any tumor greater than 60 mm in diameter and/or Salivary Gland Tumor Classification by WHO (2005)
extending to involve adjacent structure Benign epithelial tumors
N-Category ♦♦ Pleomorphic adenoma (8940/0)
♦♦ Myoepithelioma (8982/0)
N (Node)
♦♦ Basal cell adenoma (8147/0)
N0 – No palpable nodes ♦♦ Warthin tumor (adenolymphoma) (8561/0)
N1 – Equivocal node enlargement ♦♦ Oncocytoma (oncocytic adenoma) (8290/0)
N2 – Clinically palpable homolateral regional node (s) – not fixed ♦♦ Canalicular adenoma (8149/0)
♦♦ Sebaceous adenoma (8410/0)
N3 – As N2 but fixed
♦♦ Lymphadenoma (8410/0)
N4 – Clinically palpable contralateral or bilateral node (s) – not • Sebaceous
fixed • Non-sebaceous
N5 – As N4 but fixed. ♦♦ Ductal papilloma (8503/0)
678   Mastering the BDS IIIrd Year  (Last 25 Years Solved Questions)

• Inverted ductal papilloma • Ameloblastic fibro-odontoma


• Intraductal papilloma • Odontoma
• Sialadenoma papilliferum –– Complex
♦♦ Cystadenoma (8440/0) –– Compound
• Odonto -ameloblastoma
Malignant Epithelial Tumors • Calcifying cystic odontogenic tumor
♦♦ Acinic cell carcinoma (8550/3) • Dentinogenic ghost cell tumor
♦♦ Mucoepidermoid carcinoma (8430/3) ♦♦ Mesenchyme and/or odontogenic ectomesenchyme with or
♦♦ Adenoid cystic carcinoma (8200/3) without odontogenic epithelium:
♦♦ Polymorphous low-grade adenocarcinoma • Odontogenic fibroma
♦♦ Epithelial-myoepithelial carcinoma (8562/3) • Odontogenic myxoma
♦♦ Clear cell carcinoma, not otherwise specified (8310/3) • Cementoblastoma
♦♦ Basal cell adenocarcinoma (8147/3) Malignant Tumors
♦♦ Sebaceous carcinoma (8410/3)
♦♦ Odontogenic carcinomas:
♦♦ Sebaceous lymphadenocarcinoma (8410/3)
• Malignant ameloblastoma
♦♦ Cystadenocarcinoma (8440/3)
• Ameloblastic carcinoma
♦♦ Low grade cribriform cystadenocarcinoma
• Primary intraosseous carcinoma
♦♦ Mucinous adenocarcinoma (8480/3)
• Clear cell odontogenic carcinoma
♦♦ Oncocytic carcinoma (8290/3)
• Ghost cell odontogenic carcinoma
♦♦ Salivary duct carcinoma
♦♦ Odontogenic sarcoma:
♦♦ Adenocarcinoma NOS (8140/3)
• Ameloblastic fibro-sarcoma
♦♦ Myoepithelial carcinoma (8982/3)
• Ameloblastic fibro-dentino sarcoma
♦♦ Carcinoma in pleomorphic adenoma (8941/3)
• Ameloblastic fibro-odonto-sarcoma.
♦♦ Carcinosarcoma (8980/3)
♦♦ Metastatizing pleomorphic adenoma (8940/1) Bone Related Lesions
♦♦ Squamous cell carcinoma (8070/3) ♦♦ Ossifying fibroma
♦♦ Small cell carcinoma (8041/3) ♦♦ Fibrous dysplasia
♦♦ Large cell carcinoma (8012/3) ♦♦ Osseous dysplasia
♦♦ Lymphoepithelial carcinoma (8082/3) ♦♦ Central giant cell lesions
♦♦ Sialoblastoma (8974/1) ♦♦ Cherubism
Soft Tissue Tumors ♦♦ Aneurysmal bone cyst
♦♦ Simple bone cyst.
Haemangioma (9120/0)
Melanotic Neuroectodermal Tumor of Infancy
Hematolymphoma Tumors
Classification of Odontogenic Ghost Cell Lesions
♦♦ Hodgkin lymphoma (By Praetorius 2006)
♦♦ Diffuse large B-cell lymphoma
Group 1: Simple cysts — calcifying odontogenic cyst
♦♦ Extranodal marginal zone B-cell lymphoma
Group 2: Cyst associated with odontogenic hamartomas or
Secondary Tumors benign neoplasms — Calcifying cystic odontogenic tumor
(CCOT). Following are the combinations:
Classification of Odontogenic Tumors by WHO (2005) CCOT associated with odontome
Benign Tumors CCOT associated with adenomatoid odontogenic tumor
♦♦ Odontogenic epithelium with mature fibrous stroma without CCOT associated with ameloblastoma
odontogenic mesenchyme: CCOT associated with ameloblastic fibroma
• Ameloblastoma (solid, multicystic, extraosseous, CCOT associated with ameloblastic fibro-odontome
peripheral, desmoplastic, unicystic)
CCOT associated with odonto - ameloblastoma
• Squamous odontogenic tumor
• Calcifying epithelial odontogenic tumor CCOT associated with odontogenic myxofibroma
• Adenomatoid odontogenic tumor Group 3: Solid benign odontogenic neoplasms with similar cell
• Keratocystic odontogenic tumor morphology to that in calcifying odontogenic cyst and with
♦♦ Odontogenic epithelium with odontogenic ectomesenchyme with dentinoid formation — Dentinogenic ghost cell tumor
or without hard tissue formation: Group 4: Malignant odontogenic neoplasms with features
• Ameloblastic fibroma similar to those of dentinogenic ghost cell tumor — Ghost cell
• Ameloblastic fibro-dentinoma odontogenic carcinoma
Section 3: Oral Pathology  679

Classification of Cyst by Mervin Shear ♦♦ Arena virus, i.e. lassa fever


♦♦ Calicivirus, i.e. upper respiratory tract infection
I. Cysts of the Jaws
♦♦ Corona virus
♦♦ Epithelial ♦♦ Bunya virus
• Developmental: ♦♦ Picorna virus
–– Odontogenic: ♦♦ Reo virus
- Gingival cyst of infants ♦♦ Toga virus
- Odontogenic keratocyst (neoplasm) ♦♦ Retro virus
- Dentigerous cyst
- Eruption cyst DNA Virus
- Lateral periodontal cyst ♦♦ Herpes virus
- Gingival cyst of adults • Herpes simplex
- Botryoid odontogenic cyst • Herpes zoster
- Glandular odontogenic cyst • Epstein-Barr virus
- Calcifying odontogenic cyst (neoplasm) ♦♦ Pox virus
– Non-odontogenic • Small pox
- Nasopalatine duct cyst • Monkey pox
- Nasolabial cyst • Adeno virus, i.e. pharyngoconjunctival fever
- Midpalatal raphe cyst of infants • Parvo virus
- Median palatine, median alveolar • Irido virus
- Median mandibular cyst • Papova virus, i.e. papillomas.
- Globulomaxillary cyst
– Inflammatory Classification of Ulcerative Lesions
- Radicular cyst, apical and lateral Microbial Origin
- Residual cyst
- Paradental cyst and mandibular infected buccal ♦♦ Bacterial:
cyst • Streptococcal
- Inflammatory collateral cyst • Tuberculosis
• Syphilis
• Non epithelial (pseudo cysts)
• Scarlet fever
–– Solitary bone cyst
• Diphtheria
–– Aneurysmal bone cyst
• Typhoid
Cyst Associated With Maxillary Antrum • Noma
♦♦ Fungal:
♦♦ Benign mucosal cyst of the maxillary antrum
• Histoplasmosis
♦♦ Post operative maxillary cyst
• Blastomycosis
Cyst of the Soft Tissues Of Mouth, Face And Neck • Paracoccidiomidomycosis
• Coccidiomidomycosis
♦♦ Dermoid and epidermoid cyst
• Cryptococcus
♦♦ Lymphoepithelial cyst (branchial cyst)
• Zygomycosis
♦♦ Thyroglossal duct cyst
• Aspergillosis
♦♦ Anterior medial lingual cyst (intra lingual cyst of foregut
♦♦ Viral:
origin)
• Herpes
♦♦ Oral cyst with gastric or intestinal epithelium
• HIV
♦♦ Cystic hygroma
• Pox virus
♦♦ Nasopharyngeal cyst
♦♦ Protozoal:
♦♦ Thymic cyst
• Entamoeba histolytica
♦♦ Cyst of salivary glands—mucous extravasation cyst,
• Leishmaniasis
mucous retention cyst, ranula, polycystic disease of the
• Toxoplasmosis
parotid.
♦♦ Parasitic cyst—hydatid cyst, cysticercus cellulosae, trichinosis. Physical Origin
Classifications of Viral Lesions in Oral Cavity ♦♦ Cheek bite (Morsicatio Buccarum)
♦♦ Traumatic
RNA Virus ♦♦ Thermal
♦♦ Orthomyxovirus, i.e. influenza ♦♦ Electrical
♦♦ Paramyxovirus, i.e. measles and mumps ♦♦ Osteoradionecrosis
♦♦ Rhabdovirus, i.e. rabies ♦♦ Anaesthetic
680   Mastering the BDS IIIrd Year  (Last 25 Years Solved Questions)

Chemical Origin Yellow lesions


♦♦ Phenol ♦♦ Lipoma
♦♦ Silver nitrate ♦♦ Fordyce’s granules
♦♦ Hydrogen peroxide ♦♦ Verruciform xanthoma
♦♦ Aspirin ♦♦ Jaundice
♦♦ Superficial abscess
Immunological
♦♦ Behcet’s syndrome Gray-black lesions
♦♦ Reiter’s syndrome ♦♦ Amalgam tattoo
♦♦ Erythema multiforme ♦♦ Graphite tattoo
♦♦ Erosive lichen planus (secondary ulcer) ♦♦ Hairy tongue
♦♦ Lupus erythematosus ♦♦ Heavy metal ingestion
♦♦ Sarcoidosis ♦♦ Racial pigmentation
♦♦ Cyclic neutropenia ♦♦ Slough of ANUG
♦♦ Ulcerative colitis
♦♦ HIV Red Lesions
♦♦ Pemphigus ♦♦ Reactive lesions
♦♦ Epidermolysis bullosa • Pyogenic granuloma
• Peripheral giant cell granuloma
Metabolic Ulcers
♦♦ Premalignant – malignant tumors
♦♦ Diabetes • Erythroplakia
♦♦ Uremia • Hemangioma
♦♦ Neutropenia • Kaposi’s sarcoma
♦♦ Sickle cell anemia ♦♦ Lesions associated with dermatologic conditions
♦♦ Agranulocytosis • Erythema migrans
♦♦ Crohn’s disease • Systemic lupus erythematosus
Nonspecific Ulcers • Erythema multiforme
• CREST syndrome
♦♦ HIV ulcers
♦♦ Metabolic
♦♦ Graft vs host reaction
• Vitamin B complex
♦♦ Necrotizing sialometaplasia
• Anemia
♦♦ Reynaud’s phenomenon
• Burning mouth syndrome
♦♦ Bacterial angiomatosis
♦♦ Blood disorders
Neoplastic • Hemophilia
• Thrombocytopenic purpura
♦♦ Squamous cell carcinoma
• Leukemia
Classification of Pigmented Lesions ♦♦ Infectious conditions
Blue lesions • Gingivitis
• Stomatitis
♦♦ Mucocele
• Pharyngitis
♦♦ Ranula
• Scarlet fever
♦♦ Salivary duct cyst
• Koplik spots
♦♦ Submucosal hemorrhage
• Infectious mononucleosis
♦♦ Eruption cyst
• Syphilis
♦♦ Blue nevus
• Candidiasis
Brown Lesions • Median rhomboid glossitis
♦♦ Macules ♦♦ Immunological abnormalities
♦♦ Cellular nevus • Plasma cell gingivitis
♦♦ Malignant melanoma • Drug reactions and contact allergies
♦♦ Drug induced melanoma • Wegener’s granulomatosis
♦♦ Café au lait spots ♦♦ Others
♦♦ Smoker’s melanosis • Traumatic ulcer
♦♦ Endocrine pigmentation • Petechiae
♦♦ Pigmented lichen planus • Ecchymosis
♦♦ HIV – oral melanosis • Radiation mucositis
♦♦ Peutz Jeghers syndrome • Thermal and chemical burns
Section 3: Oral Pathology  681

Classification of Oral Manifestations of HIV/AIDS by Classification of Gingival Hyperplasia


EC-Clearinghouse On basis of Etiological Factors and Pathologic Changes
Group 1: Strongly associated with HIV infection ♦♦ Inflammatory enlargement
♦♦ Candidiasis: Erythematous, pseudomembranous, angular • Chronic
cheilitis • Acute
♦♦ Hairy leukoplakia ♦♦ Drug induced enlargement
♦♦ Kaposi’s sarcoma ♦♦ Enlargement associated with systemic disease
♦♦ Non-Hodgkin’s lymphoma • Conditional enlargement
♦♦ Periodontal diseases: Linear gingival erythema, necrotizing –– Pregnancy
gingivitis, necrotizing periodontitis –– Puberty
Group 2: Less commonly associated with HIV infection –– Vitamin C deficiency
♦♦ Bacterial infections: Mycobacterium avium-intracellular, –– Plasma cell gingivitis
Mycobacterium tuberculosis –– Non-specific conditioned enlargement (Pyogenic
♦♦ Melanotic hyperpigmentation granuloma)
♦♦ Necrotizing ulcerative stomatitis • Systemic diseases causing gingival enlargement
♦♦ Salivary gland disease: dry mouth, unilateral or bilateral –– Leukemia
swelling of major salivary glands –– Granulomatous disease (E.g. Wegner’s Granulo­
♦♦ Thrombocytopenic purpura matosis, sarcoidosis)
♦♦ Oral ulcerations NOS (not otherwise specified) ♦♦ Neoplastic enlargement
♦♦ Viral infections: herpes simplex, human papilloma virus, • Benign tumors
• Malignant tumors
varicella—zoster
♦♦ False enlargement
Group 3: Seen in HIV infection
Using the criteria of location and distribution gingival
♦♦ Bacterial infections: Actinomyces israelii, Escherichia coli,
enlargement is designated as follows:
Klebsiella pneumoniae.
♦♦ Cat-scratch disease (Bartonella henselae). – Localized: Gingival enlargement limited to one or more teeth.
♦♦ Epithelioid (bacillary) angiomatosis (Bartonella henselae) – Generalized: Involving the gingiva throughout the mouth.
♦♦ Drug reactions: Ulcerative, erythema multiforme, lichenoid, – Marginal: Confined to marginal gingiva.
toxic epidermolysis. – Papillary: Confined to interdental papilla.
♦♦ Fungal infections other than candidiasis: Cryptococcus neofor- – Diffuse: Involving the marginal and attached papillae.
mans, Geotrichum candidum, Histoplasma capsulatum, Muc-
– Discrete: Isolated sessile or pedunculated tumor like enlargement.
oraceae (mucormycosis/zygomycosis), Aspergillus flavus.
♦♦ Neurologic disturbances: Facial palsy, trigeminal neuralgia On basis of degree of gingival enlargement
♦♦ Recurrent aphthous stomatitis. Grade 0: No sign of gingival enlargement.
♦♦ Viral infections: Cytomegalovirus, Molluscum contagiosum. Grade I: Enlargement confirmed to interdental papilla
Grade II: Enlargement involves papilla and marginal gingiva.
Classification of Candidiasis by Axell Et Al 1997
Grade III: Enlargement covers three quarters or more of the crown.
♦♦ Primary oral candidiasis: Classification of Pulpal Diseases
• Acute form: Grossman’s Clinical Classification
–– Pseudomembranous candidiasis
♦♦ Pulpitis (Inflammation)
–– Erythematous candidiasis
• Reversible pulpitis
• Chronic form:
–– Symptomatic (acute).
–– Hyperplastic candidiasis
–– Asymptomatic (chronic).
–– Erythematous candidiasis
• Irreversible pulpitis
–– Pseudomembranous candidiasis –– Acute
♦♦ Candida associated lesion: - Abnormally responsive to cold.
• Denture stomatitis - Abnormally responsive to heat.
• Angular stomatitis –– Chronic
• Median rhomboid glossitis - Asymptomatic with pulp response.
♦♦ Keratinized primary lesion super-infected with candida: - Hyperplastic pulpitis.
• Leukoplakia - Internal resorption.
• Lichen planus ♦♦ Pulp degeneration
• Lupus erythematosus • Calcific (Radiographic diagnosis)
♦♦ Secondary candidiasis: • Others (Histopathologic diagnosis).
Candidal endocrinopathy syndrome. ♦♦ Pulp necrosis.
682   Mastering the BDS IIIrd Year  (Last 25 Years Solved Questions)

Seltzer and Bender’s Classification • Syphilis


• Granuloma inguinale
This classification is based on clinical tests and histological
• Brucellosis
diagnosis
• Cat scratch disease
♦♦ Treatable without pulp extirpation and endodontic treatment
• Tularemia
• Intact uninflamed pulp
• Glanders
• Transition stage
• Actinomycosis
• Atrophic pulp
♦♦ Fungal:
• Acute pulpitis
• Blastomycosis
• Chronic partial pulpitis without necrosis.
• Cryptococcosis
♦♦ Untreatable without pulp extirpation and endodontic treatment
• Coccidioidomycosis
• Chronic partial pulpitis with necrosis
• Histoplasmosis
• Chronic total pulpitis
♦♦ Parasitic:
• Total pulp necrosis.
• Schistosomiasis
Ingle’s Classification
Non-specific
♦♦ Inflammatory changes
• Hyper-reactive pulpalgia ♦♦ Sarcoidosis
–– Hypersensitivity ♦♦ Crohn’s disease
–– Hyperemia ♦♦ Silicosis
• Acute pulpalgia ♦♦ Berylliosis
–– Incipient ♦♦ Foreign body granuloma
–– Moderate ♦♦ Orofacial granulomatosis
–– Advanced
Classification of Osteomyelitis
• Chronic pulpalgia
• Hyperplastic pulpitis ♦♦ Acute osteomyelitis
• Pulp necrosis • Acute suppurative osteomyelitis
♦♦ Retrogressive changes • Acute subperiosteal osteomyelitis
• Atrophic papulosis • Acute periostitis
• Calcific papulosis. ♦♦ Chronic osteomyelitis
• Non- Specific Type
Classification of Dental Caries
–– Chronic intramedullary osteomyelitis
♦♦ Based on location of the lesion –– Chronic focal sclerosing osteomyelitis
• Pit and fissure caries –– Chronic diffuse sclerosing osteomyelitis
–– Occlusal –– Chronic osteomyelitis with proliferative peri­
–– Buccal or lingual pit ostitis
• Smooth surface caries –– Chronic subperiosteal osteomyelitis
–– Proximal –– Chronic periostitis
–– Buccal or lingual surface • Specific type
• Root caries –– Tuberculous osteomyelitis
♦♦ Based on tissue involved –– Syphilitic osteomyelitis
• Enamel caries –– Actinomycotic osteomyelitis
• Dentinal caries ♦♦ Radiation induced osteomyelitis
• Cementum caries ♦♦ Idiopathic osteomyelitis
♦♦ Based on virginity of the lesion
• Primary caries Classification of Fibro-Osseous Lesions
• Secondary caries ♦♦ Developmental
♦♦ Based on progression of lesion • Solitary bone cyst
• Progressive caries • Gigantiform cementoma
–– Rapidly progressive like nursing caries and radia­ • Cherubism
tion caries ♦♦ Reactive/Reparative
–– Slowly progressing • Aneurysmal bone cyst
• Arrested caries • Central giant cell granuloma
Classification of Granulomatous Lesions or Diseases • Garre’s osteomyelitis
• Osseous dysplasia
Specific or Infective Type –– Florid osseous dysplasia
♦♦ Bacterial: –– Cemental osseous dysplasia
• Tuberculosis –– Focal osseous dysplasia or sclerosing osteomy­
• Leprosy elitis
Section 3: Oral Pathology  683

• Osseous keloid –– Splenic sequestration and destruction


• Traumatic periostitis –– Associated diseases like lymphomas
♦♦ Neoplasms –– Drugs, chemical and physical agents
• Benign cementoblastoma –– Trauma to RBC
• Ossifying fibroma ♦♦ Intracorpuscular hemolytic diseases
–– Conventional • Hereditary
–– Juvenile trabecular –– Disorders of glycolysis
–– Juvenile psammomatoid –– Faulty synthesis or maintenance of reduced
• Osteoma glutathione.
• Osteoid osteoma –– Qualitative or quantitative abnormalities in the
• Osteoblastoma synthesis of globulin
♦♦ Endocrinal/Metabolic –– Abnormalities in RBC membrane
• Brown tumor of hyperparathyroidism –– Erythropoietic porphyria
♦♦ Idiopathic • Acquired
• Fibrous dysplasia –– Paroxysmal nocturnal hemoglobinuria
• Paget’s disease –– Lead poisoning
♦♦ Impaired blood production resulting from deficiency of
Classification of Bone Disorders of Face and Jaw substances essential for erythropoiesis
♦♦ Developmental defects of bone formation of face and jaw • Iron deficiency
• Agnathia • Deficiency of various B vitamins: Vitamin B12 and folic
• Micrognathia acid (pernicious anemia and megaloblastic anemia);
• Macrognathia pyridoxine responsive anemia
• Facial hemiatrophy • Protein deficiency
• Facial hemihypertrophy • Possibly ascorbic acid deficiency
• Cleft palate. ♦♦ Inadequate production of mature erythrocytes
♦♦ Benign and malignant lesions of bone • Deficiency of erythroblast
• Osteoma –– Atrophy of bone marrow: Aplastic anemia
• Osteosarcoma - Chemical or physical agents
• Ewing’s sarcoma. - Hereditary
♦♦ Fibrosseous lesions - Idiopathic
• Fibrous dysplasia of bone • Isolated erythroblastopenia
• Ossifying fibroma –– Thymoma
• Cemetifying fibroma –– Chemical agents
• Paget’s disease of bone –– Antibodies
• Cherubism • Infiltration of bone marrow
• Osteogenesis imperfecta –– Leukemia, lymphomas
• Cleidocranio dysplasia –– Multiple myeloma
• Hurler’s syndrome –– Carcinoma, sarcoma
• Garre’s osteomyelitis –– Myelofibrosis
• Jaw lesions in hyperparathyroidism • Endocrine abnormalities
• Aneurysmal bone cyst. –– Myxedema
–– Addison’s disease
Classification of Anemia –– Pituitary insufficiency
Etiological classification of anemia (By Lea and Febiger 1981) –– Sometimes hyperthyroidism
♦♦ Loss of blood • Chronic renal failure
• Acute posthemorrhagic anemia • Chronic inflammatory disease
• Chronic posthemorrhagic anemia –– Infectious
♦♦ Excessive destruction of red blood corpuscles –– Non-infectious including granulomatous and
• Extracorpuscular causes collagen disease
–– Antibodies • Cirrhosis of liver
–– Infections like malaria
684   Mastering the BDS IIIrd Year  (Last 25 Years Solved Questions)

Morphological Classification of Anemia Classification of Vesiculobullous Lesions


Vesiculobullous Lesions
Type of anemia Description Common causes
Macrocytic Increased MCV, MCH Lack of erythrocyte- Fitz Patrick classification
and normal MCHC maturating factors
♦♦ According to anatomical plane:
(intrinsic and extrinsic
factors) • Lntra epidermal blister granular layer
–– Pemphigus foliaceous
Normocytic Reduction only in Hemorrhage, hemo-
–– Frictional blisters
RBC number, normal lysis, lack of blood
MCV, MCH MCHC formation and dilution –– Staphylococcus scalded syndrome
of blood with fluid • Spinous layer
Simple microcytic Reduced MCV, MCH Associated with in-
–– Eczematous dermatitis
and normal MCHC fections and inflam- –– Secondary to heat and cold
matory diseases –– Herpes virus infection
Hypochromic micro- Reduced MCV, MCH Iron deficiency –– Familial benign pemphigus
cytic and MCHC • Suprabasal
–– Pemphigus vulgaris
Abbreviations: MCV, mean corpuscular volume; MCH, mean corpuscular
–– Pemphigus vegetans
hemoglobin; MCHC, mean corpuscular hemoglobin concentration.
–– Darier’s disease
Classification of Acute Leukemia (FAB Classification) • Basal layer
–– Erythema multiforme
Acute myeloblastic leukemia • Toxic epidermal necrolysis
M inimally differentiated: Myeloblasts lack definite
M0:  • Lupus erythematosis
cytologic and cytochemical features but have myeloid • Lichen planus
lineage antigens • Epidermolysis bullosa simplex
AML without maturation: Myeloblasts predominate with
M1:  ♦♦ Dermal-epidermal junction zone:
• Lamina lucida
distant nucleoli, few granules or auer rods are present.
–– Bullous pemphigoid
AML with maturation: Myeloblasts wtih promyelocytes
M2:  –– Cicatrical pemphigoid
predominate and auer rods may be present. –– Epidermolysis bullosa junctional
A cute promyelocytic leukemia: Hypergranular
M3:  • Below basal lamina
promyelocytes often with multiple auer rods are seen. –– Erythema multiforme
Acute myelomonocytic leukemia: Mature cells of both
M4:  –– Epidermolysis bullosa dystrophica
myeloid and monocytic series in peripheral blood;
myeloid cells resemble M2 Vesiculobullous Lesions
Acute monocytic leukemia: Promonocytes or
M5:  ♦♦ Primary blistering:
undifferentiated blast. • Pemphigus
A cute erythroleukemia: Erythroblast predominate;
M6:  • Bullous pemphigoid
myeloblasts and promyelocytes also increased • Cicatrical pemphigoid
A c u t e m e g a k a r y o c y t i c l e u k e m i a : Pl e o m o r p h i c
M7:  • Epidermolysis bullosa acqusita
undifferentiated blast cells predominate and react ♦♦ Secondary blistering:
with antiplatelet antibodies. • Contact
• Erythema mutltiforme
Acute Lymphoblastic Leukemia • Toxic epidermal necrolysis
L1:  Acute lymphoblastic (Seen in children): Homogeneous ♦♦ Infection:
small lymphoblasts; scanty cytoplasm, regular round • Varicella zoster
nuclei, inconspicuous nucleoli. • Herpes simplex
L2:  Acute lymphoblastic (Seen in adults): Heterogeneous • Bullous impetigo
lymphoblasts; variable amount of cytoplasm, irregular ♦♦ Systemic disease:
or cleft nuclei, large nucleoli. • Infection-cutaneous emboli
L3: B urkitt’s type (Uncommon): Large homogenous • Metabolic
lymphoblasts; Nuclei are round to oval, prominent –– Diabetic with bullae
nucleoli, cytoplasmic vacuolation. –– Porphyria cutanea tarda
Section 3: Oral Pathology  685

Classification of Mucocutaneous Lesions of Oral Cavity ♦♦ Pigmentation


• Anemia
Genodermatosis
• Albert syndrome
♦♦ Darier’s disease • Addison’s disease
♦♦ White sponge nevus • Racial Pigmentation
♦♦ Hereditary benign intraepithelial dyskeratosis • Endocrinopathy
♦♦ Peutz-jeghers syndrome
♦♦ Pachyonychia congenita
♦♦ Dyskeratosis congenita Classification of Bite Marks
♦♦ Pseudoxanthoma elasticum
MacDonald’s Classification
Non-Infective Disease He had given an etiological classification
♦♦ Vesicular ♦♦ Tooth pressure marks: Marks on tissue due to ‘direct appli­
• Bullous pemphigoid cation of pressure by teeth’. Incisal and occlusal surfaces
• Benign mucous membrane pemphigoid produce these marks.
• Pemphigus ♦♦ Tongue pressure marks: If sufficient amount of tissue is taken
• Erythema multiforme in mouth tongue presses the tissue against rigid areas i.e.
• Lichen planus lingual surface of teeth as well as palatal rugae. Marks left
• Epidermolysis Bullosa over skin are called as suckling.
♦♦ Non-vesicular ♦♦ Tooth scrape marks: They occur due to scrapping of teeth
• Geographic tongue over bitten material. They are caused by anterior teeth
• Lichen planus
♦♦ Collagen disorders Classification of Lip Prints
• Wegners granulomatosis
• Midline lethal granuloma By Tsuchihashi
• Polyarteritis nodosa Type I: Clear cut vertical grooves that run across entire lip
• Scleroderma Type I: Similar to type I but do not cover entire lip
• Lupus erythematosus
Type II: Branched grooves
• Vasculitis
♦♦ Degenerative disorder Type III: Intersected grooves
• OSMF Type IV: Reticular grooves
• Amyloidosis Type V: Grooves that cannot be morphologically
• Solar elastosis differentiated.

Histological section shows beauty of pathology decorated with blue and pink colors, mimicking colors of life and
rejuvenating my eyes.
The epithelium connective tissue junction seems to be like horizon, where I stand alone, asking god: sir: why you should
have created pathology for every aspect of histology?
He answered me back: my son! If pathology would not have been there, what is the importance of being normal, men never
would have taken care.
Plate 1

Page 45, Q. 13: Tetralogy of Fallot Page 493, Q. 2: Microdontia

Page 494, Q. 3: Geographic tongue Page 495, Q. 6: Anodontia

Page 496, Q. 7: Amelogenesis imperfecta Page 496, Q. 8: Mottled enamel


Plate 2

Page 498, Q. 11: Fusion Page 499, Q. 13: Supernumerary teeth

Page 499, Q. 14: Dentinogenesis imperfecta Page 500, Q. 15: Median rhomboid glossitis

Page 500, Q. 16: Lingual thyroid Page 501, Q. 17: Perleche


Plate 3

Page 507, Q. 26: Ghost teeth


Page 501, Q. 19: Concrescence

A.  Mild dysplasia B.  Moderate dysplasia


Page 513 Q. 1: Leukoplakia

Page 511, Q. 41: Mesiodens


Page 514, Q. 3: Hairy leukoplakia
Plate 4

Page 515, Q. 4: Oral Submucous fibrosis Page 515, Q. 5: Intraepithelial carcinoma

Page 516, Q. 6: Carcinoma of tongue Page 517, Q. 7: Peripheral giant cell granuloma

Page 517, Q. 8: Capillary hemangioma Page 517, Q. 8: Cavernous hemangioma


Plate 5

Page 518, Q. 11: Verrucous carcinoma Page 519, Q. 13: Central giant cell granuloma

Page 519, Q. 15: Moderately differentiated squamous cell carcinoma Page 521, Q. 17: van Gieson’s stain

Page 521, Q. 16: Giant cell


Plate 6

Page 521, Q. 17: Mallory stain Page 521, Q. 17: PAS stain

Page 522, Q. 17: Masson’s trichrome stain Page 525, Q. 20: Malignant melanoma

Page 526, Q. 22: Hodgkin’s lymphoma


Plate 7

Page 527, Q. 23: Well-differentiated squamous cell carcinoma Page 527, Q. 23: Poorly-differentiated squamous cell carcinoma

Page 528, Q. 25: Osteosarcoma Page 529, Q. 29: Fibroma

Page 530, Q. 31: Fibrosarcoma Page 531, Q. 32: Keratoacanthoma


Plate 8

Page 533, Q. 38: Lipoma Page 533, Q. 39: Ewing’s sarcoma

Page 534, Q. 41: Neurilemmoma

Page 536, Q. 44:  Papilloma (H and E stain) Page 536, Q. 45:  Ewing’s sarcoma (H and E Stain)
Plate 9

Page 537, Q. 47:  Multiple myeloma Page 540, Q.50: Lymphangioma

Page 541, Q. 1: Pleomorphic adenoma Page 543, Q. 4: Mucoepidermoid tumor

A.  Mucous retention cyst B.  Mucous extravasation cyst


Page 545, Q. 9: Mucocele
Plate 10

Page 544, Q. 5: Adenoid cystic carcinoma Page 547, Q. 15: Warthin’s tumor

Page 549, Q. 1: Follicular ameloblastoma Page 549, Q. 1: Plexiform ameloblastoma

Page 549, Q. 1: Acanthomatous ameloblastoma Page 550, Q. 1: Granular cell ameloblastoma


Plate 11

Page 551, Q. 5: Adenomatoid odontogenic tumor Page 551, Q. 6: Pindborg’s tumor

Page 552, Q. 7: Complex composite odontome Page 552, Q. 7: Compound composite odontome

Page 554, Q. 1:  Dentigerous cyst Page 556, Q. 4: Odontogenic keratocyst


Plate 12

Page 557, Q. 5: Radicular cyst Page 560, Q. 20: Calcifying odontogenic cyst

Page 563, Q. 5: Actinomycosis Page 564, Q. 6: Tuberculosis

Page 566, Q. 9: Pyogenic granuloma Page 574, Q. 1: Candida albicans


Plate 13

Page 583, Q. 3: Dentinal caries H and E (DS) Page 587, Q. 14: Pit and fissure caries (GS)

Page 587, Q. 15: Streptococcus mutans Page 590, Q. 1: Acute pulpitis

Page 591, Q. 2: Pulp polyp/chronic hyperplastic pulpitis


Plate 14

Page 592, Q. 3: Chronic pulpitis Page 592, Q. 5: Periapical granuloma

Page 594, Q. 8: Osteomyelitis Page 603, Q. 1: Abrasion

Page 604, Q. 2: Sclerotic dentin


Plate 15

Page 604, Q. 3: Pulp fibrosis Page 605, Q. 3: True pulp stones

Page 605, Q. 3: False pulp stones Page 605, Q. 3: Diffuse calcification

Page 606, Q. 4: Internal resorption of teeth or pink tooth


Plate 16

Page 606, Q. 5: Hypercementosis Page 607, Q. 6: Attrition

Page 617, Q. 5: Scurvy Page 619, Q. 5: Fibrous dysplasia

Page 620, Q. 6: Paget’s disease of bone


Plate 17

Page 622, Q. 10: Cherubism Page 625, Q. 1: Peripheral smear showing pernicious anemia

Page 628, Q. 8: Peripheral smear showing iron deficiency anemia Page 630, Q. 10: Acute myeloid leukemia

Page 630, Q. 10: Acute lymphoblastic leukemia Page 630, Q. 10: Chronic myeloid leukemia
Plate 18

Page 631, Q. 10: Chronic lymphocytic leukemia Page 632, Q. 1: Lichen planus

Page 633, Q. 3: Pemphigus Page 633, Q. 5: Erythema multiforme

Page 636, Q. 13: Lupus erythematosus


Plate 19

Page 641, Q. 22: LE Cell inclusion phenomenon

Page 644, Q. 2: Lip patterns

You might also like